You are on page 1of 1233

Updates Needed

Basically updates from all of the china topic stuff

North Korea

China (war/relations/rise/econ)

Japan (war with china/relations/econ)

Cyberwar

Warming

Air Pollution

Manufacturing

NATO

Global Alliances (trump updates)

Trump agenda?

Venezuela

Reformat
Things to Add
Zoonotic Disease

Ebola

Econ Stuff from main impact D file

International Economic Stuff

Biodiveristy – extensions

China Pollution

Honeybees

Invasive Species

Medicine

Methane Releases

Phytoplankton

Separate file for systemic stuff? Ethical obligations?

Basically, consolidate old file and this file

Add Analytical Extensions

Other List:

Fix SoPo Extensions

Russia War Extensions


Impact Defense
Accidents and Miscalc Answers
Frontline

Zero risk of accidents

Quinlan 9 (Sir Michael Quinlan, Former Permanent Under-Secretary of State UK Ministry of Defense, Thinking About Nuclear
Weapons: Principles, Problems, Prospects, p. 63-69, The book reflects the author's experience across more than forty years in
assessing and forming policy about nuclear weapons, mostly at senior levels close to the centre both of British governmental
decision-making and of NATO's development of plans and deployments, with much interaction also with comparable levels of United
States activity in the Pentagon and the State department)

There have certainly been, across the decades since 1945, many known accidents involving nuclear weapons,
from transporters skidding off roads to bomber aircraft crashing with or accidentally dropping the weapons they carried (in past days
when such carriage was a frequent feature of readiness arrangements it no longer is). A few of these accidents may have released
into the nearby environment highly toxic material. None however has entailed a nuclear detonation. Some
commentators suggest that this reflects bizarrely good fortune amid such massive activity and deployment over so many years. A
more rational deduction from the facts of this long experience would however be that the probability of any accident triggering a
nuclear explosion is extremely low. It might be further nested that the mechanisms needed to set of such an explosion are
technically demanding, and that in a large number of ways the past sixty years have seen extensive improvements in safety
arrangements for both the design and the handling of weapons. It is undoubtedly possible to see respects in which, after the cold
war, some of the factors bearing upon risk may be new or more adverse; but some are now plainly less so. The
years which the
world has come through entirely without accidental or unauthorized detonation haveincluded early decades in which
knowledge was sketchier, precautions were less developed, and weapon designs were less
ultra-safe than they later became, as well as substantial periods in which weapon numbers were larger, deployments immure
widespread arid diverse, movements more frequent, and several aspects of doctrine and readiness arrangements more tense.
Similar considerations apply to the hypothesis of nuclear war being mistakenly triggered by false alarm. Critics again point to the
fact, as it is understood, of numerous occasions when initial steps in alert sequences for US nuclear forces were embarked upon, or
at least called for, by indicators mistaken or misconstrued. In none of these instances, it is accepted, did matters get at all near to
nuclear launch—extraordinary good fortune again, critics have suggested. But the rival and more
logical inference from
hundreds of events stretching over sixty years of experience presents itself once more: that the probability
of initial misinterpretation leading far towards mistaken launch is remote. Precisely because any nuclear weapon
processor recognizes the vast gravity of any launch, release sequences have many steps, and human decision
is repeatedly interposed as well as capping the sequences. To convey that because a first step was prompted the world
somehow came close to accidental nuclear war is wild hyperbole, rather like asserting, when a tennis champion has lost his opening
History anyway scarcely offers any ready example of
service game, that he was nearly beaten in straight sets.
major war started by accident even before the nuclear revolution imposed an order-of-
magnitude increase of caution. In was occasion conjectured that nuclear war might be triggered by the real but
accidental or unauthorized launch of a strategic nuclear-weapon delivery system in the direction of a potential adversary. No such
launch is known to have occurred in over sixty years. The probability of it is therefore very low. But even if it did happen, the further
hypothesis of it initiating a general nuclear exchange is far-fetched. It fails to consider the real situation of decision-makers, as pages
63-4 have brought out. The
notion that cosmic holocaust might be mistakenly precipitated in this way
belongs to science fiction .
---Accidents- No Escalation

No escalation from accidents

- safeguards, no use or lose pressures, and rational leaders all prevent them and history proves
the scenario is science fiction

Quinlan 9 (Sir Michael Quinlan, Former Permanent Under-Secretary of State UK Ministry of Defense, Thinking About Nuclear
Weapons: Principles, Problems, Prospects, p. 63-69, The book reflects the author's experience across more than forty years in
assessing and forming policy about nuclear weapons, mostly at senior levels close to the centre both of British governmental
decision-making and of NATO's development of plans and deployments, with much interaction also with comparable levels of United
States activity in the Pentagon and the State department)

supposition of inexorable momentum in a developing


Even if initial nuclear use did not quickly end the fighting, the
exchange, with each side rushing to overreaction amid confusion and uncertainty, is implausible. It fails to consider
what the situation of the decision-makers would really be. Neither side could want escalation . Both would be
appalled at what was going on. Both would be desperately looking for signs that the other was
ready to call a halt. Both, given the capacity for evasion or concealment which drive modern
delivery platforms and vehicles can possess, could have in reserve significant forces invulnerable
enough not to entail use-or-lose pressures. (It may be more open to question, as noted earlier, whether newer
nuclear weapon possessors can be immediately in that position; but it is within reach of any substantial state with advanced
technological capabilities and attaining it is certain to be a high priority in the development of forces.) As a result ,
neither side
can have any predisposition to suppose, in an ambiguous situation of fearful risk, that the right
course when in doubt is to go on copiously launching weapons. And none of this analysis rests on any
presumption of highly subtle or pre-concerted rationality. The rationality required is plain. The argument is reinforced if we consider
the possible reasoning of an aggressor at a more dispassionate level. Any substantial nuclear armoury can inflict destruction
outweighing any possible prize that aggression could hope to seize. A state attacking the possessor of such an armoury must
therefore be doing so (once given that it cannot count upon destroying the armoury pre-emptively) on a judgment that the
possessor would be found lacking in the will to use it. If the attacker possessor used nuclear weapons, whether first or in response to
the aggressor’s own first use, this judgment would begin to look dangerously precarious. There must be at least a substantial
probability of the aggressor leaders’ concluding that their initial judgment had been mistaken—that the risks were after all greater
than whatever prize they had been seeking, and that for their own country’s survival they must call off the aggression. Deterrence
planning such as that of NATO was directed in the first place to preventing the initial misjudgment and in the second, if it were
nevertheless made, to compelling such a reappraisal. The former aim had to have primacy, because it could not be taken for granted
that the latter was certain to work. But there was no ground for assuming in advance, for all possible scenarios, that the chance of its
working must be negligible. An aggressor state would itself be at huge risk if nuclear war developed, as its leaders would know. It
may be argued that a policy which abandons hope of physically defeating the enemy and simply hopes to get him to desist is pure
gamble, a matter of who blinks first; and that the political and moral nature of most likely aggressors, almost ex hypothesi, makes
them less likely to blink. One response to this is to ask what is the alternative—it can be only surrender. But a more hopeful answer
lies in the fact that the criticism is posed in a political vacuum. Real-life conflict would have a political context .

No accidental or miscalc escalation

– history and disaster behavior studies prove

Mueller ’10 [John Mueller – Woody Hayes Chair of National Security Studies, Mershon Center, and professor of Political Science,
at Ohio State University, Atomic Obsession: Nuclear Alarmism from Hiroshima to Al-Qaeda, p. 100
However, even if a bomb, or a few bombs, were to go off, it does not necessarily follow that war would result. For that to happen, it
is usually assumed, the accident would have to take place at a time of high war- readiness, as during a crisis, when both sides are
poised for action and when one side could perhaps be triggered—or panicked—into major action by an explosion mistakenly taken
to be part of, or the prelude to, a full attack." This means that the unlikely happening—a nuclear accident—would have to coincide
precisely with an event, a militarized international crisis, something that is rare to begin with, became more so as the cold war
progressed, and has become even less likely since its demise. Furthermore ,
even if the accident takes place during a crisis, it
does not follow that escalation or hasty response is inevitable , or even very likely. As Bernard Brodie
points out, escalation scenarios essentially impute to both sides "a well-nigh limitless concern with
saving face" and/or "a great deal of ground-in automaticity of response and counterresponse." None of this was in
evidence during the Cuban missile crisis when there were accidents galore . An American spy plane was
shot down over Cuba, probably without authorization, and another accidentally went off course and flew threateningly over the
Soviet Union. As if that weren't enough, a Soviet military officer spying for the West sent a message, apparently on a whim, warning
that the Soviets were about to attack."
None of these remarkable events triggered anything in the way of
precipitous response. They were duly evaluated and then ignored . Robert Jervis points out that "when
critics talk of the impact of irrationality, they imply that all such deviations will be in the direction of emotional impulsiveness, of
launching an attack, or of taking actions that are terribly risky. But
irrationality could also lead a state to passive
acquiescence:" In moments of high stress and threat, people can be said to have three psychological alternatives: (1) to
remain calm and rational, (2) to refuse to believe that the threat is imminent or significant, or (3) to panic, lashing out frantically and
incoherently at the threat. Generally, people react in one of the first two ways.
In her classic study of disaster behavior,
MarthaWolfenstein concludes, "The usual reaction is one of being unworried ."32 In addition, the
historical record suggests that wars simply do not begin by accident . In his extensive survey of
wars that have occurred since 1400, diplomat-historian Evan Luard concludes, "It is impossible to identify a
single case in which it can be said that a war started accidentally; in which it was not, at the time the war broke out,
the deliberate intention of at least one party that war should take place." Geoffrey Blainey, after similar study, very much agrees:
although many have discussed "accidental" or "unintentional" wars, "it is difficult," he concludes, "to find a war which on
investigation fits this description." Or, as Henry Kissinger has put it dryly, "Despite popular myths, large military units do not fight by
accident.""
---Accidents- No Escalation Spiral

No escalation spiral

- history proves

Reiter ‘95 (Dan Reiter, Political Science at Emory,. "Exploding the powder keg myth: preemptive wars almost never
happen." International Security 20.n2 Fall 1995)

The spiral model sees preemption as the end result of a self-sustaining dynamic of conflict. One side feels
threatened and mobilizes, the other side responds by mobilizing, both sides fear the other will preempt so
each is then motivated to strike first, and one of them eventually does, Preemptive war becomes a self-fulfilling prophecy, as the
fear of being the target of a preemptive attack encourages a state to strike. Whilethis vision has a certain intuitive
appeal, the historical record demonstrates that in the modem era such spirals have rarely
caused crisis to escalate to preemptive war. Rather, it might be more accurate to view
preemptive war as a self-denying prophecy rather than a self-fulfilling one. Leaders often
understand the danger of stumbling into preemptive war, and rather than accepting a spiral to
war as inevitable and therefore striking first, they try to decrease the dangers of preemptive
war by taking actions intended to alleviate the opponents fear of a surprise attack. In other
words, political leaders, as analysts of international relations, often subscribe to the spiral
model of war, leading them to take measures to avoid such spirals . One study of crisis behavior
found that crisis decision-makers are usually worried about sparking an escalation spiral even
exaggerating the risk of doing so.(53) Sometimes these mutual fears of preemption lead to a
peaceful resolution of a crisis. War is avoided because each side is careful to avoid provoking the
other. In the Cuban Missile Crisis, for example, U.S. decision-makers were quite wary of backing the Soviets into a comer or taking
actions that might force the Soviets into preempting. Even after the naval quarantine was announced, President Kennedy let pass a
Soviet tanker approaching Cuba, in order to give Khrushchev more time.(54) Some have argued that it was fear of the consequences
of scaring the other side into preemptive attack that facilitated a peaceful resolution of the crisis. In the Cuban Missile Crisis, for
example, U.S. decision-makers were quite wary of backing the Soviets into a corner or taking actions that might force the Soviets
into preempting. Even after the naval quarantine was announced, President Kennedy let pass a Soviet tanker approaching Cuba, in
order to give Khrushchev more time.54 Some have argued that it was fear of the conse-quences of scaring the other side into
preemptive attack that facilitated a peaceful resolution of the crisis.55 It is worth noting that U.S. fears of preemption in the Cuban
Missile Crisis were strongly driven by the shadow of World War I; the president's "Executive Committee" (ExComm) feared a
repetition of the July Crisis of 1914. President Kennedy had just read Barbara Tuchman's The Guns of August, which argues that
World War I was a war of preemption. During the crisis, this interpretation of World War I steered Kennedy and the ExComm to
focus on the dangers of belligerence. Specifically, Kennedy believed that the Russians must not be backed into a corner: "I am not
going to follow a course which will allow anyone to write a comparable book about this time, The Missiles of October. If anybody is
around to write after this, they are going to understand that we made every effort to find peace and every effort to give our
adversary room to move. I am not going to push the Russians an inch beyond what is neces-sary"56 Since 1962, a number of analysts
have followed Kennedy's example, analogizing by drawing lessons from the Cuban Missile Crisis (and the July Crisis) to argue for the
importance of crisis stability and the dangers of pre-emption. It may be that the high salience of the Cuban Missile and July crises
accounts for the popularity of the preemptive war scenario in discourse about the causes of war, in spite of the fact that preemptive
wars almost never happen.57 The Soviets, too, were cautious about pushing the Americans too far. Khru-shchev's first letter to
Kennedy spoke specifically of the danger of an escalating spiral of hostility: we and you ought not to pull on the ends of the rope in
which you have tied the knot of war, because the more the two of us pull, the tighter the knot will be tied. And a moment may come
when that knot will be tied so tight that even he who tied it will not have the strength to untie it, and then it will be necessary to cut
that knot, and what that would mean is not for me to explain to you, because you yourself understand perfectly of what terrible
forces our countries dispose.58 It is worth noting that members of the Soviet leadership became convinced at various times during
the crisis that a U.S. attack against the Soviet Union was imminent, yet the Soviets did not undertake preemptive action against
either the U.S. mainland or Western Europe.59 One might argue that the Soviets did not preempt because they recognized their
own inferiority. However, they still might have seen advantages in striking first once war was seen as inevitable. The high stress of a
feared U.S. attack might have led the Soviet leadership either to believe that a preemptive attack would be successful because the
Americans would back down after having their nose bloodied from a first strike, or to overestimate their own capabilities. Egypt's
leaders in 1967, for example, were so confident of their military's ability to defeat Israel that they were willing to grant Israel the first
blow. The stunning and decisive defeat dealt to Egypt demonstrated how wrong they were.60 Sometimes a state is so fearful of
provoking its opponent into preemption that it avoids taking preparatory military measures important for the national defense.
The dilemma states often face is that those actions that raise the risks of preemptive war, such
as the mobilization of military forces, are also those that increase a state's military
preparedness. Though actions such as mobilization improve military preparedness and raise the
prospects for successful deterrence, the fear of preemptive war is often so great that it
overrides these concerns, and states forgo such moves . Consider, for example, Soviet actions
prior to the German invasion in 1941. Ignoring a stream of warnings and high-quality
intelligence, Stalin refused to believe that a German attack was imminent . Further, he declined
to take prudent defensive measures for fear of provoking Hitler . Even as German aircraft began to bomb
Soviet cities on June 22, Moscow ordered the Red Army not to return fire, hoping that these actions were taken by German generals
acting without authority.61 Two decades later, during the Cuban Missile Crisis, even as the Soviets came
to believe that U.S. military action was likely, they probably did not upgrade the military
readiness of their forces even though the United States was (very noisily) preparing to fight a
global nuclear war. Soviet caution was in contrast to the vision of quick and automatic escalation offered by the spiral
model; "their failure to make any preparations for general war was linked to a fear of provoking American preemptive action ."62
Other states have undercut their military preparedness to reduce the risks of preemption. During the Cold
War, some argued that NATO would have been very reluctant to raise the readiness of its nuclear
forces in the event of a crisis, fearing that such a move might encourage preemption by the Warsaw
Pact.63

Their World War One example is wrong- it's the other way around

Reiter ‘95 (Dan Reiter, Professor of Political Science at Emory, "Exploding the powder keg myth: preemptive wars almost never
happen." International Security 20.n2 Fall 1995

German actions in the July Crisis reflected this hesitance to preempt. After Russian partial
mobilization against Austria, the Germans refrained from mov-ing to full mobilization. They also
encouraged Austria to let Russia be the first to attack. The Germans knew that Russian mobilization would be substantially slower
than their own, so there would be relatively little sacrifice of military advantage by letting the Russians mobilize first .
Russia's
partial mobilization did not even lead to a hardening of the German diplomatic position .33
The German chief of staff, Helmut von Moltke, was also hesitant to react to the first warnings of
Russian general mobilization. As the German High Command began to pick up bits of evidence that Russia had initiated its
general mobili-zation sequence, von Moltke looked for alternate explanations for the intelli-gence reports, speculating that they
indicated maneuvers or a test mobilization.

Prefer our methodology

Reiter ‘95, Dan, polisci prof at emory. "Exploding the powder keg myth: preemptive wars almost never happen." International
Security 20.n2 Fall 1995
Which of these three visions of the frequency of preemptive war is best supported by the historical record? The set of cases includes
wars listed in the Correlates of War (COW) data set, which are military conflicts between recognized states in which the
participants suffered at least 1,000 battle casualties. The 1,000-casualty threshold effectively separates wars from minor, non-
escalating military clashes; such lesser conflicts deserve exclusion as they are often low-level probes or flare-ups of border tension,
and do not reflect decisions for war on the part of the national leadership. The COW project lists 67 wars between 1816 and 1980.
(18) A war is preemptive if a primary motivation for the attack is that the attacker thinks that the target is likely to strike first within
Using the COW list offers two
60 days.(19) Attacks occurring after war has broken out cannot be considered preemptive.
important advantages. First, it is a comprehensively inclusive data set for the modern era. By
looking at all wars in the modern era rather than just a small sample, we can have more
confidence in the validity and generalizability of the empirical findings. Second, the COW list was
not drawn up for the purposes of assessing the frequency of preemption. This decreases the
chances of sampling bias, as the cases in the COW data set could not have been selected to
encourage a particular conclusion about the frequency of preemption. The results are surprising:
preemptive wars almost never happen. Out of 67 wars in the COW list, only 3 (4.5 percent) are
preemptive: World War I (specifically, Russo-German interactions in July 1914), the Chinese intervention in the Korean War in
1950, and the Israeli attack on Egypt in 1967, which began the Six Day War. As a first cut, this finding supports the third, most
skeptical view of the role preemption plays in international politics. It is worth noting that though a number of scholars have made
empirical observations roughly in agreement with my coding,(20) at least one work comes to different conclusions.(21) Some might
question the accuracy of this coding, speculating that some wars in which preemption was an important motivation were left out by
declaring preemption to be not a "primary" motivation. However, the case discussions below demonstrate that even in these three
cases, which are certainly the most preemptive of this set of wars, it is arguable that preemption was not the primary motive. In
each, there were other important factors that led to war, and for each case scholars have argued that fears of an imminent attack
were not necessary for war. Additionally ,
one could argue that three cases of preemption is actually an
inflated estimate, as the attacks in World War I and the Korean War that I coded as preemptive
were examples of nations joining ongoing wars rather than initiations of war, so neither case
should be coded as an instance of preemption . In 1914, the German preemption against Russia escalated the
already begun Austro-Serb conflict, and in 1950 China intervened on the side of North Korea in its ongoing war with South Korea and
the United States. By this more restrictive definition, the correct count would be one case of preemption rather than three. In sum,
we can be quite confident that the count of three cases of preemption is not an underestimation. A criticism of assessing the
frequency of preemptive wars by looking only at wars themselves is that this misses the non-events, that is, instances in which
preemption would be predicted but did not occur. However, excluding non-events should bias the results in favor of finding that
preemptive war is an important path to war, as the inclusion of non-events could only make it seem that the event was less
frequent. Therefore, if preemptive wars seem infrequent within the set of wars alone, then this would have to be considered strong
evidence in favor of the third, most skeptical view of preemptive war, because even when the sample is rigged to make preemptive
wars seem frequent (by including only wars), they are still rare events. Below, a few cases in which preemption did not occur are
discussed to illustrate factors that constrain preemption. The rarity of preemptive wars offers preliminary support for the third,
most skeptical view, that the preemption scenario does not tell us much about how war breaks out. Closer examination of the three
cases of preemption, set forth below, casts doubt on the validity of the two preemption hypotheses discussed earlier: that hostile
images of the enemy increase the chances of preemption, and that belief in the dominance of the offense increases the chances of
preemption. In
each case there are motives for war aside from fear of an imminent attack,
indicating that such fears may not be sufficient to cause war. In addition, in these cases of war the
two conditions hypothesized to stimulate preemption -- hostile images of the adversary and
belief in the military advantages of striking first -- are present to a very high degree. This implies
that these are insubstantial causal forces, as they are associated with the outbreak of war only
when they are present to a very high degree. This reduces even further the significance of these
forces as causes of war. To illustrate this point, consider an analogy: say there is a hypothesis that saccharin causes cancer.
Discovering that rats who were fed a lot of saccharin and also received high levels of X-ray exposure, which we know causes cancer,
had a higher risk for cancer does not, however, set off alarm bells about the risks of saccharin. Though there might be a relationship
between saccharin consumption and cancer, this is not demonstrated by the results of such a test.
---Accidents- AT: Technical Fail

No technical failure- PALS

Slocombe ‘9 (De-Alerting: Diagnoses, Prescriptions, and Side-Effects* By The Honorable Walter B. Slocombe Discussion paper
presented at the seminar on “Re-framing De-Alert: Decreasing the Operational Readiness of Nuclear Weapons Systems in the U.S.-
Russia Context” in Yverdon, Switzerland, 21-23 June 2009. Caplin & Drysdale Attorneys; Former Under Secretary of Defense for
Policy

Let’s start with Technical Failure – the focus of a great deal of the advocacy, or at least of stress on past incidents of failures of safety
and control mechanisms.4 Much of the “de-alerting” literature points to a succession of failures to follow proper procedures and
draw from that history the inference that a relatively simple procedural failure could produce a nuclear detonation. The argument is
essentially that nuclear weapons systems are sufficiently susceptible of pure accident (including human error or failure at
operational/field level) that it is essential to take measures that have the effect of making it necessary to undertake a prolonged
reconfiguration of the elements of the nuclear weapons force for a launch or detonation to be physically possible. Specific measures
said to serve this objective include separating the weapons from their launchers, burying silo doors, removal of fuzing or launching
mechanisms, deliberate avoidance of maintenance measures need to permit rapid firing, and the like. . My view is that this line of
action is unnecessary in its own terms and highly problematic from the point of view of other aspects of the problem and that there
is a far better option that is largely already in place, at least in the US force – the requirement of external information – a code not
held by the operators -- to arm the weapons. Advocates of other, more “physical,” measures often describe the current
arrangement as nuclear weapons being on a “hair trigger.” That is – at least with respect to US weapons – a highly
misleading characterization. The “hair trigger” figure of speech confuses “alert” status – readiness to act quickly on orders
-- with susceptibility to inadvertent action. The “hair trigger” image implies that a minor mistake – akin to jostling a gun – will fire the
weapon. TheUS StratCom commander had a more accurate metaphor when he recently said
that US nuclear weapons are less a pistol with a hair trigger than like a pistol in a holster with the safety
turned on – and he might have added that in the case of nuclear weapons the “safety” is locked in place by a
combination lock that can only be opened and firing made possible if the soldier carrying the pistol receives a message from his
chain of command giving him the combination. Whatever other problems the current nuclear posture of the US nuclear force may
present, it cannot reasonably be said to be on a “hair trigger.” Since
the 1960s the US has taken a series of
measures to insure that US nuclear weapons cannot be detonated without the receipt of both
external information and properly authenticated authorization to use that information. These
devices – generically Permissive Actiƒon Links or “PALs” – are in effect combination locks that keep the weapons
locked and incapable of detonation unless and until the weapons’ firing mechanisms have been
unlocked following receipt of a series of numbers communicated to the operators from higher
authority. Equally important in the context of a military organization, launch of nuclear weapons (including insertion of the
combinations) is permitted only where properly authorized by an authenticated order. This combination of reliance on
discipline and procedure and on receipt of an unlocking code not held by the military personnel in charge of the launch
operation is designed to insure that the system is “fail safe ,” i.e., that whatever mistakes occur, the
result will not be a nuclear explosion.

No accidental launch

Williscroft ‘10 (Six patrols on the John Marshall as a Sonar Technician, and four on the Von Steuben as an officer – a total of
twenty-two submerged months. Navigator and Ops Officer on Ortolan & Pigeon – Submarine Rescue & Saturation Diving ships.
Watch and Diving Officer on Oceanographer and Surveyor. “Accidental Nuclear War” http://www.argee.net/Thrawn
%20Rickle/Thrawn%20Rickle%2032.htm)

Isthere a realistic chance that we could have a nuclear war by accident? Could a ballistic
submarine commander launch his missiles without specific presidential authorization? Could a
few men conspire and successfully bypass built-in safety systems to launch nuclear weapons ?
The key word here is “realistic.” In the strictest sense, yes, these things are possible. But are they
realistically possible? This question can best be answered by examining two interrelated questions. Is there a way to launch
a nuclear weapon by accident? Can a specific accidental series of events take place—no matter how remote—that will result in the
inevitable launch or detonation of a nuclear weapon? Can one individual working by himself or several individuals working in
collusion bring about the deliberate launch or detonation of a nuclear weapon? We are protected from accidental launching of
nuclear weapons by mechanical safeguards, and by carefully structured and controlled mandatory procedures that are always
employed when working around nuclear weapons. Launching a nuclear weapon takes the specific
simultaneous action of several designated individuals. System designers ensured that
conditions necessary for a launch could not happen accidentally . For example, to launch a missile
from a ballistic missile submarine, two individuals must insert keys into separate slots on
separate decks within a few seconds of each other. Barring this, the system cannot physically
launch a missile. There are additional safeguards built into the system that control computer
hardware and software, and personnel controls that we will discuss later, but—in the final analysis—without the keys
inserted as described, there can be no launch—it’s not physically possible. Because the time window for key
insertion is less than that required for one individual to accomplish , it is physically impossible for
a missile to be launched accidentally by one individual. Any launch must be deliberate . One can postulate a
scenario wherein a technician bypasses these safeguards in order to effect a launch by himself. Technically, this is possible, but such
a launch would be deliberate, not accidental. We will examine measures designed to prevent this in a later column.
Maintenance procedures on nuclear weapons are very tightly controlled. In effect always is
the “two-man rule.” This rule prohibits any individual from accessing nuclear weapons or their
launch vehicles alone. Aside from obvious qualification requirements, two individuals must be present. No matter
how familiar the two technicians may be with a specific system, each step in a maintenance
procedure is first read by one technician, repeated by the second, acknowledged by the first (or
corrected, if necessary), performed by the second, examined by the first, checked off by the first, and
acknowledged by the second. This makes maintenance slow, but absolutely assures that no errors happen. Exactly the
same procedure is followed every time an access cover is removed, a screw is turned, a weapon is moved, or a controlling
publication is updated. Nothing, absolutely nothing is done without following the written guides exactly , always
under two-man control. This even applies to guards. Where nuclear weapons are concerned, a minimum of two guards—always fully
in sight of each other—stand duty. There is no realistic scenario wherein a nuclear missile can be accidentally
launched...ever...under any circumstances...period !
---Accidents- AT: China/Russia

No risk of Chinese or Russian accidents

-intelligence reports prove

Rudney and Stanley ‘2K (Robert and Willis, Dealerting proposals for strategic nuclear forces: A critical analysis,
Comparative Strategy, 1521-0448, Volume 19, Issue 1, 2000)

Senior U.S. officials downplay allegations by Blair and other dealerting advocates that there is a significant risk of accidental or
unauthorized launch of missiles by the Russians. In a December 1998 speech, Robert D. Walpole, National Intelligence Officer for
the CIA's annual report to Congress on foreign missile developments,
Strategic and Nuclear Programs, summarized
which found that "an unauthorized or accidental launch of a Russian or Chinese strategic missile is
highly unlikely, as long as current security procedures and systems are in place. Russia employs an extensive array
of technical and procedural safeguards and China keeps its missiles unfueled and without
warheads mated" [ 110]. In communications to Congress in 1998, both the CIA and the Defense Intelligence Agency
(DIA) asserted that the possibility of an unauthorized launch Russian strategic weapons was very low
because of the many safeguards built into the system [ 111 ].
Aerospace Industry Answers
Frontline

Aerospace is resilient

Farrow 11 – degree in journalism from the University of Toronto, Past-Chair of The Writers' Union of Canada, Writer-in-
Residence at the University of Alberta, (John, October 12, 2011, “The aerospace sector is more resilient than most others”, Derby
Telegraph, Lexis)

With the aerospace industry making a steady climb out of the economic turmoil, things are set for Pattonair to increase its global
footprint in the aerospace and defence market. The firm's general manager John Farrow is quietly confident the company is in the
right industry at the right time. IT
may be economic doom and gloom elsewhere but as far as the aerospace
industry is concerned the future looks brighter than ever. Pattonair's growing customer base has a large order book.
Sales of new passenger aircraft, such as the Boeing 787 Dreamliner and the Airbus A350 XWB are guaranteeing work for years to
come. This means that demand for Pattonair's services, sourcing and delivering components, will hopefully ramp up a
number of notches. The launch customer for the long-awaited Dreamliner, All Nippon Airways, has just received the first aircraft.
And a number of other airlines will start receiving theirs. Meanwhile, development work on the Airbus A350 XWB is carrying on
apace. Advanced orders for the aircraft are mounting up. It is all good news for Pattonair and as orders grow, the firm will be aiming
to grow with it. At present, Pattonair provides approximately 100 million parts per year to over 1,000 customers across the world.
But with the aerospace sector on an upward trajectory, these numbers could increase. John Farrow, Pattonair's general
manager, said: "Many industries go through peaks and troughs. "One of the most obvious examples being the automotive sector,
which seems to lurch from major downturn to exciting new models and a scramble for extra capacity in the space of very few years -
take Jaguar as a great example. "However, in
the view of many economic experts, the aerospace industry
is far more resilient than most. "Yes, major effects such as 9/11 had far-reaching consequences for the industry, but in
the main, the highs and lows are far more smoothed out.

(See air power answers)


Afghanistan Answers
Frontline

Can’t solve Afghanistan

Walt ‘3-15
(Stephen, Robert and Renée Belfer professor of international affairs at Harvard University's Kennedy School of Government, where
he served as academic dean from 2002-2006. “The REAL reason the U.S. failed in Afghanistan.” Foreign Policy.)

Both Nasr and Chayes make useful points about the dysfunction that undermined the AfPak effort, and I'm not going to try to
adjudicate between them. Rather, I think both of them miss the more fundamental contradiction that bedeviled the entire
U.S./NATO effort, especially after the diversion to Iraq allowed the Taliban to re-emerge. The
key problem was essentially
structural: US. objectives in Afghanistan could not be achieved without a much larger commitment of
resources, but the stakes there simply weren't worth that level of commitment. In other words, winning wasn't worth
the effort it would have taken, and the real failure was not to recognize that fact much earlier and to draw the appropriate policy
conclusions. First, achieving a meaningful victory in Afghanistan -- defined as defeating the Taliban and creating an effective,
Western-style government in Kabul -- would have required sending far more troops (i.e., even more than the Army requested during
the "surge"). Troop levels in Afghanistan never approached the ratio of troops/population observed in more
successful instances of nation-building, and that deficiency was compounded by Afghanistan's ethnic
divisions, mountainous terrain, geographic isolation, poor infrastructure, and porous borders.
Second, victory was elusive because Pakistan continued to support the Taliban, and its territory provided
them with effective sanctuaries. When pressed, they could always slip across the border and live to fight another day. But
Washington was never willing to go the mattresses and force Pakistan to halt its support, and it is not
even clear that we could have done that without going to war with Pakistan itself. Washington backed off for very good reasons: We
wanted tacit Pakistani cooperation in our not-so-secret drone and special forces campaign against al Qaeda, and we also worried
about regime stability given Pakistan's nuclear arsenal. Unfortunately, these factors made victory even harder to achieve. Third, we
couldn't get Karzai to reform because he was the only game in town, and he knew it. Unless the U.S. and NATO were
willing to take over the whole country and try to govern it ourselves -- a task that would have made occupying Iraq seem easy -- we
counterinsurgencies require effective and legitimate
were forced to work with him despite his many flaws. Successful
local partners, however, and we never had one. In short, the U.S. was destined to lose because it didn't go
all-out to win, and it shouldn't have. Indeed, an all-out effort would have been a huge mistake, because the stakes were in fact
rather modest. Once the Taliban had been ousted and al Qaeda had been scattered, America's main interest was continuing to
degrade al Qaeda (as we have done). That mission was distinct from the attempt to nation-build in Afghanistan, and in the end
Afghanistan's importance did not justify a substantially larger effort. By the way, I am not suggesting that individual commanders
and soldiers did not make enormous personal sacrifices or try hard to win, or that the civilians assigned to the Afghan campaign did
not do their best in difficult conditions. My point is that if this war had been a real strategic priority, we would have fought it very
differently. We would not have rotated commanders, soldiers, and civilian personnel in and out of the theatre as often as we did, in
effect destroying institutional memory on an annual basis and forcing everyone to learn on the job. In a war where vital interests
were at stake, we certainly wouldn't have let some of our NATO partners exempt the troops they sent from combat. And if the war
had been seen aa a major priority, both parties would have been willing to raise taxes to pay for it. Thus, the real failure in
Afghanistan was much broader than the internal squabbles that Nasr and Chayes have addressed. The entire national security
establishment failed to recognize or acknowledge the fundamental mismatch between 1) U.S. interests (which were limited), 2) our
stated goals (which were quite ambitious), and 3) the vast resources and patience it would have required to achieve those goals.
Winning would have required us to spend much more than winning was worth, and to undertake exceedingly risky and uncertain
actions towards countries like Pakistan. U.S. leaders wisely chose not to do these things, but they failed to realize what this meant
for the war effort itself. Given this mismatch between interests, goals, and resources, it was stupid to keep trying to win at a level of
effort that was never going to succeed. Yet no one on the inside seems to have pointed this out, or if they did, their advice was not
heeded. And that is the real reason why the war limped on for so long and to such an unsatisfying end.

No Afghan impact
Silverman ‘9 - PhD in international relations-government and, as a Ford Foundation Project Specialist (11/19/09, Jerry Mark,
The National Interest, “Sturdy Dominoes,” http://www.nationalinterest.org/Article.aspx?id=22512)

Many advocates of continuing or racheting up our presence in Afghanistan are cut from the same domino-theory cloth as those of
the Vietnam era. They posit that losing in Afghanistan would almost certainly lead to the further "loss" of the entire South and
central Asian region. Although avoiding explicit reference to "falling dominos," recent examples include S. Frederick Starr [3] (School
of Advanced International Studies, Johns Hopkins University); Sir David Richards [4] (the UK's relatively new Chief of the General
Staff); and, in The National Interest, Ahmed Rashid [5]. The fear that Pakistan and central Asian governments are too weak to
withstand the Taliban leads logically to the proposition-just as it did forty years ago-that only the United States can defend the
region from its own extremist groups and, therefore, that any loss of faith in America will result in a net gain for pan-Islamist
movements in a zero-sum global competition for power. Unfortunately, the resurrection of "falling dominos" as a metaphor for
predicted consequences of an American military withdrawal reflects a profound inability to re-envision the nature of today's global
political environment and America's place in it. The current worry is that Pakistan will revive support for the Taliban [6] and return to
its historically rooted policy of noninterference in local governance or security arrangements along the frontier. This fear is
compounded by a vision of radical Islamists gaining access to Pakistan's nuclear arsenal. Those concerns are fueled by the judgment
that Pakistan's new democratically elected civilian government is too weak to withstand pressures by its most senior military officers
to keep its pro-Afghan Taliban option open. From that perspective, any sign of American "dithering" would reinforce that
historically-rooted preference, even as the imperative would remain to separate the Pakistani-Taliban from the Afghan insurgents.
Further, any significant increase in terrorist violence, especially within major Pakistani urban centers, would likely lead to the
imposition of martial law and return to an authoritarian military regime, weakening American influence even further. At its most
extreme, that scenario ends with the most frightening outcome of all-the overthrow of relatively secular senior Pakistani generals by
a pro-Islamist and anti-Western group of second-tier officers with access to that country's nuclear weapons. Beyond Pakistan,
advocates of today's domino theory point to the Taliban's links to both the Islamic Movement of Uzbekistan and the Islamic Jihad
Union, and conclude that a Taliban victory in Afghanistan would encourage similar radical Islamist movements in Kyrgyzstan,
Tajikistan, Turkmenistan and Uzbekistan. In the face of a scenario of increasing radicalization along Russia's relatively new, southern
borders, domino theorists argue that a NATO retreat from Afghanistan would spur the projection of its own military and political
power into the resulting "vacuum" there. The primary problem with the worst-case scenarios predicted by the domino theorists is
that no analyst is really prescient enough to accurately predict how decisions made by the United States today will affect future
outcomes in the South and central Asian region. Their forecasts might occur whether or not the United States withdraws or,
alternatively, increases its forces in Afghanistan. Worse, it is entirely possible that the most dreaded consequences will occur only as
the result of a decision to stay. With the benefit of hindsight, we know that the earlier domino theory falsely represented interstate
and domestic political realities throughout most of Southeast Asia in 1975. Although it is true that American influence throughout
much of Southeast Asia suffered for a few years following Communist victories in Cambodia, Laos and Vietnam, we now know that
while we viewed the Vietnam War as part of a larger conflict, our opponent's focus was limited to the unification of their own
country. Although border disputes erupted between Vietnam and Cambodia, China and the Philippines, actual military conflicts
occurred only between the supposedly fraternal Communist governments of Vietnam, China and Cambodia. Neither of the two
Further, no serious threats to install Communist regimes
competing Communist regimes in Cambodia survived.
were initiated outside of Indochina, and, most importantly, the current political situation in Southeast Asia now conforms
closely to what Washington had hoped to achieve in the first place [7]. It is, of course, unfortunate that the transition from military
conflict in Vietnam to the welcome situation in Southeast Asia today was initially violent, messy, bloody, and fraught with revenge
and violations of human rights. But as the perpetrators, magnitude, and victims of violence changed, the level of violence eventually
declined. This time around, there are at least two questionable assumptions underlying the resurrection of the domino theory. First,
the Taliban is no longer the unified group that emerged during 1994. Instead, the term "Taliban" is applied to several
groups engaged in the current insurgency against the Karzai government and NATO forces. Those groups collaborate through a
complex set of shifting alliances that extend across the disputed Afghanistan/Pakistan border. Second, given that local Taliban have
demonstrated their capacity to effectively engage NATO forces without the equivalent of NATO military and civilian trainers or
logistical support,
other indigenous groups opposed to the Taliban and/or al-Qaeda are also likely to
be stronger than domino theorists assume and are likely to proactively defend themselves
against radical Islamists once we are no longer there to do it for them. A retrospective view of America's involvement in Vietnam and
its ultimate consequences for U.S. interests reinforces the aphorism that all politics are local. That
truism seems lost on
American foreign-policy decision makers who tend to see international threats in global rather
than local terms. Further, the danger remains that the metaphor of falling dominos might resonate with governments in the
region that face their own increasingly radical domestic opposition. Our fears of regional collapse might also speak to Russian and
regional threats, even if they do
Chinese policy makers fearful of potentially greater instability along their borders. But such
arise, do not threaten the core national interests of the United States-the substantially exaggerated fears of
terrorist "safe-havens" notwithstanding. Those worries simply do not justify the overwhelmingly disproportionate and
financially ruinous military response that has characterized our involvement there. The "fall of dominos" is no more inevitable in
South and central Asia now than it was in Southeast Asia more than a half century ago. True, the earlier circumstances in Vietnam
and Southeast Asia are not, in most respects, similar to the current situation in Afghanistan, Pakistan, or the remainder of South and
central Asia. Nonetheless, the emphasis in both cases on external interstate threats-rather than on autonomous non-state actors-
has been a mistake because it does not reflect the actual source of most violent conflicts since the 1960s .
In an exponentially
complex world characterized by multiple actors, the domino theory does not help predict the
future course of political relations in the region-nor would any other simplistic metaphor. Despite the view that the alliance
between various Taliban and al-Qaeda factions is both strategic and long-term, a consensus is forming that most Taliban groups are
either nationalists who want to seize formal authority within recognized sovereign-states, or more localized groups that merely want
to be left alone by any pretenders to centralized state-authority. Perversely, the desire of nationalist Taliban to seize sovereign-state
power represents an acceptance of a largely secular European system of interstate relations. In that conversion will likely be found
the seeds of their eventual undoing-as local community-based groups continue to oppose any attempts, whether sponsored by
Americans or Islamic radicals, to establish centralized state authority there.

Afghan stability is impossible

NYT ‘12 ["Peace talks with the taliban" -- www.nytimes.com/2012/10/05/opinion/peace-talks-with-the-taliban.html?_r=0]

American military commanders long ago concluded that the Afghan war could only end in a negotiated settlement with the Taliban,
not a military victory. But now the generals and civilian officials say even this hope is unrealistic before 2015 —
after American and coalition troops are withdrawn. They are, instead, trying to set the stage for eventual peace talks between the
Afghan government and the insurgency sometime after their departure.¶ President Obama’s failure to make headway
in talks with the Taliban is a serious setback . Of course, persuading militants to negotiate a peace deal was always
a daunting challenge. But the Obama administration has not been persistent enough in figuring out how to initiate talks with a
resilient, brutal insurgency that continues to carry out deadly attacks against American and NATO forces.¶
During the 2010 surge, when the United States added 33,000 troops to the 68,000 in Afghanistan and put maximum military
pressure on the Taliban, the administration was conflicted and too cautious about pressing for talks. Top generals resisted
negotiations, saying the focus should be on military gains. Even after the administration decided in February 2011 to pursue talks, it
took officials months to agree on the details of their approach.¶ The
talks between the United States and the Taliban began early
this year but soon collapsed when the administration , faced with bipartisan opposition in Congress, could not
complete a proposed prisoner swap. The Taliban wanted five of their leaders released from Guantánamo Bay, Cuba,
in exchange for the sole American held by the insurgents, Sgt. Bowe Bergdahl. The risky deal was supposed to be a confidence-
building mechanism to encourage more serious talks. But its collapse has made talks even harder.¶ The Taliban
are internally divided and unwilling to meet Washington’s demands to sever all ties to Al Qaeda,
renounce violence and accept the commitments to political and human rights in Afghanistan’s Constitution. Pakistan has long
played a destructive role, enabling Taliban groups and refusing to support negotiations . Even a
more basic outreach to the Taliban — the so-called reintegration program that seeks to get lower-level fighters to lay down their
arms — has enticed only 5,000 of an estimated 20,000 to 30,000 insurgents off the battlefield.
---Afghanistan- Can’t Solve

Stability efforts will fail- too many variables are required

Wadhams ‘13

[Caroline Wadhams is a Senior Fellow at American Progress. She focuses on U.S. national security, Afghanistan, Pakistan, and
terrorism.¶ Prior to American Progress, she served as a legislative assistant on foreign policy issues for Sen. Russ Feingold (D-WI).
Wadhams also worked at the Council on Foreign Relations in Washington, D.C., as an assistant director and in New York as a
research associate on national security issues, “Managing Afghanistan’s Political Transition Between Now and 2014,”
http://www.americanprogress.org/issues/security/news/2013/01/07/49079/managing-the-political-transition-between-now-and-
2014/]

Beyond 2014 a small reserve force of less than 10,000 U.S. and international military personnel may be necessary to protect
international diplomatic staff, conduct isolated counterterrorism operations if necessary, and offer some continued training
assistance and enabling capabilities to the Afghan National Security Forces. This reserve force should also be considered within the
context of a negotiation process, as one way to bolster the Afghan government’s bargaining hand in discussions with insurgents.
Ultimately, the reserve force could be removed entirely if a larger political agreement within Afghanistan and in the region can
provide sufficient guarantees of Afghan stability and a protection of U.S. counterterrorism security interests. A limited, longer-term
financial commitment to the Afghan government and security services—with conditions—beyond 2014 will also most likely be
required.¶ But these military
aspects—troop numbers, training of the Afghan forces, and financial support to the security
services—won’t be enough to ensure Afghanistan’s security and stability over the long term.
Leaving behind an unprepared and expensive force to battle an insurgency that NATO has
struggled to contain is more likely to create instability than lasting security. Instead, our efforts must be
synchronized with broader efforts at conflict resolution . In other words, the drivers of violence—
mainly predatory and exclusive governance and regional malfeasance—must be reduced in
order for the Afghan forces to be able to better secure the country. Reducing these drivers will
require a new political consensus that has greater buy-in from key Afghan actors, Pakistan, and the broader region.

Rapid withdrawal makes Afghan instability inevitable and proves the plan isn’t fast enough

Lignet ‘13 [Langley Intelligence Group Network, 1-14-13 “Afghanistan: Obama’s New Exit Plan Jeopardizes Stability,”
http://www.lignet.com/ArticleAnalysis/Afghanistan--Obama-s-Quick-U-S--Exit-Plan-Jeopardi]

President Barack Obama’s surprise announcement that the U nited States will draw down its forces in
Afghanistan in a matter of months — rather than in 2014 as expected — seriously jeopardizes
Afghan stability. The decision, made public last Friday following a meeting with Afghan President Hamid Karzai
at the White House, imperils U.S. efforts to adequately train Afghan security forces, conduct
effective counterterrorism operations and deter future Taliban attacks. Obama announced that the
United States would speed up the planned 2014 withdrawal of troops from Afghanistan and transition security responsibility to
Afghan forces, despite remaining Taliban strongholds across the country. Left uncertain was the number of U.S. troops that will
remain in Afghanistan after 2014.
The plan doesn’t solve Taliban reconciliation- means they can’t solve stability

Gharekhan and Inderfurth ‘12 [Chinmaya R. Gharekhan served as India’s special envoy for West Asia and is a former U.N.
under secretary general. Karl F. Inderfurth served as U.S. assistant secretary of state for South Asian affairs in 1997-2001 and is a
senior advisor at the Center for Strategic and International Studies in Washington, “An Umbrella for Afghan Stability,” June 29, 2012,
http://www.nytimes.com/2012/06/30/opinion/an-umbrella-for-afghan-stability.html?_r=0]

While the focus is on the preparedness of the security forces ,


less is being heard about the other, equally
important pillar of the transition process: reconciliation. One does not know who, if anyone, is talking
to the Taliban.¶ The so-called office for the Taliban in Doha is not known to be open for business .
The assassination of former Afghan president Burhanuddin Rabbani has eroded whatever effectiveness the High Peace Council,
appointed by President Hamid Karzai to negotiate with the Taliban, had to conduct talks.¶ The
Taliban laid down certain
conditions before they would agree to serious talks, one of which was the transfer of five
detainees from Guantánamo to Qatar or somewhere close by. The United States has not complied with the
demand. The Taliban have also refined their public diplomacy over the years and can be expected to play hard or soft, as required.
They have not given up on their ambition to set up an Islamic emirate in Afghanistan.¶ It is imperative that the two tracks — getting
the Afghan national security forces up to speed and political reconciliation — make significant and parallel progress before 2014.
Though U.S. forces will remain in significant numbers beyond 2014, they will be there in a training capacity, and it is not clear
whether they would intervene to help Afghan forces.
---Afghanistan- No Impact

Instability is inevitable but won’t escalate

Finel ‘9
[Dr. Bernard I. Finel, an Atlantic Council contributing editor, is a senior fellow at the American Security Project, “Afghanistan is
Irrelevant,” Apr 27 http://www.acus.org/new_atlanticist/afghanistan-irrelevant]

It is now a deeply entrenched conventional wisdom that the decision to “abandon” Afghanistan after the Cold War was a tragic
war, state collapse, the rise of the Taliban, and inevitably terrorist
mistake. In the oft-told story, our “abandonment” led to civil
attacks on American soil. This narrative is now reinforced by dire warnings about the risks to Pakistan from
instability in Afghanistan. Taken all together, critics of the Afghan commitment now find themselves facing a nearly
unshakable consensus in continuing and deepen our involvement in Afghanistan. The problem with the consensus is that virtually
every part of it is wrong. Abandonment did not cause the collapse of the state. Failed states are not always a
threat to U.S. national security. And Pakistan’s problems have little to do with the situation across the border. First, the
collapse of the Afghan state after the Soviet withdrawal had little to do with Western abandonment. Afghanistan has
always been beset by powerful centrifugal forces. The country is poor, the terrain rough, the population divided into
several ethnic groups. Because of this, the country has rarely been unified even nominally and has never really had a strong central
government. The dominant historical political system in Afghan is warlordism. This is not a consequence of Western involvement or
lack thereof. It is a function of geography, economics, and demography. Second, there is no straight-line between state
failure and threats to the United States. Indeed, the problem with Afghanistan was not that it failed but rather that it “unfailed” and
becameruled by the Taliban.Congo/Zaire is a failed state. Somalia is a failed state. There are many parts of the
globe that are essentially ungoverned. Clearly criminality, human rights abuses, and other global ills flourish in these
spaces. But the notion that any and all ungoverned space represents a core national security threat
to the United States is simply unsustainable. Third, the problem was the Taliban regime was not that it existed. It was that
it was allowed to fester without any significant response or intervention. We largely sought to ignore the regime — refusing to
recognize it despite its control of 90% of Afghan territory. Aside from occasional tut-tutting about human rights violations and
destruction of cultural sites, the only real interaction the United States sought with the regime was in trying to control drugs.
Counter-drug initiatives are not a sound foundation for a productive relationship for reasons too numerous to enumerate here. Had
we recognized the Taliban and sought to engage the regime, it is possible that we could have managed to communicate red lines to
them over a period of years. Their failure to turn over bin Laden immediately after 9/11 does not necessarily imply an absolute
inability to drive a wedge between the Taliban and al Qaeda over time. Fourth, we are now told that defeating the Taliban in
Afghanistan is imperative in order to help stabilize Pakistan. But, most observers seem to think that Pakistan is in worse shape now
— with the Taliban out of power and American forces in Afghanistan — than it was when the Taliban was dominant in Afghanistan.
For five years from 1996 to 2001, the Taliban ruled Afghanistan and the Islamist threat to Pakistan then was unquestionably lower.
This is not surprising actually. Insurgencies are at their most dangerous — in terms of threat of contagion — when they are fighting
for power. The
number of insurgencies that actually manage to sponsor insurgencies elsewhere
after taking power is surprising low. The domino theory is as dubious in the case of Islamist
movements as it was in the case of Communist expansion . There is a notion that “everything changed on
9/11.” We are backing away as a nation from that concept in the case of torture. Perhaps we should also come to realize that our
pre-9/11 assessment of the strategic value and importance of Afghanistan was closer to the mark that our current obsession with it.
We clearly made some mistakes in dealing with the Taliban regime. But addressing those mistakes through better intelligence, use of
special forces raids, and, yes, diplomacy is likely a better solution than trying to build and sustain a reliable, pro-Western
government in Kabul with control over the entire country.

Regional cooperation deescalates conflict


Innocent ‘9
(Malou, foreign policy – Cato, http://www.cato.org/pubs/wtpapers/escaping-graveyard-empires-strategy-exit-afghanistan.pdf)

Additionally, regional stakeholders, especially Russia and Iran, have an interest in a stable Afghanistan. Both
countries possess the capacity to facilitate development in the country and may even be willing to assist Western
forces. In July, leaders in Moscow allowed the United States to use Russian airspace to transport troops and lethal military
equipment into Afghanistan. Yet another relevant regional player is the Collective Security Treaty Organization, made up of Russia,
Kazakhstan, Tajikistan, Kyrgyzstan, Uzbekistan, Armenia, and Belarus. At the moment, CSTO
appears amenable to
forging a security partnership with NATO. CSTO secretary general Nikolai Bordyuzha told journalists in March 2009
of his bloc’s intention to cooperate. “The united position of the CSTO is that we should give every kind of aid to the anti-terror
coalition operating in Afghanistan. . . . The interests of NATO and the CSTO countries regarding Afghanistan conform
unequivocally.”83 Mutual interests between Western forces and Afghanistan’s surrounding neighbors
can converge on issues of transnational terrorism, the Caspian and Central Asia region’s abundant energy
resources, cross-border organized crime, and weapons smuggling. Enhanced cooperation alone will not stabilize Afghanistan,
but engaging stakeholders may lead to tighter regional security .

Afghanistan will never fully collapse

Kaplan 10—snr fellow, Center for New American Security. Frmr distinguished prof in nat. sec., US Naval Academy (Robert, Man
Versus Afghanistan, 11 March 2010, http://www.cnas.org/node/4214)

Yet another reality points to an entirely different conclusion. The dispersal of Afghanistan’s larger population over greater territory
than Iraq’s is basically meaningless, British Army Major General Colin Boag told me: because 65 percent of the population lives
within 35 miles of the main road system, which approximates the old medieval caravan routes, only 80 out of 342 districts are really
key to military success. Afghanistan is not some barbaric back-of-beyond, but the heart of a cultural continuum connecting the
cosmopolitan centers of Persia and India. In fact, Afghanistan has been governed from the center since the
18th century: Kabul, if not always a point of authority, has been at least a point of arbitration. Especially between the early
1930s and the early 1970s, Afghanistan experienced moderate and constructive government under the constitutional monarchy of
Zahir Shah. A highway system on which it was safe to travel united the major cities, while estimable health and development
programs were on the verge of eradicating malaria. Toward the end of this period, I hitchhiked and rode buses across Afghanistan. I
never felt threatened, and I was able to send books and clothes back home through functioning post offices. There was, too, a strong
Afghan national identity distinct from that of Iran or Pakistan or the Soviet Union. Pashtunistan might be a real enough geographic
construct, but so, very definitely, is Afghanistan. As Ismail Akbar, a writer and analyst in Kabul, told me: “Thirty years of war and
even the
Pakistani interference have weakened Afghan national identity from the heights of the Zahir Shah period. But
mujahideen civil war of the early 1990s, in which the groups were split along ethnic lines, could not break up
Afghanistan. And if that couldn’t, nothing will.” Afghans were so desperate for a reunited
country after the internecine fighting of the mujahideen era that they welcomed the Taliban in Kandahar in 1994 and in Kabul in
1996, as a bulwark against anarchy and dissolution. Afghanistan, frail and battered over the years, is nevertheless
surprisingly sturdy as a concept and as a cynosure of identity.
---Afghanistan- AT: Pakistan Scenario

No Afghan impact on Pakistan

Royner ‘11
(Joshua, assistant professor of strategy and policy at the U.S. Naval War College. The National Interest, June 14, “Our Current
Strategy in Afghanistan Is Built on Strategic Myths.”

http://nationalinterest.org/blog/counterinsurgency/our-current-strategy-afghanistan-built-strategic-myths-5466)

Despite such claims, the war in Afghanistan has little to do with the issue of nuclear terrorism. Pakistani
militants,
emboldened or not, have shown nothing like the sophistication needed to forcibly remove nuclear
material from heavily guarded facilities. Recent suicide attacks on Pakistani military bases led to renewed
concerns about the security of the arsenal, but the attacks themselves did not show that the Pakistani nuclear complex is
vulnerable; they only showed that militants are willing to commit suicide. The fact that militants are aggressive does not mean
that they are smart, and we shouldn’t fear them any more than we would fear bank robbers who die at the scene and never get
away with any cash. Analysts of the Pakistani nuclear program believe a more realistic concern is insider-outsider collusion.
Growing radicalization and anti-American sentiment in the officer corps have led some observers to
reasonably worry that some workers may clandestinely share nuclear material and know-how with extremists. Militants need not
launch spectacular attacks in order to get their hands on nuclear weapons if they can reliably accumulate fissile material over time.
Pakistan’s control of its nuclear materials is undoubtedly a serious concern, and the United States should continue quiet efforts to
help it implement a recent series of personnel and procedural reforms meant to guarantee civilian oversight of the nuclear complex.
But this
problem will exist regardless of what happens in Afghanistan. U.S. officials should not continue to
justify the war in terms of the threat of nuclear terrorism. The issues are unrelated. Safe
havens and loose nukes are
the strategic bogeymen justifying the war in Afghanistan. Widely believed and rarely challenged in the
foreign policy community, these misguided assumptions have justified an impractical strategy with predictably frustrating results.
The death of bin Laden and the coming troop drawdown provide an important opportunity for strategic reassessment, and a chance
to overcome the enduring myths of the war.

Afghanistan doesn’t matter for Pakistan

Haass 10 (Richard N, President, Council on Foreign Relations, 12/20, “Let's Un-Surge in


Afghanistan”, http://www.cfr.org/publication/23669/lets_unsurge_in_afghanistan.html )

The second interest at stake is Pakistan . Some argue that we must stabilize Afghanistan lest it become a staging
ground for undermining its more important neighbor, one that hosts the world's most dangerous terrorists and possesses more than
100 nuclear weapons. This defies logic. Pakistan is providing sanctuary and support to the Afghan
Taliban who have not demonstrated an agenda to destabilize Pakistan. Why should we be more
worried than the Pakistanis themselves? Viewing Afghanistan as holding the key to Pakistan shows a
misunderstanding of Pakistan. It is, to be sure, a weak state. But the threats to it are mostly
internal and the result of deep divisions within the society and decades of poor governance. If
Pakistan ever fails, it will not be because of terrorists coming across its western border.
No Pakistan collapse

Bandow 9 – Senior Fellow @ Cato, former special assistant to Reagan (11/31/09, Doug, “Recognizing the Limits of American
Power in Afghanistan,” Huffington Post, http://www.cato.org/pub_display.php?pub_id=10924)

From Pakistan's perspective, limiting the war on almost any terms would be better than prosecuting it for years, even to "victory,"
whatever that would mean. In fact, the least likely outcome is a takeover by widely unpopular Pakistani
militants. The Pakistan military is the nation's strongest institution; while the army might not be able to rule alone,
it can prevent any other force from ruling. Indeed, Bennett Ramberg made the important point: " Pakistan, Iran and the former
Soviet republics to the north have demonstrated a brutal capacity to suppress political violence to ensure
survival. This suggests that even were Afghanistan to become a terrorist haven, the neighborhood
can adapt and resist." The results might not be pretty, but the region would not descend into chaos. In
contrast, warned Bacevich: "To risk the stability of that nuclear-armed state in the vain hope of salvaging Afghanistan would be a
terrible mistake."

No impact to Afghanistan- their impacts are inflated and incorrect

Friedman ‘9 (Hawks and Havens Posted by Benjamin H. Friedman Benjamin H. Friedman • October 6, 2009 @ 8:59 am
Filed under: Foreign Policy and National Security

The Washington Post’s oped page is a safe haven for hawks. Today we have Michael O’Hanlon and Richard Cohen fighting for the
war in Afghanistan. O’Hanlon is for generals respecting the president’s policy decisions, except when he isn’t — cases where the
general is obviously right, in that he agrees with O’Hanlon. (To me, this McChrystal incident shows the robustness of civilian control.
McChrystal spoke too freely and got rebuked. The Republic seems OK. So does the Army.) O’Hanlon’s other goal is to attack those
who want to limit the objectives in Afghanistan to counter-terrorism. To do so, he imputes his nation-building goals to the less
ambitious strategy. He says we tried the narrow mission under Bush and it failed. A. Not really. Does this, for example, sound like
counter-terrorism? B. It only failed to achieve the counterinsurgency strategy’s (maybe impossible) objectives of a stable,
centralized state in Afghanistan. A counter-terrorism (or go small) strategy sacrifices some probability of heightened stability for less
cost in blood and dollars. We have been doing fine at counter-terrorism all along, largely because al Qaeda is overrated. Afghanistan
is not a terrorist haven anymore. O’Hanlon also says that we won’t collect as much intelligence without a full-scale
counterinsurgency. Again, this is true, but insufficient. A smaller footprint provides benefits (less radicalization, less cost) that we
exchange, in a sense, for lost intelligence-gathering opportunities. In any case, intelligence needed to target airstrikes can come from
allies on the ground, intercepts, and overhead surveillance, as in Pakistan. Progress in surveillance and strike capability and the will
to use it means that a rerun of the 1990s, where al Qaeda was safe in Afghan camps, is a phony nightmare. O’Hanlon also claims that
absent a large U.S. ground force, we would have to offshore all UAV bases that range western Pakistan. This is a pessimistic
assessment; we could defend most of our airfields with a limited force in Afghanistan, and we have at least one UAV base in
Pakistan. Cohen calls Obama soft for letting McChrystal run amok, ignoring the fact that both the Secretary of Defense and the
National Security Adviser publicly rebuked him. Cohen approvingly cites Obama’s foolish claim that Afghanistan is a war of
necessity. One can’t say enough that this is senseless; even wars of pure self-defense aren’t strictly necessary, and Afghanistan, at
this point, isn’t that. He then drops the dominos.
Should we leave, he says, the Taliban will take over
Afghanistan and then Pakistan, grabbing nukes. India then invades Pakistan, and we get 1947, but nuclear. He
doesn’t say how the Taliban columns advancing on Kabul will suppress our airpower. The
widespread Afghan and Pakistani hostility to the Taliban — especially among the non-
Pashtuns who support and dominate both governments — doesn’t impress him. He doesn’t
mention the fact that the Pakistani military keeps close hold on its nukes, no matter who is
officially in power. One could go on, but suffice it to say that there is an equally plausible worst-case scenario that results
from following Cohen’s advice and expanding the war. To be fair though, Cohen is a clear-eyed realist compared to Daniel Twining,
who writes for Foreign Policy’s Shadow Government blog.
Twining sees the war in Afghanistan as a means to
keep Russia in a box, China down, India up, world trade humming, and the current international
order, whatever that is, intact. I’m not going to bother to explain how all this works, but I picture
the causal diagram as somewhat psychedelic. It’s almost like a parody of Jack Synder’ work on
imperial myths, like he missed the part of the story where it says these aren’t theories you
copy but BS people use to sell wars. For a relatively coherent version of the idea that we should fight in
Afghanistan to help Pakistan, read Steve Biddle. Biddle has two arguments that he thinks are one — that instability in Afghanistan
provide extremists a
will spread to Pakistan and that Taliban power in Afghanistan (not the same thing as instability) will
base to attack Pakistan’s government. One problem with the first claim is that historically
Afghanistan’s troubles have not destabilized Pakistan. The second argument struggles with the fact that we
likely cause insurrection among Pashtun Pakistanis by warring with their Afghan cousins. And, as Justin Logan and Matt Yglesias
note, the Taliban was in power for Afghanistan for years while Pakistan did OK, and many Pakistani elites want
it back.

No Pakistan Impact

AP 10 (“Pakistan's stability, leadership under spotlight after floods and double dealing accusations,” August 6th,
http://www.foxnews.com/world/2010/08/06/pakistans-stability-leadership-spotlight-floods-double-dealing-accusations/)

Not for the first time, Pakistan appears to be teetering on the edge with a government unable to
cope. Floods are ravaging a country at war with al-Qaida and the Taliban. Riots, slayings and arson are gripping the largest city.
Suggestions are flying that the intelligence agency is aiding Afghan insurgents. The crises raise questions about a nation crucial
to U.S. hopes of success in Afghanistan and to the global campaign against Islamist militancy. Despite
the recent
headlines, few here see Pakistan in danger of collapse or being overrun by militants — a fear
that had been expressed before the army fought back against insurgents advancing from their base in the Swat Valley early last
year. From its birth in 1947, Pakistan has been dogged by military coups , corrupt and inefficient
leaders, natural disasters, assassinations and civil unrest. Through it all, Pakistan has not prospered — but it
survives. “There is plenty to be worried about, but also indications that when push comes to shove the state
is able to respond," said Mosharraf Zaidi, an analyst and writer who has advised foreign governments on aid missions to
Pakistan. "The military has many weaknesses, but it has done a reasonable job in relief efforts. There have been
gaps in the response. But this is a developing a country, right?" The recent flooding came at a sensitive time for Pakistan, with
Western doubts over its loyalty heightened by the leaking of U.S. military documents that strengthened suspicions the security
establishment was supporting Afghan insurgents while receiving billions in Western aid. With few easy choices, the United
States has made it clear it intends to stick with Pakistan. Indeed, it has used the floods to demonstrate its commitment to the
country, rushing emergency assistance and dispatching helicopters to ferry the goods. The Pakistani government's response to
the floods has been sharply criticized at home, especially since President Asif Ali Zardari departed for a European tour. With so
many Pakistanis suffering, the trip has left the already weak and unpopular leader even more vulnerable politically. The flooding
was triggered by what meteorologists said were "once-in-a-century" rains. The worst affected area is the northwest, a
stronghold for Islamist militants. Parts of the northwest have seen army offensives over the last two years. Unless the people
are helped quickly and the region is rebuilt, anger at the government could translate into support for the militants. At least one
charity with suspected links to a militant outfit has established relief camps there. The extremism threat was highlighted by a
suicide bombing in the main northwestern town of Peshawar on Wednesday. The bomber killed the head of the Frontier
Constabulary, a paramilitary force in the northwest at the forefront of the terror fight. With authorities concentrating on flood
relief, some officials have expressed concern that militants could regroup. The city of Karachi has seen militant violence and is
rumored to be a hiding place for top Taliban and al-Qaida fighters. It has also been plagued by regular bouts of political and
ethnic bloodletting since the 1980s, though it has been calmer in recent years. The latest violence erupted after the
assassination of a leading member of the city's ruling party. More than 70 people have been killed in revenge attacks since then,
paralyzing parts of the city of 16 million people. While serious, the unrest does not yet pose an immediate
threat to the stability of the country. Although the U.S. is unpopular, there is little public support for
the hardline Islamist rule espoused by the Taliban and their allies . Their small movement has been
unable to control any Pakistani territory beyond the northwest, home to only about 20 million of the country's 175 million
people.
Africa War Answers
Frontline

Wars will stay small

Straus ’13 (Scott Straus for African Arguments, part of the Guardian Africa Network, Scott Straus is a professor in the
Department of Political Science at the University of Wisconsin, “Africa is becoming more peaceful, despite the war in Mali”,
http://www.guardian.co.uk/world/2013/jan/30/africa-peaceful-mali-war, January 30, 2013)

The bigger point is that we may be witnessing significant shifts in the nature of political violence on the
continent. Wars are on the decline since the 1990s, but the character of warfare is also changing. Today
there are fewer big wars fought for state control in which insurgents maintain substantial control of territory and put up
well-structured armies to fight their counterparts in the state – Mali not withstanding. Such wars were modal into the
1990s. From southern Africa in Angola, Mozambique, Namibia, and even Zimbabwe to the long wars in the Horn in Ethiopia,
Eritrea, and Sudan to the Great Lakes wars in Rwanda and Uganda, the typical armed conflict in Africa involved two major, territory-
holding armies fighting each other for state control. Today's
wars typically are smaller. They most often involve
small insurgencies of factionalised rebels on the peripheries of states . Today's wars also play out
differently. They exhibit cross-border dimensions, and rather than drawing funding from big external states they depend on illicit
trade, banditry, and international terrorist networks. Typical of today's wars are the rebels in Casamance, in
the Ogaden region of Ethiopia, various armed groups in Darfur, and the Lord's Resistance Army. The latter typifies an
emerging trend of trans-national insurgents. The LRA moves across multiple states in the Great Lakes region. Northern Mali is
another case in point – prior to seizing control of the north, the Islamists moved across multiple countries in the Sahel. Once they
gained territorial control in 2012, they attracted fighters from Nigeria and across North Africa. Moreover, these are not non-
ideological wars, as Gettleman claims. The jihadis in Mali and Somalia, the separatists in Casamance, and the rebels in Darfur are
certainly fighting for a cause.

Alt causes

Juma 2 [Monica, Project Coordinator for the International Peace Academy, September, “The Infrastructure of Peace in Africa,”
http://www.ciaonet.org/wps/jum01/jum01.pdf]

Since the end of the Cold War, Africa has been embroiled in a plethora of intra- and interstate conflicts . Most
of these conflicts have a transnational character and generate consequences that have implications for regions beyond those in
major causes of these conflicts are: the weak democratization process; deep-seated
which they occur. Among the
environmental problems; competition for resources; breakdown in the rule of law; and
proliferation of private armies, militias, and the attendant problem of illicit trade in, and use of, illegal arms. In
addition, the nature and dynamics of conflicts are shaped by the interplay of features peculiar to each subregion.

Africa won’t escalate

Barrett 5 [Robert, MA in Conflict Analysis and Management, Jun 1, “Understanding the Challenges of African Democratization
through Conflict Analysis,” http://papers.ssrn.com/sol3/papers.cfm?abstract_id=726162]

This is a problem, as Western nations may be increasingly wary of intervening in Africa hotspots after experiencing firsthand the
unpredictable and unforgiving nature of societal warfare in both Somalia and Rwanda. On a costbenefit basis, the West
continues to be somewhat reluctant to get to get involved in Africa’s dirty wars , evidenced by its political
hesitation when discussing ongoing sanguinary grassroots conflicts in Africa. Even as the world apologizes for bearing witness to the
Rwandan genocide without having intervened, the United States, recently using the label ‘genocide’ in the context of the Sudanese
conflict (in September of 2004), has only proclaimed sanctions against Sudan, while dismissing any suggestions at actual intervention
traditional military and diplomatic approaches at separating combatants
(Giry, 2005). Part of the problem is that
and enforcing ceasefires have
yielded little in Africa. No powerful nations want to get embroiled in
conflicts they cannot win – especially those conflicts in which the intervening nation has very
little interest.

Empirics prove this

Salisu ‘2 (Major Yushau A Salisu, April 18, 2002, “NIGERIA’S RISING HEGEMONY IS ESSENTIAL TO
PEACE AND STABILITY IN WEST-AFRICA,” http://stinet.dtic.mil/cgi-bin/GetTRDoc?
AD=ADA420533&Location=U2&doc=GetTRDoc.pdf, p. 2

The international community has of late shown lack of interest in intervening in conflicts within
sub-Saharan Africa in general and West Africa in particular as the Liberian, Sierra Leone and Burundi and
Rwandan civil wars proved. For instance the United Nation’s peacekeeping force in Africa which stood at about
40,000 in 1993 was just about 1600 as of June 19992. And during the Liberian conflicts, the then United Nation
(UN) Secretary General, Javier Perez de Cuellar categorically stated that the UN would not intervene. The
United States (U.S) which had about 2000 Marines off the Liberian coast also refused to intervene and separate the
warring faction

Don’t evaluate their Deutsch impact –

A. It has no logical warrant – no African nation has nukes and he’s predicting a scenario.

B. He’s an unqualified racist – calls Africans “rabid tigers.”


---Africa War- Alt Causes

Endless list of alternate causalities the aff doesn’t address – weak democracy, the
environment, resources, breakdown in the rule of law, private armies, and illegal arms. That’s
Juma.

And these internal links outweigh – they are descriptive of the status quo, while theirs are
predictive of future conditions.

(__) More warrants –

1. Weak states.

Mutisi ‘6
Martha, Lecturer and Staff Development Fellow at the Institute of Peace, Leadership and Governance, at Africa University, 2006,
http://www.accord.org.za/ct/2006-4.htm, p. 18

Since the violent conflicts in Africa reflect the chal- lenges of weak and failed states, the search for security
and development should be accompanied by attempts to enable the states to resuscitate and revitalise their governance structures
and operational machinery such as the legislature, executive, judiciary and security forces. It is apparent that weak
states are
in themselves unable to create conditions for stability, security, development and ultimately
durable peace. Successful democratisa- tion and development depends to a large extent on state capability and strength. In
Africa, the problem of weak and failed states is a reality, with Somalia and Sudan topping the list on the weak states scale. Weak
states are not only unstable, they also struggle to address the issues of poverty, unemployment, HIV/AIDS and environ-mental
degradation, the major factors in development. Subsequently, this process leads to a “legitimation crisis”, wherein citizens become
discontented with the state. Discontented citizens usually choose from an array of options, ways of expressing their disgruntlement.
In most cases, the frustration has been manifested through rebellion, riots, crime and coups.

2. Debt.

Jubilee.’7 (Jubilee organization for African debt relief. Bring it all back home. The impact of the debt crisis on all of us.
http://www.jubileeusa.org/truth-about-debt/debt-related-issues/bring-back-home)

Britain uses export credits to subsidize arms sales to the South. In 1993/4, 50 per cent of all export credits provided by the DTI to
exporters were for arms sales. In time, these credits become debts for poor countries. 96 per cent of the debts owed to Britain by
poor countries are owed to the Export Credit Department of the DTI. Many Third World countries have become deeply indebted
because of high military spending. And as wars escalate, they are less able to repay the money they owe. One estimate suggests that
between 1960 and 1987 Third World governments borrowed around $400 billion dollars to fund arms imports from industrial states.
The Third World arms trade has declined after a peak in the late 1980s. Most of the dictators who invested so heavily in arms are no
Debt
longer in power and today's governments are not buying as many arms as they once did. But the debts are still left to pay.
can also lead to and contribute to war. As countries become poorer because of their debts, one route that people
take is violence and protest. As this escalates, it can end in war - and does in many countries of
the Third World. As the debt crisis broke in the early 1980s, violence in many indebted countries around the Third World
erupted into war or escalated dramatically.

3. AIDS.

Brower ‘3 & Chalk 0– RAND Co-Project Director & RAND Political Scientist specializing in emerging threats [Jennifer Brower &
Peter Chalk, The Global Threat of New and Reemerging Infectious Diseases: Reconciling U.S.National Security and Public Health
Policy, www.rand.org/pubs/monograph_reports/MR1602/]

The contemporary HIV/AIDS crisis in South Africa represents an acute example of how infectious diseases can
undermine national resilience and regional stability. Roughly 25 percent of the country’s adult population is currently
infected with HIV, which makes South Africa one of the most severely affected AIDS states in the world. The consequences of the
disease have been as marked as they have been pervasive, negatively impacting on virtually all levels of the country’s security—
broadly defined—as well as significant aspects of Pretoria’s frontline neighbors. As James Wolfenson, president of the World Bank,
has remarked: Many of us used to think of AIDS as a health issue. We were wrong. AIDS can no longer be confined to the health or
AIDS is turning back the clock on development. Nothing we have seen is
social sector portfolios. Across Africa,
a greater challenge to the peace and stabilities of African societies than the epidemic of AIDS. . . . We face a major
development crisis, and more than that a security crisis. For without economic and social hope we will not have
peace, and AIDS surely undermines both.1
---Africa War- No Escalation

There’s no chance of great power conflict over Africa – Strauss and Barret says countries
perceive intervention as ineffective and have no strategic interests in the country.

No draw-in

Taire ‘4 (Morenike, Vanguard (Nigeria), Global News Wire – Asia Africa Intelligence Wire, 4-9-2004)

Defining our role may not have to be as difficult as it might first seem. In the first instance, in spite of Libya feat in WMD technology,
borrowed and invented, and despite the feat of others who, like Libya, has flirted and romanced with terrorism in the past, it
is
unlikely that Africa would be in a position to involve itself in any conflicts with any States outside
its own shores. [It] She does not have the technology, and might have trouble summoning the collective
will. And so while America grapples with impending energy troubles or rumours of it and Europe battles with the European Union,
Africa battles with hunger, and pretty much everything else that has ceased to be of any significance to anyone
in the first world. It was Sting, appropriately enough, who’d coined the lyrics and sang the song: “We have just one world, but
we live in different ones”. Indeed, we do. Unfortunately, we live, also, in perpetual danger of being sucked into the faster, more
complicated vortex of the worlds of others. We can no longer be calm, cool and collected.

Regional cooperation checks

Straus ’13 (Scott Straus for African Arguments, part of the Guardian Africa Network, Scott Straus is a professor in the
Department of Political Science at the University of Wisconsin, “Africa is becoming more peaceful, despite the war in Mali”,
http://www.guardian.co.uk/world/2013/jan/30/africa-peaceful-mali-war, January 30, 2013)

Finally,
conflict reduction mechanisms, in particular international peacekeeping and regional
diplomacy, have substantially increased on the continent. Peacekeeping is more prevalent and
especially more robust than in the 1990s. Regional bodies such as the African Union, Eccowas, Eccas, IGAD,
and SADC are quite active in most conflict situations. They have exhibited greater resolves in conflicts as
diverse as Côte d'Ivoire, Sudan, the Central African Republic, and Madagascar.

Specifically the US

Miranda ’13 (Luis Miranda, Journalist for the Real Agenda, “U.S. Military increases involvement in African conflict”, http://real-
agenda.com/2013/01/30/22156/, January 30, 2013)

As confessed last week by Hillary Clinton, the world can expect the United States to continue balkanizing sensible regions of the
planet indefinitely. With major combat operations ending in the Middle East, recently growing economic and
politicaltension in Africa opened the door for the U.S. to launch another operation in a supposed effort
to curb the spread of Al-Qaeda and its affiliates in that continent. Now, the United States reached an agreement with
the Government of Niger for immediate installation in that country of a drone base, which will be used to ‘support’ France’s
military operation in Mali, which means the beginning of a greater U.S. military involvement in the fight in North
Africa. With this agreement, the Pentagon will start reconnaissance flights over Malian territory and deploy any
number any number of troops anywhere in Mali or even neighboring countries. It is possible that, at a later stage, the drones could
be used to directly attack the groups identified as enemies, as is being done in Pakistan, Afghanistan and Yemen, where the U.S.,
almost on a daily basis, murders men, women and children are thought to be members of terrorist organizations or who are deemed
as collateral damage — as the military says. The U.S. military presence in Niger, whose scope has not been officially confirmed in
Washington, represents a significant shift in the so-called war against terrorism, so far concentrated in the Middle East and Asia. The
steps taken by the Pentagon now open a new front in Africa. So far, the U.S. only had one official base in the small state of Djibouti,
where the military stations about 2,000 soldiers and from where it launches attacks over Yemeni territory. This base, however, is too
far away for operations in Mali, Algeria, Libya and Mauritania, where the U.S. Al-Qaeda affiliated groups concentrate their forces.
The agreement with Niger, which was confirmed by official sources in the country, will allow the U.S. to have military installations in
the desert area of Agadez, in northern Niger, near the borders with Mali and Algeria. “Niger has given the green light for the use of
its territory for collecting surveillance to improve data collection of Islamist movements,” said a source quoted by Reuters. Other
U.S. media say that the U.S. is negotiating a similar agreement with Burkina Faso, on the southern border of Mali, and that the
permanent presence of drones could be extended even to Algeria, a country with which Washington maintains good relations and
that Secretary of State Hillary Clinton visited last October to discuss the security situation and the supposed extremist threat.

Wars decreasing in Africa- no risk of war

Straus ’13 (Scott Straus for African Arguments, part of the Guardian Africa Network, Scott Straus is a professor in the
Department of Political Science at the University of Wisconsin, “Africa is becoming more peaceful, despite the war in Mali”,
http://www.guardian.co.uk/world/2013/jan/30/africa-peaceful-mali-war, January 30, 2013)

In fact, as I show in a recent piece in African Affairs, looked at since the end of the cold war, wars
are not becoming more
frequent in sub-Saharan Africa. To the contrary: according to the Uppsala Armed Conflict Data Program, the pre-
eminent tracker of warfare worldwide, wars in the 2000s are substantially down from their peak in the
early 1990s. Even if one counts an uptick during the past two years, there were about one-
third fewer wars in sub-Saharan Africa in the period compared to the early-to-mid 1990s.

No external actor involvement

Straus ’13 (Scott Straus for African Arguments, part of the Guardian Africa Network, Scott Straus is a professor in the
Department of Political Science at the University of Wisconsin, “Africa is becoming more peaceful, despite the war in Mali”,
http://www.guardian.co.uk/world/2013/jan/30/africa-peaceful-mali-war, January 30, 2013)

First, the end of the cold war meant that the opportunities for rebels to receive substantial
weaponry and training from big external states declined. To be sure, states across Africa still
meddle in the affairs of their neighbors, but insurgent funding from neighbouring states is
usually enough to be a nuisance to, but not actually overthrow, existing governments.
---Africa War- AT: Deutsch

And don’t evaluate Deutsch –

A. No African nation has nuclear weapons – they don’t have a card on this question – takes
out the internal link to foreign escalation because there’s not a significant security threat to
justify intervention.

B. He’s an unqualified racist with a website – the text of the card literally refers to Africans as
“rabid tigers.” Qualifications outweigh warrants – prerequisite to evaluation since no one
responds to unqualified authors.

C. Cut out of context – it’s a hypothetical prediction for 2025.

Deutsch 2

We've been discussing various aspects of the present around the world. Let's take a step back - to the future. THE
RABID TIGER PROJECT'S MAJOR PREDICTION FOR 2025 By 2025... Nuclear weapons very likely will have been
used in anger. There are just too many weapons, components, units of nuclear material and people who know how to use them.
Despite the best efforts of the current members of the nuclear club and of the International Atomic Energy Agency (IAEA), nuclear
proliferation persists and expands. Knowledge itself is the most fluid nuclear component: it's the easiest to spread, and it cannot be
retracted. We at the Rabid Tiger Project simply do not believe that all or even most people in power around the world are morally
or otherwise restrained from causing the most terrible damage possible to the people they hate - and very few people hate no one.
In the 21st century, independence and survival may be limited to those countries that strike hard and strike first. Barring a good
ballistic missile defense (BMD) system, sometimes scornfully referred to as "Star Wars," nuclear war is like a gunfight without body
armor. There's a reason why gunfighters emphasize being quick on the draw. On the one hand, pre-emptive nuclear strikes pose the
risk of attacking a country whose leadership in fact had no aggressive designs at the moment. However, if a nuclear strike is fully
successful, the perpetrator likely won't care. On the other hand, waiting for irrefutable evidence of another's aggressive intent may
be rewarded with a surprise attack ending in annihilation or conquest. In that case, one will have avoided unfairly attacking an
enemy, but the victims likely won't care. Nuclear proliferation increases the number of sets of enemies to whom this dynamic can
apply, and exponentially increases the likelihood that one will choose to make the first move. The United States and the Soviet
Union, the first nuclear adversaries, developed a set of norms and procedures - especially in the aftermath of the 1962 Cuban Missile
Crisis - to help ensure that perceptions of momentary hostility and/or immediate threats did not escalate into a nuclear exchange.
For example, a "hot line" was established directly connecting the US president and the Soviet general secretary, so that direct,
immediate communication between the two top leaders could remove any misunderstandings before they could become self-
fulfilling prophecies. When people feel more secure that others will not feel it necessary to launch an attack, they will feel it less
necessary to launch a pre-emptive strike. To a great extent, one's attack is another's pre-emptive strike, so security can be self-
reinforcing. However, so can insecurity. Hence, the introduction of nuclear weapons into new conflicts increases the chances that
they will actually be used. India and Pakistan have averted nuclear war - for now. They have not had the US-Soviet experience with
mutual problem-solving and joint action to damp down nuclear threats. One factor reinforcing security - and which potentially can
do the opposite - is what we call the "virginity effect". Right now, no one has used nuclear weapons in anger since World War II.
National leaders can feel pressure to maintain this record. On the other hand, once nuclear weapons are used in a war, the "novelty"
wears off, and nuclear weapons become just another part of the well-armed leader's arsenal. So, nuclear proliferation exponentially
- perhaps logarithmically - increases the threat of nuclear war. SETTING THE STAGE FOR WORLD WAR III The Rabid Tiger Project
believes that a nuclear war is most likely to start in Africa. Civil wars in the Congo (the country formerly known as
Zaire), Rwanda, Somalia and Sierra Leone, and domestic instability in Zimbabwe, Sudan and other countries, as well as occasional
brushfire and other wars (thanks in part to "national" borders that cut across tribal ones) turn into a really nasty stew .
We've got all too many rabid tigers and potential rabid tigers, who are willing to push the button rather than risk being seen as
wishy-washy in the face of a mortal threat and overthrown. Geopolitically speaking, Africa
is open range. Very few
countries in Africa are beholden to any particular power. Thus, outside powers can more easily
find client states there than, say, in Europe where the political lines have long since been drawn,
or Asia where many of the countries (China, India, Japan) are powers unto themselves and don't
need any "help," thank you. Thus, an African war can attract outside involvement very quickly .
Of course, a proxy war alone may not induce the Great Powers to fight each other. But an African nuclear strike can ignite a much
broader conflagration, if the other powers are interested in a fight. Certainly, such a strike would in the first place have been
facilitated by outside help - financial, scientific, engineering, etc. Africa is an ocean of troubled waters, and some people love to go
fishing. Asia is a close second, due to the competition of major powers. For example, in an Indo-Paki confrontation, China may be
tempted to side with Pakistan, since China and India are major nuclear powers sharing a long border. However, the Asian powers are
The things to watch for are domestic economic and political
basically stable internally, at least for now.
instability in a nuclear power, the spread of nuclear weapons to new countries and new national
antagonisms and great-power ties either weak or nonexistent enough to enable opportunistic
alliances and destabilization, or strong enough that the great powers feel compelled to follow
their client states.
Agriculture Answers
Frontline
Agriculture fails because essential ground nutrients are lost when harvesting
crops
Drechsel et al. 01 [Pay Drechsel, Senior Scientist (Environmental and Soil Science) and IWMI's
Subregional Director for West Africa , Dagmar Kunze, FAO Regional Office for Africa (RAF), Frits
Penning de Vries, was Principal Researcher and Douglas Merrey was Director for Africa, “Soil
nutrient depletion and population growth in sub-Saharan Africa: A Malthusian nexus?”
Population and Environment. New York: Mar 2001. Vol. 22, Iss. 4; pg. 411, proquest]// CG

In comparison with erosion, nutrient leaching appears to be a minor contributor to N and


K depletion and is negligible in view of P, but may become substantial with respect to
other nutrients such as Mg (Pieri, 1992; Poss & Saragoni, 1992). If we calculate scenarios
for upland SSA and assume zero erosion, runoff, and leaching (and avoid related
assessment errors), we still get a negative N and K balance on average. The reason is that
the amount of nutrients lost with the harvested crop and its residues is not balanced
through the current level of inputs (fertilizer, manure). This means although the control
of soil erosion is crucial, it can only reduce the speed of nutrient depletion under the
current level of inputs, which corresponds with empirical evidence (Drechsel & Penning
de Vries, in press).

Export laws are an alt cause


Bradsher and Martin, 08 (Keith Bradsher and Andrew Martin June 30, 2008 [“Hoarding
Nations Drive Food Costs Ever Higher” The New York Times
http://www.nytimes.com/2008/06/30/business/worldbusiness/30trade.html?
_r=1&partner=rssnyt&emc=rss&oref=slogin]

BANGKOK — At least 29 countries have sharply curbed food exports in recent months,
to ensure that their own people have enough to eat, at affordable prices. When it
comes to rice, India, Vietnam, China and 11 other countries have limited or banned exports.
Fifteen countries, including Pakistan and Bolivia, have capped or halted wheat exports. More
than a dozen have limited corn exports. Kazakhstan has restricted exports of sunflower seeds.
The restrictions are making it harder for impoverished importing countries to afford the food
they need. The export limits are forcing some of the most vulnerable people, those who
rely on relief agencies, to go hungry. “It’s obvious that these export restrictions fuel
the fire of price increases,” said Pascal Lamy, the director general of the World Trade
Organization. And by increasing perceptions of shortages, the restrictions have led to
hoarding around the world, by farmers, traders and consumers . “People are in a panic,
so they are buying more and more — at least, those who have money are buying,” said
Conching Vasquez, a 56-year-old rice vendor who sat one recent morning among piles of rice at
her large stall in Los Baños, in the Philippines, the world’s largest rice importer. Her customers
buy 8,000 pounds of rice a day, up from 5,500 pounds a year ago.
Agriculture Competitiveness Answers
US agriculture exports are at an all-time high—we postdate
HPJ 11 – High Plains Midwest Journal, cites a secretary of agriculture (High Plains Midwest
Journal, “U.S. farm exports reach all-time high” June 2011,
http://www.hpj.com/archives/2011/jun11/jun27/0512AgExportsSetRecordsr.cfm) // CB

Secretary of Agriculture Tom Vilsack made the following statement on data released showing
that U.S. farm exports reached an all-time high of $75 billion during the first half of fiscal year
2011:

"Today's trade data demonstrate that, once again, America's farmers and ranchers are helping
lead the way to recovery from the worst economic recession in decades. The gains in U.S.
agricultural exports are particularly encouraging news for those who live in rural America or
earn a living in farming, ranching and agriculture-related industries, because exports are
creating jobs here at home. Farm exports alone will support more than one million jobs in
America this year. Strong U.S. farm exports will be a key contributor to building an economy that
continues to grow, innovate and out-compete the rest of the world.

"At $75 billion, U.S. agricultural exports for FY 2011 are 27 percent higher than the same period
in last year. This puts us on track to reach the current USDA export forecast of $135.5 billion by
the end of the year.

"As expected, China is our top export market. With $15.1 billion in farm exports, China
accounted for nearly 20 percent of all U.S. agricultural exports. Canada is our second-largest
market.

"Both the value and volume of exports rose in the first half of the year, with the volume of bulk
shipments up 5 percent from last year. Wheat and cotton volumes were especially robust, with
increases of 64 percent 44 percent, respectively.

"March was the highest-grossing month for U.S. agricultural exports ever. During that month
alone, U.S. farmers and ranchers exported $13.3 billion worth of U.S. agricultural goods. That's
$407 million more than the previous record set in November 2010.

"Congress can help U.S. farmers and ranchers sustain their record growth by moving
expeditiously to pass the South Korea, Colombia and Panama trade agreements. When fully
implemented, those three agreements have potential to add more than $2 billion per year to
our exports and support job creation here at home. Gains like these will help farmers and
ranchers continue to contribute to President Obama's National Export Initiative goal of doubling
all U.S. exports by 2014."
Air Pollution Answers
Frontline

Air quality is improving - their evidence flawed

Hayward ‘4 (Steven F., Senior Fellow – Pacific Research Institute, Index of Leading Indicators,
http://www.pacificresearch.org/pub/sab/enviro/04_enviroindex/Enviro_2004.pdf)

Average vehicle emissions are dropping about 10 percent per year as the fleet turns over to
inherently cleaner vehicles, including modern SUVs. · Since 1985, nitrogen oxides (NOX) emissions from cars
have dropped 56 percent and volatile organic compounds (VOCs) are down 67 percent, according to the
most recent EPA data. · Stories touting an uptick in ozone pollution are based largely on the
weekend effect, a paradoxical situation in which the weekend drop in NOX emissions, from 10 to 40
percent, causes an increase in ozone levels. · Asthma rates in children under the age of five rose more than 160
percent between 1980 and 1994, while air pollution rates fell from 25 to 80 percent . Was 2003 the year we
started losing the battle against ozone smog? That is what you would think if you read the media headlines. .Smog Woes Back
on Horizon,. trumpeted an abovethe- fold Los Angeles Times headline in mid-July.1 .It.s One Smoggy Summer,. declared the
Associated Press. And USA Today joined the chorus in October with .Smoggy Skies Persist Despite Decade of Work..2
Unfortunately, a reader of these articles will learn very little about what is behind the recent uptick in ozone levels. To the
contrary, most media stories convey loads of misinformation . The USA Today story, for example, offers this
explanation of stubborn ozone levels: .One likely reason why the smog isn.t lifting: Americans are driving more miles than they
did in the 1980s. And they.re driving vehicles that give off more pollution than the cars they drove in the .80s. (emphasis added).
USA Today needs a better fact-checking department.

Global air pollution inevitable

Watson ‘5 (Traci, Staff Writer – USA Today, “Air Pollution From Other Countries Drifts into USA”, USA Today, 3-13,
http://www.usatoday.com/weather/resources/climate/2005-03-13-pollution-_x.htm)

Americans drive imported cars, wear imported clothes and chug imported beers. Now scientists are discovering another, less
welcome import into the USA: air pollution. Mercury from China, dust from Africa, smog from Mexico
— all of it drifts freely across U.S. borders and contaminates the air millions of Americans breathe, according to
recent research from Harvard University, the University of Washington and many other institutions where scientists are
studying air pollution. There are no boundaries in the sky to stop such pollution, no Border Patrol agents to capture it. Pollution
wafting into the USA accounts for 30% of the nation's ozone, an important component of smog, says researcher David Parrish of
the National Oceanic and Atmospheric Administration. By the year 2020, Harvard University's Daniel Jacob says, imported
While the United States is cutting its
pollution will be the primary factor degrading visibility in our national parks.
own emissions, some nations, especially China, are belching out more and more dirty air.
As a result, overseas pollution could partly cancel out improvements in U.S. air quality that
have cost billions of dollars.

The impact’s empirically denied

Lomborg ‘5 (Bjorn, Adjunct Prof at the Copenhagen Business School, Director of the Copenhagen Consensus Centre, and Former
Director of the Environmental Assessment Institute in Copenhagen, 8-15, “Take a Deep Breath…Air Quality is Getting Better,” THE
GUARDIAN, Lexis)
We often assume that air pollution is a modern phenomenon , and that it has got worse in recent times.
However, air pollution has been a major nuisance for most of civilisation , and the air of the
western world has not been as clean as it is now for a long time . In ancient Rome, the statesman
Seneca complained about "the stink, soot and heavy air" in the city. In 1257, when Henry III's wife visited Nottingham, she found
the stench of smoke from coal burning so intolerable that she left for fear of her life, and in 1285 London's air was so polluted
that Edward I established the world's first air pollution commission. Shelley wrote: "Hell must be much like London, a smoky and
populous city." For London, the consequences were dire. In the 18th century it had 20 foggy days a year, but this had increased
to almost 60 by the end of the 19th century: this meant that London got 40% less sunshine than the surrounding towns, and the
number of thunderstorms doubled in London from the early-18th to the late-19th century. We have data for air pollution in
London since 1585, estimated from coal imports till 1935 and adjusted to measured pollution from the 1920s till today. This
shows how levels
of smoke and sulphur pollution increased dramatically over the 300 years
from 1585, reaching a maximum in the late 19th century, only to have dropped even faster
ever since, such that the levels of the 1980s and1990s were below the levels of the late
16th century. And despite increasing traffic, particulate emissions in the UK are expected to
decrease over the next 10 years by 30%. Smoke and particles are probably by far the most
dangerous pollutant, and London's air has not been so free of them since the middle ages.

Doesn’t cause extinction

Schwartz ‘3 Adjunct Scholar @ Competitive Enterprise Institute


[Joel Schwartz, “Particulate Air Pollution: weighing the risks” April 2003 http://cei.org/pdf/3452.pdf/] Kevin W. Prep ‘11

Studies that have attempted to estimate directly when death occurs in relation to increases in pollution by estimating the
size of this frail population have concluded that acute changes in pollution levels shorten life expectancy
by a matter of days at most.113 The putative effects of PM based on epidemiologic results are consistent with the
harvesting hypothesis. For example, if daily variations in pollution mainly affect an already-frail population, it may be that it’s
not so much the type of external stress that is important, but that any modest external stress would be enough to cause death.
This is consistent with the finding that many different types of pollution—e.g., fine and coarse PM, various gases—appear to
have effects on mortality of similar magnitude, as do changes in temperature, atmospheric pressure and other weather
variables.114 If PM and other pollutants were shortening healthy people’s lives by months or years, it would be an odd
coincidence if several different pollutants, each with a different intrinsic toxicity and each present at different levels in different
cities, all happened to exert roughly the same effects, regardless of the pollutant or its ambient concentration. On the other
hand, if PM is actually shortening life by months or years in otherwise healthy people, biological plausibility is still an issue.
Various pollutants are always present at some level in ambient air , and pollution levels vary from day
to day. It is not clear why apparently healthy people would be suddenly killed on a given day by
relatively low PM levels that they have experienced many times in the past .115 The frailpopulation
hypothesis would explain the possible lack of a threshold for the effect of PM on mortality, since changes in pollution, even at
low levels, might be enough to cause death in very frail people.116
---Air Pollution- Quality Improving

Pollution is and will continue to decrease

Schwartz ‘3 (Joel, Adjunct Scholar at the Competitive Enterprise Institute, “Particulate Air Pollution: weighing the risks” April
2003 http://cei.org/pdf/3452.pdf)

America’s air quality has vastly improved in recent decades due to progressive emission
reductions from industrial facilities and motor vehicles. The country achieved this success
despite substantial increases in population, automobile travel, and energy production. Air
pollution will continue to decline, both because more recent vehicle models start out cleaner and stay cleaner as
they age than earlier ones, and also because already-adopted standards for new vehicles and existing power plants and
industrial facilities come into effect in the next few years

It’s being solved now

Lomborg ‘1- Statistician at the University of Aarhus, Denmark. Author of “The Skeptical Environmentalist”
[Bjorn Lomborg, “Environmentalists tend to believe that, ecologically speaking, things are getting worse and worse.”August 9. 2001.
http://www.warwickhughes.com/climate/lomborg2.htm]

Fourth, pollution is also exaggerated. Many analyses show that air pollution diminishes when a
society becomes rich enough to be able to afford to be concerned about the environment .
For London, the city for which the best data are available, air pollution peaked around 1890 (see chart 2). Today, the air is
cleaner than it has been since 1585. There is good reason to believe that this general
picture holds true for all developed countries. And, although air pollution is increasing in many developing
countries, they are merely replicating the development of the industrialized countries. When they grow sufficiently rich they,
too, will start to reduce their air pollution.
---Air Pollution- Alt Causes
Tons of alt causes

Brook ‘4 (Robert D. M.D., et al, “Air Pollution and the Cardiovascular Disease”, Circulation: Journal of the American Heart
Association, 6-1, http://circ.ahajournals.org/cgi/content/full/109/21/2655#SEC1/)

A brief description of several individual air pollutants is provided first for background. A complete discussion is beyond the
scope of this statement, and interested readers may find a more comprehensive review on this subject elsewhere.26
Particulate Matter Airborne Particulate Matter consists of a heterogeneous mixture of solid and liquid particles suspended in air,
continually varying in size and chemical composition in space and time (Figure 1). Primary particles are emitted directly into the
atmosphere, such as diesel soot, whereas secondary particles are created through physicochemical transformation of gases,
such as nitrate and sulfate formation from gaseous nitric acid and sulfur dioxide (SO2), respectively. The numerous natural
and anthropogenic sourcesof PM include motor vehicle emissions, tire fragmentation and resuspension
of road dust, power generation and other industrial combustion, smelting and other metal processing,
agriculture, construction and demolition activities, residential wood burning, windblown soil, pollens
and molds, forest fires and combustion of agricultural debris, volcanic emissions, and sea spray. Although
there are thousands of chemicals that have been detected in PM in different locations, some of the more common constituents
include nitrates, sulfates, elemental and organic carbon, organic compounds (eg, polycyclic aromatic hydrocarbons), biological
compounds (eg, endotoxin, cell fragments), and a variety of metals (eg, iron, copper, nickel, zinc, and vanadium).
---Air Pollution- No Impact
No impact

Schwartz ‘3 (Joel, Adjunct Scholar – Competitive Enterprise Institute, “Particulate Air Pollution: Weighing the Risks”, April,
http://cei.org/pdf/3452.pdf)

Nonetheless, both the Bush Administration and congressional Democrats have proposed sweeping new measures to further
crack down on power plant emissions. The Administration’s Clear Skies Initiative and a more stringent Democratic alternative
are largely justified by claims that current levels of particulate matter (PM) pose a serious public health
threat. Supporters of these bills promise substantial benefits from additional PM reductions. Nevertheless, the benefit
claims for PM reductions rest on a weak foundation. EPA based its new annual fine PM (PM2.5) standard on a study
known as the American Cancer Society (ACS) study of PM and mortality, which assessed the association between the risk of
death between 1982 and 1998 with PM2.5 levels in dozens of American cities. Although the ACS study reported an association
between PM and mortality, some odd features of the ACS results suggest that PM is not the culprit. For example, according to
the ACS results, PM increased mortality in men, but not women; in those with no more than a high school degree, but not those
with at least some college education; in former- smokers, but not current- or never-smokers; and in those who said they were
moderately active, but not those who said they were very active or sedentary. These odd variations in the
relationship
between PM 2.5 and mortality seem biologically implausible. Even more surprising, the ACS study
reported that higher PM2.5 levels were not associated with an increased risk of mortality due to respiratory disease; a
surprising finding, given that PM would be expected to exert its effects through the respiratory system. EPA also ignored the
results of another epidemiologic study that found no effect of PM2.5 on mortality in a cohort of
veterans with high blood pressure, even though this relatively unhealthy cohort should have been more susceptible to the
effects of pollution than the general population. The evidence therefore suggests that the existing annual standard for PM2.5 is
unnecessarily stringent. Attaining the standard will be expensive, but is unlikely to improve public health.
Air Power Answers
Frontline

Air power not key -to overall power or deterrence – their evidence is biased
Axe ‘9 (David, military correspondent, regular contributor to The Washington Times, C-SPAN, and Wired, 3/30/9,
http://www.warisboring.com/2009/03/30/f-22s-versus-russias-rusting-ramshackle-air-force/)

Analyst Gregory Martin, a retired Air Force general, said the erosion of world influence is largely the result of weak public support for
the F-22 and F-35 stealth fighters, which are built by Lockheed Martin, Boeing and Northrop Grumman. “If you can’t afford that
[mix], then your national objectives have to be scaled back,” Martin said. In
other words, stealth fighters equal
national power. And the absence of stealth fighters equals weakness. Hogwash. The economic crisis is having an effect on
every country, unevenly. Arguably, the U.S. is faring better than most as investors flee to the comparative safety of the dollar.
Power in the world is a relative thing: if everyone else gets much weaker, and we stay the same or only grow a little weak,
then we are, in fact, more powerful than we were before. Get it? The global recession, alone, does not mean we are losing influence.
In fact, the recession might even boost our influence, by underscoring just how much the world depends on America as a consumer
market. But more importantly, American national power does not hinge on fighter jets. We could retire every
single fighter in the U.S. Air Force, tomorrow, and still remain the most powerful nation in the world, by
far. National power is a complex and shifting thing, comprising military force, financial and cultural influence, leadership in
international coalitions and organizations and even language. Every country in the world teaches American English to its business
students, aviators and sea captains. Does that have anything to do with the F-22? Do some of our biggest exports — music, movies
and television — depend on a squadron of F-35s flying orbits over North Dakota? Ignore the noise coming out of Washington’s
punditocracy as the Obama Administration shapes its first defense budget. And when that budget is published, and it (inevitably)
Nearly everyone telling
includes cuts to Air Force fighter programs, take a deep breath before panicking and consider:
you we must buy a given quantity of stealth fighters, or lose global influence, has a financial stake in
advocating such purchases. Of the speakers at the Wednesday confab: * Loren Thompson, from the Lexington Institute, runs
a private consultancy for the defense industry, with clients including Lockheed Martin * Thompson’s colleague, Rebecca
Grant, also runs her own consultancy for the defense industry * Gregory Martin has been a Northrop
Grumman consultant The U.S. Air Force is in deep trouble, but it’s trouble of its own making. And it’s testimony to just
how overwhelming, and sustainable, is America’s military, cultural, linguistic and financial
dominance in the world that our primary military air service can commit slow, institutional
suicide without alarming too many people, aside from a few hardware nerds like me and the consultants who get
rich gabbing about certain pointy airplanes on behalf of wealthy corporate clients.

Air power high and no impact to declines

Friedman and Preble ‘10 (Benjamin Friedman is a research fellow in defense and homeland security studies at the Cato
Institute, Christopher Preble is director of foreign policy studies at the Cato Institute, Budgetary Savings from Military Restraint,
September 22, 2010 Cato Policy Analysis No. 667 September 23, 2010 http://www.cato.org/pubs/pas/PA667.pdf)

We would also eliminate six fighter wing equivalents from the Air Force. There are three justifications for
this cut. First, the Navy already provides enough airpower from the sea to deal with most wars.14 Second,
the Air Force lacks enemies that challenge its air superiority. Third, advancements in weapons
guidance greatly increased the destructive power of each airframe. These factors mean that the fighter
capability we maintain is more than what is needed to support likely ground conflicts or conduct
bombing raids. Because we want an offshore posture rather than a forward defense, we retain our current bomber and refueling
tanker procurement plans. We also maintain the Air Force’s spending on unmanned aerial vehicles, given their flexibility and low
cost relative to manned aircraft.
---Air Power- Inevitable/No Challengers

ZERO challengers to US airpower

Weitz ’11 -- Hudson Institute Senior Fellow and Director of the Center for Political-Military Analysis (Richard, 9/2/11, “U.S. Air
Superiority Remains Safe,” China US Focus, http://www.chinausfocus.com/slider/u-s-air-superiority-remains-safe/)

In any case, the Pentagon’s report on the Chinese military makes clear that China’s air force and defense aerospace
sector still lags considerably behind that of the United States. According to the U.S. Department of Defense, the People’s
Liberation Army Air Force (PLAAF) has made considerable progress in its efforts to expand its role from its traditional focus on
homeland air defense to encompass extra-territorial missions such as anti-access and air-denial operations designed to prevent the
U.S. military from intervening in battles along China’s periphery. ThePLAAF has also demonstrated a limited
ability to operate even further from China, at least in terms of participating in military exercises in Central Asia and Turkey
last year or helping to evacuate Chinese nationals from the Middle East. Yet, most Chinese military aircrafts are at least
one generation behind those available to the United States. The PLA’s air defense systems comprise mostly Soviet-era
surface-to-air missile launchers purchased from Russia (such as the SA-10 and SA-20 PMU1/PMU2) and Chinese systems based on
these S-300 vairants (e.g., the HQ-9). The PLAAF’s best fighter planes, like those in the PLA Navy’s air component, also originate from
Soviet-era technology (e.g., Su-27/F-11, Su-30 variants, and the F-10), while the PLAAF’s long-range bomber force relies on even
more obsolete Soviet technology. China’s unmanned aerial vehicles and early warning aircraft also have inferior
equipment compared to those in the NATO or Russian fleets. Overall, the Pentagon assesses only about one-quarter of
China’s military aircraft as being modern combat planes equivalent to what one finds in Western fleets. As for the
Chengdu J-20, the Pentagon’s China military power report says that “the Defense Department does not expect the
J-20 to achieve an effective operational capability prior to 2018.” The report acknowledges that the J-20 “will
eventually give the PLA Air Force a platform capable of long range, penetrating strikes into complex air defense environments.” But
this very language suggests that the J-20 will serve primarily as a tactical strike plane, such as the original U.S. F-117 NightHawk that
initiated the air campaigns against Iraq and in Kosovo, rather than as a fighter that will compete with the latest U.S. stealth planes,
the F-22 and F-35, for air superiority. In this role, the plane will simply complement China’s offensive surface-to-surface missiles by
“improve[ing] the PLA's ability to strike regional air bases, logistical facilities, and other ground-based infrastructure.” Furthermore,
the report shares the ambiguous judgment of many analysts regarding whether the J-20 really is a fifth-generation aircraft on par
with these U.S. planes. It states that the J-20 "highlights China's ambition to produce a fighter aircraft that incorporates stealth
attributes, advanced avionics, and super-cruise capable engines over the next several years."[italics added] The report further notes
that “China
faces several hurdles as it moves toward J-20 production, including the mastery of high
performance jet engine production.” Indeed, although the improving quality of China’s defense industry has resulted
in the PRC’s ending its previous billion-dollar annual purchases of Soviet-era weapons from the Russian Federation, China still must
use Russian engines for its most advanced warplanes.

Air superiority is high and unmatched

Schanz ’11 -- Senior Editor for Air Force Magazine (Marc V, “AirSea Battle's Turbulent Year,” October 2011,
http://www.airforce-magazine.com/MagazineArchive/Pages/2011/October%202011/1011airsea.aspx)

AirSea Battle, the operational concept recently assembled by the Air Force and Navy, is an ambitious
effort with great implications for how the air and sea services plan for, equip, and prepare to fight future high-intensity conflicts. ASB
is born out of a need for the US military to address perceived threats and strategic concerns across the
globe, in environments far different from the two largely "low intensity" wars fought over the last decade. At its core, a finalized
AirSea Battle concept will protect America’s ability to project power and secure areas of the "global
commons"—the sea and air lanes vital to the nation’s interests—while relying heavily on air and sea
superiority. "Over the last several decades, the US military has developed and maintained an unrivaled
ability to establish and maintain air superiority and sea control," said Air Force Chief of Staff Gen. Norton A.
Schwartz in an address at the National Defense University in December 2010. The US has been so successful in
projecting expeditionary power, both from long distances and from forward bases, that its ability to do so
has been largely unchallenged, Schwartz added. Despite the lack of information, there is some evidence USAF and the
Navy are already coordinating their exercise and experimentation plans to match up with ASB concepts. The Air
Force’s Joint Expeditionary Force Experiment franchise, a series of live, virtual, and constructed experiments run by the Air Force
Command and Control Integration Center, plans on focusing on AirSea Battle concepts with the Navy in Fiscal 2012.

Even if other countries are rising it will take decades to match the US

Margolis ‘7 - contributing foreign editor at Sun National Media in Canada (Eric, 7/31/‘7,
http://www.lewrockwell.com/margolis/margolis85.html)

The US has also developed reconnaissance capability of formidable capacity and coverage. US satellites can read license plates
through clouds, smoke, rain or foliage, and track human infrared signatures. Drones, U-2 spy planes and a fleet of electronic warfare
aircraft provide unblinking, 24/7 "eyes in the sky" over almost all of Afghanistan and Iraq. The flood of data from all these sensors is
consolidated and distributed to field commands or shared with HQ units in what is called "actionable" information. The US Air Force
has become to the American Imperium what the Royal Navy was to the British Empire, the source of its might, and means of power
projection. While the Royal Navy ruled only the waves and littoral regions, the
USAF can today reach and strike any
point on the globe with devastating accuracy, speed and force. It is the mightiest, most
technologically accomplished military force in history. In fact, the USAF, with its new stealthy F-22
and upcoming F-35, are now so technologically advanced, they are at least 1.5–2 generations
ahead of the rest of the world. Russia has advanced technology and anti-stealth systems on the drawing
board but cannot yet afford to deploy them in sufficient numbers. Russia, China, and India are
unlikely to catch up with US military technology for the next 25 years – if ever. The US accounts for
50% of total global military spending, and is simply too far ahead for any other powers to catch up – unless some radical new military
technologies suddenly emerge that neutralize or make obsolete today’s advanced weapons systems. Only Europe could compete
militarily, had it the will, which it does not. In fact, America’s air force and naval aviation have enjoyed near
absolute air superiority since 1943 with only temporary challenges during the Korean and Vietnam Wars. The USAF
also has the US military’s smartest, best educated, and most forward-thinking officers. The US Army’s thankless role – and I say this
as a former Army infantryman – has become to pin down enemy units so they can become targets for the USAF’s smart bombs.

Military power inevitable- relative superiority, latent power, and flexibility

Kagan 1/17 (Robert Kagan, Senior Fellow, Foreign Policy, Center on the United States and Europe, Brookings, “Not Fade Away:
Against the Myth of American Decline”, http://www.brookings.edu/opinions/2012/0117_us_power_kagan.aspx, January 17, 2012)
Is the United States in decline, as so many seem to believe these days? Or are Americans in danger of committing pre-
emptive superpower suicide out of a misplaced fear of their own declining power? A great deal depends on the answer to these
questions. The present world order—characterized by an unprecedented number of democratic nations; a greater global prosperity,
even with the current crisis, than the world has ever known; and a long peace among great powers—reflects American principles
and preferences, and was built and preserved by American power in all its political, economic, and military dimensions. If American
power declines, this world order will decline with it. It will be replaced by some other kind of order, reflecting the desires and the
qualities of other world powers. Or perhaps it will simply collapse, as the European world order collapsed in the first half of the
twentieth century. The belief, held by many, that even with diminished American power “the underlying foundations of the liberal
international order will survive and thrive,” as the political scientist G. John Ikenberry has argued, is a pleasant illusion. American
decline, if it is real, will mean a different world for everyone. Iraqis wave behind a U.S. flag on the dashboard of a Mine Resistant
Ambush Protected (MRAP) vehicle from the 3rd Brigade Combat Team, 1st Cavalry Division as part of the last U.S. military convoy to
leave Iraq December 18, 2011. But how
real is it? Much of the commentary on American decline these days rests
on rather loose analysis, on impressions that the United States has lost its way, that it has abandoned the virtues that made it
successful in the past, that it lacks the will to address the problems it faces. Americans look at other nations whose economies are
now in better shape than their own, and seem to have the dynamism that America once had, and they lament, as in the title of
Thomas Friedman’s latest book, that “that used to be us.” The perception of decline today is certainly understandable, given the
dismal economic situation since 2008 and the nation’s large fiscal deficits, which, combined with the continuing growth of the
Chinese, Indian, Brazilian, Turkish, and other economies, seem to portend a significant and irreversible shift in global economic
power. Some of the pessimism is also due to the belief that the United States has lost favor, and therefore influence, in much of the
world, because of its various responses to the attacks of September 11. The detainment facilities at Guantánamo, the use of torture
against suspected terrorists, and the widely condemned invasion of Iraq in 2003 have all tarnished the American “brand” and put a
dent in America’s “soft power”—its ability to attract others to its point of view. There have been the difficult wars in Iraq and
Afghanistan, which many argue proved the limits of military power, stretched the United States beyond its capacities, and weakened
the nation at its core. Some compare the United States to the British Empire at the end of the nineteenth century, with the Iraq and
Afghanistan wars serving as the equivalent of Britain’s difficult and demoralizing Boer War. With this broad perception of decline as
the backdrop, every failure of the United States to get its way in the world tends to reinforce the impression. Arabs and Israelis
refuse to make peace, despite American entreaties. Iran and North Korea defy American demands that they cease their nuclear
weapons programs. China refuses to let its currency rise. Ferment in the Arab world spins out of America’s control. Every day, it
seems, brings more evidence that the time has passed when the United States could lead the world and get others to do its bidding.
Powerful as this sense of decline may be, however, it deserves a more rigorous examination. Measuring changes in a
nation’s relative power is a tricky business, but there are some basic indicators: the size and the influence of its
economy relative to that of other powers; the magnitude of military power compared with that of potential
adversaries; the degree of political influence it wields in the international system—all of which make up what the Chinese
call “comprehensive national power.” And there is the matter of time. Judgments based on only a few years’
evidence are problematic. A great power’s decline is the product of fundamental changes in the international
distribution of various forms of power that usually occur over longer stretches of time. Great powers rarely decline
suddenly. A war may bring them down, but even that is usually a symptom, and a culmination, of a longer process. The decline
of the British Empire, for instance, occurred over several decades. In 1870, the British share of global manufacturing
was over 30 percent. In 1900, it was 20 percent. By 1910, it was under 15 percent—well below the rising United States, which had
climbed over the same period from more than 20 percent to more than 25 percent; and also less than Germany, which had lagged
far behind Britain throughout the nineteenth century but had caught and surpassed it in the first decade of the twentieth century.
Over the course of that period, the British navy went from unchallenged master of the seas to sharing control of the oceans with
rising naval powers. In 1883, Britain possessed more battleships than all the other powers combined. By 1897, its dominance had
been eclipsed. British officials considered their navy “completely outclassed” in the Western hemisphere by the United States, in
East Asia by Japan, and even close to home by the combined navies of Russia and France—and that was before the threatening
growth of the German navy. These were clear-cut, measurable, steady declines in two of the most important measures of power
over the course of a half-century. Some of the arguments for America’s relative decline these days would be more potent if they had
not appeared only in the wake of the financial crisis of 2008. Just as one swallow does not make a spring, one recession, or even
a
severe economic crisis, need not mean the beginning of the end of a great power. The United States
suffered deep and prolonged economic crises in the 1890s, the 1930s, and the 1970s. In each case, it
rebounded in the following decade and actually ended up in a stronger position relative to other
powers than before the crisis. The 1910s, the 1940s, and the 1980s were all high points of American global power and influence. Less
than a decade ago, most observers spoke not of America’s decline but of its enduring primacy. In 2002, the historian Paul Kennedy,
who in the late 1980s had written a much-discussed book on “the rise and fall of the great powers,” America included, declared that
never in history had there been such a great “disparity of power” as between the United States and the rest of the world. Ikenberry
agreed that “no other great power” had held “such formidable advantages in military, economic, technological, cultural, or political
capabilities.... The preeminence of American power” was “unprecedented.” In 2004, the pundit Fareed Zakaria described the United
States as enjoying a “comprehensive uni-polarity” unlike anything seen since Rome. But a mere four years later Zakaria was writing
about the “post-American world” and “the rise of the rest,” and Kennedy was discoursing again upon the inevitability of American
decline. Did the fundamentals of America’s relative power shift so dramatically in just a few short years? The answer is no. Let’s start
with the basic indicators. In economic terms, and even despite the current years of recession and slow growth,
America’s position in the world has not changed. Its share of the world’s GDP has held remarkably
steady, not only over the past decade but over the past four decades. In 1969, the United States produced roughly a quarter of
the world’s economic output. Today it still produces roughly a quarter, and it remains not only the largest but also
the richest economy in the world. People are rightly mesmerized by the rise of China, India, and other Asian
nations whose share of the global economy has been climbing steadily, but this has so far come almost entirely at the
expense of Europe and Japan, which have had a declining share of the global economy. Optimists about China’s
development predict that it will overtake the United States as the largest economy in the world sometime in the next two decades.
This could mean that the United States will face an increasing challenge to its economic position in the future. But the sheer
size of an economy is not by itself a good measure of overall power within the international system. If it were, then
early nineteenth-century China, with what was then the world’s largest economy, would have been the predominant power instead
of the prostrate victim of smaller European nations. Even
if China does reach this pinnacle again—and Chinese
leaders face significant obstacles to sustaining the country’s growth indefinitely—it will still remain
far behind both the United States and Europe in terms of per capita GDP. Military capacity
matters, too, as early nineteenth-century China learned and Chinese leaders know today. As Yan Xuetong recently noted,
“military strength underpins hegemony.” Here the United States remains unmatched. It is far and
away the most powerful nation the world has ever known, and there has been no decline in
America’s relative military capacity—at least not yet. Americans currently spend less than $600 billion a year on
defense, more than the rest of the other great powers combined . (This figure does not include the deployment
in Iraq, which is ending, or the combat forces in Afghanistan, which are likely to diminish steadily over the next couple of years.)
They do so, moreover, while consuming a little less than 4 percent of GDP annually—a higher percentage than the other great
powers, but in historical terms lower than the 10 percent of GDP that the United States spent on defense in the mid-1950s and the 7
percent it spent in the late 1980s. The superior expenditures underestimate America’s actual superiority in military capability.
American land
and air forces are equipped with the most advanced weaponry, and are the most
experienced in actual combat. They would defeat any competitor in a head-to-head battle. American naval power
remains predominant in every region of the world. By these military and economic measures, at least, the
United States today is not remotely like Britain circa 1900, when that empire’s relative decline began to become apparent. It is more
like Britain circa 1870, when the empire was at the height of its power. It is possible to imagine a time when this might no longer be
the case, but that moment has not yet arrived. But what about the “rise of the rest”—the increasing economic clout of
nations like China, India, Brazil, and Turkey? Doesn’t that cut into American power and influence? The answer is, it depends. The fact
that other nations in the world are enjoying periods of high growth does not mean that America’s position as the predominant
power is declining, or even that “the rest” are catching up in terms of overall power and influence. Brazil’s share of global GDP was a
little over 2 percent in 1990 and remains a little over 2 percent today. Turkey’s share was under 1 percent in 1990 and is still under 1
percent today. People, and especially businesspeople, are naturally excited about these emerging markets, but just because a nation
is an attractive investment opportunity does not mean it is a rising great power. Wealth matters in international politics, but there
is no simple correlation between economic growth and international influence. It is not clear that a
richer India today wields greater influence on the global stage than a poorer India did in the 1950s under Nehru, when it was the
leader of the Non-Aligned Movement, or that Turkey, for all the independence and flash of Prime Minister Recep Tayyip Erdoğan,
really wields more influence than it did a decade ago. As for the effect of these growing economies on the position of the United
States, it all depends on who is doing the growing. The problem for the British Empire at the beginning of the twentieth century was
not its substantial decline relative to the United States, a generally friendly power whose interests did not fundamentally conflict
with Britain’s. Even in the Western hemisphere, British trade increased as it ceded dominance to the United States. The problem was
Britain’s decline relative to Germany, which aimed for supremacy on the European continent, and sought to compete with Britain on
the high seas, and in both respects posed a threat to Britain’s core security. In the case of the United States, the dramatic and rapid
rise of the German and Japanese economies during the Cold War reduced American primacy in
the world much more than the more recent “rise of the rest.” America’s share of the world’s GDP, nearly 50 percent after World War
II, fell to roughly 25 percent by the early 1970s, where it has remained ever since. But that “rise of the rest” did not
weaken the United States. If anything, it strengthened it. Germany and Japan were and are close democratic allies,
key pillars of the American world order. The growth of their economies actually shifted the balance irretrievably against the Soviet
bloc and helped bring about its demise. When gauging the impact of the growing economies of other countries today, one has to
make the same kinds of calculations. Does the growth of the Brazilian economy, or of the Indian economy, diminish American
global power? Both nations are friendly, and India is increasingly a strategic partner of the United
States. If America’s future competitor in the world is likely to be China, then a richer and more powerful India will be an
asset, not a liability, to the United States. Overall, the fact that Brazil, India, Turkey, and South Africa are enjoying a
period of economic growth—which may or may not last indefinitely—is either irrelevant to America’s strategic position
or of benefit to it. At present, only the growth of China’s economy can be said to have implications for American power in the
future, and only insofar as the Chinese translate enough of their growing economic strength into military strength.

No military challengers – basing, allies, and capabilities

Dorfman ‘12 (Zach, Assistant editor of the Carnegie Council's journal, MA in


international relations from the University of Chicago, “What We Talk about
When We Talk about Isolationism, Carnegie Council)

The rise of China notwithstanding, the United States remains the world's sole superpower. Its military (and to
a considerable extent, political) hegemony extends not just over North America or even the Western hemisphere,
but also Europe, large swaths of Asia, and Africa . Its interests are global; nothing is outside its potential sphere of
influence. There are an estimated 660 to 900 American military bases in roughly 40 countries
worldwide, although figures on the matter are notoriously difficult to ascertain, largely because of subterfuge on the part of the
military. According to official data there are active-duty U.S. military personnel in 148 countries ,
or over 75 percent of the world's states. The United States checks Russian power in Europe and Chinese
power in South Korea and Japan and Iranian power in Iraq, Afghanistan, and Turkey . In order to
maintain a frigid peace between Israel and Egypt, the American government hands the former $2.7 billion in military aid every year,
and the latter $1.3 billion. It also gives Pakistan more than $400 million dollars in military aid annually (not including
counterinsurgency operations, which would drive the total far higher), Jordan roughly $200 million, and Colombia over $55 million.
---Air Power- No Impact

Airpower not key to military effectiveness – empirically proven

Mueller ’10 -- senior political scientist at RAND Corporation (Karl P., , adjunct associate professor in the Security Studies
Program, Georgetown University, 2010, “Air Power,” RAND Corporation,
http://www.rand.org/pubs/reprints/2010/RAND_RP1412.pdf)

Strategic bombing campaigns failed to produce the sort of rapid, decisive results originally envisioned by
many of their proponents. Populations subjected to terror bombing did not rise up against their governments, demanding
capitulation in order to stop the carnage as Douhet had predicted. The British and German war economies proved
to be resilient under attack, the latter finally collapsing only late in the war when Allied planners who had been overly
influenced by the models of their own economies finally worked out which target sets truly represented its key vulnerabilities
(Brodie 1959; Mierzejewski 1988). Yet ultimately economic collapse did come in both Germany and Japan, through the combined
effects of bombing, blockade, losses on the battlefield, and Axis economic mismanagement (Bernstein 1995; Tooze 2007). In addition
to punishment, Pape declares that denialstrategies based on bombing war production do not work, nor
do decapitation or “strategic paralysis” strategies , such as those inspired by the theories of Warden (1989) and
John Boyd (Osinga 2006). He concludes therefore that strategic bombing as a whole is ineffective, and advises that air
power investment should concentrate on providing capabilities for interdiction, close air support, and other missions to defeat
enemy military forces. Attacks against enemy leaders have indeed failed with remarkable frequency to
produce their desired results (Hosmer 2001b), while strategic paralysis has proved to be an elusive goal – and one that in some
cases, such as the 2003 invasion of Iraq, might have been counterproductive in any event (Hosmer 2007). The situation is murkier
with respect to denial campaigns against war industry – for example, Pape’s claim that this was ineffective in the German case
depends on classifying the destruction of the German petroleum industry as something other than strategic attack because its
principal effect was to cripple German tactical and operational mobility, and he does not address the possibility that the ultimate
collapse of the German war machine could have been achieved earlier if different targeting choices had been made. However, it is
true that bombing enemy military production will be irrelevant to denial except in prolonged conflicts
against states that are consuming their war materiel and cannot import more of it. In the end, however, whether strategic
bombing can be decisive is less important than whether particular types of strategies are likely to succeed or fail, particularly since
the aircraft and weapons used for strategic attack today are in many respects not
fundamentally different from those needed for other types of air campaigns.

Conventional strategy is becoming obsolete

Haddick ’12 -- Managing Editor of the Small Wars Journal (Robert, writes the “This Week at War” column for
Foreign PolicyShipping Out, Foreign Policy, 8/31/12,
http://www.foreignpolicy.com/articles/2012/08/31/shipping_out?page=0,0)

For decades, aircraft carriers have been the tool-of-choice for crisis response . Policymakers in
Washington and four-star commanders in the field invariably have turned to carriers when they needed to signal U.S.
intentions, quickly reinforce military power, or provide decision-makers with options during a predicament. The Navy
has responded to the enduring demands of these customers by making the aircraft carrier strike group the prime
organizing feature of the Navy's surface and aviation forces, thereby drawing the biggest share of the service's
manpower, budget, support, and training resources. And until recently, the Air Force seemed happy to cede this
crisis-response role, because then it could focus on its own priorities. However, new and disruptive weapons
and technologies will soon upset long-standing assumptions and cozy inter-service arrangements. In
particular, the spread of long-range anti-ship missiles threatens the ability of aircraft carriers to
perform their traditional missions. What's more, these disruptions are occurring at the moment when
U.S. policymakers are under pressure to find cheaper ways of performing essential military missions.
And the Air Force could develop the technology and the long-range platforms to carry out many of the carrier's
missions at less cost. All these factors could force planners to rethink air power from first principles,
leading to stormy times for aircraft carriers and inter-service harmony. The aircraft carrier's combat
debut in the Pacific theater in 1941 instantly made the battleship obsolete. Aircraft carriers delivered more firepower,
over longer ranges, with more speed and flexibility, over a wider variety of targets at sea and ashore. After World War
II, the power of U.S. aircraft carriers forced adversaries to focus their naval spending on submarines rather than major
surface ships, a trend still visible today. Without enemy surface ships to sink, the Navy's carrier pilots focused on
projecting air power ashore, which they did against North Korea, Vietnam, Iraq (twice), and Afghanistan. Over the
past half-century, the Navy's carriers also became well-suited to crisis response. Carrier strike groups could typically
arrive at trouble spots within days and without the need for tedious negotiations with host countries over
permissions and basing rights. The Air Force was fine with this arrangement because, although its tactical fighter
wings could theoretically perform a similar role, the service's doctrine called for large, well-established, and well-
supplied bases from which it could reliably generate a high sortie rate. Such ponderous guidance could not deal well
with fleeting contingencies, many of which occurred in austere locations. But the proliferation of cheap but
deadly long-range anti-ship missiles promises to upset these assumptions and arrangements. For
example, China is putting anti-ship missiles on submarines, patrol boats, surface ships, aircraft, and trucks, giving it
the ability to dominate its nearby seas. For the price of a single major warship, China can buy
hundreds or even thousands of anti-ship missiles. And as it perfects its own reconnaissance
drones, China will be able to thoroughly patrol neighborhood waters, identifying targets for
these missiles. The Navy's aircraft carriers will come under pressure to retreat from this
missile zone. However, there is a limit to how far they can retreat while still remaining in the game. As large as U.S.
aircraft carriers are, they can only launch relatively small short-range fighter-bomber aircraft. For example, the F-35C,
the carrier version of the Joint Strike Fighter, has a combat radius of just 615 miles. Mid-air refueling can extend this
range. But refueling is not possible in hostile air space, and even with it, small fighters are constrained by the
physiological limits of their single pilot.
Alliances Answers
Frontline

Alliance collapse inevitable – Econ crisis

Zakheim ‘9 (Dov, Trustee – Foreign Policy Research Institute, “Security Challenged for the Crisis”, 3-11,
http://www.isn.ethz.ch/isn/Current-Affairs/Security-Watch/Detail/?ots591=4888CAA0-B3DB-1461-98B9-
E20E7B9C13D4&lng=en&id=98001)

The economic crisis is likely to further diminish the already weak appetite of allies and friends
both to increase or even maintain their current levels of defense expenditure, and to contribute
to coalition operations in Afghanistan. Few of our major allies and friends spend as much as 3 percent. of their GDP on
defense. Their GDPs, like ours, are in decline and in several cases, such as Japan, are declining at a far faster rate than ours.
Korea and Taiwan, like Japan, are suffering from a drop in exports, notably in the automobile sector. Iceland’s financial collapse
has received widespread attention. Economic constraints have at times been an excuse for allies not to do more for the
common defense of the West; today, that excuse is being buttressed by reality. Whether excuse or reality, the
net result will be exactly the same: the U nited S tates will be forced to bear an even heavier burden to
defend western interests, at a time when it will have fewer resources enabling it to do so. The case of the F-35 provides a
distinct example of the interplay between pressures on the US defense budget and alliance relationships. The F-35 program
could be one of those affected by the redistribution of defense spending priorities. There are eight countries that currently are
co-developing this aircraft, including key allies Britain, Canada and Australia, and many more planning to purchase it, among
them Israel, Singapore, and many of the European allies that currently fly F-16s. Any slowdown of the program will increase its
costs, and could put it beyond the purchasing power of several F-35 partners. It could also could embitter states that have
contributed to its development, furnishing them with yet another reason to be even less inclined to contribute to coalition
efforts if Afghanistan, and potentially elsewhere, than they are today.
Alt Energy Answers
Frontline

Can’t solve fast enough

Heinberg, 11 (Richard, fellow at the Post-Carbon Institute and renowned author, “Earth’s Limits”, 2/14/11,
http://www.postcarbon.org/article/254838-earth-s-limits-why-growth-won-t-return)

Can other energy sources replace fossil fuels? Some alternatives, such as wind, are seeing rapid growth rates, but still account
are unlikely to
for only a minuscule share of current global energy supplies. Even if they maintain high rates of growth, they
become primary energy sources in any but a small handful of nations by 2050. Report coverIn 2009 Post
Carbon Institute and the International Forum on Globalization undertook a joint study to analyze
18 energy sources (from oil to tidal power) using 10 criteria (scalability, renewability, energy density, energy returned on
energy invested, and so on). While I was the lead author of the ensuing report (Searching for a Miracle: Net Energy Limits and the
Fate of Industrial Societies), my job was essentially just to synthesize original research and analysis from many energy experts.[22] It
was, to my knowledge, the first time so many energy sources had been examined using so many essential criteria. Our
conclusion was that there is no credible scenario in which alternative energy sources can
entirely make up for fossil fuels as the latter deplete. The overwhelming likelihood is that, by 2100, global society will
have less energy available for economic purposes, not more.[23] Here are some relevant passages from that report: A full
impossible over
replacement of energy currently derived from fossil fuels with energy from alternative sources is probably
the short term; it may be unrealistic to expect it even over longer time frames. . . . [U]nless energy
prices drop in an unprecedented and unforeseeable manner, the world’s economy is likely to become increasingly energy-
constrained as fossil fuels deplete and are phased out for environmental reasons. It is highly unlikely that the entire world will ever
reach an American or even a European level of energy consumption, and even the maintenance of current energy consumption
levels will require massive investment. . . . Fossil
fuel supplies will almost surely decline faster than
alternatives can be developed to replace them. New sources of energy will in many cases have lower net
energy profiles than conventional fossil fuels have historically had, and they will require expensive new
infrastructure to overcome problems of intermittency.[24] Some other studies have reached different, more
sanguine conclusions. We believe that this is because they failed to take into account some of the key criteria on which we focused,
including the amount of energy returned on the energy that’s invested in producing energy (EROEI). Energy sources with a low EROEI
cannot be counted as potential primary sources for industrial societies.[25] As a result of this analysis, we believe that the world has
reached immediate, non-negotiable energy limits to growth.[26]
Amazon Answers
Frontline

Amazon strong and resilient

Glusing 9 (Jens, Spiegel Online, " Can New Growth Save the Amazon Rainforest?," 8/13,
http://www.spiegel.de/international/world/0,1518,642199,00.html)

Is the Amazon rainforest recovering? New studies suggest that the long-term consequences of
deforestation may not be as bad as predicted , as vegetation makes a comeback on abandoned
agricultural land. Felipe Garcia's shack backs up against a wall of forest. "My neighbors abandoned their farm seven years
ago," says Garcia, a farmer. "Now the jungle has taken over their property once again." He strokes his round
belly, and says: "And if I don't till my field, it'll look the same way in a few years." Garcia is a member of the Ngöbe tribe, the largest
indigenous group in Panama. He is one of the few residents of the small town of Chilibre who still makes a living farming. Most of
the local farmers abandoned their farms long ago. "Farming is too hard work for the young people," Garcia complains. "They prefer
to work in the capital." Panama City's office buildings are about an hour's drive away. Chilibre has become a bedroom community as
residents choose to commute to the capital. It has also become a field laboratory for botanists and ecologists fascinated by the
dense vegetation that has returned to abandoned farms in the former Canal Zone within just a few years. Undervalued Growth In
the past, scientists scorned the "secondary forests," as the new growth is called . There is no doubt that
they are not nearly as spectacular as the species-rich primary forests, with their giant trees which are often centuries old, and that
they are not home to nearly as many animal and plant species. But
now a growing number of biologists are
interested in this previously ignored vegetation. According to a United Nations study, the ecological
importance of these new forests, which are "growing dramatically" all over the world, is
"undervalued."

No impact to Amazon

Morano and Washburn 2k (Marc and Kent, Producers of American Investigator's "Amazon Rainforest: Clear-Cutting the
Myths" “Shaky Science Behind Save-Rainforest Effort”, 6-26, http://www.worldnetdaily.com/news/article.asp?ARTICLE_ID=17543)

Another familiar claim of the environmentalist community is that the Amazon constitutes the "lungs of
the earth," supplying one-fifth of the world's oxygen. But, according to Antonio Donato Nobre of INPE, and other
eco-scientists, the Amazon consumes as much oxygen as it produces, and Stott says it may
actually be a net user of oxygen. "In fact, because the trees fall down and decay, rainforests
actually take in slightly more oxygen than they give out ," says Stott. "The idea of them soaking up
carbon dioxide and giving out oxygen is a myth . It's only fast-growing young trees that actually take
up carbon dioxide." Stott maintains that the tropical forests of the world are "basically irrelevant" when
it comes to regulating or influencing global weather. He explains that the oceans have a much greater
impact. "Most things that happen on land are mere blips to the system, basically insignificant,"
he says.

Too many alt causes- conclusive study

Lane 8 (Jim, “World Wildlife Fund Exonerates Ethanol on Amazonian Deforestation and Food Production”, Biofuels Digest, Lexis)
May 27, 2008 (Biofuels Digest delivered by Newstex) -- The World Wildlife Fund has concluded, in a new study profiled on the
"ethanol production is not having a significant impact on food production, and that it is not contributing
BBC, that
to deforestation in the Amazon." The report concludes that sugar cane ethanol has a positive impact on the
environment. The report called for strict monitoring to protect remaining rainforest areas." In Brazil, the federal government
announced a crackdown on illegal deforestation in the Amazonian rainforest. Biofuels producers have been accused of causing
deforestation, however the authorities are targeting soy farmers, cattle ranchers and illegal timber operators in 36 pockets
where increased deforestation has occurred. An emergency meeting of the Brazilian cabinet had been been called by President
Luiz Inà cio Lula da Silva after a 50 percent jump in deforestation rates, following a steady three-year decline. A
German
academic has analyzed the factors that are causing deforestation of the Amazon, and concluded
that sugarcane ethanol production in south-central Brazil is not pushing cattle and soy farming into the Amazon region. Peter
Zuurbier, Associate Professor and Director of the Wageningen UR Latin America Office, said that the problem
is unclear land titles, unscrupulous timber companies, and poor soil conservation practices
by cattle ranchers. He said that after illegal clear cutting by timber companies, the land is occupied by nomadic cattle
herds that, over a period of 3 to 4 years, ruin the thin soil of the Amazon areas, which causes fertilizer-based soy farming to be
brought into the area to improve productivity. Researchers say that Amazonian deforestation has increased in pace in 2007 and
is likely to rise throughout 2008. Carlos Nobre, a scientist with Brazil's National Institute for Space Research, said that 2,300
square miles of forest had been converted to farmland in the past four months, compared with 3,700 square miles in the 12
months ending last July.

Cattle ranching outweighs and can’t be stopped

Felsinger 9 (Alex, Writer @ Planet Save, " 80% of Amazon Deforestation Stems from Cattle Ranching," Jan 29,
http://planetsave.com/2009/01/29/80-percent-of-amazon-deforestation-stems-from-cattle-ranching-2/)

Greenpeace Brazil has released a report at the World Social Forum in Belém showing that up to 80 percent
of deforestation in the Amazon rainforest is due to an increase in raising cattle for human
consumption. Brazil has quickly become the largest exporter of beef in the world, but they are not satisfied with their
current market share and plan to increase production. The plan flies in the face of their supposed commitment to tackle
climate change. The country is currently the fourth biggest emitter of greenhouse gases, 75% of which stem from deforestation.
Source: Planetsave (http://s.tt/12tFw) The map of Brazil above shows the concentration of cattle, mostly focused on the Mato
Grosso region in the southwest. Greenpeace used a unique technique to prepare this and other maps in the report: Our Amazon
team used specialized techniques to analyze and compare satellite images of vegetation (or lack thereof) on the ground, and data
showing the growth of infrastructure, such as roads and agribusiness. It is the first time that these two types of data have been put
together in this way. Our maps in the report Amazon cattle footprint, clearly show how much former rainforest is now used for cattle
pastures in Mato Grosso. Unfortunately, similar as in the United States, the
government fully backs the
agriculture industry with many politicians firmly in the pockets of agribusiness . They’ve even recently
proposed legislation to prevent environmental groups from setting up in the Amazon. Source: Planetsave (http://s.tt/12tFw)
---Amazon- No Impact

Amazon is safe – their evidence is hype

Morano and Washburn 2k (Marc and Kent, Producers of American Investigator's "Amazon Rainforest: Clear-Cutting the
Myths" “Shaky Science Behind Save-Rainforest Effort”, 6-26, http://www.worldnetdaily.com/news/article.asp?ARTICLE_ID=17543)

Today the environmental scientist and leader of a group called Greenspirit has a new cause -- alerting the public to what he calls
the "myth" that the Amazon rainforest is endangered by development and deforestation. "The Amazon is actually the
least endangered forest in the world," states Moore in American Investigator's television newsmagazine
documentary, "Clear-cutting the myths," hosted by former CBS and CNN newsman Reid Collins. Moore explains that,
in the 20 years of warnings about deforestation, "only 10 percent of the Amazon has been
converted to date from what was original forest to agriculture and settlement." The finding that the Amazon
rainforest threat is a myth based on bad science and political agendas -- especially by unlikely critics
such as Moore, other scientists and inhabitants of the region -- is not expected to sit well with a movement that has enlisted
schoolchildren throughout the United States and celebrities ranging from Sting to Alec Baldwin to Chevy Chase to Tom Jones
and Tony Bennett. And which has also raised tens of millions of dollars for environmental activist groups. "This is where I really
have a problem with modern-day environmentalism," says Moore. "It confuses opinion with what we know to be true, and
disguises what are really political agendas with environmental rhetoric. The fact of the matter is: There
is a larger
percentage of the Amazon rain forest intact than there are most other forests in this
world."

No species extinction and no spillover

Morano and Washburn 2k (Marc and Kent, Producers of American Investigator's "Amazon Rainforest: Clear-Cutting the
Myths" “Shaky Science Behind Save-Rainforest Effort”, 6-26, http://www.worldnetdaily.com/news/article.asp?ARTICLE_ID=17543)

Most of these estimates are rooted in the research of Harvard's Edward O. Wilson, featured by Time magazine as an
environmental "hero" in its special Earth Day 2000 edition. In the accompanying article, Wilson argues passionately to
stem the tide of extinctions "now 100 to 1,000 times as great as it was before the coming of humanity" -- neglecting to mention
that his estimates of 50,000 extinctions per year are based on his own computer models. "There is no
scientific basis for saying that 50,000 species are going extinct ," says Greenspirit's Moore. "I want a list
of Latin species." Moore maintains no one can name these species that are said to be going extinct. "The only place
you can find them is in Edward O. Wilson's computer at Harvard University. They're actually electrons on a hard drive," Moore
states. When asked if he can name a single species of the 50,000 that are said to go extinct, Keating admits: "No we cannot,
because we don't know what those species are." Moore is flabbergasted by such statements. "You're telling me that I'm
supposed to prove that those species didn't go extinct when they're not there anymore and we never knew they were there in
the first place?" Moore asks rhetorically. "That's impossible. I don't know how Wilson can truck out the number 50,000 and
keep a straight face." Stott agrees that the
focus on species loss is misguided from a scientific point of
view. "The earth has gone through many periods of major extinctions, some much bigger, let me
emphasize, than even being contemplated today and 99.9999 percent (of all species) and I wouldn't know the
repeating decimal have gone extinct. Extinction is a natural process," he asserts.
---Amazon- Alt Causes

Alt cause- Mining

Butler 6 (Rhett, “Deforestation in the Amazon”, Monga Bay, http://www.mongabay.com/brazil.html)

Mining has impacted some parts of the Amazon Basin. During the 1980s, over 100,000 prospectors
invaded the state of Para when a large gold deposit was discovered, while wildcat miners are still active in the
state of Roraima near the Venezuelan border. Typically, miners clear forest for building material, fuelwood
collection, and subsistence agriculture.

Alt cause- Climate change

Lovejoy 8 (Thomas, President – The Heinz Center, 2-14, Impact Of Tropical Forest Destruction On The Climate, CQ Congressional
Testimony, Lexis)

In addition climate change can have a major impact on the Amazon. Current estimates from the
Inter-Governmental Panel on Climate Change (IPCC) are that at 4.5 degrees Fahrenheit increase in global
average temperature, Amazon dieback will occur -- not the entire Amazon but large parts of it. Indeed, the
most severe drought in recorded Amazon history occurred in 2005, and was associated with
changes in the circulation of the Atlantic that could, in a sense, have been a preview of what climate change
could bring. This of course would be a positive feedback releasing yet more greenhouse gases to the atmosphere.
Amazon dieback is not that far a distant possibility : at current concentrations, we are currently
automatically slated for 2.7 degrees Fahrenheit of increase in average global temperature because of the lag time between
increase in greenhouse gas concentrations and radiant energy being trapped by them, and most projections bring us
close to 3.6 degrees Fahrenheit by 2030.
Angola Economy Answers
Frontline
Angola’s economy is resilient- oil isn’t key

World Finance ’12 [“Investment in Angola,” http://www.banking-awards-2012.worldfinance.com/investment-in-angola]

Today, investment in Angola is by no means restricted to the oil sector . Angola is one of only four
African countries in which annual foreign investment totals more than $3bn. Angola’s government
has made clear that it will continue to focus on major investment projects that develop
infrastructures in areas such as roads, railways, housing and retail developments. The sheer
resilience of the Angolan economy is also seen as offering numerous opportunities to the
commercial sector, supported by the domestic market, which clearly has even more growth
potential. According to the IMF, the per capita GDP in Angola grew from $585 in 2000 to $5,061 in 2011, and could attain $6,392
by 2016.

Angolan economy is resilient- years of turbulence proves no impact

Sogge ’09 [David, worked in the field of foreign aid for more than 30 years in both aid organisations and as an independent
advisor, visiting professor at the Universities of Harvard and Princeton, and a fellow of the Transnational Institute in Amsterdam,
“Angola: 'Failed' yet 'Successful',” http://www.gsdrc.org/go/display&type=Document&id=3266]

The combined impact of violent conflicts, rising revenues and the massive political influence of
the oil industry in Western democracies has made oil decisive in Angola’s political economy.
Oil dependence is commonly thought to make states unstable. But the Angolan case offers no
clear confirmation of this hypothesis; on the contrary it is consistent with statistical studies indicating that oil wealth tends
to prolong and stabilise autocratic regimes. In terms of coercive power over national territory, Angola’s state is now more
resilient than fragile. The political game may be played in part by nontransparent, arbitrary and personalised rules, but there
is at present no alternative vision. In terms of control over economic life, the state shows resilience in a
number of key respects, although abundant petrodollars may mask structural weaknesses.
Arctic Biodiversity Answers
Frontline

Artic biodiversity loss doesn’t spillover

National Academy of Engineering 03 – American National Academies (“Cumulative Environmental Effects of Oil and
Gas Activities on Alaska’s North Slope”, 2003; < http://dels-old.nas.edu/dels/rpt_briefs/north_slope_final.pdf>)//AB

Alaska’s North Slope is underlain by permafrost— a thick layer of earth material that stays
frozen year round. The permafrost is covered by a thin active layer that thaws each summer and
supports plant growth for a brief period. If permafrost thaws, the ground surface and the
structures it supports will settle. To minimize disruption to the ground surface, the North Slope
industrial infrastructure is specially built—pipelines are generally elevated rather than buried,
and roads and industrial facilities are raised on thick gravel berms. For a variety of reasons,
nearly all of the roads, pads, pipelines and other infrastructure ever built are still in place. The
environmental effects of such structures on the landscape, water systems, vegetation, and
animals are manifest not only at the “footprint” itself (physical area covered by the structure)
but also at distances that vary depending on the environmental component being affected. The
petroleum industry continues to introduce technological innovations to reduce its footprint,
for example, directional drilling and the use of ice roads and pads, drilling platforms, and new
kinds of vehicles. For some areas of concern, the committee found no evidence that effects
have accumulated. For example, despite widespread concern regarding the damaging effects of
frequent oil and saltwater spills on the tundra, most spills to date have been small and have
had only local effects. Moreover, damaged areas have recovered before they have been
disturbed again. However, a large oil spill in marine waters would likely have substantial
accumulating effects on whales and other receptors because current cleanup methods can
remove only a small fraction of spilled oil, especially under conditions of broken ice.
Arctic War Answers
Frontline

Economics trump—competition won’t escalate

IISS ’12, (International institute for Strategic Studies, 11/28/12, Russia in the Arctic: Economic Interests Trump Military
Ambitions,
www.realclearworld.com/articles/2012/11/28/russia_in_the_arctic_economic_interests_trump_military_ambitions_100373-2.html)

A recent mission by a Russian nuclear submarine to the floor of the Arctic Ocean has threatened to reignite
the media narrative that regional disputes over the right to unlock the economic potential of the Arctic could result in
military confrontation. But it is their mutual economic interests that mean that the five Arctic coastal
states are motivated to pursue legal and diplomatic avenues to achieve their aspirations, and
have no desire to jeopardise the status quo. During the Russian operation, known as Arktika-2012, geological
material was collected from one of the two underwater mountain ranges that extend from the Russian landmass towards the North
Pole. Russia wants to prove that the Lomonosov and Mendeleev ridges are extensions of Russia's continental shelf and part of the
Eurasian plate, which, according to the current legal framework, would allow Russia exclusive rights to any potential future
resources under the seabed. The details of the project were intended to remain secret, but in November 2012 several news stories
about the submarine appeared, citing a Russian defence ministry source. Despite efforts to build good regional relations among
Arctic countries, Russia's neighbours do have concerns about its increasing military presence in the Arctic and its sometimes
assertive, anti-Western rhetoric. However, considered
in the wider context of Russia's post-Cold War military re-
development, its Arctic positioning is not as confrontational as it may seem. The Arctic is a key part of Russia's reassertion of what it
sees as its rightful place in international affairs, and it has far greater territory, presence and capability in the Arctic than its neighbours. Rich in hydrocarbons, the region was highlighted in Moscow's Arctic policy of 2008 as the country's primary source of energy for
the twenty-first century: approximately 15% of the country's GDP and 25% of its exports come from there, while 80% of the gas in the Arctic lies within Russia's exclusive economic zone (EEZ). There are major on-shore gas installations, and plans to further develop
off-shore drilling, though these have met with some logistical difficulties with international partners. Along with hydrocarbons, maritime transport is a major economic development priority. The Northern Sea Route, the new shipping route most likely to become
commercially viable in the coming decades as the summer ice recedes, and promises to connect Europe and Asia, runs through Russia's territorial waters or EEZ. However, the lack of infrastructure along the route will hold back the development of commercial
shipping. Arktika-2012 The planting of a titanium Russian flag on the floor of the Arctic Ocean during a previous mission, Arktika-2007, created a powerful image, but it had no legal significance. It did, however, pique international interest in the Arctic and encouraged
a media narrative about competition over the region's territory and resources. The objective of Russia's latest mission, Arktika-2012, was to prove that its landmass extends to the North Pole by drilling into the sea floor to collect rock samples for scientific analysis. In
September, the Kalitka, a Losharik-class nuclear-powered auxiliary submarine, was used to guide the Kapitan Dranitsyn and Dickson ice breakers in drilling three boreholes at two different sites on the Mendeleev ridge, collecting over 500kg of rock samples. This was
the first known mission for the Kalitka. Equipped with space-station-grade air and water regeneration systems, the submarine can remain submerged for months. During this operation, it remained 2.5-3 kilometres below the surface for 20 days. (Though the battery-
powered civilian Mir stations used in the Arktika-2007 expedition can also operate at such depths, they can only stay submerged for 72 hours.) It was mounted to the underside of a larger nuclear-powered auxiliary submarine (the Orenburg, a redesigned Kalmar or
Delta III stretch) to transport it to the drilling site and was supported by the larger boat during the operation. Continental-shelf claims and maritime borders In collecting the geological samples, Russia was responding to a request by the United Nations Commission
on the Limits of the Continental Shelf (CLCS) that it submit supporting evidence for its claim to a broad continental shelf that extends beyond its landmass under the Arctic Ocean. The five Arctic coastal states - Russia, Canada, the United States, Norway and Denmark
- in 2008 issued a joint statement, known as the Ilulissat Declaration, committing to settling territorial claims diplomatically, using existing legal mechanisms. The primary legal body for maritime border delimitation in the Arctic is the UN Convention on the Law of the
Sea (UNCLOS), which rules that maritime countries' EEZs extend 200 nautical miles from their shore. The CLCS covers continental-shelf claims beyond that zone, up to a maximum of 350nm. hould it be determined that the claimed portion of the ocean floor has the
same geological makeup as the Russian continental landmass, then the CLCS will rule that it is an extension of Russia's continental shelf, granting Russia sovereign rights to resources under the seabed up to 350nm from its shoreline. In a submission to CLCS in 2001,
Russia claimed the Lomonosov and Mendeleev ridges, as well as the seabed below the North Pole. If this claim is verified, Russia's continental shelf would be extended by 1.2 million square kilometres, and give Russia exclusive rights to the resources below the
seabed. Russia's Ministry of Natural Resources and Environment tested the samples and found that they did match the make-up of its landmass. Its next submission to CLCS will likely be ready by the end of 2013, to be submitted in 2014. (The CLCS's ruling will be
final and binding.) Following Russia's lead, all Arctic countries are preparing to submit claims to the CLCS: Norway's is already complete, while the United States is going ahead with its preparations even though it has not yet ratified UNCLOS and is, therefore, not a
party to its adjudication. There is considerable support for acceding to UNCLOS within the US State Department and Department of Defense, and the claim is being put together in anticipation of eventual ratification. Further sources of friction between Arctic nations
on the issue of maritime border delimitation include bilateral disagreements between the US and Canada, and between Denmark and Canada, and a trilateral dispute between Russia, Canada and Denmark. The 2,000km-long Lomonosov ridge, meanwhile, is
particularly contentious: Canada claims that the ridge is an underwater extension of Ellesmere Island, while Denmark argues that it is an extension of Greenland's landmass. The US, in turn, has stated that Lomonosov is an oceanic ridge and thus cannot be an
extension of any country's continental shelf. However, joint efforts to map the seabed in more detail are under way. In 2011, the US and Canada concluded a five-year mapping operation of their continental shelf, and Canada and Denmark conducted a seismic
exploration in 2007. In September 2012, Russian President Vladimir Putin called for the creation of a joint scientific council with Canada to allow potentially overlapping continental-shelf claims to be discussed, and Canada responded positively. In addition to the joint
surveys, scientists and officials from Arctic nations have met annually since 2007 to discuss issues related to their continental shelves, which may overlap. Of the known oil and gas deposits in the region, 97% lie within the Arctic states' EEZs, meaning there is not
much competition between states for access to them. Most of these deposits may not be recoverable in the near term, due to the difficulties of hydrocarbon extraction in remote, harsh and ecologically sensitive environments. But in making maximal continental-
shelf claims, Arctic states are hedging that there may be new discoveries or technological developments that will make these deposits more accessible in future. The area that the Russian Federation is claiming is not thought to be rich in hydrocarbons, but does
include the North Pole, which has symbolic value. Receive email alerts International Institute for Strategic Studies Arctic Russia Cooperation likely to produce best results Though the CLCS will rule on the extent of the continental shelf in the Arctic, it will not draw the
boundaries within the area designated as continental shelf. It is for the countries concerned to come to an agreement on the division of that continental shelf, and the outer boundaries of their national claims, as Russia and Norway did over their Barents Sea border
in 2010. However, the CLCS requires that conflicting claims be resolved before it makes its recommendation on the boundary between international oceanic space and national jurisdiction. Differences of opinion among Arctic states over the extent of their shelves
could be resolved by discussing CLCS claims before they are submitted, reaching mutually agreeable findings and submitting parallel or joint applications. Each country submits its data to the CLCS confidentially, and its meetings are held in private. UNCLOS scholars
believe that Russia has been in communication with Canada and possibly Denmark on the division of their respective claims to the Lomonosov ridge, but there is no information in the public domain about these negotiations. Potential joint submissions are likewise
not being prepared openly. Military activity in the Arctic Apart from its economic potential, the strategic importance of the Arctic is not lost on any regional state and all of them have increased the number and complexity of their military exercises there. After a long
period of stagnation, Russia is devoting considerable resources to rebuilding and streamlining its military forces. Military exercises have increased for all of the Russian military, including the Northern Fleet, which is based on Russia's northwest coast, inside the Arctic
Circle, and is the main locus of its sea-based nuclear deterrent. Its air assets include long-range bombers and maritime reconnaissance aircraft. On the ground, its capabilities include naval infantry and an army brigade on the Kola Peninsula. In 2009, Russia
announced its plans to develop further specialised forces to protect its Arctic territory. Russia expressed its unease about the further militarisation of the region in 2009, when Norway moved its armed forces' headquarters to Reitan, in the north of the country. It
considered Cold Response - a 15-country exercise that took place in northern Norway and Sweden in March 2012 and involved 16,300 troops - a provocation, and reacted with an exercise involving its 200th motor rifle brigade from Murmansk, including T-80 tanks
with gas-turbine engines suited for the Arctic climate. However, Russia has also undertaken joint exercises with both Norway and the US. Confidence-building measures such as these, as well as forums to openly address security matters, have been considered
constructive. Moscow's 2008 Arctic policy placed its emphasis not on a military build-up but on maritime law enforcement duties. It also focused on enforcing shipping and fishing regulations, and providing search-and-rescue capabilities. Russia's northern border
includes almost 40,000km of coastline, which is becoming more exposed as summer sea ice retreats and economic activity increases. Though Russia has a coastal border guard, only a few of its ships are suitable for Arctic operations, and its ability to monitor its coast
and EEZ, and enforce regulations, is limited. As with other Arctic countries, meeting its constabulary requirements is a more immediate and pressing challenge than rebuilding military structures to tackle comparatively notional security threats. Mutual economic

Despite the signs of heightened military activity in the region, the greatest stabilising factor in the
interest

region is mutual economic interest, and the points of friction around border delimitation and military activity are

unlikely to override this. Russia, in particular, is eager to open up the Northern Sea Route for trade purposes, as it
perceives great potential for commerce along its otherwise remote northern coast and the possibility of imposing transit fees for
shipping through the route. Russia's relations with NATO and the US will have a major impact on levels of cooperation or mistrust in
the Arctic. Rebuilding its decaying infrastructure and managing the Northern Sea Route that can connect Europe and Asia
will advance Russia's strategic goals in the region more effectively than an unnecessary military
build-up.
No arctic conflict

-cooperation increasing

Fries ‘12 (Tom Fries, Nonresident Senior Fellow at the Arctic Institute, “Perspective Correction: How We Misinterpret Arctic
Conflict,” http://www.thearcticinstitute.org/2012/04/perspective-correction-how-we.html, April 18, 2012)

War and conflict sell papers -- the prospect of war, current wars, remembrance of wars past. Accordingly, a
growing cottage industry devotes itself to writing about the prospect of conflict among the
Arctic nations and between those nations and non-Arctic states, which is mostly code for “China.” As a follower of Arctic
news, I see this every day, all the time: eight articles last week, five more already this week from the Moscow Times, Scientific
American or what-have-you. Sometimes this future conflict is portrayed as a political battle, sometimes military, but the
portrayals of the states involved are cartoonish, Cold-War-ish...it’s all good guys and bad guys.
I’m convinced that this is nonsense, and I feel vindicated when I see the extent to which these countries'
militaries collaborate in the high North. From last week's meeting of all eight Arctic nations'
military top brass (excepting only the US; we were represented by General Charles Jacoby, head of NORAD and
USNORTHCOM) to Russia-Norway collaboration on search & rescue; from US-Canada joint
military exercises to US-Russia shared research in the Barents...no matter where you look,
the arc of this relationship bends towards cooperation.

Arctic cooperation even if military build ups

Rogers, ’12
(Will, Bacevich Fellow at the Center for a New American Security. “New Study Highlights Military Capabilities in the Arctic,” CNAS,
http://www.cnas.org/blogs/naturalsecurity/2012/04/new-study-highlights-military-capabilities-arctic.html.)

A new report from the Stockholm International Peace Research Institute (SIPRI) finds that the build-up of
Arctic military capabilities is limited, with few indications that conflict is looming. According to the
study, all five Arctic states – Canada, Denmark, Norway, Russia and the United States – have increased their military
capabilities in the Arctic in recent years in response to growing accessibly to the region owed largely to climate
change.¶ Some of the increased military activity is likely a response to the changing geostrategic environment that
will make military capabilities increasingly important for power projection that states need to maintain in order to
secure access to lucrative natural resources and other national interests. According to the SIPRI study, for example,
“Russia’s Arctic policy underlines the importance of the Arctic as a principal source of natural resources by 2020,” and
“Denmark’s defence policy underlines the changing geostrategic significance of the Arctic.”¶ Despite the
increased deployment of military assets, Arctic states are continuing to pursue new avenues of
cooperation, mollifying concerns – at least for the time being – that tensions will worsen as the region
becomes more accessible. Last year, the Arctic Council – an intergovernmental forum for Arctic states to address
challenges in the High North – hosted a high-level forum that led to an agreement for countries in the
region to increase search-and-rescue cooperation given the growing concerns surrounding increased eco-
tourism and commercial shipping that could portend future law enforcement challenges. Some states’ newly
deployed military assets are intended for search-and-rescue purposes, according to the SIPRI study.
Canada, for example, will replace older C-130s and other aging aircraft with 17 new search-and-rescue aircraft in the
next several years.

No Arctic war

Young ’11 (Professor – Institutional and International Governance, Environmental Institutions @ UCSB, Arctic expert, PhD –
Yale, ‘11 (Oran R, “The future of the Arctic: cauldron of conflict or zone of peace?” International Affairs 87:1, p. 185-193)

Popular accounts of the Arctic’s jurisdictional issues are regularly couched in terms of provocative
phrases like the afore-mentioned ‘who owns the Arctic’ or ‘use it or lose it’. But these phrases turn out to be highly
misleading in this context. There are virtually no disputes in the Arctic regarding sovereignty over
northern lands; no one has expressed a desire to redraw the map of the Arctic with regard to the terrestrial
boundaries of the Arctic states. Most of the disagreements are to do with jurisdiction over marine areas
where the idea of ownership in the ordinary sense is irrelevant. While some of these disagreements are of long
standing and feature relatively entrenched positions, they are not about establishing ownership, and they do not
indicate that some level of ‘use’ is required to avoid the erosion of sovereignty. There is little prospect that these
disputes will spawn armed clashes. As both Michael Byers and Shelagh Grant make clear in their excellent
analyses of Arctic sovereignty, recent efforts to address matters involving sovereignty in the Arctic are marked by
a spirit of rule-based problem-solving, rather than an escalating spiral of politically charged claims and
counterclaims. The process of delineating jurisdictional boundaries regarding the seabed beyond the limits of
Exclusive Economic Zones (EEZs) is taking place in conformity with the rules and procedures set forth in Article 76 of
UNCLOS. Norway and Russia have signed an international treaty resolving their differences regarding
jurisdictional boundaries in the Barents Sea. There are signs that Canada and the United States are interested in a
similar approach with regard to the Beaufort Sea. The Russians, whose much ballyhooed 2007 initiative to plant the
Russian flag on the seabed at the North Pole is widely discussed in the books under review, have acted in conformity with
the relevant rules of international law in addressing jurisdictional matters and repeatedly expressed their readiness to move
forward in a cooperative manner in this realm. There are, of course, significant sensitivities regarding the legal status of the Northern
Sea Route and especially the Northwest Passage. But given that commercial traffic on these routes is likely to be
limited during the near future, and that the use of these routes will require the active cooperation of the
coastal states, regardless of their formal legal status, opportunities arise for devising pragmatic arrangements
governing the use of these waterways. The progress now being made regarding the development of a mandatory
Polar Code covering Arctic shipping is good news. The fact that ‘hot spots’ in the search for oil and gas in the
Arctic are located, for the most part, in areas that are not subject to jurisdictional disputes is also helpful.
Overall, it seems fair to conclude that the Arctic states are living up to their promises to deal with jurisdictional
issues in the region in a peaceful manner.

All studies prove


IN ‘9 (Ice News – Iceland national news source, 11/29/’9 (“Military dispute over Arctic resources unlikely,”
http://www.icenews.is/index.php/2009/12/29/military-dispute-over-arctic-resources-unlikely/)

The natural resources of the Arctic region are unlikely to lead to any military conflict in the region
according to new research by the Fridtjof Nansen Institute (FNI) of Norway. The study further found that a
diplomatic solution to any dispute resolution is far likelier and more rational than armed action.
In a statement posted on their website, FNI downplays the threat of lawlessness in the Arctic. “Contrary to the general
picture drawn by the media and some commentators over the last couple of years, the Arctic region does not
suffer under a state of virtual anarchy. The era when states could claim rights to territory and
resources by simply planting their flag is long gone,” the statement reads. International law largely
regulates any issues in the Arctic region that have been dubbed “security policy challenges” in the past, SikuNews reports,
while adding that the report claims that regional states prefer an observation-based approach over any
desire for military conflict. Those issues which arise that are not clearly governed by international law in respect to resolution
procedures are generally only minor, say researchers. The focus of the majority of the case studies contained in the
findings was on relations in the Barents Sea, between Russia and Norway. These included the management of ocean resources, the
status of the continental shelf and waters around Svalbard and the delimitation of unresolved boundaries. These case studies
collectively found
little or no threat of armed dispute likely and concluded that the Arctic region has
little rationale or legal space for military conflict resolution.
---Arctic War- No Conflict

No Arctic war- cooperation high now

Aruliah ’12 (9-28 -- Asia Pacific Foundation of Canada Post-Graduate research fellow (Charles, "The Cold Truth: Why the
Arctic isn’t the same as Asia’s island disputes," iPolitics, 9-28-12, www.ipolitics.ca/2012/09/28/charles-aruliah-the-cold-truth-why-
the-arctic-isnt-the-same-as-asias-island-disputes/, accessed 10-7-12, mss)

But if one looks past such public displays, it becomes increasingly clear that, unlike territorial disputes in
Asia, Arctic relations remains primarily characterized by cooperation rather than conflict. And here’s
why: First and foremost, despite the fact that in August, the Arctic melted at an unprecedented 91,700 km2 per day, it
remains one of the harshest environments on the planet. While it’s true that sailing through the Arctic could
potentially cut the distance for international shipping in half, it can only be achieved during the late summer melt – less than one
quarter of the entire year. Even then, ships must be wary of left-over multi-year ice, icebergs, and floating growlers, some of which
can be as hard as concrete. Ships hoping to traverse the passage will still require constant monitoring and icebreaker escorts, all of
which incur significant additional costs. This is why Arctic states are closely cooperating in areas such as Search and
Rescue. Contrast this with the significantly busier Malacca Straits located near the South China Seas, which draws about 50 percent
of the world’s oil tanker traffic, and saw some 70,000 transits in 2007 (compared with the Northwest Passage’s 26 in 2010). The East
China Sea too, remains a busy waterway and central hub located between some of the world’s busiest ports. In general, the
cost
of controlling Arctic shipping just isn’t worth the risk of provoking conflict through the exercise of such
dominance. Secondly, unlike the Arctic, territorial disputes in East Asia remain intimately linked to historical grievances and
nationalistic passions from the region’s conflict-ridden past. South Korea attributes Japanese claims to the Dokdo/Takeshima islands
to its imperial annexation of the Korean peninsula in 1905. China too has argued that the Senkaku/Diaoyu islands were historically
administered by China, until the territories were ‘unfairly’ redistributed to Japan by the post-war powers following Japan’s defeat in
the Second World War. Throw in other long-standing disputes like China-Taiwan relations and it’s no wonder why mobs of zealous
citizens have taken to the streets in anger over supposed incursions of national territory. In the Arctic, the main sources
of territorial aggravation exists between long-standing allies (United States and Canada in the Beaufort Sea),
and peace-minded Middle Powers (Canada and Denmark over Hans Island) whose idea of conflict involves
marking territory with a bottle of Schnapps or Canadian Club. Even the ‘Great Power’ of the region, Russia, has
gone through great lengths with Norway to settle a 40 year territorial dispute in the Barents Sea which has also
laid the foundations for future joint economic ventures in the area. On the contrary, nationalist rhetoric
may actually be driving Arctic cooperation . The encroachment of Arctic ‘outsiders’ such as the EU,
China, Japan, South Korea and India, some of whom have argued that the Arctic be declared as ‘a common heritage
of mankind’ has led Arctic states, who fear losing territorial integrity, to adopt an ‘us vs. them’ mindset. This has
partly resulted in the denial of these countries’ applications for permanent observer status in the exclusive Arctic council, the
preeminent intergovernmental forum on the Arctic. Finally, the
prominence of scientific/environmental issues and
community sustainability in Arctic discussions has mitigated potential nationalistic posturing. The Arctic
Council remains geared towards Arctic preservation and studying the effects of environmental change – issues
where international scientific collaboration is the norm. Furthermore, the Arctic Council’s endeavor to promote the well-being of
indigenous communities, as evidenced by the inclusion of six indigenous organizations as permanent participants in Council
discussions, means that Arctic issues are dispersed amongst a variety of actors, and are not the sole realm of national governments.

Military build-up has been limited to defense of territory --- cooperation is still the norm.

Wezeman ’12 (Siemon- Senior Fellow with the SIPRI Arms Transfers Programme. “Increased military capabilities in
the Arctic reflect border demarcations,” Stockholm International Peace Research Institute,
http://www.sipri.org/media/pressreleases/26-mar-increased-military-capabilities-in-the-arctic-region-reflect-
territorial-consolidation.

The background paper, entitled Military Capabilities in the Arctic, is based on the findings of SIPRI Senior Researcher
Siemon Wezeman and shows that while governments of the five Arctic states have made protection of their Arctic
territory a priority, the military build-up is limited. ¶ The effects of climate change are making the Arctic more
accessible to economic activity—including exploitation of oil, gas and fish—and increased commercial traffic. Arctic
governments have responded with increased attention to the region in several fields, including the military. ¶
However, rather than projecting power over the Arctic as a whole, the increased military
capabilities described in the background paper are generally limited to forces and equipment for policing and
protection of recognized national territories and territorial waters.¶ Military build-up occurring but
cooperation remains the goal¶ Military interest in the region does exist. Canada, Denmark and Norway are moving
forces into their respective Arctic regions and acquiring weapons and equipment for specific Arctic use. Russia has
also started to expand its Arctic military capabilities, while the USA’s Arctic security concerns still play only a minor
role in its overall defence policy.¶ Although some tensions have emerged in the region, cooperation, not
conflict, is more visible in the Arctic. Norway and Russia have settled a 40-year border dispute in
the Barents Sea and Arctic states are enjoying stable and peaceful bilateral relations. Meanwhile, the
Arctic Council is coming into its own as an important sub-regional organization.¶ The so-called
‘scramble for the Arctic’, whereby Arctic states compete for the region's resources, has not proven to be a
military affair. Rather, the littoral states remain committed to follow existing legal frameworks to
settle border issues and claims on Arctic exclusive economic zones (EEZs) and continental shelves.

No Arctic resource exploitation- not economically or environmentally viable

Raspotnik ‘12 (Andreas, Analyst of Arctic Security and International Law at the Arctic Institute, “No Race for Arctic
Hydrocarbon Resources? A Possible Scenario for an Unclear Development”, http://www.thearcticinstitute.org/2012/02/no-race-for-
arctic-hydrocarbon.html)

Yet there are alternatives to how one can interpret the development of the region . This article presents
a scenario where hydrocarbon resources may not be the key driver of near-future Arctic
development. Many studies, most prominently the U.S. Geological Survey (USGS) Circum-Arctic Resource Appraisal (2008),
portray the area as one of the biggest unexplored energy region in the world. The USGS estimated that 90 billion barrels of oil, 1,669
trillion cubic feet of natural gas (approximately 30% of the world’s undiscovered conventional gas) and 44 billion barrels of natural
the USGS emphasized the low data density and the
gas liquids might remain undiscovered in the Arctic.[3] Yet
high geological uncertainty . Additionally, economic considerations (e.g. reference to costs of
exploration and development) were not included in the initial estimates. Nevertheless, these figures are
often used in Arctic energy debates to support theories for the likelihood of a new Cold War period and the battle for energy
resources in a purportedly lawless area. Recent drillings confirmed the appraisal’s suggestions that the Arctic is gas-prone , with
the highest potential in the Russian South Kara Sea. With regard to undiscovered oil, the Alaska Platform holds the
largest estimates. Yet the actual oil estimates will not shift the concerned global production balance .[4]
Arctic hydrocarbon resources may account for a bigger share of the world’s hydrocarbon production in the future if the reserves can
technical challenges must be addressed and the
be exploited in an economically viable way. In this context
actual costs for infrastructure (e.g. exploration, exploitation, and transportation) lowered.[5]
Future development of Arctic energy resources will also be dependent on the actual energy
price, climate conditions, production from other regions, alternative fuels developments, and
the way in which climate change will alter the accessibility of the High North . All-encompassing
cost-benefit analyses will be required to determine the economic feasibility of Arctic drilling. With
regard to the Russian territory, the International Energy Agency’s (IEA) World Energy Outlook 2011 already stated that d ue to
logistical challenges the Russian Arctic continental shelf might not become a major production area
until 2035.[6] Additionally, justified environmental concerns represent an obstacle for resource
developments in the region. Oil spill recovery efforts would be challenged by low temperatures, icy
conditions, lack of daylight and visibility during the winter months, wave heights, rough sea
conditions, limited infrastructure and the lack of available personnel resources. These
uncertainties will significantly reduce the economic incentives for oil companies to invest in the
region. Consequently future scenarios are hard to predict. Alternative fuel developments such as unconventional gas resources
(e.g. shale gas, coal bed methane, tight gas, natural gas hydrates) can be considered a dark horse for the future development of
Arctic hydrocarbon resources. An economically viable exploitation of shale gas resources is still considered a rather new
phenomenon.[7] Yet since the U.S. Energy Information Administration (EIA) published a first initial assessment on world shale gas
resources [8], one question has often been raised: “is shale gas a game changer in the global energy play?” In a previous TAI article
[9], Kathrin Keil already argued that Arctic gas reserves are becoming less and less attractive for the U.S.
market, because shale gas has proven to be a crucial game changer. Two technological developments, 1) horizontal drilling and 2)
hydraulic fracturing (=fracking) have made shale gas exploitation easier and essentially more cost-effective. Shale gas fields, which
have previously been deemed unprofitable to develop, have now become highly attractive. The EIA concluded that the significant
international potential of shale gas could play an increasingly important role in global natural gas markets. In addition to the U.S.,
China, Europe and South Africa are considered the most promising regions; yet the Middle East and Russia were not included in the
study. The initial estimate of technically recoverable shale gas resources currently accounts for 6,622 trillion cubic feet (tcf). The
worldwide technically recoverable gas resources, excluding shale gas, are roughly 16,000 trillion cubic feet.[10] However, technical
feasibility does not equate to economic viability. The U.S. and Europe, both considered the two main markets for Norwegian[11] and
Russian (Arctic) gas and oil, hold similar reserves (862 tcf and 639 tcf, respectively). Yet, due
to a multitude of issues, e.g.
economic and environmental concerns, different regulatory regimes, and the basic problem of
actual space, due to high population density in Europe, one has to be more critical about the
European prospects of shale gas. In addition analysts have already started to question the shale gas
production forecasts in the U.S . and criticized the optimistic perception of the country’s
resources and stressed the long-term costs of extracting shale gas .[12] Currently prices are too low to make
shale gas production profitable on a long-term scale. Furthermore, environmental concerns with regard to fracking, including the
potential contamination of ground water, minor earthquakes, and risks to air quality, prominently influence the debate. A recent
study asserts that emissions from shale gas rival those from coal.[13]

No war- Russia is peaceful and diplomacy solves

Vasiliev ‘12 [Anton, Russian Envoy to the Arctic Council, “International Cooperation in the Arctic
and Russian Approaches”, http://webcache.googleusercontent.com/search?
q=cache:f8Dm5779OHAJ:www.cerbanet.org/intranet/Documents/Regional%2520Office%2520-
%2520Toronto/Events/2012_CanadaRussiaNorthernTrade/International%2520cooperation%2520in%2520the%2520Arctic%2520and
%2520Russian%2520Approaches.pdf+&cd=1&hl=en&ct=clnk&gl=us]

Russia is the first Arctic state to adopt in 2008 a long-term Arctic strategy in a new historic
environment. It clearly defines four national priorities: use Arctic resources for country’s social and economic
development; preserve the Arctic as an area of peace, stability and cooperation; preserve the fragile
Arctic ecosystems and protect the interest of aboriginal people; use the advantages of the
Northern Sea Route (NSR), a national transport artery. All Arctic states followed the Russian suit
to adopt their national Arctic strategies. Denmark was the last to do that in August 2011 . Despite of
their specific national features, these strategies have much in common, namely: assertion of national sovereignty in the Arctic,
attempts to find a reasonable combination of the Northern economic development with environment protection and support to
aboriginal people, the necessity of public programs to support Northern regions more in depth scientific study of natural, climatic,
physical and other processes and changes in the Arctic to better understand and take them into consideration in the states’ policies.
But, may bethe main common thing in all those strategies is the recognition that each arctic
state’s interest may be honoured to the full through multifaceted and multiformat cooperation
with Arctic states only. Here is the interstate reflection of a simple human truth that under all changes North remains North with
natural and other conditions so harsh not to survive alone. And here is also a good basis for further enhancement and growth of
cooperation between countries. The
situation in the Arctic is generally seems positive, stable and predictable.
There are no issues between Arctic states that could call for a military solution. There is no necessity in
the presence of military and political blocs in the Arctic. All regional issues are and will be settled
within the norms of international law and in the spirit of good will. Certainly, Russia attentively follows the
developments in the Arctic and doesn’t tend to artificially simplify it. Nevertheless, it would be an unjust exaggeration to speak
about any militarization processes (meaning conventional, not strategic armaments). Indeed, all Arctic states to one extent or
another pay attention to fostering their forces and assets in the Arctic. However, to a known extent, it is a natural understandable,
limited and undistabilizing process. With the enhanced human presence and economic activities in the Arctic, with the new
challenges and threats Arctic states should have an opportunity to defend their sovereignty (for example, oil platform in the Arctic
Ocean may be an attractive aim for terrorists). The problem of so-called “opening” of coastal Arctic states’ borders earlier covered
by ice which is rapidly melting. And searcl? and rescue? The answer is evident. I think that in
this environment we should
build measures of trust and mutual understanding in the military sphere between Arctic states.
The first steps are already made. For example, during recent visits to Moscow by supreme
military commanders of Canada and Norway important agreements are reached on enhanced
bilateral ties in the military sphere, including mutual visits of military ships and marine force
exercises. Interaction between coastal Arctic states’ coast guards is underway.
---Arctic War- No Escalation

Cooperation prevails- no escalation

Chrastansky and Kriz ‘11 (Filip Chrášťanský graduated in 2011 at the Faculty of Law, Masaryk University and also at the
Faculty of Social Studies in the past. In 2010 he took part as a student assistant in the project “Armed conflicts in post-Cold War era”.
Zdenek Kriz is An associate professor at the Department of International Relations and European Studies at the Faculty of Social
Studies, Masaryk University in Brno. In 2003–2004, he worked as the head of the Department of Social Studies at the FVŠ Military
Academy in Brno. He focuses on international relationships and security policy.”, “Perception of the Situation in the Arctic by Key
Actors and the Possibily of Conflict Escalation”, 6/15/11, http://www.defenceandstrategy.eu/cs/aktualni-cislo-1-
2011/clanky/perception-of-the-situation-in-the-arctic-by-key-actors-and-the-possibily-of-conflict-escalation.html)

In recent years, a new conflict has emerged in the Arctic over the continental shelf beneath the North Pole .
Along with other events, this conflict definitely turned worldwide attention to the north. In 2007, we witnessed a minor escalation in
tensions in the region. At the same time, attention was shifted from individual existing conflicts to the power struggle over the entire
region. This was caused by politicians referring to the situation in the region as a whole. Nevertheless, increasing tensions once again
did not exceed the threshold of a violent confrontation and actors limited themselves to symbolic gestures and bold statements. The
subsequent events – culminating in the adoption of the Ilulissat Declaration and the Russian-Norwegian
border dispute settlement – then led to the de-escalation of tensions. The situation in the Arctic
is thus characterized by deepening cooperation, which is occasionally spiced up by a strong statement from one of
the Russian or Canadian representatives, the detention of an illegally fishing ship, or a demonstration of military power. Newly
introduced qualities which alter the character of state interests in the region bring new
challenges. They will need to be solved through a common approach . A number of international actors,
who have been regionally inactive until recently, are ever more loudly calling for influence in the Arctic. The most important
question of the near future will thus rest in the distribution of competences between the involved institutions and states (for
example the question if maritime transport will be regulated by each nation separately, by the Arctic five, or by the IMO, and what
form of regulation will be implemented). This will eventually lead to the convergence of the interests of the Arctic five who wish to
maintain their dominant position in the region. Even though future development in the region cannot be predicted with certainty
we can state with considerable certainty
and even though sovereign states can resort to the use of force at any time,
that in the near future, the escalation of conflicts in the Arctic on a greater scale is improbable. If
conflicts were to escalate, they would most likely only result in limited confrontations and not war ,
or even a grave crisis. If we take into consideration the risk of escalation from the point of view of objectively incompatible
interests, then the character of the interests in question – namely the national interest in fully utilizing the
economic potential of the region – clearly encourages cooperation and stabilization. Peaceful
development is furthermore supported by the complex network of institutionalized relations and
the fact that all territorial disputes so far have been resolved peacefully. The current nature of
international relations can thus be categorized in the cooperative behavioral pattern introduced by
Morton Deutsch. [53] Behavioral patterns supporting conflict escalation are only present in a limited number of cases and actors.

No Arctic conflict and no escalation

Ackerman 11 (Spencer, National Security Reporter @ WIRED, " War For the Arctic: Never Mind," June 8th,
http://www.wired.com/dangerroom/2011/06/war-for-the-arctic-never-mind/,
It wasn’t long ago that the press was running wild with hyperbolic claims of the U.S. losing out in an
impending Arctic conflict. After all, global warming is freeing up access to large deposits of oil, gas and minerals right in the
backyard of the Russians. But the press forgot to tell other polar nations to freak out. Indeed, at a forum
convened on Wednesday by the Center for Strategic and International Security, ambassadors from four polar nations,
including some traditionally menaced by Russia, were sanguine about the future of polar
exploration. “We actually think we handled these areas for decades during the Cold War rather well ,”
said Wegger Strommen, Norway’s man in Washington. The U.S Geological Survey assesses that the North Pole holds about 13
percent of the world’s untapped oil supplies. Companies and nations are champing at the bit to expand exploration as the ice caps
melt. The Russians have an advantage: a fleet of six nuclear powered icebreakers on its northern shore. By contrast, the U.S. Coast
Guard has just one, the cutter Healy. But no one’s sweating it. Should there actually be an arctic sea conflict, the
U.S. submarine fleet is second to none, as my colleague David Axe has pointed out. And a massive Arctic oil rush is
“years off,” Strommen added, since the “climate is harsh, the conditions are difficult and it’s incredibly expensive.” Beyond that,
the Russians are warm in the Arctic. Russia finalized a maritime border with Norway on Tuesday that
took 30 years to negotiate. Strommen’s colleagues from Greenland, Canada and Sweden gave high marks to a
meeting last month of the Arctic Council, the diplomatic contact group of arctic nations, in which
Russia signed onto an accord for search and rescue missions in the cold waters. Think of it as a diplomatic
thaw.

Won’t escalate.

Smirnov ‘9 (Alexei Smirnov, @ Defense and Security, 1-23-09 [“The Arctic is Hot,” lexis]

The Arctic faces the threat of armed conflicts , possibly involving Russia, in the years ahead. This conclusion
has been arrived at independently by the Danish Institute of Military Studies and Australian Armed Forces analysts. The open part of
a classified report from the Australian military says: "The Arctic is melting, and mining on the sea bed is becoming profitable. If
disputes over mining rights cannot be resolved by peaceful means, armed force may be used." Similar concerns are expressed in the
Five countries are claiming areas close to the North Pole: Denmark, Russia, the
Danish report.
United States, Canada, and Norway. Four of them are NATO members. " It's hard to imagine
them fighting each other," said Major Henrik Edig Jorgensen, a co-author of the Danish military
report. "Based on available maps of the Arctic regions that Moscow is claiming, up to 70% of
their oil and gas reserves would end up in the hands of the Russians. The stakes are too high -
they have too much to lose. If the Arctic is full of warships monitoring each other, no oil
platforms could be built there. So militarization of the North is not in Moscow's interes ts." But if
there is another situation like the recent five-day war in the Caucasus, and a NATO country is involved, the Arctic could go up in
flames.

There evidnece is just alarmist fear-mongering - there's no risk of Arctic conflict and the US
would win quickly anyway

Axe 11 (David, Military Correnspondent @ WIRED, Washington Times, " How the U.S. Wins the Coming Arctic War," Jan 11th,
http://www.wired.com/dangerroom/2011/01/how-the-u-s-wins-the-coming-arctic-war/,
The strategic implications of steadily melting Arctic ice. It’s [is] one of those perennial stories of the U.S.
defense trade, alongside “the end of U.S. air supremacy,” “cyber Pearl Harbor” and “China conquers the world.” The story always
starts and ends the same way. Up top, how global climate change will, by 2015 or so, result in ice-free Arctic summers — allowing
shipping and oil and natural-gas extraction. At the bottom, how the U.S. isn’t doing enough to secure its slice of the Arctic pie. I
should know: in weaker moments, I’ve written this tale, too. But these
tales, my versions included, usually omit two vital
points: that Arctic conflict is unlikely to occur at all; and even if it does, the U.S. will have an
overwhelming advantage over any rival. The Washington Post was the latest to repeat the Arctic-war theme, in a
story published yesterday. “The Arctic is believed to hold nearly a quarter of the world’s untapped natural resources and a new
passage could shave as much as 40 percent of the time it takes for commercial shippers to travel from the Atlantic to the Pacific,”
Jacquelyn Ryan wrote. But, she added, “government and military officials are concerned the United States is not moving quickly
enough to protect American interests in this vulnerable and fast-changing region.” Specifically, the U.S. does not have enough
icebreakers or permanent bases on the Alaskan north slope. Canada and Russia, by contrast, are buying ice-hardened Arctic ships
and building new facilities to enforce their Arctic claims, Ryan pointed out. The thing is, it’s not icebreakers and patches of wind-
blasted tarmac that would really matter in some future North Pole showdown. In
the Arctic, as in any sea battle, American
nuclear attack submarines — quiet, versatile and lethal — would make all the difference . U.S. subs
have been sneaking around under the Arctic ice, and occasionally surfacing, for decades. Today, they even carry geologists and other
scientists in order to help map Arctic mineral deposits. “In addition to being more heavily armed than most foreign boats, U.S.
submarines generally have superior quieting and combat systems, better-trained crewmen, and much more rigorous maintenance
standards,” Bob Work wrote in 2008, before becoming Navy undersecretary. “As a result, the
U.S. submarine force has
generally been confident that it could defeat any potential undersea opponent, even if significantly
outnumbered.” But in the Arctic, facing only the Canadians, Russians, Danes and Norwegians —
none of whom have large or healthy sub fleets — the U.S. Navy ’s 50 Los Angeles-, Seawolf- and Virginia-class
subs would be more numerous as well as more powerful. And besides, an Arctic war is highly unlikely,
at best. “Militarized conflict over the Arctic is unlikely, and regional disputes are unlikely to cause an overall
deterioration in relations between or among polar nations,” the Carnegie Endowment for International Peace
concluded in a 2009 conference. “Security issues should not be sensationalized in order to attract
attention towards the Arctic.” But it’s rare anyone writes stories about how we’ve got enough weapons — and don’t
really need them, besides. After all, it’s the sensational stories about shortages and looming disaster that
sell newspapers.
---Arctic War- Institutions Check

International dispute settlement checks—no escalation and media is hype.

Baker ‘8 (Betsy Baker, prof. International Law @ Vermont Law School, 9-14-08 [“Arctic Mapping and the Law of the Sea,”
http://arctic-healy-baker-2008.blogspot.com/2008/09/conflict-in-arctic-tenacity-of-media.html]

Just hours after I returned, a week ago, from my trip to the Arctic Ocean,I was dismayed to open the New York
Times and find on its editorial page hyperbole verging on that which other media sources use to
perpetuate the myth of "fierce disputes over territory and natural resources" in the Arctic . ("Arctic
in Retreat", September 8, 2008). As the sea-ice retreats, states are turning not to arms but to existing
legal structures and a tradition of scientific and diplomatic cooperation to address common problems
as well as disagreements. Immediately after transporting our mapping crew to shore last week, The Healy turned right around
and began breaking ice for a Canadian icebreaker, the Louis Saint Laurent. This month-long joint mission to map
parts of the Arctic Ocean floor is scientific and diplomatic cooperation at its international best . Like
the Russian mapping the NYT mentions in its editorial, the US and Canada are gathering data in
preparation not for conflict but for submission in a staid and stable legal process designed to provide
certainty for all states involved. The L aw o f the S ea C onvention establishes this orderly mechanism of
rigorous scientific vetting for states seeking to extend their authority over larger portions of the
continental shelf. The United States is the only Arctic state not party to the Convention but is nonetheless
mapping for its potential shelf extension in keeping with procedures agreed by the international
community.

Current framework solves.

Banker ‘8 (Betsy Baker, prof. International Law @ Vermont Law School, 9-14-08 [“Arctic Mapping and the Law of the Sea,”
http://arctic-healy-baker-2008.blogspot.com/2008/09/conflict-in-arctic-tenacity-of-media.html]

The territorial disputes referenced in the NYT editorial are also resolved not by conflict but by diplomacy .
In June 1990 Russia (then still the Soviet Union) and the United States signed a brilliantly conceived
single maritime boundary treaty that precludes the need to renegotiate the boundary once the
extended continental shelf limits are determined. Canada’s recent announcement that it plans to extend enforcement jurisdiction
from 100 to 200 miles beyond its shores should raise concern. But it must also be viewed within the context of the long-standing
friendship and shared interests of the United States and Canada on such matters as environmental protection, trade (ca. $1.5 billion
daily) and common security.Their disagreement over the Northwest Passage has never flared out of
control and continues to be the subject of diplomatic attention . Other existing legal and
diplomatic structures provide an imperfect but solid basis for Arctic states to resolve potential
disagreements. The Arctic Council is a cooperative forum for states and the Inuit Circumpolar Conference to address a range of
environmental and economic problems in the region. The Ilulissat (Greenland) Declaration , signed in May 2008,
confirms the will of the five coastal Arctic states – Canada, Denmark, Norway, Russia and the
United States – to strengthen existing cooperation based on mutual trust and transparency .
Treaties in force in the Arctic cover issues ranging from polar bear protection to pollution by dumping from vessels to biological
diversity. Activists and diplomats alike should be concerned and asking hard questions about whether these agreements will be
sufficient, or sufficiently enforced,to protect the Arctic, but to pretend that it is a lawless region up for grabs
ignores the facts.

Only a legal conflict—rule of law checks.

Paul ‘8 (Scott Paul, @The Washington Note, 9-15-08 [‘Cooling Down the Rhetoric Surrounding Arctic “Conflicts”,’ online]

While I'm taking pot shots at traditional media outlets (see below), I'll second this important post by Betsy Baker, which I found via
TWN reader Caitlyn Antrim's invaluable Ocean Law Daily. The writer apparently just returned from her trip on the Coast Guard
as dismayed as I was to read this New York Times editorial, which gives
Icebreaker Healy and was
readers the impression that five hostile nations are staring down gun barrels at each other
across the Arctic Circle. In fact, what's currently happening is these five countries -- Russia,
Canada, Norway, Denmark, and the United States -- are using legal means to advance their
interests both aggressively and fairly. All are committed to the rule of law in the Arctic , with
America's absence from the Law of the Sea Convention the only gap in a universally recognized and respected legal regime.

Dispute resolution mechanisms solve

Yalowitz et al 8 (Kenneth, Director of the Dickey Center for International Understanding @ Dartmouth, James Collins, Senior
Associate and Director of the Russia and Eurasia Program @ Carnegie, and Ross Virginia, Director of the Institute for Arctic Studies @
Dartmouth, " The Arctic Climate Change and Security Policy Conference: Final Reporting and Findings," Dec 1-3,
http://www.carnegieendowment.org/files/arctic_climate_change.pdf,

Despite alarmist predictions of a new “great game,” the prospect for a significant confrontation
among Arctic powers over resources, boundaries, or claims is now low. The United Nations Convention on
the Law of the Sea (UNCLOS) provides a significant body of rules and precedent to adjudicate the
majority of disagreements. Most regions with resources at stake are governed under the rules of
the 200-mile economic zone. Where there are disputed regimes or boundaries, these remain subject to
accommodation by the parties involved, in particular the U.S. and Canada.
---Arctic War- Their Ev is Hype

Their scenario is media hype – Prefer this evidence. We cite the Russian foreign ministry and
Arctic Council.

Global Security.org ‘8 October 22, “Russia says media reports on possible Arctic conflict 'alarmist'”
http://www.globalsecurity.org/wmd/library/news/russia/2008/russia-081022-rianovosti04.htm

22/10/2008 19:12 MOSCOW, October 22 (RIA Novosti) - Russia's Foreign Ministry believes media speculation
about the possibility of a war over the Arctic are ungrounded and "alarmist ," an ambassador
at large said Wednesday. "Media assessments of possible aggression in the Arctic, even a
third world war, are seen as extremely alarmist and provocative. In my opinion, there are no grounds for such
alarmism," Anton Vasilyev, who is also a high-ranking official on the Arctic Council, said. "We are following the
situation in the region, this also includes the military activity of other countries, but we hope
cooperation will be the main feature," he said
Arctic Leadership Answers
Frontline

Unilateral efforts can’t solve Arctic leadership

Smith ’11 [Colonel Reginald R. Smith, USAF, is Professor of National Security Affairs and Senior Developmental Education
Student (Strategy and Policy) at the Naval War College, “The Arctic: A New Partnership Paradigm or the Next "Cold War"?”
http://www.ndu.edu/press/lib/images/jfq-62/JFQ62_117-124_Smith.pdf]

The United States must move outside the construct of unilateral action in order to preserve its
sovereign rights in the Arctic, capitalize on the opportunities available, and safeguard vital national interests in the region.
In today's budget-constrained environment and as a Nation at war with higher resource priorities in Iraq and Afghanistan than in the
Arctic, it is unrealistic to believe that any significant allocation will be programmed for addressing this issue.3 Since
the United
States is too far behind in actions necessary to preserve its critical interests as compared to the
other Arctic countries, the Nation must take the lead to cultivate a new multilateral partnership
paradigm in the region.

Failure to ratify LOST is the single greatest factor in US Arctic influence

Smith ’11 [Colonel Reginald R. Smith, USAF, is Professor of National Security Affairs and Senior Developmental Education Student
(Strategy and Policy) at the Naval War College, “The Arctic: A New Partnership Paradigm or the Next "Cold War"?”
http://www.ndu.edu/press/lib/images/jfq-62/JFQ62_117-124_Smith.pdf]

The significance of the declaration is paramount to cooperation in that UNCLOS provides the
international rallying point for the Arctic states . 78 Similarly important, by virtue of the unanimous and strong
affirmation of UNCLOS, the declaration effectively delegitimized the notion to administer the Arctic along the lines of an Antarctic-
like treaty preserving the notions of sovereignty and resource exploitation in the region. 79 With U.S. participation and declaration
of support for UNCLOS in these venues, failure to ratify the treaty suggests that U.S. credibility and
legitimacy, and hence the ability to build cohesive multilateral partnerships, are appreciably
degraded. This conclusion is illustrated in Malaysia’s and Indonesia’s refusal to join the Proliferation Security Initiative using the
U.S. refusal to accede to UNCLOS as their main argument. 80 Accession to the treaty appears to be a key first
step to preserving U.S. vital interests in the Arctic and building necessary credibility for regional
and global partnerships in the political spectrum . Equally important to political partnerships in the region are
those available through military collaboration of the Arctic nations.

Alt cause to Arctic leadership- equipment shortages and no strategic capacity

Bert ‘12 [Melissa Bert is a captain in the U.S. Coast Guard and a military fellow at the Council on Foreign Relations, “A Strategy to
Advance the Arctic Economy,” February, http://www.cfr.org/arctic/strategy-advance-arctic-economy/p27258]

U.S. Capacity in the Arctic Is Lacking Traffic and commercial activity are increasing in the region. The NSR was not
navigable for years because of heavy ice, but it now consists of water with floating ice during the summer months. As the icebergs
decrease in the coming years, it will become a commercially profitable route, because it reduces the maritime journey between East
Asia and Western Europe from about thirteen thousand miles through the Suez Canal to eight thousand miles, cutting transit time by
ten to fifteen days. Russian and German oil tankers are already beginning to ply those waters in the summer months. Approximately
150,000 tons of oil, 400,000 tons of gas condensate, and 600,000 tons of iron ore were shipped via the NSR in 2011. Oil, gas, and
mineral drilling, as well as fisheries and tourism, are becoming more common in the high latitudes and are inherently dangerous,
because icebergs and storms can shear apart even large tankers, offshore drilling units, fishing vessels, and cruise ships. As a result,
human and environmental disasters are extremely likely. Despite the dangerous conditions, the Arctic has no mandatory
requirements for those operating in or passing through the region. There are no designated shipping lanes, requirements for ice-
strengthened hulls to withstand the extreme environment, ice navigation training for ships' masters, or even production and
carriage of updated navigation and ice charts. Keeping the Arctic safe with the increased activity and lack of regulations
presents a daunting task. The U.S. government is further hindered by the lack of ships, aircraft, and
infrastructure to enforce sovereignty and criminal laws, and to protect people and the marine
environment from catastrophic incidents . In the lower forty-eight states, response time to an oil spill or capsized
vessel is measured in hours. In Alaska, it could take days or weeks to get the right people and resources
on scene. The nearest major port is in the Aleutian Islands, thirteen hundred miles from Point
Barrow, and response aircraft are more than one thousand miles south in Kodiak , blocked by a
mountain range and hazardous flying conditions. The Arctic shores lack infrastructure to launch
any type of disaster response, or to support the growing commercial development in the region .

US is already expanding Arctic influence

Spotts ‘12 [Pete, Christian Science Monitor, “Is US finally developing a grand strategy for its Arctic frontier?”
http://www.alaskadispatch.com/article/us-finally-developing-grand-strategy-its-arctic-frontier]

The United States is set to unveil a five-year scheme for offshore oil and gas leases that it says
will open more of the Arctic Ocean to exploration while protecting the environment and the livelihoods of
indigenous peoples. The US is planning the move as Royal Dutch Shell prepares to sink two
exploration wells in US Arctic Ocean waters – one in the Chukchi Sea between Alaska and Siberia and north of the
Bering Strait, the other in the Beaufort Sea north of Alaska. “We are currently in the final stages of a rigorous review of
Shell’s proposal to drill exploratory wells ... this summer,” said David Hayes, deputy secretary of the Interior, in a conference call
with reporters on Tuesday, adding that he anticipated Shell would be granted permission to proceed. In granting new offshore oil
and gas leases outside the Gulf of Mexico, the Obama administration is seeking to minimize impact on the environment and
indigenous people with a policy dubbed targeted leases. Instead of opening vast tracts of sea floor for leases, the
administration is trying to focus on areas with the highest potential for oil and gas, while excluding
exploration in environmentally sensitive areas or areas important to the livelihoods of
indigenous people along the coast of the Arctic Ocean . Most details await the Interior Department’s release of
the five-year program, but federal officials, including Interior Secretary Ken Salazar, previewed the broad outlines in the conference
call. The
approach, which aims to put science, environmental issues, and indigenous uses on a more equal footing with barrels
of oil and cubic feet of gas, comes
at a time when countries bordering the Arctic are trying to craft what
Secretary Salazar calls an integrated approach to oil and gas development at the top of the
world. Russia already has drilled a handful of exploratory wells in its Arctic waters . On Tuesday,
Norway announced plans to issue oil and gas exploration permits for up to 86 offshore tracts, most of them in Arctic waters, by the
end of 2013. “We have seen very strong interest in the Arctic ... and the oil industry is clearly moving north,” Norwegian Petroleum
and Energy Minister Ola Borten told Reuters.
---Arctic Leadership- LOST Alt Cause

Inaction on LOST kills all Arctic influence

Bert ‘12 [Captain Melissa – USCG, 2011-2012 Military Fellow, U.S.Coast Guard, “A Strategy to Advance the Arctic Economy”,
February, http://www.cfr.org/arctic/strategy-advance-arctic-economy/p27258]

Critics argue that the LOSC cedes American sovereignty to the United Nations. But the failure to ratify it has the opposite effect:
it leavesthe United States less able to protect its interests in the Arctic and elsewhere. The
diminished influence is particularly evident at the International Maritime Organization (IMO), the
international body that "operationalizes" the LOSC through its international port and shipping
rules. By remaining a nonparty, the United States lacks the credibility to promote U.S. interests
in the Arctic, such as by transforming U.S. recommendations into binding international laws .

Failure to ratify LOST kills US Arctic leadership

Bert ‘12 [Melissa Bert is a captain in the U.S. Coast Guard and a military fellow at the Council on Foreign Relations, “A Strategy to
Advance the Arctic Economy,” February, http://www.cfr.org/arctic/strategy-advance-arctic-economy/p27258]

Governance in the Arctic requires leadership. The U nited States is uniquely positioned to provide
such leadership, but it is hampered by its reliance on the eight-nation Arctic Council . However, more
than 160 countries view the LSOC as the critical instrument defining conduct at sea and
maritime obligations. The convention also addresses resource division, maritime traffic, and pollution regulation, and is
relied upon for dispute resolution. The LOSC is particularly important in the Arctic, because it stipulates
that the region beyond each country's exclusive economic zone (EEZ) be divided between bordering
nations that can prove their underwater continental shelves extend directly from their land borders. Nations will have exclusive
economic rights to the oil, gas, and mineral resources extracted from those outer continental shelves, making the convention's
determinations substantial. According to geologists, the U.S. portion is projected to be the world's largest underwater extension of
land—over 3.3 million square miles—bigger than the lower forty-eight states combined. In
addition to global credibility
and protection of Arctic shelf claims, the convention is important because it sets international
pollution standards and requires signatories to protect the marine environment .
Asian Energy Answers
Frontline
China is taking steps to prevent Asian energy conflict

Xuegang 07 Southeast Asia and Energy:¶ Gateway to Stability¶ Zhang Xuegang. (Zhang Xuegang is an associate professor in
China Institutes of Contemporary¶ International Relations (CICIR). He focuses on Sino-U.S.-ASEAN security ¶ and Singaporean,
Malaysian and Thailand studies.) China Security, Vol 3 No 2 pp. 18 - 35¶ ©2007 World Security Institute.

In recent years, China


has taken a number of important steps to address ¶ the inherent risks of the
Malacca Strait. To begin with, China has actively¶ developed closer cooperative relationships with
Singapore, Malaysia and¶ Indonesia, to secure China’s interest in the Strait and to alleviate the growing¶
rivalries among the world’s major powers. Since 2005, China has played an¶ active role in
maintaining and strengthening security in the strait as well as¶ the prevention of oil leaks in the region.7¶ This
approach stresses the fact that since¶ trade and energy transport to China make ¶ up a majority
of the strait’s traffic, sea-lane¶ security is becoming a core interest to¶ China and as such, China is
pursuing a¶ leading role in its security. China’s active¶ engagement in this regard remains via¶
international cooperation and diplomatic efforts. At the same time, this approach¶ is not unconditional, and as
the Malacca Strait is of great importance¶ to China’s energy security, China is certainly considering the use of force as¶ a last option
of defense against direct threats – such as a blockade during a¶ Taiwan crisis.¶ The potential bottleneck of the Malacca Strait, on
which China is overly¶ dependent for energy and trade transport, lends itself easily to worst case¶ scenarios. Therefore, to minimize
these risks, China is also rigorously planning¶ for the long term with a diversified transportation strategy that utilizes alternative
channels, routes and¶ countries to import energy to¶ China.8 While such alternatives¶
create new opportunities for
a¶ sound energy strategy, there are¶ important limitations that affect¶ their development and functional¶ capacity in the
short term.¶ The development of a canal¶ or pipeline across the Isthmus of¶ Kra in southern Thailand would¶ significantly shift the
regional¶ strategic energy landscape. The¶ flat terrain, with a minimum¶ width of 50 km and lying mainly¶ within Thailand’s territory
and¶ partially in Burma’s, is ideal for a world-class canal. It would effectively redirect an estimated 90 percent of the ¶ vessels from
the Malacca Strait since it would save approximately half the¶ fuel.9 Unfortunately, despite enthusiasm within the Thai government,
it is¶ unlikely that progress will be made on the canal in the near future due to economic ¶ and technical constraints as well as the
more recent domestic political¶ upheaval among Thailand’s southern Muslim populations. A more simplified¶ plan developed in
2004, consisting of roads, rail and underground pipelines, ¶ would greatly reduce transport capacity and would likely play a significant
role¶ in reducing the current overdependence on the Malacca Strait.10 Even these ¶ less ambitious plans have been put on hold since
the 2006 military coup.¶ In recent years, China and Burma have also proposed a plan for building ¶ an oil pipeline. According to the
scenario, China can build a pipeline from¶ Sittwe – a city on the southwest coast of Burma near the Indian Ocean – to¶ the Chinese
city of Kunming in Yunnan Province. This shortcut would reduce¶ the Malacca voyage by 1200 km11 and could help relieve China’s
dependence¶ on the Malacca Strait by one-third or more.12 However, this plan has also ¶ stagnated recently because the Burmese
junta worries about being excessively¶ dependent on exporting oil to China and is inclined to build an alternative ¶ gas pipeline
toward China rather than the oil pipeline (the former is a less sensitive issue). This suggests a current strategy of hedging against
China’s¶ excessive influence as well as Burma’s growing tendency in recent years to seek ¶ closer energy cooperation with India. The
junta’s decision on this remains in¶ flux, and prospects for a denouement are uncertain.¶ A third alternative energy transportation
route that has been given more attention¶ is the development of a comprehensive 5,500 km trans-Asian railway ¶ network that
would link China to many ASEAN nations, creating an efficient¶ means to transport energy and other goods. Talk of the rail system
began in¶ 1995 at the 5th ASEAN Summit and the idea has evolved into an elaborate¶ three-line system that would reduce
transportation time and costs significantly.13 ¶ The plan is listed in China’s National Plan for Railway Network Constructions¶ 2003-
2020, and construction is set to commence as early as 2008. This¶ network will largely be constructed during the next five to 10
years. The trans-¶ Asian railway network will also build up the infrastructure necessary to link ¶ Southeast Asia and China with
Northeast Asia, Central Asia, South Asia, and¶ eventually create a trans-Eurasian ¶ railway system that would connect¶ Asia and
Europe.14 However,¶ a number of problems plague¶ this endeavor. For example, Laos¶ lacks the railway infrastructure to¶
effectively participate in the plan¶ and the different rail diameters¶ used in various ASEAN countries¶ will prove an additional
technical¶ hurdle.15 With a price tag of $11¶ billion USD, however, a shortage¶ of committed investment remains the primary
obstacle.
No South Asian Conflict, Economic interdependence, DOC agreement, regional,
Chinese, and ASEAN support.
Arsyad 11 South China Sea Disputes: The Road to Peace, Stability and Development. Sun 13 Nov 2011.
http://www.rosihanarsyad.com/blog_detail/2/18/south-china-sea-disputes--the-road-to-peace--stability-and-development.html.
Mr. Rosihan Arsyad. (Senior member of the Indonesian Naval Academy, Governor of South Sumatra, Indonesia, retired Admiral,
President of Bukit DBaiduri Energy, recipient of numerous international awards, spoken at mant international seminars on South
Asian security.

As the world economy becomes more integrated with increasing globalization of trade, finance, and communications, the
utilization of sea¶ transportation will continue to expand. With
increasing global economic
interdependency, it is critical for the international community to maintain safe and
undisrupted sea transportation, a crucial factor for trading activities. As South China Sea is very
strategic in the global economic development, it is very important to address safety and
security of the South China Sea in a stronger and more comprehensive cooperative effort .
For mutual benefits, regional countries must take active part through cooperation in the creation of a sustained equilibrium
that promotes peace, stability, and security in the South China Sea. This initiative must be supported by major power. ¶ It is
quite encouraging to observe that despite the looming shadow of competition in South China Sea ,the US and China
have until now appeared willing to open a positive dialogue and exchange visits by top
ranking military and defense officials as well as military personnel. Successive U.S. administrations have sought
to develop a more regular engagement with the People's Liberation Army, as concerns inside
the Pentagon mounts up on the scale of China's military buildup in recent years.10 ¶ The guidelines for the implementation of
the Declaration on Conduct of Parties (DOC) agreed to by China and ASEAN in July in Bali this year
provide the basis to accelerate the implementation of actions as proposed by the declaration.¶ The DOC, as a political text
is intended to prevent further tensions over the disputed territories and to reduce the
risks of military conflict in the South China Sea. Although non-binding, the signing of the DOC indicated that
China and ASEAN recognize the importance of regional security and economic
development in the era of globalization and that the international community including
the US and other major players such as Japan, India, Russia and Australia must support
it.¶ In the declaration, the Parties reaffirmed their commitment to the purposes and principles of the Charter of the United
Nations, the 1982 UN Convention on the Law of the Sea, the Treaty of Amity and Cooperation in Southeast Asia, the Five
Principles of Peaceful Coexistence, and other universally recognized principles of international law which shall serve as the
basic norms governing state-to-state relations. The
Parties are committed to exploring ways for
building trust and confidence in accordance with the above-mentioned principles and on
the basis of equality and mutual respect.¶ Furthermore, in the DOC, the Parties reaffirm their respect for and
commitment to the freedom of navigation in and over-flight above the South China Sea, agree to resolve their territorial and
jurisdictional disputes by peaceful means through friendly consultations and negotiations without resorting to the threat or
use of force. This consensus is very important and must be adhered t o, since existing disputes can be
escalating into conflicts and further can spread into a prolong sea battle, which certainly will disrupt the world economy. ¶
According to the Declaration on Conduct of Parties in South China Sea, pending the peaceful settlement of
territorial and jurisdictional disputes, the Parties concerned undertake to intensify efforts
to seek ways in the spirit of cooperation and understanding to build trust and confidence
between and among themselves, including holding dialogues and exchange of views as
appropriate between their defense and military officials, notifying each other on
voluntary basis any impending joint/combined military exercise and exchanging relevant
information. Most importantly, as first step the Parties concerned agree to explore or undertake cooperative activities in
marine environmental protection, marine scientific research, safety of navigation and communication at sea, search and
rescue operation and combating transnational crime, including but not limited to trafficking in illicit drugs, piracy and armed
robbery at sea, and illegal arms trade.¶ THE ROAD TO PEACE, STABILITY AND DEVELOPMENT ¶ The 2002 Declaration on
Conduct of Parties in the South China Sea (DOC) has provided a solid base for cooperation in defusing tension in the region
regarding dispute in South China Sea, and it should be supported by all parties concerned and honored by the international
community. To ensure DOC's success, it requires the continuing political commitment of all parties so that it can contribute to
maintaining peace and stability in the region. In signing the DOC, all parties are seeking ways to gain common security and
mutual benefits, based on "the needs to promote a peaceful, friendly and harmonious environment in the South China Sea
between ASEAN and China for the enhancement of peace, stability, economic growth and prosperity in the region." ¶ In July
2011, both ASEAN and China started a process of an agreement to adopt guidelines to implement the DOC and then to
develop a code of conduct to settle disputes in the South China Sea. The document, "Guidelines on the Implementation of the
Declaration on the Conduct of Parties in the South China Sea," will have to be approved by the government of China and
ASEAN countries. Both the Guidelines and the DOC indicate that cooperation should be enhanced in a step-by-step approach,
starting with the least delicate issue, such as exchange of personnel, training and survey of maritime area. Activities in the
area of education, research and development in marine science can and should provide the basis for broadbased regional
security cooperation in the area. Marine research and development can be first conducted outside the disputed area, such as
the Anambas Expedition, which is a great success and needs to be continued to include the disputed areas. ¶ Cooperation in
social and economic aspect must be considered especially in the field that could enhance capacity-building in security
measure. There is a pressing need to identify areas of overlapping claims in order to map out a comprehensive plan for
developing cooperation. The claimant countries should delineate which areas are in dispute and which are not. Exploration
could then go ahead in the undisputed areas, while the disputed areas would be turned into an area for joint cooperation. ¶
Confidence building measures (CBMs) are of particular importance because they could help remove inter-state suspicion and
promote and develop a sense of secured community. CBMs should first be aimed at increasing transparency among states in
their security policies and plans, thus creating mutual trust and understanding, which can subsequently be developed into a
more solid economic cooperation for the regional states in utilizing the rich resources of the South China Sea. ¶ Indeed, there
is great expectation and hope that the DOC and its guidelines can be developed into a binding Code of Conduct.¶ The
benefits of maintaining free and smooth traffic flow in the South China Sea much
outweigh the benefits of attempting to gain ownership of the waters by any party, as any
attempt to use military force will certainly not go unopposed .¶ Regional countries and user states
must put trust and confidence and recognize China as a major player, which has a big interest to solve the dispute peacefully
on the basis of balance-of-interest principles. China's position on settling the South China Sea disputes has consistently placed
emphasis on direct negotiations between disputing parties based on valid historical data and accepted international law. Thus
far, China's
attitude has demonstrated maximum restraint towards other claimants and
seeks to minimize differences with joint developments .¶ As the most powerful country in the region,
China also depends on the safety and security in the South China Sea. The volume of trade between China and the world is
growing rapidly. Potential trade between China and ASEAN alone could achieve USD 4.5 trillions, not to mention ASEAN
market of 600 million people. Therefore, it is very important for regional countries, especially with overlapping claim to
always encourage and persuade China to heed to the international community's expectation. Perhaps it is also time to
recognize that China has been a responsible member of "nuclear club" for 55 years and has signed the "Non-Proliferation
Treaty". Hence, China as a major power in the region can be and must be 'persuaded' to lead the disputes settlement in a
peaceful way.¶ The littoral and all user states will suffer if there is calamity in the South China Sea for they are dependent on
those sea lines. Therefore, it is necessary for the regional and user states to restraint themselves from arm conflict. All Parties
involved should realize that the greatest misconception in the South China Sea dispute is that the prerequisite for progress is
the resolution of jurisdictional stalemate. It is true that boundaries may provide some answers but they do not adequately
address the broader issues of maritime management. It is only appropriate if the Parties involved undertake to exercise self-
restraint in the conduct of activities that would complicate or escalate disputes and affect peace and stability including,
among others, refraining from inhabiting the presently uninhabited islands, reefs, shoals, cays, and other features and to
handle their differences in a constructive manner.¶ Military
intervention to settle disputes in the South
China Sea is by no means a desired 'diplomatic' option by any country. Defusing the
tensions in the South China Sea is now the top priority if peace and stability in the region
were to be maintained. Internationalizing disputes and taking unilateral actions can only
further aggravate the already strained relationship. Claimant countries must remain committed and
confident that bilateral engagements between China and ASEAN without outsiders' involvement can solve the disputes in the
South China Sea.¶ ASEAN and China should agree on the plan to create a "zone of peace, freedom, friendship and
cooperation" in the South China Sea. The idea is to identify the area that multiple countries are claiming, and jointly turn
these disputed areas into economically viable sites that benefits not only all claimant nations, but also the rest of the world.
Asian War Answers
Frontline

No Asian war- China creates stability

Carlson ’13 (Allen Carlson is an Associate Professor in Cornell University’s Government Department. He was granted his PhD
from Yale University’s Political Science Department. His undergraduate degree is from Colby College. In 2005 his Unifying China,
Integrating with the World: Securing Chinese Sovereignty in the Reform Era was published by Stanford University Press. He has
also written articles that appeared in the Journal of Contemporary China, Pacific Affairs, Asia Policy, and Nations and Nationalism.
In addition, he has published monographs for the National Committee on U.S.-China Relations and the East-West Center
Washington. Carlson was a Fulbright-Hays scholar at Peking University during the 2004-2005 academic year. In 2005 he was
chosen to participate in the National Committee’s Public Intellectuals Program, and he currently serves as an adviser to Cornell’s
China Asia Pacific Studies program and its East Asia Program. Carlson is currently working on a project exploring the issue of
nontraditional security in China’s emerging relationship with the rest of the international system. His most recent publications are
the co-edited Contemporary Chinese Politics: New Sources, Methods and Field Strategies (Cambridge University Press, 2010) and
New Frontiers in China’s Foreign Relations (Lexington, 2011). China Keeps the Peace at Sea China Keeps the Peace at Sea Why the
Dragon Doesn't Want War Allen Carlson February 21, 2013

At times in the past few months, China and Japan have appeared almost ready to do battle over the Senkaku
(Diaoyu) Islands --which are administered by Tokyo but claimed by both countries -- and to ignite a war that could be bigger than
any since World War II. Although Tokyo and Beijing have been shadowboxing over the territory for years, the standoff reached a
new low in the fall, when the Japanese government nationalized some of the islands by purchasing them from a private owner.
The decision set off a wave of violent anti-Japanese demonstrations across China. In the wake of these events, the conflict quickly
reached what political scientists call a state of equivalent retaliation -- a situation in which both countries believe that it is
imperative to respond in kind to any and all perceived slights. As a result, it may have seemed that armed engagement was
imminent. Yet, months later, nothing has happened . And despite their aggressive posturing in the disputed
territory, both sides now show glimmers of willingness to dial down hostilities and to reestablish stability .
Some analysts have cited North Korea's recent nuclear test as a factor in the countries' reluctance to engage in military conflict. They
argue that the detonation, and Kim Jong Un's belligerence, brought China and Japan together, unsettling them and placing their
differences in a scarier context. Rory Medcalf, a senior fellow at the Brookings Institution, explained that "the nuclear test gives the
leadership in both Beijing and Tokyo a chance to focus on a foreign and security policy challenge where their interests are not
diametrically at odds." The nuclear test, though, is a red herring in terms of the conflict over the disputed islands. In truth, the
roots of the conflict -- and the reasons it has not yet exploded -- are much deeper . Put simply,
China cannot afford military conflict with any of its Asian neighbors. It is not that China believes it would lose
such a spat; the country increasingly enjoys strategic superiority over the entire region, and it is difficult to imagine that its forces
over the islands , in the South China Sea or in the disputed
would be beaten in a direct engagement
regions along the Sino-Indian border. However, Chinese officials see that even the most
pronounced victory would be outweighed by the collateral damage that such a use of force would cause
to Beijing's two most fundamental national interests -- economic growth and preventing the
escalation of radical nationalist sentiment at home. These constraints, rather than any external
deterrent , will keep Xi Jinping, China's new leader, from authorizing the use of deadly force in the Diaoyu
Islands theater. For over three decades , Beijing has promoted peace and stability in Asia to facilitate
conditions amenable to China's economic development . The origins of the policy can be traced back to the late 1970s,
when Deng Xiaoping repeatedly contended that to move beyond the economically debilitating Maoist period, China would have
to seek a common ground with its neighbors. Promoting cooperation in the region would allow China to spend less on military
preparedness, focus on making the country a more welcoming destination for foreign investment, and foster better trade
relations. All of this would strengthen the Chinese economy. Deng was right. Today, China's economy is second only to that of the
United States. The fundamentals of Deng's grand economic strategy are still revered in Beijing. But any war in the region would
erode the hard-won, and precariously held, political capital that China has gained in the last several decades. It would also disrupt
trade relations, complicate efforts to promote the yuan as an international currency, and send shock waves through the country's
economic system at a time when it can ill afford them. There is thus little reason to think that China is readying for war with
Japan. At the same time, the specter of rising Chinese nationalism, although often seen as a promoter
of conflict, further limits the prospects for armed engagement . This is because Beijing will try to
discourage nationalism if it fears it may lose control or be forced by popular sentiment to take an action it deems unwise. Ever
since the Tiananmen Square massacre put questions about the Chinese Communist Party's right to govern before the
population, successive generations of Chinese leaders have carefully negotiated a balance between
promoting nationalist sentiment and preventing it from boiling over. In the process, they cemented the legitimacy of their rule. A
war with Japan could easily upset that balance by inflaming nationalism that could blow back against China's leaders. Consider a
hypothetical scenario in which a uniformed Chinese military member is killed during a firefight with Japanese soldiers. Regardless
of the specific circumstances, the casualty would create a new martyr in China and, almost as quickly, catalyze popular protests
against Japan. Demonstrators would call for blood, and if the government (fearing economic instability) did not extract enough,
citizens would agitate against Beijing itself. Those in Zhongnanhai, the Chinese leadership compound in Beijing, would find
themselves between a rock and a hard place. It is possible that Xi lost track of these basic facts during the fanfare of his rise to power
and in the face of renewed Japanese assertiveness. It is also possible that the Chinese state is more rotten at the core than is
understood. That is, party elites believe that a diversionary war is the only way to hold on to power -- damn the economic and social
consequences. But Xi does not seem blind to the principles that have served Beijing so well over the last few decades. Indeed,
although he recently warned unnamed others about infringing upon China's "national core interests" during a foreign policy speech
to members of the Politburo, he also underscored China's commitment to "never pursue development at the cost of sacrificing other
country's interests" and to never "benefit ourselves at others' expense or do harm to any neighbor." Of course, wars do happen --
and still could in the East China Sea. Should either side draw first blood through accident or an unexpected move, Sino-Japanese
relations would be pushed into terrain that has not been charted since the middle of the last century. However, understanding
that war would be a no-win situation, China has avoided rushing over the brink. This relative restraint seems to have surprised
everyone. But it shouldn't. Beijing will continue to disagree with Tokyo over the sovereign status of the islands, and will not
cannot take the risk of going to war over a
budge in its negotiating position over disputed territory. However, it
few rocks in the sea. On the contrary, in the coming months it will quietly seek a way to shelve the
dispute in return for securing regional stability, facilitating economic development, and keeping a
lid on the Pandora's box of rising nationalist sentiment. The ensuing peace , while unlikely to be deep, or
especially conducive to improving Sino-Japanese relations, will be enduring.

No risk of Asia war – Peaceful China and multilateral institutions

Bitzinger and Desker ‘9


[Richard, Senior Fellow at the S. Rajaratnam School of International Studies, Barry, Dean of the S. Rajaratnam School of International
Studies and Director of the Institute of Defense and Strategic Studies, Nanyang Technological University, Singapore, “ Why East Asian
War is Unlikely,” Survival | vol. 50 no. 6 | December 2008–January 2009

The Asia-Pacific region can be regarded as a zone of both relative insecurity and strategic stability. It
contains some of the world’s most significant flashpoints – the Korean peninsula, the Taiwan Strait, the Siachen Glacier – where
tensions between nations could escalate to the point of major war. It is replete with unresolved border issues; is a breeding ground
for transnational terrorism and the site of many terrorist activities (the Bali bombings, the Manila superferry bombing); and contains
overlapping claims for maritime territories (the Spratly Islands, the Senkaku/Diaoyu Islands) with considerable actual or potential
wealth in resources such as oil, gas and fisheries. Finally, the Asia-Pacific is an area of strategic significance with many key sea lines
of communication and important chokepoints. Yet despite all these potential crucibles of conflict, the Asia-
Pacific, if not an area of serenity and calm, is certainly more stable than one might expect. To be sure, there
are separatist movements and internal struggles , particularly with insurgencies, as in Thailand, the Philippines and
Tibet. Since the resolution of the East Timor crisis, however, the region has been relatively free of
open armed warfare. Separatism remains a challenge, but the break-up of states is unlikely. Terrorism is a
nuisance, but its impact is contained. The North Korean nuclear issue, while not fully resolved, is
at least moving toward a conclusion with the likely denuclearisation of the peninsula. Tensions
between China and Taiwan, while always just beneath the surface, seem unlikely to erupt in open conflict
any time soon, especially given recent Kuomintang Party victories in Taiwan and efforts by
Taiwan and China to re-open informal channels of consultation as well as institutional
relationships between organisations responsible for cross-strait relations . And while in Asia there is no
strong supranational political entity like the European Union, there are many multilateral organisations and
international initiatives dedicated to enhancing peace and stability, including the Asia-Pacific Economic
Cooperation (APEC) forum, the Proliferation Security Initiative and the Shanghai Co-operation Organisation . In
Southeast Asia, countries are united in a common geopolitical and economic organisation – the
Association of Southeast Asian Nations (ASEAN) – which is dedicated to peaceful economic, social and cultural development, and
to the promotion of regional peace and stability. ASEAN has played a key role in conceiving and establishing broader regional
institutions such as the East Asian Summit, ASEAN+3 (China, Japan and South Korea) and the ASEAN Regional Forum. All this
suggests that war in Asia – while not inconceivable – is unlikely. This is not to say that the region will not undergo
significant changes. The rise of China constitutes perhaps the most significant challenge to regional security and stability – and, from
Washington’s vantage point, to American hegemony in the Asia-Pacific. The United States increasingly sees China as its key peer
challenger in Asia: China was singled out in the 2006 Quadrennial Defense Review as having, among the ‘major and emerging powers
… the greatest potential to compete militarily with the United States’.1 Although the United States has been the hegemon in the
Asia-Pacific since the end of the Second World War, it will probably not remain so over the next 25 years. A rising China will
present a critical foreign-policy challenge , in some ways more difficult than that posed by the Soviet Union during the
Cold War.2 While the Soviet Union was a political and strategic competitor, China will be a formidable political,
strategic and economic competitor. This development will lead to profound changes in the strategic environment of
the Asia-Pacific. Still, the rise of China does not automatically mean that conflict is more likely; the
emergence of a more assertive China does not mean a more aggressive China . While Beijing is
increasingly prone to push its own agenda , defend its interests, engage in more nationalistic – even chauvinistic –
behaviour (witness the Olympic torch counter-protests), and seek to displace the United States as the regional hegemon, this
does not necessarily translate into an expansionist or warlike China. If anything, Beijing appears
content to press its claims peacefully (if forcefully) through existing avenues and institutions of
international relations, particularly by co-opting these to meet its own purposes. This ‘soft power’ process can be described
as an emerging ‘Beijing Consensus’ in regional international affairs. Moreover, when the Chinese military build-up is
examined closely, it is clear that the country’s war machin e, while certainly worth taking seriously, is not
quite as threatening as some might argue.

No Asian War

Alagappa 8 (Muthia, Distinguished Fellow @ Fletcher School of Law and Diplomacy @ Tufts, “The Long Shadow,” International
Affairs p. 512)

Despite this, therole of force in Asian international politics is becoming more limited due to a number of
developments. First, the traditional need for force to protect the territorial integrity of states has
declined in importance. With Iesv exceptions (Taiwan, North Korea, and South Korea) state survival is not
problematic. The Asian political map is for the most part Internationally accepted. although some
boundaries are still in dispute. Such disputes are being settled through negotiations or shelved in the interest of promoting better
bilateral relations (Wang 2003) Second, the political, diplomatic. strategic, military, and economic cost of using force has
increased dramatically. Over the past several decades, a normative framework has developed in Asia that ddegitimizcs the
use of force to invade and occupy another country or to annex territory that is internationally recognized as belonging to another
state. The use of force to invade and occupy another country or to annex territory will incur high costs. For example. If China were to
invade Taiwan without serious provocation, it can expect civil and military resistance in Taiwan, U.S. military intervention,
international condemnation, and a setback to its image as a responsible power. Such action would also incur huge economic costs
resulting from international and domestic disruptions. Unless military action were swift and surgical, it would also result iii
substantial physical damage that would only increase as Asian countries continued to modernize and urbanize. Further. military
action that is not successful can have negative domestic political consequences as well. Third. most Asian countries benefit from
participation in the regional and global capitalist marketplace. The 1997—98 financial crisis sensitized Asian countries to the vagaries
and negative consequences of globalization but did not turn them away from liberalization and participation in the global economy.
Preserving international stability has become a key goal of major powcrs. Economic
growth. modernization, and
growing economic interdependence have increased the cost of the force option and restrained
the behavior of states even when major political issues are at stake , as for example in cross-Strait relations.
Economic interdependence does not close the force option in all cases, hut the high costs of economic disruption can restrain
military action, Further, force is no longer relevant for the attainment of economic goals such as access to resources, labor, and
markets, Energy security, (‘or example, is sought through the market, national stockpiling. and sourcing arrangements. Finally,
resolution of existing disputes through the use of force is not practical . Except for the United States,
none of the Asian states can marshal the necessary military power to impose a settlement by
force. The experience in Iraq and Afghanistan suggests that even the United States suffers limitations
and that the use of force carries much risk. These considerations explain the reluctance of the United States to undertake preventive
action against North Korea, the reluctance of China w carry out its threat of using force to unify Taiwan with the PRC, and the
continuing stalemate in the India-Pakistan confiict over Kashmir. Force may still be used iii these cases, but the attendant strategic,
political, diplomatic, and economic costs and risks are high.

Multiple structural factors check

Alagappa 8 (Muthia, Distinguished Fellow @ Fletcher School of Law and Diplomacy @ Tufts, “The Long Shadow,” International
Affairs p. 512)

International political interaction among Asian states is for the most part rule governed, predictable,
and stable. The security order that has developed in Asia is largely of the instrumental type, with certain
normative contractual features (Alagappa 2003b). It rests on several pillars. These include the
consolidation of Asian countries as modern nation-states with rule-governed interactions, wide-
spread acceptance of the territorial and political status quo (with the exception of certain boundary disputes and a few
survival concerns that still linger), a regional normative structure that ensures survival of even weak
states and supports inter- national coordination and cooperation, the high priority in Asian countries given to
economic growth and development, the pursuit of that goal through partici- pation in regional and
global capitalist economies, the declining salience of force in Asian international politics, the
largely status quo orientation of Asia's major powers, and the key role of the U nited States and
of regional institutions in pre- serving and enhancing security and stability in Asia.
---Asian War- Extensions

Interdependence and democracy check Asia war

Vannarith 10—Executive Director of the Cambodian Institute for Cooperation and Peace. PhD in Asia Pacific Studies,
Ritsumeikan Asia Pacific U (Chheang, Asia Pacific Security Issues: Challenges and Adaptive Mechanism,
http://www.cicp.org.kh/download/CICP%20Policy%20brief/CICP%20Policy%20brief%20No%203.pdf)

Large scale interstate war or armed conflict is unthinkable in the region due to the high level of
interdependency and democratization. It is believed that economic interdependency can reduce conflicts
and prevent war. Democracy can lead to more transparency, accountability, and participation that can reduce
collective fears and create more confidence and trust among the people in the region. In addition, globalism and
regionalism are taking the center stage of national and foreign policy of many governments in the region except
North Korea. The combination of those elements of peace is necessary for peace and stability in the region
and those elements are present and being improved in this region.

No East Asian war --- informal processes secure

Weissmann ‘9 --- senior fellow at the Swedish School of Advanced Asia Pacific Studies (Mikael Weissmann, “Understanding the
East Asian Peace: Some Findings on the Role of Informal Processes,” Nordic Asia Research Community, November 2, 2009,
http://barha.asiaportal.info/blogs/in-focus/2009/november/understanding-east-asian-peace-some-findings-role-informal-
processes-mi)

The findings concerning China’s role in keeping peace in the Taiwan Strait, the South China Sea,
and on the Korean Peninsula confirm the underlying hypothesis that various informal processes
and related mechanisms can help explain the relative peace. Virtually all of the identified processes and
related mechanisms have been informal rather than formal. It should be noted that it is not necessarily the same types of processes
that have been of importance in each and every case. In different ways these
informal processes have
demonstrated that the relative lack of formalised security structures and /or mechanisms have
not prevented the region from moving towards a stable peace. Informal processes have been
sufficient both to prevent tension and disputes from escalating into war and for moving East
Asia towards a stable peace.

No escalation --- economic interdependence checks

Weissmann ‘9 --- senior fellow at the Swedish School of Advanced Asia Pacific Studies (Mikael Weissmann, “Understanding the
East Asian Peace: Some Findings on the Role of Informal Processes,” Nordic Asia Research Community, November 2, 2009,
http://barha.asiaportal.info/blogs/in-focus/2009/november/understanding-east-asian-peace-some-findings-role-informal-
processes-mi)
Economic integration and interdependence   (EII) and the interlinked functional cooperation have been
important, as they have pushed positive relations towards a durable peace. This includes not only
increasing cooperation and economic growth and development, but also developing a feeling of security as the
economic integration and interdependence decreases the fear of others. EII and functional cooperation
have also encouraged and created a need for diplomatic relations and intergovernmental communication
and agreements. They have also been catalysts for all forms of cross-border contacts including
being a driving force for regionalisation. This is clearly seen in Sino–ASEAN relations and the
ASEAN+3 process, but also across the Taiwan Strait where it was part of the cause of the shift in power in the
2008 elections. Together with the Chinese acceptance of multilateralism and its shift from big-power oriented foreign policy to a
focus on soft power and the building of good relations with China’s neighbours, EII has been essential for the medium
to longer-term overarching peace-building process in East Asia. In this context, what has been of
particular importance for peace is both the high degree of economic interdependence that has
developed, as well as the forces of the pan-regional ‘economics first’ policy focus. Here, the general acceptance of the ASEAN
Way as the norm for diplomacy, with its emphasis on conflict avoidance, has worked together
with the economic incentives in preventing conflict escalations and building peace.

Mutual understanding prevents Asian conflicts

Weissmann ‘9 --- senior fellow at the Swedish School of Advanced Asia Pacific Studies (Mikael Weissmann, “Understanding the
East Asian Peace: Some Findings on the Role of Informal Processes,” Nordic Asia Research Community, November 2, 2009,
http://barha.asiaportal.info/blogs/in-focus/2009/november/understanding-east-asian-peace-some-findings-role-informal-
processes-mi)

Elite interactions – i.e. personal networks, track-two diplomacy, and other forms of elite socialisation – have been
essential both on the official and unofficial levels. Firstly, these interactions have been essential for trust and confidence
building, which is of high importance in a region where trust and confidence building are not only
key features of the accepted diplomatic norm, but are also deeply embedded in the regional cultures and
societies. Elite interactions have been essential for peace in all three cases. They have also been important for the possibility
to use back-channel negotiations, something that has been beneficial for conflict prevention across the
cases. Elite interactions have also been important for the development of multilateralism and
the building of peaceful relations. They have also been essential for enhancing the understanding of the
other side(s). Understanding is important, because without an understanding of the others’ thinking, perceived
interests and intentions it is very difficult to prevent conflict escalation, and virtually impossible
to build a longer-term peace. Understanding is also important to be able to overcome the range of
historical issues.

US solves for regional stability and Asian escalation

Weissmann ‘9 --- senior fellow at the Swedish School of Advanced Asia Pacific Studies (Mikael Weissmann, “Understanding the
East Asian Peace: Some Findings on the Role of Informal Processes,” Nordic Asia Research Community, November 2, 2009,
http://barha.asiaportal.info/blogs/in-focus/2009/november/understanding-east-asian-peace-some-findings-role-informal-
processes-mi)
Lastly, theUSA has contributed to peace by working as a frame for acceptable behaviour,
safeguarding against conflict escalation over the war threshold. It has helped to ensure that negative
relations do not escalate into or beyond (temporary) crises. This is important, as little has been done to
address and resolve underlying incompatibilities, tensions, and disputes. By its presence, the US A also gives space
for the range of other processes beneficial for peace to develop in a positive direction. In short,
as the USA is perceived as a safeguard against violent confrontations, the regional parties can
focus on developing good relations and continue to increase cooperation in the economic and
other spheres.
---Asian War- AT: Arms Races

No Asian arms races

Feng ‘10 (professor at the Peking University International Studies. [Zhu, “An Emerging Trend in East Asia: Military Budget
Increases and Their Impact”, http://www.fpif.org/articles/an_emerging_trend_in_east_asia?utm_source=feed]

As such, the
surge of defense expenditures in East Asia does not add up to an arms race. No
country in East Asia wants to see a new geopolitical divide and spiraling tensions in the region.
The growing defense expenditures powerfully illuminate the deepening of a regional “security dilemma,” whereby the
“defensive” actions taken by one country are perceived as “offensive” by another country, which in turn takes its own
“defensive” actions that the first country deems “offensive.” As long as the region doesn’t split into rival blocs, however, an
arms race will not ensue. What is happening in East Asia is the extension of what Robert Hartfiel and Brian Job call “competitive
arms processes.” The history of the cold war is telling in this regard. Arm races occur between great-power rivals only if the
rivalry is doomed to intensify. The
perceived tensions in the region do not automatically translate into
consistent and lasting increases in military spending. Even declared budget increases are
reversible. Taiwan’s defense budget for fiscal year 2010, for instance, will fall 9 percent. This is a convincing case of how
domestic constraints can reverse a government decision to increase the defense budget. Australia’s twenty-year plan to
increase the defense budget could change with a domestic economic contraction or if a new party comes to power. China’s two-
digit increase in its military budget might vanish one day if the type of regime changes or the high rate of economic growth
slows. Without a geopolitical split or a significant great-power rivalry, military budget increases will not likely evolve into “arms
races.” The security dilemma alone is not a leading variable in determining the curve of military expenditures . Nor
will
trends in weapon development and procurement inevitably induce “risk-taking” behavior. Given the
stability of the regional security architecture—the combination of U.S.-centered alliance
politics and regional, cooperation-based security networking—any power shift in East Asia will
hardly upset the overall status quo. China’s military modernization, its determination to “prepare for the worst and
hope for the best,” hasn’t yet led to a regional response in military budget increases. In contrast, countries in the region
continue to emphasize political and economic engagement with China, though “balancing China” strategies can be found in
almost every corner of the region as part of an overall balance-of-power logic. In the last few years, China has taken big strides
toward building up asymmetric war capabilities against Taiwan. Beijing also holds to the formula of a peaceful solution of the
Taiwan issue except in the case of the island’s de jure declaration of independence. Despite its nascent capability of power
projection, China shows no sign that it would coerce Taiwan or become militarily assertive over contentious territorial claims
ranging from the Senkaku Islands to the Spratly Islands to the India-China border dispute.
---Asian War- AT: Maritime Disputes

No impact to Asian maritime disputes

Fravel ’12 (M. TAYLOR FRAVEL is an associate professor of political science and a member of the Security Studies Program at
the Massachusetts Institute of Technology. All Quiet in the South China Sea Why China is Playing Nice (For Now) M. Taylor Fravel
March 22, 2012

Little noticed, however, has been China's recent adoption of a new -- and much more moderate
-- approach. The primary goals of the friendlier policy are to restore China's tarnished image in
East Asia and to reduce the rationale for a more active U.S. role there. The first sign of China's new
approach came last June, when Hanoi dispatched a special envoy to Beijing for talks about the countries' various maritime
disputes. The visit paved the way for an agreement in July 2011 between China and the ten members of
the Association of Southeast Asian Nations (ASEAN) to finally implement a declaration of a code of conduct they
had originally drafted in 2002 after a series of incidents in the South China Sea. In that declaration, they agreed to "exercise self-
Since the summer, senior Chinese
restraint in the conduct of activities that would complicate or escalate disputes."
officials, especially top political leaders such as President Hu Jintao and Premier Wen Jiabao, have repeatedly
reaffirmed the late Deng Xiaoping's guidelines for dealing with China's maritime conflicts to focus on
economic cooperation while delaying the final resolution of the underlying claims. In August 2011, for example, Hu echoed
Deng's approach by stating that "the countries concerned may put aside the disputes and actively explore forms of common
Authoritative Chinese-language media, too, has begun to underscore
development in the relevant sea areas."
the importance of cooperation. Since August, the international department of People's Daily (under the pen name
Zhong Sheng) has published several columns stressing the need to be less confrontational in the South China Sea. In January 2012,
for example, Zhong Sheng discussed the importance of "pragmatic cooperation" to achieve "concrete results." Since the People's
Daily is the official paper of the Central Committee of the Chinese Communist Party, such articles should be interpreted as the
party's attempts to explain its new policy to domestic readers, especially those working lower down in party and state
bureaucracies. In terms of actually setting aside disputes, China has made progress . In addition to the July
consensus with ASEAN, in October China reached an agreement with Vietnam on "basic principles guiding the settlement of
maritime issues." The accord stressed following international law, especially the UN Convention on the Law of the Sea. Since then,
China and Vietnam have begun to implement the agreement by establishing a working group to demarcate and develop the
southern portion of the Gulf of Tonkin near the disputed Paracel Islands. China has also initiated or participated in several working-
level meetings to address regional concerns about Beijing's assertiveness. Just before the East Asian Summit last November, China
announced that it would establish a three billion yuan ($476 million) fund for China-ASEAN maritime cooperation on scientific
research, environmental protection, freedom of navigation, search and rescue, and combating transnational crimes at sea. The
following month, China convened several workshops on oceanography and freedom of navigation in the South China Sea, and in
January it hosted a meeting with senior ASEAN officials to discuss implementing the 2002 code of conduct declaration .
The
breadth of proposed cooperative activities indicates that China's new approach is probably more
than just a mere stalling tactic. Beyond China's new efforts to demonstrate that it is ready to pursue a more cooperative
approach, the country has also halted many of the more assertive behaviors that had attracted attention
between 2009 and 2011. For example, patrol ships from the Bureau of Fisheries Administration have rarely detained and held any
Vietnamese fishermen since 2010. (Between 2005 and 2010, China detained 63 fishing boats and their crews, many of which were
Vietnamese and Philippine vessels have been able to conduct
not released until a hefty fine was paid.) And
hydrocarbon exploration without interference from China. (Just last May, Chinese patrol ships cut the towed
sonar cable of a Vietnamese ship to prevent it from completing a seismic survey.) More generally, China has not
obstructed any recent exploration-related activities, such as Exxon's drilling in October of an exploratory
well in waters claimed by both Vietnam and China. Given that China retains the capability to interfere with such
activities, its failure to do so suggests a conscious choice to be a friendlier neighbor. The
question, of course, is why did the Chinese shift to a more moderate approach? More than
anything, Beijing has come to realize that its assertiveness was harming its broader foreign
policy interests. One principle of China's current grand strategy is to maintain good ties with great powers, its immediate
neighbors, and the developing world. Through its actions in the South China Sea, China had undermined this principle and tarnished
the cordial image in Southeast Asia that it had worked to cultivate in the preceding decade. It had created a shared interest among
countries there in countering China -- and an incentive for them to seek support from Washington. In so doing, China's actions
provided a strong rationale for greater U.S. involvement in the region and inserted the South China Sea disputes into the U.S.-
Chinese relationship. By last summer, China had simply recognized that it had overreached . Now,
Beijing wants to project a more benign image in the region to prevent the formation of a group of Asian states allied against China,
reduce Southeast Asian states' desire to further improve ties with the United States, and weaken the rationale for a greater U.S. role
in these disputes and in the region. So far, Beijing's new approach seems to be working, especially with
Vietnam. China and Vietnam have deepened their political relationship through frequent high-level exchanges. Visits by the
Vietnamese Communist Party general secretary, Nguyen Phu Trong, to Beijing in October 2011 and by the Chinese heir apparent, Xi
Jinping, to Hanoi in December 2011 were designed to soothe spirits and protect the broader bilateral relationship from the
unresolved disputes over territory in the South China Sea. In October, the two also agreed to a five-year plan to increase their
bilateral trade to $60 billion by 2015. And just last month, foreign ministers from both countries agreed to set up working groups on
functional issues such as maritime search and rescue and establish a hotline between the two foreign ministries, in addition to
starting talks over the demarcation of the Gulf of Tonkin.
---Asian War- AT: Asian Pivot

Asia pivot fails

Auslin ‘12 Michael, scholar at the American Enterprise Institute, columnist at wsj [“America Doesn't Need a Pivot to Asia,” 8/27,
http://online.wsj.com/article/SB10000872396390444506004577614941100974630.html?mod=googlenews_wsj]

It is time to bury the Obama administration's pivot to Asia. This reallocation of military and diplomatic resources was
supposed to guarantee stability in a region seeking to balance China's rise . In reality, this strategic
shift is less than it appears. It won't solve Asia's problems and may even add to the region's uncertainty by over-
promising and under-delivering. Everything wrong with the pivot can be summed up by Four R's: rhetoric; reality;
resourcing; and raising expectations and then doubts. So far, the first and perhaps biggest problem with the idea of the pivot
—or, as the Defense Department calls it, the rebalancing—is that it remains largely rhetorical, vague and
aspirational. True, there are some laudable moves, such as basing U.S. Marines in northern Australia
and agreeing to port new U.S. warships in Singapore. These, however, hardly add up to a breakthrough .
The world still wonders what the purpose is: to contain China, to promote democracy, to make the United States the de facto
hegemon of Asia, or simply to reassure nervous nations about China's rise? The reality is that not much will change in America's
actions. The pivot says nothing about taking on new commitments, for example toward the Association
of Southeast Asian Nations or to countries with whom America does not currently have formal alliances. Just as importantly,
Washington has made clear in recent months that it will not take sides in the territorial disputes that have roiled the East and South
China Seas, even when allies like Tokyo and Manila are involved. Further evidence for this reality comes from the resource
constraints imposed on this grand project. The Obama administration is trying to do it on the cheap. Pivot funding is in danger from
sequestration—forced budget cuts resulting from larger budget politicking in Washington—that, if allowed to proceed, will cut
another $500 billion from a defense budget already reduced by $900 billion since 2009. The administration claims that America's
military presence in Asia will not be affected by these budget cuts. If that is so, then U.S. military posture in the rest of the world will
be cut back. More likely, any buildup will be difficult to sustain. The shifting of more planes and ships to the Pacific will soon slow
down, as the size of the Air Force and Navy shrink, and as other world problems such as Iran and Syria continue to dominate the
attention of American policy makers. This, in turn, is raising doubts about the pivot in Asia, so soon after the rhetoric from
Washington had raised expectations. Countries such as Vietnam and the Philippines led themselves to believe that the pivot would
have concrete results, such as quickly increasing American presence in the region and perhaps even American support in their
maritime territorial disputes with China. Both accordingly reached out to Washington, holding new military exercises or discussing
greater security cooperation. Yet this enthusiasm makes it all the worse when those hopes turn out to be dashed by Washington's
failure to act. As one Philippines senator asked during his country's standoff this spring with China over the Scarborough Shoal, what
unrealistic
good is the alliance with the U.S. if America refuses to back up its partners in times of need? By appearing to make
promises, the Obama administration has created new diplomatic headaches for itself in managing the fall-out
from its failure to deliver. What then is the point of the pivot? By not getting involved in maritime disputes, other than
rhetorically, Washington is actually taking the most realistic approach possible. No administration, Republican or Democratic,
is going to risk a crisis with China short of any overt attempt by Beijing to take over territory clearly controlled by other
nations. Building up U.S. forces in Asia, were it even possible, would not change that political calculation. The current
American military posture can be diversified to a few more countries, but essentially, Washington has had the right balance for the
past several decades. While it would be a mistake to shrink the U.S. air and naval presence in Asia, all Washington could do is slightly

increase it, and that will change nothing in the region . Moreover, there are few realistic options for new partners in
Asia, especially ones such as Japan and Australia that can provide some level of regional security cooperation. That means America's
current grouping of allies and partners is right-sized for the political and security realities of the Asia-Pacific for the foreseeable
future.
Asia pivot is inflated- the status quo solves literally every impact

Innocent ‘12 [Malou Innocent is a foreign policy analyst at the libertarian Cato Institute, “Talk of a U.S.-Asia 'Pivot' Is
Overblown,” http://www.usnews.com/opinion/blogs/world-report/2012/12/13/talk-of-a-us-asia-pivot-is-overblown]

The greatest misperception surrounding Washington's "pivot" to Asia is that America's dominant
presence is not already felt there on a regular basis. It is.¶ The United States plays a considerable role
in the Far East, despite the Obama administration's proclamations last autumn that it would "pivot" or
"rebalance" there in the future. For one, the United States maintains forward-deployed forces in South
Korea, with 28,500 U.S. troops; Guam, with 4,500 U.S. troops; and Japan, with 40,000 U.S. troops. Guam, of course, is part of
America as a non-self-governing, unincorporated tervritory. South Korea and Japan, however, after decades of proven internal
stability and peaceful democratic transitions, are equipped to defend themselves.¶ [See a collection of political cartoons on defense
spending.]¶ Once upon a time, South Korea was incapable of surviving without America's support. That began to change in the
1980s. Today, its economy ranks around 13th in the world, it has twice the North's population, and, if South Korea's leaders chose
to, could be spending on defense the equivalent of the North's entire annual GDP.¶ As for Japan, despite its recent economic woes,
it had the fifth highest defense budget in the world in 2011, according to the International Institute for Strategic Studies. Japan
surpassed Russia, India, and Brazil, and fell only behind the United States, China, the United Kingdom, and France. Moreover, as
scholars Shinichi Ogawa and Michael Schiffer have pointed out, in criticism of its policy, Japan possesses a nuclear "breakout''
capacity, meaning its civilian nuclear fuel cycle is so advanced "that, at the flip of the switch, [it] could be militarized."¶ Save for a
planned contingent of 2,500 U.S. Marines in Australia, four littoral combat ships stationed in Singapore, and rotating troops and
surveillance aircraft in the Philippines, it is unclear whether U.S. troop deployments will grow more robust in Japan and South Korea.
They should not. Such prosperous allies can live without the generous welfare of American taxpayers.¶ [Read the U.S. News Debate:
Should Cuts Be Made to Domestic Social Programs to Protect the Defense Budget?]¶ Aside from these forward-deployed forces,
the Far East feels Washington's constant presence with the United States Pacific Command. This regional
unified military structure consists of about one-fifth of total U.S. military strength. It includes six aircraft
carrier strike groups, about two-thirds of U.S. Marine Corps combat strength, and the U.S.
Pacific Fleet, which goes on frequent patrols conducting joint, military-training exercises with
America's allies and partners.¶ Talking about partners, Uncle Sam has a lot of them in a region home to over 50 percent
of the world's population. The United States has been cultivating warmer relations with India, most
especially after accommodating New Delhi's nuclear expansion with a symbolic, 2008 agreement
facilitating civilian nuclear cooperation between them. Moreover, despite recent hand wringing over U.S.-Russia relations,
Washington's so-called "reset" has rebounded ties from their 2008 low, particularly with regard to Moscow's help supplying NATO's
war effort in Afghanistan.¶ [Read the U.S. News Debate: Are Cuts to the Defense Budget Necessary?]¶ Elsewhere, the
United
States has forged better relations with Vietnam, Myanmar, Indonesia, Singapore, and the
Philippines, and has embraced existing multilateral organizations and trade agreements, like the
East Asia Summit, the Trans-Pacific Partnership, and the Association of Southeast Asian
Nations.¶ In the end, Washington's obsessive fixation on the Middle East and North Africa should warrant serious
reconsideration. More of America's attention should be paid to the future of the Asia-Pacific, since maintaining peace in that region
will be the challenge of the 21st century. However, what foreign policy planners in Washington should be asking themselves is what
the United States should be willing to defend in this region, and at what cost? What implicit commitments should Washington make
to prosperous, populous countries eminently capable of defending themselves? Allies are intended to supplement a nation-state's
power, not hinder or jeopardize it.¶ [See a collection of political cartoons on the Middle East.]¶ Primarily, America's deepening
involvement in Asia is meant to reassure allies nervous over China's growing assertiveness and increased military spending.
However, the United States can both value being a strong military power and allow other countries in the Far East to assert a greater
leadership role. These policies are neither zero-sum nor mutually exclusive.¶ For more than half a century, the United States has
played a prominent military and economic role in the Asia-Pacific. The American people should not be led to believe that their
country was a never a force to be reckoned with there. Indeed, the
biggest tale proponents of U.S. prominence in
Asia ever sold was the intimation that we do not already have it .
The Asia pivot is a lie- no military shift and diplomatic pivot solves

Moss ‘12 [Trefor Moss is an independent journalist based in Hong Kong and former Asia-Pacific
editor for Jane's Defence Weekly, “Military the weakest link in US 'pivot' to Asia,”
http://www.scmp.com/comment/insight-opinion/article/1106840/military-weakest-link-us-
pivot-asia]

Worried about the rise of China? Anxious about the disappearance of the world you know? Fear not. Uncle Sam has a plan. The
"pivot" to Asia - the United States' historic shift in emphasis from the Atlantic to the Pacific - has been in motion for a year now. ¶ The
US' goal is to stress-proof the Asia-Pacific region, to enable it to bend but not break under the strain of rapid geopolitical change.
This can be done, the Obama administration reckons, by revamping America's regional presence (military and otherwise) and
sending out a strong signal that America will remain the Pacific's pre-eminent power. The economic and security benefits of the
rebalancing would then be felt both in the US and Asia.¶ While the pivot is a cross-government plan with many dimensions, its
military element has received the most attention. This is unfortunate because the military aspect is the pivot's weak
point. It's also dangerous, because the US risks deceiving its allies and deluding itself about what the pivot in its military guise can
realistically achieve.¶ Faced with the rise of China as a military power, many Asian countries have enthusiastically endorsed
Washington's plans, favouring the US system they know over the Chinese system they don't. Australia, the Philippines and Singapore
are among those keen to help Washington deliver what Leon Panetta, the US defence secretary, has couched as "enhanced
presence, power projection and deterrence in the Asia-Pacific". ¶ It all sounds very reassuring. But the strategy itself isn't the
problem. The crux is whether the US retains the dynamism and material resources needed to deliver the enhanced Asia-Pacific
security that its Asian friends are hoping for. What they will find, to their great disappointment, is a worrying disconnect between
the US' growing military ambitions in Asia and its diminishing power to realise them. ¶ Militarily,
the US is doing what it
can: marines are being deployed to Australia, and several small navy ships will be based in
Singapore. But these are modest changes - not the large-scale deployments that would alter
Asia's strategic landscape.¶ Right now, they are all America can afford. After a free-wheeling decade of
wars and blank cheques, the US military is entering a cycle of bruising cutbacks, not global expansion.
Around US$500 billion has already been sliced from the Pentagon's budget over the next decade
as the federal government faces up to its debts. Under the axe are 70,000 troops, hundreds of
aircraft and more than a dozen naval ships. The US navy , whose expansion was to have enabled the pivot, is
set to shrink as ship-building programmes are stretched to save money .¶ Now, like a masochist warming
to the first lashings of pain, Washington seems determined to visit further damage on itself, as President Barack Obama and senior
Republicans bang heads over the 2013 federal budget. If they fail to reach a deal, they will in early January tip the country over the
"fiscal cliff", triggering a second round of deep budget cuts. For the Defence Department, this exercise in national self-harm will
mean a second US$500 billion hit.¶ These extra cuts, if they happen, will hardly shatter the US military's world. The Pentagon is
planning a raft of creative cost-saving measures to spare the front line, like forcing civilian staff to take unpaid leave. Overall, US
defence spending over the decade would shrink from around US$6 trillion to US$5 trillion. That's still a huge allocation. ¶ And yet, the
loss of US$1 trillion hardly prepares the ground for an Asian build-up sufficient to change the global game. Nor does it reassure your
allies or deter your competitors. Deputy Secretary of Defence Ashton Carter recently insisted that the pivot would still be militarily
meaningful: 70,000 additional US soldiers could be stationed in the Asia-Pacific, he revealed. But where these troops might go, and
whether the US could even afford to deploy them if confronted by the extra cuts, remains moot. ¶ Where the US really is
ramping up its efforts in Asia is diplomacy and trade - the things that don't cost very much .¶ In
particular, Washington is forging ahead with the T rans-Pacific Partnership, a new free-trade
agreement involving eight other countries on either side of the Pacific. Others, perhaps China,
could join later. Critics of the pivot's military emphasis think Obama should have pursued a strategy of economic engagement
all along; a military build-up will only antagonise China, they argue, while expanding free trade will promote stability and
prosperity.¶ The pivot to Asia, it's now clear, is a primarily political and economic exercise . Maybe that's the
right approach, anyway.¶ But let's end the confusion about the pivot's military import. The US isn't really pivoting to
Asia militarily: it's struggling to maintain the presence it already has her e. To pretend otherwise would be
for Washington to take China's rising-power formula - "hide your strength, and bide your time" - and turn it upside down, muttering
"exaggerate your strength, and do it now" behind its own hand. That would be the mantra of a power in decline. ¶ The
US can
remain a force in Asia. But it's no good denying that US military power in Asia has already
reached its high-water mark.
Asteroids Answers
Frontline

No asteroid threat

Wall ‘3-14
(Michael, Michael was a science writer for the Idaho National Laboratory and has been an intern at Wired.com, The Salinas
Californian newspaper, and the SLAC National Accelerator Laboratory. He has also worked as a herpetologist and wildlife biologist.
He has a Ph.D. in evolutionary biology from the University of Sydney, Australia, a bachelor's degree from the University of Arizona,
and a graduate certificate in science writing from the University of California, Santa Cruz. “Deflecting Killer Asteroid Could Be
Geopolitical Nightmare” by Mike Wall, SPACE.com Senior Writer: 14 March 2013)

While astronomers have spotted 95 percent of the 980 near-Earth asteroids at least 0.6 miles (1 km) wide, which
might end civilization if they hit us, many smaller but still hazardous space rocks remain undetected. Researchers have discovered
less than 30 percent of the close-flying 330-foot-wide (100 m) objects, for example, which could destroy an area the size of a state if
they hit us. And they've mapped out the orbits of less than 1 percent of the 130-footers thought to be out there, which could wipe
out a city. In all, just 9,700 near-Earth asteroids have been catalogued to date, out of a population numbering in the millions. Many
astronomers and politicians are thus calling for more resources to be put toward asteroid-detection efforts, so that we have a better
idea of what's headed our way in the future. Something big will hit us again, experts say, and we'll probably need years or decades of
warning to do something about it. [Meteor Explodes Over Russia (Video)] How to deflect an asteroid Scientists think they
know how to deflect an asteroid headed directly for Earth, given enough lead time. The strategy involves mounting at
least two coordinated space missions, Schweickart said. The first would slam a kinetic impactor into the asteroid to knock it off
course. The second would launch a "gravity tractor" probe to fly alongside the space rock, nudging it farther via a tiny but
persistent gravitational tug. "You always need a gravity tractor there to ensure that any deflection does not end up having the
asteroid go through a [gravitational] 'keyhole,' which would simply bring it back later," Schweickart told SPACE.com. This
approach can avert more than 98 percent of projected collisions, according to a 2008 report called "Asteroid
Threats: A Call for Global Response." The report was put together by the Association of Space Explorers' International Panel on
Asteroid Threat Mitigation, which Schweickart chaired. [Photos: Asteroids in Deep Space] A different strategy — such as a nuclear
blast — may be needed for asteroids more than 1,300 feet (400 m) wide, or for those detected with little warning time, the report
adds. But
such dire cases are likely to come along just once every 100,000 years or so. Political hurdles
Humanity has successfully demonstrated both aspects of the impactor-tractor technique. In 2005, for
example, NASA smashed a probe into the Comet Tempel 1 to investigate the icy body's composition. And several spacecraft, such as
NASA's Dawn probe and Japan's Hayabusa craft, have rendezvoused with asteroids in deep space. But the main challenges of an
asteroid-deflection mission will probably be more political than technical, Schweickart said. And perhaps the biggest hurdle of all will
be getting the world to agree which way to steer the asteroid. Each incoming asteroid whose orbit has been mapped will threaten
Earth along a specific "risk corridor" — a line of potential impact sites that extends about 180 degrees across the planet's surface but
is just a few tens of kilometers wide. A deflection campaign would not be able to make huge changes to the dangerous asteroid's
orbit. Rather, it could merely drag the projected impact point along the risk corridor to the left or to the right, by slowing the space
rock down or speeding it up slightly. The goal, of course, would be to drag the impact site off the planet entirely. But determining
how exactly to achieve this — to go left or right, to push the asteroid or to pull it — would be difficult, for any decision would
necessarily put some nations at greater risk than others, Schweickart said. "If you start a deflection and something goes wrong, you
have now shifted that impact point along the risk corridor one way or the other," Schweickart said. "And now people are in danger
who were not in danger before you started this operation." And that's just one of many tricky geopolitical issues a potential
asteroid-deflection mission would present, he added. "Who does it? Who pays for it? Who handles the liability?" Schweickart said.
"How does the decision get made? Do we deflect the 40-meter object, or do we evacuate? Who makes that decision? And who pays
for the evacuation? Everybody, because we all collectively made the decision? Or do the people who were unlucky enough to
evacuate have to handle the cost?" Getting the ball rolling Schweickart and many of his colleagues think it's imperative to develop an
international framework for dealing with the asteroid threat, so that the world can work together to predict and prevent
catastrophic impacts. The Association of Space Explorers (ASE) — a group of veteran astronauts and cosmonauts — lays out a
proposed framework in its 2008 report, calling for the United Nations to oversee a three-tiered program consisting
of 1) an international asteroid detection and warning network; 2) a mission-planning group that would
assess deflection options; and 3) a mission-authorization body tasked with developing "the policies and guidelines
that represent the international will to respond to the global impact hazard." The United Nations' Action Team on Near-Earth
Objects — better known as Action Team 14 — embraced much of this plan in a set of recommendations it drew up late last
year. These recommendations are likely to be adopted by the U.N.'s Working Group on Near-Earth Objects, then go up the chain
from there, ASE officials said. "Given the work methods of the [UN's] Committee on the Peaceful Uses of Outer Space, the
recommendations of the Working Group will likely be adopted by the Committee in June," ASE officials wrote in a
Feb. 27 update. "The [UN] General Assembly will likely follow suit in October without further discussions." The U.N.'s ponderous
pace and many layers of bureaucracy may be frustrating to scientists who have been working the technical side of the hazardous-
asteroid problem for years. But Schweickart voiced optimism that the political progress made will eventually be worth the wait.
"When you get there, the whole world has bought into it," he said. "I mean, you're moving the whole world."

No asteroid impact and long timeframe

Chinese Academy of Sciences ’13 (“Earth Safe From Asteroids for 100 More Years” 2013)

A number of asteroids have flown by Earth in recent days, but astronomers say it’s unlikely to see devastating
asteroid impact on Earth at least in the next 100 years. Scientists have found 1,345 potentially Earth-
threatening asteroids with a diameter of at least 140 meters and whose closest distance from Earth is less than 7 million kilometers,
or 18.5 times the average Earth-moon distance, says Li Chunlai, a researcher at the Center of Lunar and Deep Space Explorations
under China's National Astronomical Observatories. "But we do not need to worry about it. Astronomers have collected
accurate information about the orbits of these potentially Earth-threatening asteroids. We are unlikely to see devastating asteroid
impact at least in the next 100 years," Li says. "All areas on Earth are equally possible to be hit by an asteroid. Oceans cover
about 70 percent of Earth’s surface, and most of the land is uninhabited. This has greatly reduced the
possibility of an asteroid hitting inhabited areas,” Li says. "An asteroid impact may injure people mainly through the shock wave that
is formed while the object falls off," Li says. Before the asteroid hits the ground, the shock wave shows its power and shatters the
windows of buildings. Lin Yangting, a researcher at the Institute of Geology and Geophysics under the Chinese Academy of Sciences,
says that more than 1,000 people were injured during a recent meteor event in Russia, and their injuries were mainly caused by
collapsed buildings and broken window glass shattered by the shock wave. "Some people are worried that the meteoroid may also
cause injuries through radiation. In fact, asteroids and meteoroids contain few radioactive substances , and
they usually burst dozens of kilometers above the ground, so their radiation is negligible," Lin says. Wang Sichao, a researcher at the
Purple Mountain Observatory under the Chinese Academy of Sciences, says that a large
asteroid is unlikely to hit Earth
without being noticed by astronomers beforehand. An explosive device sent to a potentially
Earth-threatening asteroid by spacecraft several years in advance can change the speed and direction of the
asteroid through explosion. It may have only a little impact on the orbit of the asteroid, but several years later, it will move far away
from Earth and pose no threat to mankind.
---Asteroids- Squo Solves
Status quo solves asteroids

Mann ‘2-22
(2013, Adam, Wired Science staff writer, “Efforts to Protect Earth From Asteroids Are Under Way. But Will It Be Enough?” Wired)

While no new money is likely to be coming to asteroid-spotting activities, the


Russian strike may cause “a shift in
priorities to looking at this more than we have at the past,” said space policy expert Henry Hertzfeld of George
Washington University. Many ground-based telescopes are already actively watching the heavens, most
coordinated through NASA’s Spaceguard program. These include the Catalina Sky Survey, the U.K. Spaceguard Centre, and the
Japanese Spaceguard Association. A similar undertaking from the European Space Agency, the Space Situational Awareness
program, works with amateur astronomers to monitor dangers from above, including near-Earth asteroids. A new endeavor coming
online in 2015 named the Asteroid Terrestrial-impact Last Alert System Project ( ATLAS)
will provide an early warning
system that could provide one week’s notice for city-destroying 45-meter asteroids and three week’s notice for potentially
devastating 140-meter objects. But the next and most important frontier is watching from space . By sheer
coincidence, a Canadian Space Agency project that started in 2008 is scheduled to launch on Feb. 25. Named the Near-Earth Object
Surveillance Satellite (NEOSSat), this suitcase-sized spacecraft will
circle the planet every 100 minutes,
scanning space for asteroids that may someday pass near the Earth. Being in space means that NEOSSat will
solve one of the main problems with current asteroid-watching techniques: You can’t do them during
the day. A telescope aimed at the sky when the sun is up will see nothing other than bright sunlight. This is the reason the
Chelyabinsk meteor went unnoticed — it came in from the dayside of our planet. The NEOSSat project will begin operation shortly
after launch and look for some of the most dangerous objects larger than half a kilometer. Even more robust space-
based telescopes are now being readied by the private sector. Planetary Resources, Inc., a company that
aims to mine asteroids, intends to launch a small telescope named the Arkyd-100 that will sit in low-Earth orbit and scan for nearby
space rocks. Though its primary goal will be to find valuable asteroids, the spacecraft will be able to watch for possibly dangerous
objects coming from the sunlit side of Earth. Arkyd-100 is expected to launch in 18 to 20 months. A more targeted effort comes from
the B612 Foundation, which plans to launch the
Sentinel telescope in late 2016. This spacecraft would sit inside the
orbit of Venus and constantly be on the lookout for killer asteroid s, whichever direction they come from.
Sentinel will spot nearly all asteroids 150 meters or larger and identify a significant portion of those down to 30 meters in diameter.
With a price tag of several hundreds of millions, the project is comparable to a large municipal undertaking like a new highway
overpass. But it is still much more than anything accomplished by a private non-profit in space and so there has been some doubt
whether or not B612 could wrangle enough people into backing their idea. Since Chelyabinsk, their telescope is on surer
footing. “There’s been a crush of people contacting us,” said Lu. “Suddenly it’s on everyone’s
mind.” Online donations have increased tenfold, he added. While most of these are fairly small, generally around $100, many
richer potential donors who were sitting on the fence have now fully signed on to the company’s plans. Entrepreneur Peter
Diamandis, co-founder of Planetary Resources, reported a similar effect with investors who were previously hesitant but are now
more willing to back in their venture. Should one of these telescope projects one day spot a doomsday asteroid, there
are
already many proposals for how to deflect the object. The method most favored by the B612 Foundation is
fairly simple – place a modest-size spaceship near the asteroid and have its gravitational pull slowly alter the object’s course,
averting catastrophe. The technique is relatively low cost and would certainly work but would require
several years or possibly decades of advanced notice. For a large rock that’s headed right for us fairly quickly, B612 recommends
smashing a giant projectile into the asteroid. This
kinetic impactor would provide a strong kick to shuffle an
asteroid off its current trajectory. Later, a gravity tractor could be used to make any more precise
changes necessary.

Lasers solve now


Kumar ‘2-17
(2013, Prashant, Administrator/writer for Vivid Times, “Scientists Propose System to Vaporize Asteroids That Threaten Earth”)

It has been an eventful year for space enthusiasts, with it seeming like a new asteroid is announced every week, each one coming
closer and closer to our blue marble. Then, just a few days ago, a large meteorite smashed into Russia, injuring over 1000 people.
Now a researcher and a physicist have teamed up to propose a solution to the problem – a laser-beam generating solar-based
asteroid destroyer that could destroy a space rock half the size of a football field in 30 minutes. UC Santa Barbara physicist
and professor Philip M. Lubin, and Gary B. Hughes, a researcher and professor from California Polytechnic State University, San
Luis Obispo, conceived DE-STAR, or Directed Energy Solar Targeting of Asteroids an exploRation, as a realistic means of
mitigating potential threats posed to the Earth by asteroids and comets. “We have to come to grips with discussing these
issues in a logical and rational way,” said Lubin, who began work on DE-STAR a year ago. “We need to be proactive rather than
reactive in dealing with threats. Duck and cover is not an option. We can actually do something about it and it’s credible to do
something. So let’s begin along this path. Let’s start small and work our way up. There is no need to break the bank to start.” The
system will work by using solar panels to turn energy from the sun into a “phased array” of individual laser beams
that are channeled into a single mega-laser beam that eradicates the asteroids, a la the Death Star. If the
asteroid is too large, DE-STAR would work by deflecting it away from our fragile planet. The best part? The concept is based on
technology that is widely available now. “This system is not some far-out idea from Star Trek ,”
Hughes said. “All the components of this system pretty much exist today. Maybe not quite at the scale that we’d
need –– scaling up would be the challenge –– but the basic elements are all there and ready to go. We just need to put them into a
larger system to be effective, and once the system is there, it can do so many things.” “These are not just back-of-the-envelope
numbers,” Hughes concurred. “They are actually based on detailed analysis, through solid calculations, justifying what is possible.
And it’s all available under current theory and current technology. “There are large asteroids and comets that cross the Earth’s orbit,
and some very dangerous ones going to hit the Earth eventually,” he added. “Many have hit in the past and many will hit in the
future. We should feel compelled to do something about the risk. Realistic solutions need to be considered, and this is definitely one
of those.”

Deflection solves

Britt 8 (Robert Roy, “Will an Asteroid Hit Earth?”, Live Science, 8-7,
http://www.livescience.com/mysteries/070116_asteroid_hit.html)

But no,a continent-destroying asteroid is not likely to hit during your lifetime. Most of 1,100 or so that
could do the job have been found. And none are on their way . Okay, there is one mid-sized rock—called
Apophis—that has a small chance of striking Earth in 2036 and wreaking some regional havoc. But astronomers are watching
it and, if
future observations reveal it really could hit us, scientists are confident they can
devise a mission to deflect it. And if all else fails, some futurists suggests, humanity could simply set
up shop elsewhere.
---Asteroids- No Impact

No impact to asteroids

Woods 2 (Michael, National Bureau – PPG, “Scientists Gather This Fall to Study Preventing an Asteroid Strike on Earth”,
Pittsburgh Post-Gazette, 6-17, http://www.post-gazette.com/healthscience/20020617asteroids0617p2.asp)

Objects the size of 2002 EM7 make similarly close approaches to Earth several times a month, Williams said. They hit
Earth every 30 to 100 years, but usually burn up in the atmosphere . Such impacts, however, create
an air burst, or powerful shock wave, that can cause great damage on the ground. "The 1908 Tunguska event was an example of
the local damage that would occur under and around the air burst of such an object," Williams explained. In that incident near
the Stony Tunguska River in Siberia, a mysterious air explosion -- now believed to be an asteroid impact -- leveled a section of
forest half the size of Rhode Island. Scientists estimate it caused as much destruction as a 15-megaton nuclear bomb.
"Impacts by such objects are not
likely to cause major loss of human life," Williams said. "About 70
percent of the world's surface is water, and much of the land mass is either uninhabited or very
sparsely populated."

No asteroid is likely to hit Earth

Easterbrook 3 (Gregg, Senior Fellow – New Republic, “We’re All Gonna Die!”, Wired Magazine, July,
http://www.wired.com/wired/archive/11.07/doomsday.html?pg=1&topic=&topic_set=)

Estimates by Alan Harris of the Space Science Institute of Boulder, Colorado, suggest that 500,000 asteroids
roughly the size of the Tunguska rock wander through Earth's orbit. Much spookier are asteroids big enough to
cause a Chicxulub-class strike. At least 1,100 are believed to exist in Earth's general area, some capable of plunging the planet
into a years-long freeze while showering the globe with doomsday rain as corrosive as battery acid. None of these killer
rocks is known to be on a collision course with Earth - but then, the courses of hundreds have yet to be charted.

The probability is 1 in 2.5 millions

Thomson 8 (Keith, Reporter – Garden & Gun, “Chances You'll Be Killed by an Asteroid in 2009”, Huffington Post, 12-30,
http://www.huffingtonpost.com/keith-thomson/chances-youll-be-killed-b_b_154138.html)

A computer video circulating the internet has rekindled fears that an asteroid will hit Earth and send
mankind the way of the brontosaurus. Based on NASA projections, there is indeed a chance that such an asteroid
will impact Earth in the next year. It is 1 in 2,518,072 This number is derived from NASA calculations of the
likelihood of a strike by any one of the six substantial Near-Earth Objects (NEOs) whose current course could intersect our
planet's in 2009. The most likely of the bunch, an NEO named 2008 AO112, alone has a 1 in 4,000,000 chance of impacting
Earth. In other words, there's a 99.999975% chance the thing will miss us. By comparison, in the new year, based on recent
National Safety Council data, chances are less that you will be killed by an asteroid than by the
following: Motor vehicle accident: 1 in 6,539 Exposure to noxious substances: 1 in 12,554 Assault by firearm: 1 in 24,005
Accidental drowning: 1 in 82,777 Exposure to smoke, fire or flames: 1 in 92,745 Exposure to forces of nature (lightning, flood,
storms, etc.): 1 in 136,075 Falling out of bed or off other furniture: 1 in 329,819 Choking on food: 1 in 343,179 Air
and space transport accidents: 1 in 502,554 Exposure to electric current, radiation, temperature, and pressure: 1 in 705,969
Being bitten, stung or crushed by another person or animal: 1 in 1,841,659 Conclusion: It would be statistically unwise to sell
your home (your chances of selling it aside) and use the proceeds for a pre-asteroid splurge in the tropics. Alternatively, if you
are considering fleeing Earth, you are more likely to die by spacecraft accident than by asteroid . And if
you do so anyway, given the chance of being bitten, stung or crushed by another person or animal, your chances are even worse
if you bring company.

It’s one of the least likely existential risks - default to ours

Bostrom ‘6 (Nick, director of the Future of Humanity Institute at the University of Oxford, Global
Agenda, www.globalagendamagazine.com/2006/Bostrom.asp)

It is sad that humanity as a whole has not invested even a few million dollars to improve its thinking about how it may best
ensure its own survival. Some existential risks are difficult to study in a rigorous way but we will not know what insights
we might develop until we do the research. There are also some sub-species of existential risk that can be measured,
such as the risk of a species-destroying meteor or asteroid impact. This particular risk turns
out to be very small. A meteor or an asteroid would have to be considerably larger than 1km in
diameter to pose an existential risk. Fortunately, such objects hit the Earth less than once in 500,000
years on average.
Balkans War Answers
Frontline

No Balkans escalation – stability now

ANSAmed, ’12 (2/1/12,


http://www.ansamed.info/ansamed/en/news/sections/politics/2012/02/01/visualizza_new.html_72797168.html)

(ANSAmed) - BELGRADE, 1 FEBRUARY - Strengthening of regional cooperation in the Balkan area, especially
in the fight to organized crime and the common target of European integration were the main
focus of a summit of Foreign Ministers of some South-East Europe Cooperation Process (SEECP) member
states. The summit took place last night in Belgrade. The summit was chaired by Serbian Foreign Minister Vuk Jeremic and several
foreign miniters took part in it: Croatian Vesna Pusic, Milan Rocen from Montenegro, Greek Stavros Dimas, Macedonian Nikola
Poposki and Ahmet Davutoglu from Turkey. Jeremic also proposed to celebrate the one hundredth anniversary of the end of Balkan
wars with a joint ceremony: "from Balkan wars to peace in the Balkans". Jeremic maintains that this would be a
tangible way to improve cooperation among Balkan countries after a century of conflicts and
clashes. The summit participants unanimously supported Serbia's EU access application, hoping
that the EU summit scheduled for the beginning of March will have a positive outcome in regard. Opportunities for
strengthening cooperation among the region's countries were also analysed during several meetings
and bilateral talks between the Foreign Ministers. The South-East Europe Cooperation Process was initiated in 1996 with
a view to transforming South-Eastern Europe into a region of stability , safety and cooperation,whose
target is European integration . There are twelve SEECP member countries: Albania, Bosnia and Herzegovina, Bulgaria,
Montenegro, Greece, Croatia, Macedonia, Moldova, Romania, Serbia, Slovenia and Turkey. Serbia is the Chairman in office until next
June (ANSAmed)

Balkans conflict won’t escalate.

Bandow ’99 [Doug, Senior Fellow of the CATO Institute, Mar 10, “The U.S. Role in Kosovo,”
http://www.cato.org/testimony/ct-db031099.html]

Most important, it would put U.S. troops at risk without any serious, let alone vital, American interest at stake. To paraphrase
German Chancellor Otto von Bismarck, the
Balkans is not worth the bones of a single healthy American rifleman. Yugoslavia
obviously poses no direct threat to the U.S. or any U.S. ally.  Some argue that there are indirect dangers: failing to
act risks another continental, if not global, conflict. Contended former German Foreign Minister Klaus Kinkel: "Everything must be
done to insure that another awful conflagration does not explode in Europe."  It is a paranoid fantasy to imagine
Serbia alone inaugurating such a conflict, however. Serbian legions will not be marching on Ankara, Athens, or
Tirana, let alone Berlin, Moscow, or Paris. Only if other states joined in could the war become a serious one.  This was, of course, the
same argument used for Western intervention in Bosnia. Yet the Yugoslavian civil war, running from Slovenia through Bosnia, lasted
longer than World War I without expanding beyond Yugoslavia. The lesson is obvious: it is better for surrounding states to remain
aloof rather than to intervene in ethnic strife, thereby building firebreaks to rather than transmission belts for war.  Even
if the
conflict in Kosovo spilled over into Albania and Macedonia, no major power would join the
conflict, in stark contrast to World War I. The worst case would be a Greco-Turkish war, but both countries have
made clear both privately and publicly that neither is interested in intervening in the Balkans. In fact, violence in Kosovo is
the least likely spark for such a conflict. Should Ankara and Athens exchange blows, it is far more likely to occur over the
Aegean islands, Cyprus, or territorial sea claims. Moreover, as noted earlier, NATO's intervention on behalf of the KLA would only
energize advocates of a greater Albania.  The most important point, however, is that any resulting instability is a European, not an
American, problem. The U.S. has a vital interest in preventing a hostile hegemonic power from dominating Europe. Washington does
not have even a minor interest in preventing Europe from having to deal with the detritus in the Balkans left over from the Cold War.
Instability on the periphery of Europe has other consequences--economic and cultural, for instance--but they are minimal. To call
this a vital interest, as does the administration, suggests that it is incapable of setting priorities.  

International community solves the impact

Wolff ’6 (Stefan, PhD & German political scientist, Ethnic Conflict, p. 138]

This link of prevention, management, and settlement is part of the reason why the international
community has, since the end of the Kosovo conflict, been more successful in the western Balkans, and it contains
important lessons. Success crucially depended on a number of factors—a developed institutional
framework and a set of policies that enable decisions to be made quickly , to provide adequate funds
and personnel, and to cooperate and coordinate activities are some of them. Another factor is the effectiveness of
the policies employed: in the western Balkans, for example, EU-imposed conditionality is so much more
effective vis-à-vis countries where the promise of closer association with , and potential accession to,
the EU is credible and where both political elites and the general public are ready to make compromises in order to attain what
many believe to be a panacea for all their problems. This leverage often does not exist elsewhere because there is no clear long-
term commitment to a particular country or region, but rather an ad hoc and often belated response to an emerging crisis, a lack
familiarity with, and sensitivity towards, the situation on the ground and no credible intelligence sources. Comparing the situation in
Macedonia with that in Bosnia and Herzegovina or Rwanda in the early 1990s illustrates this point.
---Balkans- Extensions

Won’t go nuclear and status quo solves

Tanter 99 (Raymond, Professor of Political Science – University of Michigan and Research Associate – Middle East Center and
John Psarouthakis, President and Trustee – Paideia Foundation and Adjunct Business Professor – University of Michigan, Balancing in
the Balkans, p. 50)

Despite the lack of American ground forces, however, U.S. diplomacy began to have an effect. That the war in Bosnia changed
course once the Americans arrived on the diplomatic scene is largely a reflection of the inadequacy of the security structure for
Europe in the post-Cold War era. Europe’s military power had been dedicated to opposing the Soviet Union, and the Americans
controlled most of the nuclear potential. Because it
was unlikely that conflict in the Balkans would
escalate into nuclear war, American diplomatic involvement was not as great as it would have been during the Cold
War. Even without the fear of nuclear conflict, however, Washington decided to involve itself in
stabilizing the Balkans.

And if Balkan conflict does break out plan can’t solve it – many alt causes.

Nikolic 5 (Dragan, New Balkan Conflict Brewing, BCR Issue 222, September 6, 2005, http://iwpr.net/report-news/new-balkan-
conflict-brewing)

New Balkan Conflict Brewing The fourth summit of the heads of states and governments of south-eastern Europe last
week, was overshadowed by terrorist activities in the southern Balkans. The glamorous gathering and the participants'
optimism starkly contrasted with the dark reality of ever more frequent incidents in this part of the region. The summit concluded
that terrorism must be eliminated and economic cooperation encouraged. No-one, however, knows whether these declarations will
ever be implemented. As over 200 cooks and waiters attended to the delegates and some 650 journalists reported on their every
utterance, a new Balkan conflict between the Macedonian army and Albanian militants was
brewing just ten kilometers away in the village of Tanusevci, on the Kosovo border Amid rising tension, dozens of tractors
prepared to evacuate around 140 local Albanians to safety in Kosovo. The well-intentioned declarations at the Skopje summit, it
seems, have done little to diffuse what appears to be a pre-war atmosphere. The conference reminded Macedonian citizens
of the good old times of pompous communist plenary sessions, when Macedonia was part of Yugoslavia. Those gatherings also
ended with declarations which were never implemented. During last week's summit, the Macedonian army was on high alert in the
vicinity of Tanusevci, dispatching 600 police commandos to the region after the conference closed. At the same time, across the
border in the Presevo Valley, in southern Serbia, Albanian high school students enlisted en masse in the Liberation army of Presevo,
Medvedja and Bujanovac, OVK PMB, to fight for the three municipalities with Kosovo. A week before the summit, suspected
Albanian extremists in Podujevo, Kosovo, detonated a mine under a bus carrying Serbian women and children, killing ten and
seriously injuring 40 of them. The troubles in Kosovo and southern Serbia appear to be spreading to
Macedonia, suggesting Albanian nationalists are in the process of attempting to realise their long-held dream of a Greater Albania. In
a related development, a Serbian-Macedonian agreement on border changes, promoted at the Skopje summit as evidence of
growing regional cooperation, has been rejected by the Democratic Alliance of Kosovo, the province's principal party. Albanians here
see the agreed frontier as confirming Kosovo as part of Serbia. Skopje and Belgrade, clearly anticipating trouble, are to request the
help of NATO to implement their agreement along that part of the 260 km frontier bordering Kosovo. But what's increasingly clear is
that Western patience with Albanian intransigence and militarism is beginning to wear. European Commissioner for Security and
Political Issues Xavier Solana said international assistance to Pristina has been brought into question by the Albanian terrorist actions
in Kosovo and southern Serbia. Meanwhile, European Commissioner for Foreign Policy Chris Patten warned that the broad
autonomy promised Kosovo may be jeopardised by the "barbarian " and "cowardly" attack on the Podujevo bus. Albanian Prime
Minister Ilir Meta has also strongly condemned the recent spate of terrorism, and called for Albanians in southern Serbia to make
peace with the authorities in Belgrade. But despite the condemnations, delegates at the Skopje conference failed to get to grips with
the real problem - the unsolved issue of Kosovo's future status . Although the summit message was clear - the
Albanian terrorist campaign will not result in a change of borders - no-one knows how to bring an end to the
violence. Serbia's hands are tied. It does not want to resort to the old Milosevic tactic of intervening militarily. The
Macedonian armed forces, meanwhile, lack the strength to deal with the insurgents. All eyes are fixed on the NATO, as the only
guarantor of peace in the region. However, not
even the Alliance seems prepared to tackle the problem .
Why? Because NATO members realise that force will be required to resolve it - a prospect none of
them likes. There are those who envisage a type of conflict similar to the Vietnam war and want
to avoid it at any price. Yugoslav President Vojislav Kostunica, grim and agitated throughout the summit, was clearly aware
of NATO's reluctance to get involved, but urged it and the EU to introduce measures to stem the violence in southern Serbia.
---Balkans- AT: History

Historical examples don’t apply

Eland 99 (Ivan, Director of Defense Policy Studies – Cato Institute, 5-3, http://www.cato.org/cgi-
bin/scripts/printtech.cgi/dailys/05-03-99.html)

In reality, the ostensible humanitarian justification for the war is secondary at best. It's the underlying perception that European
security is threatened that's really driving this military intervention. The United States rarely intervenes militarily when there is
no perception that its interests are at stake. So the military operation advertised as a NATO mission to relieve human suffering is
actually a ham-handed U.S. attempt to defend perceived American security interests. Those perceived interests flow
from the Clinton administration's domino theory of instability and concerns about preserving NATO's credibility.
Instead of a fear of communism spreading from country to country, the administration's refurbished domino theory sees
"instability" -- unless checked -- spreading and engulfing large parts of Europe. Instability
has always existed in
the volatile and remote Balkan nations, but it hasn't spread outside the region since 1914.
The administration constantly alludes to the specter of World War I. But in the events leading
up to that war, two powerful and hostile alliances exploited instability in the region --
[was] a situation much different from the one that exists today. At present, instability in
the Balkans has no relationship to American vital interests .
Biodiversity Answers
---Biodiversity Defense - Frontline
No impact to biodiversity:

a. It’s inevitable—
1. Warming—increasing temps, volatile weather

2.Land use and development

3. Policy barriers—planning, consensus, public opinion, and uncertainties.

European Commission, 08 (July 17, 2008, “Planning for the inevitable: the impact of climate change on
biodiversity,” http://ec.europa.eu/environment/integration/research/newsalert/pdf/116na6.pdf, Hensel)

Many scientific reports suggest that unavoidable changes in climate will happen over the
next 40-50 years as a result of past emissions. Areas seen as most vulnerable to climate change include the
Mediterranean and southern Europe, mountain and sub-arctic areas, and densely occupied floodplains and coastal zones. Annual
temperatures could increase by 2.0-6.3 degrees centigrade by 2100. Rainfall could also
increase by 1-2 per cent per decade for northern Europe and decrease by 1 per cent in southern Europe. Events
affecting habitats and biodiversity will include heat waves, droughts, storms and rising
sea levels. The impact may cause species to move towards the north and an increase in extinction
rates. Mitigation remains the key focus of climate change policy, with less attention given to understanding how to adapt to
inevitable rising temperatures. The pressures of climate change present a major challenge , not just for
biodiversity policy, but also for land use policy, which affects biodiversity. The EU’s 2006 Biodiversity Communication and its
Action Plan set an agenda for action to halt the loss of biodiversity by 2010, as agreed in the Gothenburg summit, 2001. However,
biodiversity continues to decline under pressure from land use change and
d evelopment . For example, as water supplies for urban populations shrink, building new infrastructures may place stress on
existing ground and surface water systems and the flora and fauna that rely on it. The research 1 reviewed land use plans and policy
in three countries: France, the Netherlands and the UK. It looked at their use of natural resources, management of water and coastal
zones, plans for designated sites and case studies on urban, rural, inland and coastal sites. The policies were examined for their
ability to account for biodiversity adaptation to climate change and to identify ways of integrating ‘spatial planning’ and biodiversity
policy. Spatial planning has a broader sense than ‘land use’, in that it accounts for all activities and interests that concern a particular
area. The authors found that although dynamic biodiversity is becoming more fully realised in spatial planning policy, existing EU
directives such as the Birds Directive (CEC 1979), the Habitats Directive (CEC 1992), and the Natura 2000 network set up to create a
network of protected sites, by themselves cannot fully protect landscape features necessary to support biodiversity under a period
of prolonged climate change. They recommend ‘climate-proofing’ plans through the use of Environmental Impact Assessment and
Strategic Environmental Assessment. Land use plans should be integrated with the adoption of common objectives, time horizons
and boundaries. The study also highlighted the need for more flexible responses to climate change, with stakeholders safeguarding
habitats in between protected areas. This would result in more robust conservation planning across whole landscapes, reducing
fragmentation of sites and creating corridors and networks for wildlife. International cooperation was also found to be critical, as
wildlife moves across national boundaries. Integration with agriculture, transport and water sectors would also lead to a better
capacity to adapt to climate change. Barriers
to putting a fully effective policy in place include: planning
time-scales that are too short, a lack of consensus on intervention measures,
uncertainty on the actual impact of climate change impacts, conflicts of interest and
public opinion which is sensitive to change, especially in treasured landscapes.

b. Traits determine ecological stability


Mokani et al, 08 (Karel, School of Botany and Zoology, Australia National University, *AND Julian Ash, School of Botany
and Zoology, Australia National University, *AND Stephen Roxburgh, Bushfire Cooperative Research Centre, School of Biological,
Earth, and Environmental Sciences, University of New South Wales and ENSIS, May 7, 2008, “Functional identity is more important
than diversity in influencing ecosystem processes in a temperate native grassland,” Journal of Ecology Volume 96, Issue 5, Wiley,
Hensel)

Our results indicate that the mass ratio hypothesis (Grime 1998) provides a more appropriate
framework for explaining how the biota influences key ecosystem processes in
comparison to the diversity hypothesis, for the native grassland studied. Mean trait values best explained
variation in five of the eight ecosystem processes (Table 2a,b; Fig. 1), supporting Grime’s (1998) hypothesis that it is the traits
of the most abundant species which largely determine ecosystem processes. Our results
correspond with previous research demonstrating the power of mean trait values to explain variation in key
ecosystem processes (Garnier et al. 2004; Vile et al. 2006). The trait-based functional diversity indices
(especially FD, FDQ and FDvar) also performed well, often approaching and occasionally exceeding mean trait
values in their explanatory power (Table 2a,b). Most notable of the functional diversity indices was FDvar (Mason et
al. 2003), which explained more variation than any other diversity measure for seven of the eight ecosystem processes, and had the
highest r2 values of all the diversity/trait indices for three of the ecosystem processes (green shoot biomass, root biomass, soil
moisture) (Table 2a,b). Functional diversity indices which weigh the traits of species by the
abundances of those species (namely FRO, FDQ and FDvar) are essentially fusing elements of both
the diversity hypothesis and the mass ratio hypothesis. Implicit in these indices is the assumption that the
diversity of traits is important, but is relative to the abundances of the species possessing those traits (Ricotta 2005). The fact
that these diversity measures (FRO, FDQ, FDvar) tend to perform intermediately between the
pure diversity indices (e.g. species richness) and mean trait values supports our suggestion that
it is the traits of the abundant species that are most important in influencing ecosystem
processes.

c. alt causes
Ricciardi 10 (Michael, former prof. of ecology and natural science @ Cape Cod, Mass. 
June 16, http://planetsave.com/blog/blog/2010/06/16/worlds-biodiversity-loss-not-
slowing-major-analysis-finds-video/) LL
In this International Year of Biodiversity, a multinational group of zoologists, biologists
and ecologists has assessed 24 biodiversity indicators and found that global declines in
these key indicators are either fluctuating or continuing. The Convention on Biological
Diversity (CBD), adopted in 2002, was a commitment to achieve significant reductions
in biological diversity loss by 2010. With the recognition that biodiversity plays a
significant role in human well-being and quality of life measurements, the convention’s
goal was subsequently included in the United Nations Millennium Development Goals.
Publishing the results of their integrative study in the May 28, 2010 edition of Science
Magazine (Global Biodiversity: Indicators of Recent Declines, Butchart et al), the group’s
aggregate analysis “suggests that biodiversity has continued to decline over the past
four decades, with most state (of biodiversity) indicators showing negative trends.” A
new, aggregate study of key biological diversity indicators shows the rate of decline in
genes, species loss, population loss, and ecosystem services is continuing. Previous
analyses of gene, species and population losses have been published, but the group’s
study is the first empirical analysis to integrate a broad spectrum of biodiversity
indicators. In an effort to determine whether the CBD goal is being met, the team
calculated “aggregate indices” reflecting the state of biodiversity (in a given
region/area), the biodiversity “pressure” (impacting factors), environmental/ecological
policy and management, and the state of “ecosystem services” (that people derive
benefit from). The starting year for calculating (positive/negative) biodiversity trends
was 1970. A schematic image illustrating the relationship between biodiversity,
ecosystem services, human well-being, and poverty. The illustration shows where
conservation action, strategies and plans can influence the drivers of the current
biodiversity crisis at local, regional, to global scales. More specifically, the analysis found
there to be continued declines in population trends of vertebrates, habitat-specialized
birds, shore birds (worldwide), the extent of forested land, mangroves, seagrass beds,
and the condition of coral reefs. The study also found that “aggregate species’
extinction risk (i.e., biodiversity loss at the species level) has accelerated.” This finding
included data from the Red List Index–compiled annually by the International Union for
the Conservation of Nature (IUCN)–showing the rate of change (in species loss) to be in
a negative trend. Most of the indicators reflecting “pressures on biodiversity” show
increasing trends (since 1970), and include: increases in “aggregate human
consumption of ecological assets”, increases in the deposition of reactive nitrogen
(typically through the over-use of synthetic fertilizers), the number of alien (invading)
species (in Europe), the proportion of fish stocks that are over-harvested, and the
impact of climate change on (European) bird populations.
---Biodiversity Offense
And biodiversity loss is good:
a. Key to evolutionary change – causes more diversity over time.
Boulter 2 (Michael, professor of paleobiology at the University of East London, Extinction:
Evolution and the End of Man, p. 170)

The same trend of long-drawn-out survival of the final relicts has been further considered by Bob May’s group at Oxford, particularly
Sean Nee. The Oxford group are vociferous wailers of gloom and doom: ‘Extinction episodes, such as the anthropogenic one
currently under way, result in a pruned tree of life.’ But they go on to argue that the vast majority of groups
survive this pruning, so that evolution goes on, albeit along a different path if the environment is changed. Indeed,
the fossil record has taught us to expect a vigorous evolutionary response when the
ecosystem changes significantly. This kind of research is more evidence to support the idea that evolution
thrives on culling. The planet did really well from the Big Five mass-extinction events. The
victims’ demise enabled new environments to develop and more diversification took
place in other groups of animals and plants. Nature was the richer for it. In just the
same way the planet can take advantage from the abuse we are giving it . The harder the
abuse, the greater the change to the environment. But it also follows that it brings forward the extinctions of a whole selection of
vulnerable organisms.

b. Solves extinction
Boulter 2 (Michael, professor of paleobiology at the University of East London, Extinction:
Evolution and the End of Man, p. 67)

If biological evolution really is a self-organised Earth-life system there are some very important consequences. One is that life on
this planet continues despite internal and external setbacks, because it is the system that
recovers at the expense of some of its former parts. For example, the end of the dinosaurs
enabled mammals to diversify. Otherwise if the exponential rise were to reach
infinity, there would not be space or food to sustain life. It would come to a stop.
Extinctions are necessary to retain life on this planet.

c. Stabilizes the ecosystem


Heath 99 (Jim, Orchids Australia, December, http://www.orchidsaustralia.com/whysave.htm)

Some people say we can’t afford to lose any species , no matter what species they are. Everything needs
everything else, they say, to make nature balance. If that were right, it might explain why the six orchid species should be saved.
Alas, no. We could pour weedkiller on all the orchids in Australia and do no ecological
damage to the rest of the continent’s biology. But wouldn’t the natural ecological
systems then become less stable, if we start plucking out species - even those orchids? Not
necessarily. Natural biological systems are hardly ever stable and balanced anyway.
Everything goes along steadily for a time, then boom - the system falls apart and
simplifies for no visible reason. Diverse systems are usually more unstable than the
less diverse ones. Biologists agree that in some places less diversity is more stable (in the Arctic, for
example). Also, monocultures - farms - can be very stable. Not to mention the timeless grass of a salt marsh. In
other words, there’s no biological law that says we have to save the orchids because they add diversity, and that
added diversity makes the biological world more stable.
Bioweapons Answers
Frontline

The worst case scenario happened – no extinction

Dove 12 [Alan Dove, PhD in Microbiology, science journalist and former Adjunct Professor at New York University, “Who’s Afraid
of the Big, Bad Bioterrorist?” Jan 24 2012, http://alandove.com/content/2012/01/whos-afraid-of-the-big-bad-bioterrorist/]

The second problem is much more serious. Eliminating the toxins, we’re left with a list of infectious
bacteria and viruses.
With a single exception, these organisms are probably near-useless as weapons, and history proves it.¶ There have
been at least three well-documented military-style deployments of infectious agents from the list, plus one deployment of an agent
that’s not on the list. I’m focusing entirely on the modern era, by the way. There are historical reports of armies catapulting plague-
ridden corpses over city walls and conquistadors trying to inoculate blankets with Variola (smallpox), but it’s not clear those
“attacks” were effective. Those diseases tended to spread like, well, plagues, so there’s no telling whether the targets really caught
the diseases from the bodies and blankets, or simply picked them up through casual contact with their enemies.¶ Of the four
modern biowarfare incidents, two have been fatal. The first was the 1979 Sverdlovsk anthrax incident, which
killed an estimated 100 people. In that case, a Soviet-built biological weapons lab accidentally released a large plume of
weaponized Bacillus anthracis (anthrax) over a major city. Soviet authorities tried to blame the resulting fatalities on “bad meat,” but
in the 1990s Western investigators were finally able to piece together the real story. The second fatal incident also
involved anthrax from a government-run lab : the 2001 “Amerithrax” attacks. That time, a rogue employee (or
perhaps employees) of the government’s main bioweapons lab sent weaponized, powdered anthrax through the US postal service.
Five people died.¶ That gives us a grand total of around 105 deaths, entirely from agents that were
grown and weaponized in officially-sanctioned and funded bioweapons research labs. Remember
that.¶ Terrorist groups have also deployed biological weapons twice, and these cases are very instructive. The
first was the 1984 Rajneeshee bioterror attack, in which members of a cult in Oregon inoculated restaurant
salad bars with Salmonella bacteria (an agent that’s not on the “select” list). 751 people got sick, but nobody
died. Public health authorities handled it as a conventional foodborne Salmonella outbreak ,
identified the sources and contained them. Nobody even would have known it was a deliberate attack if a member of the cult hadn’t
come forward afterward with a confession. Lesson: our
existing public health infrastructure was entirely
adequate to respond to a major bioterrorist attack.¶ The second genuine bioterrorist attack took
place in 1993. Members of the Aum Shinrikyo cult successfully isolated and grew a large stock of anthrax
bacteria, then sprayed it as an aerosol from the roof of a building in downtown Tokyo. The cult was

well-financed, and had many highly educated members, so this release over the world’s largest
city really represented a worst-case scenario .¶ Nobody got sick or died. From the cult’s perspective, it
was a complete and utter failure. Again, the only reason we even found out about it was a post-hoc confession. Aum members later
demonstrated their lab skills by producing Sarin nerve gas, with far deadlier results. Lesson: one of the top “select agents” is
extremely hard to grow and deploy even for relatively skilled non-state groups. It’s a really crappy bioterrorist weapon.¶ Taken
together, these events point to an uncomfortable but inevitable conclusion: our biodefense industry is a far greater threat to us than
any actual bioterrorists.

No impact to bioweapons

Easterbrook ‘3 (Gregg Easterbrook, senior fellow at The New Republic, July 2003, Wired, “We’re All Gonna Die!”
http://www.wired.com/wired/archive/11.07/doomsday.html?pg=2&topic=&topic_set=
3. Germ warfare!Like chemical agents, biological weapons have never lived up to their billing in popular
culture. Consider the 1995 medical thriller Outbreak, in which a highly contagious virus takes out entire towns. The reality is quite
different. Weaponized smallpox escaped from a Soviet laboratory in Aralsk, Kazakhstan, in 1971; three people
died, no epidemic followed. In 1979, weapons-grade anthrax got out of a Soviet facility in Sverdlovsk (now
called Ekaterinburg); 68 died, no epidemic. The loss of life was tragic, but no greater than could have been
caused by a single conventional bomb. In 1989, workers at a US government facility near Washington
were accidentally exposed to Ebola virus. They walked around the community and hung out with family
and friends for several days before the mistake was discovered. No one died. The fact is, evolution has spent
millions of years conditioning mammals to resist germs . Consider the Black Plague. It was the
worst known pathogen in history, loose in a Middle Ages society of poor public health, awful sanitation, and no
antibiotics. Yet it didn’t kill off humanity. Most people who were caught in the epidemic survived. Any
superbug introduced into today’s Western world would encounter top-notch public health,
excellent sanitation, and an array of medicines specifically engineered to kill bioagents . Perhaps
one day some aspiring Dr. Evil will invent a bug that bypasses the immune system. Because it is possible some novel superdisease
could be invented, or that existing pathogens like smallpox could be genetically altered to make them more virulent (two-thirds of
those who contract natural smallpox survive), biological agents are a legitimate concern. They may turn increasingly troublesome as
time passes and knowledge of biotechnology becomes harder to control, allowing individuals or small groups to cook up nasty germs
as readily as they can buy guns today. But no
superplague has ever come close to wiping out humanity
before, and it seems unlikely to happen in the future.

Can’t disperse bioweapons

Smithson 5 (Amy E., PhD, project director for biological weapons at the Henry L. Stimson Center. “Likelihood of Terrorists
Acquiring and Using Chemical or Biological Weapons”. http://www.stimson.org/cbw/?SN=CB2001121259]

Terrorists cannot count on just filling the delivery system with agent, pointing the device, and
flipping the switch to activate it. Facets that must be deciphered include the concentration of
agent in the delivery system, the ways in which the delivery system degrades the potency of the
agent , and the right dosage to incapacitate or kill human or animal targets. For open-air delivery, the
meteorological conditions must be taken into account. Biological agents have extreme
sensitivity to sunlight, humidity, pollutants in the atmosphere, temperature, and even
exposure to oxygen, all of which can kill the microbes. Biological agents can be dispersed in either dry or wet
forms. Using a dry agent can boost effectiveness because drying and milling the agent can make the particles very fine, a key factor
since particles must range between 1 to 10 ten microns, ideally to 1 to 5, to be breathed into the lungs. Drying
an agent,
however, is done through a complex and challenging process that requires a sophistication of
equipment and know-how that terrorist organizations are unlikely to possess. The alternative is
to develop a wet slurry, which is much easier to produce but a great deal harder to disperse effectively. Wet
slurries can clog sprayers and undergo mechanical stresses that can kill 95 percent or more of the microorganisms.
Blackouts Answers
Frontline
New Tech solves- multiple responses available to grid collapse.
Business Wire ‘1 (Dec. 17. “Innovative technologies can improve national security; optimal
technologies software able to make nation’s power grid more secure” written by business
editors/high-tech and energy writers.
http://findarticles.com/p/articles/mi_m0EIN/is_2001_Dec_17/ai_80858553)

Optimal Technologies announced this week the ability to improve national security with
breakthrough electric power system technologies. If one part of a power grid were to
fail due to intentional disruption -- or accident, operating error, or natural disaster --
Optimal's tools could allow multiple responses to avoid grid collapse , including automated
recontrolling of key connections, rerouting of power flows, and precise management of loads.
Optimal's new Aempfast(TM) (pronounced aim-fast ) software, now being tested, has the
unique ability to "see" the power grid as a whole and in great detail. Aempfast can
swiftly find blockages in power flow, identify and direct system adjustments eliminating
the congestion points, and reroute power -- in seconds, as opposed to hours -- thereby
avoiding blackouts and brownouts. "This software is fundamental for electric power
contingency planning and crisis management," said Roland Schoettle, founder and CEO of
Optimal Technologies.

Blackouts inevitable
Apt and Lave ‘4 (Jay, Lester. August 10. “Blackouts are Inevitable” Washington Post.
http://wpweb2.tepper.cmu.edu/ceic/pdfs_other/Blackouts_Washington_Post_8_10_04.pdf)

As we approach the first anniversary of the Blackout of '03, we're reminded of the many times
that officials, from New York Gov. Nelson Rockefeller in 1977 to Gov. George Pataki now --along
with a host of senators and representatives -- have assured us that they will take steps to
prevent future blackouts. Yet roughly every four months, the United States experiences a
blackout large enough to darken a half-million home s. Now the pressure is on Congress to enact
an energy bill that will protect us from the lights going out. There's just one problem: It can't be
done.
Brazil-US Relations Answers
Frontline

US-Brazil relations resilient

Hakim 4 (Peter, President – Inter-American Dialogue, “The Reluctant Partner”, Foreign Affairs, January / February, Lexis)

Most Brazilians, in contrast, supported their government on all of these fronts. On issue after issue, the key questions are the
same: How much dissent and independence will the United States tolerate from Brazil? And how much is Brazil prepared to
accommodate U.S. views and interests? A LIGHT TOUCH? o far, trade has been the only issue to provoke open and potentially
damaging friction between the
two countries. They have been able to cooperate, at least minimally, on
thorny issues such as Venezuela and Colombia, and they have managed to swallow harsh
rhetoric and avoid public quarrels on others, such as Cuba and the Iraq war. And although
they have strikingly different backgrounds, personal styles, and political perspectives, the two
presidents have apparently developed sincere respect for each other.

Multiple alt causes

Hakim 4 (Peter, President – Inter-American Dialogue, “The Reluctant Partner”, Foreign Affairs, January / February, Lexis)

Another prickly issue for U.S.-Brazil relations is Washington's ongoing war against terrorism
and rogue states. Although Brazil was supportive of the United States after September 11, 2001 -- it invoked the collective
security agreement of the Organization of American States (OAS) to declare that the terrorist attacks were acts of aggression
against all OAS members -- it has since become one of the region's harshest critics of the Iraq war . Last
September, Lula made plain to the UN General Assembly that Brazil, like most Latin American states, opposes Washington's
unrelenting unilateralism, its doctrine of military preemption, and its frequent disregard for the UN. Washington has largely
ignored Brasilia's objections so far, but playing deaf will become much more difficult this year, as Brazil takes a seat on the UN
Security Council for a two-year term. The key questions are whether the United States will continue to take Brazil's criticism in
stride and keep it from affecting the rest of their relationship and whether Brazil will intensify its criticism or tone it down.
Cuba is another issue on which the two countries do not see eye to eye . Last year, before both the
UN and the OAS, Brazil refused to criticize Cuba's brutal treatment of dissidents, let alone endorse U.S. resolutions condemning
Castro's appalling human rights record. During a visit to the island last September, Lula made clear that he intended to maintain
his long-standing personal friendship with Castro and declined to raise any political issues with him or meet local dissidents. Lula
did seem to take account of American sensibilities, however, by limiting his visit to a single day and asking that anti-American
displays be avoided then. Brasilia's relations with Havana may irritate Washington, but they are not likely to cause major
friction, especially as Cuba's prominence in U.S. foreign policy is waning. Brazil's
involvement in Venezuela, on the
other hand, is likely to be a more important feature of U.S.-Brazil relations. For the past year, Brazil
has chaired the "friends of Venezuela," a six-country group that includes the United States and has urged the
Venezuelan government and insurgents to resolve their political differences peaceably by holding a constitutionally authorized
recall vote on President Chavez's term. At the same time, however, Lula has pursued direct negotiations with the Chavez
administration, to foster bilateral economic ties and closer integration among South American states. Brazil has managed this
precarious double act so far, but should the situation in Venezuela deteriorate, Brasilia might have difficulty
pursuing both tracks at once without alienating Washington .
Canada-US Relations Answers
Frontline

Relations have a laundry list of alt causes and resiliency solves the impact

De Nesnera ‘4 (Andre de Nesnera, VOA News, December 11, ‘4, The Epoch Times, “Some Trade Issues Divide US, Canada,”
http://english.epochtimes.com/news/4-12-11/24897.html

President Bush recently visited Canada, his first trip abroad since his re-election. The two neighboring countries are strong allies and
have deep ties that bind them. But there are some issues, especially dealing
with trade, that still divide Ottawa and
Washington. Trade is the most important component of U.S.-Canada relations. Each country is the
other’s biggest trading customer. Eighty-four percent of Canada’s exports go the United States and Canada buys more than 70
percent of its imports from its neighbor. So it was no surprise that when President Bush visited Canada, trade issues - and especially
contentious trade issues - were high on the agenda in discussions with Canadian Prime Minister Paul Martin. Charles Doran is
Director of Canadian Studies at the Johns Hopkins School of Advanced International Studies in
Washington, DC. He says one major disagreement between the two countries deals with Washington’s tariffs
on the import of Canadian softwood lumber, such as pine. “There is a huge amount of trade in lumber between
Canada and the United States. Canadians sell a large amount, billions of dollars, and the argument has been on the part of a small
group of producers in the United States that Canada has subsidized this. Now the NAFTA (North American Free Trade Agreement)
and the World Trade Organization, in dispute resolution panels, have denied that there is unfair subsidy. But in fact, every President
for some time has been unable to unravel the legal challenges and so on, to get rid of that issue,” he says. Following the Bush-Martin
meeting, the softwood lumber
issue remains unresolved. Professor Doran says another problem stems from
the US action to ban beef imports from Canada because of mad cow disease. “There was one cow found
in Alberta with this disease, but the consequence of that has been enormous in the sense that trade for beef, for the United States
and Canada has been affected and third markets like Japan and Europe. They are trying to get around this problem. They are trying
to establish common standards, but it’s hard to believe, it’s almost hard to imagine how one cow could cause that much catastrophe
to this industry in North America,” he says. Canadian statistics indicate that the 18-month ban has cost the Canadian beef industry
more than $4 billion in lost revenues. That issue, too, still remains to be solved following the Bush-Martin summit.
Tied to those two trade issues, is the question of security along the Canadian-American border - at
nearly 9,000 kilometers the world’s longest undefended frontier. Both countries have stepped up cooperation in the
security field, especially after the terrorist attacks of September 11, 2001. Kim Nossel, Director of Political Studies at
Queen’s University in Kingston, Ontario, says Americans and Canadians are approaching the border
security issue from different angles. “From the American perspective, there is the concern about
the porousness of that long, undefended border and the ease with which one could in fact get across the border. From a
Canadian perspective, the major concern is an absolute fear that there will be a terrorist incident in the United States that will
openly and manifestly have come from Canada, that will lead to, essentially, a closing of the border. And of course that border and
the openness of that border is absolutely crucial for Canadian wealth.” Experts say Ottawa and Washington have to
find a delicate balance between the free flow of commerce and legitimate security concerns. Gill
Troy is a U.S.-Canada expert at McGill University in Montreal. He says despite various disagreements
between the two countries, one overriding issue must be kept in mind. “Even if there is an agreement to disagree,
even if the United States says: ‘look, we can’t do this because of internal constituency pressures or external trade pressures,’ the
awareness that nevertheless, while we might part on some issues, we are still fundamentally
friends, we are still fundamentally linked in so many ways - economically, ideologically,
intellectually, culturally, socially - is important,” he says. Experts agree that President Bush’s trip to Canada
was an attempt to improve relations between the two countries - relations that were strained in
recent years, during the tenure of Canadian Prime Minister Jean Chretien. Analysts say based on the recent Bush-
Martin meeting, things are looking up.
Caribbean Drug Answers
Frontline
Alternate modes of Carribean drug delivery –most of which are already
mapped.
DEA 11 STATEMENT FOR THE RECORD OF RODNEY G. BENSON ASSISTANT ADMINISTRATOR
CHIEF OF INTELLIGENCE DRUG ENFORCEMENT ADMINISTRATION BEFORE THE

SENATE CAUCUS ON INTERNATIONAL DRUG CONTROL ENTITLED “U.S. – ANDEAN SECURITY COOPERATION”
PRESENTED OCTOBER 19, 2011- pg. 3

http://www.justice.gov/dea/speeches/111019_testimony.pdf

Cocaine is exported from the Andean Region using a variety of routes and
conveyances. Although the U.S. continues to be the world’s largest consumer of
cocaine, DTOs are increasingly exploiting lucrative drug markets elsewhere in South
America as well as in Europe, Asia, and Australia . Cocaine originating in Colombia typically transits
the Central America or the Caribbean corridors with onward shipment to Mexico and the U.S., while much of the
cocaine produced in Bolivia and Peru is shipped to Brazil or through the Southern Cone en route to Europe and West
DTOs use all manner of conveyances to deliver their product to market,
Africa.
including human couriers, go-fast vessels, fishing vessels, containerized cargo, non-
commercial aircraft, self-propelled semi submersibles (SPSS). The basic drug trafficking routes
and conveyances used in the Andean region are fairly well-known ; however, DTOs are
constantly adapting to law enforcement efforts, and frequently shift their operations
to minimize risk and maximize profit. The recent seizure of two self-propelled fully submersible vessels
in Colombia and a third in Ecuador, for example, underscores the incredible lengths drug traffickers will take to move
their product.

Drug Trade good—key to local economies


Steiner and Corcheulo 99ECONOMIC AND INSTITUTIONAL REPERCUSSIONS OF THE DRUG
TRADE

IN COLOMBIA Roberto Steiner and Alejandra Corchuelo CEDE- Universidad de los Andes
December, 1999 http://www.mamacoca.org/feb2002/DrugTrade.PDF

The illegal aspect of drug trafficking and the corresponding absence of reliable data has given
rise to a great deal of speculation with respect to the amount of production, consumption, and
revenue generated by this trade. Many of the figures used by the various organizations and its
officials and by the press have come into the popular domain, creating a distorted image of the

true size of the drug trade, and indeed, of its scope and effects on producer and consumer
countries. The methodology for calculation and sources of information for these estimates are
never specified. 2In terms of the magnitude of the global drug trade, numbers in the
order of us$500 billion have been mentioned, and in terms of the United States, a figure of
us$100 billion has been stated. In an article in The Economist, a US senator argued that his
country imports about 2,500 tons of cocaine annually , a numberthat he uses to justify
American anti-drug policies. As a producer and exporter of drugs, Colombia is not inmune to
these exaggerations. Recently, a Colombian magazine mentioned that “Colombian drug
dealers have expanded their cocaine exports from some 600 tons a year to 1000 tons ."
Due to the size and, especially, due to the growth in these numbers, they point to a failure of
policies in the fight against the drug trade .Faced with a world market of the size mentioned
above, it is not surprising that disproportionate numbers are thrown around when referring to
the profits of the Colombian drug trade. Steiner (1997) refers to articles in which estimates
range from us$18 billion to us$25 billion in annual profits from the sales of illegal drugs (IDs) by
Colombians. In 1995, profits of us$25 billion would had represented the equivalent of 31% of
Colombia’s GNP!The foregoing partially explains the fact that in various studies on
theColombian economy – both by foreign authors as well as by local ones – it is
asserted that drugs constitute the main source of foreign income for the economy,
and therefore, it is drug trafficking funds that have made possible a stable economy
with acceptable growth levels. It has also been said that Colombia would not have a
viable economy, were it not for this activity. These comments neglect the existence of
various studies which show that a gradual and prudent economic policy, led by competent civil
servants, not by politicians, is what has allowed Colombia to maintain its characteristic
macroeconomic stability. Even worse, they undermine the results of moreacademic research,
according to which the ID trade, even though important, does not at all reach the numbers that
are frequently mentioned in the mass media. In some circles there is the perception that
because of its involvement in the illegal drug trade, Colombia receives an enormous
amount of hard currency, which over the past two decades has been largely
responsible for the fact that the Colombian economy has enjoyed one of the best
economic performances in the entire region. It has even been suggested that the
Colombian economy depends so heavily on the resources generated by the drug trade,
that without illegal drugs, the Colombian economy would not be viable. Based on these
precepts, some believe that Colombia is not a reliable partner in the international fight against
the illegal drug trade.
Caspian War Answers
Frontline
(Caspian War) Won’t Escalate

Olcott 98 (Martha, Senior Associate – Carnegie Endowment for International Peace, Foreign Policy, 6-22, Lexis)

So what are outside powers likely to be willing, and able, to do if the Caspian states begin to
destabilize from within? The answer, for a variety of reasons, is, "not much." Barring cataclysmic developments
in the major oil-producing states, global markets will not become dependent on Caspian oil in the foreseeable future. In 2010,
Caspian oil will account for less than 5 percent of global oil consumption. Thus, although
the Georgian
government may ask for NATO security guarantees for the proposed pipeline across its territory, it is
hard to imagine that NATO will give them. For now, NATO is satisfied with providing limited
training assistance and symbolic demonstrations of Western capacity, such as the jump of paratroopers from the 82nd
Airborne, who flew nonstop from Fort Bragg to Kazakhstan as part of a Partnership for Peace joint exercise with CENTBAT, a
new joint Uzbek, Kazakh, and Kyrgyz conflict-management force that NATO is helping organize.

Russia won’t intervene

Olcott 98 (Martha, Senior Associate – Carnegie Endowment for International Peace, Foreign Policy, 6-22, Lexis)

It is also unclear how much longer Russia's rulers will continue to believe that developments in
the Caspian affect their vital interests, if defending those interests requires more than threat and bluster. Although
the various Caspian states are still bound to Russia through various bilateral and multilateral security agreements, Moscow
is unlikely to intervene unless it believes Russia's security will be directly threatened , and
even then it might opt to seal its own borders instead.

US won’t intervene

Rogozin 5 (Dmitri, What The Papers Say (Russia), 4-7, Lexis)

Dmitri Rogozin: The Americans are very active in the Caucasus, but in reality they can only entrench
themselves there if we are complete fools and surrender everything to them. America doesn't really need the Caucasus. The
Americans aren't
interested in the Caucasus as such - but in preventing Russia from growing stronger. They
have other preoccupations. They are bogged down in Iraq, and Iran is looming ahead. That is why no
matter what we do in Georgia, the Americans will never intervene substantially.
Central Asia War Answers
Frontline

Powers will cooperate - contains the impact - empirically proven

Collins and Wohlforth 4 (Kathleen, Professor of Political Science – Notre Dame and William, Professor of Government –
Dartmouth, “Defying ‘Great Game’ Expectations”, Strategic Asia 2003-4: Fragility and Crisis, p. 312-313)

Conclusion The popular great game lens for analyzing Central Asia fails to capture the declared interests of
the great powers as well as the best reading of their objective interests in security and economic growth .
Perhaps more importantly, it fails to explain their actual behavior on the ground, as well the specific reactions
of the Central Asian states themselves. Naturally, there are competitive elements in great power relations.
Each country’s policymaking community has slightly different preferences for tackling the challenges presented in the region,
and the more influence they have the more able they are to shape events in concordance with those preferences. But these
clashing preferences concern the means to serve ends that all the great powers share. To be
sure, policy-makers in each capital would prefer that their own national firms or their own government’s budget be the
beneficiaries of any economic rents that emerge from the exploitation and transshipment of the region’s natural resources. But
the scale of these rents is marginal even for Russia’s oil-fueled budget. And for taxable profits to be created, the projects must
make sense economically—something that is determined more by markets and firms than governments. Does it matter? The
great game is an arresting metaphor that serves to draw people’s attention to an oft-neglected region. The problem is the great-
game lens can distort realities on the ground, and therefore bias analysis and policy. For when great powers are locked in a
competitive fight, the issues at hand matter less than their implication for the relative power of contending states. Power itself
becomes the issue—one that tends to be nonnegotiable. Viewing an essential positive-sum relationship through zero sum
conceptual lenses will result in missed opportunities for cooperation that leaves all players—not least the people who live in the
region—poorer and more insecure. While cautious realism must remain the watchword concerning an impoverished and
potentially unstable region comprised of fragile and authoritarian states, our analysis yields at least conditional and relative
optimism. Given the confluence of their chief strategic interests, the major powers are in a better position
to serve as a stabilizing force than analogies to the Great Game or the Cold War would suggest. It is
important to stress that the region’s response to the profoundly destabilizing shock of coordinated
terror attacks was increased cooperation between local governments and China and Russia, and—multipolar
rhetoric notwithstanding—between both of them and the United States. If this trend is nurtured and if the initial signals about
potential SCO-CSTO-NATO cooperation are pursued, another destabilizing shock might generate more
rather than less cooperation among the major powers. Uzbekistan, Kyrgyzstan, Tajikistan, and Kazakhstan [The
Stans] are clearly on a trajectory that portends longer-term cooperation with each of the great powers. As military and
economic security interests become more entwined, there are sound reasons to conclude that “great game” politics will not
shape Central Asia’s future in the same competitive and destabilizing way as they have controlled its past. To the contrary,
mutual interests in Central Asia may reinforce the broader positive developments in the great powers’ relations that have taken
place since September 11, as well as reinforce regional and domestic stability in Central Asia.

No Central Asian impact

Reuters 11 (“Riches, Fear Ensure Central Asia Stability,” Feb 9th, http://www.themoscowtimes.com/news/article/riches-fear-
ensure-central-asia-stability/430628.html,

ALMATY, Kazakhstan — Central Asia’s authoritarian leaders, having crushed dissent during
decades in power, are likely to use a mixture of oil and gas revenues, repression and cosmetic
reforms to meet any threat of Egyptian-style protests. Few in the strategic region, which covers an area twice
the size of Saudi Arabia, expect their entrenched and aging leaders to succumb to the wave of public
anger sweeping parts of the Arab world. However, in a region riven by ethnic tensions and poverty, where one
country — Kyrgyzstan — has twice overthrown a president, authorities would be remiss in ignoring this warning, political
analysts and opposition politicians say. “The most important lesson? Don’t take your country to the brink,” said Mukhiddin
Kabiri, chairman of the opposition Islamic Revival Party of Tajikistan. Authoritarian presidents rule four of the
five ex-Soviet states in Central Asia, a resource-rich and majority Muslim region, which serves as a key supply conduit
for U.S. military operations in Afghanistan. Kazakhstan holds slightly more than 3 percent of the world’s recoverable oil
reserves, while the world’s fourth-largest reserves of natural gas lie under the desert of Turkmenistan. These resources
generate prosperity. Kazakhstan, the region’s largest economy, boasts per capita gross domestic product of more than
$9,000, four times that of Egypt. In Ashgabat, capital of Turkmenistan, low utility bills help appease a
population in a country where political dissent is not tolerated . “We have free gas, water and lighting,” said
Aibibi, 34, a bookseller in an Ashgabat market. Inflation remains a region-wide threat. Unrest in Kyrgyzstan led to a colossal
19.2 percent surge in prices last year, while in Kazakhstan food prices rose 3 percent in January alone. Turkmen pensioner
Gulsenem, 57, said, “To cook with our free gas, we also need meat — and that’s becoming more expensive.” The riches of
Kazakhstan and Turkmenistan can be spread among relatively small populations, but Uzbekistan, a top-10 world gold miner and
major cotton exporter, is home to 28 million people. State figures portray a robust economy and the International Monetary
Fund forecast 8 percent GDP growth in 2010. However, perhaps nowhere in Central Asia are fear and repression more apparent.
President Islam Karimov, 73, says tough measures are needed to curb the threat of Islamist militancy. Human rights activists
speak of religious persecution and torture. Mukhammad Salikh, 61, stood against Karimov in a 1991 election. He now lives in
exile in Norway. “The danger of a social explosion has not only existed for the last 20 years. It has grown bigger with every
year,” Salikh said in a recent interview with Ferghana News Agency, a private, Russian-language agency covering Central Asian
affairs. Could an “explosion” take place in Uzbekistan, immune to Western criticism of its rights record, where state television
is strictly controlled and a mainly rural population has limited access to the Internet? United Nations data show 36 percent of
Uzbekistan’s population is urbanized, compared with 43 percent in Egypt. In Tunisia, whose president was ousted in a popular
Fear is a strong deterrent to would-be demonstrators.
uprising in January, the figure is 67 percent.
Uzbek government troops shot into crowds that took to the streets of Andijan in 2005. Witnesses say
hundreds were killed. In Tajikistan, protests in Dushanbe in 1992 lit the fuse for a five-year civil war in which tens of thousands
of people were killed. Taxi driver Turakul, 55, would rather swallow his discontent with President Emomali
Rakhmon than risk a repetition. “I’ll never go out on the streets,” he said. “I have food on the table and my four sons
work in Russia. We’ll tolerate this as long as we have our small wages.”

None of the great powers would risk an all-out war - the region serves no strategic importance

Collins and Wohlforth 4 (Kathleen, Professor of Political Science – Notre Dame and William, Professor of Government –
Dartmouth, “Defying ‘Great Game’ Expectations”, Strategic Asia 2003-4: Fragility and Crisis, p. 312-313)

Although Central Asia’s strategic salience has been on the rise, the major powers’ strategic
priorities lie elsewhere. For each of the three major outside players, bilateral relationships with the
others are far more important than any stake they hold in Central Asia. As the chapters on China
and Russia in this volume stress, the most pressing grand strategic objectives of China and Russia
remain economic development and modernization. While both are animated by a quest for great power prestige,
the current consensus among officials in both capitals is that for the foreseeable future prestige
concerns must take a back seat to the drive for modernization whenever the two aims come into
conflict. Moreover, for China, Russia, and the United States, more immediate strategic concerns put other
regions above Central Asia in their hierarchy of interests. The U.S. war on terrorism has already
shifted to the Persian Gulf. Russia’s most pressing security concerns remain in Chechnya and
the Caucasus, while China remains focused on Taiwan. Developments in Central Asia are, of course, related to the
powers’ most pressing immediate strategic concerns to a greater (Russia) or lesser (U.S., China) degree, but in no capital can
zealous officials or policy advocates make the case that any outcome in the region is pivotal to the country’s core security.
---Central Asia- Coop Not Conflict

It’s empirically proven by the last decade

Collins and Wohlforth 4 (Kathleen, Professor of Political Science – Notre Dame and William, Professor of Government –
Dartmouth, “Defying ‘Great Game’ Expectations”, Strategic Asia 2003-4: Fragility and Crisis, p. 312-313)

Often seen as an arena for a new “great game,” Central


Asia was widely expected to undergo a new
round of geopolitical rivalry after the United States entered the region in force following
September 11. 1 Contrary to these expectations, relations among the United States, Russia
and China both regionally and globally have improved dramatically. The fundamental question this chapter
addresses is whether this state of affairs reflects a potential long-term outcome or just a temporary interlude in the great
powers’ regional competition. In other words, to what degree does the “great game” lens capture the real dynamics of the
major powers’ strategic interaction in Central Asia? To address this question, we present three distinct but mutually supporting
analyses: a balance sheet of the major powers’ competitive versus their mutual interests in the region; an assessment of their
behavior in the region after September 11 to determine whether it is consistent with our analysis of their interests; and an
analysis of the Central Asian states’ response to the great powers’ actions, in order to compare their perceptions and behavior
to our reading of the great powers’ role. Our conclusion is that a
new “great game” is not underway in
Central Asia. 2 Although elements of rivalry and competition shadow some relationships, shared strategic
interests dominate the concrete actions of the major powers to a remarkable degree.
Sustaining this state of affairs is clearly in the United States’ interest, but it will require active management.

China and Russia prove this - they are cooperating and wouldn’t risk screwing that up in a war

Weitz 6 (Richard, Senior Fellow and Associate Director of the Center for Future Security Strategies – Hudson Institute, “Averting
a New Great Game in Central Asia”, Washington Quarterly, 29(3), Summer, Lexis)

To many observers’ surprise, Central Asia’s newly independent states have not become objects of
rivalry between Moscow and Beijing but rather a major unifying element in Sino-Russian relations.
The two governments cooperate more closely in Central Asia than in any other world
region. Through the multilateral SCO and their extensive bilateral dialogue, Russian officials acknowledge
China’s legitimate interests in Central Asia, while Beijing has institutional mechanisms to
promote its regional objectives in close cooperation with Moscow. China also does not want
to jeopardize security ties, including purchases of advanced Russian military technologies, by challenging
Russian policies in a region of still limited importance for Beijing. Because Chinese leaders share many
important goals with Russia in Central Asia, they have been able to benefit from Russian initiatives in these areas and redirect
resources to other priorities.

Multilateral organizations check

Weitz 6 (Richard, Senior Fellow and Associate Director of the Center for Future Security Strategies – Hudson Institute, “Averting
a New Great Game in Central Asia”, Washington Quarterly, 29(3), Summer, Lexis)
Concerns about a renewed great game are thus exaggerated. The contest for influence in the
region does not directly challenge the vital national interests of China, Russia, or the United
States, the most important extraregional countries in Central Asian security affairs. Unless restrained, however,
competitive pressures risk impeding opportunities for beneficial cooperation among these countries. The three external great
powers have incentives to compete for local allies, energy resources, and military advantage, but they also share substantial
interests, especially in reducing terrorism and drug trafficking. If properly aligned, the
major multilateral security
organizations active in Central Asia could provide opportunities for cooperative
diplomacy in a region where bilateral ties traditionally have predominated. 
Chemical Industry Answers
Frontline

Multiple factors ensure chemical industry resiliency

Outlook ‘12; Economic world economic review, “Economic Outlook — Economic Outlook No.2-2012”
http://www.mydigitalpublication.com/display_article.php?id=1058343

Rebound in the US Benefiting from the impact of the last two massive public budget support plans for
industry, the American chemical industry was also helped in 2011 by favourable dollar /euro
exchange rate and by the restored health of the Auto sector, one of its leading user industries.While
construction, the chemical industry’s second major customer, has not yet genuinely recovered, its decline has at least
halted, stabilising demand at levels which are manageable in the end for its chemical suppliers. The willingness of
American politicians to support a forced march to US economic growth offers a reassuring
outlook for activity in the sector in 2012 . Additional factors include relatively stable oil prices , the
good health of the inorganic chemical sector – notably fertilisers – and the improved financial
structure of actors in the industry after their restructuring efforts implemented during the 2008-
2009 crisis. On top of this, there are the prospects of the juicy but more distant benefits of innovations
in green chemistry.

Chemical industry resilient

CNI 8 (Chemical News & Intelligence, “This Week in ICIS Chemical Business”, 8-18, Lexis)

Engineering and construction companies are expanding to specialties and photovoltaics Global engineering and construction
companies report that the
projects are changing, but the chemical sector continues to show a
surprising amount of resilience Profitability analysis reveals North American petro chemical
industry's demise is exaggerated Profits in the North American petrochemical industry are expected to
decline sharply following Middle Eastern and Asian capacity additions. But contrary to the
prevailing view, fears of its long-term demise will prove to be exaggerated . Shell's Omega MEG process
kicks off in South Korea The big goal for a process engineer could be the development of a technology that converts all the raw
materials to the desired end product with the minimum theoretical energy consumption, no emissions and the lowest capital cost.

A laundry list of alt causes overwhelm

Swift, 12 -- American Chemistry Council chief economist and managing director


(Thomas, "What Will 2012 Bring?" 1-13-12, www.chemicalprocessing.com/articles/2012/what-will-2012-bring/?show=all, accessed
9-22-12, mss)

A two-speed manufacturing sector, with about one-half of industries soft and others doing well, has emerged. The boom in oil
and gas is creating opportunities both on the demand side (e.g., for pipe and oilfield machinery) and the supply side (e.g.,
for chemicals, fertilizers and direct iron reduction). There's strength in light vehicles and aircraft as well as in industries involved with
business investment (iron and steel, foundries, computers, etc.), and a recovery in construction materials. Elsewhere,
structural issues are sapping dynamism in a number of industries (textiles, paper, printing, etc.). Forward
momentum depends upon demand for consumer goods, which ultimately drives factory output. However,
weakening foreign demand (chemicals are early on in supply chain and exports to Europe have
evaporated) presents challenges for the manufacturing sectors. Balance sheets are strong and lower raw material
costs have benefited manufacturers. Nonetheless, an uncertain business and regulatory
environment is constraining business optimism — and hiring. Light vehicles represent an
important market for chemicals (nearly $3,000 per vehicle), and production has experienced temporary
disruptions from the disaster in Japan. US light vehicle sales should rise to 13.5 million units in 2012 as pent-up demand
fosters growth. Sales will improve even further during 2013, exceeding 14.5 million units then. However, housing, the other
large consumer of chemicals (over $15,000 per start), faces ongoing challenges. New homebuilding
remains depressed as foreclosures continue to flood inventories. Only a minor gain in housing starts should occur in 2012 and
the recovery in this sector will be quite slow. Housing activity should begin to stir in 2013. It remains well below the previous peak of
2.07 million units in 2005 and below the long-term underlying demand of 1.5 million units per year as suggested by demographics
and replacement needs. Unfortunately, today's massive housing inventory will delay a full recovery until later this decade.

Impact empirically denied- Chem industry has been weak in the past

Asia chemical industry solves the impact and is an alt

CEN ‘4 Chemical and Engineering News 01-12 http://pubs.acs.org/cen/coverstory/8202/html/8202asia.html

China's outstanding economic performance is having a major impact on the Asian chemical industry. The
country is mentioned in nearly every financial statement released by chemical companies in the region. Based on numbers for the
first 10 months of 2003, China last year increased its imports of chemicals by more than 40%. Most of this
increase was due to increased imports of organic chemicals, a category that includes polymers. As it experiences high profitability in
its chemical operations, China Petroleum & Chemical Corp. (Sinopec) is speeding up the renovation of an ethylene cracker at its Qilu
Petrochemical unit as well as a coal gasification project at the same site. Sinopec profits surged 69% compared with a year earlier in
the fiscal year's first nine months, which ended Dec. 31. Chemicals
account for approximately 20% of the oil
company's sales and 7% of its operating income. THE BUOYANT Chinese economy is helping to prop up Japan. A large portion
of the electronic materials, components, plastics, machinery, and engineered goods that China needs to fuel its growth is supplied by
Japan, Witte says. Based on numbers for the first 10 months of 2003, it appears that Japanese exports of chemicals--excluding
photographic materials--grew 25% in 2003 to nearly $40 billion., the highest amount ever. Unlike in previous years when Japanese
production of most chemicals was in decline, a major portion of Japanese-made chemicals has experienced
growth in 2003. And as incomes rise, Japanese consumers purchase more Chinese-made goods, thus creating a "virtuous" circle of
growth. Evidence of this circle was not clearly visible in the mixed bag of financial statements released by Japanese chemical
companies in their first half. JSR, for example, increased its net profit by 173% over the past year's first half, but Sumitomo's net
profit declined by 31% on high depreciation charges. However, chemical companies are generally optimistic
about their full-year results--as of March 31. Sumitomo is expecting to boost its net profit by 6% over last year; Mitsui, by
13%; and JSR, by 50%. The Indian economy is being propelled by strong industrial and agricultural
growth. ADB reports that industrial growth reached 6% in the first few months of 2003. Agricultural production received
additional impetus from a "normal" monsoon--neither too long nor too short. Agriculture is more important to India than industry,
so the rise in rural income had a more positive effect on the economy than the increase in industrial output. ADB expects stronger
growth this year as India further benefits from the strengthening of the world economy. With rising incomes, Indian demand for
petrochemicals is strong. Reliance reported a 23% increase in net profit for the half-year ended Sept. 30. This happened
despite an unscheduled shutdown of its Jamnagar p-xylene facilities. Overall, Reliance says it experienced 16% growth in demand for
its petrochemical products, which it was able to supply by having most of its plants producing beyond their nameplate capacities. In
an upbeat forecast in October, Chairman and Managing Director Mukesh D. Ambani said, "We are seeing signs of an upturn in the
petrochemical cycle and are confident of achieving even better performance in the future."
---Chem Industry- Resilient

1NC CNI ev and Fulp ev—they conflate lack of demand with collapse—not true—chem
industry is not going to collapse but lack of demand proves they can’t solve now—no impact
to spikes—prefer our statistical profitability analysis over their fear-mongering industry
propaganda

Multiple reasons industry is resilient:

A). financial reserves

KPMG, 12 ("Commercialization Of Shale Gas Points To Unprecedentedly Bright Outlook For U.S. Chemical Industry: KPMG
Report," 6-27-12, www.kpmg.com/us/en/issuesandinsights/articlespublications/press-releases/pages/commercialization-of-shale-
gas-points-to-unprecedentedly-bright-outlook-for-u-s-chemical-industry-kpmg-report.aspx, accessed 9-22-12, mss)

Efficient, Cash-strong companies eyeing Mergers & Acquisitions for Growth

Chemical companies have spent much of the past four years examining a range of other cost cutting
measures within the organization. This cost cutting and operating efficiency combined with the impact of cheap gas
feedstock has driven profitability and cash generation across the industry. The environment of
economic uncertainty and a strong desire to achieve financial flexibility in the face of continued market turbulence has led many
of the top U.S. chemical companies to build up significant war chests and financial reserves that are
now being cracked open to enhance shareholder value and take advantage of synergies in the market.

B). outsourcing

Shotter, 12 – Financial Times staff (James, "Chemical industry warns of green threat," Financial Times, 6-24-12,
www.ft.com/intl/cms/s/0/2139861c-baec-11e1-81e0-00144feabdc0.html#axzz277hhJ3Ln, accessed 9-21-12, mss)

“We are a commoditised industry where people compete on price, which means that companies locate
where it is cheapest to produce. If it becomes too expensive to produce here , companies will
just go abroad, to the US or the Far East,” said Mr Eastwood. However, the Department of Energy and Climate Change pointed
out that carbon taxes were only one factor in rising energy prices.

C). pass on costs to consumers

Darner, 8 -- Akzo Nobel Board Member responsible for Chemicals (Leif, "Q4/Full Year 2007," FD (Fair Disclosure) Wire, 3-6-8, l/n,
accessed 9-22-12, mss)

Yes, first of all, we've been working very hard in optimizing the Chemicals portfolio as such, focusing on
performance and focusing on the strategic fit of the segments as we are supporting for growth, and that is also there focusing on at
the value type of businesses. In respect of the margin management, we
have embarked on a very, very focused
program to pass on any raw material costs. We have tools that our sales forces are operating in across all
segments so that we, to a very early stage, are able to pass on raw material costs, and we've been very
successful in doing so. And that goes for four of our five market segments in Chemicals.
---Chem Industry- Alt Cause

Gut check—all aff claims are empirically denied—should have taken place before gas boom—
proves other barriers exist

Alt cause- EU and demand-lag offset natgas benefits

CID, 12 ("The Chemical Industry Scenario: Will the slowdown continue or reverse in 2012?," Chemical Industry Digest 1-31-12, l/n,
accessed 9-21-12, mss)

Paul Harmick, chief operating officer for chemicals and performance technologies at global business advisory
company KPMG says uncertainty generated by the eurozone crisis has complicated decision making
throughout the world. This is having an inhibiting effect on economic activity with companies deciding not to
do anything with their cash pile until they see what plays out in the global economy. According to a report by Black Rock
Investment Institute, despite the slowdown, China and India could continue to contribute more than half of the world's economic
growth. Therefore, "China and India are especially important given both their size and growth rates. We expect China will account
for more than 40% of global growth with India and US each accounting for about 15%. Until recently, increasing inflation in emerging
markets had caused policy makers to raise interest rates and/or reserve requirements in an attempt to slow inflation, with the effort
of dampening growth. We expect the process will begin to reverse itself sometime in 2012", the report says. The US Chemical
Industry In its year-end situation report, the ACC forecasts an anaemic increase of 1.2% in 2012 in U.S. chemical
production compared with 1.9% rise in 2011, before a stronger recovery takes hold in 2013. U.S. chemical output is expected to
increase to 2.1 percent in 2013. Most major end-use markets for chemicals have recovered in the U.S., helping to maintain the $720
billion industry's contribution to 26 percent of U.S. gross domestic product (GDP), the ACC says. However, the growth in the US
manufacturing sector, the largest consumer of chemicals has slowed down since the third quarter of 2011. A
boom in oil and
gas is creating both demand-side and supply-side opportunities, and the ACC says this is likely to continue. Yet
growth lags in a number of industries, including textiles, paper and printing, offsetting demand from
the oil and gas industry.

Outweighs- chemical industry extremely sensitive to global econ

Zacks, 12 (Zacks Equity Reserach, "Chemical Industry Stock Outlook - Jan 2012," 1-24-12,
www.zacks.com/stock/news/68371/chemical-industry-stock-outlook-jan-2012, accessed 9-21-12, mss)

Additionally, given
the industry’s sensitivity to the global economy, a diminution of growth outlook will
be a materially negative development. The recent turmoil in Europe and its impact on global growth
remain sources of near-term uncertainty. German chemical company BASF (BASFY - Analyst Report), in the third
quarter of 2011, reported a net income of EUR 1,192 million versus last year’s quarter of EUR 1,245 million (+46.3%). On January 12,
2012, BASF announced it has invested $50 million to acquire an equity ownership position in privately held Sion Power, the global
leader in the development of lithium-sulfur (Li-S) batteries, based in Tucson, Arizona.
---Chem Industry- AT: Green Chem

Multiple factors block chem sustainability development

Elkington 12 (John, executive chairman of Volans and non-executive director at SustainAbility. “Chemical industry isn't doing
enough to embrace sustainability,” 9-12-12, http://www.guardian.co.uk/sustainable-business/sustainability-with-john-
elkington/chemical-industry-embrace-sustainability-environment?newsfeed=true)

One speaker showed a slide headed 'Sustainability is …', spotlighting Shin-Etsu, a Japanese chemical
company that suffered a major explosion. Instead of clamming up, as Japanese corporate leaders are wont to do, the
CEO took a voluntary pay-cut and went out to apologise to the local community. Apologising to people when you have accidentally
blown them up makes sense, most of the time, but in the context of the global challenges we face I struggle to see this as a definitive
(indeed, even a legitimate) case of sustainability in practice. Then another speaker, this time from ExxonMobil Chemical, asserted
that – based on the latest life-cycle assessment data – shopping bags made out of high density polyethylene (HDPE) are the
sustainable option. Paper bags, he insisted, should be dropped because of the energy and water consumption involved. Ah. When
the discussion period came, I asked whether the data had taken into account the great swirling gyres of plastic debris that now scar
large areas of the world ocean? No, he admitted. For such people, as a speaker from BASF assured us, sustainability means we "are
on a journey". Like many others, this German company has talked to a considerable number of stakeholders (350, by their
reckoning) and boiled it all down to a shortlist of issues (just 40 of those). The main conclusion seems to be that we must all create
more shared value while, simultaneously, shrinking our environmental footprints. Good, but by how much? That's a question that
the sector finds it hard to answer, except in areas where there is a legal requirement that the use of particular chemicals be driven
to zero, like hexavalent chromium. And, while most participants intensely dislike the idea of further regulation, there were those –
including Peter Kunze of the European Automobile Manufacturers Association – who argued for much clearer signals on which
chemicals would be banned ultimately, coupled with "smart legislation" to ensure that the process of conversion didn't undermine
industrial or regional competitiveness. It was intriguing to see successive speakers through the lenses of vested interests. A panel of
renewable feedstocks were very unlikely to make much of an
four speakers, for example, agreed that
impression on the industry in the next decade or two . Then a colleague from another chemical company
whispered in my ear that three of the four companies were backwards-integrated into the oil
sector , effectively making them fossil fuel junkies. Hardly surprising, then, that they find it hard to
imagine – or at least publicly admit the possibility of – a radically different future. Behind the scenes
people spoke quietly of lobbying that is underway by parts of the industry: in the US, for example, chemical

companies are fighting tooth-and-nail to ensure suspect chemicals and products like formaldehyde
and styrene continue to be allowed in LEED-certified buildings. On the upside, Nicholas Denis of McKinsey & Co reported results of
their recent market survey showing that green products are now seen much more positively by both consumers and industry
executives, with between 82 and 93% of both categories saying they want to go greener, even though "the road to green
chemicals is harder than we thought initially" and the notion of a "green premium is still a Holy Grail for most
companies." Procter & Gamble promptly disagreed, to a degree, noting that their efforts to promote greener products
like compact detergents had been stymied by the unwillingness of most consumers to change to
seemingly smaller products at the same price-point. So the detergent industry went to government, asked for permission to avoid
anti-trust rules, and moved as a group of companies to strip non-compact products from the shelves. "I would love it if consumers
wanted greener products, mused P&G's Peter Kunze, "because we would then have a business model!"
---Chem Industry- AT: Regulations Kill Chem

Regulations don’t affect gas enough to hurt chemical industry

KPMG, 12
("Commercialization Of Shale Gas Points To Unprecedentedly Bright Outlook For U.S. Chemical Industry: KPMG Report," 6-27-12,
www.kpmg.com/us/en/issuesandinsights/articlespublications/press-releases/pages/commercialization-of-shale-gas-points-to-
unprecedentedly-bright-outlook-for-u-s-chemical-industry-kpmg-report.aspx, accessed 9-22-12, mss)

According to KPMG’s Shannon, “one would be hard pressed to overestimate the impact of the
commercialization of shale gas on the U.S. Chemical Industry.” At its root, the discovery of abundant reserves of shale gas in the
U.S. has driven down the natural gas price and created a massive competitive advantage for U.S. companies. The cost implications
for the U.S. Chemical industry have been impressive. Generally,
a ratio of 5-1 between crude oil and gas prices is
enough to make the U.S. Chemical environment ‘favorable’. At today’s prices, the disparity is
more like 9-1, creating lasting advantages for U.S. producers. Despite the ongoing regulatory
debate , the commercialization of shale gas has already heralded in a new era of growth and prosperity
for the U.S. chemical industry. And while some risks still remain on the horizon, there is little doubt that the U.S. industry
is embarking on a path that will lead to massive competitive advantage and significant transformation within the industry itself.
China Econ Answers
Frontline

China growth stable – comparatively best position.

Roach ’12 (8/29 [Stephen S. Roach was Chairman of Morgan Stanley Asia and the firm's Chief Economist, and currently is a senior
fellow at Yale University’s Jackson Institute of Global Affairs and a senior lecturer … Full profile Aug. 29, 2012 http://www.project-
syndicate.org/commentary/china-is-okay-by-stephen-s--roach]

CommentsThese worries are overblown. Yes, China’s economy has slowed. But the slowdown has been
contained, and will likely remain so for the foreseeable future. The case for a soft landing remains solid. CommentsThe
characteristics of a Chinese hard landing are well known from the Great Recession of 2008-2009. China’s annual GDP growth
decelerated sharply from its 14.8% peak in the second quarter of 2007 to 6.6% in the first quarter of 2009. Hit by a monstrous
external demand shock that sent world trade tumbling by a record 10.5% in 2009, China’s export-led growth quickly went from
boom to bust. The rest of an unbalanced Chinese economy followed – especially the labor market, which shed more than 20 million
jobs in Guangdong Province alone. CommentsThis time, the descent has been far milder. From a peak of 11.9% in the first quarter of
2010, China’s annual GDP growth slowed to 7.6% in the second quarter of 2012 – only about half the outsize 8.2-percentage-point
deceleration experienced during the Great Recession. CommentsBarring a disorderly breakup of the eurozone, which seems unlikely,
the International Monetary Fund’s baseline forecast of 4% annual growth in world trade for 2012 seems reasonable. That would be
subpar relative to the 6.4% growth trend from 1994 to 2011, but nowhere near the collapse recorded during 2008-2009. With
the Chinese economy far less threatened by export-led weakening than it was three and a half years
ago, a hard landing is unlikely. CommentsBut the pessimists’ hype overlooks one of the most important
drivers of China’s modernization: the greatest urbanization story the world has ever seen . In 2011, the urban
share of the Chinese population surpassed 50% for the first time, reaching 51.3%, compared to less than 20% in 1980. Moreover,
according to OECD projections, China’s already burgeoning urban population should expand by more than 300 million by 2030 – an
increment almost equal to the current population of the United States. With rural-to-urban migration averaging 15 to 20 million
people per year, today’s so-called ghost cities quickly become tomorrow’s thriving metropolitan areas. Related Articles Yu Yongding
Jul. 31, 2012 How Should China Respond to the Slowdown? BEIJING – China’s annual GDP growth slowed to 7.6% in the second
quarter of 2012, down from 8.1% in the first quarter and the lowest growth … Andrew Sheng, Geng Xiao Aug. 16, 2012 China’s Next
Transformation HONG KONG – During three decades of favorable global economic conditions, China created an integrated global
production system unprecedented… CommentsShanghai Pudong is the classic example of how an “empty” urban construction
project in the late 1990’s quickly became a fully occupied urban center, with a population today of roughly 5.5 million. A McKinsey
study estimates that by 2025 China will have more than 220 cities with populations in excess of one million, versus 125 in 2010, and
that 23 mega-cities will have a population of at least five million. CommentsChina cannot afford to wait to build its new cities.
Instead, investment and construction must be aligned with the future influx of urban dwellers. The “ghost city” critique misses this
point entirely. CommentsAll of this is part of China’s grand plan. The producer model, which worked brilliantly for 30 years, cannot
take China to the promised land of prosperity. The Chinese leadership has long known this, as Premier Wen Jiabao signaled with his
famous 2007 “Four ‘Uns’” critique – warning of an “unstable, unbalanced, uncoordinated, and ultimately unsustainable” economy.
CommentsTwo external shocks – first from the US, and now from Europe – have transformed the Four Uns into an action plan.
Overly dependent on external demand from crisis-battered developed economies, Chinahas adopted the pro-consumption
12th Five-Year Plan, which lays out a powerful rebalancing strategy that should drive development for
decades. CommentsThe investment and construction requirements of large-scale urbanization are a key pillar of this strategy.
Urban per capita income is more than triple the average in rural areas. As long as urbanization is coupled with job creation – a
strategy underscored by China’s concomitant push into services-led development – labor income and consumer purchasing power
will benefit. CommentsContrary to the China doubters, urbanization is not phony growth. It is an essential ingredient of the “next
China,” for it provides China with both cyclical and structural options. When faced with a shortfall of demand – whether owing to an
external shock or to an internal adjustment, such as the housing-market correction – China can tweak its urbanization-led
investment requirements accordingly. With
a large reservoir of surplus savings and a budget deficit of less
than 2% of GDP, it has the wherewithal to fund such efforts. There is also ample scope for
monetary easing; unlike central banks in the West, the People’s Bank of China has plenty of
ammunition in reserve. CommentsA growth slowdown is hardly shocking for an export-led economy. But China is in
much better shape than the rest of the world. A powerful rebalancing strategy offers the structural and cyclical
support that will allow it to avoid a hard landing.

China econ’s resilient---government control checks

Katsenelson ‘9 (Vitaliy N., director of research at Investment Management Associates in Denver, Colo., and the author
of Active Value Investing: Making Money in Range-Bound Markets, “The China Bubble's Coming -- But Not the One You Think”,
Foreign Policy, http://www.foreignpolicy.com/articles/2009/07/23/the_china_bubbles_coming_but_not_the_one_you_think, July
23, 2009)

Despite everything, the


Chinese economy has shown incredible resilience recently. Although its biggest
customers -- the United States and Europe -- are struggling (to say the least) and its
exports are down more than 20
percent, China is still spitting out economic growth numbers as if there weren't a worry in the
world. The most recent estimate put annual growth at nearly 8 percent. Is the Chinese economy operating in a different economic
reality?  Will it continue to grow, no matter what the global economy is doing?  The answer to both questions is no. China's fortunes
over the past decade are reminiscent of Lucent Technologies in the 1990s. Lucent sold computer equipment to dot-coms. At first, its
growth was natural, the result of selling goods to traditional, cash-generating companies. After opportunities with cash-generating
customers dried out, it moved to start-ups -- and its growth became slightly artificial. These dot-coms were able to buy Lucent's
equipment only by raising money through private equity and equity markets, since their business models didn't factor in the
necessity of cash-flow generation. Funds to buy Lucent's equipment quickly dried up, and its growth should have decelerated or
declined. Instead, Lucent offered its own financing to dot-coms by borrowing and lending money on the cheap to finance the
purchase of its own equipment. This worked well enough, until it came time to pay back the loans. The United States, of course, isn't
a dot-com. But a great portion of its growth came from borrowing Chinese money to buy Chinese goods, which means that Chinese
growth was dependent on that very same borrowing. Now the United States and the rest of the world is retrenching, corporations
are slashing their spending, and consumers are closing their pocket books. This means that the consumption of Chinese goods is on
the decline. And this is where the dot-com analogy breaks down. Unlike Lucent, China has nuclear weapons. It can print money at
will and can simply order its banks to lend. It is a communist command economy, after all. Lucent is now a $2 stock. China won't go
down that easily. The Chinese central bank has a significant advantage over the U.S. Federal Reserve. Chairman
Ben Bernanke and his cohort may print a lot of money (and they did), but there's almost nothing they can do to speed the velocity of
money. They simply cannot force banks to lend without nationalizing them (and only the government-sponsored enterprises have
been nationalized). They also cannot force corporations and consumers to spend. Since China isn't a democracy, it doesn't suffer
these problems. China's
communist government owns a large part of the money-creation and money-
spending apparatus. Money supply therefore shot up 28.5 percent in June. Since it controls the banks, it can
force them to lend, which it has also done. Finally, China can force government-owned corporate entities to
borrow and spend, and spend quickly itself. This isn't some slow-moving, touchy-feely democracy. If the Chinese government
decides to build a highway, it simply draws a straight line on the map. Any obstacle -- like a hospital, a school, or a Politburo
member's house -- can become a casualty of the greater good. (Okay -- maybe not the Politburo member's house). Although China
can't control consumer spending, the
consumer is a comparatively small part of its economy. Plus, currency control
diminishes the consumer's buying power. All of this makes the United States' TARP plans look like child's play. If
China wants
to stimulate the economy, it does so -- and fast. That's why the country is producing such robust economic numbers.

No Chinese economic collapse and no impact

Coonan ‘8 (10/25, Clifford, IrishTimes.com, “China's stalling boom has globe worried,”
http://www.irishtimes.com/newspaper/opinion/2008/1025/1224838827729.html)
All of this downbeat news feeds into a growing suspicion that China has had its cake and eaten for way too long, and that there
is simply no precedent for a country growing and growing without some kind of respite. Establishing what that pause will look
like and what it means to the rest of the world is the latest challenge facing global analysts. A hangover is considered inevitable
and the Olympics, while meaningless economically, are widely considered the psychological trigger for China to face a
slowdown. Despite all this gloom, however, writing China off is premature. The Beijing government is well placed to
help protect the economy from the worst ravages of a global downturn. It has spent the last two years trying
to fight inflation and cool the overheating economy, so it's a lot easier for it to take the foot off
the brakes than it is to put them on in the first place. The central bank has lowered its benchmark interest rate twice in the
past two months, the first time in six years. The State Council is increasing spending on infrastructure, offering tax rebates for
exporters and allowing state-controlled prices for agricultural products to rise. Expect significant measures to kick-start the
property market to avoid house prices falling too drastically. China has a lot of plus points to help out. Chinese banks did not
issue subprime loans as a rule, and the
country's €1.43 trillion in hard-currency reserves is a useful war
chest to call on in a downturn. The currency is stable and there are high liquidity levels , all of which
give China the most flexibility in the world to fend off the impact of the global financial crisis, says JP
Morgan economist Frank Gong. China is now a globalised economy, but its domestic market is still massively underexploited,
and it is to this market that the government will most likely turn. While it is a globalised economy committed to the WTO, China
is also a centralised economy run by the Communist Party, and it has no real political opposition at home to stop it acting
however it sees fit to stop sliding growth. Should the economy start to worsen significantly, public anger will increase, but China
has been so successful in keeping a tight leash on the internet and the media that it is difficult for opposition to organise itself in
a meaningful way. Recent years of surging growth in China have certainly done a lot to keep global economic data looking rosy,
but perhaps China's
influence has been somewhat oversold. It is not a big enough economy by itself to
keep the global economy ticking over, accounting for 5 per cent of the world economy,
compared to the United States with a muscular 28 per cent. And whatever about slowing growth, 9 per cent is
still an admirable rate, one that European leaders gathered this weekend in Beijing for the Asian-Europe Meeting would give
their eye teeth to be able to present to their constituencies.

No impact to Chinese economy

Blackwill, ‘9 – former associate dean of the Kennedy School of Government and Deputy Assistant to the President and Deputy
National Security Advisor for Strategic Planning (Robert, RAND, “The Geopolitical Consequences of the World Economic Recession—
A Caution”, http://www.rand.org/pubs/occasional_papers/2009/RAND_OP275.pdf, WEA)

Next, China. Again, five years from today. Did the recession undermine the grip of the Chinese Communist Party on the People’s
Republic of China (PRC)? No. Again, as Lee Kuan Yew stressed in the same recent speech, “ China
has proven itself to
be pragmatic, resilient and adaptive. The Chinese have survived severe crises —the Great Leap
Forward and the Cultural Revolution—few societies have been so stricken . These are reasons not to be
pessimistic.” Did the crisis make Washington more willing to succumb to the rise of Chinese power because of PRC holdings of
U.S. Treasury Bonds? No. Did
it alter China’s basic external direction and especially its efforts, stemming from its
own strategic analysis, to undermine the U.S. alliance system in Asia? No.
Did it cause the essence of Asian
security to transform? No.
---China Econ- Resilient
China can take a hit

Global Insight 8 (“Momentum of Chinese Growth Proves Resilient to Natural Disasters, Global Risks”, 7-17,
http://www.globalinsight.com/SDA/SDADetail13363.htm)

Growth in the Chinese economy moderated in the first half of the year, but proved durable in the face of
cataclysms at home and an increasingly grim outlook for external demand. Although momentum moderated,
the Chinese economy showed resilience in the first half of 2008 in the face of a string of natural disasters and
mounting downside risks in the global economic outlook. Data released by the National Bureau of Statistics (NBS) today
revealed that the economy expanded by 10.4% y/y in the first half of the year, after expanding by 10.1% in the second quarter.
In the three months through March, the economy expanded by 10.6%. Severe snowstorms at the beginning of the year, the
huge earthquake in Sichuan province in May, and recent flooding in other areas had been expected to rob growth of some
traction, compounded by reversals in U.S.-led global demand. The second-quarter outturn marked the slowest rate of growth
since 2005, but also the 14th consecutive quarter of double-digit growth.

More evidence

Asia Times 2 (Francesco Sisci, “China and the Global Security We”, 7-25,
http://www.atimes.com/atimes/China/DG25Ad01.html)

Furthermore, growth in the past 20 years in China


has proved not only buoyant but resilient. In spite of
crisis in one year or another, the economy has never plunged into a real recession , and the nation has
forfeited the whole socialist welfare system in a matter of a couple of years. Education and health assistance are now organized
on a strictly profit bases, without state support, housing has been privatized and jobs are no longer for life. These changes
would have caused more than one revolution in any other country, but in China they were digested without major uprisings.
Therefore in the future China can well be expected to carry on with economic reforms that appear
modest compared with the ones it has already achieved, and
continue its high growth.

China’s economy is extraordinarily resilient

Overholt 4 (Dr. William H., Asia Policy Chair – Center for Asia Pacific Policy at the RAND Corporation, “China’s Economy,
Resilience and Challenge”, Harvard China Review, Spring, http://www.rand.org/pubs/reprints/2005/RP1116.pdf)

China's economy has demonstrated extraordinary resilience in the face of the global economy
slowdown, the SARS tragedy, and the stresses of WTO entry. This resilience results from the successful
shift to domestic-led growth and from rising productivity caused by economic reform, rising competition, and highly
entrepreneurial economic structure, and high levels of foreign direct investment . China's successes are
being achieved by reforms that overcome severe challenges. Among these challenges are a pressures on
China to revalue its currency and China's rapidly expanding money supply and overheating economic expansion. What
distinguishes China from other countries facing similar challenges is that it has chosen a process of gradual reform and opening
that has proved successful in other Asia countries. It has also demonstrated an ability to form a workable leadership consensus
regarding its most important problems, to implement solutions in the face of great political and social stress, and to overcome
the stress by delivering large benefits for most of the Chinese people.
---China Econ- No Impact

Chinese econ decline doesn’t cause war- 97 crisis proves

SCMP 2 (South China Morning Post, 11-2, Lexis)

Apparently oil-rich islands in the South China Sea have for decades been a sticking point in relations between China and
Southeast Asian nations. At times from the early to mid-1990s, the issue even teetered on the brink of military confrontation.
But the meltdown of Asian economies in 1997 steered the region's focus from territorial claims to
matters closer to home - fiscal survival. China and the Association of Southeast Asian Nations
moved towards one another, seeking mutual benefits as a matter of necessity . The mainland
assisted by providing financial support and maintaining the value of the yuan.

No impact from econ decline

Pei ‘9 (Minxin, Senior Associate in the China Program at the Carnegie Endowment for International Peace, 3/12. “Will the Chinese
Communist Party Survive the Crisis?” Foreign Affairs. http://www.foreignaffairs.com/articles/64862/minxin-pei/will-the-chinese-
communist-party-survive-the-crisis)

It might seem reasonable to expect that challenges from the disaffected urban middle class, frustrated college graduates, and
unemployed migrants will constitute the principal threat to the party's rule. If those groups were in fact to band together in a
powerful coalition, then the world's longest-ruling party would indeed be in deep trouble. But that is not going to happen. Such a
revolutionary scenario overlooks two critical forces blocking political change in China and similar
authoritarian political systems: the regime's capacity for repression and the unity among the elite .
Economic crisis and social unrest may make it tougher for the CCP to govern, but they will not loosen the
party's hold on power. A glance at countries such as Zimbabwe, North Korea, Cuba, and Burma shows that a relatively
unified elite in control of the military and police can cling to power through brutal force, even in the face of abysmal economic
failure. Disunity within the ruling elite, on the other hand, weakens the regime's repressive capacity and usually spells the rulers'
doom. The CCP has already demonstrated its remarkable ability to contain and suppress chronic
social protest and small-scale dissident movements . The regime maintains the People's Armed Police, a well-
trained and well-equipped anti-riot force of 250,000. In addition, China's secret police are among the most capable in the world and
are augmented by a vast network of informers. And although the Internet may have made control of information more difficult,
and thoroughly to end the dissemination of dangerous news . Since the
Chinese censors can still react quickly
Tiananmen crackdown, the Chinese government has greatly refined its repressive capabilities. Responding to tens of
thousands of riots each year has made Chinese law enforcement the most experienced in the
world at crowd control and dispersion . Chinese state security services have applied the tactic of "political
decapitation" to great effect, quickly arresting protest leaders and leaving their followers disorganized, demoralized, and impotent.
If worsening economic conditions lead to a potentially explosive political situation, the party will stick to these tried-and-true
practices to ward off any organized movement against the regime.
China Trade War Answers
Frontline

Zero risk of trade war- the concept isn’t real

Alden ‘12 (Edward, Bernard L. Schwartz senior fellow at the Council on Foreign Relations (CFR), specializing in U.S. economic
competitiveness “What Exactly Is a “Trade War”? Time to Abolish a Silly Notion,” http://blogs.cfr.org/renewing-
america/2012/10/23/what-exactly-is-a-trade-war-time-to-abolish-a-silly-notion/, October 23, 2012)

I have a suggestion for everyone who writes about international trade: itis time to bury, once and for all, the concept of
a “trade war.” The phrase is so ubiquitous that it will be awfully hard to abolish; I have probably been guilty myself from time
to time. Indeed, it is almost a reflex that every time the United States or some other nation takes any action that
restricts imports in any fashion, reporters and editorial writers jump to their keyboards to warn that a trade war is
looming. But it is a canard that makes it far harder to have a sensible discussion about U.S. trade policy. No sooner had President
Obama and Mitt Romney finished their latest round of “who’s tougher on trade with China?” in their final debate Monday night than
the New York Times – to take one of many possible examples – warned that “formally citing Beijing as a currency manipulator may
backfire, economic and foreign-policy experts have said. In the worst case, it could set off a trade war, leading to falling American
exports to China and more expensive Chinese imports.” But what exactly is a “trade war”? To take the U.S.-China example, the
notion seems to be that, if the United States restricts Chinese imports, China will respond by restricting imports of U.S. goods, in turn
leading to further U.S. restrictions and so on and so on until trade between the two countries plummets. The closest historical
example is the reaction to the infamous Smoot-Hawley tariff act of 1930, which raised the average U.S. tariff on imports to
historically high levels. As trade historian Douglas Irwin of Dartmouth has show persuasively, Smoot-Hawley did not cause the Great
Depression, and probably did not even exacerbate it very much since trade was a tiny part of the U.S. economy. But Smoot-Hawley
did result in Great Britain, Canada and other U.S. trading partners raising their own tariffs in response. Irwin suggests that the higher
tariffs were probably responsible for about a third of the 40 percent drop in imports between 1929 and 1932, and perhaps a slightly
higher percentage of export losses. And the new trade barriers put in place took many decades to dismantle. With imports and
exports today comprising roughly a third of the U.S. economy, and the few remaining tariffs mostly in the single digits, the
consequences of similar tit-for-tat tariff increases today would be far more severe. But what are the
chances of such a “trade
war” actually occurring? Pretty close to zero, for two big reasons. First, in 1930, there was no World Trade
Organization, no North American Free Trade Agreement, no European Community/Union – in short, no rules to prevent
countries from jacking up tariffs or imposing quotas whenever governments felt domestic political pressure to do so. Today,
such unilateral action is largely forbidden. Indeed, the tit-for-tat measures we have seen in the U.S.-China trade relationship
have all been taken within the framework of WTO rules. When the Obama administration curbed purchases of
Chinese steel in 2009 under the “Buy America” provisions of the stimulus, for example, China responded with an
“anti-dumping” case which led to tariffs on imports of U.S. steel. But the United States challenged that action in the
WTO, and just last week the WTO ordered China to lift the duties. No trade war – instead the phrase “see you
in court” comes to mind. Secondly, almost every nation in the world seems fully aware of the dangers of aggressive
protectionism. One of the striking things about the Great Recession– which resulted in global trade volumes plunging by more
than 12 percent in 2009, the biggest drop since World War II – is how little of the protectionism that is permitted under WTO rules
actually occurred. Chad Bown of the World Bank has documented the surprising low level of new trade barriers imposed during the
recession and its aftermath. The danger of competitive currency devaluations – which are not clearly covered under WTO rules – is a
greater threat than tariffs. This is one of the reasons that Romney’s pledge to label China a currency manipulator
could be playing with fire, particularly after more than seven years in which the value of the renminbi has been creeping up steadily
against the dollar. And his suggestion that the United States would impose tariffs in response is
just silly – it would be a
blatant violation of WTO rules and would quickly be slapped down as such. Again, however, no trade
war – just an unfavorable WTO decision with which a Romney administration would quickly comply.

Trade conflicts inevitable and won’t escalate


Feigenbaum ’10 (BY EVAN A. FEIGENBAUM | OCTOBER 19, 2010 Evan A. Feigenbaum is head of the Asia practice group at the
Eurasia Group and adjunct senior fellow for Asia at the Council on Foreign Relations. From 2001 to 2009, he worked on East, Central,
and South Asia at the U.S. State Department.

The United States and China are deeply interdependent, with trade in goods between the two countries reaching a whopping $366
billion in 2009. But a growing number of influential people on both sides find that reality deeply alarming, albeit for different
reasons. In the United States, campaign ads this election season routinely blame trade with China for U.S. job losses. And in China,
rising stars like Wang Yang, the Communist Party boss who governs China's booming southern province of Guangdong, fret that
China's "traditional model is excessively dependent on international demand." In just the latest sign of this growing tension, the U.S.
House of Representatives last month passed legislation seeking to raise the cost to China for its currency policies. All signs at the
moment point toward increased trade and financial tension between the world's two economic giants. A full-fledged trade war
between the United States and China would be disastrous; thankfully, it's far from likely. Decision makers on both sides appear to
have concluded that their trade disputes can be managed without undermining the entire U.S.-China relationship. Trade
conflict is here to stay, but it is fast becoming a "new normal" in relations between Washington and
Beijing. What is fueling this growing tension on trade issues? Unemployment and flat growth in the United States are one part of the
story. But four
underlying factors are dramatically changing the U.S.-China economic relationship and will ensure
that conflicts persist into the future. First, U.S. and Chinese firms increasingly compete head-to-head because China is
moving up the value chain far more quickly and across a wider array of sectors -- from electric vehicles to solar energy to high-speed
rail -- than many in the United States once expected. As China seeks both to "indigenize" technology -- not simply rely on technology
transfers -- and to compete globally, it is forcing U.S. firms to confront a fast-changing and vastly more competitive landscape.
Chinese firms already offer cutting-edge technology in high-speed rail and are in the hunt for contracts in developed markets such as
Australia and California. And U.S. companies that once assumed a grand bargain -- providing U.S. technology in exchange for market
access in China -- must now fight Chinese competitors for the same market share. Many in China, not least Premier Wen Jiabao,
argue that China came late to both the industrial and information revolutions, and they are determined to ride the next
technological wave. So, China (like other states before it) is using government policy to support its ambitious goals -- for example,
favoring domestic companies in government procurement and offering preferential financing to homegrown national champions
such as the Commercial Aircraft Corporation of China, which might soon challenge Boeing in the narrow-body passenger jet market.
The bottom line is that U.S. firms face a more vigorous challenge from China. And they are working to meet that challenge in two
ways: First, by seeking to move up the value chain faster -- companies like Apple, for example, have upped their game and
succeeded, even in places like Japan, which is a wonderland of indigenously produced consumer electronics. And
second, U.S.-
based multinationals are teaming up with Western diplomats to push back against discriminatory market-access
policies in an effort to level the playing field in China. Even though China's undervalued currency preoccupies Congress and smaller
manufacturers, U.S. firms complain more often about the business climate in China -- a problem
that will not go away
even if the currency issue disappears. A second trend -- partly a result of intensifying competition -- is that old
coalitions that once provided ballast to U.S.-China relations are breaking down. In China, interest groups divide over nearly every
economic issue: Chinese exporters, bankers, and political leaders -- who once coalesced around trade-related issues -- are
increasingly at odds. Thus, China's central bank initiated a revaluation of its currency in June despite opposition from China's
Ministry of Commerce and export lobbies. Chinese interest groups are split over anti-dumping and protectionist trade measures.
And Chinese interest groups are divided, too, about access for foreigners to sensitive sectors, such as Shell's partnership with
PetroChina to explore for shale gas in Sichuan province. Meanwhile, despite the fact that many U.S. companies are deepening their
engagement with China, the old political-business coalition that helped Beijing gain permanent normal trading status in the 1990s is
fraying. It would likely be impossible to reassemble the alliance that worked to promote closer trade links in the Clinton and early
Bush years. And new areas of trade conflict are emerging, such as in clean energy, which might produce
new groups of skeptics. Just last week, the United States opened an investigation into Chinese support for clean-energy producers at
the urging of the United Steelworkers, prompting a vigorous verbal challenge from China. A third trend is the growing tolerance for
trade tensions in both Washington and Beijing. This confidence has made both governments less restrained in pursuing trade
disputes. But it also means the
United States and China have largely separated security issues, such as
North Korea and Iran, from the minutiae of Section 301 and 421 filings and market-access
disputes. The relationship will not collapse, even in the face of an avalanche of anti-dumping
suits , as both governments work to delink the various issues on an increasingly complex bilateral menu. Beijing, having grown
more comfortable with the World Trade Organization's dispute-resolution procedures (and having learned to leverage the system to
its own advantage), is now prepared to vigorously fight U.S. suits in many of these areas. It has investigated numerous anti-dumping
cases brought by Chinese producers, lent its ear to a proliferation of Chinese business lobbies, and is investigating a countervailing
duties case into U.S. subsidies for the Big Three automakers. Finally, U.S. demands for access to China's 1.3 billion consumers are
growing in both scope and intensity, particularly as China's indigenous innovation policies threaten the proprietary technologies of
U.S. companies. And demands for market access now flow both ways. A China already resistant to U.S. pressure will become even
more so the more Chinese investments in the United States are blocked. Taken together, these four factors guarantee that U.S.-
China trade relations are certain to become more fraught in the months and years ahead. But U.S.-China relations can
probably weather a proliferation of such acrimonious trade disputes , especially if they are channeled
through the WTO and other rules-based mechanisms. The bilateral relationship is extremely diverse ; both sides have
strong incentives not to let trade friction undermine every other form of cooperation. And it's
worth noting that virtually no U.S. company plans to flee China -- not even those that stand to lose the most from China's indigenous
innovation policies. Meanwhile, Beijing has two good reasons to keep the overall relationship with Washington on track. For one,
China's economy is not yet "decoupled" from America's; China continues to run large trade surpluses with the United States and,
because of its own stabilized exchange rate, is bound to U.S. monetary policy as its dollar reserves accumulate. For another, Beijing
has more trade and investment options with more countries than ever before; China can now weather conflict with the United
States more easily -- thus Beijing need not treat trade conflict with Washington as a strategic threat. Still, to keep frictions from
escalating, both sides must make sure they stick as much as possible to WTO and rules-based mechanisms for resolving their
differences, avoiding purely punitive actions not linked to specific commercial grievances. The likely course for the United States
probably involves pursuing a mix of anti-dumping and countervailing duties cases -- and continuing to search for a more systemic
remedy to press, persuade, and sometimes coerce China to level its playing field. That will produce very real tensions. But rules-
based spats, though contentious, will not likely result in underlying strategic conflict . Indeed, the
essential strategic reality of Asia today is this: China is fast becoming the central player in a new economic regionalism, but Asian
countries are deepening defense and political coordination with the United States as a hedge against Beijing's growing strategic
weight.
China Collapse Answers
Frontline

China unsustainable- collapse inevitable

Dickson ’12 (Micah Dickson, SeekingAlpha, Investor Trading Online News, “The Cracks In The Great Economic Wall Of China”,
November 27, 2012)

China has just gone through their once in a decade power transition. While
the transition of power has appeared to
have gone smoothly, it does not mean that the challenges facing China have diminished at all. Xi
Jinping and his regime face a host of challenges. These challenges vary from economic to
societal in nature. The current course China is on is utterly unsustainable . The question is, can the new
Chinese leadership make the necessary reforms to keep the country from a political and economic collapse? Investors must consider
the size and scope of the challenges facing China as they make decisions on where to allocate their assets for the coming year.
Economic Challenges Any true economic growth is based upon investors and consumers acting on information. The accuracy of that
information can decide if that economic growth is sustainable or not. Many of the basic economic numbers coming from China have
largely been called into question. Li Kepiang, possible future premier of China, said in 2012 that the GDP figures were "man-made".
There has also been documented cases of the growth in many Chinese industries being quite different from the overall GDP
numbers that are reported. Unfortunately, China's
state owned enterprises are becoming a prime example
of the failure of accurate information from China itself. State owned enterprises are filled with
Communist Party leaders who use them to bolster the Party's power. I ncluded in the list of state owned
enterprises are banks that provide loans to businesses. These businesses include other non-financial state owned enterprises. These
loans are given at lower interest rates and in unlimited amounts. This
incestuous relationship gives state owned
enterprises an advantage over other smaller enterprises inside the country . Besides being
incredibly corrupt, this system has led to what has been referred to as " zombie companies".
These are companies that should be going bankrupt because they are unable to repay their
debt. The Chinese government is not allowing these companies to go bankrupt. Instead the state
owned banks are being forced to continue to lend money to the enterprises despite their inability to repay the debt. Matthew
Boesler from the Business Insider commented on the effects of these practices in this way, "This is causing a deterioration in asset
quality on banks' balance sheets, and increases the chances that the government will have to bail them out down the road ".
Some estimate that the debt to equity ratio of many state owned enterprises exceed 230%. This
is a staggering figure. Even with all of these negative developments, the "official" amount of non-performing loans in the
Chinese banking sector is only 0.9%. This obvious contradiction is why so many of the numbers out of China are deceptions. The
banking sector numbers are not the only numbers that are troubling. The
state owned enterprises have been
showing weakness for a while. These enterprises make up 40% to 50% of GDP. From 2001 to 2009, these state owned
enterprises made 5.8 trillion Renminbi (RMB). This would equal $931.1 billion in the United States. Normally, this would be a tale of
their success. But if you remove the government subsidies for that same time period, the real average return on equity for the state
owned enterprises would be a negative 6.29%. These problems are compounded by a growing real estate
bubble. Part of China's growth has come from the government's investment in the building of
infrastructure. Robin Banerji and Patrick Jackson of the BBC describe the expansion like this, "The country is said to have built
the equivalent of Rome every two months in the past decade". The problem with this rapid expansion is that
supply is beginning to overtake demand . Satellite images are showing entire Chinese cities empty many years after
their construction. The World Bank's Holly Krambeck gave a frightening example of this in the city of Chenggong. She says, "In
Chenggong, there are more than 100,000 new apartments with no occupants". This is becoming the story all over China as new
buildings, office spaces, and other projects are lying empty due to the lack of occupants able to fill these empty structures. These
factors should cause investors to be cautious about their positions in China. A
red flag to any investor should be the
inability for Chinese companies to be audited by firms outside of China. If these large economic
challenges are not addressed, China may see anemic economic growth as Japan did in the 1990s
or w orse, an economic catastrophe that could rock the world markets as investors begin to
move their capital to other parts of the world. This could be hastened by the growing perception
of many in the United States, China's largest customer, that companies that do business there
are hurting American workers. Political Turmoil China is currently finishing their once in a decade
transition of political leadership. This however has not come without serious hiccups in the
road. There is serious tension inside the Chinese hierarchy which is beginning to reveal itself . As
Dean Cheng reported about the 2012 National People's Congress session, "As this year's session came to a close, outgoing Premier
Wen Jiabao warned of the potential for chaos and cited the Cultural Revolution of 1966-1976". This statement immediately
preceded the ousting of Chongqing Party Secretary Bo Xilai from the Communist Party. Bo, his wife, and many extended family
members were also charged with a variety of crimes including corruption, murder, and adultery. His populist tone and rising star in
the Communist Party made him an attractive candidate for higher office and many reports say he was campaigning for a position on
the CCP Politburo Standing Committee. This committee is the most important and power part of the Chinese leadership. Bo's rising
star quickly extinguished after his former police chief tried to defect to the United States. Because of how common corruption is
inside the Party, many find it strange that Bo Xilai was ousted and charged so quickly and publicly. Dean Cheng makes this comment
regarding the scandal: "Such major developments-occurring in the midst of one of China's most public political events-suggest that
Chinese politics are in major turmoil." While
the event with Bo Xilai is scandalous, it is an extension of the
ongoing concern of many inside the Party of the increasing corruption and the deterioration of
the perceived legitimacy of the Party . Premier Wen, who is exited his position during this most recent transition has
even publicly called for the power of the Communist Party to be reduced. In the 2011 World Economic Forum in Davos, Switzerland,
Premier Wen was quoted as saying: A ruling Party's most important duty is to follow the constitution and the law, and restrict its
activities within the constitution and the law…. This requires changes in the use of the Party as a substitute for the government and
in the phenomenon of over-concentration of power. For this, we need to reform the leadership system of the Party and the country.
But these reforms
will be close to impossible to carry out as the 70 wealthiest members of the
National People's Congress are ten times wealthier than the top 660 government officials here
in the United States. This is due to the fact that state owned enterprises are run by members of the
National People's Congress or by a close relative of those members . Any reforms would mean
these members would have to give up their sources of wealth and power. As we've seen in many cases,
economic troubles can strain political relations even further . If China does not make changes,
the corruption and decadence in the ruling Communist Party could become the scapegoat for
any "hard landing" China experiences. If China experiences a hard landing, it would lead to the second largest
economy in the world falling into political chaos. This would create uncertainty that would trump the uncertainty experienced from
the problems in the European Union. Societal Challenges The
political problems in China are compounded by
the fact that there is growing unrest among the average citizen in China. The largest problem is
that of forced evictions by the Chinese government. After the Financial Crisis of 2008, the Chinese government
began implementing an extremely large stimulus package. The main thrust of the stimulus package is to build up infrastructure
across the country. In order to do this, many
Chinese cities are forcibly and violently evicting citizens who
live on land that is going to be used for new government building projects. The stories of these
forced evictions have caused outrage throughout the Chinese population. An example of how outraged
many citizens are is the Chinese fishing village of Wukan. The citizens of the village became fed up with land grabs from the
government. In response, they rushed the offices of the local government during a protest. After the protest, one of the protest
leaders died while in custody. This led to the village ousting the Communist Party leadership in the village and democratically
electing local leaders. While this rebellion is an extreme example, what caused the outrage is still there and is becoming prevalent
among the Chinese people. Land grabs are not the only problems, income disparity, working conditions,
and many more social ills are beginning to bubble over. In 2010, China experienced 180,000
protests, riots, and mass demonstration. This is staggering . Unfortunately, many of the complaints are too
narrow to begin a nationwide movement that will cause sweeping reforms inside China. That will not last for long. More
and
more of the protest leaders admit that the underlying problem with the country is the one party
system that has dominated the government for so long. This growing public anger combined
with the political turmoil inside the country could combine to create a deadly chemical
reaction . Conclusion The Chinese model is quickly becoming a potential Chinese nightmare . While it
has created incredible wealth inside China, it
has created a monster that does not seem to be able to make
the necessary changes. China has to go back to the path of reforms that Deng began in 1970s in which their economy
becomes freer. Unfortunately, the incredible corruption that has sprang up from China's economic
growth is beginning to insulate itself. Communist Party leaders have shown hostility toward any
change. Investors must consider these factors when looking toward China for the growth that is missing in the United States.
While that growth may be advantageous in the short to medium term, it could be an incredibly risky bet in the long term. If China
does not address its economic, political, and societal challenges, the Great Wall that is the rising
Chinese economy may have a mighty fall .
---China Collapse– Inevitable
Most qualified economist agree- reason to prefer- our guy is the Nate Silver of economic
predictions since the 70s - you cannot trust contrarian economic reports

Droke 12 (Clif Droke is the editor of the daily Gold & Silver Stock Report. Published daily since 2002, the report provides forecasts
and analysis of the leading gold, silver, uranium and energy stocks from a short-term technical standpoint. He is also the author of
numerous books, including 'How to Read Chart Patterns for Greater Profits’, “The Coming China Economic Crisis”,
http://www.marketoracle.co.uk/Article38147.html, Dec 20, 2012)

When Bert Dohmen talks, smart investors listen . In 2007 when most investment analysts and economists were
downplaying the developing credit market troubles, Bert warned investors that the probability was very high

that the troubles would escalate into full-blown crisis and would produce a crash of historic
proportions . He chronicled the developing credit crisis in the pages of his newsletter and also published a book in
early 2012 entitled, The Coming China Crisis, which provided his insightful views on the
emerging crisis in depth. Dohmen writes the widely read Wellington Letter and China Boom-Bust
Analyst investment advisories. His Wellington Letter has provided top-notch forecast and
analysis of U.S. and global financial and economic trends since January 1977 . His newsletter
has received many #1 ratings by the top ratings services and has forecasted every bear
market using sophisticated technical analysis . Bert also frequently appears as a guest on
financial television, including CNN's Moneyline, CNBC and FOX News. Over the last 30 years he
has been a favorite speaker at the largest investment conferences . On December 18, I spoke with Bert
concerning his forecast of the coming China crisis, the global economy, the U.S. "fiscal cliff" and the likelihood of another worldwide
financial crisis. Following is a transcript of that interview. Q: You just returned from a trip to China. What can you tell us about it?
Dohmen: I learned something that just confirmed what I already knew . You can learn a lot more from a
country from your desk using the Internet than you can visiting a country and being wined and dined. I spoke with hedge fund
managers, institutional investors, heads of corporations, etc. The first story they give you is that 'everything is wonderful in China'
and that the country is only experiencing a short-term lull with the economy. But when they find out you know more, they open up
and give you the true story: 'We're stagnating, the economy is weakening, etc.' Q: In your book, The Coming China Crisis, you
mentioned that your work strongly indicates China is now going through what the U.S. did in 2007-2008. Dohmen: Yes they are, but
it will be a different kind of crisis because China's institutions are different. What we experienced was a financial crisis where
institutions went out of business or had to be bailed out. The repercussions were actually a downturn in the U.S. economy. In my
opinion, the next crisis will be an economic crisis for the U.S. and globally where all the stimulus that has been put into system is
wearing off and has less and less effect in helping the economy. I see signs that it's being counterproductive. Q: Please elaborate.
Dohmen: The
Fed's monetary policy is destructive if you look behind the scenes. It gives the central
banks no way out. The Chinese government is doing everything it can to keep its banks afloat . For
instance, the banks in China have 21 trillion dollars worth of loans. That's a huge amount considering there are only 3 trillion in
China's reserves. It's
estimated by accounting firms that bad loans on banks' books is as much as 40-
50 percent of total loans. That's 10 trillion in bad loans on the books total and it's unsustainable.
This could cause a huge crisis. The Chinese government may not be able to continue these
bailouts eventually. In the future that will be the limiting factor. Emerging markets like China have always been a problem
when money starts flowing out. Foreign capital flows out and then the problems begin. We saw this late last year and earlier this
year and discussed it in our China Boom-Bust Analyst newsletter. Foreign direct investment has since gone from
largely positive to negative. Money is flowing out of China. For emerging markets that's always
the first big warning sign of an approaching crisis. China has more foreign currency reserves than
others; it will delay the problem but not cancel out an eventual crisis. When China goes into
severe contraction, the world economy will suffer. Q: Chinese industrial demand obviously influences the price
levels of major commodities. Where do you see commodity prices headed in 2013? Dohmen: Raw material prices will be on the
weak side. China will start new stimuli for various sectors. But they can only build infrastructure that use raw materials and they
don't use enough of it. They have huge stockpiles of copper and steel and are still producing but unable to sell it. Many larger cities
have populations of more than 1 million people. These cities have steel mills and are large employers. Governments of these cities
hesitate to shut down because they don't want unemployment, so they keep producing. When
you're analyzing China
don't look at production numbers, look at sales numbers which are much harder to get. There's a
big difference between steel production and steel sales. For instance, a big headline a few months ago was that car sales hit new
records in the U.S. But the fine print is that these sales are by manufacturers which stuff the cars into retail channel and sell to auto
shops. The retailers were screaming, 'Stop sending us these cars, we can't unload them!' It's called channel stuffing in the U.S. and
they do it in other countries, too. These are the numbers you see reported, not the actual numbers. Q: Can China become truly great
under its present Communist regime? Dohmen: A
year-and-a-half ago we wrote that the Great Leap Forward
had hit the Great Wall of Communism . For China the easy stuff is over, such as low labor rates and wages. They were
able to get all this wonderful Western technology free of charge. The Western multinationals all signed over the details of their
patents to their Chinese partners. That's gone now. China got about 100 years of development in the West for free. Now there's a
point in the lifecycle of any country where entrepreneurs would be taking over and helping develop, but in
China they don't
have freedom. In China you see it every step of the way; everything is controlled by the
government. Unless China finds someone like Gorbachev to dismantle the Communist regime, China will languish and
its growth will decelerate. The private sector is over there is already in recession. The GDP numbers
which they advertise at 7 percent is totally phony . The government says people shouldn't use it
as being accurate, but as a 'guideline' [laughs]. It's being overstated and the numbers are
unreliable . To measure what's really happening we use private sector numbers like electrical consumption, which has declined
for 1 ½ years. How can this be if China's economy is growing? Q: How big
is the real estate bubble in China and has
it burst? Dohmen: Yes. People are still talking about Beijing apartment prices being sky high, but
the rest of country has had big declines. Development companies have been selling units and dropping prices 30-50
percent. This makes prior buyers angry and they want to cancel sales. There are no land sales in
China because only government can own land. They're 50 year leases, long-term leases. Recently there
have been very big new leases at record rates. These are also phony. Companies take lands and lease to
developers, which gives them 50% revenues to run hospitals, fire departments and other municipal services. In the real estate
plunge last year developers weren't buying anymore and municipalities lack of income. So they make deals with developers. The
lease rates has to be at new highs so people get enthused and think real estate is rising. It's another form of market manipulation. Q:
On page 119 of your book you noted that the growth rate in China's money supply had declined from a hefty 30% annual rate of
growth to around 12.5% in late 2011. What does China's current money supply growth rate look like? Dohmen: I don't have the

numbers in front of me right now but it continues to decline, money velocity is declining. Everything is
ratcheting down. The only entities getting bank loans are State Owned Enterprises (SOEs). The
'princelings' in China are the children of high party officials, and it's only they who get the
loans . It's total corruption . Small entrepreneurs have a difficult time competing with larger
companies and can't get these favorable bank loans or any loans. When they start encroaching on SOE
territory, the SOEs have a way of getting them out of business. Credit lines get cut. They're better connected and can eliminate
competition easily. You can't have a growing economy with this going on. Q: Recently there have been signs of a Chinese economic
rebound. Do you see this as being a temporary "dead cat bounce" or can this reversal be sustained? Dohmen: The head of the
Communist Party Congress will manipulate a bounce with statistics, as we said last month in our Wellington Letter. When you
compile the numbers you can make the economy do whatever you want. The real economy doesn't respond to that, however. In
March 2013 China will have a new 5-year plan. Until then things will look rosier, then reality will hit. You just have to look at free
market numbers like freight indices. The Baltic Dry Freight Index is a good one to look at. That is now scraping at the bottom of 2008
crisis. In 2008 that index collapsed 93 percent. Freight rates dropped by that much. You could rent a large 1,000 foot carrier at that
time for the same cost of a 35-foot boat on Lake Tahoe in 2008! When goods aren't being shipped they're not being used. Freight
rates are a much better indicator than GDP numbers. Q: If China goes into a major recession, what effects will this have on the U.S.?
Dohmen: This will be like a tsunami going through the economies of the globe. China has been the big locomotive for the world
economy. China's stimulus was four times the size per GDP than that of the U.S. They were four times as aggressive as the U.S. Fed in
stimulating their economy. This caused a commodities rebound, stock market rebound, etc. Australia was also affected by China's

demand for commodities. Looking forward, I can't understand where any good news is going to come
from. We have our analysis and scenarios we go through and every day we review everything
and ask if anything has changed that would make us wrong in our predicdtions or confirm our
analysis. Today we found out that China has cancelled a 300 ton soybeans order from the U.S. What does it mean? Does China
have too many soybeans? Are the Chinese people not hungry? Are prices too high? Another factor is that we have
some proverbial 'canaries in the mine' in that China is trying to conserve foreign currency
outflows. When China buys goods from West they pay in foreign currency, not renminbi. Are they
starting to conserve foreign currencies because they're being depleted of all that's leaving the country? We're seeing
cancellations and reductions in China left and right. One of our clients sells high grade seafood to China. He told
us recently that a big order from a major client from China was cancelled because they couldn't get dollars, foreign currency, from
their bank. So when demand from China goes down it effects the global economy. You don't see this in the daily newspapers or on
TV. You don't get this information from a visit to China, either. The China bulls are always touting the fact that China has 1.3 billon
people and those numbers will supposedly translate into obscene riches. But the number of people doesn't necessarily mean the
country in question will have a beautiful economy. Q: Before we go I have to ask you the question of the hour. What are your views
on the U.S. "fiscal cliff"? Dohmen: In our Wellington Letter for December we discussed that. Nobody knows if there's going to be an
agreement among the Democrats and Republicans. The popular view is that there will be a compromise by year end. But as you
know, the popular view is usually wrong. I can see a situation where there's a lot to be gained by Democrats by letting the country go
over the cliff and then blaming the Republicans. 'The Republicans made us do it' will be the excuse. I just don't see why so many
analysts are bullish on 2013. Companies are going to be hit by huge new costs for employees. There have already been huge price
increases for restaurants, etc., with prices up 20-30% in the last six months. I think we'll continue to see inflation in supermarket
prices and deflation in good we can do without, durable goods. For instance I've noted that Costco has made price increases of 25 to
35 percent recently. How will the Federal Reserve deal with this? A trillion dollars next year they'll put into the system. It's nothing to
them. This new stimulus, how are they ever going to exit this policy of zero percent interest rates? Someday the market will put an
end to that and paper money will become worthless and the market will know it. Right now we have 80 to 90 percent of Treasury
securities purchases being conducted by Fed through their stimulus. When the Treasury finances all their expenditures with freshly
printed money, you can see in history how this will end. There will be no happy ending to this. Can you imagine when you see real
inflation rising, could be 10% or higher, what will people say? Finally they'll have to stop inflation. When there's the smallest sign
that the Fed is reversing course you'll see a collapse in financial markets. Whenever it comes, be it next year, 5 years or more; it's
coming.

Its past the point of no return

Winnifrith ’12 (Tom Winnifrith founded the share tipping website t1ps.com in 2000 after a ten year careeer as an analyst,
journalist and TV presenter. He quit t1ps in September 2012 and now produces his own blog http://www.TomWinnifrith.com and
writes at http://www.advfn.com/newspaper/tom-winnifrith, “China Crash 2012/2013 - Overcapacity And Bubbles (Part One)”,
http://seekingalpha.com/article/860921-china-crash-2012-2013-overcapacity-and-bubbles-part-one, September 11, 2012)

There can be no doubt now that China is undergoing a "hard landing." But that hard landing will
soon turn into a crash . Although this is not the consensus view of analysts, the experts have called the slowdown badly
wrong so far and are still wrong. No amount of fiscal and monetary stimulus to the domestic economy
can dodge the pending crash. That is because the crash will be driven by chronic overcapacity
(driven by capital misallocation) in the production of commodity goods for export; structural
weakness in the domestic banking system and the existence of vast speculative bubbles, which
must at some stage burst. Hence, stimulating domestic consumption will change nothing. Many of us
have invested in China based or exposed assets because we have been told "you must have some China exposure, long-term growth,
etc, etc." There may be long-term growth but not for a while and ahead of a violent crash the sensible China asset allocation is 0%. I
have already offered up five small-cap China shorts on SeekingAlpha here, here, here and here) but in a second article later this
week I shall outline a specific list of larger investment calls based on the inevitability of China suffering a great economic crash during
the next 12 months. The debate during the summer was whether China would suffer a soft landing or a hard one. In the soft landing
camp were most of the sell-side economists and the Government of the PRC. Thus when second quarter GDP growth was
announced at 7.6% in early July, Xianfang Ren, an economist at IHS Global Insight in Beijing told AP that "overall, this is a soft
landing, but we can see that the Chinese economy is undergoing serious pain." Ren
confuses a soft landing (official
numbers) with serious pain (the real economy). That is a common error. The consensus among
analysts was (and still is) that, as in 2008, the Chinese Government could revive economic growth by a
combination of cutting base rates and starting huge infrastructure programs . This analysis is
wrong on two counts: it fails to address the nature of the problems and, consequently, the
policies are not working. There have always been grave doubts as to the reliability of
Chinese GDP statistics anyway - they appear to be created in an overly optimistic manner . Thus,
while second-quarter GDP was reported to have grown by 7.6% (down from 8.1% in Q1), Platts reported that
demand for oil grew by just 0.5% year on year in May. That is hardly what one expects of an economy meant to be growing at 8%
per annum. Meanwhile the China Coal Industry Asociation admits that coal stockpiles have been at record highs (and growing) since
May - there simply is not the demand. That is not exactly what one expects in an economy meant to be growing at 8% a
year. Electricity demand growth has now fallen to c5% per annum and is still falling. It is not just Western critics who
seem to think that official GDP data and what is happening in the real economy do not add up. Li
Keqiang, a Senior Chinese politician admitted in 2010 that the numbers are " man made ." And so
when I refer to a hard landing, I
do not refer to Chinese GDP data but to what is happening in the real
economy. Although, I do expect even the man-made GDP growth numbers to shrink rapidly. In the real economy, the issue
China faces is that it has enjoyed a vast misallocation of capital. This is partly a result of Central
Government diktats, the 5-year plans that identify growth industries. The truth is that Chinese politicians are as
useless at picking winners as we are in the West. And it is partly due to the nature of the Chinese banking system,
which is driven by state "encouragement" as to where it lends and which appears to have much looser lending criteria than banks in
the West. Thus it takes $7 invested by debt in China to add $1 to GDP while in the U.S. that number is $4-5. China may have many
attractions for investors but when it comes to allocating capital, it is highly inefficient. And this is particularly pertinent because what
has driven Chinese economic growth in the past decade has been a combination of huge investment spending and exports - this is
not an economy driven by domestic consumption. The practical result of the misallocation of capital is (if we ignore the trifling
matter of large-scale theft and fraud) that there has been excessive real economy investment either in commodity markets where
China has created global overcapacity or in industries where there really is no economic case at all. And as such, quite a lot of those
banking assets will at some stage have to head off to money heaven. And so what is the state of the real economy? Over the past
few months I have run a series of articles detailing the scale of the slowdown in everything from underpants and sock production to
steel production to ship building. The
story is the same across the board: record inventories, collapsing
sales, factory shutdowns, order levels which were predicted just six months ago to rise by 10%
this year actually falling by 10%. On a daily basis data comes out that shows that industry after
industry in China is seeing a collapse in demand, although that is perhaps not yet reflected in
actual GDP data. Shall we start with Steel? Courtesy of Proactive Investors: Raw steel production in China recorded a 2%
increase in early August this year taking the country's steel production to a record 400% increase over a period of 10 years. In 2012
China's steel production is expected to touch 715 million tons. Despite production cuts, slackening demand and rising stockpiles
China's output for 2012 is expected to be 5.2% up from 2011. The paradox here is that though the demand for steel is down and
reaching lower levels daily, Chinese steel mills are still producing to achieve record steel output figures. According to China Iron and
Steel Association (CISA), the country's steel stockpile is up by 26% from last year. The CISA also reports that the country's
steelmakers saw profits fall by 96% on the back of slowing demand, triggering speculations about a possible revival of tax breaks for
Chinese steel producers. However, the country's steel production shows no signs of letting up barring a few production cuts. Many
analysts believe that perhaps the numbers don't reveal the full story as China's steel production figures are largely based on the
output of state owned manufacturers whose primary objective is to meet the government's production targets, irrespective of

market conditions. So just how is that sustainable? It is pretty much the same across all sectors.
What has caused the slowdown? It is not that China is uncompetitive. Labor costs have risen a little (but are still pretty
low) but it is that all key export markets have seized up. China's biggest export market is the EU. I do not need to elaborate on what
is going on there. Then there is the U.S. where economic growth is anemic and there is growing anti-China rhetoric. A poor third is
Hong Kong whose fate is pretty closely tied to that of China. In commodity markets, a slump in demand can only cause prices to

slump and that can only be corrected by a loss of productive capacity. But that poses a secondary problem for China. Its banks .
Charlene Chu from Fitch Ratings issued a note last week warning that China's banking sector assets are now almost $21 trillion, up

from $9 trillion in late 2008. This is an extraordinary rate of banking growth. Even a modest shock could "wipe out
the sector's entire earnings," she warned. In an earlier note from last December ( I give Chu gold stars for foresight) she
warned: "Recent stress in informal lending and among property developers, SMEs, and local governments has not reached systemic
levels. Nonetheless, Fitch Ratings believes these are not isolated cases of distress, but rather emblematic of excesses from the credit
boom and a policy orientation that overly relies on credit controls and low fixed-interest rates; prioritises the state sector above
private companies and savers; and favours forbearance and support over restructuring. In this light, recent problems could be only
the beginning." And
where could that shock come from? Well how about the collapse of tens of
thousands of loss making producers of commodity products for export . A report out on September 9,
about the crisis facing "Sock City," a new town that became the centre of China's sock industry demonstrates the problem. Sock
producer Xu Liele claims "73 clothing firms have gone under this year, with some leaving behind debts of billions of yuan. Things

have not been so bad since 2003 , when foreign buyers stayed away because of the Sars crisis."
Sock City is small but 73 sets of bad debts in socks in one small town can be multiplied many thousands of times across China as a
whole as this crisis continues. At some stage the real economy crisis may well turn into a banking crisis as well. B ut there is a
secondary aspect of the capital misallocation. I am ignoring the elements of political fraud and
corruption raised by the Bo Xilai case. He salted away $1.3 billion and it is believed that some other senior party
officials have trousered more (Mr 4%, the former Railway Minister, is believed to have relieved the PRC of $2.8 billion). This will
provide much newspaper fodder but the real crisis is in the various bubbles China has created. The policy of loose money designed
to recover from the 2002-3 and 2008 crises prompted some to invest in hard assets but others to back entirely speculative schemes.
The Chinese banks have been only too willing to assist. The property market is an obvious disaster waiting to happen. All the major
cities now contain office blocks that have never been occupied and never will be occupied but have been traded as speculative
counters. Taking this lunacy one stage further China has allowed the construction of near ghost new Cities. In 2010 there were an
estimated 64 million empty homes in China. And yet 20 new Cities were being built each year and property prices in established
Cities moved ahead fast. In Shanghai between 2005 and 2012 second hand house prices increased in value by 73%. But the real
lunacy is in the ghost towns - vast new creations of homes which lie empty . One million people were
meant to live in Ordos in inner Mongolia. All the flats and houses have been sold to investors. But it lies empty. At what stage
to the investors earning a negative yield (after maintenance costs) on an asset that will never generate a cent in cash realize that it is

worthless ? This is a bubble. But housing and office space is only one bubble of many. Perhaps the most ludicrous is the
great walnut bubble. Prices of this nut (supplies of which are virtually unlimited) have gone up by 1000% in a decade as it has come
to be regarded as a hot alternative asset class. This is the lunacy of the Dutch tulip bulb bubble of 1637 transplanted out East. The

full gory details of this insanity can be found here. With all bubbles it is not if, but when they burst. When the
various China bubbles do burst the knock on effects on both the real economy and the banking
sector are not going to be good. For the latter it will be yet more bad debts to write off against balance sheets that look
increasingly stretched. For the former it means ever weaker domestic demand. The China bulls argue that mammoth
fiscal and monetary stimulus will turn things around. Neither policy will cure the problem of
capital misallocation. If anything they will make it worse as cheap money is pumped into
Walnut accumulation, etc. The bubbles may be sustained and allowed to grow a bit more (making
their final implosion even more dramatic) but the real economy needs a revival of exports not domestic
stimulus. Indeed given easy money but an economy that is facing a dramatic implosion there is no evidence that this cash will be
invested or consumed. China bulls also argue that much of this is discounted in a domestic stock market valued on just 11 times
historic earnings. I would disagree. In 2011 the economy was still growing rapidly. In 2012 and 2013 corporate earnings across most
sectors will collapse. In many industries few firms are profitable at all. Earnings visibility for the Chinese market right now is minimal
and thus it cannot be viewed as a value investment. I would add that, by weighting, 40% of the Shanghai composite Index is in
financial stocks where there have to be real concerns about viability for many companies, let alone earnings visibility. As an investor
I have never felt the need to have China exposure. I simply do not trust economies where capital is largely
allocated by the state. And the one-child policy means that its long-term demographic profile is
not that attractive either. But even if I believed in the long-term story, right now I'd be selling all my China-related assets
and looking to return in early 2014 or perhaps a bit earlier, after the crash that lies ahead. In Part two of this piece I shall make some
specific investment calls for those who share the view that a hard landing in China in 2012 will rapidly turn into a crash.

Demographics

Moghein ’12 (Gaurav Moghein, Policy Mic, Online Policy Science Journal, “China Economy Will Collapse Eventually: The
Glorious Days of the Dragon Are Over”, http://www.policymic.com/articles/10871/china-economy-will-collapse-eventually-the-
glorious-days-of-the-dragon-are-over, June 2012)

China’s economic growth has been both impressive and stable since the 1980s, and many argue that the picture will remain same for

decades to come. But this decade China will go through a massive negative transition, defying such
argument. The negative transition will take roots in political institution, demography and export
markets, the very elements that have carved the success story of China. Talking about political institution,
China is the largest Communist regime which has aimed for ‘growth at any cost.’ This motto has largely helped China to grow at
double digit pace, build world class infrastructure, road and rail networks, and telecommunication, but at the same time it has
fuelled ire in China’s population for the regime as human rights are often disregarded, no check is in place on growing inequality,
government spends less on social welfare, and the country has no such thing as ‘voice of the people.’ Of late, that ire has surfaced
against one party rule and a growing wealth gap. Signs of political chaos are visible and it will only grow further as export market – a
major source of economic growth and a cushion for China’s leaders in times of public protest– is likely to decline. China is the largest
exporter in the world, with huge trade surplus. That
golden age of China is on the verge of ending. Population
has started growing old, which has led to the vacancy of many jobs; as a result, wages have gone up
ending the period of cheap labors which played a crucial role in China’s growth story. With
the rise in wages companies will elevate their products price to compensate the increased
wages which will cause a damaging effect on export markets. The other damaging effect of
aging population is the rise of pensioners. As the youth is almost disappearing, it will be a hard
decade for China as it will confront the challenge of paying to an increased amount of
pensioners at a time when productivity and export market is on the decline. A parallel challenge
that will come with it, is public protest. Chinese govt. will try to stabilize political institution, suppressing protest by
spending more on public welfare keeping economic growth at stake. China has definitely
nothing in its favor right now. The deadly trio that is growing public protests against a
communist regime, an aging population, and the rise in prices leading to decline in exports may
cut China’s three-decade long growth streak. However, its economy will not collapse and will grow somewhere
between 6% and 7%, given its strong economy. But for a country with such a large population, 6% growth looks like a dire situation.
2012 may be Year of the Dragon, but the coming decades will definitely be hard for the Dragon.

Boom goes the dynamite

Main 12 (Beth Main, Shanghaiist in News, “Experts: China's economy to crash (Make up your minds!)”,
http://shanghaiist.com/2012/12/11/chinas_economy_to_crash_make_up_you.php, December 11, 2012)

In a stark contrast to this week's news that China is going to overtake the US economy within 20 years, a different
group of
experts have warned that China might actually be heading for a crash due to the strain on
exports caused by the rest of the world's economic woes and China's over eager construction
fever. The Chinese government and many economists are now expecting growth to slip below 8%
this year and of course some are predicting a full on crash , just to keep things interesting. One major problem is
that Chinahas focused too heavily on construction (making up 25% of all activity and about 15% of all jobs) which
while it provided jobs reportedly left four million apartments standing empty in Beijing alone .
This has caused the property market to slow and land sales to collapse . A large proportion of
the construction boom was funded by loans from state banks , still happy to lend to state owned enterprises
(SOEs) at bargain rates. Due to its stimulus spending, China's total outstanding debt has increased by

around 50 percentage points of g ross d omestic p roduct over the past few years. The Chinese
economy is also over reliant on exports. Domestic consumption needs to rise from its current
level of little more than one-third of GDP if it is to compensate for falling international demand.
However, it is unlikely that consumption will rise while the property bubble is deflating and
construction jobs are being lost. In the worst case scenario the Eurozone debt crisis and a slow economic recovery in
America dampens exports further and really leaves China up shit creek. Despite all the naysaying, the economy is still growing, albeit
slowly. For one group the future is looking bright, with starting salaries for graduates expected to rise 13% in 2012.
China CCP Collapse Answers
Frontline

No CCP collapse

Yuan, 11 – associate professor and acting director of the Center for International Security Studies at the University of Sydney
(12/20, Jingdong, “The Arab Spring and China's Evolving Middle East Policy,” World Politics Review,
http://www.worldpoliticsreview.com/articles/10992/the-arab-spring-and-chinas-evolving-middle-east-policy?page=1)

While Beijing has its concerns over the Arab Spring and its potentially infectious impacts on social and economic stability in China,
there are strong reasons to believe that any imminent threat to Communist Party rule remains
minimal and manageable. First, in most Middle Eastern countries, the autocratic ruler has personally reigned for decades
and has instilled a political order that is typically repressive and nonrepresentative. The lack of any meaningful political participation
provides ample frustration and is one of the principal reasons behind the various uprisings. By contrast, although China
remains under one-party rule, managed term limits and an institutionalized leadership
succession have been put in place. Second, there is a major difference between the Chinese
economy and those of the Arab world. Chinese reforms over the past three decades have opened the
country’s economy to the world, lifting hundreds of millions of Chinese people out of poverty and
creating a sizable middle class that is more interested in gains in personal welfare than in
politics. By contrast, the stagnation in many Middle Eastern economies, despite plentiful resources and oil revenues, has
infuriated ordinary citizens, especially the restless young, who find employment elusive. However, perhaps the most critical
difference is that Beijing retains total control over the military, the paramilitary and the police forces, on
whose loyalty it can count. Having learned the lessons of the 1989 Tiananmen student uprising, the Chinese authorities
quickly introduced and enforced censorship of social media after the initial Arab unrest and were resolute in stopping any organized
protests from growing into massive social movements. These
actions contrast sharply with the militaries and
security forces in countries such as Egypt and Libya, which either split or abandoned the regimes
they were supposed to protect, leading to the fall of Mubarak and Gadhafi .

No regime collapse.

Platt ’12 (8/30 [STEPHEN PLATT & JEFFREY WASSERSTROM - Stephen Platt is the author of Autumn in the Heavenly Kingdom:
China, the West, and the Epic Story of the Taiping Civil War. Jeffrey Wasserstrom is the author of China in the 21st Century and co-
editor of Chinese Characters: Profiles of Fast-Changing Lives in a Fast-Changing Land. AUG 30 2012, 12:14 PM ET
http://www.theatlantic.com/international/archive/2012/08/chinas-long-history-of-defying-the-doomsayers/261783/]

Still, not all of the CCP's efforts have been so defensive in nature. The Party has also made some positive changes, such as loosening
controls on private life, helping boost living standards, and raising China's global influence, all of which have likely made it easier for
Chinese citizens to tolerate or even support the Party's rule. The Party is talented at adapting incrementally, changing course a bit at
a time. This can work for a while, even a long while, but that doesn't mean it can go on indefinitely. Both of the CCP's two most
recent predecessors, struggling to maintain their legitimacy, eventually attempted their own complete reinvention. In the early
1900s, the Qing dynasty, in a failed bid to outrun the forces of revolution from within, abolished the Confucian examinations that
legitimized it for more than two centuries and tried to reinvent itself as a constitutional monarchy. Taiwan, under Nationalist control
from the late 1940s on, began its transformation into a thriving democracy under the watch of Chiang Kai-shek's son. Today, a Party
China's military is presently powerful enough
president rules Taiwan not as a dictator but as an elected official.
and its diplomacy stable enough that the Communist Party faces no realistic threats from
outside. Internally, its control over society is effective enough that, while unrest and discontent
may be widespread, there are neither well-organized opposition parties nor rebellious armies
that might seriously challenge the central government. For now, the Communist Party finds itself in a position
that would be enviable to the officials of the late Qing. It could, if it wished, reinvent itself with a new legitimizing narrative, or even
open the way to a new multiparty political structure as the Nationalists did in Taiwan, likely without fear of being overthrown in the
process. If it does not make such changes, however, then it seems likely that the corruption and internal dissent of today will
continue to mount. If that happens, then it is likely only a matter of time until that dissent and corruption reach a critical mass
necessary to end the regime. But, as the world learned from the late Lord Macartney's failed prediction, those processes can take
many generations longer than we might expect. Even
if the Communist Party's legitimacy does weaken
enough for the party to fall, it might not be in any of our lifetimes.

No lashout- CCP would fear retaliation AND even if the order was issuds the PLA would not
obey

Gilley 5 (Bruce, Professor of International Affairs @ New School University and Former Contributing Editor @ the Far Eastern
Economic Review, “China’s Democratic Future,” mss)

More ominous as a piece of "last ditchism" would be an attack on Taiwan. U.S. officials and many overseas democrats believe
that there is a significant chance of an attack on Taiwan if the CCP is embattled at home. Indeed, China's strategic journals make
frequent reference to this contingency: "The need for military preparations against Taiwan is all the more pressing in light of
China's growing social tensions and unstable factors which some people, including the U.S. might take advantage of under the
flag of 'humanism' to paralyze the Chinese government," one wrote. Such a move would allow the government to impose
martial law on the country as part of war preparations, making the crushing of protest easier. It would also offer the possibility,
if successful, of CCP survival through enhanced nationalist legitimacy. Yet the
risks, even to a dying regime, may
be too high. An unprovoked attack on Taiwan would almost certainly bring the U.S. and its
allies to the island's rescue. Those forces would not stop at Taiwan but might march on
Beijing and oust the CCP, or attempt to do so through stiff sanctions, calling it a threat to regional and
world peace. Such an attack might also face the opposition of the peoples of Fujian, who would
be expected to provide logistical support and possibly bear the worst burdens of war. They, like much of
coastal China, look to Taiwan for investment and culture and have a close affinity with the
island. As a result, there are doubts about whether such a plan could be put into action . A failed
war would prompt a Taiwan declaration of independence and a further backlash against the
CCP at home, just as the May Fourth students of 1919 berated the Republican government for weakness in the face of foreign
powers. Failed wars brought down authoritarian regimes in Greece and Portugal in 1974 and in
Argentina in 1983. Even if CCP leaders wanted war, it is unlikely that the PLA would oblige. Top
officers would see the disastrous implications of attacking Taiwan. Military caution would also
guard against the even wilder scenario of the use of nuclear weapons against Japan or the U.
S. At the height of the Tiananmen protests it appears there was consideration given to the use of nuclear weapons in case the
battle to suppress the protestors drew in outside Countries .41 But even then, the threats did not appear to gain even minimal
support. In
an atmosphere in which the military is thinking about its future, the resort to
nuclear confrontation would not make sense.

No CPP collapse

Time 8 [SIMON ELEGANT / BEIJING Thursday, Dec. 25, 2008 Will the Financial Crisis Bring Upheaval to China?
www.time.com/time/world/article/0%2C8599%2C1868402%2C00.html]
Just as the political elite is united, the forces that would have to oppose them in any move to
change the country's political order are fragmented, says David Zweig, a political science
professor at Hong Kong's University of Science and Technology. Though it is miserable for those thrown out
of work, millions of peasants going back to their villages are highly unlikely to pose a threat to Beijing . "Remember, Beijing
has done this before: between 1998 and 2000, the government put tens of millions of workers
at state-owned enterprises out of work. There were plenty of strikes and protests that made the
government a little nervous, but overall, they were able to survive pretty well." Currently, the official
figure is 4 million unemployed; but other estimates have the number at twice or three times that. (See pictures here of China's dust
bowl.)"For regimes to be overthrown you need an overriding ideolog y like democracy or the
mysticism of the Taiping and Boxer rebellions in the mid 1800s and early 20th Century," Says
Zweig. "For regimes to collapse now you also need the middle class, and I just can't see that
happening. They have been the core of Communist Party support for a decade or more and
their future is still very much tied up with the Party's.”
---CCP Collapse- Resilient

No CCP collapse—the government represses instability

Pei ‘9 (Minxin, Senior Associate in the China Program at the Carnegie Endowment for International Peace, 3/12. “Will the Chinese
Communist Party Survive the Crisis?” Foreign Affairs. http://www.foreignaffairs.com/articles/64862/minxin-pei/will-the-chinese-
communist-party-survive-the-crisis)

It might seem reasonable to expect that challenges from the disaffected urban middle class, frustrated college graduates, and
unemployed migrants will constitute the principal threat to the party's rule. If those groups were in fact to band together in a
powerful coalition, then the world's longest-ruling party would indeed be in deep trouble. But that is not going to happen. Such a
revolutionary scenario overlooks two critical forces blocking political change in China and similar
authoritarian political systems: the regime's capacity for repression and the unity among the elite .
Economic crisis and social unrest may make it tougher for the CCP to govern, but they will not
loosen the party's hold on power. A glance at countries such as Zimbabwe, North Korea, Cuba,
and Burma shows that a relatively unified elite in control of the military and police can cling to
power through brutal force, even in the face of abysmal economic failure . Disunity within the ruling elite,
on the other hand, weakens the regime's repressive capacity and usually spells the rulers' doom. The CCP has already
demonstrated its remarkable ability to contain and suppress chronic social protest and small-
scale dissident movements. The regime maintains the People's Armed Police , a well-trained and well-
equipped anti-riot force of 250,000. In addition, China's secret police are among the most capable in the
world and are augmented by a vast network of informers. And although the Internet may have made control of information more
difficult, Chinese censors can still react quickly and thoroughly to end the dissemination of dangerous
news. Since the Tiananmen crackdown, the Chinese government has greatly refined its repressive capabilities. Responding
to tens of thousands of riots each year has made Chinese law enforcement the most
experienced in the world at crowd control and dispersion . Chinese state security services have applied the
tactic of "political decapitation" to great effect, quickly arresting protest leaders and leaving their followers disorganized,
demoralized, and impotent. If worsening economic conditions lead to a potentially explosive political situation, the party will stick to
these tried-and-true practices to ward off any organized movement against the regime.

No regime change in China

Sautman ‘5 (Barry, Associate Prof. Social Science @ Hong Kong U. of Science, Asian Affairs: An American Review, “China’s
Strategic Vulnerability to Minority Separatism in Tibet” 32:2)

When a breakup of the Soviet Union first seemed likely, a leading American journalist wrote, “If China ever crumbles, Tibet is likely
to be the first piece to break off.”1 Talk of a “China collapse” continues : George Friedman, founder of the U.S. strategic
forecasters Stratfor.com, said in 2000 that “China would break up in the next two to three years, not unlike the Soviet Union, but
more violently, and would revert to a group of competing local warlords.” 2 Political scientist Ross Terrill writes of a “climactic
moment still to come, when a Chinese Boris Yeltsin shouts a few truths and the machinery falls apart.”3 Few
predictions by
scholars have focused on ethnic minority separatism as a major cause of a breakup , however, and a
bestselling book on China’s “coming collapse” devotes only a few anecdotal pages to Tibet.4 Yet the idea of China as vulnerable
through Tibet has a purchase on the popular Western imagination, because the Chinese Communist Party (CCP) system is thought to
be inherently self-destructive, while spiritually based Tibetan separatism is seen as inherently virtuous. This paper examines China’s
levels of vulnerability to separatism in Tibet. It argues that most
pronouncements on the question have focused
on efforts to mobilize support for separation as opposed to how events are likely to unfold.
Separatism in Tibet is unlikely to contribute to regime change in China , which would not in any case result in
a “free” Tibet,5 nor is it apt to foster disintegration. As Yang Dali and Wei Houkai observe, “China will
disintegrate only if the central government itself falls apart”6; that is, becomes so weak it
cannot prevent local elites from detaching both minority and key Han Chinese areas . That
scenario is unlikely: even China’s warlords, during and after World War I, wanted their regions to
remain part of the country and proclaimed provinces independent only for negotiating
purposes.7 Disintegration might occur if China lost a major war, had a sustained depression, or was wracked by violent factional
struggle leading to massive elite disaffection, popular mobilization, and the conviction that no acceptable central regime can be
formed.Absent sustained organizing by a national political force, however, the current
combination of gross inequality, mismanagement, and corruption is unlikely to lead to a national
breakup or even to regime change in China. Many countries subsist with worse problems, and
many survive despite being weaker states than China . Moreover, protests in China are localized and
not directed against national elites, who are instead often cast as potential allies against lower-level officials and
bosses.8 The already low-order vulnerability to separatism in Tibet will probably diminish further, due mainly to separatism’s
identification with the Dalai Lama, whose political vitality will likely ebb as he ages. This paper points out measures Beijing might
adopt to reduce vulnerability to a “nuisance, not a threat,” while benefiting Tibetans and building the confidence needed to bring
about negotiations with the Tibetan émigrés.

The CCP will not collapse

Epstein 10 (Gady, Beijing Bureau Chief for Forbes, “The Chinese Communist Party Is Like God,” Forbes, 5-21,
http://www.forbes.com/2010/05/21/china-communist-party-god-book-opinions-contributors-beijing-dispatch_2.html,

BEIJING -- A common parlor game in the China-watching crowd used to be debating how long the
Communist Party could hold on to power. A number of bets on the party's demise were made a decade ago
that can't be called until they expire in 2020 or 2030, but today those bets look silly. Few would bet against
the party now without getting lottery-like odds. How did that come to be? How has China's ruling party
become a seemingly durable authoritarian regime? Why, two decades after a period of
uncertainty and upheaval, do the people seem satisfied enough to acquiesce to being ruled by the
party? A fascinating and ambitious nonfiction book helps answer the how, and a clever Chinese dystopian novel seeks to answer
the why. Together they pull back the curtain on one of the most intriguing characters of our time, the Communist Party of
China. In his revealing new book The Party, author Richard McGregor, who covered China for London's Financial Times, lays
bare the secretive machinery of the
party, how it operates far more pervasively in public life and
commerce than many suspect. A powerful symbol of this hidden reach of the party is the secure internal network
of "red machines," phones that sit on the desks of some 300 of the nation's most elite officials. Even more interesting is on
whose desks you'll find those phones: not only members of the Politburo, but also the chief executives of 50 of the nation's
biggest companies. Paranoid conspiracy theorists on China will have plenty to feast on in McGregor's accounts of the failed
deals of Chinalco and Unocal. (Though you're not really paranoid if Beijing really is calling the shots for their big resources
companies.) "The Party is like God," a professor from People's University in Beijing tells McGregor. "He is
everywhere. You just can't see him." (Read an essay adapted from The Partyhere.) The Party is not simply an account
of how the party succeeds in ruling through its mechanisms of autocracy . The party's Achilles' heel--its
lack of any independent check on its power--undermines at every turn its efforts to police corruption, vet its members, reform
its bureaucracy and respond to crises. The maneuvering required to conduct a high-level corruption investigation sounds like it
is out of a mafia movie. Taking down a Politburo member, former Shanghai Party Secretary Chen Liangyu, required a side deal to
keep hands off of the family of former General Secretary Jiang Zemin, whose consent for the takedown was required because he
was the reigning kingpin of the Shanghai faction, despite the fact that he no longer held any official leadership posts. The
party's apparatus of control dominated every stage of decision-making in the disastrous Sanlu milk powder scandal, from
covering up melamine contamination that poisoned thousands of babies to censoring media coverage that could have saved
lives to blocking legal action that could have given families some measure of justice and deterred future corporate misbehavior.
At every stage where some check or balance might exist in a democratic system, the one-party system failed its people. But, as
McGregor notes, the
party does succeed at its main task, protecting itself. In exchange for
acquiescence to autocratic rule, the party delivers on the economy and largely stays out of
people's private lives. And as Chan Koon Chung writes in his novel The Fat Years, the people indeed
acquiesce.

CCP is invincible and escalation is empirically denied

Cody 5 (Edward, WP Foreign Service, “China Grows More Wary Over Rash Of Protests: Cell Phones, Internet Spread The Word,
Magnify Fallout”, Washington Post, 8-10, http://www.washingtonpost.com/wp-
dyn/content/article/2005/08/09/AR2005080901323.html)

Public Security Minister Zhou Yongkang said last month that the number of what he called "mass incidents" was
rising fast across China, according to an official who heard Zhou speak at a closed meeting. Zhou said that 3.76 million
Chinese took part in 74,000 such protests last year, which he characterized as a dramatic increase. Perhaps more
worrisome, Zhou continued, is a "noticeable" trend toward organized unrest, rather than the spontaneous
outbursts that traditionally have led to violent clashes between citizens and police. The minister added, however, that most
protests erupt over specific economic issues rather than political demands, suggesting they
are not coordinated or directed at bringing down the one-party system that has been in place in
China since 1949. Rural protesters have recently cited farmland seizures by local governments working with developers, or
pollution of fields and irrigation sources by locally licensed factories or mines as the reasons for their uprisings. Other protests
have erupted over clashes between factory managers and the millions of youths who leave their villages to work in assembly
plants in big city suburbs. Provincial, municipal and county governments have often proven unable to handle these complaints
because local officials, eager for economic growth in partnership with businessmen, regard the aggrieved people as obstacles to
success. Kang Xiaoguang, a Tsinghua University professor and political specialist at the Chinese Academy of Sciences, noted that
the protesters' lack of national coordination or broad political goals is an indication the government can probably bring the riots
under control. Hu and Wen, he said, regard the unrest as a major problem but inevitable, the fruit of economic disparities
caused by reforms over the past 25 years. As a result, Kang added, they want to rein in the poorly regulated capitalism that, in
many respects, has replaced socialism and have gone out of their way to demonstrate concern for the underdogs in China's
hybrid system. As
far as is known, even the most violent protesters have been armed only with
farming tools in the spate of unrest over the last several years. Similarly, police responding to riots
have generally been equipped only with clubs, staffs and tear gas. There have been no
reports of firearms being used.

No desire to overthrow the CCP - it’s resilient

Tanner 4 (Murray Scott, Senior Political Scientist – RAND Corporation, “China Rethinks Unrest”, Washington Quarterly, 27(3),
Summer, Lexis)

For those concerned about China's internal stability, the raw numbers of protests are less
important than an escalation in their size, level of organization, severity of demands, or degree of violence. The
CCP's remarkable capacity thus far to avoid the fate of its late Leninist brethren in Europe
owes much to the party's skill not only in preventing large-scale, well-organized protests with broad
anti-regime demands but also in suppressing organized opposition or civil society groups that can mobilize
such protests. Having absorbed the brutal lessons of the Tiananmen demonstrations, Chinese
protestors throughout the mid- to late 1990s self-consciously restrained their actions. Most disgruntled citizens
declined to establish permanent underground organizations that might have threatened the party. Their protests rarely
included more than a few dozen people, usually from the same work unit or village. Protest tactics
remained scrupulously peaceful, and demands focused on concrete local issues rather than
broad systemic changes. Indeed, to avoid official wrath, many petitioners took pains to reaffirm their support for the
party's central leadership, claiming that they only wanted local officials to obey Beijing's laws. Political scientist Kevin O'Brien
has thoughtfully dubbed this style of protest "rightful resistance," pointing out that it not only presented little threat to CCP
authority but also offered Beijing the chance to portray itself as a savior for citizens plagued by lawless, predatory, local party
officials. n6

Chinese government is stable – many factors check opposition

Nathan 2k (Andrew, Professor of Government – Columbia University, China and Democracy, Ed. Diamond, p. 25-26)

Many believe that a military coup or a recurrence of regional militarism is a possibility today. My
reading of the tea leaves is different. First, China lacks a Latin American—type tradition of military rule
or “guaranteeism” that would make a coup legitimate in the eyes of the people, the civilian leaders and
bureaucrats, or the military itself. Second, Chinese military people appear reluctant to take
responsibility for solving China’s political and economic problems since they do not believe they have solutions
for these problems. Third, the enormous size of the Chinese officer’s corps would make it difficult
to coordinate a coup without leaks and intramilitary opposition . A military coup would benefit some
commanders—probably certain department heads in the Central Military Affairs Commission, and the Beijing garrison
commander and Beijing military region commander—more than others, creating jealousy among those left out, those who
might even be more senior and command larger forces than the members of the coup coalition. Fourth, one of the factors
historically facilitating regional militarism was the existence of foreign spheres of interest. Contrary to popular images, the
Western powers were not opposed to democratic institutions in republican China, did not rake direct steps to frustrate them,
and gave hut slight support to the warlords. In fact, the foreign powers offered consistent diplomatic recognition to China’s
successive central governments and provided important financial support through loans, and customs and salt revenues. Still,
the foreign presence helped warlords, even if only marginally, through arms sales, by providing the occasional haven ol the
foreign concessions, and with whatever it contributed to the weakness of the central government. This historical element, of
course, is absent today. Finally, the most important factor working against a coup is that the [PLA]
People’s liberation Army already exercises strong influence on central party politics through its direct
representatives among the Elders and in the Politburo. In the past, the military’s interests were represented by leaders like Mao
Zedong, Un Biao, and Ye Jianying. whose roots were as much in the military as in the party. In
this way, the military
was able to have its say without directly taking power , as in the arrest of the Gang of Four and the fall of
Hu Yaobang and 7.hao Ziyang. This tradition is carried on today by Deng Xiaoping and Yang Shangkun. After the death of Deng
and Yang. the military will no longer have senior leaders whose careers have followed a dual military and civilian track. who can
wear two hats as central parry leaders and representatives of military interests. Yet instead of directly taking power. future
military and parry leaders will probably agree on some senior officers who can formally or informally enter the highest levels of
government to represent military interests. What are those interests? Given its national security mission and the increasing
professionalization of the military, the
officer corps appears to give priority to political stability ,
economic development, and the technical upgrading of the economy. The officers probably disagree about how to
achieve these goals, but younger officers seem to favor following reform wherever it leads, even if it involves abandoning
traditional ideas of socialism. If democratization could promote stability and reform, they would have no reason to oppose it.
---CCP Collapse- AT: Protests

Protests aren’t coordinated - our evidence assumes their scenario

Pan 5 (Esther, Staff – Council on Foreign Relations, “China’s Angry Peasants”, 12-15, http://www.cfr.org/publication/9425/)

The central government's greatest fear is that a charismatic leader will arise and unify the protesters. If unified, the rural
unhappiness over corruption, overdevelopment, and income inequality—combined with urban worker and student grievances
—could be a tremendously powerful social force. But experts
say the revolution is not quite here yet. "It's
clear the protests are becoming bigger and there are more of them, but there's no risk of them
overthrowing the Communist Party because there's no coordination between them ," Segal
says. He says any group that attempted to link the protests or reach across regional lines would be dealt
with very severely by the authorities .

The CCP will crush any protests

Pei 8 (Minxin, Fellow – Carnegie Foundation, “How is China Ruled”, April, http://www.carnegieendowment.org/files/Pei1.pdf)

At the same time, theCCP has adopted a more refined and subtle approach to the use of the state’s
repressive apparatus in defending its political monopoly. The Chinese government now permits
an unprecedented degree of personal freedom for the majority, but it targets opponents more efficiently and
effectively. Whenever possible, well-known dissidents are “encouraged” to go into exile abroad instead of languishing in jail
and becoming annoying symbols of human rights abuse. (Less wellknown dissidents are not so lucky.) Enormous
resources have been invested in the manpower and technology required to maintain effective surveillance
of groups and individuals suspected of anti-government inclinations. The Chinese Internet police unit, allegedly 30,000-strong, is
an illuminating example of this strategy. Selective press censorship ensures control of political information
without suffocating coverage of pop culture and business. In addition, new riot-police forces are now equipped
to suppress the tens of thousands of riots that erupt throughout China each year. Believing that the
Tiananmen crisis could have been averted had the government acted immediately and decisively, the Party now emphasizes
rapid response to incipient signs of crisis: Authorities at all levels are to suppress all sudden, potentially
destabilizing incidents. Local officials who fail to perform satisfactorily in handling such events risk dismissal. Of course,
the Party would not hesitate to use overwhelming force to crush any organized challenge
to its authority, as it did in rounding up the members of the tiny China Democracy Party in 1998 and
in banning the Falun Gong in 1999.
China Relations Answers
Frontline

No cooperation and no conflict—structural issues overwhelm

Harding ’11 (Harry Harding 11, founding dean of the School of Leadership and Public Policy at the University of Virginia, “Are
China and the U.S. on a collision course?”, June 14, http://thinkingaboutasia.blogspot.com/2011/06/are-china-and-us-on-collision-
course.html

In my judgment, it is highly unlikely for the relationship between the US and China to be primarily
cooperative, at least in the short to medium term. The differences in values, political systems, interests,
levels of development, and perceptions of the existing international order are simply too great for the two
countries to find common ground on all issues, or even to find a mutually agreeable allocation of costs and
benefits when they try to pursue common interests. Only a common interest that was massively compelling –
say a widespread pandemic, another financial crisis, a global outbreak of terrorist activity targeted at both countries, or increasingly
severe consequences of climate change – might produce a predominantly cooperative relationship. Fortunately,
an essentially
confrontational relationship is also unlikely, especially if one is primarily concerned with the risks of military conflict.
The high degree of economic interdependence between the two countries has already created a relatively
resilient relationship. The cost of military conflict, especially given the fact that both China and the US are nuclear
powers, will be a significant deterrent against military conflict. Equally important, the probability of the most worrying of
the trigger events identified above– a unilateral declaration of independence by Taiwan – is presently quite low, as is the risk that
China would try to compel unification through the use of force.

Too many alt causes to relations

Stokes and Hatchigian, 12 [U.S.-China Relations in an Election Year Taking the Long View in a Season of Heated Rhetoric,
Jacob, Research Assistant at the Center for a New American Security (CNAS), where his research focuses on U.S. national security
and defense policy. His writing has appeared in CNN.com, Politico, BusinessWeek, The Baltimore Sun, The Guardian and The
American Prospect, among other publications, Senior Fellow at American Progress.¶
http://webcache.googleusercontent.com/search?
q=cache:QG6048mP53AJ:www.americanprogressaction.org/issues/2012/03/pdf/us_china_relations.pdf+&hl=en&gl=us]

This report examines the 10 most debated challenges in the U.S.-China relation-¶ ship in the 2012 presidential and congressional
campaign season, exploring¶ differences between progressive and conservative approaches to China. We¶ detail these 10 issues in

the pages that follow, but briefly, here is a summation of the top challenges and the different approaches

advocated by conservatives and¶ taken by progressives.¶ • Ensuring fair trade. The Obama administration’s policy of vigorous
enforce-¶ ment and results-oriented dialogue beats conservatives’ refusal to invest in¶ American competitiveness at home; empty,
antagonistic rhetoric toward China;¶ and highly inconsistent positions on trade cases. The Obama administration has ¶ announced a
new trade-enforcement unit and has brought more major trade¶ cases against China than any of its predecessors.¶ • Progress
on currency. The Obama administration’s efforts, on its own and with¶ other nations, to pressure China to deal with its
undervalued currency have¶ resulted in progress, though more remains to be done. The administration is keeping the
pressure on. The conservative answer is both needlessly antago-¶ nistic and ineffective.¶ • China owning U.S. debt.
China owning just more than 8 percent of our federal¶ debt is not leverage China can use without unacceptably harming its own
interests.¶ Conservative hysterics and fearmongering about this complex issue is misplaced. ¶ • Chinese direct investment .
Chinese investment in our country can be a major¶ source of capital and jobs going forward. We should allow proven national ¶ 4
Center for American Progress Action Fund | U.S.-China Relations in an Election Year¶ security processes to weed out threats to our
nation and avoid excessive¶ paranoia around Chinese purchases, lest we miss investment-led growth¶ opportunities. Conservatives
should take heed.¶ • Championing human rights. The Obama administration has consistently called China
out on human rights , speaking privately and publicly with Chinese¶ leaders, meeting with the Dalai Lama twice, and giving
our diplomats new¶ forums to engage fully with their Chinese counterparts and the Chinese peo- ¶ ple to improve human rights and
religious freedoms in China. Conservatives’¶ only answer is even more forceful browbeating of Chinese leaders—emotion- ¶ ally
satisfying, but not an effective tactic to make real change.¶ • America the Pacific power. Under the Obama administration new trade
part-¶ nerships, defense arrangements, and serious connections with regional orga-¶ nizations all support deeper U.S. engagement
in Asia. Extremist conservative¶ rhetoric claiming the administration is not investing adequately in defense in¶ Asia is nonsense.¶ •
Addressing China’s military. China’s military has grown rapidly in recent years,¶ albeit from a very low base. While some
technologies are worrisome, the¶ United States retains a huge advantage over China. The Obama administration ¶ is responding to
China’s military buildup but is not exaggerating the threat, in¶ contrast to conservative efforts to use the “China threat” to justify
unsustain-¶ able increases in military spending.¶ • Supporting regional allies. Asian nations continue to turn to America to ensure¶
peace and security. The United States is meeting that need by strengthening rela- ¶ tions with our Pacific friends and allies.
Relationships with Japan, South Korea,¶ and Australia are rock-solid, and the United States joined with regional players ¶ to push
back on Chinese belligerence. Conservatives ignore this track record in¶ desperate attempts to tag the Obama administration as
abandoning our allies.¶ • A
friend to Taiwan. The Obama administration has sold unprecedentedly large¶
packages of arms to Taiwan , including major fighter upgrades, while also upping¶ outreach to the island in ways that will
not destabilize cross-Strait relations.¶ Conservatives are left complaining that the current administration, like the Bush ¶
administration before it, did not sell Taiwan the most advanced jet fighters.¶ The Obama¶ administration¶ is responding to¶ China’s
military¶ buildup but is¶ not exaggerating¶ the threat, in¶ contrast to¶ conservative efforts¶ to use the “China¶ threat” to justify¶
unsustainable¶ increases in military¶ spending.¶ 5 Center for American Progress Action Fund | U.S.-China Relations in an Election
Year¶ • Tackling cybersecurity. From the start the Obama administration has identi- ¶ fied cybersecurity as an issue of grave concern
and mounted a comprehensive¶ response. Conservatives who condemn the administration’s response do not¶ understand its scope;
they also offer little in the way of new ideas for combat-¶ ing the threat.¶ In the pages that follow, we will present in more detail
these 10 challenges along-¶ side the response of the Obama administration and the misplaced criticisms and¶ hostile rhetoric of
many conservatives.

Cooperation is impossible

Xinbo, 12 [Wu Xinbo is Professor at the Center for American Studies, Fudan University, Forging Sino–US Partnership in the
Twenty-First Century: opportunities and challenges, Journal of Contemporary China, p. UM libraries, preview available at
http://www.tandfonline.com/doi/abs/10.1080/10670564.2011.647429#preview]

Whether China and the US can forge a genuine partnership depends on their capability to
overcome bilateral differences and expand their cooperation in areas of common interests, while
the latter will be even more essential given the fact that the Sino–US relationship is still growing. However, common interests
do not necessarily guarantee common actions. In fact, efforts to forge a Sino–US partnership are
confronted with a series of challenges arising from both sides.¶ On the US side, one problem lies in the
lack of experience in working with a rising power like China . Since moving to the center of the world stage
following World War II, the United States has accumulated experience in dealing with rising powers like the Soviet Union, Japan and
Germany. While the containment strategy proved successful in coping with Soviet expansion during the Cold War, the alliance
strategy worked well to secure Japanese and German acceptance of US leadership when both countries reemerged as major
economic powers in the 1960s. China, however, is different from those rising countries. Unlike the Soviet Union, it is not pursuing an
antagonistic relationship with the US, nor is it, like Japan and Germany, following US leadership in international affairs as a small
brother. For Washington, Beijing is neither a complete enemy nor a sheer friend. Both competitive and cooperative
dimensions exist in Sino–US relations. The competitive factors may not lead to strategic confrontation if well managed,
yet confrontation may happen if not well managed. Meanwhile, the cooperative factors may not automatically
lead to cooperation, as it requires such serious efforts as hard bargaining, skillful trade-offs and the demonstration of a spirit
of respect and equality. Given its cultural and historical background, the United States lacks such sophistication to deal with a
country like China. Although the US has learned a lot since the mid-1990s about how to deal with a rising China, it still has a long way
to go to enrich experiences, improve skills and adjust mentality.¶ Another outstanding challenge originating on
the US side is the constraint of its domestic politics. While US democracy may arguably provide
a good example for internal good governance, its foreign policy lacks continuity and credibility due
to political cycles coming out of election politics and the interplay of interest group politics.28
China policy in particular has fallen victim to internal politics from time to time. While Chinese leaders
always emphasize the need to adopt strategic and long-term perspective on bilateral relations, US leaders, driven by
political cycles, invariably pay more attention to tactical and short-term gains in interactions
with China. It is true that China's domestic politics also increasingly works to affect its handling of
relations with the US, but such impact is largely manageable and has caused much less volatility than US domestic politics
does to bilateral ties. It is the volatility in US China policy that frustrates Chinese efforts and desires
to secure a steady development of relations with the US. It also undermines endeavors to build mutual trust
between the leaderships in both countries. ¶ On the Chinese side, a series of political, economic and
security factors constrain its capability to extend the cooperation that the US expects. Politically,
China appears more sympathetic with some authoritarian regimes that the US may find less tolerable, and Beijing may resist
Washington's efforts to exert pressure on them through the United Nations. As a result, China is often accused of protecting those
‘rogue’ or ‘repressive’ regimes. Economically, although China now ranks as the second largest world economy, it is still a developing
country in terms of per capital GDP and overall level of social–economic development, hence China refuses to shoulder international
responsibilities that it views beyond its capacity, and the US may perceive China as unwilling to live up to its major power status. On
the security front, given the differences in respective geopolitical interests between China and the US in the Asia–Pacific,
Beijing's approach to some regional issues, such as the Korean peninsula issue, differs from that of the US. Such differences
highlight bilateral competition rather than cooperation in the region.¶ Mutual trust holds the
key to partnership-building. However, the lack of mutual trust is an outstanding feature of current
Sino–US relations. This should be attributed not only to the real differences in respective national interests, but also to
misperceptions that each possesses toward the other. A primary US misperception is that China aspires to undermine its position in
the Asia–Pacific. China, on the other hand, always suspects that the US intends to contain it. Both sides are
aware of the other's major concerns and try to assure each other. For instance, in both joint statements of 2009 and 2011, the
United States reiterated that ‘it welcomes a strong, prosperous, and successful China that plays a greater role in world affairs’, while
China suggested that it ‘welcomes the United States as an Asia–Pacific nation that contributes to peace, stability and prosperity in
the region’. In spite of these assurances , however, those misperceptions remain strong, and both sides
continue to try to find supporting evidence from the other's words and deeds.¶ Finally, some conceptual gaps between two
countries also complicate their efforts to forge partnership in world affairs. What is China's international identity and responsibility?
How to deal with the issue of sovereignty in the era of globalization and information? How strictly should the principle of non-
interference in a sovereign country's internal affairs be abided by? How should foreign aid be best provided? What should a
preferred international order looks like? And so on. Such differences
will affect both the objectives the two
countries seek to advance and the means they employ.

Relations are resilient and both sides know their limitations

Bader ’11 (Jeffrey A. Bader 11, visiting scholar at the China Center at Brookings, “U.S.-China Senior Dialogue: Maintaining the
Balance”, May 6, http://www.brookings.edu/opinions/2011/0506_strategic_economic_dialogue_bader.aspx

The S&ED comes at a time when U.S.-China relations are in fundamentally sound condition. President Hu
Jintao’s visit to the United States was generally assessed as setting a realistic tone and achieving successes
in a relationship that will always be marked by frictions. President Obama, who will be involved in the S&ED,
has put a high priority on U.S.-China relations, and the two sides have cooperated, within limits, on major security
issues, including Iran, Korea, Sudan, Libya, and nuclear security. From the U.S. perspective, it will certainly not hurt that the
meeting comes only a week after the successful raid that eliminated Osama bin Laden, which sends a message of U.S. strength and
credibility in a relationship where those qualities are always the subject of Chinese scrutiny. The United States and China
have developed reasonable expectations about both the possibilities and limits of cooperation,
which will reduce the chances of future miscalculation. All of these subjects, plus broader developments in the
Middle East, will be on the agenda of the S&ED.
---China Rels- Alt Causes

1NC Stokes and Hatchigian—too many alt causes to relations—trade, currency, debt,
direct investment, human rights, military exercises, and Taiwan all thump the link—
prefer it, most recent ev and from China research specialist

Means they’re in a double bind, either issue don’t spill over and plan can’t solve or they do
and plan is overwhelmed

Alt causes institutionalize and inevitable—undermine trust

Lawrence & Lum 11 – Beijing Bureau Chief for the Far Eastern Economic Review & specialist in Asian Affairs at the
Congressional Research Service (Susan V. & Thomas, "US-China Relations: Policy Issues" Congressional Research Service, January 12,
http://www.dtic.mil/cgi-bin/GetTRDoc?AD=ADA535780&Location=U2&doc=GetTRDoc.pdf)

The U.S.-China relationship remains dogged, however, by long-standing mutual mistrust. That
mistrust stems in part from the two countries’ very different political systems. Many in the United
States are uncomfortable with China’s authoritarian system of government and see continued Communist Party rule in a post-Cold
War world as an anachronism. Many Communist Party elites in China are suspicious that the United
States seeks to constrain China’s rise , and in the longer-term, to foist multi-party democracy on China and
push the Communist Party from power. The two countries’ different economic models have led to mistrust,
too. Some in the United States believe that China has achieved its economic successes by
playing by a different, and not always fair, set of rules. Such critics point to China’s alleged strong reliance on
exports for growth and the PRC government’s policy of keeping China’s currency artificially weak, in part to make Chinese exports
more attractive to importing nations. Other points of contention include the PRC government’s direct and indirect subsidies and
other forms of support for its state-owned corporations, and its inability or unwillingness to prevent violations of foreign intellectual
property by Chinese entities. For their part, PRC officials have sometimes criticized the United States for its high levels of
consumption, low savings rate, and long-term debt. Chinese officials have also criticized the United States’ allegedly loose monetary
policy. Mistrustis particularly pronounced on security matters. The United States increasingly sees
China’s military modernization as aimed at constraining the U.S. military’s freedom of
movement in Asia and deterring any U.S. intervention in the case of a Chinese use of force against Taiwan.1 In mirror image,
China sees the United States as intent on thwarting its unification with Taiwan and constraining
the activities of its own military throughout Asia . Although China stated in a 2009 U.S.-China Joint Statement that
it “welcomes the United States as an Asia-Pacific nation that contributes to peace, stability and prosperity in the region” (see “The
2009 U.S.-China Joint Statement,” below), many in China chafe at the surveillance activities undertaken by the U.S. military along
China’s coast and at U.S. military exercises in waters near China, and regard U.S. military alliances in Asia as aimed to a significant
degree at China. With U.S. troops deployed in Afghanistan, which shares a short border with China to the west, and with U.S.
military and intelligence agencies deeply engaged in Pakistan, also a neighbor to the west, some Chinese commentators speak of
their country being “encircled” by U.S. forces.2 The
bilateral relationship was strained in 2010 by long-
standing issues of disagreement between the United States and China such as Taiwan and Tibet
and China’s currency policy, as well as by each side’s responses to unexpected new crises in Asia .
China condemned the Obama Administration’s January 2010 announcement of a long-planned
sale of a package of arms to Taiwan. It also strongly protested President Obama’s February 2010
meeting with the Dalai Lama, although in hosting the meeting, the President was following in the footsteps of his
predecessors in the White House. Among the unexpected crises that undermined U.S.-China relations was
a series of North Korean provocations, starting in March 2010 with North Korea’s apparent
sinking of a South Korean naval vessel. China incurred U.S. criticism for its efforts to shield North Korea from the most
serious diplomatic consequences for its actions, while the United States unsettled China when it responded to the North Korean
actions by scheduling military exercises in the Yellow Sea that China charged were meant in part as a threat to China. In
September 2010, the collision of a Chinese fishing trawler and Japanese Coast Guard vessels
near disputed islands in the East China Sea set off a new crisis. The United States angered China
by voicing its support for Japan in the ensuing China-Japan spat, and by clarifying that that the
U.S. military alliance with Japan covers the disputed island s. Also contributing to bilateral tensions in 2010
were China’s reassertion of expansive claims to territory in the South China Sea, and the United States’ subsequent July 2010
declaration of a “national interest” in freedom of navigation in the sea. The 2009 Joint Statement, issued during President Obama’s
November 2009 state visit to China, laid down some significant new statements of principle. In the document, the U.S. side declared
that the United States “welcomes a strong, prosperous and successful China that plays a greater role in world affairs,” while the
Chinese side stated that China “welcomes the United States as an Asia-Pacific nation that contributes to peace, stability and
prosperity in the region.” This was the first time China agreed to put in writing such a positive characterization of the U.S. presence
in Asia, although the wording of the Chinese-language version of the statement was more conditional than the English. While the
English text appeared to signal an acceptance that the United States presence contributes to peace, stability, and prosperity in the
region, the Chinese text stated that China welcomes U.S. “efforts” for peace, stability, and prosperity in the region, leaving open the
question of how China sees the U.S. presence as contributing to peace, stability, and prosperity currently.10 The
2009 Joint
Statement also included the controversial statement that, “respecting each other’s core
interests is extremely important to ensure steady progress in U.S.-China relations .”11 “Core
interests” was left undefined. Critics have suggested that the language may have raised
unrealistic expectations on the Chinese side of greater U.S. deference to China on issues that
China considers part of its core interests, such as Taiwan, Tibet, and the maintenance of
domestic stability through suppression of dissent .12 The “core interests” language in the Joint
Statement may also have encouraged China to experiment with referring to the South China Sea
as a “core national interest” in some closed-door meetings with foreign officials in 2010 . According
to Secretary of State Hillary Clinton, those statements contributed to the U.S. decision to declare a “national interest” in freedom of
navigation in the South China Sea at a meeting in Hanoi in July 2010.13 Finally, in the 2009 Joint Statement, the two sides stated
that, “they are committed to building a positive, cooperative and comprehensive U.S. China relationship for the 21st Century.”14
While this statement may seem formulaic, officials in both governments are careful to include it in almost every official statement
related to the relationship. Its repetition is considered to provide reassurance of each country’s commitment to the relationship. The
“positive, cooperative, and comprehensive” formulation marked an evolution from President George W. Bush’s description of the
relationship as “constructive, cooperative, and candid,”15 with “candid” upgraded to “positive” and “comprehensive” added to
reflect the broad range of issues on which the two countries expected to work together. The Obama-Hu language was a departure
from the language agreed to by President Bill Clinton and his counterpart, Chinese President Jiang Zemin, in a 1997 Joint Statement,
in which they pledged, “to build toward a constructive strategic partnership.”16
---China Rels- Don’t Solve

1NC Xinbo—their cooperation is impossible—institutional and political factors undermine


legitimate engagement—interest alignment is a far-cry from actual action—prefer it,
postdates and empirical/structural analysis from a Chinese professor of American studies

No impact to U.S.-China cooperation---it’s impossible to sustain

Friedberg ’12 (Aaron L. Friedberg, Professor of Politics and International Affairs at the Woodrow Wilson School of Public and
International Affairs at Princeton University, “Bucking Beijing,” Foreign Affairs, Vol. 91, No. 5, p. 48-58, September/October 2012)

Recent events have raised serious doubts about both elements of this strategy. Decades of trade and talk have not
hastened China's political liberalization. Indeed, the last few years have been marked by an intensified crackdown on domestic
dissent. At the same time, the much-touted economic relationship between the two Pacific powers has become a major source of
friction. And despite hopes for enhanced cooperation, Beijing has actually done very little to help
Washington solve pressing international problems, such as North Korea's acquisition of nuclear weapons or Iran's attempts to
develop them. Finally, far from accepting the status quo, China's leaders have become more forceful in attempting
to control the waters and resources off their country's coasts. As for balancing, the continued buildup of China's military capabilities,
coupled with impending cuts in U.S. defense spending, suggests that the regional distribution of power is set to shift sharply in
Beijing's favor. WHY WE CAN'T ALL JUST GET ALONG TODAY, CHINA'S ruling elites are both arrogant and insecure. In their view,
continued rule by the Chinese Communist Party (CCP) is essential to China's stability, prosperity, and prestige; it is also, not
coincidentally, vital to their own safety and comfort. Although they have largely accepted some form of capitalism in the economic
sphere, they remain committed to preserving their hold on political power. The CCP'S determination to
maintain control informs the regime's threat perceptions, goals, and policies. Anxious about their legitimacy, China's rulers are
eager to portray themselves as defenders of the national honor. Although they believe China is on track to become a world power
on par with the United States, they remain
deeply fearful of encirclement and ideological subversion . And
despite Washington's attempts to reassure them of its benign intentions, Chinese leaders are
convinced that the United States aims to block China's rise and, ultimately, undermine its one-party system of
government.¶ Like the United States, since the end of the Cold War, China has pursued an essentially constant approach toward its
greatest external challenger. For the most part, Beijing has sought to avoid outright confrontation with the United States while
pursuing economic growth and building up all the elements of its "comprehensive national power," a Chinese strategic concept that
encompasses military strength, technological prowess, and diplomatic influence. Even as they remain on the defensive, however,
Chinese officials have not been content to remain passive. They have sought incremental advances, slowly expanding China's sphere
of influence and strengthening its position in Asia while working quietly to erode that of the United States. Although they are careful
never to say so directly, they seek to have China displace the United States in the long run and to restore China to what they regard
as its rightful place as the preponderant regional power. Chinese strategists do not believe that they can achieve this objective
quickly or through a frontal assault. Instead, they seek to reassure their neighbors, relying on the attractive force of China's massive
economy to counter nascent balancing efforts against it. Following the advice of the ancient military strategist Sun-tzu, Beijing aims
to "win without fighting," gradually creating a situation in which overt resistance to its wishes will appear futile. The
failure to
date to achieve a genuine entente between the United States and China is the result not of a lack of effort but of a
fundamental divergence of interests. Although limited cooperation on specific issues might be possible, the
ideological gap between the two nations is simply too great, and the level of trust between them too low, to
permit a stable modus vivendi. What China's current leaders ultimately want -- regional hegemony -- is not something
their counterparts in Washington are willing to give. That would run counter to an axiomatic goal of U.S. grand strategy, which has
remained constant for decades: to prevent the domination of either end of the Eurasian landmass by one or more potentially hostile
powers. Thereasons for this goal involve a mix of strategic, economic, and ideological considerations
that will continue to be valid into the foreseeable future .

Relationship isn’t reverse causal—strong ties doesn’t solve

Blumenthal 11 (Dan, Resident fellow at AEI, Current commissioner and former vice chairman of the U.S.-China Economic and
Security Review Commission, where he directs efforts to monitor, investigate, and provide recommendations on the national
security implications of the economic relationship between the two countries. Previously, he was senior director for China, Taiwan,
and Mongolia in the Secretary of Defense's Office of International Security Affairs and practiced law in New York prior to his
government service. At AEI, in addition to his work on the national security implications of U.S.-Sino relations, he coordinates the
Tocqueville on China project, which examines the underlying civic culture of post-Mao China. Mr. Blumenthal also contributes to
AEI's Asian Outlook series and is a research associate with the National Asia Research Program. 10/3/2011, “The top ten unicorns of
China policy”, http://www.aei.org/article/foreign-and-defense-policy/regional/asia/the-top-ten-unicorns-of-china-policy/)

9) We need China's help to solve global problems. This is further down on my list because it is not really a
fantastical unicorn. It is true. What is a fantasy is that China will be helpful. We do need China to
disarm North Korea. They do not want to, and North Korea is now a nuclear power. The same may soon be
true with Iran. The best we can get in our diplomacy with China is to stop Beijing from being
less helpful. It is a fact that the global problems would be easier to manage with Chinese help.
However, China actually contributing to global order is a unicorn.

Domestic politics and blame gaming undermine co-op

Czarnezki, 11 [Jason J. Professor of Law in the Environmental Law Center and Faculty Director of the U.S.-China¶ Partnership
for Environmental Law at Vermont

Law School; A.B., J.D, “CLIMATE POLICY &¶ U.S.-CHINA RELATIONs”, Published After April 4th 2011.
http://www.vermontlaw.edu/Documents/Jason%20Czarnezki%20Climate%20Policy%20and%20China.pdf]

Both the United States and China are hindered by the reality of domestic politics and their ability
to blame the other for lack of progress. Professor¶ Cinnamon Carlarne, increasing future political pressure, described
the 2010¶ Cancun Climate Change Conference as “a determinative point for both a 2¶ degree world and the continuing validity of
the UNFCCC process,”44 but¶ COP-16 in Cancun has come and gone with little fanfare. The Cancun¶ process avoided the high-stakes
drama of Copenhagen, successfully set up a¶ fund for adaptation measures in poor countries, created a mechanism for ¶ technology
transfer, approved a deal to protect tropical forests, and ensured¶ adherence to the goals put forward in the Copenhagen Accord.45
IV. DOMESTIC POLITICS¶ The United States and Chinese governments have significant domestic¶ political
pressures that limit their ability and desire to come to a progressive¶ international agreement on
climate change, and these pressures create the¶ type of chaos and self-interested behavior seen at Copenhagen.¶ China
does
not want to limit its amazing and historic economic growth¶ and development. The domestic
justifications are sound and¶ understandable. Economic prosperity defines global power, many
Chinese¶ still need to be brought out of poverty, and economic success provides the ¶ necessary
stability for the ruling Communist party to stay in power . As a¶ result, China is happy to become far more
energy efficient, but will make¶ no emissions limitations promises that have the potential to limit overall ¶ economic growth.¶ To this
end, China has developed “carbon intensity” targets in an effort¶ to slow its greenhouse gas emissions and become more energy
efficient.¶ China proposes to reduce carbon intensity—the amount of CO2 emitted per¶ unit of economic output—by forty to forty-
five percent, compared with¶ 43. Agence France-Presse, China and U.S. Blame Each Other as Climate Talks Conclude, ¶ PORTFOLI
(Oct. 9, 2010, 7:39PM), http://portfo.li/o/255346-china-and-u-s-blame-each-other-as-climatetalks- ¶ conclude.¶ 44. Carlarne, supra
note 37, at 149.¶ 45. John M. Broder, Climate Talks End with Modest Deal on Emissions, N.Y. TIMES, Dec. 11,¶ 2010,
http://www.nytimes.com/2010/12/12/science/earth/12climate.html. ¶ 670 VERMONT JOURNAL OF ENVIRONMENTAL LAW [Vol. 12¶
2005.46 Unfortunately, under this plan, even though the rate of emissions ¶ will slow, overall emissions will continue to rise. This will
eventually rub¶ up against “The China Problem”—that even if other countries reduce¶ emissions to zero, China’s growth and
emissions alone, despite improving¶ energy intensity, have the potential to push global temperature above the¶ two degree Celsius
threshold goal, and potentially further.47¶ Similar to China, the
United States has domestic political and
economic¶ considerations that have created roadblocks for international climate¶ agreements
and domestic initiatives. These roadblocks include concerns ¶ about limiting economic growth, a
culture and infrastructure that support ¶ high levels of driving and energy consumption , strong
lobbying by energy¶ and automobile industries against greenhouse gas regulation, dismissal of ¶
climate science, and anti-internationalism among both politicians and ¶ citizens. As a result, the
U.S. government has not enacted a single law ¶ explicitly requiring any public or private entity to
mitigate its greenhouse¶ gas impact on the global climate.¶

Leverage is limited and other alliances solve


Economy 11 – C.V. Starr Senior Fellow and Director for Asia Studies at the Council on Foreign Relations (Elizabeth C., 01/14,
“Reality in U.S.-China Relations,” http://www.cfr.org/china/reality-us-china-relations/p23803)

Second, Washington needs to remember that its bilateral leverage is--and always has been--limited.
The United States' greatest leverage arises from working with its allies to engage (and
sometimes pressure) China. Success, whether on advancing climate change or rolling back unfair Chinese

trade and investment policies, has come when the U nited S tates finds common cause with others.
Finally, progress in the U.S.-China relationship ultimately depends on China as well. Chinese foreign policy elite debate all matter of
policy. Already in private conversations, Chinese analysts are suggesting that in the wake of China's 2010 policy travails, they are
more interested in seeking common ground with the United States. On the sensitive issue of how to handle North Korea, important
voices such as Fudan professor Shen Dingli and Beida professor Zhu Feng are proposing a rethink of China's policy. These domestic
voices are the real key to future effective U.S.-China cooperation. The
dream of a robust U.S.-China
partnership to lead the world through the thicket of ever-proliferating global challenges
remains. But for now, dreaming is no substitute for the hard work of negotiating reality.

Domestic political and economic realities means no co-op


Evans 11 – based at the Center on International Cooperation at New York University, where he heads CIC’s program on resource
scarcity, climate change and multilateralism. He has also published research on these issues with the World Bank, Chatham House
and the Brookings Institution, and works on them with the UN and a range of national governments. From 2003 to 2006, Alex was
Special Adviser to Hilary Benn, then the UK Secretary of State for International Development (Alex, 07/02, “What the ‘powershift’
narrative overlooks on US-China relations,” http://www.globaldashboard.org/2011/07/02/the-us-and-china-leaders-of-the-g-zero-
world/)

But reading Jamil Anderlini’s excellent FT comment piece today on the state of the Chinese Communist Party at its 90th birthday, I
was struck that the ubiquity of the powershift narrative obscures a more interesting story: that of how the
two countries’
situations are remarkably similar – and stand right in the way of making progress on the most important
global issues. Consider: Economically, both countries are caught between a rock and a hard place. Each is pursuing a
growth strategy that is essentially unsustainable: China remains reliant on its export sector, while
America’s [anaemic] growth is built on public and private debt. As they pursue these growth strategies, the two
countries continue to create vast global economic imbalances that create enormous risk for everyone else. Both also have
screwed financial sectors with massive bad debts, which their policymakers are studiously ignoring. This
economic mess in turn results in part from each country’s political sclerosis. Both countries have relied on the aforementioned
unsustainable growth strategies to secure political support and legitimacy. But both countries now face truly breathtaking gulfs
between their haves and have-nots that risk undermining their social contracts - a challenge amplified by weak social protection
provision in both cases. And both
have institutional and political systems that look increasingly moribund
and unable to cope with the complexity and intensity of the challenges confronting them.
---China Rels- Resilient

1NC Rosecrance—multiple strong ties mean no relations collapse—interdependency, prolif,


terror, climate, energy, and democracy—internal party shifts have locked in bilateralism—best
ev from doctorial China policy experts

No risk of collapse--multiple institutional reasons why relations are resilient


Dongxiao 12 – Vice President of Shanghai Institutes for International Studies (Chen, 01/05, “China-US Relations in 2012:
Caution Ahead,” http://chinausfocus.com/slider/no-reason-for-chagrin-over-china-us-relations-but-cautious-management-needed-
in-2012/)

The year of 2011 brought many unexpected, globally altering events. This year, non-stop crises and sea changes in the
international arena; chaos and revolution in the Middle East and West Africa; catastrophic Tsunami and nuclear-leak crisis
in Fukushima; paralysis of leadership of EU confronting the evolving debt predicament in Euro-Zone; and the sudden death of Kim
Jong-il and its unpredictable repercussions on the Korean Peninsula and Northeast Asia were just a few of the tumultuous events
that led global economic and political instability this year. Bilateral relations between China and the US, in contrast
have been relatively stable, and increasingly positive. Three driving forces have contributed to the improvement in
US-China relations in 2011: mutual commitment, multi-function mechanisms, and increasing interdependence. Beijing and
Washington both stressed their commitment to building a cooperative partnership based on mutual respect and mutual benefit
following a rocky year of bilateral relations in 2010. Both
sides have stressed that the relationship between China and
the United States should
be cooperative and mutually beneficial rather than zero-sum, and that the two
sides should stand together in the face of difficulty and carry out cooperation on an equal footing. The mutual
commitment between China and the US has been bolstered by an increasing number of bilateral mechanisms with policy
communication, coordination, and implementation functions (“C2I”). 2011 has seen of the growth of “C2I” mechanisms intensify.
with a number of new initiatives, including High-level Consultation on People-to-People Exchanges, the US-China Governors Forum,
and the Strategic Security Dialogue and Asia-Pacific Affairs Consultation under the framework of Strategic and Economic Dialogue
(S&ED). While the former two initiatives have either reflected thriving interaction in cross-cultural domains or tapped the huge
potential of sub-national cooperation across the Pacific, the latter two mechanisms have greatly upgraded capacity to address
difficult and sensitive military and security issues in bilateral relations n and build confidence in US-China relations. The 60
plus
bilateral mechanisms, plus frequent exchanges of informal visits and workshops between senior officials
have built an impressive level of institutionalization in US-China bilateral relations that has
enhanced the predictability of relations between the two countries and helped consolidate the foundation of the
relations. The substance of the bilateral relationship, in essence, is not to follow the two presidents’ agreements in words, but to
follow the roadmap in action, and those bilateral mechanisms have built significant capacity to do this. Thirdly and perhaps most
fundamentally, the growing interdependence across the Pacific and emerging agenda of global governance has served
as the “ballast” in the bilateral relationship. Despite numerous trade disputes between the two countries, economic
interdependence has been steadily enhanced, manifested either by the hike of bilateral trade and investment volume, symbiotic
financial relations, or the economic restructuring now underway in both countries. This interdependence has transcended
economics, and is growing increasingly comprehensive in nature.

History proves—elites willing to overcome much larger barriers than plan—no will to collapse
relations
Lavin 11 – Undersecretary for International Trade of the the United States Department of Commerce (Franklin, 06/28,
“Consequential China: U.S.–China Relations in a Time of Transition,”
http://www.heritage.org/research/reports/2011/06/consequential-china-us-china-relations-in-a-time-of-transition)

There are any number of differences, challenges, and even friction points in the bilateral relations, but I want
to underscore my optimism because the policy emphasis is such that it requires that most of our time be spent discussing the
problems or challenges. However, before I get to that, I want to talk a bit about what is working. For example, it is interesting to me
that both China and the U.S. have a national-interest-focused foreign policy. Neither country, I think, subscribes to a philosophy that
threatens the other. Neither country, as they say in China, tries to put sand in the other’s rice bowl. So I think there is a reasonably
positive functional relationship between the two countries. From a U.S. point of view, if we look over the modern era, since the
Nixon-to-China moment, we
have about four decades of relations, across eight presidents, both political
parties, a range of philosophies, different challenges, and different times. But, there is a high degree of
continuity in that relationship and I think there are two pillars that allow for that continuity. One is the pillar of engagement that,
regardless of the issue or the challenge, we were not going to break off or try to diminish relations but
always try to find a way to improve them. The second pillar is respect for China’s one-China policy, that we would not seek
to undermine that, although we certainly have interests vis à vis Taiwan. But we never tried to directly do something to diminish the
one-China policy.

No relations collapse—close ties on multiple issues

Rosecrance and Qingguo ‘10 – *political science professor at Cal and senior fellow at Harvard’s Belfer Center for Science
and International Affairs, former director of the Burkle Center for International Relations at UCLA, **PhD from Cornell, Professor and
Associate Dean of the School of International Studies of Peking University (Jia Qingguo and Richard Rosecrance, Global Asia, 4.4,
“Delicately Poised: Are China and the US Heading for Conflict?”, http://www.globalasia.org/l.php?c=e251, WEA)
Sustained Cooperation? 

The fact that the rise of China is unlikely to lead to armed conflict with the US does not necessarily mean that the two countries can
achieve a wholly cooperative relationship in the long term. For that to happen, the
two need to have shared interests,
aspirations, and mutually acceptable approaches to promoting their national goals. It appears that these conditions
are increasingly becoming a reality. To begin with, after years of interaction, China and the US have
developed a shared stake in cooperation. Their relationship has deepened to the point where
their economic futures have become closely interlinked. Western demand, principally from the US,
sustains a whole range of Chinese industries. Chinese investments support America’s deficit financing,
with China holding more than $1 trillion of US government debt. The US, meanwhile, contributes greatly to China’s foreign trade
surplus. If America stopped buying Chinese goods, it would put a serious crimp in Chinese
economic growth. Chinese sovereign wealth funds are also moving into the US financial market to rebalance the amount of
foreign direct investment on each side. The Emergence of Shared Values Chinese-American ties now range well beyond
economics. As major beneficiaries of existing international arrangements, both China and the US have an
important stake in many areas, including defending a free trade system, maintaining international peace and
stability, opposing proliferation of weapons of mass destruction, fighting terrorism, ensuring secure
energy supplies and reversing global warming. In addition, as a result of changes within China, the
two countries increasingly find themselves sharing similar aspirations in the world. Among other things,
China has replaced its centrally-planned economy with a market -oriented one. It has attached increasing
importance to the rule of law. It has publicly advocated protection of human rights and has adopted many
measures to improve its human rights situation. It has also tried to introduce democratic reforms such as nationwide village-level
elections and measures to broaden participation in the selection of leaders at various levels of the Chinese government and in the
policy making process. Recently, Chinese Premier Wen Jiabao said that China wants democracy and will make
more efforts in this regard. These and other changes on the part of China have narrowed the
value differences between the two countries and provided an expanding political basis for
China-US cooperation. Finally, leaders of the two countries have learned how to cooperate after years of
interaction. With the scope and depth of contacts increasing, China and the US find themselves with greater
understanding and appreciation of each other’s legitimate interests and political sensitivities than ever before.
Policy makers in the two countries not only know each other as counterparts, but also increasingly as personal
friends. Many become acquainted long before they become important in their respective policy making institutions. Previous
misunderstandings at the policy level are no longer serious. This has made miscalculation between
the two countries less likely and facilitated cooperation. 

Only a risk that assertive US stances help relations—no risk of decline

Cooke, 11 [ Clean Energy: U.S.-China Cooperation and Competition ¶ By Merritt T. (Terry) Cooke ¶ Terry Cooke is owner and
principal of www.terrycooke.com, a corporate seminar/scenario firm and GC3 Strategy, an international advisory/consultancy
business and is a Senior Fellow at FPRI. He is the author of the forthcoming monograph Sustaining U.S.-China Cooperation in Clean
Energy. He also writes the U.S.-China Clean Energy blog at www.mterrycooke.wordpress.com, http://www.fpri.org/pubs/Obama-
Hu.Summit2011.cooke.pdf]

The U.S.’s tougher tone in the traditional politics of bilateral relations and in the new politics of economic
statecraft has not tripped up U.S.-China cooperation in clean energy or triggered a combative
competitive response from China. If anything, it seems to have given China’s leaders a clearer
sense of a more assertive and comprehensible American president. China now seems to see Obama as
playing an established and recognizable “American tune” on the global stage. During his January state visit to Washington, Hu took
pains to show the “smiling face” of Chinese “peaceful rise” diplomacy, replacing the “angry face” that had been on view after the
Nobel Peace Prize award to Liu Xiaobo and a series of incidents in the South and East China Seas. Hu also skillfully brandished “China,
Inc.’s” checkbook, presiding over more than US $45 billion of commercial deals during his visit with one-quarter of that amount
going to clean energy deals with major U.S. firms.4 In negotiations during the state visit, China also appears to have ceded ground in
the highly-charged dispute over China’s “indigenous innovation” policy in government technology procurement (which U.S. critics
saw as disadvantaging U.S. providers or pressuring them to transfer intellectual property rights to Chinese firms). This approach by
China—a purring voice in response to twin U.S. growls— is understandable. The Chinese leadership, over many decades,
has come to expect , and tends to respect, clear and principled postures of strength and clear
assertions of legitimate interests from the United States. Chinese state-owned companies know that they cannot hope
to become world-class if they do not acquire global market experience and global management skills. Access
to U.S. markets
provides an indispensable proving-ground . Chinese state-owned and private manufacturers depend on sales to U.S.
markets in key areas, including, in the clean energy sector, photovoltaic solar products. They need U.S. markets to grow while they

wait for a domestic market to be developed. Public attitudes in China are deeply confused by all the talk they hear
of from U.S. sources about “Sputnik moments” and about the United States losing the innovation race to the Chinese. To their
minds, innovation is in the U.S. market’s DNA and is the most notable feature missing from the Chinese market. The notion that
Chinese innovation is an existential “Sputnik”-like threat to the United States, thus, does not describe for Chinese observers a
recognizable reality. That may make it all the more alarming and effective as a rallying cry for U.S. action taking a tougher line
against, and seeking to out compete, China in clean energy and other innovation-intensive sectors.
Leaders committed to strong relations—single issues can’t tank them
Wan 10- Staff Writer at Wall Street Journal
(William, “China: U.S. relations 'sound',” http://www.washingtonpost.com/wp-
dyn/content/article/2010/09/06/AR2010090603660.html, 9/7)

At a time of tension in U.S.-China relations, a three-day visit by senior U.S. officials to Beijing began Monday with signs that
Chinese leaders want to smooth over some key frictions. " Sound" and "stable" was how a top Communist Party
official described the two countries' relationship while receiving the U.S. delegation, which included National
Economic Council Director Lawrence H. Summers and deputy national security adviser Thomas Donilon. The meeting comes
after the U.S.-China relationship has been battered on several fronts. The
United States has fought with China
during the past few months over China's trade surplus and currency valuation, with little to show for it. U.S.-
South Korea military exercises near the Chinese coast have incensed Chinese officials, as did President Obama's
meeting with the exiled Tibetan spiritual leader, the Dalai Lama, and a U.S. arms sale to Taiwan, both of
which happened earlier this year. Rhetoric on both sides has ratcheted up in recent weeks on national security issues - with
China's state-owned party papers denouncing U.S. interference in South China Sea issues, and Secretary of State Hillary Rodham
Clinton responding in July at an Association of Southeast Asian Nations meeting that the area is part of her country's "national
interest," which set off more fuming in party papers. On Monday, both sides expressed hope that meetings between U.S. and
Chinese officials scheduled in coming weeks may help thaw some of the recent difficulties. " Although there were
some disturbances in China-U.S. relations, in April and May after President Obama and President Hu Jintao had
two meetings, our relations have gotten back on a sound track," Li Yuanchao, head of the section in the
Chinese government that oversees senior party appointments, said before the closed-door talks began. Later Monday,
according to the Associated Press, Summers told Vice Premier Wang Qishan that Obama "has emphasized for us
the importance he attaches to a very strong relationship between the United Sates and
China." Among this visit's top issues, Summers added, is setting up a trip for Hu to Washington. Still to come in the next few
months are Chinese Premier Wen Jiabao's visit in a few weeks to the U.N. General Assembly in New York, a meeting of the U.S.-
China Joint Commission on Commerce and Trade and the Group of 20 summit, where additional China-U.S. talks may occur. Hu
is likely to visit the United States in January. Some of the most pressing issues in this week's meetings involve the Korean
Peninsula, said Shi Yinhong, an international studies professor at Beijing's Renmin University. There have been icy feelings all
around, he said, since the Cheonan, a warship belonging to U.S. ally South Korea, was sunk near the border of China's ally, North
Korea. "Neither China nor the U.S. wants to make a concession on this," Shi said, "but the two countries also don't seem to want
the relationship to deteriorate again.

Resiliency inevitable--economics trump—empirical data

China Daily 3 (2-13, Lexis)

During President Jiang Zemin's visit to the United States last year, he and Bush stated that China and the United States had
extensive and crucial common interests and should expand their exchanges and co-operation in various areas to develop a
constructive and co-operative Sino-US relationship. ImprovingSino-US relations thus became the inevitable
option in Washington's China policy. Since the mid-1980s, economic and trade exchanges have been a
vital factor in bilateral ties and remain the most resilient chain. Bearing in mind the huge economic
interests arising from China's entry to the World Trade Organization (WTO), Bush emphasized the importance of Sino-US
economic and trade relations, even when pursuing a hard-line China policy in the initial period of his tenure. History
has
proved that, despite some twists and turns, common interests have overweighed differences
in Sino-US relations.
No issue spill-over—empirics

IPS 3 (Inter-Press Service, 11-4, Lexis)

Indeed, it now appears that, despite rising tensions over the bilateral trade balance and the value of the yuan, the
realists centered in the State Department have decisively taken control over U.S. China policy, thanks largely to Beijing's own
behavior and rapidly growing influence. "The administration has come to the conclusion that strategic engagement is the only
viable option on relations with China," says Garrett. ThatWashington's major problem today is over
currency, he adds, illustrates the degree to which Sino-U.S. relations have stabilized. "This is the
kind of problem we have with Japan," Garrett said. "We're at the point where we can have differences in one area
without it threatening other aspects of the relationship."
---China Econ Co-Op

US-China economic cooperation is growing and one issue can’t derail it

Fung ’12
(Brian, associate editor at The Atlantic. He has written previously forForeign Policy, The Washington Post, and Talking Points Memo.
“Don't Panic: U.S.-Chinese Economic Cooperation Isn't About to Break Down.” Oct. 16 - The Atlantic)

Xi is clearly issuing something of a warning -- Hey, America: don't say or do something you'll regret later!-- but what's important is
that he acknowledges domestic politics puts pressure on politicians to behave in ways that create unintended problems at the
international level. As long as Beijing remains aware of that, and it's hard to think it won't, given how much of China's
own governance strategy revolves around maintaining domestic stability, we can probably expect some amount
of understanding from the Chinese regime that what looks like a snub is sometimes just part of doing
business. And what a business it is: 34 U.S. states were engaged in some kind of business relationship with
China last year. The number of Chinese deals with U.S. companies is growing annually at a rate of 130
percent. In 2011, Air China opened a $5 million ticket-booking facility in Los Angeles that's staffed by 30 American employees. Even
if some attempts at Chinese investment are being blocked by the White House, there's still a lot of other
activity going on that's greasing the U.S.-China relationship.
---China Nuke Co-Op

Thorium doesn’t lock out US—being developed collaboratively

Halper ‘12, Mark Smart Planet, 6/26/12, U.S. partners with China on new nuclear, www.smartplanet.com/blog/intelligent-
energy/us-partners-with-china-on-new-nuclear/17037

The U.S. Department of Energy is quietly collaborating with China on an alternative nuclear power design known as a
molten salt reactor that could run on thorium fuel rather than on more hazardous uranium, SmartPlanet understands.

DOE’s assistant secretary for nuclear energy Peter Lyons is


co-chairing the partnership ’s executive committee,
along with Jiang Mianheng from the Chinese Academy of Sciences (CAS), according to a March presentation by CAS
on thorium molten salt reactors. Beijing-based CAS is a state group overseeing about 100 research institutes. It and
the DOE have established what CAS calls the “CAS and DOE Nuclear Energy Cooperation
Memorandum of Understanding.”

Nuclear coop now

Halper ‘12, Mark Smart Planet, 7/3/12, Westinghouse enters U.S.-China nuclear collaboration,
www.smartplanet.com/blog/intelligent-energy/westinghouse-enters-us-china-nuclear-collaboration/17252

Pittsburgh-based Westinghouse Electric Co. is


playing a supporting role in the U.S. Department of Energy’s and
China’s collaborative development of an alternative and potentially safer nuclear reactor - a project for which
DOE has funded three U.S. universities, SmartPlanet has learned.

As I reported last week, DOE and the Chinese Academy of Sciences (CAS) have been quietly
working together on a reactor design that uses a molten salt coolant auguring safer, more
efficient and lower cost reactors that operate at higher temperatures than conventional water-
cooled reactors.

The Chinese also intend to use liquid thorium molten salt fuel in a molten salt cooled reactor.
Some experts believe that the combination of a liquid thorium fuel and a molten salt-coolant
would provide a reactor that is much more efficient than today’s reactors, and that cannot melt
down. Supporters claim that thorium molten salt reactors would yield waste that lasts for only
hundreds of years instead of uranium’s tens of thousands, and from which it is far more difficult
to build a bomb. Acting Westinghouse CEO Shigenori Shiga. The U.S. developed a thorium
molten salt reactor in the 1960s at Oak Ridge National Laboratory, but abandoned it in favor of
more weapons-prone uranium reactors during the Cold War, a story which author Richard
Martin tells vividly in his new book, SuperFuel. Following my report last week based on a
purportedly leaked Chinese Academy of Sciences presentation, a DOE spokeswoman confirmed
for me that DOE signed an agreement with CAS last December for “cooperation in nuclear
energy sciences and technologies.” Pete Lyons, assistant DOE secretary for nuclear energy, said
in an email sent by the spokesperson that, “These collaborations will strengthen cooperation
between the U.S. and China around next generation nuclear technology, helping to advance
mutually beneficial technological advancements and grow civilian nuclear power as a safe,
reliable and clean source of energy for both countries.”
---China Climate Co-Op

Cooperation is paralyzed now

– domestic politics, overconsumption and Chinese view of inevitability

Czarnezki, 11 [Jason J. Professor of Law in the Environmental Law Center and Faculty Director of the U.S.-China¶ Partnership
for Environmental Law at Vermont Law School; A.B., J.D, “CLIMATE POLICY &¶ U.S.-CHINA RELATIONs”, Published After April 4 th
2011. http://www.vermontlaw.edu/Documents/Jason%20Czarnezki%20Climate%20Policy%20and%20China.pdf]

United States climate policy is paralyzed by domestic politics and a¶ culture of over-consumption .
China, despite scientific evidence that its¶ emissions alone could lead to catastrophic climate events, employs a¶ climate
policy arguably based on a cold (though perhaps correct) reality in¶ which the climate crisis is inevitable
and only the economically strong will¶ survive. Thus, this Essay has a simple thesis: to date, both the United States¶
and Chinese governments have failed to show leadership in responding to¶ the climate crisis, and without such leadership the
countries’ continued¶ paths will make the potential crisis a reality.¶ During the 2009-2010 academic year, I was a J. William
Fulbright¶ Scholar, teaching at Sun Yat-sen University (also known as Zhongshan¶ University) in Guangzhou, China. Guangzhou sits
as the capital of¶ Guangdong province in south China, a global manufacturing power in the¶ Pearl River Delta, near the boomtown
of Shenzhen, and two hours north of Hong Kong by train. While there, I taught courses on the U.S. legal system was fortunate
enough to give numerous lectures, participate in roundtables¶ with Chinese faculty and officials, have discussions with American¶
academics in China, and build relationships with U.S. government officials.¶ Nearly all these experiences involved, some exclusively,
discussion of¶ climate change and the political roles and responsibilities of the United¶ States and China, the two largest emitters of
greenhouses gases in the¶ world.¶ While I was in China, this quote appeared in the China Daily, China’s¶ English language
newspaper: “China ‘could not and should not’ set an¶ upper limit on greenhouse gas emissions at the current phase, said Su Wei,¶
the chief negotiator of China for climate change talks . . . .”1 Similar views¶ were expressed by Chinese academics and policy-makers
participating in a¶ “China-U.S. Relations Roundtable” held by the Center for Asia-Pacific¶ Studies at Sun Yat-sen University in May
2010. The question is whether¶ China’s policy is irresponsible given the scope and pace of Chinese¶ development and energy
consumption (a view held by the Europeans at the¶ Copenhagen Climate Conference in 2009).¶ The reasons behind
China’s public stance, and unwillingness to curb overall emissions, are well-known: China deserves
its turn to develop;¶ China is only a developing country; China wants to be seen as the leader of ¶ the
developing world (i.e., the king of the BASIC countries, Brazil, South¶ Africa, India, and China), not a member of the fully
developed world;¶ China’s per capita carbon emissions pale in comparison to the United¶ States’; China remains a poor
country; China’s foreign policy is noninterventionist ¶ and does not tell other countries what to
do, and China¶ expects the same autonomy in return; economic stability is key to social¶ stability and nationalism;
absent a strong economy there will be civil unrest¶ and Communist Party leaders may lose power; and the list goes on.¶ The
Chinese stance, that no cap on carbon emissions will ever exist no¶ matter how high, may be a product of China’s
belief in a cold and hard, and¶ potentially true, reality—that global economic power is paramount and will¶
provide the only avenue to adapt to an inevitable climate crisis , as well as¶ achieve the milestones of
superpower status, many of which they have¶ already achieved (e.g., Olympic Games, World Expo, United Nations¶ Security
Council). While China’s policy remains problematic, as is United¶ States’ failure to lead in the international community on the issue
of climate¶ change, China’s actions, while globally irresponsible, may be very reasonable if solely defined by Chinese domestic
interests. The question is¶ whether China’s dramatic economic rise comes with more responsibility,¶ and what is the responsibility
of the United States in light of its existing and¶ historical economic prowess and level of energy consumption.
U.S. and Chinese tariffs are a huge alt cause

Xinhua ’11 (Jianhua and Yunlu, Xinhua, 2011 Wang and Li, 10-13-11, “China-US trade war no good for anyone”
http://english.peopledaily.com.cn/90780/7616581.html

Economic and trade analysts say that if the United States passes into law a bill concerning the RMB exchange rates, long-held fears

of a brewing trade war between the U nited S tates and China will become a horrible reality, hurtling the
world economy into disorder and recession. If the U.S. Senate passes the bill on Oct. 12 Beijing time, the bill still requires approval
from the U.S. House. After the Senate and the House both pass the bill, it ultimately must be signed into law by the U.S. president.
According to the bill, if the exchange rates of major trade partners are considered undervalued, the
U nited S tates will levy punitive tariffs on the goods imported from the trade partners. Furthermore, if the currency
value of a trade partner is considered so low that it has become a means to subsidize their exports, the U.S. businesses can request
the U.S. government to impose higher tariffs on overseas competitors. Zhao Jinping, vice director of the Foreign Economic
Department of the Development Research Center (DRC) of the State Council, said that the bill is mainly aimed at China. Given the
situation concerning the U.S. presidential election in 2012, the possibility of turning the bill into a law is increasingly great because of
the political motives of the U.S. side. Similarly, the risks that China will impose high penalty tariffs on the goods
imported from the U nited S tates are also increasing. "If the United States insists on doing what it wishes, a
trade war between China and the United States will be inevitable ," he said.

Multiple barriers ensure energy cooperation fails and undermines cooperation

Segal ‘9 (The G-2 Mirage. By: Economy, Elizabeth C., Segal, Adam, Foreign Affairs, 00157120, May/Jun2009, Vol. 88, Issue 3 By
Elizabeth C. Economy and Adam Segal Elizabeth C. Economy is C. V. Starr Senior Fellow and Director for Asia Studies at the Council
on Foreign Relations. Adam Segal is Maurice R. Greenberg Senior Fellow for China Studies at the Council on Foreign Relations.

But elevating the bilateral relationship is not the solution. It will raise expectations for a level of
partnership that cannot be met and exacerbate the very real differences that still exist between
Washington and Beijing. The current lack of U.S.-Chinese cooperation does not stem from a failure on Washington's
part to recognize how much China matters, nor is it the result of leaders ignoring the bilateral relationship. It derives from
mismatched interests, values, and capabilities . The United States must accordingly resist the temptation to
initiate a high-profile, high-stakes bilateral dialogue and instead embrace a far more flexible, multilateral approach to China. In other
words, Obama should continue to work with China in order to address global problems, but he also needs to enlist the world to deal
with the problems created by the rise of China. The good news is that the United States and China do share some fundamental
foreign policy goals: kickstarting economic growth and maintaining an open global economy, maintaining peace and stability in East
Asia, and halting climate change. There is already a robust process of government-to-government exchange, with more than 60
bilateral dialogues and working groups in existence, including the Strategic Economic Dialogue, the U.S.-China Senior Dialogue, and
the Defense Policy Coordination Talks. The United States and China have cooperated on counterterrorism, negotiated with North
Korea through the six-party talks, and undertaken joint research on alternative energy. Recently, the Pentagon welcomed the
deployment of the Chinese navy for antipiracy operations in the Gulf of Aden, where both the United States and China depend on
the same shipping lanes. Recognition of China's importance, however, cannot paper over the difficulties the two countries have
faced--and will continue to face--as they try to convert shared strategic goals into concrete cooperation. Even after 30 years of
engagement, the United States and China still disagree about how the world should work. When there is agreement on the
principles of global governance, more narrow economic interests or differences in political values typically make a common front
even when shared values and interests allow the two sides to move forward, the vast gap
elusive. And
in governance and implementation capabilities often leads to mutual frustration and
recriminations. THE GREAT STALL A SIGNIFICANT obstacle to effective U.S.-Chinese cooperation is the dramatically different
view of sovereignty, sanctions, and the use of force that each country brings to the table. Beijing's need for resources and export
markets, along with its oft-repeated mantra of not mixing business with politics, clashes with the West's efforts to prevent human
rights abuses and improve governance in the developing world. For example, Chinese state-owned companies have vast resource
holdings and development interests in countries such as Angola, the Democratic Republic of the Congo, and Myanmar (also called
Burma), where human rights abuses and problems of poor governance are rampant. Likewise, China's refusal to stop its growing
arms trade has contributed to instability in Sudan and Zimbabwe, even as the rest of the world has urged restraint in weapons sales
to those nations. Yet Beijing repeatedly refuses to exert its economic leverage for political ends, arguing that the political and
economic realms should remain separate. Although Washington and Beijing share some common interests in Darfur, Myanmar, and
other regions in which serious human rights violations are occurring, their opposing perspectives on sovereignty and humanitarian
intervention lead to very different policy prescriptions. In September 2007, for example, Beijing, along with Moscow, blocked a UN
Security Council resolution sponsored by the United States and Europe that forcefully condemned the Myanmar government for the
use of force against Buddhist monks who were leading antigovernment protests. China, which has strong political ties to Myanmar's
rulers and large investments in oil and gas projects in the country, insisted that the crackdown was an internal affair, called for
restraint, and finally voted for a much-watered-down resolution. Beijing's growing dependence on imported oil and gas has also
been an obstacle to broader U.S.-Chinese cooperation on limiting Iran's nuclear program. China has generally rejected calls for tough
sanctions against Iran and undermined efforts by the United States and Europe to stop the flow of Iranian money through foreign
banks. As Iran's trade with the European Union (EU) has declined, it has picked up with China. In addition, China's authoritarian but
decentralized political and economic system also makes cooperation on issues such as product safety and environmental protection
difficult. Beijing is often incapable of following through on its international obligations because local actors have strong economic
incentives to maintain the status quo. Even when the central government has the best intentions, a lack of transparency and
accountability further undermines the implementation of laws. Over the past several years, for example, China's food- and product-
safety records have come under increasing international scrutiny. China's vast exports ensure that almost no country goes
unaffected when tainted pet food, toothpaste, dairy products, toys, or pharmaceutical ingredients are released onto the market.
These poisonous products have entered the EU, Japan, Panama, the United States, and Vietnam--to name just a few. Efforts to
address China's ongoing problems in the realm of food and product safety are hampered not only by bureaucracies with weak
oversight capacity but also by a government that refuses to permit the media and on-the-ground watchdogs to investigate and warn
the general public. Reporters from Southern Weekend and the Oriental Morning Post knew two months before the story became
public, in September 2008, that melamine--a chemical that suppliers were adding to low-quality dairy products in order raise the
protein level and pass inspections-- had sickened children. But they were prohibited from reporting the news, in order to ensure a
positive atmosphere for the Olympics. Although the press, bloggers, and nongovernmental organizations are becoming increasingly
assertive, they are unable to act consistently as a check on local officials due to censorship and political harassment.
Cooperation on climate change may prove even more challenging. As Washington weighs the value of
pursuing a bilateral climate deal with China, Beijing's ability to enforce regulations will be called into question. Effective climate
policy depends on transparency in reporting emissions, yet in the run-up to the Olympics, Beijing's published air-pollution index did
not include two of the most dangerous pollutants: ozone and small particulate matter. A bilateral climate deal would
also necessitate large-scale transfers of energy -related technologies, such as advanced materials for wind
turbines, from the United States to China. However, China lacks the legal infrastructure needed to enforce
intellectual property rights--a weakness that will worry the European, Japanese, and U.S. firms
that design new green technologies . Even basic policy initiatives, such as the implementation of energy-
efficient building codes, require a degree of enforcement capacity and official accountability that
does not exist within the Chinese government.

US China energy cooperation fails and trades off with more effective initiatives by other
countries

Segal ‘9 (The G-2 Mirage. By: Economy, Elizabeth C., Segal, Adam, Foreign Affairs, 00157120, May/Jun2009, Vol. 88, Issue 3 By
Elizabeth C. Economy and Adam Segal Elizabeth C. Economy is C. V. Starr Senior Fellow and Director for Asia Studies at the Council
on Foreign Relations. Adam Segal is Maurice R. Greenberg Senior Fellow for China Studies at the Council on Foreign Relations.

Despite insistent calls for a bilateral U.S.-Chinese effort to address climate change, cooperation would be
managed best by involving other nations. The United States and China are the two largest emitters of carbon
dioxide, and each is using the other as an excuse for inaction. China is currently calling for all the advanced
industrialized nations to devote one percent of their GDPs to a climate fund for developing countries--
a mechanism that would subsidize the transfer of clean-energy technologies to Beijing. The United States simply does not
have the financial resources available to respond to this challenge. Meanwhile, Japan is pursuing a
different tack and has already started to provide development assistance to China for clean-
energy projects thanks to its highly sophisticated public-private partnerships that provide government
financing for privately held Japanese technology companies. Beijing and Tokyo are moving ahead with technical
cooperation and demonstration projects to capture and store carbon, enhance energy efficiency, and
exploit renewable energy resources. It makes sense for the United States to duplicate some of these efforts. After all,
China is a large country, and there will always be unmet demand. In addition, the United States has a comparative advantage when
it comes to training Chinese officials, monitoring greenhouse gas emissions, and designing some clean-energy technologies. But
Washington does not want to undermine European and Japanese efforts by competing to
cooperate with China. There is frequently a cost to cooperating with Beijing: Chinese government agencies often require
donations or impose high overhead costs on foreign partners, and these fees could well rise as the Chinese play foreign actors off
against one another. Moreover, Chinese energy and environmental agencies are woefully understaffed and
often unable to meet the demands of in-depth cooperation with a number of different
partners. There is also a very real danger that U.S. officials will raise expectations within China but fail to deliver if, for example,
the U.S. government does not provide adequate funding for demonstration projects or training programs, as has happened with past
cooperative energy and environmental ventures.

International agreements on climate fail

Loris and Schaefer ’13 (Nicolas D. Loris is Herbert and Joyce Morgan Fellow in the Thomas A. Roe Institute for Economic
Policy Studies and Brett D. Schaefer is Jay Kingham Fellow in International Regulatory Affairs in the Margaret Thatcher Center for
Freedom, a division of the Kathryn and Shelby Cullom Davis Institute for International Studies, at The Heritage Foundation, “Climate
Change: How the U.S. Should Lead”, http://www.heritage.org/research/reports/2013/01/climate-change-how-the-us-should-lead,
January 24, 2013)

The past four years have seen successive annual U.N. conferences (Copenhagen in 2009, Cancun in 2010, Durban in
2011, and Doha in 2012) frantically trying to reach agreement among nearly 200 countries on a successor to
the Kyoto Protocol. In essence, these conferences have succeeded only in wresting vague pledges from
developed countries to reduce emissions, contribute funds to help developing countries adapt to climate change, and meet again to
try to negotiate a binding treaty by 2015. The problem is that the basic approach is unworkable. International
negotiations have centered on placing the economic burden of addressing climate change on a few dozen developed countries while
asking nothing from more than 150 developing countries. But the
primary source of greenhouse gas emissions is
increasingly the
developing world. Any approach to effectively address increasing emissions of greenhouse gases
must capture emissions from developed and developing countries . This notion was the central feature of
the 1997 Byrd–Hagel Resolution, which unanimously passed the Senate, establishing conditions for the U.S. becoming a signatory to
the Kyoto Protocol and remains the primary reason why the U.S. never ratified that treaty. But developing countries, primarily
India and China, have made it quite clear that they have no appetite to slow economic growth or curb
use of conventional fuels to control emissions. For this reason, Canada, Japan, and Russia refused to sign onto a
new agreement committing them to emissions reductions unless major developing country emitters were also included.
Until and unless this issue is resolved, the U.S. would be foolish to consider unilateral restrictions on the U.S.
economy that, in the end, would be merely symbolic without significant effect on global emissions
reductions.

Alt causes to tensions, and more US energy expansion make it worse


Fox News ’12 (8/21/12 (“China declares US energy Projects Violate Free Trade, Stepping Up Trade Spat”)

Monday’s announcement said U.S. government subsidies to wind, solar and hydro projects in
Washington state, Massachusetts, Ohio, New Jersey and California acted as a barrier to imports but gave no details. It
called on the U.S. government to give Chinese renewable energy products “fair treatment.” The
Chinese probe was launched last November two weeks after Washington said it would
investigate whether Beijing is inappropriately subsidizing its own makers of solar panels,
allowing them to flood the U.S. market with low-priced products and hurt American competitors. The Commerce Ministry said it was
trade groups for Chinese producers of clean energy technology that said they
acting on complaints by
were hurt by the U.S. policies.
---China Climate Co-Op Extensions

Domestic issues block successful climate cooperation

Hale ‘11 (Thomas Hale, PhD Candidate in the Department of Politics at Princeton University and a Visiting Fellow at LSE Global
Governance, London School of Economics, Center for Strategic and International Studies, The Washington Quarterly, 34:1 pp. 89-
101, “A Climate Coalition of the Willing,” http://www.twq.com/11winter/docs/11winter_Hale.pdf, 2011)

Intergovernmental efforts to limit the gases that cause climate change have all but failed. After the unsuccessful 2010
Copenhagen summit, and with little progress at the 2010 Cancun meeting, it is hard to see how major emitters
will agree any time soon on mutual emissions reductions that are sufficiently ambitious to prevent a
substantial (greater than two degree Celsius) increase in average global temperatures. It is not hard to see why. No deal excluding
the United States and China, which together emit more than 40 percent of the world’s greenhouse gases (GHGs), is worth the
paper it is written on. But domestic politics in both countries effectively block ‘‘G-2’’ leadership on climate .
In the United States, the Obama administration has basically given up on national cap-and-trade legislation. Even the
relatively modest Kerry-Lieberman-Graham energy bill remains dead in the Senate. The Chinese government, in turn, faces an
even harsher constraint. Although the nation has adopted important energy efficiency goals, the Chinese Communist
Party has staked its legitimacy and political survival on raising the living standard of average Chinese.
Accepting international commitments that stand even a small chance of reducing the country’s
GDP growth rate below a crucial threshold poses an unacceptable risk to the stability of the regime. Although the
G-2 present the largest and most obvious barrier to a global treaty, they also provide a convenient excuse for other governments to
avoid aggressive action. Therefore, the international community should not expect to negotiate a worthwhile successor to the Kyoto
Protocol, at least not in the near future.

And, our domestic politics arguments outweigh and block solvency

Lieberthal, 09 [U.S. CHINA CLEAN ENERGY COOPERATION:¶ THE ROAD AHEAD, Kenneth G, Kenneth Lieberthal is director of
the John L. Thornton China Center and senior fellow in Foreign Policy and Global Economy and Development at Brookings. Lieberthal
was a professor at the University of Michigan for 1983-2009, http://www.brookings.edu/~/media/research/files/papers/2009/9/us
%20china%20energy%20cooperation%20lieberthal/09_us_china_energy_cooperation_lieberthal]

In view of the above complications, it


is difficult¶ to be very optimistic about making adequate¶ progress on
climate change during the remainder¶ of 2009. U.S.-China bilateral cooperation should¶ be the easier task. The number of
players is small,¶ both sides see potentially welcome side benefits in¶ terms of strengthening their overall relationship,¶ and it
should be possible to focus in particular on¶ those activities that clearly are beneficial to both¶ sides. The
global talks enjoy
none of these advantages¶ and are in addition weighed down by larger political considerations that revolve
around both¶ the negotiating history and the relationship between ¶ developed and developing
countries.¶ As indicated above, astute U.S.-China cooperation¶ can make expectations about Copenhagen¶ more
realistic and the meeting itself more likely¶ to lay the groundwork for an eventual full agreement.¶ But it will take astute
leadership at the¶ highest levels in both Washington and Beijing —¶ and effective management of
domestic politics in¶ both countries—to achieve these results. The issue¶ could not be more important;
unfortunately,¶ the chances of success are at this point quite uncertain.
China-Saudi Relations Answers
Frontline

China-Saudi rels are high and stable for decades

Arab News 12 [“Working together for brighter future of Saudi-China relations,” http://www.arabnews.com/working-together-
brighter-future-saudi-china-relations]

This year marks the 22nd anniversary of the establishment of diplomatic relations between China and Saudi Arabia .
Over the
past two decades, despite the complex and profound changes of the regional and international situations, China-Saudi
strategic friendly relations maintained the momentum of rapid development, and bilateral
cooperation in various fields has yielded fruitful results. China and Saudi Arabia have had frequent
exchanges of high-level visits, which served to enhance mutual political trust . Both leaders have
been attaching great importance to the development of China-Saudi relationship . With a view to
deepening mutually beneficial cooperation and safeguarding the common benefits of the two peoples, both countries took
full advantage of these high-level visits to promote mutual understanding and trust, broaden
and deepen bilateral cooperation in all fields and continuously advance China-Saudi relationship
to new high.

China-Saudi relations are strong

Denning 12 [Dan, editor of The Daily Reckoning Australia and the Publisher of Port Phillip Publishing, 1-16, “The Growing Energy
and Oil Alliance Between China and Saudi Arabia,” http://www.dailyreckoning.com.au/how-the-energy-and-oil-alliance-between-
china-and-saudi-is-growing/2012/01/16/]

The oil and energy alliance between China and Saudi Arabia has just been strengthened. On
Saturday - in the wake of Europe's debt crisis - Saudi state oil company Aramco signed a deal with China's
Sinopec to build an oil refinery in the Red Sea city of Yanbu. The refinery will process 400,000 barrels of oil per day,
some of which will presumably end up in China. "Saudi Aramco will hold a 62.5 percent stake with Sinopec
holding the balance in the venture that highlights China's growing role as an infrastructure
developer in the oil rich kingdom," according to AFP. As an intriguing sidenote, Chinese Premier, Wen Jiabao - who made
Saudi Arabia the first stop on his Middle East tour - signed a series of "agreements and cooperation programs"
with Saudi King Abdullah. One of those was, "An agreement between the two governments on the peaceful usage of
nuclear energy was also signed. Signatories for the two parties were Dr. Hashem Yamani, President of King Abdullah City for Atomic
and Renewable Energy, and Chang Peng, President of the National Commission for Development and Reforms of China." Hmm. Isn't
that interesting? This confirms our idea that thanks to America's shale gas revolution - which makes Middle East oil imports a lot less
important to the United States - the
Saudis are shopping for a new strategic patron in the region and the
Chinese are keen to be that patron . Aramco CEO Khalid al-Falih said the deal "Represents a strategic
partnership in the refining industry between one of the main energy producers in Saudi Arabia
and one of the world's most important consumers ."
China US War Answers
Frontline

No US-China war

Rosecrance et al ‘10 (Richard, Political Science Professor @ Cal and Senior Fellow @ Harvard’s Belfer Center and Former
Director @ Burkle Center of IR @ UCLA, and Jia Qingguo, PhD Cornell, Professor and Associate Dean of School of International
Studies @ Peking University, “Delicately Poised: Are China and the US Heading for Conflict?” Global Asia 4.4,
http://www.globalasia.org/l.php?c=e251)

Will China and the US Go to War? If one accepts the previous analysis, the answer is “no,” or at least not likely.
Why? First, despite its revolutionary past, China has gradually accepted the US-led world order and become
a status quo power. It has joined most of the important inter-governmental international organizations. It has
subscribed to most of the important international laws and regimes. It has not only accepted the current world order, it has
become a strong supporter and defender of it. China has repeatedly argued that the authority of the United Nations and
international law should be respected in the handling of international security crises. China has become an ardent advocate of
multilateralism in managing international problems. And China has repeatedly defended the principle of free trade in the global
effort to fight the current economic crisis, despite efforts by some countries, including the US, to resort to protectionism. To be
sure, there are some aspects of the US world order that China does not like and wants to reform. However, it wishes to improve
that world order rather than to destroy it. Second, China has clearly rejected the option of territorial
expansion. It argues that territorial expansion is both immoral and counterproductive: immoral because it is imperialistic
and counterproductive because it does not advance one’s interests. China’s behavior shows that instead of trying to expand its
territories, it has been trying to settle its border disputes through negotiation . Through persistent efforts,
China has concluded quite a number of border agreements in recent years. As a result, most of its land borders are now clearly
drawn and marked under agreements with its neighbors. In addition, China is engaging in negotiations to resolve its remaining
border disputes and making arrangements for peaceful settlement of disputed islands and territorial waters. Finally , even on
Taiwan , which China believes is an indisputable part of its territory, it has adopted a policy of
the question of
peaceful reunification . A country that handles territorial issues in such a manner is by no means expansionist. Third,
China has relied on trade and investment for national welfare and prestige, instead of military
conquest. And like the US, Japan and Germany, China has been very successful in this regard. In fact, so successful that it
really sees no other option than to continue on this path to prosperity . Finally, after years of reforms,
China increasingly finds itself sharing certain basic values with the US, such as a commitment to the free market, rule of law,
human rights and democracy. Of course, there are still significant differences in terms of how China understands and practices
these values. However, at a conceptual level, Beijing agrees that these are good values that it should strive to realize in practice.
A Different World It is also important to note that certain changes in international relations since the end of World War II have
made the peaceful rise of a great power more likely. To begin with, the emergence of nuclear weapons has drastically reduced
the usefulness of war as a way to settle great power rivalry. By now, all great powers either have nuclear weapons or are under
a nuclear umbrella. If the objective of great power rivalry is to enhance one’s interests or prestige, the sheer destructiveness of
nuclear weapons means that these goals can no longer be achieved through military confrontation. Under these circumstances,
countries have to find other ways to accommodate each other — something that China and the US have been doing and are
likely to continue to do. Also, globalization has made it easier for great powers to increase their national welfare and prestige
through international trade and investment rather than territorial expansion. In conducting its foreign relations, the US relied
more on trade and investment than territorial expansion during its rise, while Japan and Germany relied almost exclusively on
international trade and investment. China, too, has found that its interests are best served by adopting the same approach.
Finally, the development of relative pacifism in the industrialized world, and indeed throughout the world since World War II,
has discouraged any country from engaging in territorial expansion. There is less and less popular support for
using force to address even legitimate concerns on the part of nation states. Against this background, efforts
to engage in territorial expansion are likely to rally international resistance and condemnation. Given all this, is the rise of China
likely to lead to territorial expansion and war with the US? The answer is no.
China can’t catch up and no risk of war

Zenko and Cohen ‘12 (Micah Zenko, Fellow in the Center for Preventive Action at the Council on Foreign Relations, and
MIchael Cohen, Senior Fellow at the American Security Project, serves on the board of the National Security Network and has taught
at Columbia University’s School of International and Public Affairs, served in the U.S. Department of State, former Senior Vice
President at the strategic communications firm of Robinson, Lerer and Montgomery, bachelor’s degree in international relations
from American University and a master’s degree from Columbia University, 3/14/2012, "Clear and Present Safety",
yaleglobal.yale.edu/content/clear-and-present-safety)

As the threat from transnational terrorist groups dwindles, the United States also faces few risks from other states. China is the
most obvious potential rival to the United States, and there is little doubt that China’s rise will pose a challenge to U.S. economic
interests. Moreover, there is an unresolved debate among Chinese political and military leaders about China’s proper global role,
and the lack of transparency from China’s senior leadership about its long-term foreign policy objectives is a cause for concern.
However, the present security threat to the U.S. mainland is practically nonexistent and will remain so.
Even as China tries to modernize its military, its defense spending is still approximately one-ninth that of the
United States. In 2012, the Pentagon will spend roughly as much on military research and development alone as China will spend
on its entire military. While China clumsily flexes its muscles in the Far East by threatening to deny access to disputed maritime
resources, a recent Pentagon report noted that China’s military ambitions remain dominated by “regional
contingencies” and that the People’s Liberation Army has made little progress in developing capabilities that “extend global
reach or power projection.” In the coming years, China will enlarge its regional role, but this growth will only threaten U.S. interests
if Washington attempts to dominate East Asia and fails to consider China’s legitimate regional interests. It is true that China’s
neighbors sometimes fear that China will not resolve its disputes peacefully, but this has compelled Asian countries to
cooperate with the United States, maintaining bilateral alliances that together form a strong security
architecture and limit China’s room to maneuver . The strongest arguments made by those warning of Chinese
influence revolve around economic policy. The list of complaints includes a host of Chinese policies, from intellectual property
theft and currency manipulation to economic espionage and domestic subsidies. Yet none of those is likely to lead to
direct conflict with the United States beyond the competition inherent in international trade, which does not produce
zero-sum outcomes and is constrained by dispute-resolution mechanisms, such as those of the World Trade Organization. If
China’s export-driven economic strategy, along with its large reserves of U.S. Treasury bonds, suggests
anything,
that Beijing will continue to prefer a strong United States to a weak one.

No China military threat

Erickson & Liff ‘3-7


(Andrew, S. & Adam, P. 2013, Andrew is an associate professor in the U.S. Naval War College's China Maritime Studies Institute.
Adam is a doctoral candidate in Princeton University's Department of Politics. Foreign Policy. “A Player, but No Superpower: Why
China's military is a threat to its neighbors, but shouldn't concern the United States on its home turf.”

http://www.foreignpolicy.com/articles/2013/03/07/a_player_but_no_superpower_china_military)

Even with this surging investment, there are several major obstacles to China's developing military capabilities
potent far beyond the Near Seas. Inefficiencies still weaken the PLA, which has an estimated 2.285 million active-duty
personnel, and dilute the impact of spending increases. Some commentaries in influential Chinese military journals have
charged that the PLA's procurement strategy increasingly focuses on mimicking overseas developments in arms and
equipment technology, even though more basic strategic goals, like Beijing's island and maritime claims in the Near
Seas, remain unresolved. The PLA lacks robust internal mechanisms for analyzing or evaluating its
equipment procurement needs, and a growing percentage of the defense budget appears to be
earmarked toward prestige-driven, highly publicized, and extremely expensive programs that don't yield
top-end military capabilities. China's aircraft "starter carrier," for example, is not only extremely vulnerable
to missiles and submarines, but is also years away from reaching the capabilities that the United
States possesses today. And the reconnaissance options that China's manned space program offers could be provided more
cheaply via unmanned satellites. Corruption remains a serious problem in the military. "No country can defeat
China," PLA Gen. Liu Yuan said in meeting with roughly 600 officers in December 2011. "Only our own corruption can destroy us and
cause our armed forces to be defeated without fighting." In February, Xi Jinping, China's soon-to-be president, launched a campaign
to impose stricter discipline and oversight in the military. Yet it will take more than limiting military banquets to "four dishes and a
soup," a policy Xi has called for, to solve the PLA's problems and enable it to become one of the world's most sophisticated
militaries. China scholar Minxin Pei has warned that by
pursuing gradualist, incomplete reforms, Beijing risks a
"trapped transition" instead of transformation into a full-fledged market economy. Signs of an analogous
trap are also apparent in the military, as it strives to transition from a domestic, Near Seas-focused,
personnel-intensive force to one characterized by a broader geographical mandate, advanced
technology, and innovation. A slowdown in enhancement of military capabilities looms: The PLA's
rapid progress in recent years means that fewer easy improvements and fixes remain. But the closer
the PLA gets to possessing cutting-edge capabilities in defense technology, the more difficult it becomes to advance further -- much
ofthe low-hanging fruit has already been plucked. Well-educated and technologically capable
personnel cost significantly more to attract, train, and retain than China's erstwhile massive peasant land army,
particularly when private-sector alternatives provide significantly greater compensation. Other personnel expenses -- including
health care and retirement costs -- which constitute major challenges to the U.S. military budget, may hit China
even more rapidly given its less-favorable demographics. Despite its progress in modernizing and some remarkable
new hardware, the PLA's war-fighting capabilities still significantly trail those of the U.S. military. They may
get trapped there, even though a growing number of influential actors have called for China to expand its military horizons. The
likelihood that the PLA will get trapped in its region -- with respect to high-end war-fighting capabilities -- will increase if
Beijing's
growing military power and assertiveness lead its neighbors to further accelerate their own counterbalancing .
China's geographical, economic, and (in some cases) technological advantages do not transfer to
capabilities that would allow it to engage in high-intensity combat beyond the country's
immediate periphery. In other words, China can stir up the Near Seas, but can't make tsunamis
beyond that.

Economic barriers prevent rise

Scissors ‘12, Derek Research Fellow in Economics at the Asian Studies Center of the Heritage Foundation and Adjunct
Professor of Economics at George Washington, “The Wobbly Dragon”, Foreign Affairs, January/February

Arvind Subramanian claims that China will unquestionably replace the United States as the dominant global power in the next
two decades ("The Inevitable Superpower," September/October 2011). He is right that if the U.S. economy continues on its current
trajectory, the United States will not be able to maintain its position of global leadership. But he is
far too bullish on China.
Subramanian overlooks Chinese policies that will complicate the country's economic rise and
ignores the possibility that Chinese growth will simply stop . And he uses a definition of "dominance" that bears
little resemblance to the U.S.-style preeminence he sees China assuming. Consider how Subramanian measures China's growing
power. He cites the ability of Beijing to convince African countries to recognize it instead of Taipei, but out-muscling
Taiwan
diplomatically is hardly a sign of global leadership . He sees the ease with which China undervalues the yuan by
pegging it to the dollar as proof of the country's strength, but hiding behind a foreign currency is not a sign of
economic might. He forecasts that China in 2030 will have an economy that is one-third larger than the United States',
yet he admits that it will remain only half as wealthy. These are notable trends, to be sure, but not ones that
indicate China will attain anything close to the position the U nited States has held over the past 60
years. The biggest flaw in Subramanian's index of dominance is the importance he assigns to
China's status as a net creditor . Based on this alone, he is prepared to say that China's economic strength is already
comparable to that of the United States. But China's creditor status does not make up for the fact that its
economy is presently less than half the size of the U nited States' and its people are barely one-tenth
as wealthy as Americans. Creditor status is also a misleading metric by which to judge China
because it is usually used to describe financially open economies, and China is largely closed .
Countries with open economies can invest their money in many places. Beijing, because it cannot spend its foreign
reserves at home, is forced to keep buying U.S. Treasury bonds. China's creditor status arises
largely from its weaknesses, not its strengths . The country's $3.2 trillion worth of foreign currency
holdings represents an imbalance between investment and consumption . Instead of loaning
money to rich countries, China should be importing capital in order to speed its domestic development and meet its
sizable needs, starting with properly capitalized pension and financial systems. China's financial books are strictly divided, with huge
assets in foreign currency (primarily dollars) on one side and huge liabilities in local currency on the other. Local governments
have incurred high debts by spending heavily on programs such as railroad expansion and by borrowing to fund the 2009
stimulus (which came mostly from local, not national, government coffers). Beijing should be paying down this debt
and addressing other domestic shortfalls with its mountain of foreign currency, but it cannot do so under its present
balance-of-payments rules, which are designed to keep foreign currency in the hands of the
national monetary authorities. Due to a closed capital account, domestic holders cannot send
money overseas, and foreign currency can be converted to yuan only through the state financial
system. The Chinese government has not let money flow freely because doing so would
undermine its control of domestic interest rates, reducing its ability to influence economic
cycles, and it would expose the domestic banking sector to devastating competition . If domestic
entities were allowed to send money abroad, hundreds of billions of dollars would flee the
country for financial institutions that operate commercially, unlike Chinese banks . Such a stark
fear of competition does not suggest a country ready to exert dominance anytime soon . Lastly,
Subramanian inflates China's financial influence over the United States, forgetting that influence in a buyer-seller relationship is
determined not by what-if scenarios but by who has better alternatives. TheUnited States has already diversified away
from Chinese debt by having the Federal Reserve flood the U.S. financial system with liquidity .
This is hardly ideal, but it has driven down the Chinese share of U.S. debt while keeping interest
rates historically low. In contrast, Beijing, despite its best efforts to diversify, still holds 70 percent of its
foreign currency reserves in dollars . The reason is simple: those reserves are so large and growing so
quickly that there is no alternative . The United States needs China to keep U.S. interest rates below historic norms;
China needs the United States to maintain its entire balance-of-payments system. Even if Subramanian acknowledges that China's
lopsided financial system is holding the country back now, he assumes that Beijing will soon rewrite its balance-of-payments rules
and become an open economy. This assumption underestimates the Communist Party's antipathy to change. In fact, the principal
advocate for such reforms has been Washington, which hopes to encourage China's transformation from an investment-led to a
consumption-led economy. Such a transition would undermine China's net creditor status--what Subramanian sees as its main claim
to dominance. But implementing market reforms would also allow China to keep growing at its blistering pace and surpass the
United States in GDP. If
China insists on maintaining government control over development , on the other
hand, its long-term growth prospects will be dim . Salvatore Babones ("The Middling Kingdom," September/October
2011) warns against drawing conclusions about China's trajectory by simply projecting its growth rates forward. Indeed, it is
entirely possible that Chinese GDP growth will simply stop. Growth depends on land, labor, capital, and
innovation. China has depleted its ecology, its labor surplus will soon begin to erode, and vast
overspending has driven down the return on capital --all discouraging trends from the standpoint
of growth. As for innovation, Subramanian praises China's growing technology sector and its ability to absorb new advances. But
a true economic leader must create, not absorb, and Beijing's favoritism toward large state firms will hinder
innovation. Moreover, the quality of the Chinese higher-education system is poor and not necessarily
improving. A no-growth scenario is a genuine danger--just ask the Japanese. By underemphasizing or
ignoring China's various weaknesses, Subramanian underestimates the United States' ability to
influence the competition with China. That said, his criticisms of the United States are valid; indeed, his baseline
prediction of U.S. growth at 2.5 percent annually may be too optimistic. Crippled by debt, the United States faces a period of
stagnation. If the overall economy remains sluggish, a lack of import growth will cause trade to lag and further reduce the United
States' global influence. Still, the
Chinese dragon will not fly forward indefinitely , as Subramanian suggests; it
may even crash. For the foreseeable future, China will not attain the kind of dominance the
United States has long held. The world should not expect to crown a new global leader but prepare
for the absence of one.

No China war

Goldstein 11 - Professor and Director of the China Maritime Studies Institute @ US Naval War College [Dr. Lyle J. Goldstein,
“Resetting the US–China Security Relationship,” Survival | vol. 53 no. 2 | April–May 2011 | pp. 89–116

Weighed in the aggregate, China’s rise remains a peaceful process, and the record to date should
engender significant confidence . Beijing has not resorted to a significant use of force against another
state in more than three decades . Its deployments of troops as UN peacekeepers to hot spots such as
Lebanon and the Democratic Republic of the Congo have played a helpful role , as have the counter-piracy operations of its
fleet in the Gulf of Aden. When dealing with weak and occasionally unstable states on its borders, such as
Kyrgyzstan or Tajikistan, Beijing has not resorted to military intervention , nor even flexed its military
muscles to gain advantage. Chinese maritime claims , whether in the South or the East China
seas, are generally being enforced by unarmed patrol cutters , a clear signal that Beijing does not
seek escalation to a major crisis on these matters. Contrary to the perception that China’s senior
military officers are all irreconcilable hawks , one influential People’s Liberation Army Navy (PLAN) admiral
recently said in an interview, with reference to lessons learned from recent border negotiations on China’s periphery: ‘ If there
are never any concessions or compromises, there is simply no possibility of reaching a
breakthrough in border negotiations.’2 pg. 90

No war or escalation- nuclear primacy prevents

Ross ‘5 Robert S., Staff Writer for the National Interest, Fall, (Assessing the China Threat. The National Interest. Lexis)

At the strategic level, after decades of research and testing, China is preparing to deploy solid-fuel ballistic missiles that can target
U.S. allies in East Asia and may be nearing completion of an intercontinental ballistic missile that can target the continental United
States. It is also making advances in development of its next-generation submarine-launched ballistic missiles. None of these
developments should come as a surprise; U.S. intelligence has been following these programs since their inception. Moreover,
these programs should not be considered a challenge to U.S. military superiority . Once these weapons
are fully operational, perhaps by the end of the decade, China will have a more credible minimal second-strike capability. Despite
recent Chinese bravado, not only is it hard to imagine a scenario in which China would use
nuclear weapons in response to conventional hostilities, but U.S. retaliatory capabilities would
make Chinese first-use suicidal. Continued modernization of its nuclear forces and massive
quantitative superiority over China give the U nited S tates a far more robust deterrent posture vis-a -vis
China than it ever possessed vis-a -vis the Soviet Union. Similarly, overwhelming U.S. nuclear superiority
provides greater strategic security for our East Asian allies than U.S. nuclear capabilities ever
provided for our European allies during the Cold War.
---China War- Peaceful Rise

1NC Rosecrance—no risk of US-China war—peaceful rise means accommodation rather than
conflict to preserve economic gains—China wants to rise through trade and economics—
engaged in liberal world order—prefer our evidence, it’s a joint study composed of multiple
policy experts from both countries

Economic rise prevents war—no brinksmanship, no flashpoints

Bremmer 10 [Ian, president of Eurasia Group and the author, most recently, of The End of the Free Market: Who Wins the
War between States and Corporations?, Gathering Storm: America and China in 2020 July/August 2010
http://www.worldaffairsjournal.org/articles/2010-JulyAugust/full-Bremmer-JA-2010.html]

In addition, Beijing has no incentive to mount a global military challenge to U.S. power. China will
one day possess a much more substantial military capacity than it has today, but its
economy has grown so quickly over
the past two decades, and its living standards improved so dramatically , that it is difficult to imagine the
kind of catastrophic, game-changing event that would push Beijing to risk it all by posing the West a
large-scale military challenge. It has no incentive to allow anything less than the most serious
threat to its sovereignty to trigger a military conflict that might sever its expanding network of commercial
ties with countries all over the world—and with the United States, the European Union, and Japan, in particular. The more
familiar flash points are especially unlikely to spark a hot war: Beijing is well aware that no
U.S. government will support a Taiwanese bid for independence, and China need not invade an island
that it has largely co-opted already , via an offer to much of Taiwan’s business elite of privileged access to investment
opportunities on the mainland.

Diplomatic ties check

Eland 5 (Ivan, Senior Fellow and Director of the Center on Peace and Liberty at the Independent Institute and Former Director of
Defense Policy Studies at the Cato Institute, “Coexisting with a Rising China,” 4-12, http://www.antiwar.com/eland/?articleid=5522)

The Bush administration is often guilty of running a reckless, overly militaristic foreign policy but deserves qualified praise for its
recent dealings with China. The
Chinese have requested—and the United States has accepted—a
regular dialogue at senior levels to discuss security , political, and possibly economic issues.
But the administration must go farther than merely symbolic meetings in accepting China’s rise—it must translate that new-
found respect into real world actions. Unlike the Bush administration’s threatening behavior to smaller countries, such as Iraq
and Taliban-controlled Afghanistan, the bark of the Bush administration’s policy toward the nuclear-armed China has always
exceeded its bite. President Bush took office and stridently labeled China a “strategic competitor,” but then a few months later
essentially apologized and paid ransom to get back a U.S. flight crew and spy plane, which was harassed and damaged by
Chinese fighters in international airspace. Subsequently, after 9/11, China and the United States have been
cooperating more closely. Improved relations between the two powers are a very positive development for global
security. Regular high-level meetings are important for two reasons. First, such talks provide a forum for two nuclear-
armed powers to nip tensions or problems in the bud before they turn into crises . Second, and
most important, such meetings signal that the status quo superpower has respect for the rising East Asian power. Prior to 1914,
Britain failed to acknowledge the new prestige of a rising Germany, one contributing factor to the horrific and unnecessary First
World War. China, with a rapidly growing economy and a huge population, desperately wants to be recognized as a great power
by the United States and the world.

Economics check—China’s rational

Brezinski 5 (Zbigniew, Counselor at the CSIS, Foreign Policy, Jan/Feb, http://www.worldthreats.com/Asia/Clash%20of


%20Titans.htm)

Today in East Asia, China is rising—peacefully so far. For understandable reasons, China harbors resentment and even
humiliation about some chapters of its history. Nationalism is an important force, and there are serious grievances regarding
external issues, notably Taiwan. But conflict is not inevitable or even likely. China's leadership is not inclined to
challenge the United States militarily, and its focus remains on economic development and winning acceptance
as a great power. China is preoccupied, and almost fascinated, with the trajectory of its own ascent. When I met with the top
leadership not long ago, what struck me was the frequency with which I was asked for predictions about the next 15 or 20 years.
Not long ago, the Chinese Politburo invited two distinguished, Western-trained professors to a special meeting. Their task was
to analyze nine major powers since the 15th century to see why they rose and fell. It's an interesting exercise for the top
leadership of a massive and complex country. This focus on the experience of past great powers could lead to the conclusion
that the iron laws of political theory and history point to some inevitable collision or conflict. But there are other political
realities. In the next five years, China will host several events that will restrain the conduct of its foreign policy. The 2008
Olympic Games is the most important, of course. The scale of the economic and psychological investment in the Beijing games is
staggering. My expectation is that they will be magnificently organized. And make no mistake, China intends to win at the
Olympics. A second date is 2010, when China will hold the World Expo in Shanghai. Successfully organizing these international
gatherings is important to China and suggests that a cautious foreign policy will prevail. More broadly, China
is
determined to sustain its economic growth. A confrontational foreign policy could disrupt
that growth, harm hundreds of millions of Chinese, and threaten the Communist Party's
hold on power. China's leadership appears rational, calculating, and conscious not only of
China's rise but also of its continued weakness. There will be inevitable frictions as China's regional role increases and as a
Chinese "sphere of influence" develops. U.S. power may recede gradually in the coming years, and the unavoidable decline in
Japan's influence will heighten the sense of China's regional preeminence. But to have a real collision, China needs a military
that is capable of going toe-to-toe with the United States. At the strategic level, China maintains a posture of minimum
deterrence. Forty years after acquiring nuclear-weapons technology, China has just 24 ballistic missiles capable of hitting the
United States. Even beyond the realm of strategic warfare, a country must have the capacity to attain its political objectives
before it will engage in limited war. It is hard to envisage how China could promote its objectives when it is acutely vulnerable to
a blockade and isolation enforced by the United States. In a conflict, Chinese maritime trade would stop entirely. The flow of oil
would cease, and the Chinese economy would be paralyzed.

War unthinkable—cooler heads prevail on both sides

Guangqian ‘7 (Peng Guangqian, special researcher with China Institute of International Studies, 6-5-07 [CHINA POSES NO
THREAT IN POST-COLD WAR WORLD, lexis]

China's military development and its defense modernization drive, which are tuned to a
moderate pace , are defensive in nature. China does not challenge anyone, nor does its military
strength pose a threat to anyone. In the nuclear era, it is impossible to imagine that any country
could rise by resorting to military means. A wide gap exists between Chinese and US military
strength. China is not foolish enough to challenge the position of Uncle Sam by using force. So, the
theory that China's military power constitutes a threat to the US is at the very least based on
ignorance, if not on ill intent. It is also impossible to imagine that the US could get away with
using force to rob China of its right to peaceful development without paying a price in the nuclear
era. In the Cold War era, the United States and the Soviet Union maintained a balance of terror based on nuclear parity. In the Post-
Cold War period, nuclear parity has lost its significance in the face of the overkill power of over-stockpiled nuclear weapons. As a
result, the balance of nuclear strike effects is replacing the equilibrium of nuclear strength. So, no country can emerge a victor from
a nuclear confrontation. China and the US have no reason to be locked in confrontation, let alone
nuclear confrontation. We should be on the alert against those with ulterior motives who are trying to lure the two
countries into confrontation US Defense Secretary Robert Gates stated clearly at a news briefing on March 7
that he did not regard China as the United States' strategic foe and that engagement with China
in various areas was very important. General Pace remarked, in his meeting with the leaders of China's
Nanjing Military Command on March 23, that the US and China both had strong military strength but
neither party wanted to go to war with the other. He went on to say that he did not see any
threat from China. He also remarked that the two countries should not focus on how to fight a war but should focus on how
to prevent war. This is quite to the point. Pushed by far-sighted Chinese and American politicians and
military leaders, Sino-US military ties are showing signs of strong momentum . Apart from that, military
leaders from both countries maintain ever closer liaison; recently high-level military visits have been frequent ;
and the Chinese and US navies have staged a joint maritime search-and-rescue exercise. The
Chinese and US military are discussing establishing a hotline.

China will focus on growth and trade first - zero risk of conflict

Mulvenon et al 5 (James, RAND Corporation, “China on the Move: A Franco-American Analysis of Emerging Chinese
Strategic Policies and Their Consequences for Transatlantic Relations”
http://www.rand.org/pubs/conf_proceedings/2005/RAND_CF199.pdf)

Apart from the specific circumstances that suggest value in cooperation with the United States, the
Chinese undoubtedly
understand that sustainable growth will both require and foster growing economic
interdependence between China and America. The two economies are quite complementary:
America the source of new technology and insatiable consumer demand, and China an engine of production with a seemingly
inexhaustible labor supply. True, this growing economic interdependence constrains the United States as well as China, which
might embolden the Chinese to be less compliant. At the same time, awareness that the United States has an
immense economic stake in China might cause the Chinese to feel that challenging the
United States politically and militarily is not only fundamentally unwise but also fundamentally
unnecessary. Opting to expand cooperation with the United States for the long haul would enable China to avoid a massive
military buildup and thus to concentrate investment on internal development. At the same time, the Chinese can be expected to
continue to expand their military capabilities, especially those relevant to the United States and Taiwan--their most powerful
potential adversary and their most coveted symbol of national unity, respectively. Military modernization is not incompatible
with a strategy of long-term political cooperation. Indeed, it could be viewed as important both as a hedge and as a way to
avoid having to cooperate from a severely inferior position.
---China War- Deterrence/Escalation

1NC Ross Zenko and Liff—China deterred by US primacy—nuclear escalation unthinkable


because modernization and quantitative superiority make strikes a suicide mission for China—
at best they access a conventional war, which we’ll outweigh

High costs across board means no escalation

Saunders, 9 —Senior research professor @ National Defense University (Phillip, Managing strategic competition with
China, http://findarticles.com/p/articles/mi_m0QZY/is_242/ai_n35574297/?tag=content;col1_ )

The impact of greater U.S. military vulnerability on U.S. and Chinese behavior may rest somewhat on the balance of vulnerability
at the strategic, operational, and weapons system levels. At
the strategic level, factors such as extensive
economic interdependence, the importance of Sino-U.S. relations for regional and global
stability, and the potential for nuclear escalation mean that a major U.S.-China military
conflict would impose high absolute costs and risks on both countries, regardless of the
eventual military outcome. The high absolute costs of a conflict to both sides will likely dwarf
considerations about who would suffer more, making leaders in Washington and Beijing
cautious and extremely reluctant to authorize the use of force in situations that might
escalate into a broader war. These considerations do not make war impossible, but they are likely to produce more
restrained behavior and may dampen competitive military dynamics to some degree

No draw-in arguments—China has no willing allies

Record ‘1 Jeffrey Record, Professor at the US Air War College, Senior Fellow at CISP, Institute for Foreign Policy Analysis at
Brookings, Winter, 2001 (Thinking About China and War. Aerospace Power Journal. Infotrac)

In addition to naval and air inferiority, China


would approach war with the U nited States with significant
strategic disadvantages. Regionwide suspicion of China's imperial ambitions has deprived
Beijing of significant allies and even friends in East Asia, whereas the U nited S tates is rich in both.
India remains a strategic competitor, and Chinese behavior in the South China Sea has alienated most of Southeast
Asia. The post--Cold War rapprochement between China and Russia has not eliminated centuries-
old national and racial animosities between the two countries, animosities that can be heightened only by
the growth of Chinese economic influence and demographic "aggression" in the RFE. In any event, Russian military power
has virtually evaporated in Asia. A robust, land-based strategic nuclear deterrent is the only real asset that Moscow
could make available to China in a Sino-American war, but it staggers the mind to imagine that Russia would
invite its own destruction on behalf of promoting Chinese interests in East Asia.

At worst, war is limited—constraints on both sides


Record ‘1 Jeffrey Record, Professor at the US Air War College, Senior Fellow at CISP, Institute for Foreign Policy Analysis at
Brookings, Winter, 2001 (Thinking About China and War. Aerospace Power Journal. Infotrac)

Assuming the absence of mindless escalation to a general nuclear exchange, a war between
China and the U nited S tates would be constrained by limited military capacity and political
objectives. For openers, neither China nor the U nited S tates is capable of invading and subjugating the
other, and even if the U nited S tates had the ability to do so, avoidance of a land war on the Asian
mainland has long been an injunction of American strategy. The objectives of a Sino-American
war over Taiwan or freedom of navigation in the South China Sea would be limited-just as they were in the
Sino-American war in Korea. And since the outcome in either case would be decided by naval and air
forces, with regular ground forces relegated to a distinctly secondary role, a war over Taiwan or the
South China Sea would also be limited in terms of the type of force employed. This was not the case in the
Korean War, in which ground combat dominated. (To be sure, the US position on the ground would have been untenable without air
dominance.)

Even if they win nuclear use, US first strikes mean no escalation and limited scope

Lieber and Press ‘7 (Keir A. Lieber is an assistant professor of political science at the University of Notre Dame and the author
of War and the Engineers: The Primacy of Politics Over Technology (2005). Daryl G. Press has worked as a consultant on military
analysis projects for the U.S. Department of Defense for 13 years, and he is an associate professor of government at Dartmouth
College, July 1, 2007, (The Atlantic Monthly, HEADLINE: Superiority complex: why America's growing nuclear supremacy may make
war with China more likely, p. Lexis)

From a military perspective, this modernization has paid off: A U.S. nuclear
first strike could quickly destroy
China's strategic nuclear arsenal. Whether launched in peacetime or during a crisis , a preemptive
strike would likely leave China with no means of nuclear retaliation against American territory.
And given the trends in both arsenals, China may live under the shadow of U.S. nuclear primacy for years to come . This assessment is
based on unclassified information, standard targeting principles, and formulas that defense analysts have used for decades. (And we systematically chose conservative estimates
for key unknowns, meaning that our analysis understates U.S. counterforce capabilities.) The simplest version of an American preemptive strike would have nuclear-armed
submarines in the Pacific launch Trident II missiles at the Chinese ICBM field in Henan province. The Navy keeps at least two of these submarines on "nard alert" in the
Pacific at all times, meaning they're ready to fire within 15 minutes of a launch order. Since each submarine
carries 24 nuclear-tipped missiles with an average of six warheads per missile, commanders have
almost 300 warheads ready for immediate use. This is more than enough to assign multiple
warheads to each of the 18 Chinese silos . Chinese leaders would have little or no warning of the
attack. During the Cold War, U.S. submarines posed little danger to China's silos, or to any other hardened targets. Each warhead on the Trident I missiles had little chance--roughly 12 percent--of success.
Not only were those missiles inaccurate, their warheads had a relatively small yield. (Similarly, until the late 1980s, U.S. ICBMs lacked the accuracy to carry out a reliable disarming attack against China.) But the
Navy's new warheads and missiles are far more lethal. A Trident II missile is so accurate, and the newer W88 warhead so powerful, thatif the warhead and missile function normally, the destruction of thesilo is
virtually assured (the likelihood is calculated as greater than 99 percent). In reality, American planners could not assume such near-perfect results. Some missiles or warheads could malfunction: One missile's

So a
realistic counterforce plan might assign four warheads to
rockets might fail to ignite; another's guidance system might be defective.

each silo. The U.S. would "cross-target" the missiles, meaning that the warheads on each missile
would each go to different silos, so that a silo would be spared only if many missiles
malfunctioned. Even assuming that 20 percent of missiles malfunctioned --the standard, conservative
assumption typically used by nuclear analysts--there is a 97 percent chance that every Chinese DF-5 silo
would be destroyed in a 4-on-1 attack. (By comparison, a similar attack using Cold War-era Trident I missiles would have produced less than a 1 percent
chance of success. The leap in American counterforce capabilities since the end ofthe Cold War is staggering.) Beyond bolstering the ability to conduct a first strike, the
improvements to U.S. counterforce weapons also allow war planners to design nuclear options that will make the weapons more "usable" during high-stakes crises. Nuclear
planners face many choices when they consider striking a given target. First, they must choose a warhead yield. The American arsenal includes low-yield weapons such as the B-
61 bomb,which can detonate with as little explosive force as 0.3 kilotons (one-fiftieth the power of the bomb that destroyed Hiroshima), and high-yield weapons such as the B-
83 bomb, which can yield 1,200 kilotons (80 times the strength of the Hiroshima bomb). For a military planner, high-yield weapons are attractive because they're very likely to
Low-yield warheads, on the other hand, can be more discriminating, if
destroy the target--even if the weapon misses by some distance.

planners want to minimize civilian casualties . A second key decision for war planners is whether to set the weapon to
detonate at ground level or in the air above the target. A groundburst creates enormous overpressure and ground shock, ideal for
destroying a hardened target. But groundbursts also create a lot of radioactive fallout. Dirt and other matter is sucked up into the
mushroom cloud, mixes with radioactive material, and, after being carried by the wind, falls to earth in the hours after the blast,
spreading lethal radiation. Airbursts create smaller zones of extremely high overpressure, butthey also generate very little
fallout. If the detonation occurs above a threshold altitude (which depends on the weapon yield), virtually no heavy particles from the ground mix with the radioactive material in the fireball. The
In the
radioactive material rises into the high atmosphere and then falls to earth over the course of several weeks in a far less dangerous state and over a very wide area, greatly reducing the harm to civilians.

past, a nuclear attack on China's arsenal would have had horrific humanitarian consequences. The weapons were less accurate, so an
effective strike would have required multiple high-yield warheads,detonating on the ground, against each target . The Federation of American
Scientists and the Natural Resources Defense Council modeled the consequences of such an attack--similar to the submarine attack described above--and published their findings in 2006. The

results were sobering. Although Chinas long-range missiles are deployed in a lightly populated region, lethal fallout from an attack would travel hundreds of miles and kill more than 3 million

Chinese civilians. Americanleaders might have contemplated such a strike, but only in the most dire circumstances. But things are changing radically . Improved accuracy now

allows war planners to target hardened sites with low-yield warheads and even airbursts . And the United States is pushing its breakthroughs in accuracy even
further. For example, for many years America has used global-positioning systems in conjunction with onboard inertial-guidance systems to improve the accuracy of its conventionally armed (that is, nonnuclear)
cruise missiles. Although an adversary may jam the GPS signal near likely targets, the cruise missiles use GPS along their flight route and then--if they lose the signal--use their backup inertial-guidance system for
the final few kilometers. This approach has dramatically improved a cruise missile's accuracy and could be applied to nuclear-armed cruise missiles as well. The United States is deploying jam-resistant GPS
The
receivers on other weapons, experimenting with GPS on its nuclear-armed ballistic missiles, and planning to deploy a new generation of GPS satellites--with higher-powered signals to complicate jamming.

payoff for equipping cruise missiles (or nuclear bombs) with GPS is clear when one estimates the civilian casualties from a lower-yield,
airburst attack. We asked Matthew McKinzie, a scientific consultant to the N atural R esources D efense C ouncil and coauthor of the 2006 study, to rerun the
analysis using low-yield detonations compatible with nuclear weapons currently in the U.S. arsenal . Using three warheads per target to increase the odds
of destroying every silo, the model predicts fewer than 1,000 Chinese casualties from fallout. In some low-yield scenarios, fewer than 100

Chinese would be killed or injured from fallout. The model is better suited to predicting fallout casualties than to forecasting deaths from the blast and fire, but given the low
population in the rural region where the silos are, Chinese
fatalities would be fewer than 6,000 in even the
most destructive scenario we modeled. And in the future, there may be reliable nonnuclear options for destroying
Chinese silos. Freed from the burden of killing millions, a U.S. president staring at the threat of a
Chinese nuclear attack on U.S. forces, allies, or territory might be more inclined to choose preemptive action .
---China War- AT: Glaser

Glaser agrees deterrence checks

Glaser 11 (Charles, Professor of PoliSci and International Affairs and Director of the Institute for Security and Conflict
Studies @ George Washington University, “Will China’s Rise Lead to War?” March/April Foreign Affairs,

What does all this imply about the rise of China? At the broadest level, the news is good. Current
international
conditions should enable both the United States and China to protect their vital interests
without posing large threats to each other. Nuclear weapons make it relatively easy for major powers to
maintain highly effective deterrent forces. Even if Chinese power were to greatly exceed U.S. power
somewhere down the road, the United States would still be able to maintain nuclear forces that
could survive any Chinese attack and threaten massive damage in retaliation. Large-scale
conventional attack by China against the U.S. homeland, meanwhile, are virtually impossible because the
United States and China are separated by the vast expanse of the Pacific Ocean, across which it would be difficult to
attack. No foreseeable increase in China’s power would be large enough to overcome these
twin advantages of defense for the United States. The same defensive advantages, moreover,
apply to China as well. Although China is currently much weaker than the United States militarily, it will soon be able
to build a nuclear force that meets its requirements for deterrence . And China should not find the
United States’ massive conventional capabilities especially threatening, because the bulk of U.S. forces, logistics, and support lie
these conditions is to greatly moderate the security dilemma . Both
across the Pacific. The overall effect of
the United States and China will be able to maintain high levels of security now and through any
potential rise of China to superpower status. This should help Washington and Beijing avoid truly strained geopolitical
relations, which should in turn help ensure that the security dilemma stays moderate, thereby facilitating cooperation. The
United States, for example, will have the option to forego responding to China’s modernization of its nuclear force. This
restraint will help reassure China that the United States does not want to threaten its security - and thus help head off
a downward political spiral fueled by nuclear competition.
---China War- AT: Resource War

No China resource wars

Barnett ’11 (10/17/11 (Thomas P.M., chief analyst at Wikistrat and a contributing editor for Esquire magazine,“The New Rules:
Debunking the Pentagon’s Chinese Nationalism Hype”, http://www.worldpoliticsreview.com/articles/10354/the-new-rules-
debunking-the-pentagons-chinese-nationalism-hype)

There exists within the Pentagon an unshakeable line of reasoning that says the Chinese military
threat to the United States in Asia is profound and growing, that the most likely great-power war
conflict will be over Taiwan or the South China Sea, and that the primary trigger will be China's burgeoning -- and
uncontrollable -- nationalism. Objectively, China's military capabilities are certainly growing
dramatically, but our conventional wisdom tends to break down in the structural plausibility of the
scenarios. That's why the firm belief that rampant nationalism will trigger an eventual conflict becomes so crucial, especially
when considered in combination with an additional line of speculation that emerged earlier this year, after the Chinese military
trotted out a fifth-generation fighter jet the same day that former U.S. Defense Secretary Robert Gates arrived in Beijing for
confidence-building talks: At the time, Gates suggested that maybe the People's Liberation Army was getting too big for its britches,
and according to those who emphasize the Chinese threat, when the C hinese Communist Party eventually
caves in the face of out-of-control popular nationalism, the PLA will step in and take matters into its own
hands. Clearly, the U.S. Navy and Air Force desperately need a big-war foe in China so they can do
effective bureaucratic battle against the Army and Marines in the tight budgetary years ahead. But beyond
that cynical dynamic, we can't really come up with any objective reasons why China would seek war
with the U.S. As a result, we reduce our strategic thinking to scenarios driven by sheer emotion.
First, let me dismiss the straw men. China doesn't need to do battle with the U.S. over allegedly "dwindling" world
resources because, quite frankly, it's got the money to buy whatever it needs . To the extent that it suffers
strategic vulnerability over unstable source countries, America has historically been more than happy to step in on the world's behalf
to stabilize things.
If that U.S. role diminishes over time , as most observers expect it will, then China will be
forced at least to consider taking on more security responsibilities in these fragile zones. But
swapping out American "quagmires" for Chinese ones doesn't exactly portend conflict between
the two superpowers. Applying the same conflict logic to food and water, certainly the fact that China must support 22
percent of the world's population with just 7 percent of its water supply puts Beijing at a strategic disadvantage. The only problem
here is that America is highly incentivized to exploit that growing vulnerability simply by directing more of
its voluminous agricultural exports to China , seeing as we, along with Canada, are essentially the Saudi Arabia of grain
exporters. Across the spectrum of strategic resources, the resource-war equation doesn't add up when it comes to
China and the United States given the tremendous degree of financial interdependency we already "suffer."
But if that line doesn't convince, "China threat" alarmists can always turn to scenarios forecasting Chinese domination of Asia.
However, here, too, the strategic logic breaks down, as China
has no need to resort to bullying to penetrate
regional markets -- it is already accomplishing just that with some ease. And longer term, China
needs those regional markets to flourish , for as its own population rapidly ages and its economic trajectory slows
along the normal "S-curve" lines, its primary advantage over the U.S. will be that its neighbors are likely to grow more dynamically
than ours. Then there's the additional reality that China will end up exporting many of its low-end manufacturing jobs to these same
countries as their "demographic dividend" surpasses China's now-historically spent version. The ease with which these threat
scenarios can be dismissed explains why the Pentagon is left hanging its big-war hopes on Chinese nationalism exceeding the
Chinese Communist Party's ability to keep it in check. China will go to war with America because its leaders have no choice: better to
sacrifice the U.S. to the unruly "dragon" that is the Chinese people than to go down themselves trying to tame it. For some insight on
the domestic component of this particular equation, James Reilly's new book, "Strong Society, Smart State," is noteworthy. Reilly
examines how Beijing has handled the potentially explosive issue of its Japan policy over the past decade or so of China's meteoric
economic rise. And if you buy the notion that China's rise will fuel a barely controllable nationalism, you can't pick a better test case
than Japan for examining how the party manages that dynamic, for there is no other country in the world with which China has had
a more conflicted recent past -- remembering that Chinese culture does not forget past "humiliations." As Reilly notes, in the early
2000s, Chinese society experienced a great wave of anti-Japan nationalism that led to popular protests and all manner of sensational
media coverage. For a good stretch, the government went along with the populist outpouring, essentially allowing it to vent. But
once it began damaging bilateral diplomatic negotiations and economic cooperation, the party's leadership stepped in to
simultaneously curb the worst excesses while persuading the public toward a less nefarious view. Dubbing this approach "responsive
authoritarianism," Reilly persuasively argues that the West underestimates Beijing's soft-power capabilities when dealing with its
domestic constituency. Reilly not only makes a convincing case for appreciating Beijing's capacity for molding public opinion on
Japan in recent years, he lays out the regime's fairly nuanced approach for doing the same "across the range of China's major power
relations." Among other examples, he cites similarly deft opinion-management responses to the anti-Chinese violence that erupted
in Indonesia in 1998, as well as to the series of U.S. Chinese clashes starting with the bombing of the Chinese embassy in Belgrade in
1999 through the spy plane incident of 2001. Reilly concludes that a sort of cyclical modus vivendi has been reached between the
party and the public, which should be strangely comforting for both skeptics and champions of China's future democratization. How
long can this continue? With roughly three-quarters of a billion Chinese still trapped in poverty, we could easily be looking at
another generation before China has advanced economically to the point where true democratization makes sense. While that may
seem like an eternity to those who expect the party to begin liberalizing immediately, it would put China in the norm for Asia,
compared to similar transitions in modern Japan, South Korea, Indonesia, Malaysia, the Philippines and even recently awakening
Singapore. But in the event of an economic "hard landing" any time in the near future, would the pressure from China's nationalism
then become too much for the party to handle, requiring the opening of a release valve in the form of external aggression? Here, the
longer historical record is rather clear: When China suffers economic downturns, its foreign policy has tended to become more
conservative and even extremely careful. In the meantime, given our harsh fiscal realities and the messy world we still find ourselves
largely managing on our own militarily, the U.S. Defense Department needs to dial down its growing -- and costly -- fixation with the
Chinese threat.
---China War- AT: Nationalism

No China threat

Barnett ’11 (10/17/11 (Thomas P.M., chief analyst at Wikistrat and a contributing editor for Esquire magazine,“The New Rules:
Debunking the Pentagon’s Chinese Nationalism Hype”, http://www.worldpoliticsreview.com/articles/10354/the-new-rules-
debunking-the-pentagons-chinese-nationalism-hype)

There exists within the Pentagon an unshakeable line of reasoning that says the Chinese military
threat to the United States in Asia is profound and growing, that the most likely great-power war
conflict will be over Taiwan or the South China Sea, and that the primary trigger will be China's burgeoning -- and
uncontrollable -- nationalism. Objectively, China's military capabilities are certainly growing
dramatically, but our conventional wisdom tends to break down in the structural plausibility of the scenarios. That's
why the firm belief that rampant nationalism will trigger an eventual conflict becomes so crucial, especially when considered in
combination with an additional line of speculation that emerged earlier this year, after the Chinese military trotted out a fifth-
generation fighter jet the same day that former U.S. Defense Secretary Robert Gates arrived in Beijing for confidence-building talks:
At the time, Gates suggested that maybe the People's Liberation Army was getting too big for its britches, and according
to
those who emphasize the Chinese threat, when the C hinese Communist Party eventually caves in the
face of out-of-control popular nationalism, the PLA will step in and take matters into its own hands. Clearly,
the U.S. Navy and Air Force desperately need a big-war foe in China so they can do effective
bureaucratic battle against the Army and Marines in the tight budgetary years ahead. But beyond that cynical
dynamic, we can't really come up with any objective reasons why China would seek war with the
U.S. As a result, we reduce our strategic thinking to scenarios driven by sheer emotion. First, let me
dismiss the straw men. China doesn't need to do battle with the U.S. over allegedly "dwindling" world resources because, quite
frankly, it's got the money to buy whatever it needs. To the extent that it suffers strategic vulnerability over unstable source
countries, America has historically been more than happy to step in on the world's behalf to stabilize things. If that U.S. role
diminishes over time, as most observers expect it will, then China will be forced at least to consider taking on more security
responsibilities in these fragile zones. But swapping out American "quagmires" for Chinese ones doesn't exactly portend conflict
between the two superpowers. Applying the same conflict logic to food and water, certainly the fact that China must support 22
percent of the world's population with just 7 percent of its water supply puts Beijing at a strategic disadvantage. The only problem
here is that America is highly incentivized to exploit that growing vulnerability simply by directing more of its voluminous agricultural
exports to China, seeing as we, along with Canada, are essentially the Saudi Arabia of grain exporters. Across the spectrum of
strategic resources, the resource-war equation doesn't add up when it comes to China and the United States given the tremendous
degree of financial interdependency we already "suffer." But if that line doesn't convince, "China threat" alarmists can always turn to
scenarios forecasting Chinese domination of Asia. However, here, too, the strategic logic breaks down, as China has no need to
resort to bullying to penetrate regional markets -- it is already accomplishing just that with some ease. And longer term, China needs
those regional markets to flourish, for as its own population rapidly ages and its economic trajectory slows along the normal "S-
curve" lines, its primary advantage over the U.S. will be that its neighbors are likely to grow more dynamically than ours. Then
there's the additional reality that China will end up exporting many of its low-end manufacturing jobs to these same countries as
their "demographic dividend" surpasses China's now-historically spent version. The ease with which these threat scenarios can be
dismissed explains why the Pentagon is left hanging its big-war hopes on Chinese nationalism exceeding the Chinese Communist
Party's ability to keep it in check. China will go to war with America because its leaders have no choice: better to sacrifice the U.S. to
the unruly "dragon" that is the Chinese people than to go down themselves trying to tame it. For some insight on the domestic
component of this particular equation, James Reilly's new book, "Strong Society, Smart State," is noteworthy. Reilly examines how
Beijing has handled the potentially explosive issue of its Japan policy over the past decade or so of China's meteoric economic rise.
And if you buy the notion that China's rise will fuel a barely controllable nationalism, you can't
pick a better test case than Japan for examining how the party manages that dynamic, for there
is no other country in the world with which China has had a more conflicted recent past --
remembering that Chinese culture does not forget past "humiliations." As Reilly notes, in the
early 2000s, Chinese society experienced a great wave of anti-Japan nationalism that led to
popular protests and all manner of sensational media coverage. For a good stretch, the
government went along with the populist outpouring, essentially allowing it to vent. But once it
began damaging bilateral diplomatic negotiations and economic cooperation, the party's leadership
stepped in to simultaneously curb the worst excesses while persuading the public toward a less
nefarious view. Dubbing this approach "responsive authoritarianism," Reilly persuasively argues
that the West underestimates Beijing's soft-power capabilities when dealing with its domestic
constituency. Reilly not only makes a convincing case for appreciating Beijing's capacity for molding public opinion on Japan in
recent years, he lays out the regime's fairly nuanced approach for doing the same "across the range of China's major power
relations." Among other examples, he cites similarly deft opinion-management responses to the anti-Chinese violence that erupted
in Indonesia in 1998, as well as to the series of U.S. Chinese clashes starting with the bombing of the Chinese embassy in Belgrade in
1999 through the spy plane incident of 2001. Reilly concludes that a sort of cyclical modus vivendi has been reached between the
party and the public, which should be strangely comforting for both skeptics and champions of China's future democratization. How
long can this continue? With roughly three-quarters of a billion Chinese still trapped in poverty, we could easily be looking at
another generation before China has advanced economically to the point where true democratization makes sense. While that may
seem like an eternity to those who expect the party to begin liberalizing immediately, it would put China in the norm for Asia,
compared to similar transitions in modern Japan, South Korea, Indonesia, Malaysia, the Philippines and even recently awakening
Singapore. But in
the event of an economic "hard landing" any time in the near future , would the
pressure from China's nationalism then become too much for the party to handle , requiring the
opening of a release valve in the form of external aggression? Here, the longer historical record is
rather clear : When China suffers economic downturns, its foreign policy has tended to
become more conservative and even extremely careful . In the meantime, given our harsh fiscal
realities and the messy world we still find ourselves largely managing on our own militarily, the
U.S. Defense Department needs to dial down its growing -- and costly -- fixation with the Chinese
threat.
---China War- AT: Spratley’s
No escalation—EP-3 and Impeccable proves.

Womack 11 – Professor of Foreign Affairs @ University of Virginia [Dr. Brantly Womack (PhD in Poli Sci from University of
Chicago), “The Spratlys: From Dangerous Ground to Apple of Discord,” Contemporary Southeast Asia: A Journal of International and
Strategic Affairs, Volume 33, Number 3, December 2011, pp. 370-387

It is difficult to imagine a Spratly scenario in which a crisis would go beyond a specific incident
and threaten the current overall pattern of mixed occupation. Accidents happen, so incidents cannot be ruled out, though the
sustained confrontation of two or more militaries are increasingly unlikely. Accidental incidents are
likely to lead to a blamestorm, but not to prolonged conflict or to escalation . A premeditated fait
accompli against other claimants, as argued earlier, would not accomplish much. The victor (let us assume China)
would have alienated the entire region and it would have alarmed the rest of its neighbours and
international partners. International cooperation in resource development would be unlikely, and the
logistics of transportation, supply and defence would be formidable . If China’s overall foreign policy made a
radical change towards aggressive regional hegemony perhaps the Spratlys could become a battleground. But the ramp-up in
aggressiveness would take time to develop, Spratly controversies would be derivative rather than the leading element, and there
would no longer be a need for a synecdoche of anxiety. The currently foreseeable future is based on a quarter
century of broad and peaceful development/// in which the Spratlys have been a grain of sand .
A militarized incident in the South China Sea between China and the United States is more likely, but it is not
likely to originate in the Spratlys nor is it likely to escalate . The direct confrontation has been over the
definition of innocent passage in the context of freedom of navigation in EEZs, and an incident in the Spratlys is unlikely
to generate a restriction of general freedom of navigation since traffic goes around the islands rather
than through them. Incidents such as those involving the EP-3 surveillance aircraft incident of April
2001 or the USNS Impeccable hydrographic ship in March 2009 are possible, but these do not relate specifically to
the Spratlys and are only indirectly related to Southeast Asia. It would be surprising if Southeast Asian states would be happy with an
American solution that would consider intelligence operations (by China as well as by the United States) legitimate up to a twelve
mile limit. The reverberations
from such incidents are likely to be restricted to tit-for-tat responses
rather than general escalation. The days of the War of Jenkins’s Ear are long past .35 pg. 381-383
China Taiwan War Answers
Frontline

Economics and relations prevent China-Taiwan war

Rosenberg 9 (David, Professor of Political Science – Middlebury College and Research Fellow at the Research School of Pacific
and Asian Studies – Australian National University, “Dire Straits: Competing Security Priorities in the South China Sea”, The Asia-
Pacific Journal, 3-20, http://japanfocus.org/-David-Rosenberg/1773)

There is a curious pattern of accommodation in PRC-Taiwan relations. On the one hand, the PRC views
Taiwan as a renegade province while Taiwan views the mainland with cultural empathy but political disdain. On many
South China Sea issues, however, they are often in agreement. They have not had any direct
confrontations in the South China Sea. They make the same claims, use the same definitions, baselines, and maps in
stating their interests in the region. There is even some direct cooperation between China and Taiwan on technical issues.
Beyond these governmental links, there are very substantial corporate and personal links between
China and Taiwan. Taiwanese firms have invested over US $100 billion on the mainland, more than any other country.
Much of this involves the relocation of Taiwanese industries to the Shanghai-Suzhou and Fujian areas. To a large extent,
Taiwan's continued economic prosperity is tied to reintegration with the mainland . These
economic links of investment and trade are reinforced by millions of personal visits as well as mail and email
correspondence. Bonds of marriage also strengthen these ties. Nearly 10% of Taiwanese men marry mainland brides, further
tying migrant generations to ancestral origins. These deeply-rooted, long-term economic and demographic trends provide a
counterbalance to the often strident political clashes. The longer and broader the cross-Strait engagement, the better the
prospects for peaceful coexistence. Unfortunately, the cross-Strait issue has become immersed in domestic politics in Taiwan
and China. The recent spate of threats and counter-threats over Taiwan's status is linked to maneuvering among domestic
political forces seeking popular support. For example, in March 2005, after China passed its anti-secession law, there were
widespread protest demonstrations in Taiwan led by Prime Minister Chen Shui-bian's Democratic Progressive Party (DPP). Soon
after, a large delegation of Taiwan's main opposition party, the Kuomintang (KMT, Nationalist Party), visited the mainland to
encourage trade and political dialogue with China and to pay respects to the memorial shrine of Sun Yat-sen, KMT's founder.
This, in turn, was followed in early April by the visit of right-wing Taiwan Solidarity Union party leaders to the Yasukuni shrine,
the Japanese war memorial in Tokyo. Clearly issues of national identity and national sovereignty can generate volatile reactions.
The big danger across the Taiwan Strait is that misunderstanding and miscalculation, fueled by distrust, xenophobia, and
opportunism, may lead to escalating conflict. Senior leaders on both sides of the Strait are beginning to realize the potential
consequences if instability erupts into violence. Hu Jintao has recently been signaling that he advocates a long-term policy of
stability for eventual reunification. Chen Shui-bian has recently dropped his independence demands. Several Southeast Asian
leaders have opposed Taiwan's independence; most explicitly, Singapore's Prime Minister, Lee Hsien Loong. Lee bluntly stated,
"If Taiwan goes for independence, Singapore will not recognize it. In fact no Asian country will recognize it. China will fight. Win
military confrontation between the mainland and Taiwan is
or lose, Taiwan will be devastated." The prospect of a
unlikely, in part because the consequences of such a conflict would be extremely destructive for
both sides. Diplomatic efforts are needed to avoid even this remote risk. In the March/April 2005 issue of Foreign Affairs,
Kenneth Lieberthal offered a useful proposal to change the focus of negotiations over "independence" and "reunification" to a
pragmatic question: what is needed to achieve long-term stability and peaceful coexistence between China and Taiwan? What
confidence building measures are needed to reassure security strategists that defensive military developments are not
offensive? What legal and administrative means are necessary to resolve routine conflicts that will inevitably occur as
commercial and civil relations thicken? The current U.S. attempts to help Taiwan "contain" China and to mobilize support in its
global war on terrorism threaten to complicate if not weaken regional security developments. As Ronald Montaperto notes,
"the almost daily manifestations of Chinese economic power, the effort to demonstrate commitment to the 'new' principle that
the economic development of individual nations is inseparable from the development of the region as a whole, and the broad
perception within the region that the Chinese are willing to engage actively in multilateral, cooperative policies have combined
Beijing regime is obsessed
to provide Beijing with an unprecedented measure of influence and even clout."[6] The
with economic stability, because it fears that a severe downturn would trigger social and political upheaval. The last
thing it wants is a military confrontation with its biggest trading partner , the United States, or with
Japan or Taiwan, each of which are major trade and investment partners. It may go on playing the nationalist card over Taiwan
to curry domestic political favor, but there has been no massive military build-up and there is no
plausible threat of impending war. [7] To the contrary, China is investing heavily in creating a regional security
framework to pursue its domestic development. The U.S. goal of achieving genuine regional maritime security would best be
served through cooperation with China -- one of its most important creditors, suppliers, and markets -- rather than
confrontation.

No China-Taiwan war --- no one wants it

Pei ’6 (Minxin, senior associate and director of the China program at the Carnegie Endowment for International Peace, 2/8.
“Chen’s Gamble to Stay Relevant.” Straits Times, Carnegie Endowment online.)

Not too long ago, the nightmarish scenario of an armed conflict between mainland China and Taiwan captured the attention of East
Asia. After winning his re-election to the presidency under controversial circumstances in March 2004, Taiwan's Chen Shui-bian
began a high-stakes gamble to test China's bottom line. He not only escalated the rhetoric about making Taiwan a 'normal nation',
but also backed up his words with a plan to hold an island-wide referendum on a new Constitution as a legal vehicle to solidify
Taiwan's permanent separation from mainland China. Two years later, things could hardly be more different. The spectre of a
war across the Taiwan Strait has receded. In the much improved Sino-American relationship , the
contentious Taiwan issue no longer dominates the agenda. In fact, Taiwan was largely an
afterthought in recent high-level exchanges between Chinese and American leaders. Topping the
discussions between Washington and Beijing today are more pressing global and regional
security issues: curbing North Korea's nuclear ambition, pressuring Iran to give up its plans for uranium enrichment
and, more importantly, searching for a new framework for US-China relations. The reduction of tensions
across the Taiwan Strait comes as welcome news to East Asia . In the past year, a combination of
developments has turned the tide against the pro-independence Democratic Progressive Party
(DPP). The political fortunes of the DPP , which rose to power in 2000 by championing a new Taiwanese identity and
recklessly challenged the fragile status quo in the Taiwan Strait, has been waning. Its leadership has lost credibility, both with
a majority of Taiwan's voters and with Washington. Indeed, two years before he moves out of the presidential palace in Taipei, Mr
Chen is struggling to stay relevant. Broadly speaking, three seismic changes since President Chen's re-election victory two years ago
have greatly altered the short- to medium-term political landscape both in Taiwan and across the Taiwan Strait. First, alarmed by Mr
Chen's thinly disguised ploy to seek de jure independence through the passage of a new Constitution enacted by a plebiscite,
Taiwan's voters decided to end the President's gambit by refusing to give the DPP a majority in the island's legislative chamber (a
condition which would be necessary to give a new Constitution any realistic chance of passage) in the watershed election of
December 2004. The DPP's electoral nemesis, the so-called pan-blue alliance, consisting of two opposition parties - the Kuomintang
and the People First Party – that advocate a moderate approach to mainland China, managed to retain its slim legislative majority.
This stunning rebuke by Taiwan's democratic process halted the momentum of the pro-independence movement almost overnight.
Constrained by an opposition-controlled legislature and rising public discontent with his poor governing record, President Chen lost
his ability to set Taiwan's policy agenda and direction. Of course, things went from bad to worse at the end of last year when the
DPP suffered a massive defeat in local elections. Second, China's new leadership adjusted its Taiwan policy in
two dramatic directions. On the one hand, Beijing's new leaders concluded that they must make their threat of military
action credible. Consequently, the mainland accelerated military preparations for a conflict with Taiwan in light of Mr Chen's vow to
pass a new Constitution. Chinese leaders also set in motion a legislative process to obtain pre-authorisation for the use of force -
which culminated in the passage of an 'anti-secession law' in March last year. On the other hand, China's President
Hu Jintao
coupled the threat of the use of force with a charm offensive, inviting the leaders of Taiwan's
main opposition parties to visit the mainland and offering a package of economic benefits and
goodwill gestures (a pair of pandas) to Taiwan. While wooing the Taiwanese opposition and business community, Beijing
also intensified the isolation of Mr Chen, refusing to deal with him unless he accepts the 'one China' principle, which stipulates that
the mainland and Taiwan both belong to the same China. Caught offguard by Beijing's 'panda offensive', Mr Chen's government
was unable to counter the mainland's new policy initiatives and could
offer no reassuring message to a Taiwanese
public that had grown increasingly weary of the DPP's divisive ethno-nationalist policies and was interested in
returning the cross-strait relationship to a more stable footing . Third, President George W. Bush, perhaps
the most pro-Taiwan American president in history, re-adjusted his policy in late 2004. Although the Bush administration approved
the largest arms package for sale to Taiwan in 2001 and substantially upgraded ties with Taiwan in the past five years,
Washington was greatly alarmed by Mr Chen's apparent strategy of taking advantage of US
support and seeking a dangerous confrontation with mainland China . Obviously, the United States has
no interest in fighting for Taiwan's de jure independence even though it continues to deter
China from seeking reunification through military means. In addition, with its strategic attention focused
on Iraq, the war on terrorism, Iran and North Korea, the Bush administration needs China's
cooperation on a wide range of issues and wants to prevent a needless conflict between the
mainland and Taiwan. Washington has also grown increasingly impatient with Mr Chen, who has surprised the Bush
administration on numerous occasions with statements that were viewed as irresponsible, fickle and reckless. Consequently,
Washington cooled its support for Taipei and became explicit in its opposition to the so-called 'unilateral change of the status quo', a
veiled reference to Mr Chen's plans to alter Taiwan's constitutional and political status. The cumulative effects of these
developments significantly undermined Mr Chen's effectiveness and increased his frustrations. Struggling to regain the political
initiative after the DPP's disastrous performance in last December's local polls, Mr Chen recently reshuffled his government. He
appointed two heavyweight loyalists, Mr Su Tseng-chang and Ms Tsai Ing-wen, as Premier and Vice-Premier respectively. Both are
viewed as hardliners on China policy. Defying public expectations that, chastened by his party's electoral losses, he would adopt a
more conciliatory tone towards the mainland, the Taiwanese President has apparently decided to escalate tensions with Beijing (and
Washington) again. In the past month, Mr Chen has vowed to tighten cross-strait trade and investment, scrap the symbolic National
Reunification Council, seek admission to the United Nations under the name 'Taiwan' (not the Republic of China) as well as enact a
new Constitution through a plebiscite. All these steps, if carried out, would re-ignite tensions across the Taiwan Strait. Mr Chen
conceivably could benefit from the tensions because these acts would energise his base and allow him to dominate Taiwan's policy
agenda again. It is too early to tell whether Mr Chen's gamble will pay off. So far ,
Beijing has reacted coolly to his latest
provocations, relying instead on Washington to restrain Taipei. The Bush administration, surprised again by Mr
Chen's pronouncements, has made its irritation public and criticised Taipei for trying to change the
status quo. But in Taiwan, Mr Chen's confrontational stance has failed to rally the public. For the short term, his gambit has got
him enough public attention to show his political relevance.

No escalation – China won’t use nuclear weapons

Pike ’4 (John, Global Security, China’s Options in the Taiwan Confrontation, http://www.globalsecurity.org/military/ops/taiwan-
prc.htm)

China would almost certainly not contemplate a nuclear strike against Taiwan , nor would Beijing
embark on a course of action that posed significant risks of the use of nuclear weapons. The mainland's long term goal is
to liberate Taiwan, not to obliterate it, and any use of nuclear weapons by China would run a
substantial risk of the use of nuclear weapons by the United States . An inability to control escalation
beyond "demonstrative" detonations would cause utterly disproportionate destruction.

US deterrence checks

Ross 1 (Robert S., Professor of Political Science – Boston College, National Interest, Fall, Lexis)

There can never be total confidence that deterrence will work. Yet U.S.
deterrence of any actual Chinese use of
force against Taiwan-outside of a Taiwan declaration of independence-is highly stable. Overwhelming
U.S. superiority means that the strategic, economic and political costs to China of U.S. military intervention
would be astronomical. U.S. conventional superiority and its strong political commitment to Taiwan mean that the
credibility of the U.S. threat to intervene is very high. In an insecure world, the U.S. deterrent posture in the
Taiwan Strait is an unusually secure one.

Chinese conventional weakness checks

Nolt 2k (James A., Senior Fellow – World Policy Institute, The China-Taiwan Military Balance, Taiwan Security Research, January,
http://taiwansecurity.org/IS/IS-012000-Nolt.htm)

China's weak naval and air forces provide it no ability to invade Taiwan. If Taiwan were not an island,
China might be able to threaten it with its large (but inefficient) army. To invade Taiwan across the 80-mile-wide Taiwan
Strait, China would first have to win control of the sea and air. This would be extremely
unlikely, as I argue in the next section, given Taiwan's considerable qualitative advantage in naval and air
forces, even if the US did not aid Taiwan . Taiwan's qualitative advantage has increased recently. If China could
gain control of the sea and air, a blockade of Taiwan might be possible, though it would almost certainly require direct
confrontation with US ships defying the blockade. However, even if China somehow could gain complete
control over the sea and air in the Taiwan Strait, for example, by nuclear strikes against Taiwan's naval ports and military
airfields that somehow avoided US intervention, a successful invasion of Taiwan would be virtually
impossible because of China limited sea and air lift capability relative to the size of the defending
forces. Amphibious invasions can succeed only if the attacker can land enough troops by sea and air either to overwhelm the
defender in the initial attack, or, more usually, to hang onto a beachhead long enough so that reinforcements can be landed
before the defender can build up overwhelming strength to crush the beachhead. Ports are usually too strongly defended to be
attacked directly, so supplies must initially be brought in by the same means as the troops: by parachute, helicopter and, mostly,
by specially-built amphibious warfare vessels landing on a beach. Once a port and airports are secured and repaired, forces and
supplies can be landed more efficiently using regular transports, cargo ships, and large air transports. Thus it is not the size of
China's army, or even its navy or air force, that ultimately constrain its ability to invade Taiwan, but its capacity to transport
troops and supplies to a hostile beach and nearby air landing sites.
---China Taiwan- No Escalation

Won’t escalate to nuclear war

Pike 4 (John, Global Security, “China’s Options in the Taiwan Confrontation”, http://www.globalsecurity.org/military/ops/taiwan-
prc.htm)

China would almost certainly not contemplate a nuclear strike against Taiwan, nor would
Beijing embark on a course of action that posed significant risks of the use of nuclear
weapons. The mainland's long term goal is to liberate Taiwan, not to obliterate it, and any use
of nuclear weapons by China would run a substantial risk of the use of nuclear weapons by
the United States. An inability to control escalation beyond "demonstrative" detonations would cause utterly
disproportionate destruction.

Economic check

CNA 2 (Channel News Asia, 9-17, Lexis)

Concerns over the possibility of war following the recent exchange of words between China and
Taiwan are unfounded, according to analysts in the Chinese capital. Given the advanced economic
integration, they believe heated words over recent remarks by the Taiwanese President may be substitutes
for, rather than preludes to, military hostilities. It has been described as Taiwan's show of defiance against
China's intimidation. Taiwanese President Chen Shui-bian told a pro-independence group that Taiwan was a sovereign state. He
also called for a referendum to decide the island's future. Sheer audacity, according to Beijing. Li Weiyi, Spokesman for the State
Council Taiwan Affairs Office, said: "He exposed his true colours when he stubbornly championed Taiwanese independence. This
openly provokes all Taiwanese and Chinese. It has open defiance of the 'One-China' principle embraced by the international
community." China has refused to renounce the possibility of using force to achieve reunification. It has also issued a defence
White Paper that makes it clear reunification cannot be put off indefinitely. But analysts say the
use of force is
unthinkable, given the growing economic integration. The Chinese market accounted for almost 20
percent of Taiwan's exports last year. There are now some 25,000 Taiwanese projects in China, with a contracted investment of
US$54 billion. Professor Sun Yan, Peking University, Department of International Relations, said: "Chen Shui-bian knows his
comments will invite criticisms from Beijing and lead to cross-strait tensions. "He can then say that as China is hostile towards
Taiwan, it is risky to invest in mainland China. His main aim is to stem the flow of investments into China and to reduce the over-
reliance on China." But there's only so much that Taiwan authorities can do to stem the flow of investments. At the end of the
day, in spite of the lack of direct links between the two sides, Taiwanese businessmen are still going to put money where it
makes economic sense to do so. Many experts argue that close economic ties will encourage peaceful reunification. But they
agree this can take place only when the political systems and economic development in China and Taiwan are more closely
matched. For now, they believe the use of force is unlikely, given that China's primary focus is on internal modernisation,
economic liberalisation, and increasingly, on hosting the 2008 Olympic Games. So even
though heated exchanges
continue, observers say Beijing understands that too much is at stake.
---China Taiwan- No War Now
High relations check - there evidence doesn’t assume Taiwan’s New President

AFP 9 (Agency France Presse, “China Offers Peace Talks With Taiwan,” 3-5, The Australian,
http://www.theaustralian.news.com.au/story/0,25197,25142947-2703,00.html)

Premier Wen Jiabao said today that China


was ready for talks on political and military issues with Taiwan,
seeking to further a rapprochement between the rivals. “In the coming year, we will continue to adhere to the
principle of developing cross-strait relations and promoting peaceful reunification of the motherland,” Wen said at
the start of the annual full session of parliament. “We are ... ready to hold talks on cross-strait political and military issues and
create conditions for ending the state of hostility and concluding a peace agreement.” China usually refers to relations with
Taiwan as “cross-strait”, referring to the narrow Taiwan Strait separating the island and the mainland. China and Taiwan have
been governed separately since the end of a civil war in 1949, but Beijing considers the island part of its territory, insisting that
reunification is only a question of time. Their relationship has long been considered one of Asia's potential war flashpoints, and
both sides have engaged in an expensive military build-up in the event of a conflict. From 2000 to 2008, relations between China
and Taiwan were particularly bad, as Beijing watched the island's independence-minded president Chen Shui-bian push for
greater autonomy. However
relations have improved markedly since the more China-friendly Ma Ying-jeou
became president of Taiwan in May last year, highlighted by the introduction of direct regular flights between
the two sides.

Hostile relations are disappearing - our evidence cites momentum

Khaleej Times 9 (Khaleej Times, “China’s Olive Branch to Taiwan,” 3-7, http://www.khaleejtimes.com/Display ArticleNew.asp?
xfile=data/editorial/2009/March/editorial_March14.xml&section=editorial&col=)

China’s readiness to talk to Taiwan to end mutual hostilities is a welcome initiative made by the Prime Minister Wen Jiabo in his
address to the National Peoples Congress.  As Beijing seeks a cessation of tensions with Taipei since a civil war between the
Communist party and the Nationalists in 1949, the question remains how this could be achieved. Taiwan President Ma Ying-
jeou since assuming office in May 2008 has demonstrated a remarkable departure in policy from his predecessor, Chen Shui-
bian who had escalated hostilities by building defence relations with the United States and actively pursuing independence from
China. Even though President Ying-jeou has managed to obviate hostilities to a large extent by opening economic channels that
include direct flights, shipping and postal links with China, he remains cautious on political differences with Beijing. Under the
One China policy that is interpreted differently by Beijing and Taipei, China includes the mainland, Hong Kong, Taiwan and
Macau. Increased hostilities over the sovereignty issue had led to China positioning 1300 missiles towards Taiwan. Furthermore,
an anti-secession law that was adopted by China in 2005 thus legalising non-peaceful means to protect China’s sovereignty and
territory, is perceived to be specifically aimed at Taiwan and Tibet. However, of
late one discerns a lessening of
hostilities across the Straits, which can be attributed to the conciliatory attitude of the new
Taiwanese leadership and the robust economic relations that continue to flourish despite
political tensions. Since China is Taiwan’s largest trading partner and a primary destination for Taiwanese investors, it is
only natural for Taiwan to seek better economic relations with China and vice versa. The economic measures that both China
and Taiwan are looking at include the drafting of a comprehensive agreement on economic cooperation that is expected to pave
the way for a Free trade agreement. It also entails gradual integration of banking and other financial services across the Taiwan
Straits. China’s avowal of developing economic ties with Taiwan is expected to yield positive results for economic investors in
Taiwan that has suffered a fall back during recession. Both Beijing and Taipei are testing waters and trying to
build mutual trust before embarking on issues that pose a conflict of interests. This may be the reason why Wen chose
not to elaborate on Chinese President Hu Jintao’s recent suggestion about the need to improve communication on military
issues with Taiwan. Even as Taiwanese president welcomed Wen’s support for economic talks, he stressed on the need to build
trust before addressing political issues. The Chinese have in short offered an olive branch to Taiwan, and an opportunity to
formally cease decades of hostility.
---China Taiwan- Reunification

Peaceful Reunification Now – no risk of conflict

Ross ‘6 (Robert S. Ross, 2006, Professor of Political Science at Boston College and an Associate at the John King Fairbank Center
for East Asian Research at Harvard University, March/April 2006, (Foreign Affairs, Taiwan's Fading Independence Movement, p.
Lexis)

Political developments in Taiwan over the past year have effectively ended the independence
movement there. What had been a major source of regional instability – and the most likely
source of a great-power war anywhere in the world -- has become increasingly irrelevant . The peaceful
transformation of relations between China and Taiwan will help stabilize eastern Asia, reduce
the likelihood of conflict between China and the U nited S tates, and present an opportunity for
Beijing, Taipei, and Washington to adjust their defense postures -- all without hurting Taiwan's
security or threatening U.S. interests.

Political Momentum is for reunification – China will be patient

Rosemont ‘8 (Henry Rosemont, 2008, Jr., is distinguished professor emeritus at St. Mary's College of Maryland and a visiting
scholar at Brown University, February 7, 2008 Thursday, (Foreign Policy in Focus, HEADLINE: Is China a Threat?, p. Lexis)

The PLA's air force capabilities, meanwhile, are no match in quality for the United States either defensively or offensively. Many of
China's aircraft models are over 40 years old.4 Certainly the mainland forces pose a threat to Taiwan, but Taiwan's own modern air
force should not be underestimated.5 And with
the recent electoral defeat of Chen Shui-bian's D emocratic
P rogressive P arty by the pro-mainland Kuomintang, political and military tensions are likely to
decrease markedly. Beijing can be very patient in waiting for a rapprochement with Taiwan.

China won’t spark conflict – they don’t want to scare Taiwan away

Armstrong ‘8 (Herbert W. Armstrong, 2008, the late founder and editor in chief of the Trumpet’s predecessor, the Plain Truth,
January 15, 2008, (TheTrumpet.com, New Taiwanese Government Hopes to Improve Ties With China,
http://www.thetrumpet.com/index.php?q=4691.2955.0.0, date accessed February 2, 2008)

The Kuomintang vision for Taiwan-China relations, however, will not guarantee Taiwan’s ongoing independence;
in fact, it will bring Taiwan one step closer to complete reunification with the mainland. China verbally claims
Taiwan as its own and has threatened military force to keep it from officially declaring otherwise. In the current fracas, it is being
pretty quiet—likely in an attempt not to scare the Taiwanese away from their new, more pro-
China stance. Taiwan’s new government represents progress toward China’s goal of reunification. China will let Taiwan get
closer, but will not permit it to move further away.
That means no risk of conflict

Ross ‘6 (Robert S. Ross, 2006, Professor of Political Science at Boston College and an Associate at the John King Fairbank Center
for East Asian Research at Harvard University, March/April 2006, (Foreign Affairs, Taiwan's Fading Independence Movement, p.
Lexis)

The demise of Taiwan's independence movement has removed the only conceivable source of
war between the U nited S tates and China. The two countries will continue to improve their military capabilities and
compete for influence in eastern Asia, but as the threat of war over Taiwan recedes, they may moderate
their defense policies toward each other. As Beijing gains greater confidence that Taipei seeks not
independence but cooperation, it should be able to relax its military posture . Unilateral freezes
on new missile deployments and redeployments of missiles away from the Taiwan Strait by
Beijing would increase support among Taiwan's voters for the KMT's policy of engagement. Such
actions would also promote good relations between China and other countries in the region, serving China's declared objective of a
"peaceful rise." The United States will also be relieved of the imperative to prepare for war with China. The United States will be able
to reduce its pressure on Taipei to buy costly U.S. weapons that are ill suited for Taiwan's defensive needs and politically
controversial. In fact, Washington should develop a new defense package for Taiwan that is more sensitive to Taiwan's strategic and
budget realities and that could promote more cooperative political ties between Taipei and Washington by removing a source of
acrimony from their relationship. The easing of tensions between Taipei and Washington would contribute to the emergence of less
contentious relations between China and the United States and facilitate Washington's cooperation with other U.S. allies in the
region. Although there is regionwide apprehension over China's threat to use force against Taiwan, there is also little sympathy in
the region for Taiwan's independence movement; indeed Washington's commitment to Taiwan has been a divisive issue in U.S.
relations with South Korea and Australia. Once freed from the immediate threat of war, Taiwan will be able to focus on promoting
economic development and consolidating its still-young democracy. U.S. interests in Taiwan are undiminished by Taiwanese voters'
support for the status quo in cross-strait relations, and an unprovoked Chinese attack on Taiwan would challenge those fundamental
interests. Washington will remain the guarantor of Taiwan's security: U.S. defense ties with Taiwan and the United States' security
commitment to the island are stronger today than at any time since the Nixon administration. Washington has long considered
Taiwan's moves toward independence a threat to U.S. security because they could lead to war. And so it was unsurprising when U.S.
Deputy Secretary of State Robert Zoellick's September 2005 comprehensive review of U.S. relations with China included only three
brief neutral sentences on Taiwan, signaling U.S. satisfaction with current trends in cross-strait relations. Now
that Taiwan's
independence movement is waning, and the risks of war between China and the United States
are receding, defending U.S. interests in the region will become far easier.

China won’t attack Taiwan – new developments in relations solve – and Taiwan wouldn’t ask
for US intervention anyway

China Daily 10 [5/4/10, “Taiwan pledges not to seek US help in war”, http://www.chinadaily.com.cn/china/2010-
05/04/content_9804241.htm]

Beijing - Mainland experts on Monday hailed Taiwan leader Ma Ying-jeou's pledge that the island will
never ask the United States to help fight a war with the mainland, saying it demonstrates Ma's
determination to push for better cross-Straits ties. In a CNN interview, conducted entirely in English via video
conference and broadcast on April 30, Ma, speaking from his office in Taipei, said that "we will continue to reduce the risks so
that we will purchase arms from the United States, but we will never ask the Americans to fight for Taiwan.
This is something that is very, very clear." Chen Xiancai, a researcher at the Taiwan Studies Center in Xiamen University, said Ma
has been the first Taiwan leader who dared to say "never" to US help since former leader Lee Teng-hui introduced direct
"presidential" elections in late 1990s. According
to its 1979 Taiwan Relations Act, the US has the
obligation to help defend the island, but as it seeks better ties with Beijing, the US has hedged on
saying how far it would go in the event of a war, Reuters commented on Monday. "The society of Taiwan has a strong
dependence on the US, while Ma's comment of never asking for US help to fight for Taiwan indicates his determination to
ease cross-Straits relations, which can
be interpreted as goodwill toward the mainland ," Chen told China
Daily. He said the move proves Ma will continuously promote cross-Straits exchanges and
cooperation, including the proposed comprehensive trade pact between the two sides. Chen, however,
emphasized that Ma was very cautious to appease the US by reiterating the demand for arms sales, which are strongly opposed
by the mainland. Li Jiaquan, a senior researcher with the Institute of Taiwan Studies at the Chinese
Academy of Social Sciences, also expressed his appreciation for Ma's comments. He said a consistent and
determined mainland policy is also helpful in winning more support for Ma. Li said while the pro-independence
opposition Democratic Progressive Party (DPP) always attempted to drag America into a war to
help its push for "Taiwan independence", Ma is trying to rule out such a possibility . Ma also said
during the CNN interview that the risk to the US of a conflict between the mainland and Taiwan is
the lowest in 60 years. "In the last two years, as a result of our efforts to improve relations
with the mainland, we have already defused the tension to a great extent," he said. Direct
transportation, mail and trade have been achieved across the Taiwan Straits since Ma took
office in May 2008. Hsiao Bi-khim, director of the International Affairs Department of the DPP, accused Ma on Sunday of
undermining "national security" by eliminating the vagueness Washington has deliberately maintained on how it would respond
to a possible mainland attack against Taiwan, the Taipei-based "central news agency" reported on Sunday. Washington, which
had no immediate comment on Ma's remarks, could decide on its own whether to help Taiwan, Taiwan's "cabinet" spokesman
Johnny Chiang said on Monday following protests from the opposition DPP. The US decided in January to sell Taiwan more than
$6 billion worth of weapons, causing a tough response from Beijing.
China Japan War Answers
Frontline

No China-Japan War

Carlson ’13 (Allen Carlson is an Associate Professor in Cornell University’s Government Department. He was granted his PhD
from Yale University’s Political Science Department. His undergraduate degree is from Colby College. In 2005 his Unifying China,
Integrating with the World: Securing Chinese Sovereignty in the Reform Era was published by Stanford University Press. He has
also written articles that appeared in the Journal of Contemporary China, Pacific Affairs, Asia Policy, and Nations and Nationalism.
In addition, he has published monographs for the National Committee on U.S.-China Relations and the East-West Center
Washington. Carlson was a Fulbright-Hays scholar at Peking University during the 2004-2005 academic year. In 2005 he was
chosen to participate in the National Committee’s Public Intellectuals Program, and he currently serves as an adviser to Cornell’s
China Asia Pacific Studies program and its East Asia Program. Carlson is currently working on a project exploring the issue of
nontraditional security in China’s emerging relationship with the rest of the international system. His most recent publications are
the co-edited Contemporary Chinese Politics: New Sources, Methods and Field Strategies (Cambridge University Press, 2010) and
New Frontiers in China’s Foreign Relations (Lexington, 2011). China Keeps the Peace at Sea China Keeps the Peace at Sea Why the
Dragon Doesn't Want War Allen Carlson February 21, 2013

At times in the past few months, China and Japan have appeared almost ready to do battle over the
Senkaku (Diaoyu) Islands --which are administered by Tokyo but claimed by both countries -- and to ignite a war that could be
bigger than any since World War II. Although Tokyo and Beijing have been shadowboxing over the territory for years, the standoff
reached a new low in the fall, when the Japanese government nationalized some of the islands by purchasing them from a private
owner. The decision set off a wave of violent anti-Japanese demonstrations across China. In the wake of these events, the conflict
quickly reached what political scientists call a state of equivalent retaliation -- a situation in which both countries believe that it is
imperative to respond in kind to any and all perceived slights. As a result, it may have seemed that armed engagement was
imminent.
Yet, months later, nothing has happened . And despite their aggressive posturing in the
disputed territory, both sides now show glimmers of willingness to dial down hostilities and to
reestablish stability . Some analysts have cited North Korea's recent nuclear test as a factor in the countries' reluctance to
engage in military conflict. They argue that the detonation, and Kim Jong Un's belligerence, brought China and Japan together,
unsettling them and placing their differences in a scarier context. Rory Medcalf, a senior fellow at the Brookings Institution,
explained that "the nuclear test gives the leadership in both Beijing and Tokyo a chance to focus on a foreign and security policy
challenge where their interests are not diametrically at odds." The nuclear test, though, is a red herring in terms of the conflict over
In truth, the roots of the conflict -- and the reasons it has not yet exploded -- are
the disputed islands.
much deeper . Put simply, China cannot afford military conflict with any of its Asian neighbors. It
is not that China believes it would lose such a spat; the country increasingly enjoys strategic
superiority over the entire region, and it is difficult to imagine that its forces would be beaten
in a direct engagement over the islands , in the South China Sea or in the disputed regions
along the Sino-Indian border. However, Chinese officials see that even the most pronounced
victory would be outweighed by the collateral damage that such a use of force would cause to
Beijing's two most fundamental national interests -- economic growth and preventing the
escalation of radical nationalist sentiment at home. These constraints, rather than any
external deterrent , will keep Xi Jinping, China's new leader, from authorizing the use of deadly
force in the Diaoyu Islands theater. For over three decades , Beijing has promoted peace and stability
in Asia to facilitate conditions amenable to China's economic development. The origins of the policy
can be traced back to the late 1970s, when Deng Xiaoping repeatedly contended that to move beyond the economically
debilitating Maoist period, China would have to seek a common ground with its neighbors. Promoting cooperation in the region
would allow China to spend less on military preparedness, focus on making the country a more welcoming destination for foreign
investment, and foster better trade relations. All of this would strengthen the Chinese economy. Deng was right. Today, China's
economy is second only to that of the United States. The fundamentals of Deng's grand economic strategy are still revered in
Beijing. But any war in the region would erode the hard-won, and precariously held, political capital that China has gained in the
last several decades. It would also disrupt trade relations, complicate efforts to promote the yuan as an international currency,
and send shock waves through the country's economic system at a time when it can ill afford them. There is thus little reason to
specter of rising Chinese nationalism,
think that China is readying for war with Japan. At the same time, the
although often seen as a promoter of conflict, further limits the prospects for armed
engagement . This is because Beijing will try to discourage nationalism if it fears it may lose control or be forced by popular
sentiment to take an action it deems unwise. Ever since the Tiananmen Square massacre put questions
about the Chinese Communist Party's right to govern before the population, successive
generations of Chinese leaders have carefully negotiated a balance between promoting nationalist
sentiment and preventing it from boiling over. In the process, they cemented the legitimacy of their rule. A war with Japan could
easily upset that balance by inflaming nationalism that could blow back against China's leaders. Consider a hypothetical scenario
in which a uniformed Chinese military member is killed during a firefight with Japanese soldiers. Regardless of the specific
circumstances, the casualty would create a new martyr in China and, almost as quickly, catalyze popular protests against Japan.
Demonstrators would call for blood, and if the government (fearing economic instability) did not extract enough, citizens would
agitate against Beijing itself. Those in Zhongnanhai, the Chinese leadership compound in Beijing, would find themselves between a
rock and a hard place. It is possible that Xi lost track of these basic facts during the fanfare of his rise to power and in the face of
renewed Japanese assertiveness. It is also possible that the Chinese state is more rotten at the core than is understood. That is, party
elites believe that a diversionary war is the only way to hold on to power -- damn the economic and social consequences. But Xi does
not seem blind to the principles that have served Beijing so well over the last few decades. Indeed, although he recently warned
unnamed others about infringing upon China's "national core interests" during a foreign policy speech to members of the Politburo,
he also underscored China's commitment to "never pursue development at the cost of sacrificing other country's interests" and to
never "benefit ourselves at others' expense or do harm to any neighbor." Of course, wars do happen -- and still could in the East
China Sea. Should either side draw first blood through accident or an unexpected move, Sino-Japanese relations would be pushed
into terrain that has not been charted since the middle of the last century. However, understanding that war would be a no-win
situation, China has avoided rushing over the brink. This relative restraint seems to have surprised everyone. But it shouldn't.
Beijing will continue to disagree with Tokyo over the sovereign status of the islands, and will not budge in its negotiating
position over disputed territory. However, it cannot take the risk of going to war over a few rocks in the sea.
On the contrary, in the coming months it will quietly seek a way to shelve the dispute in return for
securing regional stability, facilitating economic development, and keeping a lid on the Pandora's box
of rising nationalist sentiment. The ensuing peace , while unlikely to be deep, or especially conducive to improving
Sino-Japanese relations, will be enduring.

Bilateral cooperation and strong Sino-Japanese relations solves for stability --- mitigates risk of
impact

ISDP ‘8 (Institute for Security and Defensive Policy, “Sino-Japanese Relations,” China Initiative, http://www.isdp.eu/programs-a-
initiatives/china-initiative/sino-japanese-relations.html)

Throughout history, the relationship between China and Japan has more often than not been marked by mistrust and animosity, or
even violent conflict. Despite three decades of normalized bilateral relations, several past and present issues serve to complicate the
relation between the two states. Since
a positive and functioning relationship between China and Japan,
the two great powers in Northeast Asia, in many ways is a prerequisite for peace and stability in
the region, a souring bilateral relationship is not only problematic for the states involved, but
has implications for neighboring states and the international community at large. Against this
background, it has become increasingly important to understand, identify and implement measures that can prevent and manage
conflicts and disputes between these two states. This said, the Sino-Japanese relations have been on the
mend since Shinzo Abe (安倍 晋三) assumed the Prime Minister's office in September 2006. His visit to China in October 2006 and
the reciprocal visits of Chinese Prime Minister Wen Jiabao (温家宝) in April 2007 and President Hu Jintao in May 2008
facilitated the further thawing of bilateral relations under the framework of "mutually beneficial
relationship based on common strategic relationship." A substantial number of additional events
have indicated the continuation of the positive trend in the strengthening of the bilateral
relations. As one example, in one attempt to initiate debate on the issue of historical perception
on the 20th century Sino-Japanese relations, a joint committee of Chinese and Japanese
historians was established in an effort to reach a certain understanding of each other´s
perception of common history, mainly the atrocities from the Second World War. On the
military side, the establishment of a hot-line in November 2007 and the port visits by the fleets
represented important confidence building measures. Furthermore, China and Japan are in fact
sustaining injured US dollar economy under current severe financial crisis. This may provide
more opportunity for cooperation between the two countries. All of these bilateral efforts have
been very positive.

No risk of China-Japan war

Sutter 2 (Robert, Professor – Georgetown, “China and Japan: Trouble Ahead?”, Washington Quarterly, Autumn, Lexis)

Little appears to be on the horizon that will substantially change the recent balance between
friction and cooperation in Sino-Japanese relations in a way that would pose serious
challenges for U.S. leadership in Asia or U.S. interest in regional stability and development. The shock of the
September 11 attacks on the United States along with the U.S.-led war in Afghanistan had the effect of somewhat reducing
China's relative influence in Asia while providing Japan an opportunity to expand its role in South and Central Asia. Policy
changes after the presidential elections in South Korea late this year could upset the delicate equilibrium on the peninsula,
though few see viable alternatives to some continued South Korean engagement with the North. The Chinese leadership
transition in 2002 -- 2003 is not expected to result in significant changes in policy toward Asia, as Beijing strives to maintain a
calm external environment and focuses on internal priorities. An Indo-Pakistani nuclear war, a U.S.-led attack against Iraq, a
terrorist attack using weapons of mass destruction against the United States, or other conceivable international conflicts would
strongly affect the United States, though the
impact on Sino-Japanese friction in Asia would probably be
relatively small. Realistically, the probability is low that a Sino-Japanese entente may emerge
that would seriously complicate the existing U.S. security architecture in Asia or possibly challenge
the leading U.S. economic role in the region. Thus, Sino-Japanese wariness probably means that the United States has little to
worry about from ASEAN Plus Three or other Japan and China -- led groups that endeavor to exclude the United States.
Although increased Sino-Japanese friction could divide Asian governments, with some feeling compelled to side with Japan (and
presumably the United States) and others seemingly pressed to side with China, neither Beijing nor Tokyo sees such rivalry as in
its broad national interests. Both
powers appear more likely to continue pursuing priorities focused
on domestic issues and economic development that require broad regional cooperation and
avoiding confrontation and conflict.

China-Japan relations are up and check conflict.

Kwok ‘8 (Kristine Kwok, @ South China Morning Post, 5-8-08 [Rivals put past behind them to focus on future relations;
Japan, China sign joint declaration that avoids thorny issues, lexis]

The long-time rivals agreed in the document that they would not pose a threat to each other,
while a peaceful relationship would bring about " great opportunities and benefits" to Asia and the
world. "The two sides are determined to squarely face history, face towards the future and
continuously create new prospects for a strategic and mutually beneficial Sino-Japanese
relationship," it said. In contrast to the three previous joint declarations, which got bogged down by the two sides'
interpretations of Japan's wartime militarism, the latest document is forward-looking and shows willingness
on both sides to improve ties , despite their differences. When former president Jiang Zemin made his state visit to Japan
in 1998, attempts to rev up relations became mired in the history issue. Japan's prime minister at the time, Keizo Obuchi, refused a
In the
demand by Mr Jiang that Japan apologise for wartime atrocities in the joint statement. As a result, it was never signed.
latest document, China and Japan touch only briefly on thorny issues. Beijing went as far as
saying it "positively assessed" Japan's commitment to peace and its contribution to world peace
and stability 60 years after the second world war. Both China and Japan are important regional powers
with growing influences in global affairs. The two governments pledged to work together on
climate change and to ensure regional peace and stability . Co-operation will be increased in trade, investment,
information technology and finance, according to the statement. Cultural exchanges involving young people, media and private
organisations will also be promoted.

Interdependence checks

Kang 5 (David, Associate Professor of Government – Dartmouth College, Washington Post, 5-4, Lexis)

David Kang: North Korea has tested these short-range (100km, or 62 mile) missiles a number of times in the past few years.
They don't even have the range to hit Japan, and are aimed mostly at South Korea. So it's not quite as destabilizing as we think it
is. As to the issue of unresolved history, that's important, for sure. As
to Japan and China getting in an actual
military conflict in the future, that seems to be an unlikely scenario. The economic ties
between the two states are deeper than they ever have been , and while they are trying to work out
their political relationship, the economic relationship continues to thrive.
Cultural ties check

Harrison 2k (Selig, Director of the Asia Program – Center for International Policy and Senior Scholar – Woodrow Wilson
International Center for Scholars, China’s Future, p. 105-106)

This Sino-Indian rivalry differs from the more complex Sino-Japanese competition for influence. Economically, Tokyo has set a
faster pace than has Beijing, but ingrained feelings of cultural subordinalion to China make it
impossible for Japan to look on China as a junior power. Powerful psychological bonds ,
summed up in the Japanese expression dobun dosyu (same race, same letters), coupled with mutualities of
economic interest, temper military tensions and offset China’s memories of past Japanese
aggression.’6 India, by contrast, has historically regarded itself, as has China, as the “Middle Kingdom.” Thus, New Delhi’s
ambitions for a global superpower status comparable to that of Beijing are a constant affront to the PRC.

Chinese moderation solves

Sutter 5 (Robert G., Visiting Professor, School of Foreign Service – Georgetown University, China’s Rise in Asia, p. 127)

A contrasting perspective gave greater weight to the common interests and forces that continued to bind Sino-
Japanese relations and to limit the chances of serious confrontation or conflict. This perspective
was more in line with the overall moderate approach China adopted toward its neighbors
under the leadership of Jiang Zemin, and especially the improvement in China’s relations with the United States and its
allies and associates, including Japan, at the beginning of the twenty-first century. Mutual interests centered on
strong, growing economic and strategic interdependence between Japan and China, and the
influence of the United States and other third parties, including other national powers in Asia—all of whom favored and could
be expected to work to preserve Sino-Japanese stability. Specific elements of the argument against the development of serious
Sino-Japanese rivalry involved:
---China Japan- Relations REsilient

Sino-Japanese relations resilient – economics

Sutter 2 (Robert, Professor – Georgetown University, “China and Japan: Trouble Ahead?”, Washington Quarterly, Autumn, Lexis)

Despite these signs of conflict, binding forces still exist. A comprehensive assessment of Sino-
Japanese relations fairly quickly gets beyond expressions of angst and signs of friction to
focus on strong and often growing areas of mutual interest. Strong external forces also are
likely to dampen any nascent rivalry between China and Japan in Asia for some time to come.
Countervailing Factors The most important of these factors is that both the Japanese and Chinese
governments are domestically focused on the economic development of their countries.
They believe that economic development requires a prolonged, peaceful, and cooperative relationship with their Asian
neighbors, notably one another. China
depends heavily on Japan for economic assistance, for technology
and investment, and as a market for Chinese goods. Japan
is increasingly dependent on China as a market,
a source of imports, and an offshore manufacturing base.

No spillover

Sutter 2 (Robert, Professor – Georgetown University, “China and Japan: Trouble Ahead?”, Washington Quarterly, Autumn, Lexis)

Fortunately, China's and Japan's recent policy and behavior in Asia show little sign of becoming
seriously divisive for the foreseeable future. Neither Beijing nor Tokyo appears to give
primary attention to offsetting the influence of the other in seeking their respective
regional goals; they are focused on more general priorities and concerns. Although Sino-
Japanese differences may flare from time to time over issues grounded in the two countries' changing
power and influence in Asian and world affairs, the differences are bounded within confines that help to
avoid serious disruption and to preserve regional stability and prosperity.
China India War Answers
Frontline

No China-India war

Carlson ’13 (Allen Carlson is an Associate Professor in Cornell University’s Government Department. He was granted his PhD
from Yale University’s Political Science Department. His undergraduate degree is from Colby College. In 2005 his Unifying China,
Integrating with the World: Securing Chinese Sovereignty in the Reform Era was published by Stanford University Press. He has
also written articles that appeared in the Journal of Contemporary China, Pacific Affairs, Asia Policy, and Nations and Nationalism.
In addition, he has published monographs for the National Committee on U.S.-China Relations and the East-West Center
Washington. Carlson was a Fulbright-Hays scholar at Peking University during the 2004-2005 academic year. In 2005 he was
chosen to participate in the National Committee’s Public Intellectuals Program, and he currently serves as an adviser to Cornell’s
China Asia Pacific Studies program and its East Asia Program. Carlson is currently working on a project exploring the issue of
nontraditional security in China’s emerging relationship with the rest of the international system. His most recent publications are
the co-edited Contemporary Chinese Politics: New Sources, Methods and Field Strategies (Cambridge University Press, 2010) and
New Frontiers in China’s Foreign Relations (Lexington, 2011). China Keeps the Peace at Sea China Keeps the Peace at Sea Why the
Dragon Doesn't Want War Allen Carlson February 21, 2013

At times in the past few months, China and Japan have appeared almost ready to do battle over the
Senkaku (Diaoyu) Islands --which are administered by Tokyo but claimed by both countries -- and to ignite a war that could be
bigger than any since World War II. Although Tokyo and Beijing have been shadowboxing over the territory for years, the standoff
reached a new low in the fall, when the Japanese government nationalized some of the islands by purchasing them from a private
owner. The decision set off a wave of violent anti-Japanese demonstrations across China. In the wake of these events, the conflict
quickly reached what political scientists call a state of equivalent retaliation -- a situation in which both countries believe that it is
imperative to respond in kind to any and all perceived slights. As a result, it may have seemed that armed engagement was
imminent.
Yet, months later, nothing has happened . And despite their aggressive posturing in the
disputed territory, both sides now show glimmers of willingness to dial down hostilities and to
reestablish stability . Some analysts have cited North Korea's recent nuclear test as a factor in the countries' reluctance to
engage in military conflict. They argue that the detonation, and Kim Jong Un's belligerence, brought China and Japan together,
unsettling them and placing their differences in a scarier context. Rory Medcalf, a senior fellow at the Brookings Institution,
explained that "the nuclear test gives the leadership in both Beijing and Tokyo a chance to focus on a foreign and security policy
challenge where their interests are not diametrically at odds." The nuclear test, though, is a red herring in terms of the conflict over
In truth, the roots of the conflict -- and the reasons it has not yet exploded -- are
the disputed islands.
much deeper . Put simply, China cannot afford military conflict with any of its Asian neighbors. It
is not that China believes it would lose such a spat; the country increasingly enjoys strategic
superiority over the entire region, and it is difficult to imagine that its forces would be beaten
in a direct engagement over the islands , in the South China Sea or in the disputed regions
along the Sino-Indian border. However, Chinese officials see that even the most pronounced
victory would be outweighed by the collateral damage that such a use of force would cause to
Beijing's two most fundamental national interests -- economic growth and preventing the
escalation of radical nationalist sentiment at home. These constraints, rather than any
external deterrent , will keep Xi Jinping, China's new leader, from authorizing the use of deadly
force in the Diaoyu Islands theater. For over three decades , Beijing has promoted peace and stability
in Asia to facilitate conditions amenable to China's economic development. The origins of the policy
can be traced back to the late 1970s, when Deng Xiaoping repeatedly contended that to move beyond the economically
debilitating Maoist period, China would have to seek a common ground with its neighbors. Promoting cooperation in the region
would allow China to spend less on military preparedness, focus on making the country a more welcoming destination for foreign
investment, and foster better trade relations. All of this would strengthen the Chinese economy. Deng was right. Today, China's
economy is second only to that of the United States. The fundamentals of Deng's grand economic strategy are still revered in
Beijing. But any war in the region would erode the hard-won, and precariously held, political capital that China has gained in the
last several decades. It would also disrupt trade relations, complicate efforts to promote the yuan as an international currency,
and send shock waves through the country's economic system at a time when it can ill afford them. There is thus little reason to
specter of rising Chinese nationalism,
think that China is readying for war with Japan. At the same time, the
although often seen as a promoter of conflict, further limits the prospects for armed
engagement . This is because Beijing will try to discourage nationalism if it fears it may lose control or be forced by popular
sentiment to take an action it deems unwise. Ever since the Tiananmen Square massacre put questions
about the Chinese Communist Party's right to govern before the population, successive
generations of Chinese leaders have carefully negotiated a balance between promoting nationalist
sentiment and preventing it from boiling over. In the process, they cemented the legitimacy of their rule. A war with Japan could
easily upset that balance by inflaming nationalism that could blow back against China's leaders. Consider a hypothetical scenario
in which a uniformed Chinese military member is killed during a firefight with Japanese soldiers. Regardless of the specific
circumstances, the casualty would create a new martyr in China and, almost as quickly, catalyze popular protests against Japan.
Demonstrators would call for blood, and if the government (fearing economic instability) did not extract enough, citizens would
agitate against Beijing itself. Those in Zhongnanhai, the Chinese leadership compound in Beijing, would find themselves between a
rock and a hard place. It is possible that Xi lost track of these basic facts during the fanfare of his rise to power and in the face of
renewed Japanese assertiveness. It is also possible that the Chinese state is more rotten at the core than is understood. That is, party
elites believe that a diversionary war is the only way to hold on to power -- damn the economic and social consequences. But Xi does
not seem blind to the principles that have served Beijing so well over the last few decades. Indeed, although he recently warned
unnamed others about infringing upon China's "national core interests" during a foreign policy speech to members of the Politburo,
he also underscored China's commitment to "never pursue development at the cost of sacrificing other country's interests" and to
never "benefit ourselves at others' expense or do harm to any neighbor." Of course, wars do happen -- and still could in the East
China Sea. Should either side draw first blood through accident or an unexpected move, Sino-Japanese relations would be pushed
into terrain that has not been charted since the middle of the last century. However, understanding that war would be a no-win
situation, China has avoided rushing over the brink. This relative restraint seems to have surprised everyone. But it shouldn't.
Beijing will continue to disagree with Tokyo over the sovereign status of the islands, and will not budge in its negotiating
position over disputed territory. However, it cannot take the risk of going to war over a few rocks in the sea.
On the contrary, in the coming months it will quietly seek a way to shelve the dispute in return for
securing regional stability, facilitating economic development, and keeping a lid on the Pandora's box
of rising nationalist sentiment. The ensuing peace , while unlikely to be deep, or especially conducive to improving
Sino-Japanese relations, will be enduring.

Sino-India conflict settled with diplomacy now – antagonism is a misperception

The Hindu ’10 (1/8/2010 (Ties with china on even keel, p. http://beta.thehindu.com/news/national/article77301.ece)

While maintaining that ties with China were on an even keel, India on Thursday was hopeful of
amicably settling the border dispute through dialogue. “We have a long border with China and talks are being
held between the Special Representatives. We are looking forward to an amicable settlement,” said External
Affairs Minister S.M. Krishna here on Thursday. The complicated issue was being handled by seasoned diplomats proficient in
issues relating to national security, he said in response to a spate of media questions on alleged Chinese intrusions in the eastern
and western sections of the Sino-India border. Dialogue
with China had helped preserve peace and
tranquillity on the border for over two decades and the confidence building measures intended
to reduce or eliminate the perception of threat from each other had worked satisfactorily well. “Let
me reiterate that India does not view China or its development as a threat. That said, however, we
recognise that cooperation and competition can overlap, as it is not possible to have a perfect congruence of interests between two
nations as vast and diverse as India and China. Such
competition or lack of cooperation must not be
misunderstood as antagonism . Our differences, when they exist, must be handled with
dialogue and diplomacy,” he said. As the border is not marked, both sides go by perceptions. And there are differences
in this, he said, while agreeing with a questioner on the lack of understanding in India about the contours of the relationship.

No chance of India China war – Copenhagen summit proves cooperation is more likely

Ramachandran ‘9 (12/24/2009 (Sudha - independent journalist/researcher based in Bangalore, India keeping up with the
neighbor, The Asia Times, p. http://www.atimes.com/atimes/South_Asia/KL24Df06.html)

China and India engaged in a heated war of words in October and November, when India permitted the
Dalai Lama to visit Tawang - territory in India's northeast over which China lays claim. A little over a month
later, the two were coordinating their strategies a t the Copenhagen climate summit, signaling there
are areas where they can join together . Will conflict or cooperation define the Sino-Indian relationship in the coming
decades? While obstacles strew the path to cooperation across the board, the fact that China and India are rising
powers in an interdependent world rules out the possibility of all-out conflict . Relations in the past
have been far from smooth. Frosty interactions resulting from the 1962 war over their disputed frontier only began to
thaw in the 1980s. Delhi and Beijing have come a long way since. The two have signed agreements to
maintain peace along their frontier and on guiding principles for current negotiations to settle the dispute. Their
armies have even engaged in joint exercises on Chinese and Indian soil . Although tension along the Line of
Actual Control (LAC) erupts from time to time, the chances of all-out war have receded .

Deterrence checks

Shihai 2 (Sun, Institute of Asia-Pacific Studies, CASS, China-Indian Relations in the 21 st


Century, 9-7)

Secondly, bothcountries have nuclear capabilities and massive sophisticated conventional


weapons. This does not only mean that they have established strategic deterrence, but it also
indicated that even a conventional war could be extremely costly. The losses would far
exceed any possible geopolitical gain. Loss and gain calculations will enable both sides
remain self-restraint at occasion of crises. In today’s world the concept of power has greatly changed. Status
and influence is not just made by military capability and economic wealth but also by responsible and respectable roles in
international affairs. To
solve problems and difference by using force or other irresponsible means would
obviously damage their international images.

CBMs check

Shihai 2 (Sun, Institute of Asia-Pacific Studies, CASS, China-Indian Relations in the 21 st


Century, 9-7)

Thirdly, it should be admitted that the


Sino-Indian relationship has increasingly become mature and
practical. Since 1980’s the two countries have begun their rapprochement process and several CBMs were
established in 1990’s. In order to understand each other’s security concerns and avoid any accidental
conflict a security dialogue mechanism was initiated in March 2000. This mechanism, if it is function
well, will greatly reduce the misperceptions and distrust as well as the possibilities of
potential conflict between the two countries.

Won’t go nuclear

Shihai 2 (Sun, Institute of Asia-Pacific Studies, CASS, China-Indian Relations in the 21 st


Century, 9-7)

It cannot say that China has no concern with India’s nuclear weapons development and deployment. As a close neighbor, China
will remain watchful of India’s nuclear policy and take reactions accordingly. However, there
will be no any
possibilities for the two countries to initiate nuclear attack against each other. Both China
and India have committed no first use nuclear weapons and the border problem and other
differences are not big enough to have a war, not to say a nuclear exchange.
---China India- Econ Checks

Economics check

UPI 5 (2-9, Lexis)

The economic dimension of the China-U.S and India-China bilateral relationships militates against
any potential for war between the competitors. The volume of U.S.-China trade, and of U.S. investments is
enormous - and growing more so with each passing year. Simultaneously, the magnitude of Chinese support in financing U.S.
deficits completely rules out any inimical posture on the part of the United States. Likewise, India-China
trade is on an
upward trajectory, and poised to grow further. In other words, it is in the economic self-interest of both
these countries to refrain from inimical moves against China.

Mutual interests check

Shihai 2 (Sun, Institute of Asia-Pacific Studies, CASS, China-Indian Relations in the 21 st


Century, 9-7)

Firstly, The development of the Sino-Indian relations basically decided by their national
interests and their respect strategic preferences. For the foreseeable future, the utmost
national interests and the primary objective of the strategy of both China and India are to
develop their economy and to safeguard a favorable security environment . Both of them
want to maintain peace along their borders, stability and access to resources in Central Asia and a stable Asian
order. It is natural that a healthy and friendly Sino-Indian relations are necessary and in the
interest of both the countries. There is no doubt that development and security concerns of both sides outweigh
their aspirations for influence and power status. In the foreseeable future the possibility of direct conflict
does not exist.

No China-India War

Malone and Mukherjee ‘10


[David M. Malone, a former Canadian Ambassador to the UN and High Commissioner to India, is president of Canada's International
Development Research Centre. He is completing a survey of Indian foreign policy called Does the Elephant Dance? (forthcoming in
2011 from Oxford University Press). Rohan Mukherjee is a senior research specialist at Princeton University. He has also worked with
the Centre for Policy Research, New Delhi, and the National Knowledge Commission, Government of India. “India and China: Conflict
and Cooperation” Survival, Volume 52, Issue 1 February 2010 , pages 137 – 158]

The Sino-Indian border dispute is long running and fairly intractable, despite shows of flexibility in the past. It
periodically prompts both sides to rake up decades-old grievances . Yet India and China have
taken meaningful steps towards an institutionalised process for its resolution. Since 1988 they
have for the most part managed to separate border issues from the overall bilateral
relationship. The long-standing relationship between China and Pakistan presents a further obstacle to closer ties between
China and India. However, China has begun to adopt a more even-handed stance , evident during the Kargil War,
the attack on the Indian Parliament in 2001, and the 2008 terrorist attacks in Mumbai. The underlying logic is that
Pakistan's growing instability and India's growing power compel China to take a middle path .
China's nuclear and missile-technology assistance to Pakistan is of particular concern to India. Future tensions between India and
Pakistan could fuel a nuclear arms race on the subcontinent. In light of the Mumbai attacks and setbacks for the Pakistani
government's efforts to contain Islamist influence in the country, however, it would be surprising if Beijing
were not
becoming somewhat wary of Islamabad, given unrest in its own Xinjiang region and the
country's persistent fear of terrorism .30 Moreover, the prospect of nuclear or military conflict
between India and China is diminished by the sizeable gap in capabilities between the two .31 Tibet
is a significant security concern. Indian parliamentarian and author Arun Shourie argues that 'India's security is inextricably
intertwined with the existence and survival of Tibet as a buffer state and to the survival and strengthening of Tibetan culture and
religion'.32 For India, the Chinese role in Tibet presents both a threat and a tactical opportunity. The presence of the Dalai Lama and
thousands of Tibetan refugees in India sometimes allows New Delhi to indirectly apply pressure on Beijing, just as China's policies
toward Pakistan sometimes do to India.33 This lever is not often used, however. In 2008, the Indian government took great pains to
ensure that Tibetan protestors did not cause any embarrassment to Beijing during the passage of the Olympic Torch through New
Delhi.34 On the other hand, at the height of tensions between the two countries over border issues during autumn 2009, a visit by
the Dalai Lama to the Buddhist temple community in the disputed Tawang, nestled in northwestern Arunachal Pradesh, can only
have been perceived as provocative by Beijing.35 Perhaps the biggest challenge to Sino-Indian rapprochement, and a source of
impetus, is the rapidly improving US-Indian relationship. While a much-improved relationship with Washington has helped India
counter the traditional pro-Pakistan tilt in US foreign policy, it has also made Sino-Indian rapprochement a greater priority for
Beijing.36 This echoes some of the history of Chinese overtures towards India in the 1970s, which were likely made in part with an
eye to diminishing Indo-Soviet cooperation. As the global contest for influence between the United States and China intensifies,
India is likely to become an important factor in this strategic triangle. US approaches to China oscillate between policies of
containment and engagement. The former has given birth to a new triangle between the United States, India and China, whereby
Washington cultivates closer ties with India, as an established democracy and as a regional bulwark against a potentially aggressive,
communist China.37 On the other hand, the Obama administration's approach to China has reinvigorated engagement enthusiasts
in Washington. Indian commentators have observed with some alarm the renewed cooperation between China and the United
States in tackling the global economic crisis, as well as increased US-Chinese interdependence resulting from Chinese creditors
holding large amounts of US Treasury Bills and US debtors providing the single largest market for Chinese manufactured goods. This
has prompted some to question the logic of picking a side in the unpredictable Sino-US relationship.38 Ultimately,
neither
China nor India stands to gain from sparking a regional conflict . Both nations are deeply engaged
in the domestic sphere, including generating economic reform, maintaining state legitimacy and
juggling ethno-nationalism. Even the ostensible machinations of the United States have done
little to hamper the current upswing in Sino-Indian relations . In some key international forums,
including those addressing climate change, trade, labour laws, arms control and human rights, China and India have found
common ground in countering Western positions, though their tactical alliances have often proved unstable in the heat of
negotiation.
China Russia War Answers
Frontline

China won't invade the Russian Far East.

Harding, ‘9 [Luke, Writer for the Observer, "Russia fears embrace of giant eastern neighbour," 8/2,
http://www.guardian.co.uk/world/2009/aug/02/china-russia-relationship]

Most experts believe China's own strategic goals do not include Russia's far east , or primitive
territorial expansion. Instead Beijing's priorities lie elsewhere. They include development, reunification
with Taiwan and internal stability , which experts suggest is more of a priority than ever
following last month's ethnic riots against Han Chinese in Xinjiang. According to Dr Bobo Lo, a lecturer on
Chinese-Russian relations at the Centre for European Reform, Beijing's real challenge to Moscow is rather different. He argues that
the rise of China will lead to the "steady marginalisation of Russia from regional and global decision-making". The Chinese do
not want to invade Russia militarily because , he points out, they would lose.

Shared interests solve

Weitz ‘11 (Richard, Director, Center for Political-Military Analysis Senior Fellow Hudson Institute, PhD in pol sci from Harvard,
China-Russia relations and the United States: At a turning point?, http://en.rian.ru/valdai_op/20110414/163523421.html, 2011)

Since the end of the Cold War, the


improved political and economic relationship between Beijing
and Moscow has affected a range of international security issues. China and Russia have expanded their
bilateral economic and security cooperation. In addition, they have pursued distinct, yet parallel,
policies regarding many global and regional issues. Yet, Chinese and Russian approaches to a range of significant
subjects are still largely uncoordinated and at times in conflict. Economic exchanges between China and Russia remain minimal
compared to those found between most friendly countries, let alone allies. Although stronger Chinese-Russian ties could
present greater challenges to other countries (e.g., the establishment of a Moscow-Beijing condominium over Central Asia),
several factors make it unlikely that the two countries will form such a bloc. The relationship between the Chinese and Russian
governments is perhaps the best it has ever been. Theleaders of both countries engage in numerous
high-level exchanges, make many mutually supportive statements, and manifest other
displays of Russian-Chinese cooperation in what both governments refer to as their developing
strategic partnership. The current benign situation is due less to common values and shared interests than to the fact that
Chinese and Russian security concerns are predominately directed elsewhere. Although both countries have experienced a
geopolitical resurgence during the past two decades, Chinese and Russian security concerns are not directed at each other but
rather focus on different areas and issues, with the notable exceptions of maintaining stability in Central Asia and constraining
North Korea’s nuclear activities. Most Chinese policy makers worry about the rise of separatist movements and Islamist
terrorism in western China and about a potential military clash with the United States in the Asia-Pacific region, especially
regarding Taiwan and the contested maritime regions of the South China and East China Seas. In contrast, most Russian analysts
see terrorism in the North Caucasus, maintaining influence in Europe, and managing security relations with Washington as the
main security challenges to their country. Neither Chinese nor Russian military experts perceive a near-term
military threat from the other’s country. The Russian government has even provided sophisticated navy, air,
and air defense platforms to the Chinese military , confident that the People’s Liberation Army (PLA) would only
employ these systems, if at all, against other countries. In addition, China and Russia have resolved their
longstanding border disputes as well as contained their rivalries in Central Asia, the Korean
Peninsula, and other regions. Since the Soviet Union’s disintegration in the early 1990s, China and
Russia have resolved important sources of their Cold War-era tensions. Through protracted
negotiations, the two governments have largely solved their boundary disputes, which had erupted in
armed border clashes in the late 1960s and early 1970s. The stoking of anti-Chinese sentiment by politicians in the Russian Far
East impeded the ability of Russia’s first President, Boris Yeltsin, to make substantial progress during the 1990s in demarcating
the Russia-China border. These politicians sought to rally local support by accusing Moscow of planning to surrender territory to
Beijing. By the mid-2000s, Yeltsin’s successor, Vladimir Putin, managed to centralize sufficient political power in the Kremlin to
ignore these local sentiments. Furthermore, Russia and China have demilitarized their lengthy shared frontier
through a series of arms control and disarmament measures. Chinese and Russian leaders share a
commitment to a philosophy of state sovereignty (non-interference) and territorial integrity (against separatism). Although
Russian and Chinese leaders defend national sovereignty by appealing to international law, their opposition also reflects more
pragmatic considerations---a shared desire to shield their human rights and civil liberties practices, and those of their allies,
from Western criticism. Chinese and Russian officials refuse to criticize each other’s foreign and domestic policies in public. They
also have issued many joint statements calling for a multi-polar world in which no one country (e.g., the United States)
dominates. During the past few years, their leaders have commonly blamed American economic mismanagement for
precipitating the global recession. They regularly advocate traditional interpretations of national sovereignty that exempt a
government’s internal policies from foreign criticism. Beijing and Moscow oppose American democracy promotion efforts, U.S.
missile defense programs, and Washington’s alleged plans to militarize outer space. The two countries strive to uphold the
authority of the United Nations, where the Chinese and Russian delegations frequently collaborate to dilute resolutions seeking
to impose sanctions on Burma, Iran, Zimbabwe, and other governments they consider friendly. In July 2008, they finally
demarcated the last pieces of their 4,300-km (2,700 mile) frontier, one of the world’s longest land borders, ending a decades-
long dispute. Chinese and Russian officials have expressed concern about the efforts by the United States and its allies to
strengthen their ballistic missile defense (BMD) capabilities. Their professed fear is that these strategic defense systems, in
combination with the strong American offensive nuclear capabilities, might enable the United States to obtain nuclear
superiority over China and Russia. Both governments have also expressed unease regarding U.S. military programs in the realm
of outer space. Russian and Chinese experts claim that the United States is seeking to acquire the means to orchestrate attacks
in space against Russian and Chinese reconnaissance satellites and long-range ballistic missiles, whose trajectories passes
through the upper atmosphere. In response, the Russian and Chinese governments have proposed various arms control
initiatives purportedly aimed at preventing the militarization of space. For example, the Russian and Chinese representatives
have unsuccessfully sought for years at the UN Conference on Disarmament to negotiate a treaty on the “Prevention of an Arms
Race in Outer Space,” which would seek to prohibit the militarization of outer space. More recently, China and Russia have
submitted a joint Space Treaty to the Conference on Disarmament in Geneva, which would impose legal constraints on how the
United States could use outer space. They have sought to link progress on other international arms control initiatives to the
adoption of these space limitations. The bilateral defense relationship has evolved in recent years to become more
institutionalized and better integrated. As befits two large and powerful neighbors, the senior military leaders of Russia and
China now meet frequently in various formats. Their direct encounters include annual meetings of their defense ministers and
their armed forces chiefs of staff. Since 1997, they have also organized yearly “strategic consultations” between their deputy
chiefs of the general staff. In March 2008, the Chinese defense minister established a direct telephone line with his Russian
counterpart, the first suchministerial hotline ever created by China and another country. In December 2008, the chiefs of the
Chinese and Russian general staffs created their own direct link. Senior Russian and Chinese defense officials also typically
participate in the regular heads of government meetings between Russia and China, which occur about once a year as bilateral
summits. They also confer frequently at sessions of multinational gatherings, such as at meetings of the SCO, which host regular
sessions for defense ministers. Contacts are even more common among mid-level military officers, especially those in charge of
border security units and military units in neighboring Chinese and Russian territories. Russian and Chinese military experts also
engage in regular direct discussions related to their functional expertise such as communications, engineering, and mapping.
Substantial academic exchanges also regularly occur. More than 1,000 Chinese students have studied at over 20 Russian military
academies since 1996. The two defense communities conduct a number of larger exchanges and engagements. The best known
are the major biennial military exercises that they have been holding since 2005, but smaller-scale engagements also frequently
occur. Chinese and Russian leaders also have developed shared perspectives and independent offensive capabilities regarding
governmental activities in the cyber domain. The two governments have been developing their information warfare capabilities
and now possess an extensive variety of offensive and defensive tools in this domain. Furthermore, recent revelations regarding
Chinese cyber-espionage activities suggest the extent to which Chinese operatives have penetrated Western information
networks. In Russia’s case, cyber attacks against Estonia, Georgia, and other countries illustrate the extensive offensive
capabilities available to that country’s forces. Russia’s hybrid August 2008 campaign against Georgia was particularly effective in
disabling Georgia’s infrastructure as well as demonstrating a potential capacity to inflict widespread physical damage. Both
countries appear to have already conducted extensive surveying of U.S. digital vulnerabilities and to have prepared targeted
campaign plans to exploit U.S. network vulnerabilities if necessary. Although these offensive and defensive preparations are
being conducted independently, the Chinese and Russian governments are collaborating, along with other Eurasian allies in the
SCO, to deny Internet resources to civil liberties groups and other opponents of their regimes. Central Asia perhaps represents
the geographic region where the security interests of China and Russia most overlap. Although China and Russia
often compete for Central Asian energy supplies and commercial opportunities, the two governments
share a desire to limit potential instability in the region. They especially fear ethnic separatism in their
border territories supported by Islamic fundamentalist movements in Central Asia. Russian authorities dread the prospect of
continued instability in the northern Caucasus, especially Chechnya and neighboring Dagestan. China’s leaders worry about
separatist agitation in the Xinjiang Uighur Autonomous Region. The shared regional security interests between
Beijing and Moscow have meant that the newly independent states of Central Asia---Kazakhstan, Kyrgyzstan, Tajikistan,
Turkmenistan, and Uzbekistan---have become a generally unifying element in Chinese-Russian
relations. Their overlapping security interests in Central Asia have manifested themselves most
visibly in the Shanghai Cooperation Organization (SCO).

China isn’t stupid enough to attack Russia

Menon 2003 (Rajan, Rathbone Professor of International Relations at Lehigh University, The
National Interest, Fall)

By contrast, China's military, which was quite recently a giant horde of foot soldiers, is modernizing steadily-chiefly
with Russian weaponry, much of it supplied from cash-starved military industries in Khabarovsk, Komsomol'sk and
Vladivostok. It may lag far behind the United States, but in force projection, speed, accuracy and lethality it is a wholly
different force than it was a decade ago, thanks to Russian fighter jets , submarines, tanks and
missiles, many of them built in the Russian Far East. Yet the chances that China will attempt to conquer
Russia's Far East are slim. Such a brazen power play would damage China's wider interests . Taiwan
might recoil in terror and treat Beijing's proposals for a negotiated reunification with even greater skepticism and wariness. The
prevailing Western rationale for economic engagement with China-that commerce will
transform and co-opt that country-would be shredded. China would likely face a
counterbalancing, encircling coalition of the United States, India, Japan, Russia and Vietnam .
Would such setbacks justify the burdens of ruling the vast, problem-infested Russian Far East? The Chinese leaders know their Sun
Tzu: what they seek from the Russian Far East (access to resources and a benign northern front) can be had by means of silk-gloved
hegemony. Chinese interests can be served without its formal occupation of the territory . Indeed,
what may emerge could be a "reverse Manchurian" scenario, where the Russian Far East remains a titular part of Russia but is
increasingly integrated into Beijing's sphere of influence. That is precisely what the conspiracy among geography, demography,
power and time may create in Russia's Far East.

Negotiations solve

Chicago Tribune 4 (10-15, Lexis)

China and Russia settled the last of their decades-old border disputes Thursday during a visit to Beijing by
President Vladimir Putin, signing an agreement fixing their 2,700-mile-long border for the first time .
The struggle over border areas resulted in violent clashes in the 1960s and 1970s, when
strained Sino-Soviet relations were at their most acrimonious, feeding fears abroad that the conflict could erupt
into nuclear war. Beijing and Moscow had reached agreements on individual border sections as relations warmed in the
past decade. But a stretch of river and islands along China's northeastern border with Russia's Far East had remained in dispute.
Fear of escalation checks

Moriarty 4 (Tom, Military Intelligence Analyst – U.S. Air Force, World Affairs, 9-22, Lexis)

However, the Soviet Union ultimately chose to forgo a preemptive attack and attempted to defuse tensions
through diplomatic channels. Numerous reasons led Soviet leaders to decide against preemptive attack. The main reason
was the Soviet Union's fear that even if they could destroy all of China's nuclear weapons
capability (which, in itself, was a big assumption), they feared a conventional attack by China. Like the United
States during the Cuban missile crisis, the Soviet Union understood that they would lose the ability to prevent the crisis from
escalating into a full-blown war. Soviet leaders grew concerned that China would respond with a
prolonged people's war against the Soviet Union. Knowing that a prolonged war against a country with
more than one billion people and a proven resiliency would exhaust the Soviet Union and would require forces to
be withdrawn from Eastern Europe, Soviet leaders chose to ignore the Chinese provocations and let the
confrontation defuse naturally. (12)

China will not invade Russia – they would lose

Open Democracy 8 (“Russia-China: Axis of Convenience”, http://www.cer.org.uk/articles/lo_opendemocracy_20may08.html)

The real threat is this: China's rise will lead to Russia's steady marginalisation from regional and global decision making. The
Chinese do not intend to invade Russia militarily, because they would lose . The
consequences would be too horrific to be contemplated . They are not going to fill the Russian Far East
with lots of Chinese. Those northern regions have always been considered a barbarian outland. The Chinese who go
there want to make a quick buck before returning home . Although they receive half the salary of local
Russians, it is still much more than they would get in northeastern China or in the countryside. But very few Chinese go to the
Russian Far East to live there.
China Soft Power Answers
Frontline
Chinese soft power down now- expansionism

Ackerman, 1-31 – Signal magazine editor-in-chief


[Robert, "China Behavior Increasingly Troublesome to Neighbors," Signal Online, 1-31-13, www.afcea.org/content/?q=node/10625,
accessed 2-2-13, mss]

As the People’s Republic of China grows in economic and military stature, it is generating ill will among
neighbors who increasingly fear an expansionist budding superpower. Ironically, the greatest effect this
is having on the Asia-Pacific region is that it is driving many nations into the arms of the U nited States. This was just
one of many observations offered by a panel on China at AFCEA/USNI West 2013 in San Diego. A mix of academics and military
officers offered different perspectives on where China might be headed in the coming years. Capt. Jim Fanell, USN, deputy chief of
staff for intelligence and information operations, U.S. Pacific Fleet, said that China
has taken control of areas outside
its borders that never have been administered to, or controlled by, any government of China in recent history.
China’s coastal cutters seem to have no other mission than to harass others to submit to its territorial
claims. The result is that the countries of East Asia “now remember why they like the U nited States,”
he said. Dr. Jacqueline Deal, president and chief executive officer, Long-Term Strategy Group, related how China’s foreign minister
told then-Secretary of State Hillary Clinton that, “there are great powers, and there are small powers—and that’s a fact.” This
statement amounted to tacit approval for the Middle Kingdom to push its neighbors around, Deal said. Maj. Christopher I. Johnson,
USMC, Olmsted scholar, Hong Kong University, and logistics officer, Marine Barracks Washington, D.C., observed that China’s leaders
believe in hard power—“you cannot export soft power.” Yet, Johnson believes that China currently is a competitor, not an enemy.

Chinese hard power kills soft power approach

Feffer, 1-21 -- Open Society fellow


[John, "When Soft Power Fails," Huff Post, 1-21-13, www.huffingtonpost.com/john-feffer/china-soft-power_b_2519954.html,
accessed 2-2-13, mss]

China, after all, has not restricted itself solely to the exercise of soft power to increase its influence in the
region. In 1994, in an attempt to claim disputed territory in the South China Sea, China built structures on Mischief Reef, well within
the Exclusive Economic Zone of the Philippines. Manila didn't respond militarily to this provocation. But it has asserted its claims to
the disputed region in other ways, for instance by arranging a seismic survey to determine underwater oil resources. Chinese patrol
boats attempted to disrupt the survey. Later, Manila seized Chinese fishermen operating in the area, and China retaliated by refusing
boatloads of Philippine bananas. Although China has not asserted its claims to the South China Sea through overwhelming military
force, its rhetoric can be quite overreaching. Beijing's "nine-dash line" concept of Chinese sovereignty takes about as large a bite as
possible out of the South China Sea. In 2012, Chinese military spending went up 11 percent , pushing its official
spending over $100 billion. For the United States, which spends about seven times as much on the military, Chinese military
increases don't represent a significant threat. But for the Philippines and other Southeast
Asian countries, Chinese
spending has contributed to a major arms race in the region. With its soft-power overtures, China has
tried to expand its influence without disquieting its neighbors and trade partners. But through its hard-power
posturing, China has achieved the exact opposite. The Philippines, for instance, has moved inexorably closer to
the United States in an effort to balance China. Manila negotiated a doubling in Foreign Military Financing from the United States in
2012 and has made its military bases more accessible to U.S. forces. This in a country that unceremoniously kicked the United States
out of the Subic and Clark military bases back in 1991. The Philippines has also sought U.S. help in regional organizations to push
back against China on the issue of the disputed territory. The
South China Sea is not the only place where
Chinese hard power is undercutting its soft power. Currently China and Japan are escalating their claims over the
Diaoyu/Senkaku islands. The Japanese government recently "bought" the islands from a private owner; China has sent planes to
monitor Japanese fighter jets in the area. The fact that China is Japan's leading export market has not seemed to moderate
statements coming out of Tokyo.
China Hegemony Answers
Frontline
China heg decline now

Pei ‘12
Minxin, professor of government at Claremont McKenna College and a nonresident senior fellow at the German Marshall Fund of
the United States. “Everything You Think You Know About China Is Wrong”Are we obsessing about its rise when we should be
worried about its fall? FOREIGN POLICY

For the last 40 years,


Americans have lagged in recognizing the declining fortunes of their foreign
rivals. In the 1970s they thought the Soviet Union was 10 feet tall -- ascendant even though corruption and inefficiency were
destroying the vital organs of a decaying communist regime. In the late 1980s, they feared that Japan was going to economically
overtake the United States, yet the crony capitalism, speculative madness, and political corruption evident throughout the 1980s led
to the collapse of the Japanese economy in 1991. Could the same malady have struck Americans when it comes to China? The
latest news from Beijing is indicative of Chinese weakness: a persistent slowdown of economic
growth, a glut of unsold goods, rising bad bank loans, a bursting real estate bubble, and a
vicious power struggle at the top, coupled with unending political scandals. Many factors that have
powered China's rise, such as the demographic dividend, disregard for the environment,
supercheap labor, and virtually unlimited access to external markets, are either receding or
disappearing. Yet China's declining fortunes have not registered with U.S. elites, let alone the
American public. President Barack Obama's much-hyped "pivot to Asia," announced last November, is premised on the continuing
rise of China; the Pentagon has said that by 2020 roughly 60 percent of the Navy's fleet will be stationed in the Asia-Pacific region.
Washington is also considering deploying sea-borne anti-missile systems in East Asia, a move reflecting U.S. worries about China's
growing missile capabilities.

Their authors are misinformed political pandering – China’s decline is deeply rooted

Pei ‘12
Minxin, professor of government at Claremont McKenna College and a nonresident senior fellow at the German Marshall Fund of
the United States. “Everything You Think You Know About China Is Wrong”Are we obsessing about its rise when we should be
worried about its fall? FOREIGN POLICY

In the lead-up to the Nov. 6 U.S. presidential election, both


Democrats and Republicans have emphasized
perceived Chinese strength for reasons of both national security and political expediency.
Democrats use China's growing economic might to call for more government investment in education and
green technology. In late August, the Center for American Progress and the Center for the Next Generation, two left-leaning
think tanks, released a report forecasting that China will have 200 million college graduates by 2030. The report (which also
estimates India's progress in creating human capital) paints a grim picture of U.S. decline and demands decisive action.
Republicans justify increasing defense spending in this era of sky-high deficits in part by citing predictions that China's
military capabilities will continue to grow as the country's economy expands. The 2012 Republican Party platform, released in late
August at the Republican National Convention, says, "In the face of China's accelerated military build-up, the United States and our
allies must maintain appropriate military capabilities to discourage any aggressive or coercive behavior by China against its
neighbors." The disconnect between the brewing troubles in China and the seemingly unshakable perception of Chinese
strength persists even though the U.S. media accurately cover China, in particular the country's inner fragilities. One
explanation for this disconnect is that elites and ordinary Americans remain poorly informed about China
and the nature of its economic challenges in the coming decades. The current economic slowdown in Beijing is
neither cyclical nor the result of weak external demand for Chinese goods. China's economic ills are far more deeply
rooted: an overbearing state squandering capital and squeezing out the private sector, systemic
inefficiency and lack of innovation, a rapacious ruling elite interested solely in self-enrichment
and the perpetuation of its privileges, a woefully underdeveloped financial sector, and
mounting ecological and demographic pressures. Yet even for those who follow China, the prevailing wisdom is
that though China has entered a rough patch, its fundamentals remain strong.
China Navy/A2AD Answers
Frontline
We control uniqueness- A2AD threat exaggerated, US has maritime dominance now

WCT, 12 – citing a report from the Swiss think tank, Center for Security Studies and Conflict Research
[Want China Times, staff reporter, "China unable to defeat US at sea: Swiss thinktank," 12-22-12, www.wantchinatimes.com/news-
subclass-cnt.aspx?id=20121222000049&cid=1101, accessed 1-27-13, mss]

China unable to defeat US at sea: Swiss thinktank

An article from the Zurich-based Center for Security Studies and Conflict Research on Dec. 18 says the PLA Navy's
ability to wage a nti- a ccess/ a rea d enial warfare against the United States in the Western Pacific has been
highly exaggerated. Even though China would be able to withstand a US attack during the early stages of a campaign, it
would be unable to defeat the US and its allies in a war of attrition, the piece said. Electronic
countermeasures would also render China's much-vaunted DF-21 "carrier killer" missile useless, the article
said, concluding that China would be unable to prevent an American intervention in East Asia, and
furthermore that it would be unable to capture Taiwan without confronting US forces. While analysts around the globe are taking
note of the significance of Chinese naval bases in Sri Lanka, Myanmar and Pakistan, the article said it
is already hard enough
for the PLA Navy to fight a war against the U nited States near the China coast. China would not be
able to take on the US in the Western Pacific even if it builds two of its own aircraft carriers by 2020.
Although China is trying to increase its sub marine fleet from 60 to 75, the article doubted the ability of
the PLA to fight an anti-submarine campaign against the US Pacific Fleet, suggesting that Chinese submarines
may not be able to provide sufficient protection for a carrier battle group. Since the targeting computer of the DF-
21 can be scrambled by the US electronically, options such as ballistic missiles may not be effective. As China is surrounded by the
United States and its regional allies in the Asia-Pacific, the article stated that the
country is unable to become a
genuine maritime superpower even it has the ability to build warships to a high standard.

No china naval challenge now

O'Rourke, 12 -- CRS Naval Affairs specialist


[Ronald, "China Naval Modernization," CRS/FAS, 12-10-12, www.fas.org/sgp/crs/row/RL33153.pdf, accessed 1-27-13, mss]

The PLAN is acquiring the hardware it needs to prosecute a major regional naval showdown. Simultaneously, an increasingly-
capable, but still limited number, of vessels can fight pirates, rescue Chinese citizens trapped by violence abroad, and make “show-
the-flag” visits around the world. But the PLAN is
not set up to confront the U.S. at sea more than 1,000
miles from China. Even if the PLAN surged production of key vessels such as replenishment ships, the
resources and steps needed to build a globally-operational navy leave Beijing well over a decade
away from achieving such capability in hardware terms alone. Building the more complex human software and
operational experience needed to become capable of conducting large-scale, high-end out-of-area deployments could
require at least another decade. Meanwhile, however, China’s challenges at home and on its
contested periphery remain so pressing as to preclude such focus for the foreseeable future .

Naval upgrades are slow and moderate


Goldstein 11—Professor and Director of the China Maritime Studies Institute @ US Naval War College [Dr. Lyle J. Goldstein,
“Resetting the US–China Security Relationship,” Survival | vol. 53 no. 2 | April–May 2011 | pp. 89–116

To be sure, Beijing has made some progress in key areas of military technology, but its naval building programme is
moderate, not radical. China will not have a strategically significant aircraft-carrier force for at
least another decade, and has taken a measured and rather unhurried approach to upgrading
both its fleet of surface combatants and its amphibious attack ships. Significantly, China is still far from wielding a
strong nuclear-submarine force. Instead, its major focus has been on building conventional
submarines that are more oriented toward defensive operations in the Chinese littorals than on
power-projection missions in blue water. Of course, the recent unveiling of the fifth-generation J-20 fighter, alongside many
other advanced systems, adds additional evidence to an already long list of new capabilities that illustrate that China is quite capable
of building sophisticated weaponry. Taking the long historical view, this should not be shocking, nor should Beijing’s determination
to build armed forces commensurate with its new status. What
has been truly shocking, again taking the long view, is
how weak China has been for so long in the modern era – a major historical anomaly . Pg. 90-91
China Rise Answers
Frontline

China rise won’t be hostile

Feffer ’12 (John Feffer, February 21, 2012, Our Man in Beijing? http://www.fpif.org/articles/our_man_in_in_beijing?
utm_source=feedburner&utm_medium=feed&utm_campaign=Feed%3A+FPIF+%28Foreign+Policy+In+Focus+%28All+News%29%29

When Hu Jintao took over as the leader of China in 2002, U.S. companies welcomed his accession as a “good sign
for American business.” Political analysts described Hu as a fourth-generation member of the
Communist party leadership who might very well turn out to be a “closet liberal .” Playing it safe, the
media tended to portray him as a pragmatic enigma. In the wake of 9/11 and high-level cooperation on counter-terrorism , Hu
proved to be a reliable U.S. partner, prompting Colin Powell to remark in 2003 that U.S.-China relations were the best
since 1972. It didn’t take long, however, before the media and the punditry turned sour on Hu. By 2005, The Economist was labeling
him a “conservative authoritarian” for tightening party discipline and cracking down on intellectuals. Hu also came under fire for
holding firm against the United States around disputes over trade, currency, intellectual property, and human rights. On
counter-terrorism, U.S.-Chinese interests converged . But on this issue and most others, Hu turned out not to be a
closet liberal at all. And when it came to prosecuting the “global war on terror,” the Bush administration didn’t want a liberal. Now,
with China gearing up for another leadership transition, Hu’s putative successor Xi Jinping has embarked on his own grand tour of
the United States. As with Hu, Western sources admit that they don’t know very much about Xi beyond his generally “pro-business”
approach. He has a celebrity wife; he doesn’t like corruption; he’s a basketball fan. His father was a Party loyalist until he began to
sympathize with the Tiananmen Square protestors. Aside from these tidbits, journalists have been forced to sift through Xi’s U.S.
appearances – his meetings with the Obama administration, his return to the Iowa town he visited 25 years ago, his attendance at an
LA Lakers game – for clues to the new Chinese leader’s true political nature. Xi Jinping did what he could to frustrate the media.
He was careful to tailor his remarks in Washington to satisfy both his Western hosts and his colleagues back home. So, for instance,
he spoke of U.S.-Chinese relations as an "unstoppable river that keeps surging ahead" and of
Beijing’s willingness to engage with Washington on a broad agenda of issues from counter-
terrorism to North Korea. At the same time he was careful to warn his hosts to “respect the interests and the concerns of
China.” This latter point, that China has its own national interests, invariably eludes Western observers no matter how often Chinese
leaders repeat it. Sure, a Chinese leader might like American basketball or admire American business. But the essential fact is that he
leads a political, economic, and military apparatus dedicated to preserving itself and the country’s territorial integrity. The same can
be said for the leaders of most countries, including the United States. Certainly no one in Beijing expects the 2012 U.S. elections to
produce an American president who embraces state capitalism, a global trade order that disproportionately favors Chinese
economic growth, or a ceding of U.S. military position in the Pacific to the up-and-coming superpower. And yet for some bizarre
reason, U.S. observers expect the latest Chinese leader to suddenly tear off his clothes and reveal a Captain America suit
underneath. China’s national interests are perhaps most visibly on display around security issues. During the early Hu years, the
discussion in the West centered on China’s “peaceful rise.” More recently, the talk has gotten darker, as
pessimists point to China’s recent purchase of an old Ukrainian aircraft carrier, its ambitions in the South China Sea, its confrontation
with Japan over the disputed Senkaku/Diaoyu islands, and of course its increased spending on the military. By 2015, according to IHS
Jane’s, Chinese military spending will reach $238 billion, more than all the projected spending in the Asian region as a whole. But
there are no real indications that Beijing has abandoned its “peaceful rise” approach. The
refurbished aircraft carrier is not terribly impressive (particularly compared to the U.S. Navy’s 10
modern vessels). South Korea and Japan have a similar row over a disputed island, which might lead to the conclusion that it’s Japan,
not China, that’s abandoning its “peaceful rise.” China’s claims to islands in the South China Sea, however dubious, are longstanding
and date back to the pre-communist era. And it’s been more than 30 years since China has conducted a significant military
intervention overseas, an overall pattern of risk-averse behavior it shows no sign of abandoning. In any case, what might tip the
region into conflict is not China’s territorial ambitions but climate change. “As sea temperatures in the South China Sea continue to
rise, large quantities of fish will migrate north into even more heavily disputed waters,” writes Foreign Policy In Focus (FPIF)
contributor Derek Bolton in Shifting Winds in the South China Sea. “As fishermen are forced to follow suit, the probability of future
confrontations will increase, raising the likelihood of a more serious conflict.” The
United States, meanwhile,
continues to outspend China militarily by at least five-fold and is in the midst of a “Pacific pivot”
to reorient its security policy away from the Middle East and toward Asia. Increased U.S. military
cooperation with Australia, the Philippines, and even Vietnam makes China nervous. China’s increased military spending is not a
happy sign, but the
leadership believes it has a long way to go before achieving even rough parity
with its major rival. The overarching priorities of Chinese leaders remain nationalist: to keep a vast and fractious country
together, maintain influence in Taiwan, and ensure a steady supply of energy through its neighboring regions to sustain high levels
of economic growth. Hu
and now Xi consistently tell their U.S. interlocutors that closer U.S.-Chinese
relations are possible and desirable as long as Washington recognizes these national imperatives.

No China military threat

Erickson & Liff ‘3-7


(Andrew, S. & Adam, P. 2013, Andrew is an associate professor in the U.S. Naval War College's China Maritime Studies Institute.
Adam is a doctoral candidate in Princeton University's Department of Politics. Foreign Policy. “A Player, but No Superpower: Why
China's military is a threat to its neighbors, but shouldn't concern the United States on its home turf.”

http://www.foreignpolicy.com/articles/2013/03/07/a_player_but_no_superpower_china_military)

Even with this surging investment, there are several major obstacles to China's developing military capabilities
potent far beyond the Near Seas. Inefficiencies still weaken the PLA, which has an estimated 2.285 million active-duty
personnel, and dilute the impact of spending increases. Some commentaries in influential Chinese military journals have
charged that the PLA's procurement strategy increasingly focuses on mimicking overseas developments in arms and
equipment technology, even though more basic strategic goals, like Beijing's island and maritime claims in the Near
Seas, remain unresolved. The PLA lacks robust internal mechanisms for analyzing or evaluating its
equipment procurement needs, and a growing percentage of the defense budget appears to be
earmarked toward prestige-driven, highly publicized, and extremely expensive programs that don't yield
top-end military capabilities. China's aircraft "starter carrier," for example, is not only extremely vulnerable
to missiles and submarines, but is also years away from reaching the capabilities that the United
States possesses today. And the reconnaissance options that China's manned space program offers could be provided more
cheaply via unmanned satellites. Corruption remains a serious problem in the military. "No country can defeat
China," PLA Gen. Liu Yuan said in meeting with roughly 600 officers in December 2011. "Only our own corruption can destroy us and
cause our armed forces to be defeated without fighting." In February, Xi Jinping, China's soon-to-be president, launched a campaign
to impose stricter discipline and oversight in the military. Yet it will take more than limiting military banquets to "four dishes and a
soup," a policy Xi has called for, to solve the PLA's problems and enable it to become one of the world's most sophisticated
militaries. China scholar Minxin Pei has warned that by
pursuing gradualist, incomplete reforms, Beijing risks a
"trapped transition" instead of transformation into a full-fledged market economy. Signs of an analogous
trap are also apparent in the military, as it strives to transition from a domestic, Near Seas-focused,
personnel-intensive force to one characterized by a broader geographical mandate, advanced
technology, and innovation. A slowdown in enhancement of military capabilities looms: The PLA's
rapid progress in recent years means that fewer easy improvements and fixes remain. But the closer
the PLA gets to possessing cutting-edge capabilities in defense technology, the more difficult it becomes to advance further -- much
ofthe low-hanging fruit has already been plucked. Well-educated and technologically capable
personnel cost significantly more to attract, train, and retain than China's erstwhile massive peasant land army,
particularly when private-sector alternatives provide significantly greater compensation. Other personnel expenses -- including
health care and retirement costs -- which constitute major challenges to the U.S. military budget, may hit China
even more rapidly given its less-favorable demographics. Despite its progress in modernizing and some remarkable
new hardware, the PLA's war-fighting capabilities still significantly trail those of the U.S. military. They may
get trapped there, even though a growing number of influential actors have called for China to expand its military horizons. The
likelihood that the PLA will get trapped in its region -- with respect to high-end war-fighting capabilities -- will increase if
Beijing's
growing military power and assertiveness lead its neighbors to further accelerate their own counterbalancing .
China's geographical, economic, and (in some cases) technological advantages do not transfer to
capabilities that would allow it to engage in high-intensity combat beyond the country's
immediate periphery. In other words, China can stir up the Near Seas, but can't make tsunamis
beyond that.

No China threat

Desker and Bitzinger ‘8 *Senior Fellow at the S. Rajaratnam School of International Studies, **Dean of the S. Rajaratnam
School of International Studies and Director of the Institute of Defense and Strategic Studies, Nanyang Technological University,
Singapore (Richard and Barry, Survival 50:6, "Why East Asian War is Unlikely", pages 105-28, EBSCO, WEA )

Overall, most Western assessments agree that the PLA has made considerable progress over the past decade in adding new weapons to its arsenal, and
that China has noticeably improved its military capabilities in several specific areas – particularly missile attack, power projection over sea and in the air,
and information warfare. Most predict that Chinese military power relative to its likely competitors in the
Asia-Pacific region – especially Taiwan – and the United States will continue to increase
significantly over the next ten to 20 years. There are, however, some striking differences of opinion when it comes to
interpreting the significance of these hardware developments. Many Western analysts assert that the PLA continues to

suffer from considerable deficiencies and weaknesses that limit its ability to constitute a major
military threat: in spite of all its efforts, China is still at least two decades behind the United
States in terms of defence capabilities and technology. In particular, the PLA still lacks the
logistical and lift capacity – both by sea and by air – for projecting force much beyond its
borders. China also lags far behind the West in areas such as C4I architectures and surveillance
and reconnaissance capabilities. Some therefore argue that China’s current rearmament programme is
an incremental, long-term modernisation process that must be understood in the context of
competing force-modernisation activities taking place among China’s likely rivals.

Moreover, what
the PLA has been engaged in over the past 10–15 years may not really be a
revolution in military affairs at all. First, there is very little evidence that the Chinese military is
engaged in an revoutionary overhaul of its organisational or institutional structures . According to Jane’s
Defence Weekly, the PLA, despite its emphasis on the ‘importance of information technology and knowledge-based warfare on the battlefield ... has yet
to promulgate a definitive military doctrine to guide the development of capabilities and operations in this area’.38 Most
of China’s
ground forces remain traditional infantry units, hobbled by a shortage of rapid-mobility assets
such as helicopters, airlift or amphibious lift .39 The PLA’s highly hierarchical and top-down command
structure and its inter-service compartmentalisation does not seem to have changed, and even the Pentagon acknowledges the
PLA’s deficiencies when it comes to things like interoperability.40

Secondly, while the Chinese military is certainly acquiring new and better equipment, little of it could be construed as particularly revolutionary, or be
seen as leapfrogging a generation of weapons development. For example, using shortand medium-range ballistic missiles as precision-attack systems
may be a unique approach, but in China’s case this may be more a matter of making a virtue out of a necessity – the PLA simply lacks sufficient numbers
of other types of precision-guided munitions, particularly for land attack. In addition, systems such as the J-10 fighter jet, the Song-class diesel-electric
submarine and the Type-52C Luyang II-class destroyer (which is equipped with an Aegis-type air-defence radar), while advanced for the PLA, are
basically 1980s-era weaponry, technologically speaking. The J-10, for instance,is operationally comparable to the US F-16C, which first entered service
in the mid 1980s. Even the equipment the Chinese have acquired from Russia (Su-27 fighters, Sovremennyy-class destroyers, Kilo-class submarines, and
S-300 surface-to-air missiles) – arguably the sharpest edges of the PLA spear – are hardly cutting-edge, transformational weapons systems. Moreover,
most Chinese weapons systems coming online today were developed more or less sequentially – that is, following traditional patterns of incremental
research and development. For example, Chinese fighter-aircraft development has moved in a fairly routine fashion from second-generation (J-7/ MiG-
21) to third-generation (J-8) to fourth-generation (J-10) systems – acknowledging, of course, the 20-year period of near-total absence of new research
from the mid 1960s to the mid 1980s. Overall, therefore, Chinese military transformation looks fairly mundane, consisting mostly of buying new
platforms, such as fighter jets, warships, submarines, missile systems, tanks and other weapons.

Thirdly, modernisation does not seem to be accelerating. Indeed, over the past few years the pace of PLA arms
acquisitions has declined in some areas. The
Chinese have not laid down a new destroyer in more than three
years. In 2006 and 2007, Chinese overseas arms purchases were worth, respectively, $100 million and $150m, a far cry from the $2.8bn-worth of
foreign weapons systems it bought in 2005. The most recent edition of the US Defense Department’s annual report on Chinese military power shows
that, while the PLA has made considerable progress in incorporating ‘modern’ weapons systems into its forces, as of 2007, 70% of its surface
combatants, 60% of it submarine force and 80% of its fighter jets were still considered ‘old’.41 Clearly, Chinese military modernisation has a long way to
go.

Our carriers would crush china

Henderson ‘9 (Bruce, “China’s “Kill Weapon” – Don’t Make Me Laugh”, http://andstillipersist.com/2009/03/chinas-kill-weapon-


dont-make-me-laugh/, WEA)

The Dongfeng (East Wind) 21 is a land based medium range ballistic missile. It is a two stage,
solid fuel rocket. By using solid fuel, it is easier to set up and launch as it does not require
cryogenic cooling or propellents. China likes to deploy these on mobile launchers that they can
easily move around to hide them from anyone who might want to make sure they never launch
them.

What the US Naval Institute is implying, without spelling it out, is that China has claimed that
they have developed a very accurate warhead that can be targeted against ships at sea , a land-
based anti-ship ballistic missile based on the DF-21. This warhead would be nuclear, and with a
big enough yield, the Chinese could “Kill” a US carrier. While it is true that if you can toss a live
nuke close enough to a carrier, you can sink it, the premise of the entire write up is so far from
credible as to be a joke.

One has to wonder how and why this article surfaced. Is the Navy or it’s supporters attempting
to stoke support for development money for the Aegis systems? There was at one time a
proposal to enhance Aegis to attempt terminal intercept capability. Or is it just time to once
again realize that China is steadily moving to expand its sphere of influence in Asia?

Make no mistake, the events that would lead to a DF-21 fired at a carrier battle group would be
well down the road of a full fledged and escalating war with China, and would signal the
beginning of a nuclear exchange that would see hundreds of millions die, mostly in China. While
it is not public knowledge, China knows that the location of their missiles, command and control
and launch authorities have been infiltrated, measured to within the inch, and can be targeted
within seconds should anyone ever be so stupid as to use a DF-21 against US assets. The chances
of this ever happening are so close to zero as to not even merit consideration , let alone the level
of concern Raymond Pritchett has raised.
Chinese influence isn’t zero sum with the west --- shared regional values mitigate the risk of
conflict

Bitzinger and Desker ‘8 (senior fellow and dean of S. Rajaratnam School of International Studies respectively (Richard A.
Bitzinger, Barry Desker, “Why East Asian War is Unlikely,” Survival, December 2008,
http://pdfserve.informaworld.com-/678328_731200556_906256449.pdf)

The argument that there is an emerging Beijing Consensus is not premised on the rise of the
East and decline of the West, as sometimes seemed to be the sub-text of the earlier Asian-
values debate.7 However, like the earlier debate, the new one reflects alternative philosophical traditions. The issue is the
appropriate balance between the rights of the individual and those of the state. This emerging debate will highlight
the shared identity and values of China and the other states in the region, even if conventional
realist analysts join John Mearsheimer to suggest that it will result in ‘intense security competition with
considerable potential for war’ in which most of China’s neighbours ‘will join with the United
States to contain China’s power’.8 These shared values are likely to reduce the risk of conflict
and result in regional pressure for an accommodation of and engagement with an emerging
China, rather than confrontation.

China isn’t a threat --- conservatives blow a hypothetical Chinese conflict way out of
proportion

Guardiano ‘10 (Writer and analyst who focuses on political, military, and public-policy issues (John Guardiano, “Overstating the
China Threat,” FrumForum, May 13th, 2010, http://www.frumforum.com/overstating-the-china-threat)

Devore, in fact, has it exactly backwards: We have to prepare for the real enemy, and it’s not China. The real and immediate enemy
is a network of Islamic radicals determined to destabilize the world and wreck havoc and destruction on America and the West. Yet,
China is what preoccupies the Weekly Standard’s Noonan, Goldfarb and indeed, most conservative defense
hawks. To be sure, China is a potential military threat. The United States certainly should maintain military superiority over China;
and we certainly should guarantee the independence of Taiwan. But the Right’s obsession with a hypothetical and
distant Chinese military threat is seriously misplaced and inappropriate — especially given the
wartime exigencies of today. American Soldiers and Marines are being targeted and killed, after
all, not by China, but by Islamic radicals in Iraq and Afghanistan. And it is this global war against the Islamists — and not a
distant, hypothetical war with China — that is the future of warfare. It’s a future involving lots of messy asymmetric fights in which
American troops are integral to stability, security, and gradual, long-term democratization. It is not, however, a future that
conservatives like or wish to accept. Conservatives don’t like messy asymmetric fights which involve counterinsurgency and nation
building: because to many on the Right, that’s not “real war.” That’s not the role and purpose of the U.S. military. The
Right
dreams or imagines, instead, of a conventional “big war” with China. Dream on, because it ain’t gonna
happen, not in our lifetime anyhow. The Chinese are interested in making money, not war. Their
increasing military prowess is a natural and inevitable reflection of their growing economic
strength and vitality. Indeed, as a country modernizes and develops, so, too, does its military.
Again, I’m not suggesting that we let our guard down with China. I’m simply saying that we view the potential Chinese military threat
in context and with perspective and that we plan and budget accordingly. Unfortunately, the Right’s misplaced obsession with China
has deleterious real-world consequences. It causes conservatives to too often give short shrift to the existential Jihadist threat that
now confronts us, and too little attention to the war we are now fighting in Iraq, Afghanistan and elsewhere . What’s more, because
the Right has yet to come to terms with the nature of 21st irregular asymmetric warfare, it has been AWOL and ineffective in the
defense budget battles of recent years. For example, when President Obama and Defense Secretary Robert Gates last year pushed
dramatic defense budget cuts in the name of canceling “Cold War weapon systems,” most conservatives were flummoxed and
stymied. They rightly sensed that eliminating some of our most advanced weapon systems was a bad idea. However, conservatives
also realized that the world and warfare had changed, and that defense budget reform might well be necessary. Conventional set-
piece battles, after all, are largely a thing of the past. Except that they’re not, because in the minds of conservative
hawks, the Chinese military threat is always looming .  Thus, the Right fell back on old and dated Cold War modes
of analysis, lamenting the loss of aircraft like the F-22 — even though the F-22 has not been used in either Iraq or Afghanistan, and
even though modern-day conflicts are inherently land-based and ground-force intensive. My point is not that we don’t need any
more F-22s, because we might. My point is that conservatives should focus their intellectual and rhetorical firepower on more
relevant and urgent military priorities like the need for ground-force modernization, a new Army combat vehicle, and networking
our Army and Marine Corps with state-of-the-art communication capabilities. But the sad reality is that most conservative defense
hawks — and certainly most conservative politicians and elected officials — haven’t a clue about U.S. military requirements. And
they are especially clueless about the needs of our ground-force Soldiers and Marines. That’s why conservatives last year lost the
defense budget battle; and that’s why they’re still losing and losing badly: because they have yet to come to terms with new
geostrategic and military realities. They’re
stuck in a Cold War time warp and are mistakenly focused on
China. But the Chinese are eager to sell us commercial goods; they are not eager to destroy our
cities and our people. The same cannot be said, however, of the Jihadists who plan and plot for our destruction. You’d think
that nine years after the terrorist attacks on the Pentagon and World Trade Center, and with wars still raging in Iraq and Afghanistan,
conservatives would understand this. But alas, you would be wrong. The Right still doesn’t get it. But they should and they must. The
fate of American national security, and the survival of our Soldiers and Marines, hangs in the balance. Time to modernize our
thinking. Now.

Even if China was aggressive, the impact is mitigated --- only wants to tweak and anything else
would take too long

Jones ‘7 – foreign affairs at University of St. Andrew (“China’s Rise and American Hegemony: Towards a Peaceful Co-Existence?”
E-International Relations, 2007, http://www.e-ir.info/?p=149)

However, the degree to which a state attempts to change the status quo can vary. Thus, China
does not currently
demonstrate a fundamental revolutionary wish to overthrow the entire international system,
but rather a minor tweaking. Indeed, China’s rise has come by playing by Western capitalist rules .
Therefore, this essay cautions against sensationalism. In the regional sphere, China now appears unimpeded by either Japan or
Russia for the first time in two centuries, and thus is beginning to project its influence in the region. Cooperation
on North
Korea illustrates that the United States is willing to collaborate with China to reach its regional
security goals. Additionally, China has also used liberal institutionalism to increase political
power and further engage with the region. The recent October 2006 ASEAN-China Commemorative Summit
sought to deepen political, security and economic ties, and concluded that the strategic
partnership had ‘boosted…development and brought tangible benefits to their peoples, [and]
also contributed significantly to peace, stability and prosperity in the region.’ China’s gradual,
natural progression of influence should not be feared. Alluding to soft power, liberal theorist Joseph
Nye illustrates China’s slow shift by contending that ‘ it will take much longer before [China] can
make an impact close to what the U.S. enjoys now.’
---China Rise- ASEAN Checks
No Chinese Aggression --- ASEAN checks

Weissmann ‘9 --- senior fellow at the Swedish School of Advanced Asia Pacific Studies (Mikael Weissmann, “Understanding the
East Asian Peace: Some Findings on the Role of Informal Processes,” Nordic Asia Research Community, November 2, 2009,
http://barha.asiaportal.info/blogs/in-focus/2009/november/understanding-east-asian-peace-some-findings-role-informal-
processes-mi)

ASEAN’s attempt to socialise China into becoming a responsible big power in the regional
It has been important for
community, in order to ensure that the Chinese interests would gradually become integrated with the
interests of East Asia as a whole. Over time, China has re-interpreted its role and interests as a rising
power and has engaged in the ASEAN+3 process and embraced multilateralism and the ASEAN
Way. This has been a reciprocal process between China’s ‘soft power diplomacy’ and ASEAN’s ‘constructive
engagement’ policies. It is difficult to say what has caused what, i.e., to what extent China has been socialised by ASEAN to accept
current practices and to become what seems to be a more benign power, and to what extent the Chinese policies have influenced
ASEAN’s increased acceptance of China as a partner and a (relatively) benign, peacefully rising power. It is most likely that it is not an
either–or question, but a transformation where there have been synergy effects between ‘soft-power diplomacy’ and “constructive
engagement”. Regionalisation has also ensured that China (and others) adheres to an ‘economic first’
foreign policy focus, and that the overall peaceful relations in East Asia have developed and
have been institutionalised. Although multilateralism and institutionalisation have only been identified in
the South China Sea and Sino–ASEAN relations, they still have a spill over effect on Chinese behaviour in other
conflicts. If China would behave badly in one case, it would risk losing its laboriously built trust
towards ASEAN.
---China Rise- Economics

No risk of Chinese aggression

Kaplan, 10 (Robert, National Correspondent – The Atlantic and Senior Fellow – Center for a New American Security, “Don’t
Panic About China”, 1/28/2010, http://www.theatlantic.com/doc/201001u/china-threat)

China is rising as a great power, that’s for sure. I


see China’s rise as similar to that of the United States after
the Civil War. From the end of the Civil War to the outbreak of World War I, the U.S. economy (under forgettable
presidents – Hayes, Garfield, Arthur…) expanded steadily, with high growth rates for decades. We closed the frontier and
would eventually build the Panama Canal. And as our power grew, we developed interests around the world that we never
previously had, and that led to Navy and Marine landings in South and Central America, and in the Pacific, as we became a two-
ocean Navy. We didn’t explicitly seek power so much as we naturally followed our interests . We
rose legitimately, in other words. And China is doing likewise. It needs to be emphasized that China is not
Iran under Mahmoud Ahmedinejad: it is not threatening to destroy any nation. It does not promote a
radical philosophy. Its nationalism may at times be assertive, but it also sends out signals of
benevolence. A case in point, which may indicate the kind of military power China will turn out to be in the 21st century,
involves its recent commemoration of Zheng He, the early-Ming Dynasty explorer. Zheng He sailed his treasure fleet through the
Strait of Malacca and out across the Indian Ocean as far as the Horn of Africa, stopping in Ceylon, the Indian Subcontinent, and the
Arabian Peninsula along the way. Zheng He’s naval fleet brought trade and prosperity and suppressed piracy, and his
commemoration is a way for China to indicate two points: that it intends eventually, if all goes well, to build a two-ocean
navy – covering the Western Pacific and the Indian Ocean (a development that would signal China’s rise as a great military power
to go along with its economic clout); and that it intends this projection of power to be benign . But China is a
long way from having a two-ocean navy. It does give significant amounts of military and economic aid to Indian Ocean
littoral countries like Pakistan, Bangladesh, Sri Lanka, and Burma, and is involved in port building projects in all these places. But
whether these projects evolve into overt naval bases for the Chinese is highly questionable. More likely, port authorities from third
countries will end up running these harbors. And China will be careful not to provoke India, with whom its
relations are already quite delicate. What’s more likely to happen – and this is a clue to power relationships as a whole
in this new century – is that rather than official Cold War-era style military bases, navies and air forces like our own and China’s
will have subtle access agreements, whose use will depend upon the health of the bilateral
relationships in question. In The Grand Chessboard (1997), Zbigniew Brzezinski presents a map of what Greater Chinese
influence is likely to look like in the future, and shows it extending through parts of the Indian Ocean, all of Southeast Asia and the
Indonesian archipelago, and the First Pacific Island Chain. But he also suggests that, in keeping with Chinese imperial history, Beijing
will seek to apply its influence very indirectly.

Chinese integration into the global economic order checks conflict

Deng and Moore, 4 (Yong, Prof. Pol. Sci. – US Naval Academy, and Thomas, Prof. Pol. Sci. – U. Cincinnati, Washington
Quarterly, “China Views Globalization: Toward a New Great-Power Politics”, 27:3, Summer,
http://thewashingtonquarterly.com/04summer/docs/04summer_deng-moore.pdf)

Thus, Chinese analysts have focused their attention on defining a position for their country within a global system of U.S. hegemony.
It is in this context that the Chinese leadership has conceptualized the impact of globalization on China’s economic agenda and
security environment. By
transforming the geo-economic context of interstate competition,
globalization has created powerful incentives for China’s participation in transnational economic
structures and multilateral institutions. Pursuit of a balancing strategy , on the other hand, would
require China to divert huge sums of scarce resources to a concerted arms buildup, to establish
military alliances against Washington, and to withdraw from (and perhaps even actively undermine) the U.S.-led
liberal international economic system—all to China’s disadvantage. Such confrontational policies
are likely to prove futile and self-defeating . Rather, a Chinese foreign policy that accommodates
economic globalization and works toward active participation in international institutions is essential to
maintaining the robust economic growth critical both to social stability and the political legitimacy
of the Chinese Communist Party, let alone China’s rising status and influence in international politics.
China REM Answers
Frontline

Recent trends show China has lost monopoly on REMs- no geopolitical risks

MT ’13 (Mining-Technoloy, Online Mining/ Mineral Investment Website, “Rare earth minerals – China loses its grip”,
http://www.mining-technology.com/features/featurerare-earth-minerals-china/, February 18, 2013)

Blockbuster video game Black Ops II last year enthralled gamers, with its premise that the world could be brought to the brink of war
over China's dominance of rare earth minerals (REM). The premise is based on the scarcity of these minerals which are used for,
among many other things, powerful batteries, camera lenses, MRI scanners, modern electronics, such as iPods, TVs and computers,
and for renewable energies, such as solar panels and wind turbines, meaning they are integral to modern life. Although obviously far
fetched, at its inception Black Op II's narrative didn't seem so implausible. For many years, China had been responsible for producing
97% of all REMs. In recent years it has been known to use its monopoly of the industry as a geopolitical weapon, and to drive up the
price of REMs. However, more recently China's dominance has diminished and in November last year, the
country closed its largest mine, Baotou Steel mine, in a bid to maintain falling prices. As a direct result of China's tactics,
the exact opposite to Black Op II's narrative has occurred - the world hasn't fought China for its REM riches,
but found its own. Just recently, two REM companies, Molycorp and Lynas, have come online and
reserves of REM have been discovered all over the world. A 2011 British Geological survey
outlines 74 potential deposits worldwide. In January, the US Department of Energy announced it
will invest $120bn in a Critical Materials Centre which is, according to its director Alex King: "Spurred by the rare
earth crisis, in 2010 - 2011, when the price went up by a factor of ten, maybe 20" and because: "They [China] also showed a
China, Baotou Mine
willingness to use the availability of rare earths as a tool of international political relations". Meanwhile, in
has announced it is expecting its 2012 profits to falls by between 50 and 60%. Economy blogger and consultant and
trader in rare earth scandium, Tim Worstall, says that, after 50-odd years, China's "dominance of basic supply is
now gone." He adds: "Once China decided to throw its weight around, of course, the incentive was to
get into the business. Which is exactly what has happened."
---China REM Extensions

China doesn’t have dominance over REM’s- other countries have funded research and China’s
reserves have collapsed- our evidence postdates which is key to prefer- that’s MT

No scarcity

MT ’13 (Mining-Technoloy, Online Mining/ Mineral Investment Website, “Rare earth minerals – China loses its grip”,
http://www.mining-technology.com/features/featurerare-earth-minerals-china/, February 18, 2013)

Which leads to another important myth about REMs; they are not actually rare. "Several of the rare earths are
individuallyas common as copper on this planet. We produce millions of tonnes of copper a year," says Worstall. Now, in
2013, the REM market has been blown wide open , with Molycorp and Lynas leading the way. There are also
estimated to be another 400 small REM companies looking to exploit rare earth deposits all around the
world. Below is a detailed examination of where they might find them. US - racing towards diversified supply "China's dominance
has diminished and in November last year, the country closed its largest mine, Baotou Steel mine." Before China, America
supplied most of the world's REM needs, when it opened Mountain Pass mine in the 1960s. The
mine closed in the 1990s due to poor market conditions but, as of January this month, it has come online again
under Molycorp as Project Phoenix. The company, which bought the mine in 2008, said it expects to reach or exceed its phase I run
rate of 19,050 metric tonnes per year (mt/year) of rare earth oxide equivalent (REO) by mid-year, before moving onto phase II,
which will see as much as 40,000 mt/year of rare earth oxide equivalent produced. The US will also use its new $120m Critical
Materials Institute, located in Ames, Iowa, to diversify supply and to ensure, says King: "Whatever vulnerability a single source has,
whether that be geopolitical or environmental...our goal is to make sure multisources can operate...[and] to try and address the
technical barriers, which obviously become economic barriers." Any breakthrough research discovered at the centre will be available
to companies wanting to invest in extracting America's REM deposits, which, it seems, there is no shortage of. For example,
Ucore Rare Metals Inc plans to commence construction of a mine located 60km south-west of Ketchikan, Alaska, where it discovered
Dysprosium, Terbium and Yttrium. It hopes to be online by 2016. Ucore CEO Jim McKenzie has said: "We view the rare-earth space
the Bear Lodge Project in Wyoming which,
as a sort of race." Meanwhile, Rare Earth Elements owns and is working on
according to the US Geological Survey, is estimated to contain one of the largest deposits of disseminated
rare-earth elements in North America. Other areas in the US currently being investigated for REMs are Missouri and
Texas.

Jamaica checks

MT ’13 (Mining-Technoloy, Online Mining/ Mineral Investment Website, “Rare earth minerals – China loses its grip”,
http://www.mining-technology.com/features/featurerare-earth-minerals-china/, February 18, 2013)

Jamaica - and its fruitful red mud Jamaica's abundant red mud is one of the latest places where REMs
have been found. Jamaica's science, technology, energy and mining minister, Philip Paulwell, told
The Associated Press in January that it contains "high concentrations of rare-earth elements."
"Before China, America supplied most of the world's REM needs, when it opened Mountain Pass mine in the 1960s." Japanese
firm, Nippon Light Metal, have now partnered with the Jamaican Government in a pilot project to
see how financially viable the deposits are. Nippon is investing $3m (£1.9m) to develop methodology for
neutralising the red mud, with the objective of extracting some of the rare earth elements. The pilot will also map the potential
impact on land, water and air and the effect of neutralising the by-products of the extraction process. Nippon Light Metal's ultimate
objective is to extract some 1,500 metric tons per annum. Head of the Jamaica Bauxite Institute, Dr Parris A Lyew-Ayee, is quoted by
the Jamaica Observer as saying: "We earnestly hope that the path to full commercial production will be cleared by this venture."

Japan and Vietnam check

MT ’13 (Mining-Technoloy, Online Mining/ Mineral Investment Website, “Rare earth minerals – China loses its grip”,
http://www.mining-technology.com/features/featurerare-earth-minerals-china/, February 18, 2013)

Japan and Vietnam - partnering for a prosperous future It is believed Vietnam has scores of
untapped REM resources. In a bid to uncover and extract these resources, Japan partnered
with Vietnam to build a centre for research on REMs in June last year. This follows an agreement signed
in 2010 between the two countries agreeing to cooperate in the discovery of REMs. The two countries working together will help
with the production of not only Vietnam's REMs but also Japan's, which discovered last year that it has about 6.8 million tonnes of
minerals, including dysprosium, laden in the seabed 2,000km from Tokyo. Japan has been most affected by China's dominance in
REMs. The country imports about 60% of China's supply, and in March last year, along with the European Union and the US, Japan
submitted an official complaint to the World Trade Organisation about China's export restrictions Japan
and Vietnam's
partnership and combined reserves could result in the two countries becoming market leaders
in the supply of REMs.

Greenland checks

MT ’13 (Mining-Technoloy, Online Mining/ Mineral Investment Website, “Rare earth minerals – China loses its grip”,
http://www.mining-technology.com/features/featurerare-earth-minerals-china/, February 18, 2013)

Greenland - ice covered REM goldmine Greenland is basking in the glory of significant REM
finds, which it hopes will boost its economy, which is mainly reliant on fishing and tourism. The European Commission estimates
that its resources could total 9.16% of the global rare earth deposits, including large deposits of
niobium, platinum, rare earths and tantalum, which are all on the EU "critical raw materials"
list. In many cases the REMs are stuck beneath 150 metres of ice, but melting ice caps are making it easier for the
minerals to be extracted.
CMR Answers
Frontline
Structural planning problems means they can’t solve.
-“normalized” theory of Civil-Military relations

-military organizational cultures makes it very difficult for others to influence change

-Outdated structural legislation

Owens 13 (Mackubin T., Associate Dean of Academics for Electives and Directed Research and
Professor of National Security Affairs at the Naval War College, “Civil-Military Relations And The
U.S. Strategy Deficit”, Eurasia Review, http://www.eurasiareview.com/201006133105/civil-
military-relations-and-the-us-strategy-deficit.html)

The failure of American civil-military relations to generate strategy can be attributed to the
confluence of three factors. The first of these is the continued dominance within the American system of
what Eliot Cohen has called the “normal” theory of civil-military relations , the belief that there is a
clear line of demarcation between civilians who determine the goals of the war and the
uniformed military who then conduct the actual fighting. Until President George W. Bush abandoned it when
he overruled his commanders and embraced the “surge” in Iraq, the normal theory has been the default position of most
presidents since the Vietnam War. Its longevity is based on the idea that the failure of President Lyndon Johnson and Secretary
of Defense Robert McNamara to defer to an autonomous military realm was the cause of U.S. defeat in Vietnam. The normal
theory can be traced to Samuel Huntington’s The Soldier and the State, in which he sought a solution to the dilemma that lies at
the heart of civil-military relations: how to guarantee civilian control of the military while still ensuring the ability of the
uniformed military to provide security. His solution was a mechanism for creating and maintaining a professional, apolitical
military establishment, which he called “objective control.” Such a professional military would focus on defending the United
States but avoid threatening civilian control. But as Cohen has pointed out, the
normal theory of civil-military
relations has rarely held. Indeed, storied democratic war leaders such as Winston Churchill and Abraham Lincoln
“trespassed” upon the military’s turf as a matter of course, influencing not only strategy and operations but also tactics. The
reason that civilian leaders cannot simply leave the military to its own devices during war is that war is an iterative process
involving the interplay of active wills. What appears to be the case at the outset of the war may change
as the war continues, modifying the relationship between political goals and military means. The fact remains that wars
are not fought for their own purposes but to achieve policy goals set by the political leadership of the state. The Iraq case
reinforces Cohen’s argument. Former secretary of defense Donald Rumsfeld may have been wrong on a number of issues but
the uniformed military was no more prescient than he. For instance, while Rumsfeld did not foresee the insurgency and the shift
from conventional to guerrilla war, neither did his critics in the uniformed services, who remained wedded to operational
doctrines focused on conventional war rather than counterinsurgency. Critics also charged that Rumsfeld’s Pentagon
shortchanged the troops in Iraq, in part by failing to provide them with armored “humvees.” Yet a review of Army budget
submissions makes it clear that the service did not immediately ask for the vehicles; the Army’s priority, as is usually the case
with the uniformed services, was to acquire “big ticket” items. It was only after the insurgency began and the threat posed by
improvised explosive devices (IEDs) became apparent that the army began to push for supplemental spending to “up-armor”
the utility vehicles. And while it is true that Rumsfeld downplayed the need to prepare for post-conflict stability operations, it is
also the case that in doing so he was merely ratifying the preferences of the uniformed military. Only recently has the
uniformed military begun to shed the “Weinberger Doctrine,” a set of principles long internalized by the U.S. military that
emphasize the requirement for an “exit strategy.” But if generals are thinking about an exit strategy they are not thinking about
“war termination”—how to convert military success into political success, which is the purpose of post-conflict planning and
stability operations. This cultural aversion to conducting stability operations is reflected in the fact that operational planning for
Operation Iraqi Freedom took eighteen months, while planning for postwar stabilization began half-heartedly only a couple of
The second factor, strongly reinforced by the normal theory of civil-military relations, is
months before the invasion.
the influence of the uniformed services’ organizational cultures . Each military service is built around a
“strategic concept,” which according to Samuel Huntington constitutes “the fundamental element of a military service,” the
basic “statement of [its] role . . . or purpose in implementing national policy.” A clear strategic concept is critical to the ability of
a service to organize and employ the resources that Congress allocates to it. It also largely determines a service’s organizational
culture. Some years ago, the late Carl Builder of the RAND Corporation wrote a book called The Masks of War, in which he
demonstrated the importance of the organizational cultures of the various military services in creating differing “personalities,”
identities, and behaviors. His point was that each service possesses a preferred way of fighting and that “the unique service
The organizational culture of a service, in turn, exerts a
identities . . . are likely to persist for a very long time.”
strong influence on civil-military relations, frequently constraining what civilian leaders can
do and often constituting an obstacle to change and innovation . The critical question here is this: who
decides whether the military instrument is effective, the civilian policymakers or the military itself? An illuminating illustration
of this phenomenon at work has been the recent attempt to institutionalize counterinsurgency doctrine within the U.S. Army, a
difficult task, given the service’s focus on the “operational level of war,” which manifests itself as a preference for fighting large-
scale conventional war—despite the fact that throughout most of its existence, the conflicts in which the U.S. Army engaged
were actually irregular wars. Beginning in the late 1970s, the Army embraced the idea of the operational level of war as its
central organizing concept. As Hew Strachan has observed, “the operational level of war appeals to armies: it functions in a
politics-free zone and it puts primacy on professional skills.” And herein lies the problem for civil-military relations: the
disjunction between operational excellence in combat and policy, which determines the reasons for which a particular war is to
be fought. The combination of the dominant position of the normal theory of civil-military relations in the United States and the
U.S. military’s focus on the non-political operational level of war means that all too often the conduct of a war disconnected
from the goals of the war. As an essay published by the U.S. Army War College’s Strategic Studies Institute puts it, the
operational level of war has become an “alien” that has devoured strategy. Rather than meeting its original purpose of
contributing to the attainment of campaign objectives laid down by strategy, operational art—practiced as a “level of war”—
assumed responsibility for campaign planning. This reduced political leadership to the role of “strategic sponsors,” quite
specifically widening the gap between politics and warfare. The result has been a well-demonstrated ability to win battles that
The political
have not always contributed to strategic success, producing “a way of battle rather than a way of war.
leadership of a country cannot simply set objectives for a war, provide the requisite materiel,
then stand back and await victory. Nor should the nation or its military be seduced by this prospect. Politicians
should be involved in the minute-to-minute conduct of war; as Clausewitz reminds us, political considerations are “influential in
the planning of war, of the campaign, and often even of the battle. The task of strategy is to bring doctrine —
concerned with fighting battles in support of campaigns— into line with national policy. But instead of
strategy, we have Gray’s “black hole.” The third factor contributing to the perseverance of the American
strategic black hole is one that was, ironically, intended to improve U.S. strategic planning: t he
Goldwater-Nichols Department of Defense Reorganization Act of 1986 . In passing Goldwater-Nichols,
Congress sought to address two central concerns: 1) the excessive power and influence of
the separate services; and 2) the mismatch between the authority of the combatant
commanders and their responsibilities. The act increased the authority of the chairman of the Joint Chiefs of Staff
while reducing that of the Joint Chiefs themselves and increased the authority of the theater commanders. Congress
expected that such reorganization would, among other things, improve the quality of military
advice to policymakers. The Joint Chiefs of Staff are responsible for integrating theater strategy and national policy.
But if they are marginalized, as they were during much of the time during the Bush administration, such integration does not
occur. This is an institutional problem illustrated by the case of Gen. Tommy Franks, the commander of US Central Command,
who in directing the war in Afghanistan after 9/11 and the first phase of the war in Iraq, was able to bypass the Joint Chiefs of
Staff. His justification is found in his memoirs, An American Soldier: “Operation Enduring Freedom in Afghanistan had been
nitpicked by the Service Chiefs and the Joint Staff, and I did not intend to see a recurrence of such divisiveness in Iraq.” He sent a
Of course, such an
message: “Keep Washington focused on policy and strategy. Leave me the hell alone to run the war.”
attitude is a dysfunctional consequence of the well-intentioned institutional arrangement
created by Goldwater-Nichols reinforcing the idea that there is an autonomous realm of
military action within which civilians have no role . The result of such a disjunction between the military
and political realms is that war plans may not be integrated with national policy and that strategy,
despite lip service to its importance, in practice becomes an orphan. And in the absence of
strategy, other factors rush to fill the void, resulting in strategic drift .

Empirically denied by 200 years


Peabody 1 – Lieutenant Colonel in the U.S. Army, 4-10-1 (John, “The ‘Crisis’ in American Civil Military Relations: A Search for
Balance Between Military Professionals & Civilian Leaders,” USAWC Strategy Research Project, http://www.dtic.mil/cgi-
bin/GetTRDoc?AD=ADA390551&Location=U2&doc=GetTRDoc.pdf)

Alarmists, who have attempted to examine the nature of current civil-military relations and the basis for military subordination to
civilian control have generally started with the development of the national security state after World War II (Kohn), followed by the
traumatic social upheavals of the Vietnam War (Feaver). Some even go back to the Civil War (Weigley). However, examples of
significant civil-military turmoil are not limited to the dramatic and oft-cited Truman-MacArthur example, but
permeate the entire American history. Cases include Andrew Jackson's martial law over New Orleans in the War
of 1812, the popular protests against the Mexican American War, draft riots and other Civil War protests, Lincoln's suspension of
civil liberties, including habeas corpus, military control of the South during Reconstruction, Sherman's disagreements on Indian
policy after the Civil War, and significant
involvement by military professionals in running governments in
various countries in the twentieth century, including Cuba, the Philippines, Nicaragua, Haiti, Panama, and the Dominican
Republic. Twentieth century examples include the remarkable presidential bid on the Republican ticket by active duty General
Leonard Wood in 1920, Billy Mitchell's overt challenge to civilian authority, the crushing of the bonus marchers in 1932, the
Admiral's revolt of the late 1940s, Army resistance to Eisenhower's Doctrine of Massive Retaliation, and periodic instances of general
officers being fired for repudiating presidential authority, such as General Singlaub's removal over President Carter's proposed Korea
withdrawal in the late 1970s. Thus, despite some considerable efforts at broader consideration of the historical record,48 few
studies consider the broad pantheon of civil-military discourse and disruption common to the
American experience. Placing the ideal of military subordination to civilian control in a more
retrospective context will help correct the alarmists' inaccurate assumptions of what constitutes
appropriate military behavior, and aid us in better understanding the limits of the alarmists' arguments.

CMR tension inevitable and no impact

Peabody 1 – Lieutenant Colonel in the U.S. Army, 4-10-1 (John, “The ‘Crisis’ in American Civil Military Relations: A Search for
Balance Between Military Professionals & Civilian Leaders,” USAWC Strategy Research Project, http://www.dtic.mil/cgi-
bin/GetTRDoc?AD=ADA390551&Location=U2&doc=GetTRDoc.pdf)

While the alarmists are incorrect in important aspects regarding the tradition of civil-military relations, their concerns
have had the positive benefit of starting a serious debate and deeper examination over the nature and current condition of
American dvil-milrtary relations. Fortunately, this debate has sparked a deeper examination of American civil-military history and
tradition that illuminates a more balanced judgment of their current status, and helps guide us in outlining some considerations for
what characterizes truly appropriate civil-military relations. Even some of the alarmists have suggested the need for "restoring the
tradition of loyal dissent,*7* yet the general tone the alarmists sounded is inaccurate, as the TISS Study indicates:
"Beyond [normal] tensions and conflicts, we see no real signs of crisis, no indicators of loss of effective
civilian control nor of undue influence by military leaders in decisions properly the domain of elected or
appointed political leaders."*0 Indeed. Don Snider makes a key point that signs of discord indicate there is a stark, but
potentially healthy tension between the two imperatives and the character and ethos of their respective cultures ... between
freedom and individualism ... and the corporate nature of the military that demand sacrifice ... to the higher good of the
mission. ...Not all observed gaps are dangerous; at the same time, not all convergences between the two cultures are
functional and thus desirable."1 Whatever the origins of and solutions to the current schisms, it is important to understand
that they represent the necessary and inevitable tension in the fundamentally contradictory
nature of civil-military relations in a democratic society. Furthermore, these problems are
neither unique to the dawning twenty-first century, nor do they portend gloomy civil-military relations
in the future, as some of the alarmists depict. Reduced civilian defense expertise and the increased insertion of military officers
into the national security bureaucracy to deal with policy issues has expanded the limits of military participation in policy-making far
beyond the mythical notion of the alarmists.
Coal Answers
Frontline
Coal hurts the economy – studies prove
Apergis and Payne 10 (Nicolas and James E., March, “Coal consumption and economic growth: Evidence from a panel
of OECD countries” pdf [Table 5 omitted])

The results of the short- and long-run Granger-causality tests are reported in Table 5. Eq. (2a) shows that real gross fixed capital
formation and the labor force each have a positive and statistically significant impact on economic growth in the short-run
whereas coal consumption has a negative and statistically significant impact. As alluded to in Section 2, the negative impact of
coal consumption on economic growth may be attributed to an inefficient and excessive use of coal consumption as well as the
possibility that the environmental costs of coal usage outweigh the immediate economic benefit of coal usage on real GDP. In
terms of Eq. (2b), economic growth has a significant negative influence on coal consumption, real gross fixed capital formation
has a significant positive impact, and the labor force has a statistically insignificant impact. As mentioned in Section 2, the
negative impact of economic growth on coal consumption may suggest that an economy is becoming less coal intensive, for
instance, electricity production from coal as a proportion of total electricity production may be in decline. In the case of Eq. (2c),
both economic growth and the labor force each have a significant positive impact on real gross fixed capital formation while
coal consumption has a statistically insignificant impact. With regard to Eq. (2d), both economic growth and real gross fixed
capital formation have a significant influence on the labor force whereas coal consumption is statistically insignificant. Thus,
from the short-run causality results, it appears there is bidirectional causality between coal consumption
and economic growth in which an increase in coal consumption has a negative impact
on economic growth and likewise an increase in economic growth has a negative impact on coal consumption. The
long-run dynamics displayed by the error correction terms from Eqs. (2a) and (2d) reveal that economic growth, coal
consumption, real gross fixed capital formation, and the labor force respond to deviations from long-run equilibrium given the
statistically significance of their respective error correction terms. However, the speed of adjustment is rather slow given the
magnitude of the coefficients on the error correction terms. Overall, the results suggest there is bidirectional causality between
coal consumption and economic growth in both the short- and long-run. The presence of bidirectional causality lends support
for the feedback hypothesis whereby coal consumption and economic growth are interdependent. The interdependence
between coal consumption and economic growth indicates that energy policies designed to decrease coal intensity, improve
energy efficiency, and the
use of renewable energy sources would serve to promote the
development of long-term energy and environmental strategies that help meet
global energy demands. 4. Concluding remarks While coal has been a reliable energy source in many countries
investigation of the relationship between coal consumption and economic growth is pertinent both in terms of the economic
and environmental consequences of its continued usage. Specifically, this study tests the causal relationship between coal
consumption and economic growth within the context of a multivariate panel error correction model for 25 OECD countries
over period 1980–2005. First, the Larsson et al. (2001) panel cointegration test indicates there is a unique long-run equilibrium
relationship between real GDP, coal consumption, real gross fixed capital formation, and the labor force. Though the long-run
elasticity estimates are positive and statistically significant for real gross fixed capital formation and the labor force, the
elasticity estimate for coal consumption is negative and statistically significant. This negative impact of coal consumption on real
output in the long-run may be the result of inefficient and excessive use of coal as well as the possibility that the immediate
economic benefit associated with the use of coal is outweighed by the economic costs imposed on the environment by carbon
dioxide emissions. Second, the estimation of panel error correction model reveals there is both short- and long-run bidirectional
causality between coal consumption and economic growth, which lends support for the feedback hypothesis. However, the
short-run causality results indicate that an
increase in coal consumption has a negative impact on
economic growth. In this case, policymakers should explore the feasibility of either increasing the
efficient use of coal or reducing coal consumption. For example, legislation that would restrict the growth of
carbon dioxide emissions might provide an incentive to enhance efficiency or curb excessive coal consumption. Furthermore,
greater use of sustainable coal technologies that permit carbon dioxide capture and storage may reduce the environmental
costs upon on the economy of excessive coal consumption. Similarly, an increase in economic growth has a negative impact on
coal consumption which might suggest that an economy may be less coal intensive, as alternative energy sources are employed
such as renewable energy. The use of government tax credits and subsidies for the production and use of sustainable alternative
energy sources may provide the needed incentive for both producers and consumers to substitute away from coal to more
sustainable energy sources over time.
US not key to coal market
Carr 12 Bloomberg (Mathew, June 13, 2012, “Falling Coal Use In U.S. Fails To Stem Global Growth: BP,”
http://www.bloomberg.com/news/2012-06-13/falling-coal-use-in-u-s-fails-to-stem-global-growth-bp.html)//DR

¶ Falling coal consumption in the U.S. last year failed to stem the pace of growth in the
fuel’s use globally, which was driven by China, Australia, Ukraine and South Korea, according to BP Plc. (BP/)¶ Coal represented 30 percent of
global energy consumption, the highest since 1969, the oil producer said today in its annual Statistical Review of World Energy.¶ Global use rose
5.4 percent in 2011, similar to the downgraded 5.5 percent pace in 2010, BP said. U.S. consumption dropped more than any
other nation, declining 24.2 million metric tons of oil equivalent, or 4.6 percent, to
501.9 million tons. China’s use surged 9.7 percent to 1.84 billion tons.¶ The 2.5 percent growth in global primary energy consumption was caused by increases in emerging economies,
BP said. China accounted for 71 percent of growth last year. Use in Organisation for Economic Co-operation and Development nations declined 2.5 percent, led by a 5 percent drop in Japan. The data

suggests that growth in global CO2 emissions from energy use continued in 2011, but
at a slower pace than in 2010, BP said.
Competitiveness Answers

No impact to US competitiveness- it’s all hype

Krugman ’11 [Paul, Nobel Prize-winning economist, professor of Economics and International Affairs at Princeton University,
received his B.A. from Yale University in 1974 and his Ph.D. from MIT in 1977. He has taught at Yale, MIT and Stanford. At MIT he
became the Ford International Professor of Economics, “The Competition Myth,” 1-24-11,
http://www.nytimes.com/2011/01/24/opinion/24krugman.html?_r=0]

Meet the new buzzword, same as the old buzzword. In advance of the State of the Union, President Obama has telegraphed his
main theme: competitiveness. The President’s Economic Recovery Advisory Board has been renamed the President’s Council on Jobs
and Competitiveness. And in his Saturday radio address, the president declared that “We can out-compete any other nation on
Earth.” This may be smart politics. Arguably, Mr. Obama has enlisted an old cliché on behalf of a good cause, as a way to sell a much-
needed increase in public investment to a public thoroughly indoctrinated in the view that government spending is a bad thing.¶ But
let’s not kid ourselves: talking about “competitiveness” as a goal is fundamentally misleading. At best, it’s
a
misdiagnosis of our problems. At worst, it could lead to policies based on the false idea that
what’s good for corporations is good for America.¶ About that misdiagnosis: What sense does it make to view
our current woes as stemming from lack of competitiveness?¶ It’s true that we’d have more jobs if we exported
more and imported less. But the same is true of Europe and Japan, which also have depressed economies. And we
can’t all export more while importing less, unless we can find another planet to sell to. Yes, we could demand that China shrink its
trade surplus — but if confronting China is what Mr. Obama is proposing, he should say that plainly.¶ Furthermore, while America is
running a trade deficit, this deficit is smaller than it was before the Great Recession began. It would help if we could make it smaller
still. But ultimately, we’re
in a mess because we had a financial crisis, not because American
companies have lost their ability to compete with foreign rivals.¶ But isn’t it at least somewhat useful to think of
our nation as if it were America Inc., competing in the global marketplace? No.¶ Consider: A corporate leader who increases profits
by slashing his work force is thought to be successful. Well, that’s more or less what has happened in America recently: employment
is way down, but profits are hitting new records. Who, exactly, considers this economic success?¶ Still, you might say that talk of
competitiveness helps Mr. Obama quiet claims that he’s anti-business. That’s fine, as long as he realizes that the
interests of
nominally “American”
corporations and the interests of the nation, which were never the same, are now less
aligned than ever before.¶ Take the case of General Electric, whose chief executive, Jeffrey Immelt, has just been
appointed to head that renamed advisory board. I have nothing against either G.E. or Mr. Immelt. But with fewer than half its
workers based in the United States and less than half its revenues coming from U.S. operations, G.E.’s fortunes have very
little to do with U.S. prosperity.¶ By the way, some have praised Mr. Immelt’s appointment on the grounds that at least
he represents a company that actually makes things, rather than being yet another financial wheeler-dealer. Sorry to burst this
bubble, but these days G.E. derives more revenue from its financial operations than it does from manufacturing — indeed, GE
Capital, which received a government guarantee for its debt, was a major beneficiary of the Wall Street bailout.¶ So what
does
the administration’s embrace of the rhetoric of competitiveness mean for economic policy?¶ The favorable
interpretation, as I said, is that it’s just packaging for an economic strategy centered on public investment,
investment that’s actually about creating jobs now while promoting longer-term growth. The unfavorable interpretation is that Mr.
Obama and his advisers really believe that the economy is ailing because they’ve been too tough on business, and that what America
needs now is corporate tax cuts and across-the-board deregulation.¶ My guess is that we’re mainly talking about packaging here.
And if the president does propose a serious increase in spending on infrastructure and education, I’ll be pleased.¶ But even if he
proposes good policies, the fact that Mr. Obama feels the need to wrap these policies in bad metaphors is a sad commentary on the
state of our discourse.¶ The financial crisis of 2008 was a teachable moment, an object lesson in what can go wrong if you trust a
market economy to regulate itself. Nor should we forget that highly regulated economies, like Germany, did a much better job than
we did at sustaining employment after the crisis hit. For whatever reason, however, the teachable moment came and went with
nothing learned.¶ Mr. Obama himself may do all right: his approval rating is up, the economy is showing signs of life, and his chances
of re-election look pretty good. But the ideology that brought economic disaster in 2008 is back on top — and seems likely to stay
there until it brings disaster again.
Competitiveness theory wrong

Bhide, 9 [Amar, Glaubinger Professor of Business at Columbia University, editor of Capitalism and Society, member of the
Council on Foreign Relations, and author of The Origin and Evolution of New Businesses, “ The Venturesome Economy: How
Innovation Sustains

Prosperity in a More Connected World,” Journal of Applied Corporate Finance • Volume 21 Number 1, Winter 2009]

The techno-nationalist claim that U.S. prosperity requires that the country “maintain its scientific and
technological lead” is particularly dubious: the argument fails to recognize that the development of scientific
knowledge or cutting-edge technology is not a zero-sum competition. The results of scientific
research are available at no charge to anyone anywhere in the world. Most arguments for the public
funding of scientific research are in fact based on the unwillingness of private investors to undertake research that cannot yield a
profit. Cutting-edge
technology (as opposed to scientific research) has commercial value because it can be
patented; but patent owners generally don’t charge higher fees to foreign licensors . The then tiny
Japanese company Sony was one of the first licensors of Bell Labs’ transistor patent. Sony paid all of $50,000—and only after first
obtaining special permission from the Japanese Ministry of Finance—for the license that started it on the road to becoming a
household name in consumer electronics. Moreover, if patent holders choose not to grant licenses but to exploit their inventions on
their own, this does not mean that the country of origin secures most of the benefit at the expense of other countries. Suppose IBM
chooses to exploit internally, instead of licensing, a breakthrough from its China Research Laboratory (employing 150 research staff
in Beijing). This does not help China and hurt everyone else. Rather, as I discuss at length later, the benefits go to IBM’s stockholders,
to employees who make or market the product that embodies the invention, and—above all—to customers, who secure the lion’s
share of the benefit from most innovations. These stockholders, employees, and customers, who number in the tens of millions, are
located all over the world. Ina world where breakthrough ideas easily cross national borders, the
origin of ideas is inconsequential. Contrary to Thomas Friedman’s assertion, it does not matter that
Google’s search algorithm was invented in California. After all, a Briton invented the protocols of
the World Wide Web—in a lab in Switzerland. A Swede and a Dane in Tallinn, Estonia, started Skype, the
leading provider of peer-to-peer Internet telephony. How did the foreign origins of these innovations harm
the U.S. economy?

Competitiveness not key to heg

Brooks and Wohlforth ‘8 - Brooks is Assistant Professor AND*** William C. Wohlforth is Professor in the Department of
Government at Dartmouth College [Stephen G., “World out of Balance, International Relations and the Challenge of American
Primacy,” p. 32-35]

American primacy is also rooted in the county's position as the world's leading technological power. The United States remains
dominant globally in overall R&D investments, high-technology production, commercial first decade of this century. As we noted in
chapter 1, this was partly the result of an Iraq-induced doubt about the utility of material predominance, a doubt redolent of the
post-Vietnam mood. In retrospect, manyassessments of U.S. economic and technological prowess from
the 1990s were overly optimistic; by the next decade important potential vulnerabilities were evident .
In particular, chronically imbalanced domestic finances and accelerating public debt convinced
some analysts that the United States once again confronted a competitiveness crisis.23 If concerns
continue to mount, this will count as the fourth such crisis since 1945; the first three occurred during the 1950s

(Sputnik), the 1970s (Vietnam and stagflation), and the 1980s (the Soviet threat and Japan's challenge). None of these
crises , however, shifted the international system's structure: multipolarity did not return in the
1960s, 1970s, or early 1990s, and
each scare over competitiveness ended with the American position of
primacy retained or strengthened.24 Our review of the evidence of U.S. predominance is not meant to suggest that the
United States lacks vulnerabilities or causes for concern. In fact, it confronts a number of significant vulnerabilities; of course, this is
also true of the other major powers.25 The point is that adverse
trends for the United States will not cause a
polarity shift in the near future. If we take a long view of U.S. competitiveness and the prospects
for relative declines in economic and technological dominance, one takeaway stands out:
relative power shifts slowly . The United States has accounted for a quarter to a third of global output for over a century.
No other economy will match its combination of wealth, size , technological capacity, and
productivity in the foreseeable future (tables 2.2 and 2.3). The depth, scale, and projected longevity of
the U.S. lead in each critical dimension of power are noteworthy . But what truly distinguishes the
current distribution of capabilities is American dominance in all of them simultaneously. The chief
lesson of Kennedy's 500-year survey of leading powers is that nothing remotely similar ever occurred in the historical experience
innovation, and higher education (table 2.3). Despite the weight of this evidence, elite perceptions of U.S. power had shifted toward
pessimism by the middle of the that informs modern international relations theory. The implication is both simple and
underappreciated: the counterbalancing constraint is inoperative and will remain so until the distribution of capabilities changes
fundamentally. The next section explains why.

We’re lightyears ahead in key sectors

Zakaria 8 (Fareed, Newsweek Editor, International Relations Expert, Host of Fareed Zakaria: GPS (on CNN), “The Future of
American Power,” Foreign Affairs, May/June)

This difference between the United States and Britain is reflected in the burden of their
military budgets. Britannia ruled the seas but never the land. The British army was sufficiently small that Otto von
Bismarck once quipped that were the British ever to invade Germany, he would simply have the local police force arrest them.
Meanwhile, London's advantage over the seas -- it had more tonnage than the next two navies put together -- came at ruinous
cost. The
U.S. military, in contrast, dominates at every level -- land, sea, air, space -- and spends
more than the next 14 countries combined , accounting for almost 50 percent of global defense spending. The
United States also spends more on defense research and development than the rest of the
world put together. And crucially, it does all this without breaking the bank. U.S. defense expenditure as a
percent of GDP is now 4.1 percent, lower than it was for most of the Cold War (under Dwight Eisenhower, it rose to ten
percent). As U.S. GDP has grown larger and larger, expenditures that would have been backbreaking have
become affordable. The Iraq war may be a tragedy or a noble endeavor, but either way, it will not bankrupt the United
States. The price tag for Iraq and Afghanistan together -- $125 billion a year -- represents less than one percent of GDP. The war
in Vietnam, by comparison, cost the equivalent of 1.6 percent of U.S. GDP in 1970, a large difference. (Neither of these
percentages includes second- or third-order costs of war, which allows for a fair comparison even if one disputes the exact
figures.) U.S. military power is not the cause of its strength but the consequence. The
fuel is the United States'
economic and technological base, which remains extremely strong. The United States does face larger,
deeper, and broader challenges than it has ever faced in its history, and it will undoubtedly lose some share of global GDP. But
the process will look nothing like Britain's slide in the twentieth century, when the country lost the lead in innovation, energy,
and entrepreneurship. The United States will remain a vital, vibrant economy, at the forefront of
the next revolutions in science, technology, and industry . In trying to understand how the United States
will fare in the new world, the first thing to do is simply look around: the future is already here . Over the last 20 years,
globalization has been gaining breadth and depth. More countries are making goods, communications technology has been
leveling the playing field, capital has been free to move across the world -- and the
United States has benefited
massively from these trends. Its economy has received hundreds of billions of dollars in
investment, and its companies have entered new countries and industries with great success.
Despite two decades of a very expensive dollar, U.S. exports have held ground, and the World Economic Forum currently ranks
the United States as the world's most competitive economy. GDP growth, the bottom line, has averaged just over three percent
in the United States for 25 years, significantly higher than in Europe or Japan. Productivity growth, the elixir of modern
economics, has been over 2.5 percent for a decade now, a full percentage point higher than the European average. This superior
growth trajectory might be petering out, and perhaps U.S. growth will be more typical for an advanced industrialized country for
the next few years. But the
general point -- that the United States is a highly dynamic economy at
the cutting edge, despite its enormous size -- holds. Consider the industries of the future.
Nanotechnology (applied science dealing with the control of matter at the atomic or molecular scale) is likely to lead
to fundamental breakthroughs over the next 50 years, and the United States dominates the field . It
has more dedicated "nanocenters" than the next three nations (Germany, Britain, and China) combined and has issued more
patents for nanotechnology than the rest of the world combined, highlighting its unusual strength in turning
abstract theory into practical products. Biotechnology (a broad category that describes the use of biological
systems to create medical, agricultural, and industrial products) is also dominated by the United States .

Competitiveness isn’t zero sum

Galama and Hosek 8 (Titus, PhD and Physical Scientist at the RAND Institute, James PhD and Director of Forces and
Resources Policy Center at the Rand National Security Research Division, “U.S. Competitiveness in Science and Technology,” Feb 8 th,
www.rand.org/pubs/monographs/2008/RAND_MG674.pdf)

A future in which a significant share of new technologies is invented elsewhere will benefit
the United States as long as it maintains the capability to acquire and implement
technologies invented abroad. Technology is an essential factor of productivity, and the use of new
technology (whether it was invented in the United States or elsewhere) can result in greater
efficiency, [and] economic growth, and higher living standards. The impact of globalization on U.S.
innovative activity is less clear. On the one hand, significant innovation and R&D elsewhere may increase
foreign and domestic demand for U.S. research and innovation if the United States keeps its
comparative advantage in R&D. On the other hand, the rise of populous, low-income countries may threaten this
comparative advantage in R&D in certain areas if such countries develop the capacity and institutions necessary to apply new
technologies and have a well-educated, low-wage S&T labor force.

Tech competitiveness is key to Chinese growth, but not the US- no impact

Swagel ’12 [Phillip Swagel, an economist and academic, was assistant secretary for economic policy at the Treasury Department
from 2006 to 2009, where he was responsible for analysis on a wide range of economic issues, including policies relating to the
financial crisis and the Troubled Asset Relief Program. He has also served as chief of staff and senior economist at the White House
Council of Economic Advisers and as an economist at the Federal Reserve Board and the International Monetary Fund. He is
concurrently a professor of international economics at the University of Maryland's School of Public Policy, “International
Competitiveness,” 1-18-12, http://www.aei.org/files/2012/01/17/-international-competitiveness_114810229628.pdf]

The danger, then, of an inappropriate focus on international competitiveness is perhaps one of ¶ confusion—
that the policy development process will be guided by inappropriate considerations. This is ¶ the case especially for
competitiveness indicators that are well-connected to international trade but that ¶ do not relate straightforwardly
to well-being. The danger in this case is that policies will be favored ¶ because they improve measures of competitiveness such
as by promoting exports or narrowing the ¶ trade deficit, but without reference to whether the costs of the policies are worth the
benefits.¶ It remains useful to consider so-called international competitiveness indicators that really amount to ¶
diagnostics of the domestic economy as representing benchmarks against other countries. For example, ¶
policymakers might use indicators from the WEF to consider the potential outcomes if the United States ¶ were to adopt economic
and regulatory policies that were more like those in Europe.¶ Similarly, a comparison with measures of competitiveness
in
China could be useful as an indicator of ¶ what might be possible in terms of growth, even if
the comparison does not have immediate policy implication for the United States. China’s
remarkable growth performance has reflected in various parts ¶ the catch up of that country to the
market frontier, including in terms of technology, capital, and labor. ¶ These changes include the massive
movement of workers from low-productivity activities in the interior ¶ to the market-oriented economy of the coast; the equally
massive increase in capital stock driven by ¶ investment; and the imitation (or theft) and deployment of more
advanced technologies and production ¶ techniques.

Competitiveness isn’t key to heg or the economy

Krugman 94 (Paul, Professor of Economics – Massachusetts Institute of Technology, “Competitiveness: A Dangerous Obession”,
Foreign Affairs, March / April, Lexis)

Unfortunately, his diagnosis was deeply misleading as a guide to what ails Europe, and similar diagnoses in the United States
are equally misleading. The idea that a country's economic fortunes are largely determined by its
success on world markets is a hypothesis, not a necessary truth; and as a practical, empirical matter, that
hypothesis is flatly wrong. That is, it is simply not the case that the world's leading nations are to any
important degree in economic competition with each other, or that any of their major economic problems
can be attributed to failures to compete on world markets. The growing obsession in most advanced nations
with international competitiveness should be seen, not as a well-founded concern, but as a view held in
the face of overwhelming contrary evidence. And yet it is clearly a view that people very much want to hold -- a
desire to believe that is reflected in a remarkable tendency of those who preach the doctrine of competitiveness to support
their case with careless, flawed arithmetic. This article makes three points. First, it argues that concerns about competitiveness
are, as an empirical matter, almost completely unfounded. Second, it tries to explain why defining the economic problem as one
of international competition is nonetheless so attractive to so many people. Finally, it argues that the obsession with
competitiveness is not only wrong but dangerous, skewing domestic policies and threatening the
international economic system. This last issue is, of course, the most consequential from the standpoint of public policy.
Thinking in terms of competitiveness leads, directly and indirectly, to bad economic policies on a wide range of issues, domestic
and foreign, whether it be in health care or trade.

Alt causes –

STEM shortage means US competitiveness is unsustainable

Waldron ‘12 [Travis, reporter for ThinkProgress.org at the Center for American Progress Action Fund, “REPORT: How America’s
Falling Share Of Global College Graduates Threatens Future Economic Competitiveness,”
http://thinkprogress.org/economy/2012/08/21/722571/report-us-share-of-college-graduates-dropped-over-last-decade-compared-
to-china-india/]
The United States’ shareof global college graduates fell substantially in the first decade of the 21st century
and stands to drop even more by 2020 as developing economies in China and India have graduated more college
students, presenting challenges for American workers’ ability to remain competitive in a global
economy in the future. The U.S. share of college graduates fell from nearly one-in-four to just more than one-in-five from 2000
to 2010, according to “The Competition That Really Matters,” a report from the Center for American Progress and The Center for the
Next Generation: From 2000 to 2010, the U.S. share of college graduates fell to 21% of the world’s total from 24%, while China’s
share climbed to 11% from 9%. India’s rose more than half a percentage point to 7%. Based on current demographic and college
enrollment trends, we can project where each country will be by 2020: the U.S. share of the world’s college graduates will fall below
18% while China’s and India’s will rise to more than 13% and nearly 8% respectively.

B) Outsourcing

Sneider 5 (Daniel, Foreign Affairs Writer – Mercury


News, “Hand-Wringing Over China Misses True Economic Problem”, Mercury
News, 4-24, http://www.mercurynews.com/mld/mercurynews/11476832.htm)

“In this global market, the U.S. is a leader, but we are not dominant in the market and we are not able to
control the market,'' said former Defense Secretary William Perry, also a longtime Stanford engineering professor and
venture capitalist. Rather, he and others emphasized, the U nited S tates needs to maintain its leadership as an
innovator. Key to that is protecting intellectual property such as computer software codes, equipment designs and basic
research. The
growing trend to outsource r esearch-and- d evelopment facilities to China makes this
even more difficult.
---Competitiveness - Not Behind
We’re still way ahead other nations

Segal 4 (Adam, Senior Fellow at the Council on Foreign Relations, “Is America Losing Its Edge?”
http://www.foreignaffairs.org/20041101facomment83601/adam-segal/is-america-losing-its-edge.html)

At the moment, it
would be premature to declare a crisis in the United States ' scientific or
technological competitiveness. The United States is still the envy of the world for reasons
ranging from its ability to fund basic scientific research to the speed with which its
companies commercialize new breakthroughs. This year, total U.S. expenditures on R&D are
expected to top $290 billion-more than twice the total for Japan, the next biggest spender. In 2002,
the U.S. R&D total exceeded that of Canada, France, Germany, Italy, Japan, and the United Kingdom combined (although the
United States trailed Finland, Iceland, Japan, South Korea, and Sweden in the ratio of R&D to GDP). And although scholars from
other parts of the world may write relatively more science and engineering papers than Americans do, U.S.
research
continues to be cited the most. The United States also leads the major global technology
markets, holding commanding market shares in aerospace, [and] scientific instruments,
computers and office machinery, and communications instruments. U.S. information and
communications technology producers lead almost every sector. And for the last two decades, U.S. firms have been
the top providers of high-technology services, accounting for about one-third of the
world's total.
---Competitiveness - Not Zero Sum
Foreign innovation can help U.S. competitiveness

RAND Institute 8 (The RAND Institute is a nonprofit research organization, U.S. Competitiveness in Science and Technology,
Report Prepared for the Secretary of Defense, http://www.rand.org/pubs/monographs/2008/RAND_MG674.pdf

As with other countries, U.S. economic growth, increase in standard of living, and S&T
progress depend on the United States’ ability to absorb (make economic use of) recent
innovations made at home or abroad. The rise of R&D and innovation activity in other
nations suggests that the pool of technology created outside the United States may be
growing and that the United States is likely to benefit from increased productivity from
technology invented abroad. There is no reason to believe that the globalization of S&T
and the rise of other nations impacts the United States’ capability to absorb new
technology directly, as this capability is to a large extent determined by private sector know-how, business incentives,
consumers’ willingness to try new technologies, and the legal and regulatory framework. Some technology applications do not
require much S&T capacity, or much knowledge of S&T within the user community or the general public. For example, solar
collectors or filters for water purification can significantly enhance the productivity of workers in a developing country without
the need for them to understand how these devices work. But many technology applications do require S&T capacity (see
Silberglitt et al., 2006a, 2006b). The
S&T capacity of advanced countries, including an educated and technically
astute workforce and public, is the reason why they are highly capable of implementing new
technology, and why developing nations such as China and India have partial capability, but are well ahead of Latin America,
the Middle East, and Africa in this regard.

Economies aren’t zero-sum – other’s gains help the U.S.

Krugman 94 (Paul, Professor of Economics – Massachusetts Institute of Technology, “Competitiveness: A Dangerous Obession”,
Foreign Affairs, March / April, Lexis)

How can this be in our interdependent world? Part of the answer is that the world is not as interdependent as you might think:
countries are nothing at all like corporations. Even today, U.S. exports are only 10 percent of the
value-added in the economy (which is equal to GNP). That is, the United States is still almost 90 percent an economy
that produces goods and services for its own use. By contrast, even the largest corporation sells hardly any of
its output to its own workers; the "exports" of General Motors -- its sales to people who do not work there -- are virtually all of
its sales, which are more than 2.5 times the corporation's value-added. Moreover, countries do not compete with each other
the way corporations do. Coke and Pepsi are almost purely rivals: only a negligible fraction of Coca-Cola's sales go to
Pepsi workers, only a negligible fraction of the goods Coca-Cola workers buy are Pepsi products. So if Pepsi is successful,
it tends to be at Coke's expense. But the major industrial countries, while they sell products that compete with
each other, are also each other's main export markets and each other's main suppliers of useful imports. If the
European economy does well, it need not be at U.S. expense; indeed, if anything a successful
European economy is likely to help the U.S. economy by providing it with larger markets and selling it goods of
superior quality at lower prices. International trade, then, is not a zero-sum game. When productivity rises in Japan, the
main result is a rise in Japanese real wages; American or European wages are in principle at least as likely to rise as to fall, and in
practice seem to be virtually unaffected. It would be possible to belabor the point, but the moral is clear: while competitive
problems could arise in principle, as a practical, empirical matter the major nations of the world are not to any significant degree
in economic competition with each other. Of course, there is always a rivalry for status and power -- countries that grow faster
will see their political rank rise. So it is always interesting to compare countries. But asserting that Japanese growth diminishes
U.S. status is very different from saying that it reduces the U.S. standard of living -- and it is the latter that the rhetoric of
competitiveness asserts.

Competitiveness is a fabricated concept

Krugman 94 (Paul, Professor of Economics at MIT, Nobel Prize Winner in Economics, “Competitiveness: A Dangerous
Obsession,” March)

Economists, in general, do not use the word ‘competitiveness’. Not one of the textbooks in
international economics I have on my shelves contains the word in its index. So why are
there so many councils on competitiveness, White Papers on competitiveness, and so on? Why have
most people who think about international trade come to use ‘competitiveness’ as perhaps the
central concept of their world-view? Why, indeed, does this journal have a whole issue devoted to the subject? As I said at the
beginning of this piece, most of us would like to believe that great public debates are driven by serious intellectual concerns. We
would therefore like to believe that if famed intellectuals and powerful politicians talk about ‘competitiveness’, they must have
something meaningful in mind. It seems far too cynical to suggest that the
debate over competitiveness is
simply a matter of time-honoured fallacies about international trade being dressed up in
new and pretentious rhetoric.
Credibility Answers
Frontline

Cred is irrelevant

Wohlforth 9—Daniel Webster Professor of Government,  Dartmouth.  BA in IR, MA in IR and MPhil and PhD in  pol sci, Yale
(William and Stephen Brooks, Reshaping the World Order, March / April 2009, Foreign Affairs Vol. 88, Iss. 2; pg. 49, 15 pgs)

FOR ANALYSTS such as Zbigniew Brzezinski and Henry Kissinger, the key reason for skepticism about the United States' ability to
spearhead global institutional change is not a lack of power but a lack of legitimacy. Other states may simply refuse to follow a
leader whose legitimacy has been squandered under the Bush administration; in this view, the legitimacy to lead is a fixed
resource that can be obtained only under special circumstances. The political scientist G.John Ikenberry argues in After Victory
that states have been well positioned to reshape the institutional order only after emerging victorious from some titanic
struggle, such as the French Revolution, the Napoleonic Wars, or World War I or II. For the neoconservative Robert Kagan, the
legitimacy to lead came naturally to the United States during the Cold War, when it was providing the signal service of balancing
the Soviet Union. The
implication is that today, in the absence of such salient sources of legitimacy, the
wellsprings of support for U.S. leadership have dried up for good. But this view is mistaken. For one
thing, it overstates how accepted U.S. leadership was during the Cold War: anyone who recalls the Euromissile crisis of the
1980s, for example, will recognize that mass opposition to U.S. policy (in that case, over stationing
intermediaterange nuclear missiles in Europe) is not a recent phenomenon. For another, it understates how
dynamic and malleable legitimacy is. Legitimacy is based on the belief that an action, an actor, or a political order
is proper, acceptable, or natural. An action - such as the Vietnam War or the invasion of Iraq - may come to be
seen as illegitimate without sparking an irreversible crisis of legitimacy for the actor or the
order. When the actor concerned has disproportionately more material resources than other states, the sources
of its legitimacy can be refreshed  repeatedly. After all, this is hardly the first time Americans
have worried about a crisis of legitimacy. Tides of skepticism
concerning U.S. leadership arguably rose as high or higher after the fall of Saigon in 1975 and during Ronald
Reagan's first term, when he called the Soviet Union an "evil empire." Even George W. Bush, a globally
unpopular U.S. president with deeply controversial policies,oversaw a marked improvement in relations with
France, Germany, and India in recent years - even before the elections of Chancellor Angela Merkel in Germany and
President Nicolas Sarkozy in France. Of course, the ability of the United States to weather such crises of legitimacy in the past
hardly guarantees that it can lead the system in the future. But there are reasons for optimism. Some of the apparent damage
to U.S. legitimacy might merely be the result of the Bush administration's approach to diplomacy and international institutions.
Key underlying conditions remain particularly favorable for sustaining and even enhancing
U.S. legitimacy in the years ahead. The United States continues to have a far larger share of the human and material
resources for shaping global perceptions than any other state, as well as the unrivaled wherewithal to produce public goods that
reinforce the benefits of its global role. No other state has any claim to leadership commensurate with Washington's. And
largely because of the power position the United States still occupies, there is no prospect of a counterbalancing coalition
emerging anytime soon to challenge it. In the end, the legitimacy of a system's leader hinges on whether the system's members
see the leader as acceptable or at least preferable to realistic alternatives. Legitimacy is not necessarily about normative
approval: one may dislike the United States but think its leadership is natural under the circumstances or
the best that can be expected. Moreover, history provides abundant evidence that past leading states - such as Spain, France,
and the United Kingdom - were able to revise the international institutions of their day without the special circumstances
Ikenberry and Kagan cite. Spainfashioned both normative and positive laws to legitimize its conquest of indigenous Americans in
the early seventeenth century; France instituted modern concepts of state borders to meet its needs as Europe's preeminent
land power in the eighteenth century; and the United Kingdom fostered rules on piracy, neutral shipping, and colonialism to suit
its interests as a developing maritime empire in the nineteenth century. As Wilhelm Grewe documents in his magisterial The
Epochs of International Law, these states accomplished such feats partly through the unsubtle use of power: bribes, coercion,
and the allure oflucrative long-term cooperation. Less obvious but often more important, the bargaining hands of the leading
states were often strengthened by the general perception that they could pursue their interests in even less palatable
ways - notably, through the naked use of force. Invariably, too, leading states have had the power to set the international
agenda, indirectly affecting the development of new rules by defining the problems they were developed to address. Given its
naval primacy and global trading interests, the United Kingdom was able to propel the slave trade to the forefront of the world's
agenda for several decades after it had itself abolished slavery at home, in 1833. The
bottom line is that the
UnitedStates today has the necessary legitimacy to shepherd reform of the international system.

No impact to credibility---allies won’t abandon us and adversaries can’t exploit it

Walt ‘11 Stephen M. the Robert and Renée Belfer professor of international relations at Harvard University, December 5, 2011,
“Does the U.S. still need to reassure its allies?,” online:
http://walt.foreignpolicy.com/posts/2011/12/05/us_credibility_is_not_our_problem

A perennial preoccupation of U.S. diplomacy has been the perceived need to reassure allies of
our reliability. Throughout the Cold War, U.S. leaders worried that any loss of credibility might
cause dominoes to fall, lead key allies to "bandwagon" with the Soviet Union, or result in some form of "Finlandization."
Such concerns justified fighting so-called "credibility wars" (including Vietnam), where the main concern was not the direct stakes of
the contest but rather the need to retain a reputation for resolve and capability. Similar fears also led the United States to deploy
thousands of nuclear weapons in Europe, as a supposed counter to Soviet missiles targeted against our NATO allies. The
possibility that key allies would abandon us was almost always exaggerated, but U.S. leaders remain
overly sensitive to the possibility. So Vice President Joe Biden has been out on the road this past week, telling various
U.S. allies that "the United States isn't going anywhere." (He wasn't suggesting we're stuck in a rut, of course, but saying that the
imminent withdrawal from Iraq doesn't mean a retreat to isolationism or anything like that.) There's nothing really wrong with
offering up this sort of comforting rhetoric, but I've never really understood why U.S. leaders were so worried about the credibility
of our commitments to others. For starters, given
our remarkably secure geopolitical position, whether U.S.
pledges are credible is first and foremost a problem for those who are dependent on U.S. help. We
should therefore take our allies' occasional hints about realignment or neutrality with some
skepticism; they have every incentive to try to make us worry about it, but in most cases little
incentive to actually do it.

No spillover--- Commitments are compartmentalized – at worst it boosts of credibility in


others areas

MacDonald ‘11, Paul K. Assistant Professor of Political Science at Williams College, and Joseph M. Parent, Assistant Professor
of Political Science at the University of Miami, Spring 2011, “Graceful Decline?: The Surprising Success of Great Power
Retrenchment,” International Security, Vol. 35, No. 4, p. 7-44

Second, pessimists overstate the extent to which a policy of retrenchment can damage a great power’s capabilities or
prestige. Gilpin, in particular, assumes that a great power’s commitments are on equal footing and
interdependent. In practice, however, great powers make commitments of varying degrees that are functionally
independent of one another. Concession in one area need not be seen as influencing a
commitment in another area.25 Far from being perceived as interdependent, great power commitments are
often seen as being rivalrous, so that abandoning commitments in one area may actually bolster the
strength of a commitment in another area . During the Korean War, for instance, President Harry Truman’s administration
explicitly backed away from total victory on the peninsula to strengthen deterrence in Europe.26 Retreat in an area of
lesser importance freed up resources and signaled a strong commitment to an area of greater
significance.
Best studies conclude credibility is irrelevant

Tang 5 – Shiping Tang, associate research fellow and deputy director of the Center for Regional Security Studies at the Chinese
Academy of Social Sciences in Beijing, January-March 2005, “Reputation, Cult of Reputation, and International Conflict,” Security
Studies, Vol. 14, No. 1, p. 34-62

The general validity of reputation , however, has come under assault. Whereas in 1961 Glenn Snyder touted the
virtue of drawing the line in places such as Quemoy and Matsu,4 he later all but acknowledged the flaw of his logic.5 Likewise, a
decade after claiming that "a state can usually convince others of its willingness to defend its vital interests by frequently fighting for
interests others believe it feels are less than vital,"6 Jervis was no longer so sure in 1982: "We cannot predict with great assurance
how a given behavior will influence others' expectations of how the state will act in the future."7 This assault on reputation remains
anathema for most politicians (and many political scientists). As statesman Henry Kissinger warned his colleagues, "No serious
policymaker could allow himself to succumb to the fashionable debunking of 'prestige,' or 'honor* or 'credibility.'"8 Judging from
politicians' rhetoric and behavior, Kissinger's advice has been well taken. There
seems to be a gap, therefore, between
politicians' persistent obsession
with reputation and scholars' increasing doubt about reputation's importance,
and that gap is widening. Several more recent studies have taken the case against reputation (and
credibility) even further.9 Compared to previous studies, these tend to be more systematic and
better grounded empirically. They can be divided into two categories. The first group of work focuses on the impact of
politicians' concern for reputation on state behavior and concludes that the concern for reputation has had a profound influence on
state behavior in conflicts.10 The second group of work, taking politicians'belief in reputation as a fact, argues that this
belief is unjustified because reputation in international conflicts is difficult, if not impossible, to
develop. To put it differently, this line of work contends that reputation actually does not matter as much as politicians
usually believe, if it matters at all.11
---Credibility- AT: Arab Spring

Credibility is terminally shot and the Arab Spring isn’t key – multiple other issues overwhelm –
only risk they say no to the plan.
- Extremely high expectations doom solvency
- Anger at the US means plan is spun negatively even if they like the substance
- Israel-Palestine shapes Arab countries views, not the response to the Arab Spring

Mulvany ‘11. [Lydia, Washington reporter, “Why don’t Arabs love Obama anymore?” McClatchy Newspapers -- June 20 --
http://www.mcclatchydc.com/2011/07/19/v-print/117863/why-dont-arabs-love-obama-anymore.html]

In an IBOPE Zogby International poll released last week, respondents in four out of six countries surveyed had a lower
opinion of the United States than at the end of the Bush administration in 2008. The feeling among many
in the region is that no American president can bring about change , James Zogby, founder of the Arab American
Institute and a senior adviser at the eponymous polling firm, said Tuesday. Citing conversations he had in the region during the

polling, Zogby said, "There was this sense that it's a fundamentally broken system , that (the U.S.)
can't do the right thing ." Part of the reason were sky-high expectations after Obama's
celebrated speech to the Muslim world in 2009 in Cairo. Arabs believed Obama two years ago when he said
he'd change Washington and the world, Zogby told a roundtable at the New America Foundation, a Washington-based think
tank. Now, a stunning majority — as high as 99 percent of those surveyed in Lebanon and 94 percent in Jordan — said
Obama hasn't met expectations. Even actions that could be construed as constructive , like
establishing a no-fly zone over Libya or killing Osama bin Laden, didn't win points. In the poll, the killing of bin Laden actually
worsened attitudes. Zogby said it was because those actions reinforced America's image as the bully on the block. "I use the
example of the next-door neighbor (who screws) around with your wife for a couple of years,
and then one day trims your bushes and takes your garbage out. You don't say, 'Gosh what a
great guy,'" Zogby said. Of course, U.S. approval ratings in the region have been low for a long time. Arab opinion of the United
States rose briefly after 9/11, then dropped with the invasion of Iraq. While favorability surged in many Arab countries when Obama
was elected, it was clearly on the wane by 2010. After the Cairo speech, hopes for a solution for Israel and
Palestine were high. Lack of progress on that issue was a chief disappointment , analysts said. Shibley
Telhami, a senior fellow at the center-left Brookings Institution, said that the numbers showed that the "Arab
Spring" popular uprisings this year haven't had much of an effect on how Arabs view
the U.S. Rather, they continue to view America through the prism of the Israeli-Palestinian issue .
Analysts also said that Obama gave the impression he would change foreign and security policy drastically, while such changes often
occur incrementally. Republicans jumped on the poll and said that Arab countries wanted the George W. Bush administration's more
aggressive foreign policy back. Elliott Abrams, a deputy national security adviser under Bush, said the Obama administration wasn't
supportive enough of freedom in the Middle East and wasn't with the people when they rose up against dictators in Tunisia and
Egypt. One poll question asked people to name the greatest obstacles to peace in the Middle East . The
top two answers were U.S. interference in the Arab world and the Israeli occupation of the
Palestinian territories. The issues people in the region might welcome help with are pressing
needs like the economy, health care and education — not democracy , Zogby said. Maya Berry, executive director
of the Arab American Institute, said the message she took from the poll results was: Just stop interfering. "There was a time when
you'd get questions back, like we need you to lead here differently, or we need you to be balanced — that's just gone. Now it's just

stay out of it," Berry said. " We are now reduced to interference ."
The Arab Spring isn’t key to heg.

Traub ‘11 James s a contributing writer for the New York Times Magazine and a fellow of the Center on International
Cooperation, “Twilight in Manhattan, Dawn in Tripoli,” 9/9/2011,
http://www.foreignpolicy.com/articles/2011/09/09/twilight_in_manhattan_dawn_in_tripoli?page=0,1

There is a very real danger that our reaction to the discovery that we can't do everything will be
to conclude that we can't do anything. We should rein in our hubris, tend to our own garden, patrol our own borders
(and Persian Gulf sea lanes too, of course). We should not, in John Quincy Adams's now-much-quoted phrase, go abroad in search of
monsters to destroy. But as we learned on 9/11, the world beyond our borders can do terrible harm to us, as it could not in 1821.
And it can, lest we forget, bring great benefit as well. We
have been given a second chance to get things right .
Just as the last decade began with the terrorist attacks, this one has begun with the Arab Spring.
It is, in effect, 2002 once more: We stand at the very beginning of a new moment in history, its outcome
very much unknown. And, of course, what is centrally different about this moment is that the peoples of the Arab world have
acted on their own. The irrelevance of outsiders has made it impossible for Arab autocrats to discredit
the democratic movement, as they were able to do in the face of Bush's blustering about
freedom, and as the Iranian government sought to do in 2009. And yet we have just seen overwhelming proof
that outsiders can decisively tip the scales on behalf of Arab peoples. I am thinking, of course, of Libya. The NATO bombing campaign
dislodged Muammar al-Qaddafi's regime without undermining the rebels' own legitimacy. Conservatives
now belabor
Obama for "leading from behind," or letting France -- France! -- take the lead; but Obama understands that
the Arab Spring is not some sort of test of American power or primacy . American capacities
were indispensable to the NATO effort, but no one can say that America delivered Tripoli to the
rebels. That's not a bad model for the future. It is, of course, no secret that American firepower can
work wonders. But now the hard part begins. In Libya, as in Egypt and Tunisia, the tyrant is gone,
and the burden of creating a future different from the past has fallen on people with no
experience of self-government. This is where democracy promotion becomes very real, but also very unglamorous. Libya
doesn't need money -- save for its own unfrozen assets -- but it will need a lot of diplomatic hand-holding and help with the
establishment of political parties, electoral commissions, a parliament, and so on. OK, maybe that's not so hard. Here are some hard
questions: What are we going to do in Egypt if the Muslim Brotherhood captures a plurality of seats in elections this fall? Are we
going to say that democracies disqualify themselves when people freely choose Islamists? And what about Bahrain, where Obama
was bold enough to publicly demand reform -- and where the regime has carried out a Potemkin version of dialogue with the
opposition? Is Obama prepared to threaten serious consequences, or will he back down in the name of preserving alliances, as Bush
did in Egypt in 2005? Obama
has often expressed his admiration for Niebuhr and the realists. He will
err on the side of restraint. Perhaps, with all we've been through, that's a good thing.

Link shield – there is only a risk we lose credibility in the Middle East.

Terrill ‘11, Andrew Middle East nonproliferation analyst for the International Assessments Division of the Lawrence
Livermore National Laboratory, 6-27-20, “The Arab Upheavals and the Future of the U.S. Military Policies and Presence in the
Middle East and the Gulf,” SSI, http://www.strategicstudiesinstitute.army.mil/index.cfm/articles/Arab-Upheavals-and-the-Future-of-
the-US-Military-Policies-and-Presence-in-the-Middle-East-and-the-Gulf/2011/6/27
In an insightful if sardonic comment, leading Middle East military analyst Anthony Cordesman
has stated that the
primary export of this region is blame.3 This statement particularly applies to the United States. The United
States will probably be heavily criticized by regional opinion leaders no matter what it does or
fails to do in response to the Arab Spring . Many of the regional opinion leaders making charges
against Washington will be the same regardless of the policy , although they may vary the level of shrillness
based on actual political preferences. Moreover, intense U.S. involvement in any crisis will usually be
denounced more intensively than aloofness (which will also be criticized). Arab public opinion usually
has a default position of opposing Western intervention anywhere in the region , but there are
exceptions. To some extent, the creation of a United Nations sponsored No-Fly Zone (NFZ) over Libya was one of these exceptions,
since it was preceded by an Arab League call for such a measure, and the U.S. only played a limited and brief combat role before
other states assumed the most high visibility operational combat roles. Leading Arab nations such as Egypt and Saudi Arabia have
not participated in the NFZ, although some smaller Arab states have done so, and Qatar has been so involved with helping the
Libyans it has emerged as something of a “hero-nation.”

No Impact to Credibility – no influence in the Mid East.

Walt 11. [Stephen M., Robert and Renée Belfer professor of international relations at Harvard University. “Can the United States
'control' the Middle East? (Nope)” Foreign Policy -- January 19 --
http://walt.foreignpolicy.com/posts/2011/01/19/can_the_us_control_the_middle_east_nope]

It is yet another episode in which the United States has watched -- seemingly helplessly -- as events in places like
Tunisia, Lebanon and even Iraq unfold unexpectedly and beyond its ability to control." Shadid is obviously
right, but the observation itself is banal in at least two senses. First, even a country as powerful as the United States doesn't
"control" an awful lot of events in world politics, and especially the internal maneuverings and
struggles of a country like Lebanon. And the sooner that Americans dispense with the notion that we can reliably control events
in far-away places, the better off we'll be. Second, it is hardly surprising that the United States has steadily lost
influence (note: not control) in the Middle East. We're hamstrung by the "special relationship" with
Israel, which reduces our freedom of maneuvers, makes our rhetoric about justice and democracy and human
rights look hypocritical, and angers millions of people around the Arab and Islamic world. We foolishly invaded Iraq and
then bungled the job, which made us look both aggressive and incompetent. We continue to follow a failed
policy toward Iran, which only seems to make Ahmadinejad stronger. And we help prop up authoritarian regimes that are deeply
unpopular, favoring democracy only when the candidates we like win. And
then we wonder why we aren't able to
"control" political events in Lebanon, and we're surprised that more honest brokers are acquiring greater influence?
The mere fact that this trend seems surprising is itself quite eloquent testimony to the brain-
dead nature of our Middle East diplomacy .

US influence in the Middle East is structurally limited – no spillover.

Al-Oraibi and Russell 11. [Mina, Washington Bureau Chief for Asharq Al-Awsat, the world’s largest pan-Arab daily
newspaper, Gerard, Research Fellow @ Harvard Kennedy School of Government, “The Trust Deficit: Seven Steps Forward for U.S.—
Arab Dialogue,” The Washington Quarterly -- Summer -- http://www.twq.com/11summer/docs/11summer_Al-Oraibi_Russell.pdf]
The United States cannot expect the people of the Middle East to echo its views and support its policies
without reservation. It cannot put words in their mouths. This means, though, that the U.S. government’s
capacity to encourage democratic and liberal trends in Arab society is very limited . In Cairo in
January 2011, both anti-government and pro-government protesters claimed that the other side
was backed by the United States; as Stephen Grand of the Brookings Institution puts it, it is as if the United
States has the reverse Midas touch . 22 What the United States can do, though, is encourage other governments
to play a role in encouraging democracy and liberalism. For one thing, this means the European Union increasing its engagement
with the region.
Cuba Bioweapons Answers
Frontline
Cuba doesn’t have bioweapons

CNS 8 (Center for Nonproliferation Studies at the Monterey Institute of International Studies, “Country Profiles: Cuba”, August,
http://www.nti.org/e_research/profiles/Cuba/)

While both public and private persons have with some frequency alleged that Cuba has supported a
secret biological warfare (BW) program, no convincing proof of the existence of such a program has been
presented. Cuba signed the Biological and Toxin Convention (BWC) in 19 72 and ratified it in 19 76 . Since
approximately 1981, Castro has been instrumental in Cuba acquiring a powerful biotechnology capability which is possibly the
most advanced among developing countries. Certainly, Cuba’s biotechnological and medical industries are
the largest and most sophisticated in Latin America as demonstrated by its large-scale production of
pharmaceuticals and vaccines. This capability is being used to develop and produce products for exports and the
income from these exports might be larger than all other income-generating endeavors with the exception of sugar and
tourism.

Cuba has no bioweapons

Cereijo 8 (Manuel, “State Sponsored Terrorism: Cuba”, Net For Cuba International, 9-3,
http://www.netforcuba.org/english/Columnists/Cereijo/001en-CubaandTerrorism.htm)

Cuba's biotechnological capacity places it in group four of the World Health Organization's five categories. To
reach group
five, which is formed only by the eight top industrial economies, Cuba must produce at least 20% of the 260 basic
materials. It regularly produces 18% of these and certainly has the scientific ability to produce the others with
biotech methods. · Cuba also has 160 distinct research and development units and over 10,000 researchers through out the
country · According to Cuba’s own figures, as well as those provided by scientists and engineers, both from Cuba and other
countries, the Cuban government has spent approximately $3,500 million dollars in this industry since 1986. The
return of
such investment has been approximately the sales of $200 million dollars in vaccines and medicines . The
production for domestic use has been almost nothing, since the Cuban people lack the most
basic medicines.
Culture Answers
Frontline
Cultural survival is irrelevant – no extinction

Blake 2k (Michael, Professor of Political Philosophy and Philosophy – Harvard University, Civilization, August / September, p. 51-
53)

One frequently hears that endangered cultural groups have a right to preservation , and indeed to
outside aid and legal sanctions toward that end. Anthropologists and activists have made such claims on the grounds that the
survival of these groups has inherent value. Some advocacy groups have even gone so far as to equate the absence of such
special rights with genocide. There is no great moral distinction, such rhetoric seems to suggest, between allowing a culture to
assimilate into the wider surrounding society and actually going out and killing its members en masse. This vague moral
equation has turned up of late in the discussion of issues as varied as affirmative action, Southern regionalism, Quebecois
nationalism, and the moral status of such culturally overwhelming institutions as Wal-Mart and McDonald’s. If we take these
arguments at face value, cultural survival is something very close to a moral absolute; to refuse to endorse it is to sign up on the
side of cultural atrocity and numbing global conformity This is a shame, because it
is surprisingly difficult to figure
out exactly what is morally relevant about cultural survival in itself. The first challenge is pinning
down just what the term might mean. It cannot simply mean the continued existence of the individuals
comprising the endangered culture, since their survival is entirely compatible with their complete
assimilation and hence with the destruction of their culture. Nor however, can it mean the preservation of all
existing aspects of a culture, for some degree of cultural change and adaptation is normal, indeed
inevitable. Cultural stasis is not a plausible ideal, let alone a worthy guide to policy. The messy reality of cultural
survival, then, lies somewhere between disintegration and the deep freeze. The most plausible meaning of the slogan as a
political goal might be simply the preservation of difference: the desire that whatever cultures now exist not lose their
distinctiveness and blend into surrounding society; and that they continue to serve as means by which some people make sense
of their place in the world, however much the content of their cultures may change over time. The key idea here is that the
number of cultures now present not be reduced, however much the lifeways and customs comprising each individual culture
might change over time. But what reason have we, then, to think that cultural survival is valuable in itself? One argument draws
an analogy between cultures and other threatened aspects of the social and natural world: We ought to preserve cultures
because to do otherwise is to allow something unique and irreplaceable to leave the world. Refusing to act against assimilation
might thus be thought roughly akin to, say; shooting the last of a particularly beautiful species of condor. This argument, though,
claims too much, for we feel an equivalent sense of loss when we face not the destruction of a culture but merely its reworking
from the inside—and, thereby the destruction of specific elements within it. For example, during Quebec’s Quiet Revolution—
the tumultuous postwar period during which French Canada cast off clerical authority and conservatism and fashioned itself into
a modem secular society—much of the culture was completely remade and many traditional norms and practices abandoned.
We might easily sympathize with the feeling that there was a loss to the world in what was thereby abandoned. We do
have reason to regret the fact that current ways by which the world is understood — our own
ways included—will eventually disappear. But our justifiable sadness does not give us good
reason to declare that what is now endangered ought to be preserved forever , or to forbid
ourselves from altering inherited cultural norms —abandoning some, amending others, and embracing foreign
ways and customs as our own. One could even say that this sadness is the inevitable price we pay for freedom: If we had no
choice about what norms to adopt, and knew that our children would live as our ancestors
lived before us, the world would lose one source of woe but gain many more . This approach to
defending cultural survival, then, has some serious defects. Another line of argument harnesses the value of cultural survival to
the more kindred value of cultural diversity gaining support from the undoubted attractiveness of the latter. On reflection,
however, the ideal of cultural diversity seems scarcely less mysterious and ambiguous than the notion of cultural survival itself.
The ambiguity in valuing diversity lies, on one level, in whether it means valuing people of distinct backgrounds or valuing the
diversity of backgrounds itself. The first notion—that people ought to be respected as equals regardless of their ethnicity race,
gender, and other distinguishing traits—is today a part of any plausible political philosophy But it hardly follows that we must
value and preserve diversity itself, in the abstract; we
have, I think, no reason to regret that the world does
not contain twice as many cultures as it does . We might try to defend cultural diversity in the abstract by
pointing out how much we benefit by its concrete existence. But this raises in turn another deep ambiguity—that between
diversity of cultures and diversity within cultures. Exposure to a wide variety of lifeways is clearly of great moral value; it enables
people to flourish in ways that conformity and sameness instead suppress. But there is no necessary link between the
desirability of diversity within cultures and the demand that there be a wide variety of cultures themselves. More to the point,
the latter demand can actually work against diversity. Political measures designed to foster a culture’s
survival must perforce ascribe a negative value to assimilation ; they therefore end up
penalizing those individuals within it who seek , for example, to borrow or adapt from other
cultures. In so doing, advocates of cultural survival often provoke a stilling insistence on cultural
purity and conformity; one need only think of the recurrent French crusades for linguistic purity to realize how quickly a drive
for cultural preservation can begin to resemble a paternalistic—and, if imposed from outside, patronizing—intolerance. It is
one of the sharpest ironies of the cultural survival movement that defending a diversity of
cultures tends to repress the possibilities for diversity within cultures.
Cyber War Answers
Frontline

No risk of cyber war

Clark ’12 (MA candidate – Intelligence Studies @ American Military University, senior analyst – Chenega Federal Systems,
4/28/’12 (Paul, “The Risk of Disruption or Destruction of Critical U.S. Infrastructure by an Offensive Cyber Attack,” American Military
University)

The Department of Homeland Security worries that our critical infrastructure and key resources (CIKR)
may be exposed, both directly and indirectly, to multiple threats because of CIKR reliance on the global cyber
infrastructure, an infrastructure that is under routine cyberattack by a “spectrum of malicious actors ”
(National Infrastructure Protection Plan 2009). CIKR in the extremely large and complex U.S. economy spans multiple
sectors including agricultural, finance and banking, dams and water resources, public health and emergency services,
military and defense, transportation and shipping, and energy (National Infrastructure Protection Plan 2009). The
disruption and destruction of public and private infrastructure is part of warfare, without this infrastructure conflict cannot be
sustained (Geers 2011). Cyber-attacks are desirable because they are considered to be a relatively “low cost and long range” weapon
(Lewis 2010), but prior
to the creation of Stuxnet, the first cyber-weapon, the ability to disrupt and destroy
critical infrastructure through cyber-attack was theoretical. The movement of an offensive
cyber-weapon from conceptual to actual has forced the United States to question whether
offensive cyber-attacks are a significant threat that are able to disrupt or destroy CIKR to the level that national
security is seriously degraded. It is important to understand the risk posed to national security by cyber-attacks to ensure that
government responses are appropriate to the threat and balance security with privacy and civil liberty concerns. The
risk posed
to CIKR from cyber-attack can be evaluated by measuring the threat from cyber-attack against
the vulnerability of a CIKR target and the consequences of CIKR disruption. As the only known cyber-
weapon, Stuxnet has been thoroughly analyzed and used as a model for predicting future cyber-
weapons. The U.S. electrical grid, a key component in the CIKR energy sector, is a target that has been analyzed
for vulnerabilities and the consequences of disruption predicted – the electrical grid has been used in
multiple attack scenarios including a classified scenario provided to the U.S. Congress in 2012 (Rohde 2012). Stuxnet will
serve as the weapon and the U.S. electrical grid will serve as the target in this risk analysis that concludes that there
is a low risk of disruption or destruction of critical infrastructure from a an offensive cyber-weapon
because of the complexity of the attack path, the limited capability of non-state adversaries to
develop cyber-weapons, and the existence of multiple methods of mitigating the cyber-attacks. To
evaluate the threat posed by a Stuxnet-like cyber-weapon, the complexity of the weapon, the available attack vectors for the
weapon, and the resilience of the weapon must be understood. The complexity – how difficult and expensive it was to create the
weapon – identifies the relative cost and availability of the weapon; inexpensive and simple to build will be more prevalent than
expensive and difficult to build. Attack vectors are the available methods of attack; the larger the number, the more severe the
threat. For example, attack vectors for a cyberweapon may be email attachments, peer-to-peer applications, websites, and infected
USB devices or compact discs. Finally, the resilience of the weapon determines its availability and affects its usefulness. A useful
weapon is one that is resistant to disruption (resilient) and is therefore available and reliable. These concepts are seen in the AK-47
assault rifle – a simple, inexpensive, reliable and effective weapon – and carry over to information technology structures (Weitz
2012). The evaluation of Stuxnet identified malware that is “unusually
complex and large” and required
code written in multiple languages (Chen 2010) in order to complete a variety of specific functions
contained in a “vast array” of components – it is one of the most complex threats ever
analyzed by Symantec (Falliere, Murchu and Chien 2011). To be successful, Stuxnet required a high level
of technical knowledge across multiple disciplines, a laboratory with the target equipment
configured for testing, and a foreign intelligence capability to collect information on the target
network and attack vectors (Kerr, Rollins and Theohary 2010). The malware also needed careful
monitoring and maintenance because it could be easily disrupted; as a result Stuxnet was developed with a high
degree of configurability and was upgraded multiple times in less than one year (Falliere, Murchu and Chien
2011). Once introduced into the network, the cyber-weapon then had to utilize four known
vulnerabilities and four unknown vulnerabilities, known as zero-day exploits, in order to install itself
and propagate across the target network (Falliere, Murchu and Chien 2011). Zero-day exploits are incredibly
difficult to find and fewer than twelve out of the 12,000,000 pieces of malware discovered each year
utilize zero-day exploits and this rarity makes them valuable, zero-days can fetch $50,000 to $500,000 each
on the black market (Zetter 2011). The use of four rare exploits in a single piece of malware is
“unprecedented” (Chen 2010). Along with the use of four unpublished exploits, Stuxnet also used the “first ever”
programmable logic controller rootkit, a Windows rootkit, antivirus evasion techniques, intricate process
injection routines, and other complex interfaces (Falliere, Murchu and Chien 2011) all wrapped up in
“layers of encryption like Russian nesting dolls” (Zetter 2011) – including custom encryption algorithms (Karnouskos 2011).
As the malware spread across the now-infected network it had to utilize additional vulnerabilities in proprietary
Siemens industrial control software (ICS) and hardware used to control the equipment it was designed to sabotage.
Some of these ICS vulnerabilities were published but some were unknown and required such a high
degree of inside knowledge that there was speculation that a Siemens employee had been
involved in the malware design (Kerr, Rollins and Theohary 2010). The unprecedented technical
complexity of the Stuxnet cyber-weapon, along with the extensive technical and financial resources and foreign intelligence
capabilities required for its development and deployment, indicates that the malware was likely developed by a
nation-state (Kerr, Rollins and Theohary 2010). Stuxnet had very limited attack vectors. When a computer system is connected
to the public Internet a host of attack vectors are available to the cyber-attacker (Institute for Security Technology Studies 2002).
Web browser and browser plug-in vulnerabilities, cross-site scripting attacks, compromised email attachments, peer-to-peer
applications, operating system and other application vulnerabilities are all vectors for the introduction of malware into an
Internetconnected computer system. Networks that are not connected to the public internet are “air
gapped,” a technical colloquialism to identify a physical separation between networks. Physical separation from the
public Internet is a common safeguard for sensitive networks including classified U.S.
government networks. If the target network is air gapped, infection can only occur through
physical means – an infected disk or USB device that must be physically introduced into a possibly
access controlled environment and connected to the air gapped network. The first step of the Stuxnet cyber-attack was to initially
infect the target networks, a difficult task given the probable disconnected and well secured nature of the Iranian nuclear facilities.
Stuxnet was introduced via a USB device to the target network, a method that suggests that the
attackers were familiar with the configuration of the network and knew it was not connected to the public Internet (Chen
2010). This assessment is supported by two rare features in Stuxnet – having all necessary functionality for industrial sabotage fully
embedded in the malware executable along with the ability to self-propagate and upgrade through a peer-to-peer method (Falliere,
Murchu and Chien 2011). Developing an understanding of the target network configuration was a significant and daunting task
based on Symantec’s assessment that Stuxnet repeatedly targeted a total of five different organizations over nearly one year
(Falliere, Murchu and Chien 2011) with physical introduction via USB drive being the only available attack vector. The final factor in
assessing the threat of a cyber-weapon is the resilience of the weapon. There are two primary factors that make Stuxnet non-
resilient: the complexity of the weapon and the complexity of the target. Stuxnet was highly customized for sabotaging specific
industrial systems (Karnouskos 2011) and needed a large number of very complex components and routines in order to increase its
chance of success (Falliere, Murchu and Chien 2011). The malware required eight vulnerabilities in the Windows operating system to
succeed and therefore would have failed if those vulnerabilities had been properly patched; four of the eight vulnerabilities were
known to Microsoft and subject to elimination (Falliere, Murchu and Chien 2011). Stuxnet also required that two drivers be installed
and required two stolen security certificates for installation (Falliere, Murchu and Chien 2011); driver installation would have failed if
the stolen certificates had been revoked and marked as invalid. Finally, the configuration of systems is ever-changing as components
are upgraded or replaced. There is no guarantee that the network that was mapped for vulnerabilities had not changed in the
months, or years, it took to craft Stuxnet and successfully infect the target network. Had specific components of the target hardware
changed – the targeted Siemens software or programmable logic controller – the attack would have failed. Threats are less of a
threat when identified; this is why zero-day exploits are so valuable. Stuxnet went to great lengths to hide its existence from the
target and utilized multiple rootkits, data manipulation routines, and virus avoidance techniques to stay undetected. The malware’s
actions occurred only in memory to avoid leaving traces on disk, it masked its activities by running under legal programs, employed
layers of encryption and code obfuscation, and uninstalled itself after a set period of time, all efforts to avoid detection because its
authors knew that detection meant failure. As a result of the complexity of the malware, the changeable nature of the target
network, and the chance of discovery, Stuxnet
is not a resilient system. It is a fragile weapon that
required an investment of time and money to constantly monitor, reconfigure, test and deploy over the
course of a year. There is concern, with Stuxnet developed and available publicly, that the world is on the
brink of a storm of highly sophisticated Stuxnet-derived cyber-weapons which can be used by hackers,
organized criminals and terrorists (Chen 2010). As former counterterrorism advisor Richard Clarke describes it, there is
concern that the technical brilliance of the United States “has created millions of potential monsters all over the world” (Rosenbaum
2012). Hyperbole aside, technical knowledge spreads. The techniques behind cyber-attacks are “constantly
evolving and making use of lessons learned over time ” (Institute for Security Technology Studies 2002) and the
publication of the Stuxnet code may make it easier to copy the weapon (Kerr, Rollins and Theohary 2010).
However, this is something of a zero-sum game because knowledge works both ways and cyber-security
techniques are also evolving, and “understanding attack techniques more clearly is the first step toward increasing
security” (Institute for Security Technology Studies 2002). Vulnerabilities are discovered and patched , intrusion
detection and malware signatures are expanded and updated, and monitoring and analysis
processes and methodologies are expanded and honed. Once the element of surprise is lost, weapons
and tactics are less useful, this is the core of the argument that “uniquely surprising” stratagems like Stuxnet
are single-use, like Pearl Harbor and the Trojan Horse, the “very success [of these attacks] precludes
their repetition” (Mueller 2012). This paradigm has already been seen in the “son of Stuxnet” malware – named
Duqu by its discoverers – that is based on the same modular code platform that created Stuxnet (Ragan 2011). With the
techniques used by Stuxnet now known, other variants such as Duqu are being discovered and
countered by security researchers (Laboratory of Cryptography and System Security 2011). It is obvious that the
effort required to create, deploy, and maintain Stuxnet and its variants is massive and it is not
clear that the rewards are worth the risk and effort. Given the location of initial infection and the number of
infected systems in Iran (Falliere, Murchu and Chien 2011) it is believed that Iranian nuclear facilities were the target of the Stuxnet
weapon. A significant amount of money and effort was invested in creating Stuxnet but yet the
expected result – assuming that this was an attack that expected to damage production – was minimal at best. Iran
claimed that Stuxnet caused only minor damage, probably at the Natanz enrichment facility, the Russian contractor
Atomstroyeksport reported that no damage had occurred at the Bushehr facility, and an unidentified “senior diplomat” suggested
that Iran was forced to shut down its centrifuge facility “for a few days” (Kerr, Rollins and Theohary 2010). Even
the most
optimistic estimates believe that Iran’s nuclear enrichment program was only delayed by
months, or perhaps years (Rosenbaum 2012). The actual damage done by Stuxnet is not clear (Kerr, Rollins and Theohary 2010)
and the primary damage appears to be to a higher number than average replacement of centrifuges at the Iran enrichment facility
(Zetter 2011). Different targets may produce different results. The Iranian nuclear facility was a difficult target with limited attack
vectors because of its isolation from the public Internet and restricted access to its facilities. What is the probability of a
successful attack against the U.S. electrical grid and what are the potential consequences should this critical
infrastructure be disrupted or destroyed? An attack against the electrical grid is a reasonable threat scenario since power
systems are “a high priority target for military and insurgents ” and there has been a trend towards utilizing
commercial software and integrating utilities into the public Internet that has “increased vulnerability across the board” (Lewis
2010). Yet the increased vulnerabilities are mitigated by an increased detection and deterrent
capability that has been “honed over many years of practical application” now that power systems
are using standard, rather than proprietary and specialized, applications and components (Leita
and Dacier 2012). The security of the electrical grid is also enhanced by increased awareness after a
smart-grid hacking demonstration in 2009 and the identification of the Stuxnet malware in 2010;
as a result the
public and private sector are working together in an “unprecedented effort” to
establish robust security guidelines and cyber security measures (Gohn and Wheelock 2010).

No risk of cyberattack and no impact if it does happen

Birch, 12 – former foreign correspondent for the Associated Press and the Baltimore Sun who has written extensively on
technology and public policy (Douglas, “Forget Revolution.” Foreign Policy.
http://www.foreignpolicy.com/articles/2012/10/01/forget_revolution?page=full)

"That's a good example of what some kind of attacks would be like," he said. "You don't want to overestimate the risks. You don't
want somebody to be able to do this whenever they felt like it, which is the situation now. But this is not the end of the world." The
question of how seriously to take the threat of a cyber attack on critical infrastructure surfaced recently, after Congress rejected a
White House measure to require businesses to adopt stringent new regulations to protect their computer networks from intrusions.
The bill would have required industries to report cyber security breaches, toughen criminal penalties against hacking and granted
legal immunity to companies cooperating with government investigations. Critics worried about regulatory overreach. But the
potential cost to industry also seems to be a major factor in the bill's rejection. A January study by Bloomberg reported that banks,
utilities, and phone carriers would have to increase their spending on cyber security by a factor of nine, to $45.3 billion a year, in
order to protect themselves against 95 percent of cyber intrusions. Likewise, some of the bill's advocates suspect that in the
aftermath of a truly successful cyber attack, the government would have to bail the utilities out anyway. Joe Weiss, a cyber security
professional and an authority on industrial control systems like those used in the electric grid, argued that a well-prepared,
sophisticated cyber attack could have far more serious consequences than this summer's blackouts. "The reason we are so
concerned is that cyber could take out the grid for nine to 18 months," he said. "This isn't a one to five day outage. We're prepared
for that. We can handle that." But pulling
off a cyber assault on that scale is no easy feat. Weiss agreed that
hackers intent on inflicting this kind of long-term interruption of power would need to use a tool
capable of inflicting physical damage. And so far, the world has seen only one such weapon:
Stuxnet, which is believed to have been a joint military project of Israel and the United States . Ralph Langner,
a German expert on industrial-control system security, was among the first to discover that Stuxnet was specifically designed to
attack the Supervisory Control and Data Acquisition system (SCADA) at a single site: Iran's Natanz uranium-enrichment plant. The
computer worm's sophisticated programs, which infected the plant in 2009, caused about 1,000 of Natanz's 5,000 uranium-
enrichment centrifuges to self-destruct by accelerating their precision rotors beyond the speeds at which they were designed to
operate. Professionals like Weiss and others warned that Stuxnet was opening a Pandora's Box : Once it was
unleashed on the world, they feared, it would become available to hostile states, criminals, and terrorists
who could adapt the code for their own nefarious purposes. But two years after the discovery of
Stuxnet, there are no reports of similar attacks against the United States. What has prevented the
emergence of such copycat viruses? A 2009 paper published by the University of California, Berkeley, may offer the
answer. The report, which was released a year before Stuxnet surfaced, found that in order to create a cyber
weapon capable of crippling a specific control system -- like the ones operating the U.S. electric
grid -- six coders might have to work for up to six months to reverse engineer the targeted
center's SCADA system. Even then, the report says, hackers likely would need the help of someone
with inside knowledge of how the network's machines were wired together to plan an effective
attack. "Every SCADA control center is configured differently , with different devices, running
different software/protocols," wrote Rose Tsang, the report's author. Professional hackers are in it for the
money -- and it's a lot more cost-efficient to search out vulnerabilities in widely-used computer
programs like the Windows operating system, used by banks and other affluent targets, than in one-of-a-kind SCADA systems
linked to generators and switches. According to Pollard, only the world's industrial nations have the means to
use the Internet to attack utilities and major industries. But given the integrated global
economy, there is little incentive, short of armed conflict, for them to do so. "If you're a state
that has a number of U.S. T-bills in your treasury, you have an economic interest in the United
States," he said. "You're not going to have an interest in mucking about with our infrastructure. "
There is also the threat of retaliation. Last year, the U.S. government reportedly issued a classified
report on cyber strategy that said it could respond to a devastating digital assault with
traditional military force. The idea was that if a cyber attack caused death and destruction on the scale of a military assault,
the United States would reserve the right to respond with what the Pentagon likes to call "kinetic"
weapons: missiles, bombs, and bullets. An unnamed Pentagon official, speaking to the Wall Street Journal, summed up the policy
in less diplomatic terms: "If you shut down our power grid, maybe we will put a missile down one of
your smokestacks." Deterrence is sometimes dismissed as a toothless strategy against cyber attacks because hackers have
such an easy time hiding in the anonymity of the Web. But investigators typically come up with key suspects , if not
smoking guns, following cyber intrusions and assaults -- the way suspicions quickly focused on the United States and
Israel after Stuxnet was discovered. And with the U.S. military's global reach, even terror groups have to
factor in potential retaliation when planning their operations.

Cyberattacks nearly impossible – empirics and defenses solve

Rid 12 (Thomas Rid, reader in war studies at King's College London, is author of "Cyber War Will Not Take Place" and co-author of
"Cyber-Weapons.", March/April 2012, “Think Again: Cyberwar”, http://www.foreignpolicy.com/articles/2012/02/27/cyberwar?
page=full)

"Cyberwar Is Already Upon Us." No way. "Cyberwar is coming!" John Arquilla and David Ronfeldt predicted in a celebrated
Rand paper back in 1993. Since then, it seems to have arrived -- at least by the account of the U.S. military establishment,
which is busy competing over who should get what share of the fight. Cyberspace is "a domain in which the Air Force flies and
fights," Air Force Secretary Michael Wynne claimed in 2006. By 2012, William J. Lynn III, the deputy defense secretary at the time,
was writing that cyberwar is "just as critical to military operations as land, sea, air, and space ." In
January, the Defense Department vowed to equip the U.S. armed forces for "conducting a combined arms campaign across all
domains -- land, air, maritime, space, and cyberspace." Meanwhile, growing piles of books and articles explore the threats of
cyberwarfare, cyberterrorism, and how to survive them. Time for a reality check: Cyberwar is still more hype than
hazard. Consider the definition of an act of war: It has to be potentially violent, it has to be purposeful, and it
has to be political. The cyberattacks we've seen so far, from Estonia to the Stuxnet virus, simply don't meet
these criteria. Take the dubious story of a Soviet pipeline explosion back in 1982, much cited by cyberwar's true believers as
the most destructive cyberattack ever. The account goes like this: In June 1982, a Siberian pipeline that the CIA had
virtually booby-trapped with a so-called "logic bomb" exploded in a monumental fireball that could be seen from space. The U.S. Air
Force estimated the explosion at 3 kilotons, equivalent to a small nuclear device. Targeting a Soviet pipeline linking gas fields in
Siberia to European markets, the operation sabotaged the pipeline's control systems with software from a Canadian firm that the
CIA had doctored with malicious code. No one died, according to Thomas Reed, a U.S. National Security Council aide at the time
who revealed the incident in his 2004 book, At the Abyss; the only harm came to the Soviet economy . But did it
really happen? After Reed's account came out, Vasily Pchelintsev, a former KGB head of the Tyumen region,
where the alleged explosion supposedly took place, denied the story. There are also no media
reports from 1982 that confirm such an explosion, though accidents and pipeline explosions in
the Soviet Union were regularly reported in the early 1980s. Something likely did happen, but Reed's
book is the only public mention of the incident and his account relied on a single document . Even after the
CIA declassified a redacted version of Reed's source, a
note on the so-called Farewell Dossier that describes the effort to
provide the Soviet Union with defective technology, the agency did not confirm that such an
explosion occurred. The available evidence on the Siberian pipeline blast is so thin that it shouldn't be
counted as a proven case of a successful cyberattack. Most other commonly cited cases of cyberwar
are even less remarkable. Take the attacks on Estonia in April 2007, which came in response to the
controversial relocation of a Soviet war memorial , the Bronze Soldier. The well-wired country found itself at the
receiving end of a massive distributed denial-of-service attack that emanated from up to 85,000 hijacked computers and lasted three
weeks. The attacks reached a peak on May 9, when 58 Estonian websites were attacked at once and the online services of Estonia's
largest bank were taken down. "What's the difference between a blockade of harbors or airports of sovereign states and the
blockade of government institutions and newspaper websites?" asked Estonian Prime Minister Andrus Ansip. Despite his analogies,
the attack was no act of war. It was certainly a nuisance and an emotional strike on the
country, but the bank's actual network was not even penetrated; it went down for 90 minutes
one day and two hours the next. The attack was not violent, it wasn't purposefully aimed at
changing Estonia's behavior, and no political entity took credit for it. The same is true for the vast
majority of cyberattacks on record. Indeed, there is no known cyberattack that has caused the loss of
human life. No cyberoffense has ever injured a person or damaged a building . And if an act is not
at least potentially violent, it's not an act of war. Separating war from physical violence makes it a metaphorical
notion; it would mean that there is no way to distinguish between World War II, say, and the "wars" on obesity and cancer. Yet
those ailments, unlike past examples of cyber "war," actually do kill people. "A
Digital Pearl Harbor Is Only a Matter
of Time." Keep waiting. U.S. Defense Secretary Leon Panetta delivered a stark warning last summer: "We could face a
cyberattack that could be the equivalent of Pearl Harbor." Such alarmist predictions have been ricocheting inside the
Beltway for the past two decades, and some scaremongers have even upped the ante by raising
the alarm about a cyber 9/11. In his 2010 book, Cyber War, former White House counterterrorism czar Richard Clarke
invokes the specter of nationwide power blackouts, planes falling out of the sky, trains derailing, refineries burning, pipelines
exploding, poisonous gas clouds wafting, and satellites spinning out of orbit -- events that would make the 2001 attacks pale in
comparison. But the
empirical record is less hair-raising, even by the standards of the most drastic
example available. Gen. Keith Alexander, head of U.S. Cyber Command (established in 2010 and now boasting a
budget of more than $3 billion), shared his worst fears in an April 2011 speech at the University of Rhode Island: "What I'm
concerned about are destructive attacks," Alexander said, "those that are coming." He then invoked a remarkable accident at
Russia's Sayano-Shushenskaya hydroelectric plant to highlight the kind of damage a cyberattack might be able to cause. Shortly after
midnight on Aug. 17, 2009, a 900-ton turbine was ripped out of its seat by a so-called "water hammer," a sudden surge in water
pressure that then caused a transformer explosion. The turbine's unusually high vibrations had worn down the bolts that kept its
cover in place, and an offline sensor failed to detect the malfunction. Seventy-five people died in the accident, energy prices in
Russia rose, and rebuilding the plant is slated to cost $1.3 billion. Tough luck for the Russians, but here's what the head of Cyber
Command didn't say: The ill-fated turbine had been malfunctioning for some time, and the plant's management was notoriously
poor. On top of that, the key event that ultimately triggered the catastrophe seems to have been a fire at Bratsk power station,
about 500 miles away. Because the energy supply from Bratsk dropped, authorities remotely increased the burden on the Sayano-
Shushenskaya plant. The sudden spike overwhelmed the turbine, which was two months shy of reaching the end of its 30-year life
cycle, sparking the catastrophe. If anything, the Sayano-Shushenskaya incident highlights how difficult a
devastating attack would be to mount. The plant's washout was an accident at the end of a
complicated and unique chain of events. Anticipating such vulnerabilities in advance is
extraordinarily difficult even for insiders; creating comparable coincidences from cyberspace
would be a daunting challenge at best for outsiders. If this is the most drastic incident Cyber Command can conjure
up, perhaps it's time for everyone to take a deep breath. "Cyberattacks Are Becoming Easier." Just the opposite.
U.S. Director of National Intelligence James R. Clapper
warned last year that the volume of malicious
software on American networks had more than tripled since 2009 and that more than 60,000 pieces of
malware are now discovered every day. The United States, he said, is undergoing "a phenomenon known as
'convergence,' which amplifies the opportunity for disruptive cyberattacks , including against physical
infrastructures." ("Digital convergence" is a snazzy term for a simple thing: more and more devices able to talk to each other, and
formerly separate industries and activities able to work together.) Just because there's more malware, however,
doesn't mean that attacks are becoming easier. In fact, potentially damaging or life-threatening
cyberattacks should be more difficult to pull off. Why? Sensitive systems generally have built-in
redundancy and safety systems, meaning an attacker's likely objective will not be to shut down a
system, since merely forcing the shutdown of one control system , say a power plant, could trigger a
backup and cause operators to start looking for the bug. To work as an effective weapon,
malware would have to influence an active process -- but not bring it to a screeching halt. If
the malicious activity extends over a lengthy period, it has to remain stealthy . That's a more
difficult trick than hitting the virtual off-button. Take Stuxnet, the worm that sabotaged Iran's nuclear program
in 2010. It didn't just crudely shut down the centrifuges at the Natanz nuclear facility ; rather, the
worm subtly manipulated the system. Stuxnet stealthily infiltrated the plant's networks, then hopped onto the
protected control systems, intercepted input values from sensors, recorded these data, and then provided the legitimate controller
code with pre-recorded fake input signals, according to researchers who have studied the worm. Its objective was not just to fool
operators in a control room, but also to circumvent digital safety and monitoring systems so it could secretly manipulate the actual
processes. Building
and deploying Stuxnet required extremely detailed intelligence about the
systems it was supposed to compromise, and the same will be true for other dangerous
cyberweapons. Yes, "convergence," standardization, and sloppy defense of control-systems
software could increase the risk of generic attacks, but the same trend has also caused
defenses against the most coveted targets to improve steadily and has made reprogramming highly
specific installations on legacy systems more complex, not less.

No impact to cyberwar - it's all hype - it's technically impossible and won't escalate

Rid 12 (Thomas, PhD, Reader in War Studies @ King's College London, Non-Resident Fellow at the Center for Transatlantic
Relations in the School for Advanced International Studies at Johns Hopkins, "Think Again: Cyberwar," March/April, Foreign Policy,
http://www.foreignpolicy.com/articles/2012/02/27/cyberwar?page=0,0,

Time for a reality check: Cyberwar is still more hype than hazard. Consider the definition of an act of war: It has
to be potentially violent, it has to be purposeful, and it has to be political. The cyberattacks we've seen so far, from
Estonia to the Stuxnet virus, simply don't meet these criteria. Take the dubious story of a Soviet pipeline explosion back in
1982, much cited by cyberwar's true believers as the most destructive cyberattack ever. The account goes like this: In June 1982, a
Siberian pipeline that the CIA had virtually booby-trapped with a so-called "logic bomb" exploded in a monumental fireball that
could be seen from space. The U.S. Air Force estimated the explosion at 3 kilotons, equivalent to a small nuclear device. Targeting a
Soviet pipeline linking gas fields in Siberia to European markets, the operation sabotaged the pipeline's control systems with
software from a Canadian firm that the CIA had doctored with malicious code. No one died, according to Thomas Reed, a U.S.
National Security Council aide at the time who revealed the incident in his 2004 book, At the Abyss; the only harm came to the
Soviet economy. But did it really happen? After Reed's account came out, Vasily Pchelintsev, a former KGB head of the Tyumen
region, where the alleged explosion supposedly took place, denied the story. There are also no media reports from 1982 that
confirm such an explosion, though accidents and pipeline explosions in the Soviet Union were regularly reported in the early 1980s.
Something likely did happen, but Reed's book is the only public mention of the incident and his account relied on a single document.
Even after the CIA declassified a redacted version of Reed's source, a note on the so-called Farewell Dossier that describes the effort
to provide the Soviet Union with defective technology, the agency did not confirm that such an explosion occurred. The available
evidence on the Siberian pipeline blast is so thin that it shouldn't be counted as a proven case of a successful cyberattack. Most
other commonly cited cases of cyberwar are even less remarkable. Take the attacks on Estonia in April 2007,
which came in response to the controversial relocation of a Soviet war memorial, the Bronze Soldier. The well-wired country found
itself at the receiving end of a massive distributed denial-of-service attack that emanated from up to 85,000 hijacked computers and
lasted three weeks. The attacks reached a peak on May 9, when 58 Estonian websites were attacked at once and the online services
of Estonia's largest bank were taken down. "What's the difference between a blockade of harbors or airports of sovereign states and
the blockade of government institutions and newspaper websites?" asked Estonian Prime Minister Andrus Ansip. Despite his
analogies, the attack was no act of war. It was certainly a nuisance and an emotional strike on the country, but the bank's actual
network was not even penetrated; it went down for 90 minutes one day and two hours the next. The attack was not violent, it
wasn't purposefully aimed at changing Estonia's behavior, and no political entity took credit for it. The same is true for the vast
majority of cyberattacks on record. Indeed, there
is no known cyberattack that has caused the loss of human
life. No cyberoffense has ever injured a person or damaged a building. And if an act is not at least
potentially violent, it's not an act of war. Separating war from physical violence makes it a metaphorical notion; it would mean that
there is no way to distinguish between World War II, say, and the "wars" on obesity and cancer. Yet those ailments, unlike past
examples of cyber "war," actually do kill people. "A Digital Pearl Harbor Is Only a Matter of Time." Keep waiting.
U.S. Defense Secretary Leon Panetta delivered a stark warning last summer: "We could face a cyberattack that could be the
equivalent of Pearl Harbor." Such alarmist predictions have been ricocheting inside the Beltway for the
past two decades, and some scaremongers have even upped the ante by raising the alarm about a cyber 9/11. In his 2010
book, Cyber War, former White House counterterrorism czar Richard Clarke invokes the specter of nationwide power blackouts,
planes falling out of the sky, trains derailing, refineries burning, pipelines exploding, poisonous gas clouds wafting, and satellites
spinning out of orbit -- events that would make the 2001 attacks pale in comparison. But
the empirical record is less
hair-raising, even by the standards of the most drastic example available . Gen. Keith Alexander, head
of U.S. Cyber Command (established in 2010 and now boasting a budget of more than $3 billion), shared his worst fears in an April
2011 speech at the University of Rhode Island: "What I'm concerned about are destructive attacks," Alexander said, "those that are
coming." He then invoked a remarkable accident at Russia's Sayano-Shushenskaya hydroelectric plant to highlight the
kind of damage a cyberattack might be able to cause. Shortly after midnight on Aug. 17, 2009, a 900-ton turbine was ripped out of its
seat by a so-called "water hammer," a sudden surge in water pressure that then caused a transformer explosion. The turbine's
unusually high vibrations had worn down the bolts that kept its cover in place, and an offline sensor failed to detect the malfunction.
Seventy-five people died in the accident, energy prices in Russia rose, and rebuilding the plant is slated to cost $1.3 billion. Tough
luck for the Russians, but here's what the head of Cyber Command didn't say: The ill-fated turbine had been malfunctioning for
some time, and the plant's management was notoriously poor. On top of that, the key event that ultimately triggered the
catastrophe seems to have been a fire at Bratsk power station, about 500 miles away. Because the energy supply from Bratsk
dropped, authorities remotely increased the burden on the Sayano-Shushenskaya plant. The sudden spike overwhelmed the turbine,
which was two months shy of reaching the end of its 30-year life cycle, sparking the catastrophe. If anything, the Sayano-
Shushenskaya incident highlights how difficult a devastating attack would be to mount . The plant's
washout was an accident at the end of a complicated and unique chain of events. Anticipating such vulnerabilities in advance is
extraordinarily difficult even for insiders; creating comparable coincidences from cyberspace would be a daunting challenge at best
for outsiders. If
this is the most drastic incident Cyber Command can conjure up , perhaps it's time
for everyone to take a deep breath.

Their evidence is flawed and should be rejected


Rahman ’10 (Arifeen Rahman, SEA National Security Intern, Scientist and Engineers for America, “Bringing Cybersecurity Back
to Reality”, http://www.sefora.org/2010/08/11/rahman_0811/ , August 11, 2010, LEQ)

The controversy over “cyber-rhetoric” is often reduced to a debate over the devastating magnitude of such possible scenarios
versus the risks of framing them in such language. However, the truths in both positions do not inherently contradict each
other, and can be reconciled. The great equalizer of these theories, therefore, lies in an analysis of predictions. The extent to
which we can, or arguably should, take pre-emptive action to prevent cyber-attacks has its foundations in the determining the
probability of these events. Recent trends in the cyber security discussion have forgone an
assessment of probability for a “possibilistic” worst case assessment. Most studies tend to
sensationalize the threats which cyberspace presents, forecasting imminent scenarios of
destruction. Not only have these predictions been categorically proven false over the last
ten years, but have also brought into question the reliability of these assessments overall .
Black Swan theory, coined by Nassim Nicholas Taleb, explains how major catastrophes are low probability, high
impact scenarios which were impossible to predict, but seem inevitable when looked at retrospectively. Viewing cyber security
through this lens allows for a separation of policy from threat inflation. Black Swan theory proves that obsession over
specific scenarios of cyber attack remain futile. The scenario which is never considered, which is most
unexpected, will occur. Therefore, the only logical way to maintain cyber security is to preserve defensive protection of critical
infrastructure, and avoid hyping up new threats. […] Cyber security will remain a critical issue for the current and all following
administrations. In order to create a safe and secure environment, critical vulnerabilities must be addressed without creating an
atmosphere of fear and paranoia. The “cyber-Armageddon” is not coming. It’s time for the federal government
to get its head out its science fiction novel, and get back to reality.

Zero risk of cyber attack- new studies

Leyden ’11 (The ill-informed leading the ill-informed... By John Leyden • Get more from this author Posted in Government, 17th
January 2011 11:23 GMT

Cyberwar hype is inhibiting government attempts to develop an appropriate response to


cybersecurity threats, say computer scientists. A heavyweight study by UK computer scientists
for the Organisation for Economic Cooperation and Development ( OECD) concludes that it is "highly unlikely"
there will ever be a "pure cyber war”, comparable with recent conflicts in Afghanistan or the Balkans.
Suggestions to the contrary are down to "heavy lobbying" by suppliers , the report's authors – Professor
Peter Sommer of the London School of Economics and Dr Ian Brown of the Oxford Internet Institute, University of Oxford –
conclude. It is unlikely that there will ever be a true cyberwar . The
reasons are: many critical computer systems
are protected against known exploits and malware so that designers of new cyberweapons
have to identify new weaknesses and exploits; the effects of cyberattacks are difficult to
predict – on the one hand they may be less powerful than hoped but may also have more
extensive outcomes arising from the interconnectedness of systems, resulting in unwanted
damage to perpetrators and their allies . More importantly, there is no strategic reason why any aggressor would
limit themselves to only one class of weaponry.

At worst the impact will be contained

Rid 12 (Thomas, PhD, Reader in War Studies @ King's College London, Non-Resident Fellow at the Center for Transatlantic
Relations in the School for Advanced International Studies at Johns Hopkins, "Think Again: Cyberwar," March/April, Foreign Policy,
http://www.foreignpolicy.com/articles/2012/02/27/cyberwar?page=0,0,

"Cyberweapons Can Create Massive Collateral Damage." Very unlikely. When news of Stuxnet broke, the
New York Times reported that the most striking aspect of the new weapon was the "collateral damage" it created. The malicious
program was "splattered on thousands of computer systems around the world, and much of its impact has been on those systems,
rather than on what appears to have been its intended target, Iranian equipment," the Times reported. Such descriptions
encouraged the view that computer viruses are akin to highly contagious biological viruses that, once
unleashed from the lab, will turn against all vulnerable systems, not just their intended targets. But this metaphor is deeply
flawed. As the destructive potential of a cyberweapon grows, the likelihood that it could do far-
reaching damage across many systems shrinks. Stuxnet did infect more than 100,000 computers -- mainly in Iran,
Indonesia, and India, though also in Europe and the United States. But it was so specifically programmed that it didn't actually
damage those machines, afflicting only Iran's centrifuges at Natanz. The worm's aggressive infection strategy was designed to
maximize the likelihood that it would reach its intended target. Because that final target was not networked, "all the functionality
required to sabotage a system was embedded directly in the Stuxnet executable," the security software company Symantec
observed in its analysis of the worm's code. So yes, Stuxnet was "splattered" far and wide, but it only executed its damaging payload
where it was supposed to. Collateral infection, in short, is not necessarily collateral damage . A sophisticated
piece of malware may aggressively infect many systems, but if there is an intended target, the infection will likely have a distinct
payload that will be harmless to most computers. Especially in the context of more sophisticated cyberweapons, the image of
inadvertent collateral damage doesn't hold up. They're more like a flu virus that only makes one family sick.

Defensive measures overwhelm

Rid 12 (Thomas, PhD, Reader in War Studies @ King's College London, Non-Resident Fellow at the Center for Transatlantic
Relations in the School for Advanced International Studies at Johns Hopkins, "Think Again: Cyberwar," March/April, Foreign Policy,
http://www.foreignpolicy.com/articles/2012/02/27/cyberwar?page=0,0,

"In Cyberspace, Offense Dominates Defense." Wrong again. The information age has "offense-dominant
attributes," Arquilla and Ronfeldt wrote in their influential 1996 book, The Advent of Netwar. This view has spread through the
American defense establishment like, well, a virus. A 2011 Pentagon report on cyberspace stressed "the advantage currently enjoyed
by the offense in cyberwarfare." The intelligence community stressed the same point in its annual threat report to Congress last
year, arguing that offensive tactics -- known as vulnerability discovery and exploitation -- are evolving more rapidly than the federal
government and industry can adapt their defensive best practices. The conclusion seemed obvious: Cyberattackers have the
advantage over cyberdefenders, "with the trend likely getting worse over the next five years." A closer examination of the
record, however, reveals three factors that put the offense at a disadvantage. First is the high cost of
developing a cyberweapon, in terms of time, talent, and target intelligence needed. Stuxnet, experts
speculate, took a superb team and a lot of time. Second, the potential for generic offensive weapons may be far
smaller than assumed for the same reasons, and significant investments in highly specific attack
programs may be deployable only against a very limited target set. Third, once developed, an
offensive tool is likely to have a far shorter half-life than the defensive measures put in place
against it. Even worse, a weapon may only be able to strike a single time; once the exploits of a
specialized piece of malware are discovered, the most critical systems will likely be patched and
fixed quickly. And a weapon, even a potent one, is not much of a weapon if an attack cannot be
repeated. Any political threat relies on the credible threat to attack or to replicate a successful attack. If that were in doubt, the
coercive power of a cyberattack would be drastically reduced.
---Cyber War- Threat Inflation

Their authors inflate threats and conflate completely different types of attacks

Clark’12, MA candidate – Intelligence Studies @ American Military University, senior analyst – Chenega Federal Systems, 4/28/
(Paul, “The Risk of Disruption or Destruction of Critical U.S. Infrastructure by an Offensive Cyber Attack,” American Military
University)

This increased focus on cyber-security has led to concern that the perceived risk is greater than the actual risk, a
situation that has resulted in an imbalance between security and privacy and civil liberties (American Civil Liberties Union 2012). In
1993 a Rand Corporation paper predicted that “cyberwar is coming” and twenty years later the
prediction is the same and critics argue that cyber-war is “more hype than hazard” (Rid 2012). A review of
high profile cyberattacks shows that, with the exception of Stuxnet and the limited Israeli
disruption of Syrian air defense networks, most cyber-attacks are categorized as information theft,
network compromise, or website defacement (Lewis 2012). Even the high profile threat of an “Electronic
Pearl Harbor” (Bronk 2009), despite being repeated by senior government officials like U.S. Defense
Secretary Leon Panetta (Rid 2012) , has been found to be only a slight possibility (Wilson 2005). There is no doubt
that cyber-security is important. Businesses recognize this importance and spent more than $80 billion on
computer network security in 2011 (Johnson 2012) and the federal government is expected to be spending $10.5
billion per year by 2015 (Brito and Watkins 2012). This response is appropriate when data shows that the vast majority
of cyber-attacks are focused on espionage and the theft of intellectual property. It is not clear why
senior government officials and corporate executives focus on high-impact low-probability events and
engage in “alarmist rhetoric” (Brito and Watkins 2011) that skews the public perception of risk and
creates an atmosphere of fear. The danger of an inappropriate response in reaction to an
inflated threat and prevalence of misinformation is exemplified by the politicized intelligence that led to
the invasion of Iraq in 2003 (Brito and Watkins 2011). Understanding how information on the risk posed by cyber-attacks is poorly
communicated and the public reaction to an increased perception of risk – fear – is important in identifying when the perceived risk
is greater than the actual risk; when risk is more hype than threat. Critics of current cyber-security policy believe that threats are
being conflated; this results in a threat appearing larger than it is (Brito and Watkins 2012). In essence, a
wide variety of cyber-activity – political and social activity, criminal activity for profit, espionage, and offensive cyber-attack – are
treated as presenting the same level of threat. There is a wide divide between easily mounted and easily
defended denial of service attacks on public websites and high-potential cyber-weapons capable of severely
disrupting or destroying critical infrastructure (Rid and McBurney 2012). The rise of automated tools that
allow for low-level cyber-attacks to be easily mounted has caused a significant increase in the
number of cyber-attacks, a statistic often cited as proof of increased risk, but qualified cyber-
security organizations have discarded the number of cyber-attacks as a metric and consider it to
be meaningless as a method of assessing the scope and effects of cyber-attacks (Wilson 2005). Without
differentiating between generic malicious software and highly specialized and targeted offensive cyber-attacks, the
risk of cyber-attacks on critical infrastructure systems like the electrical grid cannot be properly assessed.

They’re biased and engage in constant risk inflation


Brito ’12, director – Technology Policy Program and senior research fellow – Mercatus Center @ George Mason University, and
Watkins, research associate – Technology Policy Program and State and Local Policy Project @ GMU, 2/14/

(Jerry and Tate, “Wired Opinion: Cyberwar Is the New Yellowcake,” Wired Magazine)

Yet evidence to sustain such dire warnings is conspicuously absent. In many respects, rhetoric about cyber catastrophe resembles
threat inflation we saw in the run-up to the Iraq War. And while Congress’ passing of comprehensive cybersecurity legislation
wouldn’t lead to war, it could saddle us with an expensive and overreaching cyber-industrial complex. In 2002 the Bush
administration sought to make the case that Iraq threatened its neighbors and the United States with weapons of mass destruction
(WMD). By framing the issue in terms of WMD, the administration conflated the threats of nuclear, biological, and chemical
weapons. The destructive power of biological and chemical weapons—while no doubt horrific—is minor compared to that of nuclear
detonation. Conflating these threats, however, allowed the administration to link the unlikely but serious threat of a nuclear attack
to the more likely but less serious threat posed by biological and chemical weapons. Similarly, proponents of regulation often
conflate cyber threats. In his 2010 bestseller Cyber War, Richard Clarke warns that a cyberattack today could
result in the collapse of the government’s classified and unclassified networks, the release of “lethal clouds
of chlorine gas” from chemical plants, refinery fires and explosions across the country, midair collisions of 737s,
train derailments, the destruction of major financial computer networks, suburban gas pipeline explosions, a nationwide power
blackout, and satellites in space spinning out of control. He assures us that “these are not hypotheticals.” But the only
verifiable evidence he presents relates to several well-known distributed denial of service (DDOS) attacks, and
he admits that DDOS is a “primitive” form of attack that would not pose a major threat to national
security. When Clarke ventures beyond DDOS attacks, his examples are easily debunked. To
show that the electrical grid is vulnerable , for example, he suggests that the Northeast power blackout of
2003 was caused in part by the “Blaster” worm. But the 2004 final report of the joint U.S.-
Canadian task force that investigated the blackout found that no virus, worm, or other malicious
software contributed to the power failure. Clarke also points to a 2007 blackout in Brazil, which he says was the
result of criminal hacking of the power system. Yet investigations have concluded that the power failure was
the result of soot deposits on high-voltage insulators on transmission lines. Clarke’s readers would no doubt be as
frightened at the prospect of a cyber attack as they might have been at the prospect of Iraq passing nuclear weapons to al Qaeda.
Yet evidencethat cyberattacks and cyberespionage are real and serious concerns is not
evidence that we face a grave risk of national catastrophe , just as evidence of chemical or biological weapons
is not evidence of the ability to launch a nuclear strike. The Bush administration claimed that Iraq was close to acquiring nuclear
weapons but provided no verifiable evidence. The evidence they did provide—Iraq’s alleged pursuit of uranium “yellowcake” from
Niger and its purchase of aluminum tubes allegedly meant for uranium enrichment centrifuges—was ultimately determined to be
unfounded. Despite the lack
of verifiable evidence to support the administration’s claims, the media
tended to report them unquestioned. Initial reporting on the aluminum tubes claim, for example, came in the form of
a front page New York Times article by Judith Miller and Michael Gordon that relied entirely on anonymous administration sources.
Appearing on Meet the Press the same day the story was published, Vice President Dick Cheney answered a question about
evidence of a reconstituted Iraqi nuclear program by stating that, while he couldn’t talk about classified information, The New York
Times was reporting that Iraq was seeking to acquire aluminum tubes to build a centrifuge. In essence, the Bush administration was
able to cite its own leak—with the added imprimatur of the Times—as a rationale for war. The media may be
contributing to threat inflation today by uncritically reporting alarmist views of potential cyber
threats. For example, a 2009 front page Wall Street Journal story reported that the U.S. power grid had been penetrated by
Chinese and Russian hackers and laced with logic bombs. The article is often cited as evidence that the power grid is rigged to blow.
Yet similar to Judith Miller’s Iraq WMD reporting, the
only sources for the article’s claim that infrastructure has been
compromised are anonymous U.S. intelligence officials. With little specificity about the alleged infiltrations,
readers are left with no way to verify the claims. More alarmingly, when Sen. Susan Collins (R-Maine) took to the
Senate floor to introduce the comprehensive cybersecurity bill that she co-authored with Sen. Joe Lieberman (I-Conn.), the evidence
she cited to support a pressing need for regulation included this very Wall Street Journal story. Washington
teems with
people who have a vested interest in conflating and inflating threats to our digital security.
The watchword, therefore, should be “trust but verify.” In his famous farewell address to the nation in 1961,
President Dwight Eisenhower warned against the dangers of what he called the “military-industrial complex”: an excessively close
nexus between the Pentagon, defense contractors, and elected officials that could lead to unnecessary expansion of the armed
forces, superfluous military spending, and a breakdown of checks and balances within the policy making process. Eisenhower’s
speech proved prescient. Cybersecurity
is a big and booming industry. The U.S. government is
expected to spend $10.5 billion a year on information security by 2015 , and analysts have estimated the
worldwide market to be as much as $140 billion a year. The Defense Department has said it is seeking more than $3.2 billion in
cybersecurity funding for 2012. Lockheed Martin, Boeing,
L-3 Communications, SAIC, and BAE Systems have all
launched cybersecurity divisions in recent years. Other traditional defense contractors, such as Northrop Grumman,
Raytheon, and ManTech International, have invested in information security products and services. We should be wary of proving
Eisenhower right again in the cyber sphere. Before enacting sweeping changes to counter cyber threats, policy
makers
should clear the air with some simple steps. Stop the apocalyptic rhetoric. The alarmist
scenarios dominating policy discourse may be good for the cybersecurity-industrial complex ,
but they aren’t doing real security any favors. Declassify evidence relating to cyber threats.
Overclassification is a widely acknowledged problem, and declassification would allow the
public to verify the threats rather than blindly trusting self-interested officials. Disentangle the
disparate dangers that have been lumped together under the “cybersecurity” label. This must be done to
determine who is best suited to address which threats. In cases of cybercrime and cyberespionage, for instance, private network
owners may be best suited and have the best incentives to protect their own valuable data, information, and reputations.

Their authors are paid off

Hersh ’10, contributor – The New Yorker, Pulitzer winner and 5-time George Polk Award winner, 11/1
(Seymour M, “The Online Threat,” The New Yorker Annals of National Security)

A great deal of money is at stake. Cyber security is a major growth industry, and warnings from Clarke,
McConnell, and others have helped to create what has become a military-cyber complex. The federal
government currently spends between six and seven billion dollars annually for unclassified cyber-security
work, and, it is estimated, an equal amount on the classified portion. In July, the Washington Post published a critical assessment
of the unchecked growth of government intelligence agencies and private contractors. Benjamin Powell, who served as general
counsel for three directors of the Office of National Intelligence, was quoted as saying of the cyber-security sector, “Sometimes
there was an unfortunate attitude of bring your knives, your guns, your fists, and be fully prepared to
defend your turf. . . . Because it’s funded, it’s hot and it’s sexy.” Clarke is the chairman of Good
Harbor Consulting, a strategic-planning firm that advises governments and companies on cyber security and
other issues. (He says that more than ninety per cent of his company’s revenue comes from non-cyber-related work.) McConnell
is now an executive vice-president of Booz Allen Hamilton, a major defense contractor. Two months
after McConnell testified before the Senate, Booz Allen Hamilton landed a thirty-four-million-dollar cyber contract. It included
fourteen million dollars to build a bunker for the Pentagon’s new Cyber Command. American intelligence and security
officials for the most part agree that the Chinese military, or, for that matter, an independent hacker, is
theoretically capable of creating a degree of chaos inside America. But I was told by military, technical, and
intelligence experts that these fears have been exaggerated , and are based on a fundamental confusion
between cyber espionage and cyber war. Cyber espionage is the science of covertly capturing e-mail
traffic, text messages, other electronic communications, and corporate data for the purpose of gathering national-security or
commercial intelligence. Cyber war involves the penetration of foreign networks for the purpose of
disrupting or dismantling those networks, and making them inoperable. (Some of those I spoke to made the point that
China had demonstrated its mastery of cyber espionage in the EP-3E incident, but it did not make overt use of it to wage cyber war.)
Blurring the distinction between cyber war and cyber espionage has been profitable for defense
contractors—and dispiriting for privacy advocates.

Basic risk calculus demands you throw out their impact

Clark, ’12 MA candidate – Intelligence Studies @ American Military University, senior analyst – Chenega Federal Systems,
4/28/(Paul, “The Risk of Disruption or Destruction of Critical U.S. Infrastructure by an Offensive Cyber Attack,” American Military
University)

The Department of Homeland Security measures risk as the product of a threat, vulnerability
to the threat, and seriousness of the consequences of a successful attack (Masse, ONeil and Rollins 2007). Using
this formula, the analysis of the threat posed by the Stuxnet offensive cyber-attack, the vulnerability
of the U.S. electrical grid, and the consequences of both the Stuxnet attack and disruption of the electrical grid
shows that there is a low probability risk that a cyber-attack could severely disrupt or destroy the
electrical component of critical sensitive infrastructure to the point that it would seriously degrade
national security economically or militarily.
---Cyber War- Countermeasures Solve

Countermeasures solve

Zenko and Cohen 12 (Micah Zenko, Fellow in the Center for Preventive Action at the Council on Foreign Relations, and
MIchael Cohen, Senior Fellow at the American Security Project, serves on the board of the National Security Network and has taught
at Columbia University’s School of International and Public Affairs, served in the U.S. Department of State, former Senior Vice
President at the strategic communications firm of Robinson, Lerer and Montgomery, bachelor’s degree in international relations
from American University and a master’s degree from Columbia University, 3/14/2012, "Clear and Present Safety",
yaleglobal.yale.edu/content/clear-and-present-safety)

A more recent bogeyman in national security debates is the threat of so-called cyberwar .
Policymakers and pundits have been warning for more than a decade about an imminent
“cyber–Pearl Harbor” or “cyber-9/11.” In June 2011, then Deputy Defense Secretary William Lynn said that “bits
and bytes can be as threatening as bullets and bombs.” And in September 2011, Admiral Mike Mullen, then
chairman of the Joint Chiefs of Staff, described cyberattacks as an “existential” threat that “actually can bring us
to our knees.” Although the potential vulnerability of private businesses and government agencies to cyberattacks has increased,
the alleged threat of cyberwarfare crumbles under scrutiny. No cyberattack has resulted in the loss
of a single U.S. citizen’s life. Reports of “kinetic-like” cyberattacks, such as one on an Illinois water plant
and a North Korean attack on U.S. government servers, have proved baseless. Pentagon networks are attacked
thousands of times a day by individuals and foreign intelligence agencies ; so, too, are servers
in the private sector. But the vast majority of these attacks fail wherever adequate safeguards
have been put in place. Certainly, none is even vaguely comparable to Pearl Harbor or 9/11, and
most can be offset by commonsense prevention and mitigation efforts.
---Cyber War- No China Threat

No Impact – Our Cyber Tech is better – the “Chinese threat” is all hype.

Ross, ‘9, Robert S. is a professor of political science at Boston College, an associate of the John King Fairbank Center for East
Asian Research at Harvard University and a fellow of the Security Studies Program at the Massachusetts Institute of Technology.
September 2009 - October 2009, (The National Interest, HEADLINE: Here Be Dragons, Robert S. Ross and Aaron L. Friedberg Debate:
Is China a Military Threat?, p. Lexis)

exaggerated assessments of this capability fail to


Beijing is also developing cyber-warfare techniques, but
evaluate China’s own emerging vulnerability to such attacks . Cyber-warfare technologies and skills are readily
accessible and U.S. advanced munitions are increasingly dependent on high-technology communication and surveillance
technologies. The United States is thus vulnerable to cyber attacks, and a Chinese cyber offensive against the United States could
the reciprocal effect of Washington’s cyber-warfare
influence U.S. operations in the western Pacific. Nonetheless,
capability on Beijing’s ability to wage high-technology warfare is equally significant . The same
advanced Chinese technologies and weaponry that pessimists argue present a major threat to U.S.
security, including ASBMs, are highly dependent on advanced communication and surveillance technologies
that are particularly vulnerable to U.S. cyber attacks. And once the United States degrades the
PLA’s advanced communication technologies, China would lose its high-technology asymmetric
capability that so alarms America’s pessimists, and it would be very susceptible to a wide range
of superior U.S. sea-based forces , even if the United States suffered from an effective Chinese
cyber attack.
The US is way ahead of China and Russia in cyberwar

Rid 12 (Thomas, PhD, Reader in War Studies @ King's College London, Non-Resident Fellow at the Center for Transatlantic
Relations in the School for Advanced International Studies at Johns Hopkins, "Think Again: Cyberwar," March/April, Foreign Policy,
http://www.foreignpolicy.com/articles/2012/02/27/cyberwar?page=0,0,

Yes, but not how you think. Russia and China are busy sharpening their cyberweapons and are already well
steeped in using them. The Russian military clandestinely crippled Estonia's economy in 2007 and Georgia's government and banks
in 2008. The People's Liberation Army's numerous Chinese cyberwarriors have long inserted "logic bombs" and "trapdoors" into
America's critical infrastructure, lying dormant and ready to wreak havoc on the country's grid and bourse in case of a crisis. Both
countries have access to technology, cash, and talent -- and have more room for malicious maneuvers than law-abiding Western
democracies poised to fight cyberwar with one hand tied behind their backs. Or
so the alarmists tell us. Reality looks
quite different. Stuxnet, by far the most sophisticated cyberattack on record, was most likely a U.S.-
Israeli operation. Yes, Russia and China have demonstrated significant skills in cyberespiona ge, but
the fierceness of Eastern cyberwarriors and their coded weaponry is almost certainly overrated . When it
comes to military-grade offensive attacks, America and Israel seem to be well ahead of the curve .
---Cyber War- No Nuclear Impact

Nuclear weapons are protected from hacking

Green ‘2 (Joshua Green, editor of Washington Monthly, November, 2002, The Myth of Cyberterrorism, Washington Monthly,)

"The government is miles ahead of the private sector when it comes to cybersecurity ," says
Michael Cheek, director of intelligence for iDefense , a Virginia-based computer security company with government and
private-sector clients. "Particularly the most sensitive military systems ." Serious effort and plain good fortune
have combined to bring this about. Take nuclear weapons. The biggest fallacy about their vulnerability ,
promoted in action thrillers like WarGames, is that they're designed for remote operation. "[The movie] is
premised on the assumption that there's a modem bank hanging on the side of the computer that
controls the missiles," says Martin Libicki, a defense analyst at the RAND Corporation. "I assure you, there
isn't." Rather, nuclear weapons and other sensitive military systems enjoy the most basic form of Internet
security: they're "air-gapped," meaning that they're not physically connected to the Internet
and are therefore inaccessible to outside hackers. (Nuclear weapons also contain "permissive action
links," mechanisms to prevent weapons from being armed without inputting codes carried by the president.) A
retired military official was somewhat indignant at the mere suggestion: "As a general principle, we've been looking at this thing for
20 years. What cave have you been living in if you haven't considered this [threat]?"
---Cyber War- Ev Comparison

No Impact – cyber attack fears are based in ignorance, profit-motivation, and political
manipulation

Green ‘2 (Joshua Green, editor of Washington Monthly, November, 2002, The Myth of Cyberterrorism, Washington Monthly,)

Why all this brooding over so relatively minor a threat? Ignorance is one reason. Cyberterrorism
merges two spheres--terrorism and technology--that most lawmakers and senior administration
officials don't fully understand and therefore tend to fear, making them likelier to accede to any
measure, if only out of self-preservation. Just as tellingly, many are eager to exploit this ignorance. Numerous
technology companies, still reeling from the collapse of the tech bubble, have recast themselves as innovators
crucial to national security and boosted their Washington presence in an effort to attract federal dollars. As Ohio State
University law professor Peter Swire explained to Mother Jones, " Many companies that rode the dot-com boom need to
find big new sources of income . One is direct sales to the federal government; another is federal mandates. If we
have a big federal push for new security spending, that could prop up the sagging market ." But
lately, a third motive has emerged: Stoking fears of cyberterrorism helps maintain the level of public
anxiety about terrorism generally,which in turn makes it easier for the administration to pass its agenda.
Deforestation Answers
Frontline

No impact or scenario for deforestation

Wigmore 5 – quoting biogeography professor at London University who edits the Journal of Biogeography and a Canadian co-
founder of Greenpeace (6/9, Barry, New York Post, Posted at Cheat Seeking Missiles, date is date of post,
http://cheatseekingmissiles.blogspot.com/2005/06/stop-global-whining-2.html)

"One of the simple, but very important, facts is that the rainforests have only been around for between 12,000
and 16,000 years. That sounds like a very long time but, in terms of the history of the earth, it's hardly a
pinprick. "Before then, there were hardly any rainforests. They are very young. It is just a big mistake that people are making.
"The simple point is that there are now still - despite what humans have done - more rainforests today than
there were 12,000 years ago." "This lungs of the earth business is nonsense; the daftest of all theories,"
Stott adds. "If you want to put forward something which, in a simple sense, shows you what's wrong with all the science they
espouse, it's that image of the lungs of the world. "In fact, because the trees fall down and decay, rainforests actually
take in slightly more oxygen than they give out. "The idea of them soaking up carbon dioxide and giving out oxygen
is a myth. It's only fast-growing young trees that actually take up carbon dioxide," Stott says. "In terms of world systems, the
rainforests are basically irrelevant. World weather is governed by the oceans - that great system of ocean
atmospherics. "Most things that happen on land are mere blips to the system, basically insignificant," he says. Both scientists say
the argument that the cure for cancer could be hidden in a rainforest plant or animal - while plausible - is
also based on false science because the sea holds more mysteries of life than the rainforests. And
both say fears that man is destroying this raw source of medicine are unfounded because the rainforests are remarkably
healthy. "They are just about the healthiest forests in the world. This stuff about them vanishing at an
alarming rate is a con based on bad science," Moore says.
Democracy Answers
Frontline
Can’t solve democracy

Haass 5 (Richard, President of the Council on Foreign Relations, Former Director of Policy Planning for
the State Department, Washington Post 1-24)

It is also difficult to spread democracy. It is one thing to oust a regime, quite another to put
something better in its place. Prolonged occupation of the sort the United States carried out in Japan and West
Germany after World War II is the only surefire way to build democratic institutions and instill democratic culture. But as Iraq
the rise of modern nationalism and modern methods of resistance means that
demonstrates,
such opportunities will be rare, costly and uncertain to succeed, despite an investment of billions of
dollars and thousands of lives. Prospects for the democratic improvement of a society can prove even worse absent occupation.
Tho8se who rejoiced 25 years ago in the overthrow of the shah of Iran should reflect on the fact that unattractive
regimes can be replaced by something far worse. We thus need to be measured in what pressures we place
on such countries as Saudi Arabia and Egypt. Here as elsewhere it is important to observe the Hippocratic oath and first do no
harm. Time is a factor in another sense. There
is no realistic way that democracy will arrive in either
North Korea or Iran before nuclear weapons do. And even if "freedom" were somehow to
come to Tehran, it is almost certain that free Iranians would be as enthusiastic as the
mullahs are about possessing nuclear weapons owing to the political popularity of these weapons and their
strategic rationale given Iran's neighborhood.

Democracy doesn’t prevent war

Goldstein, ’11 (Joshua, is professor emeritus of international relations at American University and author of Winning the War
on War: The Decline of Armed

Conflict Worldwide, Sept/Oct 2011, “Think Again: War. World peace could be closer than you think”, Foreign Policy)

"A More Democratic World Will Be a More Peaceful One." Not necessarily. The well-worn observation that real democracies almost
never fight each other is historically correct, but it's also true that democracies have
always been perfectly willing to fight
nondemocracies. In fact, democracy can heighten conflict by amplifying ethnic and nationalist
forces, pushing leaders to appease belligerent sentiment in order to stay in power. Thomas Paine and
Immanuel Kant both believed that selfish autocrats caused wars, whereas the common people, who bear the costs, would be loath
to fight. But trytelling that to the leaders of authoritarian China, who are struggling to hold in
check, not inflame, a popular undercurrent of nationalism against Japanese and American historical
enemies. Public opinion in tentatively democratic Egypt is far more hostile toward Israel than the
authoritarian government of Hosni Mubarak ever was (though being hostile and actually going to war are quite different
things). Why then do democracies limit their wars to non-democracies rather than fight each other? Nobody really knows As the
University of Chicago's Charles Lipson once quipped about the notion of a democratic peace, "We know it works in practice. Now we
have to see if it works in theory!" The best explanation is that of political scientists 9/29/2011 Think Again: War - By Joshua S.
Goldst… foreignpolicy.com/…/think_again_war?… 6/9Bruce Russett and John Oneal, who argue that three elements -- democracy,
economic interdependence (especially trade), and the growth of international organizations -- are mutually supportive of each other
and of peace within the community of democratic countries. Democratic leaders, then, see themselves as having
less to lose in going to war with autocracies.
Democracy doesn’t solve war their ev is based on flawed studies

Henderson ‘2 (Errol Henderson, Assistant Professor, Dept. of Political Science at the University of Florida, 2002, Democracy and
War The End of an Illusion?)

The replication and extension of Oneal and Russet (1997), which is one of the most important
studies on the DPP, showed that democracies are not significantly less likely to fight each
other. The results demonstrate that Oneal and Russet (1997) findings in support of the DPP
are not robust and that join democracy does not reduce the probability of international
conflict of pairs of states during the postwar era. Simple and straightforward modifications of
Oneal and Russett’s (1997) research design generate these dramatically contradictory results.
Specifically, by teasing out the separate impact of democracy and political distance (or political
dissimilarity) and by not coding cases of ongoing disputes as new cases of conflict , it became
clear that there is no siginifant relationship between join democracy and the likelihood of
international war or militarized interstate dispute (MID) for states during the postwar era. These
findings suggest that the post-Cold War strategy of “democratic enlargement,” which is aimed at ensuring peace by englaring the
community of democratic states, is quite a thin reed on which to rest a state’s foreign policy- much less the hope for international
peace.The results indicate that democracies are more war-prone than non-democracies (whether
democracy is coded dichotomously or continuously) and
that democracies are more likely to initiate
interstate wars. The findings are obtained from analyses that control for a host of political,
economic, and cultural factors that have been implicated in the onset of interstate war , and focus
explicitly on state level factors instead of simply inferring state level processes from dyadic level observations as was done in earlier
studies (e.g., Oneal and Russett, 1997; Oneal and Ray, 1997). The
results imply that democratic enlargement is
more likely to increase the probability of war for states since democracies are more likely to
become involved in—and to initiate—interstate wars.
---Democracy- Studies
Other variables account for peace—DPT is statistically flawed

Rosato 11
Sebastian, Dept of Political Science at Notre Dame. “The Handbook on the Political Economy of War”, Google Books

15.3.2 Militarized Disputes There are at least two reasons to doubt the claim that pairs of democracies are less prone to conflict
than other pairs of states. First, despite their assertions, it is not clear that democratic peace theorists have established the existence
of a powerful association between joint democracy and peace. Second, there is good evidence that factors other
than democracy -many of them consistent with realist expectations - account for the peace
among democratic states.14 Significance Democratic peace theorists have yet to provide clearcut
evidence that there is a significant relationship between their independent and dependent
variables, joint democracy and peace. It is now clear, for example, that Maoz and Russett's analysis of the Cold War period, which
claims to establish the existence of a joint, separate peace, does not in fact do so. In a reassessment of that analysis,
which follows the original as closely as possible save for the addition of a control for economic interdependence, Oneal ct al.
(1996) find that a continuous measure of democracy is not significantly correlated with peace.
Moreover, a supplementary analysis of contiguous dyads those that experience most of the
conflicts also finds no significant relationship between a continuous measure of joint democracy and
peace whenever a control for economic interdependence is included or not. This finding is
particularly damaging because democratic peace theorists argue that "most theoretical
explanations of the separate peace imply a continuous effect: the more democratic a pair of
states, the less likely they are to become involved in conflict " (Oneal and Ray 1997, p. 752). Oneal and Ray
(1997, pp. 756-7) conclude that the original Maoz and Russett finding does not survive reanalysis because it is
based on a joint democracy variable that, although widely used, is poorly calculated and constructed- and they
therefore propose a new democracy measure that they claim does achieve statistical significance. Their new measure of joint
democracy uses the democracy score of the less democratic state in a dyad on the assumption that conflict is a function of the
regime type of the less constrained of two interacting states. This "weak link" specification appears to provide powerful support for
the democratic peace finding: "As the less democratic state becomes more democratic, the likelihood of conflict declines. This is
clear evidence of the pacific benefits of democracy." The new variable provides "corroboration of the democratic peace" (Oneal and
Ray 1997, pp. 764-5). Oneal and Russett concur with this conclusion in a separate analysis that also uses the weak link assumption.
An increase in democracy in the state that is "freer lo resort to violence, reduces the likelihood of dyadic conflict" (Oneal and Russett
1997, p. 279). Although
the weak link measure is widely accepted as the gold standard in studies of
the relationship between democracy and a variety of international outcomes, it does not
provide evidence that joint democracy is significantly related lo peace . Even as they developed it, Oneal
and Ray admitted that the weak link was not a pure measure of joint democracy. What it really revealed was that the probability of
conflict was "a function of the average level of democracy in a dyad ... [and] also the political distance separating the states along
the democracy-autocracy continuum" (1997, p. 768, emphasis added). The problem, of course, is that the logics
advanced to explain the democratic peace refer to the effects of democracy on state behavior;
none refer to the effects of political similarity. Thus findings generated using the weak link
specification - which is to say all the major assessments of the democratic peace - may not
actually support the central democratic peace claim that it is something about the norms and
institutions of democracies that enables them to remain at peace. This is precisely the conclusion that Errol Henderson reaches
in his compelling assessment of Oneal and Russctt's work. His analysis replicates theirs precisely with two minor modifications: he
includes only the first year of any dispute because democratic peace theory is about the incidence of disputes, not their duration,
and he introduces a political similarity variable in order to disentangle the effects of joint democracy and political distance on
conflict. His central result is striking: democracy ;*is not significantly associated with the probability of dispute onset." "What
is
apparent from the results," he concludes, "is that in the light of quite reasonable, modest, and
straightforward modifications of Oneal and Russett's . . . research design, there is no statistically
significant relationship between joint democracy and a decreased likelihood of militarized
interstate conflict" (Henderson 2002, pp. 37-9). Mark Souva (2004) reaches essentially the same conclusion in an analysis of
the relationship between domestic institutions and interstate conflict using the weak link specification. In a model that includes
variables for political and economic institutional similarity, both of which are significantly associated with peace, there is no
significant relationship between joint democracy and the absence of conflict.

Their studies are flawed- they have research design problems and ignore both extrastate
conflict and third world states

Henderson ‘2 (Errol Henderson, Assistant Professor, Dept. of Political Science at the University of Florida, 2002, Democracy and
War The End of an Illusion?, p. 14-15

To my mind, the empirical evidence in support of both the dyadic and the nomadic DPP is problematic for
several reasons. The most recent studies alluded to earlier, which indicate that democracies are less likely to
fight each other and are more peaceful, in general, than non-democracies, are beset by research design
problems that severely hinder their reliability (e.g., Oneal and Russett, 1997; Oneal and Ray, 1997; Russett and
Oneal, 2001). For example, many of them rely on a questionable operationalization of joint democracy
that conflates the level of democracy of two states with their political dissimilarity . Only by teasing
out the effects of each factor are we in a position to confidently argue that shared democracy, rather than other factors, is actually
the
the motivating force driving democratic states toward their allegedly more peaceful international relations. In addition,
findings used to support monadic DPP claims also rely on questionable research designs that exclude
whole categories of international war—namely, extrastate wars, which are usually imperialist and colonial
wars. The exclusion of these wars from recent tests of the DPP leaves us unable to determine the actual applicability of the DPP to
the full range of international war. In addition, given that some scholars suggest that the DPP is applicable to civil wars (Krain and
it is important to determine to what extent we observe a “domes tic
Myers, 1997; Rummel, 1997),
democratic peace” for the most civil war prone states— the postcolonial, or third world, states. Previous
work has not tested the DPP for this specific group of states, and it is important that our research
design address this omission.
---Democracy- Transition

Turn – transition causes war.

Taner ‘2 (Taner—PhD Poli Sci at Syracuse—2002 (Binnur Ozkececi, Fall, Alternatives: Turkish Journal of International Relations,
Vol. 1, No. 3, p. 45, http://www.alternativesjournal.com/binnur.pdf)

More seriously, however, “democratic peace” theorists cannot adequately account for the tendency towards was in democratizing
countries, especially after the end of the Cold War. As it has been demonstrated many times since the late 1980s,
democratizing states are most often very volatile and dangerous and, thus more inclined to fight
wars “than are mature democracies or stable autocracies.”32 The “rocky transitional period” to
democratization may make countries more aggressive and war-prone due to not only domestic
competition but also utilization of nationalistic feelings by political leadership and mass public
support for aggression. If the “democratic peace” theorists would want to make their cases more persuasive, then, those
authors should be more attentive to what is going on in newly democratizing countries and modify, not necessarily change, some of
their propositions.
---Democracy- DPP Wrong

Democratic peace theory is wrong - no causality

Rosato 3 (Sebastian, PhD, Asst. Professor of Political Science at the University of Notre Dame, Former Research Fellow at the
International Security Program, “The Flawed Logic of Democratic Peace Theory,” Nov, American PoliSci Review Vol 97 No 4, Muse)

Democratic peace theory is probably the most powerful liberal contribution to the debate on the causes of war and peace. In
this paper I examine the causal logics that underpin the theory to determine whether they offer compelling explanations for the
finding of mutual democratic pacifism. I find that they do not. Democracies do not reliably externalize their
domestic norms of conflict resolution and do not trust or respect one another when their
interests clash. Moreover, elected leaders are not especially accountable to peace loving publics or
pacific interest groups, democracies are not particularly slow to mobilize or incapable of surprise
attack, and open political competition does not guarantee that a democracy will reveal private information about its level of
resolve thereby avoiding conflict. Since the evidence suggests that the logics do not operate as stipulated by the theory's
proponents, there are good reasons to believe that while there is certainly peace among democracies, it may not
be caused by the democratic nature of those states.
---Democracy- Empirics

Empirics flow neg/aff - democracy does NOT prevent conflict or increase cooperation

Rosato 3 (Sebastian, PhD, Asst. Professor of Political Science at the University of Notre Dame, Former Research Fellow at the
International Security Program, “The Flawed Logic of Democratic Peace Theory,” Nov, American PoliSci Review Vol 97 No 4, Muse)

The imperialism of Europe's great powers between 1815 and 1975 provides good evidence that liberal
Although
democracies have often waged war for reasons othe than self-defense and the inculcation of liberal values.
there were only a handful of liberal democracies in the international system during this period, they were
involved in 66 of the 108 wars listed in the Cor- relates of War (COW) dataset of extrasystemic wars (Singer
and Small 1994). Of these 66 wars, 33 were "im- perial," fought against previously independent peoples, and 33 were
Several
"colonial," waged against existing colonies. It is hard to justify the "imperial" wars in terms of self-defense.
cases are clear-cut: The democracy faced no immediate threat and conquered simply
for profit or to expand its sphere of influence. A second set of cases includes wars waged as a result of imperial
competition: Liberal democracies conquered non-European peoples in order to create buffer states against other empires or
to establish control over them before another imperial power could move in. Thus Britain tried to conquer Afghanistan
(1838) in order to create a buffer state against Russia, and France in- vaded Tunisia (1881) for fear of an eventual Italian
occupation. Some commentators describe these wars as defensive because they aimed to secure sources of overseas
wealth, thereby enhancing national power at the expense of other European powers. There are three reasons to dispute this
assessment. First, these wars were often preventive rather than defensive : Russia had made no
move to occupy Afghanistan and Italy had taken no action in Tunisia. A war designed to avert possible action in the future,
but for which there is no current evidence, is not defensive. Second, there was frequently a liberal
alternative to war. Rather than impose authoritarian rule, liberal great powers could have offered non-European
peoples military assistance in case of attack or simply deterred other imperial powers. Finally, a
substantial
number of the preventive occupations were a product of competition between Britain and
France, two liberal democracies that should have trusted one another and negotiated in good
faith without compromising the rights of non-Europeans if democratic peace theory is correct. A third set of cases
includes wars waged directly against non-Europeans whose territory bordered the European empires. Because non-
Europeans some- times initiated these wars contemporaries tended to justify them as defensive wars of "pacification" to
pro- tect existing imperial possessions. Again, there are good reasons to doubt the claim that such wars were defensive.
In the first place, non-Europeans often at- tacked to prevent further encroachment on their lands; it was they and not the
Europeans that were fighting in self-defense. Moreover, there is considerable evidence that the imperial powers often
provoked the attacks or acted preventively and exploited local instabilities as a pretext for imposing control on the periphery
of their empires (Table 1). Nor were any of the extrasystemic wars fought to prevent egregious abuses of human rights
or with the express purpose of replacing autocratic rule with a more liberal alternative. The "colonial" wars, by defini- tion,
were conflicts in which imperial powers sought to perpetuate or reimpose autocratic rule. The "imperial" wars simply replaced
illiberal indigenous governmen with authoritarian rule. When imperial rule was not imposed directly, the European
powers supported lo- cal elites but retained strict control over their actions, thereby underwriting unjust political systems
and ef- fectively implementing external rule. In short, despite protestations that they were bearing the "white man's
burden," there is little evidence that liberal states' use of force was motivated by respect for human rights or that
There are, then, several examples of liberal
imperial conquest enhanced the rights of non- Europeans.s
states violating liberal norms in their conduct of foreign policy and therefore the claim that liberal
states generally externalize their internal norms of conflict resolution is open to question. Proponents of the democratic
peace have down- played the importance of these findings in three ways. First, they have restated their argument and
claimed that democracies remain at peace because they trust and respect one other and fight nondemocracies be-
cause they neither trust nor respect them. As Doyle (1997, 32) notes, "Extreme lack of public respect or trust is one of
the major features that distinguishes re- lations between liberal and nonliberal societies from relations among liberal
societies." According to this re- statement, we should not be surprised to observe Euro- pean democracies fighting non-
Europeans and the nor- mative logic can therefore accommodate the imperial evidence. This alternative presentation of
the logic is, however, ad hoc. A more satisfying logic, and the one put forward by most democratic peace theorists, is more
complex: Democracies rarely fight each other because they trust and respect one another, and they are able to do so
because they know that their democratic coun- terparts will act on the basis of democratic norms, that is, they will only
fight in self-defense or to democra- tize others. The key is that democracies must reliably
to this logic
externalize democratic norms. If they do, then trust and respect will prevail; if they do not, then we
cannot be confident that peace will obtain between them . The history of imperialism suggests
that they do not and therefore casts doubt on the normative logic's explanatory power. Second, democratic peace
theorists have claimed that Britain, France, and the United States were not sufficiently liberal in the period under review
and thus cannot be expected to reliably externalize their internal norms (e.g., Rawls 1999, 53-54). If this claim is true, the
normative logic cannot tell us a great deal about inter- national politics. Britain, France, and the United States are generally
considered to be classic liberal democra- cies; if they cannot be expected to behave in a liberal fashion, then few, if any,
states can. Finally, democratic peace theorists assert that they do not claim that liberal norms are the sole determinant of
decisions for war; factors such as power and contiguity matter as well (e.g., Russett 1995). This defense would be convincing
if I were claiming that liberal norms were not the only factors that went into decision making or that they were not as
important in the decision making process as other factors. However, the claim made here is quite different: Liberal states
have consistently violated liberal norms when deciding to go to war . It is not that liberal norms
only matter a little; they have often made no difference at all. In sum, there are good reasons to believe that one of the
normative logic's key causal mechanisms does not operate as advertised. Liberal democratic great powers have frequently
violated liberal norms in their deci- sions for war, thereby casting doubt on the claim that democracies generally
externalize their internal norms of conflict resolution.
---Democracy- Middle East

Democratic Peace Theory is empirically denied in the Middle East

Neuberger ’11 (professor of political science, Benny Neuberger, 11/2/2011, “Between Democracy and Peace”, Haaretz)

The claim that democracies are less inclined to war than other types of government is invalid
in the Middle East. The Egyptian masses calling for the establishment of a democratic regime have brought the link
between peace and democracy into the public debate. While opposition leader Mohamed ElBaradei sounds a reassuring note by
saying the peace with Israel is not in jeopardy because democracies don't go to war against democracies, the Muslim Brotherhood
speaks of a "democratic" referendum to decide the fate of the peace agreement and could lead to its annulment. Indeed, some
predict that democratization would return Egypt to the cycle of war. The argument over the connection between democracy and
peace has been going on for 20 years. Hundreds of articles have been written about the two versions of the democratic peace
theory. The first holds that democracies do not go to war against democracies. The second goes even further, to claim that
democratic regimes are less likely than others to engage in war even against dictatorships. The theories assume that the devolution
of political power characteristic of democracies leaves time for negotiations and compromise between the hawks, and that
democratic rulers are by nature less aggressive than others. Above all, the democratic peace theory, also called the theory of
democratic pacifism, is based on Immanuel Kant's "perpetual peace." The 19th-century philosopher argued that nations seek peace
and do not want to pay the price of war and that as a result democratic rulers, who are responsible to their peoples, are disinclined
to initiate war. In the Middle East there have never been democracies on both sides of the barricades. Nevertheless there are many
There is no historical evidence for the idea (which
signs that in its most literal form the theory doesn't work here.
is congruent with the democratic peace theory and which was popular in pre-state Israel and
after independence ) that the source of the Arab-Israeli conflict is the absence of democracy on
the Arab side. There is no evidence that the Arab nations wanted peace and it was only the kings and
generals who wanted war. Furthermore, the peace agreements with Egypt and with Jordan
were not the result of democratic public opinion; rather, they came from leaders who were not
democratic but who chose the road of peace. The Kantian theory of warmongering elites and
peace-loving masses has not proved itself in Israel either. The important decisions in favor of
peace did not stem from popular pressure. This is true in regard to the withdrawal from
Sinai, as part of the peace treaty with Egypt, and the 2005 disengagement from the Gaza Strip . The
decisions to recognize the Palestinian people and the Palestine Liberation Organization, as part of the Oslo Accords; the proposal to
divide Jerusalem, at the 2000 Camp David summit; and the acceptance of the "two states for two peoples" solution, were all taken
at the leadership level and were not the result of "democratic" popular pressure. We have seen courageous decisions toward peace
on the Arab side (Anwar Sadat, Hosni Mubarak, King Hussein, Mahmoud Abbas ) and the Israeli (Menachem Begin, Yitzhak Rabin,
Ariel Sharon ). But we have also had leaders (Shimon Peres, Ehud Barak, Benjamin Netanyahu ) who did not go all the way for peace
out of fear of nationalistic public opinion. This "democratic fear" of making peace is an example of the failure of the argument that
the people always want peace. Israel's wars of choice - the 1956 Sinai Campaign and the 1982 and 2006 Lebanon wars - also
undermine the claim that democracies do not tend to initiate war, as do the wars fought by the United States in Korea, Vietnam,
Panama, Granada, Iraq and Afghanistan. A key term that is absent from the theory of democratic pacifism, but extremely relevant to
Egypt's current situation, is "democratization" - the transition period from dictatorship to democracy. Whereas in entrenched
democracies, such as in Scandinavia, the leadership and the public are indeed generally moderate, in partial democracies
(and in regimes undergoing democratization ) there is sometimes unbridled nationalistic
competition among the elites, who believe this is the way to win democratic elections. Party and
government institutions are too weak to avoid the temptation of nationalistic incitement.
It is no accident that it was precisely during the era of democratization that bloody wars broke
out in the former Soviet Union and the former Yugoslavia. Israel's right wing has no reason to
rejoice in its clairvoyance if the peace with Egypt stumbles as a result of democratization in that
country. It was the right, with Netanyahu leading the way, who argued over and over that peace
had no chance as long as the Arab states were not democratic. This was a convenient excuse not
to pay the price for peace. It is possible that had the Netanyahu government done more to
advance peace with the Palestinians and the Syrians it would have significantly decreased the
threats posed by a more democratic Egypt. The theory of democratic peace does not hold water in the Middle East
in an era of democratization. Peace and democracy are the basic values of any humane society but the link between them is tangled
and complex, exactly like the link between freedom and equality.
Deterrence Theory Answers
Frontline

Deterrence theory is wrong

Wellen 13
(Russ, Foreign Policy in Focus, think tank, book review, “Ability of Nuclear Deterrence to Defuse Crises Exaggerated,” 3/8/13,
http://www.fpif.org/blog/ability_of_nuclear_deterrence_to_defuse_crises_exaggerated?
utm_source=feedburner&utm_medium=feed&utm_campaign=Feed%3A+FPIF+%28Foreign+Policy+In+Focus+%28All+News%29%29)

During the Cold War, nuclear deterrence failed to thwart crises, which were subsequently solved with good, old politics. We may
owe thanks for the absence of war (other than proxy) during the Long Peace -- aka the Cold War -- between the United
States and the Soviet Union less to nuclear deterrence, as is commonly assumed, than to the
"underlying politics." That's a thesis beginning to gain credibility which Francis J. Gavin presents as well as anyone (though
I've just begun the book) in Nuclear Statecraft: History and Strategy in America's Atomic Age (Cornell University Press, 2012).
Theories about nuclear weapons, he writes (my additions bracketed): … were based on a certain view of the world: that the
international system was no longer solely driven by geopolitical competition between the great states. While these drives still
mattered, international relations were now shaped by the existence of and interaction between rival nuclear forces. The weapons
themselves -- their lethality, their numbers, their deployments -- drove the politics, not the other way around. The interaction could
produce outcomes -- arms races, dangerous crises, and even inadvertent war -- separate from the political sources of the rivalry.
These theories implied that the most effective policy might not be focusing on the underlying political dispute between rivals but to
control their [nuclear] weapons and their interactions. [In part, it] meant that mutual efforts had to be made to limit dangers and to
negotiate, not about the core geopolitical issues driving the dispute, but control of the weapons themselves. "This is an
extraordinary way of viewing international relations," Gavin continues. But, he asks, "does it accurately reflect the way the world
works?" He then attempts to answer his own question. (Emphasis added.) It is interesting to reflect on how rarely
the ups and downs of the superpower geopolitical competition mirrored the movements of the arms
race. The Soviets pushed the United States aggressively on the issue of West Germany's military status by threatening West
Berlin's viability at a time when the USSR was not only weak but potentially open to a US first strike in the late 1950s and early
1960s. The Soviets left West Berlin alone after 1962, even as the US nuclear superiority that arguably helped protect the city
disappeared. Why? Because the core geopolitical questions surrounding West Germany's military and political status were resolved,
largely to the Soviet Union's satisfaction. In fact, it is very hard to find any evidence that … the Soviets ever considered launching a
"bolt from the blue" against the United States. Ward
Wilson also approached the failure of deterrence in
the Berlin crisis of 1948. In his book, Five Myths About Nuclear Weapons (Houghton Mifflin
Harcourt, 2013), about which we recently posted, he writes: Historians debate whether the redeployment of
[nuclear weapons-capable] B-29s to England successfully deterred the Soviets. But few ask how Stalin could have initiated the crisis
in the first place. When he ordered access to Berlin cut off, the United States had a monopoly on nuclear weapons. (the Soviet Union
would not explode its first nuclear weapons for another year). Cutting off access to Berlin carried with it a significant risk of war.
Where two large armed groups confront each other in a narrow space, there is always the possibility of accidental escalation. Or
escalation could have been intentional. One of the options considered by Washington during the crisis was sending an armored
column to force its way up the autobahn to Berlin.
Given the risk of provoking a nuclear war and the U.S.
nuclear monopoly, why wasn't Stalin deterred from initiating the blockade? If the risk of
nuclear war deters, why did Stalin start a crisis that could have led to the use of nuclear
weapons against his country? In other words, politics often proceed independently of
considerations of the threat of nuclear attacks. Meanwhile, far from lending clarity to
international relations, nuclear deterrence just creates another obstacle and adds another
layer of complexity to world peace.

See Nuclear Primacy Answers


Disease Answers
Frontline

Extinction impossible and ahistorical

Posner 5 (Richard A., Judge U.S. Court of Appeals 7th Circuit, Professor Chicago School of Law, January 1, 2005, Skeptic, Altadena,
CA, Catastrophe: Risk and Response, http://goliath.ecnext.com/coms2/gi_0199-4150331/Catastrophe-the-dozen-most-
significant.html#abstract)

Yet the fact that Homo sapiens has managed to survive every disease to assail it in the 200,000
years or so of its existence is a source of genuine comfort, at least if the focus is on extinction
events. There have been enormously destructive plagues , such as the Black Death, smallpox, and
now AIDS, but none has come close to destroying the entire human race. There is a biological reason. Natural
selection favors germs of limited lethality; they are fitter in an evolutionary sense because their
genes are more likely to be spread if the germs do not kill their hosts too quickly. The AIDS virus is an
example of a lethal virus, wholly natural, that by lying dormant yet infectious in its host for years maximizes its spread. Yet there
is no danger that AIDS will destroy the entire human race. The likelihood of a natural pandemic that
would cause the extinction of the human race is probably even less today than in the past (except
in prehistoric times, when people lived in small, scattered bands, which would have limited the spread of disease), despite
wider human contacts that make it more difficult to localize an infectious disease. The reason is improvements in medical
science. But the comfort is a small one. Pandemics can still impose enormous losses and resist prevention and cure: the lesson of the
AIDS pandemic. And there is always a lust time. That the human race has not yet been destroyed by germs created or made more
lethal by modern science, as distinct from completely natural disease agents such as the flu and AIDS viruses, is even less reassuring.
We haven't had these products long enough to be able to infer survivability from our experience with them. A recent study suggests
that as immunity to smallpox declines because people am no longer being vaccinated against it, monkeypox may evolve into "a
successful human pathogen," (9) yet one that vaccination against smallpox would provide at least some protection against; and even
before the discovery of the smallpox vaccine, smallpox did not wipe out the human race. What is new is the possibility that science,
bypassing evolution, will enable monkeypox to be "juiced up" through gene splicing into a far more lethal pathogen than smallpox
ever was.

Extinction claims are hype

Fitzpatrick ’10 Michael Fitzpatrick, General Practitioner @ Barton House Health Center, November 2010. “Pandemic Flu: Public
Health and the Culture of Fear”

http://www.rsis.edu.sg/NTS/resources/research_papers/NTS%20Working%20Paper2.pdf

Projections by leading public health officials of rates of disease and death from pandemic flu on a
catastrophic scale had a major impact. While WHO experts such as Keiji Fukuda speculated that global death rates would be in
the millions, if not tens of millions, television reports featured images of the 1918-19 pandemic and accounts of the devastating
effects of that (historically unprecedented) viral pestilence.10 Patients fearful for their own healtn and that of their children, their
elderly relatives, and family members with chronic illnesses sought medical advice and whatever preventative measures were
available. There is however little evidence that raising awareness of the emerging threat of swine flu had any protective value. Given
the rapid spread of the virus, it appears than none of the measures taken in the early 'containment' phase of the outbreak, such as
more assiduous hand-washing, face masks, social distancing measures (school closures, etc.) and the provision of prophylactic
antivirals to contacts had an appreciable effect on its spread. Pregnant women, deemed to be particularly at risk, were particularly
susceptible to pandemic fears - and their anxieties were subsequently compounded by the development of vaccines that rival
scaremongers claimed were unsafe. It soon emerged that early reports from Mexico provided unreliable figures for deaths resulting
from swine flu and an uncertain number of cases of infection to use as a denominator with which to calculate the mortality rate. As
it also became clear that most cases were mild, projections for the impact of the pandemic were
steadily scaled down." In July, British authorities anticipated that 30 per cent of the population (19 million people) would
become infected, with a complication rate of 15 per cent, a hospitalisation rate of 2 per cent and a death rate between 0.1 per cent
and 0.35 per cent (between 19,000 and 65,000 people). By September the figure of 19,000 had become the worst-case scenario; the
following month this was reduced to 1,000. In December, the official report on the mortality statistics for the first six months of the
pandemic in England estimated a mortality rate of 0.026 per cent (138 confirmed deaths, and cases of swine flu in 1 per cent of the
population), a rate substantially lower than the most optimistic scenario of six months earlier.12 The contrast with earlier influenza
pandemics was dramatic: the death rate in 1918-19 was 2-3 per cent, and that in the less severe pandemics of 1957-58 and 1967-68
around 0.2 per cent. In the judgement of the Hine Report, ministers and officials placed excessive faith in mathematical
modelling. Theyhad come to regard this as 'hard, quantitative science' that could provide 'easily
understandable figures' which had the aura of appearing 'scientifically very robust'.13 Though the
mathematicians had warned, at the first pandemic planning meeting in April, that in the absence
of reliable data their modelling capability was low, they were under pressure from the
politicians to 'produce forecasts'. The high level of uncertainty surrounding these projections
does not seem to have deterred the modellers from producing them or the politicians from
projecting them into the public realm. The Hine Report observes that by the end of the first wave of swine flu cases in
September, sufficient data were available to guarantee accurate modelling of the second wave. However, official statements still
sought to warn against complacency about future dangers and did nothing to allay the anxieties provoked by earlier doomsday
scenarios. The Hine Report is critical of the public promotion of 'reasonable worst-case scenarios', which imply 'a reasonably likely
event', focusing in particular on CMO Professor Liam Donaldson's July statement. The report says: The English CMO's citing of the
'reasonable worst-case' planning assumption of 65,000 fatalities on 16 July 2009 was widely reported in headlines in somewhat
alarmist terms.14 It seems unfair to blame the media for the alarmist tone of their reports, when it was echoed by the newly
appointed health minister Andy Bumham, who told parliament that the swine flu pandemic could no longer be controlled and that
there could be 100,000 cases a day by the end of August. It is striking that British authorities chose to promote such gloomy
projections at a time when other prominent health figures had already declared such figures improbable. A month earlier, on the
occasion of declaring the swine flu outbreak a global pandemic, WHO chief Margaret Chan had already recognised that most cases
were mild and that she did not expect to see a sudden and dramatic jump in severe or fatal infections.15 While the Hine Report is
generally highly congratulatory of the UK response to the swine flu pandemic,
it suggests that the authorities may
have adhered too strictly to the contingency plan they had developed over the previous decade
to cope with the emergence of an influenza pandemic on the scale of the 1918-19 outbreak. As a result they 'did
not consider sufficiently the possibility that a pandemic might be far less severe' than the one
envisioned in that contingency plan. Their response was 'tailored to the plan, not the nature of the virus' and thus
lacked flexibility. The report tentatively suggests that the authorities might consider as an alternative approach, a policy of preparing
for the most likely outcome, while being prepared to monitor and change tack as necessary. The
alarmist response to the
swine flu outbreak reflects the wider trend of the past decade in which 'crying wolf has emerged as
the appropriate official response to diverse real and imaginary threats, from the millennium bug
to bioterrorism, obesity to global warming.'5 For the authorities, the over-riding principle is to avoid
blame for unforeseen disasters, by always proclaiming the worst-case scenario and repeating
the mantra 'prepare for the worst, hope for the best'. From this perspective, rational contingency
planning gives way to scaremongering. Instead of making discreet preparations for probable,
predictable emergencies (snow in winter, drought in summer), the authorities engage in speculation about
the grimmest possible eventualities (massive loss of life resulting from disease or climate change) with the
aim of promoting more responsible behaviour and healthier lifestyles .17 Rather than
communicating realistic assessments of risk to the public, the authorities engage in sharing their
anxieties and promoting fears. Instead of guiding practical professional interventions in
response to real social problems, politicians and public health officials engage in dramatic
posturing.
Bad methodology is the cause of fears that virus’s are getting more severe

Garske ‘9 (Garske T, Legrand J, Donnelly CA, et al. Assessing the severity of the novel influenza A/H1N1 pandemic. BMJ. 2009;
339: b2840. Friday 17 July 2009 00.00 BST

Another issue is the time delay between becoming ill and dying. Early in an epidemic, people may have caught
the virus, but their illness won't have had time to run its course. Put bluntly, at a particular point in time there will be
people who are going to die, but who are still alive at the time statistics are calculated.
Mathematically, this type of bias is called censoring. Censoring tends to have its strongest effect early
in a pandemic when a disease is spreading faster and faster. It has the effect of making the illness look
milder at first, then more serious later as the first wave of patients either die or recover. This happened in
the SARS epidemic, and caused unwarranted fears that the virus was becoming more severe.

War increases disease

Kassalow ‘1, Jordan S. Council on Foreign Relations, May 2001, “Why Health is Important to US Foreign Policy,”
http://www.milbank.org/reports/Foreignpolicy.html

A fourth way in which health affects the international system is through the direct links between
health and war. The link from war to health is clearer: wars kill and injure soldiers and civilians,
but they also destroy infrastructure and social structures, in both cases with adverse effects on
the population's general health. In the eastern Democratic Republic of Congo, for example, war
and ill health are tightly entwined. Of 1.7 million excess deaths between August 1998 and May
2000, only 200,000 were attributable to acts of violence, and wherever the war worsened,
infectious disease and malnutrition followed (International Rescue Committee 2000). Medical
facilities are often singled out for attack in "new wars" because they provide valuable loot, easy
victims, and a way to demoralize civilian populations. War also causes exceptional mobility, and
armies, peacekeepers, and refugees act as vectors for the transmission of disease.

Antibiotic use doesn’t spike resistance

Thomas ‘8, M.E., et al., 08 ("Risk factors for the introduction of high pathogenicity Avian Influenza virus into poultry farms
during the epidemic in the Netherlands in 2003." Preventive veterinary medicine 69, (10 June 2005): 1-11. Agricola. EBSCO. [1 Aug.
2008, http://search.ebscohost.com/login.aspx?direct=true&db=agr&AN=IND43716042&site=ehost-live]

With the exception of ciprofloxacin resistance, there


is a paucity of scientific evidence to document the
association of antimicrobial agents used in veterinary medicine with increases in antimicrobial-
resistant pathogens (Phillips et al., 2004). For example, it has been suggested that the increased
prevalence of extended-spectrum cephalosporin-resistant strains is in part related to the use in food animals of
ceftiofur, which is an extended-spectrum cephalosporin approved for use in veterinary medicine (White et al., 2001);
however, scientific evidence is lacking. Antimicrobial agents used for intensive calf rearing in the 1970–1980s have
also been speculated to contribute to the emergence of multiple-antibiotic resistant Salmonella Typhimurium DT104 strains. Genes
included in the antibiotic resistance gene cluster of Salmonella Typhimurium DT104 confer resistance to four of the five
antimicrobials used during that time to treat veal calves, therefore co-selection of the entire cluster could have arisen from the use
of any one of those drugs (Velge, Cloeckaert, & Barrow, 2005). While there is no definitive evidence for this scenario, several reviews
have been published presenting contrasting views regarding the role of veterinary usage of antimicrobials in the emergence of
antibiotic-resistant foodborne pathogens. In support of a causal relationship are reviews by Angulo et al., 2004 and Mølbak, 2004,
whereas reviews by Phillips et al., 2004 and Wassenaar, 2005 advocate that veterinary usage of antimicrobial agents
are inaccurately incriminated as being a major contributor to antibiotic-resistant pathogens in
humans. Debate on this topic will continue but should consider the additional routes which lead to resistant bacterial
populations, that antimicrobial usage in animals is required for animal health and well-being, and that not every antimicrobial-
resistant pathogen has human health consequences. On this latter point, clearly not all infections caused by resistant pathogens fail
to respond to treatment. For example, in a study of 23 diarrhea cases in Thailand, nearly all were infected with ciprofloxacin-
resistant Campylobacter, yet 58% of patients receiving ciprofloxaxin treatment were cured. This response implies that treatment
with ciprofloxacin could still be effective in many cases (Sanders et al., 2002). Another consideration is that acquisition of drug
resistance could entail a biological cost to the pathogen resulting in reduced fitness and competitiveness in the absence of antibiotic
selection pressure. For example, most data on E. coli suggest that increased antibiotic resistance results in decreased fitness
(Wassenaar, 2005). Alternatively, for some foodborne pathogens such as fluoroquinolone-resistant C. jejuni, resistance can be
neutral or even beneficial in terms of fitness (Luo et al., 2005). When coinoculated into chickens, fluoroquinolone-resistant
Campylobacter isolates either outcompeted or were outcompeted by most of the fluoroquinolone-susceptible strains, with the
outcome being dependent on the genetic background of the recipient strain. These variable results
highlight the complex
nature of antibiotic resistance and the large data gaps that exist in making informed scientific
decisions on use of antimicrobials in animals used for food.
---Disease- No Extinction

No extinction

Gladwell ‘95, Malcolm writer for The New Yorker and best-selling author The New Republic,
July 17 and 24, 1995, excerpted in Epidemics: Opposing Viewpoints, 1999, p. 31-32

Every infectious agent that has ever plagued humanity has had to adapt a specific strategy but
every strategy carries a corresponding cost and this makes human counterattack possible.
Malaria is vicious and deadly but it relies on mosquitoes to spread from one human to the next,
which means that draining swamps and putting up mosquito netting can all hut halt endemic
malaria. Smallpox is extraordinarily durable remaining infectious in the environment for years,
but its very durability its essential rigidity is what makes it one of the easiest microbes to create
a vaccine against. AIDS is almost invariably lethal because it attacks the body at its point of great
vulnerability, that is, the immune system, but the fact that it targets blood cells is what makes it
so relatively uninfectious. Viruses are not superhuman. I could go on, but the point is obvious.
Any microbe capable of wiping us all out would have to be everything at once : as contagious as
flue, as durable as the cold, as lethal as Ebola, as stealthy as HIV and so doggedly resistant to
mutation that it would stay deadly over the course of a long epidemic. But viruses are not, well,
superhuman. They cannot do everything at once . It is one of the ironies of the analysis of
alarmists such as Preston that they are all too willing to point out the limitations of human
beings, but they neglect to point out the limitations of microscopic life forms.

Diseases are short term – They evolve to be benign and don't cause extinction

AMNH 98 – (The American Museum of Natural History “How did Hyperdisease cause extinctions?”
http://www.amnh.org/science/biodiversity/extinction/Day1/disease/Bit2.html)

It is well known that lethal diseases can have a profound effect on species' population size and
structure. However, it is generally accepted that the principal populational effects of disease are
acute--that is, short-term. In other words, although a species many suffer substantial loss from the
effects of a given highly infectious disease at a given time , the facts indicate that natural populations
tend to bounce back after the period of high losses. Thus, disease as a primary cause of
extinction seems implausible. However, this is the normal case, where the disease-provoking pathogen and its host have
had a long relationship. Ordinarily, it is not in the pathogens interest to rapidly kill off large numbers of
individuals in its host species, because that might imperil its own survival. Disease theorists long
ago expressed the idea that pathogens tend to evolve toward a "benign" state of affairs with
their hosts, which means in practice that they continue to infect, but tend not to kill (or at least
not rapidly). A very good reason for suspecting this to be an accurate view of pathogen-host relationships is that individuals with
few or no genetic defenses against a particular pathogen will be maintained within the host population, thus ensuring the
pathogen's ultimate survival.
No disease is powerful enough or could mutate to cause extinction

Gladwell 99 (Malcolm, The New Republic, July 17 and 24, 1995, excerpted in Epidemics: Opposing Viewpoints, p. 31-32)

Every infectious agent that has ever plagued humanity has had to adapt a specific strategy but every strategy carries a
corresponding cost and this makes human counterattack possible. Malaria is vicious and deadly but it relies
on mosquitoes to spread from one human to the next, which means that draining swamps and putting up mosquito netting can all
hut halt endemic malaria. Smallpox is extraordinarily durable remaining infectious in the environment for years, but its very
durability its essential rigidity is what makes it one of the easiest microbes to create a vaccine against. AIDS is almost invariably
lethal because it attacks the body at its point of great vulnerability, that is, the immune system, but the fact that it targets blood cells
is what makes it so relatively uninfectious. Viruses are not superhuman. I could go on, but the point is obvious. Any
microbe
capable of wiping us all out would have to be everything at once: as contagious as flue, as
durable as the cold, as lethal as Ebola, as stealthy as HIV and so doggedly resistant to
mutation that it would stay deadly over the course of a long epidemic. But viruses are not, well,
superhuman. They cannot do everything at once. It is one of the ironies of the analysis of alarmists such as
Preston that they are all too willing to point out the limitations of human beings, but they neglect to point out the
limitations of microscopic life forms.

Any mutation will lower the risk

MacPhee and Marx 98 (Ross, American Museum of Natural History and Aaron Diamond, AIDS Research Facility and
Tulane University, “How Did Hyperdisease Cause Extinctions?”,
http://www.amnh.org/science/biodiversity/extinction/Day1/disease/Bit2.html)

It is well known that lethal diseases can have a profound effect on species' population size and structure. However, it is
generally accepted that the principal populational effects of disease are acute--that is, short-term. In
other words, although a species many suffer substantial loss from the effects of a given highly infectious disease at a given time,
the facts indicate that natural
populations tend to bounce back after the period of high losses. Thus,
disease as a primary cause of extinction seems implausible. However, this is the normal case, where the
disease-provoking pathogen and its host have had a long relationship. Ordinarily, it is not in the pathogens interest to rapidly kill
off large numbers of individuals in its host species, because that might imperil its own survival. Disease theorists long ago
expressed the idea that pathogens tend to evolve toward a "benign" state of affairs with their hosts, which
means in practice that they continue to infect, but tend not to kill (or at least not rapidly). A very good
reason for suspecting this to be an accurate view of pathogen-host relationships is that individuals with few or
no genetic defenses against a particular pathogen will be maintained within the host population, thus
ensuring the pathogen's ultimate survival.

Burn out stops disease

Lederberg 99 (Joshua, Professor of Genetics – Stanford University School of Medicine, Epidemic The World of Infectious
Disease, p. 13)

The toll of the fourteenth-century plague, the "Black Death," was closer to one third. If the bugs' potential to develop
adaptations that could kill us off were the whole story, we would not be here. However, with very rare exceptions, our
microbial adversaries have a shared interest in our survival. Almost any pathogen comes to a
dead end when we die; it first has to communicate itself to another host in order to
survive. So historically, the really severe host- pathogen interactions have resulted in a
wipeout of both host and pathogen. We humans are still here because, so far, the pathogens that have
attacked us have willy-nilly had an interest in our survival. This is a very delicate balance, and it is easily disturbed,
often in the wake of large-scale ecological upsets.

Humans will adapt

Gladwell 95 (Malcolm, The New Republic, July 17, Excerpted in Epidemics: Opposing Viewpoints, p. 29)
In Plagues and Peoples, which appeared in 1977. William MeNeill pointed out that…while man’s efforts to “remodel” his
environment are sometimes a source of new disease. They are seldom a source of serious epidemic disease. Quite the opposite.
As humans and new microorganisms interact, they begin to accommodate each other. Human
populations slowly build up resistance to circulating infections. What were once virulent
infections, such as syphilis become attenuated. Over time, diseases of adults, such as measles and
chicken pox, become limited to children, whose immune systems are still naïve.

No species has ever died from disease

Regis 97 (Ed, Author – “Virus Ground Zero”, “Pathogens of Glory”, New York Times, 5-18, Lexis)

Despite such horrific effects, Dr. Peters is fairly anti-apocalyptic when it comes to the ultimate import of viruses.
Challenging the widespread perception that exotic viruses are doomsday agents bent on
wiping out the human species, he notes that "we have not documented that viruses have
wiped out any species." As for the notion that we're surrounded by "new" diseases that
never before existed, he claims that "most new diseases turn out to be old diseases "; one type
of hantavirus infection, he suggests, goes back to A.D. 960. And in contrast to the popular belief that viral epidemics result from
mankind's destruction of the environment, Dr. Peters shows how the elimination of a viral host's habitat can eradicate a killer
virus and prevent future epidemics. This is what happened when the Aswan Dam, completed in 1971, destroyed the floodwater
habitat of the Aedes aegypti mosquitoes, carriers of Rift Valley fever virus: "After the Aswan Dam was constructed, there was no
more alluvial flooding. . . . Without a floodwater mosquito, the virus can't maintain itself over the long haul. . . . By 1980, Rift
Valley fever had essentially disappeared in Egypt." Still, Dr. Peters isn't totally averse to doomsday thinking, and in his final
chapter he lays out his own fictional disease scenario, in which a mystery virus from Australia suddenly breaks out in a Bangkok
slum. Throw in Malthus, chaos theory and the high mutation rates of RNA viruses, and soon he's got the world teetering on the
brink of viral holocaust in the finest Hollywood tradition. But he doesn't know quite what to make of his own scenario. He offers
"one valid, simplified equation to describe what we can expect from viruses in the future": mutating viruses plus a changing
ecology plus increasing human mobility add up to more and worse infectious diseases. Two pages later, though, he says that "it
is impossible to gauge how the actions of man will impact on emerging infectious diseases." If that is true, it discredits the very
equation he's given us. In the end, he presents no clear or consistent picture of the overall threat posed by the viruses he
discusses. The
empirical fact of the matter is that today's most glamorous viruses -- Marburg
and Ebola -- have killed minuscule numbers of people compared with the staggering death
rates of pathogens that go back to disease antiquity . Marburg virus, discovered in 1967, has been known to
kill just 10 people in its 30-year history; Ebola has killed approximately 800 in the 20 years since it appeared in 1976. By contrast,
malaria, an ancient illness, still kills a worldwide average of one million people annually -- more than 2,700 per day. More
than three times as many people die of malaria every day than have been killed by Ebola
virus in all of history. Yet it's Ebola that people find "scary "!
---Disease- AT: South China Morning Post

Disregard their South China Morning Post evidence—

It quotes a con man who duped a reporter into believing his false show about an AIDS vaccine
— this was just part of a larger scheme to fleece innocent investors out of millions. Ben-
Abraham is a fraud — he conned his way into a medical degree and did not graduate from
high school — his hyperbolic claims about disease impacts are self-interested and not
credible.
---Disease- Responses Solve

Responses solve

Ensom 3 (Jim, Crisis Management Trainer at Business Continuity Consultants, Former Editor of Survive Magazine, Former
Journalist for the BBC, June 20, http://www.globalcontinuity.com/article/articleview/94/1/30/)

in the containment of SARS, the most severely affected countries and areas
In reaching these landmarks
have identified and rapidly corrected long-standing weaknesses in their health systems in
ways that will mean permanent improvements for the management of all diseases. In addition,
systems of data collection and reporting, and new patterns of openly and frankly communicating information to
the public will hold the world in good stead when the next new disease emerges and the next
influenza pandemic breaks out.
---Disease- AT: Spanish flu

Comparisons to the Spanish flu are dumb – your authors don’t account for scientific advances.

Eberhart 5 [Staffer @ NewsMax. “Is President Bush Hyping the Avian Flu?” NewsMax.com, 3 Nov 05]

Unlike 1918 when the avian flu became pandemic and killed millions, today national and world health authorities
are armed with a modern understanding of epidemiology and some vital tools. Most influenza experts
agree, for instance, that the prompt culling of Hong Kong's entire poultry population in 1997 probably averted a
pandemic. Furthermore, tests for diagnosing all influenza strains in animals and humans are now rapid and reliable. Many
laboratories in the WHO global influenza network have the necessary high-security facilities, as well as the
reagents and experienced teams for performing these tests. Rapid bedside tests for the diagnosis of human
influenza are also available – allowing for the immediate quarantining of any human victims. Most
significantly, the modern world has modern medicines . Newsweek recently reported that this past summer two
expert virologists developed and ran computer simulations indicating that by simply stockpiling doses of the
antiviral medication Tamiflu and maintaining local vigilance, avian flu would be contained in the event of an
outbreak. And modern medicine's approach to pandemics is all about containment - as well as
keeping flu survivors from succumbing to so-called secondary infections while in a weakened condition.
Although the numbers are uncertain, many who battled through their siege with the 1918 Spanish flu were subsequently lost to
pneumonia and other bacterial infections. "In
1918 we had no antibiotics [for secondary bacterial
infections]; we had no anti-virals, no vaccines; there was a war on, and we didn't even know
what viruses were," concluded Ira Longini, a virologist at Emory University in Atlanta. "We keep going back to 1918
and that causes panic and fear, and I don't think we need to do that. "
---Disease- Hype

Your evidence is media hype

Rooks 06 – (Kyle, thesis paper, School of Journalism, Media and Cultural Studies Cardiff University, “An Epidemic of Epidemics: A
Case for Public Relations Role in Mitigating Health Scares,” http://slb.cf.ac.uk/jomec/resources/KyleRooks_MAIPR2005_2006.pdf)

Health scares are rife in today’s society, ironically persisting during a time of unparalleled health .
Capturing the headlines and the public’s imagination, these scares have a detrimental impact on
public health as the ensuing panic invokes stress – which does a number of harmful things to our health – and
influences public health policy through the misappropriation of government resources. The ongoing panic in the UK surrounding the
measles, mumps and rubella (MMR) vaccine, one of the safest and most effective preventative health measures, highlights both the
importance of being able to effectively frame and communicate risk and the glaring inability of health authorities to maintain trust in
the vaccine. With
the media and pressure groups perpetuating health scares to satisfy their
agendas, and the scientific community poorly positioned from a traditional standpoint to rebut
scaremongering tactics, this study endeavours to determine how a public relations ( PR) strategy
can cut through misperceptions in order to foster a reasoned dialogue and appropriate public
action to health risks.
---Disease- Aids

AIDS is getting weaker - any mutation will only make it more benign

DNC 5 (Daily News Central, Health Media Company, “HIV Virus May Be Evolving to Less Deadly Form,” September 30 th,
http://health.dailynewscentral.com/content/view/1716/0)

New evidence suggests that the AIDS virus, HIV, may be weakening. Scientists in Belgium
compared samples of HIV-1, the most dangerous strain of the virus, from the 1980s and 2002. Laboratory tests showed
that the older viruses were significantly "fitter." They multiplied more easily and were better able to resist anti-
retroviral drugs. The findings appear to contradict recent trends which indicate a growth in HIV drug resistance. But they
support theories which suggest that viruses sometimes evolve to become less virulent in
order to safeguard their survival. In this respect, HIV may be following in the footsteps of the virus which causes
myxomatosis in rabbits. When the myxoma virus was deliberately introduced as a control measure in Australia in 1950, rabbit
populations were decimated. But weakened strains of the virus quickly emerged, so that many rabbits now develop a chronic
form of the disease instead of dying. Experts warn, however, that HIV infection rates are continuing to rise in the UK and
elsewhere, and there should be no scaling down of efforts to curb its spread. The researchers, led by Kevin Arien from the
Institute of Tropical Medicine in Antwerp, looked at HIV-1-infected cells obtained from patients in 1986-89 and 2002-03. The
older viruses out-competed the new ones on 176 occasions out of 238. In nine out of 12 specially controlled and carefully
matched tests, the 1980s viruses proved the stronger. The researchers wrote in the journal AIDS: "These findings suggest that
HIV-1 replicative fitness may have decreased in the human population since the start of the
pandemic." Previous models simulating the spread of infective agents have suggested that many lethal viruses and
bacteria may evolve away their virulence, even to the point of "symbiotic existence" where they
actually benefit the host. If a virus is too deadly, it risks working against itself by killing off many of its potential hosts. Becoming
less prolific may also help to shield a virus from the host's immune system.

No extinction - 10% of people cannot get it

Smith 6 (Stephen, Reporter for the Boston Globe, “A Darwinian View of AIDS,” March,
www.boston.com/news/science/articles/2006/03/13/a_darwinian_view_of_aids/?page=full)

This is the story of the rabbit and the virus. Rabbits were never meant to live in Australia. But a 19th-century game hunter
thought otherwise and imported the creatures. Over the decades, they did what rabbits do best: multiplied and multiplied, until
they became a major nuisance. So, in the 1950s, Australian authorities introduced a viral disease called myxomatosis in a bid to
eradicate the rabbits. And it worked -- but only briefly. In a matter of years, rabbits acquired the ability to resist the virus. It is a
classic tale of evolution -- a story that unspools again and again in animals, people included. The march of human civilization,
measured both by survival and suffering, is framed in many respects by how well we compete with microscopic organisms. By
exploring how humans evolve to win the battle with diseases and how they are sometimes vanquished by those pathogens,
scientists hope to find drugs that can mimic successes and stave off the worst that viruses and bacteria can produce. ''It's a
constant race between humans and their capacity to create new drugs and pathogens and their ability to evolve," said Dr.
Daniel Cohen, an infectious disease researcher at Fenway Community Health in Boston, which specializes in AIDS treatment. But
before researchers can develop new drugs, they must understand the evolutionary nature of individual diseases and how
viruses and bacteria, in turn, shape the evolution of humans and other animals. Scientists have long known, for example, that in
regions where malaria is common, most notably Africa, humans have developed an intrinsic defense against the parasitic illness.
And that defense, which involves a change in red blood cells, has proved to be a good thing for many Africans, making them less
susceptible to the disease. But the same trait has proved to be a bad thing for their descendants in the United States and other
malaria-free areas, because it results in sickle cell disease, a condition that robs organs of oxygen, causing debilitating bouts of
pain and sometimes death. There are vital history lessons, too, when it comes to HIV. As researchers unlocked the
secrets of HIV, they found a gene mutation they suspect may protect against the virus that
causes AIDS. Human cells have locks on their surface -- scientists call them receptors -- and a virus
must insert its key into these locks to gain entry. One of those is called CCR5, and HIV
needs to unlock it [CCR5] to be able to infect cells. But scientists in recent years discovered
that 5 to 10 percent of people in northern Europe don't have CCR5 receptors. ''And that's where
the story gets interesting," said Dr. Calvin Cohen, research director for Community Research Initiative of New England, which
conducts trials of AIDS drugs. In contrast, people in Africa and Asia universally possess CCR5. So researchers theorized that lower
HIV rates in northern Europe might be due in part to some people lacking the cellular lock. But why don't they have it? Right
now, it's only an informed hunch, but scientists suspect that the mutation exhibited by northern Europeans
may be an artifact of the bubonic plague. The theory goes like this: As the plague swarmed Europe starting in
the 14th century, it wiped out people who possessed CCR5 but spared those who lacked it. ''What we're talking about is a
Darwinian process," Harmit Malik, who specializes in the study of genetic conflict at the Fred Hutchinson Cancer Research
Center in Seattle. ''What
was a really rare mutation was what survived. Everyone else had fallen
prey to this particular pathogen." And the thing is people who lack CCR5 receptors appear not to
suffer any consequences. ''So we have an ideal combination," Calvin Cohen said. ''HIV needs it, but we don't. What an
ideal target for drug development." That's why drug companies have developed experimental medications designed to block
CCR5 so that HIV cannot enter cells. Cohen's Community Research Initiative is currently involved in a study of a Pfizer Inc.
medication, with several patients enrolled in Boston.

We’ve already found a way to prevent spread

Araque 10 (Horacio, Miami Technology Examiner, “Scientist believe they found a cure for AIDS/HIV,” 7-4,
http://www.examiner.com/technology-in-miami/scientist-believe-they-found-a-cure-for-aids-hiv)

U.S. Experts
have discovered a way to avoid HIV infection , a fact that would prevent further
spread of the AIDS pandemic , according to a study published in the journal "Nature Biotechnology". Currently, there
are over 33 million people in the world infected with HIV/AIDS who, so far, have not found treatment to cure it. Experts from
the University of Southern California explained in the report how they
managed to create cells in mice
immune to the virus causing the disease. They reported that if these cells could be developed in
humans, the pathogen will be controlled, Prensa Latina revealed today on its Website. The protein in question,
the CCR5 receptor, is a surface of white blood cells that HIV uses to infect cells. Some time ago it was determined that people
with a mutation in that cells are more resistant to infection. Hence, the modifying, implanted cells would control the condition,
which was found in rodents used in the test, but it is yet to be demonstrated if the same results be obtained in humans "The
challenge now is to apply the same method in humans , which would open the way to generate a virus -resistant proteins that
can produce HIV -resistant cells in all the counterparts it infects" , said Paula Cannon director of The US Labor Department. We
hope and trust we are at the dawn of a significant breakthrough period for finding a cure to such a
deadly disease as AIDS/HIV. Have a full recovery soon, South Florida AIDS patients.

No AIDS extinction- no global infection


Yu 9 (Victoria, Dartmouth science dept, 5/22, http://dujs.dartmouth.edu/spring-2009/human-extinction-the-uncertainty-of-our-
fate)

Some surveyed students mentioned AIDS as a potential pandemic-causing virus. It is true that scientists
have been unable thus far to find a sustainable cure for AIDS, mainly due to HIV’s rapid and constant evolution. Specifically, two
factors account for the virus’s abnormally high mutation rate: 1. HIV’s use of reverse transcriptase, which does not have a proof-
reading mechanism, and 2. the lack of an error-correction mechanism in HIV DNA polymerase (8). Luckily, though, there
are
certain characteristics of HIV that make it a poor candidate for a large-scale global
infection: HIV can lie dormant in the human body for years without manifesting itself, and
AIDS itself does not kill directly, but rather through the weakening of the immune system.
---Disease- Bird Flu

No bird flu pandemic (mutation unlikely, vaccines solve, and international cooperation checks)

Strait Times 9 (“Bird Flu Pandemic ‘Unlikely to Occur’”, 1-24, Lexis)

A flu pandemic - which many scientists fear could strike at any time and kill millions of people - is unlikely to
happen, says one infectious diseases expert. Professor Jonathan Yewdell, from the prestigious US
National Institutes of Health (NIH), said that the biggest disease threat - virulent bird flu viruses rampant in
poultry and wild fowl - was unlikely to mutate to a form that would spread easily among people. On top of that,
many countries have been preparing for years to deal with a potential pandemic. Also, scientific
advances would translate into speedy vaccines which would undermine an outbreak before
it swelled to the size of pandemics of the past century. A pandemic, which could be caused by a mutated strain of the avian
influenza virus, could hit up to 70 million people and cost US $3 trillion (S $4.5 trillion), according to the United Nations and
World Bank. It is also considered one of the world's biggest threats. Since 2003, the virulent H5N1 strain of bird flu has infected
about 400 people, killing around 250. It is endemic in many parts of Asia. But Prof Yewdell, chief of cellular biology at the NIH
Laboratory of Viral Diseases, believes a pandemic is not likely. Immunology is the study of the body's response to
foreign entities such as viruses. The NIH, which leads biomedical sciences research in the US, does frontline work on fighting bird
flu and other dangerous viruses. Before a pandemic can occur, the virus has to mutate to a stage where it can spread
easily from person to person, said Prof Yewdell, who took part this week in a symposium here organised by the Singaporean
Society for Immunology. But most bird flu cases so far have been through direct contact with infected
birds, though there have been rare cases of human-to-human transmissions. While he concedes that bird flu could one day
mutate to a form where it spreads among people, he believes medical
science and international political
cooperation will stop it in its tracks. 'We are in a much, much better place today than in 1968
when we had the last flu pandemic ,' he said. 'In the past, it took a few years to sequence the
genome of a virus. Now, we can do it overnight.' The information allows vaccines and
treatments to be developed very quickly - within months , compared to years in the past.

Risk declining - bird flu peaked in ‘06

Resler 8 (Staff – MJS, “Avian Flu: Was Panic Warranted?”, Milwaukee Journal Sentinel, 4-9, Lexis)

A few years ago, public health experts and even average folks were nervous about what they feared
might be the next global pandemic - avian or bird flu. While some of those fears, especially on the part of the public,
might have been exaggerated, there was good reason to be concerned. Researchers here and abroad continue
to prudently monitor the disease, but many are now questioning whether it will become the pandemic
predicted only a few years ago. The Journal Sentinel's Susanne Rust noted this week that, at the flu's peak in 2006,
there were 115 human cases worldwide; last year, that number dropped to 87
(www.jsonline/735618). Researchers, including Dennis Maki, an infectious disease expert at the University of Wisconsin-
Madison, told Rust that the virus has had millions if not trillions of opportunities over the past five
years to evolve into a pathogen that easily could make the jump to people . So far, it has
not. And while that is certainly something for which the human race can be grateful, the ultimate folly in public health is to let
down one's guard, whether it's against a virus as familiar as measles or as exotic as avian flu, which has killed more than 60% of
the people it has infected.
No mutations

Siegel 5 (Marc, Associate Professor – New York University School of Medicine, “An Epidemic of Overreaction”, Los Angeles
Times, 10-11, http://www.ahrp.org/infomail/05/10/27b.php)

The facts are these: The current H5N1 avian influenza virus has notmutated into a form that can easily
infect humans, and the 60 people in the world who have died of this bird flu have done so not because this bug is on the
road to mutation but because millions of birds throughout Asia have been infected, and the more birds that have it, the more
likely that an occasional human bird handler will be infected. Most human influenzas begin as bird flus, but many bird flus
never change to a form that can harm us. Though flu pandemics occur on the average of three times per
century, and we are clearly overdue (the last was in 1968), there is absolutely no indication that the
transformation to mass human killer is about to happen. The threat is theoretical. Unfortunately, the
attention it has received makes it feel like something terrible is inevitable.

No pandemic - their evidence exaggerates

Stratfor 5 (“Special Report: The Bird Flu and You”, 10-24, http://www.us-asean.org/Avian_Flu/stratfor.htm)

An uncomfortable but undeniable fact is that there are a


great many people and institutions in this world that have a
vested interest in feeding the bird flu scare. Much like the "Y2K" bug that commanded public attention in 1999,
bird flu is all you hear about. Comparisons to the 1918 Spanish influenza have produced death toll projections in excess of 360
million, evoking images of chaos in the streets. One
does not qualify for funding -- whether for academic research,
medical development or contingency studies -- by postulating about best-case scenarios. The strategy is to show
up front how bad things could get, and to scare your targeted benefactors into having you study the problem and manufacture
solutions. This hardly means that these people are evil, greedy or irresponsible (although, in the case of Y2K or when a health
threat shuts down agricultural trade for years, one really tends to wonder). It simply means that fear is an effective way to spark
interest and action. Current medical technology lacks the ability to cure -- or even reliably vaccinate against -- highly mutable
viral infections; the best available medicines can only treat symptoms -- like Roche's Tamiflu, which is becoming as scarce as the
oftentimes legendary red mercury -- or slow a virus' reproduction rate. Is more research needed? Certainly. But are
we on
the brink of a cataclysmic outbreak? Certainly not. A bird flu pandemic among the human
population is broadly in the same category as a meteor strike. Of course it will happen sooner or later -- and
when it does, watch out! But there is no -- absolutely no -- particular reason to fear a global flu pandemic
this flu season. This does not mean the laws of nature have changed since 1918; it simply means there is no way to predict when
an animal virus will break into the human population in any particular year -- or even if it will at all. Yes, H5N1 does show a
propensity to mutate; and, yes, sooner or later another domesticated animal disease will cross over into the human population
(most common human diseases have such origins). But there is no scientifically plausible reason to expect
such a crossover to be imminent. But if you are trying to find something to worry about, you should at least worry about
the right thing. A virus can mutate in any host, and pound for pound, the mutations that are of most interest to humanity are
obviously those that occur within a human host. That means that each person who catches H5N1 due to a close encounter of
the bird kind in effect becomes a sort of laboratory that could foster a mutation and that could have characteristics that would
allow H5N1 to be communicable to other humans. Without such a specific mutation, bird flu is a problem
for turkeys, but not for the non-turkey farmers among us. But we are talking about a grand total of 115
people catching the bug over the course of the past three years. That does not exactly produce great
odds for a virus -- no matter how genetically mutable -- to evolve successfully into a human-communicable strain.
And bear in mind that the first-ever human case of H5N1 was not in 2003 but in 1997.
No evidence supporting mutation occuring

Parkinson 6 (Caroline, Health Reporter for the BBC, “Bird Flu: Is it still a threat?”
http://news.bbc.co.uk/2/hi/health/6159600.stm)

The fear is that the H5N1 virus will combine with a human flu virus and mutate into a pandemic
strain which could spread easily between people. There have been no indications over the last
12 months that this is any closer. But the virus has not gone away. It has changed, as all viruses do. And
there have been cases, such as a cluster in Indonesia, where family members have contracted bird flu. But, importantly, it
was H5N1 they caught, and not a mutated strain. Dr Hay said: "There is some evidence of variation
in the strain, but there is no evidence the virus has an increased capacity to spread. We
haven't seen any change."

No risk of jumping the barrier and quarantines solve mutation.

AT 9 [The Abbotsford Times, “Bird Flu not Highly Contagious” 4 Feb 09, L/N]

The strain of avian influenza found in an Abbotsford turkey barn two weeks ago has been identified as H5N2, and has low
pathogenicity or a low risk of being passed on, the Canadian Food Inspection Agency reported Wednesday. Avian
influenza is a disease that affects birds but does not cross easily from birds to infect humans. There is no evidence
to date that shows this strain poses any significant risk to human health. An infected turkey flock of 60,000
birds at E & H Farms was killed at the site and CFIA workers continue to monitor the composting process. In order
to prevent the virus from spreading to other flocks, CFIA investigators have quarantined 36 poultry farms in the
vicinity of the farm where the virus was discovered in a flock of 12-week-old turkeys. Each site is under quarantine for about
three weeks. These premises are in the three-kilometre radius of E & H Farms or have had known contact with the infected farm
during the time the virus may have been present. Before the poultry or poultry products can leave these sites, birds
must be sampled, test negative and receive a permit for movement from the CFIA.

Science found the PA protein – short-circuits risk of escalation.

Bennett 9 [Sam, staffer, “Bird Flu Protein May Offer Target for New Drugs” Bloomberg, 2/5/9, L/N]

A protein that helps avian influenza replicate may be a target for new drugs against the virus and
other flu strains, according to two studies in the journal Nature. Scientists in China and France showed how the PA
protein enables flu viruses to trick cells into copying themselves instead of normal cellular proteins.
Knowing more about PA and what it does may help researchers develop drugs to interfere with it and
limit the duration and severity of human flu infections, the studies said. “This provides a feasible target for
the discovery of new anti-influenza therapeutics, which are urgently needed owing to the
increasing problems of resistance to available drugs ” such as Roche Holding AG’s Tamiflu, researchers led by
Puwei Yuan at the Chinese Academy of Sciences in Beijing wrote. Health officials are concerned the H5N1 strain of avian influenza
may mutate into a highly contagious human flu, triggering a pandemic that could kill 71 million people worldwide and lead to a
“major global recession” costing more than $3 trillion, according to a worst-case scenario outlined by the World Bank in October. A
drug that attacks parts of the flu virus that don’t change could help health officials combat the
pathogen and curb its spread until a vaccine becomes available .
---Disease- Smallpox

Smallpox unlikely

Moodie 1 (Michael, President – Chemical and Biological Arms Control Institute, FDCH, 11-7, Lexis)

An example of this kind of vulnerability assessment is one that often focuses on the agent smallpox. A scenario is posited that
begins, "Assume a terrorist has smallpox. . .," and it proceeds to describe events that lead to a global smallpox pandemic.
While it is possible that use of smallpox could have such consequences, the assessment itself says
nothing about the likelihood of such an occurrence. And yet, appreciating the likelihood of an event is critical
to effective policy planning. Where, for example, would the terrorist get the smallpox initially? Unlike anthrax, smallpox is
not present in nature since it was eradicated as an infectious disease by the World Health
Organization. There are potential sources of smallpox, but the scenario of the kind posited above does not address the issue of
acquistion. Would not the issue of availability have some bearing on the likelihood of that particular scenario and, hence, be of
interest and concern to a decision maker?

No intentional use

Washington Post 1 (10-24, http://www.washingtonpost.com/wp-dyn/articles/A42547-2001Oct23.html)

How likely, then, is a smallpox attack? Jonathan Tucker, director of the Chemical and Biological Weapons
Nonproliferation Program at the Monterey Institute of International Studies, and author of Scourge: The Once and Future
Threat of Smallpox, estimates that the risk is minuscule -- even in light of September 11. "The number
of groups that could use smallpox is very, very small," he says. "They need a motive to cause
widespread destruction" that could eventually wash over them and their backers as well. And, he said, "they have to
be able to cloak their activities ."

Many barriers to smallpox

Newsweek 1 (10-18, p. 21, Lexis)

Acquiring, producing, and delivering the smallpox virus would pose a series of challenging
technical hurdles for terrorists, making an attack with the virus unlikely--although potentially catastrophic were it to
occur. First, because the smallpox virus no longer exists in nature, terrorists would have to
acquire it from a state with undeclared laboratory stocks of the virus, or perhaps from former Soviet bioweapons
scientists who had smuggled out samples of the virus. Second, the terrorists would have to grow the virus in eggs or
animal cells, which is technically challenging. Third, they would have to find some means of
disseminating the virus as a fine, inhalable mist of microscopic particles or droplets (called an aerosol), which would
require specialized technology and know-how. For a low-tech attack, suicide terrorists might consider
infecting themselves and spreading the disease, but they would have only a few days to do so before the facial rash became
obvious. Moreover, even
terrorists prepared for instant martyrdom in an explosion might hesitate
before willingly suffering a slow, painful, and hideous death from a disease like smallpox.
---Disease- Influenza

Influenza won’t cause mass death- basic countermeasures solve

Collignon ‘9 (Monday, 25 May 2009 Take a deep breath, Swine Flu’s not that bad by Peter Collignon Peter Collignon is an
Infectious Diseases Physician and Microbiologist and Professor, School of Clinical Medicine, Australian National University.

We need to also consider what killed most people when new and virulent Flu pandemics spread across
the world previously. It is not the Flu virus itself. Most deaths were likely the result of secondary bacterial infections
especially Staphylococcus aureus and pneumococcus. The high death rate in 1919 was because there were no
antibiotics developed yet. In the late 1950’s (Asian Flu), it was because there was a lack of available and active antibiotics — 
penicillin resistance had developed and spread rapidly in Staphylococcus aureus by then. Antibiotic resistance is a major and rapidly
growing international problem, especially in developing countries. However in Australia we are fortunate because we
still have
a variety of antibiotics (especially injectables) that will work against nearly all strains of bacteria that
might complicate Flu and cause pneumonia. We also know we can slow or stop the spread of Flu virus,
even in households with close personal contact by good hygiene, hand care (alcohol hand rub and soap and
water), masks and other general infection control measures .
---Disease- Swine Flu
Swine flu will kill nobody

Collignon ‘9 (Monday, 25 May 2009 Take a deep breath, Swine Flu’s not that bad by Peter Collignon Peter Collignon is an
Infectious Diseases Physician and Microbiologist and Professor, School of Clinical Medicine, Australian National University.

There are, appropriately, public health concerns with any new strain of influenza that develops.
During the last century we have had three major events where millions of people died after new strains (H1, H2 and H3) spread to
people and to which we had little or no pre-existing immunity (eg H1N1 Spanish Flu from 1918-19). These new strains spread to
people from strains derived from birds, often after mixing their genetic components in pigs. This latter mixing made them more
adept at spreading from person to person. The current “Swine flu” strain has a mix of human, pig and bird genetic components and
thus is of some concern. However this current strain does not fulfil two essential characteristics needed for any Flu strain to be the
same problem as Spanish flu. For any Flu strain to be a problem of the same magnitude it needs: to readily spread from person to
person AND to have a more aggressive (or virulent) effect in people compared to the strains that are circulating now and causing
problems every winter. This current swine flu strain is not more virulent. It does however transmit readily and so will very likely
cause a pandemic when it spreads around the world. However there is a problem with the word pandemic. This
just means that the virus spreads widely across continents. “Pandemic” however seems to invoke a
feeling that a new epidemic disaster has hit us and with a strain of virus that will kill a large
proportion of a population (ie it is also hyper-virulent). Swine flu is not hyper-virulent. While it spreads
relatively easily it does not appear to be any more virulent that the strains we have circulating
every year around the world. The most reliable figures to judge its virulence so far come from the US.
Data from developing counties is often incomplete because of poorer testing facilitates and an
inability to find and test mild cases. This will lead to an overestimation of the virus’s virulence .
Currently in the US it appears that for every 1000 people who get infected, about 40 people need admission to hospital and
about one person dies. This is a still an aggressive virus, but no more so than the Flu viruses that change slightly every
year or so, and then circulate around the world, mainly causing problems in winter . Given this Swine
Flu strain is H1, we would expect many in the population to have some immunity because variations of H1
strains have been recirculating in people since 1918 . This appears to be the case and is reflected in the relatively
small numbers of people over the age of 30 who have been infected. This is quite different to the situation when completely new H
strains come across from pigs or birds (eg H7), where almost no one has been previously exposed and thus we have little or any pre-
However with this Swine Flu, even children and young adults, with presumably little or no
existing immunity.
pre-existing immunity, do not seem to be experiencing excessive mortality compared to seasonal
influenza. Again this reflects the relatively low virulence of this current strain.

No impact - their evidence is hyperbole

Hannan 9 (Daniel, Author and Staff Writer for The Telegraph, “Ignore the health scare professionals: you won’t die from swine
flu,” 2009, http://blogs.telegraph.co.uk/news/danielhannan/100002999/ignore-the-health-scare-professionals-you-wont-die-of-
swine-flu/)

Swine flu is a nasty disease, but no nastier than other strains of influenza. True, it has killed hundreds of
people in Mexico (possibly for these reasons); but even there, other variants of ‘flu virus have been far more
lethal. Why, then, the urgent need to inoculate the entire British population? Perhaps I’m being overly cynical, but I can’t help
wondering whether we’re being pushed into a wrong-headed course of action by the health
scare industry. We’re told that Tamiflu needs to be taken at once, without a moment’s
delay – meaning that anyone with a sniffle is likely to start glugging the stuff. We’re also told that the virus may
mutate, meaning – conveniently – that we’ll soon need a new variety of medicine. In any case,
these flu vaccines have short shelf lives. Good news for the drug manufacturers and their lobbyists ; bad
news for the taxpayer. (Incidentally, does anyone know whether the most recent fatality – the only one not to have had
complicating medical ailments – had been given Tamiflu? I can’t find the answer anywhere.) I don’t really blame the
pharmaceutical companies: they’re doing what all big corporations aim to do, namely to maximise their profits. But I do blame
the politicians who are falling for it. Ministers must suspect that the danger is being exaggerated . Yet they would
rather spend gazillions than run the slightest risk of being accused of not having done enough. And, needless to say, there isn’t a
medical advisory body in the world that will say: “Actually, minister, considering everything in the round, the danger posed by
this virus is minor, and we recommend the disbandment of this panel”.

Status quo solves

Garrett 9 (Laurie, Senior Fellow for Global Health at the Council on Foreign Relations, “Global Health Crisis: Swine Flu, 4-27,
www.cfr.org/publication/19245/global_health_crisis.html)

Laurie, the latest reports are indicating 150 dead; more than 1,600 hospitalized in Mexico; about at least eight countries
involved, overall, now. What are the most important facts for the public to focus on in this -- at this stage in this outbreak?
LAURIE A. GARRETT: I think, at
this moment, if you're in the United States you want to focus on the
fact that no one has had serious illness from this; that the infections that we know of, to
date, in the United States have been self-resolving, meaning the disease just ran its course and
the individual got well, just as would be the case with garden-variety flu . I think that's very
important to keep in mind. It doesn't mean that we have nothing to worry about, but it does mean that we
want to keep a level head on this and not rush to a worst-case scenario unless the data
guides us in that direction.

No pandemic – swine flu is responding to regular medication and their evidence is based on
hype

Carafano 9 (David, Senior Research Fellow in Defense and Homeland Security at the Heritage Foundation, “Swine Flu: A Cause
for Panic?” 4-27, http://roomfordebate.blogs.nytimes.com/2009/04/27/swine-flu-a-cause-for-panic/, JMP)

There are some basic facts that Americans ought to know. The first is that the news
coverage is driven more by the unusual nature of the disease than how serious it might be.
The swine flu appeared at what is normally considered the “end” of the flu season. A flood of cases now is not common.
Problems with a swine flu vaccine in 1976 helped sink President Ford’s re-election chances. Second, it is a flu from a strain that
normally affects pigs. Popular attention had been fixated in recent years on the threat of mutating bird flu. Americans hadn’t
heard much about swine flu in decades. In 1975, a U.S. soldier died of swine flu, fueling fears of an outbreak. A vaccine was
rushed out. No pandemic occurred but the vaccine had some bad side effects. The debacle helped sink the re-election chances
of President Ford, who had pushed for the vaccination campaign. Since then politicians didn’t like talking about swine flu. Third,
there have been an unexpected number of deaths reported in otherwise “healthy” adults in Mexico. Usually, flu kills only the
very old and young or folks with depressed immune systems, like AIDS sufferers. All these
rare facts grab headlines.
In addition, news stories play fast and loose with terms like “outbreak,” “epidemic,” and
“pandemic.” They are not interchangeable. Each describes an increasing degree of the number of infected and
the geographical scope of the infection. We are not close to having a pandemic yet. And if we do it
may not be anything like a catastrophe. The disease looks to respond to existing
medication. Finally, the swine flu makes news because we can pin it to a location. But it might be premature to call this the
“Mexican” flu, just as it turned out it was wrong to call the 1918 pandemic the “Spanish” flu (which scientists think actually first
appeared in Kansas).

U.S. is already containing swine flu

Recordnet 9 (“World Health Organization declares first global flu epidemic in 41 years,” 6/11/2009,
http://www.recordnet.com/apps/pbcs.dll/article?AID=/20090611/A_NEWS/90611003, JMP)

In the United States, where there have been more than 13,000 cases and at least 27 deaths from swine flu, officials at
the U.S. Centers for Disease Control and Prevention said the move would not change how
the U.S. tackled swine flu. "Our actions in the past month have been as if there was a
pandemic in this country," Glen Nowak, a CDC spokesman, said Thursday. The U.S. government has
already taken steps like increasing availability of flu-fighting medicines and authorizing $1
billion for the development of a new vaccine against the novel virus. In addition, new cases
seem to be declining in many parts of the country , U.S. health officials say, as North America moves out of
its traditional winter flu season.
---Disease- TB

Vaccines will solve TB

Landry and Heilman 5 (Sarah, Associate Director of Policy and Program Operations – National Vaccine Program Office in
the Department of Health and Human Services and Carole, Director of Microbiology and Infectious Diseases – National Institute of
Allergy and Infectious Disease, “Future Directions in Vaccines: The Payoffs of Basic Research”, Health Affairs, 24(4), May)

Promise of new technologies. The payoffs from these standard approaches are now beginning to plateau. In fact, most of the
"easy" vaccines have been developed, and many challenges lie ahead for new and improved vaccines. New technologies
may provide stronger, broader, and more durable immune responses than those induced by some
earlier vaccines. New vaccines are also likely to exploit genomics and high-throughput screening
approaches that are based on computational methods. These methods will allow for development of rationally based
approaches that select potential antigens more effectively and precisely. In addition, future vaccines will use these new
tools to get around the challenges of the remaining infectious diseases. [n2] These challenges include the
inherent ability of many viruses to change (antigenic variation), as is seen with HIV and influenza; the need to develop
vaccines that rely on cell-based immunity for protection for infections such as tuberculosis ; and tools for
addressing a pathogen's ability to outsmart the immune system--immune evasion strategies, such as seen with hepatitis C. [n3]
Impact of new immune concepts. Research on the immune system has helped identify new ways of fighting infections and is
helping define the mechanisms needed for successful immunization. Most currently licensed vaccines protect by producing
neutralizing antibodies, made by the B cells of the immune system. One of the advantages of stimulating this arm of the
immune system is that it can be easily measured. Researchers believe that vaccines against many of the infections that are of
highest priority (HIV, TB, and malaria) will need to have the other arm of the immune system--the cellular component, or T
cells--pulled into action. [n4] For the first time in sixty years, new TB vaccines are in clinical trials.
[n5]

Alt cause - Asia

WHO 98 (World Health Organization, 1998)

Asia is the epicentre of the world's TB emergency and must become a top priority among
international efforts to control the disease, according to Dr Gro Harlem Brundtland, Director-General of the
World Health Organization. In a statement today (23 November) at the Global Congress on Lung Health, 29th World Conference
of The International Union against Tuberculosis and Lung Disease (IUATLD), Dr Brundtland warned that unless
there is
concerted action in Asia, the epidemic will continue to rise, jeopardising global control
efforts. "Our ability to control the spread of TB pivots on Asia – now the epicentre of the world's TB
epidemic," said Dr Brundtland. "If we cannot control TB in Asia we will never stop TB globally . Factors
such as HIV, multidrug-resistant TB and the financial crisis converging in this region are increasing the complexity of the
Six high-burden countries which account for over 50
epidemic, making it far more difficult to contain."
percent of the TB epidemic are in Asia. According to WHO estimates, 4.5 million of the eight million new cases
that occur each year are in India, China, Bangladesh, Pakistan, Indonesia and the Philippines. "We are at a crossroads in TB
control," said Dr Brundtland. "We can allow the global TB epidemic to become more deadly and strengthen its grip on the
world. Or we can act now to reduce the suffering and deaths. We can and must strike back with the tools that we have.
No rapid TB spread

Collins and Fidel 7 (Lois M., and Steve, Staff – DMN, Deseret Morning News, 6-3, Lexis)

The frenzy over tuberculosis spawned by a single "extensively drug-resistant" case is


capturing headlines. But most people exposed to the airborne bacteria will never develop
active disease. The Atlanta attorney's case has health officials concerned because his TB falls into a class of infections that
resists two first-line TB drugs and some second-line drugs -- one of only 49 other extensively drug-resistant cases reported in the
United States between 1993 and 2006. There's also a class called multidrug-resistant TB, which is easier to treat than cases like
this one but more difficult than typical TB. Although it's harder to kill, it's
no easier to spread than any other
tubercolosis, according to Carrie Taylor, an infection control practice nurse at LDS Hospital.
"You have to breathe in air that's coughed." Doctors treat an average of 38 active TB cases each year in Utah,
according to the Utah Department of Health. The disease usually settles in the lungs, although it can affect the kidneys, spine,
brain and other organs. The disease is caused by Mycobacterium tuberculosis, which spreads person-to-person but only through
close contact. Taylor and her colleague Vickie Anderson, also an infection-control practice nurse at LDS Hospital, describe it as
passing from one person's lungs directly into another's. It's not like a cold that is easily spread and fairly hardy. In fact, sunlight
kills it. Unless the individual has a drug-resistant TB strain -- "not common in Utah," said Taylor -- it's very treatable, although it
takes a long time and several medications. Left untreated, it can kill. At least initially, patients are isolated to avoid spread of the
disease. Both chicken pox and measles are more contagious, said infectious disease specialist Dr. John Kriesel of University
Hospital. As an example, when a Provo High School student was recently diagnosed with tuberculosis and health officials asked
250 of the student's school contacts to be tested for it, Kriesel predicted "not one of them will test positive for TB." People
in casual contact are extremely unlikely to get the disease. Just being exposed doesn't
mean you could pass it on, Taylor said. Without symptoms, you can't spread it, even if you have a positive skin test.
People who live with a patient are at higher risk, but most won't get it, either.

New drugs solve even drug resistant TB

Vaccine Weekly 9 (1/21, "Tuberculosis; Sequella lead drug compund SQ109 selected for phase 1B clinical trial program", 1-21,
p.85, ProQuest,)

2009 JAN 21 - (<http://www.newsrx.com> NewsRx.com) -- Sequella, Inc., aclinical-stage biopharmaceutical


company focused on diseases of epidemic potential, announced that SQ109, its lead drug candidate for
the treatment of tuberculosis (TB), was the first drug approved for evaluation in a newly
awarded clinical program contract to Dynport Vaccine Company LLC and Quintiles Transnational. The
contract, awarded by the National Institute of Allergy and Infectious Diseases (NIAID), National
Institutes of Health (NIH), is part of NIAID's clinical resource infrastructure to accelerate Phase 1 studies of
promising clinical stage drugs or vaccines that address emerging and re- emerging infectious tropical diseases and
bioweapon pathogens. The Phase 1B clinical study of SQ109 should be initiated in Q1 2009 (see also <http://
www.newsrx.com/library/topics/Tuberculosis.html> Tuberculosis). SQ-109 is a new diamine antibiotic intended to replace one
or more of the current first-line anti-TB drugs and simplify patient therapy. SQ109
was granted U.S. FDA Fast
Track designation and FDA/EMEA Orphan Drug Designation in 2007. SQ109 shows activity against drug
sensitive and multi-drug resistant (MDR and XDR) Mycobacterium tuberculosis, the causative agent of TB. The
Phase 1B study will assess safety and pharmacokinetics of multiple doses of SQ109 in healthy subjects. "This is absolutely the
best of both worlds for Sequella," commented Dr. Carol Nacy, Sequella CEO. "We again successfully competed for support from
our most valued funding partner, NIAID, while retaining the capacity to work with the same industry-leading contract research
organization, Quintiles Transnational, that conducted our first-in-human Phase 1A trial for SQ109.

Existing programs solve – TB threat falling


SNS 7 (States News Service, “Global Tuberculosis Epidemic Levelling Off: New Report”, 3-22, Lexis)

The global tuberculosis (TB) epidemic has levelled off for the first time since WHO declared TB a public health
emergency in 1993. The Global Tuberculosis Control Report released today by WHO finds that the percentage of the world's
population struck by TB peaked in 2004 and then held steady in 2005. "We are currently seeing both the fruits of
global action to control TB and the lethal nature of the disease's ongoing burden," said United Nations Secretary-
General Ban Ki-moon. "Almost 60 per cent of TB cases worldwide are now detected, and out of
those, the vast majority are cured. Over the past decade, 26 million patients have been placed on effective TB
treatment thanks to the efforts of governments and a wide range of partners. But the disease still kills 4400 people every day."
Dollar Collapse Answers
Frontline

Dollar won’t decline – Chinese won’t allow it and there’s no alternative

Hugh 10 -Macroeconomist who specializes in growth and productivity theory (“Interview with Edward Hugh: The Dollar’s Demise
is Vastly Overstated,” Blog Invest, 4/30/10, http://trick-bloggerinvest.blogspot.com/2010/04/interview-with-edward-hugh-
dollars.html)

Forex Blog: You wrote a recent post outlining the US Dollar carry trade, and how you believe that the Dollar’s decline is
cyclical/temporary rather than structural/permanent. Can you elaborate on this idea? Do you think it’s possible that the fervor
with which investors have sold off the Dollar suggests that it could be a little of both? Well, first of all, there is more than one
thing happening here, so I would definitely agree from the outset, there are both cyclical and structural elements in play.
Structurally, the architecture of Bretton Woods II is creaking round the edges, and in the
longer run we are looking at a relative decline in the dollar, but as Keynes reminded us, in the long run we are all dead, while
as I noted in the Afoe post, news of the early demise of the dollar is surely vastly overstated. Put
another way, while Bretton Woods II has surely seen its best days, till we have some idea what can replace it it
is hard to see a major structural adjustment in the dollar. Europe’s economies are not strong
enough for the Euro to simply step into the hole left by the dollar, the Chinese, as we know, are
reluctant to see the dollar slide too far due to the losses they would take on dollar
denominated instruments, while the Russians seem to constantly talk the USD down, while at
the same time borrowing in that very same currency – so read this as you will. Personally, I cannot envisage a long
term and durable alternative to the current set-up that doesn’t involve the Rupee and the Real, but these currencies are surely
not ready for this kind of role at this point. So we will stagger on.

Dollar heg isn’t key to the economy

Palley 2- US based economist, Director of Globalization Reform Project Open Society Institute (Thomas Palley, “THE OVER-
VALUED DOLLAR AND THE U.S. SLUMP,” October 2002,
http://www.thomaspalley.com/docs/articles/macro_policy/overvalued_dollar.pdf)

The arguments against an over-valued dollar are compelling, yet some continue to argue that a “strong” dollar is desirable. One
argument is that the strong dollar helps keep down inflation by lowering import prices and keeping the lid on
prices of domestic manufacturers. This argument had some support in the late 1990s when the U.S. was in the midst
of a huge creditdriven boom, but that is no longer the case. Inflation is not an imminent economic
danger, and there are reasons to believe that deflation is actually the greater danger given the highly indebted
state of the U.S. economy. In these circumstances, slightly higher inflation could be a benefit to the extent that it reduces debt
burdens. A
second argument is that a strong dollar is needed to finance the trade deficit. This
argument has the reasoning backward. There is a need to finance the trade deficit because the dollar is
hugely over-valued. Absent this over-valuation, exports would be higher and imports lower,
which would diminish the trade deficit and the amount needed to finance it. The above financing
argument also links with claims that the U.S. trade deficit is the product of inadequate domestic saving rather than the over-
valued dollar. However, these undersaving claims misunderstand the nature of the national income identity from which they
derive.

Dollar hegemony is resilient


Kritzer 10 Adam is the lead editor of the Forex Blog and contributor to other leading financial news sites. B.A. in Economics from Penn, "Dollar Returns to Favor as
World’s Reserve Currency" March 16 www.forexblog.org/2010/03/dollar-returns-to-favor-as-worlds-reserve-currency.html

Rumor has it that the Dollar is about to make a run. As the credit crisis slowly subsides, (currency) investors are once again
looking at the long-term, and they like what they see when it comes to the Dollar.¶ For those that care to
remember, 2008 was a great year for the Dollar, as the credit crisis precipitated an increase in risk aversion, and investors
realized that despite its pitfalls, the Dollar was (and still is) the most stable and really the only viable
global reserve currency. [This reversed a trend which had essentially been in place since the inception of the Euro in 1999].
In 2009, meanwhile, the Dollar resumed its multi-year decline, and many analysts were quick to label the rally of 2008 as an
aberration.¶ Then came the debt crises, first in Dubai, then in Greece. Suddenly, a handful of smaller EU countries appeared
vulnerable to fiscal crises. Japan officially became the first of the Aaa economies to receive a downgrade in its credit rating. The
British Pound is dealing with crises on both the political and economic fronts. According to Moody’s, “The ratings of the Aaa
governments — which also include Britain, France, Spain and the Nordic countries — are currently ’stable’…But…their ‘distance-
to-downgrade’ has in all cases substantially diminished.” Suddenly, the Greenback doesn’t look so bad.¶ I want to point out that
in forex, everything is relative. (Novice) forex investors are often baffled by how sustained economic and financial crises don’t
immediately result in currency depreciation. The explanation is that when the crises are worse in (every) other countries, the
base currency still looks attractive.¶ This is precisely the case when it comes to the US Dollar. To be sure, theeconomy is
still flawed, financial markets have yet to fully to recover, the federal budget deficit topped $1.8 Trillion in 2009, and
government finances seem close to the breaking point. Moody’s has also identified the US as a candidate for a ratings
downgrade. And yet, when you look at the situation in every other currency that currently rivals the US for
reserve currency status, the
Dollar still wins hands down.¶ Its economy is the world’s largest. So are its
financial markets, which are also the deepest and most liquid. Its sovereign finances are still manageable from the
standpoint of debt-to-GDP and interest-to-revenue ratios. It is the only currency whose circulation can even come close to
meeting the needs of global trade. Summarized S&P – when it confirmed the AAA credit rating of the US, “ The dollar’s
widespread acceptance stems from the U.S. economy’s fundamental strength, which in our view
comes from the economy’s size and the flexibility of labor and product markets. We view U.S. banking and capital markets to be
dynamic and unfettered relative to their peers.”¶ That’s why auctions of US Treasury bonds remain heavily oversubscribed
(demand exceeds supply), despite the rock-bottom interest coupons. China has reaffirmed its commitment to Treasuries (what
other choice does it have), confirmed by some forensic accounting work. Gold might continue to rally. So will other
commodities, for all I know. Emerging market currencies are still in good shape as well, but none of these will seriously rival the
US Dollar for a long-time, if ever. In short, when it comes to the other majors, the Dollar is still King: “You can say
whatever you want, but the dollar is the currency of last resort. It’s the currency people want in a crisis.”

No impact to dollar decline---it would cause stable duopoly with the euro

Bergsten 9 C. Fred Bergsten, Peterson Institute for International Economics, “The Dollar and the Deficits: How Washington Can
Prevent the Next Crisis,” Article in Foreign Affairs, Volume 88 No. 6, November/December 2009

Both the United States and the rest of the world have an interest in continued globalization
and efficient international financial markets, and so neither has any interest in entirely
eliminating the international role of the dollar. In any case, inertia is such a powerful force in
financial matters that a sweeping step of this kind is technically impossible. Instead, the United
States should encourage two eminently feasible changes in the current international monetary order. The first is the further
evolution of a multiple-currency system in which other monies increasingly share the international position of the dollar in
private markets. The euro, based on a collective European economy as large as the United States' and with capital markets as
extensive in most respects, is the most obvious candidate. The euro already rivals the dollar in some domains, such
as currency holdings and private bond placements, and will become a full competitor whenever the eurozone countries adopt a
more common fiscal policy. The Chinese renminbi is likely to acquire a significant international role once China allows it to be
converted for financial as well as current account transactions and eases capital controls. Some observers
fear that a
system of multiple currencies is inherently unstable. However, such a regime functioned
smoothly for several decades before World War I, and a pound-dollar duopoly existed throughout
the 1920s. A dollar-euro duopoly has already begun to emerge over the last decade. Competition
between national currencies is likely to improve economic policies and performance by forcing market
discipline on the governments and central banks behind these alternative currencies.

There’s no contender to replace the dollar

Heaton 9-staff writer for Seeking Alpha (Chris Heaton, “The Dollar is Safe for Now,” 5/8/09
http://seekingalpha.com/article/136361-the-dollar-is-safe-for-now)

What's more, there's


no real contender to replace the dollar. The euro is the closest, but it has
plenty of problems of its own. And any idea that the renminbi is ready to take over is a
fantasy. China is moving to make its currency more of a factor in international finance, by
establishing swaps with other countries that would potentially allow bilateral trade to be
settled in renminbi. But these are very early steps. The renminbi is not yet freely convertible into other
currencies. And even if the Chinese government were to make it convertible, there simply aren't enough renminbi-denominated
assets yet for foreign renminbi holders to store their wealth in. A
crucial part of the US dollar hegemony is
America's deep and highly liquid capital markets.

Dollar is resilient

EH 8 (Economics Help, “US Dollar Predictions 2009”, 12-18, http://www.economicshelp.org/blog/economics/us-dollar-predictions-


2009/)

It is difficult to predict the dollar because there are few different factors pulling the dollar in different directions. Firstly, the
dollar has been surprisingly resilient since the US slipped into its worst recession and US interest rates tumbled to
near 0%. The resilience of the dollar is not based on economic fundamentals but, a general unwinding of
positions and the ‘dash for cash’ Firstly, the dollar benefit from hedge funds and investment trusts
deciding to get out of emerging economies . As they sold securities in emerging economies they generally
were sold for dollars increasing demand for dollars. Secondly, the prospect of deflation means
that people are wanting to hold more cash. In many economies it is the dollar which is seen as the reserve currency, so
there has been an increase in demand for dollar holding s as security against deflation and falling
curencies. 
Econ Answers
Frontline

Econ resilient, US isn’t key, and impact empirically denied

Lamy ’11(Pascal Lamy is the Director-General of the World Trade Organization. Lamy is Honorary President of Paris-based think tank Notre Europe.

Lamy graduated from the prestigious Sciences Po Paris, from HEC and ÉNA, graduating second in his year of those specializing in economics. “System

Upgrade” BY PASCAL LAMY | APRIL 18, 2011)

The bigger test came with the 2008-2009 Great Recession, the first truly global recession since World War II. When
the international economy went into free fall, trade went right along with it. Production and supply are today thoroughly global in nature, with most

manufactured products made from parts and materials imported from many other countries. These global value chains have a multiplier effect on trade

statistics, which explains why, as the global economy contracted by 2 percent in 2009, trade volume shrank by more than 12 percent. This multiplier

effect works the other way around as well: Growth returned to 4.6 percent and trade volume grew by a record 14.5 percent over the course

of 2010. Projections for trade in 2011 are also strong, with WTO economists predicting that trade volume will rise

6.5 percent during the current year. This sharp rebound in trade has proved two essential things: Markets stayed

open despite ever-stronger pressures to close them, and trade is an indispensible tool for economic recovery,

particularly for developing countries, which are more dependent on trade. Shortly after the crisis broke out, we in

the WTO began to closely monitor the trade policy response of our member governments. Many were fearful that

pressures to impose trade restrictions would prove too powerful for governments to resist. But this is not what

happened. Instead, the system of rules and disciplines, agreed to over 60 years of negotiations, held firm. In a

series of reports prepared for WTO members and the G-20, we found that governments acted with great

restraint. At no time did the trade-restrictive measures imposed cover more than 2 percent of world imports. Moreover, the measures used --

anti-dumping duties, safeguards, and countervailing duties to offset export or production subsidies -- were those which, in the right

circumstances, are permissible under WTO rules. I am not suggesting that every safeguard measure or countervailing duty imposed
during those difficult days was in compliance with WTO rules, but responses to trade pressures were generally undertaken within an internationally

agreed-upon framework. Countries by and large resisted overtly noncompliant measures, such as breaking legally binding tariff ceilings or

imposing import bans or quotas. As markets stayed open, trade flows began to shift, and countries that shrugged off the impact

of the crisis and continued to grow -- notably China, India, and Brazil -- became ever-more attractive markets for

countries that were struggling, including those in Europe and North America. Trade has been a powerful engine for growth in the
developing world, a fact reflected in the far greater trade-to-GDP ratios we see there. In 2010, developing countries' share of world trade expanded to a

record 45 percent, and this trend looks set to continue. Decisions made in Brasilia, Beijing, and New Delhi to open their respective economies to trade

have been instrumental in enabling these countries to lift hundreds of millions of people out of poverty.
Best study proves no conflict from econ decline

Brandt and Ulfelder ‘11 (*Patrick T. Brandt, Ph.D. in Political Science from Indiana University, is an Assistant Professor of Political Science in the
School of Social Science at the University of Texas at Dallas. **Jay Ulfelder, Ph.D. in political science from Stanford University, is an American political

scientist whose research interests include democratization, civil unrest, and violent conflict, April, 2011, “Economic Growth and Political Instability,”

Social Science Research Network)

These statements anticipating political fallout from the global economic crisis of 2008–2010 reflect a widely held view that
economic growth has rapid and profound effects on countries’ political stability. When economies grow at a healthy clip, citizens are presumed to be too

busy and too content to engage in protest or rebellion, and governments are thought to be flush with revenues they can use to enhance their own

stability by producing public goods or rewarding cronies, depending on the type of regime they inhabit. When growth slows, however, citizens and

cronies alike are presumed to grow frustrated with their governments, and the leaders at the receiving end of that frustration are thought to lack the

financial resources to respond effectively. The expected result is an increase in the risks of social unrest, civil war, coup attempts, and regime

breakdown. Although it is pervasive, the assumption that countries’ economic growth rates strongly affect their political

stability has not been subjected to a great deal of careful empirical analysis, and evidence from social science

research to date does not unambiguously support it. Theoretical models of civil wars, coups d’etat, and transitions to and from

democracy often specify slow economic growth as an important cause or catalyst of those events, but empirical studies on the

effects of economic growth on these phenomena have produced mixed results. Meanwhile, the effects of economic

growth on the occurrence or incidence of social unrest seem to have hardly been studied in recent years, as empirical analysis of

contentious collective action has concentrated on political opportunity structures and dynamics of protest and repression. This paper helps fill

that gap by rigorously re-examining the effects of short-term variations in economic growth on the occurrence of

several forms of political instability in countries worldwide over the past few decades. In this paper, we do not seek to
develop and test new theories of political instability. Instead, we aim to subject a hypothesis common to many prior theories of political instability to

more careful empirical scrutiny. The goal is to provide a detailed empirical characterization of the relationship between economic growth and political

instability in a broad sense. In effect, we describe the conventional wisdom as seen in the data. We do so with statistical models that use smoothing

splines and multiple lags to allow for nonlinear and dynamic effects from economic growth on political stability. We also do so with an instrumented

measure of growth that explicitly accounts for endogeneity in the relationship between political instability and economic growth. To our knowledge,

ours is the first statistical study of this relationship to simultaneously address the possibility of nonlinearity and problems of

endogeneity. As such, we believe this paper offers what is probably the most rigorous general evaluation of this

argument to date. As the results show, some of our findings are surprising. Consistent with conventional assumptions, we find that social unrest
and civil violence are more likely to occur and democratic regimes are more susceptible to coup attempts around periods of slow economic growth. At

the same time, our analysis shows no significant relationship between variation in growth and the risk of civil-war onset, and results from our analysis of

regime changes contradict the widely accepted claim that economic crises cause transitions from autocracy to democracy. While we would hardly
pretend to have the last word on any of these relationships, our findings do suggest that the relationship between economic growth and

political stability is neither as uniform nor as strong as the conventional wisdom(s) presume(s). We think these

findings also help explain why the global recession of 2008–2010 has failed thus far to produce the wave of coups and

regime failures that some observers had anticipated, in spite of the expected and apparent uptick in social unrest associated with the crisis.

US not key to global economy

Economist ’10 (The odd decouple Theories about why some rich-world economies are doing better than America’s don’t stand
up Sep 2nd 2010 | from the print edition http://www.economist.com/node/16943853

AMERICA is used to making the economic weather . It has the world's largest economy, its most influential central
bank and it issues the main global reserve currency. In recent months, however, some rich-world economies
(notably Germany's) have basked in the sunshine even as the clouds gathered over America . On
August 27th America's second-quarter GDP growth was revised down to an annualised 1.6%. That looked moribund compared with
the 9% rate confirmed in Germany a few days earlier. America's jobless rate was 9.5% in July (figures for August were released on
September 3rd, after The Economist went to press). But in Germany the unemployment rate is lower even than before the
downturn. Other rich countries, including
Britain and Australia, have enjoyed sprightlier recent GDP
growth and lower unemployment than America. This unusual divergence within the rich world has
fostered many competing theories to explain it, including differences in fiscal policies, exchange rates and debt levels.
Most of these do not quite fit the facts. On one account Germany and, to a lesser extent, Britain have been rewarded for taking a
firm grip on their public finances. In this view, the promise to tackle budget deficits has had a liberating effect on private spending by
reducing uncertainty. In America, by contrast, anxiety about public debt is making businesses and consumers tighten their purse
strings. The theory is a little too neat. Although credible plans to curb deficits are helpful to medium-term growth, they are unlikely
to explain sudden spurts. Britain's budget plans were announced towards the end of the quarter, on June 22nd. Germany's were set
out two weeks earlier. They could scarcely explain why GDP growth was strong. Indeed for most of the second quarter, fiscal
uncertainty hung over both countries: in Britain because of a close election, in Germany because of commitments to help Greece
and other countries. And the immediate impact of austerity is to dampen growth: witness the slump in Greece. Perhaps the
explanation is found in currency movements. One effect of the euro-area crisis was to push the euro down against the dollar in the
early months of this year—helping German firms but harming American exporters. Much of Germany's second-quarter GDP growth
came from trade, even as a wider trade gap sapped America's economy. A weak pound could also explain Britain's renewed
economic strength, much as a surge in the yen has increased worries about Japan. On August 30th Japan's central bank said it would
offer banks ¥10 trillion ($118 billion) of six-month secured loans at its benchmark interest rate of 0.1%, on top of the ¥20 trillion of
three-month loans it had already pledged. It hopes that this flood of money will push down borrowing costs, cap the yen's rise and
help exporters. The currency theory also has holes in it. The yen's surge is too recent to explain why Japan's GDP barely rose in the
second quarter. Net trade added almost nothing to Britain's GDP growth in the last quarter. Indeed America's export growth has
been much stronger (a sudden surge in imports was behind the second-quarter trade gap). And demand for the sort of exports
Germany has done well with, mostly luxury cars and specialist capital goods, tends to be insensitive to shifts in the exchange rate.
Britain is an awkward challenger to another theory: that a debt hangover is holding back consumers in countries that had housing
booms. Consumer spending in Britain (and in America) rose at about the same rate as in thriftier Germany during the second
quarter. Britain stands out in another respect, too: its unemployment rate has risen by far less than in other places that had also
racked up big mortgage debts. Divergent trends in unemployment may be better explained by the sort of recession each country had
than by variations in jobs-market flexibility, says Kevin Daly at Goldman Sachs. In America, Ireland and Spain, the collapse of labour-
intensive construction swelled the dole queues. Britain also had a housing boom but its tight planning laws kept its construction
industry small, so fewer jobs were lost when the bust came. The downturns in Japan and Germany, deeper than America's (see
chart), were mainly caused by the collapse in world trade. That hurt capital-intensive export industries—which were also more likely
to rebound quickly—so fewer jobs disappeared. Some think America's
slowness to create new jobs is leading to
undue pessimism about the rest of the world's prospects . “If US growth is not enough to give
us a big payrolls figure, it's deemed a disaster,” says Marco Annunziata at UniCredit. But fast-growing
emerging markets, such as China , have kept the world economy ticking over . Germany has done
well because its exporters have made headway there . China's vibrancy also explains why Australia's GDP rose at
its fastest rate for three years in the second quarter.

Impact empirically denied- zero risk

Barnett ‘9 (Thomas P.M. Barnett, senior managing director of Enterra Solutions LLC, “The New Rules: Security Remains Stable
Amid Financial Crisis,” 8/25/2009)

When the global financial crisis struck roughly a year ago, the blogosphere was ablaze with all
sorts of scary predictions of, and commentary regarding, ensuing conflict and wars -- a rerun of the
Great Depression leading to world war, as it were. Now, as global economic news brightens and recovery -- surprisingly led by
China and emerging markets -- is the talk of the day, it's interesting to look back over the past year and realize how
globalization's first truly worldwide recession has had virtually no impact whatsoever on the
international security landscape. None of the more than three-dozen ongoing conflicts listed by
GlobalSecurity.org can be clearly attributed to the global recession . Indeed, the last new entry (civil conflict
between Hamas and Fatah in the Palestine) predates the economic crisis by a year, and three quarters of the chronic struggles began
in the last century. Ditto for the 15 low-intensity conflicts listed by Wikipedia (where the latest entry is the Mexican "drug war"
begun in 2006). Certainly, the Russia-Georgia conflict last August was specifically timed, but by most accounts the opening ceremony
of the Beijing Olympics was the most important external trigger (followed by the U.S. presidential campaign) for that sudden spike in
an almost two-decade long struggle between Georgia and its two breakaway regions. Looking over the various databases, then, we
see a most familiar picture: the usual mix of civil conflicts, insurgencies, and liberation-themed terrorist movements. Besides the
recent Russia-Georgia dust-up, the only two potential state-on-state wars (North v. South Korea, Israel v. Iran) are both tied to one
side acquiring a nuclear weapon capacity -- a process wholly unrelated to global economic trends. And with the United States
effectively tied down by its two ongoing major interventions (Iraq and Afghanistan-bleeding-into-Pakistan), our involvement
elsewhere around the planet has been quite modest, both leading up to and following the onset of the economic crisis: e.g., the
usual counter-drug efforts in Latin America, the usual military exercises with allies across Asia, mixing it up with pirates off Somalia's
coast). Everywhere else we find serious instability we pretty much let it burn, occasionally pressing the Chinese -- unsuccessfully -- to
do something. Our new Africa Command, for example, hasn't led us to anything beyond advising and training local forces. So, to sum
up: * No significant uptick in mass violence or unrest (remember the smattering of urban riots last year in places like Greece,
Moldova and Latvia?); * The usual frequency maintained in civil conflicts (in all the usual places); * Not a single state-on-state war
directly caused (and no great-power-on-great-power crises even triggered); * No great improvement or disruption in great-power
cooperation regarding the emergence of new nuclear powers (despite all that diplomacy); * A modest scaling back of international
policing efforts by the system's acknowledged Leviathan power (inevitable given the strain); and * No serious efforts by any rising
great power to challenge that Leviathan or supplant its role. (The worst things we can cite are Moscow's occasional deployments of
strategic assets to the Western hemisphere and its weak efforts to outbid the United States on basing rights in Kyrgyzstan; but the
best include China and India stepping up their aid and investments in Afghanistan and Iraq.) Sure, we've
finally seen global
defense spending surpass the previous world record set in the late 1980s, but even that is likely to
wane given the stress on public budgets created by all this unprecedented "stimulus" spending.
If anything, the friendly cooperation on such stimulus packaging was the most notable great-
power dynamic caused by the crisis. Can we say that the world has suffered a distinct shift to political radicalism as a
result of the economic crisis? Indeed, no. The world's major economies remain governed by center-left or
center-right political factions that remain decidedly friendly to both markets and trade. In the short
run, there were attempts across the board to insulate economies from immediate damage (in effect,
as much protectionism as allowed under current trade rules), but there was no great slide into "trade wars."
Instead, the World Trade Organization is functioning as it was designed to function, and regional
efforts toward free-trade agreements have not slowed . Can we say Islamic radicalism was inflamed by the
economic crisis? If it was, that shift was clearly overwhelmed by the Islamic world's growing disenchantment with the brutality
displayed by violent extremist groups such as al-Qaida. And looking forward, austere economic times are just as likely to breed
connecting evangelicalism as disconnecting fundamentalism. At the end of the day, the economic crisis did not prove to be
sufficiently frightening to provoke major economies into establishing global regulatory schemes, even as it has sparked a spirited --
and much needed, as I argued last week -- discussion of the continuing viability of the U.S. dollar as the world's primary reserve
currency. Naturally,
plenty of experts and pundits have attached great significance to this debate,
seeing in it the beginning of "economic warfare" and the like between "fading" America and
"rising" China. And yet, in a world of globally integrated production chains and interconnected
financial markets, such "diverging interests" hardly constitute signposts for wars up ahead.
Frankly, I don't welcome a world in which America's fiscal profligacy goes undisciplined, so bring
it on -- please! Add it all up and it's fair to say that this global financial crisis has proven the great
resilience of America's post-World War II international liberal trade order.

History disproves causality between crisis and war

Ferguson 6 (Niall, Laurence A. Tisch Professor of History at Harvard, a Senior Research Fellow of Jesus College at Oxford, and a
Senior Fellow of the Hoover Institution, “The War of the World”, Penguin Books, pg. xxxviii)

Nor can economic crises explain all the violent upheavals of the century. As noted already, perhaps the
most familiar causal chain in modern historiography leads from the Great Depression to the rise of fascism
and the outbreak of war. Yet on closer inspection this pleasing story falls apart. Not all the countries
affected by the Great Depression became fascist regimes; nor did all the fascist regimes engage
in wars of aggression. Nazi Germany started the war in Europe, but only after its economy had recovered
from the Depression. The Soviet Union, which started the war on Hitler’s side, was cut off from the world economic
crisis, yet ended up mobilizing and losing more soldiers than any other combatant. For the century as a whole,
no general rule is discernible. Some wars came after periods of growth; others were the causes
rather than the consequence of economic crisis. And some severe economic crisis did not lead to wars.
Certainly, it is now impossible to argue (thought Marxists long tried to) that the First World War was the result of a crisis of
capitalism; on the contrary, it abruptly terminated a period of extraordinary global economic integration with relatively high growth
and low inflation.
---Econ- Resilient

1NC Lamy—the economy is resilient—all metrics prove governments stay open and
cooperative—keeps economy afloat—08 crisis proves shifting trade flows and flexible public
and private sectors can solve much larger internal links to the econ than theirs

Its 2013- proves global economic resilient- Sandy and EU prove

Eberly ’13 (Jan Eberly, Assistant Secretary for Economic Policy for the Treasury Borrowing Advisory Committee of the Securities
Industry and Financial Markets Association, “Statement by Assistant Secretary for Economic Policy Jan Eberly for the Treasury
Borrowing Advisory Committee of the Securities Industry and Financial Markets Association”, February 4, 2013)

WASHINGTON - Economic recovery in the U.S. continued at a moderate pace over the course of 2012, with real GDP expanding by
1.5 percent following a 2.6 percent increase during 2011. After thirteen straight quarters of growth, real GDP edged down slightly in
the final quarter of last year, as sharply lower defense spending, slower inventory growth, and a widening of the trade deficit offset a
solid increase in consumer spending and strong growth of both residential investment and business capital spending. Job creation
has accelerated in recent months. The unemployment rate declined notably over the first nine months of 2012 and has been little
changed since September. The economy sustained a number of temporary shocks last year, such as a jump
in energy prices early in 2012, a severe drought during the summer, and Hurricane Sandy in late October, and also
contended with the ongoing sovereign debt crisis in Europe and a more general slowdown in global growth. Growth
is expected to pick up in the first quarter of 2013, despite some fiscal drag. Other potential challenges this year
include the risk of renewed setbacks in Europe, the impact of continued uncertainty about the U.S. fiscal situation ,
and the possibility of additional, sequester-related fiscal tightening. Even so, private forecasters anticipate a
gradual acceleration in the pace of expansion as 2013 unfolds, as well as further progress in reducing unemployment. According
to the advance report released last week, real GDP edged down 0.1 percent at an annual rate in the fourth quarter, compared with a
3.1 percent advance in the third quarter. The swing was due in part to a 6.6 decline in government spending. Federal outlays fell
15.0 percent – the largest quarterly decline in four decades – as federal defense purchases plummeted 22.2 percent. In the third
quarter, federal spending rose sharply, boosted by a jump in defense outlays. The composition of the pronounced swing between Q3
and Q4 suggests that uncertainty about the impending sequester played a role. State and local government spending, which has
been falling nearly continuously since late 2009, declined 0.7 percent in Q4. Altogether, the decline in government expenditures cut
1¼ percentage points from real GDP growth in Q4. GDP growth in late 2012 was also held back by a sharp slowdown in private
inventory accumulation, which subtracted 1¼ percentage points from real GDP in the fourth quarter after adding 0.7 percentage
point to growth in the third quarter. The drought-related drawdown in farm inventories, which reduced GDP growth in the prior two
quarters, slowed. A wider trade deficit subtracted an additional ¼ percentage point from GDP growth in the fourth quarter.
Notwithstanding the slight dip in headline GDP, the main components of underlying private demand strengthened in the fourth
quarter. Consumer spending, which accounts for roughly two-thirds of GDP, grew by 2.2 percent at an annual rate, accelerating from
the third quarter’s 1.6 percent rise, and adding 1.5 percentage points to real growth. Business fixed investment grew 8.4 percent in
the fourth quarter, contributing 0.8 percentage point to growth. Equipment and software investment rose at a 12.4 percent pace
after falling by 2.6 percent in the prior quarter. Residential investment grew by 15.3 percent at an annual rate in the fourth quarter,
up from 13.5 percent in the third quarter, and contributed 0.4 percentage point to GDP growth. Residential investment
has increased in each of the past seven quarters -- the first such string of advances in this sector since 2005 – and has grown at
an average annual rate of almost 11 percent per quarter over this period. Private domestic final purchases (the sum of consumption,
business fixed investment, and residential investment) jumped by 3.3 percent at an annual rate in the fourth quarter, more than
double the third’s quarter’s 1.5 percent pace. Over the past three years, this marker of a private-sector led, self-
sustaining recovery has grown at an average annual rate of just under 3 percent. Labor market conditions continue to
improve at a steady but gradual pace, and the most recent data show that job creation at the end of 2012 was actually faster than
initially reported. Private-sector job growth averaged 225,000 per month during the fourth quarter, up from 142,000 in the third
quarter, and nearly double the 117,000 jobs per month created on average in the second quarter. More than 6.1 million new jobs
have been created in the private sector since the employment trough in February 2010. Moreover, underlying labor demand
appears to be improving. The average private-sector workweek stood at 34.4 hours in January, up from a low of 33.8 hours in 2009
and just 0.2 hour shorter than in December 2007. The unemployment rate stood at 7.9 percent in January, up slightly from a near
four-year low of 7.8 percent in November and December. Measures of longer-term unemployment as well as marginal attachment
to the labor force and part-time employment continue to trend lower. The median duration of unemployment fell by 4.8 weeks over
the past year to 16.0 weeks in January and is down from a high of 24.8 weeks in mid-2010. It is worthwhile to look at progress across
the country, too: in December, 25 states reported unemployment rates that were significantly below the national average. These are
all positive signs that underlying labor market conditions continue to firm. With the progress made in the housing market in the past
several months, we now appear to be approaching important milestones. For example, total housing starts rose in December to a 4½
year high and the number of residential building permits issued reached their highest level since mid-2008. As of December, total
existing home sales had retraced to a level about two-thirds of their 2005 peak, and the decline in new single-family homes during
that month was actually attributed to a lack of supply, rather than a dearth of demand – sales in this category were still up nearly 9
percent year-over-year. The inventory of unsold new homes is just above record lows for the series, which dates to the early 1960s,
and the inventory of existing homes available for sale continues to move lower and is now two-thirds below its July 2010 peak level.
During 2012, residential investment climbed 14.4 percent – the strongest yearly increase since 1983. The major house price indexes
have been moving higher on a year-over-year basis for the past ten months, and are now being supported by tighter supply and
stronger demand conditions. Record or near-record lows in mortgage rates, a relatively high level of housing affordability, and
improving household wealth are also helping to boost demand and to support broader-based improvement in the housing sector.
Looking ahead, downside risks to U.S. economic activity remain, including persistent concerns about instability in European
sovereign debt markets. Here at home, consumer sentiment faltered at the turn of the year in the face of fiscal uncertainty and the
expiration of tax cuts. Still, energy prices have eased in very recent months, and there are signs of reviving demand in Asia. While
downside risks create vulnerabilities in any economy, recent
progress within the U.S. has improved the economy’s
resilience in the face of potential challenges. The underlying and consistent strength of private demand over the
past three years constitutes an important foundation for that resilience, and the level of real GDP is now 2.4
percent higher than in the fourth quarter of 2007, at the time of the previous expansion’s peak. After five years of decline,
residential investment has added to growth in each of the past seven quarters. The workweek has lengthened to a duration close to
that last seen in December 2007, the peak month of the previous upturn, and the unemployment rate is at a four-year low. These
are important milestones for consumers as well as the housing and labor markets, and are evidence of a moderate and steady
forward movement.

Resiliency – FDIC and treasury

Amadeo ’13 (February 12, 2013 U.S. Economy Collapse By Kimberly Amadeo, About.com Guide,
http://useconomy.about.com/od/criticalssues/p/US-Economy-Collapse.htm

The U.S. economy is so large and resilient , it is highly unlikely that even these events could
create a collapse. Hyperinflation is easily tamed by the Federal Reserve's contractionary
monetary tools. The FDIC insures banks, and the Treasury can print all the money needed to make
sure depositors get their funds. Homeland Security can address the cyber-threat. If not, eventually the economy can always return
to how it functioned before the Internet. The Strategic Oil Reserves can be released to offset an oil embargo. The
U.S. military
can respond to a terrorist attack, transportation stoppage, or rioting/civil war. In other words, most Federal
government programs are designed to prevent just such an economic collapse.
debt.

No impact to another recession.


Keystone Research 11 [Main Street Newsletter, “3 Ways the Next Recession Will Be Different”,
http://keystoneresearch.org/media-center/media-coverage/3-ways-next-recession-will-be-different]

All of this has only renewed concerns among analysts and average Americans that the U.S. would suffer a dreaded double-dip
recession, but according to several economists MainStreet spoke with, even
if we do enter into another
recession later this year or in early 2012, it won’t be nearly as damaging as the Great
Recession of 2008.“If there is another recession, I think it wouldn’t be as severe and it
would also be shorter,” says Gus Faucher, senior economist at Moody’s Analytics. “And the
reason for that is a lot of the imbalances that drove the previous recession have been
corrected.” As Faucher and others point out, banks are better capitalized now, the
housing market has shed (however painfully) many delinquent homeowners who signed
up forsubprime mortgages before the recession and U.S. corporations have trimmed their
payrolls and are sitting on ample cash reserves to help weather another storm.

The economy is resilient

Economist ‘11, Economist Intelligence Unit – Global Forecasting Service, 11/16/’11 (http://gfs.eiu.com/Article.aspx?
articleType=gef&articleId=668596451&secID=7)

The US economy, by any standard, remains weak, and consumer and business sentiment are close to 2009 lows. That said, the
economy has been surprisingly resilient in the face of so many shocks. US real GDP expanded by a relatively
robust 2.5% in the third quarter of 2011, twice the rate of the previous quarter. Consumer spending rose by 2.4%,
which is impressive given that real incomes dropped during the quarter (the savings rate fell, which helps to explain
the anomaly.) Historically, US consumers have been willing to spend even in difficult times. Before the
2008-09 slump, personal spending rose in every quarter between 1992 and 2007. That resilience is again in evidence: retail sales
in September were at a seven-month high , and sales at chain stores have been strong. Business investment
has been even more buoyant: it expanded in the third quarter by an impressive 16.3% at an annual rate, and spending by
companies in September on conventional capital goods (that is, excluding defence and aircraft) grew by the most
since March. This has been made possible , in part, by strong corporate profits. According to data compiled
by Bloomberg, earnings for US companies in the S&P 500 rose by 24% year on year in the third quarter. All of this has
occurred despite a debilitating fiscal debate in Washington, a sovereign debt downgrade by a major
ratings agency and exceptional volatility in capital markets. This reinforces our view that the US economy,
although weak, is not in danger of falling into a recession (absent a shock from the euro zone). US growth will,
however, continue to be held back by a weak labour market—the unemployment rate has been at or above 9% for 28 of the last 30
months—and by a moribund housing market.

Growth despite shocks

Bloomberg 12 (“Fed’s Plosser Says U.S. Economy Proving Resilient to Shocks,” 5-9, http://www.bloomberg.com/news/2012-05-
09/fed-s-plosser-says-u-s-economy-proving-resilient-to-shocks.html)
Philadelphia Federal Reserve Bank President Charles Plosser said the U.S. economy has proven
“remarkably resilient” to shocks that can damage growth, including surging oil prices and
natural disasters. “The economy has now grown for 11 consecutive quarters ,” Plosser said today
according to remarks prepared for a speech at the Philadelphia Fed. “Growth is not robust. But growth in the past
year has continued despite significant risks and external and internal headwinds.” Plosser, who did
not discuss his economic outlook or the future for monetary policy, cited shocks to the economy last year, including
the tsunami in Japan that disrupted global supply chains , Europe’s credit crisis that has damaged
the continent’s banking system and political unrest in the Middle East and North Africa. “The U.S.
economy has a history of being remarkably resilient ,” said Plosser, who doesn’t have a vote on policy this year.
“These shocks held GDP growth to less than 1 percent in the first half of 2011, and many analysts were concerned that the economy
was heading toward a double dip. Yet, the economy proved resilient and growth picked up in the second half of the year.” Plosser
spoke at a conference at the Philadelphia Fed titled, “Reinventing Older Communities: Building Resilient Cities.” Urban Resilience His
regional bank’s research department is working on a project to measure the resilience of different cities, to learn more about the
reasons that some urban areas suffer more than others in downturns, Plosser said. He mentioned one early finding of the study:
Industrial diversity increases a city’s resilience. “I do want to caution you that resilient and vibrant communities are not just about
government programs or directed industrial planning by community leaders,” Plosser said. “ The
economic strength of our
country is deeply rooted in our market- based economy and the dynamism and resilience of its
citizenry.”

Regulations prevent crisis

Erskine ’10 (RETHINKING SECURITIES REGULATION AFTER THE CRISIS: AN ECONOMICS PERSPECTIVE Alex Erskine 1 9 July 2010 I.
INTRODUCTION AND SUMMARY

The Global Financial Crisis swept through financial markets in 2007 – 2009, leading to severe
losses of wealth and confidence and previously unexpected market failures. It caused a Great
Recession and a drastic decline in world trade and threatened an economic downturn akin to
the 1930’s Great Depression. Facing that threat, policymakers and regulators intervened on an
unprecedented scale to ward off such an outcome. While repercussions of the crisis persist, policymakers
and regulators are now considering how to ensure such a threat does not arise again. With the
benefit of hindsight, it is clear that the established ways of thinking about economic policy, prudential policy and securities
regulatory policy in part did cause the crisis and have to be rethought. To now rebuild without rethinking would expose the financial
system in future to a repeat of the crisis just past. This paper seeks to contribute to the rethinking of securities regulation, taking a
three step approach. 1. It reviews what we thought we knew. The conventional wisdom had a pro-market deregulatory mindset. 2. It
identifies what we learned from the crisis, highlighting where we were wrong and what we now know. There is a lot of new learning
to be done and the simplicities of the past have to be left behind. 3. It makes a very preliminary first pass at what this implies for the
future in terms of a new conceptual framework for securities regulators. We are definitely only near the start, and not the end, of
this journey. For simplicity, “securities regulation” is taken to refer not just to the regulation of equities, bonds and collective
investments, but also to the regulation of derivatives and other financial products. In some countries, the securities regulator also
regulates credit The paper's structure follows on from – and to some extent responds to – the staff position note "Rethinking
Macroeconomic Policy" by IMF Chief Economist Olivier Blanchard and two co-authors (Blanchard et al 2010). This paper seeks to
complement its root-and-branch rethinking. The paper sets out a preliminary view. Reticence in many ways has been the natural
order. Before the crisis economic policymaking and prudential supervision were widely seen to be more important for economic and
financial outcomes than was securities regulation. The GFC (or is it merely GFC 1?) confirmed that prejudice. The blame for the
financial instability and the main remedial actions both focused on misguided economic policy and weaknesses amongst prudentially
regulated financial intermediaries and supervision. The international discourse on reducing systemic risks has tended to focus
exclusively on banks as ‘systemically important institutions’ (e.g. IMF 2010), but is far from settled. In addition, in the three
years since the first clear signs of crisis, many immediate lessons have already been identified .
Detailed work to revise securities regulation is under way in every country and across the
world under the auspices of IOSCO and the G20. There has been progress on transparency ,
counterparty risk, hedge funds, securitisation, over-thecounter (OTC) derivatives, standards
for credit ratings agencies, unregulated entities, products and markets and accounting issues
and more.

Don’t believe the hype—pessimists will always exist—err on the side of economic data and
analysis

Globe and Mail ‘10 (5/31/10, BRIAN MILNER, "While gloom says bear, TIGER points to bull", lexis, WEA)

Even at the height of the remarkable rebound of 2009 that brought stocks back from the dead zone, the bears
never retreated to their lairs. Negative sentiment among investors remained stubbornly high, no
matter how promising the economic indicators looked. And then along came the Greeks and their little sovereign
debt problem, the Chinese and their public hand-wringing over asset bubbles and the North Koreans and their latest idiotic sabre-
ratting to remind nervous markets just how fragile the nascent global recovery could turn out to be. The latest survey of American
investors last week showed bearish sentiment hovering close to 30 per cent, with plenty of room for an uptick in the months ahead,
as the optimists come to realize that a V-shaped recovery was never in the cards after the worst global financial and economic crisis
since the Great Depression. The world's most overexposed permabear, Nouriel Roubini, is still grabbing headlines with his dire
Greece-is-just-the-tip-of-the-iceberg warnings. (Well, he does have a new book to sell.) And such high-profile Canadian bruins as
gold-loving money manager Eric Sprott and eminent strategist and data miner David Rosenberg have never veered from their
sombre outlooks. The fact that May turned into a particularly brutal month for just about everything but U.S. Treasuries - even after
last week's modest rebound, the Dow posted its worst performance for the month in 70 years - only added fuel to arguments that
worse, much worse, is yet to come. I mention all this to Eswar Prasad, when I reach the Cornell University economics professor at his
hotel in Beijing. Prof. Prasad is a noted China watcher who once headed the IMF's China division and still keeps in close touch with
top government finance officials. But on this call, I'm more interested in one of his other hats as a shrewd analyst of global economic
and market trends. "My inclination also is to be a bear," the affable academic says. "But the
data don't support my
bearishness as much as I would like. One has to be a little cautious, because these are based on
a variety of indicators. Some of them certainly show more strength than I had realized." The data
he's talking about come out of his work on a new composite index derived from a broad set of economic, market and confidence
measures in the G20 countries and designed to provide a quarterly snapshot of the global recovery. "All signs are that the recovery
has some momentum," says Prof. Prasad, who developed the index at the Brookings Institution, a Washington think tank where he is
also a senior fellow. "But I wouldn't call it solid enough momentum that we can consider it 'in the bag.'" The new index, cutely
named TIGER (Tracking Indices for the Global Economic Recovery), is a joint effort by Brookings and the Financial Times. And TIGER
shows that since the world began climbing out of the deep trough about the middle of last year,
big emerging economies have roared ahead, while the developed world has experienced much more uneven results.
Industrial production and trade have bounced back handsomely - total exports from the big emerging countries
now exceed pre-crisis levels - but the employment picture remains cloudy and consumption has yet to develop a new head of steam.
"It's
much easier at this stage to list all the things that could derail the recovery," Prof. Prasad
says. "But all of those things are still conjectural. The reality, and the data, is that things are
looking better."
It’s resilient—Central banks and global connectivity

Zakaria ‘9 (editor-at-large of Time magazine, former editor of Newsweek International, Ph.D. in political science from Harvard,
Fareed, 12/11. “The Secrets of Stability.” Newsweek.)

Beyond all this, though, I believe there's


a fundamental reason why we have not faced global collapse in
the last year.
It is the same reason that we weathered the stock-market crash of 1987, the
recession of 1992, the Asian crisis of 1997, the Russian default of 1998, and the tech-bubble
collapse of 2000. The current global economic system is inherently more resilient than we think.
The world today is characterized by three major forces for stability, each reinforcing the other and each historical in nature. The first
is the spread of great-power peace. Since the end of the Cold War, the world's major powers have not competed with each other in
geomilitary terms. There have been some political tensions, but measured by historical standards the globe today is stunningly free
of friction between the mightiest nations. This lack of conflict is extremely rare in history. You would have to go back at least 175
years, if not 400, to find any prolonged period like the one we are living in. The number of people who have died as a result of wars,
civil conflicts, and terrorism over the last 30 years has declined sharply (despite what you might think on the basis of overhyped
fears about terrorism). And no wonder—three decades ago, the Soviet Union was still funding militias, governments, and guerrillas
in dozens of countries around the world. And the United States was backing the other side in every one of those places. That clash of
superpower proxies caused enormous bloodshed and instability: recall that 3 million people died in Indochina alone during the
1970s. Nothing like that is happening today. Peace is like oxygen, Harvard's Joseph Nye has written. When you don't have it, it's all
you can think about, but when you do, you don't appreciate your good fortune. Peace allows for the possibility of a stable economic
life and trade. The peace that flowed from the end of the Cold War had a much larger effect because it was accompanied by the
discrediting of socialism. The world was left with a sole superpower but also a single workable economic model—capitalism—albeit
with many variants from Sweden to Hong Kong. This consensus enabled the expansion of the global economy; in fact, it created for
the first time a single world economy in which almost all countries across the globe were participants. That means everyone is
invested in the same system. Today, while the nations of Eastern Europe might face an economic crisis, no one is suggesting that
they abandon free-market capitalism and return to communism. In fact, around the world you see the opposite: even in the midst of
this downturn, there have been few successful electoral appeals for a turn to socialism or a rejection of the current framework of
political economy. Center-right parties have instead prospered in recent elections throughout the West. The
second force for
stability is the victory—after a decades-long struggle—over the cancer of inflation. Thirty-five years ago,
much of the world was plagued by high inflation, with deep social and political consequences .
Severe inflation can be far more disruptive than a recession , because while recessions rob you of better jobs
and wages that you might have had in the future, inflation robs you of what you have now by destroying your savings. In many
countries in the 1970s, hyperinflation led to the destruction of the middle class, which was the background condition for many of the
political dramas of the era—coups in Latin America, the suspension of democracy in India, the overthrow of the shah in Iran. But
then in 1979, the tide began to turn when Paul Volcker took over the U.S. Federal Reserve and waged war against inflation. Over two
decades, central banks managed to decisively beat down the beast. At this point, only one country in the world
suffers from -hyperinflation: Zimbabwe. Low inflation allows people, businesses, and governments to plan
for the future, a key precondition for stability. Political and economic stability have each
reinforced the other. And the third force that has underpinned the resilience of the global system
is technological connectivity. Globalization has always existed in a sense in the modern world, but until recently its
contours were mostly limited to trade: countries made goods and sold them abroad. Today the information revolution
has created a much more deeply connected global system. Managers in Arkansas can work with suppliers in
Beijing on a real-time basis. The production of almost every complex manufactured product now involves input from a dozen
countries in a tight global supply chain. And the consequences of connectivity go well beyond economics. Women in rural India have
learned through satellite television about the independence of women in more modern countries. Citizens in Iran have used cell
phones and the Internet to connect to their well-wishers beyond their borders. Globalization today is fundamentally about
knowledge being dispersed across our world. This
diffusion of knowledge may actually be the most important
reason for the stability of the current system. The majority of the world's nations have learned
some basic lessons about political well-being and wealth creation. They have taken advantage of
the opportunities provided by peace, low inflation, and technology to plug in to the global system. And they
have seen the indisputable results. Despite all the turmoil of the past year, it's important to remember that more
people have been lifted out of poverty over the last two decades than in the preceding 10. Clear-thinking
citizens around
the world are determined not to lose these gains by falling for some ideological chimera, or
searching for a worker's utopia. They are even cautious about the appeals of hypernationalism
and war. Most have been there, done that. And they know the price .

Global trading system prevents a crash

Lamy ’11(Pascal Lamy is the Director-General of the World Trade Organization. Lamy is Honorary President of Paris-based think
tank Notre Europe. Lamy graduated from the prestigious Sciences Po Paris, from HEC and ÉNA, graduating second in his year of
those specializing in economics. “System Upgrade” BY PASCAL LAMY | APRIL 18, 2011)

Any doubts about the stability and importance of the global trade architecture should have been
dispelled by the remarkable manner in which the system has endured the devastating
economic crisis that shook the world from 2008 to 2009 . That durability stands in stark contrast
with many other elements in the international architecture , which proved too brittle to
withstand this shock. For example, governments have yet to devise an international system for
managing climate change or currency volatility. Other issues heavily tinged by domestic politics,
such as migration, are not even on the international agenda and face fire even at the regional
level, as we have seen with the influx of immigrants to Europe following recent events in North
Africa. By now, it should be clear that the failure to establish a system of global governance in the area of international finance
unquestionably blunted governments' ability to respond effectively to the crisis. Yet even while a great many things went wrong in
the crisis, the
trading system responded precisely as it was intended to. Compare that with the
1930s, when the last great global economic calamity unfolded. No such system was in place, and
the global economy paid a heavy price. The United States passed the Smoot-Hawley Tariff Act in
1930, quadrupling tariffs and raising duties to at least 60 percent on more than 3,000 types of imported products. Faced with this
provocation, America's trading partners did not sit idly by; tit-for-tat retaliation rapidly ensued, closing markets and throttling trade.
Between 1929 and 1934, world trade contracted 66 percent. The bulk of this collapse was due to disintegrating demand and the
drying-up of credit. But the imposition
of prohibitive duties not only pushed the economy further into depression -- it
also fostered a profound sense of ill will between governments and contributed to the international tensions that
led to World War II. To prevent this from happening again, leaders of great vision and wisdom agreed to
create a rules-based, transparent, and nondiscriminatory trading system. Men like James Meade and
Cordell Hull succeeded in encouraging 23 countries to accept a compact known as the General Agreement on Tariffs and Trade
(GATT). Since
the GATT came into force in 1948 and since the World Trade Organization ( WTO)
opened its doors in 1995, the world has not seen protectionist outbreaks like those of the early
1930s. This is not to say that the trading system has not been tested. Protectionist pressures
surged in 1971, for example, when the gold standard for currency conversion was abandoned,
and during the 1997-1998 Asian economic crisis, when Pacific Rim countries saw their
economies contract by double-digit margins. In each case, markets stayed open to the flow of
goods and services from the affected countries, enabling them to trade their way back to
stability and prosperity.
Fundamentals are strong and rationality prevails in crisis

Boot 8 (Max, Senior Fellow at the Council on Foreign Relations, “Don’t Sell America’s Economy Short,” LA Times)

So far, Main Street has shown a surprising amount of resiliency given the problems of Wall Street. Even if
the economy
eventually succumbs to recession, as now appears more likely, it will bounce back before long. It
always has. There have been plenty of crises in the past -- the stagflation and oil-price spikes of the 1970s, the
savings and loan debacle and soaring trade and budget deficits of the 1980s, the popping of the dot-come bubble and the terrorist
attacks in the early 2000s -- that led many observers to predict that the United States would soon go the way of Rome. What the
pessimists ignore is that the fundamentals of the U.S. economy remain strong . Indeed, the World Economic
Forum has ranked the United States as the world's most competitive economy for the last two years. (The new survey comes out
next month.) Its statistics show that per-capita gross domestic product in the U.S. consistently has grown faster than in other
developed economies since 1980. Looking deeper at the causes of American competitiveness shows that we score especially
strongly not only in domestic market size (No. 1 in the world) but also in such areas as time required to start a business (No. 3),
venture capital availability (No. 1), the cost of firing an employee (No. 1), ownership of personal computers (No. 2),
university/industry research collaboration (No. 1) and quality of scientific research institutions (No. 2). The availability of venture
capital might be affected temporarily by the market turmoil, and we should worry if Democrats gain control of both ends of
Pennsylvania Avenue in November because they might exacerbate what the survey found to be the two most "problematic" issues
for doing business in the U.S. -- high tax rates and cumbersome tax regulations. But whatever happens in the next few months, most
of the other advantages that have been powering the U.S. economy forward for decades will remain unchanged. So too will another
vital statistic: population growth. According to ederal statistics, the fertility rate in the U.S., where each woman has on average 2.1
children, is now the highest among major industrialized economies. We are above replacement level while Europe, Japan and other
industrialized economies have long been beneath it. That means that, even as our major competitors have to cope with graying
populations, declining productivity and increasing pension costs, our population will remain relatively youthful and vibrant,
notwithstanding the retirement of the baby boomers. This advantage is enhanced by our ability to attract and integrate hardworking
immigrants from around the world. America's competitors display other weaknesses that become apparent in times of crisis. As
Harvard economic historian Niall Ferguson noted over the weekend in the Washington Post, while the U.S. stock market has
declined roughly 18% this year, China has seen a fall of 48% and Russia of 55%. "These figures are not very good advertisements for
the more regulated, state-led economic models favored in Beijing and Moscow," he wrote. Although the current crisis exposes
vulnerabilities in the American financial system, it also shows one of our greatest strengths: the ability of our politicos to cross party
lines and formulate a decisive response in a time of crisis. We saw that kind of bipartisan action after 9/11, and there is a good
chance that we will see it now -- assuming that lawmakers can agree on a bailout package that makes sense. Contrast that with
Japan's dithering, delayed response after its real estate and stock market bubbles burst in 1990. A sclerotic political system dragged
out its recovery for more than a decade and put paid to predictions -- heard so often in the 1980s -- that Japan would supplant the
U.S. as the world's economic powerhouse. Given
America's record of resiliency, it would be foolish to
"short" our prospects based on recent turmoil . The smart money will stay "bullish on America," even if that was
Merrill Lynch's slogan before its downfall.

The economy is resilient – their evidence is fear mongering

Lambro 8 (Donald, Washington Times Chief Political Correspondent, “Always darkest before dawn,” 7-28, Lexis)

The doom-and-gloomers are still with us, of course, and they will go to their graves forecasting that life as we
know it is coming to an end and that we are in for years of economic depression and recession. Last week, the New York
Times ran a Page One story maintaining that Americans were saving less than ever, and that their debt burden had risen by an
average of $117,951 per household. And the London Telegraph says there are even harder times ahead, comparing today's economy
to the Great Depression of the 1930s. Wall Street economist David Malpass thinks that kind of fearmongering is filled with
manipulated statistics that ignore long-term wealth creation in our country, as well as globally. Increasingly,
people are investing "for the long run - for capital gains (not counted in savings) rather than current income - in preparation
for retirement," he told his clients last week. Instead of a coming recession, "we think the U.S. is in gradual recovery after
a sharp two-quarter slowdown, with consumer resilience more likely than the decades-old expectation of a consumer
slump," Mr. Malpass said. "Fed data shows clearly that household savings of all types - liquid, financial and tangible - are still
close to the record levels set in September. IMF data shows U.S. households holding more net financial savings than the rest of
the world combined. Consumption has repeatedly outperformed expectations in recent quarters and year," he said.
The American economy has been pounded by a lot of factors, including the housing collapse (a needed correction to
bring home prices down to earth), the mortgage scandal and the meteoric rise in oil and gas prices. But this $14 trillion
economy, though slowing down, continues to grow by about 1 percent on an annualized basis, confounding the pessimists
who said we were plunging into a recession, defined by negative growth over two quarters. That has not happened - yet. Call me a
cockeyed optimist, but I do not think we are heading into a recession. On the contrary, I'm more bullish than ever on our economy's
long-term prospects.
---Econ- US Not Key

1NC Economist—US not key—emerging economies pulling three times the weight of America
—post crisis trends point to stronger comebacks and decoupling from America—no risk
domestic collapse spreads

All post-recession financial research indicates decoupling

Kohn ‘8 – PhD in economics from Michigan, Chairman of the Committee on the Global Financial System, Vice Chairman of the
Fed(Donald, speech at the International Research Forum on Monetary Policy in Frankfurt, “Global Economic Integration and
Decoupling”, http://www.federalreserve.gov/newsevents/speech/kohn20080626a.htm, WEA)

What about our more recent experience? During the first three quarters of 2007, the U.S. economy was
growing at a solid pace of about 3 percent at an annual rate. Over the next two quarters , U.S. growth
slowed to an average of about 3/4 percent, while growth in other industrialized countries stayed much
closer to trend rates at about 2-1/2 percent, and growth in the emerging market economies, at 6-1/2
percent, held up quite well. It is important to keep in mind, however, that we are still in the midst of the current
episode. Financial markets remain stressed ; housing markets in many countries are adjusting after a sharp run-up in
prices; and the effects of the turmoil on economic activity in the United States and elsewhere are still working themselves out.
Accordingly, it is too early to tell how correlated U.S. and foreign activity will have been in this period. One piece of research
on business cycles in G-7 economies, done by staff at the Federal Reserve Board, shows how
difficult it is to establish with any confidence that business cycles have become more
synchronized in recent decades, despite trade and financial integration having clearly increased.11 Other
research, which shows a modest convergence of business cycles across a larger group of industrial economies, fails to
find an increase in the correlation of industrial country cycles with emerging market economy cycles.12
The other dimension of recent linkages is financial, where the evidence is clearer. First, few question the importance
of financial linkages between the United States and other industrial economies, which is an area where decoupling
clearly has not occurred during the recent episode. While industrial country markets for stocks and bonds
have displayed a high degree of co-movement for years, in the current episode we are seeing
notable new correlations across money markets, with disruptions in funding markets showing up in the
euro area, Switzerland, the United Kingdom, and Canada, as well as in the United States. Some of the effects of the U.S.
subprime mortgage crisis on financial markets in these countries occurred as a result of direct or indirect balance sheet
exposures by their financial institutions to U.S. securities. Other adverse consequences for foreign financial institutions occurred
when entire markets, such as that for asset-backed commercial paper, became impaired. In contrast, some have pointed
to the apparent resilience of financial conditions in emerging market economies during the past year
as an example of decoupling. In particular, the disruptions in the advanced economies have had
only limited impacts on money markets in emerging market economies, and other financial market indicators in
emerging market economies appear to have held up relatively well. For example, the spreads of emerging market sovereign
bond yields over U.S. Treasury securities have risen since June of last year, but by only about 1/3 of the rise in the average U.S.
corporate high-yield spread over U.S. Treasury securities. That rise is roughly half the average in several previous episodes of
pressure on U.S. corporate bond prices over the period from 1998 to 2005; these episodes include, among others, the Russian
and Long-Term Capital Management crisis of 1998, the 2002 surge in corporate defaults and bankruptcies, and growing
concerns about U.S. auto companies in 2005. In addition, while
stock prices in some emerging market
countries have not performed well, a broad aggregate for these markets shows stock prices up
over the past year, while the advanced economy indexes have exhibited double-digit
declines, on average.13 Certainly, stock prices in the emerging market economies moved downward during acute periods of
U.S. financial stress over the past year. However, these movements were similar in scale to those seen in industrial country
equity markets, and during the intervening periods when global pressures were less intense, the prices of emerging market
equities rebounded more substantially than those of industrial countries.

Not key—no US demand, regional trading, and strong macroeconomic policies

Merrill Lynch 6 (Merrill Lynch, “US Downturn Won’t Derail World Economy”, 9-18, http://www.ml.com/index.asp?
id=7695_7696_8149_63464_70786_71164)

A sharp slowdown in the U.S. economy in 2007 is unlikely to drag the rest of the global economy
down with it, according to a research report by Merrill Lynch’s (NYSE: MER) global economic team. The good news is that
there are strong sources of growth outside the U.S. that should prove resilient to a consumer-led
U.S. slowdown. Merrill Lynch economists expect U.S. GDP growth to slow to 1.9 percent in 2007 from 3.4 percent in 2006,
but non-U.S. growth to decline by only half a percent (5.2 percent versus 5.7 percent). Behind this decoupling is
higher non-U.S. domestic demand, a rise in intraregional trade and supportive macroeconomic
policies in many of the world’s economies. Although some countries appear very vulnerable to a U.S. slowdown,
one in five is actually on course for faster GDP growth in 2007. Asia, Japan and India appear well placed to
decouple from the United States, though Taiwan, Hong Kong and Singapore are more likely to be impacted. European
countries could feel the pinch, but rising domestic demand in the core countries should help the region weather the storm much
better than in previous U.S. downturns. In the Americas, Canada will probably be hit, but Brazil is set to decouple.

US not key

The Economist 7 (November 23, “America’s Vulnerable Economy”, pg. 13)

The best hope that global growth can stay strong lies instead with emerging economies. A decade ago, the
thought that so much depended on these crisis-prone places would have been terrifying. Yet thanks largely to economic reforms,
their annual growth rate has surged to around 7%. This year they
will contribute half of the globe's GDP growth ,
measured at market exchange rates, over three times as much as America. In the past, emerging
economies have often needed bailing out by the rich world. This time they could be the
rescuers. Of course, a recession in America would reduce emerging economies' exports, but they are less vulnerable than they
used to be. America's importance as an engine of global growth has been exaggerated. Since 2000 its share of world imports has
dropped from 19% to 14%. Its vast current-account deficit has started to shrink, meaning that America
is no longer pulling
along the rest of the world. Yet growth in emerging economies has quickened, partly thanks to demand at home. In the
first half of this year the increase in consumer spending (in actual dollar terms) in China and India added more to global GDP growth
than that in America. Most emerging economies are in healthier shape than ever (see article). They are no longer financially
dependent on the rest of the world, but have large foreign-exchange reserves—no less than three-quarters of the global total.
Though there are some notable exceptions, most of them have small budget deficits (another change from the past), so they can
boost spending to offset weaker exports if need be.
---Econ- No War

1NC Brandt—no risk of war—assumes their arguments about collapse of liberalism and trade
—indicates there is zero way a country could prosper through violence in post-collapse world
—leaders are rational and domestic opinion checks

No econ impact or lash-out


Daniel Drezner 14, Professor of IR at Tufts, “The System Worked: Global Economic
Governance during the Great Recession”, World Politics, Volume 66. Number 1, January 2014,
pp. 123-164

The final significant outcome addresses a


dog that hasn't barked: the effect of the Great Recession on cross-border conflict and violence.
During the initial stages of the crisis, multiple analysts asserted that the financial
crisis would lead states to increase their use of force as a tool for staying in power.42 They voiced genuine
concern that the global economic downturn would lead to an increase in conflict— whether through greater internal repression,
diversionary wars, arms races, or a ratcheting up of great power conflict. Violence in the Middle
East, border disputes in the South China Sea, and even the disruptions of the Occupy movement fueled impressions of a surge in global public disorder.
The aggregate data suggest otherwise, however. The Institute for Economics and Peace has concluded that "the
average level of peacefulness in 2012 is approximately the same as it was in 2007."43 ¶
Interstate violence in particular has declined since the start of the financial crisis, as have
military expenditures in most sampled countries. Other studies confirm that the Great Recession
has not triggered any increase in violent conflict, as Lotta Themner and Peter Wallensteen conclude: "[T]he
pattern is one of relative stability when we consider the trend for the past five years."44 The secular decline in violence that
started with the end of the Cold War has not been reversed . Rogers Brubaker observes that "the
crisis has not to date generated the surge in protectionist nationalism or ethnic
exclusion that might have been expected."43

No diversionary theory or increased probability of war

Jervis ’11 (Robert Jervis 11, Professor in the Department of Political Science and School of International and Public Affairs at
Columbia University, December 2011, “Force in Our Times,” Survival, Vol. 25, No. 4, p. 403-425

Even if war is still seen as evil, the security community could be dissolved if severe conflicts of interest were to arise. Could the
more peaceful world generate new interests that would bring the members of the community into sharp disputes? 45 A zero-
sum sense of status would be one example, perhaps linked to a steep rise in nationalism. More likely would be a
worsening
of the current economic difficulties, which could itself produce greater nationalism, undermine
democracy and bring back old-fashioned beggar-my-neighbor economic policies . While these
dangers are real, it is hard to believe that the conflicts could be great enough to lead the
members of the community to contemplate fighting each other. It is not so much that
economic interdependence has proceeded to the point where it could not be reversed – states that were
more internally interdependent than anything seen internationally have fought bloody civil wars. Rather it is that even
if the more extreme versions of free trade and economic liberalism become discredited, it is
hard to see how without building on a preexisting high level of political conflict leaders and mass opinion
would come to believe that their countries could prosper by impoverishing or even attacking
others . Is it possible that problems will not only become severe, but that people will entertain the thought that they have to
be solved by war? While a pessimist could note that this argument does not appear as outlandish
as it did before the financial crisis, an optimist could reply (correctly, in my view) that the very
fact that we have seen such a sharp economic down-turn without anyone suggesting that
force of arms is the solution shows that even if bad times bring about greater economic
conflict, it will not make war thinkable.

No conflicts

Katoch ‘9 (Rajan Katoch, Institute for Defence Studies and Analyses, New Delhi The Global Economic Crisis Some Strategic
Implications, First Published: August 2009)

Despite the above factors, the


situation is not the same as in the 1930s, and in today’s world, it is highly
improbable that the global economic crisis could lead to a world war as it did then. The
international order is relatively more stable, with all major powers working with greater
coordination, and mostly seeking to stick to the status quo. Learning from experience , the current national economic
policy responses are better formulated, and therefore the economic crisis is unlikely to reach the
severity of and linger on for as long as the Great Depression. The greater role being played by fora like the G20
in seeking solutions to the crisis indicates the recognition amongst the key players from both industrialised and developing countries
that a broad consensus is needed to move forward. And finally, allthis is backed by the hard fact of the
overwhelming military dominance of the US; this acts as a force for stability. Localised conflicts
remain possible; perhaps a serious threat arising out of collapse of critical states at worst, but a world war—most
unlikely.

Economic decline doesn’t cause war

Miller 00 (Morris, Economist, Adjunct Professor in the Faculty of Administration – University of Ottawa, Former Executive
Director and Senior Economist – World Bank, “Poverty as a Cause of Wars?”, Interdisciplinary Science Reviews, Winter, p. 273)

The question may be reformulated. Do wars spring from a popular reaction to a sudden economic crisis that exacerbates poverty
and growing disparities in wealth and incomes? Perhaps one could argue, as some scholars do, that it is some dramatic event or sequence of such
events leading to the exacerbation of poverty that, in turn, leads to this deplorable denouement. This exogenous factor might act as a catalyst for a
violent reaction on the part of the people or on the part of the political leadership who would then possibly be tempted to seek a diversion by finding
or, if need be, fabricating an enemy and setting in train the process leading to war. According
to a study undertaken by Minxin Pei and Ariel
Adesnik of the Carnegie Endowment for International Peace, there would not appear to be any merit in this

hypothesis. After studying ninety-three episodes of economic crisis in twenty-two countries in Latin America and
Asia in the years since the Second World War they concluded that:19 Much of the conventional wisdom about the political

impact of economic crises may be wrong ... The severity of economic crisis – as measured in terms of inflation and negative

growth - bore no relationship to the collapse of regimes ... (or, in democratic states, rarely) to an outbreak of violence ... In
the cases of dictatorships and semidemocracies, the ruling elites responded to crises by increasing repression (thereby using one form of violence to
abort another).

Economic decline doesn’t cause war

Tir 10 [Jaroslav Tir - Ph.D. in Political Science, University of Illinois at Urbana-Champaign and is an Associate Professor in the
Department of International Affairs at the University of Georgia, “Territorial Diversion: Diversionary Theory of War and Territorial
Conflict”, The Journal of Politics, 2010, Volume 72: 413-425)]

Empirical support for the economic growth rate is much weaker. The finding that poor economic
performance is associated with a higher likelihood of territorial conflict initiation is significant
only in Models 3–4.14 The weak results are not altogether surprising given the findings from
prior literature. In accordance with the insignificant relationships of Models 1–2 and 5–6, Ostrom and Job (1986), for example,
note that the likelihood that a U.S. President will use force is uncertain, as the bad economy might
create incentives both to divert the public’s attention with a foreign adventure and to focus on
solving the economic problem, thus reducing the inclination to act abroad. Similarly, Fordham (1998a,
1998b), DeRouen (1995), and Gowa (1998) find no relation between a poor economy and U.S. use of
force. Furthermore, Leeds and Davis (1997) conclude that the conflict-initiating behavior of 18
industrialized democracies is unrelated to economic conditions as do Pickering and Kisangani
(2005) and Russett and Oneal (2001) in global studies. In contrast and more in line with my findings of a significant
relationship (in Models 3–4), Hess and Orphanides (1995), for example, argue that economic recessions are linked with forceful
action by an incumbent U.S. president. Furthermore, Fordham’s (2002) revision of Gowa’s (1998) analysis shows some effect of a
bad economy and DeRouen and Peake (2002) report that U.S. use of force diverts the public’s attention from a poor economy.
Among cross-national studies, Oneal and Russett (1997) report that slow growth increases the incidence of militarized disputes, as
does Russett (1990)—but only for the United States; slow growth does not affect the behavior of other countries. Kisangani and
Pickering (2007) report some significant associations, but they are sensitive to model specification, while Tir and Jasinski (2008) find
a clearer link between economic underperformance and increased attacks on domestic ethnic minorities. While none of these works
has focused on territorial diversions, my own inconsistent findings for economic growth fit well with the mixed results reported in
the literature.15 Hypothesis 1 thus receives strong support via the unpopularity variable but only weak support via the economic
growth variable. These
results suggest that embattled leaders are much more likely to respond with
territorial diversions to direct signs of their unpopularity (e.g., strikes, protests, riots) than to
general background conditions such as economic malaise . Presumably, protesters can be distracted via
territorial diversions while fixing the economy would take a more concerted and prolonged policy effort. Bad economic conditions
seem to motivate only the most serious, fatal territorial confrontations. This implies that leaders may be reserving the most high-
profile and risky diversions for the times when they are the most desperate, that is when their power is threatened both by signs of
discontent with their rule and by more systemic problems plaguing the country (i.e., an underperforming economy).

Impact empirically denied

Barnett ‘9 (Thomas P.M. Barnett, senior managing director of Enterra Solutions LLC, “The New Rules: Security Remains Stable
Amid Financial Crisis,” 8/25/2009)

When the global financial crisis struck roughly a year ago, the blogosphere was ablaze with all
sorts of scary predictions of, and commentary regarding, ensuing conflict and wars -- a rerun of the
Great Depression leading to world war, as it were. Now, as global economic news brightens and recovery -- surprisingly led by
China and emerging markets -- is the talk of the day, it's interesting to look back over the past year and realize how
globalization's first truly worldwide recession has had virtually no impact whatsoever on the
international security landscape. None of the more than three-dozen ongoing conflicts listed by
GlobalSecurity.org can be clearly attributed to the global recession . Indeed, the last new entry (civil conflict
between Hamas and Fatah in the Palestine) predates the economic crisis by a year, and three quarters of the chronic struggles began
in the last century. Ditto for the 15 low-intensity conflicts listed by Wikipedia (where the latest entry is the Mexican "drug war"
begun in 2006). Certainly, the Russia-Georgia conflict last August was specifically timed, but by most accounts the opening ceremony
of the Beijing Olympics was the most important external trigger (followed by the U.S. presidential campaign) for that sudden spike in
an almost two-decade long struggle between Georgia and its two breakaway regions. Looking over the various databases, then, we
see a most familiar picture: the usual mix of civil conflicts, insurgencies, and liberation-themed terrorist movements. Besides the
recent Russia-Georgia dust-up, the only two potential state-on-state wars (North v. South Korea, Israel v. Iran) are both tied to one
side acquiring a nuclear weapon capacity -- a process wholly unrelated to global economic trends. And with the United States
effectively tied down by its two ongoing major interventions (Iraq and Afghanistan-bleeding-into-Pakistan), our involvement
elsewhere around the planet has been quite modest, both leading up to and following the onset of the economic crisis: e.g., the
usual counter-drug efforts in Latin America, the usual military exercises with allies across Asia, mixing it up with pirates off Somalia's
coast). Everywhere else we find serious instability we pretty much let it burn, occasionally pressing the Chinese -- unsuccessfully -- to
do something. Our new Africa Command, for example, hasn't led us to anything beyond advising and training local forces. So, to sum
up: * No significant uptick in mass violence or unrest (remember the smattering of urban riots last year in places like Greece,
Moldova and Latvia?); * The usual frequency maintained in civil conflicts (in all the usual places); * Not a single state-on-state war
directly caused (and no great-power-on-great-power crises even triggered); * No great improvement or disruption in great-power
cooperation regarding the emergence of new nuclear powers (despite all that diplomacy); * A modest scaling back of international
policing efforts by the system's acknowledged Leviathan power (inevitable given the strain); and * No serious efforts by any rising
great power to challenge that Leviathan or supplant its role. (The worst things we can cite are Moscow's occasional deployments of
strategic assets to the Western hemisphere and its weak efforts to outbid the United States on basing rights in Kyrgyzstan; but the
best include China and India stepping up their aid and investments in Afghanistan and Iraq.) Sure, we've
finally seen global
defense spending surpass the previous world record set in the late 1980s, but even that is likely to
wane given the stress on public budgets created by all this unprecedented "stimulus" spending.
If anything, the friendly cooperation on such stimulus packaging was the most notable great-
power dynamic caused by the crisis. Can we say that the world has suffered a distinct shift to political radicalism as a
result of the economic crisis? Indeed, no. The world's major economies remain governed by center-left or
center-right political factions that remain decidedly friendly to both markets and trade. In the short
run, there were attempts across the board to insulate economies from immediate damage (in effect,
as much protectionism as allowed under current trade rules), but there was no great slide into "trade wars."
Instead, the World Trade Organization is functioning as it was designed to function, and regional
efforts toward free-trade agreements have not slowed . Can we say Islamic radicalism was inflamed by the
economic crisis? If it was, that shift was clearly overwhelmed by the Islamic world's growing disenchantment with the brutality
displayed by violent extremist groups such as al-Qaida. And looking forward, austere economic times are just as likely to breed
connecting evangelicalism as disconnecting fundamentalism. At the end of the day, the economic crisis did not prove to be
sufficiently frightening to provoke major economies into establishing global regulatory schemes, even as it has sparked a spirited --
and much needed, as I argued last week -- discussion of the continuing viability of the U.S. dollar as the world's primary reserve
currency. Naturally,
plenty of experts and pundits have attached great significance to this debate,
seeing in it the beginning of "economic warfare" and the like between "fading" America and
"rising" China. And yet, in a world of globally integrated production chains and interconnected
financial markets, such "diverging interests" hardly constitute signposts for wars up ahead.
Frankly, I don't welcome a world in which America's fiscal profligacy goes undisciplined, so bring
it on -- please! Add it all up and it's fair to say that this global financial crisis has proven the great
resilience of America's post-World War II international liberal trade order.

93 crises prove
Miller ‘2K (Morris Miller, economist, adjunct professor in the University of Ottawa’s Faculty of Administration, consultant on
international development issues, former Executive Director and Senior Economist at the World Bank, Winter 2000, Interdisciplinary
Science Reviews, Vol. 25, Iss. 4, “Poverty as a cause of wars?” p. Proquest

The question may be reformulated. Do wars spring from a popular reaction to a sudden economic crisis that
exacerbates poverty and growing disparities in wealth and incomes? Perhaps one could argue, as some scholars do, that it is some
dramatic event or sequence of such events leading to the exacerbation of poverty that, in turn, leads to this deplorable
denouement. This exogenous factor might act as a catalyst for a violent reaction on the part of the people or
on the part of the political leadership who would then possibly be tempted to seek a diversion by
finding or, if need be, fabricating an enemy and setting in train the process leading to war. According to a study
undertaken by Minxin Pei and Ariel Adesnik of the Carnegie Endowment for International Peace, there would not
appear to be any merit in this hypothesis . After studying ninety-three episodes of economic crisis
in twenty-two countries in Latin America and Asia in the years since the Second World War they concluded that:19 Much
of the conventional wisdom about the political impact of economic crises may be wrong ... The
severity of economic crisis - as measured in terms of inflation and negative growth - bore no relationship to the
collapse of regimes ... (or, in democratic states, rarely) to an outbreak of violence ... In the cases of dictatorships
and semidemocracies, the ruling elites responded to crises by increasing repression (thereby using one form of violence to abort
another).

Even if they win historical analysis, modern structural changes make war unthinkable

Rachman ’11 (chief foreign-affairs commentator – Financial Times, Gideon Rachman, journalist who has been the Financial
Times chief foreign affairs commentator since July 2006. GIDEON RACHMAN: Threat of war seems unreal in an age of peace,
2011/11/29]

These scenarios are not implausible. And yet,


I still cannot bring myself to believe that we are heading back
to the 1930s. First, the very knowledge of what went wrong 80 years ago may help politicians to
avoid the same mistakes this time around. China’s continued emphasis on the need for a "peaceful
rise" owes something to a knowledge of the terrible errors of Imperial Japan. Second, there is a
plausible argument that the 66 years of peace between the major powers and developed nations since 1945 reflects the progress of
Finally, the developed world is starting from a much
civilisation, rather than a lucky cycle in world history.
higher level of affluence than it did in the 1930s. In an economic crash, people might still lose
their savings, their jobs and their homes — but they are less likely to be reduced to utter
destitution. As a result, they may be less prone to political radicalisation. The Latvian economy
shrank by 18% in 2009 but in the recent election there, two centrist parties came out on top . In
Spain, unemployment is already more than 22%, and 45% for the young. And yet a moderate centre- right party won this month’s
election. So, althoughthe risk of a severe economic crisis is very real, I don’t believe that we are at
risk of sliding back into war. But that may simply be the failure of imagination of somebody lucky
enough to be raised in a period of unparalleled peace and prosperity .

All major powers won’t fight—prefer specifics

Ferguson ‘9 ( 'There will be blood' Harvard economic historian Niall Ferguson predicts prolonged financial hardship, even civil
war, before the ‘Great Recession' ends HEATHER SCOFFIELD Globe and Mail Update February 23, 2009 at 6:45 PM
The question is whether the general destabilization, the return of, if you like, political risk,
ultimately leads to something really big in the realm of geopolitics. That seems a less certain
outcome. We've already talked about why China and the United States are in an embrace they don't
dare end. If Russia is looking for trouble the way Mr. Putin seems to be, I still have some doubt
as to whether it can really make this trouble, because of the weakness of the Russian economy .
It's hard to imagine Russia invading Ukraine without weakening its economic plight. They're
desperately trying to prevent the ruble from falling off a cliff. They're spending all their
reserves to prop it up. It's hardly going to help if they do another Georgia .” “I was more struck Putin's
bluster than his potential to bite, when he spoke at Davos. But he made a really good point, which I keep coming back to. In his
speech, he said crises like this will encourage governments to engage in foreign policy aggression. I don't think he was talking about
himself, but he might have been. It's
true, one of the things historically that we see, and also when we go back to 30s, but also to
the depressions 1870s and 19980s, weak regimes will often resort to a more aggressive foreign policy , to try to
bolster their position. It's legitimacy that you can gain without economic disparity – playing the nationalist card. I wouldn't be
It's just that I don't see it producing anything comparable
surprised to see some of that in the year ahead.
with 1914 or 1939. It's kind of hard to envisage a world war.

History proves

Ferguson 6 (Niall, Professor of History – Harvard University, Foreign Affairs, 85(5), September / October, Lexis)
Nor can economic crises explain the bloodshed. What may be the most familiar causal chain in modern
historiography links the Great Depression to the rise of fascism and the outbreak of World War II. But that simple story leaves too
much out. Nazi Germany started the war in Europe only after its economy had recovered. Not all the
countries affected by the Great Depression were taken over by fascist regimes, nor did all such regimes start wars of
aggression. In fact, no general relationship between economics and conflict is discernible for the
century as a whole. Some wars came after periods of growth, others were the causes rather
than the consequences of economic catastrophe, and some severe economic crises were not
followed by wars.
---Econ- 2NC Must Reads

Prefer our authors—their evidence is biased by economic Stockholm syndrome

Dornbrook, 10 – Reporter for the Kansas City Business Journal, **Citing Brian Wesbury – Chief Economist for First Trust
Advisors and Author (James, "Economist: Ongoing rebound gives reason for optimism", January 8th 2010, May 21st 2010,
http://kansascity.bizjournals.com/kansascity/stories/2010/01/04/daily46.html, MOORHEAD)

People should start being more optimistic about the economy because it probably will continue
rebounding in 2010, said Brian Wesbury, chief economist for First Trust Advisors LP. Wesbury was
the keynote speaker at the Association for Corporate Growth Kansas City’s annual economic forecast meeting Friday morning at the
Kansas City Marriott Downtown. Wesbury was also the keynote speaker for last year’s event, and many members agreed that his
predictions for 2009 were accurate. Wesbury, author of “It’s Not as Bad as You Think,” told the crowd that too many
people
are suffering from a sort of economic Stockholm Syndrome, where they have fallen in love with
pessimism. It’s because we just experienced the first real panic in the economy since 1907, Wesbury said, and it altered
the psyche of people to the point where they expect bad things to constantly happen. But
economic data show that the economy bottomed out in March 2009 and that recovery is under
way, he said.

Their arguments are based on mistakenly optimistic economics

Spitzer, 09 (Elliot, American Lawyer and former politician - 54th Governor of New York, June 3, 2009, Green Shoots, Red Ink,
Black Hole, Truly Terrifying Data about the real state of the U.S. economy, http://www.slate.com/id/2219599/)

I have an unfortunate sense that the "green shoots" in the economy that everyone is talking about are
nothing but dandelions. Sure, forcing $1 trillion of taxpayer money—in direct capital, guarantees, and diminished cost of
borrowing—into the banking sector has permitted the major banks to claim solvency for the moment. Yet we should not forget that
this solvency has come not through a much needed deleveraging of the banking sector but rather
from a massive transfer of the obligations of private banks to the public, with the debt accruing
to future generations. And overall loan quality at U.S. banks is still the worst in 25 years and deteriorating at the fastest pace
ever. It's a terrible mistake to confuse the momentary solvency of the financial sector and the
long-term health of our economy. While we have addressed the credit collapse, we have not begun to tackle
the far more daunting, and more significant, structural problems in the economy .
---Econ- AT: Bearden/Resource Wars

Reject Bearden—their impact should've happened in 2007

Bearden, 00 – Their Impact Author, the part they cut out (Thomas, "Unnecessary Energy Crisis", 2000, June 19th 2010,
MOORHEAD) Word Doc

My personal estimate is that, beginning about 2007, on our present energy course we will have
reached an 80% probability of this "final destruction of civilization itself" scenario occurring at any
time, with the probability slowly increasing as time passes. One may argue about the timing, slide the dates a year or
two, etc., but the basic premise and general time frame holds. We face not only a world economic crisis, but also a
world destruction crisis.

We’ll answer all his warrants about war—

Economic decline is not correlated with war—historical evidence disproves their arguments—
that’s Ferguson.

Resource wars will never occur

Deudney, 99 (Daniel, Asst Prof of Poli Sci at Johns Hopkins, Contested Grounds: Security and Conflict in the New Environmental
Politics )

The hypothesis that states will begin fighting each other as natural resources are depleted and
degraded seems intuitively accurate. The popular metaphor of a lifeboat adrift at sea with declining supplies of clean water
and rations suggests there will be fewer opportunities for positive-sum gains between actors as resource scarcity grows. Many fears
of resource war are derived from the cataclysmic world wars of the first half of the twentieth century. Influenced by geopolitical
theories that emphasized the importance of land and resources for great power status, Adolf Hitler fashioned Nazi German war aims
to achieve resource autonomy. 40 The aggression of Japan was directly related to resource goals: lacking indigenous fuel and
minerals, and faced with a slowly tightening embargo by the Western colonial powers in Asia, the Japanese invaded Southeast Asia
for oil, tin, and rubber.41 Although the United States had a richer resource endowment than the Axis powers, fears of shortages and
industrial strangulation played a central role in the strategic thinking of American elites about world strategy.42 During the Cold
War, the presence of natural resources in the Third World helped turn this vast area into an arena for East-West conflict.43 Given
this record, the scenario of conflicts over resources playing a powerful role in shaping international order should be taken
seriously.However, there are three strong reasons for concluding that the familiar scenarios of resource war are of
diminishing plausibility for the foreseeable future . First, the robust character of the world trade system means
that states no longer experience resource dependency as a major threat to their military security and political autonomy. During the
1930s, the collapse of the world trading system drove states to pursue economic autarky, but the
resource needs of
contemporary states are routinely met without territorial control of the resource source. As Ronnie
Lipschutz has argued, this means that resource constraints are much less likely to generate interstate
violence than in the past.Second, the prospects for resource wars are diminished by the growing
difficulty that states face in obtaining resources through territorial conquest . Although the invention of
nuclear explosives has made it easy and cheap to annihilate humans and infrastructure in extensive areas, the spread of
conventional weaponry and national consciousness has made it very costly for an invader , even
one equipped with advanced technology, to subdue a resisting population, as France discovered in
Indochina and Algeria, the United States in Vietnam, and the Soviet Union in Afghanistan. 45 At
the lower levels of violence capability that matter most for conquering and subduing territory,
the great powers have lost effective military superiority and are unlikely soon to regain it. Third,
nonrenewable resources are, contrary to intuitive logic, becoming less economically scarce. There is strong
evidence that the world is entering what H. E. Goeller and Alvin M. Weinberg have labeled the "age of
substitutability," in which industrial technology is increasingly capable of fashioning ubiquitous
and plentiful earth materials such as iron, aluminum, silicon, and hydrocarbons into virtually everything
needed by modern societies.46 The most striking manifestation of this trend is that prices for virtually every raw material
have been stagnant or falling for the last two decades despite the continued growth in world economic output. In contrast to the
expectations widely held during the 1970s that resource scarcity would drive up commodity prices to the benefit of Third World raw
material suppliers, prices have fallen.47

Also, there’s needs to be someone that starts the conflict—Bearden assumes a world where
two major powers are forced to fight each other than that draws in other nations that
“escalate it significantly”—there’s zero explanation why economic stagnation in the U.S.
forces other nations into a war with us.

Resource scarcity promotes cooperation not conflict

Dalby, 06 (Simon, Dept. Of Geography, Carleton University, "Security and environment linkages revisited" in Globalisation and
Environmental Challenges: Reconceptualising Security in the 21st Century, www.ntu.edu.sg/idss/publications/SSIS/SSIS001.pdf)

In parallel with the focus on human security as a necessity in the face of both natural and artificial forms of vulnerability, recent
literature has emphasised the opportunities that environmental management presents for political cooperation
between states and other political actors, on both largescale infrastructure projects as well as more traditional matters of wildlife
and new concerns with biodiversity preservation (Matthew/Halle/Switzer 2002). Simultaneously, the discussion on water wars, and
in particular the key finding the shared resources frequently stimulate cooperation rather than conflict ,
shifted focus from conflict to the possibilities of environmental action as a mode of peacemaking. Both at the international level in
terms of environmental diplomacy and institution building, there is considerable evidence of cooperative action on the part of many
states (Conca/Dabelko 2002). Case
studies from many parts of the world suggest that cooperation and
diplomatic arrangements can facilitate peaceful responses to the environmental difficulties in
contrast to the pessimism of the 1990’s where the focus was on the potential for conflicts . One
recent example of the attempts to resolve difficulties in the case of Lake Victoria suggests a dramatic
alternative to the resource war scenarios. The need to curtail over-fishing in the lake and the importance of
remediation has encouraged cooperation; scarcities leading to conflict arguments have not been
common in the region, and they have not influenced policy prescriptions (Canter/Ndegwa 2002). Many
conflicts over the allocations of water use rights continue around the world but most of them
are within states and international disputes simply do not have a history of leading to wars .
---Econ- AT: Diversionary Theory (Short)

Diversionary theory is crap - 180 empirics disprove it

Gelpi 97 (Christopher, Center for International Affairs @ Harvard, "Democratic Diversions," Sage,

Students of international politics have often argued that state leaders initiate the use of force
internationally to divert attention away from domestic problems. The author contends that these
arguments concerning relationship between domestic unrest and international conflict are not supported empirically
because they focus too narrowly on the incentives state leaders have to use external force as a
diversionary tactic without considering alternative solutions to quieting domestic unrest. It is hypothesized
that democratic leaders will respond to domestic unrest by diverting attention by using force internationally. On the other hand,
authoritarian leaders are expected to repress
the unrest directly, and these acts of repression will make
them less likely to use force internationally. An analysis of the initiation of force by the challenging
states in 180 international crises between 1948 and 1982 strongly supports these hypotheses . The results of
the analyses and their implications for the literature on diversionary conflicts and the rapidly growing literature on democratic peace
are discussed.

And, diversionary war theory is false

Boehmer ‘7 – political science professor at the University of Texas (Charles, Politics & Policy, 35:4, “The Effects of Economic
Crisis, Domestic Discord, and State Efficacy on the Decision to Initiate Interstate Conflict”, WEA)

This article examines the contemporaneous effect of low economic growth and domestic instability on the threat of regime change
and/ or involvement in external militarized conflicts. Many studies
of diversionary conflict argue that lower rates
of economic growth should heighten the risk of international conflict . Yet we know that militarized interstate
conflicts, and especially wars, are generally rare events whereas lower rates of growth are not. Additionally, a growing body of
literature shows that regime changes are also associated with lower rates of economic growth. The question then becomes which
event, militarized interstate conflict or regime change, is the most likely to occur with domestic discord and lower rates of economic
growth? Diversionary
theory claims that leaders seek to divert attention away from domestic
problems such as a bad economy or political scandals, or to garner increased support prior to elections. Leaders then
supposedly externalize discontented domestic sentiments onto other nations, sometimes as scapegoats based on the similar in-
group/out-group dynamic found in the research of Coser (1956) and Simmel (1955), where foreign countries are blamed for
domestic problems. This process is said to involve a “rally-round-the-flag” effect, where a leader can expect a short-term boost in
popularity with the threat or use of force (Blechman, Kaplan, and Hall 1978; Mueller 1973). Scholarship on diversionary conflict has
focused most often on the American case1 but recent studies have sought to identify this possible behavior in other countries.2 The
Falklands War is often a popular example of diversionary conflict (Levy and Vakili 1992). Argentina was reeling from hyperinflation
and rampant unemployment associated with the Latin American debt crisis. It is plausible that a success in the Falklands War may
have helped to rally support for the governing Galtieri regime, although Argentina lost the war and the ruling regime lost power.
How many other attempts to use diversionary tactics, if they indeed occur, can be seen to generate a similar outcome? The goal of
this article is to provide an assessment of the extent to which diversionary strategy is a threat to peace. Is this a colorful
theory kept alive by academics that has little bearing upon real events , or is this a real problem that policy
makers should be concerned with? If it is a strategy readily available to leaders, then it is important to know what domestic factors
trigger this gambit. Moreover, to know that requires an understanding of the context in external conflict, which occurs relative to
regime changes. Theories of diversionary conflict usually emphasize the potential benefits of
diversionary tactics, although few pay equal attention to the prospective costs associated with such
behavior. It is not contentious to claim that leaders typically seek to remain in office. However, whether they can
successfully manipulate public opinion regularly during periods of domestic unpopularity through their states’
participation in foreign militarized conflicts—especially outside of the American case—is a question open for debate.
Furthermore, there appears to be a logical disconnect between diversionary theories and extant studies of domestic conflict and
regime change. Lower rates of economic growth are purported to increase the risk of both militarized interstate conflicts (and
internal conflicts) as well as regime changes (Bloomberg and Hess 2002). This implies that if
leaders do, in fact, undertake
diversionary conflicts, many may still be thrown from the seat of power—especially if the outcome
is defeat to a foreign enemy. Diversionary conflict would thus seem to be a risky gambit (Smith 1996). Scholars such as MacFie
(1938) and Blainey (1988) have nevertheless questioned the validity of the diversionary thesis . As noted by Levy
(1989), this perspective is rarely formulated as a cohesive and comprehensive theory, and there has been little or no
knowledge cumulation. Later analyses do not necessarily build on past studies and the discrepancies between inquiries are often
difficult to unravel. “Studies have used a variety of research designs , different dependent variables (uses of force,
major uses of force, militarized disputes), different estimation techniques, and different data sets covering different time
periods and different states” (Bennett and Nordstrom 2000, 39). To these problems, we should add a lack of theoretical precision
and incomplete model specification. By a lack of theoretical precision, I am referring to the linkages between economic conditions
and domestic strife that remain unclear in some studies (Miller 1995; Russett 1990). Consequently, extant studies are to a degree
incommensurate; they offer a step in the right direction but do not provide robust cross-national explanations and tests of economic
growth and interstate conflict. Yet a few studies have attempted to provide deductive explanations about when and how
diversionary tactics might be employed. Using a Bayesian updating game, Richards and others (1993) theorize that while the use of
force would appear to offer leaders a means to boost their popularity, a poorly performing economy acts as a signal to a leader’s
constituents about his or her competence. Hence, attempts to use diversion are likely to fail either because
incompetent leaders will likewise fail in foreign policy or people will recognize the gambit for
what it is. Instead, these two models conclude that diversion is likely to be undertaken particularly by risk-acceptant leaders. This
stress on a heightened risk of removal from office is also apparent in the work of Bueno de Mesquita and others (1999), and Downs
and Rocke (1994), where leaders may “gamble for resurrection,” although the diversionary scenario in the former study is only a
partial extension of their theory on selectorates, winning coalitions, and leader survival. Again, how often do leaders fail in the
process or are removed from positions of power before they can even initiate diversionary tactics? A few studies focusing on leader
tenure have examined the removal of leaders following war, although almost no study in the diversionary literature has looked at
the effects of domestic problems on the relative risks of regime change, interstate conflict, or both events occurring in the same
year.3
---Econ- AT: Diversionary War (Long)

This never happens—any government willing to start a world war because the economy
dipped would probably be seriously unstable anyway

And, diversionary war theory is false

Boehmer, 07 – political science professor at the University of Texas (Charles, Politics & Policy, 35:4, “The Effects of Economic
Crisis, Domestic Discord, and State Efficacy on the Decision to Initiate Interstate Conflict”, WEA)

This article examines the contemporaneous effect of low economic growth and domestic instability on the threat of regime change and/ or involvement in external militarized

studies of diversionary conflict argue that lower rates of economic growth should heighten the
conflicts. Many

risk of international conflict. Yet we know that militarized interstate conflicts, and especially wars, are generally rare events whereas lower rates of
growth are not. Additionally, a growing body of literature shows that regime changes are also associated with lower rates of economic growth. The question then becomes

Diversionary
which event, militarized interstate conflict or regime change, is the most likely to occur with domestic discord and lower rates of economic growth?

theory claims that leaders seek to divert attention away from domestic problems such as a bad
economy or political scandals, or to garner increased support prior to elections. Leaders then supposedly externalize discontented domestic sentiments onto other
nations, sometimes as scapegoats based on the similar in-group/out-group dynamic found in the research of Coser (1956) and Simmel (1955), where foreign countries are
blamed for domestic problems. This process is said to involve a “rally-round-the-flag” effect, where a leader can expect a short-term boost in popularity with the threat or use of
force (Blechman, Kaplan, and Hall 1978; Mueller 1973). Scholarship on diversionary conflict has focused most often on the American case1 but recent studies have sought to
identify this possible behavior in other countries.2 The Falklands War is often a popular example of diversionary conflict (Levy and Vakili 1992). Argentina was reeling from
hyperinflation and rampant unemployment associated with the Latin American debt crisis. It is plausible that a success in the Falklands War may have helped to rally support for
the governing Galtieri regime, although Argentina lost the war and the ruling regime lost power. How many other attempts to use diversionary tactics, if they indeed occur, can

be seen to generate a similar outcome? The goal of this article is to provide an assessment of the extent to which diversionary strategy is a threat to peace. Is this a
colorful theory kept alive by academics that has little bearing upon real events , or is this a real problem that
policy makers should be concerned with? If it is a strategy readily available to leaders, then it is important to know what domestic factors trigger this gambit. Moreover, to know

Theories of diversionary conflict


that requires an understanding of the context in external conflict, which occurs relative to regime changes.

usually emphasize the potential benefits of diversionary tactics , although few pay equal
attention to the prospective costs associated with such behavior. It is not contentious to claim that leaders typically seek to remain in office.
However, whether they can successfully manipulate public opinion regularly during periods of domestic unpopularity

through their states’ participation in foreign militarized conflicts—especially outside of the American case—is a question open for debate.
Furthermore, there appears to be a logical disconnect between diversionary theories and extant studies of domestic conflict and regime change. Lower rates of economic
growth are purported to increase the risk of both militarized interstate conflicts (and internal conflicts) as well as regime changes (Bloomberg and Hess 2002). This implies that

if leaders do, in fact, undertake diversionary conflicts, many may still be thrown from the seat of
power—especially if the outcome is defeat to a foreign enemy. Diversionary conflict would thus seem to be a risky gambit (Smith 1996).
Scholars such as MacFie (1938) and Blainey (1988) have nevertheless questioned the validity of the diversionary thesis . As noted by Levy (1989), this

perspective is rarely formulated as a cohesive and comprehensive theory, and there has been little or no knowledge
cumulation. Later analyses do not necessarily build on past studies and the discrepancies between inquiries are often difficult to
unravel. “Studies have used a variety of research designs , different dependent variables (uses of force, major uses
of force, militarized disputes), different estimation techniques, and
different data sets covering different time periods and different
states” (Bennett and Nordstrom 2000, 39). To these problems, we should add a lack of theoretical precision and incomplete model specification. By a lack of theoretical
precision, I am referring to the linkages between economic conditions and domestic strife that remain unclear in some studies (Miller 1995; Russett 1990). Consequently, extant
studies are to a degree incommensurate; they offer a step in the right direction but do not provide robust cross-national explanations and tests of economic growth and
interstate conflict. Yet a few studies have attempted to provide deductive explanations about when and how diversionary tactics might be employed. Using a Bayesian updating
game, Richards and others (1993) theorize that while the use of force would appear to offer leaders a means to boost their popularity, a poorly performing economy acts as a

attempts to use diversion are likely to fail either because


signal to a leader’s constituents about his or her competence. Hence,

incompetent leaders will likewise fail in foreign policy or people will recognize the gambit for
what it is. Instead, these two models conclude that diversion is likely to be undertaken particularly by risk-acceptant leaders. This stress on a heightened risk of removal
from office is also apparent in the work of Bueno de Mesquita and others (1999), and Downs and Rocke (1994), where leaders may “gamble for resurrection,” although the
diversionary scenario in the former study is only a partial extension of their theory on selectorates, winning coalitions, and leader survival. Again, how often do leaders fail in the
process or are removed from positions of power before they can even initiate diversionary tactics? A few studies focusing on leader tenure have examined the removal of
leaders following war, although almost no study in the diversionary literature has looked at the effects of domestic problems on the relative risks of regime change, interstate
conflict, or both events occurring in the same year.3

Low growth makes politicians cautious—they don’t want to risk war because it makes them
vulnerable

Boehmer, 07 – political science professor at the University of Texas (Charles, Politics & Policy, 35:4, “The Effects of Economic
Crisis, Domestic Discord, and State Efficacy on the Decision to Initiate Interstate Conflict”, WEA)

Economic Growth and Fatal MIDs The theory presented earlier predicts that lower
rates of growth suppress
participation in foreign conflicts, particularly concerning conflict initiation and escalation to
combat. To sustain combat, states need to be militarily prepared and not open up a second
front when they are already fighting, or may fear, domestic opposition. A good example would be when the various
Afghani resistance fighters expelled the Soviet Union from their territory, but the Taliban crumbled when it had to face the
combined forces of the United States and Northern Alliance insurrection. Yet the coefficient for GDP growth and MID initiations was
negative but insignificant. However, considering that there are many reasons why states fight, the logic
presented earlier should hold especially in regard to the risk of participating in more severe
conflicts. Threats to use military force may be safe to make and may be made with both external
and internal actors in mind, but in the end may remain mere cheap talk that does not risk
escalation if there is a chance to back down. Chiozza and Goemans (2004b) found that secure leaders were more likely to
become involved in war than insecure leaders, supporting the theory and evidence presented here. We should find that
leaders who face domestic opposition and a poorly performing economy shy away from
situations that could escalate to combat if doing so would compromise their ability to retain
power.

They’re using sloppy methodology—diversionary war theory is an unfounded assertion

Bennett and Nordstrom, 00 – department of political science at Penn State (D Scott and Timothy, The Journal of Conflict
Resolution, 44:1, “Foreign policy substitutability and internal economic problems in enduring rivalries”, ProQuest, WEA)

The nature of the results has led some to question the validity of the theories. For example, Bruce Bueno de Mesquita. (1985) posits
that the logical flaw in externalization theory lies in the psychological nature of the original
scapegoat hypothesis on which externalization theory is based. While the international relations version of this theory is indeed
written in psychological terms (in-group/out-group), it may be that "the logical foundation
for the leap from
individual psychology to national action remains non-existent " (1985, 130). Similarly, Levy (1989, 266) puts
forth four problems with the original causal process as described by Simmel (1955), Coser (1956), and other early writers on
externalization: (1) little
attention is paid to the direction of the relationship between internal
problems and external conflict, (2) attempts to develop or test alternative theoretical relationships are
rare, (3) precise specifications of when externalization should occur theoretically are rare, and (4) the
conditions under which externalization conditions should hold are unexplored. The conclusion that Levy draws from these
theoretical issues is that many of the empirical studies suffer from problems of misspecified models (1989, 267). An additional
possible source of discrepancies in findings about diversionary conflict may be attributable to differences in research design and
variable measurement. Studies have used a variety of research designs, different dependent variables (uses of force, major uses of
force, militarized disputes), different estimation techniques, and different data sets covering different time periods and different
states. Even the central concept of externalization, namely, domestic trouble, is unclear. Most studies to date have used presidential
popularity, overall presidential success, the election calendar, or a misery index composed of inflation and unemployment as
indicators of presidential problems. Cross-national studies have most frequently examined what James (19 88, 103) categorizes as
manifest conflict, a category which includes protest demonstrations, political strikes, armed attacks, and deaths from domestic
violence. This category can be opposed to latent conflict, which exists when sources of trouble are present but have not yet led to
the physical manifestations of dissatisfaction. Diversionary
conflict theory as presented is typically so general
in its discussion of internal problems that it opens a Pandora's box of possible indicators of
domestic conflict, and all of the types of measures discussed above fit with the theory. The vague
nature of the theory may be contributing to this possible problem of model misspecification , but
there are few arguments that suggest one indicator is superior to the others. Alternative relationships between domestic economic
performance and international conflict also have been proposed, perhaps most importantly by Blainey (1973, 74). Blainey offers the
alternative hypothesis about economics and war that economically challenged countries are more likely to be the target of
aggressive military acts than their initiator (1973, 86). Faced with a poor target in a bad economic situation, who is faced with an
unhappy populace and possibly limited resources, potential conflict initiators are likely to see opportunity. The argument also
parallels the historical notion that leaders would only go to war when their coffers were full- in bad times, leaders may
simply not be able to afford to go to conflict . Blainey's argument appears to pose a challenge to diversionary conflict
theory in its emphasis on what is the most likely direction of conflict. Note, however, that its prediction (weak states become
targets) differs from a strategic application of diversionary conflict theory.
---Econ- AT: Dollar Heg
Dollar hegemony is resilient

Kritzer 10 Adam is the lead editor of the Forex Blog and contributor to other leading financial news sites. B.A. in Economics from Penn, "Dollar Returns to Favor as
World’s Reserve Currency" March 16 www.forexblog.org/2010/03/dollar-returns-to-favor-as-worlds-reserve-currency.html

Rumor has it that the Dollar is about to make a run. As the credit crisis slowly subsides, (currency) investors are once again
looking at the long-term, and they like what they see when it comes to the Dollar.¶ For those that care to
remember, 2008 was a great year for the Dollar, as the credit crisis precipitated an increase in risk aversion, and investors
realized that despite its pitfalls, the Dollar was (and still is) the most stable and really the only viable
global reserve currency. [This reversed a trend which had essentially been in place since the inception of the Euro in 1999].
In 2009, meanwhile, the Dollar resumed its multi-year decline, and many analysts were quick to label the rally of 2008 as an
aberration.¶ Then came the debt crises, first in Dubai, then in Greece. Suddenly, a handful of smaller EU countries appeared
vulnerable to fiscal crises. Japan officially became the first of the Aaa economies to receive a downgrade in its credit rating. The
British Pound is dealing with crises on both the political and economic fronts. According to Moody’s, “The ratings of the Aaa
governments — which also include Britain, France, Spain and the Nordic countries — are currently ’stable’…But…their ‘distance-
to-downgrade’ has in all cases substantially diminished.” Suddenly, the Greenback doesn’t look so bad.¶ I want to point out that
in forex, everything is relative. (Novice) forex investors are often baffled by how sustained economic and financial crises don’t
immediately result in currency depreciation. The explanation is that when the crises are worse in (every) other countries, the
base currency still looks attractive.¶ This is precisely the case when it comes to the US Dollar. To be sure, the economy is
still flawed, financial markets have yet to fully to recover, the federal budget deficit topped $1.8 Trillion in 2009, and
government finances seem close to the breaking point. Moody’s has also identified the US as a candidate for a ratings
downgrade. And yet, when you look at the situation in every other currency that currently rivals the US for
reserve currency status, the Dollar still wins hands down.¶ Its economy is the world’s largest. So are its
financial markets, which are also the deepest and most liquid. Its sovereign finances are still manageable from the
standpoint of debt-to-GDP and interest-to-revenue ratios. It is the only currency whose circulation can even come close to
meeting the needs of global trade. Summarized S&P – when it confirmed the AAA credit rating of the US, “ The
dollar’s
widespread acceptance stems from the U.S. economy’s fundamental strength, which in our view
comes from the economy’s size and the flexibility of labor and product markets. We view U.S. banking and capital markets to be
dynamic and unfettered relative to their peers.”¶ That’s why auctions of US Treasury bonds remain heavily oversubscribed
(demand exceeds supply), despite the rock-bottom interest coupons. China has reaffirmed its commitment to Treasuries (what
other choice does it have), confirmed by some forensic accounting work. Gold might continue to rally. So will other
commodities, for all I know. Emerging market currencies are still in good shape as well, but none of these will seriously rival the
US Dollar for a long-time, if ever. In short, when it comes to the other majors, the Dollar is still King: “You can say
whatever you want, but the dollar is the currency of last resort. It’s the currency people want in a crisis.”

No impact to dollar decline---it would cause stable duopoly with the euro

Bergsten 9 C. Fred Bergsten, Peterson Institute for International Economics, “The Dollar and the Deficits: How Washington Can
Prevent the Next Crisis,” Article in Foreign Affairs, Volume 88 No. 6, November/December 2009

Both the United States and the rest of the world have an interest in continued globalization
and efficient international financial markets, and so neither has any interest in entirely
eliminating the international role of the dollar. In any case, inertia is such a powerful force in
financial matters that a sweeping step of this kind is technically impossible. Instead, the United
States should encourage two eminently feasible changes in the current international monetary order. The first is the further
evolution of a multiple-currency system in which other monies increasingly share the international position of the dollar in
private markets. The euro, based on a collective European economy as large as the United States' and with capital markets as
extensive in most respects, is the most obvious candidate. The euro already rivals the dollar in some domains, such
as currency holdings and private bond placements, and will become a full competitor whenever the eurozone countries adopt a
more common fiscal policy. The Chinese renminbi is likely to acquire a significant international role once China allows it to be
converted for financial as well as current account transactions and eases capital controls. Some observers
fear that a
system of multiple currencies is inherently unstable. However, such a regime functioned
smoothly for several decades before World War I, and a pound-dollar duopoly existed throughout
the 1920s. A dollar-euro duopoly has already begun to emerge over the last decade. Competition
between national currencies is likely to improve economic policies and performance by forcing market
discipline on the governments and central banks behind these alternative currencies.
---Econ- AT: Democracy

No empirical evidence—a weak economy may have helped the Nazis, but there was no global
coalition of liberal democratic states to prevent authoritarianism—past recessions disprove
this—the 1930’s had other historically-specific variables, and democracy is already entrenched
in all the key states

Doesn’t kill democracy

Brower and Carothers, 09 – *fellow in the Democracy and Rule of Law Program at the Carnegie Endowment, **vice
president for studies at the Carnegie Endowment for International Peace, founder and director of the Democracy and Rule of Law
Program (Julia and Thomas, Carnegie Endowment for International Peace, “Will the International Economic Crisis Undermine
Struggling Democracies?”, http://www.carnegieendowment.org/publications/index.cfm?fa=view&id=23031, WEA)

As the global economic crisis unfolds, many observers are wondering what political effects it will have around the world. Will it
produce significant amounts of political instability? Will its damaging political effects fall especially hard on the many weak or
struggling democracies in the world, thereby producing a reversal of democracy’s gains of recent decades? To help deepen these
discussions empirically, it
is useful to consider the findings of research on the political effects of other
economic crises of recent decades. Of course drawing clearcut lessons from the complex past record of economic crises
is a difficult and necessarily tentative undertaking. And no matter how sophisticated such research is, broader, subtle long-term
changes in global economics and politics can mean that patterns from the past may not hold in the future. Nevertheless, existing
research offers important insights about the amount of instability that might occur—including whether it may consist more of
changes of government or changes in systems of government—as well as whether democratic or authoritarian governments will
suffer more. Threeimportant findings emerge from the research : 1. In the great majority of past
cases, economic crisis did not lead to regime change . In fact, it often did not even lead to a
change of government. In the most comprehensive article on this topic, Minxin Pei and David Adesnik (2000) examine
the political effects of 93 economic crises—defined as an annual inflation rate greater than 15 percent,
and stagnant or negative annual GDP growth—in Asia and Latin America between 1945 and 1998. Contrary to what might be
expected, they find that economic crisis contributed to regime change in only 30 cases. Six of these cases fit the model of an
immediate Suharto-style regime collapse; in the rest, regime change occurred after a time lag of about eighteen to 30 months.
Perhaps most surprising, however, is their finding that in
only about 18 of the remaining 63 cases did economic
crisis lead even to a change in government . What explains these findings? Pei and Adesnik speculate that three
factors might be at work. First, the timing has to be right for economic crises to have an observable
political impact. In about one-fifth of the cases with no change, the economic difficulties had ended prior to the next election.
Second, in ten of the cases, the economic crisis was overshadowed by an existing political crisis . Finally,
economic crises were less likely to produce regime change during the 1980s and 1990s than in the previous two decades. That this
trend coincides with the most recent wave of democratization in Latin America and Asia is no coincidence, and leads to the second
major finding of the research. 2. Democracies have been more resilient against the destabilizing effects of economic
crises than nondemocratic regimes (except for one-party authoritarian regimes). Several authors have found that democracies
weather economic crises more effectively than authoritarian regimes , including Remmer (1996), who
focuses on South America between 1944 and 1994, and Haggard (2000), who focuses on the Asian financial crisis. Pei and Adesnik
(2000) again provide the most systematic evidence for this conclusion. Of the 40 economic crises that occurred in democratic
countries, sixteen led to changes of government and only ten resulted in regime change. In contrast, of the 34 crises that occurred in
restricted democracies and military regimes, half led to regime collapse. The one exception to this trend lies with one-party
authoritarian regimes (as opposed to softer authoritarian regimes), which proved to be invulnerable to economic crisis. None of the
ten crises observed by Pei and Adesnik led to a change of government or regime in such countries. Democracy’s
advantage
lies in the flexibility that institutionalized opportunities for political change provide. Regular
elections offer citizens a much simpler means of punishing politicians for the economic crisis
than regime change. Citizens can also distinguish between institutions and politicians . Surveys of
citizens of former communist regimes in Eastern Europe during economic crises have found that they still support
democracy because they have negative memories of authoritarian regimes and value having political choice (Duch 1995). In
nondemocratic regimes, regime change may be the only means to obtain a change of policy. Their legitimacy is also much more
likely to be heavily performance-based.
---Econ- AT: Econ Leadership

Other countries collapse as well- relative power sustained

and

Recession disproves it affects American interest

Blackwill ‘9 (Former associate dean of the Kennedy School of Government and Deputy Assistant to the President and Deputy
National Security Advisor for Strategic Planning (Robert, RAND, “The Geopolitical Consequences of the World Economic Recession—
A Caution”, http://www.rand.org/pubs/occasional_papers/2009/RAND_OP275.pdf)

First, the United States, five years from today.


Did the global recession weaken the political will of the
United States to, over the long term, defend its external interests? Many analysts are already forecasting a “yes”
to this question. As a result of what they see as the international loss of faith in the American market economy model and in U.S.
leadership, they assert that Washington’s influence in international affairs is bound to recede, indeed is already diminishing. For
some, the wish is the father of this thought. But where is the empirical evidence? From South Asia, through relations
with China and Russia through the Middle East peace process, through dealing with Iran’s nuclear ambitions and North Korea’s
nuclear weaponization and missile activities, through confronting humanitarian crises in Africa and instability in Latin America, the
United States has the unchallenged diplomatic lead . Who could charge the Obama Administration with
diplomatic passivity since taking office? Indeed, one could instead conclude that the current global economic turbulence is causing
countries to seek the familiar and to rely more and not less on their American connection. In any event, foreigners (and some
Americans) often underestimate the existential resilience of the United States. In this respect, George
Friedman’s new book, The Next Hundred Years,14 and his view that the United States will be as dominant a force in the 21st century
as it was in the last half of the 20th century, is worth considering. So once again, those who now predict, as they have in every
decade since 1945, American decay and withdrawal will be wrong 15— from John Flynn’s 1955 The Decline of the American Republic
and How to Rebuild It,16 to Paul Kennedy’s 1987 The Rise and Fall of Great Powers,17 to Andrew Bacevich’s 2008 The Limits of
Power: The End of American Exceptionalism,18 to Godfrey Hodgson’s 2009 The Myth of American Exceptionalism19 and many
dozens of similar books in between. Indeed, the policies of the Obama Administration, for better or worse, are likely to be far more
influential and lasting regarding America’s longer-term geopolitical power projection than the present economic decline. To sum up
regarding the United States and the global economic worsening, former Council on Foreign Relations President Les Gelb, in his new
book, Power Rules: How Common Sense Can Rescue American Foreign Policy,20 insists that a nation’s power is what it always was—
essentially the capacity to get people to do what they don’t want to do, by pressure and coercion, using one’s resources and
position. . . . The world is not flat. . . . The shape of global power is decidedly pyramidal—with the United States alone at the top, a
second tier of major countries (China, Japan, India, Russia, the United Kingdom, France, Germany and Brazil), and several tiers
descending below. . . . Among all nations, only the United States is a true global power with global reach. Lee Kuan Yew, former
Prime Minister of the Republic of Singapore, agrees: “After
the crisis, the US is most likely to remain at the top
of every key index of national power for decades . It will remain the dominant global player for the next few
decades. No major issue concerning international peace and stability can be resolved without US leadership, and no country or
grouping can yet replace America as the dominant global power.”21 The current global economic crisis will not alter
this reality. And the capitalist market model will continue to dominate international economics, not least because China and India
have adopted their own versions of it.
---Econ- AT: Heg (Short)

Not logical—absolute weakness doesn’t translate into comparative terms—collapse would


make others suffer and unable to challenge the US

Economic decline doesn’t kill heg—American leadership is unique and their predictions have
been denied for decades

Blackwill 9 – former associate dean of the Kennedy School of Government and Deputy Assistant to the President and Deputy
National Security Advisor for Strategic Planning (Robert, RAND, “The Geopolitical Consequences of the World Economic Recession—
A Caution”, http://www.rand.org/pubs/occasional_papers/2009/RAND_OP275.pdf)

First, the United States, five years from today. Did


the global recession weaken the political will of the
United States to, over the long term, defend its external interests? Many analysts are already forecasting a
“yes” to this question. As a result of what they see as the international loss of faith in the American market economy model and
in U.S. leadership, they assert that Washington’s influence in international affairs is bound to recede, indeed
is already diminishing. For some, the wish is the father of this thought. But where is the empirical evidence?
From South Asia, through relations with China and Russia through the Middle East peace process, through
dealing with Iran’s nuclear ambitions and North Korea’s nuclear weaponization and missile activities, through
confronting humanitarian crises in Africa and instability in Latin America , the United States has the
unchallenged diplomatic lead. Who could charge the Obama Administration with diplomatic passivity since taking
office? Indeed, one could instead conclude that the current global economic turbulence is causing countries to seek the familiar
and to rely more and not less on their American connection. In any event, foreigners (and some Americans) often
underestimate the existential resilience of the United States . In this respect, George Friedman’s new
book, The Next Hundred Years,14 and his view that the United States will be as dominant a force in the 21st century as it was in
the last half of the 20th century, is worth considering. So once again, those
who now predict, as they have in
every decade since 1945, American decay and withdrawal will be wrong 15— from John Flynn’s 1955
The Decline of the American Republic and How to Rebuild It,16 to Paul Kennedy’s 1987 The Rise and Fall of Great Powers,17 to
Andrew Bacevich’s 2008 The Limits of Power: The End of American Exceptionalism,18 to Godfrey Hodgson’s 2009 The Myth of
American Exceptionalism19 and many dozens of similar books in between. Indeed, the policies of the Obama Administration, for
better or worse, are likely to be far more influential and lasting regarding America’s longer-term geopolitical power projection
than the present economic decline. To sum up regarding the United States and the global economic worsening, former Council
on Foreign Relations President Les Gelb, in his new book, Power Rules: How Common Sense Can Rescue American Foreign
Policy,20 insists that a nation’s power is what it always was—essentially the capacity to get people to do what they
don’t want to do, by pressure and coercion, using one’s resources and position. . . . The world is not flat. . . . The shape of
global power is decidedly pyramidal—with the United States alone at the top, a second tier
of major countries (China, Japan, India, Russia, the United Kingdom, France, Germany and Brazil), and several tiers
descending below. . . . Among all nations, only the United States is a true global power with global reach. Lee Kuan Yew,
former Prime Minister of the Republic of Singapore, agrees: “ After the crisis, the US is most likely to remain at
the top of every key index of national power for decades . It will remain the dominant global player for the
next few decades. No major issue concerning international peace and stability can be resolved without US leadership, and no
country or grouping can yet replace America as the dominant global power.”21 The current global economic crisis will
not alter this reality. And the capitalist market model will continue to dominate international economics, not least because
China and India have adopted their own versions of it.
And, even if decline was uneven, this wouldn’t cause conflict or damage hegemony

Deudney 99 (Daniel, Asst Prof of Poli Sci at Johns Hopkins, Contested Grounds: Security and Conflict in the New Environmental
Politics )

Alterations in the relative power of states are unlikely to lead to war as readily as the lessons of history
suggest because economic power and military power are not as tightly coupled as in the past.
The relative economic power position of major states such as Germany and Japan has
changed greatly since the end of World War II. But these changes, while requiring many complex
adjustments in interstate relations, have not been accompanied by war or the threat of war. In the contemporary
world, whole industries rise, fall, and relocate, often causing quite substantial fluctuations in
the economic well-being of regions and peoples, without producing wars. There is no reason
to believe that changes in relative wealth and power positions caused by the uneven impact of
environmental degradation would be different in their effects.
---Econ- AT: Heg (Long)

Their internal link takes out their impact—if the world economy collapses, other countries will
suffer too—the U.S. may be absolutely weaker, but it will still be relatively stronger—
economic collapse would prevent other countries from militarizing to challenge us

Even if decline was uneven, this wouldn’t cause conflict or damage hegemony

Deudney, 99 (Daniel, Asst Prof of Poli Sci at Johns Hopkins, Contested Grounds: Security and Conflict in the New Environmental
Politics )

Alterations in the relative power of states are unlikely to lead to war as readily as the lessons of history suggest
because economic power and military power are not as tightly coupled as in the past . The
relative economic power position of major states such as Germany and Japan has changed
greatly since the end of World War II. But these changes , while requiring many complex adjustments in interstate relations,
have not been accompanied by war or the threat of war. In the contemporary world, whole industries rise, fall, and
relocate, often causing quite substantial fluctuations in the economic well-being of regions and
peoples, without producing wars. There is no reason to believe that changes in relative wealth
and power positions caused by the uneven impact of environmental degradation would be different in their effects.

Deterrence solves the impact—Russia proves

Deudney, 99 (Daniel, Asst Prof of Poli Sci at Johns Hopkins, Contested Grounds: Security and Conflict in the New Environmental
Politics )

Part of the reason for this loosening of the link between economic and military power has been
the nuclear revolution, which has made it relatively cheap for the leading states to deploy
staggering levels of violence capacity. Given that the major states field massively oversufficient
nuclear forces at the cost of a few percent of their GDP , environmentally induced economic decline would
have to be extreme before their ability to field a minimum nuclear deterrent would be
jeopardized. A stark example of this new pattern is the fact that the precipitious decline in Russia's
economy and defense spending in the 1990s has not diminished Russia's ability to deter great power
attack.

Economic decline doesn’t kill heg—American leadership is unique and their predictions have
been denied for decades

Blackwill, 09 – former associate dean of the Kennedy School of Government and Deputy Assistant to the President and Deputy
National Security Advisor for Strategic Planning (Robert, RAND, “The Geopolitical Consequences of the World Economic Recession—
A Caution”, http://www.rand.org/pubs/occasional_papers/2009/RAND_OP275.pdf, WEA)
Did the global recession weaken the political will of the United States
First, the United States, five years from today.

to, over the long term, defend its external interests? Many analysts are already forecasting a “yes” to this question. As a result of what they see as the
international loss of faith in the American market economy model and in U.S. leadership , they assert that Washington’s influence in international

affairs is bound to recede , indeed is already diminishing. For some, the wish is the father of this thought. But where is the empirical

evidence? From South Asia, through relations with China and Russia through the Middle East peace process, through
dealing with Iran’s nuclear ambitions and North Korea’s nuclear weaponization and missile activities, through
confronting humanitarian crises in Africa and instability in Latin America , the United States has the
unchallenged diplomatic lead. Who could charge the Obama Administration with diplomatic passivity since taking office? Indeed, one could instead
conclude that the current global economic turbulence is causing countries to seek the familiar and to rely more and not less on their American connection. In any event,

foreigners (and some Americans) often underestimate the existential resilience of the United States . In this respect,
George Friedman’s new book, The Next Hundred Years,14 and his view that the United States will be as dominant a force in the 21st century as it was in the last half of the 20th

those who now predict, as they have in every decade since 1945,
century, is worth considering. So once again,

American decay and withdrawal will be wrong 15— from John Flynn’s 1955 The Decline of the American Republic and How to Rebuild
It,16 to Paul Kennedy’s 1987 The Rise and Fall of Great Powers,17 to Andrew Bacevich’s 2008 The Limits of Power: The End of American Exceptionalism,18 to Godfrey Hodgson’s
2009 The Myth of American Exceptionalism19 and many dozens of similar books in between. Indeed, the policies of the Obama Administration, for better or worse, are likely to
be far more influential and lasting regarding America’s longer-term geopolitical power projection than the present economic decline. To sum up regarding the United States and
the global economic worsening, former Council on Foreign Relations President Les Gelb, in his new book, Power Rules: How Common Sense Can Rescue American Foreign

power is what it always was—essentially the capacity to get people to do what they don’t want to do, by pressure and
Policy,20 insists that a nation’s

coercion, using one’s resources and position. . . . The world is not flat. . . . The shape of global power is decidedly pyramidal—with

the United States alone at the top, a second tier of major countries (China, Japan, India, Russia, the United Kingdom,
France, Germany and Brazil), and several tiers descending below. . . . Among all nations, only the United States is a true global power with global reach. Lee Kuan Yew,

former Prime Minister of the Republic of Singapore, agrees: “ After the crisis, the US is most likely to remain at the top of

every key index of national power for decades . It will remain the dominant global player for the next few decades. No major issue
concerning international peace and stability can be resolved without US leadership, and no country or grouping can yet replace America as the dominant global power.”21 The

current global economic crisis will not alter this reality. And the capitalist market model will continue to dominate international economics, not
least because China and India have adopted their own versions of it.

There’s no incentive—no one will undermine the U.S.

Brooks and Wohlforth, 05 (Steven, Asst Prof and William, Professor, both at Dartmouth, International Security, Summer)

Ultimately, what appears new about the behavior analysts are calling soft balancing is not its significance, but its perceived
prominence on the agenda. What used to be considered standard diplomatic bargaining is now likely
trumpeted as balancing because real balancing of the kind that has appeared so often throughout history --
competing great-power alliances, arms buildups, brinkmanship crises, and the like -- was cleared off the international
agenda in 1989-91 with the end of the Cold War. Importantly, the concept of balancing rose to prominence in a world
in which great power security relations were dominated by the direct threat that they posed to each other. Today, by contrast, the
likelihood of great power war is exceedingly low. Weak states
and nonstates pose the main security
challenge, and the great powers argue primarily over the best way to address them. Balance of
power theory has no utility in explaining great power relations in this world .
---Econ- AT: Warming

Econ decline doesn’t turn warming

Gronewold ‘9 (Support for Ec0-Friendly Laws Remains, Despite Struggling Economy By NATHANIAL GRONEWOLD, ClimateWire
Published: February 24, 2009 NEW YORK –

Fears that governments will halt or even roll back efforts to tackle climate change as they struggle
to deal with an international financial crisis and rising unemployment appear unfounded. Instead, what is
happening is a global boom in new eco-friendly legislation. A new study published yesterday by
Deutsche Bank shows that governments' support for environmental and climate change initiatives
remains solid and strong, promising to help the clean technology industry revive from the current global
economic slump much sooner than most other industries. In a survey of recent government action around the globe
from July 2008 to February 2009, climate change business advisers and experts at the bank say that roughly 250 new
regulations supporting energy efficiency, renewable energy and climate change strategies were passed
during the period when economies were crashing. On top of an increasingly favorable regulatory
climate, new economic stimulus spending approved by multiple governments will further fuel a revitalization of
investment in clean energy and climate change mitigation, now threatened by stagnant credit markets. "Governments have been
stepping up the pace of legislation designed to help and support 'green' industries," said Mark Fulton, Deutsche Bank's head of
climate change investment research, in the report foreword. "The burst of activity on this front by the new Obama Administration in
the U.S. is particularly visible and will provide welcome leadership and focus to similar efforts across the globe." In an accounting of
various stimulus packages by governments around the world, Deutsche Bank analysts say that well over $200 billion will be spent
globally on green projects over the stimulus spending period. The lion's share of spending, $106 billion, will come from the U.S.
government, thanks to the package pushed forward by President Obama. China and the U.S. appear to be the biggest spenders The
report's authors say that the contribution to green business and climate change industries from China's announced stimulus
spending is harder to define, since that government's plan lacks enough clear specifics. But spending from Beijing should also
provide a big boost to the sector, especially for firms invested in energy efficiency technology and products. A "ramping up" of
government anti-climate change measures in other countries, as well, including Germany, the United Kingdom, France, New
Zealand, Greece and others, also bodes well for green-oriented companies and investors. Further efforts to place a price on carbon
dioxide pollution, especially regional and national developments in the United States, should also help bolster global carbon
markets, now wavering in the weak economy, analysts say. "We believe this trend towards greater regulation will
provide crucial support to climate change industries during the current global economic
downturn," Deutsche Bank says in the report. "Our research shows that, contrary to the widespread concern that
recession would force governments to abandon initiatives on this front, governments have in fact been increasing
their efforts." As evidence for their conclusion, the analysts point to a series of legislative milestones that have
occurred well into the onset of the current economic crisis.
---Econ- Bearden Indict (Short)
Bearden is a tool—he thinks zero point energy is the only way to save us, and he predicted
collapse for 2003—it’s either inevitable, or Bearden is an idiot

Bearden 2k (Lt. Col. Tom, PhD in Nuclear Engineering, “Zero-Point Energy”, April 25,
http://www.cheniere.org/correspondence/042500%20-%20modified.htm)

So in my view we
have about three years from now, counting, in which we must have changed the
scientific mindset and be in a massive production of self-powering electrical power systems
taking their energy from the vacuum. Otherwise, we will have overrun the "point of no
return" to Armageddon—an Armageddon of our own making and our own ignorance, and largely contributed to by our
organized scientific community. Science has the power to save us or destroy us, and right now it is bent upon the "business as usual"
course that will lead to our destruction.
---Econ- Bearden Indict (Long)

You should reject the Bearden evidence—

The only solution Bearden gives is zero-point energy

Bearden, 00 – Their Impact Author, the part they cut out (Thomas, "Unnecessary Energy Crisis", 2000, June 19th 2010,
MOORHEAD) Word Doc

So unless we dramatically and quickly solve the energy crisis — rapidly replacing a substantial
part of the "electrical power derived from oil" by "electrical power freely derived from the
vacuum" — we are going to incur the final "Great Armageddon" the nations of the world have been fearing for so long. I personally
regard this as the greatest strategic threat of all times — to the United States, the Western World, all the rest of the nations of the world, and civilization itself { } { }.

That is impossible—requires temperatures below absolute zero

Deck, 97 (Paul A. Deck, assistant professor of chemistry at Virginia Polytechnic Institute and State University. “FOLLOW-UP: What
is the 'zero-point energy' (or 'vacuum energy') in quantum physics? Is it really possible that we could harness this energy?” 8-18-97.
http://www.sciam.com/article.cfm?id=follow-up-what-is-the-zer&page=4)

The zero-point
Paul A. Deck, assistant professor of chemistry at Virginia Polytechnic Institute and State University, gives a chemical perspective on this question: "

energy cannot be harnessed in the traditional sense. The idea of zero-point energy is that there
is a finite, minimum amount of motion (more accurately, kinetic energy) in all matter, even at absolute zero .
For example, chemical bonds continue to vibrate in predictable ways. But releasing the energy of this motion is impossible, because

then the molecule would be left with less than the minimum amount that the laws of quantum
physics require it to have."

He bought his Ph.D. online and his theories are a joke

Skeptical Inquirer, 07 (“Dr.' Bearden's vacuum energy”, Martin Gardner, Jan-Feb, Bnet, http://www.csicop.org/si/2007-
01/fringe.html)

One of the strangest books ever written about modern physics was published in 2002 , and reprinted two
years later. Titled Energy from the Vacuum (Cheniere Press), this monstrosity is two inches thick and weighs three pounds. Its

title page lists the author as “Lt. Col. Thomas E. Bearden, PhD (U.S. Army retired).” “Dr.” Bearden is fond of
putting PhD after his name. An Internet check revealed that his doctorate was given, in his own
words, for “life experience and life accomplishment.” It was purchased from a diploma mill
called Trinity College and University—a British institution with no building, campus, faculty, or
president, and run from a post office box in Sioux Falls, South Dakota. The institution’s owner, one Albert Wainwright, calls himself the college
“registrant.” Bearden’s central message is clear and simple. He is persuaded that it is possible to extract unlimited free energy from the vacuum of space-time. Indeed, he believes the world is on the brink of its greatest technological revolution. Forget about nuclear
reactors. Vacuum energy will rescue us from global warming, eliminate poverty, and provide boundless clean energy for humanity’s glorious future. All that is needed now is for the scientific community to abandon its “ostrich position” and allow adequate funding to
To almost all physicists this quest for what is called “zero-point energy” (ZPE) is as
Bearden and his associates.

hopeless as past efforts to build perpetual motion machines . Such skepticism drives Bearden up a wall. Only monumental ignorance, he writes, could prompt
such criticism. The nation’s number two drumbeater for ZPE is none other than Harold Puthoff, who runs a think tank in Austin, Texas, where efforts to tap ZPE have been underway for years. In December 1997, to its shame, Scientific American ran an article praising
Puthoff for his efforts. Nowhere did this article mention his dreary past. Puthoff began his career as a dedicated Scientologist. He had been de¬clared a “clear”—a person free of malicious “engrams” recorded on his brain while he was an embryo. At Stanford
Research International, Puthoff and his then-friend Russell Targ claimed to have validated “remote viewing” (a new name for distant clairvoyance), and also the great psi powers of Uri Geller. (See my chapter on Puthoff’s search for ZPE in Did Adam and Eve Have
Navels?, Norton 2000.) Bearden sprinkles his massive volume with admirable quotations from top physicists, past and present, occasionally correcting mistakes made by Einstein and others. or example, Bearden be¬lieves that the graviton moves much faster than
the speed of light. He praises the work of almost every counterculture physicist of recent decades. He admires David Bohm’s “quantum potential” and Mendel Sach’s unified field theory. Oliver Heaviside and Nikola Tesla are two of his heroes. Bearden devotes
several chapters to antigravity machines. Here is a sample of his views: In our approach to antigravity, one way to approach the problem is to have the mechanical apparatus also the source of an intense negative energy EM field, producing an intense flux of Dirac
sea holes into and in the local surrounding space-time. The excess charge removed from the Dirac holes can in fact be used in the electrical powering of the physical system, as was demonstrated in the Sweet VTA antigravity test. Then movements of the mechanical
parts could involve movement of strong negative energy fields, hence strong curves of local space-time that are local strong negative gravity fields. Or, better yet, movement of the charges themselves will also produce field-induced movement of the Dirac sea hole
negative energy. This appears to be a practical method to manipulate the metric itself, along the lines proposed by Puthoff et al. [217] The 217 superscript refers to a footnote about a 2002 paper by Puthoff and two friends on how to use the vacuum field to power
spacecraft. Bearden’s anti-gravity pro¬pulsion system is neatly diagrammed on page 319. “Negatively charged local space-time,” says the diagram, “acts back upon source vehicle producing anti-gravity and unilateral thrust.” In the 1950s, numerous distinguished
writers, artists, and even philosophers (e.g., Paul Goodman, William Steig, and Paul Edwards) sat nude in Wilhelm Reich’s “orgone accumulators” to absorb the healing rays of “orgone energy” coming from outer space. Bearden suspects (in footnote 78) that orgone
energy “is really the transduction of the time-polarized photon energy into normal photon energy. We are assured by quantum field theory and the great negentropy solution to the source charge problem that the instantaneous scalar potential in¬volves this
process.” I doubt if the Reichians, who are still around, will find this illuminating. To my amazement Bearden has good things to say about the notorious “Dean drive”—a rotary motion device designed to propel spaceships by inertia. It was promoted by John

Only elementary physics is needed to


Campbell when he edited Astounding Science Fiction, a magazine that unleashed L. Ron Hubbard’s Dianetics on a gullible public and made Hubbard a millionaire.

show that no inertial drive can move a spaceship in frictionless space. On pages 448–453
Bearden lists eighty patents for inertial drives . They have one feature in common: none of them
works. Counterculture scientists tend to be bitter over the “establishment’s” inability to
recognize their genius. Was not Galileo, they like to repeat, persecuted for his great discoveries? This bitterness is sometimes accompanied by paranoid fears, not just of conspiracies to silence them, but also fears of being
murdered. Bearden’s pages 406–453 are devoted to just such delusions. Several kinds of “shooters” are described that induce fatal heart attacks. He himself, Bearden writes, has been hit by such devices. An associate, Stan Meyer, died after a “possible” hit by a
close-range shooter. Another ZPE researcher was killed by a bazooka-size shooter. Steve Marikov, still another researcher, was assaulted by a sophisticated shooter and his body thrown off a rooftop to make it appear a suicide. When his body was removed, the
pavement “glowed.” One day at a Texas airport a person three feet from Bearden was killed with symptoms suggesting he was murdered by an ice-dart dipped in curare! “That was apparently just to teach me ‘they’ were serious.” The colonel goes on to explain that
“they” refers to a “High Cabal” who were offended by a friend’s “successful transmutation of copper (and other things) into gold. . . . We have had numerous other assassination attempts, too numerous to reiterate. . . . Over the years probably as many as fifty or
more overunity researchers and inventors have been assassinated . . . some have simply disappeared abruptly and never have been heard from since.” Overunity is Bearden’s term for machines with energy outputs that exceed energy inputs. Any significant
researcher should be wary of “meeting with a sudden suicide” on the way to the supermarket. Another thing to beware of, is a calibrated auto accident where your car is rammed from the rear, and you are shaken up considerably. An ambulance just happens to be
passing by moments later, and it will take you to the hospital. If still conscious, the researcher must not get in the ambulance unless accompanied by a watchful friend who understands the situation and the danger. Otherwise, he can easily get a syringe of air into his
veins, which will effectively turn him into a human vegetable. If he goes to the hospital safely, he must be guarded by friends day and night, for the same reason, else he runs a high risk of the “air syringe” assassination during the night. Simply trying to do scientific
work, I find it necessary to often carry (legally) a hidden weapon. Both my wife and I have gun permits, and we frequently and legally carry concealed weapons. As early as the 1930s, T. Henry Moray—who built a successful COP>1.0 power system outputting 50kW
from a 55 lb power unit—had to ride in a bulletproof car in Salt Lake City, Utah. He was repeatedly fired at by snipers from the buildings or sidewalk, with the bullets sometimes sticking in the glass. He was also shot by a would-be assassin in his own laboratory, but

to experts, though I
overpowered his assassin and recovered. Obviously, I’m not competent to wade through Bearden’s almost a thousand pages to point out what physicists tell me are howlers. I leave that task

suspect very few will consider it worth their time even to read the book. , the book’s To me, a mere science journalist

dense, pompous jargon sounds like hilarious technical double-talk. The book’s annotated glossary runs to more than 120 pages. There are 305

In my opinion it is destined to be the


footnotes, 754 endnotes, and a valuable seventy-three-page index. The back cover calls the book “the definitive energy book of the twenty-first century.”

greatest work of outlandish science in both this and the previous century. It is much funnier, for instance, than Frank
Tipler’s best-seller of a few decades ago, The Physics of Immortality.

His experiment was based on unsound scientific principles

Phact.org, 05, updated 05 (Philadelphia Association for Critical Thinking, “Tom Bearden - A Critical Examination of His Claims”,
http://www.phact.org/e/z/BeardenReview.htm, REQ)

It is obvious Tom Bearden, JLN, and everyone involved in the MEG project, do not understand
basic textbook physics. This is not especially surprising, when one considers Tom Bearden has repeatedly stated everything
in standard textbooks is wrong. But without any real understanding of how to perform basic power
calculations, the MEG project was essentially flying ‘blind.’ Quite literally millions of dollars were
spent following flawed optimisation strategies, borne of simple measuring errors . If there is any
argument about this fact, the clearest indication that this is true, is that by his own admission, JLN was unable to
close the loop. If the real output as claimed exceeded input by a factor of 7, then closing the
loop ought to have been fairly simple, even with step down transformer losses accepted. The economical
charging of quality lead acid batteries is not especially difficult for anyone trained in the
scientific method, and this was clearly not possible. So experimental fact, would appear to
validate the theoretical analysis presented . I want to kindly thank Shawn Bishop for making these issues clear, and
taking the time to release a pdf document, summarizing the basic errors in the Bearden / JLN labs research claims. It is sad, but not
entirely surprising, that millions of dollars were raised, articles accepted for publication in the peer reviewed literature, all on the
basis of simple measuring errors, that were obvious all along for anyone who could be bothered to inspect the data presented.
---Econ- Lewis Answers (Short)

Their Lewis evidence says that countries will fight wars to prevent people from dropping out
of the world economy—this is wrong—

First, extend our Ferguson evidence—history shows no correlation between economic


collapse and war—there’s no data to support their argument and past economic downturns
like the Asian Flu in 1998 did not result in war. Our Sherden evidence says economist are
terrible at predictions—they inflate the risk of large downturns and miss the small ones

Second, it’s empirically false—Belarus, Cuba, Zimbabwe, Mongolia, and Burma have almost no
participation in the world economy—none of them are targets for nuclear colonial wars

And, this makes no sense—great powers would lose more than they gained by attempts to
subdue other states

Deudney 99 (Daniel, Asst Prof of Poli Sci at Johns Hopkins, Contested Grounds: Security and Conflict in the New Environmental
Politics )

Second, the prospects for resource wars are diminished by the growing difficulty that states face in
obtaining resources through territorial conquest. Although the invention of nuclear explosives has made it easy
and cheap to annihilate humans and infrastructure in extensive areas, the spread of conventional weaponry
and national consciousness has made it very costly for an invader, even one equipped with
advanced technology, to subdue a resisting population, as France discovered in Indochina and Algeria, the
United States in Vietnam, and the Soviet Union in Afghanistan. 45 At the lower levels of violence capability that matter most for
conquering and subduing territory, the great powers have lost effective military superiority and are
unlikely soon to regain it.
---Econ- Lewis Answers (Long)

Their Lewis evidence says that countries will fight wars to prevent people from dropping out
of the world economy—this is wrong—

History shows no correlation between economic collapse and war—there’s no data to support
their argument and past economic downturns like the Asian Flu in 1998 did not result in war—
that’s Ferguson.

It’s historically false—Belarus, Cuba, Zimbabwe, Mongolia, and Burma have almost no
participation in the world economy—none of them are targets for nuclear colonial wars

This makes no sense—great powers would lose more than they gained by attempts to subdue
other states

Deudney, 99 (Daniel, Asst Prof of Poli Sci at Johns Hopkins, Contested Grounds: Security and Conflict in the New Environmental
Politics )

Second, the prospects for resource wars are diminished by the growing difficulty that states face in
obtaining resources through territorial conquest . Although the invention of nuclear explosives has
made it easy and cheap to annihilate humans and infrastructure in extensive areas, the spread of
conventional weaponry and national consciousness has made it very costly for an invader , even
one equipped with advanced technology, to subdue a resisting population, as France discovered in
Indochina and Algeria, the United States in Vietnam, and the Soviet Union in Afghanistan . 45 At
the lower levels of violence capability that matter most for conquering and subduing territory,
the great powers have lost effective military superiority and are unlikely soon to regain it.

Changes in relative economic power don’t cause states to lash out or go to war—Germany and
Japan prove

Deudney, 99 (Daniel, Asst Prof of Poli Sci at Johns Hopkins, Contested Grounds: Security and Conflict in the New Environmental
Politics )

Alterations in the relative power of states are unlikely to lead to war as readily as the lessons of
history suggest because economic power and military power are not as tightly coupled as in the
past. The relative economic power position of major states such as Germany and Japan has
changed greatly since the end of World War II. But these changes , while requiring many complex
adjustments in interstate relations, have not been accompanied by war or the threat of war. In the
contemporary world, whole industries rise, fall, and relocate, often causing quite substantial
fluctuations in the economic well-being of regions and peoples, without producing wars . There is no
reason to believe that changes in relative wealth and power positions caused by the uneven impact of environmental degradation
would be different in their effects.
---Econ- Mead Answers (Short)
The only warrant in their mead card is a vague reference to World War Two—this is wrong—

First, extend our Ferguson evidence—the Depression cannot explain World War Two—it did
not cause fascism everywhere, and even when it did, this did not always cause war. Germany
only started the war after a long period of recovery, so this makes no sense

Second, World War Two was a unique event with many causes—the Treaty of Versailles,
offensive military theory, ideologies, the individual psychology of Hitler, the decline of
colonialism, appeasement, the geographic position of Germany, and the power balance in
Europe all helped cause the war—the Depression wasn’t a proximate cause of the war and it’s
impossible to predict future violence because World War II was caused by historically-specific
circumstances

Third, even if the Depression caused World War Two, this doesn’t mean it can happen again—
there have been many financial crises in the last sixty years that did not result in war, like the
1998 Asian Flu, the stagnation following OPEC’s embargo, the recession starting in 2008
---Econ- Mead Answers (Long)

The only warrant in their mead card is a vague reference to World War Two—this is wrong—

The Depression cannot explain World War Two—it did not cause fascism everywhere, and
even when it did, this did not always cause war. Germany only started the war after a long
period of recovery.

World War Two was a unique event with many causes—the Treaty of Versailles, offensive
military theory, ideologies, the individual psychology of Hitler, the decline of colonialism,
appeasement, the geographic position of Germany, and the power balance in Europe all
helped cause the war—the Depression wasn’t a proximate cause of the war and it’s
impossible to predict future violence because World War II was caused by historically-specific
circumstances

Even if the Depression caused World War Two, this doesn’t mean it can happen again—there
have been many financial crises in the last sixty years that did not result in war, like the 1998
Asian Flu, the stagnation following OPEC’s embargo, the recession starting in 2008

Mead has zero qualifications—he never went to grad school, has a B.A. in English literature
and has wrote economy impact cards for twenty years and not been right once.

Contrary historical examples don’t matter—modern economies are different and their
theories are wrong

Deudeny, 91 (Daniel, Hewlett Fellow in Science, Technology, and Society at the Center for Energy and Environmental Studies,
Princeton University, Bulletin of the Atomic Scientists, April)

Poverty wars. In a second scenario, declining living standards first cause internal turmoil, then war. If groups at all
levels of affluence protect their standard of living by pushing deprivation on other groups, class war and revolutionary upheavals could result. Faced with these pressures, liberal
democracy and free market systems could increasingly be replaced by authoritarian systems capable of maintaining minimum order. If authoritarian regimes are more war-
prone because they lack democratic control, and if revolutionary regimes are war-prone because of their ideological fervor and isolation, then the world is likely to become more

The record of previous depressions supports the proposition that widespread economic
violent.

stagnation and unmet economic expectations contribute to international conflict. Although initially
compelling, this scenario has major flaws. One is that it is arguably based on unsound economic theory .
Wealth is formed not so much by the availability of cheap natural resources as by capital formation through
savings and more efficient production. Many resource-poor countries, like Japan, are very wealthy, while
many countries with more extensive resources are poor. Environmental constraints require an end to economic growth based on growing use of raw materials, but not
economic decline does not necessarily produce
necessarily an end to growth in the production of goods and services. In addition,

conflict. How societies respond to economic decline may largely depend upon the rate at which such declines occur. And as people get poorer, they
may become less willing to spend scarce resources for military forces . As Bernard Brodie observed about the modern
era, “The predisposing factors to military aggression are full bellies, not empty ones .” The

experience of economic depressions over the last two centuries may be irrelevant, because such
depressions were characterized by under-utilized production capacity and falling resource
prices. In the 1930's, increased military spending stimulated economies, but if economic growth
is retarded by environmental constraints, military spending will exacerbate the problem.

This card uses the same language as the mead 92 card—pinning your hopes on the world
economy will not cause war even if that economy stagnates

Economists are terrible at predicting recessions—they inflate the risk of massive


collapses and fail to predict the smaller ones
Sherden, 98 (William, business consultant, The Fortune Sellers)

Today it is the generally perceived poor track record of economists that has caused the nickname to stick. Consider a 1995 Business
Week article, entitled “A D+ for Dismal Scientists? Even the Fed’s Gurus Often Goof,” and a 1996 Forbes article, “Dismal Days for the
Economic forecasters have routinely failed to foresee turning points in the
Dismal Science.”
economy: the coming of severe recessions, the start of recoveries, and periods of rapid
increases or decreases in inflation. It is jokingly said that economists have forecast nine of the last five
recessions. In fact, they have failed to predict the past four most severe recessions and most of
them predicted growth instead for these periods. After the October 1987 stock market crash,
most economists predicted a severe downturn in the economy similar to what happened after
the 1929 stock market crash, yet during the last quarter of 1987, the economy continued expanding
vigorously. Like all other types of forecasters, economists’ vision of the future is clearly clouded with
situational bias.
---Econ- Royal Answers

Their part of the Royal evidence is just a review of other peoples arguments- the conclusion
votes neg

Royal ‘10 (Jedediah Royal, Director of Cooperative Threat Reduction at the U.S. Department of Defense, 2010, “Economic
Integration, Economic Signaling and the Problem of Economic Crises,” in Economics of War and Peace: Economic, Legal and Political
Perspectives, ed. Goldsmith and Brauer)

CONCLUSION

The logic of ECST supports arguments for greater economic interdependence to reduce the
likelihood of conflict. This chapter does not argue against the utility of signalling theory. It does,
however, suggest that when considering the occurrence of and conditions created by economic
crises, ECST logic is dubious as an organising principle for security policymakers. The discussion
pulls together some distinct areas of research that have not yet featured prominently in the
ECST literature. Studies associating economic interdependence, economic crises and the
potential for external conflict indicate that global interdependence is not necessarily a conflict
suppressing process and may be conflict-enhancing at certain points. Furthermore, the conditions
created by economic crises decrease the willingness of states to send economic costly signals,
even though such signals may be most effective during an economic crisis. These two points warrant further consideration in the
debate over ECST and, more broadly, theories linking interdependence and peace. The debate takes on particular importance for
policymakers when considering the increasingly important US-China relationship and the long-term prospects for peace in the Asia-
Pacific. Recent US policy towards China, such as the ‘responsible stakeholder’ approach, assumes that greater interdependence with
China should decrease the likelihood for conflict. Some have even suggested that the economic relationship is necessary to ensure
strategic competition does not lead to major war (see, e.g., Kastner, 2006). If US or Chinese policymakers do indeed intend to rely on
economic interdependence to reduce the likelihood of conflict, much more study is required to understand how and when
interdependence impacts the security and the defence behaviour of states. This chapter contributes some thoughts to that larger
debate. NOTES I. Notable counterarguments include Barbieri (1996). Gowa (I994), and Levy and Ali I998 . 2.‘ Offi<):ial statements
have focused on this explanation as well. See, for example, Bernanke (2009). 3. For a dissenting study. see Elbadawi and Hegre
(2008). 4. Note that Skaperdas and Syropoulos (2001) argue that states will have a greater incentive to arm against those with which
it is interdependent to hedge against coercion. This argument could be extended to include protectionism in extreme cases.
Creseenzi (2005) both challenges and agrees with Copeland’s theory by suggesting that a more important indicator is the exit costs
involved in terminating an economic relationship. which could be a function of the availability of alternatives. 5.
There is also
substantial research to indicate that periods of strong economic growth are also positively
correlated with a rise in the likelihood of conflict. Pollins (2008) and Pollins and Schweller (I999) provide
excellent insights into this body of literature.

Note: ECST= Economic Costly Signaling Theory


Egypt Prolif Answers
Frontline

No Egypt prolif – lack of tech, high costs, low utility.

VOA News ’11 ( “Analysts Say Egypt Must Remain Free of Nuclear Weapons,” 2/17/2011, http://www.iiss.org/whats-new/iiss-
in-the-press/february-2011/analysts-say-egypt-must-remain-free-of-nuclear-weapons/?locale=en

But analysts agree that Egypt sought to acquire a nuclear weapons capability back in the 1960s .  Former
Defense Department official James Russell, who is now with the Naval Postgraduate School in Monterey, California says interest
in building a nuclear capability or developing an indigenous peaceful nuclear program ended during the
presidency of Gamal Abdel Nasser. "Nasser realized the deep financial costs that are going to be
required and he also realized the political costs that would accrue from an Egypt developing a
nuclear program," said Russell. "And so all these projects were canceled after the [Six Day] 1967 war. And then in 1968, Egypt
signs the Nuclear Nonproliferation Treaty [NPT]. And since then the Sadat government ratifies the NPT in February 1981. And in
1982, they have a safeguards agreement, a comprehensive safeguards agreement with the International Atomic Energy Agency
[IAEA]." Analyst Mark Fitzpatrick, from London's International Institute for Strategic Studies (IISS), says Egypt
does not have
the most modern technologies that could be used to produce fissile material for nuclear
weapons. "The issue is, though, that Egypt conducted some experiments in plutonium separation that they did not notify the
IAEA in accordance with their safeguards agreement and it later came out and the IAEA did an investigation," noted Fitzpatrick. "And
most of that was cleared up, but there are some remaining questions. And most recently, some highly-enriched uranium particles
were discovered by the IAEA and I think they are still investigating the source of that. That all has not yet come out in IAEA reporting,
so there is some small cloud hanging over Egypt's nuclear program. I am not saying they are going for nuclear weapons, but they did
some things that were not fully in accordance with the rules." Fitzpatrick says the plutonium experiments were conducted during the
past couple of decades and came to light about six or seven years ago. And the evidence concerning the highly-enriched uranium
particles was disclosed in the media within the last two years. Egypt, Iraq and WMDs On another issue, some analysts, including
James Russell, say there have been reports of Egyptians in the 1980s helping Iraq with its chemical weapons research. "There were
also rumors of the Egyptians being involved in building a chemical plant that could possibly have made items that were used for
chemical munitions," added Russell. "But again, I don't think that there is any assessment out there today
that suggests that the Egyptians are engaged in research or really have any interest in
developing chemical weapons." And Russell says Egypt does not have the necessary industrial base in
the chemical sector to build chemical weapons . "The truth of the matter is that almost all countries around the
world - the United States and Russia included - everyone has come to the conclusion that these are not just
terribly militarily useful weapons, or that it is a technology which really has limited use in the
military arena," Russell explained. Post Mubarak Concens Some experts have questioned whether a government replacing
President Hosni Mubarak might reconsider some of Cairo's policies on weapons of mass destruction. " In the chemical and
nuclear arenas,the costs to the Egyptians of attempting such programs are significant ," added
Russell. "And it just seems to me that  any political leadership in Egypt, whatever its character, is going to
have to look at these costs.  And they are a strong discouragement to them moving down this path
to sort of reconsidering the decisions which have been looked at by previous political leadership .  I
just do not see it.

Historic trends prove no proliferation


Fahmy ’11 (Nabil Fahmy, chairman of the Middle East Project at the James Martin Center for Nonproliferation Studies (CNS) and
the dean of the School of Global Affairs and Public Policy at the American University in Cairo, “The Egyptian Revolution and the
Future of WMD in the Middle East: Q&A With Nabil Fahmy,” 3/1/2011,http://cns.miis.edu/activities/110228_fahmy_egypt_qna.htm

 
Fahmy: Well, so far no date has yet been set for the conference. One must not forget that the exact timing of the event will be
defined by other considerations. Some have argued that it may not be entirely practical to try organizing the conference before the
2012 NPT Preparatory Committee (PrepCom). I don't think it matters whether it takes place before or after the PrepCom. But it
should be held before the end of 2012. With
regard to Egypt, I want to be clear that  Egypt's nuclear politics
have not fundamentally changed since 1974, notwithstanding changes in presidents and governments. Therefore, I
don't foresee substantial changes to Egypt's nuclear policy absent regional provocations. I would
also add that the Egyptian Foreign Ministry is a professional service, mostly unaffected by the
changes in the ruling party and government .

No one supports prolif in Egypt

Fahmy ’11 (Nabil Fahmy, chairman of the Middle East Project at the James Martin Center for Nonproliferation Studies (CNS) and
the dean of the School of Global Affairs and Public Policy at the American University in Cairo, “The Egyptian Revolution and the
Future of WMD in the Middle East: Q&A With Nabil Fahmy,” 3/1/2011,http://cns.miis.edu/activities/110228_fahmy_egypt_qna.htm

As for possible candidates, one would expect a leader or leaders would emerge from the
January 25th youth movement (which led the revolution). But that hasn't happened yet. Amr Moussa, the
Arab League Secretary General, has expressed interest in the job but has yet to formally announce his candidacy. Mohamed
ElBaradei, the former Director General of the International Atomic Energy Agency and a Nobel Laureate, has declared that he won't
run, but "if asked, won't disappoint the people." Of course, ElBaradei has come under a lot of fire recently in Egypt for being out of
the country when the revolution started. I personally think he deserves more credit for his role as "the godfather of the
revolution." Other candidates
may emerge in the days to come . Neither Moussa nor ElBaradei are
expected to dramatically change Egypt's foreign policy, and I don't believe that anyone who can
get elected nationally would. Whomever is elected, however, will recalibrate it with a stronger national and proactive
focus. Either Moussa or ElBaradei would retain a centrist approach but make it tougher and more constructive. Both are
"achievers." They
don't like remaining on the sidelines, or being negative, and  neither constitutes a
threat or would change Egypt's positions on WMD issues and the NPT.  In fact, Moussa has
always been a proponent of joining the NPT,  but then became a strong advocate for Israel
joining the NPT as well, while ElBaradei initially was not an advocate of Egypt becoming a party
to the NPT, although having joined he believes in fully abiding by our treaty obligations .
ElBaradei tends to realistically link progress on the regional WMD situation to the achievement
of peace in the Middle East. I agree with ElBaradei that Egypt should not have joined the NPT before Israel, but disagree
with him that now these issues should be linked to regional peace overall, even implicitly.

Long timeframe and U.S. diplomacy solves – no technical infrastructure, no country would
refuse the U.S. nuclear umbrella

Jakobsen 10 – Associate Professor at the Department of Political Science, University of Copenhagen (Peter Viggo, Ali
Rahigh-Aghsan, Assistant Professor at the Department of Society and Globalisation, Roskilde University, Autumn, “The Rise of Iran:
How Durable, How Dangerous?” ProQuest)
The danger of an arms race is not imminent, however, as the current
aspirations for nuclear energy in the region will take
10-15 years to fulfill.60 This leaves considerable time for the US to persuade Saudi Arabia, Egypt, and
Turkey not to go nuclear, and the US is in a position to give them an offer they cannot refuse. Since
all these states and the rest of the GCC countries effectively depend upon the US for their security in the interim until they can
develop their own nuclear deterrent, the US can offer to extend its nuclear umbrella to them and provide
continued military, economic, and political support contingent on their not developing nuclear weapons. This
tactic worked in similar circumstances with Taiwan and South Korea during the Cold War, and there is no reason why it should not
work in the Middle East as well.61 The penalties the US can impose if they renege on this promise are
prohibitive and since none of these countries have an interest in appeasing Iran unless forced to do so for lack of an
alternative, it is hard to see why they should not accept such an offer. It is also inconceivable that
the US would refrain from extending such an offer to its allies given the strategic importance that it
attaches to the region. Since Iran is not capable of hitting the US mainland with nuclear missiles, the US would not have to sacrifice
New York to protect Amman and this makes extended nuclear deterrence inherently more credible in the Middle East than it was in
Western Europe during the Cold War.
Ethnic Conflict Answers
Frontline

No ethnic conflict

Habyarimana et al ‘8 (Habyarimana, professor at Gtown, et al 8 [JAMES HABYARIMANA is Assistant Professor of Public Policy
at Georgetown University. MACARTAN HUMPHREYS is Assistant Professor of Political Science at Columbia University. DANIEL
POSNER is Associate Professor of Political Science at UCLA. JEREMY WEINSTEIN is Assistant Professor of Political Science at Stanford
University. RICHARD ROSECRANCE is Adjunct Professor of Public Policy at the John F. Kennedy School of Government and Senior
Fellow at the Belfer Center for Science and International Affairs, both at Harvard University. ARTHUR STEIN is Professor of Political
Science at UCLA., “Is Ethnic Conflict Inevitable?,” Foreign Affairs July/August 08,
http://www.foreignaffairs.com/articles/64457/james-habyarimana-macartan-humphreys-daniel-posner-jeremy-weinst/is-ethnic-
conflict-inevitable]

Jerry Muller ("Us and Them," March/April 2008) tells a disconcerting story about the potential for ethnic diversity to generate
violent conflict. He argues that ethnic nationalism -which stems from a deeply felt need for each people to have its own state -"will
continue to shape the world in the twenty-first century." When state and ethnic-group boundaries do not coincide, "politics is apt to
remain ugly." Muller points to the peace and stability in Europe today as evidence of the triumph of "the ethnonationalist project": it
is only because of a half century of violent separation of peoples through expulsions, the redrawing of state boundaries, and the
outright destruction of communities too weak to claim territories of their own that Europe today enjoys relative peace. Elsewhere,
the correspondence between states and nations is much less neat, and there Muller seems to agree with Winston Churchill that the
"mixture of populations [will] ...cause endless trouble." He advocates partition as the best solution to this difficult problem. If
correct, his conclusion has profound implications both for the likelihood of peace in the world and for what might be done to
promote it. But is it correct? Do ethnic divisions inevitably generate violence? And why does ethnic diversity sometimes give rise to
conflict? Infact, ethnic differences are not inevitably, or even commonly, linked to violence on a
grand scale. The assumption that because conflicts are often ethnic, ethnicity must breed
conflict is an example of a classical error sometimes called "the base-rate fallacy ." In the area of
ethnic conflict and violence, this fallacy is common . To assess the extent to which Muller falls prey to it, one
needs some sense of the "base." How frequently does ethnic conflict occur , and how often does it occur in the
context of volatile mismatches between ethnic groups and states? A few years ago, the political scientists James Fearon
and David Laitin did the math. They used the best available data on ethnic demography for every
country in Africa to calculate the "opportunities" for four types of communal conflict between independence and 1979: ethnic
violence (which pits one group against another), irredentism (when one ethnic group attempts to secede to join co-ethnic
communities in other states), rebellion (when one group takes action against another to control the political system), and civil war
(when violent conflicts are aimed at creating a new ethnically based political system). Fearon and Laitin identified tens of thousands
of pairs of ethnic groups that could have been in conflict. But they did not find thousands of conflicts (as might have
been expected if ethnic differences consistently led to violence) or hundreds of new states (which partition would have created).
for every one thousand such pairs of ethnic groups, they found fewer than three
Strikingly,
incidents of violent conflict . Moreover, with few exceptions, African state boundaries today look just as
they did in 1960. Fearon and Laitin concluded that communal violence, although horrifying, is extremely
rare. The base-rate fallacy is particularly seductive when events are much more visible than
nonevents. This is the case with ethnic conflict, and it may have led Muller astray in his account of the triumph of European
nationalism. He emphasizes the role of violence in homogenizing European states but overlooks the peaceful consolidation that has
resulted from the ability of diverse groups -the Alsatians, the Bretons, and the Provençals in France; the Finns and the Swedes in
Finland; the Genoese, the Tuscans, and the Venetians in Italy -to live together. By failing to consider the conflicts that did not
happen, Muller may have misunderstood the dynamics of those that did. Of course, ethnic divisions do lead to violent conflict in
some instances. Violence may even be so severe that partition is the only workable solution. Yet this extreme response has not been
required in most cases in which ethnic divisions have existed. Making sense of when ethnic differences generate conflict -and
knowing how best to attempt to prevent or respond to them when they do -requires a deeper understanding of how ethnicity
works.
EMP Attack Answers
Frontline

No EMP impact

Farley ‘9 (The EMP Threat: Lots of Hype, Little Traction By Robert Farley | Posted: October 16, 2009 Robert Farley is an assistant
professor at the University of Kentucky’s Patterson School of Diplomacy and International Commerce and a contributor to PRA’s
Right Web (http://rightweb.irc-online.org/).

Uncertainty regarding the effect of EMP has fed alarmist predictions about overall impact. For example, although there is agreement
that high-altitude nuclear detonations can cause widespread damage to the electric grid and to electronic and digital equipment,
there is little agreement on the size of the nuclear weapon necessary to cause significant, long-lasting destruction. The test that
damaged electronic equipment in Hawaii measured 1.4 megatons, roughly one hundred times larger than the most powerful nuclear
test attributed to North Korea. However, numerous EMP awareness advocates (and some members of the EMP Commission) have
argued that a much smaller warhead could destroy electronics from the East Coast to the Midwest. In the absence of conclusive
research and testing, the exact size of the explosion necessary to create a devastating EMP remains unknown. Many weapons
experts doubt that an EMP attack could cause lasting or irreversible damage. Stephen Younger, former
senior fellow at Los Alamos National Lab and director at the Defense Threat Reduction Agency, argues that while an EMP might
create problems in the short term, it is unlikely to cause long-term devastation. Similarly, observers have
questioned the capacity of North Korea or Iran, much less a terrorist organization, to develop a warhead
sophisticated enough to cause widespread EMP damage. Nick Schwellenbach, a former researcher
at Project on Government Oversight, suggests that the idea of a small, EMP-optimized warhead
is absurd: "You have a lot of points of failure in order to get to a warhead that is EMP optimized. … [Y]ou need
specialized machine tools, you need capital, but to create a weapon that creates the secondary effect that you're talking about,
that's something even we can't do right now.” [9] At this point, neither Iran nor North Korea possess a missile capable of delivering
an EMP attack against the United States. However, Graham, as well as Peter Pry, the president of EMPACT America and former
senior staffer with the EMP Commission, have argued in Congressional testimony that Iran could launch a medium-range ballistic
missile from an offshore barge or freighter, thus giving the Islamic Republic first-strike capability. Moreover, EMP awareness
advocates have argued that if terrorists acquired a ballistic missile and a nuclear warhead, they could conduct the same kind of
offshore attack. The strategic logic of an EMP attack on the United States remains unclear, and skeptics’ doubts
mostly focus
on the strategic implausibility of such attacks. Under the most aggressive assumptions, a first-
strike EMP attack might cause widespread economic damage. However, under no scenario
would the attack eliminate the ability of the U.S. military to respond. Al Mauroni of the defense contractor
Science Applications International Corporation argues that “the national command authority would be able to identify where a
missile came from, determine the effects of such an attack, and respond with nuclear weapons—not necessarily just for an EMP
effect—against the adversarial nation.” [10] Former Rep. Curt Weldon, who gave the EMPACT conference’s opening address, argued
back in 1997 that it would be politically difficult for the United States to respond to such an attack, as no cities will have been
destroyed and no lives lost (at least initially), a claim which other EMP awareness advocates have echoed. However, that the United
EMP awareness
States would not respond with overwhelming military force to a successful EMP attack strains credulity.
advocates have thus far failed to offer a convincing motive for why a rogue state would use its scarce
nuclear weapons in a first-strike that might not work, and that would in any case leave the
attacker open to a devastating counterattack. EMP as a second-strike deterrent fares no better;
the strategic logic of deterrence demands that any retaliatory strike be as lethal and as secure as
possible, and it is highly unlikely that any state would rely on unproven weaponry of uncertain lethality to dissuade an attack. While
terrorists may have different incentives, the road to a functional EMP capability is much rockier for a terrorist group than a state. At
a minimum, the terrorist group would need to acquire and master the operation of a nuclear weapon and a ballistic missile, two
steps further than any known group has gone. Ulterior Motives The central political purpose of the EMP awareness movement
appears to be advancement of the cause of missile defense. The
most extreme estimates of the effect of EMP
restore the Cold War-era existential fears of nuclear war. Schwellenbach argues "what's driving
it is the political global context—it gives the right an issue that allows them to justify hawkish
behavior. It is almost a perfect solution to any argument against missile defense —North Korea and
Iran.” [11] The 90 percent casualty estimate advanced by EMP awareness advocates hypes the notion that the United States faces
potential annihilation at the hands of its enemies, and goes a step farther: even the smallest nuclear power can destroy the United
States with a small number of warheads. This, in turn, reaffirms the need for both a secure missile defense shield (including space-
based interceptor weapons) and a grand strategy of preventive war against potential nuclear and ballistic missile proliferators.
Almost all EMP awareness advocates—including Gaffney, Gingrich, and Huckabee—call for increased spending on missile defense.
Gaffney and Gingrich have also called for a “robust” policy of preemptive war, including attacks on Iranian and North Korean missiles
The fact that EMP is poorly researched and not well understood works in its
on their launching pads.
favor as a scare tactic. Since evidence of EMP’s allegedly lasting impact is purely theoretical, EMP awareness advocates can
make outlandish claims regarding the threat that even the smallest nuclear arsenal poses. They can also point to allegations made by
the official EMP Commission, ignoring the fact that many outside experts dispute its findings. The Niagara conference’s emphasis on
strategic and policy considerations shows that alarmist predictions about EMP attacks serve as fodder for promotion of a larger
nuclear weapons stockpile, for missile defense, and for preventive attacks. Little Traction Despite the effort that conservatives
have devoted to this cause, it appears to have gained little traction in the mainstream media. The New York Times, the Washington
Post, CNN, Fox News, and other major television news organizations declined to cover the EMPACT
conference. Indeed, even the neoconservative Weekly Standard, which seems perpetually on
the lookout for ways to plug purported existential threats to the homeland, stayed away from
Niagara. One Standard editor said in an interview with the author, “I don't go for that EMP stuff. Kind of more interested in
dangerous scenarios that might actually happen .” [12] Nevertheless, the presence of Huckabee
and Gingrich at the conference indicates that some major Republican Party politicians see EMP
either as a splendid political opportunity, or as their latest conservative litmus test.

No EMP impact and deterrence checks

STRATFOR ‘10 (Stratfor, “ Gauging the Threat of an Electromagnetic Pulse (EMP) Attack,”
http://www.stratfor.com/weekly/20100908_gauging_threat_electromagnetic_pulse_emp_attack, September 9, 2010)

In order to have the best chance of causing the type of immediate and certain EMP damage to the United States
on a continent-wide scale, as discussed in many media reports, a nuclear weapon (probably in the
megaton range) would need to be detonated well above 30 kilometers somewhere over the American Midwest. Modern
commercial aircraft cruise at a third of this altitude. Only the United States, United Kingdom, France, Russia and
China possess both the mature warhead design and intercontinental ballistic missile (ICBM) capability to
conduct such an attack from their own territory, and these same countries have possessed that capability for decades. (Shorter
range missiles can achieve this altitude, but the center of the United States is still 1,000 kilometers from the Eastern Seaboard and
more than 3,000 kilometers from the Western Seaboard — so just any old Scud missile won’t do.) The HEMP threat is nothing new.
It has existed since the early 1960s, when nuclear weapons were first mated with ballistic missiles, and grew to be an important
component of nuclear strategy. Despite the necessarily limited understanding of its effects, both the United States and Soviet Union
almost certainly included the use of weapons to create HEMPs in both defensive and especially offensive scenarios, and both post-
Soviet Russia and China are still thought to include HEMP in some attack scenarios against the United States. However, there are
significant deterrents to the use of nuclear weapons in a HEMP attack against the United States, and nuclear weapons have not been
used in an attack anywhere since 1945. Despite some theorizing that a HEMP attack might be somehow less destructive and
therefore less likely to provoke a devastating retaliatory response, such an
attack against the United States would inherently
and necessarily represent a nuclear attack on the U.S. homeland and the idea that the United States
would not respond in kind is absurd. The United States continues to maintain the most credible and survivable nuclear
deterrent in the world, and any actor contemplating a HEMP attack would have to assume not that they might experience some
limited reprisal but that the U.S. reprisal would be full, swift and devastating. Countries that build nuclear weapons do so at great
expense. This is not a minor point. Even today, a successful nuclear weapons program is the product of years — if not a decade or
more — and the focused investment of a broad spectrum of national resources. Nuclear weapons also are developed as a deterrent
to attack, not with the intention of immediately using them offensively. Once a design has achieved an initial capability, the focus
shifts to establishing a survivable deterrent that can withstand first a conventional and then a nuclear first strike so that the nuclear
arsenal can serve its primary purpose as a deterrent to attack. The coherency, skill and focus this requires are difficult to overstate
and come at immense cost — including opportunity cost — to the developing country. The idea that Washington will interpret the
use of a nuclear weapon to create a HEMP as somehow less hostile than the use of a nuclear weapon to physically destroy an
American city is not something a country is likely to gamble on. In other words, for the countries capable of carrying out a HEMP
attack, the
principles of nuclear deterrence and the threat of a full-scale retaliatory strike
continue to hold and govern, just as they did during the most tension-filled days of the Cold War.
Environment Answers
Frontline

Environmental improvements now – their evidence ignores long term trends

Hayward ‘11 [Steven P, american author, political commentator, and policy scholar. He argues for libertarian and conservative
viewpoints in his writings. He writes frequently on the topics of environmentalism, law, economics, and public policy.2011 Almanac
of Environmental Trends¶ by Steven F. Hayward¶ April 2011¶ ISBN-13: 978-1-934276-17-4,
http://www.pacificresearch.org/docLib/20110419_almanac2011.pdf]

Quick: What’s the largest public-policy success story in American society over the last generation? The dramatic
reduction in the crime rate, which has helped make major American cities livable again? Or welfare reform, which saw the nation’s
welfare rolls fall by more than half since the early 1990s? Both of these accomplishments have received wide media attention. Yet
the right answer might well be the environment. As Figure 1 displays, the reduction in air pollution is comparable in
magnitude to the reduction in the welfare rolls, and greater than the reduction in the crime rate—both celebrated as major public-
policy success stories of the last two decades. Aggregate emissions of the six “criteria” pollutants1 regulated under the Clean

Air Act have fallen by 53 percent since 1970, while the proportion of the population receiving welfare assistance is down
48 percent from 1970, and the crime rate is only 6.4 percent below its 1970 level. (And as we shall see, this aggregate nationwide
reduction in emissions greatly understates the actual improvement in ambient air quality in the areas with the worst levels of air

pollution.) Measures for water quality , toxic -chemical exposure , soil erosion , forest growth ,
wetlands , and several other areas of environmental concern show similar positive trends , as
this Almanac reports. To paraphrase Mark Twain, reports of the demise of the environment have been
greatly exaggerated . Moreover, there is good reason to believe that these kinds of improvements will
be experienced in the rest of the world over the course of this century. We’ll examine some of the
early evidence that this is already starting to occur . The chief drivers of environmental improvement are economic
growth, constantly increasing resource efficiency, technological innovation in pollution control, and the deepening of environmental
values among the American public that have translated to changed behavior and consumer preferences. Government regulation has
played a vital role, to be sure, but in the grand scheme of things regulation can be understood as a lagging indicator, often achieving
results at needlessly high cost, and sometimes failing completely. Were it not for rising affluence and technological innovation,
regulation would have much the same effect as King Canute commanding the tides. INTRODUCTION introduction 3 figure 1 a
comparison of crime rate, Welfare, and air Pollution, 1970–2007 -60.0% -40.0% -20.0% 0.0% 20.0% 40.0% 60.0% 1970 1975 1980
1985 1990 1995 2000 2005 2007 % of Population on Welfare Crime Rate (per 100,000 population) Aggregate Emissions Source: FBI
Uniform Crime Reports, U.S. Department of Health and Human Services, EPA 4 Almanac of Environmental Trends The American
public remains largely unaware of these trends. For most of the last 40 years, public opinion about the environment has been
pessimistic, with large majorities—sometimes as high as 70 percent—telling pollsters that they think environmental quality in the
United States is getting worse instead of better, and will continue to get worse in the future. One reason for this state of opinion is
media coverage, which emphasizes bad news and crisis; another reason is environmental advocacy groups, for whom good
news is bad news. As the cliche goes, you can’t sell many newspapers with headlines about airplanes landing safely, or about an oil
tanker docking without a spill. Similarly, slow, long-term trends don’t make for good headline copy.
INTRODUCTIONintroduction 5Improving Trends:Causes and ConsequencesMost environmental commentary dwells on the laws and
regulations we have adoptedto achieve our goals, but it is essential to understand the more important role of technologyand
economic growth in bringing about favorable environmental trends. Thebest way to see this is to look at some long-term trends in
environmental quality thatpredate modern environmental legislation.To be sure, the earliest phases of the Industrial Revolution led
to severe environmentaldegradation. But the inexorable process of technological innovation andthe drive for efficiency began to
remedy much of this damage far earlier than iscommonly perceived. In addition, new technologies that we commonly regard as
environmentally destructive often replaced older modes of human activity that were far worse by comparison. A
good example is the introduction of coal for heating andenergy in Britain.

No impact to the environment- hype


Ridder ‘8 – PhD, School of Geography and Environmental Studies, University of Tasmania (Ben, Biodiversity And Conservation,
17.4, “Questioning the ecosystem services argument for biodiversity conservation”) *ES = environmental services

The low resilience assumption

Advocates of the conservation of biodiversity tend not to acknowledge the distinction between resilient and sensitive ES. This
‘low resilience assumption’ gives rise to, and is reinforced by the almost ubiquitous claim within the conservation literature that
ES depend on biodiversity. An extreme example of this claim is made by the
Ehrlichs in Extinction. They state that “all
[ecosystem services] will be threatened if the rate of extinctions continues to increase” then observe that
attempts to artificially replicate natural processes “are no more than partially successful in most cases. Nature nearly always
does it better. When society sacrifices natural services for some other gain… it must pay the costs of substitution” (Ehrlich and
Ehrlich 1982, pp. 95–96). This assertion—that the only alternative to protecting every species is a world in which all ES have
been substituted by artificial alternatives—is an extreme example of the ‘low resilience assumption’. Paul Ehrlich revisits this
flawed logic in 1997 i nhis response (with four co-authors) to doubts expressed by Mark Sagoff regarding economic arguments
for species conservation (Ehrlich et al. 1997, p. 101). The claim that ES depend on biodiversity is also notably present in the
controversial Issues in Ecology paper on biodiversity and ecosystem functioning (Naeem et al. 1999) that sparked the debate
mentioned in the introduction. This appears to reflect a general tendency among authors in this field (e.g., Hector et al. 2001;
Lawler et al. 2002; Lyons et al. 2005). Although such authors may not actually articulate the low resilience assumption,
presenting such claims in the absence of any clarification indicates its influence. That
the low resilience assumption is
largely false is apparent in the number of examples of species extinctions that have not brought
about catastrophic ecosystem collapse and decline in ES, and in the generally limited ecosystem influence
of species on the cusp of extinction. These issues have been raised by numerous authors, although given the absence of
systematic attempts to verify propositions of this sort, the evidence assembled is usually anecdotal and we are forced to trust
that an unbiased account of the situation has been presented. Fortunately a number of highly respected people have discussed
this topic, not least being the prominent conservation biologist David Ehrenfeld. In 1978 he described the ‘conservation
dilemma’, which “arises on the increasingly frequent occasions when we encounter a threatened part of Nature but can find no
rational reason for keeping it” (Ehrenfeld 1981, p. 177). He continued with the following observation: Have there been
permanent and significant ‘resource’ effects of the extinction, in the wild, of John Bartram’s great discovery, the beautiful tree
Franklinia alatamaha, which had almost vanished from the earth when Bartram first set eyes upon it? Or a thousand species of
tiny beetles that we never knew existed before or after their probable extermination? Can we even be certain than the eastern
forests of the United States suffer the loss of their passenger pigeons and chestnuts in some tangible way that affects their
vitality or permanence, their value to us? (p. 192) Later, at the first conference on biodiversity, Ehrenfeld (1988) reflected that
most species “do not seem to have any conventional value at all” and that the rarest species are “the
ones least likely to be missed… by no stretch of the imagination can we make them out to be vital
cogs in the ecological mach ine” (p. 215). The appearance of comments within the environmental literature that are
consistent with Ehrenfeld’s—and from authors whose academic standing is also worthy of respect—is uncommon but not
unheard of (e.g., Tudge 1989; Ghilarov 1996; Sagoff 1997; Slobodkin 2001; Western 2001). The low resilience assumption is also
undermined by the overwhelming tendency for the protection of specific endangered species to be justified by moral or
aesthetic arguments, or a basic appeal to the necessity of conserving biodiversity, rather than by emphasising the actual ES
these species provide or might be able to provide humanity. Often the only services that can be promoted in this regard relate
to the ‘scientific’ or ‘cultural’ value of conserving a particular species, and the tourism revenue that might be associated with its
preservation of such services is of an entirely different order compared with the
continued existence. The
collapse of human civilization predicted by the more pessimistic environmental authors. The
popularity of the low resilience assumption is in part explained by the increased rhetorical force of
arguments that highlight connections between the conservation of biodiversity, human survival and economic profit. However,
it needs to be acknowledged by those who employ this approach that a number of negative implications are associated with any
use of economic arguments to justify the conservation of biodiversity.

This impact is flawed science and is empirically denied

Campbell ‘11 (Hank, Science Writer for Science 2.0, “I Wouldn't Worry About The Latest Mass Extinction Scare,” March 8 th,
http://www.science20.com/science_20/i_wouldnt_worry_about_latest_mass_extinction_scare-76989,
You've seen it everywhere by now - Earth's sixth mass extinction: Is it almost here? and other articles discussing an article
in Nature (471, 51–57 doi:10.1038/nature09678) claiming the end of the world is nigh. Hey, I like to live in important times. So
do most people. And something so important it has only happened 5 times in 540 million years, well that is really special. But
is it real? Anthony Barnosky, integrative biologist at the University of California at Berkeley and first author of the paper,
claims that if currently threatened species, those officially classed as critically endangered, endangered, and vulnerable, actually
went extinct, and that rate of extinction continued, the sixth mass extinction could arrive in 3-22 centuries. Wait, what??
If you know anything about
That's a lot of helping verbs confusing what should be a fairly clear issue, if it were clear.
species and extinction, you have already read one paragraph of my overview and seen the
flaws in their model. Taking a few extinct mammal species that we know about and then
extrapolating that out to be extinction hysteria right now if we don't do something about
global warming is not good science. Worse, an integrative biologist is saying evolution does not happen. Polar
bears did not exist forever, they came into existence 150,000 years ago - because of the Ice Age. Greenpeace co-founder and
ecologist Dr. Patrick Moore told a global warming skepticism site, “I quit my life-long subscription to National Geographic when
they published a similar 'sixth mass extinction' article in February 1999. This [latest journal] Nature article just re-hashes this
theme” and "The fact that the study did make it through peer-review indicates that the peer review process has become
corrupted.” Well, how did it make it through peer review? Read this bizarre justification of their methodology; "If you look
only at the critically endangered mammals--those where the risk of extinction is at least 50 percent within three of their
generations--and assume that their time will run out and they will be extinct in 1,000 years, that puts us clearly outside any
range of normal and tells us that we are moving into the mass extinction realm." Well, greater extinctions occurred
when Europeans visited the Americas and in a much shorter time. And since we don't know how
many species there are now, or have ever been, if someone makes a model and claims tens of thousands of species are going
extinct today, that sets off cultural alarms. It's not science, though. If only 1% of species have gone extinct in the groups we
really know much about, that is hardly a time for panic, especially if some 99
percent of all species that have
ever existed we don't know anything about because they... went extinct. And we did not. It
won't keep some researchers, and the mass media, from pushing the panic button. Co-
author Charles Marshall, also an integrative biologist at UC-Berkeley wants to keep the panic
button fully engaged by emphasizing that the small number of recorded extinctions to date
does not mean we are not in a crisis. "Just because the magnitude is low compared to the
biggest mass extinctions we've seen in half a billion years doesn't mean they aren't
significant." It's a double negative, bad logic and questionable science, though.

Adaptation solves

Thompson et al. ‘9 (Ian Thompson et al., Canadian Forest Service, Brendan Mackey, The Australian National University, The
Fenner School of Environment and Society, College of Medicine, Biology and Environment, Steven McNulty, USDA Forest Service,
Alex Mosseler, Canadian Forest Service, 2009, Secretariat of the Convention on Biological Diversity “Forest Resilience, Biodiversity,
and Climate Change” Convention on Biological Diversity

Concerns have been expressed that predicted cli- mate changes (IPCC 2007) may occur too quickly for
species to adapt (Huntley 1991, Davis and Shaw 2001, Jump and Penuelas 2005), but genetically diverse species
are capable of rapid evolution (Geber and Dawson 1993). Many species have adapted to rapid
changes and have done so repeatedly over geo- logical time through dispersal and genetic
changes based on the extant genetic diversity within local or regional gene pools , suggesting
long-term genetic- based resilience to change. There is considerable evidence for adaptation in
the geological and fossil record (Bernabo and Webb 1977, Webb 1981, Davis 1983, Huntley and Birks 1983, and review
by Geber and Dawson 1993). Such adaptation has been demonstrated by forest plants during or following past
glacial and interglacial episodes, which were characterized by relatively rapid climate change
(Huntley and Webb 1988).

The environment is resilient and indestructible

Easterbrook ‘95 (Distinguished Fellow, Fullbright Foundation (Gregg, A Moment on Earth pg 25)

IN THE AFTERMATH OF EVENTS SUCH AS LOVE CANAL OR THE Exxon Valdez oil spill, every
reference to the environment is prefaced with the adjective "fragile ." "Fragile environment" has become a
welded phrase of the modern lexicon, like "aging hippie" or "fugitive financier." But the notion of a fragile environment is profoundly
wrong. Individual animals, plants, and people are distressingly fragile. The
environment that contains them is close to
indestructible . The living environment of Earth has survived ice ages; bombardments of cosmic
radiation more deadly than atomic fallout; solar radiation more powerful than the worst-case
projection for ozone depletion; thousand-year periods of intense volcanism releasing global
air pollution far worse than that made by any factory; reversals of the planet's magnetic poles;
the rearrangement of continents; transformation of plains into mountain ranges and of seas
into plains; fluctuations of ocean currents and the jet stream ; 300-foot vacillations in sea levels;
shortening and lengthening of the seasons caused by shifts in the planetary axis; collisions of
asteroids and comets bearing far more force than man's nuclear arsenals; and the years
without summer that followed these impacts. Yet hearts beat on, and petals unfold still. Were
the environment fragile it would have expired many eons before the advent of the industrial affronts
of the dreaming ape. Human assaults on the environment, though mischievous, are pinpricks
compared to forces of the magnitude nature is accustomed to resisting .

No impact to the environment- 99.9% can die

Sagoff 97  Mark, Senior Research Scholar – Institute for Philosophy and Public policy in School of Public Affairs – U. Maryland,
William and Mary Law Review, “Institute Of Bill Of Rights Law Symposium Defining Takings: Private Property And The Future Of
Government Regulation: Muddle Or Muddle Through? Takings Jurisprudence Meets The Endangered Species Act”, 38 Wm and Mary
L. Rev. 825, March, L/N

Although one may agree with ecologists such as Ehrlich and Raven that the earth stands on
the brink of an episode of massive extinction, it may not follow from this grim fact that
human beings will suffer as a result. On the contrary, skeptics such as science writer Colin Tudge have challenged
biologists to explain why we need more than a tenth of the 10 to 100 million species that grace the earth. Noting that
"cultivated systems often out-produce wild systems by 100-fold or more," Tudge declared
that "the argument that humans need the variety of other species is, when you think about it, a theological one." n343 Tudge
observed that "the elimination of all but a tiny minority of our fellow creatures does not affect the material
well-being of humans one iota." n344 This skeptic challenged ecologists to list more than 10,000 species (other than
unthreatened microbes) that are essential to ecosystem productivity or functioning. n345 " The human species could
survive just as well if 99.9% of our fellow creatures went extinct , provided only that we retained the
appropriate 0.1% that we need." n346   [*906]   The monumental Global Biodiversity Assessment ("the Assessment") identified
two positions with respect to redundancy of species. "At one extreme is the idea that each species is unique and important,
such that its removal or loss will have demonstrable consequences to the functioning of the community or ecosystem." n347
The authors of the Assessment, a panel of eminent ecologists, endorsed this position, saying it is "unlikely that there is much, if
any, ecological redundancy in communities over time scales of decades to centuries, the time period over which environmental
policy should operate." n348 These eminent ecologists rejected the opposing view, "the notion that species overlap in function
to a sufficient degree that removal or loss of a species will be compensated by others, with negligible overall consequences to
the community or ecosystem." n349  Other biologists believe, however, that species
are so fabulously redundant
in the ecological functions they perform that the life-support systems and processes of the
planet and ecological processes in general will function perfectly well with fewer of them, certainly fewer
than the millions and millions we can expect to remain even if every threatened organism
becomes extinct . n350 Even the kind of sparse and miserable world depicted in the movie Blade Runner could provide a
"sustainable" context for the human economy as long as people forgot their aesthetic and moral commitment to the glory and
beauty of the natural world. n351 The Assessment makes this point. "Although any ecosystem contains hundreds to thousands
of species interacting among themselves and their physical environment, the emerging consensus is that the system is driven by
a small number of . . . biotic variables on whose interactions the balance of species are, in a sense, carried along."
n352   [*907]   To make up your mind on the question of the functional redundancy of species, consider an endangered species
of bird, plant, or insect and ask how the ecosystem would fare in its absence. The fact that the creature is endangered suggests
an answer: it is already in limbo as far as ecosystem processes are concerned. What crucial ecological services does the black-
capped vireo, for example, serve? Are any of the species threatened with extinction necessary to the provision of any ecosystem
service on which humans depend? If so, which ones are they?  Ecosystems and the species that compose them have changed,
dramatically, continually, and totally in virtually every part of the United States. There is little ecological similarity, for example,
between New England today and the land where the Pilgrims died. n353 In view of the constant reconfiguration of the biota,
one may wonder why Americans have not suffered more as a result of ecological catastrophes. The cast of species in nearly
every environment changes constantly- local extinction is commonplace in nature- but the crops still
grow . Somehow, it seems, property values keep going up on Martha's Vineyard in spite of the tragic disappearance of the
heath hen.  
---Environment- Adaptation

Functional diversity – numerous species serve the same purpose

Thompson ‘9., Ian et al Canadian Forest Service, Brendan Mackey, The Australian National University, The Fenner School of
Environment and Society, College of Medicine, Biology and Environment, Steven McNulty, USDA Forest Service, Alex Mosseler,
Canadian Forest Service, 2009, Secretariat of the Convention on Biological Diversity “Forest Resilience, Biodiversity, and Climate
Change” Convention on Biological Diversity

We make reference to functional redundancy, functional types or species, and functional groups. Several studies have established
that resilience in ecosystems is related to the biological diversity in the system and the capacity that it confers to main- tain
ecosystem processes (Walker 1995, Peterson et al. 1998, Loreau et al. 2001, Hooper et al. 2005, Drever et al. 2006, Bodin and
Wimen 2007). Most ecosystem processes are controlled by, or are the result of, biodiversity. However, not
all species are
necessarily equally important in maintaining these processes (Walker 1992, 1995, Diaz et al. 2003) and
there is some redundancy at multiple levels within most ecosystems (Hooper et al. 2005). Functional
groups are assemblages of species performing similar functional roles within an ecosystem, such
as pollination, production, or decomposition , hence providing the ecosystem with a level of redundancy (e.g., see
Hooper et al. 2002). As discussed further below, functional diversity is not necessarily corre- lated with species richness (Diaz and
Cabido 2001, Hooper et al. 2005). Functional species that domi- nate ecosystem processes are not inevitably the most numerous
species in the system (e.g., Hooper and Vitousek 1997, Diaz et al. 2003), and it is important to understand which species are
contributing most to maintaining the flows of goods and services if management or protection is an objective. We are especially
interested in functional diversity (within functional groups) in ecosystems because evidence has accumulated,
especially in grassland systems, which implicates a relationship between functional diversity and
ecosystem properties, including resilience and the related system attributes of stability and
resistance (Diaz and Cabido 2001, Hooper et al. 2005). Under changed conditions, however, spe- cies that had a limited or no
functional role (“pas- senger” species) may become functionally dominant (“driver” species), hence buffering the ecosystem against
large changes and conferring resilience; that is, passengers can become the drivers (Walker 1995). This variable response has also
been termed ‘func- tional response diversity’ and is critical to ecosystem resilience (Chapin et al. 1997, Elmqvist et al. 2003). Loss of
functional species in the absence of redun- dancy has negative consequences for the ecosystem to the point of ecosystem collapse
(Chapin et al. 1997). Hooper et al. (2005) noted that there is a clear need for continued research into the relationship between
species richness and ecosystem stability.

Ecosystems are resilient

Thompson ‘9., Ian et al Canadian Forest Service, Brendan Mackey, The Australian National University, The Fenner School of
Environment and Society, College of Medicine, Biology and Environment, Steven McNulty, USDA Forest Service, Alex Mosseler,
Canadian Forest Service, 2009, Secretariat of the Convention on Biological Diversity “Forest Resilience, Biodiversity, and Climate
Change” Convention on Biological Diversity

Forests have many unique properties, related to their high rates of primary productivity and biodi- versity, which distinguish
them ecologically from other ecosystems. Such properties include biologi-
cal structures that develop in vertical
and horizontal layers of live and dead plants, complex processes at multiple vertical levels from within
soil layers up to the canopy, the capacity for self-renewal in the face of constant small and large
disturbances, co-evolved plant-animal and plant-plant interactions , and the influence forest landscapes can
have on micro- and regional climates, especially in closed-canopy tropi- cal forests. Forests are comprised of multiple ecosys- tems
that are associated with variable edaphic and microclimate conditions across broad landscapes.
Adaptation and gene diversity create resilience

Thompson ‘9., Ian et al Canadian Forest Service, Brendan Mackey, The Australian National University, The Fenner School of
Environment and Society, College of Medicine, Biology and Environment, Steven McNulty, USDA Forest Service, Alex Mosseler,
Canadian Forest Service, 2009, Secretariat of the Convention on Biological Diversity “Forest Resilience, Biodiversity, and Climate
Change” Convention on Biological Diversity

One of the best approaches, when dealing with an uncertain future, is diversification because no single approach will fit all
situations, and this applies also to the development of forest management strategies (Ledig and Kitzmiller 1992, Millar et al. 2007).
In the biological realm, maintaining species and ge- netic diversity addresses the need to be prepared for whatever environmental
changes might happen, and this is fundamental to the concept of resilience. Species
have two main means by which
they adapt to change: they can either disperse by seed or vegetative propagules in the direction
of a more favourable environment, or they can change their gene frequencies to favour
genotypes (genetic constitutions) that are better adapted to the changed environment (Bur- don and Thrall 2001, Reusch et al.
2005). Species may also adapt through phenotypic plasticity , if their genotype entails a range of permissible
responses (with respect to the species morphological, physi- ological, behavioural or life history strategies and traits) that are suited
to the new conditions (Nussey et al. 2005). Seed and pollen
dispersal, and gene frequency changes can occur
simultaneously and interact in the process of adaptation . For instance, dispersal often promotes gene
flow among highly fragmented tree populations ; thereby maintaining within-popu- lation levels of genetic diversity
and preventing the genetic drift and loss of genetic diversity that can occur through inbreeding within small, isolated or fragmented
tree populations (Hall et al. 1996, Young et al. 1996, Nason and Hamrick 1997, Cascante et al. 2002, Rajora et al. 2002, Fuchs et al.
2003, Mos- seler et al. 2004, Degen et al. 2006, Clouthier et al. 2007, O’Connell et al. 2007, Farwig et al. 2008). Seed dispersal can
occur through wind and water, or via animals such as birds, mammals, etc. Operational forestry experience and observations have
shown that seeds can be dispersed over surprisingly long distances over relatively short time
frames. Seeds of light-seeded species, such as conifers, can travel long distances from the nearest population cen- tres (Cwynar
and MacDonald 1987). Conifers with semi-serotinous cones, such as black spruce (Picea mariana), red pine (Pinus resinosa), and
pitch pine (Pinus rigida), for example, seem particularly well adapted for such long-distance dispersal over hard- packed snow and
ice. Ritchie and MacDonald (1986) have suggested that wind dispersal over snow may also explain the rapid post-glacial migration
rates of conifers that have non-serotinous cones, such as white spruce (Picea glauca). However, long-distance seed dispersal of
typically wind-dispersed conifers could also be explained through dispersal by birds (Wilkinson 1997). Large or heavy-seeded species,
such as those found in mangroves (Geng et al. 2008), and especially those in highly fragmented environ- ments, may have greater
difficulty travelling across landscapes (e.g., walnuts [Juglans spp.], hickories [Carya spp.]). Nevertheless, oaks (Quercus spp.) (Skellam
1951, Davis 1981) and American beech (Fagus grandifolia) (Bennett 1985) are capable of rapid and widespread dispersal given the
presence of certain animal species. Generally, by
dispersing their seeds and pollen, for- est species can
maintain their genetic diversity, and hence their long-term resilience to change over space and
time, by re-establishing themselves else- where in favourable climates. However, anthropo- genic changes
to landscapes and gene pools may have reduced this capacity, and population fragmen- tation has the potential to adversely affect
the genetic and reproductive status of populations.

Species adapt and migrate

Thompson ‘9., Ian et al Canadian Forest Service, Brendan Mackey, The Australian National University, The Fenner School of
Environment and Society, College of Medicine, Biology and Environment, Steven McNulty, USDA Forest Service, Alex Mosseler,
Canadian Forest Service, 2009, Secretariat of the Convention on Biological Diversity “Forest Resilience, Biodiversity, and Climate
Change” Convention on Biological Diversity

While resilience can be attributed to many levels of organization of biodiversity, the genetic composition of
species is the most fundamental. Molecular genet- ic diversity within a species, species diversity within
a forested community, and community or ecosystem diversity across a landscape and bioregion
represent expressions of biological diversity at different scales. The basis of all expressions of biological
diversity is the genotypic variation found in populations. The individuals that comprise populations at each level of
ecological organization are subject to natural se- lection and contribute to the adaptive capacity or re-
silience of tree species and forest ecosystems (Mull- er-Starck et al. 2005). Diversity at each of these levels has
fostered natural (and artificial) regeneration of forest ecosystems and facilitated their adaptation to dramatic climate
changes that occurred during the quaternary period (review by: DeHayes et al. 2000); this diversity must be maintained in the face
of antici- pated changes from anthropogenic climate warming. Genetic
diversity (e.g., additive genetic variance) within a
species is important because it is the basis for the natural selection of genotypes within popu- lations
and species as they respond or adapt to en- vironmental changes (Fisher 1930, Pitelka 1988, Pease et al. 1989, Burger
and Lynch 1995, Burdon and Thrall, 2001, Etterson 2004, Reusch et al. 2005, Schaberg et al. 2008). The potential for
evolutionary change has been demonstrated in numerous long- term programmes based on artificial
selection (Fal- coner 1989), and genetic strategies for reforestation in the presence of rapid climate
change must focus on maintaining species diversity and genetic diversi- ty within species (Ledig and Kitzmiller 1992). In the
face of rapid environmental change, it is important to understand that the genetic diversity and
adap- tive capacity of forested ecosystems depends largely on in situ genetic variation within each
population of a species (Bradshaw 1991). Populations exposed to a rate of environmental change exceeding the rate at which
populations can adapt, or disperse, may be doomed to extinction (Lynch and Lande 1993, Burger and Lynch 1995). Genetic diversity
deter- mines the range of fundamental eco-physiological tolerances of a species. It governs inter-specific competitive interactions,
which, together with dispersal mechanisms, constitute the fundamental de- terminants of potential species responses to change
(Pease et al. 1989, Halpin 1997). In the past, plants have responded to dramatic changes in climate both
through adaptation and migration (Davis and Shaw 2001). The capacity for long-distance migration of plants by seed
dispersal is particularly important in the event of rapid environmental change. Most, and probably all, species are capable
of long-distance seed disper- sal, despite morphological dispersal syndromes that would indicate morphological
adaptations primarily for short-distance dispersal (Cwyner and MacDon- ald 1986, Higgins et al. 2003). Assessments of mean
migration rates found no significant differences be- tween wind and animal dispersed plants (Wilkinson 1997, Higgins et al. 2003).
Long-distance migration can also be strongly influenced by habitat suitabil- ity (Higgins and Richardson 1999) suggesting that rapid
migration may become more frequent and vis- ible with rapid changes in habitat suitability under scenarios of rapid climate change.
The discrepancy between estimated and observed migration rates during re-colonization of northern temperate forests following
the retreat of glaciers can be accounted for by the underestimation of long-distance disper- sal rates and events (Brunet and von
Oheimb 1998, Clark 1998, Cain et al. 1998, 2000). Nevertheless, concerns persist that potential migration and ad- aptation rates of
many tree species may not be able to keep pace with projected global warming (Davis 1989, Huntley 1991, Dyer 1995, Collingham et
al. 1996, Malcolm et al. 2002). However, these models refer to fundamental niches and generally ignore the ecological interactions
that also govern species dis- tributions.
---Environment- 99.9%

No impact to biodiversity- 99.9% can die and we are fine

Tudge 89 (Colin, Biologist, Scientifict Fellow @ the Zoological Society of London, Fellow @ the Linean Society of London,
Former Visiting Fellow @ Centre for the Philosophy of the Natural and Social Sciences, London School of Economics, has given many
lectures and seminars at the Zoological Society of London; the Sanger Centre, the Linnean Society of London, the Royal Society, the
Royal Society of Medicine, The Royal Institution, the Oxford Union, the Darwin Seminars, London School of Economics, the
University of Leeds, the University of East Anglia; The Eden Project, Cornwall, The Macaulay Institute, “The rise and fall of Homo
sapiens sapiens,” Published by the Royal Society, JSTOR,

The possibility of human extinction has certainly been suggested of late, on several grounds, including nuclear winter, epidemic
(such as AIDS), and - the matter that concerns us here - because of our own destruction of the planet. In particular, it has been
suggested that we are sowing the seeds of our own destruction by destroying so many other species; that we need a planet
that is in ecological 'balance'; and that that balance depends upon the multitude of other species, perhaps between 10 and 30
million, that the Earth is thought to contain. If that argument were true, it would be very powerful from a conservationist point
of view. I take it to be self-evident that human beings are important; even being exaggeratedly detached, we can hardly deny
that our species is an interesting biological experiment, and it would be a pity if it were snuffed out before its time. But I take it
also to be self-evident that ours is not the only important species; that other creatures have a ' right' to occupy this planet, and
that we at times have to bow to their needs, even at cost to ourselves. Those self-evident 38 [ 239 ] Vol. 325. B 480 C. TUDGE
truths are the basis of' Green' philosophy. But most people, I think, take only the first of those premises to be self-evident.
Most people, if pressed, would probably maintain in a way that is not incompatible with much of the apparent teaching of the
Bible, that other animals and plants were 'put on Earth' for our convenience, and that although we shouldn't be cruel to them,
we may dispose of them at our will. In other words, the moral philosophy of the Greens is not exclusively anthropocentric,
whereas that of most of humanity is. If you are in a minority, of whatever kind, then it pays as far as possible to demonstrate
that your philosophy is compatible, and preferably congruent, with that of the majority. Thus it is that Greens have been
anxious to show, these past few years, that a moral philosophy that is not entirely anthropocentric is coincident in its effects
with one that is exclusively anthropocentric. Specifically, to bring the discussion down to earth, they have tried to show that
human beings benefit from the variousness of other creatures. Well, do we? The answer, after we've run the gauntlet of devil's
advocacy, is 'up to a point'; which is Evelyn Waugh's euphemism for 'not really'. The arguments that affect to show that a
wealth of other species is good for us are of two kinds, specific and general. Specifically, it's pointed out, for example,
that new drugs might be found in the roots of plants as yet unexamined, or in the glands of tree frogs; or that the wild relatives
of present-day crops - or even, in these days of genetic engineering, the non-relatives of crops - contain genes that may confer
resistance to disease; or that people could derive income from wild animals, by attracting tourists, for example, or by allowing
limited hunting of animals such as the black rhinoceros. All these arguments are true. The examples abound, or at least make
an impressive list. But none of them is critical. The human species is not dying for lack of drugs, and if you should say, 'what
about AIDS?' we might answer 'does anyone believe that the best strategy for seeking an AIDS therapy is to search among the
glands of tree-frogs? Wild ground nuts from South America recently supplied breeders at the International Crops Research
Institute for the Semi-Arid Tropics (ICRISAT) in India with genes that protected the domestic crop against rust (Gibbons
I985). Very valuable, but not critical; and if it came to a toss-up between saving wilderness for its possible complement of
genes, and planting that same wilderness with crops of known value, it would be perverse (if the extra food were really needed)
to opt for the wild species. Some Africans do make money from elephants, but if oil is discovered beneath the reserves, what
price the wildlife? Besides, we might argue that saving particular species may itself help to perpetrate mass extinction. True,
the coat-tail effect is well known; a reserve designed to harbour some particularly charismatic' species will also contain a huge
number of hangers-on, just as some of the tiger reserves in India also provide homes for jungle cats. But this can work the
other way. The bontebok of South Africa, a rare subspecies of the blesbok, very properly has its own small national park. It is
good for the bontebok, but the park was established on land that once was fynbos, with its fabulous assemblage of species
The more general
based upon proteas and ericas. But the fynbos has been banished locally, because bontebok prefer grass.
argument in favour of natural variety is that human beings in some way depend upon the natural food
webs that almost invariably are highly complex and rich in species. For example, it is commonly argued - in essence
- that if tropical forest is removed or decimated so that the number of species is reduced, then what remains degenerates into
desert, which is of no use to anyone. But this argument simply isn't true. A greatly simplified forest, dominated by
commercial species of Eucalyptus, dipterocarp or Aralcaria, stands up just as well, [240 ] THE RISE AND FALL OF HOMO
SAPIENS SAPIENS 481 and as far as we know for just as long, as pristine tropical forest that contains hundreds of species of
tree. True, if you replace tropical forest with grassland and then overgraze it, the grass is liable to degenerate. But it's not the
loss of species that counts, it is the change of habit; that and a level of husbandry that probably isn't properly matched to the
demands of the tropics. Mangroves seem to provide a cast-iron example of natural variety leading intricately but nonetheless
inexorably to human benefit. Mangroves contain several species of trees which, in Queensland at least, according to studies by
Tom Smith at the Australian Institute of Marine Science (T. Smith, personal communication), in turn depend oddly enough
upon un- prepossessing crabs to spread their propagules; there are algae in there, and detritus, and a host of insect larvae and
Protozoa; all providing food and shelter, eventually, for the larvae of fish that grow into the kind that people love to eat. Take
the mangrove away - or indeed, take individual elements away, such as the crabs - and the edible fish disappear as well. There
can be no argument with this. Yet a conscientious devil's advocate would point out that the fish that are nurtured in mangroves
are for the most part eaten by rich people who are over-fed to start with; and indeed might point out that fish as a whole,
including the apparently vital tilapias of Africa and the enormous yields of cod and the like from high latitudes, contribute a
remarkably small proportion of the total protein and energy intake of human beings, and that most of what is consumed is
indeed consumed by people who don't need it. An average monetarist - nothing so grand as a devil's advocate, which is a
sacred office - could point out that most of the luxury species that Queenslanders or Floridians love to eat can perfectly well be
farmed (salmon, turbot, catfish, abolone, giant clams, oysters, and numerous prawns are among the animals that take well to
life in a pond or a cage); and if they are farmed they can be fed on ground beef, raised in Illinois. The mangroves can then be
given over to hotels, as in Miami; and the tourists will pay to visit the fish farms, which can easily be turned into theme parks,
and generate far more wealth, with far more human comfort, than miles of pristine and singularly inhospitable mangrove.
Indeed, when you think about it, it is obvious that the
people-need-natural-variety argument is false, on
two grounds. The first is that cultivated systems, whether of intensive grain or for fish, are always
more productive than wild systems because they absorb a much higher level of nutrient, and process it much
more efficiently into human food. Most wild plants hate being over-nourished; and indeed, fertilizer escaping from arable
farms, even in small amounts, is in many places the greatest single threat to the marvellous,,natural variety of the Australian
bush. But because they prefer infertile conditions, the output of wild plants is bound to be relatively meagre. Indeed, cultivated
systems often out-produce wild systems by 100-fold or more. But cultivated systems are inevitably simplified. They should not,
of course, be monocultures, but there is no deep ecology in that; it's just a matter of sensible husbandry. But few cultivated
systems contain more than a dozen or so species; orders of magnitude fewer than the wild environment. Secondly, the
argument that humans need the variety of other species is, when you think about it, a theological one.
It would be likely to be true only if the Lord had indeed created the world for our express benefit. If we reject that
notion, as Green thinkers do on moral grounds and as post-Darwinian scientists are bound to do, then we must
concede that other species are for the most part totally detached from any consideration of
human welfare, and that the loss of most of them would do us no demonstrable harm, while
the loss of several - [ 241 ] 38-2 482 C. TUDGE including many of the genus Anopheles - would be a definite plus. The loss of
the Large Copper butterfly from the English Fens has done the British people no material harm at all, and unless the Fens had
been drained they could not have become one of the world's most intensive foci of arable farming. Most societies through
most of history have persecuted the wolf, and it is impossible to show that the demise of dozens of subspecies, and one or two
full species, of wolf- like animals, has had the slightest adverse effect on human material wellbeing. I wish it were not so. I wish
we could demonstrate that people need Large Coppers and wolves. But we cannot. Thus my first conclusion in this diabolically
adversarial role is that the elimination of all but a tiny minority of our fellow creatures does not affect the
material wellbeing of humans one iota; and indeed, that if human beings really want to take over the world, then they
are obliged to tidy most other living creatures away. This is what the European colonialists set out to do when they first
encountered the fauna of Africa, and it is what all farmers have done, assiduously and deliberately, since the neolithic
revolution began around 10000 years ago. In fact, if we were to appoint a committee to make a short list of creatures that
truly contributed to human wellbeing, then I doubt if it would contain more than 10000 species; one tenth of one per cent of
the number conservatively estimated now to be on Earth. And that list would include the black rhino for millionaires to hunt,
and the Lady Amherst pheasant for ordinary people to look at. There has never been such a mass extinction; but if human
beings care only about their material wellbeing and a little sport, they would not need to worry about it at
all. Indeed the only concern that human beings need have about their fellow creatures, a competent devil's advocate would
point out, is whether there are enough. Never mind the species, what's the biomass? Provided
we can produce
enough cellulose, then in an age of biotechnology we can feed ourselves . And here there are two
questions that are linked but are none the less separate, and should be treated separately. First, there is the matter of human
numbers; can the world as it now is, or as we may contrive to make it, support all the people there are liable to be in the next
few decades and centuries? Secondly, are we by our activities reducing the capacity of the world to provide biomass, and is
this putative reduction irredeemable? The two issues of course compound each other, but they are separate issues
nonetheless. Human numbers are, of course, staggering. - There is an ecological law - a simple extrapolation of bedrock
physics - which says that' large, predatory animals are rare. We break that law: we are large and have a penchant for pTedation,
yet our population now stands at five billion; and of all feasible demographic projections the one that comes nearest to
consensus says that this will double to around 10 billion by the middle of the 21st century, that it will remain at such a figure
for several centuries, and that it will then begin to decline, in theory to some figure that our distant descendants feel is
appropriate. Nuclear war or some form of super-AIDs could of course make nonsense of such figures. But these figures do
represent the ground state. If the fabric of the Earth stayed as it is throughout that time, and if we add a little more science
(as we will), and organize, the world a bit better, reducing some of the awful inequities between north and south, for example,
then there is no doubt that the world could accommodate such numbers without difficulty. Britain's farming is as intensive as
any in the world, but agricultural scientists agree that with present technologies, and without claiming more land, output
could easily be increased by at least 2500 Along with most western [242 ] THE RISE AND FALL OF HOMO SAPIENS
SAPIENS 483 countries, we give the greater proportion of our home-grown cereal and pulses to livestock. So if we farmed
competently and ate less meat we could probably feed around 200 million people in Britain alone. Much of the rest of the
world is incapable of such intensive output, but on the other hand, most of the rest makes a far worse job of realizing whatever
potential it has, than we do here. If the world really pulled its socks up and if some of us were less greedy, then even with
present techniques we could probably feed not 10 but 20 billion people fairly comfortably. This, however, is where we run into
the second consideration; whether the world can continue to be as productive as it is now. The issues are not simple. It isn't
true to argue, for example, as some Green philosophers like to, that intensive food production inevitably and invariably leads
to soil degradation. There are fields at Rothamsted, in Hertfordshire, that have produced cereal every year for 140 years,
without added manure, and they are in better heart now than at the beginning. Though the straw and grain have been
harvested, organic matter has been maintained by the rotting roots. You cannot treat heath in this way, but any soil can go on
being productive, and indeed improve in agricultural terms, provided you stay within its limits; and the limits of some soils are
very high indeed. On the other hand, we cannot ignore the general argument of Paul Ehrlich, of Stanford (Ehrlich & Ehrlich
I987), that much of present-day food production depends not upon sustaining soil but on mining it; that in many soils, if not
most, there is a steady loss of 'heart', and indeed of the soil itself, as it washes or blows into the sea; that there is a net
increase of undesirables, such as soil salinity, which can be very hard to correct; that some useful commodities such as fossil
fuels are being destroyed forever, while others, such as phosphorus and many metals, are being spread around the planet and
will become increasingly difficult to harvest. Overall, there is a degradation of the planet's fabric. To a large extent this could be
arrested, or circumvented: soil salination can be reversed, as is happening in places in Australia; the loss of fossil fuels need
not matter, as there is enough energy in surplus straw to run a tractor and fix nitrogen. But it is clear that the technologies to
correct the ill effects of over-farming are not being applied fast enough, and won't be in the foreseeable future. It is obvious,
then, that human numbers would have had to stop increasing at some point; and Professor Ansley Coale at Princeton has
pointed out that our population would have reached 17 trillion (1018) within 700 years if the rate of increase of the 1960s had
been maintained (Coale I974, I987). It is clear, too, that the numbers will level out sooner than optimists might have hoped, as
the planet's capacity to produce is underminded. Exactly where the cut-off will be, and when we will reach it, is no& clear.
What does seem to me extremely likely is that the monetarist argument that the human species will back away from disaster
for economic reasons - that as production becomes difficult so demand will reduce - is simply nonsense. Human beings are
just as capable as any other species of breeding their way into trouble; and in fact they are more so because of the principle of
momentum, which says that in a species with a generation time as long as ours the effects of overbreeding at any one time are
not felt until 30 years later, by which time the fabric of the planet could have changed dramatically for the worse (Coale I974,
I987). The general point, then, is that we cannot say that disaster for the human species and for the planet as a whole is
inevitable; the tragedy of Ethiopia in the 1980s will not necessarily be rehearsed on a global scale. But as Paul Ehrlich has
pointed out, it is simply feeble-minded to dismiss out of hand the possibility that at some time in the next few hundred years -
in a very short time, indeed - human numbers will exceed the capacity of the world to provide support [243] 484 C. TUDGE
(Ehrlich I987). What happens at that point really is anybody's guess. Mathematicians versed in the intricacies of chaos are
perhaps best qualified to comment. In fact, the likely fate of the human species over the next few hundred years might
profitably be modelled mathematically, as has been done for nuclear winter. Every known factor that might influence our
material wellbeing, and every known interaction, would be fed into a computer, to see what turns up. In practice the models
would be far more complicated than those for nuclear winter, partly because there are more material factors to feed in, but
partly because there are other dimensions to take into account. The nuclear-winter models are purely physical; they attempt
to assess what will happen after the bombs have fallen, and after human beings have done their worst. If we modelled the fate
of the human species and our fellow creatures, we would also have to take into account future intentions: what kind of a world
do we, and our immediate descendants, want to create; and also human fallibility: to what extent are we capable of achieving
the end results we find desirable? The physical factors to be fed into the human future model are complicated, as I have
already said, but they are to some extent quantifiable. But it is a sad fact, a reflection on the discipline of sociology, that to my
knowledge we have no information at all on the second set of factors we would need to feed in: information on human
intention. We don't know what kind of a world human beings want. We may guess in a general way that people nowadays are
saddened by the poaching of rhinoceroses, and wish it didn't happen; but it is doubtful if many people know that there are two
distinct races of white rhino, for instance, or indeed that there's any difference between the African species and the Asian. And
when the Javan tiger was officially declared extinct only a few years ago, the matter hardly featured in national newspapers,
though it did feature - significantly - on children's television. It is doubtful if anyone cares, in any positive way, about the
reduction in species in tropical forest; secondary forest, or even a plantation, tastefully laid out, looks much the same as a
natural wood to the untrained eye. Indeed I suspect that when politicians - Margaret Thatcher, Neil Kinnock, George Bush -
use the word 'environment', as now is mandatory in all campaigns, that all they have in mind is generalized green-ness, a
golf-course and a bit of Repton-style landscaping, or even a Disney-style theme park with, to quote the blurb of Disney-
World, 'clownish baboons and madcap macaws'. It's one thing to get politicians ostensibly on the side of environment, but it's
another thing again to determine what actually goes on inside their heads. But what does go on inside their heads, and those of
the electorate, matters; and we just don't know what kind of a world people think is 'desirable. However, the point of nuclear
winter models is not that they unequivocally predict the future, as a soothsayer would do, but that they show a range of
possibilities. More specifically, they differentiate the possible from the impossible, and the likely from the less likely. In fact,
present nuclear-winter models show that nuclear war is likely to have some effect on climate, and that this could be disastrous
if, for example, it led to midsummer frosts in the north, and delayed monsoons in the south. Extreme scenarios - a new mini-Ice
Age, as in the seventeenth century, or the total elimination of the human species - are shown to be on the cards, but very
much at the extreme tips of the probability curve. And if we made a model of future human possibilities, feeding in intention (if
we knew it) and putting an arbitrary figure on fallibility, we too would finish up with a curve, or rather a three-dimensional
curve, of possibilities. And I suspect - this being pure guess work, but I hope reasonably sensible guess work - that among the
many scenarios on that curve would be the following six: [244 ] THE RISE AND FALL OF HOMO SAPIENS SAPIENS 485 1.
Superabundance. High human population; many other species; lush vegetation. 2. Most people's ideal (the 'populist' scenario).
High human population; small, select variety of other species; abundant vegetation. 3. Fall-back position: the 'Crete' scenario.
Low but stable human population; small but select variety of other species; scenery devastated but acceptable, as in modern
Crete. 4. Failure. Low human population, but unstable; small variety of other species, with many 'desirable' types already
gone, and extinctions continuing; scenery devastated and continuing to degrade. Human extinction conceivable, though
extremely unlikely. 5. Green and pleasant. Low, stable human population arrived at by voluntary means; high variety of other
species, lush vegetation. 6. Green and unpleasant. The same as (5), but arrived at by coercion. I should like to comment briefly
on these points. I think we can say that (1) is extremely difficult and perhaps impossible to achieve. The growth of the human
population is eliminating other species, and it is hard to see how that trend could immediately stop. Scenario (2) is the kind
alluded to above; and probably what politicians have in mind, insofar as they have anything in mind, when they start pushing
environmentalism. The select band of species envisaged in (2) would be the 10000 that competent biologists might identify.
Scenario (3) represents the likely fall-back position if (2) fails. The proposal is that the world as a whole might come to resemble
present-day Crete. Crete is stunningly beautiful. But it is, ecologically speaking, a mess. The Minoans finished off the
devastation that the farmers of the neolithic began. In a hundred years time the hillsides of Malaysia might look like those of
Crete, and we may draw comfort - cold comfort - from the fact that they will be beautiful; bare rock, after the soil is gone,
shining in the sun; not so much like Crete, perhaps, as Utah. Clearly, if we treat all the world as the Minoans treated Crete, then
we will perforce have a much smaller population than now (and Crete's population is only half what it was in its heyday) but
life for those that are left could be highly agreeable, even though their lifestyle was arrived at by insouciance. On the other
hand if things go very badly wrong - in the way that Paul Ehrlich suggests is easily to be envisaged - then we would finish up
with scenario (4). Human extinction seems unlikely even in this, the worst conceivable scenario, because even though
extinction is very difficult to predict (Jablonsky, this symposium) we can make commonsense observations. And a species like
ours that is numerous, ubiquitous, heterogeneous and individually adaptable, and yet shares a common gene pool so that
different surviv'ing bands can swap genes, must be a very strong candidate for survival. But if we reach the stage of (4), then we
will never be the same again. As Paul Ehrlich has pointed out, recovery in a devastated world, with easily obtainable raw
materials already gone, will not be possible; or at least it's very difficult to see how. The Green scenario is (5). It has been
described both by Paul Ehrlich and by Michael Soule (Ehrlich I987; Soule I987). Paul Ehrlich envisages a final human
population of around one to two billion, while Michael Soule puts the figure much lower, at about 100 million, the likely world
population at around the time of Christ; a time, as he points out, of flowering genius. Both Ehrlich and Soule are humanitarians,
and envisage such low populations being achieved by voluntary means. The means need not be draconian; if married couples
averaged two children, as people in rich countries generally seem happy to do, then the population would inexorably drop,
given-that some people will elect not to have children at all, and some will die before they have children. The only problem is
that a non-draconian policy would take hundreds of years to bring about a significant decline in population, and would not
prevent the [ 245] 486 C. TUDGE rise that is imminent. Conservation thus would become a matter of tiding as many creatures
as possible over the centuries of human populousness: a period that Michael Soule has called the 'demographic winter'.
Ehrlich and Soule both argue that the diminution of human numbers is compensated by the increased quality of life of the
people that are on Earth, and by the probable increased longevity of the human species as a whole; for (5) is almost
undoubtedly the 'safest' of the scenarios here envisaged. I agree with Soule and Ehrlich that (5) is the most desirable of the
envisagable scenarios; and so, I suspect, do most people reading this paper. But although it's not known what people at large
think, I'm sure that many people would not agree that (5) is good. Some feel that to contemplate reduction in human numbers
is ipso facto inhumane, and others feel it's a kind of blasphemy. On a more secular level, people seeking public office in South
Florida at this instant, in Everglades country, are arguing the case for growth and more growth; to quote from a political
advertisement on Florida television in 1988, 'growth leads to greater consumer choice': Taco Bell as well as Kentucky Fried.
Many people would argue, in short, that (2) is the most desirable scenario, one that has lots of people, albeit living
dangerously; and that (3), which is probably more likely than (4), is not too bad as a fall-back. Crete is beautiful, after all; and
so, for that matter, is Utah. The burden of this paper, though, is that if we want (5) to come about - and this is the only realistic
scenario that allows for a reasonable proportion of our fellow species to survive - then we have to persuade vast numbers of
other people that this is worth aiming for. We cannot, however, simply rely on the materialist arguments that say that we
should preserve our fellow creatures because they are of direct benefit to us, for three reasons. The first, as I suggested
earlier, is that these arguments are, for the most part, simply untrue. The
human species could survive just as
well if 99.9 % of our fellow creatures went extinct, provided only that we retained the appropriate 0.1 %
that we need.

We can lose 95%.

Dallas Morning News 97 [10/27, L/N]

Even though populations are disappearing quickly, Hughes said that the second "Science" paper is "a bright spot in all this,"
describing how the tree of life could survive serious pruning. Even if 95 percent of all species are
lost, 80 percent of the underlying evolutionary history remains intact, write Nee and Sir Robert May, also a
biologist at Oxford. The scientists came up with equations to describe how much evolutionary history
would remain after some species went extinct. And they found that it didn't really matter whether
they killed off species at random or in a particular pattern. Choosing particular species to save
didn't preserve much more evolutionary history than saving species at random, the research shows.
The work has implications for conservation biologists, who struggle with choosing which species are the most important to protect.
"It turns out that it really doesn't make a whole lot of difference," Dr. Nee said.
---Environment- Their Ev is Hype

Environment impacts are exaggerated

Gordon ‘95 - a professor of mineral economics at Pennsylvania State University

[Gordon, Richard, “Ecorealism Exposed,” Regulation, 1995, http://www.cato.org/pubs/regulation/regv18n3/reg18n3-


readings.html

Easterbrook's argument is that although


environmental problems deserve attention, the environmental
movement has exaggerated the threats and ignored evidence of improvement . His discontent causes
him to adopt and incessantly employ the pejoratively intended (and irritating) shorthand "enviros" to describe the leading
environmental organizations and their admirers. He proposes-and overuses-an equally infelicitous alternative phrase, "ecorealism,"
that seems to mean that most environmental initiatives can be justifited by more moderate arguments .
Given the mass, range, and defects of the book, any review of reasonable length must be selective. Easterbrook's critique begins
with an overview of environmentalism from a global perspective. He then turns to a much longer (almost 500- page) survey of many
specific environmental issues. The overview section is a shorter, more devastating criticism, but it is also more speculative than the
survey of specific issues. In essence, the overview argument is that human impacts on the environment are minor,
easily correctable influences on a world affected by far more powerful forces . That is a more
penetrating criticism than typically appears in works expressing skepticism about environmentalism. Easterbrook notes that
mankind's effects on nature long predate industrialization or the white colonization of America,
but still have had only minor impacts. We are then reminded of the vast, often highly destructive
changes that occur naturally and the recuperative power of natural systems .

Robust peer reviewed evidence indicates ecosystems are resilient

McDermott, 9 [ Tree Hugger.Com,” Good news: most ecosystems can recover in one lifetime from human induced or natural
disturbance”, http://www.treehugger.com/files/2009/05/most-ecosystems-can-recover-from-disturbance-in-one-lifetime.php]

There's a reason the phrase "let nature take its course" exists: New
research done at the Yale University School of Forestry
& Environmental Science reinforces the idea that ecosystems are quiet resilient and can rebound from
pollution and environmental degradation. Published in the journal PLoS ONE, the study shows that most
damaged ecosystems worldwide can recover within a single lifetime , if the source of pollution is removed
and restoration work done: Forests Take Longest of Ecosystems Studied The analysis found that on average forest
ecosystems can recover in 42 years, while in takes only about 10 years for the ocean bottom to recover. If an area has seen multiple,
interactive disturbances, it can take on average 56 years for recovery. In general, most ecosystems take longer to recover from
human-induced disturbances than from natural events, such as hurricanes. To reach these recovery averages, the researchers
looked at data from peer-reviewed studies over the past 100 years on the rate of ecosystem
recovery once the source of pollution was removed. Interestingly, the researchers found that it appears that the rate at which an
ecosystem recovers may be independent of its degraded condition: Aquatic systems may recover more quickly than, say, a forest,
because the species and organisms that live in that ecosystem turn over more rapidly than in the forest.
No Their impact is exaggerated

Kaleita 7 (Amy, PhD, Assistant Professor of Agricultural and Biosystems Engineering @ IA State, “Hysteria’s History:
Environmental Alarmism in Context,” http://www.pacificresearch.org/docLib/20070920_Hysteria_History.pdf)

 Apocalyptic stories about the irreparable, catastrophic damage that humans are doing to the
natural environment have been around for a long time.  These hysterics often have some basis
in reality, but are blown up to illogical and ridiculous proportions. Part of the reason they’re so appealing
is that they have the ring of plausibility along with the intrigue of a horror flick. In many cases, the alarmists identify a
legitimate issue, take the possible consequences to an extreme, and advocate action on the basis of these
extreme projections. In 1972, the editor of the journal Nature  pointed out the problem with the typical alarmist
approach: “[Alarmists’] most common error is to suppose that the worst will always
happen.”82 But of course, if the worst always happened, the human race would have died out
long ago. When alarmism has a basis in reality, the challenge becomes to take appropriate action based on that reality, not on
the hysteria. The aftermath of Silent Spring  offers examples of both sorts of policy reactions: a reasoned response to a legitimate
problem and a knee-jerk response to the hysteria. On the positive side, Silent Springbrought an end to the general belief that all
synthetic chemicals in use for purposes ranging from insect control to household cleaning were uniformly wonderful, and it ushered
in an age of increased caution on their appropriate use. In the second chapter of her famous book, Carson wrote, “It is not my
contention that chemical insecticides must never be used. I do contend that… we have allowed these chemicals to be used with little
or no advance investigation of their effect on soil, water, wildlife, and man himself.” Indeed, Carson seemed to advocate reasoned
response to rigorous scientific investigation, and in fact this did become the modern approach to environmental chemical licensure
and monitoring. An hour-long CBS documentary on pesticides was aired during the height of the furor over  Silent Spring. In the
documentary, Dr. Page Nicholson, a water-pollution expert with the Public Health Service, wasn’t able to answer how long pesticides
persist in water once they enter it, or the extent to which pesticides contaminate groundwater supplies. Today, this sort of
information is gathered through routine testing of chemicals for use in the environment. 20 V: Lessons from the
Apocalypse Ironically, rigorous investigation was not used in the decision to ban DDT, primarily due to the hysteria  Silent
Spring  generated. In this example, the hysteria took on a life of its own, even trumping the author’s original intent. There was, as we
have seen, a more sinister and tragic response to the hysteria generated by Silent Spring. Certain developing countries, under
significant pressure from the United States, abandoned the use of DDT. This decision resulted in millions of deaths from malaria and
other insect-borne diseases. In the absence of pressure to abandon the use of DDT, these lives would have been spared. It would
certainly have been possible to design policies requiring caution and safe practices in the use of supplemental chemicals in the
environment, without pronouncing a death sentence on millions of people.  A major challenge in developing appropriate responses
to legitimate problems is that alarmism catches people’s attention and draws them in. Alarmism is given more weight than it
deserves, as policy makers attempt to appease their constituency and the media. It polarizes the debaters into groups of “believers”
and “skeptics,” so that reasoned, fact-based compromise is difficult to achieve. Neither of these aspects of alarmism is healthy for
the development of appropriate policy. Further, alarmist responses to valid problems risk foreclosing potentially useful responses
based on ingenuity and progress. There are many examples from the energy sector where, in the presence of economic, efficiency,
or societal demands, the marketplace has responded by developing better alternatives. That is not to say that we should blissfully
squander our energy resources; on the contrary, we should be careful to utilize them wisely. But  energy-resource hysteria should
not lead us to circumvent scientific advancement by cherry-picking and favoring one particular replacement technology at the
expense of other promising technologies. Environmental alarmism should be taken for what it is—a natural tendency of some
portion of the public to latch onto the worst, and most unlikely, potential outcome. Alarmism should not be used as
the basis for policy. Where a real problem exists, solutions should be based on reality, not hysteria.
---Environment- Oceans

Oceans resilient

Kennedy 2 (Victor, Coastal and Marine Ecosystems and Global Climate Change,
http://www.pewclimate.org/projects/marine.cfm)

There is evidence that marine organisms and ecosystems are resilient to environmental change . Steele
(1991) hypothesized that the biological components of marine systems are tightly coupled to physical
factors, allowing them to respond quickly to rapid environmental change and thus rendering
them ecologically adaptable. Some species also have wide genetic variability throughout their range,
which may allow for adaptation to climate change.
---Environment- Oceans- AT: IPSO/Rogers
IPSO inflates threats

Barange, 11 -- Plymouth Marine Laboratory director


[Manuel, ICES Science Committee chair, "Forum on the Future of the Ocean," 7-5-11, theseamonster.net/2011/07/forum-on-the-
future-of-the-oceans/, accessed 3-18-13, mss]

I feel that I need to make a brief statement on the fact that I attended the workshop, but decided to pull out of
the report at the eleventh hour. I withdrew after several intensive edits of the report (well, intensive at least on my part, as I saw
very few comments from other participants), and particularly when the draft press release came to my attention (as I felt it took a
very distinctive turn from the tone of the report). As a result of my withdrawal, the International Geosphere-Biosphere Programme
also withdrew their logo from the report and Press Release. While many of the
threats and challenges mentioned in the report
are real, they are often presented in overly simplistic ways. The report disregards scientific
uncertainty (especially with respect to climate change impacts) and does not acknowledge the diversity of
views on specific facts. When in doubt, the worse reading is the one highlighted. Assessments
that could be considered positive, and human actions in the right direction, are disregarded (and
there are large communities working very hard on the latter). The report also tiptoes around the elephant in the room: what are we
suggesting that we do to feed 9 billion people? What are the alternatives and the stakes? What does sustainability mean to us –
what are the compromises, alternatives and the non-negotiables? To me this is what would have given the report the edge that is
needed to shift a debate that has got stuck. With regards to Climate Change, I believe that the report missed the most important
point: climate change is going to create winners and losers. And this will create significant conflict (e.g. Changes in the access to
water and food). It is important to recognise this and to state that as global citizens we need global measures to ensure that those
negatively impacted are compensated. It would be easy to combat climate change if all impacts were negative. They are not (at least
not at all temporal scales) – Certainly countries in the northernmost north are looking with interest at the opening of markets,
transport routes and new resources associated with a retreat of sea ice! I mentioned that the Press release was particularly one-
sided and had statements that I did not feel I agreed with, including the first paragraph: “panel of experts warns …/…that the world’s
ocean is at high risk of entering a phase of mass extinction of marine species, which would be unprecedented in human history”. I
missed the last day of the workshop, but at no point were we asked to debate this statement and consider the evidence. I personally
do not believe it is scientifically correct. There are other points in the PR and Report that I could not agree with (including 3 out of
the 4 “conclusions” mentioned n the press release), and so while I respect the views of those that think otherwise, I pulled out.

No scientific rigor—IPSO is biased

Boesch, 11 -- University of Maryland Center for Environmental Science marine science professor
[Don, ICES Science Committee chair, "Forum on the Future of the Ocean," 7-5-11, theseamonster.net/2011/07/forum-on-the-future-
of-the-oceans/, accessed 3-18-13, mss]

From these perspectives, I found the brief report suffering problems of legitimacy and rigor, despite how
much I might agree with its observations and recommendations. First, the title clearly implies the report was produced by
international Earth system experts, yet of the 27 participants only 15 have scientific credentials (an advanced degree relevant to the
conclusions drawn), all but one of these scientists are biologists and two-thirds reside in the U.K. (2 scientists are from Canada, 1
from Germany, 1 from Australia and one from Chile, now residing in Switzerland). Lacking
are experts on the
geophysical aspects of the Earth system that are essential for understanding and projecting climate
change and the impacts described. Although the distribution of participants meets the minimum definition of
“international,” one would expect more diversity given the lofty descriptor. By the same token, the International Programme on the
State of the Ocean is apparently not a science program with international participants, as some might expect from its title, but an
effort with the aim of “saving the Earth and all life on it” involving two people in the U.K.—at least there are only individuals
mentioned on its website. The heavy participation by environmental activists—don’t get me wrong, I respect the
work these folks do and am grateful they are doing it—opens the report to the kind of criticism of agenda-driven
bias that the Climate Resistance blogger leveled. Indeed, if we are honest—now don’t shoot the messenger—the blogger is
probably correct that the participants were indeed a preselected group who shared beliefs and
assembled, not to assess the evidence critically, but assemble it in a way to make their case for a
call to action (“the scientific outcomes . . . will be used first and foremost to strengthen the case for greater action”). Now this
is a fine and noble thing to do, but it does reduce the authority founded on inclusive, objective appraisal by scientists. As a
result, although the IPSO workshop report enjoyed a press splash and thus may have affected public opinion on scope and urgency
of ocean stresses, I suspect it will have limited staying power and long-term impact on policymaking.
---Environment- Ocean Acidification

Ocean acidification will be slow and stable, proven by 1000 studies- it improves ocean
resiliency

Codling ‘11 [Jo, received a Bachelor of Science first class and won the FH Faulding and the Swan Brewery prizes at the
University of Western Australia. Her major was microbiology, molecular biology. Nova received a Graduate Certificate in Scientific
Communication from the Australian National University in 1989,[4] and she did honours research in 1990, prize-winning science
graduate, Jo has has done over 200 radio interviews, many on the Australian ABC.  She was formerly an associate lecturer in Science
Communication at the ANU and is based in Perth, Western Australia, , “Ocean Acidification — a little bit less alkalinity could be a
good thing,” Sept. 11, http://joannenova.com.au/2011/09/ocean-acidification-a-little-bit-less-alkalinity-could-be-a-good-thing/]

Studies of how marine life copes with less alkaline conditions include many experiments with water at pH values in a range beyond
anything that is likely on planet Earth — they go beyond the bounds of what’s possible. There are estimates that the pH of the ocean
has shifted about 0.1 pH unit in the last 200 years, yet some studies consider the effects of water that is shifted
by 2 or even 4 entire pH units. Four pH units means 10,000 fold change in the concentration of hydrogen ions). That’s
a shift so large, it’s not going to occur in the next few thousand years, even under the worst of
the worst case scenarios by the most sadistic models. Indeed, it’s virtually impossible for CO2
levels to rise high enough to effect that kind of change , even if we burned every last fossil,
every tree, plant microbe, and vaporized life on earth. (Yet still someone thought it was worth studying what
would happen if, hypothetically, that happened. Hmm.)¶ 1103 studies on acidification say there’s no need to panic¶ CO2
science has an extraordinary data base of 1103 studies of the effects of “acidification” on marine
life. They reason that any change beyond 0.5 pH units is “far far beyond the realms of reality”
even if you are concerned about coral reefs in the year 2300 (see Tans 2009). Even the IPCC’s highest end “scenario
A2″ estimate predicts a peak change in the range of 0 .6 units by 2300.¶ Many of the headlines
forecasting “Death to Reefs” come from studies of ocean water at extreme pH’s that will never
occur globally, and that are beyond even what the IPCC is forecasting. Some headlines come from studies of hydrothermal
vents where CO2 bubbles up from the ocean floor. Not surprisingly they find changes to marine life near the vents, but then, the pH
of these areas ranges right down to 2.8. They are an extreme environment, nothing like what we might expect to convert the worlds
oceans too.¶ Marine life, quite happy about a bit more CO2?¶ Studies
of growth, calcification, metabolism,
fertility and survival show that, actually, if things were a little less alkaline, on average, marine
life would benefit. There will be winners and losers, but on the whole, using those five measures of health,
the reefs are more likely to have more life on and around them, than they are to shrink .¶ Figure
12. Percent change in the five measured life characteristics (calcification, metabolism, growth, fertility and survival) vs. decline of
seawater pH from its present (control treatment) value to ending values extending up to the beginning pH value of "the warped
world of the IPCC" for all individual data points falling within this pH decline range.¶ How can this be?¶ First, marine
life
evolved under conditions where most of the time the world was warmer and had more CO2 in
the atmosphere than it does today. Second, like life above the water, life-below-water is based on carbon,
and putting more carbon into the water is not necessarily a bad thing . That said, the dots in the graph
above represent study results, and the ones below zero tell us there will be some losers , even though there will be
more winners (above zer0). Thirdly, watch out for some of the more devastating headlines which also come from studies where
researchers changed the pH by tossing hydrochloric acid into the tank. Chlorine, as they say, is not the same as the gas nature
breathes — CO2. (The strange thing about the studies with hydrochloric acid, is that it doesn’t seem to be bad as we might have
expected– nonetheless, it seems like a dubious practice to use in studying the health of corals.)¶ The Ocean Acidification Database is
housed at CO2 science.¶ The graph above is just one of many on their results and conclusions page.¶ The bottom line:¶ Yes, we
should watch and monitor the oceans careful. No, there is no chance the Great Barrier Reef will be gone in the
next 100 years: 1103 studies show that if the worlds oceans were slightly less basic then marine
life as a whole will be slightly more likely to grow, survive, and be fertile.

Climate change proves Oceans and marine bioD are resilient – alarmist predictions empirically
denied

Taylor ‘10 (James M. Taylor is a senior fellow of The Heartland Institute and managing editor of Environment & Climate News.,
“Ocean Acidification Scare Pushed at Copenhagen,” Feb 10 http://www.heartland.org/publications/environment
%20climate/article/26815/Ocean_Acidification_Scare_Pushed_at_Copenhagen.html)

With global temperatures continuing their decade-long decline and United Nations-sponsored global warming talks falling apart in
Copenhagen, alarmists at the U.N. talks spent
considerable time claiming carbon dioxide emissions will
cause catastrophic ocean acidification, regardless of whether temperatures rise. The latest scientific data,
however, show no such catastrophe is likely to occur. Food Supply Risk Claimed The United Kingdom’s
environment secretary, Hilary Benn, initiated the Copenhagen ocean scare with a high-profile speech and numerous media
interviews claiming ocean acidification threatens the world’s food supply. “The fact is our seas absorb CO2. They absorb
about a quarter of the total that we produce, but it is making our seas more acidic,” said Benn in his speech. “If this continues as a
problem, then it can affect the one billion people who depend on fish as their principle source of protein, and we have to feed
another 2½ to 3 billion people over the next 40 to 50 years.” Benn’s
claim of oceans becoming “more acidic” is
misleading, however. Water with a pH of 7.0 is considered neutral. pH values lower than 7.0 are
considered acidic, while those higher than 7.0 are considered alkaline. The world’s oceans have a pH of 8.1,
making them alkaline, not acidic. Increasing carbon dioxide concentrations would make the oceans
less alkaline but not acidic. Since human industrial activity first began emitting carbon dioxide into the
atmosphere a little more than 200 years ago, the pH of the oceans has fallen merely 0.1 , from 8.2 to 8.1. Following
Benn’s December 14 speech and public relations efforts, most of the world’s major media outlets produced stories claiming ocean
acidification is threatening the world’s marine life. An Associated Press headline, for example, went so far as to call ocean
acidification the “evil twin” of climate change. Studies Show CO2 Benefits Numerous recent scientific studies show higher
carbon dioxide levels in the world’s oceans have the same beneficial effect on marine life as
higher levels of atmospheric carbon dioxide have on terrestrial plant life . In a 2005 study
published in the Journal of Geophysical Research, scientists examined trends in chlorophyll
concentrations, critical building blocks in the oceanic food chain. The French and American scientists reported “an overall
increase of the world ocean average chlorophyll concentration by about 22 percent” during the prior two decades of increasing
carbon dioxide concentrations. In a 2006 study published in Global Change Biology, scientists observed higher CO2 levels are
correlated with better growth conditions for oceanic life. Thehighest CO2 concentrations produced “higher
growth rates and biomass yields” than the lower CO2 conditions . Higher CO2 levels may well
fuel “subsequent primary production, phytoplankton blooms, and sustaining oceanic food-
webs,” the study concluded. Ocean Life ‘Surprisingly Resilient’ In a 2008 study published in Biogeosciences,
scientists subjected marine organisms to varying concentrations of CO2, including abrupt
changes of CO2 concentration. The ecosystems were “surprisingly resilient” to changes in
atmospheric CO2, and “the ecosystem composition, bacterial and phytoplankton abundances and productivity, grazing rates and
total grazer abundance and reproduction were not significantly affected by CO2-induced effects.” In a 2009 study published in
Proceedings of the National Academy of Sciences, scientists reported, “Sea star growth and feeding rates increased with water
temperature from 5ºC to 21ºC. A doubling of current [CO2] also increased growth rates both with and without a concurrent
temperature increase from 12ºC to 15ºC.” Another False CO2 Scare “ Far
too many predictions of CO2-induced
catastrophes are treated by alarmists as sure to occur, when real-world observations show
these doomsday scenarios to be highly unlikely or even virtual impossibilities,” said Craig Idso, Ph.D.,
author of the 2009 book CO2, Global Warming and Coral Reefs. “The phenomenon of CO2-induced ocean acidification appears to be
no different.

No impact to ocean acidification


- doesn’t harm corals and doesn’t harm biodiversity- their evidence is alarmist non-sense- and
studies prove acidification actually helps marine biodiversity grow. And even if they win a corals
impact- destruction is inevitable from non-emissions causes

Ridley 12 (Matt Ridley has been a scientist, journalist and businessman. With BA and DPhil
degrees from Oxford University, he worked for the Economist for nine years as science editor,
Washington correspondent and American editor, before becoming a self-employed writer and
businessman. He was founding chairman of the International Centre for Life in Newcastle,
January 7 2012, “Taking Fears of Acid Oceans With a Grain of Salt”,
http://online.wsj.com/article/SB10001424052970203550304577138561444464028.html)

Coral reefs around the world are suffering badly from overfishing and various forms of pollution.
Yet many experts argue that the greatest threat to them is the acidification of the oceans from
the dissolving of man-made carbon dioxide emissions. The effect of acidification, according to
J.E.N. Veron, an Australian coral scientist, will be "nothing less than catastrophic .... What were once thriving coral
gardens that supported the greatest biodiversity of the marine realm will become red-black bacterial slime, and they will stay that
way." This is a common view. The Natural Resources Defense Council has called ocean acidification "the scariest
environmental problem you've never heard of." Sigourney Weaver, who narrated a film about the issue, said that "the scientists are
freaked out." The head of the National Oceanic and Atmospheric Administration calls it global warming's "equally evil twin." But
do the scientific data support such alarm? Last month scientists at San Diego's Scripps Institution of
Oceanography and other authors published a study showing how much the pH level (measuring alkalinity versus
acidity) varies naturally between parts of the ocean and at different times of the day, month and
year. "On both a monthly and annual scale, even the most stable open ocean sites see pH
changes many times larger than the annual rate of acidification ," say the authors of the study, adding that
because good instruments to measure ocean pH have only recently been deployed, "this variation has been under-
appreciated." Over coral reefs, the pH decline between dusk and dawn is almost half as much as
the decrease in average pH expected over the next 100 years. The noise is greater than the
signal. Another recent study, by scientists from the U.K., Hawaii and Massachusetts, concluded that "marine
and freshwater assemblages have always experienced variable pH conditions," and that "in
many freshwater lakes, pH changes that are orders of magnitude greater than those projected
for the 22nd-century oceans can occur over periods of hours." This adds to other hints that the
ocean-acidification problem may have been exaggerated. For a start, t he ocean is alkaline and in
no danger of becoming acid (despite headlines like that from Reuters in 2009: "Climate Change Turning Seas Acid"). If
the average pH of the ocean drops to 7.8 from 8.1 by 2100 as predicted, it will still be well above
seven, the neutral point where alkalinity becomes acidity. The central concern is that lower pH
will make it harder for corals, clams and other "calcifier" creatures to make calcium carbonate
skeletons and shells. Yet this concern also may be overstated. Off Papua New Guinea and the Italian
island of Ischia, where natural carbon-dioxide bubbles from volcanic vents make the sea less alkaline, and off the Yucatan, where
underwater springs make seawater actually acidic, studies have shown that at least some kinds
of calcifiers still thrive—at least as far down as pH 7.8. In a recent experiment in the Mediterranean, reported in
Nature Climate Change, corals and mollusks were transplanted to lower pH sites, where they proved
"able to calcify and grow at even faster than normal rates when exposed to the high [carbon-
dioxide] levels projected for the next 300 years." In any case, freshwater mussels thrive in Scottish rivers, where the pH is as
low as five. Laboratory experiments find that more marine creatures thrive than suffer when
carbon dioxide lowers the pH level to 7.8. This is because the carbon dioxide dissolves mainly as
bicarbonate, which many calcifiers use as raw material for carbonate. Human beings have
indeed placed marine ecosystems under terrible pressure, but the chief culprits are overfishing
and pollution. By comparison, a very slow reduction in the alkalinity of the oceans, well within the
range of natural variation, is a modest threat, and it certainly does not merit apocalyptic
headlines.
---Environment- Endocrine Disruption

No Impact

Milloy ‘3, Steven Cato adjunct scholar, 6-20FOXNews, “Pesticide-Sperm Count Link Is Impotent,”
http://www.foxnews.com/story/0,2933,89913,00.html

“Scientists for the first time have shown a link between levels of widely used agricultural pesticides in men’s bodies and the number
and quality of their sperm,” shrieked USA Today this week. Steady, USA Today (search). Underwear, rather than pesticides, is a
more probable explanation. McNews had the wool pulled over its eyes by University of Missouri-Columbia eco-activist researcher
Shanna Swan (search), who has been crusading since the mid-1990s to link pesticides with supposed reduced sperm counts. The
latest chapter in Swan’s crusade began in April when she reported that a small sample of men from Boone County, Mo., had lower
sperm quality than similarly small samples of men from Los Angeles, Minneapolis and New York City. Based on further “analysis,”
Swan now reports in a study published in the June 18 Environmental Health Perspectives (search), the 50 Missouri men had higher
levels of metabolites of widely used pesticides (search) (alachlor, diazinon, atrazine and others) in their urine than the 50
Minneapolis men. Swan concluded: “This is the first population-study to demonstrate links between specific biomarkers of
environmental exposures and biomarkers of male reproduction in humans. Given the current widespread use of these pesticides, if
further study confirms these findings, the implications for public health and agricultural practice could be considerable.” Doubtful. In
the first place, I’m not quite sure what the dreaded “implications” of Swan’s data are since all men
in the study were fertile. They were, in fact, the partners of pregnant women recruited at
prenatal clinics. Secondly, there’s no known biological support for Swan’s idea that pesticides
affect sperm quality. Tests do not indicate that alachlor, diazinon and atrazine, for example,
produce toxic effects in the reproductive systems of laboratory animals. A reproductive biologist
from the Environmental Protection Agency (search) told USA Today that rodent studies suggest that even
the highest pesticide levels found in Swan’s subjects would have been too low to affect sperm
quality. And just because the Missouri men had lower sperm counts and higher pesticide
exposure than the Minneapolis men doesn’t automatically mean that pesticides have anything
to do with sperm production. A University of Virginia fertility expert told The Associated Press that he
was skeptical of the findings because of the lack of historical documentation of the effect of
toxins on sperm. Sperm counts are known to vary geographically. There is no certain explanation for
the phenomenon, although some studies indicate that men in colder regions seem to have
higher sperm counts than men in warmer areas . And it’s really not surprising that men from the agricultural Boone
County, Mo., would have more pesticide exposure than an urban area such as Minneapolis. Swan’s data simply aren’t
unexpected and her tenuous conclusions aren’t surprising given her track record of eco-activist,
anti-pesticide “research.” Though anti-pesticide activists (search) have tried for years to link
pesticides with declining sperm counts, one key fact stands in their way -- there’s no evidence
that sperm counts are even declining, much less that pesticides are involved . In 1999,
researchers published in the Journal of Urology (search) a review of all 29 studies from 1938 to
1996 reporting semen analyses of fertile men. They concluded, “there appears to be no
significant change in sperm counts in the U.S. during the last 60 years.” Sperm counts (search) and
quality depend on many factors. One that Swan overlooked in her study was the effect of tight-
fitting underwear. In 1996, Dutch researchers reported in the British medical journal The Lancet (search) the
results of their study of the effect of underwear on sperm quality and quantity. They reported that men who wore tight-
fitting underwear produced 50 percent less sperm than men who wore loose-fitting underwear.
Sperm motility was reduced by two-thirds among the men who wore tight-fitting underwear .
Maybe the Missouri men in Swan’s study wore tighter underwear than their Minneapolis
counterparts. I don’t know. Neither does Swan -- she didn’t check. I do know that Swan has absolutely no
evidence that implicates pesticides and exculpates all other potential factors. Regardless of the
explanation for Swan’s reported observations, sperm quality differences apparently did not
affect anyone’s fertility. Perhaps it’s time for Swan to spend time reconsidering the quality of her crusade.

Tons of Alt Causes

Pan ‘4, Cindy MD, Fellowship of the Royal Australian College of General Practitioners, 7-4-Sunday Herald Sun, p. B14, l/n

The main issues in male infertility relate to the sperm. Sperm may be too few, malformed, or unable to reach the egg. Excess
alcohol and other drugs, exposure to pesticides and chemicals, smoking, exposing testicles to
overheating, such as in saunas or hot tubs, or even too tight undies, can adveresly affect sperm
production. Stress, depression, and being very overweight can also affect sperm, and there's a
gradual decline in fertility in men over the age of 35.
Environment Leadership Answers
Frontline

Can’t solve environmental leadership –alt causes

Victor 8 - law professor at Stanford's Program on Energy and Sustainable Development and adjunct senior fellow at the Council
on Foreign Relations.

(David G., "The next U.S. President won't be green", 5-1-08 http://www.newsweek.com/id/135073/)

The U.S. record on international environmental issues is highly uneven for reasons that have
little to do with George W. Bush's leadership. His administration has been tarred across the planet for reckless
leadership on international environmental issues. (Its actual record, while dreadful, is not a uniform failure. It has done useful things
in a few areas, such as a thoughtful initiative to help conserve forests in the Congo Basin.) But the signature of Bush's reckless
foreign policy in this area, his decision to withdraw from the Kyoto treaty barely three months after taking office, actually has its
roots in the Clinton administration. Clinton was highly committed to environmental issues and his vice president, Al Gore, was an
even more passionate leader. Their zealous diplomats negotiated a treaty that was larded with commitments that the United States
never could have honored. The promise to cut U.S. emissions 7 percent below 1990 levels is a good example. Because actual
emissions were rising steadily, it would have been impractical to turn them around in time to meet the 2012 Kyoto deadline. The
U.S. Congress never could have passed the requisite legislation, and no leader in the White House could have changed that voting
arithmetic. The U.S. withdrawal from the Kyoto Protocol was inevitable. What does this mean for America's credibility in the world?
When the American president promises, should anyone listen? Increasingly, other countries are learning that the answer is no—
because Americanleaders have a habit of promising a lot more than they can deliver.
Environmental issues are particularly prone to overpromising, and not just by the United States. Europe, too,
is fresh with unrealistic claims by political leaders. The European Union, for example, has launched negotiations for the post-Kyoto
agreement by claiming that Europeans will cut greenhouse-gas emissions 20 percent to 30 percent by 2020—an outrageous goal
considering that most of Europe (with the exception mainly of Britain and Germany) will fail to meet their existing targets, and
emissions are actually rising. Europe as a whole would blow through its Kyoto targets if not for its generous use of a scheme that lets
them take credit for overseas investment in low-carbon technologies—despite mounting evidence that many of those overseas
credits don't actually deliver real reductions in emissions. Smart politicians know that the benefits lie mainly in the promising today
and not in the delivery long in the future. Ironically, the more enthusiastic the leader, the less credibility he or
she has. While the Clinton administration was busy negotiating the Kyoto treaty, the U.S. Senate was passing a resolution, 95 to 0,
to signal that it would reject any treaty that didn't contain specific commitments by developing countries to control their effluent of
greenhouse gases. Since the developing countries had already rejected that outcome the Clinton administration had little room to
maneuver. The great reversal in U.S. "leadership" on global warming over the last year—signaled by President Bush's speech three
weeks ago embracing the need for limits on greenhouse gases—came from the people rather than top leaders. Public concern about
global warming is rising (though it will be checked by the even more acute worries on the economy and war). The Bush speech was
more a recognition that serious efforts to develop climate legislation are already well underway without his stamp. Many states are
already planning to regulate greenhouse gases. The Senate has a serious bill on this subject scheduled for floor debate starting June
2. Its sponsors are Joe Lieberman (the former running mate of Al Gore but now alienated from the Democratic Party for his overly
independent views) and John Warner (a Republican who has no former track record on global warming). These are ideal leaders for
this issue because often it takes the fresh faces focused on building bipartisan majorities to get things done in America. Perhaps the
most interesting signal that American presidents are losing the ability to lead is an effort to
rewrite the rules that would govern environmental treaties under American law. Committed
environmentalists have rightly noted that America's Constitution requires a two-thirds vote for treaties in the Senate. That standard
is nearly impossible to meet because one third of the Senate is usually opposed to anything interesting. Serious efforts are now
underway to reinterpret environmental "treaties" as agreements between Congress and the president, which would require only a
majority vote. Most trade agreements, for example, travel under this more lax standard and also have special voting rules that
require Congress to approve the agreement as a whole package rather than pick it apart piece by piece. Rebranding and changing
voting rules makes it easier to approve agreements, boosting the credibility of the president to negotiate agreements that serve the
country's interest.
And, the plan can’t solve leadership – not taken seriously

Business Times Singapore ‘7 [Can Bush follow through on his green policy?, lexis]

¶ WHO looks to President George Bush for leadership on global warming? When he announced
his intention last week to set the United States on to the path of reducing greenhouse gases, the
world reacted with scepticism.The sceptics see it as presidential grandstanding which in effect is
intended to stall the Group of Eight nations' talks in Germany this week. That conclave aims to adopt a unified stand on the post-Kyoto
round discussions ahead of a global in Bali later this year. ¶ Similar scepticism was heard about the president's announcement last

month about setting up an interdepartmental study on vehicle emissions in the US. Indeed, the

Bush administration had to be hauled to the US Supreme Court and its federal E nvironmental Protection Agency
had to be directed to use its power to rein in emissions. ¶ So, President Bush finds himself in a situation of being damned if you do and damned
if you don't. After having spurned the Kyoto Protocol, the main global treaty for cutting emissions, and questioning the very science involved in global warming, Mr Bush has a hard time convincing the world that
he is for real changes on emissions.¶ Earlier, there were sound bites like 'America is addicted to oil' and there was a tantalising proposal for cellulosic ethanol for the future and with funding increased for research
to support technology-backed solutions to greenhouse gas emissions. But at the same time, immediate action to require more miles from vehicles was shunned. Nor has he abandoned his opposition to the cap
and trade system to control emissions, a central plank of the Kyoto Protocol.

Obama won’t be an environmental leader and Congress would block it anyway

McDermott, 11 - edits the Business and Energy sections of TreeHugger, as well as writing about resource consumption, animal
welfare issues, and the response of religious communities to our current environmental problems, also a goddamned hippie (Mat,
“President Obama Continues Squandering Opportunities to Show Environmental Leadership” 1/26,
http://www.treehugger.com/corporate-responsibility/president-obama-continues-squandering-opportunities-to-show-
environmental-leadership.html)

By now you're likely well aware that despite


talking about clean energy and jobs , President Obama made
no mention of climate change or environmental issues as such during yesterday's State of the Union address.
Which somehow makes it an even more important time to highlight comments that scientist-activist Dr James Hansen once wrote to
Obama in advice. Obama Apparently Hasn't Listened to Hansen New York Times has the entire letter--we don't know if it was ever
actually read--but here are some of the highlights, which I personally find important and resonant: The President should use his
ascendancy to the most powerful position on the planet to help set a new sensible course for the planet and humanity. It would
have required being blunt and honest about the situation and what was needed to break our addiction and avoid the tremendous
inter-generational injustice that the present path will bring to pass. The path to a clean energy future would not be painful for the
public, but it requires standing up to special interests who benefit from business-as-usual. It is both a moral issue and a question of
where the United States will stand in the future. Our economic standing is going to become second class this century if we do not
move smartly toward a clean energy future. Further along: The other thing not mentioned above is that the most fundamental
problem, which I keep repeating, is this: as long as fossil fuels are the cheapest energy, somebody will keep burning them --
implication, we must put a rising price on carbon. (Not cap-and-trade! A simple, honest approach -- collect a fee from fossil fuel
companies at first sale, distribute that money, 100 percent, to the public.) Nevertheless, the easiest thing that he could do, and
perhaps the best that we can hope for, is for him to give a strong boost to nuclear power. Unfortunately, he seems to fall
prey to Democratic politics on this, rather than being a responsible leader. Nuclear Energy May Help
Greenhouse Gas Emissions, But Not Environment The reference to nuclear power is something that Hansen brought up earlier in the
letter--he called it a "huge mistake" that the Carter and Clinton administrations didn't more support the development of advanced
reactor designs. Personally I'm not sure nuclear power is any better solution to our greater environmental problems than continued
reliance on other non-renewable fuels--even if it does address greenhouse gas emissions. And to be fair, President Obama continues
to show strong support for clean energy, even if implementation isn't as robust as it could be or is needed to be. President's Clean
Energy Approach Neglects Why It's Important But the
part that is really lacking right now, linking together two
things that Hansen advised as symbols of this lack, is President Obama showing or even hinting
at moral leadership in establishing Hansen's "sensible course for the planet and humanity" and
breaking away from business-as-usual thinking. Pushing for 80% of US electricity coming from
clean energy sources by 2035 is somewhat stepping away from that path, breaking away from the pack of polluters
desperately trying to retain their profitable place in the world, but it is simply not enough. The failure of President
Obama to mention climate change, and the utter fingers-in-ears approach of Congress in even
acknowledging that climate change is happening (on one side of the aisle) and in even attempting to
push forward something that will effectively address it (on the other), just makes Hansen's words
about intergenerational injustice all the more poignant.
EU Relations Answers
Frontline

Status quo solves

VOA News, ’12 (“U.S. Defense Posture Towards Europe”, 1/17/12, http://www.voanews.com/policy/editorials/americas/US-
Defense-Posture-Towards-Europe--137526523.html)

President Barack Obama recently unveiled new strategic guidance on American defense in the 21st
century. The guidance is designed to clarify America’s strategic interests in the world as the U.S. approaches the end of ten years
of war and faces tighter budgets. As U.S. Assistant Secretary of State for European and Eurasian Affairs Philip Gordon recently
explained, the new strategy reaffirmsunwavering U.S. commitment to European security and close
cooperation with NATO Allies and European partners. The transatlantic relationship remains an
essential source of stability in an unpredictable world , and Europe is America’s principal partner in promoting
global and economic security. The new defense strategy will ensure the continued ability of the U.S. to
meet its commitment to collective defense under NATO ’s Article 5. It will also provide for enhanced
cooperation and interoperability as the U.S. and Europe work together on global challenges.
Concrete U.S. steps to strengthen such cooperation will include deploying missile defense assets to Europe, enhancing special
operations forces in the region, and sending an aviation detachment to Poland for training purposes. In the last year alone, the
United States and European partners have worked together on challenges from Libya to
Afghanistan, and from Kosovo to the Horn of Africa . Diplomatically, the United States and Europe continue
to cooperate on countering the proliferation of ballistic missiles and weapons of mass
destruction and working to prevent Iran from acquiring nuclear weapons. The U.S. and Europe
have been united in working to support economic and political transitions in the Middle East and
North Africa, and to increase pressure on the Assad regime in Syria to meet its commitments to
refrain from violence against its own people. In May the transatlantic Alliance will take important steps forward in
ensuring 21st Century security as President Obama hosts the 2012 NATO Summit in Chicago. At the Summit Allies will focus on three
priorities, the first of which is ensuring continued progress on transition in Afghanistan and agreeing on a vision for NATO’s post-
2014 role in Afghanistan. The second Summit priority will be to ensure NATO has the full range of capabilities to ensure security in
the 21st Century, even as Allies face difficult budgetary realities. Finally, in Chicago NATO will continue efforts to strengthen relations
with partners around the world. The recent success in Libya showed how Allies and non-NATO partners could effectively work
together in an operation. Working together, the United States and Europe will continue to ensure continued security and prosperity
on both sides of the Atlantic.

No relations impact

Leonard ‘12 (Mark Leonard is co-founder and director of the European Council on Foreign Relations, the first pan-European
think tank., 7/24/2012, "The End of the Affair", www.foreignpolicy.com/articles/2012/07/24/the_end_of_the_affair)

But Obama's stellar personal ratings in Europe hide the fact that the
Western alliance has never loomed smaller
in the imagination of policymakers on either side of the Atlantic. Seen from Washington, there is not
a single problem in the world to be looked at primarily through a transatlantic prism. A lthough
the administration looks first to Europeans as partners in any of its global endeavors -- from dealing with Iran's nuclear program to
stopping genocide in Syria -- it no longer sees the European theater as its core problem or seeks a partnership of equals with
Europeans. It was not until the eurozone looked like it might collapse -- threatening to bring down the global economy and with it
Obama's chances of reelection -- that the president became truly interested in Europe. Conversely, Europeans have never cared less
about what the United States thinks. Germany, traditionally among the most Atlanticist of European countries, has led the pack.
Many German foreign-policy makers think it was simply a tactical error for Berlin to line up with Moscow and Beijing against
Washington on Libya. But there is nothing accidental about the way Berlin has systematically refused even to engage with American
concerns over German policy on the euro. During the Bush years, Europeans who were unable to influence the strategy of the White
House would give a running commentary on American actions in lieu of a substantive policy. They had no influence in Washington,
so they complained. But now, the tables are turned, with Obama passing continual judgment on German policy while Chancellor
Angela Merkel stoically refuses to heed his advice. Europeans who for many years were infantilized by the transatlantic alliance,
either using sycophancy and self-delusion about a "special relationship" to advance their goals or, in the case of Jacques Chirac's
France, pursuing the even more futile goal of balancing American power, have finally come to realize that they can no longer
outsource their security or their prosperity to Uncle Sam. On both sides of the Atlantic, the ties that held the alliance
together are weakening. On the American side, Obama's biography links him to the Pacific and Africa but not to the old
continent. His personal story echoes the demographic changes in the United States that have reduced the influence of Americans of
European origin. Meanwhile, on the European side, the depth of the euro crisis has crowded out almost all foreign policy from the
agenda of Europe's top decision-makers. The end of the Cold War means that Europeans no longer need
American protection, and the U.S. financial crisis has led to a fall in American demand for European products (although U.S.
exports to Europe are at an all-time high). What's more, Obama's lack of warmth has precluded him from
establishing the sorts of human relationships with European leaders that animate alliances. When asked to
name his closest allies, Obama mentions non-European leaders such as Recep Tayyip Erdogan of Turkey and Lee Myung-bak of South
Korea. And his transactional nature has led to a neglect of countries that he feels will not contribute more to the relationship --
within a year of being elected, Obama had managed to alienate the leaders of most of Europe's big states, from Gordon Brown to
Nicolas Sarkozy to Jose Luis Rodriguez Zapatero. Americans hardly remember, but Europe's collective nose was put out of joint by
Obama's refusal to make the trip to Europe for the 2010 EU-U.S. summit. More recently, Obama has reached out to allies to
counteract the impression that the only way to get a friendly reception in Washington is to be a problem nation -- but far too late
to erase the sense that Europe matters little to this American president. Underlying these superficial issues is a more
fundamental divergence in the way Europe and the United States are coping with their respective
declines. As the EU's role shrinks in the world, Europeans have sought to help build a multilateral, rule-based world. That is why it
is they, rather than the Chinese or the Americans, that have pushed for the creation of institutionalized global responses to climate
change, genocide, or various trade disputes. To the extent that today's world has not collapsed into the deadlocked chaos of a "G-
zero," it is often due to European efforts to create a functioning institutional order. To Washington's eternal frustration, however,
Europeans have not put their energies into becoming a full partner on global issues. For all the existential angst of the euro crisis,
Europe is not as weak as people think it is. It still has the world's largest market and represents 17 percent of world trade, compared
with 12 percent for the United States. Even in military terms, the EU is the world's No. 2 military power, with 21 percent of the
world's military spending, versus 5 percent for China, 3 percent for Russia, 2 percent for India, and 1.5 percent for Brazil, according
to Harvard scholar Joseph Nye. But, ironically for a people who have embraced multilateralism more than any other on Earth,
Europeans have not pooled their impressive economic, political, and military resources. And with the eurozone's need to resolve the
euro crisis, the EU may split into two or more tiers -- making concerted action even more difficult. As a result, European
power is too diffuse to be much of a help or a hindrance on many issues. On the other hand, Obama's United States --
although equally committed to liberal values -- thinks that the best way to safeguard American interests and values is to craft a
multipartner world. On the one hand, Obama continues to believe that he can transform rising powers by integrating them into
existing institutions (despite much evidence to the contrary). On the other, he thinks that Europe's overrepresentation in existing
institutions like the World Bank and the International Monetary Fund is a threat to the consolidation of that order. This is leading a
declining America to increasingly turn against Europe on issues ranging from climate change to currencies. The most striking
example came at the 2009 G-20 in Pittsburgh, when Obama worked together with the emerging powers to pressure Europeans to
give up their voting power at the IMF. As Walter Russell Mead, the U.S. international relations scholar, has written, "[I]ncreasingly it
will be in the American interest to help Asian powers rebalance the world power structure in ways that redistribute power from the
former great powers of Europe to the rising great powers of Asia today." But the long-term consequence of the cooling of this
unique alliance could be the hollowing out of the world order that the Atlantic powers have made. The big unwritten story of the last
few decades is the way that a European-inspired liberal economic and political order has been crafted in the shell of the American
security order. It is an order that limits the powers of states and markets and puts the protection of individuals at its core. If the
United States was the sheriff of this order, the EU was its constitutional court. And now it is being challenged by the emerging
powers. Countries like Brazil, China, and India are all relatively new states forged by movements of national liberation whose
experience of globalization has been bound up with their new sense of nationhood. While globalization is destroying sovereignty for
the West, these former colonies are enjoying it on a scale never experienced before. As a result, they are not about to invite their
former colonial masters to interfere in their internal affairs. Just look at the dynamics of the United Nations Security Council on
issues from Sudan to Syria. Even in the General Assembly, the balance of power is shifting: 10 years ago, China won 43 percent of the
votes on human rights in the United Nations, far behind Europe's 78 percent. But in 2010-11, the EU won less than 50 percent to
China's nearly 60 percent, according to research by the European Council on Foreign Relations. Rather than being transformed by
global institutions, China's sophisticated multilateral diplomacy is changing the global order itself. As relative power flows Eastward ,
it is perhaps inevitable that the Western alliance that kept liberty's flame alight during the Cold War and then sought
to construct a liberal order in its aftermath is fading fast. It was perhaps inevitable that both Europeans and Americans should
fail to live up to each other's expectations of their respective roles in a post-Cold War world. After all, America is still too
powerful to happily commit to a multilateral world order (as evidenced by Congress's reluctance to ratify treaties). And
Europe is too physically safe to be willing to match U.S. defense spending or pool its resources. What is surprising is that
the passing of this alliance has not been mourned by many on either side. The legacy of Barack Obama is that the transatlantic
relationship is at its most harmonious and yet least relevant in 50 years. Ironically, it may take the election of someone who is less
naturally popular on the European stage for both sides to wake up and realize just what is at stake.

No spill-over

Busby 6 (Josh, Assistant Professor of Public Affairs – University of Texas, Austin, “Memo on Reputation, April,
http://www.utexas.edu/lbj/faculty/busby/papers.php)

Are there any empirical examples of separate reputations that fuse as a result of violations across issue domains? In thinking
about the legitimation problems of the U.S. government under the present administration, has
the U.S. failure to
make and its willingness to break its commitments in multiple arenas had consequences
for other arenas? One possibility is that when the George W. Bush administration signaled that the administration was
opposed to various multilateral policy initiatives across a range of unrelated substantive domains—climate change (March
2001), small arms (July 2001), anti-ballistic missiles (December 2001), steel tariffs (March 2002), the International Criminal Court
(May 2002), among other issues --and would no longer abide by previous commitments, then the segmented reputations may
well have fused into a more general impression of the United States that had cross-issue consequences. However, evidence
of foot-dragging and non-cooperation by U.S. allies is mixed, and may be difficult to link
back to non-cooperation on secondary issues like climate change. Given the support offered by
America’s allies in Afghanistan and on the war on terror, including covert support for U.S. policy in Iraq,
one could argue that these actions have not had major consequences. Denial of use of Turkish
territory for the war in Iraq, coupled with opposition by several NATO allies to troop deployments and reconstruction assistance
in Iraq, indicate the U.S. may indeed be incurring some higher costs/worse bargains than would have the case had it been a
better partner on other issues. It is too soon to make a definitive judgment on this question. Is there something about the
current environment that may empower other actors to punish the U.S. more for failure to honor commitments, through foot-
dragging on issues the U.S. cares about and for which going it alone is simply not a viable option? The bargaining leverage of
other states in an era of interdependence would seem to be improved in some ways. That said, powerful
states generally may find it easier to break promises without significant consequences
since others may find it too costly to punish the violator or to engage in self-abnegating
behavior that denies themselves future benefits because of earlier rounds of defection
(Downs and Jones 2002).

No impact to US/EU relations

Daalder 3 (Ivo H., Senior Fellow in Foreign Policy Studies – Brookings Institution, Survival, 45(2), Summer, p. 147)
The main consequence of these changes
in US and European policy priorities is to make the
transatlantic relationship less pivotal to the foreign policy of both actors. For America, Europe is a
useful source of support for American actions – a place to seek complementary capabilities and to build ad hoc coalitions of the
willing and somewhat able. But Washingtonviews Europe as less central to its main interests and
preoccupations than it was during the Cold War. For European countries, America’s protective role has
become essentially superfluous with the disappearance of the Soviet threat, while its pacifying presence is no longer
warranted, given the advance of European integration. The task of integrating all of Europe into the zone of peace now falls
squarely on Europe’s shoulders, with the United States playing at most a supporting role. Even
the stabilisation of
Europe’s periphery – from the Balkans in the south to Turkey, the Caucuses, and Ukraine in the East –
is one where Europeans will increasingly have to take the lead.

Relations resilient

Moravcsik 3 (Andrew, Professor of Government and Director of the European Union Program – Harvard University,
July/August, Foreign Affairs, Lexis)

Transatlantic optimists are also right when they argue that the recent shifts
need not lead inexorably to the
collapse of NATO, the UN, or the EU. Historically, they note, transatlantic crises have been cyclical
events, arising most often when conservative Republican presidents pursued assertive unilateral military policies. During the
Vietnam era and the Reagan administration, as today, European polls recorded 80-95 percent opposition to U.S. intervention,
millions of protesters flooded the streets, NATO was deeply split, and European politicians compared the United States to Nazi
Germany. Washington went into "opposition" at the UN, where, since 1970, it has vetoed 34 Security Council resolutions on the
Middle East alone, each time casting the lone dissent.In the recent crisis, a particularly radical American policy combined with a
unique confluence of European domestic pressures -- German Chancellor Gerhard Schrsder's political vulnerability and French
President Jacques Chirac's Gaullist skepticism of American power -- to trigger the crisis.Most Europeans -- like most Americans --
rejected the neoconservative claim that a preemptive war against Iraq without multilateral support was necessary or advisable.
Sober policy analysis underlay the concerns of the doubters, who felt that the war in Iraq, unlike the one in Afghanistan, was not
really connected to the "war on terrorism." Skeptics were also wary of the difficulties and costs likely to attend postwar
reconstruction. No surprise, then, that most foreign governments sought to exhaust alternatives to war before moving forward
and refused to set the dangerous precedent of authorizing an attack simply because the United States requested it. In
spite
of these doubts about the Bush administration's policies, however, underlying U.S. and European interests
remain strikingly convergent. It is a cliche but nonetheless accurate to assert that the Western relationship
rests on shared values: democracy, human rights, open markets, and a measure of social
justice. No countries are more likely to agree on basic policy, and to have the power to do
something about it. Even regarding a sensitive area such as the Middle East, both sides recognize Israel's right to exist,
advocate a Palestinian state, oppose tyrants such as Saddam Hussein, seek oil security, worry about radical Islamism, and fear
terrorism and the proliferation of WMD.
---EU Relations- Resiliency

Cooperation is resilient- shared values, economic ties, and issue specific cooperation

McCormick ‘6 (The War on Terror and Contemporary U.S.-European Relations James M. McCormick 1 1 Iowa State University
ABSTRACT AU: James M. McCormick TI: The War on Terror and Contemporary U.S.-European Relations SO: Politics &
Policy VL: 34 NO: 2 PG: 426-450 YR: 2006

Even if the conceptual gap were to narrow only slightly over U.S. foreign policy generally and terrorism particularly ,
powerful
international and domestic constraints remain, which may motivate both the United States and Europe
to close the action gap. In other words, certain existing constraints may actually serve as incentives to close the action gap
between these two global actors in the near term. Some of these constraints result from the common ties that already exist, but
still bound together by
others are unique to the United States and Europe.First, of course, the United States and Europe are
a set of underlying common values and beliefs that brought them together during the Cold War
after World War II, albeit no longer with the Soviet Union acting as a lone star guiding policy formulation. Those common values and
beliefs are hardly empty notions to the vast majority of Europeans and Americans, particularly not to the
new European states that have escaped communist rule since the fall of the Berlin Wall. How those values should
be advanced will surely remain as a source of disagreement both within and between Europe and America, but those values
will undoubtedly continue to serve as incentives for all parties to seek some policy
accommodations. Second, Europe and America are fundamentally tied by the significant economic links
that serve as the "sticky power" (Mead 2004, 46-53; Mead 2005, 29-36) between them. Indeed, economic ties
remain very strong, despite recent political differences and lingering disputes over access to both participants'
markets (Drozdiak 2005). Third, the often unspoken levels of cooperation on terrorism— for example, in
the areas of law enforcement, intelligence matters, or the tracking of financial matters—remain in
place, even in the face of more visible political differences over Iraq and the wider war on
terrorism. Moreover, the events of 3/11/04 in Madrid and 7/7/05 in London continue to provide very powerful incentives for
this kind of transatlantic cooperation. In this sense, these different kinds of "ties that bind"—and continue to
bind—should not be forgotten as important sources of momentum to seek common ground between
America and Europe.

Their arguments that low relations kills cooperation are complete hype and have no scholarly
backings

- Peters ‘8 (Dr. Ingo Peters, PhD- Prof. @ Center for Transnational Relations - Freie Universität Berlin, 3/26/2008 [Paper to be
presented at the International Studies Association (ISA) Annual Conference, “Cooperation, Conflict and Crisis: The Impact of the Iraq
War on European-American Relations)

The existing social science literature on transatlantic relations is dominated by policy studies with stark
statements on the immediate and long-term quality of the relationship . Pessimists (mostly neo-realists)
usually stress diverging trends and predict no viable future for transatlantic relations; optimists (mostly ascribing to various forms of
liberalism) may also find the relationship to be in trouble but emphasize political mismanagement, which makes for a temporary and
resolvable crisis and allows for a more promising future for the overall relationship, that is if the right lessons are learned and
applied to future political conflicts. Most of the studies on transatlantic relations, whether written by pessimists or optimists,
are, however, underspecified with regard to concepts and yardsticks for systematic qualifications
of the transatlantic relationship. The failure to introduce proper yardsticks for evaluation , however,
may cast doubt on the viability of the overall analysis.7 Experts have qualified the transatlantic crisis
over Iraq as "the greatest trans-Atlantic split since Suez and possibly the deepest EU divide since its
establishment,"8 "a turning point in transatlantic relations ,"9 and testify that after the Iraq war the transatlantic
relationship still "is in a deep crisis,"10 or that "the Western order is experiencing a severe crisis."11 However, the various
adjectives used in the above quotes hardly relate to any systematic frame of reference. When
do we see a "severe" or a "deep" crisis, and when a "minor, shallow or superficial" one?

We will cooperate over terrorism regardless of relations

Coessens ‘4 (Page 1 V OL. III, N O. 1 , M ARCH 04 15 At Least in Fighting Terrorism, Transatlantic Cooperation Is Working By
Philippe Coessens Philippe Coessens is the Head of Political, Academic, Justice, and Home Affairs at the Delegation of the European
Commission in Washington, D.C. From 1998 to 2003, he was Head of Unit “European Correspondent” in the External Relations
Directorate General. Before joining the Commission in 1991, he served in the Belgian Ministry of Foreign Affairs. This article first
appeared in the Fall 2003 issue of European Affairs, a publication of the European Institute.

While disagreements between the United States and the European Union on a range of global issues have
recently attracted attention, cooperation between the two sides of the Atlantic on counter-terrorism has
been relatively successful. There can be no doubt that this is an area in which the E uropean Union and
the United States share common goals. It is clear that terrorism and associated problems such as drug traffick- ing, money
laundering, illegal immigration, and organized crime are very much global issues. So the European Union’s anti-terrorism efforts can
be said to have had a positive impact on the world in general, and on the United States in par- ticular. This applies not only to the
Union’s internal achievements but also to its efforts to strengthen cooperation with other countries and its participation in
multilateral forums such as the United Nations and the Group of Eight leading industrial nations. The European Union set to work on
a new anti-terrorism initiative immediately after the terrorist attacks of September 11, 2001, convening a spe- cial meeting of the
European Council barely a week later and coming up with a comprehensive action plan to support the United States in the face of
the terror- ist attacks. The plan contained a series of actions aimed at enhancing police and judicial cooperation, developing
international legal instruments, stepping up efforts to cut off terrorist financing worldwide (by immediately freezing assets of a
substantial number of terrorist organizations), strengthening air security, and contributing to the systematic evaluation of relations
the European Union and the United States have
with third countries in the light of their position on terrorism. Since then,
concluded a series of important bilateral agreements on police and judicial cooperation
(notably agreements between Europol and the United States reinforcing the capability of law
enforcement authorities to exchange data), as well as on mutu- al legal assistance and
extradition. These, once fully in force, will make it eas- ier to bring to justice terrorists and other serious criminals in the
jurisdictions where they are wanted. Inside the European Union, the focus has been on judicial cooperation between member
states, increased cooperation between police and intelligence services, border control, and measures to counter the financing of
terrorism. Key achievements in these areas include agreement on a common European arrest warrant and an EU framework
decision on combating terrorism, which includes a definition of terrorist offenses; a more closely coordinated scale of penalties; a
common EU list of worldwide terrorist organizations; and mutual recognition of orders freezing the property of terrorists or securing
evidence against them. Plans are also under way to develop more secure visas and travel documents. None of these agreements was
easy to achieve. They are, however, already having real operational effects by improving the legal framework for fighting crime and
terrorism throughout the European Union. The
combination of these new instruments is effectively
denying safe haven to those who finance, plan, support, or commit terrorist acts . In addition, the
European Union has approved revised recommendations by the Financial Action Task Force aimed at stemming the flow of funds to
terrorists and further cracking down on money laundering. The Union has also been developing a multi-faceted and more coordi-
nated approach aimed at incorporating the fight against terrorism into all aspects of its foreign policy. For example, it is conducting
threat analyses of various countries and regions (Central and Latin America, South and Southeast Asia) that are leading to concrete
policy recommendations. In line with these recommendations, the European Union is launching pilot projects to help three priority
countries (Indonesia, Pakistan, and the Philippines) curb terrorist financing, strengthen law enforcement, and develop more effective
judicial systems. These countries and actions were chosen in consultation with the UN Counter-Terrorism Committee so as to assist
their governments to implement UN Security Council Resolution 1373 of September 2001, which called for wide-ranging measures
to fight terrorism. A second important step has been the introduction of anti-crime and anti-terrorism clauses in EU trade and
cooperation agreements. Such clauses require the parties to exchange information on terrorist groups and their support
organizations, with a view to preventing or punishing acts of terrorism. These provisions are being gradually included in agreements
with Chile, Algeria, Egypt, and Lebanon, and currently figure in difficult negotiations for agree- ments with Syria, Iran, and the Gulf
Cooperation Council. The Group of Eight (G8), which includes four EU member states (Britain, France, Germany, and Italy), along
with representatives of the EU institutions, has also been quick to work toward implementing UN Security Council Resolution 1373.
The G8 immediately recognized the need to provide technical assistance to other countries to help them to build the necessary
capa- bilities to fulfill their obligations under the resolution. At its latest summit meeting, in Evian in June 2003, the G8 adopted an
action plan that also concentrates on outreach activities and capacity building. The plan focuses on fighting terrorism in particular
regions, such as South East and Central Asia, and in certain specific fields, such as reinforcing border secu- rity and equipping
the U.S.-EU political
institutions to tackle money laundering and the financing of terrorist organizations. More generally,
dialogue on the entire issue of terrorism has deepened. Justice, home affairs, and counter-
terrorism officials now meet regularly, and cooperation has substantially improved since 9/11 .
A great deal of work is also being done on both sides of the Atlantic on measures to counter the proliferation of weapons of mass
destruction and their acquisition by terrorist groups, and on how to deal with so-called rogue states. The European Union also
adopted an important policy document that lays out guiding principles and a concrete plan for further counter-terrorist action at a
summit meeting in Thessaloniki in June. This plan sets the course for future action so that momentum in
the fight against terror will not be lost. The verdict on all these efforts to improve transatlantic cooperation in the
fight against terrorism must at this point be “so far, so good.” Since 9/11, cooperation has substantially improved . Intelligence
has been flowing across the Atlantic in unprecedented volumes, a number of terrorist cells have been
disrupted, and many suspected terrorists are being prosecuted in different European countries. The
European Union and the United States share the same values and objectives. We are now also sharing some of the same tools
discussions we have had about different approach- es to
available to curb terrorist activity. After all the
multilateralism on either side of the Atlantic, it is striking that in this field both sides are
stressing the importance of the work of the United Nations and the Counter-Terrorism
Committee. That shows that – on terrorism, at least – the European Union and the United
States see eye-to-eye as actors in the internastional community .

Cooperation over in the Balkans, Kosovo, and Georgia now- we don’t need higher relations

Hohn ‘8 (What Bush will hand over By Christiane Höhn, Christiane Höhn has been working on transatlantic relations at the
Council of the European Union since 2004 - October 30th 2008 / 09:02 CET )

In the Balkans, co-operation has been coupled with the EU assuming a more active role, taking
over the military operation in Bosnia from NATO and supplying most of the troops in NATO's KFOR force in Kosovo. EU enlargement
as the strongest incentive is the backdrop to EU policy towards the region. The
EU's 2000 strong rule of law and
police mission in Kosovo is another milestone, the first EU operation with US participation. In
Georgia, EU and US were in constant touch during the crisis and are working together to reach
a political solution.

Cooperation over the economy now- we don’t need higher relations


Hohn ‘8 (What Bush will hand over By Christiane Höhn, Christiane Höhn has been working on transatlantic relations at the
Council of the European Union since 2004 - October 30th 2008 / 09:02 CET )

On the economic front, the creation of the Transatlantic Economic Council in 2007 stands out,
producing progress on issues such as investment, accounting standards, alternatives to animal
testing and the certification of electrical equipment. The aggregate savings on both sides of
the Atlantic of closer co-operation on economic issues could be substantial, $10 billion by the
Commission's estimate. The financial crisis has been placed at the centre of EU-US cooperation, as shown by the recent visit of the
president of the European Council, Nicolas Sarkozy, and Commission President José Manuel Barroso to Camp David, and the
upcoming summit on financial markets and the world economy in Washington .

US primacy means Europe won’t jeopardize cooperation even if relations are low

McCormick ‘6 (The War on Terror and Contemporary U.S.-European Relations James M. McCormick 1 1 Iowa State University
ABSTRACT AU: James M. McCormick TI: The War on Terror and Contemporary U.S.-European Relations SO: Politics &
Policy VL: 34 NO: 2 PG: 426-450 YR: 2006

In effect, European states and the European Union will be compelled to face the reality of American global
power and presence and, in that process, will be likely to accommodate some American policies, even as
they may seek to modify them. Senator Biden made the same point more colorfully and more bluntly
after visiting Europe in January 2005. At Condoleezza Rice's confirmation hearings, the senior committee
Democrat shouted out for the benefit of Europeans , "[g]et over it. Get over it. President Bush is our
president for the next four years. So get over it and start to act in your interest, Europe" (U.S. Congress 2005 ).
EU Trade War Answers
Frontline

Zero risk of trade war- the concept isn’t real

Alden ‘12 (Edward, Bernard L. Schwartz senior fellow at the Council on Foreign Relations (CFR), specializing in U.S. economic
competitiveness “What Exactly Is a “Trade War”? Time to Abolish a Silly Notion,” http://blogs.cfr.org/renewing-
america/2012/10/23/what-exactly-is-a-trade-war-time-to-abolish-a-silly-notion/, October 23, 2012)

I have a suggestion for everyone who writes about international trade: itis time to bury, once and for all, the concept of
a “trade war.” The phrase is so ubiquitous that it will be awfully hard to abolish; I have probably been guilty myself from time
to time. Indeed, it is almost a reflex that every time the United States or some other nation takes any action that
restricts imports in any fashion, reporters and editorial writers jump to their keyboards to warn that a trade war is
looming. But it is a canard that makes it far harder to have a sensible discussion about U.S. trade policy. No sooner had President
Obama and Mitt Romney finished their latest round of “who’s tougher on trade with China?” in their final debate Monday night than
the New York Times – to take one of many possible examples – warned that “formally citing Beijing as a currency manipulator may
backfire, economic and foreign-policy experts have said. In the worst case, it could set off a trade war, leading to falling American
exports to China and more expensive Chinese imports.” But what exactly is a “trade war”? To take the U.S.-China example, the
notion seems to be that, if the United States restricts Chinese imports, China will respond by restricting imports of U.S. goods, in turn
leading to further U.S. restrictions and so on and so on until trade between the two countries plummets. The closest historical
example is the reaction to the infamous Smoot-Hawley tariff act of 1930, which raised the average U.S. tariff on imports to
historically high levels. As trade historian Douglas Irwin of Dartmouth has show persuasively, Smoot-Hawley did not cause the Great
Depression, and probably did not even exacerbate it very much since trade was a tiny part of the U.S. economy. But Smoot-Hawley
did result in Great Britain, Canada and other U.S. trading partners raising their own tariffs in response. Irwin suggests that the higher
tariffs were probably responsible for about a third of the 40 percent drop in imports between 1929 and 1932, and perhaps a slightly
higher percentage of export losses. And the new trade barriers put in place took many decades to dismantle. With imports and
exports today comprising roughly a third of the U.S. economy, and the few remaining tariffs mostly in the single digits, the
consequences of similar tit-for-tat tariff increases today would be far more severe. But what are the
chances of such a “trade
war” actually occurring? Pretty close to zero, for two big reasons. First, in 1930, there was no World Trade
Organization, no North American Free Trade Agreement, no European Community/Union – in short, no rules to prevent
countries from jacking up tariffs or imposing quotas whenever governments felt domestic political pressure to do so. Today,
such unilateral action is largely forbidden. Indeed, the tit-for-tat measures we have seen in the U.S.-China trade relationship
have all been taken within the framework of WTO rules. When the Obama administration curbed purchases of
Chinese steel in 2009 under the “Buy America” provisions of the stimulus, for example, China responded with an
“anti-dumping” case which led to tariffs on imports of U.S. steel. But the United States challenged that action in the
WTO, and just last week the WTO ordered China to lift the duties. No trade war – instead the phrase “see you
in court” comes to mind. Secondly, almost every nation in the world seems fully aware of the dangers of aggressive
protectionism. One of the striking things about the Great Recession– which resulted in global trade volumes plunging by more
than 12 percent in 2009, the biggest drop since World War II – is how little of the protectionism that is permitted under WTO rules
actually occurred. Chad Bown of the World Bank has documented the surprising low level of new trade barriers imposed during the
recession and its aftermath. The danger of competitive currency devaluations – which are not clearly covered under WTO rules – is a
greater threat than tariffs. This is one of the reasons that Romney’s pledge to label China a currency manipulator
could be playing with fire, particularly after more than seven years in which the value of the renminbi has been creeping up steadily
against the dollar. And his suggestion that the United States would impose tariffs in response is
just silly – it would be a
blatant violation of WTO rules and would quickly be slapped down as such. Again, however, no trade
war – just an unfavorable WTO decision with which a Romney administration would quickly comply.
EU-US trade relations are resilient – common interests and WTO prevent trade wars.

Brightbill ‘4 (Timothy Brightbill, Co-Chair of the American Bar Association Section of International Law and Practice,
International Trade Committee and is an Adjunct Professor at Georgetown University Law Center, Spring, 2004 [ILSA Journal of
International & Comparative Law, 10 ILSA J Int'l & Comp L 467, lexis]

Despite the many obstacles, which are formidable, the United States and European Union still
have a strong working relationship on trade issues, thanks in large part to the their common
interests, the efforts of Ambassadors Zoellick and Lamy and their respective agencies. There are some who see the broader U.S.-
EU diplomatic relationship as moving apart, and in some areas, that may be true. But in the area of trade, I think there is little
evidence of a weakening relationship. At least, not yet. We
could well be engaged in an all-out trade war on any
of the trade issues described above, but we are not. If anything, I think the United States and the
European Union will look to collaborate even more on trade negotiations at the WTO,
particularly given the emergence of the G-21 countries at Cancun.

Economic ties and shared values overwhelm trade disputes

Kennedy and Bouton ‘2 (president of the German Marshall Fund and president of the Chicago Council on Foreign Relations
—2002 (Craig and Marshall, November 1, Foreign Policy, p. 66, Lexis)

These strains, however, need not lead to gradual dissolution of the trans-Atlantic alliance. As we have emphasized several times,
Americans and Europeans want to work together. Americans view Europe as relatively more
important than Asia and as a potential ally in addressing a wide range of challenges. Europeans, for their part, show
few signs of anti-Americanism. They increasingly question U.S. strategies, as Chancellor Schroeder demonstrated during
the recent German electoral campaign. But, based on our data, this questioning comes from a desire for a more equal relationship
An ever stronger set of trans-Atlantic economic and cultural
rather than a desire to cease cooperation.
ties also supports this commitment to cooperation on both sides. The rise of the U.S.-European
corporation formed out of mergers like Daimler and Chrysler is only one sign of how integrated
the two business communities are becoming. Trade disputes arise frequently, but these
conflicts are small compared with the overall commercial traffic across the Atlantic. On a range
of social and cultural issues, Americans and Europeans are also growing closer together . In our
study, we found a remarkable degree of agreement, even on controversial issues like global
warming and biotechnology. These common economic interests and shared values may be enough to
preserve a strong trans-Atlantic relationship in the face of the differences we have highlighted.
Americans and Europeans are not from different planets. Their worldviews differ in important ways, but they also stand on the same
ground in many other areas. The challenge for political leaders in the United States and Europe will be to find areas where shared
perspectives are strong enough to offset their few--but very strong--differences.

Mutual economic ties check trade wars

Ahearn 7 (Raymond J., Specialist in International Trade and Finance Foreign Affairs, Defense, and Trade Division – Congressional
Research Service, “Trade Conflict and the U.S.-European Union Economic Relationship”, 4-11,
http://italy.usembassy.gov/pdf/other/RL30732.pdf)
Foreign Policy Conflict Unlike traditional trade conflicts, foreign policy inspired trade squabbles
tend to lack the same kind of institutional arrangements and pressures that dampen the supply of and demand for protection.
Nor are these conflicts easily framed along free trade and protectionism lines. Some of these conflicts, but not all, may
be
moderated in the future by lobbying efforts of big business on both sides of the Atlantic to
maintain stable commercial ties. However, if Brussels or Washington is determined to use trade to achieve foreign
policy objectives, lobbying efforts are unlikely to be successful in the absence of a transatlantic agreement to treat these issues
in a more consistent fashion.

Trade wars empirically don’t escsalate

Ahearn 7 (Raymond J., Specialist in International Trade and Finance Foreign Affairs, Defense, and Trade Division – Congressional
Research Service, “Trade Conflict and the U.S.-European Union Economic Relationship”, 4-11,
http://italy.usembassy.gov/pdf/other/RL30732.pdf)

Given the high level of U.S.-EU commercial interactions, trade tensions and disputes are
not unexpected. In the past, U.S.-EU trade relations have witnessed periodic episodes of rising
trade tensions and conflicts, only to be followed by successful efforts at dispute settlement. This ebb and flow of
trade tensions occurred again in 2006 with high-profile disputes involving the Doha Round of multilateral trade negotiations and
production subsidies for the commercial aircraft sector. MajorU.S.-EU trade disputes have varied causes.
Some disputes stem from demands from producer interests for support or protection. Trade conflicts involving
agriculture, aerospace, steel, and ‘contingency protection’ fit prominently into this
grouping. These conflicts tend to be prompted by traditional trade barriers such as subsidies, tariffs, or industrial policy
instruments, where the economic dimensions of the conflict predominate. Other conflicts arise when the U.S. or the EU initiate
actions or measures to protect or promote their political and economic interests, often in the absence of significant private
sector pressures. The underlying cause of these disputes over such issues as sanctions, unilateral trade actions, and preferential
trade agreements are different foreign policy goals and priorities of Brussels and Washington. Still other conflicts stem from an
array of domestic regulatory policies that reflect differing social and environmental values and objectives. Conflicts over
hormone-treated beef, bio-engineered food products, protection of the audio-visual
sector, and aircraft hushkits, for example, are rooted in different U.S.-EU regulatory
approaches, as well as social preferences.

No spillover

Ahearn 7 (Raymond J., Specialist in International Trade and Finance Foreign Affairs, Defense, and Trade Division – Congressional
Research Service, “Trade Conflict and the U.S.-European Union Economic Relationship”, 4-11,
http://italy.usembassy.gov/pdf/other/RL30732.pdf)

In sum, U.S.-EU bilateral trade conflicts do not appear to be as ominous and threatening as
the media often portray, but they are not ephemeral distractions either. Rather they appear to have real,
albeit limited, economic and political consequences for the bilateral relationship . From an
economic perspective, the disputes may also be weakening efforts of the two partners to provide strong leadership to the global
trading system.

Trade wars won’t break US/EU relations


Ahearn 7 (Raymond J., Specialist in International Trade and Finance Foreign Affairs, Defense, and Trade Division – Congressional
Research Service, “Trade Conflict and the U.S.-European Union Economic Relationship”, 4-11,
http://italy.usembassy.gov/pdf/other/RL30732.pdf)

Mark Twain reportedly once said of Wagner’s music that “it


is not as bad as it sounds.” Similarly, U.S.-EU trade
conflicts may not be as ominous and threatening as they appear. Despite the rise in trade tensions
and episodes of tit-for-tat retaliation over the past few years, the notion that the relationship
between the world’s two most powerful economic powers is constantly teetering on the
brink of a transatlantic trade war seems a stretch . Nor does it appear that the trade
conflicts represent or symbolize any kind of fundamental rift that is possibly developing between the
United States and Europe.
European War Answers
Frontline

No chance of European war

Asmus 3 (Ronald, Senior Fellow – German Marshall Fund, Foreign Affairs, September / October, Lexis)

Several factors make the recent collapse in transatlantic cooperation surprising. The crisis came on the heels of the alliance’s
renaissance in the 1990s. Following deep initial differences over Bosnia at the start of the decade, the United States
and Europe came together to stem the bloodshed in the Balkans in 1995 and again in 1999. Led by Washington, NATO
expanded to include central and eastern Europe as part of a broader effort to secure a new post-
Cold War peace. This initiative was also accompanied by the creation of a new NATO partnership
with Russia. As a result, Europe today is more democratic, peaceful, and secure than ever. For the
first time in a century, Washington need not worry about a major war on the continent -- a
testimony to the success in locking in a post-Cold War peace over the last decade.

U.S. won’t be involved

Layne 99 (Christopher, Visiting Scholar at the Center for International Studies, “Faulty Justifications and Ominous Prospects”,
Cato Analysis, http://www.cato.org/pubs/pas/pa357.pdf)

Instability in its peripheries may affect Europe, but, contrary to the U.S. foreign policy establishment’s conventional wisdom, it
has never been true that Europe’s wars invariably affect America’s security interests. Most of
Europe’s wars—even wars involving the great powers—have not affected American security.
Moreover, the counterhegemonic strategy much more accurately delineates the requirements of America’s European strategy
than does the current strategy of reassurance and stabilization. The kinds of small-scale
conflicts that have occurred this
decade in the Balkans do not threaten America’s security interests because such conflicts do not
raise the single strategic danger that Europe could pose to the United States: the emergence of a continental
hegemon. Thus, the “new” NATO represents a radical transformation of the alliance’s strategic mission— and of America’s
role in NATO.

No war and no draw in

Liberman 1 (Peter, Associate Professor of Political Science – Queens College of the City University of New York, “Ties That
Bind”, Security Studies, 10(2), Winter, p. 103-104)

With the disappearance of the Soviet Union and no other potential superpowers on the horizon, the United
States has little to fear from would-be Eurasian hegemons for the foreseeable future. The United States is the
world’s sole superpower, with an unrivaled combination of economic resources, advanced technology, and military capabilities.
China, Russia, France, and Britain each have robust nuclear deterrents, virtually precluding their
conquest or domination by others. Japan, Germany, France, Britain, and Russia are all
democracies, and—assuming they stay that way—domestically impeded from absorbing other modern
nations. If democratic peace theory is correct, they are unlikely to fight each other at all. Even if, however, China
were to absorb Japan, Russia to conquer Germany, or Europe to unify politically, this would still not create
the kind of Eurasian behemoth so feared during the First World War, the Second World War, and the cold war. At
worst, such regional hegemonies would return the international system to bipolarity , and only
at this point would threat require renewed U.S. balancing. It seems reasonable to conclude that, as Robert Jervis has
put it, “few imaginable disputes [in the post–cold war era] will engage vital U.S. interests.”

Economic and political ties make European war impossible

Smitherman 3 (Charles W. III, Faculty of Law – University of Oxford and Doctoral Studies on European Union-United States
Trade Law and Regulation, Minnesota Journal of Global Trade, Summer)

No person, no entity, no relationship remains exempt from time's paradigm, and nowhere could this model be more applicable
on a global scale than in the relations of the United States and Europe. The
Allied victory in World War II and
the end of a near half-century of continent-wide war in Europe led to the initiation of
arrangements aimed at averting the reoccurrence of the past through commonality and
economic integration, erecting supportive pillars that would form the foundation of what
would later become known as the European Union (EU). 3 The conquering of fascism spurred the rise of an equally
threatening ideology on Europe's eastern borders, a communist agenda standing in stark contrast to the democratic, capitalistic
ideals of post-war Europe and its friend and ally, the United States. 4 Throughout the latter half of the twentieth century, in
recognition of its common ideological commitment and shared foe, the
United States provided incentives and
encouragement to the nation-states of Europe to integrate their economies and foreign
policies, binding themselves so tightly that war between the European nation-states would
be all but impossible. At the same time, this integration provided the United States with an economic and political
partner in the Cold War. 5
Eurozone Answers
Frontline

Eurozone collapse doesn’t spill-over globally –


Hasenstab ’12 (Portfolio Manager and Co-Director of the International Bond Department for Franklin Templeton Fixed
Income Group., 1-18-12

[Michael, Euro Zone Woes Cannot Sink Asia: Expert, http://www.cnbc.com/id/46049692]

The recent weakness in


asset markets in Asia shows that investors now believe Europe’s woes will
put the region at risk. But whether this fear is justified depends on two possible scenarios – a breakup of the euro zone or a
European recession. Those who believe in the first scenario have reason to be worried. A euro zone breakup involving any of the
major economies would have an impact worse than the one seen after the Lehman collapse. We would see a domino effect on
sovereign debt defaults and foreign exchange markets would be plunged into chaos by the sudden disappearance of the world’s
second most important reserve currency. But how likely is this scenario? Very unlikely. Recent moves towards establishing a
stronger fiscal union in Europe, unprecedented and massive provisioning of liquidity through both the European Central Bank (ECB)
and national central banks, the massive additional balance sheet capacity of the ECB, fiscal and structural reforms in Italy, and the
prohibitive costs of an exit from the euro for any major member of the European Monetary Union, including Germany, should
ensure that the first scenario does not materialize. However, the second scenario of a painful deleveraging in many European banks
and anemically weak European growth is possible. This
outcome would be bad for Europe, but not bad
enough to undermine the outlook for stronger parts of the global economy, especially
emerging Asia. Here’s why. Europe was not the main engine of the global economy to start off
with, and it remains a relatively closed economy. A European recession, especially if deeper
and more protracted, would dampen world trade, including Asian exports, but nothing on the
scale seen in 2008. But trade only provides part of the linkage. The more important linkages come via the capital markets.
The European Banking Authority’s requirement for banks to reach a tier 1 capital ratio of 9 percent by June of this year will require
broad based deleveraging. As raising fresh capital is extremely difficult, one other avenue could be to shed assets, including assets
abroad. Asia and other emerging markets, however, should not suffer unduly. Banks Can’t Afford to Exit Asia Most foreign banks are
present in Asia via wholly owned subsidiaries, which cannot simply take capital back to their parent companies. Many of these
subsidiaries are some of the most profitable parts of their businesses, and growing profits provides one important way to
recapitalize. Shutting down all business lines in emerging markets would leave these banks without this important source of profits
and force an even greater reliance on a weak domestic banking market in Europe. Furthermore, a plan to temporarily exit and re-
enter may not be possible, as a foreign company that leaves town during hard times would not be quickly welcomed or permitted
back. Indeed, even at the height of the post-Lehman financial crisis Asia did not see a wholesale pulling out of assets. But this
deleveraging by many European banks is only part of the capital flow story. Fund Flow to Asia Will Continue Meanwhile, the ECB has
launched its version of quantitative easing, which now augments the extraordinarily loose monetary policy of the U.S., Japan and
U.K. We now see the most aggressive printing of money in modern times. While this aims to address domestic conditions, capital
cannot be contained within national borders. Abundant global liquidity will continue to flow into Asian markets blessed with strong
macro fundamentals—particularly as the region’s currencies still appear largely undervalued. Short-term volatility excluded,
monetary policy in these four major economies will ultimately facilitate net capital inflows into Asia and many other asset markets
and thus avoid the risk of a recession-induced credit crunch in Asia. Also, strong economic and political fundamentals support Asia.
Unlike Europe or the U.S., Asia has built up plenty of room to provide fiscal stimulus and to lower interest rates in response to a
worsening external environment. For example, the South Korean government’s debt levels have been slashed over the last decade
and international reserves now well exceed levels seen before the global financial crisis. And the largest countries like China, India
and Indonesia can count on a robust and resilient domestic demand to counter external demand weakness. A euro zone
disintegration would be as calamitous as it is unlikely. On the other hand, the
far more probable scenario of painful
deleveraging by European banks and weak European growth, while a serious setback for
Europe, will likely have far more modest and manageable global spillovers. The world
economy is used to powering ahead without much help from European demand. It will do so
this time as well. And Asian markets, with their strong market fundamentals and unparalleled future growth prospects, will
continue to lead the charge.
NATO solves the impacts

Goure, Lexington Institute President, ‘11


[Daniel, held senior positions in both the private sector and the U.S. Government. Most recently, he was a member of the 2001
Department of Defense Transition Team. Dr. Goure spent two years in the U.S. Government as the director of the Office of Strategic
Competitiveness in the Office of the Secretary of Defense. He also served as a senior analyst on national security and defense issues
with the Center for Naval Analyses, Science Applications International Corporation, SRS Technologies, R&D Associates and System
Planning Corporation. was the Deputy Director, International Security Program at the Center for Strategic and International Studies.,
"Coming EU Collapse Could Re-energize NATO ", http://www.defpro.com/news/details/28104/?
SID=50824a11d84c46b49aa469947308327b]

Could the EU collapse? Unthinkable! Impossible! Yet, that is what was being said just a few months ago about the possibility
that Greece would default on its debt. Now, as Europe struggles to put together a second bailout package for that country, the
conversation has increasingly focused not on how to save Greece -- and with it the Eurozone -- but when and how the default will
happen. The
consensus now is not only will Greece default but it will be messy. The unthinkable
has become the base case. A Greek default, albeit actually good for that country, will trigger a massive economic, political
and existential crisis for the European Union (EU). A default will put enormous pressure on other weak European economies,
particularly Portugal, Ireland, Spain and Italy. This will cause a crisis among the European banks. But more significantly, Europe will
be faced with a choice: save the Euro or save the EU. There won't be the time to forge a true single fiscal and political union. The
only way both to protect the viable economies of Northern and Central Europe and allow the weaker ones in the South a chance to
recover is by the former group of countries to allow, even insist that, the latter group exit the Eurozone. This
will
fundamentally and irrevocably alter the character and purpose of the EU. In fact, should the economic
contagion reach France, the EU will certainly collapse. It is ironic that the Euro’s decline and the
EU’s possible demise is likely to be a boon for NATO. Absent a strong and expanding EU, NATO
will be the sole gravitational force exerting a cohesive influence across the continent. NATO
already includes several countries who are not members of the EU; this number will grow
significantly if a Euro crisis occurs. Most important, NATO will have to stand guard over Europe
during a time that may come to resemble economic and even politically the 1930s. NATO can
help stabilize weak European governments under tremendous stress. It also can deter outside
powers from seeking to take advantage of Europe's temporary weakness. Finally, NATO provides
the best means for leveraging what is likely to be a shrinking stock of trans-Atlantic military
capabilities. Despite its contributions to the conflicts in Afghanistan and Libya, it has become fashionable in recent
years in some circles to dismiss NATO as a quaint anachronism. It may well turn out that NATO
will prove to be the most important US security relationship of the 21st Century.
Failed States Answers
Frontline

No impact to failed states

Patrick ‘11, senior fellow, director – program on international institutions and global governance @ CFR, 4/15/’11
(Stewart M, “Why Failed States Shouldn’t Be Our Biggest National Security Fear,” http://www.cfr.org/international-peace-and-
security/why-failed-states-shouldnt-our-biggest-national-security-fear/p24689)

In truth, while failed states may be worthy of America's attention on humanitarian and development grounds, most of them
are irrelevant to U.S. national security. The risks they pose are mainly to their own inhabitants.
Sweeping claims to the contrary are not only inaccurate but distracting and unhelpful, providing
little guidance to policymakers seeking to prioritize scarce attention and resources. In 2008, I collaborated with Brookings
Institution senior fellow Susan E. Rice, now President Obama's permanent representative to the United Nations, on an index of
state weakness in developing countries. The study ranked all 141 developing nations on 20 indicators of
state strength, such as the government's ability to provide basic services. More recently, I've examined whether these
rankings reveal anything about each nation's role in major global threats: transnational terrorism,
proliferation of weapons of mass destruction, international crime and infectious disease. The findings are startlingly
clear. Only a handful of the world's failed states pose security concerns to the United States. Far greater dangers emerge
from stronger developing countries that may suffer from corruption and lack of government accountability but come
nowhere near qualifying as failed states. The link between failed states and transnational terrorism, for
instance, is tenuous. Al-Qaeda franchises are concentrated in South Asia, North Africa, the Middle East and Southeast
Asia but are markedly absent in most failed states, including in sub-Saharan Africa. Why? From a terrorist's perspective,
the notion of finding haven in a failed state is an oxymoron. Al-Qaeda discovered this in the 1990s
when seeking a foothold in anarchic Somalia. In intercepted cables, operatives bemoaned the insuperable
difficulties of working under chaos, given their need for security and for access to the global
financial and communications infrastructure. Al-Qaeda has generally found it easier to maneuver in corrupt but functional
states, such as Kenya, where sovereignty provides some protection from outside interdiction. Pakistan and Yemen
became sanctuaries for terrorism not only because they are weak but because their governments lack
the will to launch sustained counterterrorism operations against militants whom they value for other purposes. Terrorists
also need support from local power brokers and populations. Along the Afghanistan-Pakistan border, al-Qaeda finds succor in the
Pashtun code of pashtunwali, which requires hospitality to strangers, and in the severe brand of Sunni Islam practiced locally.
Likewise in Yemen, al-Qaeda in the Arabian Peninsula has found sympathetic tribal hosts who have long welcomed mujaheddin back
from jihadist struggles. Al-Qaeda has met less success in northern Africa's Sahel region, where a moderate, Sufi version of Islam
dominates. But as the organization evolves from a centrally directed network to a diffuse movement with
autonomous cells in dozens of countries, it
is as likely to find haven in the banlieues of Paris or high-rises of
Minneapolis as in remote Pakistani valleys. What about failed states and weapons of mass destruction? Many U.S.
analysts worry that poorly governed countries will pursue nuclear, biological, chemical or radiological weapons; be unable to control
existing weapons; or decide to share WMD materials. These fears
are misplaced. With two notable exceptions — North
Korea and Pakistan — the world's weakest states pose minimal proliferation risks, since they have
limited stocks of fissile or other WMD material and are unlikely to pursue them. Far more
threatening are capable countries (say, Iran and Syria) intent on pursuing WMD, corrupt nations (such as Russia) that
possess loosely secured nuclear arsenals and poorly policed nations (try Georgia) through which proliferators can smuggle
illicit materials or weapons. When it comes to crime, the story is more complex. Failed states do dominate production of some
narcotics: Afghanistan cultivates the lion's share of global opium, and war-torn Colombia rules coca production. The tiny African
failed state of Guinea-Bissau has become a transshipment point for cocaine bound for Europe. (At one point, the contraband
transiting through the country each month was equal to the nation's gross domestic product.) And Somalia, of course, has seen an
explosion of maritime piracy. Yet failed
states have little or no connection with other categories of
transnational crime, from human trafficking to money laundering , intellectual property theft, cyber-
crime or counterfeiting of manufactured goods. Criminal networks typically prefer operating in functional
countries that provide baseline political order as well as opportunities to corrupt authorities.
They also accept higher risks to work in nations straddling major commercial routes. Thus narco-
trafficking has exploded in Mexico, which has far stronger institutions than many developing nations but borders the United States.
South Africa presents its own advantages. It is a country where “the first and the developing worlds exist side by side,”
author Misha Glenny writes. “The first world provides good roads, 728 airports . . . the largest cargo port in Africa, and an efficient
banking system. . . . Thedeveloping world accounts for the low tax revenue, overstretched social
services, high levels of corruption throughout the administration, and 7,600 kilometers of land and sea borders
that have more holes than a second-hand dartboard.” Weak and failing African states, such as Niger, simply cannot
compete. Nor do failed states pose the greatest threats of pandemic disease. Over the past decade, outbreaks of SARS, avian
influenza and swine flu have raised the specter that fast-moving pandemics could kill tens of millions worldwide. Failed states, in this
regard, might seem easy incubators of deadly viruses. In fact, recent fast-onset pandemics have bypassed most failed states, which
are relatively isolated from the global trade and transportation links needed to spread disease rapidly. Certainly, the world's
weakest states — particularly in sub-Saharan Africa — suffer disproportionately from disease, with infection rates higher than
in the rest of the world. But their principal health challenges are endemic diseases with local effects , such
as malaria, measles and tuberculosis. While U.S. national security officials and Hollywood screenwriters obsess
over the gruesome Ebola and Marburg viruses, outbreaks of these hemorrhagic fevers are rare and self-
contained. I do not counsel complacency. The world's richest nations have a moral obligation to bolster health systems in Africa,
as the Obama administration is doing through its Global Health Initiative. And they have a duty to ameliorate the challenges posed
by HIV/AIDS, which continues to ravage many of the world's weakest states. But poor performance by developing countries
in preventing, detecting and responding to infectious disease is often shaped less by budgetary and
infrastructure constraints than by conscious decisions by unaccountable or unresponsive regimes. Such
deliberate inaction has occurred not only in the world's weakest states but also in stronger developing
countries, even in promising democracies. The list is long. It includes Nigeria's feckless response to a 2003-05
polio epidemic, China's lack of candor about the 2003 SARS outbreak, Indonesia's obstructionist attitude to
addressing bird flu in 2008 and South Africa's denial for many years about the causes of HIV/AIDS. Unfortunately,
misperceptions about the dangers of failed states have transformed budgets and bureaucracies.
U.S. intelligence agencies are mapping the world's “ungoverned spaces.” The Pentagon has turned its regional Combatant
Commands into platforms to head off state failure and address its spillover effects. The new Quadrennial Diplomacy and
Development Review completed by the State Department and the U.S. Agency for International Development depicts fragile and
conflict-riddled states as epicenters of terrorism, proliferation, crime and disease. Yet such preoccupations reflect more
hype than analysis. U.S. national security officials would be better served — and would serve all of us better
— if they turned their strategic lens toward stronger developing countries, from which transnational threats are more
likely to emanate.

No impact to failed states - reject their flawed studies

Logan and Preble ‘10 (Justin, Associate Director of Foreign Policy Studies @ Cato, and Christopher, Director of Foreign Policy
Studies @ Cato, " Washington’s Newest Bogeyman: Debunking the Fear of Failed States," Strategic Studies Quarterly, Summer,

A survey of the formal studies of state failure reveals a methodological wasteland . Analysts have
created a number of listings of failed states, which have, in fairness, overlapped considerably; all are populated by poor countries,
many of which have been wracked by interstate or civil violence.48 However, instead of adhering to basic social-scientific standards
of inquiry, in which questions or puzzles are observed and then theories are described and tested using clearly defined independent
and dependent variables, analysts began by drawing up a category—failed state—and then attempted to create data sets from
which theoretical inferences could be induced. To take one prominent case, the authors of the State Failure Task Force Report
contracted by the Central Intelligence Agency’s Directorate of Intelligence chose to adjust their definition of “failed state” after their
initial criteria did not produce an adequate data set for the quantitative tests the researchers wanted to perform. After dramatically
expanding the definition, the task force produced almost six times more countries that could be coded “failed” as compared with
their original criteria and then proceeded with their statistical analysis. They justified this highly questionable decision on the
judgment that “events that fall beneath [the] total-collapse threshold often pose challenges to US foreign policy as well.”49
Subsequently, the task force changed its name to the “Political Instability Task Force” and appeared to back away from the term
failed state.50 Beyond methodological shortcomings, the lists of failed states reveal only that there are many countries plagued by
severe problems. The top 10 states in the 2009 Fund for Peace/Foreign Policy magazine Failed States Index include two countries the
United States occupies (Iraq and Afghanistan), one country without any central government to speak of (Somalia), four poor African
states (Zimbabwe, Chad, the Democratic Republic of the Congo, and the Central African Republic), two resource-rich but unstable
African countries (Sudan and Guinea) and a nuclear-armed Muslim country, population 176 million (Pakistan). The sheer diversity of
the countries on the lists makes clear that few policy conclusions could be drawn about a country based on its designation as a failed
state. In fact, what has happened is that analysts have seized on an important single data point—Afghanistan in the 1990s and 2000s
—and used it to justify a focus on failed states more broadly.
Because Afghanistan met anyone’s definition of
failed state and because it clearly contained a threat, analysts concluded en masse that failed
states were threatening. When confronted with the reality that the countries regularly included on lists of
failed states include such strategic non-entities as the Democratic Republic of the Congo, Liberia, and
East Timor, advocates of focusing on state failure routinely point back at the single case that can be justified directly on US
national security grounds: Afghanistan.51 Even in Afghanistan, however, remedying the condition of “state failure” would not have
eliminated the threat, and eliminating the threat—by killing or capturing Osama bin Laden and his confederates—would not have
remedied the “failure.”
The fact that expansive claims about the significance of state failure have been
used to market studies of the subject, when viewed in light of the diverse and mostly
nonthreatening states deemed “failed,” leaves the impression of a bait and switch. For instance, the
2007 update of the Failed States Index promises on the magazine’s cover to explain “why the world’s weakest countries pose the
greatest danger.” The opening lines of the article declare that failed states “aren’t just a danger to themselves. They can threaten
the progress and stability of countries half a world away.” Strikingly, then, the article does little to back up or even argue these
claims. It instead shrugs that “failing states are a diverse lot” and that “there are few easy answers to their troubles.” By 2009, the
“greater risk of failure is not always synonymous with greater consequences of
index was conceding that
failure,” and that the state failure-terrorism link “is less clear than many have come to assume.”52
Given these concessions undermining the idea that state failure is threatening , one wonders why
scholars continue to study failed states at all. As seen above, the countries on lists of failed states are so diverse that it is difficult to
draw any conclusions about a state’s designation as failed. But the purpose, one would think, of creating a new category of states
would be to unify countries that share attributes that can inform either how we think about these states or how we craft policies
toward these states. Instead, the scholarship on state failure has arbitrarily grouped together countries
that have so little in common that neither academic research nor policy work should be influenced
by this concept. Despite repeated claims to the contrary, learning that a task force has deemed a particular state “failed” is not
particularly useful. Start with the Conclusions and Work Backward Existing scholarship on state failure seems to indicate that the
conclusion led to the analysis, rather than vice versa. Scholars who argue that “failed state” is a meaningful
category and/or indicative of threat provide a rationale for American interventionism around the globe. Given the arbitrary creation
of the category “failed state” and the extravagant claims about its significance, it is difficult to avoid the conclusion that research on
failed states constitutes, as one analyst put it, “an eminently political discourse, counseling intervention, trusteeship, and the
abandonment of the state form for wide swaths of the globe.”53

Multiple states prove no impact

Logan and Preble ‘8 — associate director of foreign policy studies at the Cato Institute, AND, director of foreign policy
studies at the Cato Institute (Justin and Christopher Preble, Harvard International Review, Winter 2008, Volume 29, Issue 4, pg. 62)
Anti-sovereignty academics and pro-empire Beltway pundits frequently defend their arguments by making
assertions along the lines that "weak and failed states pose an acute risk to US and global
security," as Carlos Pascual, the US State Department's first Coordinator for reconstruction and Stabilization, and Stephen Krasner
wrote in Foreign Affairs in 2005. This is a rather dubious claim. The Fund for Peace/Foreign Policy
magazine Failed States Index, for example, includes on its top 10 "most failed" states list
Zimbabwe, Chad, Ivory Coast, the Democratic republic of the Congo, Guinea, and the Central
African republic. It is difficult to imagine what threats are emerging from these countries that
merit significant attention from US security strategists . To be sure, Afghanistan in the late 1990s was both a
failed state by any definition and a threat to the United States. It should serve as a pointed reminder that we cannot ignore failed
states. Traditional realist definitions of power reliant on conventional military capability, size of economy, and population, must now
be supplemented with a recognition that small bands of terrorists could emerge from a backward corner of the globe and strike at
the heart of the United States as well. But even here the interventionists' logic is weak. Attacking the threat that resided in failed
Afghanistan in the 1990s would have had basically no effect on the health of the Afghan state. Killing Osama bin Laden and his
comrades would have more substantially reduced the threat that bloomed on 9/11 than sending in US or international development
personnel would have done. Attacking a threat rarely involves paving roads or establishing new judicial standards. It is this
categorical error that is at the heart of the trouble with obsessing over state failure. To
the extent that a threat has
ever emanated from a failed state-and Afghanistan is essentially the only example of this-
addressing the failure is different from attacking the threat. At best, the attempt to correlate
state failure with terrorism relies on a dubious interpretation of terrorism: that terrorism is, at its root, a
result of poverty that can be eradicated by an aggressive development effort. As Alan B. Krueger and others have demonstrated,
however, terrorism is a response to political grievances, not a consequence of poverty. Accordingly, using the threat of terrorism to
justify nation building in failed states is inappropriate.

No impact to failed states

Finel ‘9 - a Contributing Editor at the Atlantic Council, is a Senior Fellow at the American Security Project (ASP) where he directs
research on counter-terrorism and defense policy ( April 27, Bernand “Afghanistan is Irrelevant”
http://www.acus.org/new_atlanticist/afghanistan-irrelevant

Second, there is no straight-line between state failure and threats to the United States . Indeed, the
problem with Afghanistan was not that it failed but rather that it “unfailed” and becameruled by the Taliban. Congo/Zaire
is a failed state. Somalia is a failed state. There are many parts of the globe that are essentially
ungoverned. Clearly criminality, human rights abuses, and other global ills flourish in these spaces. But the notion
that any and all ungoverned space represents a core national security threat to the United
States is simply unsustainable.
Federalism
Frontline

No international modeling of federalism

Moravcsik, 05 (Andrew, “Dream On America”, Newsweek, 1/31, http://www.msnbc.msn.com/id/6857387/site/newsweek/)


Not long ago, the American dream was a global fantasy. Not only Americans saw themselves as a beacon unto nations. So did much
of the rest of the world. East Europeans tuned into Radio Free Europe. Chinese students erected a replica of the Statue of Liberty in
Tiananmen Square.

Not long ago, the American dream was a global fantasy. Not only Americans saw themselves as a beacon unto nations. So did much
of the rest of the world. East Europeans tuned into Radio Free Europe. Chinese students erected a replica of the Statue of Liberty in
Tiananmen Square. You had only to listen to George W. Bush's Inaugural Address last week (invoking "freedom" and "liberty" 49
times) to appreciate just how deeply Americans still believe in this founding myth. For many in the world, the president's rhetoric
confirmed their worst fears of an imperial America relentlessly pursuing its narrow national interests. But
the greater
danger may be a delusional America—one that believes, despite all evidence to the contrary, that
the American Dream lives on, that America remains a model for the world, one whose mission is to spread the
word. The gulf between how Americans view themselves and how the world views them was summed up in a poll last week by the
BBC. Fully 71 percent of Americans see the United States as a source of good in the world. More than half view Bush's election as
positive for global security. Other studies report that 70 percent have faith in their domestic institutions and nearly 80 percent
believe "American ideas and customs" should spread globally. Foreigners take an entirely different view: 58 percent in the BBC poll
see Bush's re-election as a threat to world peace. Among America's traditional allies, the figure is strikingly higher: 77 percent in
Germany, 64 percent in Britain and 82 percent in Turkey. Among the 1.3 billion members of the Islamic world, public support for the
United States is measured in single digits. Only Poland, the Philippines and India viewed Bush's second Inaugural positively. Tellingly,
the anti-Bushism of the president's first term is giving way to a more general anti-Americanism. A plurality of voters (the average is
70 percent) in each of the 21 countries surveyed by the BBC oppose sending any troops to Iraq, including those in most of the
countries that have done so. Only one third, disproportionately in the poorest and most dictatorial countries, would like to see
American values spread in their country. Says Doug Miller of GlobeScan, which conducted the BBC report: "President Bush has
further isolated America from the world. Unless the administration changes its approach, it will continue to erode America's good
name, and hence its ability to effectively influence world affairs." Former Brazilian president Jose Sarney expressed the sentiments of
the 78 percent of his countrymen who see America as a threat: "Now that Bush has been re-elected, all I can say is, God bless the
rest of the world." The truth is that Americans are living in a dream world. Not only do others not share America's self-regard,
they no longer aspire to emulate the country's social and economic achievements. The loss of faith in
the American Dream goes beyond this swaggering administration and its war in Iraq. A President Kerry would have had to confront a
similar disaffection, for it grows from the success of something America holds dear: the spread of democracy, free markets and
international institutions—globalization, in a word. Countries
today have dozens of political, economic and
social models to choose from. Anti-Americanism is especially virulent in Europe and Latin America, where countries have
established their own distinctive ways—none made in America. Futurologist Jeremy Rifkin, in his recent book "The European
Dream," hails an emerging European Union based on generous social welfare, cultural diversity and respect for international law—a
model that's caught on quickly across the former nations of Eastern Europe and the Baltics. In Asia, the rise of autocratic capitalism
in China or Singapore is as much a "model" for development as America's scandal-ridden corporate culture. "First we emulate," one
Chinese businessman recently told the board of one U.S. multinational, "then we overtake." Many are tempted to write off the new
anti-Americanism as a temporary perturbation, or mere resentment. Blinded by its own myth, America has grown incapable of
recognizing its flaws. For there is much about the American Dream to fault. If the rest of the world has lost faith in the American
model—political, economic, diplomatic—it's partly for the very good reason that it doesn't work as well anymore. AMERICAN
DEMOCRACY: Once upon a time, the U.S. Constitution was a revolutionary document, full of epochal innovations—free elections,
judicial review, checks and balances, federalism and, perhaps most important, a Bill of Rights. In the 19th and 20th centuries,
countries around the world copied the document, not least in Latin America. So did Germany and Japan after World War II.
Today? When nations write a new constitution, as dozens have in the past two decades, they seldom look
to the American model. When the soviets withdrew from Central Europe, U.S. constitutional
experts rushed in. They got a polite hearing, and were sent home. Jiri Pehe, adviser to former president
Vaclav Havel, recalls the Czechs' firm decision to adopt a European-style parliamentary system with strict limits on campaigning.
"For Europeans, money talks too much in American democracy . It's very prone to certain kinds of corruption,
or at least influence from powerful lobbies," he says. "Europeans would not want to follow that route." They also sought to limit the
dominance of television, unlike in American campaigns where, Pehe says, "TV debates and photogenic looks govern election
victories." So it is elsewhere. After American planes and bombs freed the country, Kosovo opted for a European constitution.
Drafting a post-apartheid constitution, South
Africa rejected American-style federalism in favor of a
German model, which leaders deemed appropriate for the social-welfare state they hoped to construct. Now fledgling
African democracies look to South Africa as their inspiration, says John Stremlau, a former U.S. State Department
official who currently heads the international relations department at the University of Witwatersrand in Johannesburg: "We can't
rely on the Americans." The new democracies are looking for a constitution written in modern times and reflecting their progressive
concerns about racial and social equality, he explains. "To borrow Lincoln's phrase, South Africa is now Africa's 'last great hope'."
Much in American law and society troubles the world these days. Nearly all countries reject the United States' right to bear arms as a
quirky and dangerous anachronism. They abhor the death penalty and demand broader privacy protections. Above all, once most
foreign systems reach a reasonable level of affluence, they follow the Europeans in treating the provision of adequate social welfare
is a basic right. All this, says Bruce Ackerman at Yale University Law School, contributes to the
growing sense that
American law, once the world standard, has become "provincial." The United States' refusal to
apply the Geneva Conventions to certain terrorist suspects, to ratify global human-rights treaties
such as the innocuous Convention on the Rights of the Child or to endorse the International Criminal Court
(coupled with the abuses at Abu Ghraib and Guantanamo) only reinforces the conviction that America's
Constitution and legal system are out of step with the rest of the world .
Fish Answers
Frontline
Status quo solves over-fishing

NOAA 11 (" The Road to End Overfishing: 35 Years of Magnuson Act," Note: While no date is given in the article, the Magnuson
act was signed in 1976 and therefore this article was published in 2011,
http://www.nmfs.noaa.gov/stories/2011/20110411roadendoverfishing.htm,

I want to acknowledge and highlight the 35th anniversary of the Magnuson-Stevens Fishery Conservation and Management Act.
Simply called “the Magnuson Act”, this law, its regional framework and goal of sustainability, has proven to be a
visionary force in natural resource management - both domestically and internationally. The
Magnuson Act is, and will continue to be a key driver for NOAA as we deliver on our nation’s commitment to ocean
stewardship, sustainable fisheries, and healthy marine ecosystems. Because of the Magnuson Act, the
U.S. is on track to end overfishing in federally-managed fisheries, rebuild stocks, and ensure conservation
and sustainable use of our ocean resources. Fisheries harvested in the U nited States are scientifically
monitored, regionally managed and legally enforced under 10 strict national standards of
sustainability. This anniversary year marks a critical turning point in the Act’s history. By the end of 2011, we are on track to
have an annual catch limit and accountability measures in place for all 528 federally-managed fish stocks and complexes. The
dynamic, science-based management process envisioned by Congress is now in place, the rebuilding of our fisheries is underway,
and we are beginning to see real benefits for fishermen, fishing communities and our commercial and recreational fishing industries.

Fish stocks recovering

Economist 9 (“Grabbing It All”, 1-3, Lexis)

A variety of remedies have been tried, usually in combination. Thus regulations have been issued
about the size and type of fish to be caught, the mesh of nets to be used, the number of days a month that boats may go to sea,
the permissible weight of their catch and so on. In
some countries fishermen are offered inducements to
give up fishing altogether. Those that continue are, at least in theory, subject to monitoring both at sea
and in port. Large areas are sometimes closed to fishing, to allow stocks to recover. Others have been designated as marine
reserves akin to national parks. And some of the technology that fishermen use to find their prey is now used by inspectors to
monitor the whereabouts of the hunters themselves. Most of these
measures have helped, as the recovery of
stocks in various places has shown. Striped bass and North Atlantic swordfish have returned
along America's East Coast, for instance. Halibut have made a comeback in Alaska. Haddock, if
not cod, have begun to recover in Georges Bank off Maine. And herring come and go off the coasts of
Scotland. Those who doubt the value of government intervention have only to look at the waters off Somalia, a country that has
been devoid of any government worth the name since 1991. The ensuing free-for-all has devastated the coastal stocks, ruining
the livelihoods of local fishermen and encouraging them, it seems, to take up piracy instead.

Recent data concludes Neg – no collapse

Economist 9 (“Plenty More Fish in the Sea?”, 1-3, Lexis)

An even gloomier assessment came in an article by 14 academics in Science in 2006. The accelerating erosion of
biodiversity, often associated with overfishing, presaged a "global collapse" to the point, in 2048, where all
species currently fished would be gone, they said. Even
many scientists who are alarmed by the evidence of
overfishing find such conclusions controversial. Most non-scientists are unmoved. For a
start, fish appears to be in plentiful supply. Even cod is available; over 7m tonnes of cod-family
(Gadidae) fish are caught each year. Sushi bars have spread across the world. To cater for the aversion to red meat, and a new-
found need for omega-3 fatty acids, fish dishes are on every menu, even in steak houses.
Supermarkets and restaurants boast of "sustainable" supplies, and sandwiches are reassuringly
labelled "dolphin-friendly", however threatened the tuna within them may be. Best of all, for the ethical consumer, fish
are now farmed (see box below). Salmon has become so plentiful that people weary of its delicate taste.
Moreover, fishermen themselves seem sceptical of any long-term scarcity. They clamour for
bigger quotas and fewer restrictions (except on foreign competitors), and complain that the scientists are either ignorant
or one step behind the new reality. Those with long memories can cite previous collapses that have been followed by
recoveries. And, in truth, not all collapses are due solely to overfishing: the sudden crash of California's sardine industry 60 years
ago is now thought to have been partly caused by a natural change in the sea temperature. Plenty
of figures seem to
support the optimists. Despite the exploitation round its coasts, Britain, for instance, still landed
750,000 tonnes of Atlantic fish in 2006, two-thirds of what it caught in 1951; even cod is still being hauled from the
north-east Atlantic, mostly by Norwegians and Russians. Some British fishing communities—Fraserburgh, for example—are in a
sorry state, but others still prosper: the value of wet fish landed in Shetland, for example, rose from £21m in 1996 to £54m
($33m-99m) in 2006. Earnings from fishing in Alaska, in whose waters about half of America's catch is taken, rose
from less than $800m in 2002 to nearly $1.5 billion in 2007. And for the world as a whole, the catch in 2006
was over 93m tonnes, according to the UN's Food and Agriculture Organisation, compared with just 19m in 1950 (see
chart on next page). Its value was almost $90 billion.

No overfishing – declines are natural

Bluemink 8 (Elizabeth, Staff Writer – ADN, “Greenpeace Puts Pollock Fishery in its Cross Hairs”,
Anchorage Daily News, 12-3, http://www.adn.com/news/alaska/story/609562.html)

Federal scientists say there are fewer fish but the accusation of overfishing is false. The pollock industry
agrees with the federal scientists. "This
(population decline) was not unexpected, and the sky is not
falling," said David Benton, executive director of the Marine Conservation Alliance, which represents western Alaska fishing
fleets, processors and ports. Federal scientists have called for a dramatic reduction in the pollock industry's harvest next year --
the lowest catch in the fishery's history -- in response to the decline. Greenpeace and other conservation groups say even
deeper cuts in the catch are needed to ensure that pollock remain healthy in the long run. The federal scientists have
recommended limiting the pollock harvest to 815,000 tons -- the smallest in more than 30 years. Greenpeace is pushing for a
much smaller harvest: 458,000 tons. "Pollock is one of the most important food sources for every animal in the Bering Sea food
web," said George Pletnikoff, a Greenpeace campaigner based in Alaska. Next week, the North Pacific Fishery Management
Council will meet in Anchorage to consider next year's catch limit, among other tasks. The meetings begin Monday and are
expected to spill over into the following week. "People can go to the Anchorage Hilton hotel next week and give their opinions
and thoughts about the fishery. They should be involved in it," Pletnikoff said. National Marine Fisheries Service scientists say
the decline in Bering Sea pollock is due to natural variability in the fish population that has been
documented for decades, not too much harvesting.

Self-correction – diminishing returns means no fish extinction

Leal 2k (Donald, Senior Associate – Political Economy Research Center, “Homesteading the Oceans: The Case for Property Rights
in U.S. Fisheries”, August, http://www.perc.org/pdf/ps19.pdf)
In a commons situation, entering the fishing grounds first and capturing the fish fastest is a compelling strategy. This is the time
when search and capture costs are the lowest. Thus, each fisher is motivated to invest in equipment (e.g., faster boats and
better detection devices) that improve the chances of winning the race for the fish—equipment that would not be necessary if
the fishery were not under the strain of such competition. Not only do the stocks decline, but fishing becomes wastefully
expensive as too many fishers invest in too much capital to catch too few fish. Because
costs tend to rise rapidly as
fish become scarcer, fisheries have historically reached commercial extinction before they
are totally depleted. The additional costs of capturing the few remaining fish exceeded the
returns, so that it became unprofitable to continue.4 Thus, while extinction may be avoided,
the fishery frequently results in a lower-than-optimal (and perhaps severely depleted) fish population and an overinvestment in
fishing effort.

Magnuson-Stevens Act solves overfishing.


Pacific Fishery Management Council 10 Applicable Laws: Magnuson-Stevens Act. Pacific Fishery
Management Council. 08/12/2010 http://www.pcouncil.org/resources/applicable-laws/magnuson-stevens-act/

Background¶ The Magnuson-Stevens Fishery Conservation and Management Act is the principal law
governing marine fisheries in the United States. It was originally adopted to extend control of U.S. waters to 200 nautical miles in the
ocean; to
phase out foreign fishing activities within this zone; to prevent overfishing, especially by foreign fleets;
to allow overfished stocks to recover; and to conserve and manage fishery resources. The Act is
named after the late Senators Warren Magnuson of Washington and Ted Stevens of Alaska. ¶ Congress passed the original Magnuson
Act in 1976. It has since been amended several times, most recently in 2006. Among other things, the Act explains the role of
regional fishery management councils and describes their functions and operating procedures. The
Act includes national
standards for management and outlines the contents of fishery management plans. In addition, it
gives the Secretary of Commerce power to review, approve, and implement fishery management plans and other recommendations
developed by the councils. National Marine Fisheries Service (under the Department of Commerce) is charged with stewardship of
the nation’s living marine resources. With input from the regional councils and stakeholder groups, National Marine Fisheries Service
provides guidance for applying the National Standards of the Act.¶ Top¶ Revisions to the Act¶ In 1996, Congress passed the
Sustainable Fisheries Act (SFA), which revised the Magnuson Act and reauthorized it through 1999. This revision brought new
requirements to prevent overfishing and rebuild overfished fisheries . The law now required that each fishery
management plan (FMP) specify objective and measurable criteria for determining when a stock is overfished or when overfishing is
occurring, and to establish measures for rebuilding the stock. The SFA also added several new definitions, including definitions for
“overfishing” and “overfished”, and for fishing communities.¶ The SFA also added three new National Standards to address fishing
vessel safety, fishing communities, and bycatch. Several existing standards were revised. The MSA now contains ten National
Standards for fishery conservation and management, with which all FMPs must comply. ¶ In late 2006, Congress revised and
reauthorized the Act again. This revision (called “Fishery Conservation and Management Amendments of 2006”) did not add any
National Standards, but made a number of changes related to establishment of annual catch limits, function of the Scientific and
Statistical Committee, the environmental review (NEPA) process, rebuilding provisions, limited access privilege programs, and other
areas. The Act is now reauthorized through 2010.¶ The Magnuson Act is complemented by other federal and state laws, including
the Marine Mammal Protection Act, the Endangered Species Act, the Coastal Zone Management Act, and the National Marine
Sanctuaries Act. International agreements and organizations, such as the International Convention for the Conservation of Atlantic
Tunas, Inter-American Tropical Tuna Commission, and the United Nation’s Code of Conduct for Responsible Fisheries, also play a role
in shaping management of U.S. fisheries.¶ Top¶ MSA Downloads and Websites¶ Magnuson-Stevens Fishery Conservation and
Management Act As Amended Through January 12, 2007¶ Magnuson-Stevens Reauthorization Act of 2006¶ Final MSA
Reauthorization Bill¶ Top¶ National Standards of the Magnuson-Stevens Act (a summary)¶ Conservation and
management measures shall:¶ Prevent overfishing while achieving optimum yield.¶ Be based upon the best scientific information
available.¶ Manage individual stocks as a unit throughout their range, to the extent practicable; interrelated stocks shall be managed
as a unit or in close coordination.¶ Not discriminate between residents of different states; any allocation of privileges must be fair
and equitable.¶ Where practicable, promote efficiency, except that no such measure shall have economic allocation as its sole
purpose.¶ Take into account and allow for variations among and contingencies in fisheries, fishery resources, and catches. ¶ Minimize
costs and avoid duplications, where practicable.¶ Take
into account the importance of fishery resources to
fishing communities to provide for the sustained participation of, and minimize adverse
impacts to, such communities (consistent with conservation requirements).¶ Minimize bycatch or mortality from
bycatch.¶ Promote safety of human life at sea.
Food Shortage Answers
Frontline
Food shortages won’t cause war

Allouche 11, research Fellow – water supply and sanitation @ Institute for Development Studies, frmr professor – MIT
(Jeremy, “The sustainability and resilience of global water and food systems: Political analysis of the interplay between security,
resource scarcity, political systems and global trade,” Food Policy, Vol. 36 Supplement 1, p. S3-S8, January)

The question of resource scarcity has led to many debates


on whether scarcity (whether of food or water) will
lead to conflict and war. The underlining reasoning behind most of these discourses over food and water wars comes
from the Malthusian belief that there is an imbalance between the economic availability of natural
resources and population growth since while food production grows linearly, population increases exponentially.
Following this reasoning, neo-Malthusians claim that finite natural resources place a strict limit on the growth of human population
and aggregate consumption; if these limits are exceeded, social breakdown, conflict and wars result. Nonetheless, it seems that
most empirical studies do not support any of these neo-Malthusian arguments. Technological change and
greater inputs of capital have dramatically increased labour productivity in agriculture. More generally, the
neo-Malthusian view has suffered because during the last two centuries humankind has breached many
resource barriers that seemed unchallengeable. Lessons from history: alarmist scenarios, resource wars and
international relations In a so-called age of uncertainty, a number of alarmist scenarios have linked the increasing use of
water resources and food insecurity with wars. The idea of water wars (perhaps more than food wars) is a
dominant discourse in the media (see for example Smith, 2009), NGOs (International Alert, 2007) and within international
organizations (UNEP, 2007). In 2007, UN Secretary General Ban Ki-moon declared that ‘water scarcity threatens economic and social
gains and is a potent fuel for wars and conflict’ (Lewis, 2007). Of course, this type of discourse has an instrumental purpose; security
and conflict are here used for raising water/food as key policy priorities at the international level. In the Middle East,
presidents, prime ministers and foreign ministers have also used this bellicose rhetoric. Boutrous Boutros-Gali
said; ‘the next war in the Middle East will be over water, not politics’ (Boutros Boutros-Gali in Butts, 1997, p. 65). The question is not
whether the sharing of transboundary water sparks political tension and alarmist declaration, but rather to what extent water has
been a principal factor in international conflicts. The evidence seems quite weak. Whether by president Sadat in Egypt
or King Hussein in Jordan, none of these declarations have been followed up by military action. The
governance of transboundary water has gained increased attention these last decades. This has a direct impact on the global food
system as water allocation agreements determine the amount of water that can used for irrigated agriculture. The likelihood of
conflicts over water is an important parameter to consider in assessing the stability, sustainability and resilience of global food
systems. None of the various and extensive databases on the causes of war show water as a
casus belli. Using the International Crisis Behavior (ICB) data set and supplementary data from the University of Alabama on
water conflicts, Hewitt, Wolf and Hammer found only seven disputes where water seems to have
been at least a partial cause for conflict (Wolf, 1998, p. 251). In fact, about 80% of the incidents relating to
water were limited purely to governmental rhetoric intended for the electorate (Otchet, 2001, p. 18). As shown in
The Basins At Risk (BAR) water event database, more than two-thirds of over 1800 water-related ‘events’ fall on
the ‘cooperative’ scale (Yoffe et al., 2003). Indeed, if one takes into account a much longer period, the following figures
clearly demonstrate this argument. According to studies by the United Nations Food and Agriculture Organization (FAO),
organized political bodies signed between the year 805 and 1984 more than 3600 water-related
treaties, and approximately 300 treaties dealing with water management or allocations in international basins have been
negotiated since 1945 (FAO, 1978 and FAO, 1984). The fear around water wars have been driven by a Malthusian outlook which
equates scarcity with violence, conflict and war. There is however no direct correlation between water scarcity and
transboundary conflict. Most specialists now tend to agree that the major issue is not scarcity per se but rather the allocation
of water resources between the different riparian states (see for example Allouche, 2005, Allouche, 2007 and [Rouyer, 2000] ).
Water rich countries have been involved in a number of disputes with other relatively water rich
countries (see for example India/Pakistan or Brazil/Argentina). The perception of each state’s estimated water needs really
constitutes the core issue in transboundary water relations. Indeed, whether this scarcity exists or not in reality, perceptions of
the amount of available water shapes people’s attitude towards the environment (Ohlsson, 1999). In fact, some water
experts have argued that scarcity drives the process of co-operation among riparians (Dinar and Dinar, 2005 and
Brochmann and Gleditsch, 2006). In terms of international relations, the threat of water wars due to increasing scarcity does
not make much sense in the light of the recent historical record. Overall, the water war rationale expects conflict to
occur over water, and appears to suggest that violence is a viable means of securing national water supplies, an argument which is
highly contestable. The debates over the likely impacts of climate change have again popularised the idea of
water wars. The argument runs that climate change will precipitate worsening ecological conditions contributing to resource
scarcities, social breakdown, institutional failure, mass migrations and in turn cause greater political instability and conflict (Brauch,
2002 and Pervis and Busby, 2004). In a report for the US Department of Defense, Schwartz and Randall (2003) speculate about the
consequences of a worst-case climate change scenario arguing that water shortages will lead to aggressive wars (Schwartz and
Randall, 2003, p. 15). Despite
growing concern that climate change will lead to instability and violent
conflict, the evidence base to substantiate the connections is thin ( [Barnett and Adger, 2007] and Kevane
and Gray, 2008).

Double bind – either no food scarcity, or there are tons of alt causes

Adeline 13 – food preservation specialist (02/17, “Food Storage: The Solution to Food Shortage,”
http://beforeitsnews.com/survival/2013/02/food-storage-the-solution-to-food-shortage-2462712.html)

We humans need food for proper nutrition. In times of crisis, access to a stable food supply is the key to continued survival. In
the
presence of natural disasters, human conflicts, climate change, and overpopulation , the threat
of food shortages and total famine is not as far-fetched as it seems. Preparing long-term food supplies can buffer the
effects of these potential catastrophes. The Anatomy of a Catastrophe Many people think that food shortages may be a thing of the
past. Thanks to the marvels of modern technology and scientific farming methods, we have a constant and abundant
supply of food. It is difficult to imagine how a food shortage can happen – but it is still a possibility. During ancient times,
humans hunted and gathered for food. Then a revolution occurred and changed the course of history: we learned how to cultivate
the soil, plant crops, and domesticate animals for a stable source of food. Eventually, modern
technology has improved
farming and fishing techniques, that food production has now become large-scale. People now depend on
hard-working farmers and fishermen for their everyday supply of food. But what happens when the harvests are poor? The farmer
will keep his produce to feed his own family first – other people are left without food. Such scenario is still possible today because
there are man-made disasters and natural calamities that threaten the world’s food supply. For example, a hurricane rages across
the country and floods several states. Our access to food is restricted because travel is nearly impossible. To add to that, business
establishments like groceries and supermarkets are probably closed down due to the flood as well as a power outage. Some might
argue that this is not a real food shortage scenario because the problem is merely logistics: there is food; it is just that we have no
access to it. It is true that natural calamities and wars cause a food shortage only temporarily. However, recent studies show
that at present, we
consume more than we produce. The UN warns that grain reserves are progressively
getting lower because of droughts and crop failures in major food producing countries. The famine in
Africa may possibly be felt in other parts of the world. This is an emergency situation that requires us to
prepare beforehand. We must have a supply of food and water for us to survive.
---Food- Alt Causes

Tons of alt causes to food scarcity – natural disasters, human conflicts, warming,
overpopulation, droughts, crop failures, African famine, and the fact that people consume
more than they produce – that’s Adeline

Alt causes swamp the internal link – new report, inefficient harvesting, handling,
transportation, and infrastructure, unsuitable storage conditions, millions of tons of wasted
food, competition for land w/ ecosystem preservation and biomass, rising population, improved
nutrition standards, shifting dietary preferences

ENS 13 – Environment News Service, cites the Institution of Mechanical Engineers (01/11, “Up to Half of Global Food Production
Wasted,” http://ens-newswire.com/2013/01/11/up-to-half-of-global-food-production-wasted/)

LONDON, UK, January 11, 2013 (ENS) – Four billion tonnes of food a year are produced globally. Yet, due to poor practices in
harvesting, storage and transportation, as well as market and consumer wastage, between 30 and 50 percent of all
food produced each year never feeds a human being, finds a new report by the Institution of Mechanical
Engineers, a registered British charity. “This level of wastage is a tragedy that cannot continue if we are to succeed in
the challenge of sustainably meeting our future food demands,” the Institution of Mechanical Engineers said in a
statement Thursday releasing the report, “Global Food: Waste Not Want Not.” “The amount of food wasted and lost around the
world is staggering,” said Dr. Tim Fox, who heads the Energy and Environment Division at the Institution. “This is food that could be
used to feed the world’s growing population – as well as those in hunger today. It is also an unnecessary waste of the land, water
and energy resources that were used in the production, processing and distribution of this food,” said Fox. The reasons for this
situation range from poor engineering and agricultural practices, inadequate transport and storage infrastructure through to
supermarkets demanding cosmetically perfect foodstuffs and encouraging consumers to overbuy through buy-one-get-one free
offers. In less-developed countries, such as those of sub-Saharan Africa and Southeast Asia, wastage tends to occur primarily at the
farmer-producer end of the supply chain. Inefficient
harvesting, inadequate local transportation and poor
infrastructure mean that produce is frequently handled inappropriately and stored under
unsuitable farm site conditions. In India, the report finds, 21 million tonnes of wheat is wasted each year due to inadequate
storage and distribution systems. In mature, fully developed countries, more-efficient farming practices and better transport,
storage and processing facilities ensure that a larger proportion of the food produced reaches markets and consumers. Yet,
consumer cultural standards dictate that produce is often wasted through retail and customer behavior. Major supermarkets, in
meeting consumer expectations, will often reject entire crops of perfectly edible fruit and vegetables at the farm because they do
not meet exacting marketing standards for their physical characteristics, such as size and appearance. For example, up to 30 percent
of the UK’s vegetable crop is never harvested as a result of such practices. Globally, retailers generate 1.6 million
tonnes of food waste annually in this way. Of the produce that does appear in the markets, commonly used sales
promotions encourage customers to purchase excessive quantities which, in the case of perishable foodstuffs, generate wastage in
the home. Overall between 30 percent and 50 percent of what has been bought in developed countries is thrown away by the
purchasers. In the UK, the report estimates that seven million tonnes of food valued at about £10 billion is thrown
away from homes every year. Global food production now takes up about 50 percent of the available suitable land, the
report estimates, adding that the amount of land used for towns and cities is relatively small and, despite future growth of urban
areas, is “unlikely to become significant in proportional terms.” Land used for producing wasted food reduces the land available to
grow renewable energy crops or to support natural resources such as forests and wildlife, the report points out. “ Considerable
tensions are likely to emerge as competition develops for use of available land between the need
for food production, demands for preservation of ecosystems and the desire to produce biomass as a source
of renewable energy.” “As water, land and energy resources come under increasing pressure from competing human demands,
engineers have a crucial role to play in preventing food loss and waste by developing more efficient ways of growing, transporting
and storing foods,” said Dr. Fox. “But in order for this to happen governments, development agencies and organizations like the UN
must work together to help change peoples’ mindsets on waste and discourage wasteful practices by farmers, food producers,
supermarkets and consumers,” he said. Today, the world population stands at approximately 7,059,016,541 people. By 2075, the
United Nations’ mid-range projection for global population growth predicts that human numbers will peak at about 9.5 billion
people. Rising
population combined with improved nutrition standards and shifting dietary
preferences will exert pressure for increases in global food supply.
---Food- No Scarcity

No food scarcity – new farming and fishing tech solve – that’s Adeline

It won’t happen – global production has doubled, tech means production will meet demand,
shortage fears unfounded, production growing faster than population

Jalsevac 4 (Paul, Life site news a division of Interim Publishing, “The Inherent Racism of Population Control”,
http://www.lifesite.net/waronfamily/Population_Control/Inherentracism.pdf )

The pattern continues today. Economist Dennis Avery explained in 1995 that, food production was more than keeping pace with
population growth since the world had, “more than doubled world food output in the past 30 years. We
have raised food supplies per person by 25 percent in the populous Third World.”4 The United Nations Food
and Agriculture Organization (UNFAO) also dispelled fears of shortages in the food supply when, in preparation for the World Food
Summit in Rome in November of 1995 it reported that, “Globally food supplies have more than doubled in the last 40 years…at a
global level,
there is probably no obstacle to food production rising to meet demand.”5 The UNFAO also later
estimated that, simply with the present available technologies fully employed, the world could feed 30 to 35
billion people, i.e. roughly six times the present world population.6 It also reported that the number of people considered
malnourished has declined from 36 percent in 1961-1970 to 20 percent in 1988-90 and later proclaimed that “earli er fears of
chronic food shortages over much of the world proved unfounded.”7 The World Bank joined in to predict in 1993 that
the improvement in the world food supply would continue, while pointing out that in developing countries grain
production has grown at a faster rate than population since 1985. Grain production has slowed in the United
States, but that is because stocks have grown so large that additional production could not be stored.8 A further wealth of evidence
is available to remove any concerns about resource shortage in the modern world.

No shortages – food is abundant

Poole 6 (Holly Kavana, Institute for Food and Development Policy, “12 Myths About Hunger”, Backgrounder, 12(2), Summer, 4-9,
http://www.foodfirst.org/12myths)

Myth 1: Not Enough Food to Go Around Reality: Abundance, not scarcity, best describes the
world's food supply . Enough wheat, rice and other grains are produced to provide every
human being with 3,200 calories a day. That doesn't even count many other commonly eaten foods - vegetables,
beans, nuts, root crops, fruits, grass-fed meats, and fish. Enough food is available to provide at least 4.3 pounds of food per
person a day worldwide: two and half pounds of grain, beans and nuts, about a pound of fruits and vegetables, and nearly
another pound of meat, milk and eggs - enough to make most people fat! The
problem is that many people are too
poor to buy readily available food. Even most "hungry countries" have enough food for all
their people right now. Many are net exporters of food and other agricultural products.
---Food- No Wars

Food shortages won’t cause war – they don’t have the majority of empirical support – most
scarcity has lead to cooperation – tech and capital inputs have boosted productivity – that’s
Allouche

Empirics disprove food wars – no empirical support, Malawi food shortages, 2004 Asian
tsunami

Salehyan 7 (Idean, Professor of Political Science – University of North Texas, “The New Myth About Climate Change”, Foreign
Policy, Summer, http://www.foreignpolicy.com/story/cms.php?story_id=3922)

First, aside from a few anecdotes, there is little systematic empirical evidence that
resource scarcity and changing environmental conditions lead to conflict. In fact, several
studies have shown that an abundance of natural resources is more likely to contribute
to conflict. Moreover, even as the planet has warmed, the number of civil wars and
insurgencies has decreased dramatically. Data collected by researchers at Uppsala
University and the International Peace Research Institute, Oslo shows a steep decline in
the number of armed conflicts around the world. Between 1989 and 2002, some 100
armed conflicts came to an end, including the wars in Mozambique, Nicaragua, and
Cambodia. If global warming causes conflict, we should not be witnessing this downward
trend. Furthermore, if famine and drought led to the crisis in Darfur, why have scores of
environmental catastrophes failed to set off armed conflict elsewhere? For instance, the
U.N. World Food Programme warns that 5 million people in Malawi have been
experiencing chronic food shortages for several years. But famine-wracked Malawi has
yet to experience a major civil war. Similarly, the Asian tsunami in 2004 killed hundreds
of thousands of people, generated millions of environmental refugees, and led to severe
shortages of shelter, food, clean water, and electricity. Yet the tsunami, one of the most
extreme catastrophes in recent history, did not lead to an outbreak of resource wars.
Clearly then, there is much more to armed conflict than resource scarcity and natural
disasters.

Democracy solves the impact – democratic governments have empirically contained food
conflicts

Salehyan 7 (Idean, Professor of Political Science – University of North Texas, “The New Myth About Climate Change”, Foreign
Policy, Summer, http://www.foreignpolicy.com/story/cms.php?story_id=3922)

To be sure, resource scarcity and environmental degradation can lead to social frictions.
Responsible, accountable governments, however, can prevent local squabbles from
spiraling into broader violence, while mitigating the risk of some severe environmental
calamities. As Nobel laureate Amartya Sen has observed, no democracy has ever
experienced a famine. Politicians who fear the wrath of voters usually do their utmost to
prevent foreseeable disasters and food shortages. Accountable leaders are also better at
providing public goods such as clean air and water to their citizens.

Famine doesn’t cause war ---- it makes people too hungry to fight

Barnett ’00 (Jon, Australian Research Council fellow and Senior Lecturer in Development Studies @ Melbourne U. School of
Social and Environmental Enquiry, Review of International Studies, “Destabilizing the environment-conflict Thesis”, 26:271-288,
Cambridge Journals Online)

Considerable attention has been paid to the links between population, the environment and conflict. The standard
argument is that population growth will overextend the natural resources of the immediate environs, leading to
deprivation which, it is assumed, will lead to conflict and instability either directly through competition for scarce
resources, or indirectly through the generation of ‘environmental refugees’. For example, according to Myers: ‘so
great are the stresses generated by too many people making too many demands on their natural-resource stocks and
their institutional support systems, that the pressures often create first-rate breeding grounds for conflict’.37 The
ways in which population growth leads to environmental degradation are reasonably well known. However, the
particular ways in which this leads to conflict are difficult to prove. In the absence of proof there is a negative style of
argumentation, and there are blanket assertions and abrogations; for example: ‘the relationship is rarely causative in
a direct fashion’, but ‘we may surmise that conflict would not arise so readily, nor would it prove so acute, if the
It is possible though, that
associated factor of population growth were occurring at a more manageable rate’.38
rather than inducing warfare, overpopulation and famine reduce the capacity of a people to
wage war. Indeed, it is less the case that famines in Africa in recent decades have produced ‘first
rate breeding grounds for conflict’; the more important, pressing , and avoidable product is
widespread malnutrition and large loss of life.

Food shocks are self-correcting

Kharas 8 (Homi, The Economist Debate: Rising food prices, The Proposition’s closing statement,
http://www.economist.com/debate/index.cfm?action=article&debate_id=10&story_id=11829068)

Images of food riots and hungry people stir deep emotions. But we must debate trade-offs: will the rise in food prices generate
more food for the world and less poverty for poor people in the future? Are today’s food prices fair to producers and
consumers? Yes, because higher food prices will bring about new investments in agriculture and
higher global production. This is already happening in Asia and other parts of the world, and will accelerate
over time. Yes, because without higher food prices, land use would shift towards corn-for-ethanol
and other biofuel crops and we would have less food available . Yes, because a system with food prices
in free fall for 30 years did not produce any measurable decline in hunger and poverty. But
the last time food prices were as high as they are today we witnessed the Green
Revolution and a rapid reduction of rural poverty in one of the largest population centres of the world,
South Asia.
GMO’s will solve

Cage 8 (Sam, Reporter for Reuters, “High food prices may cut opposition to genetically modified food,” 7-8,
http://www.iht.com/articles/2008/07/08/business/gmo.php)

Like many stores in Europe, the Coop chain of supermarkets in Switzerland does not specify whether goods are genetically
modified - because none are. But a
wave of food-price inflation may help wash away popular
opposition to so-called Frankenstein foods. "I think there's a lot of resistance in Switzerland," said a shopper,
Beatrice Hochuli, as she picked out a salad for dinner at a bustling supermarket outside the main Zurich station. "Most people in
Switzerland are quite against it." Consumers, even those from relatively wealthy parts of the world, are rarely first in line to
adopt new technologies. Although food prices are up more than 50 percent since May 2006, according to the UN Food and
Agriculture Organization's Food Price Index, Europeans remain wary of foods derived from tinkering with the genetic makeup of
plants. But policy
makers and food companies are pressing the genetic modification topic in a
bid to temper aversion to biotech crops like pesticide-resistant rapeseed for oils and "Roundup-ready"
soybeans, which tolerate dousing of the Roundup herbicide. These are crops already common in the United States and other
major food exporters like Argentina and Brazil. The European Commission has said that it believes
biotech crops can alleviate the current crisis in food supply, although it added in June that expediency
should not overrule strict scientific scrutiny of the use of the technology involved. The chairman of Nestlé, the
world's biggest food group, has said it is impossible to feed the world without genetically
modified organisms. Meanwhile, the British government's former chief scientific adviser,
David King, has said over the past week that genetically modified crops hold the key to
solving the world's food crisis. He called in a Financial Times interview for a "third green
revolution," in reference to two waves of innovation that helped increase crop yields sharply in Asia over the past 50 years.
Climate change and increasing concern about fresh water supplies are helping to fuel interest in new
seed varieties likely to be more resistant to drought and able to produce reasonable yields
with significantly less water.

No famine – the poorest are insulated from global markets

Paarlberg 8 (Robert, Professor of Political Science – Wellesley College, “It's Not the Price that Causes Hunger”, The International
Herald Tribune, 4-23, Lexis)

International prices of rice, wheat and corn have risen sharply, setting off violent urban protests in roughly a dozen countries in
Asia, Africa and Latin America. But is this a ''world food crisis?'' It is certainly a troubling instance of price instability in
international commodity markets, leading to social unrest among urban food-buyers. But we
must be careful not to
equate high crop prices with hunger around the world. Most of the world's hungry people do
not use international food markets, and most of those who use these markets are not
hungry. International food markets, like international markets for everything else, are used primarily by
the prosperous and secure, not the poor and vulnerable. In world corn markets, the biggest importer by far is Japan.
Next comes the European Union. Next comes South Korea. Citizens in these countries are not underfed. In the poor countries of
Asia, rice is the most important staple , yet most Asian countries import very little rice. As recently as March , India was keeping
imported rice out of the country by imposing a 70 percent duty. Data on the actual incidence of malnutrition reveal that the
regions of the world where people are most hungry, in South Asia and Sub-Saharan Africa, are those
that depend least on imports from the world market. Hunger is caused in these countries not by high
international food prices, but by local conditions , especially rural poverty linked to low productivity in
farming. When international prices are go up, the disposable income of some import-dependent urban
dwellers is squeezed. But most of the actual hunger takes place in the villages and in the countryside ,
and it persists even when international prices are low. When hunger is measured as a balanced index of
calorie deficiency, prevalence of underweight children and mortality rates for children under five, we find that South Asia and
sub-Saharan Africa in 2007 had hunger levels two times as high as in the developing countries of East Asia, four times as high as
in Latin America, North Africa or the Middle East, and five times as high as in Eastern Europe and Central Asia. The poor in South
Asia and sub-Saharan Africa are hungry even though their connections to high-priced international food markets are quite weak.
In the poorest developing countries of Asia, where nearly 400 million people are hungry, international grain prices are hardly a
factor, since imports supply only 4 percent of total consumption - even when world prices are low. Similarly in sub-Saharan
Africa, only about 16 percent of grain supplies have recently been imported, going mostly into the more prosperous cities rather
than the impoverished countryside, with part arriving in the form of donated food aid rather than commercial purchases at
world prices. The region in Africa that depends on world markets most heavily is North Africa, where 50 percent of grain
supplies are imported. Yet food consumption in North Africa is so high (average per capita energy consumption there is about
3,000 calories per day, comparable to most rich countries) that increased import prices may cause economic stress for urban
consumers (and perhaps even street demonstrations) but little real hunger. Import dependence is also high in Latin America (50
percent for some countries) but again high world prices will not mean large numbers of hungry people, because per capita GDP
in this region is five times higher than in sub-Saharan Africa. There is a severe food crisis among the poor in South Asia and sub-
Saharan Africa, but it does not come from high world prices. Even in 2005 in sub-Saharan Africa, a year of low international crop
prices, 23 out of 37 countries in the region consumed less than their nutritional requirements. Africa's food crisis grows
primarily out of the low productivity, year in and year out, of the 60 percent of all Africans who plant crops and graze animals
for a living. The average African smallholder farmer is a woman who has no improved seeds, no nitrogen fertilizers, no irrigation
and no veterinary medicine for her animals. Her crop yields are only one third as high as in the developing countries of Asia, and
her average income is only $1 a day.
---Food- Technology Solves
New tech means farmers will always produce more food
Zubrin 11 (Dr. Robert Zubrin, president of Pioneer Astronautics, Senior Fellow with the Center
for Security Policy “WHY IT’S WRONG TO AGREE WITH THE MALTHUSIANS ABOUT ETHANOL”
May 13, 2011http://www.ilcorn.org/daily-update/182-why-it-rsquo-s-wrong-to-agree-with-the-
malthusians-about-ethanol/)// CG
In an op-ed article printed in the Denver Post May 8, editorial columnist Vince Carroll endorsed the view of population control
advocate Lester Brown that the U.S. corn ethanol program is threatening the world’s poor with starvation. This endorsement is
especially remarkable in view of the fact that, as the otherwise generally astute Mr. Carroll has correctly noted many times in the
past, all of Lester Brown’s many previous limited-resources doomsday predictions have proven
wildly incorrect. In fact, Lester Brown is wrong about the alleged famine-inducing potential of the ethanol program for
exactly the same reason he has been repeatedly wrong about the alleged famine-inducing potential of population growth. There
is not a fixed amount of grain in the world. Farmers produce in response to demand.
The more customers, the more grain. Not only that, but the larger the potential market, the
greater the motivation for investment in improved techniques . This is why, despite the
fact that the world population has indeed doubled since Lester Brown, Paul Ehrlich,
and the other population control zealots first published their manifestos during the
1960s, people worldwide are eating much better today than they were then. In the case of
America’s corn growing industry, the beneficial effect of a growing market has been especially pronounced, with corn yields per acre
in 2010 (165 bushels per acre) being 37 percent higher than they were in 2002 (120 bushels per acres) and more than four times as
great as they were in 1960 (40 bushels per acre.) Not
only that, but in part because of the impetus of
the expanded ethanol program, another doubling of yield is now in sight, as the best
farms have pushed yields above 300 bushels per acre. As a result, in 2010, the state of Iowa alone
produced more corn than the entire United States did in 1947. Of our entire corn crop, only 2 percent is actually eaten by Americans
as corn, or 12 percent if one includes products like corn chips and corn syrup. These advances in productivity do not only benefit the
United States. America’s farmers are the vanguard for their counterparts worldwide. New
seed strains and other
techniques first demonstrated on our most advanced farms, subsequently spread to
average farms, and then go global, thereby raising crop yields everywhere.
Food Prices Answers
Frontline

Tons of alt causes to food prices – extreme weather, long-term supply/demand crunch,
population growth, speculative “futures” investing, disease, less available land, feedstock
competition from the biofuels industry, warming

Wall 13 – personal finance reporter for the Daily and Sunday Telegraph and Telegraph.co.uk, citing Baring Asset Management
(Emma, 03/02, “As the price of food rises, is there profit to be made?”
http://www.telegraph.co.uk/finance/personalfinance/investing/9902374/As-the-price-of-food-rises-is-there-profit-to-be-
made.html)

Noticed the price of sugar lately? Potatoes? Fresh fruit? A weak pound, US drought and one of the wettest years on record for
Britain have all contributed to the cost of your shopping basket soaring. When sterling falls, your money buys fewer of the
commodities that need to be imported. In fact, the recent sharp falls probably haven't had their full impact in yet. But the story of
rising food prices is about much more than currency swings. And it provides tempting possibilities for investors. Extreme
weather in 2012 led to sharp price rises in the likes of corn, wheat and soya beans, and these prices look
likely to remain elevated for the next six months, according to Baring Asset Management. There is also a
long-term crunch between supply and demand. In fact, food production must increase by at least 70pc by 2050 to
meet global demand. According to the United Nations, the world's population is forecast to increase from 7 billion
to 9.3 billion over the next 40 years, and to meet this demand investment in food production is needed. The
Ecclesiastical Investment Management Amity Insight report Hungry Planet warned that our current food supply is just
not sustainable in the medium to long term, and can only be solved through extensive investment in global agriculture, which
will help increase crop yields. Neville White, socially responsible investment analyst at Ecclesiastical, said: "Increased food
production will have to be achieved with less land, water and people. Investing in companies with a focus on mechanisation, crop
production and fertilisation that aim to increase food production can not only have a real impact on food but can also ensure that
investors profit with principles." There are two ways to invest in food: you can buy commodities through trading on the future price
of a grain or crop and/or buying an exchange-traded fund. Or you can buy shares in agriculture and food-production related
companies. Sarasin AgriSar invests in the entire supply chain, from grain to supermarkets. This means that although you may miss
out on large upsurges in the soft commodities market, growth should be smoother. Henry Boucher, manager of the AgriSar fund,
said that holding food-related shares was a more ethical way of investing – handing your money to companies in the chain reduces
their capital cost and helps them invest to improve food supply. "Some speculators invest in food itself, which takes supply out of
the market [if they store it for later sale at higher prices]," said Mr Boucher. "Commodity traders invest directly in corn, pork bellies,
wheat and sugar. We're more interested in finding companies that help improve global productivity." He cites investments like
Japan's Kubota, which makes small rice transplanters, or Indian company Syngenta, whose fertiliser and seed pre-mix is designed to
improve productivity by up to four times. "Both make products for the small farmer – they can be used on land as little as one acre,"
he said. "This is not about mass-farming but helping the small businesses left in the Asian countryside." Speculative
"futures" investing in food markets can also be more volatile. Futures are short-term punts – one bad
crop season, due to disease or extreme weather, may mean significant losses. Agriculture-related shares
are held for longer and are less affected by natural disasters. Jonathan Blake, manager of the Baring Global Agriculture fund, said last
year's weather had enhanced the investment appeal of those companies providing the likes of seeds, herbicides and fertilisers,
which will enable farmers to maximise their crop output. "It will take time to address the shortfalls caused by the severe weather
events of 2012, from droughts in America to washout conditions in the UK and Europe," said Mr Blake. "We do, however, expect
crop prices to come down later in 2013, providing we have a year of 'normal' weather, as significantly improved output will allow
inventory levels to begin to be rebuilt." The Baring fund has a sizeable proportion of listed fertiliser, herbicide and seed producers.
"Crop production, through the continuous cycle of planting, growing and harvesting, robs the soil of nutrients," said Mr Blake. "As a
result, these nutrients need replenishing through the application of fertilisers. Additionally, for many farmers these nutrients are
highly affordable given the current high prices farmers are able to get for their crops." Schroders Climate Change manager Simon
Webber also likes investing in companies that offer productivity solutions which will help bring down the price of food through use
of their products to increase farming production. He also invests in Syngenta and US company Trimble Navigation, which provides
solutions for levelling fields. It is not just population growth that provides investment opportunities in the food sector, but the
change in global diets. As disposable incomes swell in emerging markets, diets tend to become more Westernised. The AgriSar fund
invests in Asian supermarket chain Dairy Farm, whose revenues have risen as the expanding middle classes change their dietary and
shopping habits. "Incomes in China are increasing at 10pc a year," said Mr Boucher. "People are no longer going to the market daily
but visit a supermarket once a week, where they will be buying more meat, dairy products and imported vegetables." Mr Webber
said that on top of the global demand for more agricultural produce are the effects on supply, where available productive land
is in decline, yield growth is reducing and there is a growing competition from the biofuel industry for
feedstock. "Climate change acts as a threat multiplier to the sector on top of the dual impacts of
increased demand and decreased supply, presenting various investment opportunities. The sectors that will benefit
from this are companies involved in agricultural production as well as food retailers, whose share price will increase as food prices
inflate."

Population growth alone swamps the internal link – so does ag slowdown – USDA forecast, 70-
100% increase in demand by 2050

Johnson 13 – writer for the Council on Foreign Relations (Toni, 01/16, “Food Price Volatility and Insecurity,”
http://www.cfr.org/food-security/food-price-volatility-insecurity/p16662)

The Global Food Market Just fifteen food crops make up 90 percent of the world's energy intake, according to the UN Food and
Agriculture Organization (FAO), with rice, maize (corn), and wheat comprising two-thirds of that number. The world grows more
grains (PDF)--also known as cereals--than any other crop type. Much of the global increase in food prices stems from staple grains,
which in some countries can represent more than half of calorie intake. According to the World Bank, due to an incredibly dry
summer in the United States and Europe, global corn and soybean prices reached all-time highs in July 2012, while wheat soared to
prices comparable to 2011 peaks. Because grains also represent a major food source for livestock, higher grain prices have
contributed to higher dairy and meat prices. The USDA predicts that domestically, prices will continue to rise in
2013 at a rate of 3 to 4 percent. A June 2011 report to G20 agriculture ministers from ten major NGOs, including the World Trade
Organization, the World Bank, and the UN World Food Program, noted that by 2050, food demand (PDF) will have
increased by between 70 percent and 100 percent to meet a projected population growth of at least 2.5
billion additional people. "This alone is sufficient to exert pressure on commodity prices," the report said.
Growth in agriculture production is largely expected to come from increased crop yields and will primarily be located in developing
countries, according to a 2009 UN report (PDF). Experts say there is plenty of opportunity to improve farming techniques in the
developing world. Meeting projected demand will require increasing cereal production by an additional one billion tons, up from
more than two billion tons currently, and more than doubling meat production from current levels. However, according to a 2011
report by the OECD, annual growthin agriculture production (PDF) in the next decade is forecast to be a third
less than the annual growth in the previous decade. The report estimates that a 5 percent increase or decrease in
harvest yield in major grains can lead to as much as a 25 percent difference in price. Food Price Volatility According to the FAO, price
volatility has been extremely rare in agricultural markets, but the
global food system is becoming increasingly
vulnerable to it. The 2011 NGO report argued that "volatility becomes an issue for concern and for possible policy response
when it induces risk-adverse behavior that leads to inefficient investment decisions and when it creates problems that are beyond
the capacity of producers, consumers, or nations to cope."
German Econ Answers
Frontline

German economy resilient – strong industrial core and work agreements

New York Times 10 (12/24/10, Michael Slackman, “Bavaria Boom, but Germans Feel Economic
Pressure,” http://www.nytimes.com/2010/12/25/world/europe/25germany.html)

By nearly every measure this industrial region in Bavaria, home to exporting giants like Siemens
Healthcare, Adidas and Puma, is at the center of Germany’s resilient economic success, driving
the nation’s growth even as much of Europe struggles to stay solvent . With an unemployment rate of 3.5
percent and Christmas markets full of shoppers and revelers, “Bavaria is the planet of the happy ,” said Katrin Schmidt,
a labor analyst with the Federal Employment Agency based in Nuremberg. Except that many people in this region, and around the
nation, seem subdued and uncertain about their own financial health. Germany’s good fortune, however conspicuous to other
Europeans, is widely viewed here as having come at the expense of its workers, who for the past decade have sacrificed wages and
benefits to make their employers more competitive. “I don’t believe that the economic situation really improved,” said Karl-Hans
Diem, a 60-year-old metal worker. Thisdisconnect — bullish economic indicators combined with
simmering discontent even among the employed — is at the core of the conflict between
Germany and some of its euro zone allies over how to handle the continuing European financial
crisis. German leaders have been confronted with the opposing forces of a strong economy and a work force that has seen its real
wages, adjusted for inflation, decline over the past decade. When Greece was in crisis, Chancellor Angela Merkel was criticized from
abroad for moving too slowly to help reassure markets that it stood behind the euro, and at home for agreeing to dole out too much
aid. With the more recent Irish crisis, she came under attack around Europe for demanding that private investors share the burden
of future bailouts, while at home for not insisting on that kind of accountability sooner. “They should better spend the money here;
we have problems of our own,” said Sabine Striegel, 45, who is living with her parents in nearby Baiersdorf, having recently lost a
temporary job with Siemens. Anti-euro sentiments have run so hot that the finance minister, Wolfgang Schäuble, told the daily
newspaper Bild this month the “danger of an anti-euro party needs to be taken seriously.” Germany’s political leaders and business
executives argue that the economy has benefited from the common currency. For
a nation that derives nearly half of
its gross domestic product from exports, a stable currency has greatly eased crossborder
commerce in the unified euro zone. And yet there is broad sentiment across Germany that the euro has not been good
for individuals. A recent survey by one of Germany’s premier research companies, Infratest dimap, of 1,004 people found this month
that 57 percent feel Germany should have kept the mark, while 60 percent said the euro was a disadvantage to them personally. “I
do think that a lot became more expensive” with the euro, said Jonas Molzberger, 21, an apprentice at the hotel Grauer Wolf on the
main street of Erlangen. “We feel that too in the hotel industry, especially in the restaurant. There are less people, less is being
ordered.” Low salaries — and higher prices — are a core complaint of German workers who are increasingly demanding wage
increases after a decade in which their real earnings dropped by 4.5 percent when adjusted for inflation, according to a recent report
by the International Labor Organization. Exports
have grown robustly in part because workers agreed years
ago to reduced wages and reduced hours to make Germany more competitive. Like workers in
other industrialized nations, including the United States, Germans also have had to accept that
the jobs available are not as secure as they once were . The number of people in nonstandard or atypical
employment in Germany increased to 7.72 million in 2008 from 5.29 million in 1998, according to the Federal Statistical Office. “I see
how families are struggling,” said Eberhard Irlinger, administrator for Erlangen-Höchstadt, a district of about 130,000 people around
the city of Erlangen. “In
fact, part of the economic prosperity comes from people not getting the social
security they should have. Germans are very reluctant to help other people when they have had
to step back from their own demands.” Yet these feelings of insecurity do not fit neatly into the picture of a nation
that has strikingly outperformed its neighbors. In fact, many Bavarian businesses say their biggest problem these days is a shortage
of skilled workers to fill positions that would help meet a growing demand for German quality goods. Across the nation, Germany
has experienced a severe shortage of skilled workers, in part because of its low unemployment rate, but also because of a low birth
rate and a relatively unwelcoming environment for immigrants, said officials, business leaders and experts. “We have lost contracts
because we just don’t have the people,” said Karl-Heinz John, chief executive officer of the small software developer called Infoteam
in the nearby village of Bubenreuth. “I know that all my industry partners are having trouble finding adequately educated people.”
Mr. John said he had to walk away from $1 million in contracts last year. But Infoteam still grew. Germany’s economy is on track to
expand at nearly 4 percent this year, while many of its southern neighbors struggle through recession. The national unemployment
rate is at an 18-year low of 7 percent. The country’s industrial output rose faster than expected and although exports fell
unexpectedly in October by 1.1 percent, imports rose to a new post-war high. That rise in imports spurred some optimism that the
consumer sector may play a larger role in fueling Germany’s future economic growth as demand for its products stalls in the
financially troubled euro zone nations. Growth is expected to continue in 2011, and last week a key monitoring institute reported
that business confidence was up compared with a year ago. German policy makers have struggled, at times awkwardly, to keep from
inflaming public sentiment at home while at the same time recognizing that German support for some weaker neighbors is vital to
preserving the euro and keeping exports strong. Mr. Schäuble, the finance minister, said that the public needed to better
understand that the euro was not the enemy of prosperity, but that it drove prosperity. “Without the euro, every German would be
poorer,” he told Bild. “Without the euro, the labor market would look a lot worse.” Many people do say that the euro is in
Germany’s broader interest, but that has not been accompanied by enthusiasm for the currency or support for spending German
money to help other nations. “I also feel that with the euro all — or let’s say much — has become more expensive,” said Peter Kern,
a hairdresser in Erlangen. “And if one also takes into account that people often do not earn more, then of course problems become
visible.”

No risk of crisis – assets check full collapse

Dauderstädt & Kellermann 07 (March 2007, Michael (director of the Division for Economic and
Social Policy at the Friedrich-Ebert-Stiftung) AND Christian (research fellow in the International
Policy Analysis Unit at the Friedrich-Ebert-Stiftung), “Controlling the Risks of a Global Economy,”
http://library.fes.de/pdf-files/iez/04744.pdf)

Costs and risks, especially in the crisis scenario, but also to some degree in the reference
scenario, lie in unemployment remaining high and prosperity having to be partially sacrificed on
the altar of competitiveness, potentially forcing Germany to participate in military action to
secure raw materials, or at least believe that it must make foreign policy concessions to suppliers, such as Russia. The
regionalization of the global economy could intensify, even though an appreciation of the Euro may pull more imports in the Euro
zone, which would not compensate, however, for the loss in U.S. consumption due to a devalued dollar, and thus not prevent the
global collapse of growth. In
Germany and in Europe a potentially strong influx of dollars would
initially lead to the Euro’s appreciation and rising prices of European/German assets (stocks, real
estate, etc.). This asset price inflation does not necessarily mean inflation in goods markets, since cheaper imports would have a
dampening effect. On one hand, the heavy influx of foreign capital might lead to a repeat performance of the `Locust´ debate, and,
on the other hand, bring about another wave of economic nationalism (fear of foreign takeovers). Rising
asset values,
however, would also strengthen consumer spending by providing an illusion of prosperity . The ECB
would have to decide if it should put a halt to this asset inflation or, in view of low consumer price inflation, tolerate it. Attempts to
arrest it by means of raised interest rates would have the fatal side effect of attracting additional capital. In
both the crisis –
and the reference scenario, there are political options to, minimize costs and risks, and make
use of opportunities. During a crisis, the most likely policies to find supporters are protectionist
policies along the line of `everybody for himself´, which generally escalate a crisis situation. In this
case it would be necessary to counterbalance the emergence of the crisis scenario while still in the reference scenario, something
rather difficult, given the existing hegemonic power structures with dominant free-market oriented actors.
Great Power Draw-in Answers
Frontline

No great power draw-in

Gelb ’10 (President Emeritus of the Council on Foreign Relations, 2010 (Leslie, former senior official in the U.S. Defense
Department and State Department, Foreign Affairs, November/December Foreign Affairs 2010)

Also reducing the likelihood of conflict today is that there


is no arena in which the vital interests of great
powers seriously clash. Indeed, the most worrisome security threats today --rogue states with
nuclear weapons and terrorists with weapons of mass destruction-- actually tend to unite the
great powers more than divide them. In the past, and specifically during the first era of
globalization, major powers would war over practically nothing. Back then, they fought over the Balkans, a
region devoid of resources and geographic importance, a strategic zero. Today, they are unlikely to shoulder their
arms over almost anything , even the highly strategic Middle East. All have much more to lose
than to gain from turmoil in that region. To be sure, great powers such as China and Russia will
tussle with one another for advantages, but they will stop well short of direct confrontation .
To an unprecedented degree, the major powers now need one another to grow their
economies, and they are loath to jeopardize this interdependence by allowing traditional
military and strategic competitions to escalate into wars. In the past, U.S. enemies--such as the Soviet Union--
would have rejoiced at the United States' losing a war in Afghanistan. Today, the United States and its enemies share an interest in
blocking the spread of both Taliban extremism and the Afghan-based drug trade. China also looks to U.S. arms to protect its
investments in Afghanistan, such as large natural-resource mines. More broadly, no great nation is challenging the balance of power
in either Europe or Asia. Although nations may not help one another, they rarely oppose one another in explosive situations.
Greece Turkey War Answers
Frontline

No Greece/Turkey war – both sides will stop short

Mann 1 (Stephen F., Lieutenant – US Navy, MA in National Security Affairs – Naval Postgraduate School, “The Greek-Turkish
Dispute in the Aegean Sea”, March, http://theses.nps.navy.mil/Thesis_01mar_Mann.pdf)

The basic issues in the Aegean and Cyprus have yet to be resolved, but relations
between Turkey and Greece
have improved, especially in the last year. Infrequent events such as the Imia/Kardak crisis still show the
escalatory nature of their relationship, but at the same time it is clear that both sides will
almost certainly always stop short of the act of war; the risks are too great, the potential rewards
to little, and the outcomes too uncertain. Both governments have some common sense in this
regard and they must now use that common sense to move toward resolution of the overall problem. How to move toward
that resolution is the question; many possibilities exist but some options and considerations, discussed in the next chapter,
seem more likely to work than others.

EU accession checks

Keefer 3 (Scott, LLM in International and Comparative Law – Georgetown University, Cardozo Journal of International and
Comparative Law, 11 Cardozo J. Int’l & Comp. L. 55, Lexis)

There will be risks in splitting the decision-making process. The implicit threat of force by Turkey could impact direct
negotiations. But the
use of force to resolve this dispute grows less likely as Turkey attempts to enter
the European Union. Greece holds a significant bargaining tool in its ability to either hamper or aid
a Turkish bid to join the European Union that may prove a greater weight than military force.
During the Kardak Crisis, the European Union solidly supported Greece, holding that Greek borders were part of the external
borders of the Union and that Turkey’s claims challenged the status quo in violation of international law.

Many incentives deter war

Mann 1 (Stephen F., Lieutenant – US Navy, MA in National Security Affairs – Naval Postgraduate School, “The Greek-Turkish
Dispute in the Aegean Sea”, March, http://theses.nps.navy.mil/Thesis_01mar_Mann.pdf)

It is generally accepted that the


dispute will not boil into hot war 119; though they’ve been at the
brink of war many times, for example in 1987 over the oil reserves (see Chapter IV, section B) and 1996
over Imia/Kardak (see Chapter II, section E), at least until now both countries always stop short. Among
the reasons for Greece is that [it] she is a much weaker power militarily. Among those for Turkey is
that she would be fighting against the UN if she aligned herself with the TRNC versus Greece and the
Republic of Cyprus, a damaging move both militarily and diplomatically. War would benefit neither
and hurt both. But both also claim that they are ready to fight, and it seems that if absolutely necessary they indeed
would. Incidents still occur every few months in the Aegean that hold the potential to escalate the dispute into conflict: both
countries still openly accuse the other of not doing enough to help solve the dispute120, harassments in the air still occur
frequently121, and infrequent events such as the Abdullah Öcalan case (where Turkey accused Greece of harboring the
Kurdistan Workers. Party (PKK) leader and terrorist who had been Turkey.s greatest domestic enemy122) do nothing but hurt.
Hopefully these kinds of incidents will not cause the two countries to do anything rash militarily , but
the simple fact that the word .hopefully. must be used in a sentence such as this is further proof that the dispute must be
settled.

Hot lines and external mediation stop war

Mann 1 (Stephen F., Lieutenant – US Navy, MA in National Security Affairs – Naval Postgraduate School, “The Greek-Turkish
Dispute in the Aegean Sea”, March, http://theses.nps.navy.mil/Thesis_01mar_Mann.pdf)

Finally, the United States (and other third parties such as the UN, the EU, and NATO) has been
intimately involved in trying to bring the two countries into accord. Though U.S. efforts have not always
been appreciated by both sides, higher levels of conflict were most likely avoided because of these third
party interventions. The Imia/Kardak crisis, for example, did not escalate to fighting because of
the intervention of U.S. personnel and initiatives. Other methods of limiting escalation of possible
minor incidents into full-scale war (such as a shooting incident) are also in place: the multinational
peacekeeping force in Cyprus, UNFICYP, for example, has helped to limit confrontations. Though no similar force exists
in the Aegean, the UN and NATO have attempted to implement several preventative measures to try to quell any uprising. Hot
lines, transparency of military exercises, and implementation of confidence building measures are UN and NATO
mechanisms that, while not optimal, have attempted to help Greece and Turkey avoid military
confrontation. Also, top-level diplomats are assigned to the region that offer their services and
opinions with a view towards reaching a solution. These are just a few of the ways in which the United
States has tried to broker peace in the region.
Grid Answers
Frontline

Grid is resilient and sustainable

Clark ’12 (MA candidate – Intelligence Studies @ American Military University, senior analyst – Chenega Federal Systems,
4/28/’12

(Paul, “The Risk of Disruption or Destruction of Critical U.S. Infrastructure by an Offensive Cyber Attack,” American Military
University)

In 2003, a simple physical breakdown occurred – trees shorted a power line and caused a fault – that had a
cascading effect and caused a power blackout across the Northeast (Lewis 2010). This singular occurrence has been
used as evidence that the electrical grid is fragile and subject to severe disruption through cyber-
attack, a disruption that could cost billions of dollars, brings business to a halt, and could even endanger lives – if compounded by
other catastrophic events (Brennan 2012). A power disruption the size of the 2003 blackout, the worst in American¶ history at that
time (Minkel 2008), is a worst case scenario and used as an example of the¶ fragility of the U.S. energy grid. This
perceived fragility is not real when viewed in the context of the robustness of the electrical
grid. When asked about cyber-attacks against the electrical grid in April of 2012, the intelligence chief of U.S. Cyber
Command Rear Admiral Samuel Cox stated that an attack was unlikely to succeed because of the
“huge amounts of resiliency built into the [electrical] system that makes that kind of catastrophic
thing very difficult” (Capaccio 2012). This optimistic view is supported by an electrical grid that has
proven to be robust in the face of large natural catastrophes. Complex systems like the
electrical grid in the U.S. are prone to failures and the U.S. grid fails frequently. Despite efforts to reduce
the risk out power outages, the risk is always present. Power outages that affect more than 50,000 people have occurred
steadily over the last 20 years at a rate of 12% annually and the frequency of large catastrophes remains relatively high
and outages the size of the 2003 blackout are predicted to occur every 25 years (Minkel 2008). In a complex system that is always at
risk of disruption, the effect is mitigated by policies and procedures that are meant to restore
services as quickly as possible. The most visible of these policies is the interstate Emergency
Management Assistance Compact, a legally binding agreement allowing combined resources to be quickly
deployed in response to a catastrophic disaster such as power outages following a severe hurricane (Kapucu,
Augustin and Garayev 2009). The electrical grid suffers service interruptions regularly , it is a large and complex
system supporting the largest economy in the world, and yet commerce does not collapse (Lewis 2010). Despite blizzards,
earthquakes, fires, and hurricanes that cause blackouts, the economy is affected but does not collapse and
even after massive damage like that caused by Hurricane Katrina, national security is not affected because
U.S. military capability is not degraded (Lewis 2010). Cyber-security is an ever-increasing concern in an increasingly
electronic and interconnected world. Cyber-security is a high priority “economic and national security
challenge” (National Security Council n.d.) because cyber-attacks are expected to become the top national security threat
(Robert S. Mueller 2012). In response to the threat Congress is crafting legislation to enhance cyber-
security (Brito and Watkins 2012) and the Department of Homeland Security budget for cyber-security has
been significantly increased (U.S. Senate Committee on Homeland Security and Governmental Affairs 2012).

The military isn’t stupid – backup capacity solves blackouts


Aimone ’12 (9-12 (Dr. Michael, Director of Business Enterprise Integration – Office of the Deputy Under Secretary of Defense
(Installations and Environment), “Statement Before the House Committee on Homeland Security, Subcommittee on Cybersecurity,
Infrastructure Protection and Security Technologies,” 2012, http://homeland.house.gov/sites/homeland.house.gov/files/Testimony
%20-%20Aimone.pdf)

DoD’s facility energy strategy is also focused heavily on grid security in the name of
mission assurance. Although the Department’s fixed installations traditionally served largely as a platform for training
and deployment of forces, in recent years they have begun to provide direct support for combat operations, such as unmanned
aerial vehicles (UAVs) flown in Afghanistan from fixed installations here in the United States. Our fixed installations also serve
as staging platforms for humanitarian and homeland defense missions. These installations are largely
dependent on a commercial power grid that is vulnerable to disruption due to aging infrastructure, weather-
related events, and potential kinetic, cyber attack. In 2008, the Defense Science Board warned that DoD’s reliance on a fragile
power grid to deliver electricity to its bases places critical missions at risk. 1 Standby Power Generation Currently, DoD
ensures that it can continue mission critical activities on base largely through its fleet of
on-site power generation equipment. This equipment is connected to essential mission
systems and automatically operates in the event of a commercial grid outage. In addition,
each installation has standby generators in storage for repositioning as required. Facility
power production specialists ensure that the generators are primed and ready to work, and that they
are maintained and fueled during an emergency. With careful maintenance these
generators can bridge the gap for even a lengthy outage. As further back up to this installed equipment,
DoD maintains a strategic stockpile of electrical power generators and support equipment
that is kept in operational readiness. For example, during Hurricane Katrina, the Air Force
transported more than 2 megawatts of specialized diesel generators from Florida, where
they were stored, to Keesler Air Force Base in Mississippi, to support base recovery.

Alt Cause – Solar Flares

Deborah Zabarenko, Environmentalist Writer, 12 [“Solar superstorm could knock out US power grid – experts,” Chicago
Tribune, August 3, http://articles.chicagotribune.com/2012-08-03/news/sns-rt-us-solar-superstormbre8721k8-20120803_1_power-
grid-geomagnetic-transformers]

WASHINGTON (Reuters) - U.S.


weather has been lousy this year, with droughts, heat and killer
storms. But a solar superstorm could be far worse. A monster blast of geomagnetic particles
from the sun could destroy 300 or more of the 2,100 high-voltage transformers that are the
backbone of the U.S. electric grid, according to the National Academy of Sciences (NAS). Even a few hundred
destroyed transformers could disable the entire interconnected system. There is impetus for a
group of federal agencies to look for ways to prepare for such a storm this year as the sun
moves into an active period called solar maximum, expected to peak in 2013. Some U.S. experts
estimate as much as a 7 percent chance of a superstorm in the next decade , which seems a
slight risk, but the effects would be so wide-ranging - akin to a major meteorite strike - that it
has drawn official concern. Although the likelihood of this kind of geomagnetic storm, like a big hit from a space rock, is
extremely low, its impact would be great. By comparison, the probability of a large meteorite hitting Earth is at some fraction of 1
percent. Power blackouts can cause chaos, as they did briefly in India when more than 600 million people lost electricity for hours on
two consecutive days in July. However, the kind of long-duration outage that might happen in the case of a massive solar storm
would have more profound and costly effects. There is disagreement on how costly the damage would be, but experts in the
U.S. government and industry acknowledge it is a complex problem requiring a coordinated
solution. A report by the NAS estimated that about 365 high-voltage transformers in the
continental United States are at risk of failure or permanent damage requiring replacement in
the event of a solar superstorm. The North American Electric Reliability Corporation, or NERC, which oversees North
America's power grid, disputed the academy's estimate that hundreds of high-voltage transformers could be lost in a solar
superstorm. In a report earlier this year, NERC said a more likely result would be voltage collapse, which would take out power but
not destroy transformers. Any power outage would be less protracted in that event. NO POWER FOR A YEAR? The
academy's
report noted that replacements for transformers might not be available for a year or more,
and the cost of damage in the first year after a storm could be as high as $2 trillion . The most
vulnerable areas are the eastern one-third of the country, from the Midwest to the East Coast, and the Northwest, as far east as
Montana and Wyoming and as far south as California. The national grid was built over decades to get energy at the lowest price
from where it is generated to where it is used. A solar superstorm has the capacity to bring that network down, the academy's
report said.

Prefer our evidence—grid is actively improving

Koerth-Baker ’12 (science editor – Boing Boing, columnist – NYT Magazine, electric grid expert, 8/3/’12
(Maggie, “Blackout: What's wrong with the American grid,” http://boingboing.net/2012/08/03/blackout-whats-wrong-with-t.html)

But this is about more than mere bad luck. The real causes of the 2003 blackout were fixable problems, and
the good news is that, since then, we’ve made great strides in fixing them. The bad news, say some grid experts, is
that we’re still not doing a great job of preparing our electric infrastructure for the future. Let’s get one thing out of the way right up
front: The North American electric grid is not one bad day away fro m the kind of catastrophic failures
we saw in India this week. I’ve heard a lot of people speculating on this, but the folks who know the grid say that,
while such a huge blackout is theoretically possible, it is also extremely unlikely. As Clark Gellings, a
fellow at the Electric Power Research Institute put it, “An engineer will never say never,” but
you should definitely not assume anything resembling an imminent threat at that scale. Remember, the
blackouts this week cut power to half of all Indian electricity customers. Even the 2003 blackout—the largest blackout in
North America ever—only affected about 15% of Americans. We don’t know yet what, exactly, caused the Indian
blackouts, but there are several key differences between their grid and our grid. India’s electricity is only weakly tied
to the people who use it, Gellings told me. Most of the power plants are in the far north. Most of the population is in the
far south. The power lines linking the two are neither robust nor numerous. That’s not a problem we
have in North America. Likewise, India has considerably more demand for electricity than it has supply.
Even on a good day, there’s not enough electricity for all the people who want it , said Jeff Dagle, an
engineer with the Pacific Northwest National Laboratory’s Advanced Power and Energy Systems research group. “ They’re
pushing their system much harder, to its limits,” he said. “If they have a problem, there’s less cushion
to absorb it. Our system has rules that prevent us from dipping into our electric reserves on a day-
to-day basis. So we have reserve power for emergencies .”

Safeguards are strong

Donna Leinwand Leger, USA Today, 12 [“Blackout Like India's Unlikely in U.S., Experts Say,”, August 1,
http://www.weather.com/news/india-blackout-unlikely-in-us-20120801]
WASHINGTON -- A massive, countrywide power failure like the one in India on Tuesday is "extremely
unlikely" in the United States, energy experts say . In India, three of the country's government-operated power
grids failed Tuesday, leaving 620 million people without electricity for several hours. The outage, the second in two days in the
country of 1.21 billion people, is the world's biggest blackout on record. The
U.S. electricity system is segmented
into three parts with safeguards that prevent an outage in one system from tripping a blackout
in another system, "making blackouts across the country extremely unlikely ," Energy Department
spokeswoman Keri Fulton said. Early reports from government officials in India say excessive demand knocked the country's power
generators offline. Experts say India's industry and economy are growing faster than its electrical systems. Last year, the economy
grew 7.8% and pushed energy needs higher, but electricity generation did not keep pace, government records show. " Weare
much, much less at risk for something like that happening here, espe cially from the perspective
of demand exceeding supply," said Gregory Reed, a professor of electric power engineering at
University of Pittsburgh. "We're much more sophisticated in our operations . Most of our issues have
been from natural disasters." The U.S. generates more than enough electricity to meet demand and
always have power in reserve, Reed said. "Fundamentally, it's a different world here," said Arshad Mansoor, senior vice
president of the Electric Power Research Institute in Washington and an expert on power grids. " It's an order of magnitude
more reliable here than in a developing country ." Grid operators across the country analyze
power usage and generation, factoring outside factors such as weather, in real time and can
forecast power supply and demand hour by hour, Mansoor said. "In any large, complex interactive
network, the chance of that interconnection breaking up is always there ," Mansoor said. "You cannot
take your eye off the ball for a minute." Widespread outages in the U.S. caused by weather are common.
But the U.S. has also had system failure s, said Ellen Vancko, senior energy adviser for the Union of Concerned
Scientists, based in Washington. On Aug. 14, 2003, more than 50 million people in the Northeast and Canada lost power after a
major U.S. grid collapsed. The problem began in Ohio when a transmission wire overheated and sagged into a tree that had grown
too close to the line, Vancko said. That caused other power lines to overheat until so many lines failed that the system shut itself
down, she said. "That
was less a failure of technology and more a failure of people, a failure of
people to follow the rules," Vancko said. "There were a whole bunch of lessons learned ." In 2005, in
response to an investigation of the blackout, Congress passed a law establishing the North American Electric Reliability Corporation
(NERC) to enforce reliability standards for bulk electricity generation. "On
the whole, we have a more reliable
electrical system with NERC," said Vancko, a former NERC official. "We have the safeguards, but we cannot
say it can't happen here. It's the most complicated system in the world."

Prefer our defense—reporting and academic work on critical infrastructure is prone to threat
inflation

Sorebo, chief cybersecurity technologist and vice president – SAIC, consultant for the government and industry in cybersecurity
and smart grid technology, MA – GW University, JD – Catholic U, 2/8/’10

(Gib, “The Many Shades of Project Grey Goose,” RSA Conference)

As I noted in my previous post about a recent 60 Minutes segment, we often rely on rumor and innuendo as the basis for
journalism in critical infrastructure. If a current or former high-ranking public official says he heard something, then it
must be true. Unfortunately, Project Grey Goose, whose stated objective was “to answer the question of whether there has been
any successful hacker attacks against the power grid, both domestically and internationally,” falls
victim to much of the same
fear, uncertainty, and doubt. As in all media reports, there are factual bases for findings that exaggerated the
true state of the electric grid. For example, their statement that “90% of the U.S. Department of Defense's
(DOD) most critical assets are entirely dependent on the bulk power grid” is presumably taken
from a Government Accountability Office (GAO) report noting that 85 percent of critical DoD assets rely on
commercial electric power. However, the “entirely dependent” statement ignores the wide
variety of backup generators that support these assets , and while not adequate, are nonetheless a
significant contribution to the reliability of critical DoD assets. So rather than sounding the alarm that
military bases, for the most part, do not have their own power plants, a better response would have been to suggest that the
military expand the use of backup generators and micro-grid technology to augment commercial power as the GAO report
does. Of course, that would not grab as many headlines. Similarly, the Grey Goose Report note that “[m]ost Grid asset
owners and operators have been historically resistant to report cyber attacks against their networks as well as make the necessary
investments to upgrade and secure their networks.” While it may be true that incidents are underreported,
the implication that the electricity industry is deficient compared to other industrial sectors is misleading or even
wrong. Most companies do not report security incidents unless legally required to or to mitigate the harm to their customers, and
even then the evidence of an intrusion and theft of data had better be definitive. Lost laptops and backup tapes are one thing. You
cannot say they are within your control if they go missing. However, organizations
in general have a horrible record
of even detecting when a successful attack has occurred let alone what was taken. Like many
industries, the electricity industry has struggled to pinpoint the source of many disruptions
associated with their network infrastructure. More often than not, the problems were
inadvertent and not malicious. We can certainly do better, and with technologies like Smart Grid, we have to. However,
calling out the electricity industry for failures that we’ve all been subjected to is not very productive. The other statements made
about the vulnerabilities in the electricity sector are misleading. While North American Electric Reliability Corporation Critical
Infrastructure Protection (NERC CIP) still does not apply to many aspects of the electrical grid for a variety of jurisdictional reasons,
where it does apply, it is not voluntary, as the many utilities subjected to rigorous and painful audits can attest. The process may not
be perfect, but utilities
are being subjected to scrutiny. Moreover, anyone receiving stimulus grants under
the Department of Energy’s Smart Grid grant program has to demonstrate a very rigorous approach to cyber
security through the entire implementation life cycle. Finally, the report cites a litany of vulnerabilities discovered
in various Smart Grid devices such as meters and perpetuates speculation about the potential impact on the grid without
considering compensating security controls. Nowhere does the report cite names of vulnerable vendors nor
does it provide any information about whether these vulnerable products have actually been
implemented. It’s like saying that tests on personal computers showed that they were
vulnerable to attack without identifying the operating system or the applications running on the device.

Zero impact to grid failures, even by cyber attacks

Birch ’12 (Douglas Birch, former foreign correspondent for the Associated Press and the Baltimore Sun who has written
extensively on technology and public policy, 10/1/12, “Forget Revolution,” Foreign Policy,
http://www.foreignpolicy.com/articles/2012/10/01/forget_revolution?page=full, October 1, 2012)

Government officials sometimes describe a kind of Hieronymus Bosch landscape when warning of the possibility of a cyber attack on
the electric grid. Imagine, if you will, that the United States is blindsided by an epic hack that interrupts power for much of the
Midwest and mid-Atlantic for more than a week, switching off the lights, traffic signals, computers, water pumps, and air
conditioners in millions of homes, businesses, and government offices. Americans swelter in the dark. Chaos reigns! Here's another
nightmare scenario: An electric grid that serves two-thirds of a billion people suddenly fails in a developing, nuclear-armed country
with a rich history of ethnic and religious conflict. Rail transportation is shut down, cutting off travel to large swathes of the country,
while many miners are trapped underground. Blackouts on this scale conjure images of civil unrest, overwhelmed police, crippled
hospitals, darkened military bases, the gravely injured in the back of ambulances stuck in traffic jams. The specter of what Defense
Secretary Leon Panetta has called a "digital Pearl Harbor" led to the creation of U.S. Cyber Command, which is tasked with
developing both offensive and defensive cyber warfare capabilities, and prompted FBI Director Robert Mueller to warn in March that
cyber attacks would soon be "the number one threat to our country." Similar concerns inspired both the Democrats and Republicans
to sound the alarm about the cyber threat in their party platforms. But are cyber attacks really a clear and present danger to
society's critical life support systems, capable of inflicting thousands of casualties? Or has fear of full-blown cybergeddon at the
hands of America's enemies become just another feverish national obsession -- another of the long, dark shadows of the 9/11
attacks? Worries about a large-scale, devastating cyber attack on the United States date back several decades, but
escalatedfollowing attacks on Estonian government and media websites during a diplomatic conflict with Russia in 2007. That digital
ambush was followed by a cyber attack on Georgian websites a year later in the run-up to the brief shooting war between Tbilisi and
Moscow, as well as allegations of a colossal, ongoing cyber espionage campaign against the United States by hackers linked to the
Chinese army. Much of the concern has focused on potential attacks on the U.S. electrical grid. "If I were an attacker
and I wanted to do strategic damage to the United States...I probably would sack electric power on the U.S. East Coast, maybe the
West Coast, and attempt to cause a cascading effect," retired Admiral Mike McConnell said in a 2010 interview with CBS's 60
Minutes. But the scenarios sketched out above are not solely the realm of fantasy. This summer, the
United States and India
were hit by two massive electrical outages -- caused not by ninja cyber assault teams but by force majeure. And, for
most people anyway, the results were less terrifying than imagined. First, the freak "derecho" storm that barreled
across a heavily-populated swath of the eastern United States on the afternoon of June 29 knocked down trees that crushed cars,
bashed holes in roofs, blocked roads, and sliced through power lines. According to an August report by the U.S. Department of
Energy, 4.2 million homes and businesses lost power as a result of the storm, with the blackout stretching across 11 states and the
District of Columbia. More than 1 million customers were still without power five days later, and in some areas power wasn't
restored for 10 days. Reuters put the death tollat 23 people as of July 5, all killed by storms or heat stroke. The second incident
occurred in late July, when 670 million people in northern India, or about 10 percent of the world's population, lost power in the
largest blackout in history. The failure of this huge chunk of India's electric grid was attributed to higher-than-normal demand due to
late monsoon rains, which led farmers to use more electricity in order to draw water from wells. Indian officials told the media there
were no reports of deaths directly linked to the blackouts. But this cataclysmic event didn't cause widespread chaos in
India -- indeed, for some, it didn't even interrupt their daily routine. "[M]any people in major cities barely noticed the disruption
because localized blackouts are so common that many businesses, hospitals, offices and middle-class homes have backup diesel
generators," the New York Timesreported. The most important thing about both events is what didn't happen. Planes didn't fall out
of the sky. Governments didn't collapse. Thousands of people weren't killed. Despite disruption and delay,
harried public officials, emergency workers, and beleaguered publics mostly muddled through. The summer's blackouts strongly
suggest that a
cyber weapon that took down an electric grid even for several days could turn out
to be little more than a weapon of mass inconvenience. That doesn't mean the United States can relax. James Lewis,
director of the technology program at the Center for Strategic and International Studies, believes that hackers threaten the security
of U.S. utilities and industries, and recently penned an op-ed for the New York Times calling the United States "defenseless" to a
cyber-assault. But he told Foreign Policy the recent derecho showed that even a large-scale blackout would not
necessarily have catastrophic consequences.
---Grid- Safeguards

No black-outs in the US

Wood 12 -- Senior Communications Advisor at Business Roundtable (Carter, 8/2/12, "The grid: After India, America? No, but
still…" http://businessroundtable.org/blog/the-grid-after-india-america-no-but-still/)

A blackout of such scale could not happen in the U nited S tates. For one thing, we don't have 600 million
people. And America's electrical grid is certainly much more resilient than the one in India , a still-
developing country with ineffective governments. Still, as The Washington Post reports today, "Aging power grid on overload as U.S.
demands more electricity." At CNBC, Jim Cramer
asked Thomas F. Farrell II, Chairman, President & CEO of
Dominion Resources, about India. Could the same thing happen in the U nited States? Farrell responded:
Our system has a lot more rigor to it and partly because we have reserve margins, meaning we
have more power stations than we need to run at any particular moment in time , so that if a
power station goes out, there's a back-up to help keep the grid stable. They don't have that
much excess power in India, and when they get to the root cause, they'll probably find that was somewhere in there.

US grid stable – frequency can be maintained by new technologies

Lamonica 12 -- senior writer covering green tech and cutting-edge technologies, contributor @ Technology Review (Martin,
8/2/12, "Outage in India Could Be a Harbinger for the Rest of the World," http://www.technologyreview.com/news/428685/outage-
in-india-could-be-a-harbinger-for-the-rest/)

The primary function of grid operators is to anticipate load and to maintain a steady balance
between power supply and demand. The grid signal operates at a set frequency —60 hertz in the U.S.
and 50 hertz in India—and when supply and demand fall out of sync , the frequency will either dip or rise .
In the U.S., grid operators have "hot" generators on standby to ramp up power in order to keep
a close-to-steady frequency, but that's not the case when generators are routinely maxed out. "In a developing world
country, it's tough to keep 10 percent of the generation capacity on contingency when you may use it once in a lifetime," Mansoor
says. "You're not using the generator, but you still pay for it. That's tough to do." More
technologies to keep that
frequency steady are emerging. Sensors called phasor measurement units are designed for real-
time measurement of grid frequency, and can flag potential problems . Grid operators in the
United States are increasingly using automation to manage demand-response programs that
lower consumption at big power users at peak times . These types of technologies as well as microgrids
(see "Microgrids Keeps the Power Local, Cheap, and Reliable") stand to make electricity grids more reliable as
more renewable resources come online and weather-related events, such as heat waves, strain generating
resources.

Smart grid investments already underway


Kemp 12 -- Reuters market analyst (John, 4/5/12, "COLUMN-Phasors and blackouts on the U.S. power grid: John Kemp,"
http://www.reuters.com/article/2012/04/05/column-smart-grid-idUSL6E8F59W120120405)

(Reuters) - Smart meters are the most visible part of the power industry's attempt to upgrade the
electricity network to cope with rising consumption and the integration of renewable sources of generation such
as wind and solar. The U.S. federal government is spending hundreds of millions of dollars to support
the roll out of smart meters that will enable more variable power pricing linked to time of use (TOU)
or peak demand to encourage ordinary households and small businesses to limit their electricity use when
demand is greatest and help manage pressure on the grid. Increasing the amount of "demand-
side response" is crucial to improving flexibility and ensuring the grid can cope with the
unpredictable output of intermittent power sources such as wind and solar, as well as small-scale
distributed generation by households and businesses themselves selling power back into the network at irregular times. But
the smart grid is about much more than just smart meters. The Smart Grid Investment Grant (SGIG) programme
established by the American Recovery and Reinvestment Act (ARRA) is also funding the deployment of 877 high-
frequency phasor monitoring units (PMUs) linked to satellite communications and the global
positioning system. The aim is to improve the robustness and reliability of the transmission grid
and prevent a repeat of the cascading power failures that rolled across the north-eastern United States and parts
of Canada in 2003.

New tech means their 2003 example no longer applies


Koerth-Baker, science editor – Boing Boing, columnist – NYT Magazine, electric grid expert, 8/3/’12

(Maggie, “Blackout: What's wrong with the American grid,” http://boingboing.net/2012/08/03/blackout-whats-wrong-with-t.html)

In 2003, it took about 30 seconds for data about what was happening on the grid to be gathered, compiled,
analyzed, and displayed in a way that grid controllers could use. That sounds pretty fast, until you consider
the fact that changes on the grid happen much, much faster***. If a power plant goes offline in Arizona, it can create a measurable
effect in Canada in about a second. If your view of the grid is updated only every 30 seconds, you miss
important details. After the 2003 blackout, grid experts went back and essentially replayed the whole thing in a computer
modeling program. The idea was to try to get a better idea of where things went wrong and how a similar event could be prevented
in the future. They found that, about an hour before the blackout, the grid was showing signs of stress that controllers didn’t see at
the time, said Carl Imhoff, manager of the Energy and Environment Sector at PNNL. It wasn’t the controllers’ fault. They simply
didn’t have the technology to see the big picture. Fixing the Grid Today, that technology exists.
Phasor Measurement Units are kind of the opposite of sexy. Also known as PMUs, they’re just anonymous little boxes that sit on
server racks in electrical substations. But phasors are linked into transmission lines. They see what’s
happening on the line—how well supply and demand are balanced, whether voltage and
frequency are stable and within the normal range. That’s just one point of data, recorded in one place. But a
network of phasors can tell you a lot. It can show you , for instance, if the stability of the grid is
changing as electricity moves from Cleveland to Columbus. And the phasors process that information far more
quickly. Today, our grid can give controllers information about the big picture in less than 10 seconds. Researchers like Massoud
Amin are working on getting that response time down to fewer than 3 seconds. If we’d had a phasor network in 2003,
grid controllers would have had that hour warning about the problem. There’s a good chance they’d
have been able to fix it , or, at least, make the resulting blackout smaller and more localized. When it
comes to PMUs, 2003was really a wake-up call. It led utilities and the government to team up to
install a true phasor network throughout the United States. That effort is currently ongoing. In 2009 there were maybe
200 phasors in operation. By the end of 2013, there will be more than 1000 installed throughout this country. Over the last five years
a partnership between federal Recovery Act funds and private industry dollars has invested $7.8 billion in
upgrading the grid, Massoud Amin said.
---Grid- Methodology

Bias colors all of their evidence

Sorebo ’9, chief cybersecurity technologist and vice president – SAIC, consultant for the government and industry in
cybersecurity and smart grid technology, MA – GW University, JD – Catholic U, 11/11/

(Gib, “More FUD from the Mainstream Media?,” RSA Conference)

In reality, much
of what passes as intelligence within the critical infrastructure sector about
security events seems to be more based on rumor and innuendo rather than sound
investigation. This is undoubtedly due in part to the fact that private sector operators who own
most of the infrastructure are loathe to report such events, particularly when there is no obvious harm. One
cannot fault journalists for reporting what they know. However, simply reporting that the Brazilian electric company disputes the
claim or has no comment might imply that the full story is not being told. There are some real threats facing our critical
infrastructure that are clearly in the public interest to be disclosed. However, such reporting, and ultimately efforts to fix the
problem, loses its credibility if operators feel that any good faith efforts to report incidents will be met
with exaggeration and grandstanding. The media does a decent job at rooting out corruption and disclosing
matters of public concern without telling a one-sided story. One would hope that they don’t feel it is okay to skimp on
their reporting duties when the subject is more obscure and less familiar to the general public.
Those of us in the information security community should work to see that the full story is told whenever possible.

Theeir method is flawed

Schneier ’7, editor – Crypto-Gram, internationally renowned security technologist and author, contributor – NYT, Forbes,
Guardian, Wired, Bulletin of the Atomic Scientists, 10/2/

(Bruce, “Staged Attack Causes Generator to Self-Destruct,” http://www.schneier.com/blog/archives/2007/10/staged_attack_c.html)

A government video shows the potential destruction caused by hackers seizing control of a
crucial part of the U.S. electrical grid : an industrial turbine spinning wildly out of control until it becomes a smoking
hulk and power shuts down. The video, produced for the Homeland Security Department and obtained by The Associated Press on
Wednesday, was marked "Official Use Only." It shows commands quietly triggered by simulated hackers
having such a violent reaction that the enormous turbine shudders as pieces fly apart and it belches black-and-white smoke. The
video was produced for top U.S. policy makers by the Idaho National Laboratory, which has studied the little-understood risks to the
specialized electronic equipment that operates power, water and chemical plants. Vice President Dick Cheney is among those who
have watched the video, said one U.S. official, speaking on condition of anonymity because this official was not authorized to
publicly discuss such high-level briefings. More here. And the video is on CNN.com. I haven't written much about SCADA
security , except to say that I think the risk is overblown today but is getting more serious all the time -- and we need to
deal with the security before it's too late. I didn't know quite what to make of the Idaho National Laboratory video; it seemed like
hype, but I couldn't find any details. (The CNN headline, "Mouse click could plunge city into darkness, experts say," was definitely
hype.) Then, I
received this anonymous e-mail: I was one of the industry technical folks the DHS
consulted in developing the "immediate and required" mitigation strategies for this problem. They
talked to several industry groups (mostly management not tech folks): electric, refining, chemical, and
water. They ignored most of what we said but attached our names to the technical parts of
the report to make it look credible. We softened or eliminated quite a few sections that may have had
relevance 20 years ago, such as war dialing attacks against modems. The end product is a work order
document from DHS which requires such things as background checks on people who have access to modems and logging their visits
to sites with datacom equipment or control systems. By the way -- they were unable to hurt the generator you see in the video but
did destroy the shaft that drives it and the power unit. They triggered the event from 30 miles away ! Then they
extrapolated the theory that a malfunctioning generator can destroy not only generators at the power company but the
power glitches on the grid would destroy motors many miles away on the electric grid that
pump water or gasoline (through pipelines). They kept everything very secret (all emails and reports encrypted,
high security meetings in DC) until they produced a video and press release for CNN. There was huge concern by DHS that
this vulnerability would become known to the bad guys -- yet now they release it to the world for their own career reasons. Beyond
shameful. Oh, and they did use a contractor for all the heavy lifting that went into writing/revising the required mitigations
document. Could not even produce this work product on their own. By the way, the vulnerability they hypothesize is
completely bogus but I won't say more about the details. Gitmo is still too hot for me this time of year.
Guam Answers
Frontline
No Guam realignment

Hornung 12, Jeffrey W. Associate Professor at the Asia-Pacific Center for Security Studies in Honolulu, HI and an Adjunct
Fellow with the Office of the Japan Chair at the Center for Strategic and International Studies, 1/5/, Time to Acknowledge the
Realignment Impasse,

http://csis.org/files/publication/120105_Hornung_RealignmentImpasse_JapanPlatform.pdf

Japanese foreign minister Koichiro Gemba and U.S. secretary of state Hillary Clinton recently
reaffirmed their countries’ commitment to an agreement to build a Futenma Replacement
Facility (FRF) in northern Okinawa and relocate over 8,000 Marines to Guam. Despite this high-
level reaffirmation, the agreement is no longer workable . Although the allies are no closer to
implementing it than they were the day negotiations began, developments over the past few
months have pushed them further from their goal. Tokyo and Washington need to acknowledge
that the conditions that were once conducive to implementation no longer exist . Because of the
impasse, it is time to temper political rhetoric with reality.

Overwhelms the aff

Freedberg, ‘12, Sydney -Pentagon, Congress Must Break 'Logjam' Over Japan, Guam Bases:
CSIS, defense.aol.com/2012/07/27/pentagon-congress-must-break-logjam-over-japan-guam-
bases-c/

Congress commissioned the study from the Center for Strategic and International Studies in the
authorization bill for fiscal year 2012. SASC chairman Sen. Carl Levin's office formally received
the report on Tuesday, according to a stamp on the posted version of the document. While the
Senators are still reviewing it, Levin, ranking member John McCain, and Virginia Democrat Jim
Webb released a statement this morning praising CSIS's work, in particular its call for the
administration to better articulate the strategic logic and practical logistics of the "rebalancing
to Asia" -- the term that has replaced the catchier but controversial "pivot to Asia."

The Senators' statement was silent on another part of the study, however: CSIS also challenges
Capitol Hill and the administration to compromise on stalled plans to move Marines from
Okinawa to Guam.

"These plans are at the center of a logjam between DoD [the Department of Defense], which
would like to implement them, and the Congress, which is reluctant to authorize funding absent
better details about cost and long-term master plans." The report argues that decades-long
commitments to Japan and Korea have resulted in a Pacific posture that puts too many forces in
the north of the region and not enough in the south, where China has become increasingly
aggressive towards its maritime neighbors in the South China Sea, especially the Philippines.
Shifting forces from Okinawa to Guam would help correct that imbalance, and China, our allies,
and neutral parties are all watching for signs of US clarity and resolve: "The current impasse
between DoD and the Congress is not cost-free in terms of US strategic influence in the region ,"
the report warns.

Pre-positioning and alternative bases sufficient to solve

Hornung 12, Jeffrey W. Associate Professor at the Asia-Pacific Center for Security Studies in Honolulu, HI and an Adjunct
Fellow with the Office of the Japan Chair at the Center for Strategic and International Studies, 1/5/, Time to Acknowledge the
Realignment Impasse,

http://csis.org/files/publication/120105_Hornung_RealignmentImpasse_JapanPlatform.pdf

What the alternative is remains open for discussion, although closing Futenma and integrating
its current assets and operations elsewhere needs to be included. Understanding that building
large military bases is unaffordable today, a joint basing option on Japanese military bases for
the marines may prove beneficial, something Sheila Smith of the Council of Foreign Relations
has advocated. Not only does it save money by utilizing existing infrastructure, it also improves
interoperability between the allies. Another option, advocated for years by Mike Mochizuki of
George Washington University and Michael O’ Hanlon of the Brookings Institution, could be the
pre-positioning of U.S. equipment and supplies. This could be at Japanese military bases (with
access agreed ahead of time), on amphibious ships at Japanese military ports, or on maritime
vessels pre-positioned in Japanese waters. Pre-positioning equipment would allow marines to
be flown in from bases abroad (or a smaller number that remain in Okinawa) to quickly access
their equipment in a crisis. Another option is looking to other states. When U.S. realignment
plans were first negotiated, one of the reasons Guam was chosen was because no other
countries offered to host U.S. forces. Yet, this past November, Washington and Canberra agreed
for the port of Darwin to host a full marine task force of up to 2,500 personnel. The implications
of having this many marines based in Australia could take some pressure off the U.S. focus on
Okinawa. Likewise, in December, Admiral Jonathan Greenert, chief of naval operations, wrote in
the U.S. Naval Institute’s Proceedings that the United States will station several of its newest
littoral combat ships in Singapore and may step up periodic aircraft deployments to the
Philippines and Thailand. While not marines, this strengthens the U.S. presence in the region
and could therefore further alleviate pressure to have a heavy military presence on Okinawa or
Guam.

Though none of these alternatives are perfect substitutes for the Roadmap, they represent
innovative approaches to maintaining U.S. forward presence in the Asia-Pacific region to ensure
rapid, flexible responses during times of crisis.

Guam doesn’t solve

Klingner 11
Bruce Klingner is Senior Research Fellow for Northeast Asia in the Asian Studies Center at The Heritage Foundation, Heritage
Foundation, May 18, 2011, " Proposed Re-Realignment for Northeast Asia Ignores Strategic Realities",
http://www.heritage.org/research/reports/2011/05/proposed-re-realignment-for-northeast-asia-ignores-strategic-realities

The Senators casually suggest that U.S. Air Force units now at Kadena could be dispersed “into
other areas of the Pacific region.” They fail to understand that all of the units are on Okinawa to
fulfill critical treaty commitments and other alliance missions and are already best positioned to
deal with the highest probability contingencies. Okinawa’s strategic location contributes to
potent U.S. deterrent and power projection capabilities and enables rapid and flexible
contingency response. Redeploying U.S. forces from Okinawa to Guam would reduce these
capabilities. Guam is 1,400 miles, a three-hour flight, and multiple refueling operations farther
from potential conflict zones in and around Japan and Korea. Some of the planes currently at
Kadena do not have aerial refueling capability, reducing their availability and effectiveness if
redeployed to Guam. Moving fixed-wing aircraft to Guam would drastically reduce the number
of combat aircraft and logistic sorties that U.S. forces could conduct during crises involving
North Korea or China while exponentially increasing and likely exceeding refueling and logistic
capabilities. Deploying additional aircraft carriers would not be sufficient. Aircraft carriers cannot
support transport or air-to-air refueling aircraft, nor can they generate the necessary combat
aircraft sorties planned for both Kadena and Futenma during contingency and combat
operations. Potentially Dangerous If Levin, McCain, and Webb have concerns about the slow
pace of DoD planning and construction or doubts over the manner in which military realignment
funds have been dispersed, then they should address those specific issues. But advocating that
comprehensive U.S. military realignment plans in two countries be brought to a screeching halt
is counterproductive at best and strategically dangerous at worst. Accepting the Senators’
recommendations would push U.S. Air Force and Marine Corps units away from potential
conflict zones. Diminishing U.S. military assets would concern the nation’s Asian allies, degrade
American deterrent and defense capabilities, embolden North Korea and China, and signal a U.S.
retreat from Asia.

There is no Guam build-up!

Hornung 12, Jeffrey W. Associate Professor at the Asia-Pacific Center for Security Studies in Honolulu, HI and an Adjunct
Fellow with the Office of the Japan Chair at the Center for Strategic and International Studies, 1/5/, Time to Acknowledge the
Realignment Impasse,

http://csis.org/files/publication/120105_Hornung_RealignmentImpasse_JapanPlatform.pdf

Mounting Delays on Guam

Unlike Okinawa, many on Guam embraced the idea of an expanded U.S. presence. This was
because of the prospect of a rapid influx of money to help rebuild the poor infrastructure and
struggling economy. Yet, while most media and scholarly attention has focused on the FRF,
problems have been increasing on Guam as well.
The U.S. Navy was not able to release up to $1.2 billion in military construction task orders for
the Guam buildup until the March 2011 signing of a Programmatic Agreement. This enabled
contractors to complete the final phases of their design work for proposals. The existing buildup
is already plagued by infrastructure concerns and cost overruns . Worse, key construction
projects are hampered by lawsuits, environmental concerns, and as I discuss below, funding
cuts. With the navy promising this past autumn to conduct new studies on proposed
construction sites, the commencement of crucial parts of the Guam buildup is now pushed back
by several years, at least.

Consider first the navy’s plans for live-fire rifle and grenade ranges for the marines. The navy
deferred its decision on a location because of an ongoing lawsuit. In November 2010, three
private groups filed a lawsuit against the Department of Defense (DOD) in an effort to prevent
the ranges on about 1,090 acres of ancestral Chamorro lands that are host to ancient graves and
medicinal plants. The area, known as Pagat, is the preferred site but is listed on the National
Register of Historic Places. Opponents encourage the navy to locate the ranges on the island of
Tinian, an option the navy has ruled out. Although navy officials have pledged to protect Pagat
Village and argued that the ranges could be built with minimal damage to Pagat, that
Guamanians can access the land when the ranges are not in use and that some areas would be
accessible at all times, Guamanians remain unconvinced. And while the government of Guam
approved the plan in March 2011, the plan has not moved forward due to continuing public
outcry and the lawsuit. Forward movement completely stopped when, in November, the navy
announced it would conduct a supplemental environmental impact statement (SEIS) to evaluate
alternatives. The SEIS will take a minimum of two years to complete and is not expected to begin
until early 2012. Once completed, the navy will select a location, pushing construction years
away.

The navy has also delayed a decision on where to locate a wharf for transient aircraft carriers. As
part of the Guam buildup, the navy plans to build a new wharf somewhere in Guam’s main
harbor, Apra Harbor. Although the navy never officially chose a site, after a one-year study its
initial plans were narrowed to two locations (Polaris Point or the former Ship Repair Facility)
that required dredging of about 71 acres of coral. However, several U.S. federal agencies,
including the Environmental Protection Agency, the Fish and Wildlife Service, and the National
Marine Fisheries Service requested that the navy conduct a new environmental impact study to
focus not just on coral, but also fishes, sponges, and protected sea turtles at 72 sites in Apra.
There was also opposition from local residents and lawmakers who pressured the navy to use
the San Luis area instead, an area west of the proposed locations that would require less
dredging. Although the initial fieldwork showed little high-quality coral would be removed, the
navy agreed this past autumn to conduct another full study and consider an alternative site.
Because this study could take several years and may require artificial reefs or other forms of
protection, the final decision on location ( let alone completion ) is now several years delayed.

As delays mount, and give rise to new requirements, costs continue to rise. With the final
number of personnel, dependents, and civilians yet to be determined, costs are expected to rise
further still. Worse, it is uncertain how Guam will be able to handle the expected population
increase. There are about 183,000 people living on Guam. According to the navy’s July 2010
Final Environmental Impact Statement, Guam’s permanent population is expected to increase
by about 30,000 (troops, dependents, civilian workers). In addition, the island will see an
estimated short-term increase of 79,000 workers. Both expansions will take a terrible toll on the
island’s infrastructure, including utilities, roads, and water supplies, as well as services, such as
dentists, doctors, clinics, and schools. Until these problems are resolved, which is still many
years away, it is uncertain how quickly the Guam buildup can proceed .

It is clear that the Guam buildup is beset by problems . The decisions by the navy this past
autumn to conduct new studies means that work cannot begin on key portions of the Guam
buildup for at least the next few years. Continuing infrastructure problems may push this back
even further. With key aspects of Guam’s buildup now uncertain, the likelihood of the
Roadmap’s success diminishes further still.

Funding was zeroed out

Jeffrey W. Hornung, Associate Professor at the Asia-Pacific Center for Security Studies in
Honolulu, HI and an Adjunct Fellow with the Office of the Japan Chair at the Center for
Strategic and International Studies, 1/5/12, Time to Acknowledge the Realignment Impasse,

Evaporating Financial Support from Tokyo and Washington

More than anything else, money is needed for the Roadmap to succeed. In 2006, costs for
facility and infrastructure development on Guam were calculated at $10.27 billion. Japan would
provide $6.09 billion and the United States $4.18 billion. Yet, in December, Tokyo and
Washington announced cuts and freezes in their funding . With funds drying up, the death bell
for the realignment plan has rung .

In the United States, key members of the Senate have been critical of DOD’s realignment plans.
In May 2011, Senators John McCain, Carl Levin, and Jim Webb criticized the plans as “unrealistic,
unworkable and unaffordable.” Acting out of a need to address ballooning costs and rising
opposition, they proposed an alternative to sync with current fiscal and political realities,
although it was bereft of many specifics. A Government Accountability Office (GAO) study the
same month supported their thinking, showing that the military did not develop an accurate
cost estimate or appropriately consider alternatives. Instead of the expected $10.27 billion, the
GAO study stated that the cost of the Okinawa and Guam relocations could be closer to $29.1
billion.

As such, it was not surprising that senators cut funding during discussions on the FY2012
National Defense Authorization Act (NDAA). Last month, they slashed the entire $150 million
allocated for the planned relocation of marines from Okinawa to Guam. Additionally, despite the
House of Representatives approving $303 million for military construction projects on Guam,
the Senate reduced it to $83.6 million but froze it until certain conditions are met. Furthermore,
they cut the entire $33 million approved by the House to fund socioeconomic projects directly
related to the Guam transfer and the influx of people (although it was revived in the
Consolidated Appropriations Act).

In the past, funding cuts have later been restored. Yet, because DOD faces automatic cuts of
$600 billion during a financial recession, it is unlikely the NDAA cuts will be restored until DOD
presents a more rigorous analysis of the need and costs of its realignment plan to convince the
Senate.

Worse, Tokyo followed Washington by cutting its funding by about 80 percent. In its initial
budget for FY2011, Tokyo dedicated $667 million (¥52 billion) for the planned relocation. Yet,
following the NDAA cuts, Tokyo decided it had no choice but to reduce its amount to less than
$128 million (¥10 billion) in FY2012. It also is examining whether it should withhold
disbursements for related spending included in the FY2011 budget.

Thus, fiscal realities in Washington and Tokyo have now significantly altered the entire
realignment plan. These developments are the most damaging because they bring the
realignment process to a halt. With little possibility that these cuts will be restored, forward
progress on the Roadmap is now unlikely.

No funding
Ritten 12
Travis J. Ritten, staff writer, Stars and Stripes, May 29, 2012, " Senate committee again blocks
funding for Marines' move off Okinawa", http://www.stripes.com/news/pacific/pacific-base-
relocation/senate-committee-again-blocks-funding-for-marines-move-off-okinawa-1.178866

CAMP FOSTER, Okinawa — Despite a new deal recently struck by the U.S. and Japan, the Senate
is poised to block any funding for the realignment of Marines in Japan and the Pacific for the
second year in a row as it attempts to force the military into offering up better planning and
cost estimates, according to the office of Sen. Jim Webb, D-Va. A Senate Armed Services
Committee draft of the National Defense Authorization Act released last week denies any money
next fiscal year for shifting thousands of Marines from Okinawa to Guam and Hawaii because
the Department of Defense has not provided a schedule of construction projects and costs as
well as an explanation how the move fits into overall strategic goals in the region. The Senate
had already blocked funding this year for the massive reorganization of Marine forces, claiming
previous plans were unaffordable and unrealistic. The effort is supported by powerful Senate
lawmakers , including Carl Levin, D-Mich., and John McCain, R-Ariz., the ranking members of the
committee. However, the defense bill must still be approved by the full Senate in the coming
months and will face a competing House version that includes some funding for Marine Corps
realignment projects in the Pacific. “The NDAA expresses the Senate Armed Services
Committee’s unwillingness to authorize funding for realignment plans until it is provided details
needed to assess the strategic impact, feasibility, and affordability of the lay-down’s initiatives,”
Webb’s office said in a news release over the weekend. In April, the U.S. and Japan announced a
new realignment plan that would cost $8.6 billion to shift about 9,000 Marines off Okinawa. But
that dollar figure is only an initial estimate that is not based on specific construction projects
that would be needed on Guam, where about 5,000 of those Marines will be relocated. The
estimate also does not include costs for moving thousands of Marines to Hawaii, or the
relocation of the controversial Futenma air station on Okinawa, which is expected to be paid for
by Japan. The Government Accountability Office cast doubt over U.S. and Japan cost estimates
last year when it reported that a previous realignment agreement estimated at $10.27 billion
would have actually cost nearly $24 billion when all expenses were tallied. The SASC has again
requested this week the military provide a master plan for the Pacific realignment project that
includes a description of costs and schedules for the facilities and infrastructure needed to shift
the Marines. A master plan has been in the works for years but never completed by the
Department of Defense. Before receiving any funding, the U.S. Pacific Command would also be
required to weigh in on whether the plans provide what is needed to fight wars and assist with
disasters in the region, according to Webb’s office.

Australia base and prepositioning now – solves signal and capability

Bruce Vaughn, CRS Specialist in Asian Affairs, 1/13/12, Australia: Background and U.S.
Relations, http://www.fas.org/sgp/crs/row/RL33010.pdf

During his visit to Australia, President Obama and Prime Minister Gillard announced that the
United States will deploy on a rotational basis up to 2,500 Marines, which are part of a Marine-
Air Ground Task Force, to the Northern Territory and that there will be additional joint air force
cooperation between the two nations. These moves are part of new force posture initiatives
that will significantly enhance defense cooperation between the two nations and will also
include the prepositioning of equipment and supplies.5 The U.S. forces will be housed in
Australian facilities and are part of a U.S. effort to diversify the U.S. military presence in Asia.6 It
was announced that the deployments would begin in the summer of 2012 with an initial group
of 250 Marines. The announcement of the decision to expand the U.S. troop presence in
Australia came after Secretary of State Hillary Clinton’s “America’s Pacific Century” article in
Foreign Policy and has been viewed by many as providing a substantive military component to a
policy that was designed to send a signal to Asia that the United States is firmly committed to
the region. As such, expanded military ties with Australia can be viewed as a key component
that will demonstrate America’s resolve in Asia. The Obama Administration’s decision to
rebalance American strategic priorities from the Middle East to Asia coincides with Australian
strategic priorities that seek to keep America fully engaged in Asia.
Heg Answers
Frontline

No decline and no challengers

Kaplan ‘11, senior fellow – Center for a New American Security, and Kaplan, frmr. vice chairman – National Intelligence Council,
(Robert D and Stephen S, “America Primed,” The National Interest, March/April)

But in spite of the seemingly inevitable and rapid diminution of U.S. eminence, to write America’s
great-power obituary
is beyond premature. The United States remains a highly capable power. Iraq and Afghanistan, as
horrendous as they have proved to be—in a broad historical sense—are still relatively minor events that America can
easily overcome. The eventual demise of empires like those of Ming China and late-medieval Venice was brought
about by far more pivotal blunders. Think of the Indian Mutiny against the British in 1857 and 1858. Iraq in particular—
ever so frequently touted as our turning point on the road to destruction—looks to some extent eerily similar. At the time,
orientalists and other pragmatists in the British power structure (who wanted to leave traditional India as it was) lost some sway to
evangelical and utilitarian reformers (who wanted to modernize and Christianize India—to make it more like England). But the
attempt to bring the fruits of Western civilization to the Asian subcontinent was met with a violent revolt against imperial authority.
Delhi, Lucknow and other Indian cities were besieged and captured before being retaken by colonial forces. Yet, the debacle did not
signal the end of the British Empire at all, which continued on and even expanded for another century. Instead, it signaled the
transition from more of an ad hoc imperium fired by a proselytizing lust to impose its values on others to a calmer and more
pragmatic empire built on international trade and technology.1 There is no reason to believe that the fate of America need follow a
more doomed course. Yes, the mistakes made in Iraq and Afghanistan have been the United States’ own, but, though destructive,
they are not fatal. If we withdraw sooner rather than later, the cost to American power can be stemmed. Leaving a stable
Afghanistan behind of course requires a helpful Pakistan, but with more pressure Washington might increase Islamabad’s
cooperation in relatively short order. In terms of acute threats, Iran is the only state that has exported terrorism and insurgency
toward a strategic purpose, yet the country is economically fragile and politically unstable, with behind-the-scenes infighting that
would make Washington partisans blanch. Even assuming Iran acquires a few nuclear devices—of uncertain quality with uncertain
delivery systems—the long-term outlook for the clerical regime is itself unclear. The administration must only avoid a war with the
Islamic Republic. To be sure, America
may be in decline in relative terms compared to some other powers, as well as
to many countries of the former third world, but
in absolute terms, particularly military ones, the U nited
States can easily be the first among equals for decades hence. China, India and Russia are the only major
Eurasian states prepared to wield military power of consequence on their peripheries. And each, in turn, faces its own obstacles on
the road to some degree of dominance. The Chinese will have a great navy (assuming their economy does not implode) and that will
enforce a certain level of bipolarity in the world system. But Beijing
will lack the alliance network Washington has, even
as China and Russia will always be—because of geography—inherently distrustful of one another.
China has much influence, but no credible military allies beyond possibly North Korea, and its authoritarian regime lives in fear
of internal disruption if its economic growth rate falters. Furthermore, Chinese naval planners look out from their
coastline and see South Korea and a string of islands—Japan, Taiwan and Australia—that are American allies, as are, to a
lesser degree, the Philippines, Vietnam and Thailand. To balance a rising China, Washington must only preserve its naval and air
assets at their current levels. India, which has its own internal insurgency, is bedeviled by semifailed states on its
borders that critically sap energy and attention from its security establishment, and especially from its land forces;
in any case, India has become a de facto ally of the United States whose very rise, in and of itself, helps to balance China. Russia
will be occupied for years regaining influence in its post-Soviet near abroad, particularly in Ukraine,
whose feisty independence constitutes a fundamental challenge to the very idea of the Russian state. China checks Russia in Central
Asia, as do Turkey, Iran and the West in the Caucasus. This is to say nothing of Russia’s diminishing population and overwhelming
reliance on energy exports. Given the problems of these other states, America remains fortunate indeed. The United States is poised
to tread the path of postmutiny Britain. America might not be an empire in the formal sense, but its obligations and constellation of
military bases worldwide put it in an imperial-like situation, particularly because its air and naval deployments will continue in a
post-Iraq and post-Afghanistan world. No country is in such an enviable position to keep the relative peace in Eurasia as is the United
States—especially if it can recover the level of enduring competence in national-security policy last seen during the administration of
George H. W. Bush. This is no small point. America has strategic advantages and can enhance its power
while extricating itself from war. But this requires leadership—not great and inspiring leadership which comes along
rarely even in the healthiest of societies—but plodding competence, occasionally steely nerved and always free of illusion.

Alt causes overwhelm

Copley ’12 (June 2012 (Gregory R., editor of Defense & Foreign Affairs’ Strategic Policy, Strategic Policy in an Age of Global
Realignment, lexis)

3. Strategic Recovery by the US. The US will not, in 2012 or 2013, show signs of any recovery of its
global strategic credibility or real strength. Its manufacturing and science and technology sectors
will continue to suffer from low (even declining) productivity and difficulty in capital formation (for
political reasons, primarily). A significant US recovery is not feasible in the timeframe given the present political and
economic policies and impasse evident. US allies will increasingly look to their own needs while attempting to
sustain their alliance relationship with the US to the extent feasible. Those outside the US alliance network, or
peripheral to it, will increasingly disregard US political/diplomatic pressures , and will seek to
accommodate the PRC or regional actors. The continued economic malaise of the US during
2012, even if disguised by modest nominal GDP growth, will make economic (and therefore
strategic) recovery more difficult and ensure that it will take longer. In any event , the fact that
the US national debt exceeds the GDP hollows the dollar and thus makes meaningful recovery
impossible in the short-term. The attractiveness of a low dollar value in comparison to other currencies in making US
manufacturing investment more feasible than in recent years is offset by declining US workforce productivity and political
constraints which penalize investment in manufacturing, or even in achieving appealing conditions for capital formation. Banks are
as afraid of such investment as are manufacturing investors themselves.

Heg doesn’t solve conflict

Fettweis 10 – Professor of national security affairs @ U.S. Naval War College (Chris, Georgetown University Press, “Dangerous
times?: the international politics of great power peace” 173-75)

Simply stated, the hegemonic stability theory proposes that international peace is only possible when there is one country strong
enough to make and enforce a set of rules. At the height of Pax Romana between 27 BC and 180 AD, for example, Rome was able to
bring unprecedented peace and security to the Mediterranean. The Pax Britannica of the nineteenth century brought a level of
stability to the high seas. Perhaps the current era is peaceful because the United States has established a de facto Pax Americana
where no power is strong enough to challenge its dominance, and because it has established a set of rules that a generally in the
interests of all countries to follow. Without a benevolent hegemony, some strategists fear, instability may break out around the
globe. Unchecked conflicts could cause humanitarian disaster and, in today’s interconnected world economic turmoil that would
ripple throughout global financial markets. If the United States were to abandon its commitments abroad, argued Art, the world
would “become a more dangerous place” and, sooner or later, that would “rebound to America’s detriment.” If the massive
spending that the United States engages in actually produces stability in the international political and economic systems, then
perhaps internationalism is worthwhile. There
are good theoretical and empirical reasons, however, the
belief that U.S. hegemony is not the primary cause of the current era of stability. First of all, the
hegemonic stability argument overstates the role that the United States plays in the system. No
country is strong enough to police the world on its own. The only way there can be stability in
the community of great powers is if self-policing occurs, ifs states have decided that their
interest are served by peace. If no pacific normative shift had occurred among the great powers that
was filtering down through the system, then no amount of international constabulary work by the United
States could maintain stability. Likewise, if it is true that such a shift has occurred, then most of what the hegemon
spends to bring stability would be wasted. The 5 percent of the world’s population that live in the United
States simple could not force peace upon an unwilling 95 . At the risk of beating the metaphor to death, the
United States may be patrolling a neighborhood that has already rid itself of crime. Stability and
unipolarity may be simply coincidental. In order for U.S. hegemony to be the reason for global stability, the rest of
the world would have to expect reward for good behavior and fear punishment for bad. Since the end of the Cold War, the United
States has not always proven to be especially eager to engage in humanitarian interventions abroad. Even rather incontrovertible
evidence of genocide has not been sufficient to inspire action. Hegemonic stability can only take credit for
influence those decisions that would have ended in war without the presence, whether physical
or psychological, of the United States. Ethiopia and Eritrea are hardly the only states that could go to
war without the slightest threat of U.S. intervention. Since most of the world today is free to
fight without U.S. involvement, something else must be at work. Stability exists in many places
where no hegemony is present. Second, the limited empirical evidence we have suggests that
there is little connection between the relative level of U.S. activism and international stability.
During the 1990s the United States cut back on its defense spending fairly substantially , By 1998 the
United States was spending $100 billion less on defense in real terms than it had in 1990. To internationalists, defense
hawks, and other believers in hegemonic stability this irresponsible "peace dividend"
endangered both national and global security "No serious analyst of American military
capabilities," argued Kristol and Kagan, "doubts that the defense budget has been cut much too
far to meet Americas responsibilities to itself and to world peace ."" If the pacific trends were due
not to U.S. hegemony but a strengthening norm against interstate war , however, one would not
have expected an increase in global instability and violence. The verdict from the past two
decades is fairly plain: The world grew more peaceful while the United States cut its forces. No
state seemed to believe that its security was endangered by a less-capable Pentagon, or at
least none took any action that would suggest such a belief. No militaries were enhanced to address power
vacuums; no security dilemmas drove mistrust and arms races; no regional balancing occurred
once the stabilizing presence of the U.S. military was diminished. The rest of the world acted as if
the threat ofinternational war was not a pressing concern, despite the reduction in U.S.
capabilities. The incidence and magnitude of global conflict declined while the United States cut its military spending under
President Clinton, and it kept declining as the Bush Administration ramped spending back up. No complex statistical
analysis should be necessary to reach the conclusion that the two are unrelated . It is also worth noting
for our purposes that the United States was no less safe. Military spending figures by themselves are insufficient to disprove a
connection between overall U.S. actions and international stability. Once again, one could presumably argue that spending is not the
only or even the best indication of hegemony, and that it is instead U.S. foreign political and security commitments that maintain
stability. Since neither was significantly altered during this period, instability should not have been expected. Alternately, advocates
of hegemonic stability could believe that relative rather than absolute spending is decisive in bringing peace. Although the United
States cut back on its spending during the 1990s, its relative advantage never wavered. However, even if it is true that either U.S.
commitments or relative spending account for global pacific trends, then at the very least stability can evidently be maintained at
drastically lower levels of both. In other words, even if one can be allowed to argue in the alternative for a moment and suppose
that there is in fact a level of engagement below which the United States cannot drop without increasing international disorder, a
rational grand strategist would still recommend cutting back on engagement and spending until that level is determined. Grand
strategic decisions are never final; continual adjustments can and must be made as time goes on. Basic logic suggests that the United
States ought to spend the minimum amount of its blood and treasure while seeking the maximum return on its investment. And if
the current era of stability is as stable as many believe it to be, no increase in conflict would ever occur irrespective of U.S. spending,
which would save untold trillions for an increasingly debt-ridden nation. It is also perhaps worth noting that if opposite trends had
unfolded, if other states had reacted to news of cuts in U.S. defense spending with more aggressive or insecure behavior, then
internationalists would surely argue that their expectations had been fulfilled .
If increases in conflict would have been
interpreted as proof of the wisdom of internationalist strategies, then logical consistency
demands that the lack thereof should at least pose a problem. As it stands, the only evidence we have
regarding the likely systemic reaction to a more restrained United States suggests that the
current peaceful trends are unrelated to U.S. military spending. Evidently the rest of the world can
operate quite effectively without the presence of a global policeman. Those who think
otherwise base their view on faith alone.

The only comprehensive study proves no transition impact.

MacDonald and Parent 11—Professor of Political Science at Williams College & Professor of Political Science at
University of Miami [Paul K. MacDonald & Joseph M. Parent, “Graceful Decline? The Surprising Success of Great
Power Retrenchment,” International Security, Vol. 35, No. 4 (Spring 2011), pp. 7–44]

In this article, we
question the logic and evidence of the retrenchment pessimists. To date there has
been neither a comprehensive study of great power retrenchment nor a study that lays out the
case for retrenchment as a practical or probable policy. This article fills these gaps by systematically
examining the relationship between acute relative decline and the responses of great powers.
We examine eighteen cases of acute relative decline since 1870 and advance three main arguments. First, we
challenge the retrenchment pessimists’ claim that domestic or international constraints inhibit the ability of declining great powers
to retrench. In fact, when states fall in the hierarchy of great powers, peaceful
retrenchment is the most common
response, even over short time spans. Based on the empirical record, we find that great powers
retrenched in no less than eleven and no more than fifteen of the eighteen cases, a range of 61–83 percent. When
international conditions demand it, states renounce risky ties , increase reliance on allies or adversaries,
draw down their military obligations, and impose adjustments on domestic populations. Second, we
find that the magnitude of relative decline helps explain the extent of great power retrenchment. Following the dictates of
neorealist theory, greatpowers retrench for the same reason they expand : the rigors of great power
politics compel them to do so.12 Retrenchment is by no means easy, but necessity is the mother of invention, and
declining great powers face powerful incentives to contract their interests in a prompt and proportionate
manner. Knowing only a state’s rate of relative economic decline explains its corresponding degree of
retrenchment in as much as 61 percent of the cases we examined. Third, we argue that the rate of decline
helps explain what forms great power retrenchment will take . How fast great powers fall contributes to
whether these retrenching states will internally reform, seek new allies or rely more heavily on old ones, and make diplomatic
overtures to enemies. Further, our analysis suggests that great powers facing acute decline are less likely to
initiate or escalate militarized interstate disputes. Faced with diminishing resources, great
powers moderate their foreign policy ambitions and offer concessions in areas of lesser strategic value.
Contrary to the pessimistic conclusions of critics, retrenchment neither requires aggression nor invites
predation . Great powers are able to rebalance their commitments through compromise, rather
than conflict . In these ways, states respond to penury the same way they do to plenty: they seek to adopt policies
that maximize security given available means . Far from being a hazardous policy, retrenchment can be successful.
States that retrench often regain their position in the hierarchy of great powers. Of the fifteen great powers that
adopted retrenchment in response to acute relative decline, 40 percent managed to recover
their ordinal rank. In contrast, none of the declining powers that failed to retrench recovered their
relative position. Pg. 9-10
---Heg- No Decline

1NC Kaplan—no risk of heg collapse—no other nation has relative power capabilities of the US
nor the economic or cultural weight—means the US controls the key alliances and trading
blocs that will keep it afloat—other nations don’t have the means or motive to challenge US
power

We control all the key alliances—deter challengers and keep world friendly to US

Bandow, 11 – senior fellow at the Cato Institute. A former special assistant to Ronald Reagan, he is the author of Foreign
Follies: America's New Global Empire (Xulon) [1-31-2011, Doug Bandow, “Solving the Debt Crisis: A Military Budget for a Republic”,
January 31st, http://www.cato.org/pub_display.php?pub_id=12746]

More than two decades after the Cold War dramatically ended, the U.S. maintains a Cold War military .
America has a couple score allies, dozens of security commitments, hundreds of overseas bases, and hundreds of thousands of
troops overseas. Yet international hegemonic communism has disappeared, the Soviet
Union has collapsed, Maoist
China has been transformed, and pro-communist Third World dictatorships have been discarded
in history's dustbin. The European Union has a larger economy and population than America does. Japan
spent decades with the world's second largest economy. South Korea has 40 times the GDP and
twice the population of North Korea. As Colin Powell exclaimed in 1991, "I'm running out of
demons. I'm running out of enemies. I'm down to Castro and Kim Il-sung." Yet America accounts for roughly half of the
globe's military outlays. In real terms the U.S. government spends more on the military today than at any time during the Cold War,
Korean War, or Vietnam War. It
is difficult for even a paranoid to concoct a traditional threat to the
American homeland. Terrorism is no replacement for the threat of nuclear holocaust . Commentator
Philip Klein worries about "gutting" the military and argued that military cuts at the end of the Cold War "came back to haunt us
when Sept. 11 happened." Yet the reductions, which still left America by far the world's most dominant power, neither allowed the
attacks nor prevented Washington from responding with two wars. And responding with two wars turned out to be a catastrophic
mistake. Evil terrorism is a threat, but existential threat it is not. Moreover, the best response is not invasions and occupations — as
the U.S. has learned at high cost in both Afghanistan and Iraq. Rather, the most effective tools are improved intelligence, Special
Forces, international cooperation, and restrained intervention. Attempts at nation-building are perhaps even more misguided than
subsidizing wealthy industrialized states. America's record isn't pretty. The U.S. wasn't able to anoint its preferred Somali warlord as
leader of that fractured nation. Washington's allies in the still unofficial and unstable nation of Kosovo committed grievous crimes
against Serb, Roma, and other minorities. Haiti remains a failed state after constant U.S. intervention. The invasion of Iraq unleashed
mass violence, destroyed the indigenous Christian community, and empowered Iran; despite elections, a liberal society remains
unlikely. After nine years most Afghans dislike and distrust the corrupt government created by the U.S. and sustained only by allied
arms. The
last resort of those who want America to do everything everywhere is to claim that the
world will collapse into various circles of fiery hell without a ubiquitous and vast U.S. military
presence. Yet there is no reason to believe that scores of wars are waiting to break out. And
America's prosperous and populous allies are capable of promoting peace and stability in their
own regions.

Empirically decline claims wrong

Joffe 9. [Josef. Editor of Die Zeit, a Senior Fellow at Stanford’s Institute for Intl Studies, and a Fellow in IR @ the Hoover Institute.
“The Default Power” Foreign Affairs. August 2009. Lexis]
Every ten years, it is decline time in the U nited States. In the late 1950s, it was the Sputnik shock,
followed by the "missile gap" trumpeted by John F. Kennedy in the 1960 presidential campaign. A decade later, Richard Nixon
and Henry Kissinger sounded the dirge over bipolarity, predicting a world of five, rather than two, global powers. At
the end of the 1970s, Jimmy Carter's "malaise" speech invoked "a crisis of confidence" that struck "at the very
heart and soul and spirit of our national will." A decade later, academics such as the Yale historian Paul Kennedy predicted the ruin
of the United States, driven by overextension abroad and profligacy at home. The United States was at risk of "imperial
overstretch," Kennedy wrote in 1987, arguing that "the sum total of the United States' global interests and obligations is
nowadays far larger than the country's power to defend them all simultaneously." But three years later, Washington
dispatched 600,000 soldiers to fight the first Iraq war -- without reinstating the draft or raising taxes. The only
price of "overstretch" turned out to be the mild recession of 1991.

No challengers

Joffe 9. [Josef. Editor of Die Zeit, a Senior Fellow at Stanford’s Institute for Intl Studies, and a Fellow in IR @ the Hoover Institute.
“The Default Power” Foreign Affairs. August 2009. Lexis]

The United States is the default power, the country that occupies center stage because there is nobody else
with the requisite power and purpose . Why not any of the others? On a speculative note, it may take a liberal,
seafaring empire to turn national interests into international public goods. The United Kingdom built a global empire for itself, but in
the process it produced a whole slew of precious public goods: free trade, freedom of the seas, and the gold standard. It
is
difficult to imagine China, India, Japan, Russia, or the EU as guardians of the larger common
interest. The EU comes close, but it has neither the means nor the will to act strategically. Japan,
although rich enough to marshal the means, will continue to huddle under the United States' strategic umbrella
as long as it is extended. India has the size and the population, but apart from being the poorest of them all, it is trapped in a
permanent conflict with Pakistan (and a latent one with China), which monopolizes its resources and attention. China
and
Russia are revisionist powers in business only for themselves. They also lack the right polity. The
United Kingdom and the United States are history's only liberal empires. To labor for a liberal order abroad requires such an order at
home, and so does the habit, sincere or selfish, of articulating the national interest in a universal language. The British Empire's rule
over India was more benign than Belgium's over the Congo under the rapacious reign of King Leopold, and it was also more pleasant
than is China's in Tibet or Russia's in its former Soviet empire. The United States has routinely intervened in Central America -- where
it once kept a lot of nasty company -- but China's rebellious students put up a replica of the Statue of Liberty in Tiananmen Square,
and not one of Lenin's mausoleum. China and Russia might shine forth as models of authoritarian modernization, but to capture a
wider swath of the political imagination, it takes a country that is not just rich but also democratic and free
---Heg- Not Stabilizing

1NC Fettweis—no hegemonic stability—best theoretical and statistical data goes our way—
international norms setting and historical analysis of defense cuts definitively proves heg
doesn’t solve conflict—prefer our ev—their claims are faith-based claims with backing in data

Unipolarity is comparatively meaningless to other factors in preventing conflict

Legro 11 (Jeffrey W. – professor of politics and Randolph P. Compton Professor in the Miller Center at the University of Virginia,
Sell unipolarity? The future of an overvalued concept in International Relations Theory and the Consequences of Unipolarity, p.
EBook)

Such a view, however, is problematic. What seems increasingly clear is that the role of polarity has been overstated or
misunderstood or both. This is the unavoidable conclusion that emerges from the penetrating chapters in this volume that probe
America’s current dominant status (unipolarity) with the question “does the distribution of capabilities matter for patterns of
international politics?”3 Despite the explicit claim that “unipolarity does have a profound impact on
international politics”4 what is surprising is how ambiguous and relatively limited that
influence is across the chapters. The causal impact of unipolarity has been overvalued for three

fundamental reasons. The first is that the effects of unipolarity are often not measured relative to the influence of other
causes that explain the same outcome. When the weight of other factors is considered,
polarity seems to pale in comparison . Second, rather than being a structure that molds states, polarity
oftenseems to be the product of state choice. Polarity may be more outcome than cause. Finally, while
international structure does exist, it is constituted as much by ideational content as by material capabilities.
Again polarity loses ground in significance .

Statistically high probability of war

Monteiro 12– Assistant Professor of Political Science at Yale (Nuno, “Unrest Assured: Why Unipolarity Is Not Peaceful”,
International Security, Vol. 36, No. 3 (Winter 2011/12), pp. 9–40,)

How well, then, does the argument that unipolar systems are peaceful account for the first two decades of unipolarity since the end
of the Cold War? Table 1 presents a list of great powers divided into three periods: 1816 to 1945, multipolarity; 1946 to
1989, bipolarity; and since 1990, unipolarity .46 Table 2 presents summary data about the incidence of war during
each of these periods. Unipolarity is the most conflict prone of all the systems, according to at least two
important criteria: the percentage of years that great powers spend at war and the incidence of
war involving great powers. In multipolarity, 18 percent of great power years were spent at war.
In bipolarity, the ratio is 16 percent. In unipolarity, however, a remarkable 59 percent of great
power years until now were spent at war. This is by far the highest percentage in all three systems.
Furthermore, during periods of multipolarity and bipolarity, the probability that war involving a
great power would break out in any given year was, respectively, 4.2 percent and 3.4 percent.
Under unipolarity, it is 18.2 percent—or more than four times higher.47 These figures provide
no evidence that unipolarity is peaceful.48 In sum, the argument that unipolarity makes for peace is heavily
weighted toward interactions among the most powerful states in the system. This should come as no surprise given that Wohlforth
makes a structural argument: peace ºows from the unipolar structure of international politics, not from any particular characteristic
of the unipole.49 Structural
analyses of the international system are usually centered on interactions
between great powers.50 As Waltz writes, “The theory, like the story, of international politics is written in terms of the
great powers of an era.”51 In the sections that follow, however, I show that in the case of unipolarity, an investigation of its
peacefulness must consider potential causes of conflict beyond interactions between the most important states in the system.

Be skeptical of their claims—no studies prove stability theory

Montiero, 12 - Assistant Professor of Political Science at Yale University (Nuno, “Unrest Assured: Why Unipolarity is Not
Peaceful” International Security, Winter, http://www.mitpressjournals.org/doi/pdf/10.1162/ISEC_a_00064)

In contrast, the question of unipolar peacefulness has received virtually no attention. Although the past decade has
witnessed a resurgence of security studies, with much scholarship on such conflict-generating issues as terrorism, preventive war,
military occupation, insurgency, and nuclear proliferation, no one has systematically connected any of them to
unipolarity. This silence is unjustified. The first two decades of the unipolar era have been anything but peaceful. U.S. forces
have been deployed in four interstate wars: Kuwait in 1991, Kosovo in 1999, Afghanistan from 2001 to the present, and
Iraq between 2003 and 2010. 22 In all, the United States has been at war for thirteen of the twenty-two years since the end of the
Cold War. 23 Put another way, the first two decades of unipolarity, which make up less than 10 percent of U.S. history,
account for more than 25 percent of the nation’s total time at war . 24 And yet, the theoretical consensus
continues to be that unipolarity encourages peace. Why? To date, scholars do not have a theory of how unipolar
systems operate. 25 The debate on whether, when, and how unipolarity will end (i.e., the debate on durability) has all but
monopolized our attention.

Psychological bias in academy—prefer our ev

Fettweis 10 – Professor of national security affairs @ U.S. Naval War College (Chris, Georgetown University Press, “Dangerous
times?: the international politics of great power peace” Google Books)

Hegemony’s Psychological Appeal

Raison d'etat cannot entirely account for the anathematic status of strategic restraint . Many
people simply prefer internationalism and enjoy the prestige it appears to confer. It is human to desire greatness,
to want to belong to the best team, political party or state . While all people everywhere take pride in their
country or their culture, Americans have long been exceptional in their exceptionalism. " The pleasure and
pride that the citizens of Rome felt toward their empire is similar to that which Americans hold
toward their republic. Like all people, they do not readily accept being second-best in anything, from
math scores to basketball to automobile quality. "Americans love a winner," Patton told his troops on the eve of D-Day, "and will
not tolerate a loser. Americans play to win, all the time .... The
very thought of losing is hateful to an American ."
Being "number one" has a cachet that will not soon weaken as long as people are competitive by nature. We all
like to bask in the reflected glory of national greatness. Triumphalism extends beyond the
masses into the halls of government and ivorytowers of academia. Some of the more strident
internationalists clearly feel hostility toward the idea of sharing the stage with other powers not
so much because of actual threats such a situation might pose (since they know better than anyone that such
threats are minimal), but rather because they recoil from the notion that the United States should
relinquish its title as de facto champion of the world without a struggle. Strategic restraint to some people
would herald the end ofthe American Century and all the glory and prestige that accompanies it.
Schlesinger wistfully implored his restraint-minded countrymen to "recognize, as we return to the womb, that we are surrendering a
magnificent dream."" Few
people make international affairs their chosen profession in order to
recommend that the United States withdraw from most international affairs. Strategists are
professionally predisposed to favor internationalism, if for no other reason than that it is more
interesting and appealing than restraint. The national honor, after all, is at stake,

Multiple safety nets solve stability—hegemony unnecessary

Preble 10 - director of foreign policy studies at the Cato Institute, taught history at St. Cloud State University and Temple
University, was a commissioned officer in the U.S. Navy, Ph.D. in history from Temple University (Christopher, 8/13, “U.S. Military
Power: Preeminence for What Purpose?”) http://www.cato-at-liberty.org/u-s-military-power-preeminence-for-what-purpose/)

Most in Washington still embraces the notion that America is, and forever will be, the
world’s indispensable nation. Some scholars, however, questioned the logic of hegemonic
stability theory from the very beginning. A number continue to do so today . They advance
arguments diametrically at odds with the primacist consensus. Trade routes need not be
policed by a single dominant power; the international economy is complex and resilient.
Supply disruptions are likely to be temporary, and the costs of mitigating their effects
should be borne by those who stand to lose — or gain — the most. Islamic extremists are
scary, but hardly comparable to the threat posed by a globe-straddling Soviet Union armed
with thousands of nuclear weapons. It is frankly absurd that we spend more today to fight Osama
bin Laden and his tiny band of murderous thugs than we spent to face down Joseph
Stalin and Chairman Mao. Many factors have contributed to the dramatic decline in the
number of wars between nation-states; it is unrealistic to expect that a new spasm of
global conflict would erupt if the United States were to modestly refocus its efforts, draw
down its military power, and call on other countries to play a larger role in their own
defense, and in the security of their respective regions. But while there are credible
alternatives to the United States serving in its current dual role as world policeman / armed social worker, the
foreign policy establishment in Washington has no interest in exploring them. The people here have grown accustomed to
living at the center of the earth, and indeed, of the universe. The tangible benefits of all this military spending flow
disproportionately to this tiny corner of the United States while the schlubs in fly-over country pick up the tab. In short, we
shouldn’t have expected that a group of Washington insiders would seek to overturn the judgments of another group of
Washington insiders. A genuinely independent assessment of U.S. military spending, and of the strategy the military is
designed to implement, must come from other quarters.

No impact to heg- can’t solve conflicts

Preble ‘9 (Christopher A. Preble is Director of Foreign Policy Studies at the Cato Institute and a former commissioned officer in
the U.S. Navy, “The Power Problem,” 3/17/2009, ISBN#: 978-0-8014-4765-5, Google Books
The conceptions of national interested foisted on the American people by leaders in both major political parties, and the
rationales and justifications put forward for military action to safeguard our supposedly tenuous
security, are based on the simple proposition that the world sitting atop a combustible log pile,
that every incipient conflict can become the spark that engulfs the planet , and that the United
States is the only country with a bucket of water to extinguish the spark before it ignites a flame.7 Madeleine
Albright’s confident assertion that “We stand tall and we see further than other countries into the future, and we see the danger
here to all of us,” nicely encapsulates the dominant worldview among policymakers in Washington. Believing
that every
simmering ethnic and sectarian conflict is likely to bloom into full-scale war, Washington
contemplates sending the U.S. military into the middle of these squabbles. Believing that demographic
trends will precipitate a pell-mell scramble for scarce resources, and that these scrambles are
likely to turn violent, U.S. policymakers offer to preserve a peaceful global economic order . Seeing
every tin-pot tyrant with a megaphone as the next Adolf Hitler, someone in Washington makes plans to whack them before they
realize their wicked ambitions. As Senator John McCain proclaimed in his defense of the first Gulf War, if America failed to act to
reverse Hussein’s aggression against Kuwait, “there will be inevitably a succession of dictators” that would present “a threat to the
stability of this entire globe.”8 In 1994, the Atlantic Monthly’s Robert Kaplan warned of an equally alarming prospect: “the coming
anarchy.” In Kaplan’s view, Western strategists needed to start concerning themselves with “what is occurring…throughout West
Africa and much of the underdeveloped world: the withering away of central governments, the rise of tribal and regional domains,
the unchecked spread of disease, and the growing pervasiveness of war.”9 Less than two years later, William Kristol and Robert
Kagan wrote, “American hegemony is the only reliable defense against a breakdown of peace
and international order.”10 That was not true in 1994, or 1996, and it is not true today. Although the
stated rationales have changed since 9/11, with various advocacy groups using the fear of future terrorist attacks as a vehicle for
pushing their pet projects—from nuclear disarmament, to removing Saddam Hussein, to averting global climate change, to ending
the so-called U.S. addiction to oil—the underlying reality has not. Most of the time, our government has no business
sending U.S. troops into the middle of foreign tangles, because most military interventions have
only the slightest connection to the problems they purport to solve , and, to the extent that
there is a connection, the use of the military is likely to exacerbate the problem . We are left,
then, with the same flawed rationales as before 9/11: the erroneous belief that the United States is
the only country on the planet with the wisdom, foresight, and capacity to propel the planet
toward the future, and the related notion that we alone are capable of preventing the world
from descending into total, bloody chaos. People who favor the United States performing the
role of global sheriff envision the world as both more threatening and simpler than it actually
is.11 Indeed, to read much of what passes for serious discussion in foreign policy circles today, one might conclude that the United
States isn’t simply the world’s indispensable nation, but rather that it is the world’s only nation, or at least the only nation with the
sense and the foresight to even have a foreign policy in the first place. But our fear of instability is largely overblown:
failed sates and civil wars rarely represent security threats to the United States. Such
conditions, however, often represent security threats to other states, usually nearby states, that
should be expected to deal with most such crises long before they engulf a particular region , let
alone consume the planet. At the same time, the world is vastly more complex than the
interventionists would have it: overthrowing undemocratic regimes, fixing failed states, or
stopping civil wars or ethnic and sectarian violence, are all exceptionally difficult tasks . It could be
reasonably argued that in an era of transnational threats and weapons of mass destruction we can’t afford to be absolutely certain
that a threat will materialize, and that in such circumstances the costs of inaction are outweighed by the costs of action. President
Bush made that argument explicitly with respect to Iraq.12 But Bush’s logic was intended to apply not merely to one case, but to all
cases. Speaking to West Point Cadets in June 2002, he declared: “In the world we have entered, the only path to safety is the path of
action. And this nation will act.”13
---Heg- No Transition Wars

1NC McDonald and Parent—no transition war—best and most comprehensive study indicates
transitions are entirely peaceful over 80% of the team—declining nations are less likely to
escalate conflicts but still have the deterrent capability to prevent flare-ups—prefer our ev,
indicts their unresearched claims with statistical empirical data

Hegemonic decline will not result in great power wars.

Ikenberry 11 (G. John – Albert G. Milbank Professor of Politics and International Affairs at Princeton University, A World of
Our Making, Democracy: A Journal of Ideas, Summer, p. http://www.democracyjournal.org/21/a-world-of-our-making-1.php?
page=all)

There are four reasons to think that some type of updated and reorganized liberal international order will
persist . First, the old and traditional mechanism for overturning international order— great-
power war—is no longer likely to occur . Already, the contemporary world has experienced the
longest period of great-power peace in the long history of the state system. This absence of great-power
war is no doubt due to several factors not present in earlier eras, namely nuclear deterrence and the
dominance of liberal democracies. Nuclear weapons—and the deterrence they generate—give great powers
some confidence that they will not be dominated or invaded by other major states. They make war among major states

less rational and there-fore less likely . This removal of great-power war as a tool of
overturning international order tends to reinforce the status quo. The U nited S tates was lucky to
have emerged as a global power in the nuclear age, because rival great powers are put at a
disadvantage if they seek to overturn the American-led system. The cost-benefit calculation of rival
would-be hegemonic powers is altered in favor of working for change within the system . But, again, the fact
that great-power deterrence also sets limits on the projection of American power presumably makes the existing international order
more tolerable. It removes a type of behavior in the system—war, invasion, and conquest between great powers—that historically
provided the motive for seeking to overturn order. If the violent over-turning of international order is
removed , a bias for continuity is introduced into the system. Second, the character of liberal international order
itself— with or without American hegemonic leadership — reinforces continuity . The complex
interdependence that is unleashed in an open and loosely rule-based order generates expanding realms of exchange and
investment that result in a growing array of firms, interest groups, and other sorts of political stakeholders who
seek to preserve the stability and openness of the system . Beyond this, the liberal order is also relatively
easy to join. In the post-Cold War decades, countries in different regions of the world have made democratic transitions and
connected themselves to various parts of this system. East European countries and states within the old Soviet empire have joined
NATO. East Asian countries, including China, have joined the World Trade Organization (WTO). Through its many multilateral
institutions, the liberal international order facilitates integration and offers support for states that are making
transitions toward liberal democracy. Many countries have also experienced growth and rising incomes within this order. Comparing
international orders is tricky, but the current liberal international order , seen in comparative perspective, does
appear to have unique characteristics that encourage integration and discourage opposition and
resistance. Third, the states that are rising today do not constitute a potential united opposition bloc to the existing order. There are
so-called rising states in various regions of the world. China, India, Brazil, and South Africa are perhaps most prominent. Russia is
also sometimes included in this grouping of rising states. These states are all capitalist and most are democratic. They all gain from
trade and integration within the world capitalist system. They all either are members of the WTO or seek membership in it. But they
also have very diverse geopolitical and regional interests and agendas. They do not constitute either an economic bloc or a
geopolitical one. Their ideologies and histories are distinct. They share an interest in gaining access to the leading institutions that
govern the international system. Sometimes this creates competition among them for influence and access. But it also orients their
struggles toward the reform and reorganization of governing institutions, not to a united effort to overturn the underlying order.
Fourth, all the great powers have alignments of interests that will continue to bring them together
to negotiate and cooperate over the management of the system. All the great powers—old and rising—are
status-quo powers. All are beneficiaries of an open world economy and the various services that the liberal
international order provides for capitalist trading states. All worry about religious radicalism and failed states. Great
powers such as Russia and China do have different geopolitical interests in various key trouble spots, such as Iran and South Asia,
and so disagreement and noncooperation over sanctions relating to nonproliferation and other security issues will not disappear.
But the opportunities for managing differences with frameworks of great-power cooperation exist and
will grow. Overall, the forces for continuity are formidable . Of course, there are many forces operating
in the world that can generate upheaval and discontinuity. The collapse of the global financial
system and an economic depression that triggers massive protectionism are possibilities. Terrorism and other forms of
transnational violence can also trigger political panic and turmoil that would lead governments to shut down borders and
reimpose restrictions on the movement of goods and people. But in the face of these seismic events in world politics,

there are deep forces that keep the system anchored and stable .

Trade routes and globalization check conflict post-decline

Preble 10 [Christopher Preble (director of foreign policy studies at the Cato Institute) August 2010 “U.S. Military Power:
Preeminence for What Purpose?” http://www.cato-at-liberty.org/u-s-military-power-preeminence-for-what-purpose/]

Most in Washington still embraces the notion that America is, and forever will be, the world’s indispensable nation. Some
scholars, however, questioned the logic of hegemonic stability theory from the very beginning. A number
continue to do so today. They advance arguments diametrically at odds with the primacist consensus. Trade routes need
not be policed by a single dominant power; the international economy is complex and
resilient. Supply disruptions are likely to be temporary, and the costs of mitigating their
effects should be borne by those who stand to lose — or gain — the most. Islamic extremists are
scary, but hardly comparable to the threat posed by a globe-straddling Soviet Union armed with thousands of nuclear weapons. It is
frankly absurd that we spend more today to fight Osama bin Laden and his tiny band of murderous thugs than we spent to face
down Joseph Stalin and Chairman Mao. Many
factors have contributed to the dramatic decline in the
number of wars between nation-states; it is unrealistic to expect that a new spasm of
global conflict would erupt if the United States were to modestly refocus its efforts,
draw down its military power, and call on other countries to play a larger role in their own
defense, and in the security of their respective regions. But while there are credible alternatives to the
United States serving in its current dual role as world policeman / armed social worker, the foreign policy establishment in
Washington has no interest in exploring them. The people here have grown accustomed to living at the center of the earth, and
indeed, of the universe. The tangible benefits of all this military spending flow disproportionately to this tiny corner of the United
States while the schlubs in fly-over country pick up the tab
Deterrence and liberalism ensure NO conflict is likely to erupt

Ikenberry 11 (G. John, “A World of our Making” http://www.democracyjournal.org/21/a-world-of-our-making-1.php?


page=all)

There are four reasons to think that some type of updated and reorganized liberal international order will persist. First,
the
old and traditional mechanism for overturning international order— great-power war—is no
longer likely to occur. Already, the contemporary world has experienced the longest period of
great-power peace in the long history of the state system. This absence of great-power war is no doubt due to several
factors not present in earlier eras, namely nuclear deterrence and the dominance of liberal democracies.
Nuclear weapons—and the deterrence they generate—give great powers some confidence that they
will not be dominated or invaded by other major states. They make war among major states less rational
and there-fore less likely. This removal of great-power war as a tool of overturning international order tends to reinforce the
status quo. The United States was lucky to have emerged as a global power in the nuclear age, because rival great powers are
put at a disadvantage if they seek to overturn the American-led system. The cost-benefit calculation of rival would-be
hegemonic powers is altered in favor of working for change within the system. But, again, the
fact that great-power
deterrence also sets limits on the projection of American power presumably makes the
existing international order more tolerable. It removes a type of behavior in the system—
war, invasion, and conquest between great powers—that historically provided the motive for
seeking to overturn order. If the violent over-turning of international order is removed, a bias for continuity is
introduced into the system.

Regional cooperation fills in—in states’ best interest

Sachs, 11 – Director of The Earth Institute, Quetelet Professor of Sustainable Development, and Professor of Health Policy and
Management at Columbia University. He is also Special Advisor to United Nations Secretary-General Ban Ki-moon (Jeffrey, “A World
of Regions,” 5/26, http://www.social-europe.eu/2011/05/a-world-of-regions/)

In almost every part of the world, long-festering problems can be solved through closer cooperation among neighboring countries. The European Union
provides the best model for how neighbors that have long fought each other can come together for mutual benefit. Ironically, today’s
decline
in American global power may lead to more effective regional cooperation. This may seem an odd time to praise
the EU, given the economic crises in Greece, Spain, Portugal, and Ireland. Europe has not solved the problem of balancing the interests of strong
economies in the North and those of weaker economies in the South. Still, the EU’s accomplishments vastly outweigh its current difficulties. The EU has
created a zone of peace where once there was relentless war. It has provided the institutional framework for reuniting Western and Eastern Europe. It
has fostered regional-scale infrastructure. The single market has been crucial to making Europe one of the most prosperous places on the planet. And
the EU has been a global leader on environmental sustainability. For these reasons, the
EU provides a unique model for other
regions that remain stuck in a mire of conflict, poverty, lack of infrastructure, and environmental
crisis. New regional organizations, such as the African Union, look to the EU as a role model for regional
problem-solving and integration. Yet, to this day, most regional groupings remain too weak to solve their members’ pressing
problems. In most other regions, ongoing political divisions have their roots in the Cold War or the colonial era. During the Cold War, neighbors often
competed with each other by “choosing sides” – allying themselves with either the United States or the Soviet Union. Pakistan tilted towards the
Americans; India towards the Soviets. Countries had little incentive to make peace with their neighbors as long as they enjoyed the financial support of
the US or the USSR. On the contrary, continued conflict often led directly to more financial aid. Indeed, the US and Europe often acted to undermine
regional integration, which they believed would limit their roles as power brokers. Thus, when Gamal Abdel Nasser launched a call for Arab unity in the
1950’s, the US and Europe viewed him as a threat. The US undercut his call for strong Arab cooperation and nationalism, fearing a loss of American
influence in the Middle East. As a result, Nasser increasingly aligned Egypt with the Soviet Union, and ultimately failed in the quest to unite Arab
interests. Today’s reality, however, is that great powers can no longer divide and conquer other regions, even if they try. The age of colonialism is
finished, and we
are now moving beyond the age of US global dominance. Recent events in the
Middle East and Central Asia, for example, clearly reflect the decline of US influence. America’s
failure to win any lasting geopolitical advantage through the use of military force in Iraq and
Afghanistan underscore the limits of its power, while its budget crisis ensures that it will cut its military resources sooner
rather than later. Similarly, the US played no role in the political revolutions underway in the Arab world ,
and still has not demonstrated any clear policy response to them. President Barack Obama’s recent speech on the Middle East is a further display of
America’s declining influence in the region. The speech drew the most attention for calling on Israel to return to its 1967 borders, but the effect was
undercut when Israel flatly rejected the US position. The world could see that there would be little practical follow-up. The rest of the speech was even
more revealing, though it drew little public notice. When Obama discussed the Arab political upheavals, he noted the importance of economic
development. Yet when it came to US action, the most that the US could offer financially was slight debt relief for Egypt ($1 billion), scant loan
guarantees ($1 billion), and some insurance coverage for private investments. The real message was that the US government would contribute very
little financially to the region’s economic recovery. The days when a country could depend on large-scale American financing are over. We are, in short,
moving to a multi-polar world. The Cold War’s end has not led to greater US dominance, but rather to the dissemination of global power to many
regions. East Asia, South Asia, Latin America, and the Middle East have new geopolitical and economic influence. Each region, increasingly, must find its
own path to economic development, energy and food security, and effective infrastructure, and must do so in a world threatened by climate change
and resource scarcity. Each region, therefore, will have to secure its own future. Of course, this should occur in a context of cooperation across regions
as well as within them. The
Middle East is in a strong position to help itself. There is a high degree of
economic complementarity between Egypt and the oil-rich Gulf States. Egypt can supply technology, manpower, and
considerable expertise for the Arab region, while the Gulf provides energy and finance, as well as some specialists. The long-delayed vision of Arab
economic unity should be returned to the table. Israel, too, should recognize that its long-term security and prosperity will be enhanced as part of an
economically stronger region. For the sake of its own national interests, Israel must come to terms with its neighbors. Other
regions also
will find that the decline of US power increases the urgency of stronger cooperation between
neighbors. Some of the greatest tensions in the world – say India and Pakistan, or North and South Korea – should
be defused as part of region-wide strengthening. As the EU shows, ancient enmities and battle lines can be turned into
mutually beneficial cooperation if a region looks forward, to resolving its long-term needs, rather than backward, to its long-standing rivalries and
conflicts.

It’s true of all rising nations – BRIC countries have a stake in the international system.

Ikenberry 11 (G. John, “A World of our Making” http://www.democracyjournal.org/21/a-world-of-our-making-


1.php?page=all)

Third,the states that are rising today do not constitute a potential united opposition bloc to
the existing order. There are so-called rising states in various regions of the world. China, India, Brazil, and
South Africa are perhaps most prominent. Russia is also sometimes included in this grouping
of rising states. These states are all capitalist and most are democratic. They all gain from
trade and integration within the world capitalist system. They all either are members of the
WTO or seek membership in it. But they also have very diverse geopolitical and regional interests and agendas. They
do not constitute either an economic bloc or a geopolitical one. Their ideologies and histories are distinct. They share an interest
in gaining access to the leading institutions that govern the international system. Sometimes this creates competition among
them for influence and access.
But it also orients their struggles toward the reform and
reorganization of governing institutions, not to a united effort to overturn the underlying
order.

Deductive and empirical evidence goes our way

Parent 11—assistant for of pol sci, U Miami. PhD in pol sci, Columbia—and—Paul MacDonald—assistant prof of pol sci, Williams
(Joseph, Graceful Decline?;The Surprising Success of Great Power Retrenchment, Intl. Security, Spring 1, p. 7)

Some observers might dispute our conclusions, arguing that hegemonic transitions are more conflict prone
than other moments of acute relative decline. We counter that there are deductive and empirical reasons to
doubt this argument. Theoretically, hegemonic powers should actually find it easier to manage acute
relative decline. Fallen hegemons still have formidable capability, which threatens grave harm to any
state that tries to cross them. Further, they are no longer the top target for balancing coalitions, and recovering
hegemons may be influential because they can play a pivotal role in alliance formation. In addition, hegemonic powers, almost by
definition, possess more extensive overseas commitments; they should be able to more readily identify and eliminate extraneous
burdens without exposing vulnerabilities or exciting domestic populations. We believe the
empirical record supports
these conclusions. In particular, periods of hegemonic transition do not appear more conflict prone
than those of acute decline. The last reversal at the pinnacle of power was the AngloAmerican
transition, which took place around 1872 and was resolved without armed confrontation. The tenor of that
transition may have been inºuenced by a number of factors: both states were democratic maritime empires, the United States was
slowly emerging from the Civil War, and Great Britain could likely coast on a large lead in domestic capital stock. Although China and
the United States differ in regime type, similar
factors may work to cushion the impending Sino-American
transition. Both are large, relatively secure continental great powers, a fact that mitigates
potential geopolitical competition. 93 China faces a variety of domestic political challenges,
including strains among rival regions, which may complicate its ability to sustain its economic
performance or engage in foreign policy adventurism. 9

No causal chain between decline and conflict

Goldstein ‘11, Professor IR at American University [Joshua S. Goldstein, Professor emeritus of international relations at
American University, “Thing Again: War,” Sept/Oct 2011,

http://www.foreignpolicy.com/articles/2011/08/15/think_again_war?print=yes&hidecomments=yes&page=full]

Nor do shifts in the global balance of power doom us to a future of perpetual war. While some
political scientists argue that an increasingly multipolar world is an increasingly volatile one -- that peace is
best assured by the predominance of a single hegemonic power, namely the United States -- recent geopolitical history
suggests otherwise. Relative U.S. power and worldwide conflict have waned in tandem over the past
decade. The exceptions to the trend, Iraq and Afghanistan, have been lopsided wars waged by the
hegemon, not challenges by up-and-coming new powers. The best precedent for today's emerging world order
may be the 19th-century Concert of Europe, a collaboration of great powers that largely maintained the peace for a century until its
breakdown and the bloodbath of World War I.
---Heg- AT: Vague Threats
Their authors overestimate the importance of the US heg

Fettweis 11 [Christopher J. Fettweis - Department of Political Science Tulane University and Professor of National Security
Affairs at the US Naval War College, “Free Riding or Restraint Examining European Grand Strategy”, Comparative Strategy;
Sep/Oct2011, Vol. 30 Issue 4, p316-332, 17p,]

Assertions that without the combination of U.S. capabilities, presence and commitments instability
would return to Europe and the Pacific Rim are usually rendered in rather vague language . If the United States
were to decrease its commitments abroad, argued Robert Art, “the world will become a more dangerous place
and, sooner or later, that will redound to America’s detriment.”53 From where would this danger arise? Who
precisely would do the fighting, and over what issues? Without the United States, would Europe really
descend into Hobbesian anarchy? Would the Japanese attack mainland China again, to see if they could
fare better this time around? Would the Germans and French have another go at it? In other words, where exactly is
hegemony is keeping the peace? With one exception, these questions are rarely addressed. That
exception is in the Pacific Rim. Some analysts fear that a de facto surrender of U.S. hegemony would lead to a rise of Chinese
influence. Bradley Thayer worries that Chinese would become “the language of diplomacy, trade and commerce, transportation and
navigation, the internet, world sport, and global culture,” and that Beijing would come to “dominate science and technology, in all
its forms” to the extent that soon theworldwould witness a Chinese astronaut who not only travels to the Moon, but “plants the
communist flag on Mars, and perhaps other planets in the future.”54 Indeed Chin a is the only other major power that has increased
its military spending since the end of the Cold War, even if it still is only about 2 percent of its GDP. Such levels of effort do not
suggest a desire to compete with, much less supplant, the United States. The much-ballyhooed, decade-long military
buildup has brought Chinese spending up to somewhere between one-tenth and one-fifth of the
U.S. level. It is hardly clear that a restrained United States would invite Chinese regional, must less
global, political expansion. Fortunately one need not ponder for too long the horrible specter of a red flag on Venus, since on the
planet Earth, where war is no longer the dominant form of conflict resolution, the threats posed by even a rising China would not be
terribly dire. The dangers contained in the terrestrial security environment are less severe than ever before. Believers in the
pacifying power of hegemony ought to keep in mind a rather basic tenet: When it comes to policymaking,
specific threats are more significant than vague, unnamed dangers. Without specific risks, it is just as
plausible to interpret U.S. presence as redundant, as overseeing a peace that has already arrived. Strategy should not be
based upon vague images emerging from the dark reaches of the neoconservative imagination.
Overestimating Our Importance One of the most basic insights of cognitive psychology provides the final
reason to doubt the power of hegemonic stability: Rarely are our actions as consequential upon
their behavior as we perceive them to be. A great deal of experimental evidence exists to support the
notion that people (and therefore states) tend to overrate the degree to which their behavior is
responsible for the actions of others. Robert Jervis has argued that two processes account for this overestimation,
both ofwhichwould seem to be especially relevant in theU.S. case. 55 First, believing that we are responsible for their
actions gratifies our national ego (which is not small to begin with; the United States is exceptional in its
exceptionalism). The hubris of the United States, long appreciated and noted, has only grown with the collapse of the Soviet
Union.56 U.S. policymakers famously have comparatively little knowledge of—or interest in—
events that occur outside of their own borders . If there is any state vulnerable to the
overestimation of its importance due to the fundamental misunderstanding of the motivation of
others, it would have to be the United States. Second, policymakers in the United States are far more familiar with
our actions than they are with the decision-making processes of our allies. Try as we might , it is not possible to fully
understand the threats, challenges, and opportunities that our allies see from their perspective.
The European great powers have domestic politics as complex as ours, and they also have competent, capable strategists to chart
their way forward. They react to many international forces, of which U.S. behavior is only one.
Therefore, for any actor trying to make sense of the action of others, Jervis notes, “in the absence of strong evidence to the
contrary, the most obvious and parsimonious explanation is that he was responsible.”57 It
is natural, therefore, for U.S.
policymakers and strategists to believe that the behavior of our allies (and rivals) is shaped largely by
what Washington does. Presumably Americans are at least as susceptible to the overestimation of their ability as any other
people, and perhaps more so. At the very least, political psychologists tell us, we are probably not as important to
them as we think. The importance of U.S. hegemony in contributing to international stability is
therefore almost certainly overrated . In the end, one can never be sure why our major allies have not gone to, and do
not even plan for, war. Like deterrence, the hegemonic stability theory rests on faith; it can only be
falsified, never proven . It does not seem likely, however, that hegemony could fully account for twenty years of strategic
decisions made in allied capitals if the international system were not already a remarkably peaceful place. Perhaps these
states have no intention of fighting one another to begin with , and our commitments are redundant.
European great powers may well have chosen strategic restraint because they feel that their security is all but assured, with or
without the United States.
---Heg- AT: BWI

Brooks Ikenberry and Wohlforth are incorrect- hegemony causes entanglement, balancing,
and nuclear weapons are sufficient

Walt ’13 (Stephen M. Walt, “More or less: The debate on U.S. grand strategy”, January 2, 2013)

If you'd like to start 2013 by sinking your teeth into the debate on U.S. grand strategy, I recommend you start with two pieces in the
latest issue of Foreign Affairs. Both are by good friends of mine, and together they nicely limn the contours of a useful debate on
America's global role. It's also worth noting that there are realists on both sides of this particular exchange, which reminds us that
agreement on fundamental principles doesn't necessarily yield agreement on policy conclusions. The first piece is Barry Posen's "Pull
Back: The Case for a Less Activist Foreign Policy," and the
second is Stephen Brooks, John Ikenberry, and
William Wohlforth's "Lean Forward: In Defense of American Engagement ." (A longer version of the B, I
& W argument can be found in the latest issue of International Security; Posen's argument is outlined at length in a forthcoming
book.) Dedicated readers of this blog know that I am largely in agreement with Posen's position, so I'm going to focus
today on what I find lacking in B, I & W. Like all of their work, it's vigorously argued and the longer version is richly
documented. But all those footnotes cannot save it from some serious weaknesses. First, B, I, & W
straw-man their target by lumping together a group of strategic thinkers whose differences
are at least as significant as their points of agreement . The "proponents of retrenchment" that
they criticize range from libertarian isolationists who want to bring virtually all US forces home to "offshore
balancers" like Posen who support a robust but less extravagant defense budget and favor not isolationism but merely more
limited forms of international engagement. Needless to say, there is a world of difference in these views (even
if both are broadly in favor of doing less), and so many of B, I & W's broad-brush charges miss
their mark. Second, there is something deeply puzzling about B, I & W's devotion to what
Ikenberry used to called "liberal hegemony," and what he and his co-authors now prefer to call "deep
engagement." B, I & W argue that deep engagement has been America's grand strategy since World War II and they believe it
was the optimal strategy for the bipolar Cold War, when the United States faced a global threat from a major great-power rival. Not
only was the USSR a formidable military power, but it was also an ideological rival whose Marxist-Leninist principles once
commanded millions of loyal followers around the world. Here's
the puzzle: the Soviet Union disappeared in
1992, and no rival of equal capacity has yet emerged. Yet somehow "deep engagement" is still
the optimal strategy in these radically different geopolitical circumstances . It's possible that U.S.
leaders in the late 1940s hit on the ideal grand strategy for any and all structural conditions, but it is surely odd that an event as
significant as the Soviet collapse can have so few implications for how America deals with the other 190-plus countries around the
globe. Third,B, I, & W give "deep engagement" full credit for nearly all the good things that
have occurred internationally since 1945 (great power peace, globalization, non-proliferation,
expansion of trade, etc.), even though the direct connection between the strategy and these
developments remains contested. More importantly, they absolve the strategy from most if not all
of the negative developments that also took place during this period. They recognize the events
like the Indochina War and the 2003 war in Iraq were costly blunders, but they regard them as
deviations from "deep engagement" rather than as a likely consequence of a strategy that
sees the entire world as of critical importance and the remaking of other societies along
liberal lines as highly desirable if not strategically essential. The problem, of course, is that U.S. leaders can
only sell deep engagement by convincing Americans that the nation's security will be fatally compromised if they do not get busy
managing the entire globe. Because the United States is in fact quite secure from direct attack and/or conquest, the only way to do
that is by ceaseless threat-mongering, as has been done in the United States ever since the Truman Doctrine, the first Committee on
the Present Danger and the alarmist rhetoric of NSC-68. Unfortunately, threat-mongering requires people in the
national security establishment to exaggerate U.S. interests more-or-less constantly and to
conjure up half-baked ideas like the domino theory to keep people nervous. And once a
country has talked itself into a properly paranoid frame of mind, it inevitably stumbles into
various quagmires, as the United States did in Vietnam, Iraq, and Afghanistan. Again, such
debacles are not deviations from "deep engagement"; they are a nearly inevitable
consequence of it. Fourth, B, I, & W largely ignore the issue of opportunity cost. Advocates of
restraint like Posen (and myself) are not saying that the United States cannot afford to intervene in lots of overseas venues, they are
saying that the
United States would be better off with a smaller set of commitments and a more
equitable division of labor between itself and its principal allies. If the United States were not spending
more than more of the world combined on "deep engagement," it could invest more in
infrastructure here at home, lower taxes, balance budgets more easily, provide more
generous health or welfare benefits, or do whatever combination of the above the public
embraced. Fifth, B, I, & W argue that deep engagement works because hardly anybody is
actively trying to balance American power. In their view, most of the world likes this strategy, and is eager for
Washington to continue along the same path. On the one hand, this isn't that surprising: why shouldn't NATO countries or Japan
prefer a world where they can spend 1-2% of GDP on defense while Uncle Sucker shoulders the main burden? More importantly,
advocates of restraint believe doing somewhat less would encourage present allies to bear a
fairer share of the burden, and also discourage some of them from adventurist behavior
encouraged by excessive confidence in U.S. protection (which Posen terms "reckless driving"). If the U.S.
played hard-to-get on occasion, it would discover that some of its allies would do more both
to secure their own interests and to remain eligible for future U.S. help. Instead of bending
over backwards to convince the rest of the world that the United States is 100 percent
reliable, U.S. leaders should be encouraging other states to bend over backwards to convince
us that they are worth supporting. Moreover, even if most of the world isn't balancing U.S.
power, the parts that are remain troublesome. For instance, "deep engagement" in the Middle
East has produced some pretty vigorous balancing behavior, in the form of Iraq and Iran's
nuclear programs, Tehran's support for groups such as Hezbollah, and the virulent anti-
Americanism of Al Qaeda. Indeed, the more deeply engaged we became in the region (especially
with the onset of "dual containment" following the first Gulf War), the more local resistance
we faced. Ditto our "deep engagement" in Iraq and Afghanistan. And given that those two
wars may have cost upwards of $3 trillion, it seems clear that at least a few people have
"balanced" against the United States with a certain amount of success. Sixth, reading B, I, &
W, one would hardly know that the nuclear revolution had even occurred. Nuclear weapons
are not very useful as instruments of coercion, but they do make their possessors largely
unconquerable and thus reduce overall security requirements considerably. Because the
United States has a second-strike capability sufficient to devastate any country foolish enough to attack us, the
core security of the United States is not in serious question. The presence of nuclear weapons
in the hands of eight other countries also makes a conventional great power war like World
War I or World War II exceedingly unlikely. Yet despite this fundamental shift in the global
strategic environment, B, I & W believe the United States must remain "deeply engaged" in
Europe, Asia, and elsewhere in order to prevent a replay of the first half of the 20th century. To
repeat: most of the strategists who reject "deep engagement/liberal hegemony" do not call for isolationism, a retreat to Fortress
America, or a slash-and-burn approach to defense spending. On the contrary: they favor continued U.S. engagement, albeit in a
more restrained, highly selective, and strategically sustainable way. They believe the United States should seek to maintain favorable
balances of power in key regions, but that it does not need to provide all the military muscle itself and certainly should not try to
dictate or control the political evolution of these areas with military force. They believe this approach would preserve core U.S.
interests at an acceptable cost, and would be far better suited to the current distribution of global power. "Deep
engagement" might have been a good strategy for the Cold War, though even that proposition
is debatable. But as you may have noticed, the Cold War is now over. Isn't it about time that
U.S. grand strategy caught up with that fact?
---Heg- AT: K to Trade
International trade and economics aren’t dependent on military intervention

Fettweis 11 [Christopher J. Fettweis - Department of Political Science Tulane University and Professor of National Security
Affairs at the US Naval War College, “Free Riding or Restraint Examining European Grand Strategy”, Comparative Strategy;
Sep/Oct2011, Vol. 30 Issue 4, p316-332, 17p,]

Second, it should be equally simple to demonstrate that these states remain vigorously engaged in
international trade and economics. In fact, fourteen of the fifteen “most globalized” countries in 2010 are in
Europe, according to the Swiss Economic Institute.10 The major U.S. allies appear to believe that the market
does not need protection, and that prosperity is no longer dependent upon active military
intervention abroad. Multinational corporations today can generally access the entire world
without much fear of undue harassment from host governments, who have strong
incentives to provide a healthy, well-regulated environment for trade and prosperity to
flourish. Threats to free trade still exist from a variety of criminal predators, but their solution, according to this point of view, hardly
requires costly military action. If and when local law enforcement agencies prove incapable of providing protection for the
businesses that operate in their territory, modern multinationals surely have the resources to either provide it for themselves, or
move out. In other words, the allies have reached the conclusion that Microsoft
does not need the Marine
Corps and great powers no longer have to use force to guard their economic interests.
Today’s market will take care of itself.
---Heg- Middle East Answers

Regional cooperation will fill in – checks heg

Sachs 11. [Jeffrey, Director of The Earth Institute, Quetelet Professor of Sustainable Development, and Professor of Health Policy
and Management at Columbia University. He is also Special Advisor to United Nations Secretary-General Ban Ki-moon, “A World of
Regions,” 5/26, http://www.social-europe.eu/2011/05/a-world-of-regions/]

In almost every part of the world, long-festering problems can be solved through closer cooperation among neighboring countries.
The European Union provides the best model for how neighbors that have long fought each other can come together for mutual
benefit. Ironically, today’s decline in American global power may lead to more effective regional
cooperation. This may seem an odd time to praise the EU, given the economic crises in Greece, Spain, Portugal, and Ireland.
Europe has not solved the problem of balancing the interests of strong economies in the North and those of weaker economies in
the South. Still, the EU’s accomplishments vastly outweigh its current difficulties. The EU has created a zone of peace where once
there was relentless war. It has provided the institutional framework for reuniting Western and Eastern Europe. It has fostered
regional-scale infrastructure. The single market has been crucial to making Europe one of the most prosperous places on the planet.
And the EU has been a global leader on environmental sustainability. For these reasons, the
EU provides a unique model
for other regions that remain stuck in a mire of conflict, poverty, lack of infrastructure, and
environmental crisis. New regional organizations, such as the African Union, look to the EU as a role
model for regional problem-solving and integration . Yet, to this day, most regional groupings remain too weak to
solve their members’ pressing problems. In most other regions, ongoing political divisions have their roots in the Cold War or the
colonial era. During the Cold War, neighbors often competed with each other by “choosing sides” – allying themselves with either
the United States or the Soviet Union. Pakistan tilted towards the Americans; India towards the Soviets. Countries had little incentive
to make peace with their neighbors as long as they enjoyed the financial support of the US or the USSR. On the contrary, continued
conflict often led directly to more financial aid. Indeed, the US and Europe often acted to undermine regional integration, which they
believed would limit their roles as power brokers. Thus, when Gamal Abdel Nasser launched a call for Arab unity in the 1950’s, the
US and Europe viewed him as a threat. The US undercut his call for strong Arab cooperation and nationalism, fearing a loss of
American influence in the Middle East. As a result, Nasser increasingly aligned Egypt with the Soviet Union, and ultimately failed in
the quest to unite Arab interests. Today’s reality, however, is that great powers can no longer divide and conquer other regions,
even if they try. The age of colonialism is finished, and we are now moving beyond the age of US global
dominance. Recent events in the Middle East and Central Asia, for example, clearly reflect the
decline of US influence. America’s failure to win any lasting geopolitical advantage through the
use of military force in Iraq and Afghanistan underscore the limits of its power , while its budget crisis
ensures that it will cut its military resources sooner rather than later. Similarly, the US played no role in the political
revolutions underway in the Arab world, and still has not demonstrated any clear policy response to them. President
Barack Obama’s recent speech on the Middle East is a further display of America’s declining influence in the region. The speech drew
the most attention for calling on Israel to return to its 1967 borders, but the effect was undercut when Israel flatly rejected the US
position. The world could see that there would be little practical follow-up. The rest of the speech was even more revealing, though
it drew little public notice. When Obama discussed the Arab political upheavals, he noted the importance of economic development.
Yet when it came to US action, the most that the US could offer financially was slight debt relief for Egypt ($1 billion), scant loan
guarantees ($1 billion), and some insurance coverage for private investments. The real message was that the US government would
contribute very little financially to the region’s economic recovery. The days when a country could depend on large-scale American
financing are over. We are, in short, moving to a multi-polar world. The Cold War’s end has not led to greater US dominance, but
rather to the dissemination of global power to many regions. East Asia, South Asia, Latin America, and the Middle East have new
geopolitical and economic influence. Each region, increasingly, must find its own path to economic development, energy and food
security, and effective infrastructure, and must do so in a world threatened by climate change and resource scarcity. Each region,
therefore, will have to secure its own future. Of course, this should occur in a context of cooperation across regions as well as within
them. The
Middle East is in a strong position to help itself . There is a high degree of economic
complementarity between Egypt and the oil-rich Gulf States . Egypt can supply technology, manpower, and
considerable expertise for the Arab region, while the Gulf provides energy and finance, as well as some specialists. The long-delayed
vision of Arab economic unity should be returned to the table. Israel, too, should recognize that its long-term security and prosperity
will be enhanced as part of an economically stronger region. For the sake of its own national interests, Israel must come to terms
with its neighbors. Other regions also will find that the decline of US power increases the urgency of
stronger cooperation between neighbors. Some of the greatest tensions in the world – say India and
Pakistan, or North and South Korea – should be defused as part of region-wide strengthening . As the EU shows,
ancient enmities and battle lines can be turned into mutually beneficial cooperation if a region looks forward, to resolving its long-
term needs, rather than backward, to its long-standing rivalries and conflicts.
Houti Rebels Answers
Frontline

No Houthi rebellion – Qatar will mediate


Terrill ‘11, professor of national security affairs – Strategic Studies Institute,
(W. Andrew, “THE CONFLICTS IN YEMEN AND U.S. NATIONAL SECURITY,” January)

Despite the problems noted above, there have also been serious efforts to maintain the truce . The
Qatari government, in particular, chose to recommit itself to the struggle for peace in northern
Yemen, falling back upon its familiar role as mediator. In late August 2010, the Qataris sponsored a meeting in
Doha in which the two sides agreed to an “explanatory appendix” associated with the earlier agreement.64
The signing of this document by representatives of both the Yemeni government and the Houthi leadership was dutifully witnessed
by Qatar’s Prime Minister and Foreign Minister. The
main goal of the Houthis in these negotiations was to obtain
the release of around 1,000 prisoners who had been taken in the fighting in the north. The government agreed to
meet this Houthi demand, and in return the Houthis agreed to surrender captured government
weapons to Qatari mediators.65

Won’t spill over to Saudi


Hill and Nonneman 11 (Ginny and Gerd, Associate Fellows of the Middle East and North Africa
Programme at Chatham House, “Yemen, Saudi Arabia and the Gulf States: Elite Politics, Street
Protests and Regional Diplomacy”, 5/11,
http://humansecuritygateway.com/documents/CH_YemenSaudiArabiaandGulfStatesPolPrtsReg
Dip.pdf)

However, the campaign also enjoyed significant public support in Saudi Arabia and was ‘spun’ by
the media as ‘a heroic and successful struggle to protect Saudi sovereignty’. 80 Some
satisfaction is derived from the fact that there have been no further incursions since the
intervention, and that Saudi Arabia was able to turn the episode to its advantage by securing the
border area. There is now a semi-permanent military complex around the southern Saudi city
of Najran. Nearly 80 border villages have been evacuated and the villagers are being re-housed
in 10,000 purpose-built units. Visible security improvements have been reported, including
earthen berms, concertina wire, floodlights and thermal cameras. 81 These measures serve
Saudi Arabia’s longer-term objective of containing AQAP, as well as constraining cross-border
flows of drugs, weapons and illegal migrants.

No proxy war – just posturing


Alaaldin 9 (Ranj, senior Middle East analyst at the Next Century Foundation, “Yemen's proxy
war that isn't,” The Guardian, 12/10/09,
http://www.guardian.co.uk/commentisfree/2009/dec/10/yemen-proxy-war-saudi-arabia)
The conflict in Yemen initially started off as a local affair between the country's Sunni-dominated central government and Shia rebels
in the north, known as the Houthis. It now has a regional dimension that pits two regional powerhouses against each other: Sunni
Saudi Arabia and Shia Iran. But for
all the hype and conspiracy theories that abound, there is no significant
Iranian support for the Houthis, despite Yemeni and Saudi protests. Exchanges between Iran
and the Saudi-Yemeni coalition fighting the rebels have certainly been heated . Iran recently named a
Tehran streetThe Martyrs of Sa'ada (Sa'ada being the province where the Houthis are based) and named another after a senior
Houthi leader. Yemen, for its part, renamed a street in Sana'a, its capital, after Neda Agha-Soltan, the Iranian protester who was shot
dead in June during the post-election unrest in Iran. The Saudi-Iranian rivalry over Yemen is also reflective of other rivalries
elsewhere. Both have supported proxies in Iraq, Lebanon and Gaza. Saudi military attacks on the Houthis along and within the
Yemeni border are still going on, in tandem with the Yemeni state's crackdown on both the rebels and Shia culture and identity.
From a purely emotional perspective, it would therefore make sense for the Iranians to support the
Houthis. This is, after all, the all-too-familiar story of a distinct minority group (the Shias) being besieged by an anti-Shia, Sunni
Arab establishment (the Saudi-Yemen alliance). Ideologically, the Houthis belong to a sect of Shia Islam
(Zaydism) that is in fact closer to Sunni Islam and at odds with the predominant and more global version of Shiism
practised in Iran and elsewhere (known asTwelver Shiism); hence the fact that senior clerics in Iran have provided
only a limited level of public backing to the rebels. Sources have suggested that Tehran is bringing Zaydis to religious
centres like Qom, in Iran, in an attempt to convert them, or provide them with the opportunity to convert to Twelver Shiism. Right
now, Iran's denunciation of Saudi interference and provocative offer to mediate stems primarily from a
humanitarian concern; the conflict also provides Tehran with an opportunity to flex its muscles and repair some of the post-
election damage inflicted to its credibility and axis of influence in the region. The Houthis require no military training
from outside, given that most members have been through the state military service system, and require no arms, with
weapons coming from a black market that underwent a boom after the 1994 Yemeni civil war. The Yemeni government did say it
had seized an Iranian-crewed vessel containing weapons near the Houthi stronghold in the north. Yet its failure to parade the
seizures made it difficult to confirm and verify its claims. Though there are allegations of Iranian financial support for the Houthis,
Yemeni officials have admitted the Houthis are more likely to be financed by non-governmental
Shia religious and economic actors . Further, Yemen is not geographically convenient for Iran;
notably, the Houthis have offices in the holy Shia centre of Najaf that could serve as a contact point with Tehran (though there is
nothing to suggest this is the case). What is clear is that one regional power, Saudi Arabia, is playing an active and, arguably,
unhelpful role in the Yemen-Houthi war. Its military incursions – which give the Houthis a bloody nose and nothing more – and
repeated, almost hyperbolic, claims of Iranian support for the Houthis has created a proxy war that does not, in any
tangible form, exist. Rebel movements seldom disappear through force alone. With both the Saudi and Yemeni courts failing to
show any serious commitment to a long-term, equitable, solution to the Houthi problem, the assumption is that both believe
resolution would be in nobody's interest. The Saudis may want to just contain the Houthis and ensure there is no prominent force of
Zaydi Islam across its borders, perhaps in the form of a federal autonomous region, supported possibly by Iran or Libya who would
welcome any opportunity to pressure the Saudis. Too much stability, coupled with social and political reform, could run counter to
this objective. WithinYemen, power is preserved through a system of patronage made up of tribal and
socio-economic complexities. Social and political reform, imperative for a long-term resolution to the
conflict, would undermine this system and therefore undermine the power held by the upper echelons of power
within the corrupt Yemeni government, and the military. Collectively, they reap the riches of the country's declining oil reserves,
smuggling networks, and Saudi financial assistance. Keeping
alive both the Houthi and the al-Qaida threat in
Yemen guarantees the flow of these lucrative sources of income, and they will do everything, and
anything, to protect it.
No war – the Houthis are backing down now – they fear Saudi forces – our card is
more recent than their internal
PressTV 2k10 (Press TV News Service, “Riyadh perceives ‘no threat’ from Houthis,” 1/31/10,
http://edition.presstv.ir/detail/117517.html)

A Saudi military source says the Houthi fighters in northern Yemen , which Riyadh waged war on November,
do not pose a threat to the Kingdom's security. "They do not really pose a threat," the source was
quoted by Reuters as saying on Sunday. Riyadh joined an already maximized operation launched by the
Yemeni government against the fighters in November. Saudi Arabia was accused of targeting
civilian areas during the campaign which lasted until Wednesday, when Houthis called for a
truce with Riyadh. The Houthis began to withdraw from their positions on the Saudi border amid
a worrying increase in the civilian mortalities from the offensives . The Saudi military, however, is reportedly
keeping up with the offensive. The Saudi official said the troops had exchanged fire with the fighters
who, he claimed, had crossed the border. "The (Houthi) snipers are still present … These people cannot be trusted,
every day they creep in and out."
Japan Econ Answers
Frontline

Empirically disproven- the earthquake and tsunami collapsed the Japanese economy before

AP 11

(CBC News, “Japanese economy slides into recession,” 5/19/11,


http://www.cbc.ca/news/world/story/2011/05/19/japan-economy-recession.html)

Japan's economy contracted sharply in the first quarter, veering back into recession as factory production
and consumer spending wilted in the aftermath of March 11 earthquake and tsunami . Real gross
domestic product — a measure of the value of all goods and services produced domestically — shrank at an annualized rate of 3.7
per cent in the January-March period, the Cabinet Office said Thursday. The result marks the second straight quarter that the
world's No. 3 economy has lost steam and undershoots an annualized 2.3 per cent fall forecast in a Kyodo News
agency survey. While there is no universally accepted definition of a recession, many economists define it as two consecutive
quarters of GDP contraction. Others consider the depth of economic decline as well as other measures like unemployment.
Martin Schulz, senior economist at Fujitsu Research Institute in Tokyo, said there is "no doubt"
that recession has returned. More surprising is just how quickly the economy crumpled, he said. The latest GDP
report includes just 20 days following the disaster, but "the impact is huge," said Schulz, who had expected to
see most of the economic fallout in the second quarte r. The magnitude-9.0 earthquake and tsunami left more
than 24,000 people dead or missing, and wiped out entire towns in the hardest-hit areas. Damage is estimated at $300
billion US, making it the most expensive natural disaster in history. Economic ripples It damaged factories in the region, causing
severe shortages of parts and components for manufacturers across Japan, especially automakers. A crippled nuclear power plant
caused widespread power shortages that added to the headaches faced by businesses and households. As a result, Japan's factory
production and consumer spending both fell the most on record in March. Exports
in March went south for the first
time in 16 months. Companies are reporting lower earnings and diminished outlooks for the rest
of the fiscal year. The recent events have deeply unnerved households, who are likely to remain cautious for the coming
months, Schulz said. "The nuclear disaster showed just how much is wrong in Japan actually," he said. "And
many things that seemed so stable and sure like electricity supply ... are looking not safe at all."

Japan Economy not key to global economy

Schuman 11 – American author and journalist who specializes in Asian economics, politics and
history. He is currently the Asia business correspondent for TIME Magazine, based in Hong Kong

(Michael, “Will Japan’s quake rock the world economy?,” 3/16/11,


http://curiouscapitalist.blogs.time.com/2011/03/16/will-japan%E2%80%99s-quake-rock-the-
world-economy/)

After a stress-inducing day on Wall Street, investors have


finally woken to the potential dangers to the world
economic order posed by the giant earthquake and tsunami in Japan . The Japanese economy is the world's
third-largest, and its companies play important roles in everything from heavy industry to high-tech to finance. There is no doubt
that the pain in Japan will somehow be felt by all of us. But how much? Despite
the jitters in the markets, the fallout
from Japan's trauma for the world economy might in the end be very limited . Here's why: I think the
main impact of Japan's crisis will show itself in supply chains . Japan is tightly integrated into global
manufacturing networks and is an important supplier of tons of stuff the world needs, from steel to semiconductors. So any
disruption of supply of components and materials in Japan will ripple through those supply chains, potentially causing shortages or
interruptions that would hamper factory operations around the world. It's hard to tell at this point how severe the overall impact
might be, but based on what's going on in Japan's industrial sector these days, problems seem inevitable. Factories are shut across
the country, and with power shortages and transport a mess, industrial production is unlikely to get back to normal for weeks. The
situation is probably most serious in electronics, since Japan produces 40% of the world's electronics components, including
materials and parts for chips and LCD TVs, as well as advanced batteries. This
supply chain problem may be short-
term, as companies find new sources of components, and there is enough spare capacity in
Japan and elsewhere to replace production lost to quake damage. But that doesn't mean the disruption won't be costly. But
beyond that, there is little reason to believe the troubles in Japan's economy will have a very
dramatic impact on the rest of the world . Clearly, industrial output and demand in Japan will take a
hit, and so will growth. But that won't translate into big losses for the global economy, even in
its current fragile condition. There are two reasons why. First, though Japan is a big economy, it exports
more than it imports, thus a decrease in demand in Japan doesn't spill over much into the global
economy. Only 5% of U.S. exports head to Japan, for example. Morgan Stanley points out that Japan takes in 8% of Chinese
exports, but Japanese demand doesn't add much to the growth of those exports. In 2010, Japan
contributed only 1.9ppt to the 31.3% year-on-year growth of China's exports.
Japan Prolif Answers
Frontline

Japan won’t proliferate --- deeply embedded in Japanese culture and society

Kamiya ‘9 – professor of international relations at the National Defense Academy of Japan (Matake Kamiya, “Realistic
Proactivism: Japanese Attitudes Toward Global Zero,” ISN, September 2009, http://www.isn.ethz.ch/isn/Digital-
Library/Publications/Detail/?ots591=cab359a3-9328-19cc-a1d2-8023e646b22c&lng=en&id=106532)

The Japanese share a deep-seated aversion to nuclear arms, a feeling that transcends
differences in political ideology and beliefs. An almost instinctive dread of, and hatred for,
nuclear weapons widely held across the spectrum of Japanese society is both one of the most
fundamental roots of Japan’s non-nuclear stance and an extremely powerful deterrent against
Japanese nuclear proliferation. The origin of such strong anti-nuclear attitudes lies in Japan’s
tragic experience as the only nation ever to suffer a nuclear attack. The two bombs dropped on Japan in
August 1945 killed about 140,000 in Hiroshima and about 70,000 in Nagasaki. In the years that followed, tens of thousands more
died from so-called atomic bomb disease—various illnesses caused by exposure to radiation. Even today, many Japanese
suffer the after-effects of this exposure . Naturally, Hiroshima and Nagasaki have greatly influenced
post-war Japanese culture. Over the past half-century, countless books, nursery tales, television and radio programs,
movies, comic books, animated features and other forms of communication about the bombs have exposed
later generations to the horrors of nuclear war .§§§ Another factor often overlooked by outsiders, but no less
important in shaping Japanese antinuclear sentiment than Hiroshima and Nagasaki, was the harm done
to Japanese fishermen by US nuclear testing in the South Pacific in March 1954. The radioactive
fallout from the first US hydrogen bomb test on Bikini Atoll severely contaminated the Fukuryu-maru No. 5,
a Japanese tuna-fishing boat known as the Lucky Dragon outside Japan, and its crew of 23, even though the boat was
located 35 kilometers from the danger zone declared by the United States at the time of the explosion. The entire crew
suffered from atomic bomb disease; one crew member died, and the rest were hospitalized for more than a year. The
Japanese were both horrified and outraged to see that their compatriots were victims of nuclear
weapons yet again, particularly because the tragedy occurred in peacetime .125 The Fukuryu-
maru incident left a deep and lasting impression among the Japanese population that one could
become a victim of nuclear weapons anywhere or anytime. Shortly afterward, the first nation-wide grassroots
movement against nuclear weapons sprang up in Japan and, by the end of 1954, more than 20 million Japanese had signed the
Suginami Appeal for the Prohibition of Atomic and Hydrogen Bombs.126 In April 1954, both houses of Japan’s Diet unanimously
passed resolutions that called for the prohibition of nuclear weapons and international control of nuclear energy. Japan’s
non-
nuclear policy has consistently reflected this profound hatred for nuclear weapons, which has
been deeply embedded in post-war Japanese culture and society.

Oversight solves and timeframe is too long

Kamiya ‘9 – professor of international relations at the National Defense Academy of Japan (Matake Kamiya, “Realistic
Proactivism: Japanese Attitudes Toward Global Zero,” ISN, September 2009, http://www.isn.ethz.ch/isn/Digital-
Library/Publications/Detail/?ots591=cab359a3-9328-19cc-a1d2-8023e646b22c&lng=en&id=106532)
In conclusion, Japan’s
nuclear infrastructure has been oriented exclusively to peaceful uses of
nuclear energy. In order to guarantee that its nuclear program would not be diverted from
peaceful purposes, Japan has taken various specific measures. Consequently, despite all its
latent nuclear know-how and potential, Japan is not capable of acquiring a militarily significant
nuclear arsenal in a short period of time. Because Japan is an open society and all of its nuclear
power activities are subject to IAEA safeguards, it would be impossible for the country to start a
project in secret to obtain all the necessary technologies described above and to build covertly a
militarily-meaningful nuclear arsenal of its own.

Uranium removal proves Japan can’t and won’t prolif

Kamiya ‘9 – professor of international relations at the National Defense Academy of Japan (Matake Kamiya, “Realistic
Proactivism: Japanese Attitudes Toward Global Zero,” ISN, September 2009, http://www.isn.ethz.ch/isn/Digital-
Library/Publications/Detail/?ots591=cab359a3-9328-19cc-a1d2-8023e646b22c&lng=en&id=106532)

As for uranium, despite having the technological capability to produce weapon-grade, highly-
enriched uranium, Japan has refrained from generating such materials. In the past, Japan
purchased a limited amounted of highly-enriched uranium from the United States and retained
it for use in peaceful research reactors . Since 1996, however, Japan has agreed to return that
highly-enriched uranium to the US in order to remove the risks of proliferation to third countries
and terrorists, and by the summer of 2008, 579.7 kilograms of highly enriched uranium, which represented “almost all of
highly enriched material in Japan’s principle research reactors ,” was actually transferred to
nuclear research facilities in the United States.95 According to Andrew Beineawski, an official of the US Department of
Energy, the repatriation operation was initiated by the US as part of a counter-proliferation project called the Global Threat
Reduction Initiative. Beiniawski said, “Japanese research reactors have been very successful in shipping their spent HEU fuel to the
United States. These shipments contribute to HEU minimization efforts worldwide.” The remaining tenodds kilograms of highly-
enriched uranium still in Japan is scheduled to be returned to the United States by 2012.96 As Ambassador Yukiya Amano, newly-
elected Director General of the IAEA, maintained, the
Japanese government judged by the end of 1970s that “using
highly enriched uranium is a matter of concern from the security point of view, is not a
necessity, and is not helpful to ensure the confidence of the international community, as far as
Japan is concerned,” and “decided and has been reducing the enrichment level” since then.97
Japan’s attitude toward highly-enriched uranium demonstrates that Japan has not entertained
any idea of diverting such materials for weapon purposes .

Japanese prolif won’t spillover

White 8 
[Hugh, Visiting Fellow at the Lowy Institute for International Policy and Professor of Strategic Studies at the Australian National
University, 7/17. “A nuclear Japan: The least bad option?” The Lowy Interpreter, http://www.lowyinterpreter.org/post/2008/07/A-
nuclear-Japan-The-least-bad-option.aspx]

But in a world with nuclear weapons, a key aim of strategic policy is to stop them being used. We therefore have to put very
high priority on reducing the risks of conflict between nuclear powers, and to reduce the risks of escalation to a nuclear
exchange if they do go to war. And this leads to some very tough choices. At times, the imperative for fewer nuclear weapons in
fewer hands must be weighed against the imperative to build an international order and a military balance which stabilises the
international order and makes the use of these weapons less likely. We might find that the risks of nuclear war in Asia would be
I am not sure that a Japanese
lower if Japan had nuclear weapons than if it did not. Second, more specifically,
nuclear capability would automatically ignite a new wave of proliferation. Developing
nuclear weapons is a big step for anyone. Who among the non-nuclear states would find their
strategic situation so profoundly altered by a Japanese nuclear capability that they would feel impelled to take
this step? The most likely, of course, is South Korea. But if, like me, you are a inclined to doubt that North Korea will surrender
its weapons, and that an eventual unified Korea is therefore like to be a nuclear power anyway, then this horse may already be
out of the stable. Beyond Northeast Asia, I think flow-on proliferation effects are much less likely: would Australia, or Indonesia,
be more likely to seek nuclear weapons because Japan had them? Thirdly, we might ask whether the
non-proliferation
regime could survive by adapting to accommodate a nuclear-armed Japan. Is this unthinkable?
Surely not, when serious thought is being given to accommodating India as a nuclear-armed country. Two other quick points.
First, Crispin suggests that Japan does not need nuclear weapons because it has formidable conventional forces. Japan’s forces –
especially air and naval forces — are strong, and I am sure they will get stronger. But they will not provide Japan with a capacity
to deter nuclear attack from any of its three nuclear-armed near neighbours. It is worth making the point that the only
legitimate purpose for a Japanese nuclear capability would be to provide an independent minimal deterrent of nuclear attack on
its own territory. But it seems only nuclear weapons can do that. 

Even if Japan proliferates, it would be useless --- lacks missile capability

Kamiya ‘9 – professor of international relations at the National Defense Academy of Japan (Matake Kamiya, “Realistic
Proactivism: Japanese Attitudes Toward Global Zero,” ISN, September 2009, http://www.isn.ethz.ch/isn/Digital-
Library/Publications/Detail/?ots591=cab359a3-9328-19cc-a1d2-8023e646b22c&lng=en&id=106532)

For Japan, tactical nuclear weapons would be nearly useless, because, as an island country, it
would find few meaningful targets for such weapons. In order to obtain a militarily-meaningful
nuclear arsenal, Japan would have to possess ballistic missiles. Although the country has advanced
rocket production and space-launch capabilities, it would take many years before the country would
actually be able to deploy ballistic missiles for military purposes. Japan has developed the solid-
fuel MV rocket, which is capable of launching probes for interplanetary missions. However, the
rocket which the Japanese government has positioned as its “primary large-scale launch vehicle”
has been the H-II, not the M-V99. In fact, production of the M-V series was discontinued after the
launch of M-V-7 in September 2006 for cost and other reasons.100 As the Federation of American Scientists simply put it, the H-II
is “ENTIRELY unsuited for conversion to ballistic missile applications [emphasis in the original]” because
it is powered by liquid oxygen and liquid hydrogen .101 According to Thompson and Self, “Japan has
invested heavily in rockets that would not make effective ballistic missiles.” They also note: To the
extent that technical consideration of military applicability entered into the engineering
context . . . it seems that the civilian rocket programs at ISAS [the Institute for Space and Aeronautical Sciences]
and NASDA [the National Space Development Agency] steered away from rather than toward, such
capabilities. Military rocketry research at the Technology Research and Development Institute (TRDI), inside the
Defense Agency, has been restricted to small rockets for tactical use, such as surface-to-air missiles.102
---Japan Prolif- Won’t Prolif- Laundry List

Multiple barriers to rearm:

a. Treaty obligations prevent it and the government is strongly opposed

Hughes 7 (Lleywelyn, Assistant Professor of International Affairs @ George Washington University, “Why Japan Will Not Go
Nuclear (Yet): International and Domestic Constraints on the Nuclearization of Japan,” International Security, Muse,

Japan's strategy to deter nuclear threats while remaining a nonnuclear weapon state has centered on its security treaty with the
United States. Strong support for multilateral nonproliferation and arms control regimes has complemented this policy. Indeed
Japan has enmeshed itself in a web of international agreements both to enhance its own security and
to signal its intention to refrain from developing an indigenous nuclear deterrent .23 Japanese
officials cite Japan's ratification of the NPT in June 1976 as the moment at which the option of developing an
indigenous nuclear deterrent was discarded.24 Under its NPT commitments, Japan is prohibited from
manufacturing, receiving the transfer of, or controlling directly or indirectly a nuclear device. Since ratifying the
NPT, Japan has been a committed participant in international agreements promoting arms control and nonproliferation of
nuclear, chemical, and biological weapons. It supports the missile technology control regimes and other export control regimes,
and since 1989 has hosted the UN Conference on Disarmament Issues.25 Japan also continues to invest resources in training
officials from developing countries in arms control and nonproliferation policies.26 [End Page 73] Evidence demonstrates that
in the 1990s Japanese officials strengthened Japan's commitment to arms control and nonproliferation, and began to engage in
more prominent public diplomacy, increasing the reputational costs of unilateral withdrawal from the NPT. Japan ratified the
Comprehensive Test Ban Treaty in 1997, and in 1998 became a signatory to the Additional Protocol of the NPT. The
latter significantly extends its reporting responsibilities to the International Atomic Energy Agency
(IAEA), and enables inspections of declared and suspected undeclared sites at short notice. Japan undergoes a
rigorous inspections program under this regime, reportedly accounting for up to one-third of the IAEA's budget.27 Japan
also maintains bilateral agreements with its nuclear suppliers banning it from using
imported materials for purposes other than its civilian nuclear energy program.28 Japan has also
increased its public rhetoric in support of nonproliferation . Since 1994 it has offered a resolution
annually at the UN General Assembly calling for the total elimination of nuclear weapons. And since 2002 it has published a
biannual white paper, in both Japanese and English outlining Japan's policy position on arms control and nonproliferation.29
Reflecting these changes, the Ministry of Foreign Affairs (MoFA), the lead ministry on disarmament and arms control issues, has
increased the amount of organizational resources invested in the nonproliferation regime. Management of Japan's disarmament
and arms control policy was initially subsumed within the United Nations Division of the Ministry of Foreign Affairs' Foreign
Policy Bureau. Institutional reforms following the 1991 Gulf War, however, saw the disarmament and arms control policy
functions shifted into a newly created disarmament and nonproliferation section. This new section included a department
charged with ensuring Japan's compliance with international obligations regarding the peaceful use of nuclear energy, and one
responsible for managing Japan's arms control and nonproliferation policies. A second round of reforms in 2004 gave greater
prominence to Japan's disarmament and arms control policy. The arms control and nonproliferation section was elevated to the
divisional level, and within this division arms control and nonproliferation functions were given prominence over nuclear energy
compliance functions. These
organizational changes reflect two factors: first, the prominence of
multilateral arms control and nonproliferation within [End Page 74] Japanese foreign policy
and increased administrative obligations to treaties to which Japan is now a party; and second,
the completion of bilateral treaty negotiations associated with Japan's civilian nuclear energy program.30

b. It would destroy their domestic energy sector


Hughes 7 (Lleywelyn, Assistant Professor of International Affairs @ George Washington University, “Why Japan Will Not Go
Nuclear (Yet): International and Domestic Constraints on the Nuclearization of Japan,” International Security, Muse,

This centrality of energy security to Japan's civilian nuclear energy program implies that the
Ministry of Economy,
Trade, and Industry and public and private organizations involved in this program are unlikely to
support the diversion of nuclear materials or the requisition of components of the program for military
use. Although not tested, bilateral contracts with uranium suppliers, for example, include clauses that
imported materials must be used for peaceful purposes. Breaking these contracts would
have a significant impact on Japan's nuclear energy program and therefore its energy security
goals.59 Planned reprocessing facilities also do not have adequate capacity to manage all spent fuel produced in Japanese
reactors, meaning that this situation is unlikely to [End Page 82] change. The likelihood of such opposition emerging was
suggested in industry comments opposing the merits of conducting a debate on nuclearization following the October 2006
North Korean nuclear test, when it was argued that debating this question would be irresponsible because of its
potential impact on Japan's spent fuel reprocessing planning.

c. The legislature wouldn’t pass it and there are regulatory commissions that prevent it

Hughes 7 (Lleywelyn, Assistant Professor of International Affairs @ George Washington University, “Why Japan Will Not Go
Nuclear (Yet): International and Domestic Constraints on the Nuclearization of Japan,” International Security, Muse,

A third domestic legal constraint on Japanese nuclearization is the Basic Law on Atomic Energy ,
enacted in 1955 as the foundational law managing Japan's extensive civilian nuclear energy program. Article 2 of the law
establishes that the research, development, and utilization of atomic energy must be limited to peaceful purposes and carried
out independently under democratic management. The Basic Law potentially [It] constrains the development
of nuclear weapons in two ways. First, amendments to the law must pass through normal
parliamentary procedures, giving the opposition to any amendment an opportunity to
block proposed changes. Second, the law establishes the Atomic Energy Commission as a body
composed of civilians that is a formal part of the cabinet, reporting to a minister without portfolio within the Cabinet
Office. The Atomic Energy Commission is charged with creating policy and coordinating with other parts of the bureaucracy on
nuclear budgetary issues. The most important policy role of the commission is drafting the long-term plan for the use of nuclear
energy, and ensuring that nuclear energy planning conforms with the articles of the Basic Law. As such, the
commission
defines its role as making sure that Japan continues to limit its use of nuclear energy to
peaceful purposes. Meeting records of the commission show that its members are united
against the development of a nuclear deterrent. Following the 2002 comments by Chief Cabinet Secretary
Fukuda on the constitutionality of nuclear weapons, for example, debate within the commission was dominated by [End Page
88] the question of how to respond to what was judged by committee members as a transgression of the Basic Law.
Nevertheless, discussions between committee members indicate that few regulatory tools are available to them to halt any
drive to revise the law to allow the diversion of nuclear materials to a nuclear weapons program; the commission does not have
the power to veto changes to the Basic Law itself. This suggests that if an effort was made to alter the law to allow the diversion
of materials used in the civilian nuclear energy program to military use, the commission could do little beyond attempting to
mobilize public opinion.

d. The Japanese public is against it and there’s strong inertia to change

Hughes 7 (Lleywelyn, Assistant Professor of International Affairs @ George Washington University, “Why Japan Will Not Go
Nuclear (Yet): International and Domestic Constraints on the Nuclearization of Japan,” International Security, Muse,
Informal constraints present a more significant barrier to Japanese nuclearization . First, public
polling in Japan consistently demonstrates an aversion to nuclearization that has not varied
significantly despite the end of the Cold War and the emergence of North Korea as a nuclear
weapons state. In 1968, polling carried out by the Asahi Shimbun newspaper found that 21 percent of the population
answered affirmatively when asked whether Japan should obtain nuclear weapons. In 1978 and 1981, supporters had fallen to
15 and 16 percent.87 The end of the Cold War and the emergence of North Korea as a nuclear weapons state have not reversed
this sentiment. A poll carried out in October 1999 by the National Institute for Research Advancement, for example, asked
respondents what position in relation to nuclear weapons Japan should take in the event that the U.S.-Japan alliance was either
dissolved or rendered meaningless. It found that support for independent nuclearization nevertheless stood
at 7 percent. This result replicated a January 1994 poll asking respondents whether they would favor Japan developing its
own nuclear weapon if North Korea did the same. Nine percent replied they "somewhat favored" or "strongly favored" such an
outcome.88 The October 2006 nuclear test by North Korea also appears not to have reversed public sentiment. In a poll
conducted on November 11–12, 2006, the Yomiuri Shimbun newspaper found that 17.6 percent of respondents agreed with the
statement that Japan's commitment to remain [End Page 89] a nonnuclear weapons state should not be absolute, but rather
Japan should reconsider its commitment to remain a nonnuclear state depending on changes in the international
environment.89 Second, a feature of the postwar period has been the repeated inability of
political actors to implement substantial changes to Japan's security policy in the face of
opposition. Most notably, the Liberal Democratic Party, which has governed Japan for all but a handful of years
since the party's creation in 1955, has suffered from a lack of intraparty cohesion on security policy ,
with both centrists and more hawkish groups coexisting within the party. Although the balance of power between these groups
is dynamic, the equilibrium between them has historically been found in a reliance on the U.S. deterrent and limited domestic
spending on autonomous defense.90
Japan Relations Answers
Frontline

Strong relations inevitable:

Their authors are hacks and empirically denied

Kato 10 (Yoichi, Asahi Shimbun Senior Staff Writer, Citing Wallace Gregson, Assistant Secretary of Defense for Asian and Pacific
Security Affairs, “The Shangri-La Dialogue,” 2010, http://www.iiss.org/whats-new/iiss-in-the-press/june-2010/new-futenma-eco-
impact-study-not-needed/,

It’s always a very interesting topic in the academic and think tank community for people to voice
opinions on the health of the U.S.-Japan alliance. Almost always, regardless of what year it
is or what day it is or what we’re discussing, the opinion is always, “Oh, the alliance is in trouble .”
And I would say, I would offer, that the persistent rumors of the demise of the U.S.-Japan security
alliance are greatly exaggerated.

Multiple alt causes deny decline

Tanaka ’11 (Senior fellow at the Japan Center for International exchange (Hitoshi, “Reinvigorating US-Japan relations,” March
2011, http://www.jcie.org/researchpdfs/EAI/6-2.pdf)

There has long been a perception gap underlying US-Japan relations . In the 50 years since the revised US-
Japan Security Treaty was signed in 1960, numerous developments have been seen to be putting the relationship in crisis . There
was the “Nixon Shock,” when the US president visited China without consulting or even notifying
Japan. There was the time when the US secretary of state proclaimed that Japan was “insensitive” because it
was importing oil from Iran during the hostage situation. There was the incident in which Toshiba Machine
violated the Coordinating Committee for Export to Communist Areas agreements by selling industrial equipment to the
Soviet Union during the height of the Cold War. The mid1980s saw economic friction between the United States and
Japan peak. And then there were tensions surrounding Japan’s contribution to the first Gulf War. On Okinawa, there have been
incidences of rape by American soldiers and the conflict over basing issues. On top of all of that, there was the Ehime
Maru incident, in which several Japanese high school students died when their fishery training boat was hit by a US Navy submarine .
In the United States, Japan is often perceived as not adequately fulfilling its role as an alliance
partner. At the same time, many in Japan hold a deep-seated perception that their country is being treated like a dependent by
the United States. The Japanese side has been particularly prone to making proclamations that the
relationship is in crisis every time there is an isolated incident. Recognizing this perception gap
between the two countries, every time there is a US-Japan summit, our leaders speak with a common voice to emphasize that the
U nited St ates and Japan enjoy an “equal partnership,” a phrase that was uttered especially often by former Prime
Minister Hatoyama
US – Japan relations are strong and resilient --- rooted in common interest

AFP ‘10 (“US, Japan relations unaffected by Prime Minister's resignation,” Agence France Presse, June 2, 2010,
http://www.google.com/hostednews/afp/article/ALeqM5jBNtvewQHZM2q35LVUaMKfsQ9ljg)

WASHINGTON — The White House said Wednesday Japan was one of America's "best friends" in the
world, and that the relationship would not be adversely affected by the departure of Prime
Minister Yukio Hatoyama. Hatoyama stepped down after a brief tenure disrupted by a political and diplomatic row over a
US air base in Japan, after taking office vowing to forge a more equal relationship with Washington."We respect the Japanese
political process and Prime Minister Hatoyama?s decision to step down," White House spokesman Robert Gibbs said in a statement.
"The selection of Japan's next prime minister is a matter for the Japanese people and political process. "The
US-Japan
bilateral relationship is very strong and deeply rooted in our common interests and values. " Our
alliance has flourished under each Japanese prime minister and US president for the past half
century and will continue to strengthen in the years to come," Gibbs said. Earlier, Gibbs's deputy Bill
Burton told reporters that Japan was "one of our best friends in the world and that alliance is
not going to change as a result of any change in leadership in that country." "We'll watch the political
process take its course and be waiting like everybody else to see who the next prime minister will be," Burton said on Air Force One
as Obama flew to Pittsburgh.

US-Japan relations strong --- Japan perceives the alliance as vital

MOFA, 10 (Ministry of Foreign Affairs – Japan, “Telephone Conversation Between Prime Minster-Elect Naoto Kan and U.S.
President Barack Obama,” June 6, 2010, http://www.mofa.go.jp/announce/announce/2010/6/0606_02.html)

Mr. Naoto Kan, Prime Minister-elect, held a telephone conversation with Mr. Barack Obama, President of the United States on
Sunday, June 6 upon the request from the US side. During the telephone conversation, issues such as Japan-U.S. relations, the
sinking of the Republic of Korea (ROK) patrol vessel and the Iranian nuclear issue were discussed. President Obama
congratulated Prime Minister-elect Kan on his election as the next Prime Minister. In response , Prime
Minister-elect Kan thanked President Obama for the telephone call when the President was busy with
the oil spill in the Gulf of Mexico. Prime Minister-elect Kan said that the Japan-U.S. Alliance remains is
the cornerstone of Japanese diplomacy . The two leaders agreed to cooperate closely not only
on bilateral issues but also on the situation of the Asia-Pacific region as well as global issues, and
to make efforts to further deepen and develop the Japan-U.S. Alliance this year, which marks
the 50th anniversary of the conclusion of the Japan-U.S. Security Treaty (Treaty of Mutual Cooperation
and Security between Japan and the United States). Concerning the relocation of the Futenma Air Station, Prime Minister-
elect Kan said that Japan will commit to tackling the issue based on the recent agreement. The
two leaders confirmed that the two sides will make further efforts on the issue. Prime Minister-elect
Kan said the incident of the sinking of the ROK navy patrol vessel is a serious issue concerning the regional security. The two
leaders agreed to continue close coordination among Japan, the United States and the ROK in
responding to the case. The two leaders affirmed that they share concern over the Iranian
nuclear issue and that Japan and the U nited States will closely cooperate with each other in the
issue, including a response at the UN Security Council. The two leaders expressed their wishes to meet again at
the G-8 Summit and the G-20 Summit toward the end of June.
---Japan Relations- Resilient
a) Fear of Korea

Harris 10 (Tobias, Asia Security Initiative, “How will regional uncertainty affect the U.S.-Japan alliance?,” 11-23,
http://asiasecurity.macfound.org/blog/entry/111how_will_regional_uncertainty_affect_the_u.s.-japan_alliance/,

The exchange of fire between the North and South Korean militaries that left two ROK Marines dead
and at least a dozen wounded (see the roundup at Wired’s Danger Room blog), following closely on the heels of
revelations regarding a new North Korean uranium reprocessing facility, strengthens hopes that
the U.S. and Japan might be able look past Futenma and strengthen their security
relationship. The relationship has, of course, had a bit more wind in its sails since the standoff between Japan and China
over the maritime collision near the Senkakus. Can we really draw a straight line from regional instability
to closer security cooperation between the U.S. and Japan ? Arguably this logic has worked in
the past, with North Korean provocations from 1994 onward stirring Japanese policymakers to bolster Japan’s capabilities
and launch new bilateral initiatives with the U.S., ballistic missile defense being perhaps the most notable example. And there
are signs that the DPJ-led government is remarkably more realist in its approach to the region
than many expected. I think Foreign Minister Maehara Seiji spoke for many in the DPJ when he told an official Chinese foreign
affairs publication that he is “by no means a hawk but a realist who values idealism.” The distinction between “hawk” and
“realist” is meaningful and says a lot about the DPJ’s approach to foreign and security policy.

b) Tsunami response

Calder ‘11 - Director of the Reischauer Center for East Asian Studies at the School of Advanced International Studies at the
Johns Hopkins University.

(Kent E., “Crisis will strengthen Japan-U.S. relations,” 3/23/11, http://globalpublicsquare.blogs.cnn.com/2011/03/23/crisis-will-


strengthen-japan-u-s-relations/)

Prime Minister Naoto Kan labeled his country's earthquake and the ensuing tsunami “Japan’s worst disaster since World War II.”
There is much to support his assessment. If there are any silver linings, however, they’re likely on the political side .
Japan’s
response to this disaster represents a potential watershed in U.S.-Japan relations , which have been
eroded by Okinawa disputes over U.S. military bases, political turmoil in Tokyo, and years of neglect. Prime Minister Kan’s
acceptance of humanitarian support from the U.S. military is unprecedented, despite the
longstanding U.S.-Japan alliance. It contrasts sharply with reformist Prime Minister Murayama Tomiichi’s rejection of
analogous support from U.S. forces in Japan following the 1995 Hanshin (or Kobe) earthquake. The U.S. Navy has dispatched a relief
flotilla of 20 vessels to waters off Japan, including the aircraft carrier Ronald Reagan and the Seventh Fleet command ship Blue
Ridge. Apart from search and rescue operations, U.S. forces have helped open Sendai Airport, which was badly damaged in the
earthquake, and they’ve cooperated with Tokyo Electric Power and Japanese Self Defense Forces to prevent a large-scale disaster at
the Fukushima nuclear plant. In that effort, U.S. forces have provided fire engines, high-pressure pumping vehicles, radiation-
Obama has amplified the positive U.S.-
resistant protective suits, and Global Hawk overhead surveillance. President
Japan atmospherics through a series of sensitive personal gestures . He was quick to express condolences
and extend offers of assistance, as was Secretary of State Hillary Clinton. He also visited the Japanese Embassy personally to convey
those sentiments - a gesture well reported in Japan . Ongoing cooperation in confronting the current crisis, coupled with
the precarious yet arguably enhanced political stability in Japan, paves the way for a productive scheduled summit between Kan and
Obama in Washington, probably late in June. Before the crisis,
plans were already underway for emphatic
security declarations strengthening alliance cooperation. But the earthquake crisis has given
them an important “human security” element that has often been missing from U.S.-Japan
dialogue. Proposals enabling enhanced cultural exchange and working holidays for students are gaining momentum, amplifying
the broadening process.

Both sides are strongly committed to the alliance

Ogawa 10 (Satoshi, Yomiuri Shimbun Correspondent, “Japan, U.S. 'to set new China plan' / Joint strategy aims at 'deeper security
goals',” 11-22, http://www.yomiuri.co.jp/dy/national/T101121002647.htm,

WASHINGTON--Japan and the United States will soon draw up new "common strategic
objectives" that will most likely focus on ways to enhance the Japan-U.S. strategy toward
China, according to diplomatic sources in Washington. The two sides hope to have the objectives worked out in connection
with a joint statement on the Tokyo-Washington alliance to be finalized by next spring, the sources said. Prime Minister Naoto
Kan and U.S. President Barack Obama confirmed the plan to adopt the new objectives when they met in Yokohama on Nov. 13,
The envisaged common objectives will be designed to define anew
the sources said Saturday.
goals Japan and the United States should pursue to enhance regional and global security,
the sources said. How Japan and the United States should cope with China , in particular its maritime
advances, will probably be high on the agenda , they said. Observers say adopting new Tokyo-Washington
strategic objectives may again bring into question the diplomatic stance of the Democratic Party of Japan, as the DPJ in its
election platform emphasized the need to strengthen Japan's ties to China and other Asian nations, while calling for a "close and
equal" Japan-U.S. relationship. In his Yokohama talks with Kan, Obama proposed that new strategic objectives be drawn
up in the process of working out the planned new Japan-U.S. security cooperation statement "to deepen [security]
goals" common to the two countries, and Kan agreed to the overture, the sources said. The two
governments will launch ministerial working-level consultations on the matter as soon as early December,
they said. A set of common strategic objectives was agreed on in the Japan-U.S. Security Consultative Committee in February
2005. The SCC, known as the two-plus-two meeting as it comprises the foreign and defense ministers from both countries, is
tasked with strengthening security arrangements of Japan and the United States. The next session of the SCC will discuss
specific plans for relocating the functions of the U.S. Marine Corps' Futenma Air Station in Okinawa Prefecture. In the 2005 two-
plus-two consultations, Japan and the United States agreed to urge China, with its growing national strength, to play a
constructive role in the international community and work for a peaceful settlement of issues surrounding Taiwan. Beijing
reacted negatively to the Tokyo-Washington common objectives, the first time that an official Japan-U.S. document had made
multiple Japanese
reference to a joint strategy toward China. As to what the planned new common objectives will be,
and U.S. sources said the two sides would confirm bilateral security cooperation , including
measures to strengthen the strategy toward China, in light of China's activities since 2005 that threaten the stability of the East
Even if the planned new objectives prove to be much the same as those of 2005,
Asian region.
it would be significant for the United States to reconfirm the common objectives with
Japan under the DPJ administration, they said.
I-Law Answers
Frontline

Citing I-Law doesn’t set a precedent

New York Times 8 (“US Court is Now Guiding Fewer Nations,” 9-18, http://www.nytimes.com/2008/09/18/us/18legal.html?
_r=2&hp=&adxnnlx=1221753717-8pdanTsDalyAfCQgzjrVvQ&pagewanted=print)

Judicial citation or discussion of a foreign ruling does not, moreover, convert it into binding
precedent. Chief Justice John Marshall, sitting as a circuit court judge, discussed the question in 1811. “It has been said that
the decisions of British courts, made since the Revolution, are not authority in this country,” he said. “I admit it — but they are
entitled to that respect which is due to the opinions of wise men who have maturely studied the subject they decide.” Indeed,
American judges cite all sorts of things in their decisions — law review articles, song lyrics,
television programs. State supreme courts cite decisions from other states, though a decision from Wisconsin is no
more binding in Oregon than is one from Italy. “Foreign opinions are not authoritative; they set no
binding precedent for the U.S. judge,” Justice Ruth Bader Ginsburg said in a 2006 address to the Constitutional
Court of South Africa. “But they can add to the story of knowledge relevant to the solution of trying questions.” But Professor
Fried said the area was a minefield. “Courts have been citing foreign law forever, but sparingly, for very good
reason,” he said. “It is an invitation to bolster conclusions reached on other grounds. It leads to more impressionistic,
undisciplined adjudication.”

Other countries support international law now

Benvenisti 8 (Eyal, Professor of Law at Tel Aviv University, “Reclaiming Democracy,” 102 AJIL 241)

In recent years, courts in several


democracies have begun to engage seriously in the interpretation and
application of international law and to heed the constitutional jurisprudence of other national courts. Most recently, this
new tendency has been demonstrated by the judicial [*242] responses to the global counterterrorism effort since the events of
September 11, 2001: national courts
have been challenging executive unilateralism in what could perhaps
be a globally coordinated move. In this article I describe and explain this shift, arguing that the chief motivation of
the national courts is not to promote global justice, for they continue to regard themselves first and foremost as national
agents. Rather, the
new jurisprudence is part of a reaction to the forces of globalization, which are placing
increasing pressure on the different domestic branches of government to conform to global
standards. This reaction seeks to expand the space for domestic deliberation, to strengthen the ability of national
governments to withstand the pressure brought to bear by interest groups and powerful foreign governments, and to insulate
the national courts from intergovernmental pressures. For this strategy to succeed, courts need to forge a united judicial front,
which entails coordinating their policies with equally positioned courts in other countries by developing common
communication tools consisting of international law and comparative constitutional law. The analysis also explains why the U.S.
Supreme Court, which does not need to protect the domestic political or judicial processes from external pressure, has still not
joined this collective effort. 3 On the basis of this insight into the driving force behind reliance on foreign law, the article
proposes another outlook for assessing the legitimacy of national courts' resort to foreign and international legal sources. It
asserts that recourse to these sources is perfectly legitimate from a democratic theory perspective, as it aims at reclaiming
democracy from the debilitating grip of globalization.
Ilaw solves nothing

Goldsmith and Posner 5 – *Harvard Law School professor and former US Assistant Attorney General and **Professor of
Law at UChicago Law School (Jack L. and Eric A., “The Limits of International Law,” April,
http://www.angelfire.com/jazz/sugimoto/law.pdf)

The Limits of International Law intends to fill that gap. The book begins with the premise that all
states, nearly all the time,
make foreign policy decisions, including the decisions whether to enter treaties and comply
with international law, based on an assessment of their national interest. Using a simple game-
theoretical framework, Goldsmith and Posner argue that international law is intrinsically weak and
unstable, because states will comply with international law only when they fear that
noncompliance will result in retaliation or other reputational injuries. This framework helps us
understand the errors of the international law advocates and their critics. On the one hand, large multilateral treaties
that treat all states as equal are unattractive to powerful states, which either refuse to enter the treaties,
enter them subject to numerous reservations that undermine the treaties’ obligations, or
refuse to comply with them. The problem with these treaties is that they treat states as equals when in fact they are
not, and they implicitly rely on collective sanctions when states prefer to free ride. Thus, many human rights
treaties are generally not enforced, and so they have little effect on states’ behavior. And the
international trade system is mainly a framework in which bilateral enforcement occurs, so powerful states may cooperate with
other powerful states but not with weaker states, whose remedies for trade violations are valueless. On the other hand,
international law is not empty or meaningless, as many critics have argued. States are able to cooperate with each other, especially
on a bilateral basis, and their patterns of cooperation eventually congeal into the customary international norms. Cooperation also
occurs within bilateral treaties and within the general frameworks set up in multilateral treaties. In the absence of a world
government, the cooperation remains relatively thin, and often erratic; its character changes as the
interests and relative power of nations change. But none of this is to claim that international law is phony or
illusory or a great public relations game. What it does suggest, however, is that international law has no life of its
own, has no special normative authority; it is just the working out of relations among
states, as they deal with relatively discrete problems of international cooperation. There is no
reason to expect states to enter treaties just for the sake of expanding the domain of
international law; and there is no reason to expect states to comply with treaties when their
interests and powers change. The aggressive international legalization expected and yearned for by
international lawyers just cannot happen as long as there are nearly 200 states with independent
interests, agendas, and ideologies. Even democratic states have no reason to commit them- selves to international
law when doing so does not serve the interests of the voters.

One plan won’t spillover – no change in US stance

Moravcsik ‘4 (Professor of Politics and International Affairs and director of the European Union Program at Princeton and
Nonresident Senior Fellow at the Brookings Institution (Andrew, “The Paradox of US Human Rights Policy” in American
Exceptionalism and Human Rights, p. 197)

This is a sobering conclusion, for it suggests that U.S.


ambivalence toward international human rights
commitments is not a short-term and contingent aspect of specific American policies. It is instead woven into the
deep structural reality of American political life. This is so not, for the most part, because international
human rights commitments are inconsistent with a particular understanding of democratic ideals like popular sovereignty, local
control, or expansive protection of particular rights shared by most Americans. It is true, rather, because a conservative
minority favored by enduring domestic political institutions has consistently prevailed in
American politics to the point where its values are now embedded in public opinion and
constitutional precedent. The institutional odds against any fundamental change in Madison’s
republic are high. To reverse current trends would require an epochal constitutional rupture –
an Ackermanian “constitutional moment” – such as those wrought in the United States by the Great Depression and the
resulting Democratic “New Deal” majority; in Germany, France, and Italy by the end of World War II; and in all European
countries through a half century of European human rights jurisprudence. Short of all that, this particular brand of American
ambivalence toward the domestic application of international human rights norms is unlikely
to change anytime soon.

Ilaw’s not binding

Casey ‘6 (Served in the Office of Legal Counsel in the U.S. Department of Justice (Lee A., “International Law and the Nation-State
at the U.N.: A Guide for U.S. Policymakers,” http://www.heritage.org/Research/Reports/2006/08/International-Law-and-the-Nation-
State-at-the-UN-A-Guide-for-US-Policymakers)

Finally, although international


law is generally considered to be part of American law, the United States, like other
sovereign nations, can derogate from the accepted rules. And, like other aspects of the nation's foreign relations, the
exercise of this authority falls-at least in the first instance-to the President. The Supreme Court's ruling in The
Paquete Habana is not to the contrary, although claims are sometimes made that it is. That case involved the U.S. Navy's
capture, during the Spanish-American War, of fishing boats in Cuba's coastal waters. The Supreme Court was called upon
to determine whether these vessels were lawful captures and concluded that they were not. Citing generally accepted rules of
international law suggesting that coastal fishermen were not to be molested by belligerent forces, the Court ruled that the
boats were not lawful "prizes" of war. However, in doing so, it specifically noted that "where there is no treaty and no
controlling executive or legislative act or judicial decision, resort must be had to the customs and usages of civilized
nations."[19] The suggestion is clear that, had
there been a formal decision by the President (or by
Congress through appropriate legislation) to ignore the otherwise applicable international rule, the United
States courts would have been bound by that decision.
Indo-Pak Answers
Frontline

No indo-pak war

Ganguly ‘8 [Sumit Ganguly is a professor of political science and holds the Rabindranath Tagore Chair at Indiana University,
Bloomington. “Nuclear Stability in South Asia,” International Security, Vol. 33, No. 2 (Fall 2008), pp. 45–70]

As the outcomes of the 1999 and 2001–02 crises show, nuclear deterrence is robust in South
Asia. Both crises were contained at levels considerably short of full-scale war. That said, as Paul Kapur
has argued, Pakistan’s acquisition of a nuclear weapons capability may well have emboldened its leadership, secure in the belief that
India had no good options to respond. India, in turn, has been grappling with an effort to forge a new military doctrine and strategy
to enable it to respond to Pakistani needling while containing the possibilities of conflict escalation, especially to the nuclear level.78
Whether Indian military planners can fashion such a calibrated strategy to cope with Pakistani probes remains an open question.
This article’s analysis of the 1999 and 2001–02 crises does suggest, however, that nuclear
deterrence in South Asia is
far from parlous, contrary to what the critics have suggested . Three specific forms of evidence can be
adduced to argue the case for the strength of nuclear deterrence. First, there is a serious problem of conflation in
the arguments of both Hoyt and Kapur. Undeniably, Pakistan’s willingness to provoke India
has increased commensurate with its steady acquisition of a nuclear arsenal. This period from
the late 1980s to the late 1990s, however, also coincided with two parallel developments that
equipped Pakistan with the motives, opportunities, and means to meddle in India’s internal
affairs—particularly in Jammu and Kashmir. The most important change that occurred was the end of the conflict with
the Soviet Union, which freed up military resources for use in a new jihad in Kashmir . This jihad, in
turn, was made possible by the emergence of an indigenous uprising within the state as a result of Indian political malfeasance.79
Once the jihadis were organized, trained, armed, and unleashed, it is far from clear whether Pakistan could control the behavior and
actions of every resulting jihadist organization.80 Consequently, althoughthe number of attacks on India did
multiply during the 1990s, it is difficult to establish a firm causal connection between the
growth of Pakistani boldness and its gradual acquisition of a full-fledged nuclear weapons
capability. Second, India did respond with considerable force once its military planners realized
the full scope and extent of the intrusions across the Line of Control . Despite the vigor of this
response, India did exhibit restraint. For example, Indian pilots were under strict instructions not to cross the Line of
Control in pursuit of their bombing objectives.81 They adhered to these guidelines even though they left
them more vulnerable to Pakistani ground ªre.82 The Indian military exercised such restraint to
avoid provoking Pakistani fears of a wider attack into Pakistan-controlled Kashmir and then
into Pakistan itself. Indian restraint was also evident at another level. During the last war in Kashmir in 1965, within a week
of its onset, the Indian Army horizontally escalated with an attack into Pakistani Punjab. In fact, in the Punjab, Indian forces
successfully breached the international border and reached the outskirts of the regional capital, Lahore. The Indian military resorted
to this strategy under conditions that were not especially propitious for the country. Prime Minister Jawaharlal Nehru, India’s first
prime minister, had died in late 1964. His successor, Lal Bahadur Shastri, was a relatively unknown politician of uncertain stature and
standing, and the Indian military was still recovering from the trauma of the 1962 border war with the People’s Republic of China.83
Finally, because of its role in the Cold War, the Pakistani military was armed with more sophisticated, U.S.-supplied weaponry,
including the F-86 Sabre and the F-104 Starfighter aircraft. India, on the other hand, had few supersonic aircraft in its inventory,
barring a small number of Soviet-supplied MiG-21s and the indigenously built HF-24.84 Furthermore, the Indian military remained
concerned that China might open a second front along the Himalayan border. Such concerns were not entirely chimerical, because a
Sino-Pakistani entente was under way. Despite these limitations, the Indian political leadership responded to Pakistani aggression
with vigor and granted the Indian military the necessary authority to expand the scope of the war. In marked contrast to the politico-
military context of 1965, in 1999 India had a self-confident (if belligerent) political leadership and a substantially more powerful
military apparatus. Moreover, the country had overcome most of its Nehruvian inhibitions about the use of force to resolve
disputes.85 Furthermore, unlike in 1965, India had at least two reserve strike corps in the Punjab in a state of military readiness and
poised to attack across the border if given the political nod.86 Despite these significant differences and advantages, the Indian
political leadership chose to scrupulously limit the scope of the conflict to the Kargil region. As K. Subrahmanyam, a prominent
Indian defense analyst and political commentator, wrote in 1993:. The awareness on both sides of a nuclear
capability that can enable either country to assemble nuclear weapons at short notice induces
mutual caution. This caution is already evident on the part of India. In 1965, when Pakistan
carried out its “Operation Gibraltar” and sent in infiltrators, India sent its army across the
cease-fire line to destroy the assembly points of the infiltrators. That escalated into a full-scale war. In 1990,
when Pakistan once again carried out a massive infiltration of terrorists trained in Pakistan, India tried
to deal with the problem on Indian territory and did not send its army into Pakistan-occupied
Kashmir.87

No Indo-Pak War

Wright ‘13 (Thomas Wright is a fellow at the Brookings Institution in the Managing Global Order project. Previously, he was
executive director of studies at the Chicago Council on Global Affairs, a lecturer at the Harris School of Public Policy at the University
of Chicago, and senior researcher for the Princeton Project on National Security, "Don’t Expect Worsening of India, Pakistan Ties,"
http://blogs.wsj.com/indiarealtime/2013/01/16/dont-expect-worsening-of-india-pakistan-ties/, January 16, 2013)

There’s no end for now to the hostile


rhetoric between India and Pakistan. But that doesn’t necessarily presage
anything more drastic. Pakistan claims another of its soldiers died Tuesday night in firing across the Line of Control in
Kashmir, the divided Himalayan region claimed by both nations. Indian army chief, Gen. Bikram Singh, on Wednesday, said Pakistan
had opened fire and India retaliated. “If any of their people have died, it would have been in retaliation to their firing,” Gen. Singh
said. ”When they fire, we also fire.” It was the latest in tit-for-tat recriminations over deaths in Kashmir that began last week.
Pakistan claimed one of its soldiers died on Jan. 6. Two days later, India said Pakistani forces killed two of its soldiers and mutilated
the bodies. Tuesday night, Indian Prime Minister Manmohan Singh said the mutilations meant it could not be “business as usual”
between the countries. That has worried some that peace talks, which have been in train for two years, could be about to break
down. Mr. Singh’s comments built on a drumbeat of anger from India. Gen. Singh, Monday called the mutilations “unpardonable”
and said India withheld the right to retaliate to Pakistan aggression when and where it chooses. Pakistan Foreign Minister Hina
Rabbani Khar, who is in the U.S., Tuesday termed the Indian army chief’s comments as “very hostile.” There are some other worrying
signs. India said Tuesday it was delaying the start of a visa-on-arrival program meant to make it easier for some Indians and
Pakistanis to visit each other’s countries. The visa program, like talks on opening up bilateral trade, is supposed to pave the way
toward broader peace talks that would encompass thornier issues, like how to solve the Kashmir problem. Also Tuesday, nine
Pakistani hockey players who had come to participate in a tournament in India were sent home due to fears of protests and violence
against them. Still, there’s little benefit for either side to escalate what is now still sporadic firing over the
Line of Control, the de facto border in Kashmir. Pakistan is embroiled in its own political meltdown sparked by
the Supreme Court’s decision Tuesday to order the arrest of Prime Minister Raja Pervez Ashraf on allegations of corruption. Tens of
thousands of protesters Tuesday took to the streets in Islamabad, and remain there today, demanding immediate elections and a
greater role for the army and Supreme Court in politics. Pakistan’s military continues to play an important political role,
dominating defense and foreign policy. But it has so far shown little sign of mounting a full-blown coup despite
persistent rumors of military intervention. Pakistan’s government must hold national elections by May,
meaning the next few months are likely to be choppy ones in Pakistan politics. In such an environment, the military is
unlikely to want to dial up tensions with India. On the Indian side, despite Mr. Singh’s unusually strident tone
Tuesday, there also will be pause before taking matters to the next level. Mr. Singh has put immense personal political capital into
trying to improve ties with Pakistan since he came to power in 2004. Last year, he hosted Pakistan President Asif Ali Zardari in New
Delhi and promised a return visit. Such a trip is clearly off the table for now. But India still has
put too much into peace
talks to throw away the progress made so far on visas, trade and other issues. Even Gen. Singh, India’s army chief,
Monday said he did not believe the latest flare-up would lead to a broader escalation in violence and an
official end to a 2003 ceasefire agreement in Kashmir. The clashes so far, he noted, have been limited to specific areas of the Line of
Control.
CBMs solve

Botez ‘11 (Radu, Writer @ Open Security, a think tank specializing in contemporary conflicts, “ India-Pakistan talks slowly move
forward,” June 29, http://www.opendemocracy.net/opensecurity/security_briefings/290611,

At the end of last week, foreign


secretaries Nirupama Rao of India and Salman Bashir of Pakistan met in Islamabad
to discuss security issues and prepare the upcoming meeting of the countries' foreign ministers in India in July. The
meeting was the first at foreign secretary level since July 2009. In February, India and Pakistan announced
they would resume peace talks that India had broken off following the Mumbai attacks in late 2008. Discussions
revolved around the issues of terrorism and the territorial dispute over Kashmir, a divided region claimed by both states in
its entirety. India has blamed Pakistan-based terrorist group Lashkar-e-Taiba (LeT) for the Mumbai attacks, supported by elements
from ISI, Pakistan's main intelligence agency, according to one of the planners, recently on trial in a Chicago court. It has repeatedly
asked Pakistan to act against militants on its territory and bring to trial those involved in plotting the attacks. The surroundings of
Osama bin Laden's killing were seen by many in New Delhi as a confirmation of Pakistan providing shelter to terrorists. However,
after talks between the states' home secretaries earlier this year, Pakistan agreed to allow Indian investigators to visit Islamabad.
Rao and Bashir also said they would look into confidence building measures (CBMs) with regard to
their nuclear and conventional weapons capability . On Tuesday, Pakistani defence minister Chaudhry Ahmad
Mukhtar said that India had a greater capacity to sustain a war. In May, Pakistan tested a new short-range ballistic missile that could
lead to the nuclear threshold being crossed early in the event of a conflict analysts say. According to observers, both parties were
cautious in addressing sensitive issues and talks have not led to any major breakthrough. The openSecurity verdict: With the recent
meeting, India and Pakistan have once again indicated their intentions to improve relations . The
most prominent episode of demonstrating goodwill took place when Pakistani prime minister Yusuf Raza Gilani visited India earlier
this year to attend the cricket world cup semi-final between the two countries' teams, joined by his Indian counter-part Manmohan
Singh. Both leaders engaged in so-called 'cricket diplomacy', spending several hours together on and off the field. As welcome as this
may be, it will not help to resolve issues both countries have gone to war over several times in the past.

No Indo-Pak war- no intent OR capability

Athale ’11 (Why an Indian threat to Pakistan is a myth Last updated on: June 1, 2011 10:32 IST Colonel (Dr) Anil Athale (Retired)
Colonel (Dr) Anil Athale (retd) is coordinator of the Pune-based Initiative for Peace and Disarmament.

This is season for busting long held myths. The Abbottabad raid by the Americans without the apparent knowledge
of Pakistanis and Osama bin Laden's long stay there in a garrison town has exploded the myth of the Inter Services Intelligence as
the world's 'premier' spy outfit. Though this was more a creation of the lazy Indian media, it had acquired a life of its own. In reality,
the ISI has been a major failure. It has witnessed a number of attacks on its own offices in Pakistan and terrorist attacks on many
targets, the latest being the one on a major naval aviation base near Karachi. The truth is that the ISI's reputation was built on the
many successful operations it carried out in/against India. But these attacks could only be carried out because the ISI had a huge
support base in India in the shape of organisations like the Students Islamic Movement of India, Indian Mujahideen, Ahle Hadith and
the Tabiligi Jamaat. When bereft of this kind of support in its own country, the ISI has moved from blunder to blunder. But the
mother (and father) of all myths is the notion of 'perennial Indian security threat to Pakistan'. This
myth has enabled the Pakistan Army to milk their country dry of all resources and also keep a firm grip on power by promoting fear
psychosis. It is time cold military logic is brought to bear on this. Threat perception is a sum total of capability plus intention. Let us
take intention first. Except for a tiny fringe element, no
major political party in India has any interest in
undoing Partition. It is not out of any charity but due to the fact that addition of turbulent,
violent 180 million people to India's existing billion is not a prospect that any Indian relishes.
Given the dismal socio-economic and educational statistics and the mess that Pakistan is, no sane Indian wants that part
of undivided India to become part of India again. Pakistanis could point out to the Indian intervention in erstwhile
East Pakistan, now Bangladesh or Sri Lanka (the Indian Peace Keeping Force episode) as evidence of Indians' ill intentions. It is
necessary to point out the fact that Bangladesh is an independent country today and there is not a single Indian soldier or Indian
base either there or in Sri Lanka. After the task of sending back the refugees was achieved, India withdrew from Bangladesh. The Sri
Lankans were sceptical of Indian intentions, so much so that the then President R Premadasa even collaborated with the Liberation
Tigers of Tamil Eelam. Yet even the most anti-Indian Sri Lankans today accept that India made great sacrifices and left on its own. It
is time to remind the Pakistanis that India returned the Shakkargarh area as well as areas in Sindh after the 1971 war along with the
92,000 prisoners of war. If India, like Israel after 1967 war, had intentions to grab Pakistani land, it could well have continued to dilly-
dally its return! Ruling a hostile country of 180 million is beyond the capability of India and there
are no attractive
economic gains or scarce resources like oil in Pakistan that could attract India. There is no earthly
'interest', economic, political or military that would serve by India conquering Pakistan . The oft-
heard argument of Pakistani military leaders and security experts is that they plan their security based not on 'intent' but on
capability. The logic is that intent can change any time. Accepting this argument, though a dubious one (for instance India has
obvious military capability to attack and capture all other six countries in South Asia but no 'interest' and therefore no intent to do
so), let us examine the military balance between India and Pakistan. The Indian Army is 1.1 million strong compared to the 0.55
million strong Pakistan Army. But it must be noted that at least 1/3 of this force is needed at all times to guard the nearly 2,000 km
of India-Tibet border. Even during the 1971 war when the Soviet Union had deployed close to 44 divisions against China (from the
normal three divisions) on the Amur-Ussuri river border, India could not withdraw much of the forces from North. The situation
today is very different -- there is no Soviet Union to force the Chinese into passivity (as in 1971) and with a rail link to Lhasa, China is
in a stronger position in Tibet. In
case of any future conflict between India and Pakistan, only around 8
lakh troops of India would be available against Pakistan to match 4 to 5 lakh granting that around 1 lakh troops
are on the Afghan border. This is much less than the normal three times superiority that is needed for a
successful aggression. Modern warfare is technology intensive. In terms of sophistication, thanks to the American supplies
post 9/11, in most of the fields, Pakistan has far better equipment than the Indians. The table below gives an idea of the various
major armaments of both. Old tanks Modern tanks Anti-tank guns/missiles Artillery guns India 3673 444 3000 10,758 Pakistan 1821
835 14400 4521 Source: Sipri and IISS. Thus while India has a certain edge in overall number of tanks, in terms of sophistication,
Pakistan is actually ahead of India. Some years ago, the Indian Army chief had openly acknowledged that most Indian tanks lacked
night fighting capability. To counter larger Indian numbers, Pakistan has far more anti-tank guns and missiles. Thus the so-called
Indian conventional edge is a myth. In artillery India does have greater numbers, but it must be remembered that at least 1/3 of
these would be on the China border. Readers may remember how in 1999, the final assault on Kargil heights was delayed as guns
had to be moved from China border. Modern wars are often called land-air battles. The air arm plays a crucial role in land campaign
as well as strategic tasks in depth areas. Here India enjoys a clear edge in the sense that to its 385 modern fighter/ground attack
aircraft, Pakistan can field only 58. India also has another 189 older aircraft while bulk of Pakistani aircraft numbering 374 are old
and obsolete. In the air war, technology is far more important and with its Sukhoi 30 MKI, India has a clear superiority. But Pakistan
has emphasised the defence aspect and has close to 226 fighters to defend its air space as compared to 112 of India. The India Air
Force is certainly capable of creating air superiority on parts of Pakistan for a limited period. When in the past the Indian Air Force
chief's stated that India can attack and destroy terror camps, he was banking on this superiority. Post bin Laden's killing as Pakistan
apprehended that India may try similar attack; the Pakistan PM went to China to redress the balance in air. In May 2011, China has
promised immediate delivery of 50 fighter aircraft to Pakistan. On sea, however, the picture is very different.The Indian Navy out-
ships and outguns Pakistan by a factor of 8, true to its size. Two factors mitigate this Indian superiority. Clear that it lacks resources
for any kind of control of sea, Pakistan went in for a sea denial strategy. It must be remembered that it was Pakistan that had the
first ever submarine in the subcontinent. Even today, while India has 16 submarines (majority of them very old and due for
retirement), Pakistan has 8 modern submarines. In addition due to its own reasons, the US navy has a very large presence in the
northern Arabian Sea. With the close alliance between the US and Pakistan, the later can be assured of a degree of protection. The
main conclusion from this very broad analysis is that despite the great disparity in all other
fields between India and Pakistan, in terms of military capability India has only a slight edge.
The picture of near parity becomes even clearer when we take account of the nuclear
weapons. It is undoubtedly true that Pakistan's small size vis a vis India and specially the fact that its strategic, fertile and
densely populated areas are all along its eastern border within 150 km belt, it is vulnerable to Prithvi missiles of India. In case of
an all out nuclear exchange, India can decimate Pakistan . On the other hand, even with limited success in
strikes with nuclear weapons, Pakistan can certainly destroy 5-10 Indian cities . The destruction of Mumbai-Pune
industrial belt alone can damage 40 percent of Indian industry. The question is, is it therefore worthwhile for
India to even think in terms of nuclear exchange? The obvious answer is NO. It is this that has
been holding Indian reaction to repeated terror strikes from Pakistan. In the foreseeable future, till India
does not acquire a capability to destroy most if not all of Pakistan's nuclear weapons and their delivery system, Pakistani
deterrence will continue to work. An effective missile defence shield is still in the embryo. As seen earlier, India
gains nothing from destruction/conquest of Pakistan. Irrespective of that it does not have the
conventional military capability to do so. Pakistan has a strong defence aided by the terrain like Punjab where canals
are a major obstacle to rapid attacks. But given its edge in the air power, India has the capability to mount a surgical attack in
retaliation to a Mumbai 26/11 type terror attack. If India alerts its nuclear forces prior to this action and clearly tells Pakistan that
this is limited punishment and should Pak think of nuclear response, then we can 'pre-empt' with a conventional attack on their
nuclear sites to be followed by nuclear one if Pakistan prepares to use them first, we should be able to call the Pakistani nuclear
bluff. This is all that the 'Cold Start' doctrine is all about. In this sense Pakistan does face a threat of a surgical and limited Indian
strike. But for Pakistan the option is very clear, to avoid this attack it has to stop creating monsters like the Lashkar-e-Tayiba or the
Jaish-e-Mohammad. If there are no terror strikes mounted from Pak against India, the 'Cold Start' doctrine will remain mere
doctrine. From the foregoing discussion it is clear that India has neither the intent nor the capability to pose a
major threat to Pakistan. It does, however, have the ability to mount raids and surgical strikes. A retired Pakistani general
Ikram Sehgal in an article in The News (May 26) has written a humorous article justifying the bogey. The general cleverly enumerates
all the Indian formations that are poised against Pakistan, completely omitting any mention what Pakistan has to oppose it! The
general wants the gullible Pakistani 'janata' to believe that India is out to do them in. The Pakistani theme song of Indian threat is
based on the root cause theory. The argument goes something like this -- Kashmir should have come to Pakistan and India is refusing
to hand it over, so India is the aggressor and in occupation of Pakistani territory in Kashmir. It is thus the Pakistan desire to get
Kashmir through force that is at the root of subcontinents problems. Even for the sake of argument one is to accept Pakistani
position, it still begs the question about its inability to take Kashmir. Pakistan has tried in 1947 and 1965, direct military means. Since
1980s, it is trying to get Kashmir by supporting an insurgency there. Given the military balance and the public opinion in India that
will not accept another partition on basis of religion, Pakistan cannot get Kashmir for another 1,000 years. The recent revelations by
David Colman Headley alias Dawood Jilani have brought into open the relationship between terrorist organisation like the LeT and
Pakistani armed forces (through its intelligence wing, the ISI). One need not be a rocket scientist to know that no amount of terror
attacks like the 26/11 on Mumbai can wrest Kashmir from India or destabilises the country. Then why these attacks like the one in
Chattisingpura on March 20, 2000, attack on Indian Parliament on 13 December 2001, the Mumbai train blasts on 11 July 2006 or
the latest Mumbai attack on 26/11, 2008? The biggest impact of these actions is to raise tension between India-Pakistan. This serves
the following Pakistani Army objectives: 1) Indian reaction to these terror strikes is described as 'threat' to keep Pakistanis united. 2)
Keep the primacy of the army and get unlimited resources. 3) In case it provokes a backlash within India, it further strengthens
support to groups like SIMI etc within India and helps the ISI. 4) Use it as an excuse to reduce efforts in war on terrorism. The
terror attacks on India are a win-win situation for the Pakistani Army. With their considerable
military resources and nuclear threat they know that India cannot pose any credible threat to
Pakistan. Hence, it is a no risk option. On the other hand the resultant tensions help the army keep its iron grip on
the hungry, turbulent and underdeveloped country. International relations are often described as 'geo-
politics'. In this the geographical element is unchanging and static, hence the politics or polices
have to change to achieve desired results.

Tension low now

Polgreen, 11 (Lydia, NYT staff writer in India, 7/27/11, http://www.nytimes.com/2011/07/28/world/asia/28india.html?_r=1_

NEW DELHI — The foreign ministers of India


and Pakistan met here on Wednesday, agreeing to a set of small but
significant concessions to ease tensions in the disputed border region of Kashmir and pledging
to work toward closer ties between their mutually wary, nuclear-armed countries. The meeting came just two weeks after
three synchronized explosions ripped through the city of Mumbai at rush hour, killing 24 people, wounding more than 100 and
raising fresh Indian suspicions about possible Pakistani subterfuge. The
decision by both sides to proceed with the
meeting anyway signaled that broad-based talks aimed at resolving issues between the
countries were back on track. They had been stalled for more than two years after Pakistani gunmen killed more than
160 people in a rampage through Mumbai, formerly Bombay. “We have reaffirmed our commitment to resolve
all outstanding issues through a comprehensive, serious and sustained dialogue,” said S. M. Krishna,
India’s foreign minister, after the meeting. “This is indeed a new era of bilateral cooperation,” said Hina Rabbani
Khar, Pakistan’s new foreign minister, the first woman to hold that position and, at 34, one of the youngest. “I believe it is the desire
of both countries to make an uninterrupted and uninterruptible process.” India and Pakistan share a long and fractious border and
have fought three wars, two of them over Kashmir, which both claim. Other issues, like sharing water from the Indus Valley rivers,
are also difficult points of contention. Both countries have nuclear weapons. The United States has long wanted India and Pakistan
to resolve their differences so that Pakistan might focus on fighting the Taliban and other Islamic militants on its western frontier.
The measures the ministers announced were relatively small but represent a significant change in tone and outlook for the
relationship between the countries, analysts said. The number of days cross-border trade will be allowed between the two sides of
Kashmir will be doubled to four per week. The two governments agreed to make it easier for Kashmiris from either side to cross the
border. Travel permits are currently issued only to people with relatives on the other side, but on Wednesday the ministers agreed
to allow people to visit for tourism and religious pilgrimages. They also pledged to reduce the amount of time it takes for applicants
to secure travel permits to 45 days or less, rather than the three to four months it currently takes. The two ministers agreed that
they would meet in the first half of 2012 to discuss progress.
---Indo-Pak- Deterrence Solves

It would never escalate

Economic Times ‘11 (“No chance of Indo-Pak nuclear war despite 'sabre rattling': Pak nuclear scientist A Q Khan,”
http://articles.economictimes.indiatimes.com/2011-05-17/news/29552014_1_nuclear-blackmail-nuclear-secrets-india-and-pakistan,
May 17, 2011)

NEW YORK: Pakistan's disgraced nuclear scientist A Q Khan has said that despite "sabre rattling" between Islamabad
and New Delhi, there is no chance of a nuclear war between the two neighbours . Khan, who has
been accused of selling nuclear secrets to Iran, Libya and Syria, wrote in Newsweek magazine that nuclear weapons in both
countries had prevented war for the last 40 years. "India doesn't need more than five weapons
to hurt us badly, and we wouldn't need more than 10 to return the favour ," he said. "That is why
there has been no war between us for the past 40 years." "India and Pakistan understand the
old principle that ensured peace in the Cold War: mutually assured destruction," he said. "The two
(India and Pakistan) can't afford a nuclear war , and despite our sabre rattling, there is no chance
of a nuclear war that would send us both back to the Stone Age ," he said. He claimed that Pakistan had to
invest in a nuclear programme "to ward off nuclear blackmail from India". "I would like to make it clear that it was an Indian nuclear
explosion in May 1974 that prompted our nuclear program, motivating me to return to Pakistan to help create a credible nuclear
deterrent and save my country from Indian nuclear blackmail," he said. "We are forced to maintain this deterrence
until our differences with India are resolved. That would lead to a new era of peace for both
countries," Khan wrote. "I hope I live to see Pakistan and India living harmoniously in the same way as the once bitter enemies
Germany and France live today," he said. Khan blasted various governments in Pakistan as well as "successive incompetent and
ignorant rulers" for not engaging in basic development of the country, and raising the people's standard of living. " We
are far
worse off now than we were 20, or even 40, years ago when we were subjected to embargoes ,"
he said.

Deterrence prevents India/Pakistan conflict.

Tepperman ‘9 ( 9/7/2009 (John - journalist based in New York Cuty, Why obama should learn to love the bomb, Newsweek,
p.lexis)

The record since then shows the same pattern repeating: nuclear-armed enemies slide toward war,
then pull back, always for the same reasons. The best recent example is India and Pakistan,
which fought three bloody wars after independence before acquiring their own nukes in 1998.
Getting their hands on weapons of mass destruction didn't do anything to lessen their animosity. But it did dramatically
mellow their behavior. Since acquiring atomic weapons, the two sides have never fought
another war, despite severe provocations ( like Pakistani-based terrorist attacks on India in 2001 and 2008). They
have skirmished once. But during that flare-up, in Kashmir in 1999, both countries were careful to keep
the fighting limited and to avoid threatening the other's vital interests. Sumit Ganguly, an Indiana
University professor and co-author of the forthcoming India, Pakistan, and the Bomb, has found that on both sides, officials' thinking
was strikingly similar to that of the Russians and Americans in 1962. The prospect of war brought Delhi and
Islamabad face to face with a nuclear holocaust, and leaders on each side did what they had to
do to avoid it.

Deterrence

Tellis ‘02 (Ashley, Foreign Policy Research Institute, Orbis, Winter, p. 24-5)

In the final analysis, this situation is made objectively "meta-stable" by the fact that neither India, Pakistan,
nor China has the strategic capabilities to execute those successful damage-limiting first strikes that might
justify initiating nuclear attacks either "out of the blue" or during a crisis. Even China, which of the three comes closest to
possessing such capabilities (against India under truly hypothetical scenarios), would find it difficult to conclude that the
capacity for "splendid first strikes" lay within reach. Moreover, even if it could arrive at such a determination, the
political justification for these actions would be substantially lacking given the nature of its current political
disputes with India. On balance, therefore, it is reasonable to conclude that a high degree of deterrence
stability, at least with respect to wars of unlimited aims, exists within the greater South Asian region.

Deterrence Solves

Altaf ’11 (Nuclear deterrence in South AsiaFrom the Newspaper | Letters | 27th September, 2011 SYED GOHAR ALTAF Quaid-i-
Azam University, Islamabad

THIS is apropos of the article ‘Security and FMCT’ by Zahir Kazmi (Sept 20). There is no doubt that South Asian peace and
stability heavily, indeed completely, rests on nuclear deterrence and the ‘balance of terror’ that exists between
nuclear-armed Pakistan and India. Moreover, one fully agrees with Mr Kazmi’s analysis that the West’s double standards
have threatened the non-proliferation regime and arms control-cum-disarmament efforts more than anything else.
Historically, the ‘second nuclear age’ is in vogue. In the first nuclear age, which started with the United States nuclear tests, the
world was about to reach the brink of demolition. However, nuclear deterrence
and the doctrine of MAD — mutual
assured destruction — emerged in the 1970s and successfully averted the ‘doomsday scenario’, which was
being drummed up by nuclear pessimists. Similarly, in today’s second nuclear and post-cold-war age, ‘credible
minimum deterrence’ can play a pivotal role in maintaining peace and stability, especially in the
South Asian context. As a matter of fact, South Asian stability is a direct outcome of a robust nuclear
deterrence that prevails between Pakistan and India.

Deterrence solves

Alagappa’9 (Distinguished Senior Fellow at the East-West Center, PhD in International Affairs from the Fletcher School of Law
and Diplomacy, 2009, Muthiah Alagappa, “Nuclear Weapons Reinforce Security and Stability in 21st Century Asia”, Vol 4 No 1)
The stabilizing effect of nuclear weapons may be better illustrated in India-Pakistan relations, as the crises between these two
countries during the 1999–2002 period are often cited as demonstrating nuclear weapon-induced instability. Rather than simply
attribute these crises to the possession of nuclear weapons, a more accurate and useful reading would ground them in Pakistan’s
deliberate policy to alter the status quo through military means on the premise that the risk of escalation to nuclear war would deter
India from responding with full-scale conventional retaliation; and in India’s response, employing compellence and coercive
diplomacy strategies. In other words, particular goals and strategies rather than nuclear weapons per se precipitated the crises.
Further, the outcomes of these two crises revealed the limited utility of nuclear weapons in bringing about even a minor change in
the territorial status quo and highlighted the grave risks associated with offensive strategies.
Recognition of these limits
contributed to the two countries’ subsequent efforts to engage in a
and the grave consequences in part
comprehensive dialogue to settle the many disputes between them . The crises also led to
bilateral understandings and measures to avoid unintended hostilities . Though it is too soon to take a long
view, it is possible to argue that, like the Cuban missile crisis in 1962, the 1999 and 2001–02 crises between India and Pakistan mark
the danger of nuclear war shifted their focus to avoiding a major
a watershed in their strategic relations:
war and to finding a negotiated settlement to bilateral problems . Large-scale military deployments along
the common border, Pakistan-supported insurgent activities in India, and cross-border terrorism continue; and the two countries
regularly conduct large-scale military exercises and test nuclear-capable missiles that have each other’s entire territory within range .
Despite these activities, the situation has become relatively less tense ; stability with the ability to
absorb shocks even like that created by the November 26 terrorist attack in Mumbai has begun
to characterize the bilateral relationship.

Expert consensus

Enders, 02 (David Enders, “Experts say nuclear war still unlikely,” Michigan Daily, January 30th, 2002, pg.
http://www.michigandaily.com/content/experts-say-nuclear-war-still-unlikely)

University political science Prof. Ashutosh Varshney becomes animated when asked about the likelihood of nuclear war
between India and Pakistan. "Odds are close to zero," Varshney said forcefully, standing up to pace a little bit in his
office. "The assumption that India and Pakistan cannot manage their nuclear arsenals as well as the U.S.S.R. and U.S. or Russia and
China concedes less to the intellect of leaders in both India and Pakistan than would be warranted." The world"s two youngest
nuclear powers first tested weapons in 1998, sparking fear of subcontinental nuclear war a fear Varshney finds ridiculous. " The
decision makers are aware of what nuclear weapons are, even if the masses are not," he said.
"Watching the evening news, CNN, I think they have vastly overstated the threat of nuclear
war," political science Prof. Paul Huth said. Varshney added that there are numerous factors working against the
possibility of nuclear war. "India is committed to a no-first-strike policy," Varshney said. "It is virtually
impossible for Pakistan to go for a first strike, because the retaliation would be gravely
dangerous." Political science Prof. Kenneth Lieberthal, a former special assistant to President Clinton at the National
Security Council, agreed. "Usually a country that is in the position that Pakistan is in would not shift
to a level that would ensure their total destruction," Lieberthal said, making note of India"s considerably larger
nuclear arsenal. "American intervention is another reason not to expect nuclear war," Varshney said. "If anything has happened
since September 11, it is that the command control system has strengthened. The trigger is in very safe hands." But the low
probability of nuclear war does not mean tensions between the two countries who have fought three wars since they were created
in 1947 will not erupt. "The possibility of conventional war between the two is higher. Both sides are looking for ways out of the
current tension," Lieberthal said.
---Indo-Pak- Econ Checks

Economic constraints solve war

Tellis ‘02
(Ashley, Foreign Policy Research Institute, Orbis, Winter, p. 19)

In any event, the


saving grace that mutes the potential for exacerbated competition between both
countries remains their relatively strong economic constraints. At the Pakistani end, these
constraints are structural: Islamabad simply has no discretionary resources to fritter away on an open-ended arms race,
and it could not acquire resources for this purpose without fundamentally transforming the nature of the Pakistani state
itself—which transformation, if it occurs successfully, would actually mitigate many of the corrosive forces that currently drive
Islamabad’s security competition with India. 21 At the Indian end, these constraints may be more self-
imposed. New Delhi commands a large pool of national resources that could be siphoned off and reallocated to security
instruments, but the current weaknesses of the central government’s public finances and its reform program, coupled with its desire
to complete the technological modernization programs that have been underway for many decades, prevents it from enlarging the
budgetary allocations for strategic acquisitions at will. 22 With
these constraints on both sides, future
nuclearization in India and Pakistan is more likely to resemble an "arms crawl" than a genuine
Richardson-type "arms race." The strategic capabilities on both sides will increase incrementally but slowly
—and in India will have further to go because of its inferior capabilities compared to China’s. This slowness may be the best outcome
from the viewpoint both of the two South Asian competitors and the United States.
---Indo-Pak- No Impact

No regional spillover

Gwynne Dyer, 02 Ph.D. in war studies from the University of London, serves on the Board of Governors of Canada’s Royal Military
College, independent journalist, May 24, 2002, Hamilton Spectator, “Nuclear war a possibility over Kashmir,” p. Lexis

For those who do not live in the subcontinent, the most important fact is that the damage would be largely
confined to the region. The Cold War is over, the strategic understandings that once tied India
and Pakistan to the rival alliance systems have all been cancelled, and no outside powers would be
drawn into the fighting. The detonation of a hundred or so relatively small nuclear weapons over India and
Pakistan would not cause grave harm to the wider world from fallout.

No nuclear winter

Ivan Eland, 03 Senior Fellow and Director of the Center on Peace & Liberty at The Independent Institute, Assistant Editor of The
Independent Review, Fall, The Independent Review, “The New Nuclear Danger: George W. Bush’s Military-Industrial Complex; Book
Review,” Vol. 8, No. 2, http://www.independent.org/tii/content/pubs/review/books/tir82_caldicott.html

Caldicott’s far-fetched arguments do the most harm to the cogent parts of her book. She asserts that in a possible
nuclear exchange between India and Pakistan, “if any of either Russia or America’s 2,250 strategic
weapons on hair-trigger alert were launched either accidentally or purposefully in response, nuclear winter would
ensue, meaning the end of most life on earth” (p. xiii). Even if the dubious nuclear-winter thesis is accepted, a
mini-nuclear war on the Asian subcontinent plus one Russian or U.S. nuclear warhead is unlikely
to bring about such cataclysmic climate change.
---Indo-Pak- Cold Start Answers

Cold Start was ended

Ahmed ’11 (INDIA - ARTICLES Print Bookmark Email Facebook #3332, 15 February 2011 Towards an Indo-Pak Nuclear Lexicon -
III: Cold Start Ali Ahmed Research Fellow, IDSA email: aliahd66@hotmail.com

The Army Chief wrote the epitaph on Cold Start, stating, “There is nothing called ‘Cold Start’.” But
it must be acknowledged that in its short shelf life of seven years it had spawned a cottage industry of
commentators on it. Like the proverbial ‘blind men of Hindoostan’, they inspected Cold Start and found it viable or
otherwise, depending on their avian nature as hawks or doves. That Cold Start is no more is to be mourned since
it did enliven strategic commentary, helped create a strategic culture and brought the Indian political and security
establishment to see the military, finally, as a potentially usable instrument. Yet, that the doctrine remained untested
owes to India’s grand strategy, characterized lately as ‘strategic restraint’ . Strategic restraint is
a coming to terms with limitations of force, brought on by things such as the economy and the
nuclear backdrop. The term ‘Cold Start’ for India’s conventional doctrine was conjured up in a
media briefing by a ‘source’ on the sidelines of the Army Commander’s conference in the summer of 2004. The doctrine had been
discussed in that conference but was adopted in the next conference in autumn that year. It was a culmination of the changes in the
Army brought on by the lessons of the two preceding crises, the Limited War in Kargil and the subsequent exercise in coercive
even as military analysts made their careers dissecting it, the
diplomacy, Operation Parakram. Incidentally,
government in the form of the Ministry of Defence, took care never to refer to it. This suggests that
it was a legacy of the previous government, with an image of being more defence-friendly. The NDA dispensation
had taken care to bring out the nuclear doctrine while in power, tying India down to its expansive formulation of ‘massive’ nuclear
retaliation against India or its forces ‘anywhere’. The threat was to create the space necessary to operationalize a proactive
conventional war doctrine. ‘Cold Start’ was a limiting description of the doctrine. Cold Start refers only to the kind of capability the
Army gave itself, of going into conventional operations from a standing start. This owed to its embarrassment in having taken three
weeks to mobilize after the parliament attack, thereby seemingly losing an opportunity to administer punishment and thereby instill
caution in Pakistan. The doctrine envisaged a series of limited thrusts across a wide front into Pakistan by divisional sized forces.
These served to capture territory, seize the initiative and provide a launch pad for strike corps to deepen the penetration. Enemy
reserves would thus be forced to react and expend themselves. Attrition from the air would enable whittling down the Pakistani
Army, seen as the center of gravity. The Army, suitably degraded, would be displaced from power post-conflict in Pakistan, enabling
a democratic peace to ensue unlike the last time round post 1971. The doctrine had much to recommend it. It brought India’s
military advantage back into the reckoning despite nuclearization. It enabled taking the first tier of defences when they were unheld
or relatively underdeveloped. This would have saved India from casualties, particularly in the mountains. This would have helped
prevent the development of an attrition match as had occurred in the earlier wars. It would be a war fought on Pakistani territory,
thus sparing India of the effects of the increasing lethality of war. Making early gains, India could call for early war termination on its
terms. Since the nuclear threshold was to be respected, there would be no call for Pakistan to resort to nuclear weapons.
Nevertheless, the doctrine had its criticism. It was seen as narrowing the opportunity for crisis management and conflict avoidance
through diplomacy. It would set South Asia a short fuse vulnerable to any passing bunch of self-interested jihadis. A default military
reaction in real time would place the two states at odds with each other, playing into the hands of non-state actors who would
expect to gain from the resulting instability in Pakistan. It
was deemed to be too deterministic about the
location of the Pakistani nuclear threshold . It was taken as high, enabling operations to a limited depth. This was
to be unmindful of the influence of a conflict environment on decision-making. It was politically naïve in its belief that knocking
down the Pakistani Army would result in democratic forces prevailing. The jihadis could instead have come to the fore. This could be
due to Pakistan using them in an Iraqi style asymmetric war against Indian conventional forces in Pakistani territory. Lastly, it was
seen as part of the inter-Service rivalry, with the Army trying to set the agenda as the lead service, in reply to the Air Force’s intent
of an ‘air alone’ strategy in an indigenous version of ‘Shock and Awe’.
---Indo-Pak- Terrorism Answers
India terror attacks won’t cause Indo-Pak war

Chaudhary ’11 (India faces chronic low-grade terror threat, but Pakistan relationship is safe Posted By Ian Bremmer
Wednesday, September 14, 2011 - 4:45 PM Share By Shamila N. Chaudhary Shamila N. Chaudhary is an analyst in Eurasia Group's
Asia practice.

The Sept. 7 bombing of the Delhi High Court that killed 13 underscores the ever-present security
threat from militants in India's major cities . But while relatively small attacks are likely to be a fact of
life for the foreseeable future, they do not fundamentally change the security picture. The
opposition Bharatiya Janata Party (BJP) is, however, likely to ramp up its criticism of the
Congress government. More importantly, the attacks will not cause a strategic shift in the
Indian government's relationship with Pakistan, despite the claims of responsibility from radical
Islamist groups with connections to Pakistan . It's still unclear who is responsible for the attack. A member of the
Pakistan and Bangladesh based Harkat-ul-Jihad al-Islami (HuJI) terror group purportedly sent an email claiming responsibility for the
blast. But another email on Sept. 8 claimed responsibility for the Indian Mujahideen (IM), the main suspect in the July 13 bombings
in Mumbai. HuJI's email claimed the purpose of the attack was to coerce the Indian government into commuting the death sentence
of Afzal Guru, convicted of conspiracy in the 2001 attack on India's parliament building. Both groups have executed a number of
similar attacks in India over the last several years. The Indian government has not identified the group responsible for the blast, but
three Kashmiri men have been arrested. Regardless of which group is culpable, the attack may boost limited domestic pressure on
the Congress government, but will have little effect on India's relationship with Pakistan. Immediately
after the attack,
Prime Minister Manmohan Singh called for cooperation, not accusations , in the face of the growing
terror threat in India. This message may have been intended more for domestic audiences than
targeted at Pakistan, given the political pressures the Congress party currently faces in light of a
series of corruption scandals and the July Mumbai bombings. While national elections will not be held until
2014, the BJP is likely to once again make the government's poor handling of terrorism a campaign issue despite calls for additional
intelligence gathering. The fragile nature of the ongoing dialogue with Pakistan also factors into Singh's calculus. Singh perceives the
dialogue as one of his legacy issues, but enjoys little political support in the Indian government outside a handful of senior officials
any derailment of the dialogue with Pakistan limits India's ability to influence
and aides. But
Islamabad for more progress on the trials related to the 2008 Mumbai attacks . India is also likely
to mute its response because of its desire to sustain its development and diplomatic presence in
Afghanistan without threat from Pakistan-based groups. Pakistan will also manage its message closely, as its
damaged relationship with the United States is still on the mend. Neither does the resurgence of hostile rhetoric with India serve the
interests of the civilian and military leadership, which are both desperate to improve their domestic image after the May 2 raid that
killed Osama bin Laden.

Empirically denied

Riedel ’11 (Bruce Riedel, Senior Fellow at Brookings (9-26-2011, senior fellow at the Saban
Center at the Brookings Institution and an adjunct professor at the School for Advanced
International Studies of Johns Hopkins University. The Daily Star, September 26, 2011)
At least twice jihadists have tried to provoke war between India and Pakistan. The first time
was in December 2001 with the attack on the Indian Parliament; then on Nov. 26, 2008, with the attack on
Mumbai. Two Indian prime ministers were too smart to take the bait.
Indian Economy Answers
Frontline

No impact to the Indian economy

Katoch ‘9 (Rajan Katoch, Institute for Defence Studies and Analyses, New Delhi The Global Economic Crisis Some Strategic
Implications, First Published: August 2009)

Going by current indications, India is likely to emerge from the crisis somewhat better off than most countries.
Though there has been a drop in exports of goods and services, a general slowdown in manufacturing
activity, and increased unemployment in the affected sectors—it could be worse. Economic growth is not very
dependent on foreign capital, and so is likely to continue, though at more modest rates than hitherto. A strong fiscal
response has also been put into place. While the government now optimistically expects a growth rate of 7.7 per cent
35 , even pessimistic forecasts do not go below 5 per cent. 36 That in itself is a good, in a year when the world economy is expected
to record negative growth, and major industrialised countries are anticipating severe contractions in their economies. For India,
economic growth has been the key to power and will remain so, hence, policies supporting economic growth remain a strategic
necessity. Multilaterism is again in fashion. The role of the rich countries club— the G7, is being replaced by the G20
in which India is represented, which is good for India. There was a general understanding at the G20 summit in April on
expanding funding and broad-basing governance of the International Monetary Fund. Again, India could expect to be a beneficiary of
the exercise in terms of an enhanced role. Due to the expectation of continued economic growth, India would not face the
guns vs. butter issue to the extent that many others are facing. Defence expenditure is down to a sustainable 2.5 per cent of
GDP. 37 There are enough funds available for defence modernisation and procurement.

Indian economy is resilient

Business Line 8 (“Opinion Not Convincing Enough”, 6-26, Lexis)

The RBI has drawn some comfort from the fact that the
Indian economy presents some positive features
such as a financial market that is largely insulated from the turmoil affecting global institutions,
adequate foreign exchange reserves, agricultural production that is poised to sustain the momentum achieved last
year, and so on. Indeed, the only silver lining, if at all, in a scenario where the inflation rate has breached a 13-year record, is
that the Indian economy presents a far more resilient look than it did in 19 95 when inflation was of
the same order. Equally, the economy is not without its share of disturbing features. The inflation is all-pervasive and not
confined to petroleum products alone. It is hobbled by supply- side constraints in many key sectors that is aggravating the firm
trend in prices caused by global factors. Above
all, there is no evidence as yet that the administrative
dynamism so essential to unshackling the economy is in place.

Indian econ is resilient.

Hindustan Times ‘8 (– 9/19/08 (“Indian economy resilient enough to stave off any spillover: FM”,
http://www.hindustantimes.com/StoryPage/StoryPage.aspx?sectionName=&id=9837186c-df47-4739-af98-
469b67304fc9&MatchID1=4813&TeamID1=6&TeamID2=1&MatchType1=1&SeriesID1=1212&PrimaryID=4813&Headline=FM+allays
+fears+over+global+financial+turmoil)
Finance Minister P Chidambaram on Thursday allayed fears over the global financial
turmoil, saying the Indian economy is resilient enough to stave off any spillover . Global markets
remained jittery for the fourth day amid reports that Morgan Stanley was preparing for a possible sell-off, and back home, a
stock broker each in Hyderabad and Indore committed suicide, allegedly because of trade losses. “Let me assure everyone
(that) there is no cause for any alarm that any Indian bank is exposed or is vulnerable like
couple of banks that have failed in the US,” Chidambaram said. His comments helped the Sensex bounce back
from a 705-point plunge in early trading and close 52 points higher. The Sensex has lost more than 1,600 points,
or 11 per cent, since September 8, when it became apparent that some of the top US
financial firms could go bankrupt. The week opened with Lehman Brothers’ bankruptcy and Merrill Lynch’s sale.
Following them was American International Group, the world’s largest insurance company, which got a $85 billion bailout from
This stoked fears that more financial firms could report trouble and set off a
the US government.
chain reaction across markets worldwide. Although India’s share market has been hit
hard, analysts say the impact on the broader economy would be limited as it is relatively
less open compared to many of its peers. “Our banking system is reasonably insulated
from what is happening in the rest of the world ,” Chidambaram said, adding there could be some short-term
squeeze on credit.
---Indian Econ- Inevitable
Only read this if their answers to econ resilient are “it is on the brink” or something like that

All of the reasons the Indian economy isn’t resilient are just reasons why collapse is inevitable

Schuman 11- American author and journalist who specializes in Asian economics, politics and
history. Asia business correspondent for TIME Magazine
(Michael, “India’s economy: Headed for trouble?” 1/18/11, http://curiouscapitalist.blogs.time.com/2011/01/18/india%E2%80%99s-
economy-headed-for-trouble/,)

Just look at the mess India finds itself in. The wholesale price index soared 8.4% in December
compared to a year earlier. Prices of onions, vegetables and other staples are rising even faster .
The latest read of the government food price index shows they jumped almost 17% in a year. That's a serious,
serious matter for a country with so many people still stuck in poverty – nothing eats into the food on a poor
man's dinner table like rapidly rising prices. Certain basic foods, like onions, are such a crucial ingredient in Indian cooking that
people just can't live without them, so rising prices at the local market hit hard. The government has been scrambling to contain the
damage, by, for example, banning the export of onions. Some of this food inflation could well be temporary – a result of unusual
weather conditions that hurt the onion crop, for example. India is also not the only country facing escalating prices, especially of
food. Prices of commodities are rising across the board , with the Food and Agriculture Organization's food price
index hitting a record in December. But India's inflation is also its own fault. A mix of loose budgets and easy money
(leftover from recession-busting efforts) with a lackluster approach towards much-needed reform s (more
deregulation, for example) and investments (i.e., in infrastructure) have created bottlenecks that spawn
inefficiencies and push up prices. Here's more from Courtis: If you have aggressive monetary and fiscal policy, together
with booming labor market expansion, you better have hugely powerful supply side policies, or inflation can only explode... guess
what, in the absence of China style aggressive supply side policies -infrastructure, deregulation, opening of the economy,
education--, inflation is exploding... and which means quickly dropping competitiveness.
---Indian Econ- Resilient

It’s resilient

Rajah 11 – managing director and CIO of Asian equities at Franklin Templeton Investments
[Sukumar, “India resilient in spite of inflationary headwinds”, Investment Week, p. 46,
proquest,]

India has emerged as one of the top investment destinations , over the last decade, helped by its
strong fundamentals, growth potential and resilience during the financial crisis. The economy
has undergone significant transformation during this period helped by drivers such as reforms, outsourcing and strong capital flows. The last few years have
witnessed the arrival of companies with global scale and while tech companies became India's poster boys, companies from other sectors such as metals, capital goods and telecoms also occupied centre stage.

Indian companies benefitted from the strong demand growth for goods and services and easy
availability of capital - large firms had increased appetite for global acquisitions. Strong earnings
growth, high RoE and low leverage have been important factors for increased confidence in
Indian firms. Recently, headwinds have emerged in the form of graft probes at various institutions with persistently high levels of inflation. The latter is driven by supply-demand side factors and the
rise in international commodity prices. Reserve Bank of India was one of the first central banks to tighten monetary policy - however, growth momentum has remained strong and growth is expected to be at

While the pace may moderate due to cyclical factors and


about 8.5% for the year ended March 2011, close to pre-crisis levels.

tightening, India is expected to remain one of the fastest growing major economies. Despite
short-term concerns, India remains an attractive growth story over the medium term. A well-
balanced growth model and the lack of excesses, high savings rate and a large young population
holds the economy in good stead. Private entrepreneurship has flourished and is spreading overseas. The breadth and depth of the equity markets and corporate
regulatory framework are key positives. The government will eventually push through further structural reforms that

address infrastructure and skill gaps. Volatility cannot be ruled out over the near term, but the direction remains positive.
Indian Relations Answers
Frontline

Relations inevitable – most frequent partners for security engagements, robust dialogues,
engagements, military exercises, personnel exchanges, defense trade, shared values, democratic
principles, cooperation in multilateral forums

ZeeNews 3/6 (“US looking for strategic partnerships with India: Locklear,” http://zeenews.india.com/news/nation/us-
looking-for-strategic-partnerships-with-india-locklear_833251.html)

Observing that the


US-India relationship is the strongest it has been since India gained its
independence in 1947, Locklear said a strengthened US-India strategic partnership is imperative to achieve US national
interests including ensuring regional security, strengthening the international trading system, protecting shared domains, countering
terrorism, and bolstering international non-proliferation. "We
remain India's most frequent partner for security
engagements. Our defence relationship is built around a robust programme of dialogues and
engagements, military exercises, personnel exchanges and defense trade, which has grown
from USD 0 to USD 9 billion dollars in less than a decade. "The Indians now operate a fleet of six C-130J cargo
aircraft; they have taken delivery of their first of eight P-8I Poseidon maritime patrol aircraft and their first of ten C-17 Strategic
Airlifters," he said. "Our relationship with India has room to grow, and we are optimistic and enthusiastic about its potential. India's
legacy of non-alignment and commitment to a policy of "strategic autonomy" is often viewed as limiting the relationship. "However,
our shared values and commitment to democratic principles inevitably place us on parallel, if
independent paths. Several of these parallel interests include cooperating in multilateral forums
which address," the PACOM Commander said.

Multiple things solve India relations

India Today ‘13 [“Obama's second term may witness new initiatives in US-India relations,”
http://indiatoday.intoday.in/story/obama-second-term-new-initiatives-in-us-india-relations/1/243159.html]

Others say factors like the discovery of shale oil and gas providing the US with a strategic boost will
influence its global relations and there could be a redefining of the notions of engagement .
Outgoing Secretary of State Hillary Clinton has already spoken about the new means of diplomacy, what
she called "smart power" that includes trade , technology and private investment, to advance US interests.¶ And
change has been a recurring motif with Obama as evidenced in the slogans of "the change we believe in" in 2008 and "forward" in
2012. It is believed that the president would be thinking of his legacy as previous re-elected presidents had pursued international
initiatives. It is therefore natural to expect some new issues and priorities that will shape US relations with the world and India.¶
One such issue is global trade. The year 2013 is going to be quite an important year since 2001, when the Doha
Round was launched and China joined the World Trade Organisation. By the end of this year, new WTO negotiations would begin on
liberalising trade in services such as consulting, banking and insurance, and on expanding the 1996 Information Technology
Agreement that eliminated tariffs on trade in devices like memory chips. These talks are likely to produce the biggest negotiated
liberalisation of trade since the Uruguay Round of early 1990s.¶ At the same time, negotiations could be completed on the first
trans-Pacific free trade agreement and started on a trans-Atlantic deal between the US and the EU.¶ India had a glimpse of
US trade agenda earlier this month when Mike Froman, an Obama administration adviser and possibly the new US
Trade Representative (USTR), said the world has "turned the page" on Doha and indicated that the Trans-
Pacific Partnership (TPP) would be pushed as an alternative.¶ According to Froman, major emerging countries like India and
Brazil want to maintain their developing country status and do not want to open their markets. Froman has also charged India with
blocking a trade facilitation agreement on infrastructure at ports and custom stations.¶ The
US and other developed countries
have also begun pushing for "early harvest" agreements - the plurilateral or select group agreements on goods
and services - ahead of the next WTO ministerial meet at Bali in December as the Doha Round is
stalled.¶ The Obama administration has also signalled it would be push forward the New Silk Road
concept mooted by Clinton -- an international infrastructure network that would remove barriers to flow of goods and
people among countries of South Asia.¶ Robert D. Hormats, Under Secretary of State for Economic Growth, Energy and
Environment, has spoken of an Indo-Pacific Corridor that would reach out to Southeast Asia, making
the region a hub of global trade.¶ There is a strategic angle to this. Following the pullout from Iraq, and
planned drawdown of troops from Afghanistan by the end of 2014 and China's growing assertiveness in East and South China sea,
Obama has been reorienting US policy towards Asia-Pacific.¶ Some believe the
US interest in making India an anchor
country for regional economic cooperation is aimed at gaining New Delhi's support for
facilitating its exit from Afghanistan. Hormat has indicated that success of Afghanistan would depend on the level of
regional cooperation.¶ The US has made a "bet" on India and would perhaps shape the context in
which India would take its decisions. In Obama's first term, joint initiatives in areas like energy
and healthcare were launched. In the second, both countries could collaborate on maritime and
cyber security.

Indian relations resilient

Schaffer 10 (Teresita Schaffer, Ambassador Teresita C. Schaffer is director of the South Asia Program at the Center for Strategic
and International Studies, “U.S.-India Initiative Series The United States and India 10 Years Out,” October 2010)
http://www.cnas.org/files/documents/publications/CNAS_10YearsOut_Schaffer.pdf

India and the United States have transformed their relationship in the past 20 years. Looking ahead a decade or more, this trend is
likely to continue. The
two countries can expect strong economic ties and a lively security
relationship, including increased defense trade and especially stronger cooperation in the
Indian Ocean. Economic issues will remain important drivers of Indian foreign policy.
Cooperation on the global scene will have ups and downs, but the two countries will gradually
find more areas where they can work together . As India’s international trade encompasses more sophisticated
and knowledge-based products, India will pursue economic interests that do not necessarily dovetail with those of the developing
countries as a group. India-Pakistan relations are likely to remain brittle. India will continue to see China as its major strategic
challenge. Over the next decade, India will become more comfortable with a higher international profile – but slowly, and with
considerable nervousness about the risks involved in departing from its comfort zone focused on the nonaligned movement. The
“wild cards” most likely to produce real discontinuities in U.S.-India ties relate to the domestic coherence and international behavior
of Pakistan and China, to international conflict involving Iran and to changes in the global distribution of power over the next
decade. In addition, externally driven changes in climate or technology could limit India’s economic growth and in the process make
India a much more inwardoriented country. Looking behind this broad-brush projection, it is instructive to take apart the elements
of continuity and change, and to see what lessons these hold for policymakers. Growing Economies, Expanding
LinkagesIndia’s economic growth after 1990 was one of the most important factors in
transforming U.S.-India relations. This economic growth had two consequences, both of which
are likely to continue through the next decade: It led to an increasingly large and vibrant U.S.-
India economic relationship, and it put economic success – trade, investment and securing
energy supplies – at the heart of India’s foreign policy and strategic calculus .Projections for India’s
economy over the next five years consistently foresee growth rates of upwards of 8 percent, possibly higher. Even if India’s
economic expansion falls somewhat short of these levels, one can expect economics to remain one of the drivers of U.S.-India
relations, and a very positive one. In the next decade, trade will continue to grow as a share of India’s economy. The United States
will remain one of India’s top three trading partners, and probably the largest when one includes services and information
technology trade as well as goods. India’s economic growth will make it an increasingly important partner for the United States,
though its share of U.S. trade will not be as impressive (about 1.4 percent of U.S. trade in 2009; even dramatic growth will leave it
well short of the top tier). The geography and composition of India’s trade will be increasingly diverse. India has already begun
implementing free-trade areas with the Association of Southeast Asian Nations (ASEAN) and Korea, and will negotiate one with
Japan. With each new trade opening, the next one will become slightly easier, and India will approach the next multilateral trade
negotiation with more flexibility and a greater expectation that it can benefit from global trade liberalization. A free-trade
agreement with the United States is conceivable toward the end of this period, but only if the U.S. economy picks up enough to
counteract the politics of getting such an agreement enacted by the U.S. Congress. Investment
flows will continue to
increase, both into and out of India. As a result, Indian companies will become increasingly active
participants in the U.S. economy. In particular, the close personal and corporate linkages that
bind the leading information technology firms in both countries will continue . Trade and
investment are private activities. They can be intensified through government policies that sustain
India’s own economic growth, but fundamentally they will carry on regardless of the ups and downs in
government-to-government ties. They will also help stabilize the rest of the relationship. Security
and the Indian Ocean By the same token, India’s growing recognition that its economy is not only a
critical domestic priority but also a determinant of its national power will reinforce ties with the
United States – not only in the economic sphere but also in the security area. One of the established
pillars of U.S.-India ties is a common interest in Indian Ocean security. For India, this is critical
not only for the safety of the immediate neighborhood but also for the security of its most
important economic supply routes. This common concern will reinforce the importance of
naval cooperation and more generally of security ties . India and U.S. security perspectives on
East Asia will remain closely aligned . Ten years hence, India will be more fully integrated into Asia than it is now.
India’s leadership will continue to see China as their primary strategic challenge. Its foreign policy will seek friendly engagement with
China, and this will lead to instances where India draws closer to China on certain global issues (as happened, for example, at the
climate change meeting in Copenhagen, Denmark). But India’s leaders are acutely conscious that an assertive China will challenge
India’s security and economic interests. India will compensate through growing economic and political ties with Japan, Korea and the
ASEAN countries, which will also make it more of a player on the larger Asian scene. This approach has strong parallels with the way
the United States looks at China. It will also, however, make India especially sensitive to any suggestion that the United States is
giving preference to China in its approach to Asian security and institutions.

No impact to relations

Tellis 7 - senior associate at the Carnegie Endowment for International Peace, specializing in international security, defense, and
Asian strategic issues. Former Department of State senior adviser to the Undersecretary of State for Political Affairs. Former senior
policy analyst at the RAND corporation

CHAPTER 8 Gauging U.S.-Indian strategic cooperation Edited By Henry D. Sokolski, Army War College (U.S.). Strategic Studies
Institute WHAT SHOULD WE EXPECT FROM INDIA AS A STRATEGIC PARTNER? Ashley J. Tellis http://74.125.155.132/scholar?
q=cache:AN0FEcbqUQ0J:scholar.google.com/+india+author:%22Ashley+Tellis%22&hl=en&as_sdt=0,23

It would not be an exaggeration to say that for the first time in recent memory Indian and American interests in each of
these eight issue-areas are strongly convergent.10 It is equally true to assert that India’s contribution ranges from important to
does not mean, however,
indispensable as far as achieving U.S. objectives in each of these issue-areas is concerned. That
that the United States and India will automatically collaborate on every problem that comes
before the two countries. The differentials in raw power between the United States and India
are still too great and could produce differences in operational objectives, even when the
overarching interests are preeminently compatible. Beyond the differentials in raw power, bilateral
collaboration could still be stymied by competing national preferences over the strategies used
to realize certain objectives. And, finally, even when disagreement over strategies is not at issue, differences in
negotiating styles and tactics may sometimes divide the two sides.

Relations are super resilient—no single issue will decide relations.

Roemer 11. Indian Express citing Timothy Roemer, US ambassador to India. No single issue can decide Indo-US relation: Roemer.
April 29 2011. http://www.indianexpress.com/news/no-single-issue-can-decide-indous-relation-roemer/783485/

After both its fighters failed to make it to India's multi-billion dollar deal, the US on Friday said
even though it is deeply disappointed, no single issue can decide the Indo-US relation . "We respect
Indian procurement process but we are deeply disappointed... But I am forever an optimist and positive. No
single issue can decide the Indo-US relation. Our partnership is resilient and global in nature. It
will make us inextricable in coming decades," US Ambassador to India Timothy J Roemer said
here. He was addressing a gathering of the US Chamber of Commerce to India. His statement comes a day after India announced
the rejection of its two aircraft - Boeing's F-18 and Lockheed Martin's F-16, from multi billion dollar 126 Medium-Multi Role Combat
Aircraft (M-MRCA) deal, in which European Eurofighter and French Rafale have been shortlisted for final selection. The US
Ambassador has resigned a day after India informed America about its decision to reject American fighters. Addressing the gathering
Roemer said, "The
US India partnership is on a trajectory that knows no limits. We are entering a
golden age in our relations that will result in us creating economic opportunities for our citizens and ensuring safe and
secure communities throughout the world." He further talked about the Indo-US partnership in areas of health, education, trade,
energy and agriculture. Calling
his two year-long stint in India as an "extraordinarily successful and
rewarding" period, Roemer said, "Our two countries are collaborating and partnering in almost
every field of human endeavour."

Relations inevitable—coinciding foundations and interest

Department of State 10. U.S.-India Strategic Dialogue Joint Statement. Citing Secretary of State Hillary Clinton and India’s
External Affairs Minister Shri Krishna. Office of the Spokesman, Washington, DC. June 3, 2010.
http://www.state.gov/r/pa/prs/ps/2010/06/142645.htm

BEGIN TEXT: U.S. Secretary of State Hillary Rodham Clinton and India’s External Affairs Minister, Shri S.M.
Krishna, met in Washington on June 3, 2010, for the inaugural meeting of the “India-U.S. Strategic
Dialogue.” President Barack Obama welcomed the Indian delegation and emphasized his commitment to strengthening the
growing U.S.-India partnership. Minister Krishna was accompanied by Minister for Human Resource Development, Shri Kapil Sibal,
Deputy Chairman, Planning Commission of India, Dr. Montek Singh Ahluwalia, Minister of State for Science and Technology, Shri
Prithviraj Chavan and high ranking officials of the Government of India. Secretary Clinton was joined by Commerce Secretary Gary
Locke, FBI Director Robert Mueller, NASA Administrator Charles Bolden, the President’s Advisor for Science and Technology Dr.
Holdren, USAID Administrator Rajiv Shah, Deputy National Security Advisor Michael Froman, and other high ranking U.S.
Government officials. The
U.S. – India Strategic Dialogue is propelled by the dynamic momentum
achieved in the U.S.-India relationship over the last decade and is in pursuance of the global
strategic partnership - for a better world that Prime Minister Singh and President Obama reaffirmed during their meetings in
November 2009 and April 2010 in Washington. Secretary Clinton and Minister Krishna pledged to deepen
people-to-people, business-to-business, and government-to-government linkages between the
world’s oldest and largest democracies, for the mutual benefit of both countries and for the
promotion of global peace, stability, economic growth and prosperity . Both recalled that the India - U.S.
partnership rests on the firm foundation of common ideals as well as security and economic interests. The guiding principles
upon which both nations were founded – democracy , mutual respect, individual liberty, rule of
law, and an appreciation for the strength we derive from being pluralistic societies – make the
India -U.S. bond strong, resilient, and uniquely important for building a peaceful, prosperous,
inclusive, and sustainable world. The Indian situation in the US today is different; there is no
baggage of unpleasant memories on which prejudice and violence can draw. Yet, like a desert cactus, hate can
thrive on very little.

No impact – cooperation is only lip service

Tellis 5 (Ashley, senior associate at the Carnegie Endowment for International Peace, specializing in international security,
defense, and Asian strategic issues. 11/16/05  “The U.S.-India ''Global Partnership'': How Significant for American Interests?
““http://www.carnegieendowment.org/publications/index.cfm?fa=view&id=17693)

Several practical implications, which ought to be of significance to the Congress as it ponders the U.S.-Indian civilian nuclear
agreement, flow from these realities. To begin with, the
strengthening U.S.-Indian relationship does not imply
that New Delhi will become a formal alliance partner  of Washington at some point in the future. It also
does not imply that India will invariably be an uncritical partner  of the United States in its global
endeavors. India’s large size, its proud history, and its great ambitions, ensure that it
will likely march to the beat of its own drummer, at least most of the time. The first question, for the Congress in
particular and for the United States more generally, therefore, ought not to be, “What will India do for us?”—as critics of the civilian
nuclear agreement often assert. Rather, the real question ought to be, “Is a strong, democratic, (even if perpetually) independent,
India in American national interest?” If, as I believe, this is the fundamental question and if, as I further believe, the answer to this
question is “Yes,” then the real discussion about the evolution of the U.S.-Indian relationship ought to focus on how the United
States can assist the growth of Indian power, and how it can do so at minimal cost (if that is relevant) to any other competing
national security objectives.

India relations are resilient

Garretson 10 – former Council on Foreign Relations (CFR) International Fellow in India, Visiting Fellow at the Institute for
Defence Studies and Analyses (IDSA) New Delhi. Previously the Chief of Future Science and Technology Exploration for Headquarters
Air Force, former Los Alamos National Laboratory (LANL) Service Academy Research Associate, recipient of the National Space
Society’s (NSS) Space Pioneer Award

Peter A. Garretson “Sky’s No Limit: Space-based Solar Power, the Next Major Step in the Indo-US Strategic Partnership”
http://www.idsa.in/sites/default/files/OP_SkysNoLimit.pdf

Despite the concerns of sceptics, the Indo-US strategic partnership seems to rest on very sound
fundamentals that are not likely to change over several decades . First, is a shared cultural
history in colonialism, with the attendant struggle for freedom, and the important influence of the
enlightenment thought, British political organisation , commerce and trade routes and prominence of the
English language in matters of science, state-craft and commerce. Second, the significant and
growing bilateral trade. Third is the asymmetric but aligned economic needs–where India needs
investment today to maintain a high rate of growth for development and cohesion, and the US is looking for high growth places to
invest, and places that provide both a market for its own goods and a costcompetitive manufacturing base to manufacture the ideas
it conceives and finances. Fourth is the large and politically active diaspora that is actively seeking to
build closer ties. Fifth is a shared interest in limiting the damage of those extremists that
undermine pluralism and sew extremism and violence. Finally, both wish to take part in the the
economic rise of a vibrant Asian market where a normative rule set prevails that allow all
members to benefit from the use of global commons and work on collective problems and
human security is possible. Within this framework, both nations see the need to make space for and engage China as it
evolves as a responsible stakeholder with greater transparency, but to ensure that accommodation takes place
respecting important equities of themselves and their neighbours, and is free of any element of coercion.

US/Indian relations are resilient

Mancuso 8 (Mario, Undersecretary of Commerce, “The Future of the U.S.-India High Technology Relationship”, 6-2,
http://www.bis.doc.gov/news/2008/mancuso06052008.htm)

The strength of today’s U.S. and India relationship is real, and underscores what visionary governments
can accomplish for their people by acknowledging change and seizing opportunities.   Shared interests and values,
and improved economic and trade relations, have transformed the U.S.-India bilateral
relationship into a "strategic partnership."  And while tender points remain, the DNA of our
partnership is more differentiated, healthy, and resilient than ever before. Of the many dialogues that nurture
our bilateral economic relationship, few have been as vibrant or had as much impact as the U.S.-India High Technology
Cooperation Group (HTCG).
Indonesia Stability
Indonesia won’t collapse – media reports are wrong

Lloyd 2k (Grayson, Foreign Affairs, Defence and Trade Group – Parliament of Australia, “Indonesia's Future Prospects:
Separatism, Decentralisation and the Survival of the Unitary State”, 6-27, http://www.aph.gov.au/library/Pubs/CIB/1999-
2000/2000cib17.htm)

Contrary to the argument of numerous media reports predicting otherwise, East Timor's independence has not been
a catalyst for general state collapse in Indonesia. Fears sponsored by the armed forces and ultra-nationalists that the loss of East
Timor would trigger the disintegration of the unitary state have so far proved unfounded. That said, the independence
movements in Aceh and Papua have certainly been rejuvenated by events in East Timor. However East Timor was a special case.
Its people are overwhelmingly Catholic, it was never a part of the Dutch East Indies (the precursor to the Indonesian Republic),
and moreover the United Nations did not recognise the territory's absorption into Indonesia. In nationalistic terms Indonesia is
thus no less Indonesian after the loss of East Timor-indeed it may be more so.(12) In other areas, such as oil-rich East Kalimantan
and the similarly oil abundant Riau in Central Sumatra, the problems centre on regional autonomy and the ever-present regional
demand for a larger return on oil or other revenues from the central Government. In Maluku (especially Ambon and
Halmahera), and West Kalimantan the central Government is faced with provinces afflicted by widespread inter-communal
violence and disorder. These are neither separatist nor independence campaigns, however they do represent a continuing
challenge to the maintenance of national unity. Indonesia has not yet reached the point where it can take its national unity for
granted. In reality it is unlikely to arrive at such a point, but it
does not follow that Indonesia will fracture
and collapse. There are at least two significant reasons why this is unlikely to happen. The first is
that no political, economic or other agenda in Indonesia would be served by disintegration .(13)
The second is that many if not most Indonesians recognise that the economic and social justice benefits
achieved by remaining united-challenging though this is in the present climate-outweigh the potential
consequences of disintegration.(14) But the resolve of Indonesians to stay together as a nation is not in itself
sufficient. This resolve will need to be accompanied by sensitive Government policies and a genuine desire to overcome real and
widespread problems.

Indonesian stability is resilient

Seth 4 (S.P., Freelance Writer – Jakarta Post, “Indonesia Needs New, Younger Leadership”, The Jakarta Post, 4-20, Lexis)

President Megawati might not be personally tainted but most, if not all, around her are under a cloud. People do not have
credible choices, thus deepening popular cynicism about their leaders and the system they preside over. It
is not to
suggest that Indonesia is doomed. Countries of Indonesia's size, population and diversity do
not collapse just like that. There is considerable resilience in old cultures. Most of its people are
tied in common destiny and are likely to swim and sink together . And the odds are in favor of
swimming together because adversity creates new bonds for survival.

No impact

Barber 99 (Ben, Staff – WT, “U.S. Ends Ties with Indonesian Army”, Washington Times, 9-10, Lexis)
However, he and other analysts said it was unlikely a stalling of the Indonesian economy would
drag down the region and spread around the world, as happened in 1997 and 1998. Gen. Shelton said
that he did not see a national security risk to the United States in Indonesia's problems that would justify
sending in U.S. troops. Thomas Pickering, undersecretary of state, told a congressional panel yesterday that " Australia is
ready to put forces in . . . but this can't be done without invitation. For a U.N. mandate, we need Jakarta's permission."
Intervention Answers
No intervention

Mueller ’11 (JOHN MUELLER: U.S. Intervention from Kosovo to Libya: 9/11 and the Iraq Syndrome Posted on September 8, 2011
by COLUMBUS – The stridently militaristic response to 9/11 by the United States should probably be seen as an aberration in
American foreign policy. JOHN MUELLER is Professor of Political Science at Ohio State University. He is the author of Atomic
Obsession: Nuclear Alarmism from Hiroshima to Al Qaeda (2010) and the author, with Mark Stewart, of Terror, Security, and Money:
Balancing the Risks, Costs and Benefits of Homeland Security (2011).

Andrew Bacevich and many others, in contrast, espy in the experience since World War II the rise of what
he calls a “new American militarism .” However it seems to me that such analysis puts too much
weight on the temporary successes in the wake of 9/11 of the neoconservative movement in the
George W. Bush administration. With the consequent sour experience in Iraq and in Afghanistan,
we seem to be witnessing the re-emergence of a perspective on intervening in overseas conflicts
somewhat like the one that prevailed before 9/11, and future policy seems likely to be carried
out under the weight of what might be called “The Iraq Syndrome.” A “syndrome” can be
defined as a general, even visceral, unwillingness, in the aftermath of a bad experience, to do anything that might lead to a
repetition. And under the impetus of the Iraq Syndrome such once-fashionable (and sometimes self-infatuated) expressions as
unilateralism, preemption, preventive war, and indispensable-nationhood have already picked up a patina of quaintness .
As part of
the process, there is growing skepticism about the notions that the United States should take
unilateral military action to correct situations or overthrow regimes it considers reprehensible but that
present no immediate threat to it, that it can and should forcibly bring democracy to other nations not now so blessed, that it has
the duty to rid the world of evil, that having by far the largest defense budget in the world is necessary and broadly beneficial, that
international cooperation is of only very limited value, and that Europeans and other well-meaning foreigners are naive and
decadent wimps. Defense Secretary Robert Gates observed in February 2010 (at West Point, no less) that
“any future defense secretary who advises the president to again send a big American land army
into Asia or into the Middle East or Africa should ‘have his head examined ,’ as General MacArthur so
delicately put it.” That certainly sounds like the Iraq or (perhaps better) the Iraq-Afghanistan
Syndrome at work. However, there is nothing really new in Gates’ statement and in the concomitant, post-Iraq unwillingness
to engage militarily only when the environment is “permissive” or when high altitude bombing can be relied upon. There has
never been much enthusiasm for sending Americans troops into hostile situations in recent
decades—or even longer—unless there was a decided provocation like Pearl Harbor or 9/11.
The syndrome can be seen at work in the hesitant approach to the chaos in Libya . The U.S.
government applied military pressure only reluctantly and tentatively, ruling out the idea of sending in ground troops, and made it a
priority that any intervention be internationally approved. Thus, the Libyan venture, and the American role in it, has followed the
pattern not of Iraq, but of Kosovo in 1999. Boxed in by their own postured huffing and puffing against a demonized regime,
American leaders reluctantly approved “kinetic military action” from a safe distance, supported by the much-underexamined hope
that this might be quickly decisive. The growth of the syndrome also shows up in public opinion data .
Beginning in 1945, a key poll question about engagement in foreign affairs has been posed periodically: “Do you think it would be
best for the future of this country if we take an active part in world affairs, or if we stayed out of world affairs?” After the campaign
in Kosovo, Americans became less keen on intervention—an interesting reaction, since the campaign was something of a success at
least in its own terms—and those choosing the “stay out” option rose to near all-time high of 34 percent. Right after 9/11, the figure
dropped to a low of 14 percent, and after a brief rise, declined again to 14 percent at the beginning of the Iraq War in 2003. Since
that time, however, the “stay out” option has become considerably more popular, so that by 2006 , the
last time the question was asked, fully 38 percent embraced the sentiment —the highest ever registered. This does not
necessarily mean that old-fashioned isolationism is emerging; the United States is unlikely to withdraw from participation in the
global economy, disengage from international political organizations, or cease to be a citizen of the world community. But it could
well be fertile ground for an Iraq Syndrome, or Iraq-Afghanistan Syndrome, to flourish .
Iran Conflict/Miscalc Answers
No escalation to great power war or nuclear use

Davis et al 11,– senior political scientist at RAND - 6/6/ Iran’s Nuclear Future: Critical U.S. Policy Choices, Prepared for the
United States Air Force, RAND, (Lynn E. Davis, Jeffrey Martini, Alireza Nader, Dalia Dassa Kaye, James T. Quinlivan, Paul Steinberg),
http://www.rand.org/content/dam/rand/pubs/monographs/2011/RAND_MG1087.pdf

Current U.S. Conventional and Nuclear Posture Is Sufficient. The case for why Iran will be deterred from using nuclear
weapons against U.S. military forces rests on a number of considerations. First, while possibilities of a proxy conflict or
limited military engagements exist, it is difficult to see a conflict between the United States and
Iran escalating to a major conventional conflict, because Iran faces overwhelming destruction. Crossing
the nuclear threshold risks further devastation for Iran, thereby directly threatening the regime’s
survival; also, in using its nuclear weapons, Iran would be using up the very weapons it had
acquired for other purposes. Second, the United States, with the deployment of long-range conventional
precision-strike systems, has credible military capabilities to inflict high levels of devastation
without resorting to the use of nuclear weapons. In this approach, existing U.S. declarations with respect to
using nuclear weapons would remain unchanged. The threat of U.S. nuclear retaliation could be made more explicit in the
event of a conventional conflict so as to reduce the prospect of Iran misreading U.S. intentions.

No escalation

Jamieson ’7 (US may attack but will Iran fight back? Asia Times, http://www.atimes.com/atimes/Middle_East/II12Ak02.html)

Even those Americans actively seeking to provoke a war with Iran have had little success in
finding or provoking a suitable incident that can be presented to the American people as a good
reason to launch an attack on Iran. Despite the seizure of Iranian personnel in Iraq, at Irbil in January and more recently
in Baghdad, the Iranians have refrained from any reckless response, although only their people seized in
Baghdad have been returned. The capture of British sailors in March by Iranian Revolutionary Guards
might have been a suitable flash point. However, Tehran soon released the sailors after
squeezing every propaganda advantage from their capture, and Britain specifically asked the
United States not to exacerbate the situation . Since the beginning of the year there have been constant US claims
of Iranian interference in Iraq and of the Iranians supplying arms to militias and insurgents in that country. However, no clear
link has ever been established between the Iranians and any particular attack on US forces. Even
if the United States chose to respond to these alleged Iranian hostile acts with "hot pursuit"
Special Forces raids into Iran or the bombing of alleged terrorist training camps in that country,
this would not precipitate the sort of crisis needed to justify a wholesale assault on Iran 's nuclear
facilities and its armed forces in the near future.

Multiple barriers to Iran heg


Savyon 11. [A, director – Iranian Media Project @ Middle East Media Research Institute, 7/4/’11 “Iran's Defeat in the Bahrain
Crisis: A Seminal Event in the Sunni-Shi'ite Conflict,” http://www.memri.org/report/en/0/0/0/0/0/0/5424.htm#_ednref6

Despite its image as a looming superpower, which revolutionary Iran has sought for years to cultivate, its actual
policy reveals a deep recognition of its weakness as a representative of the Shi'ites, who constitute a 10% minority
in a Sunni Muslim region. Historically persecuted over centuries, the Shi'ites developed various means of survival, including taqiya –
the Shi'ite principle of caution, as expressed in willingness to hide one's Shi'ite affiliation in order to survive under a hostile Sunni
rule – and passivity, reflected in the use of diplomacy alongside indirect intimidation, terrorism, etc. The ideological change
pioneered by the founder of the Islamic Revolution in Iran, Ayatollah Ruhollah Khomeini – who transformed the passive perception
characteristic of the of the Shi'a (which was based on the legend of the martyrdom of Hussein at the Battle of Karbala) into an active
perception of martyrdom (shahada)[26] – is not being carried out by Iran. Tehran is refraining from sending Iranian
nationals to carry out martyrdom operations, despite its years-long glorification of this principle. It is also not
sending Iranians to Gaza, either on aid missions or to carry out suicide attacks – and this despite the fact that regime-
sponsored organizations are recruiting volunteers for such efforts. Moreover, it appears that the Shi'ite regime in Iran is
utilizing the legend of Hussein's martyrdom solely for propaganda purposes, in order to glorify its own
might and intimidate the Sunni and Western world . Such intimidation is in keeping with Shi'ite tradition, as a way
to conceal Tehran's unwillingness to take overt military action against external challenges. Conclusion Tehran's defeat in the
Bahrain crisis reflects characteristic Shi'ite restraint, stemming from recognition of its own weakness in the face of the
vast Sunni majority. The decade during which Iran successfully expanded its strength and power exponentially via threats and
creating an image of superpower military strength has collapsed in the Bahrain crisis; Iran
is now revealed as a paper
tiger that will refrain from any violent conflict . When it came to the crunch, it became clear that
the most that Iran could do was threaten to use terrorism or to subvert the Shi'ite citizens of
other countries – in keeping with customary Shi'ite behavior – and these threats were not even
implemented. It can be assumed that the Sunni camp, headed by Saudi Arabia, is fully aware of the political and military
significance of Iran's weakness and its unwillingness to initiate face-to-face conflict. This will have ramifications on both the regional
and the global levels. In
addition to having its weakness exposed by the Bahrain situation, Tehran has
also taken several further hits to its prestige and geopolitical status . These include: the popular
uprisings in Syria against the regime of Syrian President Bashar Al-Assad, weakening the Tehran-Damascus axis;
post-revolutionary Egypt's refusal to renew relations with Iran; and the fact that the E.U. was capable of uniting and
leading a military attack against the regime of Libyan leader Mu'ammar Al-Qadhafi as well as its refusal to renew the nuclear
negotiations with Tehran based on Iran's demands. All
this, added to the serious internal rift between Iranian
Supreme Leader Ali Khameneiand his long-time ally Iranian President Mahmoud Ahmadinejad, have today left the
Iranian regime in clearly reduced circumstances .

No Iran war scenario

Riedel 12 – Senior Fellow in the Saban Center for Middle East Policy at the Brookings Institution and a professor at Georgetown
University (Bruce, 01/20, “Iran is not an existential threat,” http://thedailynewsegypt.com/global-views/iran-is-not-an-existential-
threat.html)

The danger of war is growing again over Iran's nuclear ambitions. Iran is rattling its sabers, the Republican presidential candidates
and others are rattling theirs. But even if Iran gets the bomb, Israel will have overwhelming military
superiority over Iran, a fact that should not be lost in all the heated rhetoric. Former head of the Mossad, Meir Dagan, says Iran
won't get the bomb until at least 2015. In contrast, Israel has had nuclear weapons since the late 1960s and has jealously guarded
its monopoly on them in the region. Israel has used force in the past against developing nuclear threats. Iraq
in 1981 and Syria in 2007 were the targets of highly effective Israeli air strikes against developing nuclear weapons programs.
Israel has seriously considered conducting such a strike against Iran and may well do so especially now
that it has special bunker-busting bombs from the US. Estimates of the size of the Israeli arsenal by international think tanks
generally concur that Israel
has about 100 nuclear weapons, possibly 200. Even under a crash program, Iran
won't achieve an arsenal that size for many years — perhaps decades. Israel also has multiple delivery
systems. It has intermediate range ballistic missiles, the Jericho, that are capable of reaching any target in Iran. Its
fleet of F15 long-range strike aircraft can also deliver nuclear payloads. Some analysts have suggested that it can also deliver nuclear
weapons from its German-made Dolphin submarines using cruise missiles. Israel will also continue to have
conventional military superiority over Iran and the rest of the region. The Israel Defense Forces has a demonstrated
qualitative edge over all of its potential adversaries in the region, including Iran. The Israeli air force has the capability to penetrate
air defense systems with virtual impunity as it demonstrated in 2007 when it destroyed Syria's nascent nuclear capability. The IDF's
intelligence and electronic warfare capabilities are vastly superior to its potential rivals. The 2006 Lebanon war and the 2009 Gaza
war demonstrated that there are limits to Israel's conventional capabilities but those limits should not obscure the underlying reality
of Israel's conventional military superiority over its enemies. Iran, on the other hand, has never
fully rebuilt its
conventional military from the damage suffered in the Iran-Iraq war. It still relies heavily for air and sea power on equipment
purchased by the Shah 40 years ago, much of which is antique today. Moreover, the June 2010 United Nations sanctions, UN
Security Council resolution 1929, impose a very stringent arms ban on Iran. Virtually all significant weapons systems —
tanks, aircraft, naval vessels, missiles, etc — are banned from sale or transfer to Iran. Training and technical assistance
for such systems is also banned. In other words, even if Iran wants to try to improve its conventional military capability in the next
few years and has the money to do so, the UN arms ban will make that close to impossible. Iran does not have the
capability to produce state-of-the-art weapons on its own , despite its occasional claims of self-sufficiency. It
certainly cannot build a modern air force to compete with the IDF on its own. Finally, Israel will continue to enjoy the support of the
world's only superpower for the foreseeable future. Assistance from the United States includes roughly $3 billion in aid every year.
That is the longest running financial assistance program in American history, dating back to the 1973 war. It is never challenged or
cut by Congress and permits Israeli planners to do multi-year planning for defense acquisitions with great certitude about what they
can afford to acquire. When Texas Governor Rick Perry suggested cutting aid to Israel to zero in one Republican debate, his poll
numbers plummeted. He backtracked fast. US assistance is also far more than just financial aid. The Pentagon and Israel engage in
constant exchanges of technical cooperation in virtually all elements of the modern battle field. Missile defense has been at the
center of this exchange for over 20 years now. The
U nited S tates and Israel also have a robust and dynamic
intelligence relationship, which helps ensure Israel's qualitative edge. Every American president from Richard
Nixon to Barack Obama has been a supporter of maintaining Israel's qualitative edge over its potential foes, including US allies like
Egypt and Saudi Arabia. Iran, in contrast, has no major power providing it with financial help. Its arms relationships
with Russia and China have been severed by Security Council Resolution 1929. Its only military ally is Syria, not exactly a
powerhouse. And Syria is now in the midst of a civil war; its army is dissolving. If President Bashar Al-Assad falls, Iran is the biggest
loser in the "Arab spring". Hezbollah will be the second largest loser. The deputy secretary general of Hezbollah and one of its
founders, Sheikh Naim Qassem, wrote in 2007 that Syria is "the cornerstone" of Hezbollah’s survival in the region. While Syria and
Hezbollah have their differences, the relationship is a "necessity" for Hezbollah. So don't let the hot air from Tehran or the
Republican debates confuse the reality on the ground. Iran is a dangerous country but it is not an existential threat to
either Israel or America.

Iran’s not a threat

Nader and Dobbins ’12 (1/5/12 [Alireza Nader, senior policy analyst at the RAND Corporation, and James Dobbins, a former
U.S. assistant secretary of state, is director of the International Security and Defense Policy Center at RAND, “Iran’s Self-Destructive
Gamble”, The New York Times, 1-5-2012, http://www.nytimes.com/2012/01/06/opinion/irans-self-destructive-gamble.html?_r=1]

In these circumstances, it is important to realistically judge the nature and extent of the Iranian
threat. For all its bluster, the Iranian regime is more vulnerable than at any time in its 32-year
history. Internally, Iran is constrained by deep political divisions, civil strife and a woeful
economy. President Mahmoud Ahmadinejad has directly challenged the country’s supreme leader, Ayatollah Ali Khamenei, while
Khamenei has spoken of eliminating the presidency. The life of the ordinary Iranian becomes more precarious every day, with rising
unemployment, inflation, state repression, and the country’s growing international isolation. The regime has maintained a
superficial sense of stability through repression. Legislative elections are scheduled for early March. Leaders of the reformist Green
movement are threatening to boycott the ballot, but there will still be a closely fought contest between the more religious and
secularist wings of the regime. Both this election and the presidential vote next year could well become occasions for public
demonstrations of the sort that threatened the regime three years ago and have since toppled several Middle East governments.
Iran is on the brink of losing its only real ally, Syria , as President Bashir al-Assad looks as if he
could be the next Arab dictator to fall. Tightening international sanctions are slowing Iran’s
nuclear program while limiting its ability to project power. Saudi Arabia , Iran’s principal regional
rival, is leading the other Gulf states in an ever more explicit anti-Iranian coalition. The United
States is strengthening its military and political ties with several of these states. Iran’s leaders have watched U.S. forces topple
Saddam Hussein and the Taliban with relative ease and NATO help do the same with Col. Muammar el-Qaddafi in Libya ; Iran’s
antiquated conventional forces are no match for the U.S. military . And the Iranian regime believes that the
United States remains committed to a policy of regime change, even though Washington might not presently have the appetite for a
new military intervention.

US and Iran won’t go to war

Xinhua News 12
[“Iran's nuclear issue escalates, but unlikely into conflict”, 12-29-2011, http://news.xinhuanet.com/english/indepth/2011-
12/29/c_131334180.htm]

Iran's nuclear issue has escalated again recently with the West mulling
BEIJING, Dec. 29 (Xinhua) --
sanctions on Iran's Central Bank and its crude exports, but the seeming saber-rattling on the part
of both sides is unlikely to evolve into an imminent conflict . On Dec. 14, the U.S. House of Representatives
passed a massive defense bill, which requests new sanctions on Iran, targeting foreign financial institutions that do business with the
Islamic republic's central bank. Iran's First Vice President Mohammad-Reza Rahimi said Tuesday that if the exports of Iran's oil were
hit by sanctioned from the West, "not a drop of oil" would pass through the Strait of Hormuz. Iran's Navy Chief Admiral Habibollah
Sayari also said Wednesday that Iran would find it "really easy" to shut down the Strait of Hormuz, the world's most important oil
transit channel. The comments drew a quick response from the United States. A spokeswoman from the Bahrain-based U.S. Navy
Despite the latest
5th Fleet said the Navy is "always ready to counter malevolent actions to ensure freedom of navigation."
flare-up of tensions, there is little possibility that Iran and the U.S. would rush headlong into
war, in view of the current circumstances . U.S. President Barack Obama, with his re-election bid next
year on his mind, would try to avoid any war that could become controversial. Iran , on the other
hand, is becoming more adept at playing a game of brinkmanship. While sticking to its tough stance on its
nuclear program, Tehran has apparently been ready to compromise a bit at the last minute, leaving
the door open for further negotiations. It has been almost 10 years since Iran's nuclear issue
surfaced in 2003, but neither side has ever crossed the red lines. Iran has maintained its cooperation with
the International Atomic Energy Agency (IAEA) and has never announced plans to develop nuclear weapons.
The West, for its part, has not imposed a complete embargo on Iran's crude exports, to say
nothing of launching strikes against Iran's nuclear facilities.

Iran isn’t a threat

Luttwak ’7, senior associate – CSIS, professor – Georgetown and Berkeley, 5/26/
(Edward, “The middle of nowhere,” Prospect Magazine)
Now the Mussolini syndrome is at work over Iran. All the symptoms are present, including tabulated lists of Iran’s
warships, despite the fact that most are over 30 years old; of combat aircraft, many of which (F-4s, Mirages, F-5s, F-
14s) have not flown in years for lack of spare parts; and of divisions and brigades that are so only in
name. There are awed descriptions of the Pasdaran revolutionary guards, inevitably described as “elite,” who do indeed
strut around as if they have won many a war, but who have actually fought only one—against Iraq,
which they lost. As for Iran’s claim to have defeated Israel by Hizbullah proxy in last year’s affray, the publicity was
excellent but the substance went the other way, with roughly 25 per cent of the best-trained men dead ,
which explains the tomb-like silence and immobility of the once rumbustious Hizbullah ever since the ceasefire. Then there is the
new light cavalry of Iranian terrorism that is invoked to frighten us if all else fails. The usual middle east experts now explain
that if we annoy the ayatollahs, they will unleash terrorists who will devastate our lives, even though 30 years of “death to America”
invocations and vast
sums spent on maintaining a special international terrorism department have produced only one
major bombing in Saudi Arabia, in 1996, and two in the most permissive environment of Buenos Aires, in 1992
and 1994, along with some assassinations of exiles in Europe. It is true enough that if Iran’s nuclear installations are bombed in some
overnight raid, there is likely to be some retaliation, but we live in fortunate times in which wehave only the irritant of
terrorism instead of world wars to worry about—and Iran’s added contribution is not likely to
leave much of an impression. There may be good reasons for not attacking Iran’s nuclear sites—including the very slow
and uncertain progress of its uranium enrichment effort—but its ability to strike back is not one of them. Even the seemingly
fragile tanker traffic down the Gulf and through the straits of Hormuz is not as vulnerable as it seems—Iran and Iraq
have both tried to attack it many times without much success, and this time the US navy stands
ready to destroy any airstrip or jetty from which attacks are launched. As for the claim that the “Iranians”
are united in patriotic support for the nuclear programme, no such nationality even exists. Out of Iran’s population of 70m or so, 51
per cent are ethnically Persian, 24 per cent are Turks (“Azeris” is the regime’s term), with other minorities comprising the remaining
quarter. Many of Iran’s 16-17m Turks are in revolt against Persian cultural imperialism ; its 5-6m Kurds have
started a serious insurgency; the Arab minority detonates bombs in Ahvaz; and Baluch tribesmen
attack gendarmes and revolutionary guards. If some 40 per cent of the British population were engaged in separatist
struggles of varying intensity, nobody would claim that it was firmly united around the London government. On top of this, many
of the Persian majority oppose the theocratic regime, either because they have become post-Islamic in reaction
to its many prohibitions, or because they are Sufis, whom the regime now persecu tes almost as much as the
small Baha’i minority. So let us have no more reports from Tehran stressing the country’s national unity. Persian nationalism
is a minority position in a country where half the population is not even Persian. In our times,
multinational states either decentralise or break up more or less violently; Iran is not decentralising, so its future
seems highly predictable, while in the present not much cohesion under attack is to be expected.

Prefer our evidence – most conclusive studies agree


Kaye 10. [Dalia Dassa, senior political scientists – RAND, Dangerous But Not Omnipotent, Report by RAND for the Airforce and
DOD, http://www.rand.org/content/dam/rand/pubs/monographs/2009/RAND_MG781.pdf]

Guided by these observations, this study aims to prepare the USAF leadership and the U.S. defense community to anticipate and
confront future challenges from Iran. To do so, we examine the motivations behind Iranian strategy; Iran’s military doctrine and
capabilities; Iran’s interactions with non-state Islamist groups; and the Arab public’s per ception of Iran. More broadly, we
present a framework for assessing future trends in Iranian strategy . Because Tehran often acts in ways that
are intentionally ambiguous, U.S. leaders must avoid making presumptions about Iranian intentions that are derived simply from
Iranian capabilities. Thus, any analysis of future threats from the Islamic Republic must be grounded in an
understanding of the domestic roots of Tehran’s behavior. Similarly, the United States must identify the
buffers and barriers to Iran’s power-projection efforts . Some of these limitations are present in the
regional system Iran is trying to influence, but they are also found inside Iran’s unique strategic
culture. In Chapter Two, therefore, we begin by addressing the domestic drivers for assertiveness and caution in Iranian behavior,
focusing on the regime’s perception of Iran’s place in the world and weighing the role of ideology, pragmatism, and factionalism in
its policy calculations. To set the stage for subsequent analysis, we identify three principal themes that inform Iran’s regional
strategy: deterrence, support for Islamists and non-state actors, and an appeal to Arab public opinion. In Chapter Three, we cover
Iran’s developing conventional military buildup, discussing the significant gap between its doctrinal aspirations
for asymmetric warfare and the reality of its rather limited conventional capability. In Chapter Four,
we explore Tehran’s interactions with non-state Islamists in Lebanon and Iraq, assessing the extent of Iranian control over these
groups and the resulting threat to U.S. interests. In Chapter Five, we cover Tehran’s appeal to Arab public opinion, revealing how
Arab sentiment frequently swings between acclaim for and criticism of the Islamic Republic,
making it an unstable strategic resource . Finally, our concluding chapter surveys previous U.S. policies toward the
Islamic Republic and formulates a new U.S. strategy paradigm by acknowledging the aforementioned limitations on Iranian power
and adopting a more multilateral approach. Our Methodology Is Grounded in Primary Sources Although
discerning Tehran’s motivations and future strategy is challenging, it is possible to derive insights from a number of sources. We
analyzed Iranian media, the statements of key Iranian leaders, and Persian-language publications of
Iranian think tanks and policy journals. We also drew from phone and email discussions with Iranian scholars and interactions
with former Iranian diplomats at meetings in the Middle East. 10 In assessing Arab opinions of Iran, we benefited from extensive
discussions with Arab officials, military commanders, diplomats, scholars, and religious clerics, principally in the Persian Gulf region
but also in Egypt and Jordan. Moreover, we made use of Arabic-language print and broadcast resources. Finally, our
research
drew from consultations with government analysts and USAF personnel , as well as previous RAND work
on Iran’s security policy, the behavior of nuclear-armed states, and U.S. strategies for dealing with a post-nuclear Iran. 11 Based on a
thorough examination of these sources, we present an empirically rooted analysis to inform U.S. decisionmakers who need to
anticipate patterns and variations in Iranian behavior.

Iran’s military was destroyed 20 years ago


Gerhsman 5. [John, Middle East editor for the Foreign Policy in Focus Project, 7/26/2005, “The U.S. and Iran: Democracy,
Terrorism, and Nuclear Weapons”, Foreign Policy in Focus, http://www.fpif.org/fpiftxt/173]

One litmus test of a country’s aggressive designs on its neighbors is military procurement. As
a country amasses arms,
bolsters troops, and acquires training, the chance that it may initiate war escalates, because the
probability of success rises. On this front, Iran also seems less of a threat. Iran’s military procurement
relative to the Gulf States is far less than it was during the 1970s under the shah, when the United States was actually
promoting arms sales to Iran. In addition, much of Iran’s naval capability was destroyed by the United States in
the 1987-88 tanker war, and Iran lost much of its ground weaponry during Iraq’s 1988 offensive. As
much as half of Iran’s inventory of major land-force weapons were destroyed in the course of the
war with Iraq.9 Although Iran’s defensive capabilities have improved somewhat, there is little to suggest that
Teheran poses any kind of realistic offensive threat to the region. Indeed, Iranian tanks and planes actually
number less than in 1980.10
Iran Navy Answers
No Iran navy threat

Peck ‘11
(Michael, writer for Wired magazine, “No, The Iranian Navy Isn’t About to Shell Cape Cod” 9.28.11)

Before the Department of Homeland Security triggers Orange Alerts for incoming Iranian dreadnoughts, and the peaceful citizens of
Cape Cod and Norfolk head for the hills, Danger Room wanted to bring you this public service announcement. Or, more accurately, a
reminder that Afghan opium may have wafted into the hallowed halls of the Iranian Admiralty. The
most suitable ships – that is, those that could survive more than 30 seconds off Norfolk – would be the three Kilo-class diesel subs
that Iran acquired from the Soviet Union in the 1990s. But these boats are not in such great shape, and assuming they
were not badly damaged in mid-ocean collisions with Zionist sharks, the 6,000-mile endurance of the Kilos happens to
more or less be the distance from the Persian Gulf to New York. Perhaps the subs could sail to New York, in the same way
that those old Soviet bombers could reach the Big Apple – if they flew a one-way mission. Considering there aren’t
too many overseas Iranian naval bases, or too many nations that would want to be associated with sailing
a battle fleet off Boston, this leaves North Korea (wrong ocean) and Venezuela as resupply bases. Venezuela is also run by a
dictator equally capable of pretending he has a long-distance navy, and in fact, the Tehran Times article specifically mentions Iranian
ships in the Gulf of Mexico. There are other possibilities, of course. Iranian frigates and supply ship combos have sailed to the Red
Sea. Perhaps they could reach the U.S. Or how about merchant ships carrying speedboats with RPGs that would blockade Houston?
Let your imagination run wild. The mullahs and their admirals certainly have.
Iran Prolif Answers
Frontline

No prolif and long timeframe

Kahl ’12 (Colin H. Kahl 12, security studies prof at Georgetown, senior fellow at the Center for a New American Security, was
Deputy Assistant Secretary of Defense for the Middle East, “Not Time to Attack Iran”, January
17, http://www.foreignaffairs.com/articles/137031/colin-h-kahl/not-time-to-attack-iran?page=show

Kroenig argues that there is an urgent need to attack Iran's nuclear infrastructure soon, since Tehran could "produce its first nuclear
weapon within six months of deciding to do so." Yet that last phrase is crucial. The International Atomic Energy Agency (IAEA) has
documented Iranian efforts to achieve the capacity to develop nuclear weapons at some point, but there is
no hard evidence that Supreme Leader Ayatollah Ali Khamenei has yet made the final decision
to develop them. In arguing for a six-month horizon,Kroenig also misleadingly conflates hypothetical timelines to produce
weapons-grade uranium with the time actually required to construct a bomb. According to 2010 Senate testimony by James
Cartwright, then vice chairman of the U.S. Joint Chiefs of Staff, and recent statements by the former heads of Israel's national
intelligence and defense intelligence agencies, even if Iran could produce enough weapons-grade uranium for a bomb in six
months, it would take it at least a year to produce a testable nuclear deviceand considerably longer to
make a deliverable weapon. And David Albright, president of the Institute for Science and International Security (and the source of
Kroenig's six-month estimate), recently told Agence France-Presse that there
is a "low probability" that the
Iranians would actually develop a bomb over the next year even if they had the capability  to do
so. Because there is no evidence that Iran has built additional covert enrichment plants  since the
Natanz and Qom sites were outed in 2002 and  2009, respectively, any near-term move by
Tehran to produce weapons-grade uranium would have to rely on its declared facilities. The
IAEA would thus detect such activity with sufficient time for the international community to mount a forceful
response. As a result, the Iranians are unlikely to commit to building nuclear weapons until they can do so much more quickly or out
of sight, which could be years off.

No impact to Iran prolif – their ev is biased

– rationality, nuclear deterrence and defense posture check escalation

Waltz 12 – Senior Research Scholar at the Saltzman Institute of War and Peace Studies and Adjunct Professor of Political
Science at Columbia University (Kenneth N., Jul/Aug, “Why Iran Should Get the Bomb,” EBSCO)

UNFOUNDED FEARS One reason the danger of a nuclear Iran has been grossly exaggerated is that the debate
surrounding it has been distorted by misplaced worries and fundamental misunderstandings of how
states generally behave in the international system. The first prominent concern, which undergirds many others, is that the
Iranian regime is innately irrational. Despite a widespread belief to the contrary, Iranian policy is made not by "mad
mullahs" but by perfectly sane ayatollahs who want to survive just like any other leaders. Although Iran's leaders
indulge in inflammatory and hateful rhetoric, they show no propensity for self-destruction . It would be a grave
error for policymakers in the United States and Israel to assume otherwise. Yet that is precisely what many U.S. and Israeli officials
and analysts have done. Portraying Iran as irrational has allowed them to argue that the logic of nuclear deterrence
does not apply to the Islamic Republic. If Iran acquired a nuclear weapon, they warn, it would not hesitate to use it in a first strike
against Israel, even though doing so would invite massive retaliation and risk destroying everything the Iranian regime holds dear.
Although it is impossible to be certain of Iranian intentions, it is far more likely that if
Iran desires nuclear weapons, it
is for the purpose of providing for its own security, not to improve its offensive capabilities (or destroy
itself). Iran may be intransigent at the negotiating table and defiant in the face of sanctions, but it still acts to secure its own
preservation. Iran's leaders did not, for example, attempt to close the Strait of Hormuz despite issuing blustery warnings that they
might do so after the EU announced its planned oil embargo in January. The Iranian regime clearly concluded that it did not want to
provoke what would surely have been a swift and devastating American response to such a move. Nevertheless, even some
observers and policymakers who accept that the Iranian regime is rational still worry that a nuclear weapon would embolden it,
providing Tehran with a shield that would allow it to act more aggressively and increase its support for terrorism. Some analysts
even fear that Iran would directly provide terrorists with nuclear arms. The problem with these concerns is that they contradict the
record of every other nuclear weapons state going back to 1945. History
shows that when countries acquire the
bomb, they feel increasingly vulnerable and become acutely aware that their nuclear weapons
make them a potential target in the eyes of major powers. This awareness discourages nuclear states from
bold and aggressive action. Maoist China, for example, became much less bellicose after acquiring nuclear weapons in 1964,
and India and Pakistan have both become more cautious since going nuclear. There is little reason to believe Iran would break this
mold.

No Middle East prolif from Iran

Miklos ‘3-3 (2013)

(Timothy, 2nd year M.A. student at the Elliott School of International Affairs in Security Policy Studies with a focus on nuclear
weapons. He holds a bachelor’s degree in Political Science from the University of Michigan. Prior to his undergraduate studies he
served honorably in the United States Marine Corps for 8 years. He is fluent in Russian. “Iran Proliferation Triggering a Nuclear
Domino Effect in the Middle East: An Unrealistic Scenario.” International Affairs Review. http://www.iar-gwu.org/node/468)

President Obama has stated that Iranian acquisition of a nuclear weapon will spark an arms race in the Middle East. This view is
a status quo dogma among policymakers of both the Republican and Democratic parties, and dissenting views are
generally ignored. Ari Shavit of Haaretz identifies the most at-risk states as Egypt, Saudi Arabia, and Turkey. However, a
nuclear arms race in the Middle East in response to an Iranian weapon is highly unlikely. For those countries
most likely to proliferate, the political and financial costs are too high. The nuclear domino scenario has been an
accepted doctrine since 1962 when President Kennedy warned that by the 1970s there would be around 25 nuclear weapon states.
Yet, today there are only nine. According to a recent Center for a New American Security (CNAS) report, “Cairo does not see Iran’s
nuclear ambitions as an existential threat.” Egypt’s true enemy is Israel, which has defeated Egypt in four consecutive wars. If Egypt
did not pursue a nuclear option to deter its nuclear-armed enemy Israel, then it will not do so against Iran. Egypt simply does
not have the financial resources, nuclear infrastructure, or motive to build a successful clandestine nuclear
program, as its facilities are under IAEA safeguards. As a signatory of the Treaty on the Non-Proliferation of Nuclear
Weapons (NPT), Egypt has remained committed to non-proliferation since the Treaty’s inception and would be unlikely to
withdraw. Even if Egypt had the capability and intention to pursue nuclear weapons, its security would not be enhanced. An
attempted breakout would likely be destroyed in a preemptive strike by Israel, which has proven the credibility of this threat twice
by destroying the Osirak reactor in Iraq in 1981 and the Al Kibar reactor in Syria in 2007. Unlike Iran, Egypt does not have long
distances, deep reactors, and strong air defenses to protect itself from Israeli preemption. Iran poses the largest threat to Saudi
Arabia and, as such, the Kingdom would have the strongest security motive to pursue a deterrent. Riyadh has called on a Nuclear
Weapon Free Zone (NWFZ) in the Middle East, yet has repeatedly warned that an Iranian nuclear weapon may compel it to follow
suit. This is not credible and is likely an attempt to pressure the United States to take greater action against Tehran. According
to Philipp Bleek of the Monterey Institute, “states whose rivals pursue or acquire nuclear weapons are much likely to themselves
explore a nuclear weapons option…but are no more (or less) likely to pursue or acquire nuclear weapons” ("Why do states
proliferate?," Forecasting Nuclear Proliferation in the 21st Century: Volume 1 The Role of Theory). Nasser of Egypt made a similar
threat in response to Israel’s nuclear program and explored Egypt’s nuclear possibilities, but in 1968 chose to sign the NPT instead.
Saudi Arabia has virtually no domestic nuclear infrastructure, resources, or knowledge base to conduct
a “crash” program. It is also an NPT state and has many U.S. military and foreign investors on its territory, making it
difficult to support such a program. Its only option would be to purchase a nuclear weapon from Pakistan. However, Islamabad
is unlikely to spare any weapons, as they are needed to deter India. Additionally, selling a nuclear weapon would bring
world condemnation on Pakistan and leave it a pariah state surrounded by nuclear enemies. Riyadh would risk losing the support of
the United States if it were to attempt to pursue a deterrent, leaving it open to an Israeli strike. Instead, Saudi Arabia will
likely rely
on its preferred weapons of “cash and diplomacy,” finding the U.S. nuclear umbrella a “more attractive offer.”
Turkey is a NATO member with around 70 tactical nuclear weapons on its soil and is protected by the U.S. nuclear
umbrella. An indigenous nuclear program would forfeit this position. Etel Solingen (“Domestic Models of Political Survival,"
Forecasting Nuclear Proliferation in the 21st Century: Volume 1 The Role of Theory) asserts that states with integrated economies
face greater costs to proliferating and are therefore less likely to do so. There is too much at stake for these
nations to develop nuclear weapons, as they each stand to suffer great financial and political losses and will ultimately
be less secure because of it. The United States has far greater influence over these nations than it does over Iran. Washington should
keep the pressure on Tehran to adhere to IAEA safeguards. However, alarmist rhetoric of a Middle East arms race
is unjustified and not conducive to reaching an agreeable diplomatic settlement with Iran.

Iran can’t proliferate

Hymans ‘12 [Jacques E. C. Hymans, PhD from Harvard, Associate Professor of International Relations at the University of
Southern California, his most recent book is Achieving Nuclear Ambitions: Scientists, Politicians, and Proliferation, “Botching the
Bomb: Why Nuclear Weapons Programs Often Fail on Their Own-and Why Iran's Might, Too,” Foreign Affairs91. 3 (May/Jun 2012):
44-53, Proquest]

IN THE intensifying crisis over Iran's nuclear activity,


the great proliferation slowdown has gone all but
unmentioned. Yet this robust global trend clearly indicates a need to guard against any hasty
conclusion that Iran's nuclear program is about to achieve its ultimate aims. Iran's nuclear scientists and
engineers may well find a way to inoculate themselves against Israeli bombs and computer hackers. But they face a potentially
far greater obstacle in the form of Iran's long-standing authoritarian management culture. In a study
of Iranian human-resource practices, the management analysts Pari Namazie and Monir Tayeb concluded that
the Iranian regime has historically shown a marked preference for political loyalty over professional
qualifications. "The belief," they wrote, "is that a loyal person can learn new skills, but it is much more difficult to teach loyalty
to a skilled person." This is the classic attitude of authoritarian managers. And according to the Iranian political scientist Hossein
Bashiriyeh, in recent years, Iran's "irregular and erratic economic policies and practices, political nepotism and general
mismanagement" have greatly accelerated. It is hard to imagine that the politically charged Iranian nuclear program is sheltered
from these tendencies. It is surely more difficult to assess the quality of Iran's nuclear management than it is to count the number of
Iranian centrifuge machines. But such an assessment is vital, because the progress of Iran's program will depend on how much
professional autonomy its scientists and engineers are able to retain. In the meantime, a number of broad lessons from the great
proliferation slowdown can help provide a more sober assessment of the situation. The first lesson is to be wary of narrow,
Recent alarming estimates of Iran's timeline to the
technocentric analyses of a state's nuclear weapons potential.
bomb have been based on the same assumptions that have led Israel and the U nited S tates to
consistently overestimate Iran's rate of nuclear progress for the last 20 years . The majority of official
U.S. and Israeli estimates during the 1990s predicted that Iran would acquire nuclear weapons by 2000. After that date passed with
no Iranian bomb in sight, the estimate was simply bumped back to 2005, then to 2010, and most recently to 2015. The point is not
that the most recent estimates are necessarily wrong but rather that they lack credibility. In particular ,
policymakers should
heavily discount any intelligence assessments that do not explicitly account for the impact of
management quality on Iran's proliferation timeline. The second lesson of the proliferation slowdown is that
policymakers should reject analyses based on assumptions about a state's capacity to build nuclear programs in secret. Ever since
the mid-1990s,official proliferation assessments have freely extrapolated from minimal data, a
practice that led U.S. intelligence analysts to wrongly conclude that Iraq had reconstituted its
weapons of mass destruction programs after the Gulf War. The United States must guard against the possibility of an equivalent
intelligence failure over Iran. This is not to deny that Tehran may be keeping some of its nuclear work secret. But it is simply
unreasonable to assume, for example, that Iran has compensated for the problems it has faced with centrifuges at the Natanz
uranium-enrichment facility by hiding better-working centrifuges at some unknown facility. Indeed, when Iran has tried to hide
weapons-related activities in the past, it has often been precisely because the work was at the very early stages or was going badly.

New sanctions solve

Kahl ’12 (Colin H. Kahl 12, security studies prof at Georgetown, senior fellow at the Center for a New American Security, was
Deputy Assistant Secretary of Defense for the Middle East, “Not Time to Attack Iran”, January
17, http://www.foreignaffairs.com/articles/137031/colin-h-kahl/not-time-to-attack-iran?page=show

In making the case for preventive war as the least bad option, Kroenig dismisses any prospect of finding a diplomatic solution to the
U.S.-Iranian standoff. He concludes that the Obama administration's dual-track policy of engagement and pressure has failed to
arrest Iran's march toward a bomb, leaving Washington with no other choice but to bomb Iran. But this ignores the severe economic
strain, isolation, and technical challenges that Iran is experiencing. After years of dismissing the economic effects of
sanctions, senior
Iranian officials now publicly complain about the intense pain  the sanctions
are producing. And facing the prospect of U.S. sanctions against Iran's central bank and
European actions to halt Iranian oil imports, Tehran signaled in early January some willingness to
return to the negotiating table. Washington must test this willingness and, in so doing, provide Iran with a clear strategic
choice: address the concerns of the international community regarding its nuclear program and see its isolation lifted or stay on its
current path and face substantially higher costs. In framing this choice, Washington must be able to assert that like-minded states
are prepared to implement oil-related sanctions, and the Obama administration should continue to emphasize that all options,
including military action, remain on the table.
---Iran Prolif- No Threat
Iran does not have a nuclear weapon

Zakaria and Hosenball ’12 (Tabassum Zakaria and Mark Hosenball, WASHINGTON | Fri Mar 23, 2012 4:00pm GMT

(Reuters) - The
United States, European allies and even Israel generally agree on three things about Iran's nuclear
program: Tehran does not have a bomb, has not decided to build one, and is probably years
away from having a deliverable nuclear warhead. Those conclusions, drawn from extensive interviews with current
and former U.S. and European officials with access to intelligence on Iran, contrast starkly with the heated debate surrounding a
possible Israeli strike on Tehran's nuclear facilities."They're keeping the soup warm but they are not cooking it," a U.S.
administration official said. Reuters has learned that in late 2006 or early 2007, U.S.
intelligence intercepted telephone
and email communications in which Mohsen Fakhrizadeh, a leading figure in Iran's nuclear program,
and other scientists complained that the weaponization program had been stopped. That led to a bombshell
conclusion in a controversial 2007 National Intelligence Estimate: American spy agencies had "high
confidence" that Iran halted its nuclear weapons program in the fall of 2003. Current and former U.S.
officials say they are confident that Iran has no secret uranium-enrichment site outside the purview of
U.N. nuclear inspections.They also have confidence that any Iranian move toward building a functional nuclear
weapon would be detected long before a bomb was made. These intelligence findings are what underpin
President Barack Obama's argument that there is still time to see whether economic sanctions will compel Iran's
leaders to halt any program. The Obama administration, relying on a top-priority intelligence collection program and after
countless hours of debate, has concluded that Iranian leaders have not decided whether to actively construct a nuclear weapon,
current and former officials said.

No Iran prolif – latest intelligence

CSM ’12 (1/19/12 – Christian Science Monitor (Dan Murphy, “Israel says ... Iran isn't building a nuclear weapon.”
http://www.csmonitor.com/World/Backchannels/2012/0119/Israel-says-Iran-isn-t-building-a-nuclear-weapon?
utm_source=feedburner&utm_medium=feed&utm_campaign=Feed%3A+feeds%2Fworld+(Christian+Science+Monitor+%7C+World))

To be sure, Iraq and Iran are not the same; Iran is indeed enriching uranium, a key component of a nuclear weapon. But the fear-
mongering sounds the same. What today's arguments about Iran ignore, however – much as the arguments in favor
of the Iraq war ignored – was the position of the US intelligence community that Iran is not currently
building a nuclear weapon. The US position appears to be that Iran is seeking the ability to build a weapon, without
actually taking that final step. Two weekends ago, Defense Secretary Leon Panetta said: "Are they trying to develop a
nuclear weapon? No. But we know that they're trying to develop a nuclear capability and that's what concerns us and our
red line to Iran is: Do not develop a nuclear weapon." And it's not just the US assessment. Israel's liberal newspaper
Haaretz reported yesterday that "Iran has not yet decided whether to make a nuclear bomb,
according to the intelligence assessment Israeli officials will present later this week to [visiting] Gen. Martin
Dempsey, chairman of the US Joint Chiefs of Staff." Israeli Defense Minister Ehud Barak poured cold water on
speculation that his country is planning a unilateral attack against Iran. "This entire thing is very far off. I
don’t want to provide estimates [but] it’s certainly not urgent," he said.
No threat from Iran – status quo solves

Nader and Dobbins ’12 (1/5/12 – senior policy analyst at the RAND Corporation and director of the International Security
and Defense Policy Center at RAND (Alireza and James, “Iran's Self-Destructive Gamble.” The New York Times.
http://www.nytimes.com/2012/01/06/opinion/irans-self-destructive-gamble.html)

Iran is on the brink of losing its only real ally, Syria, as President Bashir al-Assad looks as if he could be the next Arab dictator to fall.
Tightening international sanctions are slowing Iran’s nuclear program while limiting its ability to
project power. Saudi Arabia, Iran’s principal regional rival, is leading the other Gulf states in an
ever more explicit anti-Iranian coalition. The United States is strengthening its military and political ties with several
of these states. Iran’s leaders have watched U.S. forces topple Saddam Hussein and the Taliban with relative ease and NATO help do
the same with Col. Muammar el-Qaddafi in Libya; Iran’s antiquated conventional forces are no match for the U.S. military. And the
Iranian regime believes that the United States remains committed to a policy of regime change, even though Washington might not
presently have the appetite for a new military intervention. Iran’s leaders believe that having a nuclear weapons capability could
deter a U.S. military attack on Iran and safeguard a vulnerable regime. It
is unlikely that the regime, concerned with
self-preservation, will be the first party to use nuclear weapons in a conflict with the United
States or Israel. The actual use of a nuclear weapon by Iran would surely lead to the regime’s
ultimate end. Iran is often depicted as an irrational actor, with Ahmadinejad’s seemingly erratic behavior and
odious rhetoric serving as justification. Yet the clerical-led regime in Tehran is no less rational and
calculating than the former Soviet Union or Communist China, both of which were successfully
deterred and contained by the United States . What’s more, the Iranian regime faces many
internal and external challenges that will facilitate U.S. containment of Iran if it obtains nuclear
weapons. The United States possesses the overwhelming diplomatic, economic and military
power to cope with an adversary vulnerable to the very same forces that have led to the
toppling of regimes across the Middle East .

No Iran nuclearization

Warrick, ‘11 – Pulitzer Prize winning reporter for the Washington Post.  He currently covers intelligence. (Joby, “Iran’s nuclear
program suffering new setbacks, diplomats and experts say,” http://www.washingtonpost.com/world/national-security/irans-
nuclear-program-suffering-new-setbacks-diplomats-and-experts-say/2011/10/17/gIQAByndsL_story.html?wpisrc=al_national)

Iran’s nuclear program, which stumbled badly after a reported cyberattack last year, appears beset by poorly
performing equipment, shortages of parts and  other woes as global sanctions exert a mounting toll, Western
diplomats and nuclear experts say. The new setbacks are surfacing at a time when Iran faces growing international pressure,
including allegations that Iranian officials backed a clumsy attempt to kill a Saudi diplomat in Washington. Analysts say Iran has
become increasingly frustrated and erratic as political change sweeps the region and its nuclear program struggles. Although Iran
continues to stockpile enriched uranium in defiance of U.N. resolutions, two
new reports portray the country’s
nuclear program as riddled with problems as scientists struggle to keep old er equipment
working. At Iran’s largest nuclear complex, near the city of Natanz, fast-spinning machines called centrifuges churn out enriched
uranium. Butthe average output is steadily declining as  the equipment breaks down, according to an analysis
of data collected by U.N. nuclear officials. Iran has vowed to replace the older machines with models that are faster and more
efficient. Yet new
centrifuges recently introduced at Natanz contain parts made from an inferior  type of metal
that is weaker and more prone to failure, according to a report by the Institute for Science and International Security, a
Washington nonprofit group widely regarded for its analysis of nuclear programs. “Without question, they have been set back,” said
David Albright, president of the institute and a former inspector for the U.N. nuclear watchdog, the International Atomic Energy
Agency. Although the problems are not fatal for Iran’s nuclear ambitions, they have “hurt Iran’s ability to break out quickly” into the
ranks of the world’s nuclear powers, Albright said. U.S. intelligence officials have concluded that Iran’s clerical leaders are seeking to
rapidly acquire the technical capability to make nuclear weapons, though there are indications that top officials have not yet
firmlycommitted to building the bomb. Iran maintains that its nuclear intentions are peaceful. Western diplomats and
nuclear experts say Iranian officials have been frustrated and angered by the program’s numerous setbacks,
including deadly attacks on Iranian nuclear scientists.  Four Iranian scientists have been killed by unidentified
assailants since 2007, and a fifth narrowly escaped death in an attempted car-bombing. Some U.S. officials have suggested that the
alleged plot to assassinate Saudi Arabia’s ambassador to Washington was emblematic of the frustration and disarray within Iran’s
ruling elite at a time when internal unrest has destabilized the nation’s closest Arab ally, Syrian President Bashar al-Assad. U.S.
officials have said that the alleged assassination plot originated from elements within Iran’s elite Quds Force, a covert paramilitary
group. But it is not clear whether the nation’s top leaders knew about or approved the plan. The alleged $1.5 million scheme fell
apart when an Iranian American accomplice sought to hire a Mexican hit man who in reality was an undercover informant for the
Drug Enforcement Administration. “It could be an outgrowth of the fact that we’ve crossed a red line in the Iranians’ eyes,” said a
senior administration official involved in high-level discussions of Iran policy. “We’re used to seeing them do bad things, but this plot
was so bizarre, it could be a sign of desperation, a reflection of the fact that they’re feeling under siege,” said the official, who spoke
on the condition of anonymity so he could discuss the matter candidly. Albright noted that Iran has behaved erratically in other
arenas as well, using novel tactics to try to gain advanced materials and technology for its nuclear program and weapons systems.
“Theirprocurement efforts  are less thought-through, and they’re getting caught a lot more, which suggests that they are
becoming more desperate,” he said. The Obama administration has sought to use the revelations of the alleged plot to rally
international support for stronger sanctions and other measures to discourage Iran from seeking to become a nuclear power. In
Tehran, officials said Monday that they were ready to investigate allegations by the United States that the Quds Force plotted to kill
Saudi Ambassador Adel al-Jubeir. “We are ready to patiently investigate any issue, even if it’s fabricated,” Foreign Minister Ali Akbar
Salehi told the state-run Islamic Republic News Agency. “We also asked America to give us the information related to this scenario.”
Salehi and other Iranian officials, however, continued to maintain that Iran had nothing to do with the alleged plot, which they
dismissed as a “bad Hollywood script.” The plot allegations have seriously strained Iran’s already fragile relations with the United
States and Saudi Arabia. In an interview on al-Jazeera English on Monday, Iranian President Mahmoud Ahmadinejad said the Obama
administration made the allegations to divert attention from its own economic problems. “Why has the U.S. administration leveled
this accusation?” Ahmadinejad said. “The truth will be revealed in the end.” The studies of Iran’s struggling uranium program draw
on data collected by U.N. officials who conduct regular inspections of Iran’s facilities to ensure that the nation is not
diverting the enriched product into a military weapons program. The inspectors’ report documented a sharp drop in
output in 2009 and 2010, providing the first confirmation of a major equipment failure linked to a computer
virus dubbed Stuxnet.
---Iran Prolif- AT: Nuclear Terror

Won’t happen – detection, cost, risk, can’t manage or predict terrorist behavior

Waltz 12 – Senior Research Scholar at the Saltzman Institute of War and Peace Studies and Adjunct Professor of Political Science
at Columbia University (Kenneth N., Jul/Aug, “Why Iran Should Get the Bomb,” EBSCO)

As for the risk of a handoff to terrorists, no country could transfer nuclear weapons without running a high risk
of beingfound out. U.S. surveillance capabilities would pose a serious obstacle, as would the United States' impressive and
growing ability to identify the source of fissile material. Moreover, countries
can never entirely control or even predict
the behavior of the terrorist groups they sponsor. Once a country such as Iran acquires a nuclear
capability, it will have every reason to maintain full control over its arsenal. After all, building a
bomb is costly and dangerous. It would make little sense to transfer the product of that investment
to parties that cannot be trusted or managed.

ZERO risk Iran gives terrorists the bomb

Mueller 12 (John, professor of political science at Ohio State University and author, “Iran: false nuclear fears cloud the west's
judgment”, 2/16, http://www.guardian.co.uk/commentisfree/2012/feb/16/iran-false-nuclear-fears, JD)

Alarmism about nuclear proliferation is fairly common coin in the foreign policy establishment. And of late it has been boosted by
the seeming efforts of Iran or its friends to answer covert assassinations, apparently by Israel, with attacks and attempted attacks of
their own in India, Georgia and Thailand. A non-hysterical approach to the Iran nuclear issue is entirely possible. It should take
several considerations into account. If the rattled and insecure Iranian leadership is lying when it says it has no intention of
developing nuclear weapons, or if it undergoes a conversion from that position (triggered perhaps by an Israeli air strike), it will find,
like all other nuclear-armed states, that the bombs are essentially useless and a considerable waste of time, effort, money and
scientific talent. Nuclear weapons have had a tremendous influence on our agonies and obsessions since 1945, inspiring desperate
rhetoric, extravagant theorising, wasteful expenditure and frenetic diplomatic posturing. However, they have been of little historic
consequence. And they were not necessary to prevent a third world war or a major conflict in Europe: each leak from the archives
suggests that the Soviet Union never seriously considered direct military aggression against the US or Europe. That is, there was
nothing to deter. Moreover, there never seem to have been militarily compelling – or even minimally sensible – reasons to use the
weapons, particularly because of an inability to identify targets that were both suitable and could not be effectively attacked using
conventional munitions. Iran
would most likely "use" any nuclear capacity in the same way all other nuclear
states have: for prestige (or ego-stoking) and to deter real or perceived threats. Historical experience
strongly suggests that new nuclear countries, even ones that once seemed hugely threatening, like communist China in the 1960s,
are content to use their weapons for such purposes. Indeed, as strategist (and Nobel laureate) Thomas Schelling
suggests,
deterrence is about the only value the weapons might have for Iran. Such devices, he points out,
"would be too precious to give away or to sell" and "too precious to waste killing people" when
they could make other countries "hesitant to consider military action". The popular notion that nuclear
weapons furnish a country with the capacity to "dominate" its area has little or no historical support – in the main, nuclear threats
since 1945 have either been ignored or met with countervailing opposition, not timorous acquiescence. It thus seems
overwhelmingly likely that,
if a nuclear Iran brandishes its weapons to intimidate others or get its way,
it will find that those threatened, rather than capitulating or rushing off to build a compensating arsenal of their own,
will ally with others, including conceivably Israel, to stand up to the intimidation – rather in the
way an alliance of convenience coalesced to oppose Iraq's invasion of Kuwait in 1990. Iran's leadership, though hostile
and unpleasant in many ways, is not a gaggle of suicidal lunatics. Thus, as Schelling suggests, it is exceedingly
unlikely it would give nuclear weapons to a group like Hezbollah to detonate, not least because
the rational ones in charge would fear that the source would be detected, inviting devastating
retaliation. Nor is an Iranian bomb likely to trigger a cascade of proliferation in the Middle East, as many people insist.
Decades of alarmist predictions about proliferation chains, cascades, dominoes, waves, avalanches, epidemics and
points of no return have proven faulty. The proliferation of nuclear weapons has been far slower than routinely expected
because, insofar as most leaders of most countries, even rogue ones, have considered acquiring the weapons, they have come to
appreciate several defects: the weapons are dangerous, distasteful, costly and likely to rile the neighbours. And the nuclear diffusion
that has transpired has had remarkably limited, perhaps even imperceptible, consequences. As Professor Jacques Hymans has
shown, the weapons have also been exceedingly difficult to obtain for administratively dysfunctional countries like Iran .
---Iran Prolif- AT: Egypt Prolif

Iran prolif won’t cause Egyptian prolif – old evidence doesn’t apply – empirics, new Egyptian
government, aging nuclear infrastructure, leaders consumed w/ democratic transition and
economic challenges, scarce financial resources, puts peace agreement w/ Israel at risk,
international backlash

Kahl 13 – Senior Fellow at the Center for a New American Security and an associate professor in the Security Studies Program at
Georgetown University’s Edmund A. Walsh School of Foreign Service (Colin H., Melissa G. Dalton, Visiting Fellow at the Center for a
New American Security, Matthew Irvine, Research Associate at the Center for a New American Security, February, “If Iran Builds the
Bomb, Will Saudi Arabia Be Next?” http://www.cnas.org/files/documents/publications/CNAS_AtomicKingdom_Kahl.pdf)

Nevertheless, the conventional wisdom that Iranian nuclearization will inevitably spark region-wide proliferation deserves closer
scrutiny. Historically,
“reactive proliferation” has been exceedingly rare. And in the current context, neither
Egypt nor Turkey is likely to respond to a nuclear-armed Iran by pursuing the bomb. Egypt’s
new Muslim Brotherhood-dominated government views Iran as a regional rival, but Cairo does not see Iran’s
nuclear ambitions as an existential threat. Moreover, Egypt’s aging nuclear infrastructure is in
poor shape, and the country’s leaders will be consumed for the foreseeable future with completing a rocky
democratic transition and addressing almost insurmountable economic challenges. As a result, the Egyptian
government is highly unlikely to divert scarce financial resources, put its peace agreement
with Israel at risk and invite the ire of the international community by pursuing nuclear weapons.7
---Iran Prolif- AT: Turkey Prolif

Iran prolif won’t cause Turkish prolif – empirics, none of the necessary technical infrastructure,
risks, NATO/American security guarantees solve

Kahl ‘13 – Senior Fellow at the Center for a New American Security and an associate professor in the Security Studies Program at
Georgetown University’s Edmund A. Walsh School of Foreign Service (Colin H., Melissa G. Dalton, Visiting Fellow at the Center for a
New American Security, Matthew Irvine, Research Associate at the Center for a New American Security, February, “If Iran Builds the
Bomb, Will Saudi Arabia Be Next?” http://www.cnas.org/files/documents/publications/CNAS_AtomicKingdom_Kahl.pdf)

Nevertheless, the conventional wisdom that Iranian nuclearization will inevitably spark region-wide proliferation deserves closer
scrutiny. Historically, “reactive proliferation” has been exceedingly rare. And in the current context, neither
Egypt nor Turkey is likely to respond to a nuclear-armed Iran by pursuing the bomb. Egypt’s new Muslim Brotherhood-dominated
government views Iran as a regional rival, but Cairo does not see Iran’s nuclear ambitions as an existential threat. Moreover, Egypt’s
aging nuclear infrastructure is in poor shape, and the country’s leaders will be consumed for the foreseeable future with completing
a rocky democratic transition and addressing almost insurmountable economic challenges. As a result, the Egyptian government is
highly unlikely to divert scarce financial resources, put its peace agreement with Israel at risk and invite the ire of the international
community by pursuing nuclear weapons.7 Ankara may have more anxiety regarding Iranian nuclearization, seeing it as a threat to
Middle East stability and Turkey’s growing regional influence. Turkey also has considerably more financial resources than Egypt does
to devote toward a nuclear program and has ambitious plans to expand its civilian nuclear sector. However, it
would likely take
many years for Turkey to fully develop the nuclear or technical infrastructure needed to support an
advanced nuclear weapons program. And, crucially, Turkey already possesses a credible nuclear
deterrent in the form of its longstanding NATO security guarantee. If Iran crosses the nuclear
threshold, Ankara is thus likely to aggressively pursue a Middle East nuclear-free zone while sitting
comfortably under the American nuclear umbrella. 8
---Iran Prolif- AT: Saudi Prolif

Even if Iran prolif causes Saudi Arabia to go nuclear, it’ll be civilian development, not weapons
prolif – also conventional prolif and security guarantees solve

Kahl 13 – Senior Fellow at the Center for a New American Security and an associate professor in the Security Studies Program at
Georgetown University’s Edmund A. Walsh School of Foreign Service (Colin H., Melissa G. Dalton, Visiting Fellow at the Center for a
New American Security, Matthew Irvine, Research Associate at the Center for a New American Security, February, “If Iran Builds the
Bomb, Will Saudi Arabia Be Next?” http://www.cnas.org/files/documents/publications/CNAS_AtomicKingdom_Kahl.pdf)

IV. PRO SPECTS FOR SAUDI PRO L IFERATION If Iran joins the nuclear club, Saudi Arabia would likely be
motivated to explore some form of nuclear deterrent in response. But the prediction that Riyadh
will rapidly acquire nuclear weapons – either by developing them indigenously or acquiring them in an illicit transfer
from Pakistan – is probably wrong. Instead, the Kingdom would be more likely to respond by developing more
robust conventional defenses and civilian nuclear capabilities. The Kingdom is also likely to
pursue a near-term nuclear security guarantee from either Pakistan or the United States, with Washington
ultimately proving to be the more attractive alternative.

Long timeframe and Iran prolif won’t cause Saudi prolif – conservative Saudi leadership prefers
stability, Israeli and Iranian arms races, U.S. security assistance, sanction fears, Saudi
commitments to non-prolif, international reputation concerns, religious concerns, focus on
domestic economy, empirics, lack of natural resources, technical expertise, experience, human
capital, managerial expertise, safety culture and regulatory, technical and legal structures, no
national coordination authority

Kahl 13 – Senior Fellow at the Center for a New American Security and an associate professor in the Security Studies Program at
Georgetown University’s Edmund A. Walsh School of Foreign Service (Colin H., Melissa G. Dalton, Visiting Fellow at the Center for a
New American Security, Matthew Irvine, Research Associate at the Center for a New American Security, February, “If Iran Builds the
Bomb, Will Saudi Arabia Be Next?” http://www.cnas.org/files/documents/publications/CNAS_AtomicKingdom_Kahl.pdf)

Disincentives for Saudi Proliferation This is normally where the story ends: with the
conclusion that Saudi Arabia
would face overwhelming incentives to pursue nuclear weapons should Iran become a nuclear-armed
state.69 Yet this standard narrative ignores powerful pressures pushing in the opposite direction. Security
di sincentives The conservative Saudi leadership strongly prefers stability – both at home and abroad – and
there is no doubt that Saudi rulers fear that a nuclear empowered Iran would threaten the Kingdom and the wider Middle East. But
Saudi Arabia acquiring its own nuclear weapons could, on net, make the threat to stability worse, not better.70 Domestically, the
Saudis would have to consider the prospect that nuclear weapons could fall into the hands of violent jihadist extremists opposed to
the regime.71 Regionally, the Kingdom would face the possibility that Israel would strike Saudi facilities to prevent the emergence of
another nuclear state in the region, just as Israel did in Iraq in 1981 and in Syria in 2007. (Indeed, the concern over triggering an
Israeli attack may have been the primary reason the Kingdom did not respond in kind to Israel’s nuclear program.72) Even if Saudi
Arabia could avoid being the target of a preventive strike, Riyadh
would have to consider the risks associated
with engaging in a nuclear arms race with Jerusalem and Tehran , including the possibility of nuclear crises
that could pose a direct and immediate existential threat to the regime. It is unclear how seriously Saudi
leaders would take these risks, but one additional possibility could not escape their calculations: the prospect that pursuing nuclear
weapons could lead to a rupture in the vital security relationship with the United States. If the past is prologue, then the American
reaction to any Saudi proliferation decision would be swift and punitive. In 1986, for example, Riyadh purchased dozens of
intermediate- range CSS-2 ballistic missiles capable of carrying nuclear warheads from China. When Washington learned of the
missile deal in 1988, there was a crisis in the relationship.73 Congress threatened to block the sale of equipment needed to sustain
Saudi Airborne Warning and Control System aircraft and put other elements of the security assistance relationship in jeopardy. The
Israelis also warned that they might strike the missile sites. In response to pressure from the George H. W. Bush administration,
Riyadh signed the NPT, and King Fahd provided personal assurances to Washington that Saudi Arabia would not pursue nuclear or
chemical warheads for the missiles.74 This episode is often portrayed as a clear example of Saudi desires to acquire nuclear weapons
capabilities – but it also suggests that the House of Saud would have to consider the prospect of a punitive U.S. response if the
Kingdom were to pursue nuclear weapons in reaction to Iranian nuclearization. Saudi
dependence on American
security assistance provides a powerful disincentive to Saudi nuclearization. The United States
Military Training Mission (USMTM) in Saudi Arabia, founded in 1953, is the largest U.S. foreign security assistance mission in the
world. Its core functions include training, advising and assisting the Saudi Arabian Armed Forces to develop strategic plans and
policy; conducting joint and coalition operations and exercises; maintaining interoperability among U.S., Saudi and regional partner
forces; managing professional military education programs; and assisting in sustainment and modernization of Saudi forces.75 The
Saudis particularly rely on the United States for access to cutting-edge military technology, as evidenced most recently by the $30
billion arms deal to Saudi Arabia announced in December 2011 that will provide Riyadh with an advanced variant of U.S. F-15 fighter
aircraft, as well as the logistics and maintenance packages required to keep these systems operating.76 Saudi leaders know
that U.S. law requires economic and military sanctions against nuclear proliferators , and they also know that
the Kingdom lacks the sympathy on Capitol Hill required to reliably block punitive measures.77 Thus, if Riyadh were to seek nuclear
weapons, Saudi leaders would have to expect that U.S. security assistance would be dramatically curtailed. Many of USMTM’s
activities would likely stop. Because the Kingdom relies heavily on U.S. contracts for maintenance and spare parts, this would
severely undermine the Saudi military’s ability to function and protect the Kingdom from internal and external threats. The effect on
core Saudi security interests would be immediate and severe. Reputati onal concerns Potential reputational damage to the Kingdom
would weigh against status-based motivations to acquire nuclear weapons in response to an Iranian bomb. A
Saudi drive for
the bomb would fly in the face of Riyadh’s commitments to nonproliferation norms. Saudi Arabia
joined the NPT in 1988,78 signed a comprehensive safeguards agreement with the International Atomic Energy Agency (IAEA) in
200579 and has consistently voiced its support for a nuclear-free zone in the Middle East.80 Moreover, according to Thomas
Lippman, now-deceased Crown Prince Sultan bin Abdulaziz, declared that nuclear weapons contravened the tenets of Islam.81
Because King Abdullah and other Saudi leaders highly value their standing with the international community and their status as
the “Custodians of the Two Holy Mosques,” they cannot easily cast aside reputational concerns or religious
objections to nuclear weapons. Thus, although the Saudis might calculate that nuclear weapons would help to check
Iranian aggression and prevent a tilt in the regional balance of power against the Kingdom, they would also likely worry that
violations of their commitments would mark them as international outlaws and jeopardize their credibility as champions of Islamic
law. Economic sancti ons The possibility of being targeted with economic sanctions would also factor into Saudi decisionmaking.
Saudi Arabia’s economy depends almost entirely on its oil sector, which in 2011 accounted for nearly 80 percent of budget revenues,
45 percent of gross domestic product (in real terms) and 90 percent of export earnings.82 Consequently, the Kingdom could be
highly vulnerable to energy sanctions.83 Riyadh has seen the effects of economic sanctions on other nuclear offenders, including the
U.S. reaction to Pakistan’s nuclear tests in 1998 and the harsh sanctions imposed on North Korea. Perhaps more relevant for Saudi
leaders’ calculations, they have witnessed the willingness of the international community to impose crippling energy sanctions on
Iraq and Iran – two major oil exporters – for their violations of the NPT. Furthermore, Saudi Arabia is much less economically self-
sufficient than contemporary Iran, suggesting that the Kingdom would be far more vulnerable to potential sanctions.84 Saudi
Arabia’s very centrality to the global oil market, however, means that Saudi leaders are likely to doubt the international community’s
willingness to target the Kingdom with crippling energy sanctions should they pursue nuclear weapons.85 In contrast to the
unprecedented international support that has existed for sanctions on Iran, the United States and other Western governments
would have considerable difficulty encouraging other states to adopt sanctions against Saudi oil. The Kingdom supplies about three
times more oil to the world market than Iran, giving it considerable leverage in shaping global oil prices.86 Moreover, there is
currently no country or set of countries with sufficient spare production capacity to compensate for an embargo against Saudi oil.
Indeed, Iranian sanctions have been effective because global oil prices have remained steady, which is partly a consequence of
Riyadh expanding its oil production to supply Iran’s customers.87 Furthermore, despite recent energy forecasts that predict
significant increases in oil production from countries like the United States (which is expected to overtake the Kingdom as the
world’s largest oil producer by the end of the decade), Saudi Arabia will continue to play a major role in shaping the global oil
market.88 Energy analysts argue that most of the additional volume of oil produced by the United States and other nations that are
not members of the Organization of the Petroleum Exporting Counties will likely be consumed by rising demand from China, India
and other emerging economic powers, instead of adding to a surplus in global oil supply that would help buffer the market from
price spikes.89 Moreover, declining production in conventional oilfields – including those in Nigeria, Brazil and elsewhere – is
expected to contribute to tightening in the global oil market. 90 Therefore, a disruption in Saudi oil supply – as a result of sanctions
or other events – would still have global ramifications for oil consumers, including the United States. This seems to take the threat of
oil sanctions as a dissuasion tool off the table for the foreseeable future. Yet even
if Saudi leaders believe they
would not be hit with significant oil penalties, they are likely to fear other negative economic
ramifications from a proliferation decision, including possible financial sanctions and limits on foreign
investment. Population growth rates in the Kingdom remain high, and about 29 percent of the country’s population is under the
age of 14.91 This produces significant structural challenges for the Saudi economy, including high unemployment rates and low per
capita income, despite high oil prices on the international market.92 Partially to address these issues, the Saudi government would
like to increase the levels of foreign direct investment in the Kingdom and better integrate into the global economy, a desire that
was evident in the 14-year Saudi bid for membership in the World Trade Organization, which resulted in Riyadh joining the
organization in December 2005.93 Ongoing needs to reform the Saudi economy , attract foreign direct investment
and better integrate into international markets act as a disincentive to building a nuclear arsenal because
potential investors might shy away from a potentially unstable and unfavorable market – a risk that would be compounded by the
effects of economic sanctions.94 Furthermore, the growing danger of domestic unrest in the wake of the Arab Spring will inevitably
enhance Saudi sensitivity to taking any provocative steps that could lead to investor concerns or economic disruption. The gravity
that Saudi leaders attach to these risks hinges on their calculation regarding both the likelihood and possible duration of sanctions.
The House of Saud might judge that the Saudi position in the oil market provides retaliatory options that would deter other states
from targeting the nation with overly harsh financial measures. And Saudi leaders might conclude that they could ride out short-
term dislocations, betting that the international community would eventually accept their nuclear program, much as global powers
eventually accommodated themselves to the Indian and Pakistani programs. Still, given the severe structural challenges to the Saudi
economy and the acute concerns among Saudi leaders regarding political stability, they could not completely discount the possibility
of being targeted with punitive financial sanctions. Reasons for restrai nt All told, the combined risks that Saudi nuclearization would
worsen threats to domestic and regional stability, threaten critical security ties with the United States, produce significant
reputational costs and trigger damaging sanctions would leave the Kingdom’s strategic position “precarious to the point of
untenability.”95 These
are extremely powerful disincentives. Moreover, Saudi nuclear restraint would
be consistent with the country’s historical pattern of behavior when confronted with significant regional
challenges. After Israel allegedly developed nuclear weapons in the late 1960s, for example, the Kingdom did not rush to build the
bomb. For decades, Riyadh viewed Israeli occupation of Arab land as the major source of instability in the region, and to this day,
Saudi Arabia demands that Israel withdraw from Arab territories seized during the 1967 war, including East Jerusalem, the location
of Islam’s third-holiest site.96 Yet the Kingdom did not seek nuclear weapons in an attempt to counter the threat or to generate
more stature or coercive influence to push Israel toward concessions on the Palestinian issue. Nor did the Kingdom pursue nuclear
weapons in the 1990s, despite being attacked by Iraqi ballistic missiles during the 1991 Gulf War and worrying that Iran and Syria
were on the verge of becoming the dominant powers in the region. The Saudis chose instead to continue to rely on Washington for
protection and diplomatically engaged Syria and, temporarily, Iran based on their common interest in containing Iraq.97 Saudi
Technical and Bureaucratic Constraints Even if Riyadh wanted to move decisively to produce nuclear weapons in response to a
nuclear-armed Iran, the technical and bureaucratic hurdles for developing a successful, indigenous nuclear weapons program would
be monumental. As the world’s largest oil exporter and a country with enormous foreign currency reserves, Saudi Arabia has
sufficient economic resources to eventually develop a robust nuclear program should its leaders decide to do so.98 However, such
a project would take more than a decade and may not be able to succeed even if the Saudi government
devoted considerable resources to the endeavor. Saudi Arabia currently lacks the natural resources, technical
expertise and practical experience required for uranium mining, uranium conversion, uranium
enrichment, reprocessing, fuel fabrication and nuclear power production – that is, nearly every
essential civilian building block required to eventually develop a nuclear bomb (see Saudi Arabia’s Nuclear Infrastructure text box).99
Developing the technology and expertise necessary to support an indigenous nuclear weapons program would require dramatically
expanding Saudi Arabia’s civilian nuclear energy sector. Such expansion could arguably be justified to meet a number of pressing
domestic needs. Nuclear energy could help power vitally important desalination efforts. It could also address a fundamental fiscal
dilemma – created by a combination of Saudi population growth, government fuel subsidies and increased domestic oil consumption
for electricity – that could make Saudi Arabia a major oil importer as early as 2030.107 By that date, according to some estimates,
the Kingdom would require oil to be $320 a barrel for the country to simultaneously meet rising domestic energy needs and
maintain adequate revenues from oil exports to meet anticipated budget requirements.108 At least partly for these reasons, Riyadh
announced an extraordinarily ambitious plan in June 2011 to spend $100 billion on 16 nuclear reactors over the next 20 years, with
the hope of completing the first pair of reactors between 2019 and 2021.109 The Kingdom has also been actively engaged with
several countries to strengthen civilian nuclear cooperation. In December 2006, the Saudis and other GCC states announced a joint
research initiative to expand civilian nuclear power and cooperation.110 In 2008, the Kingdom signed a Memorandum of
Understanding on Civil Nuclear Energy Cooperation with the United States to expand Saudi nuclear capabilities in the areas of
medicine, industry and power generation.111 In early 2011, Saudi Arabia signed an agreement with France, a leading producer of
civilian nuclear power plants, to expand Saudi access to French nuclear expertise.112 Later that same year, Saudi Arabia reached
nuclear cooperation agreements with Argentina and South Korea to facilitate research and development, including building nuclear
power plants and research reactors, as well as associated training, safety and waste management. 113 And, in January 2012, the
Kingdom inked a deal with China to cooperate in areas such as maintaining and developing nuclear power plants and research
reactors, as well as the manufacturing and supply of nuclear fuel elements.114 Despite Riyadh’s clear desire to expand its civilian
nuclear activities, however, it remains highly uncertain whether any of these arrangements and plans will bear much fruit or how
long they might take to significantly expand Saudi Arabia’s indigenous nuclear capabilities. Indeed, most nuclear experts see Saudi
plans as highly unrealistic.115 Furthermore, even if the Kingdom has a legitimate domestic requirement for nuclear power, it has “no
basis … to claim that it has any legitimate civilian need to acquire nuclear fuel production capability, including equipment and
facilities to enrich uranium or reprocess spent fuel.”116 Any Saudi attempt to develop indigenous fuel-cycle capabilities would
therefore raise significant suspicions within the international community regarding the intentions of the program. Perhaps for this
reason, Saudi officials have repeatedly stressed the exclusively peaceful nature of their nuclear activities. Following the
announcement of the 2006 GCC joint research initiative, for example, Prince Saud al-Faisal, the Saudi foreign minister, told
reporters, “Our aim is to obtain the technology for peaceful purposes, no more no less. … We want no bombs. … Our policy is to
have a region free of nuclear weapons.”117 Even though the GCC agreement was clearly meant to signal to Tehran that the Gulf
states would seek to compete in the area of nuclear expertise, the Saudis and their GCC partners also declared that their efforts
would be fully transparent and under IAEA safeguards.118 Through in its 2008 Memorandum of Understanding with the United
States, Riyadh similarly signaled its intent to forego domestic uranium enrichment or spent-fuel reprocessing in favor of procuring
nuclear fuel from market sources, although it has not yet made any firm commitments in this regard.119 Whether Saudi Arabia will
ultimately follow through with these pledges remains to be seen; official statements could be aimed at masking more malign
intentions. However, the commitments themselves create leverage points for the international community to limit the potential
proliferation dangers emanating from the Saudi program (see Section V). Last but not least, even if the Kingdom’s technical prowess
grows over time, any Saudi attempt to develop nuclear weapons would be complicated by significant bureaucratic and managerial
challenges. Put bluntly, the
Saudi bureaucracy lacks the human capital, managerial expertise, safety
culture and regulatory, technical and legal structures necessary to nurture and sustain a robust
domestic nuclear program, and the country has no national authority capable of coordinating all
the required activities.120 The country may be able to eventually overcome these constraints, but they suggest that the
prospect of Saudi Arabia moving decisively toward an indigenous nuclear weapons program in
response to an Iranian bomb, let alone succeeding in this endeavor, is remote.
Iran Strikes Answers
No US strike

Walt 3/18 (Stephen M. Walt, “Why I hope Obama is bluffing”,


http://walt.foreignpolicy.com/posts/2013/03/18/why_i_hope_obama_is_bluffing, March 18, 2013)

What do you suppose we would do? Would President Obama (or anyone else) immediately order
a preventive war? Not on your life, because he could not be sure that Iran wouldn't find some way to get a bomb on
American soil or use it against some close U.S. ally. Would Obama immediately announce a blockade or
threaten an invasion, in order to persuade Iran to voluntarily give up its weapons? Hardly, because we couldn't
put enough pressure on them to force compliance. Would the U.S. decide to abandon its
regional allies and let Iran dominate the Persian Gulf? Of course not -- for the same reasons
that it didn't abandon NATO when the Soviets tested a bomb in 1949 and it didn't abandon
Japan and South Korea when China and North Korea tested nuclear weapons. No, if Iran ever
did cross the nuclear weapons threshold, the United States would do what it has always done
when an adversary went nuclear: It would fall back on containment and deterrence. We would
extend our far more potent nuclear umbrella over key regional allies, and we would send clear and
unmistakable messages to Tehran about the dire consequences that would befall them if their new arsenal were ever used by
anyone. Getting a bomb wouldn't transform Iran into a global superpower, and it certainly wouldn't allow them to blackmail their
neighbors or launch a war of conquest. The only thing this situation would prevent the United States from doing is forcible regime
change, which is something we shouldn't be contemplating in any case.
Iran-Israel Strikes Answers
Frontline

Zero risk of Israeli strike on Iran -this impact is a huge joke – any evidence to the contrary is
based on Israel fearmongering in order to increase pressure on the US

Rubin, ‘12 – professor at the Interdisciplinary Center in Herzliya, Israel, the Director of the Global Research and International
Affairs (GLORIA) Center, and a Senior Fellow at the International Policy Institute for Counterterrorism (Barry, “Israel Isn’t Going to
Attack Iran and Neither Will the United States.” http://pjmedia.com/barryrubin/2012/01/26/israel-is-not-about-to-attack-iran-and-
neither-is-the-united-states-get-used-to-it/)

The radio superhero The Shadow had the power to “cloud men’s minds.” But nothing clouds men’s minds like anything that has to
do with Jews or Israel. This year’s variation on that theme is the idea that Israel is about to attack Iran.
Such a claim repeatedly appears in the media. Some have criticized Israel for attacking Iran and turning the Middle East into a
cauldron of turmoil (not as if the region needs any help in that department) despite the fact that it hasn’t even happened. On the
surface, of course, there is apparent evidence for such a thesis. Israel
has talked about attacking Iran and one can
make a case for such an operation. Yet any serious consideration of this scenario — based on actual
research and real analysis rather than what the uninformed assemble in their own heads or
Israeli leaders sending a message to create a situation where an attack isn’t necessary — is this:
It isn’t going to happen. Indeed, the main leak from the Israeli government, by an ex-intelligence
official who hates Prime Minister Benjamin Netanyahu, has been that the Israeli government already
decided not to attack Iran. He says that he worries this might change in the future but there’s no hint that this has
happened or will happen. Defense Minister Ehud Barak has publicly denied plans for an imminent attack as
have other senior government officials. Of course, one might joke that the fact that Israeli leaders talk about attacking
Iran is the biggest proof that they aren’t about to do it. But Israel, like other countries, should be subject to rational analysis. Articles
written by others are being spun as saying Israel is going to attack when that’s not what they are saying. I stand by my analysis and
before December 31 we will see who was right. I’m not at all worried about stating very clearly that Israel
is not going to go
to war with Iran. So why are Israelis talking about a potential attack on Iran’s nuclear facilities?
Because that’s a good way – indeed, the only way Israel has — to pressure Western countries to
work harder on the issue, to increase sanctions and diplomatic efforts. If one believes that somehow pushing Tehran into
slowing down or stopping its nuclear weapons drive is the only alternative to war, that greatly concentrates policymakers’ minds.
Personally, I don’t participate — consciously or as an instrument — in disinformation campaigns, even if they are for a good cause.
Regarding Ronen Bergman’s article in the New York Times, I think the answer is simple: Israeli leaders are not
announcing that they are about to attack Iran. They are sending a message that the United
States and Europe should act more decisively so that Israel does not feel the need to attack Iran
in the future. That is a debate that can be held but it does not deal with a different issue: Is Israel about to attack
Iran? The answer is “no.”

No strikes – and this doesn’t assume that Iran is rational

Rubin, ‘12 – professor at the Interdisciplinary Center in Herzliya, Israel, the Director of the Global Research and International
Affairs (GLORIA) Center, and a Senior Fellow at the International Policy Institute for Counterterrorism (Barry, “Israel Isn’t Going to
Attack Iran and Neither Will the United States.” http://pjmedia.com/barryrubin/2012/01/26/israel-is-not-about-to-attack-iran-and-
neither-is-the-united-states-get-used-to-it/)
Yet given the points made above, even the Iran-as-irrational analysis — and even assuming it to be correct, the
probability of being right about Iran ever trying to launch a nuclear attack is far lower than 100 percent — does not justify an
Israeli attack at this time. And, finally, Israel has other options.  The alternative is this:  As the Iranian regime
works hard to get nuclear weapons and missiles capable of carrying them, Israel uses the time to build a multi-level
defensive and offensive capability . These layers include: U.S. early warning stations and anti-missile missile installations
in the Gulf; Israeli missile-launching submarines; Israeli long-range planes whose crews have rehearsed and planned for strikes at
Iranian facilities; different types of anti-missile missiles capable of knocking down the small number of missiles Iran could fire
simultaneously;  covert operations, possibly including computer viruses and assassinations, to slow down Iran’s development of
nuclear weapons; improved intelligence; help to the Iranian opposition (though the idea of “regime change” in the near future is a
fantasy); and other measures. If and when there was a clear Iranian threat to attack Israel, then Israel could launch a preemptive
assault. And if no such threat ever materializes, Israel need never attack. Any future Iran-Israel war will happen if Iran’s regime
makes it unavoidable, not in theory but in actual practice. Note that attacking
a limited number of missiles and
launch facilities, that must be located closer to Israel within Iranian territory, is easy. Attacking
multiple nuclear facilities buried deep in the ground anywhere in Iran is hard . Ah, but what if Iran gives
small nuclear devices to terrorists? Well ask yourself two simple questions: 1. Would an Israeli attack on Iran ensure that this didn’t
happen? Answer: Not at all. 2. Would an Israeli attack on Iran ensure that Iran would definitely give nuclear devices to terrorists and
try to strike against Israel as quickly and as frequently as possible? Absolutely yes. Does
an Israeli strategy of not
launching an attack assume that Iran’s regime is “rational” and “peace-loving” and will be
deterred by Israel’s ability to strike back? Absolutely not . Indeed, quite the opposite. No such assumption is
required. Israel will simply be ready and alert based on the assumption that Iran might attack some day. But such a war, however
possible, is not inevitable. And since Israel cannot prevent Iran from obtaining nuclear weapons by attacking, there is no point in
doing so.

No strikes – Israel’s just saber rattling – it doesn’t have the capacity or confidence

Xinhua ’11 (“Israel unlikely to strike Iran without alerting U.S.” http://news.xinhuanet.com/english2010/world/2011-
11/07/c_122243128.htm)

The Israeli ambiguity over a possible air strike on Iran 's alleged nuclear weapons program continues as the
United Nations nuclear agency, the International Atomic Energy Agency (IAEA), is scheduled to release its report on Iran's nuclear
activity later this week. Western and Middle Eastern countries are all concerned that Iran is using its intentions for civilian use of
nuclear energy as a cover for producing nuclear weapons. Israel considers the issue an existential threat, owing to numerous
statements by Iranian President Mahmoud Ahmadinejad and other leaders calling for destruction of the Jewish State. The U.S.
administration has lately been asking Israel to clarify its position on a possible military strike, perhaps even without alerting
Washington beforehand. During a visit to Israel in October, U.S. Defense Secretary Leon Panetta met with Prime Minister Benjamin
Netanyahu and Defense Minister Ehud Barak. Panetta, however, reportedly left without a clear answer regarding Israel's true
intentions. "It's
very hard for me to see them (strike Iran) without coordination with the U.S.," Prof .
Joshua Teitelbaum, of the Interdisciplinary-Center in Herzliya, told Xinhua. "It's such a big thing
and it would involve crossing airspace where the U.S. is active in. They would need some kind of
coordination with the Americans to do this," he said. In addition, an Iranian retaliation could target U.S. ground
forces stationed in Iraq and Kuwait, as well as the American naval base in Qatar, the professor said. SANCTIONS "There's an
increasing sense of urgency on the issue now, and Israel
is becoming more outspoken about it as a way to
pressure the U.S. and other countries," Teitelbaum said. " The saber rattling serves to increase
the urgency." He argued that, for the U.S., which also pushes for tougher sanctions against Iran, the image of Israel as a nervous,
unpredictable country could serve Washington's agenda, warning United Nations Security Council members to take action -- before
Israel does. Israel
seems to be trying to project an image of a state with its "back against the wall ,"
he argued, one that might take radical steps if it feels threatened. In
the long run, however, Israel's best bet would
be to take on a role as part of a collective international effort, and not as a unilateral actor,
Teitelbaum said. He added that when it comes to the military capabilities, "the assessment is that
Israel can't do as much as the U.S. could." While the Israel Air Force (IAF) in 1981 launched a
successful raid on an Iraqi nuclear reactor, a mission to Iran is a completely different story. In
Iraq there was only one target, as opposed to Iran where the alleged nuclear facilities are spread
over the country and in some cases hidden underground. REFOCUSING THE AGENDA Prof. Shlomo
Aronson, of the Hebrew University in Jerusalem, said that Netanyahu and Barak want to shift
international attention to Iran, and to do this they "agreed upon putting the military option on
the table." "They have to make the military threats more visible without having made any
decision to attack Iran, because this goes beyond the capabilities of Israel's air force and army ,"
Aronson said. The IAF is currently equipped with the American-made F-15 and F- 16 fighters, which
lack the long-range capability to strike targets as far away as Iran without refueling midair . While
Israel has ordered a squadron -- 19 planes -- of the new U.S.-made F-35 fighter jets, the first ones are scheduled to arrive no earlier
than 2016.

No strikes – their evidence is unverified speculation

Golan ‘12– veteran Israeli journalist, former editor-in-chief of Ha'aretz and Globes (Matti, “Israel won't attack Iran without US go-
ahead.” http://www.globes.co.il/serveen/globes/docview.asp?did=1000716538&fid=411)

What is really going on between Israel and the US over Iran? Is what we read and hear all that there is? In other words, is
Israel
really planning to attack Iran without coordination with the US? Does Washington really oppose an Israeli
attack on Iran? eports say yes, that is exactly the case. But these reports do not meet my test of common sense, and at the margins
the reports, at least to me, do not make sense. For example, reports that the US is protecting its facilities and bases in
Europe on the basis of an assessment that Israel will attack Iran and that a counterattack would soon follow. I read these reports
that the US has accepted the possibility of an Israeli attack, and that all the superpower can say in the matter is that it is readying for
such a possibility. Protection? That's a response? I would expect that alongside
protecting its facilities, the US
would press Israel not to attack; and not just the usual and not-so-painful pressure, but
measures that fit the deed. After all, an Israeli attack would result in an Iranian response. What this response would entail,
and whether it would target Israel or other countries as well - this cannot be known, but it is possible. The US knows this - how could
it not know - and its response is to protect facilities? Not a full assault on Israel with all the means at its disposal (non-military means
of course)? Behind the scenes? Obviously, there are people who will say that the US is undoubtedly doing just this behind the
scenes. I don’t buy it. I know a bit about how these things work , and the accepted way to work is both behind the
scenes and on the front stage. If there were real and serious pressures, we would know about them. The US would make sure that
we know, because it would undoubtedly want to create public opinion, which is an integral part of managing pressure. The bottom
line that I want to reach is that if
there are disagreements between Israel and the US, they are at the
margins. I believe that there is no such thing that Israel would embark on an action with such severe
consequences, possibly even existential consequences, without coordination with the US. There is no such
thing, in my opinion, that President Barack Obama will be woken up one night with the news of an Israeli
attack. If Israel were to attack, it will be with his consent, support, and knowledge . So how do I
explain the many reports that Israel will launch an attack without coordinating it with the US? It
seems like a game to me. It's objective? Many to give the US an alibi, as if it didn’t know, in order to avoid a confrontation
with Iran; maybe it's a smokescreen to conceal the real contacts and agreements between Washington and Jerusalem; and maybe
it's disinformation targeting Iran. Everything
is possible, except for one thing: that Israel would attack Iran
without coordinating it with the world's great power and Israel's only friend . It does not matter what is
said and asserted, I don’t believe it.

Israel wont strike --- all eyes are on it and logistical hurdles
Keiler, 10 [7/25/10, Jonathan F , former captain in the Army's Judge-Advocate General Corps, The American Thinker, “Surprise!
Why An Israeli Strike on Iran is Unlikely”, http://www.americanthinker.com/2010/07/surprise_why_an_israeli_strike.html

If Israel does launch a military strike against Iran's nuclear facilities it will be the most widely anticipated military operation in
modern history, even more so than D-Day or the 2003 campaign against Iraq. The buildup to those operations lasted a few
years. Speculation about an Israeli strike on Iran has persisted for more than a decade. And this leads one to the most obvious of
conclusions -- that if Israel has not struck yet, it won't. I don't pretend to know one way or the other, but the fact
that conditions for an Israeli strike against Iran were more favorable a few years ago than they are today is a relatively
persuasive argument that the window of opportunity, if it ever existed, may have passed. In a recent piece, The Weekly
Standard's Reuel Marc Gerecht makes this point, among many others. But the gist of Gerecht's piece is that if the leadership of
the Israeli Air Force (IAF) believes that military success is probable, then the widely bruited, supposedly disastrous
consequences of such a strike, (e.g. Iranian counter moves in the Persian Gulf and against American and Israeli interests,
strengthening of the Iranian regime, weakening of opposition movements, etc.) are much overrated. Gerecht makes a

compelling case for an Israeli strike, if the IAF leadership thinks it is feasible. Where I think he may err -- and many other
pundits as well -- is in guessing that if the IAF proposes a plan, that Israeli Prime Minister Benjamin Netanyahu would be
particularly inclined accept an optimistic IAF evaluation and launch a strike. Of course, nobody knows the exact conditions for a
successful IAF strike, although if you want a hypothetical plane-by-plane and target-by-target operational plan the Center for
Strategic and International Studies produced one for general consumption. The real question is at what point Israel's political
leadership pushes past the uncertainty. Here the threshold is likely much higher than Gerecht and other like-minded pundits
imagine. It's true historically that Israel's leadership has put great faith in the IAF, and that this confidence has generally been
well rewarded. The IAF is the world's only air force to have taken out enemy nuclear installations, and it is a perfect two-for-two
in that regard (against Iraq and Syria.) Likewise, the IAF had spectacular successes in the 1967 War and the 1982 Lebanon War.
On the other hand, over-reliance in the ability of Israel's airman to solve its military problems led to setbacks in the 1973 Yom
Kippur War and the 2006 Lebanon War. Gerecht not only places great store on what the IAF might tell Netanyahu but on the

fact that Netanyahu is an ardent Zionist and Israeli patriot. And for that matter, there is little reason to doubt the bona
fides of Defense Minister Ehud Barak, or any other element of Israel's mainstream leadership when it comes to a genuine
desire to protect the nation. The radical anti-Zionist Israeli left has yet to come to power, and hopefully never will. Gerecht also
particularly cites the Netanyahu's family background, noting that his father was a famous scholar of oppressed Spanish Jewry,
and his brother, the only commando to die at Entebbe. Here is an implication that Netanyahu might be willing gamble on the
IAF if he truly believes Iran is near to producing a nuclear weapon. But the brother who is likely to have the most influence on
Netanyahu is not his fallen older brother Jonathan, but rather his younger brother Iddo, who has over the past decades devoted
much time and effort to detailing the circumstances of Jonathan Netanyahu's death at Entebbe, the results of which are
sobering. For the truth is, the military situation vis a vis Iran is in many ways more similar to Entebbe, than it is to the surgical
anti-nuclear strikes carried out by the IAF against Iraq and Syria. And the reasons for this are the issues of complexity and
surprise. An
Israeli attack on Iran would be an enormously complex undertaking, so much so,
that the actual point the attack, dropping bombs on Iran's nuclear facilities, is but one facet
of a gigantic political, diplomatic, logistic, technical, and operational problem . It is similar to the
situation faced at Entebbe, when the problem of getting a rescue force from Israel into the heart of Africa to a large extent
subsumed the actual goal of the raid -- rescuing the hostages. The truth about Entebbe, divorced from superficial accounts of
daring, heroism and Hollywood fantasy, is that the raid, which can legitimately be considered the boldest and most successful
hostage rescue in history, came very close to becoming a tragic failure. There is insufficient space here for a full detailed account
of the matter, but the actual rescue of the hostages was arguably the weakest part of the plan, and the portion of the operation
that came closest to failure. In summary, Entebbe occurred in 1976, when Western armies were still adjusting to the problem of
suicidal hostage takers, and sophisticated hostage rescue techniques were in their infancy. Many devices rescue forces take for
granted today, such as flash bang grenades or night vision devices, were unavailable. Success, even for the best troops, was a hit
and miss affair. Two years before Entebbe, at a high school in the northern Israeli town of Ma'alot, a botched IDF rescue attempt
resulted in scores of deaths and injuries. The Entebbe rescue plan sought to avoid another Ma'alot through the element of

surprise. It called for a thirty man sayeret matkal team (led by Lieutenant Colonel Netanyahu) to immediately drive off
the first aircraft to land at Entebbe in a black Mercedes and a pair of Land Rovers meant to imitate Ugandan President Idi Amin's
motorcade. The commandos themselves were crudely disguised in Ugandan style uniforms and blackface, and carried AK-47s
like the Ugandan army. The vehicles were to drive up to the doors of the terminal where the hostages were held, whereupon
the commandos were to leap out, rush the building and rescue the hostages before the terrorists knew what was happening.
But this was just one element of a much more complex plan, that also required three other transport aircraft to reach Entebbe
via a long dangerous flight route, land unobserved and unmolested, seize the airport, destroy Ugandan fighter planes, ambush
Ugandan reinforcements, guard the rescue aircraft, treat and evacuate casualties and rescued hostages, refuel the aircraft and
withdraw, all of which required 120 or so additional troops plus vehicles. There were of course, also multiple additional political,
diplomatic, command/control and logistic considerations. In the event, Colonel Netanyahu's rescue convoy was intercepted by a
pair of Ugandan soldiers several hundred meters from the terminal. The Israelis tried to kill both with small caliber silenced
pistols, but one soldier survived the assault and fled. Commandos gunned him down with un-silenced machine guns. Ugandan
soldiers then opened fire on the convoy as it moved out again. Netanyahu, fearing that the rescue team would be annihilated in
its thin skinned vehicles, ordered the commandos to abandon them and run to the terminal, still at least fifty meters away.
Some commandos fired back as they ran, emptying their ammo magazines. They arrived at the terminal disordered and
sheltered in the lee of the building, the plan a shambles. To add to the confusion, the terminal building did not match the mock-
up upon which they'd trained. The assault came to a stop. Netanyahu then stepped out into the open to urge on the attack and
was mortally wounded. At this point the rescue at Entebbe would seem to have failed. What saved it was the still overwhelming
effect of surprise, and a bit of individual courage and initiative. Inside the terminal the German and Palestinian terrorists had
been alarmed by the shooting and shouting outside, but were so certain that they were safe from an Israeli rescue attempt that
they attributed the commotion to in-fighting among the Ugandans, whom they held in low regard anyway. This over-confidence
had been deliberately fostered by the Israeli government, which prior to the raid had essentially admitted surrender, and agreed
(at that time contrary to Israeli practice) to negotiate with the terrorists. As the terrorists stood by, a few individual commandos
acted on their own initiative and stormed the building. They killed the terrorists and rescued the hostages. Netanyahu and
Barak are former commandos themselves, and when briefed by IAF commanders they will know the story of
Entebbe, and countless other operations, many from personal involvement. They will understand that anything
in a complex plan that can go wrong likely will. And they will also know that the one thing
that saved the day at Entebbe, the element of strategic surprise, will be absent in an
assault on Iran. The only surprise the Israelis can hope for in a strike against Iran is the precise date and time, and
considering the complexities of getting scores of aircraft through hostile airspace before even reaching Iranian skies, they might
not even have that. If the Israelis were serious about attacking Iran, the best thing they could do now is stop talking about it.
Indeed, ideally, the Israelis would appear accept the position that seems to be that of the United States under President Obama
-- that a nuclear Iran is inevitable and manageable. Then maybe they could lull the Iranian leadership and military into
complacency and hope to regain a bit of strategic surprise. But right now, with every eye trained on Israeli skies
and the world expectantly awaiting an Israeli assault, the chances of Israeli success must be
dramatically reduced, a fact not lost on Netanyahu and Barak. I don't pretend to know what Israel will
do, and nobody would be happier to see a successful Israeli strike on Iran than me, but logic suggests that if the
Israelis haven't done it yet, they probably never will. And Benjamin Netanyahu is no more likely
to launch an attack than his predecessors, for the same set of complex reasons that they
were restrained.

No retaliation, war, or regional conflict

Bronner, ‘12 (Ethan, NYT staff reporter, 1/26/12, http://www.nytimes.com/2012/01/27/world/middleeast/israelis-see-irans-


threats-of-retaliation-as-bluff.html?_r=1&hp)

JERUSALEM — Israeli intelligence estimates, backed by academic studies, have cast doubt on the
widespread assumption that a military strike on Iranian nuclear facilities would set off a
catastrophic set of events like a regional conflagration, widespread acts of terrorism and sky-
high oil prices. Prime Minister Benjamin Netanyahu has said he thinks Iranian citizens will welcome an attack. The
estimates, which have been largely adopted by the country’s most senior officials, conclude that
the threat of Iranian retaliation is partly bluff . They are playing an important role in Israel’s
calculation of whether ultimately to strike Iran, or to try to persuade the United States to do so,
even as Tehran faces tough new economic sanctions from the West. “A war is no picnic,” Defense Minister
Ehud Barak told Israel Radio in November. But if Israel feels itself forced into action, the retaliation would
be bearable, he said. “There will not be 100,000 dead or 10,000 dead or 1,000 dead. The state of
Israel will not be destroyed.” The Iranian government, which says its nuclear program is for civilian purposes,
has threatened to close the Strait of Hormuz — through which 90 percent of gulf oil passes — and if attacked, to
retaliate with all its military might. But Israeli assessments reject the threats as overblown. Mr. Barak and Prime
Minister Benjamin Netanyahu have embraced those analyses as they focus on how to stop what they view as Iran’s determination to
obtain nuclear weapons. No issue in Israel is more fraught than the debate over the wisdom and feasibility of a strike on Iran.
Some argue that even a successful military strike would do no more than delay any Iranian
nuclear weapons program, and perhaps increase Iran’s determination to acquire the capability.
Security officials are increasingly kept from journalists or barred from discussing Iran. Much of the public talk is as much
message delivery as actual policy. With the region in turmoil and the Europeans having agreed to harsh sanctions
against Iran, strategic assessments can quickly lose their currency. “They’re like cartons of milk — check the sell-by date,” one senior
official said. But conversations with eight current and recent top Israeli security officials suggested several things: since Israel has
been demanding the new sanctions, including an oil embargo and seizure of Iran’s Central Bank assets, it will give the sanctions
some months to work; the sanctions are viewed here as probably insufficient; a military attack remains a very real option; and
postattack situations are considered less perilous than one in which Iran has nuclear weapons. “Take every scenario of confrontation
and attack by Iran and its proxies and then ask yourself, ‘How would it look if they had a nuclear weapon?’ ” a senior official said. “In
nearly every scenario, the situation looks worse.” The core analysis is based on an examination of Iran’s interests and abilities, along
with recent threats and conflicts. Before the United States-led war against Iraq in 1991, Saddam Hussein vowed that if attacked he
would “burn half of Israel.” He fired about 40 Scud missiles at Israel, which did limited damage. Similar fears of retaliation were
voiced before the Iraq war in 2003 and in 2006, during Israel’s war against Hezbollah in southern Lebanon. In the latter, about 4,000
rockets were fired at Israel by Hezbollah, most of them causing limited harm. “If you put all those retaliations together and add in
the terrorism of recent years, we are probably facing some multiple of that,” a retired official said, speaking on the condition of
anonymity, citing an internal study. “I’m not saying Iran will not react. But it will be nothing like London
during World War II.” A paper soon to be published by the Institute for National Security Studies at Tel Aviv University,
written by Amos Yadlin, former chief of military intelligence, and Yoel Guzansky, who headed the Iran desk at Israel’s National
Security Council until 2009, argues that the Iranian threat to close the Strait of Hormuz is largely a bluff. The paper contends that,
despite the risks of Iranian provocation, Iranwould not be able to close the waterway for any length of
time and that it would not be in Iran’s own interest to do so. “If others are closing the taps on
you, why close your own?” Mr. Guzansky said. Sealing the strait could also lead to all-out
confrontation with the United States, something the authors say they believe Iran wants to avoid. A
separate paper just published by the Begin-Sadat Center for Strategic Studies says that the fear of missile warfare against
Israel is exaggerated since the missiles would be able to inflict only limited physical damage. Most
Israeli analysts, like most officials and analysts abroad, reject these arguments. They say that Iran has been preparing for an attack
for some years and will react robustly, as will its allies, Hezbollah and Hamas. Moreover, they say, an attack will at best delay the
Iranian program by a couple of years and lead Tehran to redouble its efforts to build such a weapon. But Mr. Barak and Mr.
Netanyahu believe that those concerns will pale if Iran does get a nuclear weapon. This was a point
made in a public forum in Jerusalem this week by Maj. Gen. Amir Eshel, chief of the army’s planning division. Speaking of the former
leaders of Libya and Iraq, he said, “Who would have dared deal with Qaddafi or Saddam Hussein if they had a nuclear capability? No
way.”
---Iran Israel Strikes- No Strikes

Israel won’t attack – relations with the US

Cook ‘9 (Cook, senior fellow, Mid East studies – CFR, 6/9/’9


(Steven A, “Why Israel Won’t Attack Iran,” Foreign Affairs)

Given Israel's perception of an acute Iranian threat and its demonstrated ability to act alone, there
must be some other factor holding the Israelis back. Most likely, that factor is politics, and more
specifically, the importance that close relations with Washington has on the domestic political
calculations of Israeli leaders. Unlike 1981, when the United States had barely a toe-hold in the Middle East, Washington
occupies two countries in or adjacent to the region, maintains military facilities throughout the Persian Gulf, and
relies on Arab governments for logistical support. In the event of an Israeli attack, Washington would surely be
accused of colluding with Jerusalem, severely damaging the United States' position in the region while
provoking a ferocious Iranian response in Iraq, Afghanistan, Gaza, and southern Lebanon. The resulting
breach between Israel and the United States would be unprecedented, creating a crisis far more serious than
President Dwight Eisenhower's demand that Israel stand down after its invasion of Sinai in 1956 and Gerald Ford's "reassessment" of
1975 (which suspended all military and economic agreements between the two countries for three months when Israel proved
uncooperative in negotiating a second Sinai agreement). This is a scenario with which many Israelis, including
Netanyahu, are unlikely to be comfortable. The Israelis have always claimed that they did not want a formal defense
treaty with the United States for fear that such a pact would limit their freedom of maneuver. David Ben Gurion sought close
relations with Washington, but not at the expense of Israel's "independence or its existence." Yet, the historical record does not
track consistently with Ben Gurion's bravado. The 1956
and 1975 episodes are instructive because the Israelis
backed down, establishing an informal pattern for future relations in which Israeli prime
ministers tend to tread cautiously when it comes to the United States.

Israel won’t first strike - second strike capabilities and perception of Iran as rational check

Parsi ‘6 (Trita Parsi, Ph.D. in International Relations from Johns Hopkins University School of Advanced International Studies,
“What's to fear? A challenge to Israel's strategic primacy”, www.iranian.com/Parsi/2006/January/Nuclear/index.html, January 5,
2006,

What lies at the heart of Israel'scampaign to halt Iran's nuclear advances, however, is not necessarily the
fear of a nuclear clash, but the regional and strategic consequences nuclear technology parity in the Middle East will
have for Israel. In spite of its rhetoric, Israel views the regime in Tehran as rational (but
extremist), calculating and risk-averse. Even those Israeli officials who believe that Iran is
hell-bent on destroying the Jewish state recognize that Tehran is unlikely to attack Israel
with nuclear weapons due to the destruction Israel would inflict on Iran through its
second-strike capability. With its nuclear-equipped submarines, Israel has a guaranteed
second-strike capability. "Whatever measure [the Iranians] have, they can't destroy Israel's capability to respond,"
Ranaan Gissin, spokesperson for Israel's prime minister, told me.

Israel will not first strike

Sadr 5 – PhD Candidate @ Univ Maryland in Poli Sci (El, The Impact of Iran’s Nuclearization on Israel, Middle East Policy 12.2,
Wiley Interscience)

Despite Israel’s strategically understandable claims regarding its ability to preempt Iranian
nuclearization through unilateral military action, many experts foresee complications that make a
singlestrike success extremely difficult if not impossible .5 Furthermore, even if Iran’s nuclear
capabilities were eliminated or significantly impaired in such an attack, the military and political
ramifications for Israel and its allies are likely to be much more costly than they were in 1981.6 Thus, it is
necessary to consider whether the costs and opportunities associated with the emergence of a nuclear Iran might not be
more tolerable than those associated with military action.
Iran-Saudi War Answers
No Saudi/Iran war

Kaye 10—Senior political scientist, RAND. CFR member and former prof at George Wash. PhD in pol sci from UC Berkeley—
AND—Frederic Wehrey—Senior analyst at RAND. Former Georgetown prof. D.Phil. candidate in IR, Oxford. Master’s in near
Eastern studies, Princeton—AND—Jeffrey Martini—Middle East research project associate at RAND. Master’s in Arab studies at
Georgetown (Dalia Dassa, The Iraq Effect, Report Prepared for the Air Force,
http://www.rand.org/pubs/monographs/2010/RAND_MG892.pdf)

The result is that Iran’s Arab neighbors, particularly the smaller GCC states, have pursued
a mixed approach
toward Tehran that incorporates some elements of accommodation and engagement, regardless of
warnings from Riyadh and Washington. And indeed, even the Saudis themselves have adopted such an
approach; the regional shakeup in the aftermath of the Iraq War has not fundamentally altered the
Saudis’ rapprochement with Tehran, which dates to the mid- 1990s. When necessary, the
two states have shown the propensity for pragmatic cooperation in specific geographic areas and on
issues where their interests intersect—even if, in other areas, there is concurrently open rivalry. As one analyst of Saudi Arabia
argues, the
United States may want the Saudis to head an anti-Iranian alliance, but they will not
do this, preferring to act as mediators instead . 53 Ahmadinejad’s invitation and attendance at
the December 2007 GCC summit was a particularly public expression of the mixed approach toward
Iran among the Arab Gulf states and underscores the obstacles U.S. attempts to solidify an anti-Iranian stance
among Iran’s neighbors face.

No war

Alexander ’11 ( Alexander, 10-10-2011 Vatutin Alexander, political analyst, 10-10, “ Saudi Arabia-Iran tensions posing threat to
global oil market,” http://english.ruvr.ru/2011/10/10/58461134.html

Trying to predict the consequences of confrontation between Saudi Arabia and Iran, experts rule out the use
of force but agree that tensions will continue to grow in the region. Military expert Vladimir Anokhin continues: "Iran has been
seeking leadership in the Islamic world. Meanwhile, this status belongs to Saudi Arabia, which certainly stirs up conflict. However,
Iran will hardly let this result in a military campaign because the Saudis are strong enough to
attract the entire Arab world to their side in a war against Iran. They can boast a high level of authority, which is
exactly what the West expects from them." Mr. Anokhin thinks that neither Iran nor Saudi Arabia is interested in
military confrontation. Otherwise it might affect oil exports, which will be a serious blow to the House of
Saud. Apart from this, Mecca and Medina, the holiest places for all Muslims, attract billions into the country’s budget each year
during the Hajj. As far as Iran is concerned, it risks becoming a rogue state because of its nuclear ambitions. In its recent statements,
Riyadh said it was going to seek nukes to have a mechanism of deterrence against Tehran. Saudi Arabia even discussed the issue
with Pakistan. One should not forget that security of Saudi Arabia is also guarded by the US and NATO, which will never allow to
interrupt oil supplies from the Arabian Peninsula. King Abdullah of Saudi Arabia is not a Mideast leader the West could sacrifice like
Hosni Mubarak in Egypt, Muammar Gaddafi in Libya or Bashar Assad in Syria. Expert Sergei Demidenko believes that a part of
criticism is the only thing the King of Saudi Arabia might face: "The
Saudis are not ready to join a war. They
simply do not know how to do it. Secondly, their responsibility is a matter of concern for the US
and NATO. It means that if Iran attempts to escalate tensions further, it will have to deal with the
alliance."
US deters

Barzegar, ‘11
Kayhan Barzegar, 4-20-2011, Faculty Member, Department of International Relations, Science and Research University, Tehran,
Iran, Former Associate, Project on Managing the Atom/International Security Program, 2010–2011; Former Research Fellow, Project
on Managing the Atom/international Security Program, 2007–2010, "Iran's Interests and Values and the 'Arab Spring'"

http://belfercenter.ksg.harvard.edu/publication/20954/irans_interests_and_values_and_the_arab_spring.html

Iran will not initiate any military expedition in the Persian Gulf in the near future, because this policy
would fortify the adversarial and entrenched strategy of the United States and Israel, which represents Iran
as the main source of regional instability. Such a policy will become even more precarious when it relates
to Iran's nuclear program. It is highly unlikely that Iran will make the same mistake as Saudi
Arabia did with Bahrain, since this policy would increase political-security divisions in the region and
provide an ideal justification for foreign forces by legitimizing their continued presence in the Persian Gulf.

They’ll mess with the Saudis in Lebanon instead

Jayedanfar ‘11
Meir Javedanfar, Iranian-Israeli Middle East analyst, 3-24-2011, “Iran and Saundi Arabia cold war has entered a new era,” The
Guardian, http://www.guardian.co.uk/commentisfree/2011/mar/24/iran-saudi-arabia-bahrain-cold-war

It is unlikely that Iran would send its forces across the Gulf into Bahrain. This move would be
very risky, both politically and militarily.

Instead, the Iranians will most probably to continue to focus on undermining the Saudis in
places such as Lebanon. In fact, Iran's own actions there also fanned the flames of the cold war, especially its support for
Hezbollah, which in 2008 attacked local Sunnis, leaving 11 dead and 30 wounded. This infuriated the Saudis, who are close to
Lebanon's Sunni community and have backed them. Hezbollah's show of force boosted its leverage significantly, and thus helped it
acquire its much-desired veto in Lebanon's cabinet.

No escalation.

Gharekhan ’11 (4/28/2011 (Chinmaya R., The new great game in West Asia?, The Hindu, p.
http://www.thehindu.com/opinion/lead/article1774074.ece?homepage=true)

In the years immediately following its attack on Iraq, the U.S. tried to cobble together a coalition of ‘moderate'
Sunni states to contain Iran's growing influence in the region. Israel could not obviously be a part of this grouping
but it fully supported the effort. During the visits by this writer to the countries in the region, it was made clear to him that
Iraq's neighbours would not remain silent and inactive if the Sunnis there came under serious
danger. The situation did not escalate to that level; neither of the regional powers wanted to
risk war .The interesting point is that it is Iran, the lone Shia superpower which does not have
the economic clout of Saudi Arabia, which has adopted an aggressive posture whereas the Sunni states seem to be on
the defensive. Iran feels isolated, encircled and threatened by hostile American forces as well as by what it might perceive as
antagonistic Sunni states. It is this which perhaps makes the Iranian regime more motivated and forceful in its diplomacy and
actions. The feeble attempts by the Americans to discourage Saudi Arabia from sending its troops into Bahrain not only did not
succeed but also led the Saudis to the conclusion that they must be on their own when it came to defending their regime and
checking Iran's growing influence. If Bahrain's Shias succeed in gaining a share in the power structure, the Saudis will feel truly
threatened, given that its Shia community, accounting for about 10 per cent of the population, is concentrated in its eastern
territory where its oil assets are located. Any prospect of Iranian influence on the mainland of Saudi Arabia will be a nightmare to its
ruling dynasty.It is perhaps too late to soften the Shia-Sunni, Iran-Saudi tensions. Even if the Sunni-ruled states satisfy the demands
of their Shia populations to some extent, Iran will continue to press home the advantage that has come its way recently, consolidate
and build on it. The Americans will certainly not watch this game passively.

No risk of war—neither side wants it, empirically denied, no regional leaders

Stratfor ‘6 Analysts, future seers “What Prevents a Broad Sunni-Shiite Conflict” 7/27/2006
http://www.stratfor.com/what_prevents_broad_sunni_shiite_conflict

Relations between the Iranians and the Saudis have had their ups and downs — but even the
killing by Saudi security forces of some 400 Iranian-backed Shiite pilgrims demonstrating
during the Hajj in Mecca in the late 1980s did not spark a major conflict between the two. Ultimately,
neither side wants to fight the other . This is because both Tehran and Riyadh know the
structural limits they face with regard to taking each other on geopolitically. Each side knows
the other wants its place in the region. Riyadh knows that Iran, and the Shia in the Arab world — especially those
in Saudi Arabia’s oil-rich Eastern province — constitute a potential geopolitical threat that cannot be altogether neutralized, and
hence requires management. Similarly, Iran
knows it can go only so far in claiming the leadership
mantle of the Arab and Muslim world because of its Persian and Shiite credentials, which
mean Tehran will never be in a position to impose a settlement in the region.
Iraq Answers
Frontline

No Iraq impact

Maloney 7 –Senior Fellow, Foreign Policy, Saban Center for Middle East Policy. Suzanne 2007. “Why the Iraq War
Won’t Engulf the Mideast”. http://www.brookings.edu/opinions/2007/0628iraq_maloney.aspx

Long before the Bush administration began selling "the surge" in Iraq as a way to avert a general war in the Middle East,
observers both inside and outside the government were growing concerned about the potential for armed conflict among
the regional powers. Underlying this
anxiety was a scenario in which Iraq's sectarian and
ethnic violence spills over into neighboring countries, producing conflicts between the major
Arab states and Iran as well as Turkey and the Kurdistan Regional Government. These wars then destabilize the entire
region well beyond the current conflict zone, involving heavyweights like Egypt. This is scary stuff indeed, but with the
exception of the conflict between Turkey and the Kurds, the scenario is far from an accurate reflection of the way Middle
Eastern leaders view the situation in Iraq and calculate their interests there. It
is abundantly clear that major
outside powers like Saudi Arabia, Iran and Turkey are heavily involved in Iraq. These countries
have so much at stake in the future of Iraq that it is natural they would seek to influence political developments in the
country. Yet, the Saudis, Iranians, Jordanians, Syrians, and others are very unlikely to go to
war either to protect their own sect or ethnic group or to prevent one country from
gaining the upper hand in Iraq. The reasons are fairly straightforward. First, Middle Eastern leaders,
like politicians everywhere, are primarily interested in one thing: self-preservation. Committing
forces to Iraq is an inherently risky proposition , which, if the conflict went badly, could threaten domestic
political stability. Moreover, most Arab armies are geared toward regime protection rather than projecting power and thus
have little capability for sending troops to Iraq. Second,
there is cause for concern about the so-called
blowback scenario in which jihadis returning from Iraq destabilize their home countries,
plunging the region into conflict. Middle Eastern leaders are preparing for this possibility. Unlike in the 1990s,
when Arab fighters in the Afghan jihad against the Soviet Union returned to Algeria, Egypt and Saudi Arabia and became a
source of instability, Arab security services are being vigilant about who is coming in and going from their countries. In the
last month, the Saudi government has arrested approximately 200 people suspected of ties with militants. Riyadh is also
building a 700 kilometer wall along part of its frontier with Iraq in order to keep militants out of the kingdom. Finally,
there is no precedent for Arab leaders to commit forces to conflicts in which they are not
directly involved. The Iraqis and the Saudis did send small contingents to fight the Israelis in 1948 and 1967, but they
were either ineffective or never made it. In the 1970s and 1980s, Arab countries other than Syria, which had a compelling
interest in establishing its hegemony over Lebanon, never committed forces either to protect the Lebanese from the
Israelis or from other Lebanese. The civil war in Lebanon was regarded as someone else's fight. Indeed, this is the way
many leaders view the current situation in Iraq. To Cairo, Amman and Riyadh, the situation in Iraq is worrisome, but in the
end it is an Iraqi and American fight. As far as Iranian mullahs are concerned, they have long preferred to press their
interests through proxies as opposed to direct engagement. At a time when Tehran has access and influence over powerful
Shiite militias, a massive cross-border incursion is both unlikely and unnecessary. So Iraqis will remain locked in a sectarian
and ethnic struggle that outside powers may abet, but will remain within the borders of Iraq. The Middle East is a region
both prone and accustomed to civil wars. But given
its experience with ambiguous conflicts, the
region has also developed an intuitive ability to contain its civil strife and prevent local
conflicts from enveloping the entire Middle East. Iraq's civil war is the latest tragedy of this hapless
region, but still a tragedy whose consequences are likely to be less severe than both supporters and opponents of Bush's
war profess.

Tons of structural alt causes to stability

Cordesman ‘12 – Arleigh A. Burke Chair in Strategy at the Center for Strategic and International Studies (Anthony H., Sam
Khazai, “Patterns of Violence in Iraq,” https://csis.org/files/publication/121024_Iraq_Violence.pdf)
There are deep structural problems in Iraq’s demographics and economy that also encourage violence in
Iraq. Factors compounding these risks include high population growth, serious problems in
employment and the agriculture sector, and a per capita income that only ranks 162nd in the world in spite of record
oil revenue.5 So do abuses of human rights. The Department of State Annual Report on Human Rights Practices for 2011 states that
Iraq faced significant human rights problems over the course of the past year: 6 “During the year the most significant human rights
developments were continuing abuses by sectarian and ethnic armed groups and violations by government-affiliated forces.
Divisions between Shia and Sunni and between Arab and Kurd empowered sectarian militant
organizations. These militants, purporting to defend one group through acts of intimidation and revenge against another,
influenced political outcomes. Terrorist attacks designed to weaken the government and deepen societal divisions occurred during
the year. The three most important human rights problems in the country were governmental and societal violence reflecting a
precarious security situation, a fractionalized population mirroring deep divisions exacerbated by Saddam Hussein’s legacy, and
rampant corruption at all levels of government and society.” Iraq’s political and economic challenges now dominate both its internal
politics and relations with the US, Iran, and Iraq’s other neighbors. Iraq needs trade and cross-border support from Iran, just as it
needs aid, diplomatic, and military support from the US. Iraq’s much-reduced military capabilities make it dependent on aid, military
sales, and training from the United States, and Iraq still lacks the resources and cohesion to resist against Iranian coercion
and to defend against Iranian aggression. Moreover, Iraq’s economy remains crippled by a lack of local security in
many areas, and it is important to understand just how serious its other problems are. Iraq has a level of corruption that
Transparency International ranked 175th out of 183 countries in 2011 – making it the eighth most corrupt country in the world.7 In
spite of more than half a decade of faltering legislative efforts, Iraq
has failed to pass effective investment, tax, and property
laws to secure both domestic and foreign investment as well as to create effective security forces to protect its
infrastructure and businesses. Governance and a failure to properly encourage development remains a major problem. A budget
crisis that lasted from 2008 to 2010, and a political crisis that began long before the March 2010 election that produced a
de facto stalemate in many aspects of governance, have added to these economic problems as well as sharply delayed
critical qualitative improvements in every branch of Iraq’s national security forces. Iraq has not
been able to absorb and support many of the aid projects funded during the US occupation, and its problems in national governance
have been compounded by corruption, political infighting, and sectarian and ethnic struggles at the provincial and local levels.
---Iraq- No Impact

No impact to Iraq – outside powers are more interested in self-preservation than getting
involved – they have no incentive, and conflicts are inherently localized – that’s Maloney

No spillover

Wehrey 10 [Frederic et al, Senior Policy Analyst for Rand Corporation, 1/, Dalia D Kaye, Jessica Watkins, Jeffrey Martini,
Robert A Guffey, “The Iraq Effect: The Middle East After the Iraq War,”

http://www.ncci-library.org/jspui/bitstream/123456789/445/1/RAND%20-%20The%20Iraq%20Effect%20-%20The%20Middle
%20East%20After%20the%20Iraq%20War%20(2010).pdf ]

To be sure, whether Iraq “succeeds” (i.e., continues on its current trajectory of reduced violence and some
degree of political reconciliation) or “fails” (i.e., returns to widespread sectarian or ethnic violence and
instability) will greatly affect the long-term position and prospects of the Iraqi state . But while
regional actors are by no means insulated from such developments, regional trend lines are unlikely to shift
significantly in response to internal Iraqi outcomes . For example, renewed violence in Iraq and massive
repression and exclusion of the Sunni minority would no doubt anger Sunni Arab regimes and publics and would undermine
Iran’s outreach efforts to the broader region. But Iran’s regional influence does not depend just on its
leverage in Iraq, which, even under the best of circumstances, would still face resistance
because of Iraqi nationalist sentiment. Even in the event of failure in Iraq, Iran is likely to
continue its pursuit of other regional levers of influence that are of greater concern to its
Arab neighbors, such as its ties to militant groups fighting Israel, as well as its pursuit of
nuclear capabilities. Indeed, such levers would prove valuable to any type of Iranian leadership, but they are certainly
valuable to hard-liners, who are attempting to consolidate power after the contested 2009 elections. Or, on the other hand, if
the United States successfully withdraws from Iraq, leaving it with some level of stability, its
improved regional credibility is not likely to deter regional states from continuing to pursue
a hedging strategy with respect to Iran and to diversify extraregional security relationships by developing closer
ties to such states as China and Russia.
---Iraq- Stability Alt Causes

Those outweigh and undermine their internal link

Cordesman ‘12 – Arleigh A. Burke Chair in Strategy at the Center for Strategic and International Studies (Anthony H., Sam
Khazai, “Patterns of Violence in Iraq,” https://csis.org/files/publication/121024_Iraq_Violence.pdf)

The reality, however, is that Iraqi violence remains high and may well be increasing. The
structural causes of violence
remain a strong as ever, Iraq lacks unified and effective leadership, and sectarian and ethnic
divisions remain a critical problem. The security forces are not becoming more effective, are
corrupt, and are divided. Moreover, US-Iranian strategic competition over the future of Iraq is a further
source of division and violence, and continues to undermine and challenge US interests throughout the
region.

Also empirically disproves the impact

Cordesman ‘12 – Arleigh A. Burke Chair in Strategy at the Center for Strategic and International Studies (Anthony H., Sam
Khazai, “Patterns of Violence in Iraq,” https://csis.org/files/publication/121024_Iraq_Violence.pdf)

The overall mix of ethnic,


sectarian, and tribal tensions causes a civil war from 2005 to 2008, and
has remained a source of violence ever since. Figures 6 and 7 illustrate the level of ethnic and sectarian divisions
that help shape Iraq’s violence. The stalled implementation of the 2010 Erbil power-sharing agreements is just the latest in a series
of power struggles that have now escalated to the point where Iraq’s Vice President faces the death penalty and some Sunnis say
Iraq’s Prime Minister is seeking to become a dictator. Internal violence from extremist groups is rising again, and the NCTC reports
that peace in Iraq is close to the casualty levels of “war” in Afghanistan. Pressures from Iran and the
power struggle for the future of Syria add new tensions on Iran’s borders. Iraq’s structural domestic divisions also
interact with the broader patterns of instability in the region. Iraq is caught up in the political struggles
between the US, Arab states, and Iran. It is a key focus of the competition between the US and Iran, but also between Iran and the
Southern Gulf states. It is caught up in the civil conflict in Syria, and the broader struggles between Sunni and Shi’ite that now affect
much of the Islamic world.
---Iraq- Drones Answes

Drones definitely don’t solve diplomacy

Schmitt and Schmidt 12 (Eric, senior writer who covers terrorism and national security issues for The New York
Times, Michael S., Baghdad writer for the New York Times, “U.S. Drones Patrolling Its Skies Provoke Outrage in Iraq,”
http://www.nytimes.com/2012/01/30/world/middleeast/iraq-is-angered-by-us-drones-patrolling-its-skies.html?_r=1)

BAGHDAD — A month after the last American troops left Iraq, the State Department is operating a
small fleet of surveillance drones here to help protect the United States Embassy and consulates, as well as American personnel.
Some senior Iraqi officials expressed outrage at the program, saying the unarmed aircraft are an
affront to Iraqi sovereignty. The program was described by the department’s diplomatic security branch in a little-noticed
section of its most recent annual report and outlined in broad terms in a two-page online prospectus for companies that might bid
on a contract to manage the program. It foreshadows a possible expansion of unmanned drone operations into the diplomatic arm
of the American government; until now they have been mainly the province of the Pentagon and the Central Intelligence Agency.

The U.S. will withdraw all of its drones from Iraq now

Pegg 11 – Bureau of Investigative Journalism (David, 12/14, “Analysis: After eight years in Iraq’s skies where now for US
drones?” http://www.thebureauinvestigates.com/2011/12/14/analysis-where-us-drones-are-going-after-iraq/)

The flag of the United States military was lowered in Baghdad today during a ceremony to finally mark the
end of operations in Iraq. Defense Secretary Leon Panetta told troops they could be ‘secure in knowing that your sacrifice
has helped the Iraqi people to cast tyranny aside’. Yesterday, at a speech to returning troops in North Carolina, President Barack
Obama hailed the ‘moment of success’ and told soldiers they were departing with their ‘heads held high’. By the end of the year,
the US will have withdrawn the remainder of its forces from Iraq after eight years of combat operations. This
will include its armada of combat and surveillance drones – a fleet that has transformed during the occupation
and US presence.

More ev – Iraq doesn’t differentiate between surveillance drones and attack drones

Schmitt and Schmidt 12 (Eric, senior writer who covers terrorism and national security issues for The New York
Times, Michael S., Baghdad writer for the New York Times, “U.S. Drones Patrolling Its Skies Provoke Outrage in Iraq,”
http://www.nytimes.com/2012/01/30/world/middleeast/iraq-is-angered-by-us-drones-patrolling-its-skies.html?_r=1)

The American plans to use drones in the air over Iraq have also created yet another tricky issue for
the two countries, as Iraq continues to assert its sovereignty after the nearly nine-year occupation. Many
Iraqis remain deeply skeptical of the United States, feelings that were reinforced last week when the Marine who was
the so-called ringleader of the 2005 massacre of 24 Iraqis in the village of Haditha avoided prison time and was sentenced to a
reduction in rank. “If they are afraid about their diplomats being attacked in Iraq, then they can take them out of the country,” said
Mohammed Ghaleb Nasser, 57, an engineer from the northern city of Mosul. Hisham Mohammed Salah, 37, an Internet cafe owner
in Mosul, said he did not differentiate between surveillance drones and the ones that fire missiles.
“We hear from time to time that drone aircraft have killed half a village in Pakistan and Afghanistan
under the pretext of pursuing terrorists,” Mr. Salah said. “Our fear is that will happen in Iraq under a different
pretext.”
Latin America Instability Answers
Diplomacy and cooperation prevent escalation
Jorge Heine 12, Chair in Global Governance at the Balsillie School of International Affairs, is
Professor of Political Science at Wilfrid Laurier University, 10/26/12, “Regional Integration and
Political Cooperation in Latin America,”
http://lasa.international.pitt.edu/LARR/prot/fulltext/vol47no3/47-3_209-217_heine.pdf

Despite this fragmented picture of overlapping acronyms, schemes, and interests, there is little
doubt that the forces of convergence have prevailed over those of divergence. The launch of
the Latin American and Caribbean Community of Nations in 2010 is proof of this. Mexico,
Chile, and Colombia are as much members of this body as are Venezuela, Bolivia, and Ecuador.
Collective diplomacy, political cooperation , and a regional vision are very much the order of
the day , transcending ideological differences. As may be seen in its reaction to the coup in
Honduras in June 2009, a lack of understanding of this strong multilateral component in the
foreign policies of Latin American nations lies at the root of the difficulties that the
administration of US president Barack Obama has faced in the region, despite the enormous
expectations raised there by his election.12 By imposing a unilateral solution that in effect
condoned the coup, against the express wishes of the OAS and the overwhelming majority of
Latin American governments, the United States squandered its infl uence in Latin America. Inter-
American relations have gone downhill ever since, with the US ambassadors to Ecuador and to
Mexico being forced to leave their posts in quick succession in 2011.

Their impact is empirically denied—no escalation

Hartzell 2000 (Caroline A., 4/1/2000, Middle Atlantic Council of Latin American Studies Latin American Essays, “Latin America's
civil wars: conflict resolution and institutional change.” http://www.accessmylibrary.com/coms2/summary_0286-28765765_ITM)

Latin America has been the site of fourteen civil wars during the post-World War II era, thirteen of which
now have ended. Although not as civil war-prone as some other areas of the world, Latin America has endured some
extremely violent and destabilizing intrastate conflicts. (2) The region's experiences with civil wars and their
resolution thus may prove instructive for other parts of the world in which such conflicts continue to
rage. By examining Latin America's civil wars in some depth not only might we better understand the circumstances under which
such conflicts are ended but also the institutional outcomes to which they give rise. More specifically, this paper focuses on the
following central questions regarding Latin America's civil wars: Has the resolution of these conflicts produced significant
institutional change in the countries in which they were fought? What is the nature of the institutional change that has taken place
in the wake of these civil wars? What are the factors that are responsible for shaping post-war institutional change?

Instability’s inevitable—drug trafficking

Grudgings ‘9 (Stuart, Rueters, Latin America ex-leaders urge reform of US drug war,
http://www.reuters.com/article/latestCrisis/idUSN11358345)
RIO DE JANEIRO, Feb 11 (Reuters) - Thewar against drugs is failing and the U.S. government should break with
"prohibition" policies that have achieved little more than cram its prisons and stoke violence, three former
Latin American presidents said on Wednesday. The respected former presidents urged the United
States and Latin American governments to move away from jailing drug users to debate the
legalization of marijuana and place more emphasis on the treatment of addicts . Former Colombian
President Cesar Gaviria said there was no meaningful debate over drugs policy in the United States, despite a broad consensus that
current policies had failed. "The problem today in the U.S. is that narco-trafficking is a crime and so any politician is fearful of talking
about narco-trafficking or talking about policies because they will be called soft," he said. Gaviria has joined with former Brazilian
President Fernando Henrique Cardoso and former Mexican President Ernesto Zedillo to try to change the debate on drugs in Latin
America, where trafficking
gangs have killed tens of thousands of people and weakened democracies
through corruption. From Mexico's gang wars to the drug-funded FARC guerrilla group in
Colombia and daily shoot-outs between gangs and police in Rio de Janeiro's shantytowns, much
of the region is scarred by drug violence and many believe U.S. policies have failed. A United Nations
meeting in Vienna next month will frame international drugs policy for the next 10 years, and the three former presidents, whose
group is called the Latin American Commission on Drugs and Democracy, said it is time for change. They pointed to falling street
prices for cocaine and still high levels of consumption in the United States despite decades of policies focused on punishing users
and cutting supplies from Latin American countries such as Colombia. 'PREJUDICES, FEARS' The presidents' commission released a
report calling on governments to refocus policies toward treating users, move toward decriminalizing marijuana, and invest more in
education campaigns. It said current policies were rooted in "prejudices, fears and ideological visions" that inhibited debate. Even as
the group met in Rio on Wednesday, police arrested 51 people in a major operation in the city and other states against a suspected
drug smuggling ring that sent cocaine to Europe and brought back synthetic drugs like Ecstasy. Organized crime has
flourished around drugs and is now threatening the stability of Mexico, where a spiraling war
between rival gangs killed more than 5,700 people last year . Cardoso, one of Latin America's most respected
figures, said U.S. leadership was essential to break the cycle of drug-related crime and violence. " It will be almost
impossible to solve Mexico's problems and other countries' problems without a more ample,
comprehensive set of policies from the U.S . government," he said. Despite winning power on broad promises
of change, drugs policy featured little in U.S. President Barack Obama's election campaign and there are few indications that he will
embark on a major overhaul. Gaviria said Washington appeared increasingly isolated in its repressive approach as Latin America and
Europe move toward treating drug abuse as a health problem rather than a crime. (Editing by Raymond Colitt and Kieran Murray)
Leading from Behind Answers
Leading from Behind is inevitable – Libya approach is now US paradigm.

Brady 11. (Kyle, author of Modern America: The End of Political Discourse, “Libya shows multilateralism is new U.S. strategy”,
Policymic -- August -- http://www.policymic.com/articles/libya-shows-multilateralism-is-new-u-s-strategy)

The involvement of the United States in the Libyan conflict, justified through humanitarian concerns, has been a hallmark of the
Obama administration’s approach to foreign policy and international relations, tentatively characterized as a multilateral, reluctantly
interventionist, and respectfully protectionist. Some have already gone so far as to label this tendency to involve other powers, the
insistence on diplomacy and a (strongly misleading) deferential appearance as the "Obama Doctrine." However, this is not
just the doctrine of a single president, it is also the foundation of post-Libya U.S. foreign policy that will
continue without President Barack Obama: American multilateralism. While the apparent success of Libya is not
necessarily indicative of NATO success, or even a replicable effort to be applied to future conflicts, it does represent a
turning point in the attitude of the United States. Specifically, this post-Libya approach to foreign policy
is one that emphasizes diplomacy and multilateralism, a consideration of humanitarian and foreign interests and
a desire for cooperation. However, these actions are taken through the lens of U.S. interests , a key concept in
American multilateralism. For example, the lack of Central American military involvement in the Libyan civil war, or the desire for UN
approval, is not at all indicative of a withdrawal of U.S. power. America
is moving towards a position wherein the
power of other states is recognized, but not deferred to . In this position, the U.S. is more than willing
to allow others to bear the costs of an engagement effort , military or otherwise, so long as goals
remain in alignment — the U.S. may be a global police force, but it is no longer one that overtly demonstrates its power.
Rather than placing itself in the center of every issue, American multilateralism calls for the U.S. to be the
influence in the shadows that brokers deals in its own interest without necessarily becoming
directly involved. It is a more covert and subtle approach that misleadingly appears to be disinterested, deferential and weak.
Evidence for this development can be found in various arenas since the inauguration of Obama, including an insistence on a strictly
civilian Egyptian revolution, the demonstration of air power in Libya only on request and the diplomatic approach to Syria that has
allowed Turkey to lead as a growing, influential regional. Another year remains in Obama’s first term, with a possible second term
looming, and there is no doubt that the apparent success of American foreign policy in Libya will strengthen Obama’s resolve,
solidifying the post-Libya approach of American multilateralism into a more substantial, lasting
paradigm that will have ample chance to be tested, refined and fully institutionalized.

Obama doctrine is here to stay – defense cuts prove

Wehner ‘12, senior fellow at the Ethics and Public Policy Center, 1/6/2012
[Peter, “The Defense Budget and America’s Decline,” http://www.commentarymagazine.com/2012/01/06/defense-budget-america-
decline/]

There are many salient points to make about President Obama’s terribly unwise plan to cut $500 billion in defense
spending during the next decade. But I want to focus on what I think it reveals about the worldview of America’s
44th president. The one unequivocal area in which the federal government should be involved in is national defense. And our
military is the one area which Gallup reports Americans trust more than any other American institution. According to a recent
survey, 78 percent of those polled say they have a great deal of confidence in the U.S. military (versus 12 percent for Congress). And
that trust is well-earned; the military has performed its tasks with extraordinary skill. And yet it is the military, more than any area in
the federal government, that is now being asked to absorb the brunt of budget cuts – even though we’re still a nation at war. It is a
striking thing to witness. I’ve argued before that the Obama presidency, animated by a progressive impulse, wants to punish success.
Rewarding human excellence is in many respects an alien concept to this president (see his repeated attacks on wealth creators). It’s
therefore not surprising the president would decide to target the military when it comes to budget cuts. Its achievements have
earned it a reduction in its budget, much like bad schools are rewarded with more money (the theory being that their failures are
due to parsimony). And then there is the point made by the estimable Charles Krauthammer, who said on Fox News last night that
Obama’s budget strategy “is a roadmap of American decline.” I quite agree, and I would simply add that it’s intentional. For those
who dissent from this judgment, I would point them to an important article by Ryan Lizza in the New Yorker that includes this
paragraph: Obama may be moving toward something resembling a doctrine. One of his advisers described the president’s actions in
Libya as “leading from behind.” That’s not a slogan designed for signs at the 2012 Democratic Convention, but it does accurately
describe the balance that Obama now seems to be finding. It’s a different definition of leadership than America is known for, and it
comes from two unspoken beliefs: that the relative power of the U.S. is declining, as rivals like China rise, and that the U.S. is reviled
in many parts of the world. Pursuing our interests and spreading our ideals thus requires stealth and modesty as well as military
strength. “It’s so at odds with the John Wayne expectation for what America is in the world,” the adviser said. “But it’s necessary for
shepherding us through this phase.” Declining power, a reviled reputation, modesty, and leading from behind:
Obama sees his task as shepherding what he deems to be this deeply imperfect nation – one he
repeatedly apologized for during the early months of his presidency — through its inevitable descent. Unfortunately for us,
with Obama at the helm, America’s decline is becoming a self-fulfilling prophecy . You need look no
further than the defense budget for confirmation of that.
Low Oil Prices Good Answers
Prolonged dip in prices collapses all producer states, causes political repression and state
collapse, and unleashes wars across the world

Hulbert 12 Matthew Hulbert is an analyst at the Netherlands Institute for International Relations "The political perils of low
oil prices" July 9 2012 www.europeanenergyreview.eu/site/pagina.php?
id=3796&id_mailing=295&toegang=49182f81e6a13cf5eaa496d51fea6406

As unedifying as all that might be, the bigger problem producer states have is that internal repression
has no guarantee of success these days. It didn't work for Gadhafi in Libya, and it's unlikely to work
for Assad in Syria in the long term. As fierce as the rear-guard battles have been, they’ve not been
militarily conclusive or conducive to on-going hydrocarbon production. Follow that argument through
and it is clear that if the bulk of producer regimes were struggling to hang on in a $125/b world, they stand little
chance of pulling through in an $80/b (or less) environment. So we reach the third step, and logical
conclusion of our argument. The lower prices go, the more likely political unrest creates serious
supply disruptions affecting physical supplies, with concomitant effects on paper markets.
That obviously puts a radically new spin on what 'cyclical' means as far as price and political
instability is concerned, but when we look across producer states, it’s hard to find any major players not
sitting on a powder keg of political risk these days. More likely than not, it will be some of the
smaller players that get caught in the cross fire first. In the Gulf, Saudi Arabia is already deeply
concerned about Bahrain relative to its Eastern Province. State implosion in Yemen is seen as an
internal issue of the al-Saud to deal with, while serious deterioration in Iraq is becoming increasingly
problematic in the North. Libya could see any post-war oil gains rapidly wiped out,
Sudanese production has already fallen prey to intractable internal disputes, Kazakhstan
remains entirely 'dispensable' in Central Asia given a lack of external clout in the region, while Nigeria has
new civil strife problems to confront with Boko Haram. That’s before we consider intractable
problems in Central Africa and the Horn of Africa . Any one of these jurisdictions could end
up with a scorched earth policy if financing gaps aren’t closed. Go further up the producer state 'food
chain', and some of the world's largest players all have the same structural political problems, be
it in the Middle East, Eurasia or Latin America . Any sign that a bigger petro-beast is losing
control, and prices would rapidly lift. That might be welcome news for producer states lucky enough to ride the
price wave and remain intact, but it's a very dangerous game to play . And that's the whole problem
here - the gap between geological costs of production and the geopolitical cost of survival is
simply too wide for producers to cover without falling back on draconian measures. If this 'self-
correcting' mechanism between price and political unrest starts supporting an informal price floor then so be it, but we
shouldn't be fooled that this is serving anyone's interests - on either side of the consumer-producer ledger. Yes, it will help firm
prices when certain producers struggle to adapt to rapidly shifting economic conditions, but assuming that more and more
producer states hit political problems as prices slip, we're merely cementing the 'too big to fail' status of the very largest oil
producers. Seeing petro-states dropping like political flies as prices correct isn't a proper
'solution' for a floor, not only because prices will rebound with a vengeance when markets tighten, but because it will make
us even more dependent on a handful of key suppliers. As we all know from previous problems in Iraq (2.9
mb/d), Iran (3 mb/d), Libya (1.48 m/bd), Nigeria (2.4 mb/d) and even Venezuela (2.7 mb/d), once things go
politically wrong , it takes a very long time, if ever, to get back to optimal production levels.
It's the antithesis of where consumers want to be in terms of sourcing plentiful and fungible supplies. Final scene:
corpses all over the stage By way of reminder, as much as petro-states currently face a systemic crisis trying to set a

price floor, it was only in March that we saw how badly placed OPEC is to moderate the market at the top. Seeing
petro-
states in a pickle might warm the hearts of many right now, but markets can turn, and turn
fast. When they do, the oil weapon will shift target as well. It will no longer be pointed at
petro players heads, but directly at consumer states . That's the consequence of a dysfunctional energy
system - not just with a $50-$150/b outlook eminently possible, but swings well beyond that 'price band' all too likely.
Splitting this price directly in two and sticking close to $100/b might not be that bad an idea
after all: Mopping up the mess from producer state implosion would require an effort far
beyond the international systems capabilities and reach. Carefully agreed truces are always better than
outright wars, particularly for those squeamish about collateral damage. Corpses would litter the entire energy
stage .
MANPADS Answers
No MANPADS threat

Reuters ‘11. [“U.N. urges Libya to sell off cache of ‘yellowcake’ uranium” --
http://www.alarabiya.net/articles/2011/12/22/184055.html]

Martin also confirmed a U.S. finding from last month that Libya’s missing stocks of shoulder-fired anti-aircraft missiles –
“man portable air defense systems” or MANPADS - appear to be still in the country . “While the focus of
international concern continues to be the potential proliferation of MANPADs, as yet there seems to be
little evidence of such weapons systems appearing in neighboring countries,” he said. “Visits at
weapon storage sites and brigades throughout Libya suggest that most looted arms may be held by
revolutionary brigades or local militias within a limited distance from the looted sites , thereby
rendering it primarily a national Libyan arms control and disarmament concern,” Martin said. He added that the
U.N. Support Mission in Libya (UNSMIL) had agreed with Libya's Defence Ministry to set up a task force on
MANPADS to “facilitate a country-wide mapping of weapons and storage sites and to coordinate
the identification, collection and disabling efforts.” The U.N. mission is also working to register
MANPADS held by revolutionary brigades, Martin said.

(Ian Martin is the UN special envoy to Libya)

No weapons prolif.

Chivers ‘11 (C.J.; “How to Control Libya Missiles? Buy Them Up,” Gainesville Sun,
http://www.gainesville.com/article/20111222/ZNYT03/112223021/-1/news?Title=How-to-Control-Libya-Missiles-Buy-Them-Up)

The United States has committed $40 million to secure Libya’s arms stockpiles, much of it to prevent
the spread of Manpads. No budget has been designed for a purchase program, and the price to be paid for each missile and
its components has not been determined, the official said. If Libya agrees to a program, prices will probably be set by Libyan officials
after testing the market, he added. The official, along with others, spoke on the condition of anonymity because the program, if
approved, would be classified. Although such efforts are often called “buyback” programs, in this case even the label raises
sensitivities, officials said. After providing Stinger missiles to Afghan forces fighting the Soviets in the 1980s, the United States
organized a buyback program, trying to reduce the chance that the missiles would be used against international civilian air traffic or
Western military planes. In Libya, the program would not technically be a buyback , as these weapons were not
provided by the West, American officials said. They were purchased from Eastern bloc suppliers during Colonel Qaddafi’s long period
of arms acquisition. Matthew H. Schroeder, a researcher who covers proliferation of Manpads at the Federation of American
Scientists, said that such purchase programs had taken missiles out of circulation in Afghanistan and Iraq. “ These programs
have netted Manpads in the past, in at least quantities in the low hundreds,” he said. He emphasized that he did not
know any details of the American plans for Libya, and that he could not comment on them.

No MANPADS threat.
Haaretz ‘11. [“Libya’s nuclear materials remain intact, UN report says” -- http://www.haaretz.com/news/middle-east/libya-s-
nuclear-materials-remain-intact-un-report-says-1.403006]

Martin also struck down the fear that surface-to-air missiles could spread to neighboring
countries. He said that thousands of so-called MANPADS abandoned by troops loyal to the former Gadhafi regime
had been looted, but remained within Libya and were held by revolutionary brigades or local militias. Libya's
transitional government had also asked the Organization for the Prohibition of Chemical
Weapons to study chemical weapons stocks, Martin said. He said the organization was scheduled to
visit sites in Libya in January to verify declarations made by the Gadhafi regime.

MANPADS fail

DEFENSE WEB ‘11 “US Says 5,000 Surface-to-Air Missiles Secured in Libya,” http://www.defenceweb.co.za/index.php?
option=com_content&view=article&id=22022:us-says-5-000-surface-to-air-missiles-secured-in-libya&catid=50:Land&Itemid=105)

"The bulk of the inventory are SA-7 older Soviet models of surface-to-air missiles," said Smith. "Some
of the missiles they have recovered were corroded and non-functional . Many of the others were still in
their steel shipping containers and were fully functioning missiles. "
Manufacturing Answers
World econ thumps the adv

Schneider ‘13 (Howard, Washingon Post, "World Bank trims growth forecast,"
http://www.washingtonpost.com/business/economy/world-bank-trims-growth-forecast/2013/01/15/f5fafc0e-5f48-11e2-b05a-
605528f6b712_story.html)

The World Bank has sharply reduced its estimate of global economic growth in 2013,
projecting that the downturn in Europe and the United States’ fiscal problems will continue to weigh
on investment and spending. The bank said it expects the world economy to expand 2.4 percent this year, compared with 3
percent growth it had forecast as of June. The second half of 2012 saw a worsening of the euro crisis, fiscal
brinksmanship in the United States that left central debt and spending issues unresolved, and a recognition that
developing countries were slowing as well. Although World Bank officials said they found some developments
last year “comforting,” including the deal that avoided the worst of the “fiscal cliff” in the United States, they still saw big risks
misstep in Europe that aggravates problems there, or a collapse in fiscal talks that threatens the
ahead: a
U.S. sovereign debt rating. That alone could force global growth down a notch. Economic
weaknesses from 2012 “ended up lasting a fair bit longer” than expected, said Andrew Burns, lead author of the bank’s latest Global
Economic Prospects report. The bank forecast U.S. growth of 1.9 percent — less than the most-pessimistic estimates from the
Federal Reserve. The bank also foresaw more modest growth for China in coming years — 8.4 percent in 2013, slowing to about 8
most sour note was again reserved for Europe, where a financial and
percent the following two years. But its
sovereign debt crisis has eased but a moribund economic landscape remains. Even the German
economy, the region’s largest and the presumed pillar of growth, shrank in the final months of 2012, and leaders in
the region remain torn over the proper balance of budget-cutting austerity and programs to rebuild growth.

Manufacturing resilient – newest data – manufacturing jobs don’t solve the economy

Mallaby ‘13 (Sebastian, senior fellow at the Council on Foreign Relations, "American industry is on the move,"
http://www.ft.com/intl/cms/s/0/6709cc5c-58ed-11e2-b59d-00144feab49a.html#axzz2ILjNAQfA)

The more important technological jolt comes under the heading of “big data”. On Friday an exhaustive
survey of management practices at 30,000 US manufacturing establishments was released. Two of the authors, Nick Bloom and John
Van Reenen, had previously shown that US companies were, on average, better managed than foreign rivals.
A striking conclusion of their study is that US manufacturers continue to get better, particularly when it comes
to capturing and analysing data on everything from customer behaviour to production-line efficiencies. And there is
plenty of scope to improve further. A minority of survey respondents embraced most state-of-the-art management incentives and

monitored performance against clear targets. But a quarter of respondents adopted fewer than half of these practices. So the
stage is at least half set for a US manufacturing revival , even if obstacles – poor education, poor
infrastructure – remain. But what might a revival mean? Not, unfortunately, a cure for
unemployment. Since a trough in January 2010, the US has generated just over half a million new manufacturing jobs but
the bounce mostly reflects the collapse during the recession. For an advanced economy to create manufacturing employment
independently of a cyclical rebound is almost unheard of. Even as it boosted manufacturing as a share of output between 1993 and
2007, Sweden lost almost a 10th of its manufacturing jobs. But a manufacturing turnround is clearly desirable. Precisely because
manufacturing workers can be displaced by machines , it is factories that drive
productivity: in the US, manufacturing accounted for about 17 per cent of output between 1995 and 2005, yet contributed 37
per cent of economywide productivity gains, according to McKinsey. Higher productivity means higher pay for surviving employees:
American manufacturing workers are on average paid better than American service workers. And consumers benefit from the
productivity windfall. Since 1985 the quality-adjusted price of US durables has scarcely budged while the cost of services has more
than doubled.

US manufacturing up –conventional irrelevant

Mallaby ‘13 (Sebastian, senior fellow at the Council on Foreign Relations, "American industry is on the move,"
http://www.ft.com/intl/cms/s/0/6709cc5c-58ed-11e2-b59d-00144feab49a.html#axzz2ILjNAQfA)

If Sweden sounds impressive, here is the surprise: over the same period American
manufacturers piled up an even larger productivity gain of 69 per cent. Again, competition
contributed: the US joined the North American Free Trade Agreement and the World
Trade Organisation, and its continent-sized economy generates plenty of internal
competition. But in the US case, the impetus from trade and competition has been powerfully
reinforced by a jolt from technology. Despite much fashionable chatter, this is not mainly
about fracking. The new extraction technology has cut the price of natural gas in the US to a
fraction of the Asian level, but, as the McKinsey Global Institute observed recently, the
industries that are most energy-intensive are not actually very trade-intensive. US paper
mills and oil refineries will enjoy the cheap gas bonanza but not much production in these
sectors is likely to shift to US shores.

US manufacturing will structurally recover in the long-term

Boselovic ‘13 (Len, Heard Off the Street: Recent events temper optimism on reshoring," http://www.post-
gazette.com/stories/business/opinion/heard-off-the-street-recent-events-temper-optimism-on-reshoring-670221/)

The ExOne Co. disclosed plans last week for an initial public offering, giving investors an opportunity to place a bet on
technology many say could reverse the long, steady decline of U.S. manufacturing. The North
Huntingdon business is one of a growing number of practitioners of a technology industrial mavens have named additive
manufacturing. It
involves making complex products using digital imaging rather than bending,
molding, punching or grinding parts that must then be assembled into a finished product that additive
manufacturing can produce quicker and cheaper. Advocates say the technology could help bring
factory jobs back to the United States, a trend manufacturing gurus say has enough legs already to justify a
name: reshoring. Boston Consulting Group says reshoring could create 2.5 million to 5 million U.S.

manufacturing jobs by the end of this decade. President Barack Obama and others have based their claims of a
renaissance in U.S. manufacturing on reshoring and increasing U.S. exports.

Squo solves manufacturing

Ignatius 12 (David Ignatius writes a twice-a-week foreign affairs column and contributes to the
PostPartisan blog. Ignatius joined The Post in 1986 as editor of its Sunday Outlook section. In
1990 he became foreign editor, and in 1993, assistant managing editor for business news. He
began writing his column in 1998 and continued even during a three-year stint as executive
editor of the International Herald Tribune in Paris. Earlier in his career, Ignatius was a reporter
for The Wall Street Journal, covering at various times the steel industry, the Justice Department,
the CIA, the Senate, the Middle East and the State Department. Ignatius grew up in Washington,
D.C., and studied political theory at Harvard College and economics at Kings College,
Cambridge., 5/4/2012, "An economic boom ahead?", www.washingtonpost.com/opinions/an-
economic-boom-ahead/2012/05/04/gIQAbj5K2T_story.html)

Energy security would be one building block of a new prosperity. The other would be the
revival of U.S. manufacturing
and other industries. This would be driven in part by the low cost of electricity in the United
States, which West forecasts will be relatively flat through the rest of this decade , and one-half to one-
third that of economic competitors such as Spain, France or Germany. The coming manufacturing recovery is the subject of several
studies by the Boston Consulting Group. I’ll focus here on the most recent one, “U.S. Manufacturing Nears the Tipping Point,” which
appeared in March. What’s happening, according to BCG, is a “reshoring” back to America of
manufacturing that previously migrated offshore , especially to China. The analysts estimate that by
2015, China’s cost advantage will have shrunk to the point that many manufacturers will prefer
to open plants in the United States. In the vast manufacturing region surrounding Shanghai, total compensation
packages will be about 25 percent of those for comparable workers in low-cost U.S. manufacturing states. But given higher
American productivity, effective labor costs will be about 60 percent of those in America — not
low enough to compensate U.S. manufacturers for the risks and volatility of operating in China.
In about five years, argue the BCG economists, the cost-risk balance will reach an inflection point in
seven key industries where manufacturers had been moving to China: computers and
electronics, appliances and electrical equipment, machinery, furniture, fabricated metals,
plastics and rubber, and transportation goods. The industries together amounted to a nearly $2 trillion market in
the United States in 2010, with China producing about $200 billion of that total. As manufacturers in these “tipping
point” industries move back to America, BCG estimates, the U.S. economy will add $80 billion to
$120 billion in annual output, and 2 million to 3 million new jobs, in direct manufacturing and
spin-off employment. To complete this rosy picture, the analysts forecast that in about five
years, U.S. exports will increase by at least $65 billion annually. Hold on, Dr. Pangloss. Those are just
economists’ estimates. What do real manufacturers say ? Well, BCG has some new numbers on that, too. In April, the
consulting firm released a survey of executives at 106 U.S.-based companies with annual sales of
more than $1 billion. Thirty-seven percent of them said they were planning to reshore
manufacturing operations or “actively considering” the move. Among larger companies with sales of more
than $10 billion, the positive response rose to 48 percent. Talking about American decline has become a
national sport among policy intellectuals. The country still has severe political problems, but the numbers in
these new studies make me wonder if some of the deep pessimism is misplaced .

Energy prices are irrelevant to manufacturing

Levi 12 (Michael A. Levi David M. Rubenstein Senior Fellow for Energy and the Environment, 5/7/2012, "Oil and Gas Euphoria Is
Getting Out of Hand", blogs.cfr.org/levi/2012/05/07/oil-and-gas-euphoria-is-getting-out-of-hand/)
But there is more. Ignatius’s
column isn’t just about energy; it’s also about the resurgence of U.S.
manufacturing. Here’s how he links the two: “Energy security would be one building block of a new
prosperity. The other would be the revival of U.S. manufacturing and other industries . This would
be driven in part by the low cost of electricity in the United States, which West forecasts will be relatively flat through
the rest of this decade, and one-half to one-third that of economic competitors such as Spain, France or Germany.” Once again,
these sorts of claims have become increasingly common . Indeed the quantitative assertions are
perfectly plausible. But the big picture implications don’t make sense . As of 2010, total sales of U.S.
manufactured goods were about five trillion dollars . At the same time, the sector spent about 100
billion dollars on energy. That’s a mere two percent of total sales. You could slash energy costs
to zero, and it would barely move the needle for most U.S. manufacturers. There are, of course,
exceptions, like some iron, steel, cement, and paper makers. But even these industries care about much
more than their electricity prices. Will lower energy costs move things at the margin? Of course they will, and
that’s good news. But they are nowhere close to what’s needed for U.S. manufacturing to broadly
thrive.

Manufacturing loss inevitable

Thompson 12 (Derek Thompson is a senior editor at The Atlantic, where he oversees business coverage for the website.,
3/9/2012, "Trade My Brain, Please! Why We Don't Need to 'Make Something' to Export It",
www.theatlantic.com/business/archive/2012/03/trade-my-brain-please-why-we-dont-need-to-make-something-to-export-
it/254274/)

The president is onto something. Exports matter. A good reason to fetishize manufacturing is right in the president's
first line: "If
we do stuff here, we can sell it there ." As you might have caught on, I changed the word "make" in the
president's speech to "do" in this paragraph, because we don't need to make something and put it in a box to
sell it to foreigners. We can do stuff and sell it for foreign money, too. This sort of thing is called
a "service exports." It means selling our work, or brains, and our resources to other countries .
"Services exports" sounds like a rather silly or impossible thing -- like putting an American doctor in a small box, shipping him across
the Pacific to hospital in Mumbai, and shipping him back with the rupees. In fact, services exports
are much simpler
than that. Simpler, even, than selling actual manufactured goods. If an Argentinian student goes to
Harvard, that's an export. If a Korean uses a Kansas architect to design a building, that's an export. If Bain Capital advises a
British investor getting in on a Moroccan start-up, that's an export. Perhaps service exports seem less "pure" than
manufactured exports. In fact, there's a better case that the opposite is true . For any given
"export dollar," service exports create a great share of what economists call "U.S. value added.
That's a mouth-full, so you can call it "cold hard money in America." Think about a car shipped in a box from the United States to
Spain. That's a U.S. export. But it's not a 100% U.S. product. The car parts might have come from one country, where they were fixed
in Canada, taken south to be assembled in the United States, and shipped to Barcelona. The money made from the Spanish sale
counts as a U.S. export, but the revenue is divided across the car's global supply chain. On the other hand, if a Barcelona family goes
to Detroit for vacation, their euros stay in Detroit. "Business
service exports had 95.6 percent U.S. value-added
in 2004," the Brookings Metropolitan Policy program reported in a new study on exports. "Metropolitan areas specialized in
services, such as Des Moines, Las Vegas, and Washington, D.C. tend to have higher shares of U.S. value-added in their exports than
the rest of the largest 100 metro areas." The
United States is the second or third largest total exporter, by
various counts. But as a service exporter, we're the unambiguous world leader , commanding 14% of
the world market, twice that of second-place Germany. In 2010, private services exports represented a third of U.S. exports,
according to Brookings, and that number is going to keep growing. (As Scott Thomasson pointed out on Twitter, we even have a
trade surplus with China.) An emphasis on exports is important because it keeps us competitive in a global market and brings in
foreign money, which is especially useful for a slow economy. But we
shouldn't just think of exports as stuff we
can put into a box. We will continue to make things and put them in boxes and sell them in other
countries. But 70% of the economy is employed in the services sector and there are five times
more people working in professional services/education/leisure&hospitality than manufacturing today,
and the ratio will probably grow in the next decade. We need to talk about those exporting industries, too. You
don't need to make something to sell it "there."

Alt causes

Fitzpatrick 10/21/12 – Congressman (Fitzpatrick, Mike. “ Grow the economy, create jobs .” 21 October 2012.
http://www.phillyburbs.com/news/local/courier_times_news/election/grow-the-economy-create-jobs/article_cf0a5556-e7be-5161-
9a5e-bf939bac207f.html)

But the manufacturing sector, so vital to the American economy, is not on firm ground. I have developed a
comprehensive legislative plan that, I believe, will put us on the right path: “Made in America: A Plan to Promote American
Manufacturing and Revitalize Our Economy.” This plan includes proposals I helped introduce or supported in the 112th Congress
aimed at job creation and economic growth. Unfortunately, many of these common-sense proposals remain
gridlocked in Congress, but that cannot dissuade us from moving forward. First, let us look at the facts on local
manufacturing. The National Association of Manufacturers points to the 8th Congressional District as home to more than 700
manufacturing plants, and 36,581 manufacturing jobs, including 17,291 jobs supported by exporting manufactured goods.
Nationally, American manufacturers produce nearly $4.5 trillion worth of manufactured goods every year, or nearly 20 percent of all
goods worldwide. But it is clear that failure to address the challenges facing U.S. manufacturers today will result in the loss of
international competitiveness and slow the export engine that can fuel a true economic recovery. Here are the basic principles that
manufacturers believe will lead to economic growth and job creation: We must lower taxes and promote
certainty. The Alliance for American Manufacturing cites revising our nation’s tax policy as one of the most important
steps in the plan to save American manufacturing. It is no secret that the U.S. has one of the
highest corporate tax rates of any industrialized country, unchanged for the past 24 years.
High tax rates serve to drive investment and jobs overseas. Everyone must be required to pay their fair share
of taxes; however, we must ensure that our tax code is not punitive and allows manufacturers to thrive. We must rein in
regulations. According to the Small Business Administration, federal regulations place a $1.75 trillion strain on our economy,
annually. The SBA estimates these regulations cost an average of $10,585 per employee, and drain over
$200,000 from small businesses employing 20 people — an amount that can be reinvested in
the economy. Needed are common-sense regulations that protect consumers and the environment, while allowing
manufacturers to grow their business. We must promote American energy security. Energy independence
will help create demand for domestically manufactured goods and lower the cost of energy
for U.S. companies. Small and medium-sized firms cite energy costs as one of their heaviest burdens. We must
promote the development of a 21st Century work force to effectively compete in a 21st
Century economy. This includes promoting vocational-technical training and strengthening
science, technology, engineering and mathematics education (STEM) to prepare workers for the changing workplace.
Many manufacturers have skilled positions going unfilled even as U.S. joblessness remains at a troubling level. This problem
deserves focus at the highest levels of government. We must invest in our infrastructure. According to the American
Road and Transportation Builders Association, less than 4 percent of Stimulus funds were spent on transportation infrastructure.
This is unacceptable, particularly when 139 roads and bridges in Bucks County have been declared “structurally deficient.” With my
support, Congress passed a substantial transportation bill that funds infrastructure, streamlines the project permitting process, and
encourages capital investments necessary to create long-term jobs, boost our economy and create manufacturing orders. We
must pursue fair trade. Trade supports 90 million American manufacturing jobs — 17,000 jobs in the
8th District. Over 100 8th District companies are exporting manufactured goods to global customers.
Stats take out this impact

Thomas ‘12 – Business Journals Editor (Thomas, G. Scott. “Manufacturing employment drops, yet payrolls remain large.” 19
October 2012. http://www.bizjournals.com/bizjournals/on-numbers/scott-thomas/2012/10/manufacturing-employment-drops-
yet.html)

manufacturing sector has been on the decline for roughly 30 years. Several statistics could
America's
be cited as proof. Let's stick with the most basic of indicators -- seasonally adjusted
employment figures (as of August in the given year):

1960: 15.4 million manufacturing jobs

1970: 17.8 million

1980: 18.4 million

1990: 17.7 million

2000: 17.3 million

2010: 11.5 million

2012: 11.9 million

The drop, as you can see, began in the early 1980s, but was kept to reasonably small levels until 2001 and 2002, when the
nation lost more than 1 million manufacturing jobs in consecutive years. A slight recovery has occurred
during the past two years, with America regaining 409,000 manufacturing jobs since 2010. But nobody is predicting a
return to the employment levels of 2000, let alone 1980.

Manufacturing decline inevitable

Lewis ‘12 – Staffwriter (Lewis, Matt K. “ The incredible shrinking manufacturing sector.” 19 October 2012.
http://theweek.com/bullpen/column/235089/the-incredible-shrinking-manufacturing-sector)

During the second presidential debate, President Obama echoed those lyrics. Today, only 9 percent of Americans work
in manufacturing jobs. American industry has declined on Obama's watch — something he readily admits to.
During Tuesday night's debate, he channeled Springsteen, saying that "some jobs are going, and won't be coming
back." Part of this is unavoidable. It has to do with globalized markets and outsourcing. The
obvious benefits of high productivity and low wages overseas is irresistible to many
businesses. It has to do with immigration and technology. Meanwhile, American industry is becoming more
productive and efficient — a good thing, except that it means we can create more stuff with fewer workers. The good news, the
president tried to reassure us, is that the void left by these disappearing manufacturing jobs will be filled by high-paying, high-skills
occupations. Whether that's true or not, there's an obvious point he did not state: Many
Americans will be left behind
in the process. As the world changes, some people simply can't — or won't — adapt . And the
president's vision leaves those Americans behind. Manufacturing jobs were once the cornerstone of American industry. You could
graduate from high school (or not) and get a job in a factory that would pay you enough money to support a middle-class family.
However, duringthe Reagan era, manufacturing declined significantly, setting in motion a trend
that lasted through both Bush administrations as well as the Clinton administration. This isn't a Republican
or Democratic problem. There's plenty of bipartisan blame to go around.
Meltdowns Answers
No impact to meltdowns

Adams 12 - Former submarine Engineer Officer, Founder, Adams Atomic Engines, Inc., (Rod “Has Apocalyptic Portrayal of
Climate Change Risk Backfired?”, May 2, http://atomicinsights.com/2012/05/has-apocalyptic-portrayal-of-climate-change-risk-
backfired.html?utm_source=feedburner&utm_medium=feed&utm_campaign=Feed%3A+AtomicInsights+%28Atomic+Insights%29)

Not only was the discussion enlightening about the reasons why different people end up with different opinions about climate
change responses when presented with essentially the same body of information, but it also got me thinking about a possible way to
fight back against the Gundersens,Caldicotts, Riccios, Grossmans and Wassermans of the world. That group
of five tend to use apocalyptic rhetoric to describe what will happen to the world if we do not
immediately start turning our collective backs on all of the benefits that abundant atomic energy can
provide. They spin tall tales of deformed children, massive numbers of cancers as a result of minor
radioactive material releases, swaths of land made “uninhabitable” for thousands of years,
countries “cut in half”, and clouds of “hot particles” raining death and destruction ten thousand
miles from the release point. Every one of those clowns have been repeating similar stories for
at least two solid decades, and continue to repeat their stories even after supposedly
catastrophic failures at Fukushima have not resulted in a single radiation related injury or death .
According to eminent scientists – like Dr. Robert Gale – Fukushima is unlikely to EVER result in any
measurable increase in radiation related illness. One important element that we have to consider to assess cancer
risks associated with an accident like Fukushima is our baseline risk for developing cancer. All of us, unfortunately, have a substantial
risk of developing cancer in our lifetime. For example, a 50-year-old male has a 42% risk of developing cancer during his remaining
life; it’s almost the same for a 10-year-old. This risk only decreases when we get much older and only because we are dying of other
causes. It’s true that excess radiation exposure can increase our cancer risk above baseline levels; it’s clear from studies of the
survivors of the 1945 atomic bombings of Hiroshima and Nagasaki, of people exposed to radiation in medical and occupational
settings, and of people exposed to radon decay products in mines and home basements. When it comes to exposures like that of
Fukushima, the question is: What is the relative magnitude of the increased risk from Fukushima
compared to our baseline cancer risk? Despite our fears, it is quite small . If the nuclear industry
– as small and unfocused as it is – really wanted to take action to isolate the apocalyptic antinuclear activists, it could take a page
from the effective campaign of the fossil fuel lobby. It could start an integrated campaign to help the rest of us to remember that,

despite the dire predictions, the sky never fell , the predicted unnatural deaths never
occurred, the deformations were figments of imagination, and the land is not really irreversibly
uninhabitable for generations. The industry would effectively share the story of Ukraine’s recent decision to begin
repopulating the vast majority of the “dead zone” that was forcibly evacuated after the Chernobyl accident. It would put some
context into the discussion about radiation health effects; even if leaders shy away from directly challenging the Linear No Threshold
(LNT) dose assumption, they can still show that even that pessimistic model says that a tiny dose leads to a tiny risk. Aside: My
personal opinion is that the LNT is scientifically unsupportable and should be replaced with a much better model. We deserve far
less onerous regulations; there is evidence that existing regulations actually cause harm. I hear a rumor that there is a group of
mostly retired, but solidly credentialed professionals who are organizing a special session at the annual ANS meeting to talk about
effective ways to influence policy changes. End Aside. Most
of us recognize that there is no such thing as a zero
risk; repeated assertions of “there is no safe level” should be addressed by accepting “close
enough” to zero so that even the most fearful person can stop worrying. The sky has not fallen,
even though we have experienced complete core meltdowns and secondary explosions that did
some visible damage. Nuclear plants are not perfect, there will be accidents and there will be
radioactive material releases. History is telling me that the risks are acceptable, especially in the
context of the real world where there is always some potential for harm. The benefits of
accepting a little nuclear risk are immense and must not be marginalized by the people who
market fear and trembling .

Meltdown impacts won’t happen – empirics

WNA ’11 [World Nuclear Association, “Safety of Nuclear Power Reactors”, (updated December 2011), http://www.world-
nuclear.org/info/inf06.html]

From the outset, there has been a strong awareness of the potential hazard of both nuclear criticality and release of radioactive
materials from generating electricity with nuclear power. As in other industries, the design
and operation of nuclear
power plants aims to minimise the likelihood of accidents, and avoid major human consequences when they
occur. There have been three major reactor accidents in the history  of civil nuclear power - Three Mile Island,
Chernobyl and Fukushima. One was contained without harm to anyone, the next involved an intense fire without provision for
containment, and the third severely tested the containment, allowing some release of radioactivity. These are the only major
accidents to have occurred in over
14,500 cumulative reactor-years of commercial nuclear power operation in 32
countries. The risks from western nuclear power plants, in terms of the consequences of an accident or terrorist attack,  are

minimal  compared with other commonly accepted risks. Nuclear power plants are very robust.
 

Zero probability of meltdown attacks

Hargreaves ‘9 [Steve Hargreaves, CNNMoney.com staff writer, The threat of nuclear meltdown, November 12,
2009, http://money.cnn.com/2009/11/12/news/economy/nuclear_security/index.htm]
 

This is only a drill, but the threat they're preparing for is very real. It's one of the worst
disaster scenarios imaginable: Terrorists infiltrate a nuclear power plant and cause a  meltdown. The
government and the industry say that with all the security measures in place, the   chance of that

happening is practically zero .


 

No risk of attacks – newest evidence

NEI ’12 [Nuclear Energy Institute, “Myths & Facts About Safety”, January
2012, http://www.nei.org/newsandevents/nei-backgrounders/myths--facts-about-nuclear-
energy/myths--facts-about-safety]
 

Myth: Nuclear power plants are likely targets for terrorism. Fact: With protective measures similar to high-
security military installations, U.S. nuclear plants are among the most  highly protected facilities  in the
nation’s industrial infrastructure. It is because of their fortifications and multiple layers of security that nuclear plants
present a strong deterrent to potential threats. Myth: A nuclear power plant cannot withstand a terrorist attack. Fact: With
protective measures similar to high-security military installations, U.S. nuclear plants are among the most highly protected facilities
in the nation’s industrial infrastructure. Nuclear power plants are protected 24/7 by professional security personnel armed with
automatic weapons prepared to repel ground and airborne terrorist attacks. It is because of their fortifications and
multiple layers of security that nuclear plants are far less likely to be targets of terrorism than the
thousands of far more vulnerable potential targets across the nation. Anti-terrorism measures are regularly tested and closely
coordinated with local, state and federal authorities. Myth: A nuclear power plant cannot withstand the impact of a jetliner. Fact:
Following the terrorist attacks of Sept. 11, 2001, sophisticated computer modeling by some of the world’s leading structural
engineers showed that nuclear power facilities  that contain radioactive material can withstand a jetliner impact
without releasing radiation. Likewise, all new nuclear power plants are required to withstand the direct impact of a fully fueled
commercial jetliner. Myth: Nuclear plants are vulnerable to cyber attacks. Fact: There has never been a successful
cyber attack at any U.S. nuclear plant . Unlike industries for which two-way data flow is critical (e.g.
banking), nuclear power plants do not require incoming data flow. None of a plant’s safety and control systems are
connected to the Internet. Any additional computers utilized in a nuclear plants are strictly controlled with their content, use and
possession monitored by security personnel. Nuclear plants are protected from grid instability  and are able to
safely shut down in a variety of ways without computer controls under any condition including a total loss of off-site power.

New reactor designs solve the meltdown risk

CFR ‘6 (Lionel Beehner, “Chernobyl, Nuclear Power, and Foreign Policy”, April
25, http://www.cfr.org/publication/10534/chernobyl_nuclear_power_and_foreign_policy.html)

Repeat of Chernobyl-like meltdown. A dozen or so other Chernobyl-era nuclear plants with aging equipment are still operating—and
expected to continue to operate for the next thirty years—within the former Soviet Union. Although their design flaws have for the
most part been addressed, some experts fear human error makes a future  meltdown, however remote,
still a possibility. Elsewhere, Chernobyl-like meltdowns are becoming unlikely, thanks  mainly to
developments in technology, IAEA Deputy Director Tomihiro Taniguchi told the Associated Press. Others point
to advanced technologies like pebble-bed reactors, which use graphite pebbles and gases like helium as a coolant, and are
safer, cheaper, and more efficient but leave greater waste than traditional nuclear power plants. Plans for these kinds of
reactors are in place inSouth Africa and the United States.
Backup generators check meltdowns

Counterpunch ‘3, 8/14/2003, "Unusual Events" at Nuke Power Plants, p. online

August 14, 2003 was an historic day for the nuclear power industry, as nine nuclear reactors at
seven power plants in New York, Ohio, Michigan, and New Jersey were forced to shut down during largest and
most severe electricity blackout in U.S. history. While the rest of the world described the blackout as a massive
power failure, the Nuclear Regulatory Commission (NRC), in perhaps its most classic understatement to date, termed the
widespread loss of power as "instabilities" in the electrical transmission grid, while assuring people that All
plants are in a
safe condition, using their emergency diesel generators where appropriate.(1). The Commission elaborated on
the issue Friday, stating that "Safety systems at all the shut-down plants operated successfully, and
plants stabilized in a safe shut-down condition. Adequate safety was maintained at all times ." All
seven plants issued an "unusual" event report, which the Commission described as "the lowest of four classes of emergency, and
means an incident is in process or has occurred indicating a potential degradation of plant safety. No
releases of
radioactive material requiring off-site response or monitoring have occurred or are expected."

Even the worst case scenario will be contained and cause no harm

McGregor ‘1 (Douglas S. McGregor, Director, Semiconductor Materials and Radiological Technologies Laboratory, University of
Michigan, Ph.D in Nuclear Engineering, etc etc, April 23rd 2001, (Rethinking Nuclear Power, The New American, Vol 17, No. 9,
http://www.thenewamerican.com/tna/2001/04-23-2001/vo17no09_nuclear.htm)

The most serious accident possible is the release of radioactive material into the environment. It is not a
nuclear explosion, for the simple reason that the uranium fuel used in a nuclear power plant does not contain a
high enough concentration of U-235 to make a nuclear explosion even theoretically possible. To make
such an explosion possible, the uranium fuel inside a reactor would have to be enriched to about 90 percent U-235, but it is only
enriched to about 3.5 percent. The worst nuclear power plant disaster in history occurred when the Chernobyl
reactor in the Ukraine experienced a heat (and gas) not nuclear explosion. If such an explosion were to have
occurred in a Western nuclear power plant, the explosion would have been contained because all Western
plants are required to have a containment building a solid structure of steel-reinforced concrete that completely
encapsulates the nuclear reactor vessel. The Chernobyl plant did not have this fundamental safety structure, and so the
explosion blew the top of the reactor building off, spewing radiation and reactor core pieces into the air. But the design of the
Chernobyl plant was inferior in other ways as well. Unlike the Chernobyl reactor, Western power plant nuclear
reactors are designed, under operating conditions, to have negative power coefficients of reactivity that
make such runaway accidents impossible. The bottom line is that the flawed Chernobyl nuclear power plant would
never have been licensed to operate in the U.S. or any other Western country, and the accident that occurred there simply would
not occur in a Western nuclear power plant. The circumstances surrounding the Chernobyl accident were in many ways the worst
possible, with an exposed reactor core and an open building. Thirty-one plant workers and firemen died directly from radiation
exposure at Chernobyl. Also, it is projected that over 3,400 local residents will eventually acquire and die of cancer due to their
exposure to the radioactive fallout. By comparison, within a matter of hours more than 2,300 were killed and as many as 200,000
others injured in a non-nuclear accident when a toxic gas cloud escaped from the Union Carbide pesticide plant in Bhopal, India.
According to conventional wisdom, theworst nuclear power accident in this country occurred at the
Three Mile Island plant in Pennsylvania. Yet, in that incident, nobody was killed and nobody was
injured. One exception, perhaps, could be Dr. Edward Teller, the distinguished pro-nuclear physicist who played a key role in the
development of nuclear advancements during and after World War II. In a two-page ad appearing in the Wall Street Journal for July
31, 1979, Dr. Teller explained that, at 71 years of age and working 20 hours per day, the strain of refuting some of the anti-nuclear
"propaganda that Ralph Nader, Jane Fonda and their ilk" were "spewing to the news media" in the wake of Three Mile Island led to a
heart attack. He continued: "You might say that I was the only one whose health was affected by that reactor near Harrisburg. No,
that would be wrong. It was not the reactor. It was Jane Fonda. Reactors are not dangerous." The event at Three Mile Island
occurred from faulty instrumentation that gave erroneous readings for the reactor vessel environment. Due to a series of equipment
failures and human errors, plus inadequate instrumentation, the reactor core was compromised and underwent a partial melt. Yet
radioactive water released from the core configuration was safely confined within the
containment building structure, and very little radiation was released into the environment. The Three
Mile Island incident actually underscores the relative safety of nuclear power plants since the safety
devices worked as designed and prevented any injury from occurring to humans, animals, or the
environment. Moreover, the accident directly resulted in improved procedures, instrumentation, and safety systems, and now our
nuclear reactor power plants are substantially safer. The Three Mile Island Unit 2 core has been cleaned up and the radioactive
deposit properly stored; Three Mile Island Unit 1 is still operating with an impeccable record.

No risk of terrorist attack on nuclear power plants - plants are structurally designed and
secured against attacks

Spencer and Loris ‘7. http://www.heritage.org/research/energyandenvironment/bg2087.cfm.

reactors are designed to withstand the impact of airborne objects like passenger
FACT: Nuclear
airplanes, and the Nuclear Regulatory Commission has increased security at U.S. nuclear power plants and
has instituted other safeguards. A successful terrorist attack against a nuclear power plant could have severe
consequences, as would attacks on schools, chemical plants, or ports. However, fear of a terrorist attack is not a sufficient reason to
deny society access to any of these critical assets. The United States has 104 commercial nuclear power plants, and
there are 446 worldwide. Not one has fallen victim to a successful terrorist attack. Certainly, history should not beget com-
placency, especially when the stakes are so high. However, the NRC has heightened security and increased safeguards on site to deal
with the threat of terrorism. A deliberate or accidental airplane crash into a reactor is often cited as a threat, but nuclear reactors
are structurally designed to withstand high-impact airborne threats, such as the impact of a large passenger
airplane. Furthermore, the Federal Aviation Administration has instructed pilots to avoid circling or loitering over
nuclear or electrical power plants, warning them that such actions will make them subject to
interrogation by law enforcement personnel.[8]

No risk of reactor meltdown

UIC 7 (Uranium Information Center, “Safety of Nuclear Power Reactors”, Nuclear Issues Briefing Paper, 14,
http://www.uic.com.au/nip14.htm)

Those responsible for nuclear power technology in the west devoted extraordinary effort to ensuring that a meltdown of the
reactor core would not take place, since it was assumed that a meltdown of the core would create a major public hazard, and if
uncontained, a tragic accident with likely fatalities. In avoiding such accidents the industry has been
outstandingly successful. In 12,000 cumulative reactor-years of commercial operation in 32 countries, there have been
only two major accidents to nuclear power plants - Three Mile Island and Chernobyl, the latter being of little relevance outside
the old Soviet bloc. It was not until the late 1970s that detailed analyses and large-scale testing, followed by the 1979
meltdown of the Three Mile Island reactor, began to make clear that even
the worst possible accident in a
conventional western nuclear power plant or its fuel could not cause dramatic public harm.
The industry still works hard to minimize the probability of a meltdown accident, but it is now clear that no-one need fear a
potential public health catastrophe. The decades-long test and analysis program showed that less radioactivity escapes from
molten fuel than initially assumed, and that this radioactive material is not readily mobilized beyond the immediate internal
structure. Thus, even
if the containment structure that surrounds all modern nuclear plants
were ruptured, it would still be highly effective in preventing escape of radioactivity . It is the
laws of physics and the properties of materials that preclude disaster, not the required actions by safety equipment or
personnel. In fact, licensing approval now requires that the effects of any core-melt accident must be confined to the plant
itself, without the need to evacuate nearby residents. The two significant accidents in the 50-year history of civil nuclear power
generation are: •Three Mile Island (USA 1979) where the reactor was severely damaged but radiation was contained and there
were no adverse health or environmental consequences •Chernobyl (Ukraine 1986) where the destruction of the reactor by
steam explosion and fire killed 31 people and had significant health and environmental consequences. The death toll has since
increased to about 56. A table showing all reactor accidents, and a table listing some energy-related accidents with multiple
fatalities are appended. These two significant accidents occurred during more than 12,000 reactor-years of civil operation. Of all
the accidents and incidents, only the Chernobyl accident resulted in radiation doses to the public greater than those resulting
from the exposure to natural sources. Other incidents (and one 'accident') have been completely confined to the plant. Apart
from Chernobyl, no nuclear workers or members of the public have ever died as a result of exposure to radiation due to a
commercial nuclear reactor incident. Most of the serious radiological injuries and deaths that occur each year (2-4 deaths and
many more exposures above regulatory limits) are the result of large uncontrolled radiation sources, such as abandoned
medical or industrial equipment. (There have also been a number of accidents in experimental reactors and in one military
plutonium-producing pile - at Windscale, UK, in 1957, but none of these resulted in loss of life outside the actual plant, or long-
term environmental contamination.) It should be emphasised that a commercial-type power reactor
simply cannot
under any circumstances explode like a nuclear bomb. The International Atomic Energy Agency (IAEA) was set up
by the United Nations in 1957. One of its functions was to act as an auditor of world nuclear safety. It prescribes safety
procedures and the reporting of even minor incidents. Its role has been strengthened in the last decade. Every country which
operates nuclear power plants has a nuclear safety inspectorate and all of these work closely with the IAEA.
Methane Blowouts Answers
Frontline
Methane dissolves in water and is consumed by bacteria – their author
Ryskin, Department of Chemical Engineering, Northwestern University, 3
George, Sept. 2003, Geological Society of America, Geology, 31(9), “Methane-driven oceanic
eruptions and mass extinctions”,
https://pangea.stanford.edu/research/Oceans/GES205/methaneGeology.pdf

Methane, CH4, is continuously produced beneath the ocean floor. Most of this methane is consumed by archaea
and bacteria in the upper layer of sediments; the rest escapes into the overlying water column
as bubbles. (For present purposes, it is immaterial whether some part of this methane flux results from dissociation of methane
hydrates.) The rising bubbles diminish in size and finally disappear as methane dissolves in the
seawater (Hovland et al., 1993). In a well-oxygenated (or anoxic but sulfate rich) water column, dissolved methane, as
well as the sinking organic matter from the surface waters, is oxidized by microbes. But unlike the
rising bubbles of methane or the sinking particles of organic matter, which can traverse the entire water column in a few days,
oxygen molecules or sulfate ions can be delivered to the water column only by fluid flow (or diffusion, which is very slow). In
stagnant water masses, therefore, the oxidation potential of the water column will be eventually overwhelmed by the flux of sinking
organic matter, plus the flux of methane bubbles from the seafloor. There is strong evidence in the geological record that dissolved
methane may escape oxidation in the water column (Thomas et al., 2002). Under such conditions, methane that dissolves in the
water column will accumulate in it as time proceeds. Evidence of such accumulation exists both in direct measurements and in the
geological record. The only long-term series of the former, in the Cariaco Basin, shows that methane concentration in the deep
waters Of the basin is increasing with time, roughly linearly (Figs. 6d and 7 Of Scranton et al., 2001). The microfossil carbon isotope
data from the Santa Barbara Basin sediment core show that high levels Of dissolved methane were present periodically in the water
column (Ken- nett et al., 2000). This has been confirmed by a microbial-biomarker study (Hinrichs et al., 2003), and is likely to be
representative Of oceanic regions prone to stagnation and anoxia, such as silled basins or deep-water masses isolated by topography
and Taylor columns. (Such regions are likely to have sizes below the resolution Of ocean circulation models used in biogeochemical
studies; e.g., Hotinski et al. 120011.) The time scale Of the meridional overturning circulation, —103 yr, has little relevance for the
water masses that do not participate in it; these can only be mixed by turbulence resulting from breaking Of the internal gravity
waves (ultimately due to tides and winds). The corresponding vertical (diapycnal) turbulent diffusivity K is typically —-10-5 m2 s—l,
and can be as low as 0.5 X 10—6 m2 (Gregg et al., 2003); the latter translates into K 16 m2 yr-I. This means that it may take a million
years for turbulent mixing to penetrate vertically through (Kt)l/2 4 km (t being time). (The tracer-based estimates Of the age Of
water masses are controversial (Wunsch, 20021 and are not used here.) In stagnant water masses, accumulation Of dissolved
methane may thus continue for very long times. Assuming that dissolved methane can accumulate to high concentrations, let us
explore the consequences. Consider a water column with stable density stratification due to the temperature and/or salinity
gradients. Methane carried by a rising bubble will dissolve in the surrounding water as long as the
concentration Of methane already dissolved in it is less than the saturation value (solubility) for the local temperature T and
pressure P; here saturation means equilibrium with respect to gaseous methane at the same T and P, i.e., with respect to methane
inside the bubble. The solubility Of methane in seawater is low (Handa, 1990); in terms of mole fraction, solubility at T 25 oc and P I
bar is —2 X 10-5. However, solubility is nearly proportional to pressure (Henry's law), and also increases as temperature drops, so
that at T 5 oc and P 400 bar (depth 4 km), the predicted value Of solubility is 4.3 X 10-3. METASTABILITY AND ERUPTION A liquid
subject to gravity and completely or partially saturated with dissolved gas is,
thermodynamically, in a metastable state. Con- sider for clarity the case when the concentration Of the dissolved
gas is only slightly below saturation throughout, and thus increases down- ward in accordance with Henry's law. Then locally there
is no tendency for the dissolved gas to exsolve (to form bubbles), in spite Of the fact that nuclei are
abundant in seawater. (Exsolution would lead to a slight increase in free energy: below saturation, the chemical potential Of the gas
species is lower in solution than in the free gas phase.) At the same time, the free energy Of the system as a whole would be greatly
reduced if most Of the dissolved gas were to somehow escape from solution and collect above the liquid. ('lh1S free energy
reduction IS due to the fast decrease Of the chemical potential Of gas with a drop in pressure.) Thus, the system is in a metastable
state, albeit an unusual one. Strictly speaking, this state is not an equilibrium one even locally: the increase of the solute
concentration with depth causes a diffusion flux directed upward, which, given sufficient time, could bring the system into the above
state Of minimum free energy. However, the continuous supply of methane by the rising bubbles from the seafloor ensures that the
concentration profile will remain nonuniform, slowly approaching the saturation one. Even if that supply were to cease, the diffusion
time scales are so long that this path toward the global energy minimum can be ignored.
No impact to methane blowouts- the methane becomes CO2 in the air and
there are many other sources
Ruppel and Noserale 12- Carolyn Ruppel, a Marine Geophysicist, has a Ph.D.. from
Massachusets Institute of Technology, associate professor in earth and atmospheric sciences at
Georgia Tech, and Diane Noserale, geologist and information specialist fro the US Geological
Survey, 2012 (“Gas Hydrates and Climate Warming,” USGS, January 24th, Available online at
http://www.usgs.gov/blogs/features/usgs_science_pick/gas-hydrates-and-climate-warming/,
Accessed 9-14-14)

A Chart showing how as climate warms, more hydrates melt, releasing more methane gas,
which acts as a greenhouse gas, causing climatic warming, thus perpetuating the cycle.
Some research suggests that this has happened in the past. Extreme warming during the
Paleocene-Eocene Thermal Maximum about 55 million years ago may have been related to a
large-scale release of global methane hydrates. Some scientists have also advanced the
Clathrate Gun Hypothesis to explain observations that may be consistent with repeated,
catastrophic dissociation of gas hydrates and triggering of submarine landslides during the Late
Quaternary (400,000 to 10,000 years ago).
Methane in the Atmosphere: Current Observations
The atmospheric concentration of methane , like that of carbon dioxide, has increased since
the onset of the Industrial Revolution (fig. 5). Methane in the atmosphere comes from many
sources, including wetlands, rice cultivation, termites, cows and other ruminants, forest fires,
and fossil fuel production (fig. 6). Some researchers have estimated that up to 2 percent of
atmospheric methane may originate with dissociation of global gas hydrates. Currently,
scientists do not have a tool to say with certainty how much, or if any, atmospheric methane
comes from hydrates.
Although methane is a potent greenhouse gas, it does not remain in the atmosphere for long.
Within about 10 years, it is transformed to carbon dioxide. Thus, methane that is released to
the atmosphere ultimately adds to the amount of carbon dioxide, the main greenhouse gas .
Expected Impact of Warming Climate on Methane Hydrate Deposits
For the most part, warming at rates documented by the Intergovernmental Panel on Climate
Change for the 20th century should not lead to catastrophic breakdown of methane hydrates
or major leakage of methane to the ocean-atmosphere system from gas hydrates that
dissociate. While the vast majority of methane hydrates would require a sustained warming
over thousands of years to trigger dissociation, gas hydrates in some locations are dissociating
now in response to short-term and long-term climate processes.
--- AT: Light ev
Light’s calculations aren’t based in reality – his studies are solely an
extrapolation of a false reading
Smith 13 [Alex Smith, Host of syndicated weekly Radio Ecoshock Show - the cutting edge with
top scientists, authors & activists. Previously a researcher for a global environment group, Clips
from Guy McPherson, John D. Cox, Dr. David Archer. Interview w. John Michael Greer. Analysis
of predictions by Malcolm Light of the Arctic Methane Emergency Group (AMEG), 6/5/13, “Will
Humans Go Extinct Soon?”, http://www.ecoshock.info/2013/06/will-humans-go-extinct-
soon.html, PS]
"Muriel" says Malcolm Light cherry picked the very worst single measurement of methane release
in 2010, a startling spike reported by the American agency NOAA, from the Arctic Svalbard
base on the day of November 16th, 2010. Light uses that spike for all his subsequent
frightening calculations. But here is what New Zealand blogger, Climate Show radio host Gareth Renowden wrote in his
"Hot-Topic" blog about the graph Malcolm Light used: "The graph comes from NOAA’s Earth System Research Labs Global
Monitoring Division’s new data visualisation web page here (you’ll see a CO2 graph first, but click on the menu to the left of the
graph to get the methane version). The readings for the last year are preliminary, and shown in brown. The last five data points are
so far off the chart that they are almost certainly going to be rejected as being caused by local contamination. That’s happened
before — the green dots show when — and at the moment other Arctic sites are not showing a similar rise. However, Svalbard is
close to sea floor methane hydrate deposits that are known to be venting gas". --- end Quote from Gareth Renowden at hot-
topic.co.nz Muriel Volestrangler tells us the data points showing the big methane spike have been revised by NOAA, who always said
such measurements were provisional. I checked. The spike is gone. The steady rise over the decades is back. You can see it
for yourself at the NOAA site linked above. Therefore, most of the calculations in Malcolm Light's paper are
not based on reality. They show what could happen, maybe, if there was ever a big release of
methane in the Arctic, which continued at the new high level. But that hasn't happened at all.
There's no doubt about it. Actual methane levels in the global atmosphere have NOT spiked beyond
the gradual increase expected. No emergency. Yet. Although it could happen... Volestrangler finds several more
gross assumptions, all for the worst, and unprovable hypothesis in the Malcolm Light paper. Muriel writes: " But since the
basis for it all is a one month anomaly at one measuring station, which has since been counted
as a false reading, it is all based on a mistake. He's taken that one month error, and
extrapolated it until the world has heated up by 14 degrees C, which he says will therefore be
an extinction event."

Plan can’t solve—Light also recommends a council of world governments and


oil companies are necessary to solve
Light 12 — Malcom Light, Center for Polar Observation and Modeling, University of London,
polar climate modeling and methane hydrates in the permafrost and submarine Arctic, 2013
(“Act Now On Methane,” Arctic News, December 21st, Available Online at, Accessed 11-16-2014)

The UN/US must immediately declare an international emergency of the most extreme kind
and call for a conference of world governments and oil companies to select the fastest and
most efficient way to deal with the Arctic methane eruption threat. The Lucy Project and
suggested HAARP experiments are methods of destroying the Arctic atmospheric methane
clouds at a distance and will be our first line of defence against the extreme global warming
potential of the fast increasing concentration of methane in the atmosphere. The Angels
proposal aims to depressurize the Arctic sub seafloor methane in - situ and shut down the
methane eruption centres, but it will require massive cooperation between governments and
oil companies to achieve its objectives (Light, 2012c). We are already two years past the tipping
point in August 2010 when massive subsea atmospheric methane eruptions started in earnest
in the Arctic. The next three years are all we have left to try to put a break on the Arctic
methane emissions before the heating effects and sea level rise due to loss of Arctic sea ice and
Arctic methane induced global warming will be completely unstoppable and humanity will be
facing total extinction before the middle of this Century (2050).

Don’t trust their Light evidence- he is not qualified and his paper is based on
false science
Smith 13- Alex Smith, Host of syndicated weekly Radio Ecoshock Show - the cutting edge with
top scientists, authors & activists. Sixth year on the air. Previously a researcher for global
environment group, print journalist, and private investigator, 2013 (“Will Humans Go Extinct
Soon?,” EcoShock Radio, June 5th, Available online at http://www.ecoshock.info/2013/06/will-
humans-go-extinct-soon.html, Accessed 9-11-14)

I'm concerned with the conclusion that we will be wiped out, soon, and especially using
Malcolm Light of the AMEG group as a source.
Malcolm Light's core article, posted on The Arctic Methane Emergency Group web site is titled:
"Global Extinction within one Human Lifetime as a Result of a Spreading Atmospheric Arctic
Methane Heat wave and Surface Firestorm".
In what looks like a scientific paper, complete with complex graphs and charts, we find this
stunning announcement: "The absolute mean extinction time for the northern hemisphere is
2031.8 and for the southern hemisphere 2047.6 with a final mean extinction time for 3/4 of the
earth's surface of 2039.6."
Well that's pretty precise isn't it? On August 2031, say goodbye for those of us in the Northern
Hemisphere. There is a lot in this article. I won't call it a scientific "paper" until I can find
evidence it was peer-reviewed or published in an official journal.
This theory, and the radical solutions I'll discuss in a minute, were presented to the Britain's
Environmental Audit Committee. The UK Parliament responded to this all-party committee's
report "Protecting the Arctic". This AMEG paper went beyond the fringe into serious
consideration by a major government.
What was the Committee told, as published by AMEG? Quoting from Malcolm Light's article,
"This process of methane release will accelerate exponentially, release huge quantities of
methane into the atmosphere and lead to the demise of all life on earth before the middle of
this century."
We get slightly different estimates of when humans will go extinct in notes to one of the
complex graphs in Light's article. It reads:
"Figure 8 shows a different method of interpreting the extinction fields defined by the (12 +-3) +
6% year long lifetime of methane (IPCC, 1992) assumed to have been instantaneously injected
into the Arctic atmosphere in 2010 and the lifetime of the globally spreading methane
atmospheric veil at different methane global warming potentials. The start of extinction begins
between 2020 and 2026.9 and extinction will be complete in the northern hemisphere by 2057.
Extinction will begin around 2024 in the southern hemisphere and will be completed by 2087.
Extinction in the southern hemisphere, in particular in Antarctica will be delayed by some 30
years. This makes property on the Transantarctic mountains of premium value for those people
wish to survive the coming methane firestorm for a few decades longer."
Let's pick up on that key phrase "assumed to have been instantaneously injected into the
Arctic atmosphere in 2010". That leads to a building block of the whole theory, and the panic
to geoengineer the planet.
An unlikely source has disputed the essential point. Somewhere out there in cyberspace is a
person who made tens of thousands of posts on the site democraticunderground.com . She or
he used the name "Muriel Volestrangler" - which just happens to be an alias used by British
comedian John Cleese.
"Muriel" says Malcolm Light cherry picked the very worst single measurement of methane
release in 2010, a startling spike reported by the American agency NOAA, from the Arctic
Svalbard base on the day of November 16th, 2010. Light uses that spike for all his subsequent
frightening calculations.
But here is what New Zealand blogger, Climate Show radio host Gareth Renowden wrote in his
"Hot-Topic" blog about the graph Malcolm Light used:
"The graph comes from NOAA’s Earth System Research Labs Global Monitoring Division’s new
data visualisation web page here (you’ll see a CO2 graph first, but click on the menu to the left
of the graph to get the methane version). The readings for the last year are preliminary, and
shown in brown. The last five data points are so far off the chart that they are almost certainly
going to be rejected as being caused by local contamination. That’s happened before — the
green dots show when — and at the moment other Arctic sites are not showing a similar rise.
However, Svalbard is close to sea floor methane hydrate deposits that are known to be venting
gas".
--- end Quote from Gareth Renowden at hot-topic.co.nz
Muriel Volestrangler tells us the data points showing the big methane spike have been revised
by NOAA, who always said such measurements were provisional. I checked. The spike is gone.
The steady rise over the decades is back. You can see it for yourself at the NOAA site linked
above.
Therefore, most of the calculations in Malcolm Light's paper are not based on reality. They
show what could happen, maybe, if there was ever a big release of methane in the Arctic,
which continued at the new high level. But that hasn't happened at all. There's no doubt
about it. Actual methane levels in the global atmosphere have NOT spiked beyond the gradual
increase expected. No emergency. Yet. Although it could happen...
Volestrangler finds several more gross assumptions, all for the worst, and unprovable
hypothesis in the Malcolm Light paper. Muriel writes:
"But since the basis for it all is a one month anomaly at one measuring station, which has
since been counted as a false reading, it is all based on a mistake. He's taken that one month
error, and extrapolated it until the world has heated up by 14 degrees C, which he says will
therefore be an extinction event."
It's good news if Malcolm's paper is faulty, unless you have an agenda that requires
governments to act in panic.
WHO IS MALCOLM LIGHT? AND WHAT DOES HE WANT?
At AMEG we find only this: "Malcolm Light, specialist in earth sciences, blogger at
globalwarmingmlight.blogspot.com". He claims a PhD from the University of London, but we
are not told in what field.
I checked that blog address, but globalwarminglight has been taken over by companies selling
financial services. Dead end.
An article at the Ecologist Magzine describes Light as an inventor and retired professor. Not
much information there.
In a post at peakoil.com, Light says of himself: "Malcolm Light, retired Arctic oil, mineral and
climate researcher". Getting closer.
I followed up on papers published by Malcolm Light, which he cites in his AMEG posting. Here
is one titled "Saline Fluid Flow and Hydrocarbon Migration and Maturation Related to
Geopressure, Frio Formation." It's from the place where Malcolm Light worked: The Bureau of
Economic Geology at the University of Texas, Austin. He is a petroleum geologist who spent
most of his career in Texas preparing information to assist extraction of fossil fuels.
Now we understand the vision that Malcolm Light is advancing to solve the climate crisis - a
solution which hardly anyone in the environment community knows about.
Digging around on the Net, we find Malcolm Light has two main answers for the methane
emergency. Other scientists like David Archer at the University of Chicago dispute there is any
such emergency at this time.
PROJECT LUCY (IN THE SKY WITH DIAMONDS)
The first he calls "Project Lucy". I found it at the Iowa City Climate Advocates site. In this paper,
Malcolm Light gives himself a new description, quote: "Malcolm Light, PhD (Univ. of London),
consultant with many years of experience in methane, diamonds and numerous related
topics."
Here is Light's solution Number 1. Quote: "Project Lucy therefore aims to design, build and test a
microwave transmission system targeting low-altitude methane clouds with the aim of breaking
the first C-H bond as soon as the methane erupts into the atmosphere from the Arctic Ocean.
The transmitters can be mounted on submarines, planes and after 2015 on boats and drilling
rigs when the Arctic ice cap has melted."
Light also calls for the use of HAARP, the High Frequency Active Auroral Research Program super
radio wave broadcasting facility in Alaska to break up methane. He wants to use the multi-billion
dollar secret military facilities, in both the United States and Russia, to emit a very specific
frequency. Not only will this break up methane into less harmful chemicals, Light claims. It will
create a bi-product of nanodiamonds, tiny flecks of diamonds, in the upper atmosphere, which
would serve to reflect more sunlight away from Earth. Lucy in the sky with diamonds.
Malcolm Light bases this whole radio waves kill methane and makes diamonds theory on
experiments in making artificial diamonds, under high pressure, in laboratories. No one has
ever tried what he suggests. so far I can't find a scientist who can make any sense out of this
proposal in the real world. Perhaps someone could try it and see, if anyone would fund the
research.
At least I like the idea that radio will save the world! That matches my own hopes with Radio
Ecoshock.
ARCTIC DRILLING TO SAVE THE WORLD - THE ANGELS PROJECT
Malcolm Light's second way to save the world from the alleged Arctic Methane Emergency is far
more dire. Characteristic of the answers seen by a petroleum geologist, or big fossil fuel
companies, his solution is to install a massive array of gas drilling rigs in the Arctic, trying to
capture methane from the sea and land, before it can rise up into the atmosphere.
This second plot is has the delightful name "The Angels Project". I found this in a PowerPoint
presentation by Light, titled "A Proposal for the Prevention of Arctic Methane". "ANGELS" is an
acronym for "Arctic Natural Gas Extraction Liquefaction Sales".
The sub-title reads: "A Proposal for the Prevention of Arctic Methane Induced Catastrophic
Global Climate Change by Extraction of Methane from beneath the Permafrost - Arctic Methane
Hydrates and its Storage and Sale as a Subsidized "Green Gas“ Energy Source".
One slide says:
"This 'methane hydrate gun', which is cocked and ready to fire at any moment, is an extremely
serious scenario that will cause abrupt climate change (CCSP, 2008; IMPACTS 2008). Even if this
subsea volume of Arctic methane is released over a longer interval of some ten to twenty years
it will still result in a massive feedback on global warming and drive the Earth on an irretrievable
plunge into total extinction."
Shell Oil, Exxon/Mobil, StatOil, and all the majors will be thrilled to hear we MUST start
massive drilling for natural gas in the Arctic, right away, or go extinct!
Light continues:
"After 2015, when the Arctic Ocean becomes navigable (Figure 5. Piomass in Naumer, 2012) it
will be possible to set up a whole series of drilling platforms adjacent to, but at least 1 km away
from the high volume methane eruption zones and to directionally drill inclined wells down to
intersect the free methane below the sealing methane hydrate permafrost cap within the
underlying fault network."
He predicts huge methane plumes erupting from certain identifiable areas of the Arctic Sea
(from troubled areas of the sea bed). These should be capped instead by drilling rigs, and the
energy used instead of oil or coal.
Quote: "Separated methane is stored in LNG tankers for sale to customers as a subsidised green
alternative to coal and oil for power generation and for air and ground transport."
And here is how the world will be convinced to burn this methane:
"Support should be sought from the United nations, World Bank, national governments and
other interested parties for a subsidy (such as a tax rebate) of some 5% to 15% of the market
price on Arctic permafrost methane and its derivatives to make it the most attractive LNG for
sale compared to LNG from other sources."
"This will guarantee that all the Arctic gas recovered from the Arctic methane hydrate reservoirs
and stockpiled, will immediately be sold to consumers and converted into safer byproducts. This
will also act as an incentive to oil companies to produce methane in large quantities from the
Arctic methane hydrate reserves. In this way the Arctic methane hydrate reservoirs will be
continuously reduced in a safe controlled way over the next 200 to 300 years supplying an
abundant "Green LNG" energy source to humanity."
I note the "safer byproducts" of burning methane (natural gas) is carbon dioxide, the global
warming gas that lasts up to 100,000 years. Malcolm glosses over that.
So... we will run the world from the Arctic methane erupting from clathrates and permafrost
melting? Global warming will force us to capture this massive source of greenhouse gases and
convert our society to methane power. Forget about solar or wind power, tidal, or simple
changes to drastically lower power use. We'll turn lemons into lemonade as the Arctic warms.
We'll spray sulfates too, using aircraft, to cool the Arctic, and hope those radio waves create
some reflective "nanodiamonds" in the upper Troposphere or lower Stratosphere to further
deflect sunlight. It's a total industrial solution to the woes of industrial civilization.
Think about this for a moment or two. As I say, we are forced to extract and burn still more
climate-wrecking fossil fuels or go extinct, Malcolm Light claims. But we only have a limited
amount of carbon we can still burn and stay below two degrees. I doubt there is enough left in
that budge even to build and install this fleet of Arctic drilling rigs. Never mind the huge
emissions required to convert methane into LNG.
No such Arctic drilling has been tried. Shell Oil's attempts last year turned to disaster when
they tried to withdraw their rig before winter. It crashed aground in Alaska.
Beyond that, we haven't a clue whether the methane will really appear mainly along certain
fault lines Malcolm Light has splashed across a colorful graph in his AMEG paper.
None of that research has been done. Does anyone really believe we can capture meaningful
amounts of methane from a geologic release occuring across the vast Arctic tundra, and
coastlines tens of thousands of miles long? The whole world economy would have to be
converted to drilling rigs to capture a tiny percent of the methane. It doesn't make sense.
Adding it up, it's a huge and possibly dangerous fantasy, with near-term human extinction
nowhere in sight.
The UK government responded to the Environment Audit Committee's report by saying they
do not envisage an ice-free Arctic in the next few years, or a collapse of the major ice sheets
on Greenland.
The Government wrote:
"Geo-engineering techniques for the Arctic at present do not offer a credible long term
solution for tackling climate change. Further research is needed to understand how such
techniques work and their wider impacts on climate systems. In the meantime, therefore, we
remain unconvinced that using 'technical fixes' is the right approach and efforts should not be
diverted from tackling the fundamental drivers of global climate change."
Denied.
There are all sorts of curiosities lurking around in the AMEG plan to save the Arctic, and the
world. They suggest the move away from burning dirty bunker oil in ships is a mistake. Ships
should burn the dirtiest, most sulfur-laden fuel we can find, to add more sun-deflecting
clouds. Commercial aircraft, they suggest, should get fuel additives that create chemtrails to
ward off the sun. Unbelievably, they use the word "chemtrails" thus setting off another whole
Internet hive of paranoia.
You can download AMEG's plan for the Arctic as a .pdf file here.
In fact, I think a case can be made that Malcolm Lights paper does two things:
First, it creates a story to bring the public, and even environmentalists, on board for massive
gas drilling in the Arctic. That's something the big oil and gas companies want badly.
But it also stirs up the noise level around climate change, which may be just as important to
the major energy companies. As University of Chicago climate scientist David Archer told us in a
Radio Ecoshock interview, once the public becomes over concerned about methane, coming
from the far away Arctic, we can forget about our own roles in burning carbon every day, in
our cars, homes and factories. It's carbon dioxide that really counts in the long run. Carbon
dioxide, not methane, will last determine the heat of the world for the next 100,000 years -
for millennia after the short-lived methane has broken down.
Mexico Economy Answers
Resilient and rising

Silber, 11 – respected economics and science writer for Scientific American Mind [June,
Kenneth, “Mexico's resilient market: the Mexican stock market has a history of overcoming
economic and financial crises”, Research Magazine p. 44, gale infotrac, AL]

At
The benchmark IPC index, which at the start of 2000 was slightly above 7,000, hit a peak of 31,975 in May 2008 before falling to 19,626 in May 2009. As of December 2009, it was above its pre-crisis peak.

the end of 2010 the IPC hit an all-time high of 38,550 before retreating mildly into the vicinity of
37,000 in the second quarter of 2011. Goldman Sachs has labeled Mexico one of the N-11, a
group of 11 nations poised to follow the four BRICs of Brazil, Russia, India and China in taking on
expanded roles in the world economy. Mexico, along with South Korea, is a more developed economy than
the bulk of the N-11, a diverse set that includes frontier markets such as Nigeria and Bangladesh, and yet Mexico is included in the grouping for its high growth potential. Indeed,
some projections have placed Mexico as the world's fifth largest economy in 2050 , compared to its current
position narrowly missing the top 10. This would put Mexico behind China, the U.S., India and Brazil and ahead of Russia. Mexico may not be a BRIC--though Mexican officials have been known to grumble that the

Notably, such optimism about growth is occurring


category should be called BRICMs--but it is set to carry significant weight whatever the label.

amid a spate of grim news from Mexico about escalating violence by drug cartels. Such violence,
while it holds potential to damage tourism revenues, has had little impact thus far in dampening
investor enthusiasm. As in the past, Mexico's market is showing considerable resilience.
Mexico Instability Answers
No Mexican instability---crime and violence are effects of failed states,
not causes
Neil Couch 12, Brigadier in the British Army, July 2012, “’Mexico in Danger of Rapid
Collapse’: Reality or Exaggeration?” http://www.da.mod.uk/colleges/rcds/publications/seaford-
house-papers/2012-seaford-house-papers/SHP-2012-Couch.pdf/view

A ‘collapsed’ state, however, as postulated in the Pentagon JOE paper, suggests ‘a total vacuum
of authority’, the state having become a ‘mere geographical expression’.16 Such an extreme
hypothesis of Mexico disappearing like those earlier European states seems implausible for a
country that currently has the world’s 14th largest economy and higher predicted growth than
either the UK, Germany or the USA; that has no external threat from aggressive neighbours,
which was the ‘one constant’ in the European experience according to Tilly; and does not suffer
the ‘disharmony between communities’ that Rotberg says is a feature common amongst failed
states.17,18¶ A review of the literature does not reveal why the JOE paper might have
suggested criminal gangs and drug cartels as direct causes leading to state collapse. Crime and
corruption tend to be described not as causes but as symptoms demonstrating failure. For
example, a study for Defense Research and Development Canada attempting to build a
predictive model for proximates of state failure barely mentions either.19 One of the principal
scholars on the subject, Rotberg, says that in failed states, ‘corruption flourishes’ and ‘gangs
and criminal syndicates assume control of the streets’, but again as effect rather than
trigger.20 The Fund for Peace Failed States Index, does not use either of them as a ‘headline’
indicator, though both are used as contributory factors.¶ This absence may reflect an
assessment that numerous states suffer high levels of organised crime and corruption and
nevertheless do not fail. Mandel describes the corruption and extreme violence of the Chinese
Triads, Italian Mafia, Japanese Yakuza and the Russian Mob that, in some cases, has continued
for centuries.21 Yet none of these countries were singled out as potential collapsed or failed
states in the Pentagon’s paper. Indeed, thousands of Americans were killed in gang warfare
during Prohibition and many people ‘knew or at least suspected that politicians, judges,
lawyers, bankers and business concerns collected many millions of dollars from frauds, bribes
and various forms of extortion’.22 Organised crime and corruption were the norm in the
political, business, and judicial systems and police forces ran their own ‘rackets’ rather than
enforcing the law.23 Neither the violence nor the corruption led to state failure.

Mexico is not a failed state but instability is inevitable

Currie ‘11
(Duncan, writer for national review online, “Mexico Agonistes,” April 18)

When discussing the Mexican security crisis, U.S. officials should be careful to avoid hyperbole. Mexico
is nowhere near
being a “failed state,” and the DTO violence, however ghastly, is not a Colombian-style “insurgency.” State and local
police forces remain dangerously weak and corrupt, but Calderón has done much to professionalize the federal
police. Mexican drug mafias have expanded their presence in Central America, but that’s partly because
the Mexican government has been squeezing and battering them at home. Here’s the painful, inescapable
reality: Unless the United States either legalizes drugs or radically reduces its consumption of
them, Mexican DTOs will continue to reap gargantuan profits and exercise tremendous power.
Whenever one gang is dismantled or pushed out of a given city, other traffickers will immediately seek to grab its former territory
and market share. “That’s the thing that’s frustrating to me,” says Shirk. “Every step forward is really part of this never-ending fight
in an unwinnable war.”

No instability – reject media hype

Seelee and Shirt, 10 – *director of the Mexico Institute at the Woodrow Wilson International Center for Scholars AND **
fellow at the center and an associate professor at the University of San Diego (Andrew Selee, David Shirk, 3/27/10, " Five myths
about Mexico's drug war ", Washington Post, http://www.washingtonpost.com/wp-
dyn/content/article/2010/03/26/AR2010032602226.html)

The country has certainly seen a big rise in drug violence, with cartels fighting for control of major narcotics
shipment routes -- especially at the U.S. border and near major seaports and highways -- and branching into kidnapping, extortion
and other illicit activities. Ciudad Juarez, in particular, has been the scene of major battles between two crime organizations and
accounted for nearly a third of drug-linked deaths last year.

But the violence is not as widespread or as random as it may appear. Though civilians with no
evident ties to the drug trade have been killed in the crossfire and occasionally targeted, drug-related
deaths are concentrated among the traffickers . (Deaths among military and police personnel are an estimated 7
percent of the total.) A major reshuffling of leaders and alliances is occurring among the top organized crime groups, and, partly
because of government efforts to disrupt their activities, violence has jumped as former allies battle each other. The
bloodshed
is also geographically concentrated in key trafficking corridors, notably in the states of Sinaloa,
Chihuahua and Tamaulipas.

While the violence underscores weaknesses in the government's ability to maintain security in parts of the country, organized
crime is not threatening to take over the federal government. Mexico is not turning into a failed
state.
Middle East War Answers
Frontline

No risk of Middle East war

Maloney and Takeyh, 7 – *senior fellow for Middle East Policy at the Saban Center for Middle East Studies at the Brookings
Institution AND **senior fellow for Middle East Studies at the Council on Foreign Relations (Susan and Ray, International Herald
Tribune, 6/28, “Why the Iraq War Won't Engulf the Mideast”, http://www.brookings.edu/opinions/2007/0628iraq_maloney.aspx)

Yet, the Saudis, Iranians, Jordanians, Syrians, and others are very unlikely to go to war either to
protect their own sect or ethnic group or to prevent one country from gaining the upper hand in Iraq. The reasons are fairly
straightforward. First, Middle Eastern leaders, like politicians everywhere, are primarily interested in one thing:
self-preservation. Committing forces to Iraq is an inherently risky proposition, which, if the conflict
went badly, could threaten domestic political stability . Moreover, most Arab armies are geared
toward regime protection rather than projecting power and thus have little capability for
sending troops to Iraq. Second, there is cause for concern about the so-called blowback scenario in which
jihadis returning from Iraq destabilize their home countries, plunging the region into conflict. Middle Eastern leaders
are preparing for this possibility. Unlike in the 1990s, when Arab fighters in the Afghan jihad against the Soviet Union
returned to Algeria, Egypt and Saudi Arabia and became a source of instability, Arab security services are being
vigilant about who is coming in and going from their countries . In the last month, the Saudi government has
arrested approximately 200 people suspected of ties with militants. Riyadh is also building a 700 kilometer wall along part of its
frontier with Iraq in order to keep militants out of the kingdom. Finally, there
is no precedent for Arab leaders to
commit forces to conflicts in which they are not directly involved . The Iraqis and the Saudis did send small
contingents to fight the Israelis in 1948 and 1967, but they were either ineffective or never made it. In the 1970s and 1980s, Arab
countries other than Syria, which had a compelling interest in establishing its hegemony over Lebanon, never committed forces
either to protect the Lebanese from the Israelis or from other Lebanese. The
civil war in Lebanon was regarded as
someone else's fight. Indeed, this is the way many leaders view the current situation in Iraq. To Cairo, Amman and Riyadh,
the situation in Iraq is worrisome, but in the end it is an Iraqi and American fight. As far as Iranian mullahs are concerned, they have
long preferred to press their interests through proxies as opposed to direct engagement. At a time when Tehran has access and
influence over powerful Shiite militias, a massive cross-border incursion is both unlikely and unnecessary. So Iraqis will remain
locked in a sectarian and ethnic struggle that outside powers may abet, but will remain within the borders of Iraq. The
Middle
East is a region both prone and accustomed to civil wars. But given its experience with
ambiguous conflicts, the region has also developed an intuitive ability to contain its civil strife
and prevent local conflicts from enveloping the entire Middle East .

No escalation
Fettweis ‘7, Asst Prof Poli Sci – Tulane, Asst Prof National Security Affairs – US Naval War College, (Christopher, “On the
Consequences of Failure in Iraq,” Survival, Vol. 49, Iss. 4, December, p. 83 – 98)

Without the US presence, a second argument goes, nothing would prevent Sunni-Shia violence from sweeping into every country
where the religious divide exists. A Sunni bloc with centres in Riyadh and Cairo might face a Shia bloc headquartered in Tehran,
both of which would face enormous pressure from their own people to fight proxy wars across the region. In addition
to intra-Muslim civil war, cross-border warfare could not be ruled out. Jordan might be the first to send
troops into Iraq to secure its own border; once the dam breaks, Iran, Turkey, Syria and Saudi Arabia might follow suit. The
Middle East has no shortage of rivalries, any of which might descend into direct conflict after a destabilising US
withdrawal. In the worst case, Iran might emerge as the regional hegemon, able to bully and blackmail its neighbours with its new
nuclear arsenal. Saudi Arabia and Egypt would soon demand suitable deterrents of their own, and a nuclear arms race
would envelop the region. Once again, however, none of these outcomes is particularly likely. Wider
war No matter what the outcome in Iraq, the region is not likely to devolve into chaos . Although it might seem
counter-intuitive, by most traditional measures the Middle East is very stable. Continuous,
uninterrupted governance is the norm , not the exception; most Middle East regimes have been in
power for decades. Its monarchies, from Morocco to Jordan to every Gulf state, have generally been in power
since these countries gained independence. In Egypt Hosni Mubarak has ruled for almost three decades, and
Muammar Gadhafi in Libya for almost four. The region's autocrats have been more likely to die quiet , natural
deaths than meet the hangman or post-coup firing squads. Saddam's rather unpredictable regime, which attacked
its neighbours twice, was one of the few exceptions to this pattern of stability, and he met an end unusual for the
modern Middle East. Its regimes have survived potentially destabilising shocks before, and they would be
likely to do so again. The region actually experiences very little cross-border warfare, and even less since
the end of the Cold War. Saddam again provided an exception, as did the Israelis, with their adventures in Lebanon. Israel
fought four wars with neighbouring states in the first 25 years of its existence, but none in the 34 years since.
Vicious civil wars that once engulfed Lebanon and Algeria have gone quiet, and its ethnic conflicts do not
make the region particularly unique. The biggest risk of an American withdrawal is intensified civil war in Iraq rather than regional
conflagration. Iraq's neighbours will likely not prove eager to fight each other to determine who gets to be
the next country to spend itself into penury propping up an unpopular puppet regime next door. As much as the Saudis
and Iranians may threaten to intervene on behalf of their co-religionists, they have shown no eagerness
to replace the counter-insurgency role that American troops play today. If the United States, with its remarkable
military and unlimited resources, could not bring about its desired solutions in Iraq, why would any other country think it could do
so?17 Common interest, not the presence of the US military, provides the ultimate foundation for stability. All
ruling regimes in the Middle East share a common (and understandable) fear of instability. It is the
interest of every actor - the Iraqis, their neighbours and the rest of the world - to see a stable, functioning
government emerge in Iraq. If the United States were to withdraw, increased regional cooperation to address
that common interest is far more likely than outright warfare.

Empirically denied

Yglesisas 7 (Matthew, Associate Editor – Atlantic Monthly, “Containing Iraq”, The Atlantic, 9-12,
http://matthewyglesias.theatlantic.com/archives/2007/09/containing_iraq.php)

Kevin Drum tries to throw some water on the "Middle East in Flames" theory holding that American withdrawal from Iraq will
lead not only to a short-term intensification of fighting in Iraq, but also to some kind of broader regional conflagration. Ivo
Daalder and James Lindsay, as usual sensible but several clicks to my right, also make this point briefly in Democracy: "Talk that
Iraq’s troubles will trigger a regional war is overblown; none
of the half-dozen civil wars the Middle East has
witnessed over the past half-century led to a regional conflagration." Also worth mentioning in this
context is the basic point that the Iranian and Syrian militaries just aren't able to conduct meaningful
offensive military operations. The Saudi, Kuwait, and Jordanian militaries are even worse.
The IDF has plenty of Arabs to fight closer to home . What you're looking at, realistically, is that our allies in
Kurdistan might provide safe harbor to PKK guerillas, thus prompting our allies in Turkey to mount some cross-border military
strikes against the PKK or possibly retaliatory ones against other Kurdish targets. This is a real problem, but it's obviously not a
problem that's mitigated by having the US Army try to act as the Baghdad Police Department or sending US Marines to wander
around the desert hunting a possibly mythical terrorist organization. 

No escalation – their evidence doesn’t take into account new developments

KELLEY 2 (Jack, national security writer for the Post-Gazette and The Blade of Toledo Pittsburgh Post Gazette, April 7)

During the Cold War, there was reason to suppose an Arab-Israeli war could spark a third
world war. In those days, Israel was a client of the United States. The radical Arab states were clients of the Soviet Union. If
the proxies got into a tiff, the conflict could spread to the principals. The closest we came to this was during the Yom Kippur War
of 1973, when Egyptians, in a surprise attack, dealt a severe blow to Israeli defense forces. Only an airlift of M-60 tanks from
U.S. bases in Germany kept Israel from being overrun. Once its initial battle losses had been replaced, Israel quickly regained the
initiative, routing Egyptian and Syrian forces. Israeli troops were poised to take Cairo and Damascus. The Soviets were willing to
permit the United States to restore the status quo ante. But they threatened to intervene to prevent a decisive Israeli victory. So
we prevailed upon the Israelis to stop short of humiliating their enemies. The Yom Kippur War was a near thing for the world.
Only three times in history have U.S. forces gone to DEFCON 1, the highest war footing. The Yom Kippur War was one of those
times. Now the Cold War is over. Russia is a shadow of what we thought the Soviet Union was,
and is more or less an ally in the war on terror. Radical Arabs have lost their sponsor. And
Egypt has, after a fashion, switched sides. There is no longer good reason to suppose a
conflict between Israelis and Palestinians would spread. Another consequence of the Yom
Kippur war was the Arab oil embargo. But the oil "weapon" has lost much of its bang. We are
more dependent upon foreign oil now than we were then, but less dependent on oil from
the Persian Gulf, since new sources elsewhere have been developed. And Arab governments
have become so dependent upon oil revenues that the loss of them would harm Arabs more
than the loss of their oil would harm us.
---Middle East War- No US escalation

US won’t get involved

Ricks ’12 (Thomas E. Ricks, Wednesday, January 18, 2012 - 11:03 Best comment I have heard lately on attacking Iran's nuclear
facilities,
http://ricks.foreignpolicy.com/posts/2012/01/18/best_comment_i_have_heard_lately_on_attacking_iran_s_nuclear_facilities

Planning on attacking Iran? "Better pack a lunch," advises my friend, retired Lt. Col. Terry Daly, who knows a lot about war. His point
was that airstrikes alone against Iranian nuclear facilities wouldn't do much. If you are going to attack Iran, you need to hit its ability
to retaliate, and that means that pretty soon you have a big fat war on your hands. I can't believe we are discussing this. I am hearing
lots of depressing talk that there is a good chance that Israel will attack Iran sometime this year and that we will get sucked into the
ensuing mess. In some ways, there already is a kind of shadow war under way with Iran -- Stuxnet, the drone intrusions, the recent
explosions and assassinations, the sanctions. But for all that, I
just can't see Obama getting us involved in another
Middle Eastern war. The American people certainly have no appetite for it. I think he almost
certainly would lose reelection if a war broke out, because his base would fall apart and the left
would go into opposition. At any rate, an article by my CNAS colleague Colin Kahl that went up last night on the website of
Foreign Affairs argues well that the "containment vs. attack" mindset is a false dilemma. In fact, he
says, even if you attacked Iran, you'd still have to contain it afterward. So a series of airstrikes is not a substitute for containment,
but a prelude to it.
---Middle East War- No War

Nobody wants war

Dunn ’11 (COLLINS DUNN 9/12/11(Michael; PhD, Editor of the Middle East Journal, “Are We Facing a Gathering ‘Perfect Storm’ in
the Region?” MEI Editor’s Blog)

A week ago IDF Maj. Gen. Eyal Eisenberg, the Home Front Commander, made headlines for
saying that Arab Spring could lead to an increase in the chances of a regional war . Other Israeli
officials backpedaled quickly, but in the wake of the deepening crisis between Israel and Turkey and now the attack on the Israeli
Embassy in Egypt, there seems to be a growing sense of tightening siege in Israel. (I know, of course, that residents of Gaza would
find it ironic that Israel feels besieged when they are far more literally so, but the fact is that when Israel feels threatened —
justifiably or not — it has often resorted to military action. Two of Israel's once dependable allies, Turkey and Egypt, are no longer so
dependable for quite different reasons. And the United Nations debate on recognizing the Palestinian Authority as an independent
state is looming, with many members of the European Union likely to support the Palestinian effort, despite US and Israeli
opposition. If Israel feels that it is increasingly isolated, again rightly or wrongly, the dangers of conflict do escalate .
That this is
a dangerous time is indisputable. I may be grasping at straws, but I do find it encouraging that there
really doesn't seem to be any party that wants a war , regional or limited. Some Israelis might
welcome another round in Gaza or against Hizbullah, but probably not just now. While some in
Egypt might welcome a distraction, no one, not even the Islamists, wants a war. The Palestinian
Authority wants legitimacy, not war. Whether the UN ploy brings that closer or makes it more remote is certainly
debatable, and since it's being discussed so many places I haven't felt eager to get into it here. It is, however, going to be a rough
ride, given so many converging uncertainties. One should hope for cool heads and cautious diplomacy, with revolutions still
simmering and Israel jittery.

No impact

The Nation ‘7 [9/24/2007, “Why We Must Leave Iraq,” editorial. http://www.thenation.com/doc/20070924/editors]

As to the concern that a complete withdrawal will lead to regional war , as different countries
intervene in Iraq's civil war: This is a naïvely self-centered view of the Middle East and its
problems. For all its democratic and human rights shortcomings, the region is resilient and
capable of managing conflict. It survived fifteen years of civil war in Lebanon and almost a
decade of brutal war between Iran and Iraq . It will survive the withdrawal of US forces from
Iraq. It was the Saudis and Syrians who in 1989 brokered an end to the war in Lebanon, not us.
And Iran, Syria and Saudi Arabia have the greatest stake in keeping the Iraqi conflict contained
and therefore can be counted on to control their allies in Iraq once US forces withdraw.
---Middle East War- Alt Causes

Can’t Solve- Alt cause Demographics.

Cordesman 11. [Anthony, Arleigh A. Burke Chair in Strategy at the Center for Strategic and International Studies, national
security analyst on a number of global conflicts, former Professor of National Security Studies at Georgetown University, “Stability
and security in Egypt, Libya, Tunisia and the rest of the MENA Region” CSIS -- November 8 --
http://csis.org/files/publication/111102_MENA_Stability_Security.pdf]

The Impact of Demographics and Low Economic Growth Both Muslim and Western states have ceased to focus
on population growth for differing religious and political reasons. As the detailed numbers for population growth in Figure One
show, however, population growth has been truly explosive in virtually every MENA state. It is true that
population growth rates have been dropping, but they are dropping far more slowly than many predicted in the past, and
populations that have grow 4 to 6 times since 1950 will generally double again by 2050. They will do so in nations that have far more
restrictions in terms of water and arable land than most states in the world, and where the policies of past regimes have often kept
economic growth and diversification far below the rates of progress in Asia and Latin America. Governments have failed
both to create the social and political conditions that will reduce population growth and deal
with the growth that has already occurred . As Figure Two shows, the end result is a massive “youth
bulge” that is pouring more and more young men and women into economies that cannot offer
them productive jobs, or meaningful per capita incomes. This is as true of many oil-exporting states as it is of oil poor states.
Even if one ignores the corruption, cronyism, and nepotism that has sharply increased the gap
between rich and poor in most MENA states, and that has lowered the status and security of much of the middle class,
Figure Three shows just how low the CIA World Factbook ranks the average per capita income of
far too many MENA states. It is striking to see just how poor Algeria, Iraq, and Iran now are – largely because of history of
internal conflict, terrible economic policies, and gross over dependence on the petroleum sector. Many other states, however, have
per capita incomes ranking below 100 – a rough indication of serious overall poverty in today’s global economy. This is often
disguised by the growth of modern urban areas, but it is all too real in practice. While there is no direct correlation between poverty
and political unrest, it is all too clear even from these numbers why so many Arabs and Iranians could regard their governments as
having failed them. These
data, however, only hint at how much sharper the disparities between the
richest and poorest MENA states have become . (A sparsely populated Libya with major oil wealth still had a
poverty rate of some 30% of its population before the current political upheavals began. Models that focus on GNP growth or on per
capita income without addressing income distribution are little more than political and economic rubbish.) These
basic
economic pressures interact with hyperurbanization throughout the region which has forced
radical shifts in tribal, ethnic, sectarian and social structures in every state (Saudi Arabia has gone from 8%
urbanization in 1950 to over 80% today). Traditional elements have sometimes adapted, but traditional societies of the kind that
existed before 1950s have virtually vanished. No Arab or Persian cleric, leader, regime, or political party that keeps ignoring these
realities can serve a given nation’s people or create the groundwork for political stability. Political change in Egypt, Libya, and
Tunisia – as is the case with states that so far have been more stable – will fail, or will make the future even
worse if it does not face the need for major improvements in the economy , infrastructure,
education, and governance that can deal with these pressures . No amount of progress in
democracy or human rights can succeed unless these issues are addressed as well .
---Middle East War- No escalation

No Middle East escalation – leaders value stability and use proxy groups instead of militaries,
that’s Fettweis.

Empirics are conclusive. Prefer it --

Best prediction model

Fettweis ‘7, Asst Prof Poli Sci – Tulane, Asst Prof National Security Affairs – US Naval War College,
(Christopher, “On the Consequences of Failure in Iraq,” Survival, Vol. 49, Iss. 4, December, p. 83 – 98)

Firstly, and perhaps most obviously, policymakers should keep in mind that the unprecedented is also
unlikely . Outliers in international behaviour do exist, but in general the past is the best guide to
the future. Since the geopolitical catastrophes that pessimists expect will follow US withdrawal are all
virtually without precedent , common sense should tell policymakers they are probably also
unlikely to occur. Five years ago, US leaders should have realised that their implicit prediction for the aftermath of invasion -
positive, creative instability in the Middle East that would set off a string of democratic dominoes - was without precedent. The
policy was based more on the president's unshakeable faith in the redemptive power of democracy than on a coherent
understanding of international relations. Like all faith-based policies, success would have required a miracle; in
international
politics, miracles are unfortunately rare. Faith is once again driving predictions of post-withdrawal Iraq,
but this time it is faith in chaos and worst-case scenarios.

Secondly, imagined consequences are usually worse than what reality delivers . Human beings tend to
focus on the most frightening scenarios at the expense of the most likely , and anticipate
outcomes far worse than those that usually occur. This is especially true in the United States, which for a variety of
reasons has consistently overestimated the dangers lurking in the international system.3 Pre-war Iraq was no exception;
post-war Iraq is not likely to be either.

Best Middle East methodology

Luttwak ’7, senior associate – CSIS, professor – Georgetown and Berkeley, 5/26/
(Edward, “The middle of nowhere,” Prospect Magazine)

Why are middle east experts so unfailingly wrong? The lesson of history is that men never learn from
history, but middle east experts, like the rest of us, should at least learn from their past mistakes.
Instead, they just keep repeating them. The first mistake is “five minutes to midnight”
catastrophism. The late King Hussein of Jordan was the undisputed master of this genre. Wearing his gravest aspect, he
would warn us that with patience finally exhausted the Arab-Israeli conflict was about to explode, that all
past conflicts would be dwarfed by what was about to happen unless, unless… And then came the remedy—usually
something rather tame when compared with the immense catastrophe predicted, such as resuming this or that stalled negotiation,
or getting an American envoy to the scene to make the usual promises to the Palestinians and apply the usual pressures on Israel.
We read versions of the standard King Hussein speech in countless newspaper columns, hear identical invocations in the
grindingly repetitive radio and television appearances of the usual middle east experts, and are now faced with Hussein’s son
Abdullah periodically repeating his father’s speech almost verbatim. What actually happens at each of these
“moments of truth”—and we may be approaching another one—is nothing much; only the same old cyclical
conflict which always restarts when peace is about to break out, and always dampens down when the violence
becomes intense enough. The ease of filming and reporting out of safe and comfortable Israeli hotels inflates the media
coverage of every minor affray. But humanitarians should note that the dead from Jewish-Palestinian fighting since
1921 amount to fewer than 100,000 —about as many as are killed in a season of conflict in Darfur.

Err neg – even the most extreme wars didn’t escalate

Simon ‘7 (senior fellow, Mid East Studies – CFR, (Steven, http://www.cato.org/pubs/policy_report/v29n4/cpr29n4-4.html)

Another fear is regional chaos. This is something that National Security Advisor Steve Hadley has spelled
out in a number of places, most recently in the Washington Post. The idea is that, if the United States withdraws from
Baghdad, it will be the end of the world as we know it. If you look historically at the experience of
the Middle East, though, civil wars do not turn into regional wars. If you look at Algeria, Yemen
—even Lebanon, where the Israeli and Syrian piece of it was very carefully controlled by both
governments—you did not get a regional war from a civil war.

Regional cooperation solves

Hadar ’11 (Hadar, foreign policy studies – Cato, 7/1/’11 (Leon, http://www.cato.org/pub_display.php?pub_id=13259)

To some extent, the recognition that the United States has lost some of its ability to determine strategic outcomes in the Middle
East has already encouraged regional powers to reassess the wisdom of free riding on American power. Saudi Arabia, together with
its partners in the Gulf Cooperation Council (GCC), has deployed troops to Bahrain to provide support to the regime and
is heading the efforts to stabilize Yemen. Meanwhile, France, a major Mediterranean power, and Britain
have played a leading role in the military operation in Libya to protect their interests in the region. Turkey
has been asserting more forcefully its role as a regional power in multiple ways. Indeed, contrary to the
warning proponents of U.S. military intervention typically express, the withdrawal of American troops from Iraq and
Afghanistan would not necessarily lead to more chaos and bloodshed in those countries. Russia, India and Iran —
which supported the Northern Alliance that helped Washintgon topple the Taliban — and Pakistan (which once backed the
Taliban) all have close ties to various ethnic and tribal groups in that country and now have a common
interest in stabilizing Afghanistan and containing the rivalries. A similar arrangement could be applied to Iraq where
Turkey, Saudi Arabia and Iran share an interest in assisting their local allies and in restraining
potential rivals — Shiites, Sunnis, Kurds and Turkmen — by preventing the sectarian tensions in Iraq from
spilling into the rest of the region. Hence, Turkey has already been quite successful in stabilizing and developing
economic ties with the autonomous Kurdish area of Iraq while containing irredentist Kurdish pressures in northern Iraq and
southern Turkey and protecting the Turkmen minority. And Turkey, together with Saudi Arabia and Iran, has played a critical role
toward forming a government in Baghdad that recognizes the interests of Shiites, Sunnis, and Kurds. The United States should take
part in any negotiations leading to regional agreements on Afghanistan and Iraq, a process that could also become an opportunity to
improve the relationship with Iran. Such an approach has the potential to demonstrate that regionalism, as opposed to
American hegemonism, could be more beneficial to U.S. interests as well as to the governments and people of the
Middle East and Central Asia.
---Middle East War- Russell Answers

Russell concludes neg. (deterrence solves – this matters because he cites the aff’s article and
then refutes it)

Russell ‘9 [James, senior lecturer, National Security Affairs – Naval Postgraduate School, managing editor – Strategic Insights,
“Extended Deterrence, Security Guarantees, and Nuclear Weapons: U.S. Strategic and Policy Conundrums in the Gulf,” Strategic
Studies Institute) [footnote 26 included]

The build out of the U.S. military infrastructure points around the region provide the hosting states with
tangible evidence of the credibility of the American military commitment to their security. The
military footprint today in the Gulf is no “trip-wire” force, but is engaged in tangible military operations,
such as the multi-national maritime security operations conducted in the Gulf and the Arabian Sea by the combined task force
command operating out of the 5th Fleet Headquarters in Manama. Since the British withdrawal from the Gulf in the early 1970s,
the United States has repeatedly demonstrated its willingness to deploy its conventional forces to
the region in response to regional instability. Starting with Operation Earnest Will in 1988, the United States slowly
but inexorably inserted itself into the role played by the British for over a century as protecting the Gulf States from external threats.
Following Operation Desert Storm, the United States kept sufficient forces in theater to enforce the
United Nations’ cease fire resolutions on a recalcitrant Saddam. Last, but not least, it flowed significant forces
and absorbed the monetary costs of toppling Saddam and providing a protective conventional
force that can be readily called upon by the Iraq regime if needed. Given this history it is difficult to see how any state
could doubt the credibility of the United States’ commitments to use its conventional forces as an instrument of
regional defense. This history suggests an overwhelming emphasis on the role of conventional force in operationalizing American
security guarantees and extended deterrent commitments. Inthe Gulf—unlike Northeast Asia—the role of nuclear
weapons has never been explicitly spelled out and has very much remained in the background. However, while
reference to nuclear weapons might remain unstated, the reality is that they are explicitly committed to
defend American forces whenever the commander-in-chief might deem it necessary. The entire (and substantial)
American military regional footprint operates under a quite explicit nuclear umbrella—headlines
or no headlines. If a nuclear umbrella is indeed draped over America’s forward deployed Gulf presence,
it’s hard not to see how that umbrella is similarly draped over the states that are hosting those forces. The only
problem with Secretary Clinton’s recent statements is that she seems unaware of this fact, i.e., the United States already maintains a
nuclear umbrella backed by nuclear weapons in the region. While the United States has pledged not to use nuclear weapons
against non-nuclear signatories of the Nuclear Nonproliferation Treaty (known as negative security assurances), it maintains a
policy of calculated ambiguity in honoring those commitments if its forces are attacked by chemical or
biological weapons.[24] President Clinton reinforced this position in Presidential Decision Memorandum 60 in December 1997,
which stated: state-parties to the Treaty on the Nonproliferation of Nuclear Weapons, except in the case of an invasion or any other
attack on the United States, its territories, its armed forces or other troops, its allies, or on a state toward which it has a security
commitment carried out, or sustained by such a non-nuclear-weapon state in association or alliance with a nuclear-weapon state.
[25] As previously noted, the
United States last unsheathed this proverbial sword in 1996 with the discovery of a potential
chemical weapons plant in Libya. The sword, however, remains at the ready in the Gulf where Iran’s
development of chemical weapons, long-range missiles, and its emphasis on terrorism and asymmetric warfare constitute
prominent elements of the regional threat environment. If anything Iran’s weakened conventional forces
potentially drive Iranian military responses during an armed conflict to those weapons that would lead the United States to consider
forswearing its negative security assurances.[26] [Footnote begins] 26. As
argued in James A. Russell, “Strategic
Stability Reconsidered: Prospects for Escalation and Nuclear War in the Middle East,” Institut Français des Relations
Internationales Proliferation Papers 26 (Spring 2009). [Footnote ends] In this scenario, it seems clear that American
nuclear weapons are a component in the web of military capabilities designed to discourage
Iranian use of its unconventional weapons in war. Conclusion Nuclear weapons have historically helped
implicitly and explicitly support America’s far flung global commitments in the Gulf and elsewhere. The
system of Gulf security built by the United States reflects a time-honored template of regional defense
and security honed in decades of Cold War experience. In the Gulf, the dual tools of extended deterrence
and security assurances have proven a cornerstone of a system of regional security efficiently
administered by America’s military organizations. Nuclear weapons today undeniably form part of this system—
explicitly protecting U.S. forces and implicitly protecting regimes hosting those forces.
---Middle East War- Israel Conflict Answers

No Israel conflict- no side wants to go to war

Sappenfeld ‘6 [Mark, “Wider war in Middle East? Not likely.” Christian Science Monitor]

Of the dangers presented by the conflict between Israel and Hizbullah in southern Lebanon, the
possibility of a broader Middle East war is among the less likely . In the 1967 Arab-Israeli war –
and repeatedly since – Israel has shown its clear military supremacy. So dominant has been
Israel's advantage in both technology and tactics that former foes such as Jordan and Egypt
sued for peace in those wars, while Tel Aviv's avowed enemies – Syria and Iran – have turned to
backing terrorists. At this moment, the calculus doesn't appear to have changed . There is no
coalition of Arab governments willing to unite militarily against Israel. Syria's military prowess
has crumbled since the fall of the Soviet Union – its greatest benefactor – while Iran remains too
geographically remote to strike effectively. The result is a new paroxysm of the proxy war that
has existed in the region for a generation – ebbing and flowing as Hizbullah, armed and financed
by Iran and Syria, harass Israel without provoking a major Middle East war, military analysts say.
"No state is willing to deal with Israel conventionally," says Seth Jones, a terrorism expert at
the RAND Corp. The shape of the conflict so far – sparked by Hizbullah's raid into northern Israel and capture of two Israeli
soldiers – reveals both the capabilities and limitations of each side. Historically, Hizbullah has been able to do little more than nip at
Israel's northern border with incursions and sporadic rocket attacks. By and large, its arsenal is primitive, comprising various short-
range rockets that can destroy buildings only with a direct hit, yet are difficult to aim with any precision. It has continually fired
rockets into northern Israel. Hizbullah's longer-range rockets Yet there are signs of increasing sophistication, perhaps due to help
from Iran, experts say. On Friday, Hizbullah launched a more advanced missile, which struck an Israeli warship. Hizbullah rockets are
also penetrating deeper into Israel than ever before, with several striking Haifa, Israel's third-largest city, on Sunday. Israel claims
that four of the missiles were the Iranian-made Fajr-3, with a 28-mile range. For its part, Israel has so far relied mostly on air strikes
as its military response. Monday, Israel acknowledged that its forces had invaded Lebanon, though they returned shortly after. Israel
invaded southern Lebanon in 1982. Its army occupied the territory for three years, then withdrew because of the strain of the
occupation and broad international condemnation. History also offers a note of caution to Israel's foes. In 1967, Israel responded to
Egyptian aggression by taking the Sinai Peninsula and the Gaza Strip from Egypt, the West Bank from Jordan, and the Golan Heights
from Syria. Years later, when Syria and Israel fought over control of Lebanon in 1982, Israeli jet fighters reportedly shot down 80
Syrian planes without losing any of its own. Israel's military superiority is built on American support and a skill honed by decades of
fighting for the very existence of the nation. Israel receives the best equipment that the United States can offer its allies. " They
have some of the most highly advanced weapons systems in the world," says Dr. Jones . Israel's
air force, in particular, has no rival in the region , which makes air strikes the most effective – and most probable –
means of Israeli retaliation and aggression. Yet Israel has so far focused most of its attacks on Lebanon , despite
Hizbullah's links to Syria and Iran. Indeed, both sides have long used Lebanon as a way to harass the other, since Lebanon's military is
almost irrelevant, analysts say. Even
though Israel accuses Syria and Iran of backing Hizbullah's attacks,
it hesitates to attack them directly. The reason is simple: Though Syria's aging military is no match for Israel's, it has
missiles that could strike any part of Israel, as well as stocks of chemical weapons. Moreover, the 60 miles from the Israeli bordewr
to the Syrian capital of Damascus is one of the most heavily fortified zones in the world. "Syria doesn't have the capacity to win [a
war against Israel], but it can cause lots of suffering," says Nadav Morag, former senior director for domestic policy in the Israel
National Security Council. Iran more formidable than Syria By contrast, Iran presents a far more formidable challenge – but one that
is so remote from Israel geographically as to make hostilities difficult. As with Syria, Iran's greatest threat lies in its missiles. Yet the
prospect of firing missiles at America's greatest ally – at a time when it is surrounded by American forces in Iraq and Afghanistan – is
decidedly risky. Likewise, the
notion of an Israeli air strike against Iran presents enormous logistical
hurdles. Although Iran does not possess a credible air force and has only mid-grade Russian air-defense systems to contend with
Israeli jets, Israel would surely be denied overfly rights by the Arab countries that surround them ,
meaning it would have to take a circuitous and difficult oversea route to Iran. It would probably be a measure taken only as a last
resort., Mr. Morag says.

Israel faces no threat and nobody will provoke them to lashout

Cetron and Davies ’7 (Cetron, Marvin J. is president of Forecasting International ; and Davies, Owen former senior editor at
Omni magazine and is a freelance writer The Futurist 9-1-07

If Israel ever needed America's unquestioning patronage, it no longer does, despite its embarrassing
inability to defeat Hezbollah in the Lebanon conflict of July 2006. Prime Minister Ehud Olmert made that clear
when (apparently inadvertently) he admitted in December 2006 that his country does indeed possess nuclear
weapons. Israel's neighbors may be hostile, but if they sometimes are suicidal as individuals, as
nations they are not. Even if they are capable of destroying Israel, as Iran may soon be, they will take
no action against it that would provoke a nuclear response.
---Middle East War- Israel Palestine Answers

No spillover – self-interest overwhelms

Satloff ‘6 (Satloff, Washington Institute for Near East Policy, executive director, 2006, Robert Satloff, "Forget the Domino
Theories," 12/9/06

Linkage also has its more ominous side. The


most common is the fear that, left unresolved, the Israeli-Palestinian
conflict could explode into a Middle East-wide war . A second variation locates the epicenter of regional instability
in the Persian Gulf. A generation ago the fear was that the export of Iran's Islamic revolution would undermine pro-West Arab
states. Today, as Defense Secretary Robert Gates said during his confirmation hearings, the fear is that Sunni-Shiite violence in Iraq
will spread like a contagion through the region, leaving ethnic bloodletting in its wake.
The problem with all these theories is
that after a generation of theorizing about Middle East dominoes, the evidence is piling up: The
linkages simply don't exist. First, military success in the Gulf does not translate into diplomatic success in the region. The
Madrid process, a regional initiative, may have had a promising opening session, but once it got down to real bargaining, it ran up
against the stark realities of the Israeli-Palestinian and Israeli-Syrian divides. And the idea of building on Saddam Hussein's demise to
promote change on the Israeli-Palestinian front may have hastened Yasser Arafat's movement into irrelevance, but it ran aground on
the dismal intra-Palestinian political realities of what followed him. Second, local disasters do not translate into regional disasters.
Despite Iran's subversion, terrorism, bullying and threats, every Arab state survived the export
of the Islamic revolution. And despite the near unanimity of received wisdom about the Middle
East, there is no evidence to support the proposition that Israeli-Palestinian violence has
substantial regional repercussions, let alone that it could lead to regional war . The best evidence
for this counterintuitive conclusion comes from the Palestinian uprising that began after the collapse of
the Camp David summit in 2000. With more than 3,000 Palestinian and 1,000 Israeli fatalities, the
bloodshed in the subsequent three years was the worst in the history of the Israeli-Palestinian
conflict. Yet the regional impact was virtually zero. Not one Arab state threatened to fight
alongside the Palestinians, and none even came to their aid militarily ; indeed, only faraway Iran tried to
send weapons. The Arab "street" did not rise in protest. Neither Jordan nor Egypt severed its peace treaty with
Israel, and no Arab state faced significant protests . The conflict -- certainly a horrible experience for Israelis
and Palestinians -- was contained. The lesson of the past generation is that most states in the Arab Middle
East have grown stronger, not weaker. Arab leaders are interested first and foremost in survival,
which means protecting their national interests, not subscribing to romantic notions of ethnic or religious
ideology. That is why, for example, Gates's warning about Arab states intervening in Iraq to defend
fellow Sunni Arabs from Shiite ethnic cleansing is far-fetched.

Threats of regional spillover are hollow

Westbrook 2 (Bob, “Middle East War Scenario”, 2-19, http://www.trumpetsounds.com/warscenario.html)

Though these actions would trigger an


Israel/Palestinian war, it would not necessarily initiate a broader
Middle East war. You may have noticed that until now, most of the major players among the Arab nations
have been content to make a lot of noise but do little on behalf of the Palestinians. Whether the
Arab/Muslim nations would take up arms against Israel to "protect" the Palestinians is debatable.
Peace has no impact on regional stability

Luttwak 7 (Edward, Senior Associate – Center for Security and International Studies, “The Middle of Nowhere”, Prospect
Magazine, May, http://www.prospect-magazine.co.uk/article_details.php?id=9302)

Yes, it would be nice if


Israelis and Palestinians could settle their differences , but it would do little or
nothing to calm the other conflicts in the middle east from Algeria to Iraq, or to stop Muslim-Hindu violence in
Kashmir, Muslim-Christian violence in Indonesia and the Philippines, Muslim-Buddhist violence in Thailand, Muslim-animist
violence in Sudan, Muslim-Igbo violence in Nigeria, Muslim-Muscovite violence in Chechnya, or the different varieties of inter-
Muslim violence between traditionalists and Islamists, and between Sunnis and Shia, nor would it assuage the perfectly
understandable hostility
of convinced Islamists towards the transgressive west that relentlessly invades their minds, and
sometimes their countries. Arab-Israeli
catastrophism is wrong twice over, first because the conflict is
contained within rather narrow boundaries, and second because the Levant is just not that
important any more.
---Middle East War- Lebanon/Hezbollah/Syria

These specific conflicts won’t escalate

Ferguson 6 (Niall, Professor of History – Harvard University and Professor – Harvard Business School, “This Might Not Be a
World War, But It Still Needs a Sense of Urgency”, The Telegraph, 7-23,

http://www.telegraph.co.uk/opinion/main.jhtml?xml=/opinion/2006/07/23/do2302.xml)

Could today's quarrel between the Israelis and Hezbollah over Lebanon produce a Third
World War? That's what the former Republican Speaker of the House of Representatives, Newt Gingrich, called it last week,
echoing earlier fighting talk by Dan Gillerman, Israel's Ambassador to the United Nations.Such language can - for now, at least -
safely be dismissed as hyperbole. This
crisis is not going to trigger another world war. Indeed, I do not
expect it to produce even another Middle Eastern war worthy of comparison with those of June
1967 or October 1973. In 1967, Israel fought four of its Arab neighbours, Egypt, Syria, Jordan and Iraq. In 1973, Egypt and Syria
attacked Israel. Such
combinations are very hard to imagine today. Nor does it seem to me likely
that Syria and Iran will escalate their involvement in the crisis beyond continuing their
support for Hezbollah. Neither is in a position to risk a full-scale military confrontation with
Israel, given the risk that this might precipitate an American military reaction . Crucially,
America's consistent support for Israel is not matched by any great power support for Israel's
neighbours. During the Cold War, by contrast, the risk was that a Middle Eastern war could spill
over into a superpower conflict. Henry Kissinger, secretary of state in the twilight of the Nixon presidency, first
heard the news of an Arab-Israeli War at 6.15am on October 6, 1973. Half an hour later he was on the phone to the Soviet
ambassador in Washington, Anatoly Dobrynin. Two weeks later Kissinger flew to Moscow to meet the Soviet leader, Leonid
Brezhnev. The stakes were high indeed. At one point during the 1973 crisis, as Brezhnev vainly tried to resist Kissinger's efforts
to squeeze him out of the diplomatic loop, the White House raised America's state of military readiness to Defcon 3, putting its
strategic nuclear forces on high alert. It is hard to imagine anything like that today. In any case, this crisis may soon be over.
Most wars Israel has fought have been short, lasting a matter of days or weeks (six days in 1967,
three weeks in 1973).
---Middle East War- Egypt War Answers

Egypt will never go to war

Lachman ’11 (Sol Lachman, writing for the Jewish Reporter, “Why a new Israeli-Egyptian war isn’t likely,” 5/11/2011,
http://thejewishreporter.com/2011/05/11/why-a-new-israeli-egyptian-war-isnt-likely/

Hana Levi Julian reported Wednesday of the word on the Egyptian street favoring an end to the peace agreement with Israel; the
folk of the Arab Spring. While that is obviously true, there is more to international relations than vox populi,
especially in a country like Egypt that has yet to find its way to anything resembling democracy. I certainly don’t disagree
with anything Julian wrote, but rather come to add some additional ideas to the mix. The polls not withstanding , hating
Israel doesn’t make Egypt any more prepared for war than she was yesterday. Actively breaking
the conditions of the peace treaty with Israel could cost Egypt billions in U.S. aid, wrecking the
fragile Egyptian economy. Let’s not forget what fuels this peace. Israel pays Egypt well for the gas and oil
it receives. Gaza will not pay or be able to pay. Hatred of Israel is not likely to win over the wallet. Or will it? After all this is the
Middle East. The United Nations is not ‘’fair and balanced’’ like a U.S. cable news network claims to be. Yet it would not be
simple to abrogate an internationally brokered treaty that has within it embodied several UN
Security Council resolutions. Condemnation might not be universal, but there would likely be
some negative repercussions for Egypt in that august forum. Egypt has lost every war it has fought, and
not only with Israel. No one wants war, but it is certain that the Egyptian military will consider well the
consequences of war with Israel. In the past, Egypt has only made war with Israel when supported by other allies. Jordan
is entirely unlikely to do anything. Syria, Egypt’s strongest ally is in disarray, and Iraq is out of the picture, at least for the time being.
Libya has her own confusion. Iran? The Saudis? Both unlikely partners. Hezbollah may be the strongest power in Lebanon, but would
they risk it all to support Egypt in a war? Hamas is even less of a factor. Right now, Egypt is disorganized. It may be quite useful to
create a semblance of unity by utilizing hatred of Israel. They can all safely agree. Someone once said, “All of diplomacy comes down
to how many divisions do you have and how many are you willing to lose?” Keeping tabs on Egyptian public opinion is of great
importance, but a war is unlikely at this time.
---Middle East War- Iraq War Answers

No impact to Iraq --- all powers will cooperate and prevent civil war

Hadar ’11 (7/1/11—former professor of IR at American U and Mount Vernon-College, PhD in IR from American U Leon Hadr,
Saving U.S. Mideast Policy, )

Indeed, contrary to the warning proponents of U.S. military intervention typically express, the withdrawal of American
troops fromIraq and Afghanistan would not necessarily lead to more chaos and bloodshed in those countries.
Russia, India and Iran—which supported the Northern Alliance that helped Washintgon
topple the Taliban—and Pakistan (which once backed the Taliban) all have close ties to various ethnic
and tribal groups in that country and now have a common interest in stabilizing
Afghanistan and containing the rivalries. A similar arrangement could be applied to Iraq where Turkey,
Saudi Arabia and Iran share an interest in assisting their local allies and in restraining
potential rivals—Shiites, Sunnis, Kurds and Turkmen— by preventing the sectarian tensions
in Iraq from spilling into the rest of the region . Hence, Turkey has already been quite
successful in stabilizing and developing economic ties with the autonomous Kurdish area of
Iraq while containing irredentist Kurdish pressures in northern Iraq and southern Turkey and
protecting the Turkmen minority. And Turkey, together with Saudi Arabia and Iran, has played
a critical role toward forming a government in Baghdad that recognizes the interests of
Shiites, Sunnis, and Kurds.

See Iraq Answers


Middle East Prolif Answers
No Middle East prolif

Miklos 13
(Timothy, 2nd year M.A. student at the Elliott School of International Affairs in Security Policy Studies with a focus on nuclear
weapons. He holds a bachelor’s degree in Political Science from the University of Michigan. Prior to his undergraduate studies he
served honorably in the United States Marine Corps for 8 years. He is fluent in Russian. “Iran Proliferation Triggering a Nuclear
Domino Effect in the Middle East: An Unrealistic Scenario.” 3/3/13, International Affairs Review. http://www.iar-gwu.org/node/468)

President Obama has stated that Iranian acquisition of a nuclear weapon will spark an arms race in the Middle East. This view is
a status quo dogma among policymakers of both the Republican and Democratic parties, and dissenting views are
generally ignored. Ari Shavit of Haaretz identifies the most at-risk states as Egypt, Saudi Arabia, and Turkey. However, a
nuclear arms race in the Middle East in response to an Iranian weapon is highly unlikely. For those countries
most likely to proliferate, the political and financial costs are too high. The nuclear domino scenario has been an
accepted doctrine since 1962 when President Kennedy warned that by the 1970s there would be around 25 nuclear weapon states.
Yet, today there are only nine. According to a recent Center for a New American Security (CNAS) report, “Cairo does not see Iran’s
nuclear ambitions as an existential threat.” Egypt’s true enemy is Israel, which has defeated Egypt in four consecutive wars. If Egypt
did not pursue a nuclear option to deter its nuclear-armed enemy Israel, then it will not do so against Iran. Egypt simply does
not have the financial resources, nuclear infrastructure, or motive to build a successful clandestine nuclear
program, as its facilities are under IAEA safeguards. As a signatory of the Treaty on the Non-Proliferation of Nuclear
Weapons (NPT), Egypt has remained committed to non-proliferation since the Treaty’s inception and would be unlikely to
withdraw. Even if Egypt had the capability and intention to pursue nuclear weapons, its security would not be enhanced. An
attempted breakout would likely be destroyed in a preemptive strike by Israel, which has proven the credibility of this threat twice
by destroying the Osirak reactor in Iraq in 1981 and the Al Kibar reactor in Syria in 2007. Unlike Iran, Egypt does not have long
distances, deep reactors, and strong air defenses to protect itself from Israeli preemption. Iran poses the largest threat to Saudi
Arabia and, as such, the Kingdom would have the strongest security motive to pursue a deterrent. Riyadh has called on a Nuclear
Weapon Free Zone (NWFZ) in the Middle East, yet has repeatedly warned that an Iranian nuclear weapon may compel it to follow
suit. This is not credible and is likely an attempt to pressure the United States to take greater action against Tehran. According
to Philipp Bleek of the Monterey Institute, “states whose rivals pursue or acquire nuclear weapons are much likely to themselves
explore a nuclear weapons option…but are no more (or less) likely to pursue or acquire nuclear weapons” ("Why do states
proliferate?," Forecasting Nuclear Proliferation in the 21st Century: Volume 1 The Role of Theory). Nasser of Egypt made a similar
threat in response to Israel’s nuclear program and explored Egypt’s nuclear possibilities, but in 1968 chose to sign the NPT instead.
Saudi Arabia has virtually no domestic nuclear infrastructure, resources, or knowledge base to conduct
a “crash” program. It is also an NPT state and has many U.S. military and foreign investors on its territory, making it
difficult to support such a program. Its only option would be to purchase a nuclear weapon from Pakistan. However, Islamabad
is unlikely to spare any weapons, as they are needed to deter India. Additionally, selling a nuclear weapon would bring
world condemnation on Pakistan and leave it a pariah state surrounded by nuclear enemies. Riyadh would risk losing the support of
the United States if it were to attempt to pursue a deterrent, leaving it open to an Israeli strike. Instead, Saudi Arabia will
likely rely
on its preferred weapons of “cash and diplomacy,” finding the U.S. nuclear umbrella a “more attractive offer.”
Turkey is a NATO member with around 70 tactical nuclear weapons on its soil and is protected by the U.S. nuclear
umbrella. An indigenous nuclear program would forfeit this position. Etel Solingen (“Domestic Models of Political Survival,"
Forecasting Nuclear Proliferation in the 21st Century: Volume 1 The Role of Theory) asserts that states with integrated economies
face greater costs to proliferating and are therefore less likely to do so. There
is too much at stake for these
nations to develop nuclear weapons, as they each stand to suffer great financial and political losses and will ultimately
be less secure because of it. The United States has far greater influence over these nations than it does over Iran. Washington should
keep the pressure on Tehran to adhere to IAEA safeguards. However, alarmist rhetoric of a Middle East arms race
is unjustified and not conducive to reaching an agreeable diplomatic settlement with Iran.

No Middle East prolif

Jones ‘2-18
(Peter, an associate professor in the Graduate School of Public and International Affairs at the University of Ottawa. He is also an
Annenberg Distinguished Visiting Fellow at the Hoover Institution at Stanford University. “Rapid nuclear proliferation simply doesn’t
happen.” The Globe and Mail)

Among the many reasons why Iran should not acquire nuclear weapons (a sentiment with which any reasonable person must agree),
one hears the argument that it would initiate a cascade of proliferation across the Middle East. First Saudi Arabia, then Turkey, then
Egypt, then God knows who would inevitably acquire nuclear weapons – and quickly. So goes the conventional wisdom expressed by
Israeli Prime Minister Benjamin Netanyahu, Stephen Harper and any number of hawkish think-tank experts. There is
considerable historical evidence to suggest that this would not happen. If it did, it would take a
long time. Talk of rapid proliferation across the region is simply not apt. Since the dawn of the nuclear era, various leaders and
analysts have predicted that nuclear proliferation would take place rapidly and inexorably. Those countries that could build the
bomb would do so, and others would build it in response. It has been predicted that almost 50 countries would eventually join the
nuclear club alongside the five nuclear-weapon states recognized under the Nuclear Non-Proliferation Treaty. That prediction has
proved wrong. Only four additional countries – India, Pakistan, Israel and North Korea – have acquired nuclear
weapons. One country unambiguously tried and was stopped (Iraq, before it was foolish enough to invade Kuwait). In each case,
the reasons why these countries decided to build nuclear weapons had to do with the specifics of their security situations rather
than a reflex action. This record is hardly cause for celebration but also hardly the proliferation threat so often forecast. Moreover,
neighbours were threatened when these countries acquired nuclear weapons but decided not to build nuclear weapons in response.
Japan and South Korea did not build them after China and then North Korea did, despite
chilling rhetoric from the one-party states that easily matched anything Iranian President Mahmoud Ahmadinejad has said
about Israel. No Arab country built them after Israel did. Yes, Pakistan followed India into the nuclear club,
but no other country in the region has. Rather than build their own bombs, most countries faced with neighbours
acquiring nuclear weapons have sought alliances and protection from others – most often the United States.
Thus, contrary to popular wisdom, experience has been that most states do not build nuclear bombs, even when they have the
opportunity and, seemingly, the motive to do so. If there is a norm of international conduct regarding nuclear weapons, it is a norm
of non-proliferation. For every state that has developed nuclear weapons, there are dozens more,
including Canada, that could have but did not. South Africa did but then gave them up. There are several, including Brazil,
Argentina and Sweden, that went down the road toward nuclear weapons but stopped and went back. There are even a few –
Ukraine, Belarus and Kazakhstan – that inherited nuclear weapons when the Soviet Union collapsed but soon gave them up
voluntarily. And yet, in the Iranian case, we continue to be captivated by an argument that widespread regional proliferation is
inevitable. Why? In part, it is because of what some scholars of proliferation call a “proliferation narrative” that has gripped the
majority of analysts and practitioners of international affairs. This narrative, which focuses on power as the key element of
international affairs, holds that nuclear weapons are the ultimate expression of power; that states seek to maximize their power;
and so states, all other things being equal, will want nuclear weapons. Moreover, states facing a nuclear-armed foe will almost
certainly want their own bomb. Despite decades of evidence to the contrary, this narrative continues to hold sway in large parts of
the academic and practitioner communities. Another reason may be that pointing to an inevitable proliferation cascade is ample
justification for those who wish to attack Iran to do so. It is a powerful narrative: “Yes, an attack on Iran may be dangerous, uncertain
and could lead to a regional war, but far better to try to stop the Iranians from getting the bomb before X number of other [whisper
when you say this] Muslim countries across the Middle East decide to build bombs as well.” But would they? And could they do it
quickly? Despite some musings by some members of the Saudi Royal Family, it is debatable as to whether any other Middle East
other states in the region did
country would automatically decide to build a bomb if Iran ever did. Moreover, even if
decide to build bombs, it would take decades for them to do so. Experience seems to show that most of them
would eventually counter an Iranian bomb by moving even further into the security embrace of the United
States – an outcome profoundly at odds with Iran’s interests. The idea of a rapid and inevitable proliferation
cascade across the Middle East is simply not reasonable – but it works well as a scare tactic to justify a war
with Iran that might otherwise be a hard sell to a war-weary American public.

Capacity restraints check Middle East prolif

Cook 12 – the Hasib J. Sabbagh Senior Fellow for Middle Eastern Studies at the Council on Foreign Relations (Steven A., 04/02,
“Don't Fear a Nuclear Arms Race in the Middle East,”
http://www.foreignpolicy.com/articles/2012/04/02/don_t_fear_a_nuclear_arms_race)

Multiple nuclear powers on a hair trigger in the Middle East -- the most volatile region on earth, and one that is undergoing massive
political change -- is a nightmare scenario for U.S. and other security planners, who have never before confronted a challenge of

such magnitude. But thankfully, all the dire warnings about uncontrolled proliferation are -- if not exactly
science fiction -- further from reality than Shavit and Obama indicate. There are very good reasons for the
international community to meet the challenge that Iran represents, but Middle Eastern nuclear dominoes are not one of them.
Theorists of international politics, when pondering the decision-making process of states confronted by nuclear-armed neighbors,
have long raised the fears of asymmetric power relations and potential for nuclear blackmail to explain why these states would be
forced to proliferate themselves. This logic was undoubtedly at work when Pakistan embarked on a nuclear program in 1972 to
match India's nuclear development program. Yet for all its tribulations, the present-day Middle East is not the tinderbox that South
Asia was in the middle of the 20th century. Pakistan's perception of the threat posed by India -- a state with which it has fought four
wars since 1947 -- is far more acute than how either Egypt or Turkey perceive the Iranian challenge. And while Iran is closer to home
for the Saudis, the security situation in the Persian Gulf is not as severe as the one along the 1,800-mile Indo-Pakistani border. Most
important to understanding why the Middle East will not be a zone of unrestrained proliferation is the significant difference
between desiring nukes and the actual capacity to acquire them. Of all three states that Shavit mentioned, the one on virtually
everyone's list for possible nuclear proliferation in response to Iran is Turkey. But the Turkish Republic is already under a nuclear
umbrella: Ankara safeguards roughly 90 of the United States' finest B61 gravity bombs at Incirlik airbase, near the city of Adana.
These weapons are there because Turkey is a NATO member, and Washington's extended deterrence can be expected to at least
partially mitigate Turkey's incentives for proliferation. But even if the Turks wanted their own bomb, they have
almost no capacity to develop nuclear weapons technology. Indeed, Turkey does not even possess the capability to
deliver the 40 B61 bombs at Incirlik that are allocated to Turkish forces in the event of an attack, according to a report released by
the Carnegie Endowment for International Peace. Given the changes in Turkey's foreign policy and its drive for global influence, it is
conceivable that it will want to develop a Turkish version of France's force de frappe. However, Ankara would literally be starting
from scratch: Turkey has no fissile material, cannot mine or enrich uranium, and does not possess the technology to reprocess spent
fuel, all of which are required for nuclear weapons development. This does not mean that Turkey is not interested in nuclear
technology. Yet Ankara's efforts, to the extent that they exist beyond the two small-scale facilities in Ankara and Kucukcekmece, are
directly related to the country's predicted energy shortfall resulting from the combination of a booming economy and growing
population. The Turkish government has announced plans for civilian nuclear power to provide a quarter of Turkey's electricity
needs by 2040. But even this three-decade timeline seems overly optimistic given the inchoate nature of Turkey's nuclear research.
The Egyptians are way ahead of the Turks in developing nuclear infrastructure, but don't
expect to see the rise of a
nuclear power on the Nile anytime soon. Egypt's nuclear program is actually older than India's, and was
established only three years after Israel founded its Atomic Energy Commission. The Egyptian Atomic Energy Commission, which
Gamal Abdel Nasser established in 1955, was exclusively dedicated to the development of peaceful atomic energy, though there
were suspicions to the contrary. The 1956 nuclear cooperation agreement with the Soviet Union transferred to Egypt a 2-megawatt
light water reactor that only produced small amounts of plutonium. There were, of course, worrying signs about the Egyptian
program -- specifically Cairo's refusal to open the Inshas reactor to International Atomic Energy Agency (IAEA) inspection until after
the peace treaty with Israel. Yet neither President Anwar Sadat nor his successor, the recently deposed Hosni Mubarak, ever made
any effort to develop nuclear weapons technology. Sadat signed the Nuclear Non-Proliferation Treaty in 1980, and Mubarak
negotiated with the United States, France, Canada, and Germany for reactors and funding for Egypt's nuclear program. Nothing,
however, ever came of these discussions because of the 1986 Chernobyl disaster -- and the fact that the Egyptians never signed
what is known as the Additional Protocol, which gives the IAEA enhanced powers to inspect nuclear facilities. Given the trajectory of
Egypt's nuclear development, Cairo's rejection of the Additional Protocol had more to do with politics and sovereignty than plans for
a clandestine weapons program. Even after Mubarak's son Gamal triumphantly declared at the ruling party's 2006 convention that
Egypt was going to ramp up its nuclear development program, it is hard to believe that Egyptians ever really took him seriously.
Mubarak spent $160 million on consultants to tell him where to build 10 planned nuclear power plants, and selected a location
along the Mediterranean for the first one. But each of the power plants comes with a price tag of $1.5 billion -- and this is a country
that in the last 15 months has spent approximately $26 billion of its $36 billion foreign currency reserves just to stay afloat. One has
to wonder about the pundits' warning of an Egyptian bomb: Have they even been to Egypt lately? If so, they might have a better
grasp of Egypt's ramshackle infrastructure and the dire state of its economy, neither of which can support a nuclear program. What
about Saudi Arabia, then, the Sunni power that is on the tip of most analysts' tongues when it comes to Shiite Iran getting the bomb?
Saudi Arabia has the cash to make large-scale investments in nuclear technology. Indeed, the only factor that makes warnings
about Saudi proliferation -- such as that delivered by former Ambassador the United States Prince Turki al-Faisal last year -- even
remotely credible is the resources the Saudis can muster to buy a nuclear program. Yet, while Riyadh can outfit itself with nuclear
facilities with ease, it does not have the capacity to manage them. Mohamed Khilewi, a former Saudi diplomat, claims that the
kingdom has been developing a nuclear arsenal to counter Israel since the mid-1970s -- but he offers no substantiated evidence to
support these claims. In fact, the country has no nuclear facilities and no scientific infrastructure to support
them. It's possible that Saudi Arabia could import Pakistanis to do the work for them. But while Saudis feel comfortable with
Pakistanis piloting some of their warplanes and joining their ground forces, setting up a nuclear program subcontracted with
Pakistani know-how -- or even acquiring a nuclear device directly from Islamabad -- poses a range of political risks for the House of
Saud. No doubt there would be considerable international opprobrium. Certainly Washington, which implicitly extends its nuclear
umbrella to Saudi Arabia, would have a jaundiced view of a nuclear deal between Riyadh and Islamabad. Moreover, it's one thing to
hand the keys to an F-15 over to a foreigner, but letting them run your nuclear program is another matter altogether. The concern
about Saudi proliferation stems from fears that the kingdom would be forced to act if both Iran and Israel possessed a nuclear
arsenal. "We cannot live in a situation where Iran has nuclear weapons and we don't," an unnamed Saudi official declared to the
Guardian on the sidelines of a meeting between Prince Turki al Faisal and NATO officials in June 2011. "It's as simple as that. If Iran
develops a nuclear weapon, that will be unacceptable to us and we will have to follow suit." Yet given the fact that the Saudis have
very little nuclear infrastructure to speak of, this kind of statement is little more than posturing designed to force the U.S. hand on
Iran. Unlike similar warnings by Israel, which has the capacity to follow through on its threat to attack Iran's nuclear sites,
Riyadh's rhetoric about acquiring nuclear weapons is empty. What is amazing is how many people take the
Saudis seriously. If Khilewi had been telling the truth, now would seem like a good time for the Riyadh to give
Tehran a look at what the royal family has been hiding in the palace basement all these years -- but so far, we
have only heard crickets. Despite its flimsiness, it is hard to ignore the utility of the Middle East's nuclear dominoes theory.
For those who advocate a preventive military strike on Iran, it provides a sweeping geopolitical rationale for a dangerous operation.
But the evidence doesn't bear this argument out: If Washington decides it has no other option than an attack, it should do so
because Iran is a threat in its own right, and not because it belives it will thwart inevitable proliferation in places like Turkey, Egypt,
and Saudi Arabia. It won't, for the simple reason that there is no reason to believe these countries represent a
proliferation risk in the first place.

Empirics and deterrence disprove spillover – Israel has been nuclear for decades, fear of
overwhelming retal deters Iran

Chapman 12 – member of the Chicago Tribune's editorial board (Steve, 07/08, “The arms race that won't happen,”
http://articles.chicagotribune.com/2012-07-08/news/ct-oped-0708-chapman-20120708_1_nuclear-proliferation-iran-regional-
proliferation)

But there is no reason to think Iran would ever use such weapons, and there is little reason to
think it would spur other countries to get them. If all it takes to unleash regional proliferation is one fearsome
state with nukes, the Middle East would have gone through it already — since Israel has had them for decades. ¶ Why
would governments in the region respond differently to Iran? Many of them are allied with the U.S. — which
means Iran can't attack or threaten them without fear of overwhelming retaliation. Turkey, as a member
of NATO, enjoys a formal defense guarantee from Washington. The U.S. might offer similar assurances to Saudi Arabia, Egypt and
other nervous neighbors.¶ One way or another, they would probably find they can manage fine. Iran is no scarier than Mao's China
was in 1964, when it detonated its first atomic device. Writes Francis Gavin, a professor at the Lyndon B. JohnsonSchool of Public
Affairs at the University of Texas at Austin, "It was predicted that India, Indonesia and Japan might follow."¶ At the time, he noted in
a 2009 article in International Security, "a U.S. government document identified 'at least 11 nations (India, Japan, Israel, Sweden,
West Germany, Italy, Canada, Czechoslovakia, East Germany, Rumania and Yugoslavia)' with the capacity to go nuclear, a number
that would soon 'grow substantially' to include 'South Africa, the United Arab Republic, Spain, Brazil and Mexico.'" In recent decades,
some countries have actually given up their nukes — including Ukraine and South Africa. Others, like Brazil and Sweden, have
scrapped their weapons programs. After the Cold War, it was assumed the newly reunified Germany would want to assert its new
status by joining the nuclear club. It has yet to exhibit a glimmer of interest.¶ A
nuclear Iran would soon learn
something previous nuclear powers already know: These weapons are not much use except to deter nuclear
attack. What help have they been for the U.S. in Iraq or Afghanistan?¶ China invaded Vietnam in 1979 to force the enemy's
withdrawal from Cambodia. The Vietnamese not only refused but sent the People's Liberation Army home with its tail between its
legs. China regards Taiwan as part of its territory, but the island has remained functionally independent despite the threat of nuclear

coercion.¶ If Iran does get nukes, its neighbors that have survived without them will find that nothing
much has changed . Nuclear proliferation is the danger that lurks just over the horizon, and that's
where it is likely to stay .

No middle east prolif

Lindsay and Takeyh ’10 ( James M. Lindsay is Senior Vice President, Director of Studies, and Maurice R. Greenberg Chair at
the Council on Foreign Relations, AND Ray Takeyh is a Senior Fellow at the Council on Foreign Relations and the author of Guardians
of the Revolution: Iran and the World in the Age of the Ayatollahs, “After Iran Gets the Bomb”, Mar/Apr2010, Vol. 89, Issue 2,
EBSCO, CMR

Another danger that would have to be countered would be nuclear proliferation in the Middle East. Iran's regional rivals might try to
catch up with it. History suggests, however, that states go nuclear for reasons beyond tit for tat; many
hold back even when their enemies get nuclear weapons.China's pursuit of the bomb in the
1960s prompted fears that Japan would follow, but nearly half a century later, Japan remains
nonnuclear. Although Israel has more than 200 nuclear weapons, neither its neighbors --not even
Egypt, which fought and lost four wars with Israel--nor regional powers , such as Saudi Arabia or Turkey, have followed
its lead. An Iranian nuclear bomb could change these calculations. The U.S. National Intelligence Council concluded in a 2008
report that "Iran's growing nuclear capabilities are already partly responsible for the surge of interest in nuclear energy in the Middle
East." And nuclear energy programs can serve as the foundation for drives for nuclear weapons. But it would
not be
easy for countries in the region to get nuclear weapons. Many lack the infrastructure to develop their
own weapons and the missiles needed to deliver them. Egypt and Turkey might blanch at the
expense of building a nuclear arsenal.  The Pakistanis were willing to "eat grass" for the privilege
of joining the nuclear club, as the Pakistani leader Zulfikar Ali Bhutto once famously put it, but not
everyone is.
 

Middle east prolif will be stable

Seng ’98 (Jordan Seng (University of Chicago) June, 1998 “Strategy for Pandora’s Children: Stable Nuclear Proliferation Among
Minor States” online
In addition to the indications that the Israelis would be amenable to an opaque détente in the Middle East, there are reasons to
believe that emerging
nuclear states in the Arab world would also actively seek nuclear détente,
and that the Arabs themselves would pursue a form of opaque proliferation as Israel has done.
Naturally, the Arab states would be subject to many of the same constraining factors that have led to opacity in other minor
proliferators. I have discussed these factors at length theoretically and in case studies, and I will not here offer yet more specific
discussion of how exactly they apply to each potential proliferator in the Middle East. However, I will note that in addition to the
fundamental factors of limited nuclear proliferation among minor states there are others unique to the Middle Eastern scenario, and
these additional factors may even further encourage limitation in Middle Eastern proliferators’ nuclear weapons development.
First, Middle
Eastern proliferators will be emerging in an environment in which opacity is an
established and successful phenomenon. We will discuss the details of the deterrent success of Israel’s opaque
nuclear posture below. For now, suffice it to say that Israel’s nuclear weapons are indeed providing deterrence. One perhaps can
argue that Israel’s nuclear ambiguity has lessened the deterrent value of its weapons at some point or another, or that opacity
slowed the development of solid deterrent credibility. But even if these moderating effects have hampered Israel to some
degree, the journey will be smoother for Arab states simply because Israel has gone before. The
rules of the road for
opacity have been established in the Middle East. All concerned parties are speaking the
language of opaque proliferation already. They have experience in the hinting and tacit signaling
and understanding that comprises an opaque strategic dialogue . Secondly, as noted
above, the Arab nations have been largely complicit in preserving Israel’s opaque status . There is
a certain pattern of mutual consent that suggests both  Arabs and Israelis would be willing to
accede to an arrangement of mutual opaque deterrence should
additional nuclear proliferation occur. One might think of it in terms of a mutually beneficial quid pro quo deal, half of
which has manifested in the years of Arab complicity with respect to Israeli opacity, and half of which Arab proliferators can rightly
expect from Israel in the future. Insofar as the Arabs have reason to expect Israel to cooperate in establishing opacity, they will be
more likely to engender opacity from the outset. Thirdly, the fact that
Israel has a very sizeable nuclear lead
might well encourage emerging Middle Eastern proliferators toseek nuclear détente as soon as
possible. Even as the well-established Israeli nuclear superiority goads Arab states into pursuing
their own nuclear deterrent, it also figures to motivate some form of cold war
accommodation once proliferators have achieved a modicum of nuclear weapons capability . As
Pakistan does not want to get into an open nuclear arms competition with the technologically
superior India, so Middle Eastern states will want to avoid open arms competition with Israel.
Military Grid Answers
No grid fluctuation—segmentation, safeguards, and reserve power

Leger 12 [Donna Leinwand Leger, USA Today, “Energy experts say blackout like India's is unlikely
in U.S.,” 7-31 http://www.usatoday.com/news/nation/story/2012-07-31/usa-india-power-
outage/56622978/1]

A massive, countrywide power failure like the one in India on Tuesday is "extremely unlikely" in the
United States, energy experts say. In India, three of the country's government-operated power grids failed Tuesday,
leaving 620 million people without electricity for several hours. The outage, the second in two days in the country of 1.21 billion
people, is the world's biggest blackout on record. The
U.S. electricity system is segmented into three parts
with safeguards that prevent an outage in one system from tripping a blackout in another
system, "making blackouts across the country extremely unlikely," Energy Department spokeswoman Keri
Fulton said. Early reports from government officials in India say excessive demand knocked the country's power generators offline.
Experts say India's industry and economy are growing faster than its electrical systems. Last year, the economy grew 7.8% and
pushed energy needs higher, but electricity generation did not keep pace, government records show. " We are much, much
less at risk for something like that happening here, especially from the perspective of demand
exceeding supply," said Gregory Reed, a professor of electric power engineering at University of
Pittsburgh. "We're much more sophisticated in our operations. Most of our issues have been
from natural disasters." The U.S. generates more than enough electricity to meet demand and
always have power in reserve, Reed said. "Fundamentally, it's a different world here," said Arshad
Mansoor, senior vice president of the Electric Power Research Institute in Washington and an
expert on power grids. "It's an order of magnitude more reliable here than in a developing
country." Grid operators across the country analyze power usage and generation, factoring outside factors such as weather, in
real time and can forecast power supply and demand hour by hour, Mansoor said. "In any large, complex interactive network, the
chance of that interconnection breaking up is always there," Mansoor said. "You cannot take your eye off the ball for a minute."
Widespread outages in the U.S. caused by weather are common. But the
U.S. has also had system failures, said Ellen
Vancko, senior energy adviser for the Union of Concerned Scientists, based in Washington. On
Aug. 14, 2003, more than
50 million people in the Northeast and Canada lost power after a major U.S. grid collapsed . The
problem began in Ohio when a transmission wire overheated and sagged into a tree that had grown too close to the line, Vancko
said. Thatcaused other power lines to overheat until so many lines failed that the system shut
itself down, she said. "That was less a failure of technology and more a failure of people, a
failure of people to follow the rules," Vancko said. "There were a whole bunch of lessons
learned." In 2005, in response to an investigation of the blackout, Congress passed a law
establishing the North American Electric Reliability Corporation (NERC) to enforce reliability standards for bulk
electricity generation.

No impact to grid

Rahman ’10 (Arifeen Rahman, SEA National Security Intern, Scientist and Engineers for America, “Bringing Cybersecurity Back to
Reality”, http://www.sefora.org/2010/08/11/rahman_0811/ , August 11, 2010)
One common misconception is that a cyber-attack is easier to commit than a conventional attack – its tools easier to obtain and
deploy. This renders the false image of a lone hacker singlehandedly bringing down the US electric grid. In reality, creating such
an orchestrated attack would require over “$200 million, country-level intelligence and five
years of preparation time.” As a result, the probability of a wide-scale attack required to inflict high-
magnitude destruction remains increasingly low. Fears of a Chinese attack on this scale
appear unfounded, given the economic impact that it would have on the United States. The
economic relationship between China and the United States still remains a two-way street,
and would be foolish to compromise at the current time.

No risk of large cyber-attacks – no means or motivation and deterrence prove

Birch ‘12 (Douglas is a former foreign correspondent for the Associated Press and the Baltimore Sun who has written extensively
on technology and public policy, Forget Revolution, 10/1/12, http://www.foreignpolicy.com/articles/2012/10/01/forget_revolution?
page=0,3)

Professionals like Weiss and others warned that Stuxnet was opening a Pandora's Box: Once it was
unleashed on the world, they feared, it would become available to hostile states, criminals, and terrorists who could adapt the code
for their own nefarious purposes. But twoyears after the discovery of Stuxnet, there are no reports of similar
attacks against the United States. What has prevented the emergence of such copycat viruses? A 2009 paper published by the
University of California, Berkeley, may offer the answer. The report, which was released a year before Stuxnet surfaced, found that
in order to create a cyber weapon capable of crippling a specific control system -- like the ones
operating the U.S. electric grid -- six coders might have to work for up to six months to reverse engineer the
targeted center's SCADA system. Even then, the report says, hackers likely would need the help of someone with
inside knowledge of how the network's machines were wired together to plan an effective attack. "Every SCADA control center is
configured differently, with different devices, running different software/protocols," wrote Rose Tsang, the report's author.
Professional hackers are in it for the money -- and it's a lot more cost-efficient to search out
vulnerabilities in widely-used computer programs like the Windows operating system, used by banks and other
affluent targets, than in one-of-a-kind SCADA systems linked to generators and switches. According to Pollard,
only the world's industrial nations have the means to use the Internet to attack utilities and
major industries. But given the integrated global economy, there is little incentive, short of armed conflict, for them to do so.
"If you're a state that has a number of U.S. T-bills in your treasury, you have an economic
interest in the United States," he said. "You're not going to have an interest in mucking about with our infrastructure."
There is also the threat of retaliation. Last year, the U.S. government reportedly issued a classified report on cyber
strategy that said it could respond to a devastating digital assault with traditional military force. The idea was that if a cyber attack
caused death and destruction on the scale of a military assault, the United States would reserve the right to respond with what the
Pentagon likes to call "kinetic" weapons: missiles, bombs, and bullets. An unnamed Pentagon official, speaking to the Wall Street
Journal, summed up the policy in less diplomatic terms: "If you shut down our power grid, maybe we will put a missile down one of
your smokestacks." Deterrence is sometimes dismissed as a toothless strategy against cyber attacks because
hackers have such an easy time hiding in the anonymity of the Web. But investigators typically come up with key
suspects, if not smoking guns, following cyber intrusions and assaults -- the way suspicions quickly focused
on the United States and Israel after Stuxnet was discovered. And with the U.S. military's global reach, even terror
groups have to factor in potential retaliation when planning their operations.

Multiple checks empirically check escalation – their evidence is alarmist

Birch ‘12 (Douglas is a former foreign correspondent for the Associated Press and the Baltimore Sun who has written extensively
on technology and public policy, Forget Revolution, 10/1/12, http://www.foreignpolicy.com/articles/2012/10/01/forget_revolution?
page=0,3)First, the
freak "derecho" storm that barreled across a heavily-populated swath of the
eastern United States on the afternoon of June 29 knocked down trees that crushed cars, bashed holes in roofs,
blocked roads, and sliced through power lines. According to an August report by the U.S. Department of Energy, 4.2 million homes
and businesses lost power as a result of the storm, with the blackout stretching across 11 states and the District of Columbia. More
than 1 million customers were still without power five days later, and in some areas power wasn't restored for 10 days. Reuters put
the death toll at 23 people as of July 5, all killed by storms or heat stroke. The
second incident occurred in late July,
when 670 million people in northern India, or about 10 percent of the world's population , lost
power in the largest blackout in history. The failure of this huge chunk of India's electric grid was attributed to higher-than-normal
demand due to late monsoon rains, which led farmers to use more electricity in order to draw water from wells. Indian officials told
the media there were no reports of deaths directly linked to the blackouts. But this
cataclysmic event didn't cause
widespread chaos in India -- indeed, for some, it didn't even interrupt their daily routine. "[M]any
people in major cities barely noticed the disruption because localized blackouts are so common that many businesses, hospitals,
offices and middle-class homes have backup diesel generators," the New York Times reported. The
most important thing
about both events is what didn't happen. Planes didn't fall out of the sky. Governments didn't
collapse. Thousands of people weren't killed . Despite disruption and delay, harried public officials,
emergency workers, and beleaguered publics mostly muddled through. The summer's blackouts
strongly suggest that a cyber weapon that took down an electric grid even for several days could
turn out to be little more than a weapon of mass inconvenience . "Reasonable people would have
expected a lot of bad things to happen" in the storm's aftermath, said Neal A. Pollard, a terrorism expert who teaches at
Georgetown University and has served on the United Nation's Expert Working Group on the use of the Internet for terrorist
purposes. However, he said,
emergency services, hospitals, and air traffic control towers have backup
systems to handle short-term disruptions in power supplies. After the derecho, Pollard noted, a generator
truck even showed up in the parking lot of his supermarket. The response wasn't perfect, judging by the heat-related deaths and
lengthy delays in the United States in restoring power. But nor were the people without power as helpless or clueless as is
sometimes assumed.
Multilateralism Answers
Multilat fails

Holmes ’10 [Kim Holmes, VP, foreign policy and defense studies, Heritage. Frmr Assistant Secretary of State for International
Organization Affairs. While at the State Department, Holmes was responsible for developing policy and coordinating U.S.
engagement at the United Nations and 46 other international organizations. Member of the CFR. Frmr adjunct prof of history,
Georgetown. PhD in history, Georgetown, Smart Multilateralism and the United Nations, 21 Sept.
2010, http://www.heritage.org/research/reports/2010/09/smart-multilateralism-when-and-when-not-to-rely-on-the-united-
nations]

The need for multilateralism is obvious. Nations share concerns about many problems and issues for which coordinated efforts could
be mutually beneficial. Yet only rarely do all governments agree on the nature of a problem and the means to
address it. At times, negotiations result in a less-than-perfect, but still acceptable, course of action. Disagreements can also lead
to no action or the use of force or other confrontational measures. One of the purposes of multilateralism is to minimize the number
and intensity of such confrontations. The process itself, however, is fraught with political
challenges that can undermine potential solutions and even lead to other problems. For the United States, multilateralism faces its
greatest challenge at the United Nations, where U.S. diplomats seek cooperative action among member
nations on serious international problems. Therein lies the tension. The United Nations is first and foremost a political body made up
of 192 states that rarely agree on any one issue. Even fundamental issues, such as protecting and observing human rights, a
key purpose of the U.N. that all member states pledge to uphold when they join it, have become matters
of intense debate. A key reason for this difficulty is the fact that the voices and votes of totalitarian and authoritarian regimes
have equal weight to those of free nations at the U.N. The all-too-frequent clash of worldviews between liberty and
authoritarian socialism has stymied multilateralism more than facilitated it, frequently leading
to institutional paralysis when a unified response to grave threats to peace and security or human rights and fundamental
freedoms was needed. U.S. secretary of state John Foster Dulles, who attended the San Francisco meetings that established the
U.N., acknowledged this Achilles’ heel in 1954, when he told reporters: “The United Nations was not set up to be a reformatory. It
was assumed that you would be good before you got in and not that being in would make you good.”[1] Fifty-five years later, the
ideological fray at the U.N. has turned the terms “democracy” and “freedom” on their heads. Autocracies that deny democratic
liberties at home are all too keen to call the Security Council “undemocratic” because in their view not every region, country, or bloc
is sufficiently represented. During my time at the State Department, I was told repeatedly by other diplomats at the U.N. that the
very concept of “freedom” is taboo because the term is “too ideologically charged.” In this environment, how can the United States
or any freedom-loving country advance the purposes set forth in the U.N. Charter, including “encouraging respect for human rights
and for fundamental freedoms for all,”[2] when the word “freedom” itself is considered too controversial? More money will not do
it. No other nation contributes more to the U.N.’s regular budget, its peacekeeping budget, or the budgets of its myriad affiliated
organizations and activities than the United States. America has continued its generous support even though Americans increasingly
view the U.N. as inefficient and ineffective at best and fraudulent, wasteful, anti-American, and beyond reform at worst.[3] If the
United States is to advance its many interests in the world, it needs to pursue multilateral diplomacy in a smarter, more pragmatic
manner. This is especially true when Washington is considering actions taken through the United Nations. A decision to engage
multilaterally should meet two criteria: First, it should be in America’s interests, and second, it will serve to advance liberty. Unless
the United States can achieve both these ends acting within the U.N. system, it should find ways to work around it. Such
“smart multilateralism” is not easy, particularly in multilateral settings. It requires politically savvy leaders who can overcome
decades-old bureaucratic inertia at the State Department and in international organizations. It requires the political will
and diplomatic skill of people who are dedicated to advancing U.S. interests in difficult environments, especially where progress
will likely be slow and incremental. It requires a belief in the cause of liberty, gleaned from a thorough study of our nation’s history
and the U.S. Constitution, and a deep appreciation for the values and principles that have made America great. Smart multilateralism
requires a fundamental awareness of the strengths and weaknesses, capabilities and failings, of the U.N. and other multilateral
negotiating forums, so that the United States does not overreach. Perhaps the most critical decision is whether or not to take a
matter to the U.N. in the first place. It would be better to restrict U.S. engagement at the U.N. to situations in which success is
possible or engagement will strengthen America’s influence and reputation. Selective engagement increases the potential for
success, and success breeds success. When America is perceived to be a skillful and judicious multilateral player, it finds it easier to
press its case. Smart multilateralism thus requires well-formulated and clear policy positions and a willingness to hold countries
accountable when their votes do not align with our interests. Finally, smart multilateralism is not the same thing as “smart power,” a
term that Secretary of State Hillary Clinton has used. Suzanne Nossell, a former diplomat at the U.S. Mission to the U.N. in New York,
coined that term in 2004 and described it in an article in Foreign Affairs.[4] Smart power is seen as a takeoff of “soft power,” which
suggests that America’s leaders downplay the nation’s military might as well as its historic role in establishing an international
system based on the values of liberty and democracy, and de-emphasize its immense economic and military (“hard”) power. Smart
power seeks to persuade other countries from a position of assumed equality among nations. This assumption has become the
Achilles’ heel of the U.N. system and other Cold War–era organizations. Smart multilateralism does not make that same mistake.
Challenges to Effective U.S. Multilateralism The United States belongs to dozens of multilateral organizations, from large and well-
known organizations such as NATO, the World Trade Organization (WTO), and the International Monetary Fund to relatively small
niche organizations such as the Universal Postal Union and the International Bureau of Weights and Measures. The 2009
congressional budget justification[5] for the U.S. Department of State included line items for U.S. contributions to some fifty distinct
international organizations and budgets.[6] The United Nations and its affiliated bodies receive the lion’s share of these
contributions. While the World Bank and International Monetary Fund weight voting based on contributions, most of these
organizations subscribe to the notion of the equality of nations’ votes. With a few exceptions such as Taiwan,[7] all nations—no
matter how small or large, free or repressed, rich or poor—have a seat at the U.N. table. Every nation’s vote is equal, despite great
differences in geographic size, population, military or economic power, and financial contributions. This one-country, one-vote
principle makes the U.N. an extremely difficult venue in which to wage successful multilateral diplomacy. In this
environment, multilateralism becomes a double-edged sword. It can sometimes speed up global responses to global problems, as
with the avian flu outbreak and the Asian tsunami. At other times, it can slow or prevent timely responses, as with halting Iran’s
nuclear weapons program and stopping genocide in Darfur. Too often, multilateralism at the U.N. is
the political means
by which other countries and regional blocs constrain or block action. Groups of small nations can join together to
outvote the great powers on key issues, and this situation can often lead to bizarre outcomes and compromises. Even seemingly
noncontroversial issues, such as improving auditing of U.N. expenditures, require days of skillful, almost nonstop negotiations. The
U.N. is simply too poorly primed for American multilateralism. It is a vast labyrinth of agencies, offices, committees, commissions,
programs, and funds, often with overlapping and duplicative missions.[8] Lines of accountability and responsibility for specific issues
or efforts are complex, confused, and often indecipherable. For example, dozens of U.N. bodies focus on development, the
environment, and children’s and women’s issues. Coordination is minimal. Reliable means to assess the effectiveness of the bodies’
independent activities is practically nonexistent. Although institutional fiefdoms and bureaucratic interests strongly influence the
formulation of U.N. policy, programs, and resolutions, the most powerful actors remain the member states. Each tries to persuade
the U.N. as an institution to advocate and adopt its positions on the matters most important to it. The chaos of conflicting priorities
rarely results in consensus for decisive action. The most common result is inaction or a lowest-common-denominator outcome. Too
often, the United States also finds that other countries’ positions on an issue have been predetermined in their
regional or political groupings. 

Multilat fails- too many actors, too self-interested- Copenhagen proves

Haas 10. [Richard N., President, “The Case for Messy Multilateralism” Council on Foreign Relations -- January 5 --
http://www.cfr.org/un/case-messy-multilateralism/p21132]

No country, not even the US, can face these challenges alone. The world is simply too large and too complex to control. By their
nature, these challenges are best met by collective effort . Decisions to opt out of global arrangements (or an
inability to opt in, as we see in the case of governments too weak to combat terrorists who set up shop on their territory) can have
repercussions far beyond a country's borders. But to acknowledge that we are all multilateralists now (or at least
need to be) is only to start the conversation. Multilateralism is not one thing but many . The issue
takes on a new urgency in the aftermath of the recent Copenhagen conference, which brought
together representatives of 193 governments in an unsuccessful effort to reach a formal, binding and
comprehensive accord. Whatever its consequences for climate change, Copenhagen is but the most recent
reminder that classic multilateralism is increasingly difficult to achieve. This same reality also helps to
account for the world's inability to agree to a new global trade accord. Launched in Qatar nearly a
decade ago, the Doha round of negotiations has stalled. There are simply too many participants, too many contentious
issues and too many domestic political concerns to discuss. This problem also explains the near-total irrelevance of the
United Nations General Assembly. "One [person], one vote" may provide a sound basis for domestic politics, but
on a global scale democracy (or, more precisely, democratic multilateralism) is a prescription for doing nothing. It
is not simply the large number of participants but the fact that it makes little sense to give
countries with minuscule populations and economies equal standing with , say, China or the US.
The UN's founders predicted as much when they created the Security Council. The idea was to establish an elite body to tackle the
world's most important issues. The problem is that the
composition of the Security Council reflects what the
world looked like after the second world war. That world is now more than 60 years old. Missing from the
ranks of permanent members are India, Japan, Germany, Brazil and representatives of a more integrated Europe. It
was this weakness (along with the inability to agree on the make-up of a reformed Security Council) that in part led to the creation
of the Group of Seven and the trilateral process in the 1970s. Japan and the European Commission gained a seat at this important
table. Yet over the decades, the G7 also proved inadequate, as it left out such critical countries as China and India. Hence the
emergence of the Group of 20 in the midst of the global financial crisis and the Major Economies Forum as concerns over climate
change mounted. It is too soon to judge the impact of these latest versions of elite multilateralism. In the meantime, we are seeing
the emergence of multiple innovations. One is regionalism. The proliferation of bilateral and regional trade pacts (most recently in
Asia) is in part a reaction to the failure to conclude a global trade accord. Such arrangements are inferior - they do not, for example,
normally deal with subsidies, much less cover all products and services. They can also have the perverse effect of retarding trade by
discriminating against non-members. But some trade expansion is preferable to none. A second alternative is functional
multilateralism - coalitions of the willing and relevant. A global accord on climate will prove elusive for some time to come. But that
need not translate into international inaction. A useful step would be to conclude a global pact to discourage the cutting down and
burning of forests, something that accounts for a fifth of the world's carbon output. Copenhagen made some limited progress here,
but more needs to be done to assist such countries as Brazil and Indonesia. Yet another alternative might be described as informal
multilateralism. In many cases it
will prove impossible to negotiate international accords that will be
approved by national parliaments. Instead, governments would sign up to implementing, as best they can, a series of
measures consistent with agreed-upon international norms. We are most likely to see this in the financial realm, where setting
standards for the capital requirements of banks, accounting systems and credit ratings would facilitate global economic growth.
None of this - not elitism or regionalism or functionalism or informalism - is a panacea. Such
collective action is
invariably less inclusive, less comprehensive and less predictable than formal global accords. It can
suffer from a lack of legitimacy. But it is doable and desirable, and can lead to or complement classic multilateralism.
Multilateralism in the 21st century is, like the century itself, likely to be more fluid and, at times, messy than
what we are used to.

Dyer agrees

Dyer 4. (Gwynne, military historian and lecturer on international affairs, “The End of War”, Toronto Star, 12/30/2004,
http://www.commondreams.org/views04/1230-05.htm)

THEIR CARD BEGINS


The "firebreak" against nuclear weapons use that we began building after Hiroshima and Nagasaki has held for well over half a
century now. But the proliferation of nuclear weapons to new powers is a major challenge to the stability of the system. So are the
coming crises, mostly environmental in origin, which will hit some countries much harder than others, and may drive some to
desperation. Add in the huge impending shifts in the great-power system as China and India grow to rival the United States in GDP
over the next 30 or 40 years and it will be hard to keep things from spinning out of control. With good luck and good management,
we may be able to ride out the next half-century without the first-magnitude catastrophe of a global nuclear war, but the potential
certainly exists for a major die-back of human population. We cannot command the good luck, but good management is something
we can choose to provide. It depends, above all, on preserving and extending the multilateral system that we have been building
since the end of World War II. The rising powers must be absorbed into a system that emphasizes co-operation and makes room for
them, rather than one that deals in confrontation and raw military power. If they are obliged to play the traditional great-power
game of winners and losers, then history will repeat itself and everybody loses.

THEIR CARD ENDS


Our hopes for mitigating the severity of the coming environmental crises also depend on early and concerted global action of a sort
that can only happen in a basically co-operative international system. When
the great powers are locked into a
military confrontation, there is simply not enough spare attention, let alone enough trust, to make
deals on those issues, so the highest priority at the moment is to keep the multilateral approach alive
and avoid a drift back into alliance systems and arms races. And there is no point in dreaming that we can leap straight into some
never-land of universal brotherhood; we will have to confront these challenges and solve the problem of war within the context of
the existing state system. The solution to the state of international anarchy that compels every state to arm itself for war was so
obvious that it arose almost spontaneously in 1918. The wars by which independent states had always settled their quarrels in the
past had grown so monstrously destructive that some alternative system had to be devised, and that could only be a pooling of
sovereignty, at least in matters concerning war and peace, by all the states of the world. So the victors of World War I promptly
created the League of Nations. But thesolution was as difficult in practice as it was simple in concept .
Every member of the League of Nations understood that if the organization somehow acquired the
ability to act in a concerted and effective fashion, it could end up being used against them, so no
major government was willing to give the League of Nations any real power. Instead, they got World
War II, and that war was so bad by the end the first nuclear weapons had been used on cities that the victors made a second
attempt in 1945 to create an international organization that really could prevent war. They literally changed international law and
made war illegal, but they were well aware that all of that history and all those reflexes were not going to vanish overnight. It would
be depressing to
catalogue the many failures of the U nited Nations, but it would also be misleading. The
implication would be that this was an enterprise that should have succeeded from the start, and has failed
irrevocably. On the contrary; it was bound to be a relative failure at the outset. It was always
going to be very hard to persuade sovereign governments to surrender power to an untried world
authority which might then make decisions that went against their particular interests. In the words of
the traditional Irish directions to a lost traveler: "If that's where you want to get to, sir, I wouldn't start from here." But here is where
we must start from, for it is states that run the world. The present international system, based on heavily armed and jealously
independent states, often exaggerates the conflicts between the multitude of human communities in the world, but it does reflect
an underlying reality: We cannot all get all we want, and some method must exist to decide who gets what. That is why neighboring
states have lived in a perpetual state of potential war, just as neighboring hunter-gatherer bands did 20,000 years ago. If we now
must abandon war as a method of settling our disputes and devise an alternative, it only can be done
with the full co-operation of the world's governments. That means it certainly will be a monumentally
difficult and lengthy task: Mistrust reigns everywhere and no nation will allow even the least of its
interests to be decided upon by a collection of foreigners . Even the majority of states that are more or
less satisfied with their borders and their status in the world would face huge internal opposition from
nationalist elements to any transfer of sovereignty to the U nited Nations.

Multilateralism is structurally impossible – absence of shared great-power threat and


domestic opposition overwhelm the plan

Skidmore, 11 – Professor in the Department of Politics and International Relations and the Director of the Center for Global
Citizenship at Drake (David, 12/20. “The Obama Presidency and US Foreign Policy: Where’s the Multilateralism?” International
Studies Perspectives, Vol. 13, Issue 1, pp 43-64, February 2012. Wiley Online Library.)

Expectations that the presidential transition from George W. Bush to Barack Obama would produce a
multilateralist turn in American foreign policy have thus far proven misplaced. This is largely because
the strategic environment of the post-Cold War era places structural constraints on the ability
of any US president, of whatever ideological leanings, to pursue a consistently multilateralist
foreign policy. Internationally, the absence of a shared great power threat has undermined the
institutional bargain between the United States and allied states, thus rendering the terms of
multilateral cooperation more difficult to agree upon. At home, the end of the Cold War has
undermined presidential authority and empowered veto players whose interests are threatened
by multilateral commitments. Nevertheless, structure is not destiny. Understanding the sources of political constraint can
suggest strategies for overcoming or bypassing such obstacles to multilateral engagement in US foreign policy. A president who
wishes to exercise multilateral leadership abroad must seek to renegotiate the terms of US engagement with international
institutions while fashioning a compelling rationale that mobilizes public support at home.
Nagorno-Karabakh Answers
No escalation – allies won’t be drawn in and Russia will be diplomatic

Glashatov 7 (Oleg, “Zero Hour Approaches for Yerevan; Azerbaijan Prepares to Fight for Nagorno-Karabakh: Will There Be
War?”, What the Papers Say Part A (Russia), 7-5, Lexis)

Speaking at Johns Hopkins University, US Council on Foreign Relations analyst Wayne Merry noted that Azerbaijan cannot win,
even though military options for resolving the conflict are being discussed openly in Azerbaijan. In his view, Nagorno-
Karabakh is an impregnable fortress, further strengthened by Armenian forces, and even the
American military would have difficulty attacking that fortress . According to the analyst, this is also the
prevalent view in the Pentagon. But Azerbaijan takes an entirely different view of the situation. Zakhir Orudzh, a member of the
Azeri parliament's defense and security committee, says: "Armenia can only be superior to us in the capacities it gains from
bilateral military agreements with Russia and participation in the CIS Collective Security Treaty Organization. For all other
parameters and resources, Azerbaijan is superior to Armenia, in military terms. And don't let anyone try to intimidate Azerbaijan
with the idea that conflict escalation could have serious consequences for our country. Everyone should realize that if
Azerbaijan and Armenia were left to face each other alone, with no external support, we could rapidly prove that we are in the
right." Armed hostilities
could resume in several ways; in almost every scenario, they would be started either by
Azerbaijan or by dubious international structures that specialize in promoting the West's interests in this region (such as the
International Crisis Group). The most immediately relevant scenario could involve the United States attacking Iran, and
Azerbaijan taking advantage of the chaos to make an attempt at sorting out the Nagorno-Karabakh problem once and for all.
However, Azerbaijan could hardly expect substantial military support in these
circumstances, from either the United States (it would be too busy elsewhere) or Turkey
(which might confine its participation in the conflict to sending volunteers). All of the above leads
to the following conclusion: Azerbaijan is unlikely to succeed with a blitzkrieg in the immediate future. In
this situation (as in most modern conflicts), the time factor would be decisive. Moreover, if hostilities do break out, Russia's
military obligations would come into effect: Armenia is an ally within the CIS Collective Security Treaty Organization.
Consequently, Moscow is likely to make every effort to see that this conflict is resolved by
diplomatic or other means.

Empirically denied

Arminfo 7 (News Agency, “Arkadiy Gukasyan: Pat Situation Maybe Created In The Karabakh Negotiating Process Because
Inefficiency Of Its Format”, 7-3, Lexis)

Pat situation maybe created in the Karabakh negotiating process because inefficiency of its format, the NKR President Arkadiy
Gukasyan said in the Russian-Armenian (Slavonic) state University when making a report "Nagornyy Karabakh: prospects of
settlement". He also added that at present the OSCE Minsk Group because of contradiction of official Baku cannot make the
format of the negotiating process in line with configuration of the conflict defined by it. "The pat situation maybe created in the
negotiating process because of inefficiency of its format as the Azerbaijani party is trying to persuade the world community that
only Azerbaijan and Armenia are the parties to the conflict blaming the latter for the territorial pretensions. We see the way out
from the created situation in returning Nagornyy Karabakh to the negotiating table, and the OSCE MG co-chairs' efforts should
be directed to this goal reaching", - the NKR president said. He also added that in fact today the parties are trying to treat a
decease not knowing its diagnosis. "Today the
conflict is 20 years old, but they are still disputing about
the participants in the conflict and its parties: Azerbaijan-Armenia, Azerbaijan-Karabakh or
Azerbaijan-Armenia-Karabakh. I think this is an absolutely absurd situation. I have got a formula:
until Azerbaijan strives to speak only with Armenia without Nagornyy Karabakh, Azerbaijan does not strive to settle the conflict
and is just propagandizing", - Arkadiy Gukasyan concluded.
NATO Answers
Frontline

NATO fails and EU solves the impacts better

Hockenos ‘9 (03/09/2009 RETHINKING US-EUROPE RELATIONS Is the EU Better for Obama than NATO? By Paul Hockenos
Paul Hockenos is editor of Internationale Politik-Global Edition. His most recent book is "Joschka Fischer andthe Making of the Berlin
Republic: An Alternative History o fPostwar Germany".

The new American administration would be well served to rethink the United States’
relationship to Europe: It should move toward a strategic partnership of equals with the European Union and entertain the
possibility of new fora to address global security threats. In the long-term, a close, respectful working relationship with the European
Union would enhance America’s own security and enable it to engage much more effectively in a multipolar world. America’s long-
standing preference for NATO as the transatlantic institution of choice has several explanations. For one, it arguably had—at least
until Afghanistan—a record of success. It helped the West win the Cold War without firing a shot. NATO’s job, as British secretary-
general Lord Ismay famously put it in 1967, was “to keep the Russians out, the Americans in, and the Germans down.” But rather
than close up shop with “mission accomplished” in the early 1990s, the 1949-founded pact sought a new purpose. Because the
Europeans lacked the military hardware necessary to wage war against the Serb nationalists, NATO led the humanitarian
interventions in Bosnia in 1995 and the armed campaign against Milosevic’s Serbia in 1999. That same year, the Czech Republic,
Hungary, and Poland became the first former-Warsaw pact countries to join NATO, over Russia’s stiff objections. In the years to
follow, the Baltic states and Slovenia, Slovakia, Bulgaria, and Romania also joined. Although the United States and Great Britain
circumvented NATO to topple the Taliban government in late 2001, two years later NATO took its operations outside of Europe for
the first time in the form of the International Security Assistance Force in Afghanistan. Today the NATO-led force includes 50,000
troops from 40 countries, including all 27 of the NATO allies. Given the East-West stalemate, during the postwar decades it was
possible for NATO allies to work together in the name of collective defense, despite the many differences of opinion within the pact.
Leaving aside the question of the nature of the Soviet threat (archives in Moscow turned up no plans for an invasion), the United
States and the Western Europeans concurred that the Soviet Union was the enemy. Although the United States set the agenda and
the Europeans were effectively junior partners, the principle of collective decision-making was formally respected. Moreover, in the
aftermath of the Cold War there were no obvious alternatives to keep the United States and Europe close once American troops
withdrew and the nuclear umbrella became irrelevant. Creating something new was beyond the imagination of Washington’s
foreign policy makers at the time. Lastly, because it was and would remain primarily a military organization, NATO was one
institution that the United States, with its nuclear arsenal and vast military superiority, would be certain to continue to dominate.
by transforming the alliance into an agency for addressing international crises of all kinds,
Yet
NATO’s advocates have only called greater attention to its inadequacy for the 21st century .
NATO’s new “comprehensive approach” to security endows it with a catch-all mandate that changes as new threats or missions arise
and has grown to include responsibilities that go far beyond the exercise of military force. But while its mandate has
changed, its tools and thinking have lagged behind. There is no better example than NATO’s flagship mission in
Afghanistan, where the alliance is confronted with civilian, policing, and humanitarian duties that it cannot possibly carry out. Most
of the European NATO member states in Afghanistan argue that stability is only going to be achieved through a strategy that
combines education, rule of law programs, economic aid, and infrastructure projects. They underscore that the purpose of the
international mission is to facilitate a hand over to the Afghans and to create conditions for reconstruction. Germany and Spain point
out, for example, that Afghan poppy production—and Afghanistan’s bumper crops—cannot be checked by bombing campaigns, and
that air strikes on poor Afghan farmers could well backfire, costing the force even more good will. But “counter-narcotics” is yet
another category that has been added to NATO’s to-do list. There is growing consensus that the Afghanistan mission is make-or-
break for NATO and that, at the moment, the latter cannot be ruled out. The war in Afghanistan is only the most egregious example
of NATO’s dilemma. Whether it is cyberwar, peacekeeping, international terrorism, or energy security, NATO is invoked by
Atlanticists as the go-to institution, overburdening it with new responsibilities. In late January, NATO’s secretary general even
proposed an alliance presence in the Arctic as global warming melts the northern ice cap and major powers scramble to lay claim to
its energy resources. Others see NATO patrolling Gaza’s borders in a new Israel-Palestine peace deal. As the
Dutch political
scientist Peter van Ham argues, “NATO’s instruments have become blunt and outdated in the
light of today’s non-traditional security challenges and techniques.” Yet, he notes, contrary to
expectations its portfolio has only expanded: “Whereas not too long ago the main question was
how the European Union could use NATO’s military tools...the debate is now how should NATO
draw upon the resources of the European Union, the United Nations, the World Bank, as well as
non-governmental organizations.” But this has not caused US foreign policy makers to consider new fora or mechanisms to
address the new threats. Nor have the Europeans been enterprising or ingenuous with new ideas. For them this is the path
of least resistance: by putting these complex challenges in NATO’s hands, they appear to have
addressed the problems without actually doing so. There is also a lingering question of whether NATO is up to the
job of keeping the peace in the North Atlantic area, its original raison d'etre. Today, the threats to European security are strikingly
different from those of the Cold War years. They include ethnic conflict on Europe's frontiers, mass migration and refugee flows,
energy crises, nuclear proliferation, and transnational terrorism. Particularly in Europe, many experts see security challenges in
global warming, international trafficking, resource scarcity, and failing states. A recent EU study concluded that increased tensions
over falling water supplies in the Middle East will affect the continent's energy security and economic interests. In addition, global
warming will exacerbate poverty and spur mass migration from Africa. Neither
NATO's instruments nor its
framework is right for these kinds of problems . Under the Bush administration this did not matter -- it saw
NATO's role exclusively as part of the war on terrorism. The August 2008 conflict in Georgia, however, underscored that there are
still threats to Europe's security within and on its borders that the continent's powers will have to respond to with instruments other
than pure force. It is no secret that Russia feels deeply threatened by the alliance's expansion eastward, which it has consistently
protested since the early 1990s. Moscow perceives as hostile the advance to its borders of a foreign military alliance that was
designed to resist the Soviet Union and still sees Russia as a competitor. Although not solely accountable for Russia's authoritarian
turn, NATO's expansion into East Central Europe --contrary to US and German promises to
Gorbachev in 1989 -- has expedited the aggressive nationalism and assertiveness of Putin-era
Russia. It has fueled a new arms race and aggravated a security threat in Europe that has far-
reaching implications for the Europeans. Likewise, the further eastward enlargement of NATO to include Ukraine and Georgia, which
Obama specifically advocated in his July 2008 Berlin address, will not engender greater security -- neither for Western Europe nor for
Georgia and Ukraine. Admitting Georgia could draw NATO into a direct confrontation with Russia. Would the alliance really risk war
with Russia over Georgia's breakaway enclaves in the Caucasus? Unlikely. The Georgians should have no illusions: they have already
paid a high price for the false sense of security that American advisors gave to them prior to the recent conflict. The European Union
in the World As great as the gap across the Atlantic has been in recent years, the United States still has much more in common with
the Europeans than it does with new powers China or Russia. Europe could and should be America's closest partner in world affairs.
But this relationship would be immensely different than the current one. It must be a partnership of equals across the Atlantic and
this will require real compromises from the United States as well as the Europeans. To make this possible, the Obama
administration must begin to think anew about the European Union. For one, the Union is not teetering on the brink of
disintegration, regardless of how some American commentators interpret its disunity on many issues and the recent failures to pass
a constitution. Though institutional reform is absolutely necessary, even in its current condition the European
Union is healthy, admired by the overwhelming majority of Europeans, and will continue to perform as it has in the recent
past -- but no better than that until a constitution or new reform treaty is approved. The European Union is already a major, capable
power in world affairs. It has global interests and a sense of responsibility that goes beyond narrow self-interest. Its size and
international economic might alone make it globally relevant, especially since much of the Union's power comes from its
conditionally linked trade policies. The single market includes 450 million people, and ranks as the world's largest exporter of goods
and the second leading importer worldwide behind the United States. When the ongoing financial crisis peaked this fall, President
Bush's first call of help abroad was to the European Union. The Europeans also contribute over half the world's foreign aid to
developing countries, including €300 million a year to the Palestinian Authority, triple the resources the United States provides.
Diplomatically,
the European Union has led international negotiations with Iran over its nuclear
program since 2003. In 2004 European diplomacy helped bring about a peaceful resolution to
Ukraine's Orange Revolution and, more recently, European negotiators brokered a peace in
Georgia that sent peacekeeping troops and monitors to the Caucasus. Its greatest success by
far has been to stabilize the Western Balkans in the aftermath of the wars of the 1990s. Thus,
even though European Union foreign policies are in their infancy, they already make a
significant contribution to global security. Although the European Union in its various incarnations has long been
involved in matters beyond its borders, this took new form in 1992 with the Common Foreign and Security Policy. Since then, its
ability to engage in the wider world was boosted significantly, first with the 1999 European Security and Defense Policy (ESDP) and
then with the 2003 adoption of the European Security Strategy. ESDP endowed the European Union with military capabilities,
enabling it to launch its first mission in Macedonia in March 2003.
NATO is resilient

RIA 6 (Regulatory Intelligence Agency, 12-21, Lexis)

WASHINGTON, Dec. 21, 2006 - The N orth A tlantic T reaty O rganization is healthy and its best years lie ahead ,
Marine Gen. James L. Jones said today at the Europe Atlantic Council here. Jones stepped down as NATO's supreme allied
commander earlier this month. While some aspects of the alliance may need work, Jones said that, on the whole, it is
an "incredibly healthy organization." Jones assumed his office in January 2003 after serving as the commandant
of the Marine Corps. During his time in the position, the alliance has changed dramatically. "Perhaps the highlight of the last
four years was witnessing the accession of seven new nations into the alliance in 2004," he said. "It was a very emotional
moment for seven former Warsaw Pact countries." Membership in NATO meant acceptance in the free world to the former
communist countries, Jones said. "There was a sort of palpable enthusiasm for freedom, democracy, rule of law and just the
vast potential for those people that had been unleashed," he said. "You feel every day their enthusiasm from these new
members." During Jones' tenure, the NATO-led International Security Assistance Force in Afghanistan grew from a force
providing security in and around the Afghan capital of Kabul, to providing security for the entire country. The NATO commander
in Afghanistan now commands 32,000 troops from 32 different countries, Jones said. The
NATO mission in
Afghanistan and NATO training mission in Iraq are just two operations that show the term
"out of area operations" is obsolete, he said. During the Cold War, NATO's job was to defend Western Europe
from the menace of the Soviet Union and the Warsaw Pact. There were no "out-of-area operations, nor was the possibility even
really contemplated," he said. "It is a given that NATO is operating today on three different continents with more than 50,000
troops committed to NATO missions," he said. Troops
under NATO command operate in Asia, Africa and
Europe, and Jones said the alliance is also embracing change. "Nowhere was that more in evidence than in
establishing the NATO Response Force," he said. The force - 25,000 personnel ready to deploy at a moment's notice - is now
fully operational and capable. The general said the force is NATO's greatest commitment to transformation. The force is ready
to "take on missions at a strategic distance, but in an expeditionary manner," he said. The NATO Response Force's first real
deployment - to Pakistan to help with humanitarian relief following the earthquakes in January 2005 - is a prime example of this,
Jones said. The fact that the force's first mission was a humanitarian operation has also caused some reassessment in NATO, he
said. "NATO is reinventing itself and re-explaining itself because in this world NATO is thought of, correctly, as
principally a warfighting organization," he said. "This transformation of NATO - going from a reactive 20th-century force, which
it needed to be, to a 21st-century more expeditionary and agile force - brings with a whole lot of things" that countries didn't
realize when they signed up for the process in 2002. "It has caused a lot of pain because it gets you into such things as
multinational logistics (and) organic intelligence, which NATO has never had," he said. Other transformational aspects during
Jones' command included eliminating duplicate NATO headquarters, disestablishing the Alled Command Atlantic and replacing it
with the Allied Command Transformation and placing all operations under Allied Command Europe. This is not to say there are
not problems that NATO must address, Jones said. First and foremost is money. The per capita share of many countries has
actually gone down since the Prague Summit in 2002. NATO nations agreed during that summit to spend roughly 3 percent of
their gross domestic product on defense. Another problem is national caveats, Jones said. This is where troops assigned to a
mission has such stringent restrictions placed on them, that commanders can hardly use them. But the
alliance is
remarkably adaptable and resilient, Jones said. "The other bit of evidence that the alliance is
healthy is that I know of no countries that are trying to leave the alliance," Jones said. "And I
know quite a few that are trying to queue up and measure up to become members by as early as 2008."

Too many alt causes to NATO

Jankowski ’11 (10/31 (Domink, expert analyst at the National Security Bureau of the Republic of
Poland and is pursuing a doctorate at the Warsaw School of Economics, "A Post-Libya NATO
Assessment," http://nationalinterest.org/commentary/post-libya-nato-assessment-6016?
page=1)
From the NATO perspective, two worrisome trends have emerged. First, despite the political backing for
operation “Unified Protector,” fewer than one-third of NATO allies actually participated in strike
missions, and fewer than half contributed contingents. That raises a question: Why so few? Some countries, especially the more
recent entrants, have been extensively engaged in other NATO operations (Afghanistan, Kosovo). They may believe a
contribution to another mission would overstretch their capabilities. Others lack the necessary air and naval assets that would mesh
operationally with those of their allies. Both situations reveal, however, another pan-European weakness: the decline in
defense spending. The financial crisis has become the new normal . It changed the logic of international
relations, ushering in a new era marked by intensifying “zero-sum” geopolitical rivalries. Thus, only four European countries are
meeting the minimum threshold of 2 percent GDP expenditures on national defense. From the EU perspective, the
situation seems even worse. One of the recent issues of the prestigious European magazine Europe’s World contained an
eye-catching advertisement for NATO: “Question: Which organization adopted a new vision of its geopolitical role in Lisbon? Hint: It
wasn’t the European Union!” The ad’s not so subtle jibe has been borne out by the Libyan crisis, which caught the EU by surprise. In
fact, there is a growing sense of ambiguity about the real outcome of the EU’s crisis-management policy. Despite being the subject
of occasional good news, it is hardly an unalloyed success. That is in part because of two main operational obstacles the EU
continues to face. First and foremost, the EU still lacks adequate civilian and military capabilities. The second obstacle is inherent in
the EU’s institutional structure and how it works. The bureaucracies responsible for foreign and security policy—including the
European External Action Service, the EU’s diplomatic corps—are still essentially under construction.

No impact

Conry 95 (Barbara, Foreign Policy Analyst – Cato, Cato Policy Analysis, “The Western European Union as NATO’s Successor”, 9-18,
http://www.cato.org/pubs/pas/pa-239.html)

Europe after NATO: Bogus Nightmare Scenarios It is inaccurate to suggest, as NATO partisans often
do, that the only alternative to Atlanticism is a return to the dark ages of the interwar era: nationalized
European defenses, American isolationism, xenophobia, demagoguery, and the other evils associated with the rise of Hitler and
World War II. Former U.S. senator Malcolm Wallop (R-Wyo.) warns that weakening NATO will have dire consequences. "As we
have thrice before in this dreadful century, [we will] set in motion an instability that can only lead to war, shed blood, and lost
treasure. Pray that we are wiser."(4) Lawrence di Rita of the Heritage Foundation similarly defends NATO as an "insurance
policy" against a future world war. "If keeping 65,000 young Americans in Europe will prevent 10 times that many new
headstones in Arlington cemetery once the Europeans turn on themselves again--as they have twice this century--then it's a
small price to pay."(5) Such alarmism underestimates the significance of 50 years of economic
and political cooperation among the West European powers and the role of pan-European
institutions such as the Organization for Security and Cooperation in Europe. It also ignores the fact that a
viable institutional alternative to NATO--the Western E uropean U nion--already exists.
With the proper resources and recognition on the part of Washington and the Europeans that an independent
European defense is essential in the post-Cold War era, the WEU is a promising alternative to Atlanticism. Far
from being a lame second choice to NATO or defense on the cheap, a robust WEU would be
superior to NATO in many ways, better suited in the long run to protecting European and, indirectly,
American interests.

NATO is irrelevent

Schake 12/26 (Kori, fellow at the Hoover Institution and an associate professor at the U.S. Military Academy, "NATO After Libya:
Why the Alliance Is Worth Saving," http://www.thedailybeast.com/newsweek/2011/12/25/nato-after-libya-why-the-alliance-is-
worth-saving.htmlhttp://www.thedailybeast.com/newsweek/2011/12/25/nato-after-libya-why-the-alliance-is-worth-saving.html)
But Libya was also a warning: NATO remains utterly
dependent on American air power and munitions to
beat even a third-rate enemy like Gaddafi. How much longer will U.S. lawmakers and taxpayers
continue to provide that support? Why should they underwrite the performance of European
militaries when Europe’s own taxpayers refuse to do so? In a farewell speech this past June in Brussels, then–
U.S. defense secretary Robert Gates gave a dismal assessment of the alliance’s Libyan performance,
warning that NATO had shown itself to be at risk of “collective military irrelevance.” NATO’s
membership has more than doubled to 28 countries since its inception in 1949, but its basic principle remains the same: “The Parties
agree that an armed attack against one or more of them in Europe or North America shall be considered an attack against them all.”
Only once in NATO’s 62-year history has its mutual-defense clause been invoked: after the Sept. 11 attacks on America. More than
10 years later, NATO allies continue to fight America’s fight, with 40,000 non-U.S. troops deployed in Afghanistan. That’s loyalty.
Imagine how hard it would be for an American president to sustain involvement in someone else’s war after 10 years. Yet NATO has
been a good deal for the Europeans and Canadians: for the past six decades the world’s strongest military has been committed to
their defense. Nearly 80,000 U.S. military personnel are currently stationed in Europe, working closely and routinely under NATO’s
integrated military command, sharing expertise, plans, equipment, and training. In fact, Europeans have more influence over U.S.
policies in NATO than in any other forum. Its rules, originally drawn up for Western Europe’s defense, give the European allies so
much leverage that Washington insisted that its treaty obligations be limited to “Europe or America,” carefully excluding the
Europeans’ colonial possessions in Asia and Africa. (Times do change—in recent years the United States has been the leading
proponent of NATO’s security responsibilities around the world.) The allies can count on America to step in whenever they lack the
required capabilities. The British and the French may have flown most of the missions during Operation Unified
Protector, but it was U.S. personnel and equipment that destroyed Libya’s air defenses at the
outset, firing more than 100 Tomahawk cruise missiles on the first day. As the intervention continued, the Americans
provided 80 percent of the necessary intelligence ; dispatched targeting specialists to make up
for a shortage of qualified European technicians; and supplied additional munitions to the other
allies when they ran short. And it was practically inevitable that they would run short. Minus the United States,
NATO’s members spend roughly $150 billion a year on defense, a figure that’s not even close to
the U.S. expenditure—excluding war costs—of around $560 billion. That’s fully half the world’s total. Russia and China
each spend about $100 billion (although it’s hard to be sure about China, because its reporting is so opaque). Libya itself spent $1
billion in the year before Gaddafi’s overthrow. All told, the alliance has nearly 2 million military members, including battle-hardened
veterans of the Balkans, Iraq, and Afghanistan. Seven of the world’s 10 best militaries are European members of NATO. Even without
U.S. participation, the Europeans should be able to defeat any potential enemy. The trouble is, they no longer believe they can.
Britain openly admits its dependence on the United States. Since 1997, the United Kingdom has predicated its defense planning on
the assumption that it will not fight a war without American assistance. While
Eurochauvinists may harbor notions
of independence, the truth is that few of them are willing to fight unless America is at their
side, no matter now noble the cause may be.
---NATO- Resilient

NATO unity high now

Gordon ‘9 (Philip H. Gordon, 12/9/2009 (Assistant Secretary, Bureau of European and Eurasian Affairs,
http://www.state.gov/p/eur/rls/rm/2009/133417.htm)

As I said, I began by highlighting Afghanistan because it is


emblematic of how the United States and
Europe can and do cooperate on the most important global challenges of the day. In that respect, I want to
make two points absolutely clear tonight: First, the United States looks forward to working with a strong, cohesive Europe as a
partner in meeting the security and economic challenges of the 21st century. And second, we
have already seen in the
first year of this administration an extraordinarily high—and possibly unprecedented—level of unity
and common purpose as the United States and Europe have stood shoulder to shoulder to
face gathering global threats. I have been working on U.S.-Europe issues for several decades, and would dare say
that I don’t think there has been a time in my professional career when our global
strategies are as in sync as they are today.

Past events prove NATO is resilient

Corn 7 (Tony, Ph.D. – University of Paris and Graduate – U.S. Naval War College, “The Revolution in Transatlantic Affairs”, Real
Clear Politics, 8-21, http://www.realclearpolitics.com/articles/2007/08/the_revolution_in_transatlanti.html)

If the Alliance survived a debacle of the magnitude of Suez in 1956, it can withstand
anything. The main danger for NATO therefore is not military failure or even a Suez-like
temporary political meltdown, but something more insidious. Over time, what an ill-conceived globalization of NATO
could lead to is the transformation of the tactical coalition that the Shanghai Cooperation Organization currently is into a
strategic “NATO of the East” while at the same time perverting the Atlantic Alliance into, so to speak, a “SEATO of the West” —
namely, a make-believe alliance with no viable strategy (because a conventional military configuration is irrelevant when the
threats are of the asymmetric variety) and no coherent policy (because the interests of the global members are simply too
heterogeneous to ever converge.) The Long War promises to be a thinking man’s war. As
a full-fledged Alliance,
NATO possesses the kind of staying power that mere ad hoc coalitions cannot deliver; but
NATO still has to come to terms with the fact that thinking power will matter more than fighting power. If NATO is to avoid the
twofold danger of the SCO becoming a NATO of the East while NATO becomes a mere SEATO of the West, the Alliance will have
first of all to downgrade its “toolbox” dimension and beef up its “think-tank” dimension.
---NATO- Fails Now

NATO has no money – should have tanked credibility

EFTHYMIOPOULOS ’11 - is Visiting Scholar at the Center for Transatlantic Relations SAIS Johns Hopkins and Head of the
think tank Strategy International (Dr. Marios P, “Enhancing NATO’s Financial Performance”, December 21,
http://mariospoints.wordpress.com/2011/12/21/enhancing-natos-financial-performance/)

While NATO is on its final steps for the Lisbon Summit in November 2010, the
question of a viable practical
budgetary finance for the future needs to be clearly addressed. NATO maybe engaged still, in
reviews of its institutional, ethical and operational strategic foundations, however it needs to take into
consideration possibly practical development and application of a new Strategic Concept that
will be financially feasible, given the Alliance’s members budgetary national cuts . A new financially
futuristic viable Alliance with a greater use of technology, rather than human resources in the operational fields or elsewhere
requested, which costs a great amount of money, will project stability and development. It will have a positive impact factor on the
financial needs of the new operations and new challenges and will utterly bring closer, civil to military relations. This following article
is part of A. research on NATO and its new challenges upon application of the new strategic concept post Lisbon Summit. B. This
research is partially conducted at the Woodrow Wilson International Center for Scholars in Washington; while Southeast Europe
Project Policy. It is also part of a lecture on “NATO’s future in an era of financial crisis” that was presented at the Center. The
global economic crisis already affects the defence expenditures of member states (latest casualty to
be the UK), but also the Alliance, as an organization. What is therefore estimated is that NATO should re-examine its
financial posture and expenditure but also positively react with new methods of development at a time of much needed initiatives
for overall development of its organization. Simply put it, if NATO is to deal with global issues, while retaining its
peripheral role, NATO should become the dominant security and joint foreign political-military organization. To be internationally
secure as a dominant power and viable as organization, it
has to construct a robust and futuristic-minded
financial viable organization; as such agreed by members-states , to conduct financial operations in a
globalised world. The report of the 12 member committee that was presented on May 17th 2010 explicitly refers to an array of more
diffused challenges, without however specifying which. They range, but can be narrowed down to the following: International piracy,
terrorism, proliferation of WMD’s, energy security and cyber-attacks, demographic concerns and natural disasters, amongst others.
NATO’s credibility and value were considered to be at stake as was implied at the report. The question on how
would NATO win the support of the wider public sector, how will that be shown at this upcoming Strategic Concept was put in
question.
---NATO- EU Fill-in Solves

No impact to NATO collapse- EU Solves

Gallagher 3 (Michael, Assistant Secretary of Commerce, Winter, Houston Journal of Int’l Law)

NATO’s supporters argue that ending NATO will destabilize Europe. Ending NATO, they claim, will
destroy the transatlantic link between the United States and Europe, and isolate the United States from Europe.
The ties of history, however, prevent this outcome. The United States has long enjoyed a
“special relationship” with the United Kingdom. The United States also has strong relations with such nations as
Italy, Turkey, Germany, the Netherlands, Denmark, and Norway. Some claim that NATO is the
foremost expression of U.S. commitment to Europe. The United States, however, aided Europe in two world wars, and stood
firmly by Europe’s side during the Cold War – this commitment surpasses diplomatic formalities. The
United States will
not isolate itself from Europe merely because NATO disbands. Additionally, European
nations do not need a formal security link to the United States. Even with NATO gone,
“there is still plenty of life in, and need for, [the United States-Europe security]
partnership.”

EU fill-in solves their impact

Tertrais 4 (Bruno, Senior Research Fellow – Foundation for Strategic Research, Washington Quarterly, Spring)

A Europe-wide security guarantee already exists. In 1947, France and the United Kingdom allied themselves
against the possible resurgence of German nationalism in the Pact of Dunkirk. The following year, this arrangement was
enlarged to include three other European nations (Belgium, Luxembourg, and the Netherlands) and became the Brussels Treaty,
which includes a commitment to use military force if one of the treaty's parties is attacked. The Brussels Treaty was modified in
1954 to include Germany and Italy and remains in force today. Article 5 of the Brussels Treaty provided the basis for the
Washington Treaty's own Article 5, which is less demanding because it does not automatically commit NATO members to
military assistance. Emboldened
by their common perception of the Iraq war and the evolution of U.S.
policy, several countries in the EU, including France and Germany, are tempted to deepen their
defense cooperation independently of the United States and NATO. Although the degree to which the EU's
defense policy should be independent from NATO and the United States is still a point of contention among Europeans and a
touchy subject in transatlantic circles, consensus holds that, in the case of a regional crisis, the EU should not be impotent if
Washington chose not to intervene. More recently, the EU made the historical decision to include a security clause in its draft
constitution that amounts to a mutual security guarantee with the possibility of military assistance. The formulation agreed on
by EU governments in December 2003 states: If a Member State is the victim of armed aggression on its territory, the other
Member States shall have towards it an obligation of aid and assistance by all the means in their power, in accordance with
Article 51 of the United Nations Charter. This shall not prejudice the specific character of the security and defence policy of
certain Member States. Commitments and cooperation in this area shall be consistent with commitments under NATO, which,
for those States which are members of it, remains the foundation of their collective defence.n20 The EU thus
increasingly appears to be a real security alliance, and it may in fact be among the most solid ones
imaginable, given its roots in strong economic, legal, and political integration . Thus, in the midst of
the decline of other Western, permanent multinational alliances, the EU's emerging common defense policy appears one of the
most original and interesting developments in the long history of military alliances . If NATO were to decline further
in importance for Europeans, the continent would not be left without a security guarantee . In fact, in
spite of the wishes of all alliance members, the existence of such an EU security clause could even be a logical consistency
contributing toward NATO's decreased importance in the future.
---NATO- Expanding Bad Turn

Expanding the alliance is mission overreach- this collapses both NATO and the EU, and
undermines transatlantic cooperation

Schmidt ‘7 (John R. Schmidt is the senior analyst for Europe in the Bureau of Intelligence and Research at the Department of
State. A career U.S. Foreign Service Officer, he has served as director of the NATO office at the State Department and as director for
NATO affairs at the National Security Council. The Washington Quarterly 30.1 93-106- Winter 06/07 Last Alliance Standing? NATO
after 9/11 John R. Schmidt

While European allies have been working to establish an autonomous ESDP within the EU, the
United States has been tugging in the opposite direction, seeking new roles and missions for
the alliance as the NATO enlargement process and NATO engagement in the Balkans begin to wind down. The attacks on
September 11, 2001, were a seminal event in this regard, demonstrating that the most
important security threats to NATO members, military or otherwise, emanated from outside of
Europe and that NATO was poorly equipped to handle them. Although NATO invoked Article 5
for the first time in its history in the wake of the September 11 attacks and allies came forward
with offers of military support for the subsequent military operation in Afghanistan, the United
States found that European allies had little useful to offer . U.S. rejection of most of the offers
ruffled allied feathers and raised [End Page 97] questions about the relevance of a military alliance where only one member
could project significant, high-end, expeditionary military power. The U.S. response to this conundrum was twofold. The first was to
persuade European allies to pool their limited resources to establish a single, multinational, European-centered NATO Response
Force (NRF), trained and equipped to U.S. standards, that would be able to deploy quickly and fight effectively alongside U.S. forces.
The second, closely related to the first, was to persuade allies that NATO needed to extend its mandate beyond the traditional
borders of Europe so that NATO forces could go out-of-area to where the threats actually were. Even the French were able to
appreciate this logic, and the two initiatives were adopted without serious controversy at the Prague summit in November 2002.
Despite this effort to reinvigorate the alliance, however, NATO found itself deeply enmeshed in one of the most serious crises in its
history just three months later when France, Germany, and Belgium vetoed having NATO undertake precautionary planning to
provide military assistance to Turkey in the event of an invasion by Iraq. Although this incident and the Iraq war itself divided allies
down the middle, reflecting further unraveling of the Cold War consensus, the fact that EU-member allies were themselves divided
over Iraq resulted in very little damage being done to the Prague agenda. Indeed,
in the immediate aftermath of this
crisis, NATO agreed to assume command of the ISAF peacekeeping operation in Afghanistan, the
first out-of-area operation in the history of the alliance. Even the compulsively recalcitrant
French, despite their bitter quarrel with the United States at the time, may have supported this
decision out of a desire to avoid doing too much long-term damage to NATO. Afghanistan and
the NRF have dominated the NATO agenda ever since. Under persistent U.S. prodding, the allies have agreed to
the step-by-step expansion of the ISAF peacekeeping force from Kabul into the provinces, most recently into the former Taliban
heartland of southern Afghanistan, where NATO peacekeeping operations are being seriously tested. Although increasing Taliban
attacks have made ISAF peacemaking operations highly dangerous, the allies have steadfastly resisted U.S. efforts to get them
involved in U.S.-led Operation Enduring Freedom counterterrorism operations, which would put their forces even more seriously in
harm's way. Meanwhile, progress in developing the NRF, which is due to become fully operational in October 2006, has been
hampered by the continuing inability of European allies to devote the resources required to acquire key logistical [End Page 98]
capabilities, such as strategic lift, or to train and equip sufficient numbers of combat troops to U.S. standards. Instead, in something
of a paradox, the NRF, which was originally conceptualized as the antidote to a two-tiered alliance, has come to be used for lower-
end purposes, as in the Kashmir relief operation, because its units are available for rotational call-up. Competing U.S.-French
Although these post–September 11 U.S. initiatives have met with mixed
Ambitions Fuel Frictions
success, the crisis atmosphere that characterized the period leading up to the invasion of Iraq
has faded. NATO has even undertaken a modest training mission there. The primary fault line at NATO
now is of older and more familiar vintage: the long-standing struggle between the United States, concerned as always to maintain
strong U.S. influence in Europe through NATO, and France, just as determined to minimize that influence without actually breaking
the transatlantic link. To be fair to France, Paris is prepared to use NATO in what it regards as appropriate circumstances. Bringing
them along is almost never easy, but the French did approve the NATO takeover of ISAF as well as the NATO training mission in Iraq
and have been vocal supporters of the NRF, although their interest here may well be driven in part by the hope that it will produce
military capabilities that can be used by the EU. Nevertheless, just as the United States has sought to ensure that the ESDP remains
firmly anchored within NATO, France has pressed for greater ESDP autonomy and opposed U.S. efforts
to strengthen NATO or move it in directions that Paris believes are more properly the province
of the EU. This has precipitated a tug of war both within the EU, as described above, and within
NATO. Although France is not without allies, most EU-member NATO allies seek a middle road. They favor a strong
and autonomous ESDP and would probably be prepared to go further in this direction than the United States would like, but they are
generally not prepared to cross the U.S. redlines discussed above. This reflects an enduring commitment to maintain close relations
with the United States despite the absence of any palpable military threat to their security. Yet, just as they are unwilling to go as far
are similarly disinclined to go as far as the
as the French would like in pushing ESDP independence from NATO, they
United States would like in finding new roles and missions for the alliance, particularly if they
conflict with perceived EU prerogatives, or in funding the NRF. Not surprisingly, Franco-U.S.
disagreement over the proper roles of NATO and the EU has complicated relations between the
two organizations. Despite their overlapping mandates and the fact they have 19 members in common, there is only minimal
systematic interaction, very little transparency, and even [End Page 99] less coordination. The one exception was Bosnia, where the
handoff to the EU when it did come was accomplished relatively smoothly, in accordance with Berlin Plus procedures. The lack of
coordination between the two organizations, however, is not simply a consequence of Franco-U.S. arm wrestling over which forum
should predominate but reflects other rivalries as well. This includes long-standing Greek-Turkish animus, which has become even
more complicated following Cypriot accession to the EU. There
are also rivalries between the NATO and EU
bureaucracies in individual foreign ministries as well as between the NATO and EU international
staffs. For the most part, these frictions have precipitated little direct conflict because each
organization has tended to focus on different missions . NATO has focused on larger-scale and more difficult
peacekeeping operations, as in Bosnia (at its beginning), Kosovo, and Afghanistan, while the EU has taken on much smaller-scale or
more settled peacekeeping operations and rule-of-law missions. That there is a potential for conflict, however, was amply
demonstrated by the recent flap over who should provide air transport for African Union forces in Sudan, a controversy that
Despite these dissonances, the United States has continued to
generated much ado over relatively little.
press an ever more ambitious agenda on NATO. In Afghanistan, the United States has been steadily pressuring
allies to broaden the envelope of risk they are willing to shoulder in conducting peacekeeping operations. To boost the flagging NRF,
Washington is urging European allies to support a strategic airlift initiative that would provide the air transport and other logistical
resources needed to help wean the NRF from its dependence on the United States. In the run-up to the Riga summit in November
2006, the United States has been proposing that NATO expand its contacts with non-European Western allies, such as Japan,
Australia, and South Korea, which would give the alliance an even more global focus. Washington has also supported the call of
Secretary General Jaap de Hoop Scheffer for enhanced political discussion at NATO. German chancellor Angela Merkel picked up this
theme herself in her February 2006 Wehrkunde Conference speech, in which she supported using NATO as a forum for discussing
and possibly coordinating positions on a wide range of foreign policy and security topics, specifically mentioning the Middle East and
Iran. The United States is also pressing NATO to take on a more substantial role in the Middle East and Africa by seeking to establish
military training centers. Taken together, these
initiatives map out an ambitious vision of an increasingly
globally focused alliance taking on a progressively wider range of potential issues, activities,
and missions, with the United States firmly in the lead . [End Page 100] The EU and NATO: Fundamentally
Different Organizations If these trend lines are clear, the prospects for the future are not. U.S.
ambitions for NATO clearly conflict with French ambitions for the EU . Furthermore, even though
most EU NATO members find themselves caught in the middle, the current zero-sum nature of
NATO-EU relations seems to portend continuing turmoil on the road ahead, to the detriment
of both organizations and of transatlantic relations more generally . At the end of the day, the answer to
how far Washington can take the alliance may depend as much on U.S. preferences as it does on how far the French and other NATO
allies are prepared to have it go. Similarly, for the EU, the availability of resources, not just ambition, will have a profound effect on
In considering the art of the possible, NATO
what kind of security and defense role the EU can play in the future.
and the EU are fundamentally different kinds of organizations. NATO is a defense alliance
whereas the EU has the trappings of a supranational state . As part of a defense alliance, NATO members
agree to defend each other in case of an attack, while EU members pledge to surrender various aspects of their national sovereignty
across the full spectrum of governance, involving foreign and domestic issues. The EU's establishment of security and defense
structures under the ESDP that putatively duplicate NATO structures is best seen as part of this broader process. In the realm of
foreign affairs, the EU mandate extends well beyond that of NATO. The EU has given CSFP High Representative Javier Solana much
more authority to pursue diplomatic initiatives on behalf of the EU than NATO has ever bestowed on a secretary general. The EU
also routinely employs special representatives to act diplomatically on behalf of the organization, most prominently in Bosnia, where
the EU high representative plays a critical role in national politics, something NATO has never done. Despite the recent rejection of
the EU Constitution in two member states, which indefinitely stalled the latest attempt at further formal integration, it is reasonable
to expect that the EU will continue to play an active, perhaps increasingly active, diplomatic role in world affairs. It is difficult to
envision NATO taking on this kind of a role, at least to anywhere near the same extent. This is not simply a matter of French
opposition but of U.S. preference. Expanding the range of political issues to be discussed at NATO, as de Hoop Scheffer has
proposed, is one thing. Undertaking diplomatic initiatives under a NATO flag is quite another. This would require the United States to
subordinate its diplomatic freedom of action [End Page 101] to political oversight by NATO, something it has never been willing to
do, given the constraints this would place on U.S. flexibility, particularly on critical international issues such as the Middle East.
Should the United States decide to pursue selected diplomatic initiatives through NATO
nonetheless, prospects for success would be uncertain at best. Sympathetic EU members of
NATO might be persuaded to go along, depending on the issue, while others might welcome
the opportunity to constrain what they perceive to be unilateralist U.S. tendencies . Merkel's
speech at Wehrkunde, for example, has a hint of the latter . The French and their allies,
however, would probably reject the notion out of hand, motivated by what they would
consider to be a direct threat to EU foreign policy prerogatives . The French are happy enough
to work with the United States but seek to do so in the UN Security Council, the Group of Eight,
or U.S.-EU forums, where their own influence can be maximized relative to the United States.
The same fate could also await any U.S. effort to encourage NATO to play a leading role in
addressing security issues that are not primarily military in nature, such as terrorism or energy
security. After the September 11 attacks, a working proposal was developed within the U.S.
government to establish a counterterrorism cell at NATO to help track terrorists and coordinate
arrests. This proposal never made it to NATO, owing to a long-standing preference for using
traditional, primarily bilateral channels in some quarters of the U.S. government . The proposal
would almost certainly have encountered opposition from European allies as well, partially
because of the same concerns but also because the French and others would have perceived
such a function as falling outside the essentially military mandate of NATO . The fate of other
proposals, such as the recent suggestion that NATO discuss energy security, is likely to depend on the degree of the ambition. The
EU is already active on many of these issues, and in the face of almost certain French opposition,
the burden of proof would fall on the United States to demonstrate that meaningful NATO
involvement would add significant value rather than duplicate ongoing efforts elsewhere.
Natural Gas Spikes Answers
No price spike

Menza 12 (Justin Menza, News Writer at CNBC, Financial Journalist at UBS Investment Bank Sr.
Financial Writer at Standard & Poor's , 8/22/2012, "No Spike in Natural Gas Looming: Boone
Pickens", www.cnbc.com/id/48752448/No_Spike_in_Natural_Gas_Looming_Boone_Pickens)

The U.S. should continue to have a cheap energy advantage compared to the rest of the world , T.
Boone Pickens, BP Capital founder, told CNBC’s “Squawk Box” on Wednesday. “There's going to be a fabulous opportunity
for natural gas, but we're not there yet,” Pickens said. Pickens expects natural gas prices to climb to $4 by the
end of the year, but no major price spike. “You can make money at $4,” he said, “Of course, it's going to be better
than at $2 or $3, but you aren't going to get many wells drilled.” Instead, Pickens sees a greater chance of a spike in
crude oil prices. He’s predicting $115 a barrel on West Texas Intermediate crude by year’s end. Flare ups in the Middle East
could cause a spike, Pickens cautioned. “If (Israel) bombs Iran, you're going to have a spike up in oil price, there's no question about
that, just because they bombed Iran, not because you're going to have a shortage of oil immediately,” he said. Nonetheless,
the U.S. will continue to have the world’s cheapest energy. “U.S. crude is 15 percent cheaper
than Brent North Sea crude oil , and natural gas is 75 percent cheaper than China, the Mideast, Japan, or wherever else,”
Pickens said.

No risk of volatility – game-changers in supply and demand solve


Staple ‘11, CEO American Clean Skies Foundation, and Szydlowski, President & CEO of the
Bipartisan Policy Center, March 2011

(Gregory and Norm, “TASK FORCE ON ENSURING STABLE NATURAL GAS MARKETS,”
http://www.cleanskies.org/wp-content/uploads/2011/05/63704_BPC_web.pdf)

The findings and recommendations in this report reflect optimism that the robust supply
horizon for natural gas presents fresh opportunities—not only to move beyond prior market
concerns but to develop new tools for managing price uncertainty . Fundamental changes in
the domestic supply and demand balance for natural gas, including an unprecedented level of
available storage and import capacity, should allow markets to function more efficiently and
fluidly in the future. This should create more favorable investment conditions and significantly
dampen the potential for destructive cycles of price volatility and market instability.

Prefer our ev—consensus


Staple ‘11, CEO American Clean Skies Foundation, and Szydlowski, President & CEO of the
Bipartisan Policy Center, March 2011

(Gregory and Norm, “TASK FORCE ON ENSURING STABLE NATURAL GAS MARKETS,”
http://www.cleanskies.org/wp-content/uploads/2011/05/63704_BPC_web.pdf)
The Task Force on Ensuring Stable Natural Gas Markets (hereafter “Task Force”) was jointly
convened by the Bipartisan Policy Center and the American Clean Skies Foundation in March
2010 to examine historic causes of instability in natural gas markets and to explore potential
remedies. The membership of the Task Force is unique in its diversity and unique in the sense
that it brings together key stakeholders from both sides of the supply–demand equation .
Individual Task Force members are listed in the Preface; they represent natural gas producers,
transporters and distributors, consumer groups and large industrial users, as well as
independent experts, consumer advocates, state regulatory commissions and environmental
groups.

Prices are fine for at least five years


Kasey ‘12, writer for The State Journal (West Virginia), 9/14/2012
(Pam, “US natural gas production on track to top 2011 record in 2012,”
http://www.statejournal.com/story/19544273/us-natural-gas-production-on-track-to-top-2011-
record-in-2012)

Despite low prices, U.S. natural gas producers are on track in 2012 to top their record 2011
production.

Gas produced in the first six months of 2012 came to 11.9 trillion cubic feet, or tcf, compared
with 11.2 tcf in the first six months of 2011, according to the U.S. Energy Information
Administration.

Total production in 2011 of 23 tcf exceeded 2010 production by 7.8 percent and topped the
previous record, set in 1973, of 21.7 tcf.

Volume has been higher in every month of 2012 compared with 2011. The monthly average for
January through June 2012 was 1.98 tcf, compared with the 2011 average of 1.92 tcf — putting
the industry on track to produce 23.8 tcf this year.

Daily production has topped 65 billion cubic feet per day, or bcfd, this year, compared with 63
bcfd in 2011.

Prices dipped to a decade low below $2 per million British thermal units in April and, at under
$3, remain low.

But industry executives have explained to The State Journal in the past that leases typically are
constructed with five-year expirations, putting time pressure on producers and creating a long
lag in price responsiveness. Contracts for the use of drilling rigs, often set up on an annual basis,
also make nimble response to low prices difficult.
Naval Power Answers
Frontline
The US is so far ahead that this impact is a joke

Work 9 (Robert, VP of Strategic Studies @ Center for Strategic and Budgetary Assessments, “Strategy for the Long Haul: the US
Navy Charting A Course for Tomorrow’s Fleet”,
http://www.csbaonline.org/4Publications/PubLibrary/R.20090217.The_US_Navy_Charti/R.20090217.The_US_Navy_Charti.pdf)

On August 1, 2008, the TSBF numbered 280 ships of all types (see Figure One).3 Predictably, naval
advocates fretted that the
smaller fleet posed a great risk to US national security . For example, Seth Cropsey, a Deputy
Undersecretary of the Navy in the Reagan and George H. W. Bush administrations, cautioned that, “Without
intending it, US policy is verging toward unilateral naval disarmament.” 4 He went on to say: The Navy’s focus
is [unclear]. Its [280] combat ships — a number that House Armed Services Committee Chairman Ike Skelton called “shocking” — comprise a force that
is less than half the size achieved during the Reagan years . . . The last time the US possessed so small a fleet was sometime between December 1916
and April 1917, on the eve of the nation’s entry into World War I. While technically true, these dire comments are misleading . Of
the many ways to gauge US naval power, comparing the size of the current US battle force to that of past US fleets is the least useful. Past TSBFs are
reflections of different strategic environments, federal budgets, national grand strategies, and stages of technological development. They also reflect
the state of the contemporary global naval competition. In 1916, although the TSBF numbered only 245 ships of all types, the 36 battleships of the
Navy’s battle line placed it second among world navies behind the British Royal Navy. Despite having “only” 245 ships, it could safely assume it would
never have to fight the Royal Navy, and be relatively confident that it could fight and defeat any other navy in the world. During the 1980s, even as it
grew to a post-Vietnam high of nearly 600 vessels, the Navy was fighting off a concerted effort by the Soviet Navy to knock it out of the top spot.5 In
other words, whether today’s TSBF is as big as the US fleets in 1916 or 1987 is utterly irrelevant.Far more important is the answer
to the following question: how does the US Navy stack up against its potential contemporary
competitors? And the answer to this question paints a very different picture than comparing
today’s TSBF with that of past US fleets . SECOND TO NONE The first true indicator of US naval dominance comes from
comparing the size of the US battle force with other world navies. What alarmists over fleet size fail to mention is that although the US TSBF

is the smallest it has been in over ninety years, so too are the rest of the world’s navies. 6 At the
height of its naval dominance, Great Britain strove to achieve at least a “two-navy standard.” That is, the Royal Navy aimed
to maintain a fleet and battle line that was as large as the combined fleets of the two closest
naval powers. Today, counting those ships that can perform naval fire and maneuver in distant theaters — aviation platforms of all types,
tactical submarines (nuclear and diesel-electric attack boats and conventional guided-missile submarines), and surface combatants and amphibious
ships with full load displacements greater than 2,000 tons7 — the next two largest contemporary navies belong to Russia and the People’s
Republic of China (PRC). Together, they operate a total of 215 warships of all types. The US Navy alone

operates 203 such warships, very close to, but not quite, a two-navy standard.8 However, when factoring in a second
important indicator of naval power — aggregate fleet displacement (tonnage) — the US Navy enjoys considerably more

than a two-navy standard . As naval analyst Geoffrey Till explains, “[t]here is a rough correlation between
the ambitions of a navy and the size and individual fighting capacity of its main units, provided they
are properly maintained and manned.”9 Therefore, full load displacements and aggregate fleet warship

displacements are the best proxies available to measure a ship’s and a fleet’s overall combat
capability, respectively. Accordingly, both are useful measures for sizing up the contemporary global hierarchy of naval competitors.10 When
considering aggregate fleet displacements, the US Navy’s overwhelming advantage in combat capability is readily apparent. Besides the

United States, there are only twenty navies in the world that operate fleets with aggregate
displacements of 50,000 tons or more. In order of fleet displacement (largest to smallest), these navies are operated by: Russia, the
PRC, Japan, the United Kingdom, France, India, Taiwan, Italy, Indonesia, Spain, South Korea, Brazil, Turkey, Australia, Greece, Canada, Germany, the
Netherlands, Peru, and Singapore. Together, these
twenty navies operate a total of 719 ships with a combined displacement of
3,632,270 tons.11 In comparison, the combined displacement of the US Navy’s 203 fighting warships totals
3,121,014 tons — which exceeds the total tonnage of warships operated by the next thirteen navies combined. In other words, in terms
of overall fleet combat capability, the US Navy enjoys a thirteen-navy standard . However, it is
important to note that of the twenty countries discussed above, eighteen are formal US allies
(Australia, Canada, France, Germany, Greece, Italy, Japan, the Netherlands, South Korea, Spain, Turkey, and the United Kingdom), governments

friendly to the United States, (Peru, Brazil, Indonesia, and Singapore), or emerging strategic partners (India).
Moreover, all of these nations are either full or partial democracies. The likelihood of the
United States ever finding itself in a war or naval confrontation with any of these countries is
extremely remote. Indeed, if anything, during times of crisis the US Navy can normally count on
receiving important naval contributions from some or all of these nations. At the turn of the twentieth
century, the officers of the British Royal Navy concluded that they would never again fight the US Navy, and could remove its rapidly expanding fleet
from calculations over the minimal two-navy standard. Similarly, eight years after the turn of the twenty-first century, the US can confidently exclude
these eighteen navies from its naval force planning calculations. This is the implicit message of the Navy’s recently published Cooperative Strategy for
21st Century Seapower, which seeks to foster and sustain cooperative maritime relationships with more international partners.12

No challengers to US naval power-

Farley 7 (Roberts, Assistant Professor @ the Patterson School of Diplomacy and International Commerce, "The False Decline of
U.S. Navy," Oct 23, http://prospect.org/cs/articles?article=the_false_decline_of_the_us_navy,

We live in strange times. While the United States is responsible for close to 50 percent of aggregate world military expenditure, and
maintains close alliances with almost all of the other major military powers, a community of defense analysts continues to insist that
we need to spend more. In the November issue of The Atlantic, Robert Kaplan asserts that United States hegemony is under the
threat of “elegant decline,” and points to what conventional analysts might suggest is the most secure element of American power;
the United States Navy. Despite the fact that the U.S. Navy remains several orders of magnitude more
powerful than its nearest rival, Kaplan says that we must beware; if we allow the size of our Navy to further
decline, we risk repeating the experience of the United Kingdom in the years before World War I. Unfortunately , since no
actual evidence of U.S. naval decline exists, Kaplan is forced to rely on obfuscation, distortion,
and tendentious historical analogies to make his case. The centerpiece of Kaplan’s argument is a comparison of the
current U.S. Navy to the British Royal Navy at the end of the 19th century. The decline of the Royal Navy heralded the collapse of
British hegemony, and the decline of the U.S. Navy threatens a similar fate for the United States. The only problem with this
argument is that similarities between the 21st century United States and the 19th century United Kingdom are more imagined than
real. It’s true that the relative strength of the Royal Navy declined at the end of the 19th century, but this was due entirely the rise of
the United States and Germany. But the absolute strength of the Royal Navy increased in the late 19th and early 20th centuries, as
the United Kingdom strove to maintain naval dominance over two countries that possessed larger economies and larger industrial
bases than that of Great Britain. In other words, the position of the Royal Navy declined because the position of the United Kingdom
declined; in spite of this decline, the Royal Navy continued to dominate the seas against all comers until 1941. Britain’s relative
economic decline preceded its naval decline, although the efforts to keep up with Germany, the United States, and later Japan did
serious damage to the British economy. The United States faces a situation which is in no way similar. Returning to the present,
Kaplan takes note of the growth of several foreign navies, including the Indian, Chinese, and Japanese. He points out that the
Japanese Navy has a large number of destroyers and a growing number of submarines. He warns that India “may soon have the
world’s third largest navy” without giving any indication of why that matters. Most serious of all, he describes the threat of a
growing Chinese Navy and claims that, just as the Battle of Wounded Knee opened a new age for American imperialism, the
conquest of Taiwan could transform China into an expansionist, imperial power. The curious historical analogies aside, Kaplan is
careful to make no direct comparisons between the growing navies of foreign countries and the actual strength of the United States
Navy. There’s a good reason for this oversight; there
is no comparison between the U.S. Navy and any navy
afloat today. The United States Navy currently operates eleven aircraft carriers. The oldest and
least capable is faster, one third larger, and carries three times the aircraft of Admiral Kuznetsov, the
largest carrier in the Russian Navy. Unlike China’s only aircraft carrier , the former Russian Varyag,
American carriers have engines and are capable of self-propulsion. The only carrier in Indian
service is fifty years old and a quarter the size of its American counterparts. No navy besides the
United States’ has more than one aircraft carrier capable of flying modern fixed wing aircraft. The United
States enjoys similar dominance in surface combat vessels and submarines, operating twenty-two
cruisers, fifty destroyers, fifty-five nuclear attack submarines, and ten amphibious assault ships (vessels roughly equivalent to most
In every category the U.S. Navy combines presumptive numerical superiority
foreign aircraft carriers).
with a significant ship-to-ship advantage over any foreign navy. This situation is unlikely to
change anytime soon. The French Navy and the Royal Navy will each expand to two aircraft carriers over the next decade.
The most ambitious plans ascribed to the People’s Liberation Army Navy call for no more than
three aircraft carriers by 2020, and even that strains credulity, given China’s inexperience with
carrier operations and the construction of large military vessels. While a crash construction program might conceivably give the
Chinese the ability to achieve local dominance (at great cost and for a short time), the
United States Navy will continue
to dominate the world’s oceans and littorals for at least the next fifty years. In order to try to
show that the U.S. Navy is insufficient in the face of future threats, Kaplan argues that we on are
our way to “a 150 ship navy” that will be overwhelmed by the demands of warfighting and global economic maintenance.
He suggests that the “1,000 Ship Navy” proposal, an international plan to streamline cooperation between the world’s navies on
maritime maintenance issues such as piracy, interdiction of drug and human smuggling, and disaster relief, is an effort at “elegant
decline,” and declares that the dominance of the United States Navy cannot be maintained through collaboration with others. It’s
true that a 600 ship navy can do more than the current 250-plus ship force of the current U.S. Navy, but Kaplan’s
playing a
game of bait and switch. The Navy has fewer ships than it did two decades ago, but the ships it
has are far more capable than those of the 1980s. Because of the collapse of its competitors, the
Navy is relatively more capable of fighting and winning wars now than it was during the Reagan
administration. Broadly speaking, navies have two missions; warfighting, and maritime maintenance. Kaplan wants to confuse
the maritime maintenance mission (which can be done in collaboration with others) with the warfighting mission (which need not
be). A navy can require the cooperation of others for the maintenance mission, while still possessing utter military superiority over
any one navy or any plausible combination of navies on the high seas. Indeed, this is the situation that the United States Navy
currently enjoys. It cannot be everywhere all at once, and does require the cooperation of regional navies for fighting piracy and
smuggling. At the same time, the U.S. Navy can destroy any (and probably all, at the same time) naval challengers. To conflate these
two missions is equal parts silly and dishonest. The
Navy has arrived at an ideal compromise between the
two, keeping its fighting supremacy while leading and facilitating cooperation around the world
on maritime issues. This compromise has allowed the Navy to build positive relationships with the navies of the world, a fact
that Kaplan ignores. While asserting the dangers posed by a variety of foreign navies, Kaplan makes a
distortion depressingly common to those who warn of the decline of American hegemony; he
forgets that the United States has allies. While Kaplan can plausibly argue that growth in Russian
or Chinese naval strength threatens the United States, the same cannot reasonably be said of
Japan, India, France, or the United Kingdom . With the exception of China and Russia, all of the most
powerful navies in the world belong to American allies. United States cooperation with the navies of NATO,
India, and Japan has tightened, rather than waned in the last ten years, and the United States also retains warm relations with third
tier navies such as those of South Korea, Australia, and Malaysia. In any conceivable naval confrontation the
United States will have friends, just as the Royal Navy had friends in 1914 and 1941. Robert Kaplan wants to warn the
American people of the dangers of impending naval decline. Unfortunately, he’s almost entirely wrong on the facts. While the
reach of the United States Navy may have declined in an absolute sense, its capacity to fight and
win naval wars has, if anything, increased since the end of the Cold War. That the United States
continues to embed itself in a deep set of cooperative arrangements with other naval powers
only reinforces the dominance of the U.S. Navy on the high seas. Analysts who want to argue for greater U.S.
military spending are best advised to concentrate on the fiascos in Iraq and Afghanistan.

Naval power is good for nothing


Reed 8 [John T. Reed, West Point Graduate and platoon leader in the 82nd Airborne Division., June, 2008.<"Are U.S. Navy surface
ships sitting ducks to enemies with modern weapons?"http://www.johntreed.com/sittingducks.html]

I have read media stories that said whenever


the U.S. Navy did computer war games against the Soviet
Union, all significant U.S. Navy surface ships were destroyed by the Soviets within about 20
minutes of the start of the computerized war . How? Nukes. A reader says that the Soviet submarines
in the Cuban missile crisis had nuclear torpedoes which they would have used if we did an
amphibious landing. I have no way to confirm that. Although the Navy ships and their carrier-based
planes perform spectacularly well against third-rate enemies like Afghanistan and Iraq, I wonder
how they would do against Argentina or any other enemy equipped with modern weapons. In short, I
wonder if U.S. Navy surface vessels are obsolete . Think about it. They are large, slow-moving, metal
objects that float on the surface of the ocean—in the Twenty-First Century! Ocean liners were the main
way to get across the oceans for civilian passengers until the second half of the Twentieth Century. Since then, most people have
used planes because they are much faster and cheaper. Except the U.S. military. Civilians essentially got rid of their “navy” around
1950. Only the hidebound military would still have a Navy in the Twenty-First Century. Nowadays, civilians only ride passenger ships
for pleasure cruises. An argument can be made that the Navy does the same. Only maybe the old line, “you can tell the men from
the boys by the size of their toys” is a more accurate way to put it. Navy brass want to grow up to captain a ship. A big ship. The
bigger the better. Before WW II, they wanted to be captains of battleships. After WW II, British historian B.H. Liddell Hart said, “A
battleship had long been to an admiral what a cathedral is to a bishop.” Now Navy officers want to captain aircraft carriers. Very
exciting. Very romantic. Great fun. But obsolete. WW II in the Pacific last time they were not obsolete The last time we used them to
fight worthy opponents was in the Pacific during World War II. At that time, warring navies had to send out slow-moving patrol
planes to search for the enemy’s ships. The motion picture Midway does an excellent job of showing both the Japanese and the
Americans doing this. Low-visibility weather would often hide ships back then. Easily detected- Those days are long gone. Surface
ships are not only easily seen by the human eye absent fog or clouds, they are also easily
detected, pinpointed, and tracked by such technologies as radar, sonar, infrared detectors,
motion detectors, noise detectors, magnetic field detectors, and so forth. Nowadays, you can probably
create an Exocet-type, anti-ship missile from stuff you could buy at Radio Shack. Surface ships can no longer hide from the enemy
like they did in World War II. Satellites- Satellites and spy planes obviate the need for World War II-type patrol planes and blimps,
unless someone shoots them down, in which case planes can accomplish the same thing.. Too
slow- Anti-ship missiles
can travel at speeds up to, what, 20,000 miles an hour in the case of an ICBM aimed at a carrier
task force. Carriers move at 30 knots or so which is 34.6 miles per hour. Too thin-skinned- Can you
armor the ships so anti-ship missiles do not damage them? Nope. They have to stay relatively
light so they can float and go 34.6 miles per hour. Cannot defend themselves-Can you arm them with anti-
missile defenses? They are trying. They have electronic Gatling guns that automatically shoot down the incoming
missiles. But no doubt those Gatling guns have a certain capacity as to number of targets they can hit at a time and range and
ammunition limitations. They also, like
any mechanical device, would malfunction at times. Generally, one would
expect that if the enemy fired enough missiles at a Gatling-gun-equipped ship, one or more would
eventually get through. How many? Let’s say the capacity of an aircraft carrier and its entourage body-guard ships to stop
simultaneous Exocet-type anti-ship missiles is X. The enemy then need only simultaneously fire X + 1 such missiles to damage or sink
the carrier. In the alternative, the enemy could fire one Exocet-type missile at a time at the carrier. Unless they are programmed
otherwise, having only one such target, all the relevant guns would fire at it, thereby exhausting the carrier task force’s anti- missile
ammunition more quickly, in which case fewer than X +1 Exocet-type missiles might be enough to put the carrier out of action. As
Japan’s top WW II Admiral Yamamoto said, There is no such thing as an unsinkable ship. The fiercest serpent may be overcome by a
swarm of ants. U.S. warships also have electronic warfare
jamming devices that screw up the guidance systems of
some types of incoming missiles. These, of course, are ineffective against nuclear-tipped missiles that
need little guidance. Furthermore, if the enemy uses 20,000-miles-per-hour nuclear missiles, there is no known anti-missile
defense. They move too fast for the electronic Gatling guns and do not need to ever get within the Gatling guns’ range to destroy
the ships. Our enemy certainly would use nukes if they had enough of them and were in an all-out war against us. Cannot hide, run,
or defend themselves In summary, Navy surface ships cannot hide from a modern enemy . They cannot
run from a modern enemy. And they cannot defend themselves against a modern enemy .
Accordingly, they are only useful for action against backward enemies like Afghanistan and Iraq or drug smugglers.

Naval power not key to heg or stability

Goure 10 [Daniel, Department of Defense Transition Team, “Can the Case be Made for Naval Power?” Lexington Institute, 2 July
2010, http://www.lexingtoninstitute.org/can-the-case-be-made-for-naval-power-?a=1&c=1171]

More broadly, it appears that the nature of the security challenges confronting the U.S. has changed dramatically over the past
There are only a few places where even large-scale conventional conflict can be
several decades.
considered possible. None of these would be primarily maritime in character although U.S. naval forces
could make a significant contribution by employing its offensive and defensive capabilities over land. For example, the
administration’s current plan is to rely on sea-based Aegis missile defenses to protect regional allies and U.S. forces until a land-
based variant of that system can be developed and deployed. The sea ways, sometimes called the global commons, are
predominantly free of dangers. The exception to this is the chronic but relatively low level of piracy in some parts of the
world. So, the classic reasons for which nations build navies , to protect its own shores and its commerce or to
place the shores and commerce of other states in jeopardy , seem relatively unimportant in today’s world.

Can’t be deployed effectively

Watts ‘12 (Robert, graduate of the Coast Guard Academy, Captain Watts has served six sea tours with the Navy and Coast
Guard, most recently commanding USCGC Steadfast (WMEC 623). A qualified Surface Warfare Officer and Cutterman, he holds
advanced degrees from the Naval War College, Old Dominion University, American Military University, and the Naval Postgraduate
School, and he is currently a doctoral candidate at the Royal Military College of Canada (War Studies). The New Normalcy-Sea Power
and Contingency Operations in the Twenty-First Century, http://www.usnwc.edu/getattachment/87e866a1-24dd-4e91-9ffa-
cb0f64f15144/The-New-Normalcy--Sea-Power-and-Contingency-Operat.aspx, 2012)

The inherent mobility of sea power means largely what it does in the traditional role—modern technology
allows global reach in three dimensions and almost instant operational coordination worldwide.
But the primary barrier to mobility in crisis-contingency operations is not technological. If
mobility is to be exercised, ships must actually sail, and it is here—in the commitment of resources to a crisis —
that things become culturally problematic. Despite the need, the answer to a crisis contingency is not always to employ sea power
immediately. This cultural hesitancy has two aspects. The first is so deeply ingrained in the American psyche that it is more a matter
of legend than of practical discussion. The United States has a long-standing tradition of rejecting the use of military forces in the
domestic context, a rejection that dates back to the Revolution. It was codified in law with the passing of the Posse Comitatus Act of
1878, which directs that military forces (specifically the U.S. Army) cannot engage in domestic law enforcement.18 The legislation is
often misinterpreted as meaning that any domestic use of military forces is illegal; that is not the case, but it is nevertheless widely
believed in both civilian and military 56 NAVAL WAR COLLEGE REVIEW circles.19 Thus before
naval forces can be
committed to a crisis, a comprehensive legal review is often demanded, something that takes
time—time that is usually not available. Another cultural barrier arises from service ethos .
Bluntly, warships are designed and train to fight. In the modern high-tech era, naval warfare is a
very specific (and expensive) proposition. It demands very sophisticated and specialized equipment .
The radar on an Aegis cruiser, for example, is exceptionally good at tracking and destroying enemy aircraft—but only that. In a crisis
contingency that marginalizes that purpose of a platform’s defining systems, the purpose of the platform itself could be called into
question. According to this logic, if a vessel is employed (albeit successfully) for a purpose for which it is not designed, the door is
opened for its increasing use for that purpose and not its proper one. In the grand scheme of things, warships used for other
purposes are not training for war; in the short term this leads to a loss of readiness for combat, while in the longer term it could
mean the elimination of platforms altogether in favor of others more suitable for noncombat missions. Although this seems to be a
largely philosophical argument, in a shrinking budget environment it is not without a certain politically
compelling logic. The effects of these factors are not insignificant. In recent crisis contingencies
(the mass migration operations of 1994 and Katrina) the arrival of naval vessels was delayed while legal and
operational impact issues were addressed, in the Katrina case so long as to become a national embarrassment.20
Bureaucratic reasons, not materiel, were the culprits, ultimately to the detriment of the
response. Hesitancy can be fatal in an operation requiring rapid response, and culture and bureaucracy can conspire to
encourage just that.
---Navy- No Challengers

No challengers – pirates are a bigger threat than peer competitors

Tillman ‘9, Barrett Historian specializing in naval and aviation topics, 2009. U.S. Naval Institute Proceedings Magazine, “Fear
and Loathing in the Post-Naval Era,” http://www.usni.org/magazines/proceedings/story.asp?STORY_ID=1896

In attempting to justify a Cold War force structure, many military pundits cling to the military stature of China as proof of a possible
large conventional-war scenario against a pseudo-peer rival. Since only China possesses anything remotely approaching the prospect
of challenging American hegemony—and only in Asian waters—Beijing ergo becomes the "threat" that justifies maintaining the Cold
War force structure. China's development of the DF-21 long-range antiship ballistic missile, presumably intended for American
carriers, has drawn much attention. Yet even granting the perfection of such a weapon, the most obvious question goes begging:
why would China use it? Why would Beijing start a war with its number-two trading partner—a
war that would ruin both economies?10 Furthermore, the U.S. Navy owns nearly as many major
combatants as Russia and China combined . In tonnage, we hold a 2.6 to 1 advantage over them. No
other coalition—actual or imagined—even comes close. But we need to ask ourselves: does that
matter? In today's world the most urgent naval threat consists not of ships , subs, or aircraft, but of
mines-and pirates.11

No competitors and naval power is inevitable

Tillman 9 [Barrett, professional writer, speaker, and award-winning historian who has written nearly 50 books and hundreds of
articles. His next book is Whirlwind: Bombing Japan 1942-1945, due from Simon & Schuster in 2010. He is also a commentator on
The History Channel and National Geographic Channel. Fear and Loathing in the Post-Naval Era, Proceedings Magazine - June 2009
Vol. 135/6/1,276 [15] 2010-06-30, http://www.usni.org/print/4264]

In the absence of power projection, navies default to lesser tasks. "Presence" is an age-old naval
mission, better known as "showing the flag." More colorfully, it was called "gunboat diplomacy,"
with the duty gunboat or (in especially touchy situations) a naval squadron appearing offshore
to quell restless natives or opponents with a show of force. The U.S. Navy's Cold War mission of
deterrence largely vanished with the collapse of the Soviet Union 20 years ago. The reason is
disturbingly simple: there is no peer opponent to be deterred . That leaves the service with a reduced menu
of options for justifying its enormous expense. Moreover, if you get Sailors and Marines aside and ask them about their military
experience, many express dissatisfaction. They allude to disaster relief as "pizza delivery"—not the reason they enlisted. What,
then, is the purpose of "this people's Navy," and how should it be employed? Moreover, how has the use of a navy been perceived
throughout history? In his 1987 treatise, The Western Way of War, Victor Davis Hanson described Greek hoplite influence on
Western military thought, emphasizing the concept of the decisive battle. The Hellenic states of the 4th century BC regarded decisive
battle as necessary, avoiding prolonged attrition and adverse agricultural-economic effects. No less was true of Western naval
thought, epitomized in the early 20th-century doctrine of a decisive battle, whether in the North Sea or mid-Pacific. The world's
leading navies accepted the idea, building ships, fleets, strategy, tactics, and doctrine around it. For Imperial Japan, it led to disaster
at Midway in June 1942. Whatever the doctrine, institutional knowledge of war at sea is a precious commodity, increasingly rare:
the junior officers who fought at Leyte Gulf retired between the 1960s and the early 1980s. The average American World War II
veteran was born in 1919, making the median age 90 at this writing while the teenaged Sailors of VJ-Day now are in their early 80s.
Therefore, personal knowledge of such events is vanishing at an accelerated rate and will be gone in a decade. While few would
claim that the specifics of the Leyte Gulf battle apply in the 21st century, Navy supporters should realize that as "the greatest"
officers and Sailors depart the scene, so does much of the population disposed to support the service politically. (Only about one-
third of Naval Institute members were alive in 1944.) The China Scenario
In attempting to justify a Cold War force
structure, many military pundits cling to the military stature of China as proof of a possible large
conventional-war scenario against a pseudo-peer rival . Since only China possesses anything
remotely approaching the prospect of challenging American hegemony—and only in Asian
waters—Beijing ergo becomes the "threat" that justifies maintaining the Cold War force
structure. China's development of the DF-21 long-range antiship ballistic missile, presumably
intended for American carriers, has drawn much attention. Yet even granting the perfection of
such a weapon, the most obvious question goes begging: why would China use it? Why would
Beijing start a war with its number-two trading partner—a war that would ruin both economies?
10 [2] Furthermore, the U.S. Navy owns nearly as many major combatants as Russia and China
combined. In tonnage, we hold a 2.6 to 1 advantage over them. No other coalition—actual or
imagined—even comes close. But we need to ask ourselves: does that matter? In today's world
the most urgent naval threat consists not of ships, subs, or aircraft, but of mines-and pirates.11
[2]
---Navy- Navies Fail

Navies are obsolete – cruise missiles and submarines will take out the fleet

Burleson ‘7 – (Mike, columnist with Sea Classics magazine. “An All-Submarine Navy” 6-19-07.
http://www.opinioneditorials.com/guestcontributors/mburleson_20070619.html)

Last week, the third in a new class of underwater battleships, the USS MICHIGAN, joined the fleet after a $1 billion face lift. The 4
converted subs of the OHIO class, former Trident missile ships, are the undersea equivalent of the reborn IOWA class from the
1980’s. Armed with over 150 Tomahawk cruise missiles, plus the ability to carry special forces and unmanned vehicles, they give the
Navy an incredible ability to strike decisively from the sea. I am of the opinion that in
full-scale shooting war at sea, the
US surface navy will be devastated in the first day., by the combination of cruise missiles and
stealthy submarines. The survivors would all be forced into port, unable to participate in the
counterattack, which would likely be initiated by our own deadly nuclear attack submarines . What
this means is, our current force of colossal and pricey warships including aircraft carriers, cruisers,
destroyers, and amphibious ships are obsolete in today’s precision, push button warfare . They are
also tremendously expensive to build and operate, with only the richest of earth’s superpowers able to afford them in ever declining
numbers. If this wasn’t reason enough for maritime nations to reevaluate their shipbuilding priorities, there are few if any
jobs the surface fleet can do which the submarine cannot . I’ll elaborate: Command of the Sea Submariners say
there are only 2 types of ships: submarines and targets. There’s valid reasons for this. Since World War 2 anti-submarine
defenses have failed to match the attack boat’s advancements in speed, stealth, and weaponry.
For instance, since 1945 the average speed of destroyers have remained at 30 knots, with only nuclear vessels able to maintain this
rate for any period. In contrast, the velocity of nuclear attack submarines, beginning with the launch of USS NAUTILUS in 1954, has
tripled and quadrupled from around 10 knots submerged to 30-40 knots. Also, an antisubmarine vessel must get within a few miles
of an enemy sub to fire its rockets or torpedoes. Its only long-range defense, the helicopter, is slow and must linger in a vulnerable
hover while its sonar buoys seek out their prey. Some Russian-built boats come equipped with anti-aircraft missiles which makes this
standard ASW tactic suicidal. In contrast, a modern submarine can launch its missiles from 75 miles away
and farther. Should it choose to close the distance, as occurred when a Chinese SONG class stalked the USS KITTY HAWK last
year, to fire its ship killing torpedoes, it can do so at speeds as fast as and sometimes surpassing surface warships. Whether attacking
with cruise missiles or wake-homing torpedoes the attack boat remains submerged; the preeminent stealth vessel. The sub has
likely held this dominate position on the high seas, since the dawn of the first nuke ships beginning in the 1950’s. The only lacking
factor has been a full-scale naval war to prove it. The single example is the sinking of the Argentine cruiser BELGRANO 25 years ago
by the British submarine HMS CONQUEROR in the Falklands Conflict. Afterward, the Argentine Navy fled to port and remained there!
Commerce Raiding/Protection: This traditional role of the submarine is one which it excelled in the last century. The difference
today is, neither
America nor Britain has the capability to mass produce the thousands of anti-
submarine escorts which just barely defeated Germany’s U-boats in 2 world wars , even if it
would matter. In the next war at sea, the submarine would bring all commerce to a halt, making
a mockery of the globalized free market system . The only counter to this menace is perhaps a combination of
aircraft and submarine escorts, with the latter acting as the destroyer, shepherding its convoy through the “shark” ridden waters.
North Korea War Answers
Frontline

No Korea war – its all posturing and international powers check escalation – history proves

Fisher ‘3-12
[Max, the Post's foreign affairs blogger. Before joining the Post, he edited international coverage for TheAtlantic.com, The
Washington Post, “ Why North Korea loves to threaten World War III (but probably won’t follow through)”

But is North Korea really an irrational nation on the brink of launching “all-out war,” a mad dog of East Asia? Is Pyongyang ready to
sacrifice it all? Probably not. The North Korean
regime, for all its cruelty, has also shown itself to be shrewd, calculating,
and single-mindedly obsessed with its own self-preservation. The regime’s past behavior suggests pretty
strongly that these threats are empty. But they still matter. For years, North Korea has threatened the
worst and, despite all of its apparent readiness, never gone through with it. So why does it keep going through these
macabre performances? We can’t read Kim Jong Eun’s mind, but the most plausible explanation has to do with internal North
Korean politics, with trying to set the tone for regional politics, and with forcing other countries (including the United States) to bear
the costs of preventing its outbursts from sparking an unwanted war. Starting World War III or a second Korean War would not serve
any of Pyongyang’s interests. Whether or not it deploys its small but legitimately scary nuclear arsenal, North Korea could indeed
cause substantial mayhem in the South, whose capital is mere miles from the border. But the North Korean military is
antiquated and inferior; it wouldn’t last long against a U.S.-led counterattack. No matter how badly such a war would go
for South Korea or the United States, it would almost certainly end with the regime’s total destruction. Still,
provocations and threats do serve Pyongyang’s interests, even if no one takes those threats very seriously. It helps to rally
North Koreans, particularly the all-important military, behind the leader who has done so much to impoverish them. It
also helps Pyongyang to control the regional politics that should otherwise be so hostile to its interests. Howard French, a former
New York Times bureau chief for Northeast Asia whom I had the pleasure of editing at The Atlantic, explained on Kim Jong Il’s death
that Kim had made up for North Korea’s weakness with canny belligerence: The shtick of apparent madness flowed from his
country’s fundamental weakness as he, like a master poker player , resolved to bluff and bluff big. Kim adopted a game of
brinkmanship with the South, threatening repeatedly to turn Seoul into a “sea of flames.” And while this may have sharply raised the
threat of war, for the North, it steadily won concessions: fuel oil deliveries, food aid, nuclear reactor construction,
hard cash-earning tourist enclaves and investment zones. At the risk of insulting Kim Jong Eun, it helps to think of North
Korea’s provocations as somewhat akin to a child throwing a temper tantrum. He might do lots of shouting, make some over-the-top
declarations (“I hate my sister,” “I’m never going back to school again”) and even throw a punch or two. Still, you give the child the
attention he craves and maybe even a toy, not because you think the threats are real or because he deserves it, but because you
want the tantrum to stop. The big problem here is not that North Korea will intentionally start World War III or a second Korean
War, because it probably won’t. So you can rest easy about that. The big problem is that North Korea’s threats and provocations,
however empty, significantly raise the risk of an unwanted war. The United States, South Korea and yes Pyongyang’s all-
important ally, China, all have much more to lose in a regional war than does North Korea. It falls to those countries, then, to
keep the Korean peninsula from spiraling out of control. Even if they don’t ultimately offer Pyongyang
concessions to calm it down, as they have in the past, they’ve still got an interest in preventing future outbursts. Like parents
straining to manage a child’s tantrum, it’s a power dynamic that oddly favors the weak and misbehaving.

No korean war—it’s all rhetoric

Kang ‘10 (David Kang, professor of international relations and business and director of the Korean Studies Institute at the
University of Southern California. His latest book is East Asia Before the West: Five Centuries of Trade and Tribute (Columbia
University Press, 2010). 12/31/2010]
However, despite dueling artillery barrages and the sinking of a warship, pledges of “enormous
retaliation,” in-your-face joint military exercises and urgent calls for talks, the risk of all-out war
on the Korean peninsula is less than it has been at anytime in the past four decades . North Korea
didn’t blink, because it had no intention of actually starting a major war. Rather than signifying a
new round of escalating tension between North and South Korea, the events of the past year
point to something else—a new cold war between the two sides. In fact, one of my pet peeves is the
analogies we use to describe the situation between South and North Korea . We often call the situation a “powder
keg” or a “tinderbox,” implying a very unstable situation in which one small spark could lead to a
huge explosion. But the evidence actually leads to the opposite conclusion: we have gone sixty
years without a major war, despite numerous “sparks” such as the skirmishing and shows of
force that occurred over the past month. If one believes the situation is a tinderbox, the only explanation for six
decades without a major war is that we have been extraordinarily lucky. I prefer the opposite explanation: deterrence is
quite stable because both sides know the costs of a major war, and both sides—rhetoric and
muscle-flexing aside—keep smaller incidents in their proper perspective.

Major powers will deescalate—proves no conflict and no draw-in

Zhijiang ’12 (Kim Jong-un’s regime: facing up to domestic challenges, China and the US January 26th, 2012 Author: Wei Zhijiang,
Sun Yat-sen University Wei Zhijiang is Professor and Director of the Institute of South Korea Studies at the School of Asia-Pacific
Studies, Sun Yat-sen University.

With regard to the role of outside powers, China and US share common strategic interests in avoiding chaos
and maintaining peace and stability on the Korean Peninsula. After the death of Kim Jong-il, ROK-US
summit telephone talks declared that the US has no intention to interfere in the succession process.
This indicates that the US will not put pressure on North Korea to promote its collapse and hopes
to avoid conflict on the peninsula and to achieve peace and stability. The US Assistant Secretary
of State for East Asian and Pacific Affairs, Kurt Campbell, visited China recently in order to further exchange
views with China concerning the situation in the DPRK and to coordinate policies toward North Korea.
China’s strategy has been to maintain peace and stability on the peninsula, and to build a
harmonious and stable strategic environment in Northeast Asia conducive to national development. Kim’s
death has not changed the basic strategy of China toward the Korean Peninsula. The main basis of
China’s Korean Peninsula policy is to comprehensively strengthen and support Kim Jong-un’s new North Korean regime. The main
purpose of the US’ ‘return to Asia’ strategy is to strengthen its strategic influence in the Asia Pacific region, including the Korean
Peninsula. It also includes preventing military provocation or possible war in the East Asia region through the strengthening of US-
ROK, US-Japan and US-Australia military alliances, both bilaterally and multilaterally. Therefore, China
and the US have
common strategic interests on the Korean Peninsula issue. They do not want chaos in the North
Korean situation, the collapse of the regime, or a large-scale military conflict between the North and
South. In resolving the North Korean nuclear crisis, the missile crisis and other issues, there is a
wide range of cooperative space that China and the US can utilise. The two parties should strengthen their strategic
coordination and communication with the DPRK in order to cope with any future crises and deal with the current challenges
concerning the Korean Peninsula, and act to safeguard the peace and stability of the Korean Peninsula.
Zero risk of Korean conflict

Rowland ’10 (Ashley Rowland, Stars and Stripes, “Despite threats, war not likely in Korea, experts say,”
http://www.stripes.com/news/despite-threats-war-not-likely-in-korea-experts-say-1.127344?localLinksEnabled=false, December 3,
2010)

Despite increasingly belligerent threats to respond swiftly and strongly to military attacks, analysts say there is
one thing both North Korea and South Korea want to avoid: an escalation into war. The latest
promise to retaliate with violence came Friday, when South Korea’s defense minister-to-be said during a confirmation hearing that
he supports airstrikes against North Korea in the case of future provocations from the communist country. “In case the enemy
attacks our territory and people again, we will thoroughly retaliate to ensure that the enemy cannot provoke again,” Kim Kwan-jin
said, according to The Associated Press. The hearing was a formality because South Korea’s National Assembly does not have the
power to reject South Korean president Lee Myung-bak’s appointment. Kim’s comments came 10 days after North Korea
bombarded South Korea’s Yeonpyeong island near the maritime border, killing two marines and two civilians — the first North
Korean attack against civilians since the Korean War. South Korea responded by firing 80 rounds, less than half of the 170 fired by
North Korea. It was the second deadly provocation from the North this year. In March, a North Korean torpedo sank the South
Korean warship Cheonan, killing 46 sailors, although North Korea has denied involvement in the incident. The South launched a
series of military exercises, some with U.S. participation, intended to show its military strength following the attack. John Delury, a
professor at Yonsei University in Seoul, said South Korea is using “textbook posturing” to deter another attack by emphasizing that it
is tough and firm. But it’s
hard to predict how the South would respond to another attack. The
country usually errs on the side of restraint, he said. “I think they’re trying to send a very clear signal to North
Korea: Don’t push us again,” Delury said. “For all of the criticism of the initial South Korean response that it was too weak, in the end
I think people
don’t want another hot conflict. I think the strategy is to rattle the sabers a bit to
prevent another incident.” Meanwhile, Yonhap News reported Friday that North Korea recently added multiple-launch
rockets that are capable of hitting Seoul, located about 31 miles from the border. The report was based on comments from an
unnamed South Korean military source who said the North now has 5,200 multiple-launch rockets. A spokesman for South Korea’s
Joint Chiefs of Staff would not comment on the accuracy of the report because of the sensitivity of the information. Experts
say
it is a question of when — not if — North Korea will launch another attack. But those experts
doubt the situation will escalate into full-scale war . “I think that it’s certainly possible, but I think that what
North Korea wants, as well as South Korea, is to contain this,” said Bruce Bechtol, author of “Defiant Failed State: The North Korean
Threat to International Security” and an associate professor of political science at Angelo State University in Texas. He said North
Korea typically launches small, surprise attacks that can be contained — not ones that are
likely to escalate. Delury said both Koreas want to avoid war, and North Korea’s leaders have a
particular interest in avoiding conflict — they know the first people to be hit in a full-scale
fight would be the elites.
---Korea- No War

1NC Fisher and Kang—no risk the North and South fight—It’s all a big bluff – The North’s
military is inferior and obsessed with self preservation - flashpoints mean little in overall
strategic calculations—moving toward nonescalatory regional cold war—past sixty years
disprove any “powder keg” analysis—both sides avoid direct military confrontation at all costs

North Korea is just hype

Sang-Hun 3/13 (Choe Sang-Hun, New York Times, “S. Korea decries Pyongyang’s threats, says truce remains”,
http://www.bostonglobe.com/news/world/2013/03/12/south-korea-says-north-korea-cannot-unilaterally-nullify-korean-war-cease-
fire/UMkNihrBcyXaUUWsGInc5N/story.html, March 13, 2013)

SEOUL — SouthKorea said Tuesday that North Korea cannot unilaterally nullify the 1953 Korean
War cease-fire, calling the North’s war threats a psychological ploy to strengthen Kim Jong
Un’s leadership at home and force Washington and South Korea to make concessions to the
isolated country. Following through on a standing threat that it revived last week amid tensions over joint US-South Korean
military drills coupled with UN sanctions, North Korea declared that the armistice nullified as of Monday and that the guns of the
1950-53 Korean War, silenced for 60 years under an uneasy truce, could roar again. North Korea also severed military and Red Cross
hot lines and issued a torrent of threats, including a possible ‘‘preemptive nuclear strike’’ against the United States and South Korea.
In North Korea, the authorities were kicking up a war fever — a tool of populace control they had previously used at times of
international tension — by having people evacuate into tunnels with emergency provisions and putting up military camouflages on
buses and trucks, the South Korean Defense Ministry said Tuesday. Kim, the North Korean leader, visited front-line artillery units
twice in the last week, warning that ‘‘war can break out right now’’ and calling for merciless strikes at the South Korean marine
bases he watched across the border through binoculars, according to North Korean media. Last Friday, Hyon Yong-chol, the chief of
the General Staff of the North Korean People’s Army, visited the truce village of Panmunjom on the border, raising fears among
some South Koreans that the North might repeat what it had done there in 1996 and send hundreds of armed troops to punctuate
its claim that the armistice was no longer valid. Stressing their alliance against North Korean threats, the office of the South Korean
president, Park Geun-hye, said Tuesday that she planned to meet President Obama in Washington in early May. The announcement
came a day after the allies began their annual Key Resolve joint military exercises, which are in addition to their two-month-long Foal
Eagle drill that began on March 1. ‘‘A unilateral nullification or termination of the armistice is not allowed under its related articles
and principles of international laws,’’ said Cho Tae-young, spokesman of the South Korean Foreign Ministry. ‘‘We demand that North
Korea withdraw comments that threaten the stability and peace of the Korean Peninsula and the region.’’ Martin Nesirky, a
spokesman for the United Nations, which sent allied troops to fight for South Korea during the war, also told reporters that North
Korea could not dissolve the armistice unilaterally. KimMin-seok, spokesman of the South Korean Defense
Ministry, said Tuesday that there was no sign of imminent nuclear or missile tests by the
North or hostilities along the border. He said that the ‘‘rhetorical threats’’ flooding the North’s
state-run media were aimed at putting ‘‘psychological pressure’’ on the South. Jay Carney, a White
House spokesman, said the United States was ‘‘certainly concerned by North Korea’s bellicose rhetoric. And the threats that
they have been making follow a pattern designed to raise tension and intimidate others.’’

Rhetoric isn’t reality—past flare ups prove neither side is looking to fight

Stares ’10 Paul, CFR Center for Preventive Action Director and Conflict Prevention Senior Fellow, 8/12, “Handling Tensions on the
Korean Peninsula," http://www.cfr.org/publication/22788/handling_tensions_on_the_korean_peninsula.html, access 12/7/2010
Other than firing some coastal artillery
and detaining a South Korean fishing boat that recently strayed into
North Korea waters, Pyongyang has responded primarily with belligerent rhetoric and apocalyptic
warnings. The recent ROK-U.S. naval exercises, for example, elicited threats of a "retaliatory
sacred war." But by historical standards, such bombast is unexceptional. The recent North Korean
provocations also pale in comparison to earlier attacks and skirmishes, most notably during the late
1960s when, among other things, the Blue House--South Korea's presidential residence--was attacked, or in the 1980s when the
South Korean cabinet was bombed during a visit to Burma. These
far-worse periods of inter-Korean tensions
never ignited another war, and the incentives to prevent this from happening are even
greater today. South Korea fears losing its hard-won prosperity, while a much weaker North
knows that it would never survive another major conflict.

North concerned primarily with survival—they’re rational—no risk of conflict

Pollom ’10 (Drew, The Gonzaga Bulletin, “War in Korea unlikely,” 2010, http://www.gonzagabulletin.com/war-in-korea-unlikely-
1.1815018,

Suddenly we are transported back to the 1950s. The recent aggressive activity by North Korea, the last real remnant of the Cold
War, has once again put the U.S. and Asia on the brink of war. While the
North has always taken grandiose
actions in an attempt to gain attention, the last six months have truly been deadly. First, there was the sinking of
the South Korean warship by a North Korean torpedo. Now there is the shelling on Yeonpyeong by North Korea resulting in the
death of two civilians. Seoul demands vengeance. The U.S. conducts war games. Americans prepare themselves for war in
Korea. Before you make that tin foil hat to protect you from the nuclear radiation, you have to
first see if war with North Korea will actually happen . Beyond the black and white situation of responding to
North Korean aggression is a complex world of Asian politics, particularly involving the U.S. and China. At the end of the day,
while I still believe we
are not going to war with North Korea, we are getting close. North Korea has
always been an enigma. Described as a Stalinist regime, given its cult-like reverence for its leaders, this isolated country
is known for having a huge bark. Since the 1990s, North Korea has leveraged its nuclear weapons against the world in exchange
for an erratic list of demands. People often view the North as dangerous and unstable, both of which are very true if you look at
its behavior at face value. However, beneath
everything North Korea has ever done, there has been
one all-consuming goal: survival. The regime will do and say anything that will ensure its
continued hold on power. North Korea would pretty much lose that hold if it went to war.
Despite its numbers in the military, it cannot keep up with the well-fed and technologically superior South. Even its nuclear
arsenal can be neutralized by the tactical strikes of the U.S. Air Force. Even
in its grandest delusion, North Korea
knows that a war with the U.S. will end in disaster for it. Based on that, I believe that the
latest aggression is another act of desperation to be important on the global stage. At the
same time, the U.S. isn't exactly rushing to go to war. The war games conducted last week are an important
act to show the world that the U.S. and South Korea won't be bullied by the North. The reality is any war effort will
most likely be hamstrung from the start. There is, of course, the obvious fact that most of the troops are still
dedicated in Iraq and Afghanistan. With the current economic recession and large budget deficits, we may not want to plug
another huge chunk of money in another invasion. Instead, we are more likely to seek a diplomatic solution to the situation
before it gets out of hand. In the end, whatever we end up doing, South Korea will most likely follow.

No war- self interest and decades of peace prove


McClatchy 10 [MCCLATCHY WASHINGTON BUREAU | BY WARREN P. STROBEL AND JONATHAN S. LANDAY, “Experts say war
unlikely North, South Korea” May 25]

Although the isolated, communist North's behavior is notoriously unpredictable and sometimes seems
irrational, all-out war between it and the democratic, capitalist South still seems unlikely, analysts
said, given the stakes. Nevertheless, tensions on the Korean peninsula, where some 28,500 U.S. troops provide a
tripwire for U.S. military intervention if the North attacks, are likely to rise in coming days. North Korea would likely
lose any conflict with the South, but not before inflicting massive damage on South Korea's capital, Seoul, a 30-minute
drive south of the demilitarized zone that has divided the two Koreas since 1953. U.S. intelligence officials estimate that some
11,000 North Korean artillery pieces are in sheltered positions within range of Seoul and probably could destroy much of the city
before they could be knocked out. "The tensions certainly have increased," but there is no sign that
North Korea is mobilizing its 1.2 million-strong military, said a U.S. defense official, who requested
anonymity in order to speak more freely. "They have masses (of troops) down on the DMZ (demilitarized
zone), but they do a normal shift or rotation," he said. South Korean officials said they were bracing for fresh
provocations from the North, especially at sea. On Monday, South Korean President Lee Myung-bak blamed North Korea for the
March 26 sinking of the corvette Cheonan, which killed 46 of its crew, and said he was curbing trade with North Korea and
banning its ships from transiting South Korean waters. "That could get sort of ugly if (North Korean vessels) don't stop, and
chances are they won't," said Art Brown, formerly the top U.S. intelligence analyst for East Asia. " It's
unlikely they will
do nothing. I tend to think they're not going to try Korean War, version two ." Still, Brown and other
former top U.S. officials said that serious clashes between the Koreas during the past 57 years
haven't led to warfare -- and sometimes have provided opportunities for rapprochement.

Past 50 years disproves escalation

White 10 – Master’s in journalism from Columbia and IR degree from the London School of Economics, editor for Business Insider
and formerly wrote for MSNBC (3/26, Gregory, Business Insider, “The Long, Long History Of False Starts Of War Between South And
North Korea”, http://www.businessinsider.com/were-calling-it-this-is-not-the-start-the-restart-of-the-korean-war-2010-3)

History suggests that this sinking of a South Korean naval vessel off the coast of the country will not
be the restart of the Korean conflict. Since the end of open conflict between North and South Korea, the North has
consistently acted in an aggressive manner towards its neighbor. During the 1960s, North Korea
conducted military operations into the south , culminating in 1968 when 600 of these raids were reported. In
the 1970s, North Korea tried to assassinate key members of the South Korean government, in
an attempt to push the crisis forward. In 1999, two North Korean naval ships were blown up killing 30. In
2002, a sea battle killed and unspecified amount of North Koreans and 5 South Koreans. In November 2009, two military
vessels exchanged fire (via HuffPo). In January 2010, North Korea launched 30 shells into the country's no sail zone.
This time won't be different. Little will happen.

Tension is subsiding and talks solve

Barnett 9 (Thomas, Professor – US Naval War College, “Threat of Great Power War Recedes”, Korea Times, 3-23,
http://www.koreatimes.co.kr/www/news/opinon/2009/03/137_41779.html)

Here's where it gets tricky for President Obama: the three conflict scenarios that currently justify our military's
big-war focus are China-Taiwan; North Korea, and Iran. All three scenarios will effectively disappear over the next half-
decade. With the Kuomintang's return to power in Taipei last year, tranquility broke out between island and mainland,
triggering a concerted effort at brokering a peace treaty that matches Taiwan's already profound economic integration with
China. If only Nixon could go to China, then only Chiang Kai-shek's party could do the same for Taiwan. Setting aside political
integration, Taipei's leaders follow Hong Kong's example: separate systems integrating with one another in an expanding
economic commonwealth. We're witnessing the first steps toward an Asian economic union with
China as its natural anchor. No, it will not be a linear journey, as the current economic crisis demonstrates, but where
else can small Asian states turn? As for Kim Jong-il's North Korea, that fake state won't long survive the
Dear Leader's death, made all the more imminent by a recent stroke that Pyongyang strenuously denied.
Whatever the timetable, the key point here is that none of the concerned great powers expects North
Korea's collapse will trigger war among them. Their long-standing multilateral talks have
demystified that dire scenario.
---Korea- No Escalation

1NC Zhijiang—no draw-in and no conflict—the US and China are committed to stabilizing the
peninsula—this deters both sides from getting in a shooting war because neither want to
upset their main ally—also means no risk of draw-in should a war occur—both superpowers
have existential reasons to deescalate conflict

U.S. won’t intervene

Cohen 9 (Michael, “The Future of the US Military”, DemocracyArsenal, 3-23,


http://www.democracyarsenal.org/2009/03/the-future-of-the-us-military.html)

Gentile identifies three possible future conflicts that might entail fighting a conventional war, "A clash between Iranian forces
and an American combat brigade in Iraq could erupt in a minute. A North Korean march on Seoul will not be a
fight for hearts and minds. Nor will ground fighting on Taiwan in the event of a Chinese assault." But realistically, each of
these conflicts, and in particul ar the latter two will likely not involve a major conventional engagement. If
North Korea marches over the 38th Parallel or China invades Taiwan (and really why would either country do
something foolish) the Army is unlikely to play a leading role; instead it will be the Air Force and Navy
respectively. Does anyone believe that the US should be fi ghting another land war in Asia? And what about Iran? Well one
would hope that after 6 years and more than 4,000 dead in Iraq we would have learned our lesson in the
Mid dle East as well.

Diplomatic policy ensures de-escalation

Korea Herald 9 (“N.K. Nukes for Survival, Not War”, 2-14, Lexis)
North Korea is unlikely to use its nuclear weapons unless it feels the regime's survival is at risk, the chief U.S.
intelligence official said Thursday. The remark by Dennis Blair, director of national intelligence, signaled a softened U.S. threat
perception on North Korea's nuclear ambition. "Pyongyang probably views its
nuclear weapons as being more
for deterrence, international prestige, and coercive diplomacy than for war-fighting and would
consider using nuclear weapons only under certain narrow circumstances ," he said in a report presented
at a Senate Select Committee on Intelligence hearing. "We also assess Pyongyang probably would not attempt to use nuclear
weapons against U.S. forces or territory unless it perceived the regime to be on the verge of military defeat and risked an
irretrievable loss of control," Blair said in the report, titled "Annual Threat Assessment of the Intelligence Community." The report
said that Pyongyang "is less likely to risk selling nuclear weapons or weapons-quantities of fissile material than nuclear technology or
less sensitive equipment to other countries or non-state actors," to avoid "a regime-ending military confrontation with the United
States." Experts
and analysts here said the new Barack Obama administration may craft its North
Korea policy based on the appeased perception , which could contradict that of the conservative
South Korean government.
---Korea- AT: Korea Missiles / Strike the U.S.
No Korea missile impact

– tech hurdles

AP 12
[North Korea missiles could threaten U.S., but not yet]

WASHINGTON--North Korea's successful long-range rocket shot raised a nightmarish specter for Americans: that one day its
longtime adversary could fire a nuclear-tipped missile into the U.S. mainland. But North
Korea still has far to go to
make that threat a reality. Experts say the North still faces major technical hurdles. Dec. 12 launch showed it
can fire a rocket into space, but it has not worked out how to make a projectile return to Earth and hit a target. Nor
has it mastered how to put a nuclear warhead on a missile. Even if it solves those problems, a giant
rocket would be vulnerable to a pre-emptive attack. That's little consolation for U.S. officials who say North
Korea is progressing technologically, and unless that progress is checked, it's only a matter of time before the North has the ability to
threaten America and elsewhere with a long-range ballistic missile. There's still time to negotiate, and
Washington will be pressing China, the North's main ally, to get Pyongyang to play ball. But negotiating North Korea's
disarmament in return for much-needed aid will prove tough if the North's totalitarian leadership--ensconced since the Korean War
six decades ago--views its weapons of mass destruction as key to its survival. Six-nation talks on the North's nuclear program, also
including China, the U.S., Russia, South Korea and Japan, have been stalled since 2009, and the Obama administration's effort last
year to kick start the process by offering food aid in return for nuclear concessions collapsed when Pyongyang attempted a satellite
launch in April. That launch attempt failed, like the three other launches of three-stage rockets that had proceeded it since 1998,
leading to skepticism that the impoverished country, which subject to tough international sanctions, had the wherewithal to
succeed. But Dec. 12 launch managed to send a small communications satellite into orbit, prompting celebrations in Pyongyang. In
2010, U.S. Secretary of Defense Robert Gates warned that within five years the North could develop an intercontinental ballistic
missile capable of reaching the United States, and Dec. 12 launch suggests the North is on track for that, said former U.S. defense
official, James Schoff, now an expert on East Asia at the Carnegie Endowment for International Peace. But he and other experts say
the North must still surmount tough technical barriers. “It's worth keeping in mind that even though this launch worked, North
Korea has no confidence in the reliability of the rocket, which undermines its utility for military purposes,'' said David Wright at the
Washington-based Union of Concerned Scientists. Wright says the Unha-3 has a potential range of 8,000 to 10,000 kilometers (4,970
to 6,210 miles), which could put Hawaii and the northwest coast of the mainland United States within range. But the satellite it
mounted on the rocket weighs only 50 kilograms (110 pounds)--about a tenth the weight of a nuclear
warhead. The North would also have to develop a heat shield for a missile so it could withstand the
extreme heat and pressure of a descent back through the Earth's atmosphere. It would also need a guidance system to enable it to
hit a target. “Those are pretty serious tasks,'' Wright said. Another key technical challenge would be to miniaturize a nuclear
warhead to fit on a missile. The North Korea conducted nuclear tests in 2006 and 2009, and probably has enough
plutonium for a half-dozen or more bombs, but would will have to do more testing to advance its warhead designs. Even if it
achieves all that, experts say a liquid-fueled rocket like the 32-meter (105-foot) -tall Unha-3 on which a ballistic missile
would be vulnerable to attack in a pre-emptive
could be based takes days to assemble and hours to fuel. That
airstrike, compared to more mobile solid-fueled missiles developed by the U.S. and Soviet Union which are more easily concealed
and ready to launch within minutes.

New US BMD checks

McGreal 3/15 (Chris McGreal, “US to reinforce missile defences to counter North Korea 'provocations'”,
http://m.guardiannews.com/world/2013/mar/15/us-strengthen-missile-defence-north-korea, March 15, 2013)

The US is to strengthen its missile defence shield on the Pacific coast in response to North
Korea's strides in developing nuclear weapons and threats of war, defence secretary Chuck Hagel announced on Friday. Hagel
said the US will deploy 14 additional missile interceptors in Alaska to "make clear to the world
that the United States stands firm against aggression". The additional interceptors will bring to 44 the total
deployed in Alaska and California, although the numbers will not increase for four years and the defensive missiles have proved
unreliable in tests. The
White House has derided a surge in Pyongyang's threats, including a
declaration that the 1953 armistice agreement with South Korea is now void, as "bellicose
rhetoric". US officials believe the threats are an attempt to save face following the latest
round of United Nations sanctions over North Korea's underground nuclear test last month. But
officials have also said that the North Korean regime is highly unpredictable. Barack Obama told ABC News this week that he doubts
Pyongyang can make good on its threats, but the US needs to be prepared. "They probably can't, but we don't like margin of error,"
he said. Hagel noted that a year ago North Korea put on display what appeared to be an intercontinental ballistic missile capable of
being moved by road, and used another missile to put a satellite into orbit, "thus demonstrating progress in its development of long
range missile technology". "The United States has missile defence systems in place to protect us from
limited ICBM attacks. But North Korea in particular has recently made advances in its capabilities and is engaged in a series
of irresponsible and reckless provocations. Specifically, North Korea announced last month that it conducted its third nuclear test,"
he said. Hagel said the US
will deploy the additional missiles "in order to bolster our protection of
the homeland and stay ahead of this threat" as well as installing a second radar in Japan to
give improved early warning and tracking of any missile launched from North Korea. Hagel said it
is not clear when North Korea might be able to develop an effective intercontinental nuclear weapon but the new missiles will not
be deployed until 2017. "One of the reasons we're doing what we're doing based on the intelligence we have is to assure that
whatever their timelines are we're not reacting to those timelines, that we're ahead of any timelines of any potential threat," he
said. "We feel that to have the 30 [interceptors] in place now and the additional 14 by the end of 2017 gives our country the security
it needs and the people need to be reassured that security is there." Asked about the "very poor test performance" of the
interceptor the US plans to deploy in Alaska and whether he has confidence it would be certain to shoot down a North Korean
missile, Hagel acknowledged "there was an issue regarding our gyro guidance system". He said that the deployment would not go
ahead until "we are sure we have the complete confidence that we need". But he insisted that he had confidence in the weapon.
"But the American people should be assured that our interceptors are effective," he said. Hagel
said the Pentagon also plans to shift resources in order to speed up deployment of missiles in Europe to counter the threat from
Iran. The target date for putting the defences in place had been 2022 because of spending cuts by Congress but that is being pulled
forward in response to what Hagel called a maturing threat from Tehran.

No Korea missile threat

– regime propaganda

Ramstad 12
(Evan, WSJ Asia, citing a technical report published by Rand corporation, December 13, 2012, “After First Glance, North Korea’s
Missiles Not As Fearsome”)

North Korea’s rocket shot on Wednesday may have been the greatest success it has yet had in a decades-long effort to develop the
technology for long range missiles. But there’s also
less to it than meets the eye. A technical report
published by the Rand Corporation in September concludes that the threat posed by North Korea’s missile program is
not as significant as widely portrayed. It says the main driver the North Korea regime is political: “to create
the impression of a serious missile threat and thereby gain strategic leverage, fortify the North
Korean regime’s domestic power, and deter other countries, particularly the Republic of Korea and the
United States, from military action.” The author of the study, Markus Schiller, a missile expert at Schmucker Technologie in
Germany, said in an e-mail interview that Wednesday’s launch doesn’t change his conclusions about the North’s missile capabilities
or intentions. “There is no need to reconsider any conclusions or recommendations of the report,” he said. He noted this is the
country’s only success of long-range technology from five launches over nearly 15 years and was a “rocket that uses old Russian
engines.” “I would say that a North Korean cargo ship, or an airliner from Koryo Air, is more dangerous,” he added,
referring to the North’s state-run airline. “If they wanted an ICBM [intercontinental ballistic missile], they have to develop a
new rocket, using different technology. This would take a very long time, require a lot of work,
and cost a lot of money.” He said the rocket that North Korea launched on Wednesday is not powerful enough to carry a
nuclear warhead very far. Its satellite payload may have only weighed 100 kilograms (220 pounds), while a warhead would be five to
seven times heavier. “The whole rocket’s performance is too low to be used as an ICBM, even in three stage
configuration,” he said. Using the Soviet-era technology that was the basis for the rocket fired on Wednesday, Mr. Schiller estimates
North Korea would need to build a rocket that’s two to three times larger to hit the U.S. The alternative,
he said, is to use different technology and more efficient propellants. “But this is a completely new rocket,” he said. He added that
North Korea also needs to demonstrate that it can design complex warheads that can withstand the heat
and acceleration that an ICBM goes through when it re-enters the atmosphere. In his paper for Rand, Mr. Schiller
recommends that policymakers in other countries not get overly concerned about the North’s missile tests. “Don’t elevate the North
Korean threat,” he wrote. “This is exactly what the North Korean regime wants.” That has sparked some criticism from with other
analysts and North Korea watchers. But even the most hawkish analysts say that North Korea is far from posing a real danger with its
long-range missiles. Bruce Bechtol, a former U.S. military intelligence analyst who now teaches at Angelo State University in Texas,
said that North Korea has several hurdles to overcome, including miniaturizing a warhead and deploying its long-range missiles in a
way that can’t be detected. “It
takes several days for them to set it up,” he said. “If we see them setting it up, we
could take it out.” And he noted that, if North Korea wants want to sell the technology to countries like Iran,
they’ll face difficulties shipping parts by sea and air. “They still have several issues,” Mr. Bechtol said.
---Korea- AT: Transition War
No internal power struggles

Zhijiang ’12 (Kim Jong-un’s regime: facing up to domestic challenges, China and the US January 26th, 2012 Author: Wei Zhijiang,
Sun Yat-sen University Wei Zhijiang is Professor and Director of the Institute of South Korea Studies at the School of Asia-Pacific
Studies, Sun Yat-sen University.

After the death of Kim Jong-il in December, Kim Jong-un has officially become the supreme leader of North
Korea and the supreme commander of the Korean People’s Army. This is in addition to his position as the Vice Chairman
of the Central Military Commission of the Korean Workers’ Party, which was announced in September 2010. These
appointments demonstrate that the succession system , with Kim Jong-un as the core person,
has been stabilised. Currently, there are three major powers that are adjuvant to the Kim Jong-un regime in North Korea.
First is the Kim family system, which is now headed by Kim Jong-un’s uncle, Jang Sung-taek, and aunt, Kim Kyong-hui. Jang serves as
the Vice Chairman of the National Defence Commission, and as a Chief Minister of the Party Central Committee. Together they
control the major power of the Korean Workers’ Party, the government and the military — and have established a broad political
power base within the Korean government and military systems. Second are the old-guard politicians. They could be
represented by Kim Yong-nam, the chairman of the Supreme People’s Assembly Committee. They have been trusted by the
three generations of leaders (Kim Il-sung, Kim Jong-il and Kim Jong-un), and they also have high national prestige. They
will continue to support the Kim Jong-un regime . Third are the military forces, which could be represented
by the chief of staff, Ri Yong-ho, and the Minister of the People’s Armed Forces, Kim Yong-chun. They have all shown loyalty
to the Kim Jong-un regime. After the death of Kim Jong-il, Kim Jong-un publicly issued his ‘first orders’ as supreme
commander, proving his actual control over the army. The support of the above three powers will help
guarantee the smooth transition of power to the Kim Jong-un regime.
North Korea Regime Collapse Answers
Frontline

North Korea stable

Straits Times 9 (“No Sudden Collapse in N.Korea”, 1-15, http://www.straitstimes.com/Breaking


%2BNews/Asia/Story/STIStory_326500.html)

North Korea is unlikely to experience a sudden collapse despite growing unrest over chronic
food shortages, according to a South Korean think-tank. The state-run Korea Institute for National Unification said in a
report seen on Thursday that leader Kim Jong-Il, 66, is fully in command. South Korea and US officials have said
he suffered a stroke last August but recovered well. Mr Kim's health is closely watched since he has yet to nominate publicly his
'Chances are slim for North Korea to suddenly
successor to rule the nuclear-armed but impoverished state.
collapse, as Chairman Kim Jong-Il seems to be in command with state affairs under his control,' the institute said.
'Protracted economic woes have weakened social discipline and stirred discontent among North Korea's
citizens, but the predominant view is that it is an overreaction to read these as signs pointing
to North Korea's collapse.' The communist North suffered a full-scale famine in the 1990s which
killed hundreds of thousands. It has since relied on foreign aid to help feed its 23 million people.

North Korea resilient – no risk of collapse

Byman and Lind 8 (Daniel, Director of the Security Studies Program – Georgetown University and Jennifer, Professor of
Government – Dartmouth College, “Understanding Regime Stability in North Korea”,
http://www.allacademic.com/meta/p_mla_apa_research_citation/2/7/9/0/6/p279064_index.html)

The Kim regime has wielded the authoritarian toolbox to protect itself both from popular revolt
and from internal coups. Its social policies, use of ideas and ideology, and use of force prevent
the onset of revolution. Through numerous other tools (elite co-optation, manipulation of
foreign governments for financial aid, and the “coup-proofing” of domestic institutions ) the
regime protects itself from coups d’etat. This framework not only helps explain the past
resilience of the regime, it predicts that its position will remain secure for decades to come.
Totalitarian North Korea will not fall to popular revolt, and —because it will continue to receive the
international aid needed to co-opt elites—coups are also unlikely to occur or succeed. The regime is several
stages (and potentially decades) away from collapse.

Korean collapse is slow – no impact

Scobell 8 (Andrew, Professor of International Affairs – Texas A&M University and Director of the China Program – GHW Bush
School of Government and Public Service, “Projecting Pyongyang: The Future of North Korea’s Kim Jong Il Regime”, March,
http://www.strategicstudiesinstitute.army.mil/pdffiles/pub844.pdf)

This monograph considers the future trajectory of the Pyongyang regime and explores a range of future scenarios. It does not
consider the future of North Korea as a geographic or territorial entity. Some analysts and observers discuss the future without
clarifying whether they are discussing the country of North Korea or the Pyongyang regime. In this monograph, the focus is on
the fate of the regime dominated by the Kim Dynasty, initially ruled by Kim Il Sung and then led by his son, Kim Jong Il, following
the former’s death in 1994. A fundamental assumption is that the regime will collapse. Thus, the key question is not whether
the regime will collapse, but when and how it will collapse. The logic behind this assumption is based on this author’s
assessment that the Kim regime is a totalitarian one, and that such a regime has a limited life span. However, this
collapse
may be a long and drawn out process that could very well play out over a period of years or
even over the course of a decade or more. The purpose of this monograph is to set out an array of scenarios to
assist planners and decisionmakers in thinking about and preparing for possible future contingencies concerning North Korea.
This monograph does not dwell on war or conflict scenarios involving North Korea because military planners have already
focused considerable effort and attention on these. It is entirely possible that the fate of the country as a political, territorial,
and juridical entity is intimately bound up with the fate of the regime, but one should not assume this to be so. In other words,
the collapse of the Kim regime may not lead to the collapse of North Korea as a state.
Moreover, one should not assume that even if the regime collapse is followed by state collapse that these events would
inexorably lead to Korean unification.

North Korea won’t lash-out

Scalapino 98 (Robert, Robson Research Professor of Government, The US and the Two Koreas, p. 36)

Evidence suggests, however, that while quantitatively strong, the DPRK military is very uneven in various qualitative respects.
Energy shortages, for example, have greatly curtailed training time for pilots. The use of soldiers for a wide variety of civilian
tasks from construction to agricultural pursuits raises questions about training. Moreover, despite the high expenditures,
budgetary restraints prevent the purchase of modern equipment from abroad. Thus, obsolescence is an increasingly
troublesome problem. Most important, however, few leaders commit suicide on behalf of themselves and their country. The
DPRK leaders know that while they could exact heavy damage on the South by a sudden strike, they would
subsequently be pulverized by the combined ROK-U.S. air, sea, and ground forces. Today, the U.S. commitment is
firm-and fully credible in the North as in the South. So it must remain. The North may well initiate recurrent
military provocations and incidents, but the likelihood of an all out war seems very remote under
current conditions. Survival, not extinction, is the primary goal of the DPRK elite.
---Noko Regime Collapse- Chol Indicts
Chol’s job is to lie about the North Korean threat – he’s the spin-doctor of the regime

Straits Times 2 (11-18, Lexis)

DO YOU hear the knives sharpening? As the scraping gets louder while outraged parties dig in on North Korea
after its admissions to an uranium programme and kidnappings of Japanese citizens, it is clear that even Supreme
Leader Kim Jong Il can do with some serious public relations help. Enter spin doctor Kim
Myong Chol, 58. The interview stretches for more than 1 1/2 hours, but the man shows no signs of fatigue. In fact, he gets
more and more into the swing of it as he sells Pyongyang's policies. 'Kim Jong Il will never kowtow to foreign pressure,' he
asserts. 'The North Korean population expects Kim Jong Il as a supreme national leader to do what he can to safeguard national
dignity and sovereignty from American encroachment.' Given the fervency with which he stumps for the regime, it is surprising
to learn that he is not a government official - he receives no salary for his ceaseless beating of North Korea's drums. Dr Kim is a
born-and-bred Japanese and lives in Tokyo with his wife and two sons. He is the first son of a sea diver's family from Cheju,
southern Korea. In 1940, his parents settled in Japan. His
insider information comes from high-level
government contacts in Pyongyang cultivated through his 19 years as a journalist at People's Korea, a weekly. A
1994 paper he wrote captured the attention of analysts in the US State Department and Pentagon, and was followed by two
books: Kim Jong Il Day Of Korean Reunification, and Kim Jong Il's Strategy For National Reunification. Both instantly became
bible reading for North Korean officials. According to declassified information, claims Dr Kim, Mr Kim hailed them as the best
insights into his own policy thinking. 'North Korean officials, including ambassadors kid: 'Even graders know about you.
Everywhere in the DPRK you are a household name.',' laughs Dr Kim. 'They said I'm most widely
known in the country after Kim Jong Il.' But South Korea was not so enamoured. Kim Jong Il Day Of Korean
Unification was instantly banned, while the second book became a bestseller - before it was banned six months later. He is
considered such a threat that if any South Korean is to talk with the writer, he is required to ask the Unification Ministry of the
National Intelligence Service for authorisation, says Dr Kim. A South Korean embassy official in Singapore cannot confirm this
because such details are 'confidential', but adds it is likely because of Dr Kim's close ties to Pyongyang. Dailies such as The
New York Times bestowed upon him the title of 'unofficial spokesman for North Korea'.
Asked if he feels any conflict between his Japanese home and his North Korean roots, he says: 'Sometimes yes. I cannot help it.
My native place is in Japan, while my spiritual and physical homeland is Korea and in particular North
Korea.'

Chol sucks

Newsweek 2k (7-2, http://www.lai-aib.org/lai/article_lai.phtml?section=A3ABBSBC&object_id=7183)

Kim Myong Chol is North Korea’s top spin doctor. He heads the Center for Korean-
American Peace in Tokyo, and serves as the DPRK’s unofficial spokesman . His latest book, Kim Jong
Il’s Reunification Strategy, has raced up the best-seller list in South Korea since last month’s historic inter-Korean summit in
Pyongyang.
North Korea Proliferation Answers
North Korea can’t prolif

Hymans ‘12 [Jacques E. C. Hymans, PhD from Harvard, Associate Professor of International Relations at the University of
Southern California, his most recent book is Achieving Nuclear Ambitions: Scientists, Politicians, and Proliferation, “Botching the
Bomb: Why Nuclear Weapons Programs Often Fail on Their Own-and Why Iran's Might, Too,” Foreign Affairs91. 3 (May/Jun 2012):
44-53, Proquest]

The third lesson is that states that


poorly manage their nuclear programs can bungle even the supposedly
easy steps of the process. For instance, based on estimates of the size of North Korea's plutonium
stockpile and the presumed ease of weapons fabrication, U.S. intelligence agencies thought that by
the 1990s, North Korea had built one or two nuclear weapons. But in 2006, North Korea's first nuclear
test essentially fizzled, making it clear that the "hermit kingdom" did not have any working
weapons at all. Even its second try, in 2009, did not work properly. Similarly, if Iran eventually
does acquire a significant quantity of weapons-grade highly enriched uranium, this should not
be equated with the possession of a nuclear weapon.
South Korea Proliferation Answers
No risk of South Korean proliferation

Hibbs 3/15 (Mark Hibbs, Senior Associate Nuclear Policy Program, Hibbs is a Berlin-based senior associate in Carnegie’s
Nuclear Policy Program. Before joining Carnegie, for over twenty years he was an editor and correspondent for nuclear energy
publications, including Nucleonics Week and NuclearFuel, published by the Platts division of the McGraw-Hill Companies, “Will South
Korea Go Nuclear?”, March 15, 2013)

So will Seoul reach for nuclear weapons in response to fresh North Korean threats ? Currently,
and for an indefinite period that transcends the negotiation of a new nuclear agreement, that
is highly unlikely. The Asan Institute for Policy Studies, a South Korean think tank, announced a week after the test that two-
thirds of Koreans polled between February 13 and 15 "supported a domestic nuclear weapons program." Some news media
interpreted this as a signal that public opinion might pressure the government to develop
nuclear weapons, but in fact Asan said the result of the poll was "virtually unchanged" from a
year before. The press largely ignored results showing that the percentage of Koreans who
believed that North-South relations were the "most salient" issue facing the country --
between 8 percent and 15 percent -- paled in comparison to the 40 percent most worried about job creation.
According to Asan researchers, when media interest in North Korean missile launches and nuclear tests wanes, public concern drops.
A trickle of personalities in South Korea has gone on record since the North Korean test saying that they favor considering a nuclear
weapons option. The
most frequently-cited politician is Chung Moon Joon, a maverick legislator in
the National Assembly who for several years has urged that the United States redeploy
theater nuclear weapons on South Korean territory. For decades, the topic of an independent
South Korean nuclear capability was a taboo; Chung's lifting of this veil in recent years was
tailored to shock Beijing and Washington, and his message fills a niche in the conservative
wing of Korean politics. But by no means are Chung's views mainstream.

International backlash

Hibbs 3/15(Mark Hibbs, Senior Associate Nuclear Policy Program, Hibbs is a Berlin-based senior associate in Carnegie’s
Nuclear Policy Program. Before joining Carnegie, for over twenty years he was an editor and correspondent for nuclear energy
publications, including Nucleonics Week and NuclearFuel, published by the Platts division of the McGraw-Hill Companies, “Will South
Korea Go Nuclear?”, March 15, 2013)

What's more, in recent decades neither the South Korean government nor research organizations
close to it have seriously analyzed what acquiring nuclear weapons would mean for the country.
The reason they have not done so is evident. Increasingly, South Korea has thrown its weight behind a
policy of economic development based on international cooperation with trading partners,
participation in international organizations and compacts, a security alliance with the United States,
deployment of nuclear power technology, and -- especially since the late 1990s -- establishment of democratic
institutions and a civil society. Today, most or all of these developments stand in the way of South
Korean leaders pursuing nuclear weapons . Consider the country's growing reliance on nuclear power. Four decades
ago, South Korea turned to the atom when the cost of imported oil began to threaten the growing economy's balance of payments.
Since the 1970s, South Korea has built and is now operating two dozen power reactors generating nearly a third of the country's
electricity. South Korea is building more reactors, and it has installations for nuclear research, medicine, engineering, equipment
manufacture, waste treatment, disposal, and fuel fabrication. Leaving aside the considerable investment in human capital associated
with this effort,
South Korea's peaceful nuclear energy assets might be worth several hundred
billion dollars. Should South Korea reach for nuclear arms, its nuclear program and its energy
security would be at risk. Reactors would run out of fuel, and access to imported substitute
fossil fuels might be embargoed. It is unclear whether the United States would sustain its
security guarantee to South Korea. Outside the NPT, South Korea might be far more
vulnerable to attack from the North, and its relations with China, Japan, and Russia might be
frozen. On the other hand, with increasing wealth, perhaps South Korea might conclude in a crisis it could afford to take such
risks.

South Korea would ask for increased US defense

Hibbs 3/15 (Mark Hibbs, Senior Associate Nuclear Policy Program, Hibbs is a Berlin-based senior associate in Carnegie’s
Nuclear Policy Program. Before joining Carnegie, for over twenty years he was an editor and correspondent for nuclear energy
publications, including Nucleonics Week and NuclearFuel, published by the Platts division of the McGraw-Hill Companies, “Will South
Korea Go Nuclear?”, March 15, 2013)

Provocative statements by some Korean commentators, and instinctive public support in favor of nuclear weapons registered by
pollsters, express anxiety and vulnerability to North Korea's threat, not a strategy. In
theory, that could change should North
Korea acquire greater destructive capabilities and South Korean leaders lose confidence in U.S. promises to
defend it from such capabilities. Long before reaching that point, however, South Korea's leaders would
likely seek stronger defense commitments from the United States. Seoul and Washington
would also both press China to weigh in with Pyongyang .
Nuclear Power Answers
Nuclear power is dead

Wasserman 12 –senior advisor to Greenpeace USA and the Nuclear Information & Resource Service. He is author or co-author
of a dozen books (Harvey, 01/03, “2012 Is the Year to Finally Bury Nuke Power,” http://www.huffingtonpost.com/harvey-
wasserman/2012-is-the-year-to-final_b_1180444.html)

We are already very close. The


mythical "Nuclear Renaissance" has been gutted by Fukushima, low gas
prices and the escalating Solartopian revolution in green energy. Solar panels, wind turbines,
sustainable bio-fuels, geo-thermal, ocean thermal, increased efficiency and much more have simply priced
atomic energy out of the market. There is virtually no private money to build new reactors -- except where there are
huge government subsidies and guarantees. In 2012 we must make those all go away. Likewise, there are increasingly powerful
grassroots movements focused on shutting reactors that still operate. Germany has shut 7, and the rest will be gone by
2022, if not earlier. In Japan, just 11 of more than 50 reactors now operate. Because local governments can prevent nukes from re-
opening once they go down for refueling, Japan could emerge from 2012 without a single nuke on line. The biggest US battle is at
Vermont Yankee. March 21 is D-Day for forcing a nuclear corporation to honor a solemn contract it signed with a sovereign state,
agreeing to shut down if the state doesn't approve continued operations. The legislature wants the reactor shut, which Entergy now
refuses to do. But with some 430 reactors still operating worldwide, and with several score ostensibly on order, here are some of
2012's keys to finally ridding the planet of this radioactive curse: X The
switch to green power has become
definitive and is clearly unstoppable. Last year renewables generated more US electricity than nukes. Far more
private capital is now being invested in renewables than in nuclear or fossil fuels. General Electric says its
photovoltaic solar cells will generate electricity cheaper than coal within five years. Well-funded opponents are making it more
difficult to spread green technologies, but they can be beaten. X The breakdowns in the solar business are far fewer and further
between than in the fossil/nuke world. The lead in this technology has shifted to Asia. The much hyped Solyndra failure came not
from technological issues, but because the Chinese are underselling its American competitors -- and its own costs -- by 30-40%.
Returning at least some of the business to the US is essential to our economic survival. X A dollar invested in increased efficiency --
powered by accelerating breakthroughs such as LED lighting -- has long since produced more jobs and saved more energy than one
invested in nuclear power. X In-depth studies from the Union of Concerned Scientists, Rocky Mountain Institute, and a host of others
make it clear that investments in solar and wind energy yield better returns than nuclear. X It takes at very least and
optimistic five years to bring a nuclear plant on line assuming all permits are in order, but large-scale wind and solar
facilities regularly come on line in half that or less. X The decisions by Japan and Germany to abandon nuclear power have come
from countries long at the core of the industry. Japan manufactures many key reactor components, and maintains ownership stakes
in General Electric and Westinghouse, which have designed and/or built most of the world's commercial reactors. Germany's
corporate giant Siemens, an industry mainstay, has abandoned the technology to focus on renewables. As other major countries
and corporations follow suit, the nuke industry will waste away. X Those who "support nuclear power"
cannot guarantee the reactors they want built will be properly regulated or monitored. The world at large may not hear about the
next Fukushimas until long after the radioactive fallout spreads around the planet. Given the dismal state of regulation even in
"advanced" countries like Japan and the US, will those who support the "Renaissance" be there to monitor the Korean nukes sold to
the United Arab Emirates et. al.? X The US still has some $10 billion in designated loan guarantees for new reactors. Two reactors are
technically under construction in South Carolina, and two more at Georgia's Vogtle. Despite $8.33 billion in loan guarantees,
Georgia's rates are already soaring. Attempts
to get Congress to kick in more money have been blocked by the
grassroots No Nukes movement. X Local resistance to reactor projects has raged wherever reactors
operate or are proposed, and has been extremely effective. Richard Nixon promised 1000 US reactors by the
year 2000, but the operable number was 104. Those nearly 900 reactors that went missing were mostly stopped by local grassroots
movements. Everyproposed or operating reactor not killed financially can be ultimately stopped by
local opposition movements geared toward a long, hard struggle against "impossible" odds that ultimately prove
beatable. X As it has been from the start, nuclear power is a ward of the state. Nowhere on Earth are the builders held fully
responsible for their mess. The Japanese government has just coughed up a tip-of-the-iceberg $13 billion bailout for Fukushima's
owner, the Tokyo Electric Power Company. Hundreds of billions are yet to come. Either the company goes bankrupt, or the
government takes it over beforehand. Either way, the public pays financially, and with its health and that of its children. So it will be
everywhere nukes are built, including the US, where the 1957 Price-Anderson Act still limits owner liability in the wake of a
catastrophe. X Cost estimates for new reactors have already soared 200-300% and more over original prices just a
few years ago, and will continue go ever higher. By contrast, renewable technology prices continue their rapid, steep
decline. X France's nuclear industry has all but given up on the US market. A reactor under construction in Finland is years behind
schedule and billions of Euros over budget, as is another at Flamanville, in France itself. French public opinion has turned strongly
toward renewables. X US war hawks now want an attack on Iran for allegedly using commercial technology to build a Bomb. But it's
instructive to remember that the west once tried to sell 36 reactors to the Shah, who was overthrown by religious fundamentalists
in 1979, leading to the current crisis. Does the "Renaissance" blueprint mean pushing reactors everywhere, then launching
preemptive wars following the inevitable regime changes? X After more than 50 years, the radioactive waste problem has been
nowhere solved. Nevada's Yucca Mountain is
not revivable, and there are no usable high-level storage
sites anywhere else on the planet. X Nuclear power makes global warming worse. Greenhouse gases pour out of the
mining, milling, enrichment and waste management process. Massive quantities of direct heat threaten our rivers, lakes and oceans.
Thus more and more reactors must shut during hot summer months, when they are supposedly fighting global warming. X The
calculations on how much climate changing heat and steam have spewed into the atmosphere during the explosions at Chernobyl
and Fukushima remain to be done. Likewise the heat impacts of the liquid emissions into the ocean at Fukushima remain unknown.
X By wasting huge amounts of social capital, nuclear construction slows the conversion to renewables, which at the real core of
defeating global warming. X Fukushima is not over. Three melted cores remain problematic, and the entire complex is vulnerable to
aftershocks which could bring spent fuel pools crashing to the ground and cause other disasters impossible to foresee. X Nuclear
power is killing people in ever-greater numbers. The industry continues to mount its usual personal assaults on those who prove
that. But the killing power of radiation has been known since "mountain sickness" -- lung cancer -- began surfacing among Czech
uranium miners in the 1500s. The continuum is unbroken through the introduction of x-rays, the work of the Curies, radium watch
dial painting, definitive links to childhood leukemia, and more. The Hiroshima-based "science" used to establish a "safe" dose of
radiation has been thoroughly debunked. The medical consensus that there is no such thing is quite firm. X The nuclear industry
never accepted the burden proving this technology to be safe before being deployed amidst a civilian population. For a half-century
reactor backers have done a superb job of simply refusing to maintain or study reliable epidemiological data bases around
commercial reactors (as well as weapons production facilities). But as early as 1970 the chief medical officer of the Atomic Energy
Commission, Dr. John Gofman, branded commercial atomic power as a form of "premeditated mass murder." X The largest study so
far of the health impacts of Chernobyl, conducted by three Russian scientists, indicates upwards of a million casualties over the past
quarter-century. That first study of the US health impacts from Fukushima, indicates that many thousands more deaths are likely to
be suffered in the US above what's already apparent. Does all this add up to the end of nuke power? Worldwide,
the
industry is crumbling. The collapse of its private investment base, and the shutdowns in Japan, Germany,
Switzerland, Spain, Mexico, Israel and elsewhere are rapidly shrinking the technology's credible
reach.
Nuclear Leadership Answers
US nuclear leadership inevitable

BPC 12
Bipartisan Policy Center’s Nuclear Initiative, Co-chaired by Senator Pete Domenici and Dr. Warren F. “Pete” Miller, July 2012,
Maintaining U.S. Leadership in Global Nuclear Energy Markets, http://bipartisanpolicy.org/library/report/maintaining-us-leadership-
global-nuclear-energy-markets

Nuclear power already plays an important role in the U.S. energy supply mix: The
nation’s existing fleet of 104
reactors currently accounts for close to 20 percent of overall electricity production. In many parts of the
country, nuclear plants help to assure grid stability and have been a major source of cost-effective, low-carbon base-load power for
decades. The NRC, the industry’s chief regulatory overseer, is expected to approve extension of the operating licenses for most of
these plants to 60 years while striving for improved safety and increasingly efficient operations. At present, the domestic nuclear
industry is looking at limited opportunities for expansion in terms of increasing the number of U.S. plants. Currently, four new
Generation III+ nuclear reactors have been licensed by the NRC and are under construction in the Southeast. In addition, the
Tennessee Valley Authority has restarted construction activities at Watts Bar II. Given this near-term expansion, the
United States
will continue to be a world leader in the development of advanced reactor technologies , including
Generation III+ advanced passive reactors and SMRs. International interest in developing new nuclear-
generating capacity, on the other hand, presents potentially substantial business opportunities for the
domestic nuclear industry. Commercial nuclear exports generate obvious economic benefits for U.S.
firms and for the nation’s overall balance of trade. Importantly, they also help the United States retain a major role in
the evolution and maintenance of international nuclear safety and nonproliferation regimes .
Other nations not only look to the U.S. industry for operational expertise , they see the NRC as setting
the international gold standard for safety and physical security regulation. DOE’s National Nuclear Security
Administration, meanwhile, has a great deal of influence over the nonproliferation aspects of international
fuel-cycle issues.
Nuclear Primacy Answers
Nuclear stockpile will inevitably be stable—nuclear primacy creates security dilemma and
miscalculation—net worse for stability

Elkind ’12 (David J. is a research intern for the Project on Nuclear Issues “American Nuclear Primacy: the End of MAD or a New
START?” May 22 http://csis.org/blog/american-nuclear-primacy-end-mad-or-new-start

These results show that the United States cannot reasonably claim to have obtained nuclear
primacy. Reductions in the two nations’ respective arsenals, coupled with the large number of Russian
targets collaborate to make it exceptionally difficult to destroy the Russian arsenal in a counterforce first
strike. Even though my results demonstrate a modest level of confidence in the baseline scenario, I believe that mutually assured
destruction remains in place. Because the costs of even a single Russian warhead surviving would have such devastating
consequences for the United States, I do not believe that any President or military planner would care to wager America’s most
populous cities in conducting a nuclear first strike. While these results speak to the purely military considerations of that choice, the
political, ethical and humanitarian considerations likewise make such an action highly unlikely. Even though this article
concludes that the US could not carry out a counterforce strike on the Russian arsenal in 2012, and therefore does not
possess nuclear primacy, this should not be interpreted as a call to restart the arms race or
otherwise acquire primacy. Liber and Press write that “the shift in the nuclear balance could
significantly damage relations among the great powers and increase the probability of nuclear
war,” and outline a variety of possible mechanisms by which this could come to pass and present rebuttals to counterarguments
(interested readers should refer to Lieber and Press, “The End of MAD?” 31-38). To bridge the gap in nuclear capabilities,
Russia and China may undertake perilous activities to restore the nuclear balance, such as pre-
delegated launch authority, a launch-on-warning posture, or larger nuclear arsenals. Pre-delegated
launch authority increases the risk of unauthorized nuclear use; Cold War experience confirms that launch-
on-warning postures are vulnerable to false alarms initiating a counter-attack to imaginary
missiles; arms races carry the risk that one side will perceive that it has gained the upper hand
and undertake a nuclear first use. Furthermore, nuclear primacy carries considerable risks in times of crisis. In the
event of a political crisis or a conventional war between the US and a rival power, the threat of a disarming strike
by the United States may predispose the rival to land the first blow while it still has the means to do
so. In this way, having a reduced confidence in the ability of the US to carry out a first strike
should be read as a stabilizing feature of international politics, as strategic stability (if it had ever
departed) has been restored as a pillar of the international system . External to these considerations, achieving
nuclear primacy would be a pyrrhic victory. The preceding analysis assumes that the United States is in
possession of perfect intelligence on the locations and attributes of Russian nuclear weapons facilities and is
able to carry out such an attack unhindered by air- or missile-defenses (and concludes such an attack is ill-advised
despite possessing perfect information). Even if mobile missiles do not continuously patrol, it would make sense for Russia
to shuttle them from one garrison to another in order to decrease Russia’s opponents’ confidence in accounting for all
of them. Furthermore, Russia’s decision to deploy its mobile forces in the event of a crisis (or continuously as a matter
of policy) could spark concerns in Washington that either a Russian attack is immanent or simply that United
States’ confidence in a first-strike option has evaporated , creating further perceptions of insecurity
and upsetting the strategic environment which, in the mind of US policymakers, has assumed nuclear primacy. What’s more,
mobile deployments are a cheap, easy countermeasure that would effectively negate the
confidence gained (such as any is gained) from believing that the United States has nuclear primacy. Achieving, and then
maintaining, a position of primacy introduces several significant strategic concerns of its own, and would hardly enhance
the security of the United States or the international system. I would like to advance this line of argumentation one step further.
If this model accurately reflects reality and a Liber and Press-style counterforce strike on Russia’s nuclear arsenal is unlikely to
succeed, then deep cuts to the nuclear arsenal and the decision to abandon counterforce targeting
gains credibility. That is, deep cuts to the nuclear arsenal would not mean abandoning counterforce doctrine because that has
already happened. Simply put, attempting the counterforce attack would include an inescapable risk to the United States – and we
can rest easier knowing that this is the case.

No modernization regardless and nuclear deterrence is outdated

WaPo ‘12 [Washington Post “Aging U.S. nuclear arsenal slated for costly and long-delayed modernization”,
http://www.washingtonpost.com/world/nationa l-security/us-nuclear-arsenal-is-ready-for-overhaul/2012/09/15/428237de-f830-
11e1-8253-3f495ae70650_story.html]

There is no official price tag for the effort to upgrade and maintain the 5,113 warheads in the inventory, to replace old
delivery systems and to renovate the aging facilities where nuclear work is performed . A study this summer by the
nonpartisan Stimson Center, a Washington think tank, estimated costs would be at least $352
billion over the coming decade to operate and modernize the current arsenal. Others say the figure
could be far higher, particularly if the work is delayed even longer. The timing does not fit with the nation’s
evolving defense posture, either. Over the past decade, the U.S. military has moved away from nuclear
deterrence and major military interventions in favor of more precise tactics rooted in Special
Operations forces and quick tactical strikes deemed more effective against today’s enemies. Federal
officials and many outside analysts are nonetheless convinced that, after years of delay, the government must invest huge sums if it
is to maintain the air, sea and land nuclear triad on which the country has relied since the start of the Cold War. Failing to act before
the end of next year, they say, is likely to mean that there won’t be enough time to design and build the new systems that would be
required if the old arsenal is no longer safe or reliable. “I’ve been doing this for 20 years, and I haven’t seen a moment like this,”
Thomas P. D’Agostino, who leads the National Nuclear Security Administration (NNSA), the federal agency charged with managing
the safety of the nuclear arsenal, said in an interview. The debate over the future of the nation’s nuclear arsenal is playing out in
Congress and within the administration. Public reports, interviews with government officials and outside experts and visits to nuclear
facilities rarely seen by outsiders provided a portrait of the scope and cost of maintaining and refurbishing the nuclear stockpile
underlying the debate.

No scenario for losing deterrence – subs check

Kristensen ‘12 -- FAS nuclear weapons expert [Hans, "DOD: Strategic Stability Not Threatened Even by Greater Russian
Nuclear Forces," FAS, 10-10-12, www.fas.org/blog/ssp/2012/10/strategicstability.php, accessed 1-27-13, mss]

DOD: Strategic Stability Not Threatened Even by Greater Russian Nuclear Forces A Department of
Defense (DOD) report on Russian nuclear forces, conducted in coordination with the Director of National Intelligence and sent to
Congress in May 2012, concludes that even
the most worst-case scenario of a Russian surprise disarming first strike
against the United States would have “little to no effect” on the U.S. ability to retaliate with a devastating
strike against Russia. I know, even thinking about scenarios such as this sounds like an echo from the Cold War, but the Obama
administration has actually come under attack from some for considering further reductions of U.S. nuclear forces when Russia and
others are modernizing their forces. The point would be, presumably, that reducing while others are modernizing would somehow
give them an advantage over the United States. But the DOD report concludes that Russia “would not be able to achieve a militarily
significant advantage by any plausible expansion of its strategic nuclear forces, even in a cheating or breakout scenario under the
New START Treaty” (emphasis added). The conclusions are important because the report come after Vladimir Putin earlier this year
announced plans to produce “over 400” new nuclear missiles during the next decade. Putin’s plan follows the Obama
administration’s plan to spend more than $200 billion over the next decade to modernize U.S. strategic forces and weapons
factories. The conclusions may also hint at some of the findings of the Obama administration’s ongoing (but delayed and secret)
review of U.S. nuclear targeting policy. No Effects on Strategic Stability The DOD report – Report on the Strategic Nuclear Forces of
the Russian Federation Pursuant to Section 1240 of the National Defense Authorization Act for Fiscal Year 2012 – was obtained
under the Freedom of Information Act. It describes the U.S. intelligence community’s projection for the likely development of
Russian nuclear forces through 2017 and 2022, the timelines of the New START Treaty, and possible implications for U.S. national
security and strategic stability. Much of the report’s content was deleted before release – including general and widely reported
factual information about Russian nuclear weapons systems that is not classified. But the important concluding section that
describes the effects of possible shifts in the number and composition of Russian nuclear forces on strategic stability was released in
its entirety. The section “Effects on Strategic Stability” begins by defining that stability in the strategic nuclear relationship between
the United States and the Russian Federation depends upon the assured capability of each side to deliver a sufficient number of
nuclear warheads to inflict unacceptable damage on the other side, even with an opponent attempting a disarming first strike.
Consequently, the report concludes, “the only Russian shift in its nuclear forces that could undermine the basic framework of mutual
deterrence that exists between the United States and the Russian Federation is a scenario that enables Russia to deny the United
States the assured ability to respond against a substantial number of highly valued Russian targets following a Russian attempt at a
disarming first strike” (emphasis added). The DOD concludes that such a first strike scenario “will most likely not occur.” But even
if it did and Russia deployed additional strategic warheads to conduct a disarming first strike, even significantly
above the New START Treaty limits, DOD concludes that it “would have little to no effects on the U.S.
assured second-strike capabilities that underwrite our strategic deterrence posture” (emphasis
added). In fact, the DOD report states, the “Russian Federation…would not be able to achieve a militarily significant advantage by
any plausible expansion of its strategic nuclear forces, even in a cheating or breakout scenario under the New START Treaty,
primarily because of the inherent survivability of the planned U.S. Strategic force structure, particularly the OHIO-class ballistic
missile submarines, a number of which are at sea at any given time.” Implications These are BIG conclusions with BIG
implications. They reaffirm conclusions made by DOD in 2010
[http://www.foreign.senate.gov/publications/download/executive-report-111-06-treaty-with-russia-on-measures-for-further-
reduction-and-limitation-of-strategic-offensive-arms-the-new-start-treaty], but the new report is important because it comes after
Russia earlier this year announced plans to produce “over 400” nuclear missiles over the next decade. In the real world, however,
Russian nuclear forces are not increasing. Even with Putin’s missile production plan, simultaneous retirement of older missile will
continue the downward trend and result in a net reduction of Russian strategic nuclear forces over the next decade and a half. This
fact has not stopped some from arguing against additional U.S. nuclear reductions. Their argument is that reductions are unwise at a
time when Russia and others are modernizing their nuclear forces. Others have even argued that Russia could break out of the New
START Treaty by cheating and presumably achieve some strategic advantage. Even the U.S. Senate’s advice and consent resolution
that in 2010 approved the New START Treaty required that “the President should regulate reductions in United States strategic
offensive arms so that the number of accountable strategic offensive arms under the New START Treaty possessed by the Russian
Federation in no case exceeds the comparable number of accountable strategic offensive arms possessed by the United States to
such an extent that a strategic imbalance endangers the national security interests of the United States” (emphasis added). A similar
obsession with numbers was echoed in the 2012 report by the State Department’s International Strategic Advisory Board on future
U.S.-Russian “Mutual Assured Stability,” which concluded that it requires some “rough parity” of nuclear forces. (A similar number
obsession has evolved with NATO about non-strategic nuclear weapons, but that’s another story). But the DOD report appears to
conclude that such warnings and parity requirement are missing the point. Strategic
stability and deterrence today
are provided by a secure retaliatory capability , primarily ballistic missile submarines. In fact, although ICBMs and
bombers also play a role in the U.S. nuclear posture, they seem oddly absent from the report’s description of what is required to
maintain strategic stability based on a sufficient secure retaliatory capability. Retaining that capability, it seems, does not even
require the ballistic missile submarines to be on alert (although the report doesn’t explicitly say so). It only requires that a
sufficient number of submarines “are at sea” and secure at any given time – or perhaps even only in a crisis.
Likewise, the conclusion that a Russian disarming first strike “will most likely not occur” may be obvious to most but, if formal, seems
to remove the need for having ICBMs on alert, as long as a sufficient number of submarines are at sea to provide the basic
deterrence that underpins strategic stability.

Conventional weapons solve- don't need nukes


Gerson ‘9 (CARNEGIE ENDOWMENT FOR INTERNATIONAL PEACE RETHINKING U.S. NUCLEAR POSTURE MODERATOR: JAMES
ACTON, ASSOCIATE, NONPROLIFERATION PROGRAM, CARNEGIE ENDOWMENT SPEAKERS: MICHAEL S. GERSON, RESEARCH
ANALYST, CENTER FOR NAVAL ANALYSES Transcript by Federal News Service Washington, D.C.

The National Academy of Sciences report on the future of U.S. nuclear weapons policy advocated “no first use.” Again, along these
we didn’t need to rely on the threat of nuclear
lines that the major conventional threat had disappeared, therefore
weapons to deter – to help to bolster deterrence of a conventional attack. Moreover, the conventional
capability – U.S. conventional superiority demonstrated so well in the first Gulf War made it
such that conventional capabilities were absolutely sufficient for deterrence . Even Paul Nitze, one of
the architects of NSC- 68 in 1994 asked “is it time to junk our nukes?” His argument was smart conventional weapons should be the
principal U.S. deterrent.
They’re safer, they cause less collateral damage, they provide more flexibility,
there’s less risk of escalation, and perhaps most importantly, they’re highly credible. So people
who had once concocted rather elaborate scenarios for nuclear war-fighting came around to
this view, that smart conventional weapons would be the principal deterrent , whereas those in favor
of no-first-use advocated one set of use.

Deterrence is sufficient in the status quo – primacy is a paper tiger


Bin 06 (Li Bin, a Chinese physicist, works on arms control and international security, professor of the Department of International
Relations and Institute of International Studies, Tsinghua University, directs the Arms Control Program at Tsinghua University, 2006,
“Paper Tiger with Whitened Teeth”, Issue 4, http://www.chinasecurity.us/index.php?
option=com_content&view=article&id=213&Itemid=8&lang=zh)

The Lieber and Press paper also raises the concern that China might use nuclear weapons to
destroy American cities if the United States supports the separatists in Taiwan in a war for separation, a
suggestion which arose from a Chinese military scholar.6 In fact, a more accurate interpretation of this comment
is that China could extend its nuclear deterrence to dissuade mass conventional attack from the
United States in a Sino-U.S. war over Taiwan. The idea is that China could compensate for its conventional inferiority vis-
à-vis the United States by adding the influence of nuclear weapons. However, the United States should not be
concerned about this for two reasons. First, China’s leaders fully understand that nuclear weapons are
a paper tiger in this kind of conventional conflict. No matter who is defeated in conventional war
(if it ever came to that), neither China nor the United States would be able to alter the outcome using
nuclear weapons. The second reason is that to deter a nuclear attack (minimum deterrence) does not require
nuclear primacy. A retaliatory nuclear force larger than the base criterion described by Robert McNamara should
be sufficient for this purpose.7 The coercive power of minimum nuclear deterrence (deterring others from
using nuclear weapons) has been held by the United States for over half a century. If the United States would
achieve nuclear primacy today, it would make little contribution to the U.S. minimum nuclear deterrence. 

History proves

Blair and Yali 06 [Bruce G. Blair, President of the World Security Institute, author of numerous articles and books on security
issues including the Logic of Accidental Nuclear War and Global Zero Alert for Nuclear Forces, and Chen Yali, editor in chief of
Washington Observer, a Program Manager of Chen Shi China Research Group, Autumn 2006, “The Fallacy of Nuclear Primacy”, pp.
51-77, World Security Institute, http://www.wsichina.org/cs4_4.pdf .]
All sides all but ignored the theoretical first strike capability of the United States during the past
15 years (and much longer in the case of China). This history is not a perfect crucible for testing all of the professors’ hypotheses,
but the preponderance of evidence so far refutes their argument. What this recent history really
seems to be suggesting is that U.S. nuclear primacy is an academic artifice that was and is practically
useless for understanding America’s relations with other nuclear powers . Nuclear primacy in
modern times offers no exploitable political leverage. Russia and China appear quite confident in
their deterrent arsenals in spite of the lopsided U.S. advantage estimated by models of nuclear
war.

Can’t solve irrational actors

Blair and Yali 06 –[Bruce G. Blair, President of the World Security Institute, author of numerous articles and books on security
issues including the Logic of Accidental Nuclear War and Global Zero Alert for Nuclear Forces, and Chen Yali, editor in chief of
Washington Observer, a Program Manager of Chen Shi China Research Group, Autumn 2006, “The Fallacy of Nuclear Primacy”, pp.
51-77, World Security Institute, http://www.wsichina.org/cs4_4.pdf .]

In all calculations of nuclear primacy and deterrence, the players


are assumed to be rational. However, rational
actors might lapse into irrational behavior in readily imaginable ways that are completely obtuse
to the nuclear primacy framework. The obvious scenario in this regard concerns the defeat of
China’s military force in a potential Taiwan conflict. The Taiwan issue has been a core national
interest of China, one that arouses such fervent emotions throughout the country that irrational
behavior in its use of nuclear weapons cannot be ruled out.

Their assumptions are wrong

Can’t surprise first strike


Starr and Yarynich ‘7 (Valery Yarynich, retired Colonel and served at the Center for Operational and Strategic Studies of the
Russian General Staff, Steven Starr, US nuclear weapons expert, "'Nuclear Primacy' is a Fallacy", 4/4/07,
http://www.globalresearch.ca/index.php?context=va&aid=4991)

We believe such reactions can lead to a deterioration of relations between the U.S. and the Russian Federation , particularly in the
area of nuclear arms control. This is both unfortunate and unnecessary, because the “Nuclear Primacy” argument is
based upon flawed logic and questionable methodology. The conclusions reached by Lieber and
Press about a U.S. “Nuclear Primacy” over Russia and the corresponding results of their calculations in tables are
erroneous. Although their set of initial data is sufficiently full and correct (Russian nuclear forces and American offensive means),
both their model and method of assessing final results are incorrect. We share their concern about the
(potential) danger of such a phenomenon as U.S. “Nuclear Primacy” over Russia , but nevertheless we believe that it is absent
today and cannot exist in the future. Our arguments are as follows. One should not estimate the strategic military
results of a massive nuclear strike without first conducting a preliminary assessment of the ecological consequences of such an
attack, because these consequences can be clearly unacceptable for both an attacker and the world as a whole. Lieber and Press
ignored this consideration. An ecological examination must include an assessment of all possible aspects of this attack, including the
consequences of: hundreds of American nuclear warheads detonating on Russian soil; the destruction of thousands of Russian
nuclear warheads and the corresponding secondary effects; the interception of Russian retaliation warheads by U.S. Ballistic Missile
Defenses (BMD); and the explosions of Russian warheads on American territory, if U.S. BMD failed. In any case, the results of this
examination must be made public, because the final decision about their admissibility must belong to the people rather than to a
handful of politicians and high-ranking military officers. Lieber and Press examine only one scenario: a Surprise
Attack at Peacetime Alert levels (SAPTA). Although they concede that this event is not “likely”,
they use this variant as the basis for all their serious conclusions. We will not talk about the moral and
ethical reasons, but rather focus upon the political and military-technical issues which render this approach unworkable. First, to
implement SAPTA the National Command Authority (NCA) must have in place a set of
legislatively approved special conditions authorizing this action. No such set now exists.
Secondly, the NCA is obliged to inform the nation about this critical decision before a first strike
is launched. This must be done if only to provide a time-buffer in which its citizens could implement some measures of protection
against the possible negative consequences of the attack. Third,
in order to conduct a first strike it is necessary to
implement a number of organizational and technical procedures within the strategic nuclear forces. This is because in
peacetime there are numerous procedural and technological blocks in place which are designed to protect
nuclear weapons against human error, accidents and sabotage. In order to remove such barriers as a preliminary step towards
launching a nuclear first strike, it would require the participation of a significant number of crews on duty working at different
operational levels. The implementation of all the above mentioned circumstances as preparations
for a “surprise” first
strike would be technically impossible to hide. Therefore, the opposite side would have a certain
amount of time to raise the combat readiness of its strategic nuclear forces. If Russia did that, then, as
Lieber and Press recognize themselves, nuclear retaliation is inevitable.

Can’t disarm all of their nukes

Acton ‘10 (James M. Acton, senior associate in the Nuclear Policy Program at the Carnegie Endowment and a Stanton Nuclear
Security Fellow, physicist who specializes in deterrence, disarmament, nonproliferation, and nuclear energy, joint UK member of the
International Panel on Fissile Materials, 5/1/10, “MANAGING VULNERABILITY”, http://goliath.ecnext.com/coms2/gi_0199-
12578010/Second-strike-is-the-U.html.)

The main problem with Lieber and Press’ argument is that no state actually has a small arsenal consisting solely
of silo-based missiles. China, Iran, and North Korea are all focusing on the development of road-
mobile missiles (in fact, the latter two do not appear to have any silo-based weapons at all). Although China has very
few road-mobile missiles that could reach U.S. soil, and Iran and North Korea have none, each
country has plenty that could reach the territory of key U.S. friends and allies. The challenge
with destroying road-mobile weapons is locating them. If their location is known, conventional
munitions will suffice. If their location is not known, even nuclear weapons are useless (discounting
the possibility of wide-area nuclear bombardment, which Lieber and Press would presumably not advocate). Locating mobile
ballistic missiles is exceptionally hard. According to the Gulf War Air Power Survey (an official
analysis of U.S. Air Force operations during the Persian Gulf War), the United States launched
about 1,500 sorties against Scud launchers in Iraq during the 1991 war; not a single mobile
launcher was confirmed destroyed. Granted, U.S. capabilities and doctrine have improved markedly since then.
Nonetheless, it is still fiendishly diffcult to locate mobile missiles hidden by a well-prepared enemy. The bottom line is that
because the bulk of China’s, Iran’s, and North Korea’s missile forces are mobile, the United
States could not eliminate their entire arsenals with a high probability. Increasing the United States’
ability to eliminate only silo-based weapons would add very little to deterrence
China can move weapons

Li, 6 – Director of the Arms Control Program and Professor of the Institute of International Studies (2006, Bin, China Security
“Paper Tiger with Whitened Teeth,” http://www.chinasecurity.us/index.php?option=com_content&view=article&id=213, Sawyer)

The Lieber and Press paper tries to prove that the United States can destroy all Russian or Chinese long-range nuclear weapons in a
surprise preventive nuclear strike in peacetime. But in any security dispute some form of threat signaling is
necessary. For example, after Russia receives strong signals of a nuclear threat from the United States, it may disperse its mobile
ICBMs and nuclear submarines or launch its silo-based ICBMs when its early warning systems detect even unclear signals of
incoming warheads. China may relocate its cave-based ICBMs when it interprets strong nuclear
signals by the United States. These efforts can reduce the effectiveness of the preemptive U.S.
nuclear strike and therefore make the number of survivable Russian and Chinese long-range nuclear weapons greater than
zero. In fact, the authors acknowledge that “(a) preemptive strike on an alerted Russian arsenal would still likely fail, but a surprise
attack at peacetime alert levels would have a reasonable chance of success.” 

Russia can retaliate

Starr and Yarynich ‘7 (Valery Yarynich, retired Colonel and served at the Center for Operational and Strategic Studies of the
Russian General Staff, Steven Starr, US nuclear weapons expert, "'Nuclear Primacy' is a Fallacy", 4/4/07,
http://www.globalresearch.ca/index.php?context=va&aid=4991)

Lieber and Press also assume that the Russian Early Warning System will be completely unable to
reveal a massed American attack capable of destroying all Russian nuclear forces . “A critical issue for
the outcome of a U.S. attack [they say] is the ability of Russia to launch on warning (i.e., quickly launch a retaliatory strike before its
forces are destroyed). It is unlikely that Russia could do this.” We believe this important conclusion demands more serious
calculations than the mere statement that “it is unlikely ”. It's necessary to prove that the
Russian EWS will be completely incapable of revealing such massed American attack which is capable
of destroying all Russian nuclear forces. Admittedly, the Russian EWS is now weakened. However, if it is able to detect even
a small part of the American attack, then it is impossible to rule out the possibility that Russia
will react by utilizing the policy of Launch on Warning (LoW), i.e., launching its missiles before the attack is
confirmed by nuclear detonations. The number of nuclear warheads in a Russian LoW strike will be far more than in case of a pure
LuA (Launch under Attack) variant.
Oil Shocks Answers
Frontline

And even if there are collapses, Weiss says oil can fill in—ample reserves in both US and
foreign countries—we’ve had excess since Bush One—strategic reserve is 96 percent full

No vulnerability – spare capacity, alternative routes, and structural insulation

Kahn ‘11 (Jeremy Kahn, journalist, “Crude Reality,” February 13, 2011,
http://www.boston.com/bostonglobe/ideas/articles/2011/02/13/crude_reality/?page=full, )

There’s no denying the importance of Middle Eastern oil to the US economy. Although only 15 percent of imported US oil comes directly from the
Persian Gulf, the region is responsible for nearly a third of the world’s production and the majority of its known reserves. But the
oil market is
also elastic: Many key producing countries have spare capacity, so if oil is cut off from one country, others tend to

increase their output rapidly to compensate. Today, regions outside the Middle East, such as the west coast of Africa, make up an
increasingly important share of worldwide production. Private companies also hold large stockpiles of oil to smooth over shortages — amounting to a
few billion barrels in the United States alone — as does the US government, with 700 million barrels in its strategic petroleum reserve. And the

market can largely work around shipping disruptions by using alternative routes; though they are more
expensive, transportation costs account for only tiny fraction of the price of oil. Compared to the 1970s, too, the structure of the US

economy offers better insulation from oil price shocks. Today, the country uses half as much
energy per dollar of gross domestic product as it did in 1973, according to data from the US Energy Information
Administration. Remarkably, the economy consumed less total energy in 2009 than in 1997, even though its GDP rose and the population grew. When
it comes time to fill up at the pump, the average US consumer today spends less than 4 percent of his or her disposable income on gasoline, compared
with more than 6 percent in 1980. Oil, though crucial, is simply a smaller part of the economy than it once was.

Empirically proven, no impact to shocks

Jaffe ‘8 [ Amy Myers Jaffe is the Wallace S. Wilson Fellow for Energy Studies at the James A. Baker III Institute for Public Policy at
Rice University, “ Opportunity, not War,” Survival | vol. 50 no. 4 | August–September 2008 | pp. 61–82 ]

We’ve heard the argument before: scarcity of future oil supplies is a danger to the global international system and will create
international conflict, death and destruction. In 1982, noted historian and oil-policy guru Daniel Yergin wrote that the energy
question was ‘a question about the future of Western society’, noting that ‘stagnation and unemployment and depression tested
democratic systems in the years between World War I and World War II’ and asserting that if there wasn’t sufficient oil to drive
economic growth, the ‘possibilities are unpleasant to contemplate’.1 His words proved typical prose foreboding the top of a
commodity cycle. A year later, oil prices began a four-year collapse to $12 a barrel. That oil is a cyclical industry is not in question.
Since 1861, oil markets have experienced more than eight boom-and-bust cycles . In 1939, the US
Department of the Interior announced that only 13 years of oil reserves remained in the United States. In more recent history,
Middle East wars or revolutions produced oil price booms in 1956, 1973, 1979, 1990 and 2003. Each
time, analysts rushed to warn of doomsday scenarios but markets responded and oil use was
curtailed both by market forces and government intervention rather than by war and massive global
instability. The question Nader Elhefnawy raises in ‘The Impending Oil Shock’ is whether this time will be different.

Adaptation empirically resolves supply problems


Kahn ‘11 (2/13/11, Jeremy, Boston Globe, “Crude reality”, http://articles.boston.com/2011-02-13/news/29336191_1_crude-oil-
shocks-major-oil-producers SW)

Among those asking this tough question are two young professors, Eugene Gholz, at the University of Texas, and Daryl Press, at
Dartmouth College. To find out what actually happens when the world’s petroleum supply is
interrupted, the duo analyzed every major oil disruption since 1973. The results, published in a recent issue of the journal
Strategic Studies, showed that in almost all cases, the ensuing rise in prices, while sometimes steep, was
short-lived and had little lasting economic impact. When there have been prolonged price rises,
they found the cause to be panic on the part of oil purchasers rather than a supply shortage .
When oil runs short, in other words, the market is usually adept at filling the gap. One striking example
was the height of the Iran-Iraq War in the 1980s. If anything was likely to produce an oil shock, it was this: two major
Persian Gulf producers directly targeting each other’s oil facilities. And indeed, prices surged 25
percent in the first months of the conflict. But within 18 months of the war’s start they had
fallen back to their prewar levels, and they stayed there even though the fighting continued to rage for six more
years. Surprisingly, during the 1984 “Tanker War” phase of that conflict — when Iraq tried to sink oil
tankers carrying Iranian crude and Iran retaliated by targeting ships carrying oil from Iraq and its Persian Gulf allies —
the price of oil continued to drop steadily. Gholz and Press found just one case after 1973 in which the market
mechanisms failed: the 1979-1980 Iranian oil strike which followed the overthrow of the Shah, during which Saudi Arabia, perhaps
hoping to appease Islamists within the country, also led OPEC to cut production, exacerbating the supply shortage. In their paper ,
Gholz and Press ultimately conclude that the market’s adaptive mechanisms function
independently of the US military presence in the Persian Gulf, and that they largely protect the American
economy from being damaged by oil shocks. “To the extent that the United States faces a national security
challenge related to Persian Gulf oil, it is not ‘how to protect the oil we need’ but ‘how to assure consumers that there is nothing to
fear,’ ” the two write. “That is a thorny policy problem, but it does not require large military deployments and costly military
operations.” There’s no denying the importance of Middle Eastern oil to the US economy. Although only 15 percent of
imported US oil comes directly from the Persian Gulf, the region is responsible for nearly a third
of the world’s production and the majority of its known reserves. But the oil market is also elastic: Many key
producing countries have spare capacity, so if oil is cut off from one country, others tend to
increase their output rapidly to compensate . Today, regions outside the Middle East, such as the
west coast of Africa, make up an increasingly important share of worldwide production . Private
companies also hold large stockpiles of oil to smooth over shortages — amounting to a few billion barrels
in the United States alone — as does the US government, with 700 million barrels in its strategic
petroleum reserve. And the market can largely work around shipping disruptions by using
alternative routes; though they are more expensive, transportation costs account for only tiny fraction of the price of oil.

Prefer the consensus of new economic research

Kahn ‘11 (Jeremy Kahn, journalist, “Crude Reality,” February 13, 2011,
http://www.boston.com/bostonglobe/ideas/articles/2011/02/13/crude_reality/?page=full, )

Economists have a term for this disruption: an oil shock. The


idea that such oil shocks will inevitably wreak havoc on
the US economy has become deeply rooted in the American psyche, and in turn the United States has
made ensuring the smooth flow of crude from the Middle East a central tenet of its foreign policy. Oil security is one of the primary reasons America
has a long-term military presence in the region. Even aside from the Iraq and Afghan wars, we have equipment and forces positioned in Oman, Saudi

Arabia, Kuwait, and Qatar; the US Navy’s Fifth Fleet is permanently stationed in Bahrain. But a growing body of economic
research suggests that this conventional view of oil shocks is wrong. The US economy is far less
susceptible to interruptions in the oil supply than previously assumed, according to these
studies. Scholars examining the recent history of oil disruptions have found the worldwide oil market to be
remarkably adaptable and surprisingly quick at compensating for shortfalls. Economists have found that
much of the damage once attributed to oil shocks can more persuasively be laid at the feet of bad government policies. The US economy, meanwhile,
has become less dependent on Persian Gulf oil and less sensitive to changes in crude prices overall than it was in 1973.

Empirics prove a multitudes of mechanisms generate resiliency

API ‘4 (American Petroleum Institute, “Achieving Energy Security in an Interdependent World,” most recent date cited – 2004,
http://www.api.org/policy-and-issues/policy-items/safety/achieving-energy-security-in-an-interdependent-world.aspx, )

Progress has been made in this area. While global dependence


on oil imports has generally increased since 1980,
vulnerability to short-term interruptions has not. The world has weathered several major
interruptions since 1980, namely the tanker war in the late 80s, the invasion of Kuwait in 1990, and the invasion of Iraq in 2003, none of
which has produced economic damage approaching either the magnitude or the duration of the 70s disruptions. In part, this is
attributable to measures adopted to manage such risks, including building strategic reserves, promoting
free trade and investment, and developing traditional diplomatic and military instruments to secure
that trade. In part, it is attributable to favorable market or political trends, such as the decline in the share of oil in GDP and the
increased access to potentially productive lands as a result of the breakup  of the Soviet Union. But primarily, it was due to the fact that OPEC since

1980 has had available a large volume of excess capacity, which it has generally used to offset any such shortfalls.
---Oil Shocks- Empirics Disprove

Global spare capacity is huge – zero risk of serious shortages

Gholz and Press 8 (Eugene, Professor of Public Affairs – University of Texas at Austin, and Daryl G., Professor of
Government – Dartmouth College, “All the Oil We Need”, The New York Times, 8-21, Lexis)

WHILE oil prices have declined somewhat of late, the volatility of the market and the political and religious unrest in major oil-
producing countries has Americans worrying more than ever about energy security. But they have little to fear -- contrary to
common understanding, there
are robust stockpiles of oil around the globe that could see us
through any foreseeable calamities on the world market. True, trouble for the world's energy supplies
could come from many directions. Hurricanes and other natural disasters could suddenly disrupt oil production or
transportation. Iran loudly and regularly proclaims that it can block oil exports from the Persian Gulf. The anti-American rhetoric
of President Hugo Chavez of Venezuela raises fears of an export cutoff there. And ongoing civil unrest wreaks havoc with
Nigeria's output. Even worse, this uncertainty comes in the context of worrisome reports that oil producers have little spare
capacity, meaning that they could not quickly ramp up production to compensate for a disruption. But such fears rest on a
misunderstanding. The world actually has enormous spare oil capacity. It has simply moved. In the
past, major oil producers like Saudi Arabia controlled it. But for years the world's major consumers have bought extra oil to fill
their emergency petroleum reserves. Moreover, whereas the world's reserve supply once sat in relatively inaccessible
pools, much of it now sits in easily accessible salt caverns and storage tanks. And consumers
control the spigots. During a supply disruption, Americans would no longer have to rely on the good will of foreign
governments. The United States alone has just more than 700 million barrels of crude oil in its Strategic
Petroleum Reserve. Government stockpiles in Europe add nearly another 200 million barrels of crude and more than 200
million barrels of refined products. In Asia, American allies hold another 400 million barrels. And China is
creating a reserve that should reach more than 100 million barrels by 2010. Those figures only count the government-
controlled stocks. Private inventories fluctuate with market conditions, but American commercial inventories alone include well
over a billion barrels. Adding up commercial and government stockpiles, the major consuming countries around
the world control more than four billion barrels. Some policy makers and analysts worry that these emergency
stocks are too small. For example, they sometimes compare the American strategic reserve to total American consumption,
so the reserves appear dangerously inadequate. The United States consumes about 20 million barrels of oil every day, so the
Strategic Petroleum Reserve could only supply the country for 35 days. (Furthermore, the United States could not draw oil out
of the reserve at anything approaching a rate of 20 million barrels per day.) This is why President Bush in his 2007 State of the
Union address called for doubling the strategic reserve. But
this vulnerability is a mirage. The size of
plausible disruptions, not total consumption, determines the adequacy of global reserves. The worst oil
disruptions in history deprived global markets of five million to six million barrels per day.
Specifically, the collapse of the Iranian oil industry during the revolution in 1978 cut production by nearly five million barrels a
day, and the sanctions on Iraq after its conquest of Kuwait in 1990 eliminated 5.3 million barrels of supply. If
a future
disruption were as bad as history's worst, American and allied governments' crude oil stocks alone
could replace every lost barrel for eight months .

No risk of shocks—massive reserves and safeguards solve

Weiss ’12 – Senior Fellow and Director of Climate Strategy at the Center for American Progress (Weiss, Daniel J.. “Preparing for
the Next Oil Price Shock” May 18, 2012. http://www.americanprogress.org/issues/2012/05/spr_g8.html)
Some argue there should be no sale of reserve oil unless there is actually a severe supply disruption, rather than selling in response
to high oil prices driven up by Wall Street speculators in anticipation that there might be a Persian Gulf supply interruption. But the
United States and its allies have ample oil reserves that could be used in the event of another huge
price spike or a supply disruption so as to pop the speculative bubble and provide economic
relief. The United States has significant oil reserves. The Strategic Petroleum Reserve is 96 percent full with
nearly 700 million barrels of oil. The Organisation for Economic Co-operation and Development nations had more
than 800 million barrels of oil reserves at the end of 2011. Selling 30 million barrels from each reserve would
reduce total reserves by less than 4 percent. In addition, U.S. reserve oil has been sold under every
president beginning with George H.W. Bush. He sold 17.2 million barrels of reserve oil in advance of the 1991 Gulf
War in anticipation of supply disruption that did not occur. In 1996 the Republican Congress led by Speaker of the House Newt
Gingrich (R-GA) sold 23 million barrels of oil to reduce the federal budget deficit at a time when it was less than 80 percent full. In
other words, the oil in the U.S. Strategic Petroleum Reserve is not some sacred oil supply only
to be used during an oil embargo or pipeline destruction .

Newest empirical study proves no risk—multiple market solutions

Khadduri ’11 - Former Middle East Economic Survey Editor-in-Chief (Walid, "The impact of rising oil prices on the economies of
importing nations", http://english.alarabiya.net/views/2011/08/23/163590.html)

What is the impact of oil price shocks on the economies of importing nations? At first glance, there appears
to be large-scale and extremely adverse repercussions for rising oil prices. However, a study
published this month by researchers in the IMF Working Paper group suggests a different picture
altogether (it is worth mentioning that the IMF has not endorsed its findings.) The study (Tobias N. Rasmussen & Agustin
Roitman, "Oil Shocks in a Global Perspective: Are They Really That Bad?", IMF Working Paper, August 2011) mentions that
“Using a comprehensive global dataset […] we find that the impact of higher oil prices on oil-importing
economies is generally small: a 25 percent increase in oil prices typically causes GDP to fall by
about half of one percent or less.” The study elaborates on this by stating that this impact differs from one
country to another, depending on the size of oil-imports, as “oil price shocks are not always costly for oil-importing countries:
although higher oil prices increase the import bill, there are partly offsetting increases in external receipts [represented in new and
additional expenditures borne by both oil-exporting and oil-importing countries]”. In other words, the
more oil prices
increase, benefiting exporting countries, the more these new revenues are recycled , for
example through the growth in demand for new services, labor, and commodity imports. The
researchers argue that the series of oil price rallies (in 1983, 1996, 2005, and 2009) have played an
important role in recessions in the United States. However, Rasmussen and Roitman state at the same time
that significant changes in the U.S. economy in the previous period (the appearance of combined elements, such as
improvements in monetary policy, the institution of a labor market more flexible than before
and a relatively smaller usage of oil in the U.S. economy) has greatly mitigated the negative effects of
oil prices on the U.S. economy.
No impact - empirically proven

Joongang Daily ’11 (Joongang Daily in association with the International Herald Tribune, Kim Yeong-ook, Editorial writer of
the JoongAng Ilbo, “Lessons from oil shocks”, http://joongangdaily.joins.com/article/view.asp?aid=2932926, March 3, 2011,

The oilshocks in the 1970s provided valuable lessons for the global economy. They encouraged
energy awareness and efficiency in economies worldwide and spurred the race to develop
alternative energy sources. They built more resilience against oil turbulence . The recent spike had been
unannounced, but prices are still well below the 2008 record-high of $147 per barrel. There is no need to
raise a fuss over the possibility of a third global shock. The Lee Myung-bak administration is facing an energy crisis
while it is fumbling with the president’s signature platform of “green growth” through aggressive programs on new energy sources
and efficiency campaigns. The increase in prices may deal an immediate blow to the economy and consumer lives, but in
the
longer run it can serve as an impetus to push the economy toward more energy conservation
and a more clean-energy consuming structure. It may, in reality, be a valuable gift from the fall of Libyan
strongman Muammar el-Qaddafi.
---Oil Shocks- Backups solve

Backups check

Economist ‘11 (“The 2011 oil shock”, http://www.economist.com/node/18281774, March 3, 2011,

So far, theshocks to supply have been tiny. Libya’s turmoil has reduced global oil output by a
mere 1%. In 1973 the figure was around 7.5%. Today’s oil market also has plenty of buffers.
Governments have stockpiles, which they didn’t in 1973. Commercial oil stocks are more ample than
they were when prices peaked in 2008. Saudi Arabia, the central bank of the oil market, technically has enough spare
capacity to replace Libya, Algeria and a clutch of other small producers . And the Saudis have
made clear that they are willing to pump.

Reserves check- increases in prices just causes more to be tapped

Vaitheeswaran ‘7 (Vijay V. Vaitheeswaran 7 is correspondent for The Economist and coauthor, with Iain Carson, of Zoom: The
Global Race to Fuel the Car of the Future (New York: Twelve Books, 2007). Foreign Policy, November 2007,
http://www.foreignpolicy.com/

"The World Is Running Out of Oil")

Hardly. The world has more proven reserves of oil today than it did three decades ago, according
to official estimates. Despite years of oil guzzling and countless doomsday predictions, the
world is simply not
running out of oil. It is running into it. Oil is of course a nonrenewable resource and so, by definition, it will run dry
some day. But that day is not upon us, despite the fact that a growing chorus of "depletionists" argue that we've
already reached the global peak of oil production. Their view, however, imagines the global resource base in
oil as fixed, and technology as static. In fact, neither assumption is true. Innovative firms are
investing in ever better technologies for oil exploration and production, pushing back the oil
peak further and further. The key is understanding the role of scarcity, price signals, and future
technological innovation in bringing the world's vast remaining hydrocarbon reserves to
market. Thanks to advances in technology, the average global oil recovery rate from reservoirs has grown from about 20
percent for much of the 20th century to around 35 percent today. That is an admirable improvement. But it also means that
two thirds of the oil known to exist in any given reservoir is still left untapped. The best rebuttal
shale, tar
to the depletionists' case lies in the world's immense stores of "unconventional" hydrocarbons. These deposits of
sands, and heavy oil can be converted to fuel that could power today's ordinary automobiles.
Canada, for example, has deposits of tar sands with greater energy content than all the oil in Saudi Arabia. China, the United
States, Venezuela, and others also have large deposits of these energy sources. The problem is that the conversion comes at a
much greater environmental and economic cost than conventional crude oil. But the
very same high oil prices that
doomsters claim are a sign of imminent depletion also provide a powerful incentive for the
development of these mucky deposits-and for the technology that will allow us to extract them in a cleaner
fashion.
---Oil Shocks- No impact to econ

No oil shocks – economic impact would be minimal

Bolton ’12 (2/23/2012 (Alexander, The Hill staff reporter, http://thehill.com/homenews/administration/212239-possible-israeli-


strike-on-iran-poses-risks-for-us-economy-obama)

Sarah Emerson, president of Energy Security Analysis Inc., an independent research firm, said the attack
itself would not affect global oil supply but warned of Iran’s response. She noted that 15 percent to 20 percent of the
world’s oil output is shipped through the Strait of Hormuz, a narrow channel on Iran’s border. It’s an even higher percentage of the
oil in transit between countries. While Iran could not seize control of the strait indefinitely, it could effectively shut down traffic for
weeks until the U.S. Navy could establish safe passage. “I don’t think it can do anything but increase oil prices,” she said. But
Emerson said the economic impact of an oil shock would not be as severe as was experienced after
the Iraqi invasion of Kuwait and the oil embargo. “ We don’t use oil in many parts of the U.S.
economy anymore. Virtually none is used in electricity and heating,” she said. While transportation,
including airline, train and shipping traffic, still depends on oil to fuel it, she believes the nation can lower its
consumption by changing travel habits. The nation is also better positioned to soften the
economic blow by tapping the Strategic Petroleum Reserve, which did not exist in the late '70s
and which the president was reluctant to tap in the 1990s. “Now we have bigger reserves and we’re much more
willing to use them because there’s not as much spare capacity as we used to have,” she said, predicting an
announcement to release oil from the reserve would come “within days if there was a problem
with the strait.” The White House has been encouraged by a rising stock market, which could
improve the confidence of consumers seeing the balance of their retirement funds rise . Daniel
Alpert, managing partner of Westwood Capital, based in New York City, said an Israeli attack “would be disruptive to oil prices” and
that the stock market had not priced in that possibility. But Alpert said military action would not necessarily send the cost of energy
soaring. “If someone drops heavy ordnance on Iran, it’s going to send the world into a highly concerned state but it’s not going to
necessarily justify oil prices hitting new highs,” he said. Alpert believes excess liquidity caused by the European Central Bank’s effort
to avert a credit crisis is an equally notable factor pushing up oil prices. “You’re starting to see oil prices go up again. I think what
we’re going to see is a redux of what we saw last year at this time,” he said. Alpert said the Fed’s monetary stimulus last year, known
as QE2, sent up the prices of commodities, which inhibited economic growth. He said that is a reason he doubts the Fed will embark
on a third round of quantitative easing in the months ahead.

Status quo gas prices aren’t triggering consumer prices inflation

Lahart ‘11 (Justin, Wall Street Journal, “Fed Sees Recovery Picking Up”,
http://online.wsj.com/article/SB10001424052748704615504576172133517901722.html, March 1, 2011)

One of the Federal Reserve's top officials offered an upbeat assessment of the U.S. economy's
ability to weather the recent rise in oil prices, saying that the recovery is gaining momentum and policy
makers so far see no reason to curb their efforts to stimulate growth. "We may be much closer to establishing a
virtuous circle in which rising demand generates more rapid income and employment growth, which in turn bolsters confidence
and leads to further increases in spending," Federal Reserve Bank of New York President William Dudley said Monday at a
gathering at New York University's Stern School of Business. But he suggested the economy probably has a long way to grow
The comments from Mr. Dudley, who is vice chairman of the Fed
before inflation becomes a problem.
committee that sets short-term interest rates, come at a time when recent reports have shown stronger
consumer spending, rising manufacturing demand and improved confidence. The Commerce
Department reported Monday that personal income rose a full percentage point in January from December, as the cut in Social
Security taxes boosted paychecks. Consumer spending grew a muted 0.2%—likely the temporary effect of a bout of cold
weather. The missing piece has been stronger jobs growth. But improved labor-market indicators, such as a drop in the number
of people filing for unemployment claims, suggest "that employment growth will increase considerably more rapidly in the
coming months," said Mr. Dudley. Economists surveyed by Dow Jones expect Friday's report to show the economy added
200,000 jobs in February, as hiring rebounded after January's snowstorms. Still, it would take a substantial rise in employment
for inflation to become a problem, Mr. Dudley said. He estimated that the unemployment rate would have to slip down to
between 6% and 7% before workers could demand the kind of wage increases that would fuel worrisome rises in prices. That is
higher than the level of unemployment that has sparked inflation in the past, but still well below recent levels. Economists
estimate the unemployment rate edged up to 9.1% in February from 9% in January. Journal CommunityDISCUSS “The money
from the tax cuts mostly goes to rich people, who save it. Meanwhile, stagnant hourly wages and government cutbacks
eviscerate the incomes of others. Under these circumstances, why should anyone expect consumption to grow very much? ” —
Lawrence Beck Mr. Dudley noted, though, that the recent increase in energy and other commodity prices merits greater
vigilance. Unrest in the Middle East and North Africa has pushed oil prices higher this year, and recently those
increases have begun to show up in the price of gasoline. The Energy Information Administration reported that regular gasoline
averaged $3.38 a gallon at the pump Monday, up from $3.19 a week earlier. As it cuts into Americans' ability to buy other items,
the increase in fuel costs could slow the economy. Commodity prices could also create higher inflation, Mr. Dudley
said, if
they start to feed through into higher prices for other items. But he said that it was also important that the Fed not
overreact to the recent price increases, recognizing that some of them are likely to be temporary and
that commodities represent a relatively small part of overall U.S. inflation measures. Through
January, at least, inflation remained tame. The Commerce Department said that its "core" index of consumer
prices, which excludes food and energy, rose 0.1% from a month earlier, putting it up just 0.8%
compared with a year earlier. That's below the Fed's informal inflation target of just under
2%. Overall prices, including food and energy, rose 0.3% on the month , and were up 1.2% compared
to a year earlier.

Libya empirically denies their oil impacts

Wales Online ‘11 (British Newspaper, “Oil prices rising as Libya supply disrupted further by Gaddafi field attacks,” 2011,
http://www.walesonline.co.uk/business-in-wales/business-news/2011/04/09/oil-prices-rising-as-libya-supply-disrupted-further-by-
gaddafi-field-attacks-91466-28487117/,

THERE was more bad news on the commodity price front yesterday as oil prices hit a two-
and-a-half year high and new data showed factory-gate prices rising at the fastest rate since October 2008. Brent crude
has now hit $123.17 a barrel, while benchmark crude for May delivery was up 1% at $111.28 on the New York Mercantile
Exchange. The surge in oil prices is likely to be passed on at UK petrol pumps and add further pressure to already-squeezed
disposable incomes. Oil output slowed to a trickle as forces loyal to Muammar Gaddafi struck the
eastern Sarir and Messla fields. The two fields are in the massive Sirte Basin region, which holds roughly 80% of
Libya’s 46.4bn barrels in proven reserves of conventional crude oil.

No impact to oil spikes


Layne 6 (Christopher, Associate Professor of International Affairs – Texas A&M University, The Peace of Illusions: American
Grand Strategy from 1940 to the Present, p. 178-179)

Domestic instability in a major oil-producing state is another threat to U.S. interests in the Gulf. In the form of civil unrest,
instability could temporarily reduce the flow of oil from an affected country and drive up prices. However, because the oil
industry is globally integrated, other oil producers would increase their own production to
make up for the lost capacity. Thus, any spike in oil prices would be temporary, and lost
supplies would be replenished by other producers. In fact, past experience shows that this is
precisely what happens when internal instability in an oil-producing state causes a temporary
disruption in oil supplies.63 Instability in any of the Gulf oil producers, of course, could bring a hostile regime to power.
Here, there are two things to keep in mind. First, it is unlikely that U.S. military intervention could forestall such an event, and
indeed it might make things worse. Second, the economic consequences of such an event are exaggerated. In
an integrated, global oil market it is immaterial whether a hostile regime would sell oil directly to the United States. Because
oil is fungible, all that matters is that such a regime make its oil available to the market. The chances
of a hostile regime embargoing its oil are very low. All the major oil producers in the Gulf are economically dependent on their
oil revenues. Even if a hostile regime in the Gulf wanted to embargo oil shipments to the United States or the West, it could not
long do so without shooting itself in the foot economically. Moreover, if a hostile regime chose to behave in an economically
irrational fashion by sacrificing income to achieve political or economic objectives, markets would adjust. Higher oil
prices caused by an embargo would lead oil-consuming states like the United States to switch to
alternative energy sources and use energy more efficiently, and also provide an incentive for other oil-
producing states to increase the supply of oil in the market.64 Simply put, in relatively short order the supply-
demand equilibrium would return to the marketplace, and oil prices would return to their natural
marketplace level.

Shocks won’t collapse the economy

Dechaux 7 (Delphine, “Less Damage in Third Oil Shock”, Herald Sun Australia, 5-5, Lexis)

THE world is enduring a third ''oil shock'' as crude prices trade at record levels close to $US100 a barrel after a sustained surge
over the past three years, according to economists. But unlike the oil shocks of 1973 and 1980, this time the
global economy remains solid, even amid the added threat of the US housing crisis. ''There is no doubt we are in the
third oil price shock,'' said Leo Drollas, chief economist at the Centre for Global Energy Studies in London. ''Because since
2004 . . . prices have gone from $US30 to almost $US100.'' OPEC ministers meeting in Abu Dhabi today to decide on output
quotas for the cartel argue that the oil price spike does not reflect the supply-demand situation. Rather they believe prices have
surged because of geopolitical concerns, such as that over Iran's nuclear program. In the run-up to the 1980 oil shock prices had
more than doubled. Francois Lescaroux, an economist at IFP, a French state-run energy research body, said majority opinion
was that the first two oil shocks were due to supply factors. ''Everyone agrees this time that demand factors are pulling up
prices,'' he said. The oil price shock of 1973 occurred after Arab members of OPEC halted shipments of crude to the United
States, Western Europe and Japan for their perceived support of Israel in its battle against Syria and Egypt during the Yom
Kippur War. Following the oil embargo, the price of crude jumped above $US10 a barrel for the first time. The second oil crisis,
in 1979, followed the Iranian Revolution. By the start of 1981, oil prices had surged to $US39, which, adjusted for inflation, is the
equivalent of $US101 a barrel today. Yahia Said, a professor at the London School of Economics, said political unrest was a
common factor in all three oil shocks. ''In the first case, it was the Yom Kippur War of 1973, in the second case it was the Iraqi
invasion of Iran (after the Iranian Revolution). In this case it is tensions around Iraq and Iran,'' he added. ''The
shock this
time has not had the same negative repercussions in terms of inflation or in terms of recession. ''It
means that the
economies as the result of the previous two shocks have managed to reduce the
impact of high oil prices, especially in developed countries,'' he added.

Empirically proven

Nordhaus 7 (William, Professor of Economic – Yale University, “Who’s Afraid of a Big Bad Oil Shock”, September,
http://www.econ.yale.edu/~nordhaus/homepage/Big_Bad_Oil_Shock_Meeting.pdf)

When the U.S. invaded Iraq in March 2003, many economists feared that the war would lead to a sharp decline in Iraqi oil
production, a spike in oil prices, and a woeful economy that would follow the scripts of the oil shocks of 1973, 1978, and 1990.
There was in fact a moderate decline in world oil production, and real oilprices increased from $20 in 2001:4 to $62 in
2006:3. But the ailments associated with earlier oil-price increases did not appear. Instead, output continued
to grow rapidly, inflation was moderate, unemployment fell, consumers remained happy,
and productivity grew at close to its postwar norm.1 Macroeconomists would be out of business were there no
surprises. The business of this paper is to inquire into the explanations for the surprising oil non-crisis of the early 2000s. The
robustness of the economy following the latest oil shock can perhaps be seen in the context of one of the most important
developments in all of macroeconomics, the Great Moderation. This little-noticed theory holds that the economy has shown
much less volatility of inflation, unemployment, and output growth since World War II. (See particularly Stock and Watson
[2002].) Perhaps the moderated response of the economy to the latest oil shock is part of the declining volatility of the overall
economy. While much has been written about the macroeconomic impacts of price shocks, little analysis is available on the
impact of the most recent shock. The most comprehensive study is a preliminary paper by Blanchard and Gali [2007]. Most
research focuses on the role of monetary policy and inflation dynamics in the post-shock periods. There seems to be no
consensus as to whether the macroeconomic fear of oil-price shocks continues to be warranted.
Overpop Answers
Frontline

No population bomb- statistics, changing norms, and empirics all prove

Longman ’10 (Think Again: Global Aging A gray tsunami is sweeping the planet -- and not just in the places you expect. How did
the world get so old, so fast? BY PHILLIP LONGMAN | NOVEMBER 2010 Phillip Longman, a fellow at the New America Foundation
and the Washington Monthly, is author The Empty Cradle.

"The World Faces a Population Bomb."

Yes, but of old people. Not so long ago, we were warned that rising global population would
inevitably bring world famine. As Paul Ehrlich wrote apocalyptically in his 1968 worldwide bestseller, The
Population Bomb, "In the 1970s and 1980s hundreds of millions of people will starve to death in spite
of any crash programs embarked upon now . At this late date, nothing can prevent a substantial increase in the
world death rate." Obviously, Ehrlich's predicted holocaust , which assumed that the 1960s global
baby boom would continue until the world faced mass famine, didn't happen. Instead, the
global growth rate dropped from 2 percent in the mid-1960s to roughly half that today, with
many countries no longer producing enough babies to avoid falling populations. Having too
many people on the planet is no longer demographers' chief worry; now, having too few is. It's
true that the world's population overall will increase by roughly one-third over the next 40 years, from 6.9 to 9.1 billion, according to
the U.N. Population Division. But this will be a very different kind of population growth than ever before -- driven not by birth rates,
which have plummeted around the world, but primarily by an increase in the number of elderly people. Indeed, the global
population of children under 5 is expected to fall by 49 million as of midcentury, while the number of people over 60 will grow by 1.2
billion. How did the world grow so gray, so quickly? One reason is that more people are living to advanced old age. But just as
significant is the enormous bulge of people born in the first few decades after World War II. Both the United States and Western
Europe saw particularly dramatic increases in birth rates during the late 1940s and 1950s, as returning veterans made up for lost
time. In the 1960s and 1970s, much of the developing world also experienced a baby boom, but for a different reason: striking
declines in infant and child mortality. As these global baby boomers age, they will create a population explosion of seniors. Today in
the West, we are seeing a sharp uptick in people turning 60; in another 20 years, we'll see an explosion in the numbers turning 80.
Most of the rest of the world will follow the same course in the next few decades. Eventually ,
the last echoes of the
global baby boomers will fade away. Then, because of the continuing fall in birth rates, humans
will face the very real prospect that our numbers will fall as fast -- if not faster -- than the rate
at which they once grew . Russia's population is already 7 million below what it was in 1991. As for Japan, one expert has
calculated that the very last Japanese baby will be born in the year 2959, assuming the country's low fertility rate of 1.25 children
per woman continues unchanged. Young Austrian women now tell pollsters their ideal family size is less than two children, enough
to replace themselves but not their partners. Worldwide, there is a 50 percent chance that the population will be falling by 2070,
according to a recent study published in Nature. By 2150, according to one U.N. projection, the global
population could be half what it is today . That might sound like an appealing prospect: less traffic, more room at
the beach, easier college admissions. But be careful what you wish for. "Aging Is a Rich-Country Problem." NO. Once, demographers
believed, following a long line of ancient thinkers from Tacitus and Cicero in late Rome to Ibn Khaldun in the medieval Arab world,
that population aging and decline were particular traits of "civilized" countries that had obtained a high degree of luxury. Reflecting
on the fate of Rome, Charles Darwin's grandson bemoaned a pattern he saw throughout history: "Must civilization always lead to the
limitation of families and consequent decay and then replacement from barbaric sources, which in turn will go through the same
experience?" Today, however, wesee that birth rates are dipping below replacement levels even in
countries hardly known for luxury . Emerging first in Scandinavia in the 1970s, what the experts call
"subreplacement fertility" quickly spread to the rest of Europe, Russia, most of Asia, much of South
America, the Caribbean, Southern India, and even Middle Eastern countries like Lebanon, Morocco, and Iran. Of the
59 countries now producing fewer children than needed to sustain their populations, 18 are characterized by the United Nations as
"developing," i.e., not rich. Indeed, most developing countries are experiencing population aging at
unprecedented rates. Consider Iran. As recently as the late 1970s, the average Iranian woman had nearly seven children.
Today, for reasons not well understood, she has just 1.74, far below the average 2.1 children needed to sustain a population over
time. Accordingly, between 2010 and 2050, the share of Iran's population 60 and older is expected to increase from 7.1 to 28.1
percent. This is well above the share of 60-plus people found in Western Europe today and about the same percentage that is
expected for most Northern European countries in 2050. But unlike Western Europe, Iran and many other developing regions
experiencing the same hyper-aging -- from Cuba to Croatia, Lebanon to the Wallis and Futuna Islands -- will not necessarily have a
chance to get rich before they get old. One contributing factor is urbanization; more than half the world's population now lives in
cities, where children are an expensive economic liability, not another pair of hands to till fields or care for livestock. Two other oft-
cited reasons are expanded work opportunities for women and the increasing prevalence of pensions and other old-age financial
support that doesn't depend on having large numbers of children to finance retirement. Surprisingly, this graying of the world is not
by any means the exclusive result of programs deliberately aimed at population control. For though there are countries such as
India, which embraced population control even to the point of forced sterilization programs during the 1970s and saw dramatic
reduction in birth rates, there are also counterexamples such as Brazil, where the government never promoted family planning and
yet its birth rate went down even more. Why? In both countries and elsewhere, changing
cultural norms appear to be
the primary force driving down birth rates -- think TV, not government decrees. In Brazil, television was
introduced sequentially province by province, and in each new region the boob tube reached, birth rates plummeted soon after.
(Discuss among yourselves whether this was because of what's on Brazilian television -- mostly soap operas depicting rich people
living the high life -- or simply because a television was now on at night in many more bedrooms.)

No impact to overpop

Geracioti and Pilzer ’7 (David A. Geracioti, staff writer for Registered Rep, Paul Zane Pilzer, White House advisor and
environmentalist professor, “Fertile Human Minds, Fertile Human Earth,” July 1, LexisNexis)

Registered Rep: How did you come upon the concept of unlimited wealth, the idea you describe in your book God Wants You To
Be Rich Paul Zane Pilzer: I came of age in the 1970s. I graduated college in 1974, got an MBA from Wharton in 1976. And if you
came of age in the 1970s, why that was the age when we were running out of gasoline, we were running out of oil, we were
running out of natural resources. Whatever your parents had, halve it and you'll be lucky to get that, that was the feeling then.
RR: Scarcity has long been a central tenant of human activity. You're saying that's wrong? PZP: Economics is the study of
scarcity. And, when I was studying economics at Wharton, I was thinking, "What's scarce?" They told me land was scarce, and
that there was only a limited amount of food. Yet every year food productivity went up - by a factor of 300 times per acre from
1930 to 1980, creating effectively 300 times more land. RR: Back in the late 1700s and early 1800s, Malthus [the English
economist and demographer] predicted that hordes of people were going to starve to death: too many people, not enough
food. Who knew that fewer and fewer farmers could produce more and more food? PZP:
Yeah, Malthus looked at
the amount of farmland, saw the population going up, and said the population would be
controlled by the limited food supply. The worldwide population was almost one billion in
1800, when Malthus said it can't get any bigger, because the farms can only produce so much
food. And he was right at that time. What he didn't have a way of seeing was productivity
growth. The same farm 100 years ago was producing half as much food , and half as much
before that. And he couldn't foresee the real green revolution . Remember, that's what it
used to be called? RR: He didn't factor in technology . PZP: Yes, Malthus was wrong. His
mistake was that he held technology constant. He looked around at the farms and he didn't
have data that showed that farm productivity would far outstrip humans' ability to
reproduce - all the way to 6 and a half billion people today. RR: There must be an investing
lesson in there. PZP: From an investment standpoint, the classic mistake made throughout
history is holding technology constant. If you remember in the 1970s, we were running out
of oil, running out of copper, we were running out of all these things. But the pessimists
missed two things. One is that if we run out of a resource, we find cheaper ways to get it,
because prices start going up. Or, secondly, we invent something completely new outside the
industry that just completely displaces the "scarce" resource.

Population boom not root cause of impacts

Angus and Butler ‘11 (Ian Angus is coauthor of Too Many People? Population, Immigration, and the Environmental Crisis. He
is editor of the ecosocialist journal Climate and Capitalism. Simon Butler is coauthor of Too Many People? Population, Immigration,
and the Environmental Crisis. He is editor of Green Left Weekly, “Population crisis: blame the 1 per cent”
http://www.abc.net.au/unleashed/3610642.html)

But most of the 7 billion are not endangering the earth . The majority of the world's people don't
destroy forests, don't wipe out endangered species, don't pollute rivers and oceans, and emit essentially no
greenhouse gases. Even in the rich countries of the Global North, most environmental destruction is caused not by
individuals or households, but by mines, factories, and power plants run by corporations that care more about profit
than about humanity's survival. No reduction in US population would have stopped BP from poisoning the Gulf of
Mexico last year. Lower birth rates won't shut down Canada's tar sands, which Bill McKibben has justly called one of the most
staggering crimes the world has ever seen. Universal
access to birth control should be a fundamental human
right - but it would not have prevented Shell's massive destruction of ecosystems in the Niger River
delta, or the immeasurable damage that Chevron has caused to rainforests in Ecuador. Ironically, while populationist groups focus
attention on the 7 billion, protestors in the worldwide Occupy movement have identified the real source of environmental
destruction: not the 7 billion, but the 1 per cen t, the handful of millionaires and billionaires who own more, consume
more, control more, and destroy more than all the rest of us put together.

No population crunch – we adapt

Goklany 10, policy analyst for the Department of the Interior – phd from MSU, “Population, Consumption, Carbon Emissions,
and Human Well-Being in the Age of Industrialization (Indur, Part IV – There Are No PAT Answers, or Why Neo-Malthusians Get It
Wrong”, April 26, http://www.masterresource.org/2010/04/population-consumption-carbon-emissions-and-human-well-being-in-
the-age-of-industrialization-part-iv-there-are-no-pat-answers-or-why-neo-malthusians-get-it-wrong/)

Moreover, fears
that the world’s population would continue to increase exponentially have failed
to materialize. The world’s population growth rate peaked in the late 1960s . Population increased by
10.6% from 1965–70, but only 6.0% from 2000–05. Many countries are now concerned that fewer young people means that their
social security systems are unsustainable. Projections nowsuggest that the world’s population may peak at
around 9 billion around mid-century (see here). The slowdown in the population growth rate,
unanticipated by Neo-Malthusians, can be attributed to the fact that population (P) is dependent
on affluence (or the desire for affluence) and technology (A and T in the IPAT equation). Empirical data show that
as people get wealthier or desire greater wealth for themselves or their offspring, they tend to
have fewer children. Cross-country data shows that the total fertility rate (TFR), which measures the number of
children per women of child-bearing age, drops as affluence (measured by GDP per capita) increases (see Figure 1).
Moreover, for any given level of affluence, TFR has generally dropped over time because of changes
in technology, and societal attitudes shaped by the desire for economic development (see here).
Most importantly, it is not, contrary to Neo-Malthusian fears, doomed to rise inexorably , absent coercive
policies. Neo-Malthusians also overlook the fact that, in general, affluence, technology and human well-
being reinforce each other in a Cycle of Progress (Goklany 2007a, pp. 79-97). If existing technologies
are unable to reduce impacts or otherwise improve the quality of life, wealth and human capital
can be harnessed to improve existing technologies or create new ones that will . HIV/AIDS is a case in
point. The world was unprepared to deal with HIV/AIDS when it first appeared. For practical purposes, it was a death sentence for
anyone who got it. It took the wealth of the most developed countries to harness the human capital to develop an understanding of
the disease and devise therapies. From 1995 to 2004, age-adjusted death rates due to HIV declined by over 70 percent in the US
(USBC 2008). Rich countries now cope with it, and developing countries are benefiting from the technologies that the former
developed through the application of economic and human resources, and institutions at their disposal. Moreover, both technology
and affluence are necessary because while technology provides the methods to reduce problems afflicting humanity, including
environmental problems, affluence provides the means to research, develop and afford the necessary
technologies. Not surprisingly, access to HIV therapies is greater in developed countries than in developing countries. And in
many developing countries access would be even lower but for wealthy charities and governments from rich countries (Goklany
2007a, pp. 79–97). Because technology is largely based on accretion of knowledge, it ought to advance with time, independent of
affluence — provided society is open to scientific and technological inquiry and does not squelch technological change for whatever
reason. Consequently, indicators of human well-being improve not only with affluence but also with
time (a surrogate for technology). This is evident in Figure 1, which shows TFR dropping with time for any specific level of GDP per
capita. It is also illustrated in Figure 2 for life expectancy, which shows that wealthier societies have higher average life expectancies,
and that the entire life expectancy curve has been raised upward with the passage of time, a
surrogate for technological change (broadly defined). Other indicators of human well-being — e.g., crop
yield, food supplies per capita, access to safe water and sanitation, literacy, mortality — also improve with affluence
and, separately, with time/technology (see here and here). This indicates that secular technological
change and economic development, rather than making matters worse, have actually enhanced
society’s ability to solve its problems and advanced its quality of life. Moreover, population is not
just a factor in consumption. It is the basis for “human capital.” No humans, no human capital .
Humans are not just mouths, but also hands and brains . As famously noted by Julian Simon, they are the
Ultimate Resource. This is something Neo-Malthusians have difficulty in comprehending . Notably, a
World Bank study, Where is the Wealth of Nations?, indicated that “human capital and the value of institutions … constitute the
largest share of wealth in virtually all countries.” A population that is poor, with low human capital, low affluence, and lacking in
technological knowhow is more likely to have higher mortality rates, and lower life expectancy than a population that is well
educated, affluent and technologically sophisticated, no matter what its size. These
factors — human capital,
affluence and technology — acting in concert over the long haul, have enabled technology for
the most part to improve matters faster than any deterioration due to population, affluence (GDP
per person) or their product (GDP). This has helped keep environmental damage in check , (e.g., for
cropland, a measure of habitat converted to human uses) or even reverse it (e.g., for water pollution, and indoor and
traditional outdoor air pollution), particularly in the richer countries. Note that since the product of population (P) and affluence (A
or GDP per capita) is equivalent to the GDP then according to the IPAT identity, which specifies that I = P x A x T, the technology term
(T) is by definition the impact (I) per GDP (see Part II in this series of posts). I’ll call this the impact intensity. If the impact is specified
in terms of emissions, then the technology term is equivalent to the emissions intensity, that is, emissions per GDP. Therefore the
change in impact intensity (or emissions intensity) over a specified period is a measure of technological change over that period.
Since matters improve if impact/emissions intensity drops, a negative sign in front of the change in impact intensity denotes that
technological change has reduced the impact. Table 1 shows estimates of the changes in impacts intensity, or technological change,
over the long term for a sample of environmental indicators for various time periods and geographical aggregations. Additional
results regarding technological change over different time periods and countries are available from the original source (here). These
results indicate that in
the long run, technological change has, more often than not, reduced impacts.
The reduction in many cases is by an order of magnitude or more ! Thus, notwithstanding plausible Neo-
Malthusian arguments that technological change would eventually increase environmental impacts, historical data suggest
that, in fact, technological change ultimately reduces impacts , provided technology is not rejected through an
inappropriate exercise of the precautionary principle or compromised via subsidies (which usually flow from the general public to
politically favored elements of society). To summarize, although
population, affluence and technology can
create some problems for humanity and the planet, they are also the agents for solving these
very problems. In the IPAT equation, the dependence of the I term on the P, A and T terms is not
fixed. It evolves over time. And the Neo-Malthusian mistake has been to assume that the
relationship is fixed, or if it is not, then it changes for the worse . A corollary to this is that projections of
future impacts spanning a few decades but which do not account for technological change as a
function of time and affluence, more likely than not, will overestimate impacts, perhaps by
orders of magnitude. In fact, this is one reason why many estimates of the future impacts of
climate change are suspect, because most do not account for changes in adaptive capacity
either due to secular technological change or increases in economic development (see here and here).
Famously, Yogi Berra is supposed to have said, “It’s tough to make predictions, especially about the future.” Most analysts recognize
this. They know that just because one can explain and hindcast the past, it does not guarantee that one can forecast the future.
Neo-Malthusians, by contrast, cannot hindcast the past but are confident they can forecast the
future. Finally, had the solutions they espouse been put into effect a couple of centuries ago, most of us alive today would be
dead and those who were not would be living poorer, shorter, and unhealthier lives, constantly subject to the vagaries of nature,
surviving from harvest to harvest, spending more of our time in darkness because lighting would be a luxury, and our days in the
drudgery of menial tasks because under their skewed application of the precautionary principle (see here, here and here) fossil fuel
consumption would be severely curtailed, if not banned. Nor would the rest of nature necessarily be better off. First, lower
reliance on fossil fuels would mean greater demand for fuelwood, and the forests would be
denuded. Second, less fossil fuels also means less fertilizer and pesticides and , therefore, lower
agricultural productivity. To compensate for lost productivity,, more habitat would need to be
converted to agricultural uses. But habitat conversion (including deforestation) — not climate change —
is already the greatest threat to biodiversity !

Not approaching carrying capacity, best studies prove

Lombog ‘1 (Bjorn Lomborg, Ph.D, University of Aarhus, Denmark, 2001 (August 2, "The Truth About the Environment", The
Economist, http://www.klamathbucketbrigade.org/Lomborg_TheTruthAboutTheEnvironment062005.htm)

The trouble is, the evidence does not back up this litany. First, energy and other natural
resources have become more abundant, not less so since the Club of Rome published “The Limits to Growth” in
1972. Second, more food is now produced per head of the world's population than at any time
in history. Fewer people are starving. Third, although species are indeed becoming extinct,
only about 0.7% of them are expected to disappear in the next 50 years, not 25-50%, as has so often been
predicted. And finally, most forms of environmental pollution either appear to have been
exaggerated, or are transient—associated with the early phases of industrialisation and therefore best cured not by
restricting economic growth, but by accelerating it. One form of pollution—the release of greenhouse gases that causes global
warming—does appear to be a long-term phenomenon, but its total impact is unlikely to pose a devastating problem for the
future of humanity. A bigger problem may well turn out to be an inappropriate response to it. Take these four points one by
one. First, the exhaustion of natural resources. The early environmental movement worried that the mineral resources on which
modern industry depends would run out. Clearly, there
must be some limit to the amount of fossil fuels
and metal ores that can be extracted from the earth: the planet, after all, has a finite mass. But that limit is
far greater than many environmentalists would have people believe. Reserves of natural
resources have to be located, a process that costs money. That, not natural scarcity, is the main limit on
their availability. However, known reserves of all fossil fuels, and of most commercially important
metals, are now larger than they were when “The Limits to Growth” was published. In the case of oil, for example,
reserves that could be extracted at reasonably competitive prices would keep the world economy running for about 150 years
at present consumption rates. Add to that the fact that the price of solar energy has fallen by half in
every decade for the past 30 years, and appears likely to continue to do so into the future, and
energy shortages do not look like a serious threat either to the economy or to the
environment. The development for non-fuel resources has been similar . Cement, aluminium,
iron, copper, gold, nitrogen and zinc account for more than 75% of global expenditure on raw
materials. Despite an increase in consumption of these materials of between two- and ten-fold over the past 50
years, the number of years of available reserves has actually grown. Moreover, the increasing
abundance is reflected in an ever-decreasing price: The Economist's index of prices of industrial raw materials
has dropped some 80% in inflation-adjusted terms since 1845. Next, the population explosion is also turning out
to be a bugaboo. In 1968, Dr Ehrlich predicted in his best selling book, “The Population Bomb”, that “the battle to feed
humanity is over. In the course of the 1970s the world will experience starvation of tragic proportions—hundreds of millions of
people will starve to death.” That did not happen. Instead, according to the United Nations, agricultural
production in
the developing world has increased by 52% per person since 1961. The daily food intake in poor
countries has increased from 1,932 calories, barely enough for survival, in 1961 to 2,650 calories in 1998, and is
expected to rise to 3,020 by 2030. Likewise, the proportion of people in developing countries who are starving has
dropped from 45% in 1949 to 18% today, and is expected to decline even further to 12% in 2010 and just 6% in 2030. Food, in
other words, is becoming not scarcer but ever more abundant. This is reflected in its price. Since 1800 food prices have
decreased by more than 90%, and in 2000, according to the World Bank, prices were lower than ever before. Dr Ehrlich's
prediction echoed that made 170 years earlier by Thomas Malthus. Malthus claimed that, if unchecked, human population
would expand exponentially, while food production could increase only linearly, by bringing new land into cultivation. He was
wrong. Population growth has turned out to have an internal check: as people grow richer and healthier, they have
smaller families. Indeed, the growth rate of the human population reached its peak , of more
than 2% a year, in the early 1960s. The rate of increase has been declining ever since . It is now
1.26%, and is expected to fall to 0.46% in 2050 . The United Nations estimates that most of the world's
population growth will be over by 2100, with the population stabilising at just below 11 billion (see chart 1). Malthus also
failed to take account of developments in agricultural technology. These have squeezed
more and more food out of each hectare of land . It is this application of human ingenuity that has boosted
food production, not merely in line with, but ahead of, population growth. It has also, incidentally, reduced the need to take
new land into cultivation, thus reducing the pressure on biodiversity.
---Overpop- Squo Solves

World is underpopulated now- their evidence is hype

Eberstadt ‘1 (Nicholas Eberstadt, Henry Wendt Chair in Political Economy at the American Enterprise Institute. This article is
adapted from a longer one in the current issue of Foreign Policy magazine, “Underpopulation, Not Overpopulation, The Real Global
Problem”, http://www.abortiontv.com/Lies%20&%20Myths/underpopulation.htm, March 18, 2001)

It may not be the first way we think of ourselves, but all of us alive today are children of the "world population explosion."
Thanks to sweeping mortality declines, human numbers leapt from about 1.6 billion or 1.7 billion in 1900 to more than 6 billion
in 2000.In certain circles within Washington (and outside the United States), that unprecedented leap in
human numbers fueled an anti-natalist obsession. But continuing preoccupation with high
fertility and rapid population growth leaves us poorly prepared to comprehend (much less respond
to) emerging demographic trends. Three of these are poised to refigure our global profile in surprising -- and not
always beneficial -- ways. The first is the spread of "sub-replacement" fertility regimens: patterns of
childbearing that will eventually result, all else being equal, in indefinite population decline.
According to the U.S. Census Bureau, 83 countries and territories are now thought to experience
below-replacement fertility. Those places encompass nearly 2.7 billion people -- roughly 44 percent
of the world's total population. Today's global march toward smaller family size flies in the face of many prevailing
assumptions about when rapid fertility decline can and cannot occur. Poverty and illiteracy (especially female illiteracy) are
widely regarded as impediments to fertility decline, yet they have not prevented Bangladesh from reducing its fertility rate by
more than half over the past quarter-century. By the same token, "traditional" religious attitudes are commonly seen as a
barrier against low fertility. Yet over the past two decades, Iran, under the tight rule of a militantly Islamic clerisy, has slashed its
fertility level by fully two-thirds, and now apparently it stands on the verge of sub-replacement. What accounts for the
worldwide plunge in fertility? The honest answer is that nobody really knows -- at least, with any degree of confidence. If you
can find the shared determinants of fertility decline in such disparate below-replacement societies as the United States,
Guadeloupe, Thailand and Tunisia, then your Nobel Prize is in the mail. While causes might be uncertain, results are quite
predictable. Global
population growth will decelerate markedly over the coming generation.
By current projections, in fact, slightly fewer babies will be born worldwide in the year 2025
than at any point over the previous four decades. Thanks to extreme birth dearth,
depopulation is now imminent for both Europe and Japan . In Europe, immigration must nearly
quadruple -- to an average of almost 4 million net entrants a year -- to prevent a decline in the size of the 15- to-64-year-old
"working age" population over the next 50 years. In Japan, where net immigration approximates zero, more than 600,000
newcomers a year will be needed to keep the working age population from shrinking. Will these territories opt for indefinite
decline -- or for ethnic transformation? Given the arithmetic, they have no other options. Low and decreasing
fertility levels will accelerate the tempo of social aging -- the second great demographic
trend of the coming era. We all know about the coming pensioner problem in Western countries -- but Western
countries are rich. Many of today's developing countries, by contrast, will become "gray" before they become "rich." One of
the most arresting cases of population aging is now set to unfold in China. Between 2000 and
2025, China's median age will soar -- in fact, it may exceed America's within 25 years. By 2025, roughly 200 million
Chinese will be 65 or older. Caring for China's elderly will inexorably become a domestic, and global, political issue --
for nothing remotely resembling a national pension system is yet in place in that country. The third, and most
ominous, demographic trend of the coming era involves unexpected and brutal mortality
spikes. In our era, we have come to presume that death rates inevitably decline during times of peace and order. That happy
presumption must now be discarded. By Census Bureau projections, nearly 40 countries and territories will
have lower life expectancies in 2010 than they enjoyed in 1990. More than 750 million people -- one-
sixth of the world's current population -- live in such spots. Many of these countries are today's sub-Saharan victims of the HIV-
AIDS epidemic. But the international health setback is not just about Africa and AIDS. In Russia -- an urbanized, industrialized,
peacetime society -- lifespans are shorter today than 40 years ago. In a dozen other post-Communist
countries, life expectancy is lower today than in the 1970s. Since virtually no one predicted these
foreshortenings of national lifespan, we cannot yet claim to know which countries will be afflicted by -- or spared from --
uncontrollable bouts of mortality in the years to come. Before
too long, unfortunately, our current era's widespread
anxiety about health-driven global population growth may look remarkably quaint and
naive.

Squo solves O-Pop

Friedman 10 (George, Founder and CEO of STRATFOR, the World’s Leading Private Intelligence and Forecasting Company,
Media Expert, “The Next 100 Years,” January, Originally Published January 27 th 2009, p.8)

But underlying all of this will be the single most important fact of the twenty-first century: the
end of the population
explosion. By 2050, advanced industrial countries will be losing population at a dramatic
rate. By 2100, even the most underdeveloped countries will have reached birthrates that
will stabilize their populations. The entire global system has been built since 1750 on the
expectation of continually expanding populations. More workers, more consumers, more soldiers - this was
always the expectation. In the twenty-first century, however, that will cease to be true. The entire
system of production will shift. The shift will force the world into a greater dependence on technology - particularly
robots that will substitute for human labor, and intensified genetic research (not so much for the purpose of
extending life but to make people productive longer).

No crunch - new evidence proves

Wattenberg 2k (Ben, Senior Fellow at the American Enterprise Institute, “Doomsay for Doomsaying,” 1-1,
http://www.aei.org/publications/pubID.16039/pub_detail.asp)

A long-awaited world crisis of overpopulation is not developing . Indeed, declining population looms as a
problem for many countries. Finally, after all these years of demographic doomsaying , population
proliferationism, and exponential extrapolated explosionism, comes a new report from the United Nations and a
headline in the New York Times: "World Is Less Crowded Than Expected." Really? Than expected by whom? Than expected by
whom--when? Apparently, not expected when it should have been expected by Joseph Chamie, director of the United Nations
Population Division, who is quoted thusly in the Times story: "We had some glimmer that this was occurring several years ago,
but we weren't sure if it was simply a blip. Now we actually have concrete results showing
this is a global trend."
Several years ago? A glimmer? A blip? Actually, global
fertility and birth rates have been declining--
rapidly--beyond expectations--for more than a quarter of a century! The United Nations' own
population data show that worldwide fertility rates were at a stratospheric 5.0-children-per-woman level in the 1950 to 1955
period. Fifteen years later, from 1965 to 1970, the level was still 4.9 children per woman. The cry of the Population Explosionists
reached full throat. After all, it only takes a little more than two children per woman to merely "replace" a society at a long-term
stable rate. Five children per woman would end up, it was said, yielding a global population of 10 billion, no, 12 billion, no, 15
billion people--even 20 billion people! (Such were the estimates, from establishment organizations and scare groups alike.)
Then fertility began falling like a heavy stone, and it has continued to fall right through the
1990 to 1995 period, as reported in the United Nations' new "World Population Prospects: The 1996 Revisions." The
current global total fertility rate is 2.96 children per woman. Consider what that means: In just the last twenty-five years, the
world has gone from about five kids per woman to about three per woman, which is
roughly two-thirds of the way to demographic stability ! (Fertility in Bangladesh dropped from 6.2 children
in 1980 to 1985, to 3.4 in 1990 to 1995.) But there is no particular reason to think that global fertility will
stop at the 2.1-children-per-woman "replacement rate." It is not a magic number. The new data in from
Europe are somewhere between astounding and terrifying. From 1990 to 1995, from an already low below-replacement base,
37 of the 39 European nations have seen further reductions in fertility rates. (The demographic behemoths of Luxembourg and
Finland had minuscule increases, but remain in the 1.7 to 1.8 range, about 15 percent to 20 percent below the rate needed to
maintain stability.) Italy, Germany, and Spain have rates in the 1.2 to 1.3 range. At that level, according to one estimate, Italy's
population would decrease from about 57 million today to about 37 million by the year 2030, and continue to fall rapidly.
Moreover, just about every developed country in the world has a below-replacement fertility
rate (including the United States, with an estimated rate of 1.97 for 1996 that has fallen for six straight years). There are
already twenty-seven countries officially designated as "developing" that have below-replacement rates. Nothing like this has
happened before in history. But neither the United Nations nor the most important population and environmental
organizations choose to stress this condition. The United Nations chooses a 2.1 global rate in the year 2050 and labels it
"medium-variant," which is commonly interpreted to mean "most likely." That yields a population stabilizing at near 11 billion
people, roughly doubling from the 5.7 billion people today. Forget it. It's not going to happen. The United Nations' "low-
variant" assumption is closer to the mark and ends up with a population that tops out at
about 7.7 billion in 2040, and then sinks. Why such ongoing, systemic distortion? There are lots of answers bruited
about. Popucrats need their budgets for legitimate family-planning services; apocalypse-mongering is seen as a way to get such
budgets. Politics plays a role; proud nations do not want to be seen as shrinking. Environmentalists
live off of
crises; diminishing long-term population takes the air out of the panic balloon . Feminists don't
like the whole idea of a birth dearth. It's reached a point where most everyone believes in a population crisis. But you needn't.
---Overpop- No Impact

No impact – multiple countries prove

Roney 1 (Stephen, Staff Writer – News Magainze, 12-17, Ebsco)

World population could rise, on present trends, from 6 billion to 10.9 billion by 2050--in the worst-
case. But UN "worst-case" population scenarios have so far always been wrong . A study this
summer in the journal Nature projects a peak world population below 9 billion. And, even in the UN's projections,
after mid-century it will decline. "Growth," Nature notes, will probably end "in the foreseeable
future." In fact the end has begun, in the developed world. The German population could drop 20% by 2050. Italy will need
2.2 million immigrants a year to hold population steady. Japan will be down between 17% and 25% on current trends. Every
country in the developed world is now below replacement level. So too the Second World. Russia may shrink from 147 million to
below 115 million in 15 years. Central and Eastern Europe, on the UN's own figures, may drop by one-third by 2050. Which
leaves the Third World. But fertility rates are falling everywhere, even there: worldwide, they have dropped from six per woman
in the 1960s to 2.8 today, and are still dropping. China and Brazil are now below replacement level. Elsewhere, AIDS is suddenly
altering population trends. In South Africa, one-quarter of young women and one in five university students have HIV. In
Botswana, the figure is 36%. Some see sub-Saharan Africa "becoming depopulated within decades." HIV in China is spreading at
30% to 60% per annum. We humans may not be an endangered species yet, but "population explosion" hardly seems the
proper term. The UN's own figures, in sum, suggest a population decline as great as 25% in each generation, worldwide, from
2050 or a bit beyond. That sounds more like an implosion. So, even
given the "worst-case" brief population
peak of 10.9 billion, will the world starve? I expect not. If crowding causes starvation,
famine reports should be coming now from the Netherlands, South Korea and Taiwan--
three of the world's four most densely populated nations . Yet the densest populations are, usually, the
richest. Developed western Europe has a higher population density (429 per square mile) than poor Africa (65), Latin America
(66) or Asia (303). Japan is denser than China or India.

No overshoot

Hugh 6 (Edward, “Rethinking the Demographic Transition”,


http://www.edwardhugh.net/rethinking_the_demographic_.pdf)

In the short-run there is no real global problem. The population range in 2050 will be only
from 7.4 to 10.6 billion, numbers which at first can certainly be fed and, given proper safeguards, are
not likely to cause a major upset to such global systems as the atmosphere.

No impact

Hartmann 95 (Betsy, Director of the Population and Development Program – Hampshire College, Reproductive Rights and
Wrongs: The Global Politics of Population Control, p. 36)
The relaxation of the Malthusian position was reflected in a 198 U.S.National Academy of Sciences report,
which retreated substantially from past alarmist assessments of population growth . While
concluding that population growth was more likely to impede progress than promote it, the report found that it was
not the unmitigated environmental and economic evil it had been portrayed to be . According
to the report, there is no "necessary relation" between population growth and resource
exhaustion, and the effect of population growth on the economy is mixed . Even when population
growth has a negative impact, slower growth alone will not guarantee progress?

Markets solve

Kahl 2 (Colin, PhD, Foreign Affairs Expert at Georgetown University, “International Affairs,” Fall, p.257)

Neo-classical arguments related to resource scarcity have substantial merit. After centuries of debate, it is clear that traditional
Malthusian claims about the inevitable relationship between population growth and
resource scarcity have proven false. Indeed, as Birdsall and Sinding, two scholars
sympathetic with the neo-Malthusian view, note, “[t]he effects of markets and institutions
—sometimes good, sometimes bad—can easily swamp the effect of population change on
resource use, degradation, and depletion.” Nevertheless, neo-classical economists tend to
be overly optimistic about the prospects for adaptation. Indeed, while markets and
institutions have frequently adapted to DES-induced pressures at the global level and within wealthy
industrialized countries, serious local scarcities continue to emerge within developing countries. Moreover, and somewhat
ironically, adaptation has been much more successful in heading off shortages of non-renewable resources (e.g., oil and other
minerals) than renewable ones.
Ozone Answers
Ozone is recovering

Science Daily 6 (Science Daily Magazine, Citing a NASA Research Report and Eun-Su Yang, Professor at Georgia Institute of
Technology, “Earth’s Ozone Layer Appears to be on the Road to Recovery,” May 27,
http://www.sciencedaily.com/releases/2006/05/060527093645.htm)

People were understandably alarmed, then, in the 1980s when scientists noticed that
manmade chemicals in the atmosphere were destroying this layer . Governments quickly enacted an
international treaty, called the Montreal Protocol, to ban ozone-destroying gases such as CFCs then found in aerosol cans and
air conditioners. Today, almost 20 years later, reports continue of large ozone holes opening over Antarctica, allowing
dangerous UV rays through to Earth's surface. Indeed, the 2005 ozone hole was one of the biggest ever, spanning 24 million sq
km in area, nearly the size of North America. Listening to this news, you might suppose that little progress has been made. You'd
be wrong. While the ozone hole over Antarctica continues to open wide, the
ozone layer around the rest of the
planet seems to be on the mend. For the last 9 years, worldwide ozone has remained
roughly constant, halting the decline first noticed in the 1980s. The question is why? Is the Montreal
Protocol responsible? Or is some other process at work? It's a complicated question. CFCs are not the only things that can
influence the ozone layer; sunspots, volcanoes and weather also play a role. Ultraviolet rays from sunspots boost the ozone
layer, while sulfurous gases emitted by some volcanoes can weaken it. Cold air in the stratosphere can either weaken or boost
the ozone layer, depending on altitude and latitude. These processes and others are laid out in a review just published in the
May 4th issue of Nature: "The search for signs of recovery of the ozone layer" by Elizabeth Westhead and Signe Andersen.
Sorting out cause and effect is difficult, but a group of NASA and university researchers may have made some headway. Their
new study, entitled "Attribution of recovery in lower-stratospheric ozone," was just accepted for publication in the Journal of
Geophysical Research. It concludes that about
half of the recent trend is due to CFC reductions . Lead
author Eun-Su Yang of the Georgia Institute of Technology explains: "We
measured ozone concentrations at
different altitudes using satellites, balloons and instruments on the ground. Then we
compared our measurements with computer predictions of ozone recovery, [calculated
from real, measured reductions in CFCs]." Their calculations took into account the known
behavior of the sunspot cycle (which peaked in 2001), seasonal changes in the ozone layer,
and Quasi-Biennial Oscillations, a type of stratospheric wind pattern known to affect
ozone. What they found is both good news and a puzzle. The good news: In the upper stratosphere (above roughly 18 km),
ozone recovery can be explained almost entirely by CFC reductions. "Up there, the Montreal Protocol seems to be working,"
says co-author Mike Newchurch of the Global Hydrology and Climate Center in Huntsville, Alabama. The puzzle: In the lower
stratosphere (between 10 and 18 km) ozone has recovered even better than changes in CFCs alone would predict. Something
else must be affecting the trend at these lower altitudes. The "something else" could be atmospheric wind patterns. "Winds
carry ozone from the equator where it is made to higher latitudes where it is destroyed. Changing wind patterns affect the
balance of ozone and could be boosting the recovery below 18 km," says Newchurch. This explanation seems to offer the best
fit to the computer model of Yang et al. The jury is still out, however; other sources of natural or manmade variability may yet
prove to be the cause of the lower-stratosphere's bonus ozone. Whatever the explanation, if
the trend continues, the
global ozone layer should be restored to 1980 levels sometime between 2030 and 2070. By
then even the Antarctic ozone hole might close--for good.

Ozone stable – and no impact

Lieberman 7 (Ben, Senior Policy Analyst – Heritage Foundation, “Ozone: The Hole Truth”, The Washington Times, 9-19, Lexis)
Environmentalists have made many apocalyptic predictions over the last several decades.
Virtually none has come to pass. Yet each time, the greens and their political allies proclaim victory, arguing their
preventive prescriptions averted disaster. Such is the case with the 1987 Montreal Protocol On Substances That Deplete The
Ozone Layer (Montreal Protocol). The luridpredictions of ozone depletion -induced skin cancer epidemics,
ecosystem destruction and others haven't come true, for which Montreal Protocol proponents congratulate themselves.
But in retrospect, the evidence shows ozone depletion was an exaggerated threat in the first place. As the treaty
parties return to Montreal for their 20th anniversary meeting it should be cause for reflection, not celebration, especially for
those who hope to repeat this "success story" in the context of global warming. The treaty came about over legitimate but
overstated concerns that chlorofluorocarbons (CFCs, a then-widely used class of refrigerants) and other compounds were rising
to the stratosphere and destroying ozone molecules. These molecules, collectively known as the ozone layer, shield the Earth
from excessive ultraviolet-B radiation (UVB) from the sun. The Montreal Protocol's provisions were tightened in 1990 and again
in 1992, culminating with a CFC ban in most developed nations by 1996. So what do we know now? As far as ozone depletion is
concerned, the thinning of the ozone layer that occurred throughout the 1980s apparently stopped in the
early 1990s, too soon to credit the Montreal Protocol. A 1998 World Meteorological Organization (WMO) report said: "Since
1991, the linear [downward] trend observed during the 1980s has not continued, but rather total column ozone has been
almost constant." However, the same report noted that the stratospheric concentrations of the offending compounds were
still increasing through 1998. This lends credence to the skeptical view, widely derided at the time of the Montreal Protocol,
that naturalvariations better explain the fluctuations in the global ozone layer. More importantly, the
feared increase in ground level UVB radiation has also failed to materialize. Keep in mind that ozone
depletion, in and of itself, doesn't really harm human health or the environment. It was the concern that an
eroded ozone layer will allow more of the sun's damaging UVB rays to reach the Earth that led to the Montreal Protocol. But
WMO concedes no statistically significant long-term trends have been detected , noting earlier this year
that "outside the polar regions, ozone depletion has been relatively small, hence, in many places, increases in UV due to this
depletion are difficult to separate from the increases caused by other factors, such as changes in cloud and aerosol." In short,
the impact of ozone depletion on UVB over populated regions is so small it's hard to detect. Needless to
say, if UVB hasn't gone up, then the fears of increased UVB-induced harm are unfounded. Indeed, the much-hyped acceleration
in skin cancer rates hasn't been documented. U.S. National Cancer Institute statistics show malignant melanoma incidence and
mortality, which had been undergoing a long-term increase that predates ozone depletion, has actually been leveling off during
the putative ozone crisis. Further, no
ecosystem or species was ever shown to be seriously harmed
by ozone depletion. This is true even in Antarctica, where the largest seasonal ozone losses, the so-called Antarctic
ozone hole, occur annually. Also forgotten is a long list of truly ridiculous claims, such as the one from Al Gore's 1992 book
"Earth in the Balance" that, thanks to the Antarctic ozone hole, "hunters now report finding blind rabbits; fisherman catch blind
salmon."

Natural variation determines ozone

Carnacchio 97 (CJ, Staff – The Review, “The Sky Falls on Environmental Myths”, Michigan Review, 10-8,
http://www.umich.edu/~mrev/archives/1997/10-8-97/environment.htm)

Myth #2: The Hole in the Ozone Layer: Contrary to the environmentalists' claims, there
is no permanent hole in the
ozone layer and no ozone shortage. Ozone is constantly created and destroyed. The interaction of
ultraviolet radiation with oxygen molecules is what produces ozone. In the stratosphere, 10 to 40 kilometers above the earth's
surface, several tons of ozone are produced every second. The amount of ozone present at any one time is influenced
by many factors. For example, the amount of ultraviolet radiation reaching the stratosphere (and ultimately
producing ozone) depends upon latitude, solar cycle, and season. Concentrations of ozone may
differ drastically from one day to the next, sometimes by as much as 50 percent, depending on the
weather. Ozone holes are natural reactions to these ultraviolet light variations. Ozone levels can also be
affected by the amount of volcanic matter in the stratosphere. Each volcanic eruption emits roughly a thousand times the
amount of ozone-depleting chemicals than all the CFCs man has ever produced. The ozone hole that appeared over Antarctica
and caused all the panic is a natural and annual phenomena. The annual ozone hole was first measured in 1956-57, long before
the ozone-destroying CFCs were in common use. The hole appears at the end of the dark, cold Antarctic winter, lasts about
three to five weeks, and then disappears. There is no overall or permanent depletion of the ozone layer.

Ozone keep’s gettin’ better

Lomborg 1 (Bjorn, Associate Professor of Political Science – University of Aarhus, The Skeptical Environmentalist, p. 274)

The international cooperation has rapidly borne fruit: as can be seen in Figure 143, total production in 1996 was down below
the production in 1960. At the same time, the
total combined abundance of ozone-depleting
compounds in the lower atmosphere peaked in about 19 94 and are now slowly declining –
actually faster than was predicted by the UN just four years earlier . The concentration of the ozone-
depleting chlorine and bromine was predicted to peak in the stratosphere before the year 2000. The latest synthesis report of
the UNEP ozone assessment predicts that “the
ozone layer will slowly recover over the next 50 years.”
Likewise, the Antarctic ozone hole will slowly recover. Thus, today we have pretty much
done what we can, ozone depletion is at its maximum and it will recover within the next 50
years.
Pakistan Economy Answers
Their economy is strong and resilient and US cooperation high- newest evidence

Desk ’13 (Web Desk, The Express Tribune, “Economic stability of Pakistan an encouraging sign: Olson”,
http://tribune.com.pk/story/491648/economic-stability-of-pakistan-an-encouraging-sign-olson, January 9, 2013)

ISLAMABAD: US Ambassador to Pakistan Richard Olson in a meeting with finance minister Dr


Abdul Hafeez Sheikh on Wednesday said that economic stability of Pakistan is an encouraging
sign, Radio Pakistan reported. Dr Sheikh said that despite energy scarcity and security issue in the
country‚ economic indicators are showing positive trends which reflect resilience of the
economy. The Finance Minister added that due to economic policies of the
government‚ Pakistan is currently witnessing the lowest inflation rate in the region and the
Karachi Stock Exchange has emerged as the best performing Stock Exchange in the world . Both
the sides reaffirmed their commitment to enhancing economic relations. Olson said that the
United States is assisting Pakistan in many public welfare projects and will continue to do so in
future to further cement the relations between the two people. The Ambassador said that the US values its
relations with Pakistan and would continue to move forward in a number of mutually
beneficial areas.
Pakistan Instability Answers
Frontline
No Pakistani collapse

Sunil Dasgupta 13 Ph.D. in political science and the director of UMBC's Political Science
Program and a senior fellow at Brookings, 2/25/13, "How will India respond to civil war in
Pakistan," East Asia Forum, http://www.eastasiaforum.org/2013/02/25/how-will-india-respond-
to-civil-war-in-pakistan/
Bill Keller of the New York Times has described Pakistani president Asif Ail Zardari as overseeing ‘a ruinous kleptocracy that is spiraling deeper into
economic crisis’. But in contrast to predictions of an unravelling nation, British journalist-scholar
Anatol Lieven argues that the Pakistani state is likely to continue muddling through its many
problems, unable to resolve them but equally predisposed against civil war and consequent
state collapse. Lieven finds that the strong bonds of family, clan, tribe and the nature of South Asian
Islam prevent modernist movements — propounded by the government or by the radicals —
from taking control of the entire country.¶ Lieven’s analysis is more persuasive than the widespread view that Pakistan is
about to fail as a state. The formal institutions of the Pakistani state are surprisingly robust given the

structural conditions in which they operate. Indian political leaders recognise Pakistan’s
resilience. Given the bad choices in Pakistan, they would rather not have anything to do with
it. If there is going to be a civil war, why not wait for the two sides to exhaust themselves before thinking about intervening? The 1971 war
demonstrated India’s willingness to exploit conditions inside Pakistan, but to break from tradition requires strong, countervailing logic, and those
elements do not yet exist. Given the current conditions and those in the foreseeable future ,
India is likely to sit out a Pakistani
civil war while covertly coordinating policy with the United States.

No impact to Pakistan instability- their ev is hype

Hundley ’12 (Before joining the Pulitzer Center, Tom Hundley was a newspaper journalist for 36 years, including nearly two
decades as a foreign correspondent for the Chicago Tribune. During that time he served as the Tribune’s bureau chief in Jerusalem,
Warsaw, Rome and London, reporting from more than 60 countries. He has covered three wars in the Persian Gulf, the Arab-Israeli
conflict and the rise of Iran’s post-revolutionary theocracy. His work has won numerous journalism awards. He has taught at the
American University in Dubai and at Dominican University in River Forest, Illinois. He has also been a Middle East correspondent for
GlobalPost and a contributing writer for the Chicago News Cooperative. Tom graduated from Georgetown University and holds a
master’s degree in international relations from the University of Pennsylvania. He was also National Endowment for the Humanities
journalism fellow at the University of Michigan. Published September 5, 2012

With both sides armed to the teeth, it is easy to exaggerate the fears and much harder to pinpoint where the real
dangers lie. For the United States, the nightmare scenario is that some of Pakistan's warheads or its fissile material falls into the
hands of the Taliban or al Qaeda -- or, worse, that the whole country falls into the hands of the Taliban. For example, Rolf Mowatt-
Larssen, a former CIA officer now at Harvard University's Belfer Center for Science and International Affairs, has warned of the
"lethal proximity between terrorists, extremists, and nuclear weapons insiders" in Pakistan. This is a reality, but on the whole,
Pakistan's nuclear arsenal appears to be reasonably secure against internal threats, according to those who know the country best.
To outsiders , Pakistan appears to be permanently teetering on the brink of collapse. The fact that
large swaths of the country are literally beyond the control of the central government is not reassuring. But a weak state does
not mean a weak society, and powerful internal dynamics based largely on kinship and tribe make
it highly unlikely that Pakistan would ever fall under the control of an outfit like the Taliban. During
the country's intermittent bouts of democracy, its civilian leaders have been consistently incompetent and corrupt, bu t even in
the worst of times , the military has maintained a high standard of professionalism. And there is
nothing that matters more to the Pakistani military than keeping the nuclear arsenal -- its crown
jewels -- out of the hands of India, the United States, and homegrown extremists. "Pakistan struggled to acquire
these weapons against the wishes of the world. Our nuclear capability comes as a result of great sacrifice. It is our most precious and
powerful weapon -- for our defense, our security, and our political prestige," Talat Masood, a retired Pakistani lieutenant general,
told me. "We keep them safe." Pakistan's nuclear security is in the responsibility of the Strategic Plans Division,
which appears to function pretty much as a separate branch of the military. It has its own training facility and
an elaborate set of controls and screening procedures to keep track of all warheads and fissile
material and to monitor any blips in the behavior patterns of its personnel. The 15 or so sites where
weapons are stored are the mostly heavily guarded in the country. Even if some group managed to
steal or commandeer a weapon, it is highly unlikely the group would be able to use it . The greater
danger is the theft of fissile material, which could be used to make a crude bomb. "With 70 to 80 kilos of highly enriched uranium, it
would be fairly easy to make one in the basement of a building in the city of your choice," said Pervez Hoodbhoy, a distinguished
nuclear physicist at Islamabad's Quaid-i-Azam University. At the moment, Pakistan has a stockpile of about 2.75 tons -- or some 30
bombs' worth -- of highly enriched uranium. It does not tell Americans where it is stored. "All nuclear countries are conscious of the
risks, nuclear weapons states especially so," said Gen. Ehsan ul-Haq, who speaks with the been-there-done-that authority of a man
who has served as both chairman of Pakistan's Joint Chiefs of Staff Committee and head of the ISI, its controversial spy agency. "Of
course there are concerns. Some are genuine, but much of what you read in the U.S. media is irrational and
reflective of paranoia. Rising radicalism in Pakistan? Yes, this is true, and the military is very conscious of this." Perhaps
the most credible endorsement of Pakistan's nuclear security regime comes from its most steadfast
enemy. The consensus among India's top generals and defense experts is that Pakistan's nukes
are pretty secure. "No one can be 100 percent secure, but I think they are more than 99 percent secure ," said
Shashindra Tyagi, a former chief of staff of the Indian Air Force. "They keep a very close watch on personnel. All of the steps that
could be taken have been taken. This business of the
Taliban taking over -- it can't be ruled out, but I think it's
unlikely. The Pakistani military understands the threats they face better than anyone, and they are smart
enough to take care it." Yogesh Joshi, an analyst at the Institute for Defense Studies and Analyses in New Delhi, agrees:
"Different states have different perceptions of risk. The U.S. has contingency plans [to secure Pakistan's nukes] because its
Pakistan,
nightmare scenario is that Pakistan's weapons fall into terrorist hands. The view from India over the years is that
probably more than any other nuclear weapons state, has taken measures to secure its weapons .
At the political level here, there's a lot of confidence that Pakistan's nuclear weapons are secure."

No Pakistani collapse

AP 10 (“Pakistan's stability, leadership under spotlight after floods and double dealing accusations,” August 6th,
http://www.foxnews.com/world/2010/08/06/pakistans-stability-leadership-spotlight-floods-double-dealing-accusations/,

Not for the first time, Pakistan appears to be teetering on the edge with a government unable to
cope. Floods are ravaging a country at war with al-Qaida and the Taliban. Riots, slayings and arson are gripping the largest city.
Suggestions are flying that the intelligence agency is aiding Afghan insurgents. The crises raise questions about a nation crucial
to U.S. hopes of success in Afghanistan and to the global campaign against Islamist militancy. Despite
the recent
headlines, few here see Pakistan in danger of collapse or being overrun by militants — a fear
that had been expressed before the army fought back against insurgents advancing from their base in the Swat Valley early last
year. From its birth in 1947, Pakistan has been dogged by military coups , corrupt and inefficient
leaders, natural disasters, assassinations and civil unrest. Through it all, Pakistan has not prospered — but it
survives. “There is plenty to be worried about, but also indications that when push comes to shove the state
is able to respond," said Mosharraf Zaidi, an analyst and writer who has advised foreign governments on aid missions to
Pakistan. "The military has many weaknesses, but it has done a reasonable job in relief efforts. There have been
gaps in the response. But this is a developing a country, right?" The recent flooding came at a sensitive time for Pakistan, with
Western doubts over its loyalty heightened by the leaking of U.S. military documents that strengthened suspicions the security
establishment was supporting Afghan insurgents while receiving billions in Western aid. With few easy choices, the United
States has made it clear it intends to stick with Pakistan. Indeed, it has used the floods to demonstrate its commitment to the
country, rushing emergency assistance and dispatching helicopters to ferry the goods. The Pakistani government's response to
the floods has been sharply criticized at home, especially since President Asif Ali Zardari departed for a European tour. With so
many Pakistanis suffering, the trip has left the already weak and unpopular leader even more vulnerable politically. The flooding
was triggered by what meteorologists said were "once-in-a-century" rains. The worst affected area is the northwest, a
stronghold for Islamist militants. Parts of the northwest have seen army offensives over the last two years. Unless the people
are helped quickly and the region is rebuilt, anger at the government could translate into support for the militants. At least one
charity with suspected links to a militant outfit has established relief camps there. The extremism threat was highlighted by a
suicide bombing in the main northwestern town of Peshawar on Wednesday. The bomber killed the head of the Frontier
Constabulary, a paramilitary force in the northwest at the forefront of the terror fight. With authorities concentrating on flood
relief, some officials have expressed concern that militants could regroup. The city of Karachi has seen militant violence and is
rumored to be a hiding place for top Taliban and al-Qaida fighters. It has also been plagued by regular bouts of political and
ethnic bloodletting since the 1980s, though it has been calmer in recent years. The latest violence erupted after the
assassination of a leading member of the city's ruling party. More than 70 people have been killed in revenge attacks since then,
paralyzing parts of the city of 16 million people. While serious, the unrest
does not yet pose an immediate
threat to the stability of the country. Although the U.S. is unpopular, there is little public support for
the hardline Islamist rule espoused by the Taliban and their allies . Their small movement has been
unable to control any Pakistani territory beyond the northwest, home to only about 20 million of the country's 175 million
people.

No chance that Pakistan will collapse

Bandow ‘9- Senior Fellow @ Cato, former special assistant to Reagan (11/31/09, Doug, “Recognizing the Limits of American
Power in Afghanistan,” Huffington Post, http://www.cato.org/pub_display.php?pub_id=10924)

From Pakistan's perspective, limiting the war on almost any terms would be better than prosecuting it for years, even to "victory,"
whatever that would mean. In fact, the
least likely outcome is a takeover by widely unpopular Pakistani
militants. The Pakistan military is the nation's strongest institution; while the army might not be
able to rule alone, it can prevent any other force from ruling . Indeed, Bennett Ramberg made the important
point: "Pakistan, Iran and the former Soviet republics to the north have demonstrated a brutal
capacity to suppress political violence to ensure survival. This suggests that even were
Afghanistan to become a terrorist haven, the neighborhood can adapt and resist ." The results might
not be pretty, but the region would not descend into chaos . In contrast, warned Bacevich: "To risk the stability of that
nuclear-armed state in the vain hope of salvaging Afghanistan would be a terrible mistake."

No risk of an Islamist takeover in Pakistan- experts agree.

Rediff, 2008 [‘A jihadi takeover of Pak is unlikely” Rediff Interview South Asia expert Brude Riedel, March 13,
http://www.rediff.com/news /2008/mar/13inter.htm

Bruce Riedel recently joined The Brookings Institution as Senior Fellow, on the back of his expertise as
an expert on the Middle East and South Asia, and more than three decades as a policymaking
veteran in regional diplomacy and counter-terrorism in the government. He believes an Islamist takeover of
Pakistan, and consequently Islamist control over that country's nuclear weapons arsenal, is a
non-issue even though jihadists have become increasingly dangerous. It is more realistic, he says, to be concerned about a
weapon or two being spirited away by military personnel sympathetic to the jihadist cause, and passed on by them into a terrorst
group. Against the backdrop of reports that administration and Congressional circles are considering unilateral US action to secure
Pakistan's nuclear weapons, Riedel believes any such pre-emptive strike would be highly counter-productive; it could, he says, make
a bad situation infinitely worse. Instead, he suggests that the US work actively to help restore democracy and end the military
dictatorship. An elected civilian government, with oversight over the nuclear weapons program, is the best possible solution, he
argues. Riedel speaks on the back of quantifiable experience : For much of his government career, he has served
with the Central Intelligence Agency, the National Security Council, the Department of State and the Department of Defense. He has
served the past three Presidential administrations -- those of George H W Bush and Bill Clintons two terms on the National Security
Council. In this capacity, he was a central player during the 1999 Kargil crisis that put nuclear-armed India and Pakistan in a
confrontational posture. He was the only official to sit in on the July 4, 1999 meeting between then President Clinton and then
Pakistan Prime Minister Nawaz Sharif, who was given a stern warning to immediately withdraw Pakistani military forces from Kargil
or face Washington's wrath. He spoke to rediff India Abroad Managing Editor Aziz Haniffa in Washigton, DC. At Brookings the other
day, you were dismissive of the belief that Pakistan is on the verge of being taken over by Islamists who will have access to the
country's nuclear weapons. Why do you believe such a scenario is untenable? The
notion that Pakistan as a whole
could fall into the hands of jihadists, and that you could have an Islamicised state, is a very, very
remote possibility. This is more so because there is no indication to believe that the Islamists are on
the verge of any major electoral victory and that they are going to be marching on Islamabad,
although there is no denying that they have become increasingly dangerous.

No risk that Pakistani nukes are stolen

Innocent ‘10 - foreign policy analyst at the Cato Institute (Malou, “Away from McChrystal and Back to the Basics,” Huffington
Post, 6/28, http://www.cato.org/pub_display.php?pub_id=11934)

Pakistan has an elaborate command and control system in place that complies with strict
Western standards, and the country's warheads, detonators, and missiles are not stored fully-
assembled, but are scattered and physically separated throughout the country . In short, the danger
of militants seizing Pakistan's nuclear weapons in some Rambo-like scenario remains highly
unlikely.
---Pakistan- No Nuclear Giveaway

Terrorists couldn’t break through or infiltrate Pakistan’s nuclear security.

Kanwal, ‘8 [Gurmeet, director, Centre for Land Warfare Studies, New Delhi, “Are Pakistan’s nuclear warheads safe?”
http://www.rediff.com/news/2008/oct/14guest.htm]

The third tier comprises a well-guarded and fortified perimeter fence with strictly controlled entry. Most of these sites have air
defence assets allotted to them to defend against attacks from the air. Personnel selected for the security of the
outer perimeter are reported to belong to elite infantry battalions of the Pakistani army. The
possibility of any of these personnel being subverted is guarded against by counter-intelligence
teams. Military regimes have very strong survival instincts and the Musharraf regime has ensured that hard-line
radical elements are ruthlessly weeded out from the nuclear security detail . Hence, it can be
concluded that if some rogue elements were to try to gain control over the nuclear warheads, they would have to be
prepared to fight their way through several layers of highly motivated personnel who are
armed to the teeth. The delivery systems of Pakistan's Strategic Forces Command, comprising Chinese supplied M-11
and M-9 and the North Korean Nodong and Taepo Dong nuclear-capable surface-to-surface missiles and their launchers, are
based at separate locations. These sites or "hides" are well-dispersed to ensure that maximum warheads survive a
conventional air attack during war. They are also well defended against possible commando raids. In the improbable eventuality that
radical hard-liners take over Pakistan, their rag-tag
fighters will have to fight the elite army guards to the
bitter end before they can lay their hands on the delivery systems . A terrorist organisation must get hold of
both a nuclear warhead and a launch system and must acquire the expertise to mate the warhead with the launcher. Or, it must
smuggle a warhead undetected to the target and somehow break the electronic code to activate it. These are all extremely complex
challenges as highly sophisticated expertise is required to test, mate, activate and launch a nuclear warhead.

No Pakistan terror nukes impact

Tepperman ‘9 (Jonathan, Deputy Editor at Newsweek, Former Deputy Managing Editor, Foreign Affairs, LLM, i-law, NYU,
MA, jurisprudence, Oxford, “Why Obama Should Learn to Love the Bomb”,
http://jonathantepperman.com/Welcome_files/nukes_Final.pdf, 2009)

Note – Michael Desch = prof, polsci, Notre Dame

As for Pakistan,
it has taken numerous precautions to ensure that its own weapons are
insulated from the country’s chaos, installing complicated firing mechanisms to prevent a
launch by lone radicals, for example, and instituting special training and screening for its nuclear personnel
to ensure they’re not infiltrated by extremists. Even if the Pakistani state did collapse entirely
—the nightmare scenario— the chance of a Taliban bomb would still be remote. Desch argues that
the idea that terrorists “could use these weapons radically underestimates the difficulty of
actually operating a modern nuclear arsenal. These things need constant maintenance and they’re very easy to
disable. So the idea that these things could be stuffed into a gunnysack and smuggled across the Rio Grande is preposterous.”

No Pakistan impact- they’ve gone through worse


Stolar ‘7 (Alex Jr., Peace Fellow in the South Asia Program – Stimson Center, “The Implications of Unrest in Pakistan for Nuclear
Security”, http://www.stimson.org/southasia/?SN=SA200705181263, May 18, 2007)

It has been a very painful week in Pakistan. On May 12th, over forty died when violence broke out in Karachi between political
factions supporting and opposing President Musharraf. Three days later, a suicide bomber entered a hotel restaurant in Peshawar
and detonated the explosives strapped to his body, killing at least twenty persons. Recent
turbulence has renewed
fears that Pakistan’s nuclear weapons and materials may be vulnerable to breakdowns in
command and control or theft. The good news is that these nightmare scenarios are unlikely to occur
during the current political unrest. The bad news is that Pakistan’s domestic unrest will continue and grow worse
without the restoration of a representative government, and that extremists have many ways to further destabilize Pakistan. Are
Pakistan’s bombs safe? In theory, Pakistan’s nuclear weapons could be vulnerable to theft, illicit transfer, or unintentional use if the
army’s discipline and command and control structure faltered. Concerns about the security of Pakistan’s weapons are greatest in the
West when Pakistani politics enter a rough patch and during leadership changes. Fortunately, these worst case scenarios
are highly unlikely. Pakistan has been through worse passages of political unrest. Intimidation, politically-driven
violence, and sectarian strife are all too common in Pakistani politics. If past experience is any guide, the current
unrest will not lead to anarchy or chaos in Pakistan. The vast majority of Pakistanis desire a
moderate and stable state, and the army has an institutional interest to prevent the
breakdown of national authority and cohesion. Pakistan’s weapons were secure during
previous periods of political instability, and they are likely to remain the most protected national
assets during the current unrest. There are no signs of a breakdown in command and control in the Pakistan Army.

It’s impossible for terrorists to steal nukes from Pakistan.

Asian Tribune, ‘7 [Adnan Gill, “Loose Nukes or Loose Talk,” 11-23, http://www.asiantribune.com/index. php?q=node/8348]

These experts are also of the opinion that Pakistan does not use the electronic systems that require the input of access codes to arm
warheads; instead these weapons are stored disassembled, with key components kept in separate, secure vaults. The US twice, once
after Pakistan's nuclear tests and then right after 9/11, debated on whether to share with Pakistan its one of the most sophisticated
and guarded technology, known as “Permissive Action Links,” or PALS to enhance the security of its nukes . What
most of experts do not understand, or do not want to disclose is the fact that such a technology is no longer difficult to mimic. It
doesn't take billion dollars in research to integrate a bank ATM access-code circuit board with the firing circuits/triggers. Such
devices can also cause a small explosion within the warhead to safely render it useless after
certain number of incorrect tries, or due to an unauthorized tampering. In a worst case scenario, even if
some militant group is somehow able to get their hands on a Pakistani nuclear device, they will have to either smuggle it out to the
target or they will have to get a viable delivery system too (which are stored separately). In addition, they
will need
expertise in matting the warhead with the delivery system and arming the device. Since the
components of Pakistani nuclear devices are allegedly stored disassembled and separately, the
rouge group will have to be lucky in securing all of the components. This will be next to
impossible to do without triggering security countermeasures . In case, they are able to smuggle out highly
enriched material and they have all the expertise at their disposal, still it will be next to impossible to fashion a nuclear bomb. It
takes a highly sophisticated infrastructure and other materials to build a nuclear weapon. It may be worth noting, that even the
states bent on developing nuclear weapons can rarely put together such an infrastructure; or today there would have been a lot
more nuclear weapons states than 10 (declared and undeclared) nations.
The Pakistan army won’t allow the transfer of nuclear weapons to the Taliban

Simon, and Stevenson ‘9 * adjunct Senior Fellow at the Council on Foreign Relations, AND **Professor of Strategic Studies
at the US Naval War College, (Steven and Jonathan, “Afghanistan: How Much is Enough?” Survival, 51:5, 47 – 67, October 2009
http://www.informaworld.com/smpp/section?content=a915362559&fulltext=7132409)

The United States' next logical move would be to intensify pressure, raising civilian casualties, increasing political pressure on the
Kabul and Islamabad regimes, and ultimately weakening them, which would only help al-Qaeda and the Taliban. In fact, some
evidence of this dynamic has already materialised, as the Pakistani government has faced difficulties in dealing with hundreds of
thousands of Pakistanis displaced by the military campaign, undertaken at Washington's behest, in the Swat Valley. Certainly worries
about Islamabad's ability to handle the Taliban on its own are justi fied. Some Taliban members are no doubt keen on regime change
in favour of jihadists, as noted by Bruce Riedel, who headed up the Obama administration's 60-day policy review.29 But
Pakistan's military capabilities should not be given short shrift. The Pakistani army, however
preoccupied by India, is seasoned and capable, and able to respond decisively to the Taliban
should its activities reach a critical level of destabilisation. Inter-Services Intelligence, devious
though it may be, would be loath to allow the transfer of nuclear weapons to the Taliban .

Pakistan loves its nukes- It won’t give them away.

The Washington Times, ‘8 [Nicholas Kralev, “India, Pakistan says nukes secure,”
http://www.washingtontimes.com/news/2008/dec/12/india-pakistan-say-nukes-safe-from-terrorists/]

Retired diplomats and generals from India and Pakistan, who visited Washington to promote the elimination of all nuclear weapons,
said Thursday that their countries' atomic weapons remain safe from terrorists. "There
is no way that any militant can
get any fissile material or weapons and use them," said Lt. Gen. Talat Masood of the Pakistani army.
"Pakistan has gone to the extent of eating grass to acquire nuclear weapons, so it's not going to
hand them to militants." Retired Air Chief Marshal Shashindra Pal Tyagi, former head of the Indian air force, expressed the
same confidence about both the Indian and Pakistani arsenals. "In India, the controls of the weapons are under different
organizations," he said. "Pakistan's armed forces are very professional -- we know, we fought them -- and from all accounts, they
have put a lot of safety measures in place."

Pakistan has an incentive not to give its nukes to terrorists.

Mueller ‘8 (John Mueller, Department of Political Science @ Ohio State University, January 1, 2008, The Atomic Terrorist:
Assessing the Likelihood, p. http://polisci.osu.edu/faculty/jmueller/APSACHGO.PDF

Pakistan, which has been repeatedly threatened by al-Qaeda, has a similar very
The government of
strong interest in controlling its nuclear weapons and material--and scientists. Notes Stephen
Younger, former head of nuclear weapons research and development at Los Alamos and
director of the Defense Department's Defense Threat Reduction Agency from 2001 to 2004,
"regardless of what is reported in the news, all nuclear nations take the security of their
weapons very seriously" (2007, 93; see also Kamp 1996, 22; Milhollin 2002, 47-48).
---Pakistan- No Electoral Islamists

No chance of an Islamic takeover in Pakistan.

Arms Control Today, ‘7 [Kenneth N. Luongo, Naeem Salik, “Building confidence in Pakistan’s nuclear security,” December 1]
Islamist takeover as a result of elections or collapse of government. At the heart of the current crisis in Pakistan is the question of
political elections. A serious question is whether Islamic extremist groups and Islamist political parties could gain power in Pakistan
through the election process. According to the International Crisis Group, "Poll
after poll has found that if fair and
free elections were held under constitutional protections and monitored by national and international observers, the
result would be a moderate, pro-Western, anti-extremist government in Pakistan." (2) Extremist
Islamist parties have never won more than 11 percent of the total votes in a Pakistani election .
(3) Questions have also been raised about the reliability of the Pakistani military, given the ethnic diversity that exists within its
ranks. The military in Pakistan has become more ethnically diverse in recent decades and contains Baluchis, Pashtuns, Punjabis, and
Sindhis. This has not been a cause for concern about potential factionalism as the troops are professionally trained and have proven
to be cohesive in the current political crisis.
---Pakistan- No Collapse

Extremists have no support

Hamid 7 (Shadi, Director of Research – Project on Middle East Democracy and Associate – Truman National Security Project,
“The Nuclear Scenario in Pakistan”, Democracy Arsenal, 12-27, http://www.democracyarsenal.org/2007/12/the-nuclear-sce.html)

Well, yes it might be. But the chances of the 'nuclear scenario' actually happening is so slim that
treating it as the overriding policy question is, at best, a diversion and distraction from the real risks
Pakistan faces. How exactly did this become the conventional wisdom? On one hand, you have Al-Qaeda and other associated
terrorist groups. Al-Qaeda - I hope I am stating the obvious - is not going to take over the Pakistani government anytime soon.
Extremist groups have the capability to terrorize the population, assassinate leaders, and destablize the
country, but there are few indications that they have made enough inroads into the military
or ISI to threaten an actual internal coup. The other possibility is that the various Islamist parties might somehow
come to power through free elections. Maybe this is what people are really referring to when they talk about an "Islamist
takeover," a term which has long been a staple of Middle East-related fearmongering, and one that has been employed to great
effect by the Muslim world's predominantly secular (and often brutal) dictators, including many of our allies. Well, the chances
of this scenario occurring are even slimmer. Islamist parties in Pakistan have not made much an effort to moderate
(in contrast to their Arab and Turkish counterparts), and they are, in fact, a frightening bunch. However, they do not
command significant support in a country dominated by well-established secular parties.
Their peak electoral support is around 15%, give-or-take. In other words, not much of a threat. With all
that said, we are talking about the Muslim world, an area of the world that tends to surprise when surprises are least expected
and not particularly welcome. So I could be wrong. But the point remains that we
shouldn't overexaggerate the
threat of nuclear oblivion ushered in by Pakistan's Islamic extremists. And then there's the other
question of why Islamic extremists have been able to wreak so much havoc in the first place. Didn't Mush promise us he would
defeat the extremists or something? Oh, wait. Every dictator in the Muslim world promises us that. And, every time, we end up
dissapointed.
Peace Process Answers
It will fail regardless of presidential action – multiple reasons

Black ’11 (4/16 (Conrad, National post, (f/k/a The Financial Post) (Canada), Lexis, “A recipe for peace; We know what a future
Israeli-Palestinian peace deal will look like. The question is: When will the Arabs say yes?”, April 16, 2011,

There is something faintly nostalgic about former U.S. national security advisor Brent Scowcroft's Financial Times op-ed,
published Wednesday, calling for Barack Obama to "broker a new Mideast Peace." The man, the media and the message are all,
as Hillary Clinton would say, "so yesterday." It's a little like watching vintage films from the era when American leaders were first
"brokering peace" in the Mideast, such as American Graffiti or The Graduate. Brent Scowcroft was the U.S. national security
advisor to President Gerald Ford, and he returned to the post under President George H.W. Bush. He is a distinguished foreign-
policy and strategicpolicy specialist, but has never been considered overly original, a reputation that will not be shaken by his
new suggestion. is a justly respected newspaper, but its editorial line is always the urbane, gentlemanly, British impulse to speak
softly, move in increments, generally advance the conventional wisdom; and don't stretch the imagination, catch a cold thinking
outside the box, or get seriously riled up over anything short of a genuine outrage. In this case, however, the conventional
wisdom is nonsense. The counsel for President Obama to "broker peace" is on par with Pakistani President Musharraf 's advice
to Tony Blair to "do Palestine." The Palestinians could have peace with Israel tomorrow if they wanted it. In any event, Barack
Obama is not trusted by Israel. His chief initiative in the area to date has been to deny the existence of the
agreement George W. Bush made with Ariel Sharon, whereby Israel would vacate Gaza, would dismantle some West Bank
settlements, and would confine extensions of other settlements in contested areas to natural population growth. (The world's
obsession with settlement abandonment as the key to peace -which Obama seems to share -is foolish: If
Israel dismantled every settlement, or even turned them over for occupation by returning Palestinians, a new
pretext to keep the pot boiling would be devised.) The larger , more basic, problem is rooted in
history. Amid the desperate conditions of the First World War, the British sold the same real estate twice: to the Jews to
promote pro-Allied opinion in the United States and the international Jewish community, and to the Arabs to incite rebellion
against the occupying Ottoman Turks. The predictable happened when both buyers tried to take possession, and there has
never been any answer except to divide the diminutive space in two. This has been the subject of three wars, all of them
sparked by the Arabs, and none successfully (although the intervention of Richard Nixon in the 1973 war -after he had supplied
the Israelis with a virtually new air force -enabled the Egyptians to claim a partial success). Always, it is claimed that a return to
the 1967 borders is the basis of peace, although those borders would leave Israel nine miles wide on its Mediterranean shore,
and the West Bank and Gaza sections of Palestine separated by 50 miles. The Arabs effectively had those borders, under the
control of Jordan and Egypt, in 1967. Yet they went to war and and lost -events that would not normally be expected to
generate reverence for the status quo ante. The facts, which must be perfectly well-known to Brent Scowcroft, are that it
is
impossible to deal with the Palestinians while Hamas controls Gaza and the PLO the West
Bank; that it is impossible to broker anything while the surrounding Arab powers are in
turmoil; that this U.S. administration is not taken seriously by anybody in the area after the
denial of the Bush-era settlements arrangement and the failure of its Iran policy; and the solution, when the
Palestinians are ready, has, as Scowcroft himself notes, largely been identified already. There will have to be some exchanges of
territory, to make Israel wider between the Mediterranean and the West Bank. Scowcroft envisions a united Jerusalem serving
as the capital of both countries. I don't think so; I think sideby-side Jerusalems, with the Arabs controlling their area beyond
Orient House and a special arrangement and assured access to designated holy sites for all faiths throughout both countries.
The Palestinian right of return would be to Palestine, and this fairy tale of one big happy Holy Land where all would be brothers,
but in fact the Muslims would outnumber the Jews and expel or massacre them yet again, should finally have a silver stake
driven through its heart. A clairvoyant is not required to see that this is where it will end up. Nor is one required to see that, as
Arab populations have begun to stop being distracted by the red herring of Israel and have focused on the misgovernment from
which they have suffered, no Israeli flags have been burned nor Palestinian flags waved about by non-Palestinians. Israel is
absolutely legitimate as a Jewish state, and was so constituted by the unanimous permanent members of the United Nations
Security Counsel, on the motion of Stalin's ambassador, seconded by Truman's. The borders have been open to legitimate
debate. But when the Palestinians determine that they will no longer be used as cannonfodder, a cause célèbre that enables the
leaders of the Muslim powers to misgovern, oppress and pillage their countries, and deflect discontent by waving the bloody
shirt of Palestine, the borders could be quickly established along the lines mentioned. If the Palestinians could draw the lesson
of the spectacular economic growth of the West Bank, which Israel has assisted, and where Prime Minister Salam Fayyad
favours peace and is the first Palestinian leader whose CV does not contain a long stint as an extremist or terrorist; and of the
contrast with the collapsed economy in Gaza, which has happily served as a launch-site for rockets aimed at Israeli civilians since
Sharon vacated it; then peace would be imminent. For a sensible and experienced man such as Brent
Scowcroft to suggest that President Obama is in any position to broker anything in a Middle East
where the Arab governments are fighting for their lives with their own people and Hamas is still trying to kill all the Jews, is
disconcerting. Even Richard Goldstone, the token anti-Israel Jew recruited by the United Nations to write a smear job on
Israel's hugely provoked reprisals against Gaza, has recanted his fraudulent report. Israel has its faults, but it is a legitimate
Jewish state, a successful society of laws and enterprise. The Palestinians have grievances, but a remedy is at hand. Israel
will take half a loaf. If Palestine would also, there would be peace. But
it won't happen until it is clear what
Egypt, Syria and Lebanon will look like, and whether anything will be done to curb the baleful
influence in the area of Iran. If he was minded to, President could do something about that, but it isn't a matter of brokerage
and there is no sign of it coming.

See Israel-palestine answers in Middle East War Answers


Peak Coal Answers
Coal is abundant – it’ll last 200 years

SA 8 (Seeking Alpha – Stock Market and Investment Magazine, “Coal Prices Set to Go Up”, 9-29,
http://seekingalpha.com/article/97771-coal-prices-set-to-go-up)

Furthermore, with the steep increase in oil and gas prices, coal's importance in the world energy mix is set to increase in the
future. According to the International Energy Agency, there are abundance of coal resources of
approximately 200 years worth of coal reserves, evenly distributed in the US (27%), Russia (17%), China
(13%) and India (10%). It is estimated that by 2030, coal will account for 27% of the global energy mix, up from the 24% that
it holds today. Given the abundance and accessibility of coal resources, the increased usage of coal will
facilitate minimizing the global energy crisis. Going forward, the demand-supply mismatch is expected to last
for at least two to three years before it recovers from the supply bottlenecks in Australia and ease power shortages in South
Africa. Hence, coal price outlook in FY 2009 is likely to remain strong with coking coal prices expected to increase to US$320 a
metric ton and thermal coal to increase to US$135 a metric ton for 2009 coal year (Source: Merrill Lynch & Co Inc).

Even conservative estimates show no coal shortage

Casey 5 (Doug, Author, Publisher, and Professional Investor, “Too Late for Coal?”, 2-2,
http://www.kitcocasey.com/articles/13/)

But very few people have been in on this run from the ground floor. At the turn of the millennium, no one wanted anything to
do with coal. Back then, the market was still laboring under 20-year low prices, with export coal indices yo-yoing in a $10-$15
range for much of the last two decades. Peaks and troughs in price came roughly 3-4 years apart, with lows in 1987 and 1994,
and peaks in 1990 and 1997. When the 1997 Asian crisis hit, just as coal was set to begin one of its downswings, it lead to 20-
year-low export prices in 2000. Why such a rollercoaster ride? A large part of the reason is the ease with which coal suppliers
enter and leave the market. There is a lot of coal in the world: over a trillion tons of recoverable
reserves, according to the presumably conservative estimates of the U.S. Energy Information Administration
(EIA). World coal consumption , by comparison, is about 5 billion annual tons, meaning that the coal we
already know exists could supply us for 200 years at current rates-or even 135 years, using the EIA's highest
projection for economic growth by 2025. During periods of high prices, miners crank up production on the
more than ample reserves, creating an oversupply in the markets followed shortly by lower prices. Marginal coal mines
then shut down, allowing demand to catch up. Prices head back up again, prompting suppliers to produce more, and the cycle
repeats.

Global coal supply is strong

Rosenthal 8 (Elisabeth, Staff – NY Times, “Europe Turns Back to Coal, Raising Climate Fears”, New York Times, 4-23, Lexis)

Enel and many other electricity companies say they have little choice but to build coal plants to replace aging infrastructure,
particularly in countries like Italy and Germany that have banned the building of nuclear power plants. Fuel costs have risen 151
percent since 1996, and Italians pay the highest electricity costs in Europe.
In terms of cost and energy security,
coal has all the advantages, its proponents argue. Coal reserves will last for 200 years , rather than 50
years for gas and oil. Coal is relatively cheap compared with oil and natural gas, although coal prices have tripled in the past few
years. More important, dozens of countries export coal — there is not a coal cartel — so there is more room
to negotiate prices. “In order to get over oil, which is getting more and more expensive, our plan is to convert all oil plants to
coal using clean-coal technologies,” said Gianfilippo Mancini, Enel’s chief of generation and energy management. “This will be
the cleanest coal plant in Europe. We are hoping to prove that it will be possible to make sustainable and environmentally
friendly use of coal.”
Peak Natural Gas Answers
New supplies solves

Casselman 9 (Ben, “U.S. Gas Fields Go From Bust to Boom”, Wall Street Journal, April 30,
http://online.wsj.com/article/SB124104549891270585.html)

A massive natural-gas discovery here in northern Louisiana heralds a big shift in the nation's
energy landscape. After an era of declining production, the U.S. is now swimming in natural gas. Even conservative
estimates suggest the Louisiana discovery -- known as the Haynesville Shale, for the dense rock formation that
contains the gas -- could hold some 200 trillion cubic feet of natural gas . That's the equivalent of
33 billion barrels of oil, or 18 years' worth of current U.S. oil production. Some industry
executives think the field could be several times that size. "There's no dry hole here ," says Joan
Dunlap, vice president of Petrohawk Energy Corp., standing beside a drilling rig near a former Shreveport amusement park.
From Rock to Gas Huge new fields also have been found in Texas, Arkansas and Pennsylvania.
One industry-backed study estimates the U.S. has more than 2,200 trillion cubic feet of gas
waiting to be pumped, enough to satisfy nearly 100 years of current U.S. natural-gas
demand. The discoveries have spurred energy experts and policy makers to start looking to
natural gas in their pursuit of a wide range of goals: easing the impact of energy-price spikes,
reducing dependence on foreign oil, lowering "greenhouse gas" emissions and speeding the transition to renewable fuels.
Peak Oil Answers
Peak oil is a myth- empirics

Planes ’13 (A Big Day for the Dow and a Bigger Peak Oil Myth By Alex Planes | More Articles | Save For Later January 19, 2013 |
Comments (3)

The peak oil myth begins Peak oil has a longer history than you think. Although the models that define the American peak oil
hypothesis were first advanced in the 1950s, predictions of the imminent depletion of American oil reserves can be
found much earlier. In fact, one of the earliest known warnings that the United States would run out of oil was released on
Jan. 19, 1922, when the U.S. Geological Survey warned the public that only two decades of oil
remained in the ground, if present consumption patterns held steady. That day, the survey reported that
there remained 9.2 billion barrels of oil in the ground but warned that not all of it could be discovered, let alone extracted. The
survey's director somberly announced: "Unlike our reserves of coal, iron, and copper, which are so large that apprehension of their
early exhaustion is not justified, the oil reserves of the country ... [appear] adequate to supply the demand for only a limited number
of years. The annual production of the country is now almost 500 million barrels, but the annual consumption, already well beyond
the half-billion mark, is still growing." Of the 9.2 billion known barrels of oil, the survey estimated that all but 200 million were
recoverable, with Oklahoma, California, and the Texas-led Gulf Coast region dominating the available reserves. The director claimed
that the estimates were "undoubtedly the best that have ever been made for the United States, and better than have hitherto been
prepared for any oil country or district of the world." This was a world only 11 years removed from the Standard Oil breakup, and
only nine years removed from Ford's (NYSE: F ) first assembly line. Ford had not yet built its 10 millionth car, which would roll off the
assembly line to terrific fanfare two years after the Geological Survey's report. The United States had approximately 12.3 million
registered automobiles in 1922, many of which were Ford's Model T's with fuel economies ranging from 13 to 21 miles per gallon.
The value of Standard Oil's successor companies had grown fourfold in the decade following its dissolution, spurred in part by a
spike in oil prices that had tripled the cost of a barrel of oil between 1916 and 1920. Oil
exploration companies had
managed to double the amount of oil extracted annually between 1911 and 1921, but yearly automobile
registrations had increased by more than an order of magnitude (from fewer than 900,000 in 1911) in the same period. At the same
time, American electric utilities had been increasing their capacity by about 12% a year from the turn of the century onward, and
many of these plants were oil-fueled -- as were an increasing number of factories. America's voracious appetite for cars, electricity,
and mass production seemed nearly limitless at the beginning of one of the most incredible booms in history. At the time, it seemed
that these dire warnings might be worth heeding. However, as you're well aware, this doomsday scenario did not come
to pass. In the two decades leading up to World War II, from 1922 to 1941, America's total oil production would total 19.3 billion
barrels, and the price per barrel declined from a nominal price of $1.61 in 1922 to $1.14 in 1941 -- the equivalent of a real price drop
of 20%. America's oil production didn't reach its annual peak until 1970, during which the total oil extracted from American fields
was 3.5 billion barrels, or nearly 40% of all the oil the U.S. Geological Survey thought remained five decades earlier. In
90 years
of production since this colossally wrong prediction, the United States has extracted a total of
198 billion barrels of oil, more than 20 times the amount originally thought available for
extraction. In 2011, the five largest oil companies operating in the U.S., including former Baby Standards ExxonMobil, Chevron, and
ConocoPhillips (NYSE: COP ) , combined to generate revenues that were equivalent to more than 10% of U.S. GDP for that year. The
three American-based oil supermajors alone combined for annual oil-equivalent production of
3.2 billion barrels in 2011 -- much of which was natural gas, as total U.S. oil production by itself accounted for only 2.1
billion barrels in 2011. The peak oil hypothesis may have merit over the very long run, but it's been disproven so
often on shorter time frames that we have to accept that it's folly to predict the availability of
resources in the future using only the knowledge available to the present.
Peak oil flawed- experts

ENGLAND ‘12 – Staff Writer (England, Cameron. “ Peak oil debate is over, say experts”. May 14, 2012.
http://www.adelaidenow.com.au/business/peak-oil-debate-is-over/story-e6frede3-1226354771053)

THE debate about peak oil is over and the world has used just a fraction of the petroleum it
will be possible to extract, an expert believes. Speaking at the Australian Petroleum Production and Exploration
Association (APPEA) 2012 conference in Adelaide, oil major Total's chief executive Christophe de Margerie said
new sources of petroleum, such as tight gas and shale oil, meant that the world had ample
supplies of petroleum. Mr de Margerie said while there were economic and environmental issues which would affect how
quickly resources were exploited, there was "definitely not a concern about reserves''. His comments were echoed by Saudi
Arabia's Petroleum and Mineral Resources Minister Ali I. Naimi, who told the conference new
technology would continue to drive the petroleum sector. "It is estimated that the world has consumed
something like one trillion barrels of oil since the industry started in the nineteenth century,'' he said. "It is thought there are at least
five trillion barrels of petroleum potentially recoverable. " But
it is not just that oil continues to be discovered.
It is that technology, partly driven by prices, enables ever greater reserves to be booked, and
eventually recovered.

No Peak oil

YERGIN ’12 - is chairman of IHS Cambridge Energy Research Associates, an energy research and consulting firm. (Daniel, “There
Will Be Oil “. September 17. http://online.wsj.com/article/SB10001424053111904060604576572552998674340.html)

But there is another way to visualize the future availability of oil: as a "plateau." In this view, the world has decades of
further growth in production before flattening out into a plateau—perhaps sometime around midcentury
—at which time a more gradual decline will begin. And that decline may well come not from a
scarcity of resources but from greater efficiency, which will slacken global demand. Those sounding
the alarm over oil argue that about half the world's oil resources already have been produced and that the point of decline is
nearing. "It's quite a simple theory and one that any beer-drinker understands," said the geologist Colin Campbell, one of the leaders
of the movement. "The glass starts full and ends empty, and the faster you drink it, the quicker it's gone." This
is actually the
fifth time in modern history that we've seen widespread fear that the world was running out
of oil. The first was in the 1880s, when production was concentrated in Pennsylvania and it was said that no oil would be found
west of the Mississippi. Then oil was found in Texas and Oklahoma. Similar fears emerged after the two world wars. And in the
1970s, it was said that the world was going to fall off the "oil mountain." But since
1978, world oil output has
increased by 30%. Just in the years 2007 to 2009, for every barrel of oil produced in the world,
1.6 barrels of new reserves were added. And other developments—from more efficient cars
and advances in batteries, to shale gas and wind power—have . Yet the fear of peak oil maintains its
powerful grip.

No Peak Oil
Wolfgram 5, Ann F. junior fellow at Massey College – Phd in history from Toronto, “Population, Resources & Environment: A
Survey of the Debate”, January 1, http://www.voxfux.com/features/malthusian_theory/malthusian_theory.htm

The resource category of minerals is, by nature, varied and broad, encompassing minerals such as copper and coal. In recent years,
the mineral that has drawn the most public attention has been petroleum, particularly in reference to consumption and perceived
scarcity. Because it is such a well-known mineral, let us take petroleum as a case-in-point for minerals as related to the population-
resources question. Neo-Malthusian approach: In years past, the main concern coming from this sector was fear of total mineral
resource depletion. In an on-going public debate between Lester Brown, of the Neo-Malthusian school, and Julian Simon, Simon
wagered that mineral resources were not being depleted, because price, which reflects scarcity,
did not rise but declined in the long-term. Simon won the wager . (Simon’s position will be discussed later in
this section.) In recent years, the neo-Malthusian argument, especially with regard to petroleum has shifted from concern over
resource depletion to effects of mining and mineral usage on the environment. Fears over land degradation due to mining, air
pollution due to burning petroleum, water pollution due to oil spills and industry waste, among other things, are now the main
thrust of the neo-Malthusian argument with regard to minerals resources, petroleum in particular. These will be discussed in a later
section devoted to population and environment. Scientific evidence: According
to the U.S. Department of Energy (DOE),
domestic oil reserves have declined over the past decade . However, this should not naively be
thought to be a sign that the world is rapidly running out of oil. Rather, it means that less oil
was being produced by oil companies. The DOE pointed to several economic and industry trends that impacted
domestic reserves, such as the sharp decrease in drilling due to the collapse of crude oil prices in 1986, the shift within the
petroleum industry to drilling for natural gas, and restrictions on oil exploration in oil-prone places in the United States. (32)
Domestic and world oil resources are difficult to quantify in that, in addition to known high-grade resources, there
are lower-
grade oil reserves which can be tapped using new technologies, as well as oil fields that have yet
to be discovered. In 1995, the Department of Interior’s estimate for undiscovered recoverable
oil plus inferred resources of domestic crude oil was 132 billion barrels, which was six times
larger than the 1995 proven reserves. (33) It must also be remembered that the most oil reserves lie outside
of the United States. People-as-Problem-Solvers: Predictably, one of the responses of the human creativity/ technological
advancement proponents is that technological development will allow for a greater efficiency in the use
of minerals resources. However, there is a second dimension to technological development that they point to:
technological advancements may also mean less dependence on a given resource. For instance,
historically, wood and steam were the primary sources of energy prior to oil. With the advent of the internal combustion engine,
petroleum became the primary energy resource. Thus, the development of new technologies caused a shift in
the demand for certain resources. In the future, our sources of energy may be nuclear power, solar power or wind
power. As Julian Simon, a self-described optimist in these matters, argues, # trends in energy costs and scarcity have
been downward over the entire period for which we have data . And such trends are usually the
most reliable bases for forecasts. From these data we may conclude with considerable
confidence that energy will be less costly and more available in the future than in the past. The
reason that the cost of energy has declined in the long-run is the fundamental process of (1)
increased demand due to growth of population and income, which raises prices and hence
constitutes opportunity to entrepreneurs and inventors; (2) the search for new ways of supplying
the demand for energy; (3) the eventual discovery of methods which leave us better off than if
the original problem had not appeared . (34) Thus, according to Simon theory based on historical data, either new
technologies will develop, thereby lessening the need for more petroleum, or scarcity will
eventually arise, thus spurring invention and development of new technologies.

No peak – their studies are trash


Williams 3 (Bob, Executive Editor – Oil and Gas Journal, Oil and Gas Journal, 7-14, Lexis)

A number of prominent energy consultants, economists, and petroleum scientist have taken issue with the notion
that the world awaits an imminent peak in oil production. Thomas Ahlbrandt, world energy project chief with the US Geological
Survey in Denver, objects to the concept underlying the Hubbert curve. "Is
there an imminent oil peak? The
short answer is no," he said. "I believe in the plateau concept, which reconciles the need for additional resources within
the constraints of infrastructure and capital investment. "The symmetric rise and fall of oil production is not technically
supportable, as Hubber, Laherrere, and others have published, although generally not recognized by (Colin) Campbell, (Kenneth)
Deffeyes, and others who have been making draconian end-of-civilization claims since 1989
and every year since , , , Why is there no accountability for these failed forecasts either by Hubbert or
disciples such as Campbell, Laherrere, etc.?" Instead, Ahlbrandt and others point to even mature areas such as the UK North
Sea, which in the past 20 years has repeatedly defied forecasts of a bell-curve-style decline
(Figs. 6-7). And peak-oil critics also noted the surge in discoveries in areas deemed critical
for future supply, such as the deepwater Gulf of Mexico (Fig. 8). Sarah Emerson, managing
director, Energy Security Analysis Inc., Wakefield, Mass., is one of many energy economists
who contend that the Hubbert modelers disregard the roles of oil supply, demand, and
prices as well political and regulatory impacts. "I do not believe the peak in global oil
production is imminent," she told OGJ. ", , , The geologists who present the resource
scarcity argument tend to ignore changes in the economic context. For example, foreign
investment laws can change in countries with large reserves and limited access to capital or
technology. This means places where we never expected development (or expected slow development) suddenly open up. A list
of the countries who have opened up to foreign investment is an impressive who's who of producers: Russia, Azerbaijan,
Kazakhstan, Venezuela, now Iraq, and maybe even someday Kuwait and Saudi Arabia. New-found access to capital and
technology requires a total reappraisal of resource development." She contends that the global oil industry and market is
"incredibly dynamic, constantly changing as it responds to regulation and innovation. "The Hubbert curve analysis is
far too static to stand as a guiding assessment of the future of global oil supply. As with any 'model' results, it should be
one input into a broader, more comprehensive market analysis."

No solvency – change would have to be global and would, at minimum, take 50 years

Hertsgaard 4 (Mark, Renowned Author, Wash Post, 6-13, Lexis)

According to Roberts, a contributor to Harper's who visited Saudi Arabia and Azerbaijan and interviewed a range of industry,
government and private experts for this book, there is little dispute among insiders that an oil peak is inevitable someday; oil is,
after all, a finite resource. But there is considerable disagreement about when that will occur .
Optimists, such as those in the U.S. Geological Survey and the Energy Information Agency, foresee no peak before 2035.
"Pessimists, by contrast," Roberts writes, "a group whose members include geologists, industry analysts, and a surprising
number of oil industry and government officials, believe that a peak may come much sooner -- perhaps as soon as 2005." That
30-year difference is crucial. Toavoid the disaster scenario outlined above, the world must put in place
substitute sources of energy, and a system for delivering them, before the peak occurs.
Otherwise, shortages are certain and chaos likely. Establishing an alternative system will be no small
challenge, however, for it must displace the 40 percent of global energy demand that is
currently met by oil. Historically, human societies have needed about 50 years to shift from
one energy foundation to another. Wood, for example, gave way to coal during the early 19th century and coal to
oil in the mid-20th. Given how little the United States in particular has done so far to develop successful alternatives, one must
hope the optimists are right in saying that we have decades, not months or years, to leave oil behind.
Won’t cause war

Victor 7 (David G., Professor of Law – Stanford Law School and Director – Program on Energy and Sustainable Development,
“What Resource Wars?”, The National Interest, 11-12, http://www.nationalinterest.org/Article.aspx?id=16020)

RISING ENERGY
prices and mounting concerns about environmental depletion have animated fears that the
world may be headed for a spate of “resource wars”—hot conflicts triggered by a struggle to grab valuable
resources. Such fears come in many stripes, but the threat industry has sounded the alarm bells especially loudly in three areas.
First is the rise of China, which is poorly endowed with many of the resources it needs—such as oil, gas, timber and most
minerals—and has already “gone out” to the world with the goal of securing what it wants. Violent conflicts may follow as the
country shunts others aside. A second potential path down the road to resource wars starts with all the money now flowing into
poorly governed but resource-rich countries. Money can fund civil wars and other hostilities, even leaking into the hands of
terrorists. And third is global climate change, which could multiply stresses on natural resources and trigger water wars, catalyze
the spread of disease or bring about mass migrations. Most
of this is bunk, and nearly all of it has focused on the wrong
lessons for policy. Classic resource wars are good material for Hollywood screenwriters. They rarely occur in the
real world. To be sure, resource money can magnify and prolong some conflicts, but the root causes
of those hostilities usually lie elsewhere. Fixing them requires focusing on the underlying
institutions that govern how resources are used and largely determine whether stress explodes into violence. When
conflicts do arise, the weak link isn’t a dearth in resources but a dearth in governance .

Oil is plentiful and mutual interests check violence

Odell 4 (Peter, Professor Emeritus of International Energy Studies – Erasmus University, Why Carbon Fuels Will Dominate the 21 st
Century’s Global Energy Economy, p. xii-xiii)

Over the 21st century as a whole, a total of some 1660 Gigatons (_ 1660 x 10' tons) oil equivalent of
carbon energy will be produced and used, compared with a cumulative total in the 20th century of just under
500 Gigatons. This more-than-three-fold increase in the use of carbon energy in the 21st
century reflects not only the bountiful nature of the world's endowment of carbon energy
fuels, but also the willingness of the nations which are rich in coal, oil and/or natural gas to accept the depletion of their
"natural" resources, in return for the economic growth it generates for the countries concerned and the rising incomes it
secures for their populations. It also indicates
the managerial and technological achievements which
can be anticipated through the multitude of global regional and local entities responsible
for the extraction, the transportation and the processing of the world's energy resources .
The fundamental mutuality of the interests of the very many parties already involved in
such activities - albeit with temporary disturbances between them arising from economic and/or political difficulties (as
over the past 100 years) - will virtually ensure supply continuity at the levels required by demand
developments. In this set of defined circumstances for the exploitation of carbon energies ,
the concept of "resource wars" (Klare, 2002; Kleveman, 2003) becomes invalid, as such phenomena
are likely only in the context of a terminal scarcity of coal, oil and/or natural gas. This study
demonstrates that such scarcity is excludable, except on a local or regional scale from time to time, for the 21st
century.
Multiple factors check

Ismael 8 Hossein-zadeh, author of the recently published The Political Economy of U.S. Militarism teaches economics at Drake
University, Des Moines, July 9, 2008, Are They Really Oil Wars?, Iowa.http://www.counterpunch.org/zadeh07092008.html

Peak oil theory is based on a number of assumptions and omissions that make it less than
reliable. To begin with, it discounts or disregards the fact that energy-saving technologies have
drastically improved (and will continue to further improve) the efficiency of oil consumption. Evidence shows that, for
example, “over a period of five years (1994-99), U.S. GDP expanded over 20 percent while oil usage rose by only nine percent.
Before the 1973 oil shock, the ratio was about one to one.” Second,
Peak Oil theory pays scant attention to
the drastically enabling new technologies that have made (and will continue to make) possible
discovery and extraction of oil reserves that were inaccessible only a short time ago. One of
the results of the more efficient means of research and development has been a far higher success rate in finding new oil fields.
The success rate has risen in twenty years from less than 70 percent to over 80 percent.
Computers have helped to reduce the number of dry holes. Horizontal drilling has boosted extraction. Another important
development has been deep-water offshore drilling, which the new technologies now permit. Good examples are the North Sea,
the Gulf of Mexico, and more recently, the promising offshore oil fields of West Africa. Third, Peak Oil theory also
pays short shrift to what is sometimes called non-conventional oil. These include Canada's giant
reserves of extra-heavy bitumen that can be processed to produce conventional oil. Although this was originally considered cost
inefficient, experts
working in this area now claim that they have brought down the cost from
over $20 a barrel to $8 per barrel. Similar developments are taking place in Venezuela. It is thanks to
developments like these that since 1970, world oil reserves have more than doubled, despite the extraction of hundreds of
millions of barrels.
Peak Phosphorus Answers
No peak phosphorous

Schlaefi ‘11
(Samuel, worked for five years as a laboratory assistant in international companies before moving into journalism. He studied
journalism and organisational communications at the University of Applied Sciences Winterthur and Hamburg University of Applied
Sciences in Germany. “Phosphorus: Unsung bedrock of prosperity.” April 27. http://ourworld.unu.edu/en/phosphorus-unsung-
bedrock-of-prosperity/)

Roland Scholz, a professor from the Institute for Environmental Decisions, however, does not believe a peak is on the horizon: “As it
is, we know far too little about the natural reserves to be able to make a prediction like that.” The global oil
deposits are a different kettle of fish altogether; they have been researched thoroughly, so we can already make sound quantitative
estimates. As far as the calculations of the phosphate reserves are concerned, however, we are reliant upon data from
the industry. But mining is not the only way to obtain phosphorus; it exists in various forms and qualities.
Scholz found an indication of just how uncertain today’s estimations really are in a recent publication on
Morocco’s reserves: the authors concluded that they are actually twice as big as originally assumed. A trained
mathematician, Scholz especially criticises the type of the calculations used thus far. Like peak oil, peak phosphorus is calculated
using the Hubbard curve. However, this only makes sense under certain conditions; the available resources have to be known and
limited, and the demand function constant, for instance. According to Schulz, however,a model for calculating the
phosphorus peak also needs to include possible technological innovations in mineral
extraction. In calculating the copper deposits, for example, for a long time the estimations were too low because the experts
only assumed it could be extracted from ores containing at least two percent copper. Today, we process ores with a 0.2-percent
copper content. “But because there haven’t been many incentives for phosphorus until now, the potential sources of
innovation aren’t being exploited fully,” explains Scholz. Potential technical innovations in phosphorus recycling also
need to be factored into the equation, he says. After all, unlike with oil, phosphorus does not dwindle away.

No solvency: peak is inevitable. Plan only slows use – but they have no impact to a marginally
faster pace.

Phosphorus reserves are constantly growing – no chance of shortages

Lomborg 1 (Bjorn, Head – Copenhagen Consensus Center and Adjunct Professor – Copenhagen Business School, The Skeptical
Environmentalist, p. 144)

Phosphorus is a constituent of DNA and consequently indispensible for all forms of life. Often, the amount of phosphorus
available sets the limit of biological activity in natural systems. Phosphorus makes up about 1 percent of our raw material
expenditure. The phosphorus
reserves stand at about 90 years at current consumption, but
because phosphate rocks look like ordinary shales and limestones, even to exports, we can
anticipate discoveries of new, large deposits in the future. Recently, the US Geological
Survey announced the finding of phosphatic crusts and nodules in the offshore continental shelf of
Florida containing very large deposits, single-handedly doubling the phosphorus reserves to
about 180
years. Consequently, it is not expected that the availability of phosphorus will
become a limitation to food production.

Recapture solves

Hudnell 8 (Dr. Kenneth, Vice President and Director of Science – Solarbee Inc., Federal News Service, 7-10, Lexis)

But what we're trying now is where you have this floating platform pumping up all this water to put a metal ring around the
water where it's coming into the unit and run solar-powered electric charges that pulse positive and negative off this thing. And
what you can do is magnetize the phosphorous and attract it to these bars. And so if you can develop
a system that will attract the phosphorous in and then you can go and periodically exchange the rings, you can
take all the phosphorous off those metal rings -- (off mike). And you know this is just a very small-scale first attempt to do this,
but I think that potentially technology
will allow us to do very large-scale things like this at water
treatment plants, for example, and recapture that phosphorous. And we really have no alternative in the
future because the phosphorous is going to be depleted, and we cannot farm without phosphorous; there's no synthetic
alternative.

No shortage for 100 years

Hudnell 8 (Dr. Kenneth, Vice President and Director of Science – Solarbee Inc., Federal News Service, 7-10, Lexis)

So when you're talking about solutions for controls, you look at the causes. You can do something about nutrients -- you can
reduce nutrient input into our water. We need to do that for many reasons -- not only the HABs and water quality, but we
need to reduce nutrient input because phosphorous we're predicted to peak production in 30
years and be out of resources for phosphorous in 50 to 100 years. We need to be recapturing and
reusing these instead of let them run into the water and cause problems.

Phosphorous can be recaptured

Cordell 8 (Dana, Ph.D. Scholar – Institute for Sustainable Futures, “Phosphorus Recovery”,
http://phosphorusfutures.net/index.php?option=com_content&task=view&id=18&Itemid=32)

Unlike oil, which is lost once it is used, phosphorus can be recovered and used over and over again if
present in sufficient concentrations. Between the phosphate rock mine and the food in our stomachs, up to 80% of P is lost in
the process from fertilizer production, application on fields, uptake by crops, food processing and retailing and final
consumption. This tells us that in
addition to increasing efficiency in the entire process we can look for
opportunities to increase recovery of P in crop residues, food waste in supermarket
dumpsters and household bins, manure, human excreta, struvite and other sources such as bone
meal, ash and algae.
Poverty Answers
Poverty is rapidly decreasing

Wolf 3 (Martin, Associate Editor and Chief Economics Commentator – Financial Times “Are Global Poverty and Inequality Getting
Worse?”, Prospect Magazine, March, http://www.prospect-magazine.co.uk/article_details.php?i d=4982)

All data on incomes and income distribution are questionable, above all those generated in developing countries. But, contrary
to what you say, World Bank researchers have calculated the numbers in extreme poverty-less than
$1 a day-ona consistent basis, in recent studies . The data shows a decline since 1980 of 200m
people in the category of the absolutely poor. This is a fall from 31 per cent of the world's population to 20
per cent (not 24 per cent, which is the proportion in developing countries alone). That is a spectacularly rapid
fall in poverty by historical standards. It makes a nonsense of the idea that poverty alleviation has been blighted
by globalisation. Now turn to the even murkier area of inequality. Here you argue that if we exclude China and India, there is no
obvious trend in inequality. But why would one want to exclude two countries that contained about 60 per cent of the world's
poorest people two decades ago and still contain almost 40 per cent of the world's population today? To fail to give these giants
their due weight in a discussion ofglobal poverty alleviation or income distribution would be Hamlet without the prince. You
then write that changes in relative average incomes across countries are not what we are really interested in, "which is the
income distribution among all the world's people or households." This is wrong in itself. If a country's average income rises
rapidly, it does also possess greater means for improving the lot of the poor. Maybe the government refuses to use the
opportunity, but a successor government could. In any case, we do possess data on relative household incomes. In a Foreign
Affairs article, David Dollar and Aart Kraay of the World Bank report a big decline in world-wide income inequality since its peak
in about 1970. The study builds on work that goes back to 1820. The underlying method is to calculate the percentage gap
between a randomly selected individual and the world average. The more unequal the distribution of world income, the bigger
that gap becomes. They report that this gap peaked at 88 per cent of world average income in 1970, before falling to 78 per
cent in 1995, roughly where it was in 1950. The chief driver of changes in inequality among households is changes between
countries, not within them. This was also the finding of Branko Milanovic's study of global household income distribution
between 1988 and 1993, which you cite approvingly. You rely on this study to support the thesis of rising household inequality.
But it contains at least four defects. First, there are well-known inconsistencies between data on household expenditures and
national accounts. Second, the methods used generate no increase in rural real incomes in China, which is inconsistent with
most views of what actually happened. Third, the period of five years is very short. Fourth and most important, this was an
atypical period, because India had an economic upheaval in 1991, while China's growth was temporarily slowed by the
Tiananmen crisis. My conclusion is that the last two decades saw a decline not just in absolute poverty but
also in world-wide inequality among households. The chief explanation for this was the fast growth of China and, to a lesser
extent, of India. This progress was not offset by rising inequality within them. In the case of India there was no such rise. In
China there has been a rise in inequality in the more recent period of its growth, largely because of controls on the movement
of people from the hinterland to the coastal regions.

Poverty inevitable – alt causes overwhelm

Baker and Weisbrot 3 (Dean and Mark, Co-Directors – Center for Economic and Policy Research, “False Promises on Trade”,
7-25, http://www.commondreams.org/views03/0725-02.htm)

Similarly, most of sub-Saharan Africa is suffering from an un-payable debt burden. While there has been some limited relief
offered in recent years, the remaining debt burden is still more than the debtor countries spend on health care and education.
The list of problems imposed on developing countries can be extended at length bans on the
industrial policies that led to successful development in the west, the imposition of patents on drugs and
copyrights on computer software and recorded material, inappropriate macro-economic policies imposed
by the IMF and the World Bank. All of these factors are likely to have far more severe consequences
for the development prospects of low and middle-income countries than the agricultural policies of
rich countries.

The official poverty rate statistics distort actual poverty, overall trends show massive
improvement

Eberstadt, 08 – senior fellow at the American Enterprise Institute (Nicholas, The Poverty of “The Poverty Rate”
http://www.aei.org/docLib/20081117_PovertyofthePovertyRate.pdf OPR = Official Poverty Rate

Taken at face value, these stark numbers would seem to be a cause for dismay, if not outright
alarm. To go by the OPR, modern America has failed stunningly to lift the more vulnerable
elements of society out of deprivation— out from below the income line where, according to the author of the
federal poverty measure, “everyday living implied choosing between an adequate diet of the most economical sort and some
other necessity because there was not money enough to have both.”3 This statistical portrait of an apparent long-term rise in
absolute poverty in the contemporary United States evokes the specter of profound economic, social, and political dysfunction
in a highly affluent capitalist democracy. All the more troubling is the near-total failure of social policy implied by such numbers.
Despite the War on Poverty and all subsequent governmental antipoverty initiatives, official poverty rates for the nation have
mainly moved in the wrong direction over the past three decades. If these numbers cast a disturbing and unfamiliar picture of
their country for American citizens and policymakers, it is worth remembering that they would be regarded as utterly
unsurprising by many of the most convinced critics of the American system, who would regard such results as exactly what they
would expect. The apparent paradox of steady economic growth and persistent or increasing poverty, for example, conforms
rather well with some of the Marxian and neo-Marxist critiques of industrial and global capitalism, which accused such systems
of inherently generating “immiserating growth.”4 Among non-Marxists, these contours of the U.S. tableau are viewed today as
affirming the critique of what is known internationally as “neoliberal reform,” and providing powerful particulars for vigorously
rejecting the “American model.”5 But, as we shall demonstrate in the following chapters, the
social and economic
portrait afforded by America’s official poverty statistics is woefully distorted—almost
bizarrely miscast. In reality, the prevalence of absolute deprivation in the United States has
declined dramatically over the decades since the debut of the official poverty rate. In
reality, the purchasing power of lower-income households is far higher today than it was in
the 1960s or ’70s. In reality, the standard of living of the poverty population itself has
improved manifestly, decade by decade, since the federal poverty measure was first
introduced. The problem is, the statistical measure our democracy has devised for charting
our national performance against poverty does not register these basic realities—and
worse, cannot even recognize them.
Presidential Leadership Answers

Presidential leadership is impossible and has no impact

Miller ’12 (Foreign Policy Magazine MAGAZINE ARCHIVE FOLLOWLOGIN Where Have All the George Washingtons Gone? Five
reasons why America doesn’t have great presidents anymore. BY AARON DAVID MILLER | FEBRUARY 20, 2012

Greatness in the presidency is driven by severe crisis that encumbers the nation for a sustained period of time.
Washington, Lincoln, and Roosevelt were America's greatest presidents because they faced and overcame the three greatest
challenges in America's history: the birth and consolidation of the republic; civil war; and the Great Depression and World War II.
The Founders wanted a strong executive, but one who was accountable also, constrained by checks and balances as well as shared
and separated powers. The American system moves only when it's shocked, and it allows a president to tame what is an inherently
unruly political structure. But crisis only opens the door . Unless a president also has the character and the capacity to
know what to do and how to do it, they will fail. James Buchanan and Herbert Hoover weren't up to the challenges; Lincoln and FDR
were. Since FDR, America hasn't had the kind of emergency that has offered the chance for both
heroic crisis management and the fashioning of some transformational legacy that would change
the country in a fundamental way. America has had plenty of crises, to be sure, but none that have been
inescapable, relentless, and nation-encumbering. In fact, America's modern challenges have become routinized .
That's what happened with the Cold War, even with the 9/11 attacks. They became what presidential scholar Richard Neustadt
called "inside government crises" rather than outside crises. Indeed, where presidents have had great moments -- John F. Kennedy
over Cuba, Lyndon B. Johnson in the wake of JFK's death pushing transformative change on civil rights and social programs -- they
resulted from real crisis or trauma. The 9/11 attacks might have been such a moment, but it was wasted by a president who took the
country into discretionary war and away from using the tragedy to promote a sense of sustained national identity or purpose.
Obama had his economic crisis -- but without Depression-era 25 percent unemployment and bread lines, it wasn't sufficient to force
contemporary challenges are severe, but
Republicans to cooperate or allow for dramatic presidential action. America's
they'reslower bleeds. Debt, deficits, and decaying infrastructure produce delay and division -- not
consensus. And problems of this nature don't afford much space for heroic presidential action .
Balancing the budget isn't quite defeating Hitler, but the structural polarization in modern
politics makes even the former very hard to achieve. 4. Media trivializes. Greatness in the presidency requires a
certain amount of distance, detachment, even mystique. The media has been tough on presidents since George Washington -- but
not nearly as intrusive as it is now. FDR could hide his affairs and the extent of his disability because a willing press allowed him to;
Kennedy had the same free pass. Not today. America's 24/7, in-your-face media is relentless -- and presidents contribute to it by
believing they need to keep ahead of the curve and project their presence constantly. Lincoln gave four major speeches during his
presidency; FDR gave only four fireside chats during his first year. Barack Obama gave over 500 speeches and major remarks during
his first 365 days in office. Presidents today also succumb to the Oprah-style need to reveal and share things about themselves, a
curious feature of our modern age. Eighteenth- and 19th-century politicians didn't share. Washington, Lincoln, and FDR were public
men with impenetrable private interiors. Jefferson burned his letters to his late wife. Obama had written two bestselling
autobiographies before he set foot in the White House. A presidential Twitter news conference (July 6, 2011) may be smart politics
and communications, but somehow the notion of being the president in 140 characters just doesn't compute. The media offer
presidents great opportunities to get the message out. But the media also suck the mystery and aura out of the presidency. It's hard
to imagine now, as presidential historian Michael Beschloss reminds us, but when JFK addressed the country during the Cuban
missile crisis, the networks went back to normal programming. That meant there was no media mediation, no talking-head
commentary interposing itself between the public and the president. Americans were left to come to terms with the president and
his words, by themselves. 5. It's a big world out there. Modern
presidents face a world that's largely beyond
their control. The new globalized, integrated world impinges significantly, directly, and often immediately on important
domestic issues in a way their predecessors could never even imagine. As an expanding continental power, America faced troubles
enough. Both Washington and Lincoln had to deal with galactic challenges of nation-building and civil war, and they even had to
contend with the great powers of their day in North America and at sea. As monumental as these challenges were, they were still
contained. They were continental problems that offered up at least the possibility of continental solutions. With largely
nonpredatory neighbors to its north and south, and oceans and fish to its east and west ("our liquid assets," one historian once
called them), U.S. presidents had more control and a better margin for success. The foreign-policy challenges -- crisis with Spain,
intervention in Mexico, preventing European influence in Latin America, and World War I -- faced by the first three presidents to get
America's feet wet in the world (McKinley, Theodore Roosevelt, and Woodrow Wilson) paled before the complexity of those that the
second Roosevelt would encounter, let alone his successors. Put aside for just a minute the gigantic size of America's economic,
political, cultural, and security footprint in the world -- the latter, alone, which has the country deployed militarily to hundreds of
bases around the world. Just think about the dependency and interconnectedness of the United States in the world today. Nineteen
terrorists attacked the Twin Towers and the Pentagon; within a couple of years, America found itself bogged down in what would
become the two longest wars in its history. Or what about Greece, a place the founders looked to as a source of philosophical and
intellectual inspiration? It now exports bad debt that can roil U.S. financial markets with even a hint of possible default. America
doesn't control the world. And its presidents aren't action-adventure superheroes who can
impose their will with words and deeds. The gap between expectations and delivery has always
been a tough one to close, and the demand for great leaders has always exceeded the supply .
Americans need to get a grip and dial down what they expect of the presidency and those who
occupy it. Maybe then Americans can allow their presidents to be good without expecting them always to be great.
Prolif Answers
Frontline

No prolif cascades and long timeframe – their ev is biased

Kahl 13 – Senior Fellow at the Center for a New American Security and an associate professor in the Security Studies Program
at Georgetown University’s Edmund A. Walsh School of Foreign Service (Colin H., Melissa G. Dalton, Visiting Fellow at the Center for
a New American Security, Matthew Irvine, Research Associate at the Center for a New American Security, February, “If Iran Builds
the Bomb, Will Saudi Arabia Be Next?” http://www.cnas.org/files/documents/publications/CNAS_AtomicKingdom_Kahl.pdf)

***cites Jacques Hymans, USC Associate Professor of IR***

I I I . LESSONS FROM HISTORY Concerns over “regional proliferation chains,” “falling nuclear dominos” and “nuclear tipping points”
are nothing new; indeed, reactive proliferation fears date back to the dawn of the nuclear age.14 Warnings of an inevitable deluge
of proliferation were commonplace from the 1950s to the 1970s, resurfaced during the discussion of “rogue states” in the 1990s and
became even more ominous after 9/11.15 In 2004, for example, Mitchell Reiss warned that “in ways both fast and slow, we may
very soon be approaching a nuclear ‘tipping point,’ where many countries may decide to acquire nuclear arsenals on short notice,
thereby triggering a proliferation epidemic.” Given the presumed fragility of the nuclear nonproliferation regime and the ready
supply of nuclear expertise, technology and material, Reiss argued, “a single new entrant into the nuclear club could catalyze similar
responses by others in the region, with the Middle East and Northeast Asia the most likely candidates.”16 Nevertheless,
predictions of inevitable proliferation cascades have historically proven false (see The Proliferation Cascade
Myth text box). In the six decades since atomic weapons were first developed, nuclear restraint has proven far more common than
nuclear proliferation, and cases of reactive proliferation have been exceedingly rare. Moreover, most countries that have
started down the nuclear path have found the road more difficult than imagined, both technologically
and bureaucratically, leading the majority of nuclear-weapons aspirants to reverse course. Thus, despite frequent
warnings of an unstoppable “nuclear express,”17 William Potter and Gaukhar Mukhatzhanova astutely note that the “train to date
has been slow to pick up steam, has made fewer stops than anticipated, and usually has arrived much later than expected.”18 None
of this means that additional proliferation in response to Iran’s nuclear ambitions is inconceivable, but the empirical record does
suggest that regional chain reactions are not inevitable. Instead, only certain countries are candidates for reactive proliferation.
Determining the risk that any given country in the Middle East will proliferate in response to Iranian nuclearization requires an
assessment of the incentives and disincentives for acquiring a nuclear deterrent, the technical and bureaucratic constraints and the
available strategic alternatives. Incentives and Disincentives to Proliferate Security considerations, status and reputational concerns
and the prospect of sanctions combine to shape the incentives and disincentives for states to pursue nuclear weapons. Analysts
predicting proliferation cascades tend to emphasize
the incentives for reactive proliferation while ignoring or
downplaying the disincentives. Yet, as it turns out, instances of nuclear proliferation (including reactive
proliferation) have been so rare because going down this road often risks insecurity, reputational
damage and economic costs that outweigh the potential benefits.19 Security and regime survival are
especially important motivations driving state decisions to proliferate. All else being equal, if a state’s leadership believes that a
nuclear deterrent is required to address an acute security challenge, proliferation is more likely.20 Countries in conflict-prone
neighborhoods facing an “enduring rival”– especially countries with inferior conventional military capabilities vis-à-vis their
opponents or those that face an adversary that possesses or is seeking nuclear weapons – may be particularly prone to seeking a
nuclear deterrent to avert aggression.21 A recent quantitative study by Philipp Bleek, for example, found that security threats, as
measured by the frequency and intensity of conventional militarized disputes, were highly correlated with decisions to launch
nuclear weapons programs and eventually acquire the bomb.22 The Proliferation Cascade Myth Despite repeated warnings since the
dawn of the nuclear age of an inevitable deluge of nuclear proliferation, such fears have thus far proven largely unfounded.
Historically, nuclear restraint is the rule, not the exception – and the degree of restraint has actually increased over time. In the first
two decades of the nuclear age, five nuclear-weapons states emerged: the United States (1945), the Soviet Union (1949), the United
Kingdom (1952), France (1960) and China (1964). However, in the nearly 50 years since China developed nuclear weapons, only four
additional countries have entered (and remained in) the nuclear club: Israel (allegedly in 1967), India (“peaceful” nuclear test in
1974, acquisition in late-1980s, test in 1998), Pakistan (acquisition in late-1980s, test in 1998) and North Korea (test in 2006).23 This
significant slowdown in the pace of proliferation occurred despite the widespread dissemination of nuclear know-how and the fact
that the number of states with the technical and industrial capability to pursue nuclear weapons programs has significantly
increased over time.24 Moreover, in the past 20 years, several states have either given up their nuclear weapons (South Africa and
the Soviet successor states Belarus, Kazakhstan and Ukraine) or ended their highly developed nuclear weapons programs (e.g.,
Argentina, Brazil and Libya).25 Indeed, by one estimate, 37 countries have pursued nuclear programs with possible weaponsrelated
dimensions since 1945, yet the overwhelming number chose to abandon these activities before they produced a bomb. Over time,
the number of nuclear reversals has grown while the number of states initiating programs with
possible military dimensions has markedly declined.26 Furthermore – especially since the Nuclear Non-Proliferation Treaty
(NPT) went into force in 1970 – reactive proliferation has been exceedingly rare. The NPT has near-universal membership among the
community of nations; only India, Israel, Pakistan and North Korea currently stand outside the treaty. Yet the actual and suspected
acquisition of nuclear weapons by these outliers has not triggered widespread reactive proliferation in their respective
neighborhoods. Pakistan followed India into the nuclear club, and the two have engaged in a vigorous arms race, but Pakistani
nuclearization did not spark additional South Asian states to acquire nuclear weapons. Similarly, the North Korean bomb did not lead
South Korea, Japan or other regional states to follow suit.27 In the Middle East, no country has successfully built a nuclear weapon in
the four decades since Israel allegedly built its first nuclear weapons. Egypt took initial steps toward nuclearization in the 1950s and
then expanded these efforts in the late 1960s and 1970s in response to Israel’s presumed capabilities. However, Cairo then ratified
the NPT in 1981 and abandoned its program.28 Libya, Iraq and Iran all pursued nuclear weapons capabilities, but only Iran’s program
persists and none of these states initiated their efforts primarily as a defensive response to Israel’s presumed arsenal.29 Sometime
in the 2000s, Syria also appears to have initiated nuclear activities with possible military dimensions, including construction of a
covert nuclear reactor near al-Kibar, likely enabled by North Korean assistance.30 (An Israeli airstrike destroyed the facility in
2007.31) The motivations for Syria’s activities remain murky, but the nearly 40-year lag between Israel’s alleged development of the
bomb and Syria’s actions suggests that reactive proliferation was not the most likely cause. Finally, even countries that start on the
nuclear path have found it very difficult, and exceedingly time consuming, to reach the end. Of the 10 countries that launched
nuclear weapons projects after 1970, only three (Pakistan, North Korea and South Africa) succeeded; one (Iran) remains in progress,
and the rest failed or were reversed.32 The successful projects have also generally needed much more time than expected to finish.
According to Jacques Hymans, the
average time required to complete a nuclear weapons program has
increased from seven years prior to 1970 to about 17 years after 1970, even as the hardware, knowledge and industrial
base required for proliferation has expanded to more and more countries.33 Yet throughout the nuclear age, many states with
potential security incentives to develop nuclear weapons have nevertheless abstained from doing so.34 Moreover, contrary to
common expectations, recent statistical research shows that states with an enduring rival that possesses or is pursuing nuclear
weapons are not more likely than other states to launch nuclear weapons programs or go all the way to acquiring the bomb,
although they do seem more likely to explore nuclear weapons options.35 This suggests that a rival’s acquisition of nuclear weapons
does not inevitably drive proliferation decisions. One reason that reactive proliferation is not an automatic response to a rival’s
acquisition of nuclear arms is the fact that security calculations can cut in both directions. Nuclear weapons might deter outside
threats, but leaders have to weigh these potential gains against the possibility that seeking nuclear
weapons would make the country or regime less secure by triggering a regional arms race or a preventive
attack by outside powers. Countries also have to consider the possibility that pursuing nuclear weapons will
produce strains in strategic relationships with key allies and security patrons. If a state’s leaders conclude that their overall
security would decrease by building a bomb, they are not likely to do so.36 Moreover, although security considerations are often
central, they are rarely sufficient to motivate states to develop nuclear weapons. Scholars have noted the importance of other
factors, most notably the perceived effects of nuclear weapons on a country’s relative status and influence.37 Empirically, the most
highly motivated states seem to be those with leaders that simultaneously believe a nuclear deterrent is essential to counter an
existential threat and view nuclear weapons as crucial for maintaining or enhancing their international status and influence. Leaders
that see their country as naturally at odds with, and naturally equal or superior to, a threatening external foe appear to be especially
prone to pursuing nuclear weapons.38 Thus, as Jacques Hymans argues, extreme levels of fear and pride often “combine to produce
a very strong tendency to reach for the bomb.”39 Yet here too, leaders contemplating acquiring nuclear weapons have to balance
the possible increase to their prestige and influence against the normative and reputational costs associated with violating the
Nuclear Non-Proliferation Treaty (NPT). If a country’s leaders fully embrace the principles and norms embodied in the NPT, highly
value positive diplomatic relations with Western countries and see membership in the “community of nations” as central to their
national interests and identity, they are likely to worry that developing nuclear weapons would damage (rather than bolster) their
reputation and influence, and thus they will be less likely to go for the bomb.40 In contrast, countries with regimes or ruling
coalitions that embrace an ideology that rejects the Western dominated international order and prioritizes national self-reliance and
autonomy from outside interference seem more inclined toward proliferation regardless of whether they are signatories to the
NPT.41 Most countries appear to fall in the former category, whereas only a small number of “rogue” states fit the latter. According
to one count, before the NPT went into effect, more than 40 percent of states with the economic resources to pursue nuclear
programs with potential military applications did so, and very few renounced those programs. Since the inception of the
nonproliferation norm in 1970, however, only 15 percent of economically capable states have started such programs, and nearly 70
percent of all states that had engaged in such activities gave them up.42 The prospect of being targeted with economic sanctions by
powerful states is also likely to factor into the decisions of would-be proliferators. Although sanctions alone proved insufficient to
dissuade Iraq, North Korea and (thus far) Iran from violating their nonproliferation obligations under the NPT, this does not
necessarily indicate that sanctions are irrelevant. A potential proliferator’s vulnerability to sanctions must be considered. All else
being equal, the more vulnerable a state’s economy is to external pressure, the less likely it is to pursue nuclear weapons. A
comparison of states in East Asia and the Middle East that have pursued nuclear weapons with those that have not done so suggests
that countries with economies that are highly integrated into the international economic system – especially those dominated by
ruling coalitions that seek further integration – have historically been less inclined to pursue nuclear weapons than those with
inward-oriented economies and ruling coalitions.43 A state’s vulnerability to sanctions matters, but so too does the leadership’s
assessment regarding the probability that outside powers would actually be willing to impose sanctions. Some would-be
proliferators can be easily sanctioned because their exclusion from international economic transactions creates few downsides for
sanctioning states. In other instances, however, a state may be so vital to outside powers – economically or geopolitically – that it is
unlikely to be sanctioned regardless of NPT violations. Technical and Bureaucratic Constraints In addition to motivation to pursue the
bomb, a state must have the technical and bureaucratic wherewithal to do so. This capability is partly a function of wealth. Richer
and more industrialized states can develop nuclear weapons more easily than poorer and less industrial ones can; although as
Pakistan and North Korea demonstrate, cash-strapped states can sometimes succeed in developing nuclear weapons if they are
willing to make enormous sacrifices.44 A country’s technical know-how and the sophistication of its civilian nuclear program also
help determine the ease and speed with which it can potentially pursue the bomb. The existence of uranium deposits and related
mining activity, civilian nuclear power plants, nuclear research reactors and laboratories and a large cadre of scientists and engineers
trained in relevant areas of chemistry and nuclear physics may give a country some “latent” capability to eventually produce nuclear
weapons. Mastery of the fuel-cycle – the ability to enrich uranium or produce, separate and reprocess plutonium – is particularly
important because this is the essential pathway whereby states can indigenously produce the fissile material required to make a
nuclear explosive device.45 States must also possess the bureaucratic capacity and managerial culture to successfully complete a
nuclear weapons program. Hymans convincingly argues that many recent would-be proliferators have weak state institutions that
permit, or even encourage, rulers to takea coercive, authoritarian management approach to their nuclear
programs. This approach, in turn, politicizes and ultimately undermines nuclear projects by gutting the
autonomy and professionalism of the very scientists, experts and organizations needed to successfully
build the bomb.46 Alternative Sources of Nuclear Deterrence Historically, the availability of credible security guarantees by
outside nuclear powers has provided a potential alternative means for acquiring a nuclear deterrent without many of the risks and
costs associated with developing an indigenous nuclear weapons capability. As Bruno Tertrais argues, nearly all the states that
developed nuclear weapons since 1949 either lacked a strong guarantee from a superpower (India, Pakistan and South Africa) or did
not consider the superpower’s protection to be credible (China, France, Israel and North Korea). Many other countries known to
have pursued nuclear weapons programs also lacked security guarantees (e.g., Argentina, Brazil, Egypt, Indonesia, Iraq, Libya,
Switzerland and Yugoslavia) or thought they were unreliable at the time they embarked on their programs (e.g., Taiwan). In contrast,
several potential proliferation candidates appear to have abstained from developing the bomb at least
because of formal or informal extended deterrence guarantees from the United States (e.g., Australia,
partly
Germany, Japan, Norway, South Korea and Sweden).47 All told, a recent quantitative assessment by Bleek finds that security
assurances have empirically significantly reduced proliferation proclivity among recipient countries.48 Therefore, if
a country perceives that a security guarantee by the United States or another nuclear power is both available and credible, it is less
likely to pursue nuclear weapons in reaction to a rival developing them. This option is likely to be particularly attractive to states that
lack the indigenous capability to develop nuclear weapons, as well as states that are primarily motivated to acquire a nuclear
deterrent by security factors (as opposed to status-related motivations) but are wary of the negative consequences of proliferation.

Prolif doesn’t cause war

Forsyth ’12 [James Wood Forsyth Jr., PhD, currently serves as professor of national security studies, USAF School of Advanced
Air and Space Studies, Maxwell AFB, Alabama. He earned his PhD at the Josef Korbel School of International Studies, University of
Denver. He has written on great-power war, intervention, and nuclear issues, “The Common Sense of Small Nuclear Arsenals,”
Summer, Strategic Studies Quarterly, http://www.au.af.mil/au/ssq/2012/summer/forsyth.pdf]

Whatever its logical shortcomings, it is important to stress that deterrence worked—it kept the Cold War “cold”
and allowed international life to go on without a catastrophic nuclear war. After 70 years, most
analysts agree on the basic dynamics of deterrence , and the contemporary debate regarding deterrence, when
not addressing the problem of nonstate actors, tends to pivot on force structure considerations. 19 Here, the behavior of
states with small nuclear arsenals is instructive . As previously mentioned, most states with nuclear
arsenals have not acquired large numbers of nuclear weapons. Instead, they appear content
with a relatively small arsenal capable of warding off an attack as well as dissuading others
from interfering in their internal and external affairs . But of the two roles nuclear weapons seem to play—
deterrence and dissuasion—is one more important than another? For India and Pakistan, nuclear weapons play a decidedly deter
rent role. But if one were to free Britain of its NATO obligations, who exactly would Britain be deterring today? What about France?
Neither of these countries is as hard-pressed in the security arena as India or Pakistan, yet both hold on to nuclear weapons. While
nuclear weapons still “hold power at bay,” one must wonder whose power is being held at bay and how. It is important not to
overinterpret this. Nuclear weapons serve a purpose. How else can one explain why nine states have them, while others appear to
want them? But what purpose do they serve, in general? To answer that question, one must look at what nuclear weapons do for
states. Among other things, nuclear
weapons socialize leaders to the dangers of adventurism and , in
effect, halt them from behaving or responding recklessly to provocation. 20 Statesmen may not want
to be part of an international system that constrains them, but that is the system that results among nuclear powers. Each is
socialized to the capabilities of the other, and the
relationship that emerges is one tempered by caution
despite the composition, goals, or desires of its leaders. In short, nuclear weapons deter and
dissuade.

No prolif—countries don’t want nukes

Mueller ‘11 [John, professor of Political Science at Ohio State University, “'Clocking' Nuclear Weapons” International Relations
and Security Network, online]

It is difficult to see how any country that has possessed nuclear weapons has found them
beneficial since World War II. They have supplied little diplomatic advantage, and no nuclear-armed
country has discovered an effective use for them in the many wars waged in places like Vietnam, Korea, Afghanistan, the Falklands,
Algeria, Lebanon, Kashmir, Kosovo, Iraq, Grenada, Panama and Hungary. Nor have they been useful in deterring war. Their supposed
chief achievement was to prevent World War III during the Cold War, but this notion continues to be undercut with each leak from
Soviet archives. Although highly sympathetic to revolutionary and civil war violence, Soviet ideology dismissed direct war against the
capitalist world, whether nuclear or not, as stupendously stupid. That is, there was nothing for the nukes to deter. Those who
experienced World War II scarcely needed visions of mushroom clouds to realize that it was imperative to be cautious about major
war. The weapons' uselessness also helps to explain why alarmists have been wrong for
decades about the pace of nuclear proliferation. Dozens of countries have been technologically
capable of obtaining nuclear arsenals, but very few have done so . Although international
efforts have often been given credit for this, the chief rea son seems to be that countries
considering developing the weapons have found them to be a foolish and wasteful expenditure.
Most, like Japan, joined the nonprolifera tion regime only after deciding they didn't want the
weapons anyway. And some of those that tried to get the weapons eventually gave up in
frustration . Thus when Libya's self-important dictator, Muammar Gaddafi, abruptly abandoned his nuclear program in 2003,
inspectors found much of his laboriously-acquired material still in its packing boxes. In the last decades, in fact, more countries have
abandoned nuclear weapons programs than have taken them up. Although happy to take the bribes offered to them for so doing,
none has yet found that abandoning the weapons has been disadvantageous.

Prolif will fail and will be incredibly slow- too many barriers

Hymans ‘12 [Jacques E. C. Hymans, PhD from Harvard, Associate Professor of International Relations at the University of
Southern California, his most recent book is Achieving Nuclear Ambitions: Scientists, Politicians, and Proliferation, “Botching the
Bomb: Why Nuclear Weapons Programs Often Fail on Their Own-and Why Iran's Might, Too,” Foreign Affairs91. 3 (May/Jun 2012):
44-53, Proquest]

THE CHRONIC problem of nuclear proliferation is once again dominating the news. A fierce debate has developed over how to
respond to the threat posed by Iran's nuclear activities, which most experts believe are aimed at producing a nuclear weapon or at
least the capacity to assemble one. In this debate, one side is pushing for a near-term military attack to damage or destroy Iran's
nuclear program, and the other side is hoping that strict sanctions against the Islamic Republic will soften it up for a diplomatic
solution. Both sides, however, share the underlying assumption that unless outside powers intervene in a dramatic fashion, it is
inevitable that Iran will achieve its supposed nuclear goals very soon. Yet there is another possibility. The Iranians
had to
work for 25 years just to start accumulating uranium enriched to 20 percent, which is not even
weapons grade. The slow pace of Iranian nuclear progress to date strongly suggests that Iran
could still need a very long time to actually build a bomb -- or could even ultimately fail to do
so . Indeed, global trends in proliferation suggest that either of those outcomes might be more
likely than Iranian success in the near future. Despite regular warnings that proliferation is spinning
out of control, the fact is that since the 1970s, there has been a persistent slowdown in the
pace of technical progress on nuclear weapons projects and an equally dramatic decline in
their ultimate success rate . The great proliferation slowdown can be attributed in part to U.S.
and international nonproliferation efforts. But it is mostly the result of the dysfunctional
management tendencies of the states that have sought the bomb in recent decades. Weak
institutions in those states have permitted political leaders to unintentionally undermine the
performance of their nuclear scientists, engineers, and technicians. The harder politicians have
pushed to achieve their nuclear ambitions, the less productive their nuclear programs have
become . Meanwhile, military attacks by foreign powers have tended to unite politicians and scientists in a common cause to
build the bomb. Therefore, taking radical steps to rein in Iran would be not only risky but also potentially counterproductive, and
much less likely to succeed than the simplest policy of all: getting out of the way and allowing the Iranian nuclear program's worst
enemies -- Iran's political leaders -- to hinder the country's nuclear progress all by themselves. NUCLEAR DOGS THAT HAVE NOT
BARKED "TODAY, ALMOST any industrialized country can produce a nuclear weapon in four to five
years," a former chief of Israeli military intelligence recently wrote in The New York Times,
echoing a widely held belief. Indeed, the more nuclear technology and know-how have diffused
around the world, the more the timeline for building a bomb should have shrunk. But in fact,
rather than speeding up over the past four decades, proliferation has gone into slow motion.
Seven countries launched dedicated nuclear weapons projects before 1970, and all seven succeeded in relatively short order. By
contrast, of the ten countries that have launched dedicated nuclear weapons projects since 1970, only three have achieved a bomb.
And only one of the six states that failed -- Iraq -- had made much progress toward its ultimate goal by the time it gave up trying.
(The jury is still out on Iran's program.) What is more, even the successful projects of recent decades have needed a long time to
achieve their ends. The average timeline to the bomb for successful projects launched before 1970 was about seven years; the
average timeline to the bomb for successful projects launched after 1970 has been about 17 years. International security experts
have been unable to convincingly explain this remarkable trend. The first and most credible conventional explanation is that the
Nuclear Nonproliferation Treaty (NPT) has prevented a cascade of new nuclear weapons states by creating a system of export
controls, technology safeguards, and on-site inspections of nuclear facilities. The NPT regime has certainly closed off the most
straightforward pathways to the bomb. However, the NPT became a formidable obstacle to would-be nuclear states only in the
1990s, when its export-control lists were expanded and Western states finally became serious about enforcing them and when
international inspectors started acting less like tourists and more like detectives. Yet the
proliferation slowdown
started at least 20 years before the system was solidified . So the NPT, useful though it may be,
cannot alone account for this phenomenon.
---EXT No Prolif

1NC Kahl and Mueller—no acquisition—leaders are dissuaded from nukes because costs
outweigh benefits—seen as wasting resources without significant diplomatic gains—

No prolif- too many disincentives to go nuclear

Mueller ‘9 (John – Woody Hayes chair of national security studies at Ohio State University, Atomic Obsession, p. 103)

It rather appears that, insofar as most leaders of most countries (even rogue ones) have considered
acquiring the weapons, they have come to appreciate several defects. Among them the
weapons are dangerous, distasteful, costly, and likely to rile the neighbors . If one values
economic growth and prosperity above all, the sensible thing seems to be to avoid the weapons unless
they seem vital for security or are required to stoke a leader’s extravagant ego. That is, as one observer puts it, “ there have
always been quite powerful disadvantages to acquiring nuclear weapons, costs that countries
would not wish to bear unless they felt extremely vulnerable or extremely cocky.” And the result, notes weapons
inspector David Kay tersely, is that a considerable number of states “ have largely on their own
decided that nuclear arms do not offer them any real benefits .”2 This chapter assesses the quite
considerable and significant consequential disincentives to go nuclear.

No prolif- their evidence is based on flawed predictions

Gavin 10 (Francis, Tom Slick Professor of International Affairs and Director of the Robert S. Strauss Center for International
Security and Law @ the Lyndon B. Johnson School of Public Affairs @ the University of Texas at Austin, “Sam As It Ever Was; Nuclear
Alarmism, Proliferation, and the Cold War,” Lexis)

Fears of a tipping point were especially acute in the aftermath of China's 1964 detonation of an atomic bomb: it
was predicted that India, Indonesia, and Japan might follow, with consequences worldwide, as "Israel,
Sweden, Germany, and other potential nuclear countries far from China and India would be affected by
proliferation in Asia." 40 A U.S. government document identified "at least eleven nations (India, Japan, Israel, Sweden, West
Germany, Italy, Canada, Czechoslovakia, East Germany, Rumania, and Yugoslavia)" with the capacity to go nuclear, a number
that would soon "grow substantially" to include "South Africa, the United Arab Republic, Spain, Brazil and Mexico." 41 A top-
secret, blue-ribbon committee established to craft the U.S. response contended that "the [1964] Chinese nuclear explosion has
increased the urgency and complexity of this problem by creating strong pressures to develop independent nuclear forces,
These predictions were
which, in turn, could strongly influence the plans of other potential nuclear powers." 42
largely wrong. In 1985 the National Intelligence Council noted that for "almost thirty years the Intelligence Community has
been writing about which nations might next get the bomb." All of these estimates based their largely pessimistic
and ultimately incorrect estimates on factors such as the increased "access to fissile materials,"
improved technical capabilities in countries, the likelihood of "chain reactions ," or a "scramble" to
proliferation when "even one additional state demonstrates a nuclear capability." The 1985 report goes on, "The most striking
characteristic of the present-day nuclear proliferation scene is that, despite the alarms rung by past Estimates, no additional
overt proliferation of nuclear weapons has actually occurred since China tested its bomb in 1964." Although"some
proliferation of nuclear explosive capabilities and other major proliferation-related developments have taken place
in the past two decades," they did not have "the damaging, systemwide impacts that the
Intelligence community generally anticipated they would." 43 In his analysis of more than sixty
years of failed efforts to accurately predict nuclear prolif eration, analyst Moeed Yusuf concludes
that "the pace of proliferation has been much slower than anticipated by most." The majority of countries suspected of
trying to obtain a nuclear weapons capability " never even came close to crossing the
threshold . In fact, most did not even initiate a weapons program ." If all the countries that were
considered prime suspects over the past sixty years had developed nuclear weapons, "the world would have at least 19 nuclear
powers today." 44 As Potter and Mukhatzhanova argue, government and academic experts frequently
"exaggerated the scope and pace of nuclear weapons proliferation." 45 Nor is there compelling
evidence that a nuclear proliferation chain reaction will ever occur. Rather , the pool of
potential proliferators has been shrinking . Proliferation pressures were far greater during
the Cold War. In the 1960s, at least twenty-one countries either had or were considering nuclear
weapons research programs. Today only nine countries are known to have nuclear weapons. Belarus,
Brazil, Kazakhstan, Libya, South Africa, Sweden, and Ukraine have dismantled their weapons
programs. Even rogue states that are/were a great concern to U.S. policymakers--Iran, Iraq, Libya, and North Korea--
began their nuclear weapons programs before the Cold War had ended. 46 As far as is known, no
nation has started a new nuclear weapons program since the demise of the Soviet Union in
19 91 . 47 Ironically, by focusing on the threat of rogue states, policymakers may have underestimated the potentially far more
destabilizing effect of proliferation in "respectable" states such as Germany, Japan, South Korea, and Taiwan.

No prolif- countries don’t want nukes empirics prove

Tepperman 9 [Jonathan, Newsweek International's first Assistant Managing Editor (now Deputy Editor), “Why Obama Should
Learn to Love the Bomb” 8-29, http://www.newsweek.com/2009/08/28/why-obama-should-learn-to-love-the-bomb.html]

The risk of an arms race—with, say, other Persian Gulf states rushing to build a bomb after Iran got one—is a bit harder to
years, the most nuclear-weapons states we've ever
dispel. Once again, however, history is instructive. "In 64
had is 12," says Waltz. "Now with North Korea we're at nine. That's not prolif eration; that's
spread at glacial pace ." Nuclear weapons are so controversial and expensive that only
countries that deem them absolutely critical to their survival go through the extreme trouble
of acquiring them. That's why South Africa, Ukraine, Belarus, and Kazakhstan voluntarily
gave theirs up in the early '90s, and why other countries like Brazil and Argentina dropped
nascent programs. This doesn't guarantee that one or more of Iran's neighbors—Egypt or Saudi Arabia, say—might not
still go for the bomb if Iran manages to build one. But the risks of a rapid spread are low, especially given Secretary of State
Hillary Clinton's recent suggestion that the United States would extend a nuclear umbrella over the region, as Washington has
over South Korea and Japan, if Iran does complete a bomb. If one or two Gulf states nonetheless decided to pursue their own
weapon, that still might not be so disastrous, given the way that bombs tend to mellow behavior. 

Better studies
Potter and Mukhatzhanova ‘8 (William, Sam Nunn and Richard Lugar Prof. Nonproliferation Studies and Dir. James
Martin Center for Nonproliferation Studies – Monterey Institute of International Studies, and Guakhar, Research Associate – James
Martin Center, International Security, “Divining Nuclear Intentions: A Review Essay”, 33:1, Summer, Project Muse)

Today it
is hard to find an analyst or commentator on nuclear proliferation who is not pessimistic about
the future. It
is nearly as difficult to find one who predicts the future without reference to metaphors
such as proliferation chains, cascades, dominoes, waves, avalanches, and tipping points.42 The lead
author of this essay also has been guilty of the same tendency , and initially named an ongoing research
project on forecasting proliferation he directs "21st Century Nuclear Proliferation Chains and Trigger Events." As both authors
proceeded with research on the project, however, and particularly after reading the books by Hymans and
Solingen, we became convinced that the metaphor is inappropriate and misleading, as it implies
a process of nuclear decisionmaking and a pace of nuclear weapons spread that are unlikely to
transpire. The current alarm about life in a nuclear-armed crowd has many historical antecedents and can be
found in classified National Intelligence Estimates (NIEs) as well as in scholarly analyses. The 1957 NIE, for example,
identified a list of ten leading nuclear weapons candidates , including Canada, Japan, and Sweden.43
Sweden, it predicted, was "likely to produce its first weapons in about 1961, " while it was estimated
that Japan would "probably seek to develop weapons production programs within the next decade."44 In one of the [End Page 159]
most famous forecasts, President John Kennedy in 1963 expressed a nightmarish vision of a future world
with fifteen, twenty, or twenty-five nuclear weapons powers.45 A number of the earliest scholarly projections of
proliferation also tended to exaggerate the pace of nuclear weapons spread. A flurry of studies between 1958 and
1962, for example, focused on the "Nth Country Proble m" and identified as many as twelve candidates capable
of going nuclear in the near future.46 Canada, West Germany, Italy, Japan, Sweden, and Switzerland were among the states most
The "peaceful nuclear explosion" by India in 1974 was seen by
frequently picked as near-term proliferators.
many analysts of the time as a body blow to the young NPT that would set in motion a new wave
of proliferation. Although the anticipated domino effect did not transpire, the Indian test did precipitate a marked increase in
scholarship on proliferation, including an innovative study developed around the concept—now in vogue—of proliferation chains.
Rarely cited by today's experts, the
1976 monograph on Trends in Nuclear Proliferation , 1975–1995, by Lewis
Dunn and Herman Kahn, set forth fifteen scenarios for nuclear weapons spread, each based on the assumption
that one state's acquisition of nuclear weapons would prompt several other states to follow suit, which in turn would trigger a
succession of additional nuclearization decisions.47 Although lacking any single theoretical underpinning and accepting of the notion
that proliferation decisions are likely to be attributed to security needs, the Dunn-Kahn model rejected the exclusive focus by realists
on security drivers and sought to probe [End Page 160] beneath the rhetoric to identify the possible presence of other pressures and
constraints. To their credit, Dunn and Kahn got many things right and advanced the study of proliferation. Their forecasts, however,
were almost without exception wildly off the mark. Why,
one may inquire, were their pessimistic projections
about nuclear weapons spread—and those of their past and subsequent counterparts in the intelligence community—
so often divorced from reality? Although Hymans and Solingen appear not to have been familiar with the research by
Dunn and Kahn on proliferation trends at the time of their books' publications, their national leadership and domestic political
survival models offer considerable insight into that dimension of the proliferation puzzle.48 The Four Myths of Nuclear Proliferation
Hymans is keenly aware of the deficiency of past proliferation projections, which he attributes
in large part to the "tendency to use the growth of nuclear capabilities, stances toward the non-
proliferation regime, and a general 'roguishness' of the state as proxies for nuclear weapons intentions " (p.
217). Such intentions, he believes, cannot be discerned without reference to leadership national identity conceptions, a focus that
appears to have been absent to date in intelligence analyses devoted to forecasting proliferation.49 Hymans is equally critical of the
popular notion that "the 'domino theory' of the twenty-first century may well be nuclear."50 As he points out, the new
domino theory, like its discredited Cold War predecessor, assumes an over-simplified view about
why and how decisions to acquire nuclear weapons are taken .51 Leaders' nuclear preferences, he
maintains, "are not highly contingent on what other states decide, " and, therefore, "proliferation
tomorrow will probably remain as rare as proliferation today, with no single instance of
proliferation causing a cascade of nuclear weapons states" (p. 225). In addition, he argues, the domino
thesis embraces "an exceedingly dark picture of world trends by lumping the truly dangerous
leaders together with the merely self-assertive [End Page 161] ones ," and equating interest in
nuclear technology with weapons intent (pp. 208–209). Dire proliferation forecasts, both past and present, Hymans
believes, flow from four myths regarding nuclear decisonmaking: (1) states want the bomb as a deterrent; (2) states seek the bomb
as a "ticket to international status"; (3) states go for the bomb because of the interests of domestic groups; and (4) the international
regime protects the world from a flood of new nuclear weapons states (pp. 208–216). Each of these assumptions is faulty, Hymans
contends, because of its fundamental neglect of the decisive role played by individual leaders in nuclear matters. As discussed
earlier, Hymans argues that the need for a nuclear deterrent is entirely in the eye of the beholder—a leader with an oppositional
nationalist NIC. By the same token, just because some leaders seek to achieve international prestige through acquisition of the
bomb, it does not mean that other leaders "necessarily view the bomb as the right ticket to punch": witness the case of several
decades of Argentine leaders, as well as the Indian Nehruvians (pp. 211–212). The case of Egypt under Anwar al-Sadat, though not
discussed by Hymans, also seems to fit this category. Hymans's focus on the individual level of analysis leads him to discount
bureaucratic political explanations for nuclear postures, as well. Central
to his argument is the assumption that
decisions to acquire nuclear weapons are taken "without the considerable vetting that political
scientists typically assume precedes most important states choices" (p. 13). As such, although he is
prepared to credit nuclear energy bureaucracies as playing a supporting role in the efforts by Australia, France, and India to go
nuclear, he does not observe their influence to be a determining factor in root nuclear decisions by national leaders. Moreover,
contrary to a central premise of Solingen's model of domestic political survival, Hymans finds little evidence in his case studies of
leaders pursuing nuclear weapons to advance their political interests (p. 213). For example, he argues, the 1998 nuclear tests in India
were as risky domestically for Vajpayee as they were internationally (p. 214).
---EXT Khal & Hymans No Prolif

1NC Khal and Hymans—prolif will fail and be incredibly slow—too many tech and
management barriers prevent prolif- the harder countries push the LESS likely they are to get
nukes

Prolif will fail and be incredibly slow—barriers are too large to overcome

Hymans 12
Jacques Hymans, USC Associate Professor of IR, 4/16/12, North Korea's Lessons for (Not) Building an Atomic Bomb,
www.foreignaffairs.com/articles/137408/jacques-e-c-hymans/north-koreas-lessons-for-not-building-an-atomic-bomb?page=show

Washington's miscalculation is not just a product of the difficulties of seeing inside the Hermit Kingdom. It is also a result of the
broader tendency to overestimate the pace of global proliferation . For decades, Very Serious
People have predicted that strategic weapons are about to spread to every corner of the earth. Such
warnings have routinely proved wrong - for instance, the intelligence assessments that led to the
2003 invasion of Iraq - but they continue to be issued . In reality, despite the diffusion of the relevant
technology and the knowledge for building nuclear weapons, the world has been experiencing a great
proliferation slowdown. Nuclear weapons programs around the world are taking much longer to
get off the ground - and their failure rate is much higher - than they did during the first 25 years
of the nuclear age. As I explain in my article "Botching the Bomb" in the upcoming issue of Foreign Affairs, the key
reason for the great proliferation slowdown is the absence of strong cultures of scientific
professionalism in most of the recent crop of would-be nuclear states, which in turn is a consequence of their
poorly built political institutions. In such dysfunctional states, the quality of technical
workmanship is low, there is little coordination across different technical teams, and technical
mistakes lead not to productive learning but instead to finger-pointing and recrimination. These problems
are debilitating , and they cannot be fixed simply by bringing in more imported parts through illicit supply networks. In
short, as a struggling proliferator, North Korea has a lot of company.

We have the only predictive evidence based on tested empirical research and theory

Potter 8
William C. Potter is Sam Nunn and Richard Lugar Professor of Nonproliferation Studies and Director of the James Martin Center for
Nonproliferation Studies at the Monterey Institute of International Studies, Summer 2008, Divining Nuclear Intentions,
http://muse.jhu.edu/journals/international_security/v033/33.1.potter.pdf

For much of the nuclear age, academic experts, intelligence analysts, and public commentators periodically have forecast
rapid bursts of proliferation, which have failed to materialize. Central to their prognoses, often
imbued with the imagery and metaphors of nuclear dominoes and proliferation chains, has been the assumption that one
state's nuclearization is likely to trigger decisions by other states to "go nuclear" in quick succession. Today the
proliferation metaphors of choice are "nuclear cascade" and "tipping point," but the implication is the same—we are on the cusp of
rapid, large-scale nuclear weapons spread. It is with some justification, therefore, that the study of proliferation has been labeled
Although proliferation projections abound, few of them are founded
"the sky-is-still-falling profession."1
on, or even informed by, empirical research and theory.2 This deficiency, though regrettable, is
understandable given the small body of theoretically or empirically [End Page 139] grounded research on forecasting proliferation
developments, and the underdeveloped state of theory on nonproliferation and nuclear decisionmaking more generally. Also
contributing to this knowledge deficit is the stunted development of social science research on foreign policy–
oriented forecasting and the emphasis on post hoc explanations, rather than predictions on the part
of the more sophisticated frameworks and models of nuclear decisionmaking. Two important exceptions to this general
paucity of nonproliferation theory with predictive value are recent books by Jacques Hymans, The
Psychology of Nuclear Proliferation: Identity, Emotions, and Foreign Policy, and Etel Solingen, Nuclear Logics: Alternative Paths in
East Asia and the Middle East.3 These studies merit careful attention because of their solid grounding in
comparative field research and social science theory, their challenges to prevailing conceptions
about the sources of nuclear weapons decisions, and their promise for predicting proliferation developments .
As such, they go well beyond the influential but historically oriented explanatory frameworks developed by scholars such as Peter
Lavoy, Ariel Levite, T.V. Paul, Scott Sagan, and James Walsh.4 Although the approaches advanced by Hymans and Solingen have
their own limitations, these two books represent
the cutting edge of nonproliferation research and should be of
great interest to both policy practitioners and scholars. In particular, a careful review of their studies
sheds new insights
into why past predictions of rapid proliferation have proved faulty, why the current alarm over
impending proliferation doom is largely without merit, and why we should not count on single theories of
international relations—at least in their [End Page 140] current state—to offer much guidance in explaining or predicting the
dynamics of nuclear weapons spread.
---EXT Hymans- AT: Bomb Transfer

Trasfer Fails

Hymans ‘12 [Jacques E. C. Hymans, PhD from Harvard, Associate Professor of International Relations at the University of
Southern California, his most recent book is Achieving Nuclear Ambitions: Scientists, Politicians, and Proliferation, “Botching the
Bomb: Why Nuclear Weapons Programs Often Fail on Their Own-and Why Iran's Might, Too,” Foreign Affairs91. 3 (May/Jun 2012):
44-53, Proquest]

IRAQ'S EXPERIENCE notwithstanding, many proliferation analysts insist that although technologically backward states might not have
been capable of nuclear weapons development in the past, they can now simply purchase all they need in the freewheeling
globalized marketplace. Admittedly, illicit nuclear entrepreneurs -- such as A. Q. Khan, the rogue Pakistani scientist who sold nuclear
technology to Iran, Libya, and North Korea -- do pose a threat. But
international nuclear technology transfers often
fail because the dysfunctional states that are trying to get the bomb are hardly any better at
exploiting foreign nuclear know-how than they are at developing their own. Libya's
misbegotten nuclear weapons project reflects this general pattern. Despite buying all the items in
Khan's catalog, Libya was unable to "put them together and make them work," according to a
2005 U.S. government report. Indeed, when IAEA inspectors gained access to Libyan nuclear
facilities after Libya's president, Muammar al-Qaddafi, abandoned the project in 2003, they
found much of the imported merchandise still in its original packing crates.
---EXT Hymans- AT: Rogue Scientists

Rogue scientists are a Hollywood myth- no prolif

Hymans ‘12 [Jacques E. C. Hymans, PhD from Harvard, Associate Professor of International Relations at the University of
Southern California, his most recent book is Achieving Nuclear Ambitions: Scientists, Politicians, and Proliferation, “Botching the
Bomb: Why Nuclear Weapons Programs Often Fail on Their Own-and Why Iran's Might, Too,” Foreign Affairs91. 3 (May/Jun 2012):
44-53, Proquest]

As for some analysts' terrifying predictions of ex-Soviet nuclear scientists and technicians leaving home
en masse to further the nuclear ambitions of rogue regimes, this is more the stuff of Hollywood than a
genuine problem. Ex-Soviet researchers vastly prefer the professional establishments of the West
over the secret lairs of brutal dictators. Moreover, developing-state rulers need to be wary of
recruiting outsiders, since the few genuine nuclear experts available can be hard to distinguish
from the scores of frauds and spies also on the market . Take, for instance, the case of Argentine President
Juan Perón's post-World War II recruitment of Nazi scientists. This was perhaps the most successful effort to produce a reverse
scientific brain drain in history. Yet Ronald Richter, the Austrian physicist whom Perón chose to head his nascent nuclear program,
turned out to be part con man and part madman. Perón realized his error only after the snickering worldwide reaction to his 1951
announcement that Richter had succeeded in producing controlled fusion.
---EXT Hymans- Iran Specific

Iran can’t proliferate

Hymans ‘12 [Jacques E. C. Hymans, PhD from Harvard, Associate Professor of International Relations at the University of
Southern California, his most recent book is Achieving Nuclear Ambitions: Scientists, Politicians, and Proliferation, “Botching the
Bomb: Why Nuclear Weapons Programs Often Fail on Their Own-and Why Iran's Might, Too,” Foreign Affairs91. 3 (May/Jun 2012):
44-53, Proquest]

IN THE intensifying crisis over Iran's nuclear activity,


the great proliferation slowdown has gone all but
unmentioned. Yet this robust global trend clearly indicates a need to guard against any hasty
conclusion that Iran's nuclear program is about to achieve its ultimate aims. Iran's nuclear scientists and
engineers may well find a way to inoculate themselves against Israeli bombs and computer hackers. But they face a potentially
far greater obstacle in the form of Iran's long-standing authoritarian management culture. In a study
of Iranian human-resource practices, the management analysts Pari Namazie and Monir Tayeb concluded that
the Iranian regime has historically shown a marked preference for political loyalty over professional
qualifications. "The belief," they wrote, "is that a loyal person can learn new skills, but it is much more difficult to teach loyalty
to a skilled person." This is the classic attitude of authoritarian managers. And according to the Iranian political scientist Hossein
Bashiriyeh, in recent years, Iran's "irregular and erratic economic policies and practices, political nepotism and general
mismanagement" have greatly accelerated. It is hard to imagine that the politically charged Iranian nuclear program is sheltered
from these tendencies. It is surely more difficult to assess the quality of Iran's nuclear management than it is to count the number of
Iranian centrifuge machines. But such an assessment is vital, because the progress of Iran's program will depend on how much
professional autonomy its scientists and engineers are able to retain. In the meantime, a number of broad lessons from the great
proliferation slowdown can help provide a more sober assessment of the situation. The first lesson is to be wary of narrow,
Recent alarming estimates of Iran's timeline to the
technocentric analyses of a state's nuclear weapons potential.
bomb have been based on the same assumptions that have led Israel and the U nited S tates to
consistently overestimate Iran's rate of nuclear progress for the last 20 years . The majority of official
U.S. and Israeli estimates during the 1990s predicted that Iran would acquire nuclear weapons by 2000. After that date passed with
no Iranian bomb in sight, the estimate was simply bumped back to 2005, then to 2010, and most recently to 2015. The point is not
that the most recent estimates are necessarily wrong but rather that they lack credibility. In particular ,
policymakers should
heavily discount any intelligence assessments that do not explicitly account for the impact of
management quality on Iran's proliferation timeline. The second lesson of the proliferation slowdown is that
policymakers should reject analyses based on assumptions about a state's capacity to build nuclear programs in secret. Ever since
the mid-1990s,official proliferation assessments have freely extrapolated from minimal data, a
practice that led U.S. intelligence analysts to wrongly conclude that Iraq had reconstituted its
weapons of mass destruction programs after the Gulf War. The United States must guard against the possibility of an equivalent
intelligence failure over Iran. This is not to deny that Tehran may be keeping some of its nuclear work secret. But it is simply
unreasonable to assume, for example, that Iran has compensated for the problems it has faced with centrifuges at the Natanz
uranium-enrichment facility by hiding better-working centrifuges at some unknown facility. Indeed, when Iran has tried to hide
weapons-related activities in the past, it has often been precisely because the work was at the very early stages or was going badly.

See Iran Prolif Answers


---EXT Hymans- North Korea Specific

North Korea can’t prolif

Hymans ‘12 [Jacques E. C. Hymans, PhD from Harvard, Associate Professor of International Relations at the University of
Southern California, his most recent book is Achieving Nuclear Ambitions: Scientists, Politicians, and Proliferation, “Botching the
Bomb: Why Nuclear Weapons Programs Often Fail on Their Own-and Why Iran's Might, Too,” Foreign Affairs91. 3 (May/Jun 2012):
44-53, Proquest]

The third lesson is that states that


poorly manage their nuclear programs can bungle even the supposedly
easy steps of the process. For instance, based on estimates of the size of North Korea's plutonium
stockpile and the presumed ease of weapons fabrication, U.S. intelligence agencies thought that by
the 1990s, North Korea had built one or two nuclear weapons. But in 2006, North Korea's first nuclear
test essentially fizzled, making it clear that the "hermit kingdom" did not have any working
weapons at all. Even its second try, in 2009, did not work properly. Similarly, if Iran eventually
does acquire a significant quantity of weapons-grade highly enriched uranium, this should not
be equated with the possession of a nuclear weapon.
---EXT No War

1NC Forsyth—prolif not destabilizing—deterrence is right—countries become socialized to


risks—survival is priority amongst new proliferators—any country with capabilities to form
legitimate arsenal has integrated requisite stabilizing institutions

New prolif won’t cause war- the best evidence all agrees

Tepperman ‘9 ( 9/7/2009 (John - journalist based in New York Cuty, Why obama should learn to love the bomb, Newsweek,
p.lexis)

A growing and compelling body of research suggests that nuclear weapons may not, in fact,
make the world more dangerous, as Obama and most people assume. The bomb may actually make us safer. In this era
of rogue states and transnational terrorists, that idea sounds so obviously wrongheaded that few politicians or policymakers are
willing to entertain it. But that's a mistake. Knowing the truth about nukes would have a profound impact on government policy.
Obama's idealistic campaign, so out of character for a pragmatic administration, may be unlikely to get far (past presidents have
tried and failed). But it's not even clear he should make the effort. There are more important measures the U.S. government can and
should take to make the real world safer, and these mustn't be ignored in the name of a dreamy ideal (a nuke-free planet) that's
both unrealistic and possibly undesirable. The argument that nuclear weapons can be agents of peace as well as destruction rests
on two deceptively simple observations. First, nuclear
weapons have not been used since 1945. Second,
there's never been a nuclear, or even a nonnuclear, war between two states that possess them .
Just stop for a second and think about that: it's hard to overstate how remarkable it is, especially given the singular viciousness of
the 20th century. As Kenneth Waltz, the leading "nuclear optimist" and a professor emeritus of political science at UC Berkeley puts
it, "We now have 64 years of experience since Hiroshima. It's striking and against all historical precedent that for that substantial
period, there has not been any war among nuclear states." To understand why--and why the next 64 years are likely to play out the
same way--you need to start by recognizing that all
states are rational on some basic level. Their leaders may
be stupid, petty, venal, even evil, but they tend to do things only when they're pretty sure they
can get away with them. Take war: a country will start a fight only when it's almost certain it can
get what it wants at an acceptable price. Not even Hitler or Saddam waged wars they didn't
think they could win. The problem historically has been that leaders often make the wrong
gamble and underestimate the other side--and millions of innocents pay the price. Nuclear
weapons change all that by making the costs of war obvious , inevitable, and unacceptable .
Suddenly, when both sides have the ability to turn the other to ashes with the push of a button--and everybody knows it--the basic
math shifts. Even
the craziest tin-pot dictator is forced to accept that war with a nuclear state is
unwinnable and thus not worth the effort. As Waltz puts it, " Why fight if you can't win and
might lose everything ?" Why indeed? The iron logic of deterrence and mutually assured destruction is so compelling,
it's led to what's known as the nuclear peace: the virtually unprecedented stretch since the end of World War II in which all the
world's major powers have avoided coming to blows. They did fight proxy wars, ranging from Korea to Vietnam to Angola to Latin
America. But these never matched the furious destruction of full-on, great-power war (World War II alone was responsible for some
50 million to 70 million deaths). And since the end of the Cold War, such bloodshed has declined precipitously. Meanwhile, the
nuclear powers have scrupulously avoided direct combat, and there's very good reason to think they always will. There have been
some near misses, but a close look at these cases is fundamentally reassuring--because in each instance, verydifferent
leaders all came to the same safe conclusion . Take the mother of all nuclear standoffs: the Cuban missile
crisis. For 13 days in October 1962, the United States and the Soviet Union each threatened the other with destruction. But both
countries soon stepped back from the brink when they recognized that a war would have meant
curtains for everyone. As important as the fact that they did is the reason why: Soviet leader Nikita Khrushchev's aide Fyodor
Burlatsky said later on, "It is impossible to win a nuclear war, and both sides realized that, maybe for the first time." The record
since then shows the same pattern repeating: nuclear-armed enemies slide toward war, then
pull back, always for the same reasons . The best recent example is India and Pakistan, which
fought three bloody wars after independence before acquiring their own nukes in 1998. Getting their hands on weapons
of mass destruction didn't do anything to lessen their animosity. But it did dramatically mellow
their behavior. Since acquiring atomic weapons, the two sides have never fought another war,
despite severe provocations (like Pakistani-based terrorist attacks on India in 2001 and 2008). They have skirmished
once. But during that flare-up, in Kashmir in 1999, both countries were careful to keep the fighting limited and to avoid threatening
the other's vital interests. Sumit Ganguly, an Indiana University professor and coauthor of the forthcoming India, Pakistan, and
the Bomb, has found
that on both sides, officials' thinking was strikingly similar to that of the
Russians and Americans in 1962. The prospect of war brought Delhi and Islamabad face to face with a nuclear holocaust,
and leaders in each country did what they had to do to avoid it.

Reject their evidence - irrational fear

Tepperman 9 [Jonathan, Newsweek International's first Assistant Managing Editor (now Deputy Editor), “Why Obama Should
Learn to Love the Bomb” 8-29, http://www.newsweek.com/2009/08/28/why-obama-should-learn-to-love-the-bomb.html, SM]

Put this all together and nuclear weapons start to seem a lot less frightening. So why have so few people in Washington
recognized this?Most of us suffer from what Desch calls a nuclear phobia, an irrational fear that's
grounded in good evidence—nuclear weapons are terrifying—but that keeps us from making clear, coldblooded
calculations about just how dangerous possessing them actually is. The logic of nuclear peace rests on a
scary bargain: you accept a small chance that something extremely bad will happen in exchange for a much bigger chance
that something very bad—conventional war—won't happen. This may well be a rational bet to take , especially if
that first risk is very small indeed. But it's a tough case to make to the public. 

No impact - multiple checks prevent use

Cha 1 (Victor, Associate Professor of Government and School of Foreign Service @ Georgetown, “The second nuclear age:
Proliferation pessimism versus sober optimism in South Asia and East Asia,” Journal of Strategic Studies, InformaWorld)

Proliferation pessimists do not deny the existence of the nuclear taboo; they do, nevertheless, see this taboo as shared only by
First World proliferators. Is this a fair assessment? As Tannenwald argues, a
taboo takes effect when the agent
realizes (1) the exceptionalist nature of the weapon (i.e., in terms of its destructive power); (2)
the absence of effective defenses (i.e., vulnerability); (3) and fears the political and social
consequences of taking such an action. All of these conditions readily hold for new nuclear
powers. Moreover, the revulsion against nuclear weapons use (first-use) has become so
institutionalized in an array of international agreements and practices such that new NWS
states operate in an environment that severely circumscribes the realm of legitimate nuclear
use.90 Proliferation pessimists therefore underestimate the transformative effects of nuclear
weapons on these new proliferators. They assume that the interests for aspiring nuclear powers remain constant in the pre-
and post-acquisition phases. They do not consider that once states cross the nuclear threshold, they
become acutely aware of the dangers and responsibilities that come with these new
awesome capabilities. The likelihood of such a learning process occurring is even higher if nuclear weapons are valued
for their political currency. As noted above, while security needs certainly drive proliferation in Asia, a predominant factor that
cannot be disentangled from this dynamic is the striving for prestige and international recognition as an NWS state. Moreover,
if the taboo equates the use of nuclear weapons with an 'uncivilized' or 'barbarian' state,"
then those states that are status-conscious will be that much more attuned to the taboo. The
effects of the taboo on Asian proliferators are therefore both regulative and constitutive. In the
former sense, as these states further embed themselves in the international community (discussed below), this change
heightens the costs of breaking any rules regarding nuclear use. The taboo's constitutive effects also are evident in that any use
would undermine one of the primary purposes for which the capabilities were sought (e.g., prestige, badge of modernity).
Although it is still relatively early in the game, there is some evidence that the acquisition of nuclear capabilities has been
accompanied by a change in preferences about what is acceptable behavior. While India has rejected any notions that it might
roll back its newfound capability, it had readily admitted that as an incipient nuclear weapons state, it now has certain
responsibilities that include a no-first-use policy and not sharing nuclear weapons technology with other irresponsible states.92
Similarly, Pakistan previously placed little value and even resented nonproliferation norms as these were seen as inhibiting and
degrading to the national character.93 Otherwise, they might have been swayed by the benefits of not responding to the Indian
tests as a shining example of a country adhering to nuclear nonproliferation norms. Arguably it is only after becoming an
incipient nuclear weapons state that such arguments about nonproliferation gain value. Nowhere is this perverse dynamic more
evident than in both sides' views of the CTBT. Previously perceived as an instrument intended to preempt nuclear spread
beyond the first age, the CTBT is now arguably seen by India and Pakistan in less antagonistic terms, and even among some, as a
responsibility to be borne as a nuclear state. 

Empirical studies prove it moderates behavior

Sechser ‘09 [Todd, Dept. of Politics at UVA, “Should The United States or the International Community Aggressively Pursue
Nuclear Nonproliferation Policies? No.” In: Haas et al Controversies in Globalization: Contending Approaches to International
Relations, CQ Press, http://faculty.virginia.edu/tsechser/Sechser-Haas-2009.pdf]

The optimist camp’s first and most important claim is that the presence of nuclear weapons suppresses international conflicts.
Nuclear weapons, in this view, differ from conventional military tools in two central ways. First, nuclear weapons carry enormous
destructive power. Whereas the targets of conventional weapons necessarily tend to be small in size (for instance, an airfield,
communications center, or ammunition depot), the most powerful nuclear weapons can place entire cities at risk. The use of even a
few nuclear weapons could destroy hundreds of thousands (if not millions) of human lives in a short span of time. Second, defenders
have little control over the level of destruction they endure during a nuclear conflict. Without a reliable means to destroy incoming
ballistic missiles or to shield cities from nuclear attack— neither of which exists today—nuclear combatants must rely on an enemy’s
restraint to limit the amount of damage they suffer. These two
characteristics—colossal destructive capacity and
the lack of an effective defense— combine to induce caution among leaders facing the prospect
of nuclear retaliation. Leaders will behave less aggressively and will more eagerly seek peaceful
solutions to crises, the logic goes, since they do not want to endure even a small risk that a conventional
war might become nuclear. These propositions can be evaluated empirically by comparing the rates at which proliferators
have participated in interstate conflicts both before and after their acquisition of nuclear weapons. If the optimists are correct,
nuclear states should experience fewer conflicts after they acquire nuclear weapons. One way to measure the
turbulence of a state’s foreign affairs is to calculate its participation in militarized interstate
disputes, defined here as conflicts involving at least one military fatality. Figure 1 considers five proliferators and charts how
much their involvement in military conflicts changed after they became nuclear states. Israel, for instance, participated in an
average of 1.21 conflicts per year as a nonnuclear state, but entered into only 0.33 conflicts per year
after becoming a nuclear state in 1972, so its bar in figure 1 drops below zero to illustrate that Israel has been involved in
fewer interstate conflicts since acquiring nuclear weapons. Optimists predict that states will participate in fewer conflicts after going
nuclear, since they expect nuclear
weapons to deter aggression and dissuade opposing leaders from
escalating crises. And indeed, four of the five states examined here participated in fewer interstate
conflicts, on average, once they became nuclear states. For example, Israel fought four interstate wars against its
neighbors before acquiring nuclear weapons, but just two afterward. India and Pakistan have gone to war against one another four
times since achieving independence, but only one of those wars occurred after the two rivals acquired nuclear weapons. Indeed,
India and Pakistan saw the average incidence of militarized disputes between them decline by
half(from 0.55 disputes per year to 0.27) once both states had acquired nuclear weapons. Only South Africa experienced an
increase in its conflict participation rate after achieving nuclear status, although the magnitude of this change (+0.06) was the
smallest of the five proliferators considered here. These data tell us that proliferation optimists are right to expect a decline in the
frequency of interstate wars as more states acquire nuclear weapons.

Proliferators are not aggressive – they care about the economy and regime survival

Alagappa ‘8 (Muthiah – distinguished senior fellow at the East-West Center, The Long Shadow, p. 508-509)

Another major conclusion of this study is that although nuclear weapons could have destabilizing consequences in certain
situations, on net they have reinforced national security and regional stability in Asia. It is possible to argue that
fledgling and small nuclear arsenals would be more vulnerable to preventive attacks; that the related strategic compulsion for early
use may lead to early launch postures and crisis situations; that limited war under nuclear conditions to alter or restore the political
status quo can intensify tensions and carry the risk of escalation to major war; that inadequate command, control, and safety
measures could result in accidents; and that nuclear facilities and material may be vulnerable to terrorist attacks. These are
legitimate concerns, but thus far nuclear weapons have not undermined national security and regional stability in Asia.
Instead, they have ameliorated national security concerns, strengthened the status quo, increased
deterrence dominance, prevented the outbreak of major wars, and reinforced the regional
trend to reduce the salience of force in international politics. Nor have nuclear weapons had the predicted domino
effect. These consequences have strengthened regional security and stability that rest on multiple pillars. The grim scenarios
associated with nuclear weapons in Asia frequently rely on worst-case political and military situations; often they are seen in
isolation from the national priorities of regional states that emphasize economic development
and modernization through participation in regional and global economies and the high priority accorded to stability
in domestic and international affairs. The primary goal of regional states is not aggrandizement through military
aggression but preservation of national integrity, state or regime survival, economic growth and
prosperity, increase in national power and international influence, preservation or incremental change in the status quo,
and the construction of regional and global orders in which they are subjects rather than objects. Seen in this broader
perspective, nuclear weapons and more generally military force are of greater relevance in the defense,
deterrence, and assurance roles than offensive ones . This does not imply that offensive use of force or military
clashes will not occur; only that force is not the first option, that military clashes will be infrequent, and that when they do occur
they will be limited in scope and intensity. Security interaction in Asia increasingly approximates behavior associated with defensive
realism.

History proves acquisition will be slow and doesn’t cause war

Bennett 5 (Drake, Boston Globe, “Give nukes a chance”,


http://www.boston.com/news/globe/ideas/articles/2005/03/20/give_nukes_a_chance?pg=full)

KENNETH N. WALTZ, adjunct professor of political science at Columbia University, doesn't like the phrase ''nuclear
proliferation.'' ''The
term proliferation' is a great misnomer,'' he said in a recent interview. ''It refers to
things that spread like wildfire. But we've had nuclear military capabilities extant in the world
for 50 years and now, even counting North Korea, we only have nine nuclear countries. '' Strictly
speaking, then, Waltz is as against the proliferation of nuclear weapons as the next sane human
being. After all, he argues, ''most countries don't need them.'' But the eventual acquisition of nuclear weapons
by those few countries that see fit to pursue them, that he's for. As he sees it, nuclear weapons prevent
wars. ''The only thing a country can do with nuclear weapons is use them for a deterrent ,''
Waltz told me. ''And that makes for internal stability, that makes for peace, and that makes for
cautious behavior.'' Especially in a unipolar world, argues Waltz, the possession of nuclear
deterrents by smaller nations can check the disruptive ambitions of a reckless superpower.
As a result, in words Waltz wrote 10 years ago and has been reiterating ever since, ''The gradual spread of
nuclear weapons is more to be welcomed than feared. ''
---Prolif- AT: Snowball

Their authors are rooted in outdated Cold War paranoia

Van der Meer ’11 [Sico, Research Fellow at the Netherlands Institute of International Relations ‘Clingendael’ and a PhD
Candidate at the Erasmus University Rotterdam; his PhD project on nuclear proliferation dynamics is financially supported by the
Dutch non-governmental organisation IKV Pax Christi. “Not that bad: Looking back on 65 years of nuclear non-proliferation efforts,”
Security and Human Rights 2011 no.1]

Since the invention and first use of nuclear weapons, predictions on the spread of these weapons have been traditionally
pessimistic. Especially during the Cold War, from 1945 to 1991, the persistent pessimism among experts and policymakers is — with
the knowledge of looking backwards — surprising. During the first decades of the Cold War it was generally expected that far more
countries would acquire a nuclear weapons arsenal rather soon. This pessimism was not that strange, considering that nuclear
weapons were generally seen as acceptable, desirable and even necessary among political and military elites in many nations during
the 1950s and early 1960s.2 Nuclear weapons are considered as the ultimate weapon that would deter any enemy from attacking.
Moreover, nuclear weapons offer not only military power: they are also considered to increase a state’s political power
internationally. Having nuclear weapons grants a state — and its leadership — international prestige, and a nuclear weapon state
will automatically be considered and treated as a (regional) superpower. Based on this positive attitude towards nuclear weapons,
forecasts in these years were therefore easily predicting that 20 to 25 states would become nuclear weapon powers within the next
few decades; countries like Sweden, West Germany and Japan are examples of countries that were often considered would soon
cross the nuclear threshold, but they never did. One of the reasons for the alarming forecasts during much of the Cold War period
was the failure of many estimates to distinguish between the capacity of states to develop nuclear weapons and the desire of these
states to do so.3 Even nowadays, however, political
and academic forecasts often tend to be rather
pessimistic, predicting nuclear domino effects, or chain reactions, when new nuclear weapon powers
(for example, Iran) will emerge and cause other states to develop nuclear weapons too.4 Despite all
the pessimistic forecasts, however, only nine states nowadays possess nuclear weapons.5 Although
more states have employed nuclear weapons programmes at some point in the past 65 years,
most of them have sooner or later ended their ambition to acquire these weapons. Some states
even destroyed their nuclear arsenal (South Africa) or gave up inherited arsenals (Ukraine,
Belarus and Kazakhstan). Especially since the second half of the 1980s the number of states with
nuclear weapons-related activities has become very marginal .6

No chain reactions – the domino effect never occurs

Alagappa ‘8 (Muthiah – distinguished senior fellow at the East-West Center, The Long Shadow, p. 521-522)

It will be useful at this juncture to address more directly the set of instability arguments advanced by certain policy makers
and scholars: the domino effect of new nuclear weapon states, the probability of preventive action against new nuclear
weapon states, and the compulsion of these states to use their small arsenals early for fear of losing them in a preventive or
preemptive strike by a stronger nuclear adversary. On
the domino effect, India's and Pakistan 's nuclear
weapon programs have not fueled new programs in South Asia or beyond. Iran's quest for
nuclear weapons is not a reaction to the Indian or Pakistani programs. It is grounded in that country's
security concerns about the United States and Tehran's regional aspirations. The North Korean test has evoked
mixed reactions in Northeast Asia. Tokyo is certainly concerned; its reaction, though, has not
been to initiate its own nuclear weapon program but to reaffirm and strengthen the American extended
deterrence commitment to Japan. Even if the U.S. Japan security treaty were to weaken, it is not certain
that Japan would embark on a nuclear weapon program . Likewise, South Korea has sought
reaffirmation of the American extended deterrence commitment, but has firmly held to its nonnuclear posture .
Without dramatic change in its political, economic, and security circumstances, South Korea is highly unlikely to embark on a covert
(or overt) nuclear weapon program as it did in the 1970s. South Korea could still become a nuclear weapon state by inheriting the
nuclear weapons of North Korea should the Kim Jong Il regime collapse. Whether it retains or gives up that capability will hinge on
the security circumstances of a unified Korea. The North Korean nuclear test has not spurred Taiwan or
Mongolia to develop nuclear weapon capability. The point is that each country's decision to embark on and
sustain nuclear weapon programs is contingent on its particular security and other circumstances. Though appealing, the
domino theory is not predictive ; often it is employed to justify policy on the basis of alarmist
predictions. The loss of South Vietnam, for example, did not lead to the predicted domino effect
in Southeast Asia. In fact the so-called dominos became drivers of a vibrant Southeast Asia and brought about a fundamental
transformation in that subregion (Lord 1993, 1996). In
the nuclear arena, the nuclear programs of China, India,
and Pakistan were part of a security chain reaction, not mechanically falling dominos . However, as
observed earlier the Indian, Pakistani, and North Korean nuclear tests have thus far not had the domino effect predicted by alarmist
analysts and policy makers. Great
caution should be exercised in accepting at face value the sensational
predictions of individuals who have a vested interest in accentuating the dangers of nuclear
proliferation. Such analysts are now focused on the dangers of a nuclear Iran. A nuclear Iran may or may not have destabilizing
effects. Such claims must be assessed on the basis of an objective reading of the drivers of national and regional security in Iran and
the Middle East.

No domino

Potter & Mukhatzhanova ‘8 [William C. and Gaukhar, * Sam Nunn and Richard Lugar Professor of Nonproliferation
Studies and Director of the James Martin Center for Nonproliferation Studies at the Monterey Institute of International Studies and
** Research Associate at the James Martin Center, “Divining Nuclear Intentions: a review essay.” International Security, Vol. 33, No.
1 (Summer 2008), pp. 139–169, Google scholar]

Today it is hard to find an analyst or commentator on nuclear proliferation who is not


pessimistic about the future. It is nearly as difficult to and one who predicts the future without reference to
metaphors such as proliferation chains, cascades, dominoes, waves, avalanches, and tipping points.42 The lead author of this
essay also has been guilty of the same tendency, and initially named an ongoing research project on forecasting proliferation he
directs “21st Century Nuclear Proliferation Chains and Trigger Events.” As both a thors proceeded with
research on the project, however, and particularly after reading the books by Hymans and Solingen, we became convinced that
the metaphor is inappropriate and misleading, as it implies
a process of nuclear decisionmaking and a pace
of nuclear weapons spread that are unlikely to transpire. The current alarm about life in a nuclear-armed
crowd has many historical antecedents and can be found in classified National Intelligence Estimates (NIEs) as well as in
scholarly analyses. The 1957 NIE, for example, identified a list of ten leading nuclear weapons candidates, including Canada,
Japan, and Sweden.43 Sweden, it predicted, was “likely to produce its first weapons in about
1961,” while it was estimated that Japan would “probably seek to de- velop weapons production
programs within the next decade.”44 In one of the most famous forecasts, President John Kennedy in 1963
expressed a nightmarish vision of a future world with afteen, twenty, or twenty-ave nuclear weap- ons powers.45 A number of
the earliest scholarly projections of proliferation also tended to exaggerate the pace of nuclear
weapons spread. A ourry of studies between 1958 and 1962, for example, focused on the “Nth
Country Problem” and identified as many as twelve candidates capable of going nuclear in
the near future.46 Canada, West Germany, Italy, Japan, Sweden, and Switzerland were among the states most frequently
picked as near-term proliferators. The “peaceful nuclear explosion” by India in 1974 was seen by many
ana- lysts of the time as a body blow to the young NPT that would set in motion a new wave
of proliferation. Although the anticipated domino effect did not transpire , the Indian test did precipitate
a marked increase in scholarship on proliferation, including an innovative study developed around the concept— now in vogue
—of proliferation chains. Rarely cited by today’s experts, the 1976
monograph on Trends in Nuclear
Proliferation, 1975–1995, by Lewis Dunn and Herman Kahn, set forth fifteen scenarios for nuclear
weapons spread, each based on the assumption that one state’s acquisition of nuclear
weapons would prompt several other states to follow suit, which in turn would trigger a
succession of additional nuclearization decisions.47 Although lacking any single theoretical
underpinning and accepting of the notion that proliferation de- cisions are likely to be
attributed to security needs, the Dunn-Kahn model rejected the exclusive focus by realists on
security drivers and sought to probe beneath the rhetoric to identify the possible presence of
other pressures and constraints. To their credit, Dunn and Kahn got many things right and advanced the study of
proliferation. Their forecasts, however, were almost without exception wildly off the mark. Why, one
may inquire, were their pessimistic projections about nuclear weapons spread—and those of
their past and subsequent counterparts in the intelligence community—so often divorced
from reality? Although Hymans and Solingen appear not to have been familiar with the re- search by Dunn and Kahn on
proliferation trends at the time of their books’ publications, their national leadership and domestic political survival models
offer considerable insight into that dimension of the proliferation puzzle.48

No domino theory—nonproliferation has zero utility

Potter 8
William C. Potter is Sam Nunn and Richard Lugar Professor of Nonproliferation Studies and Director of the James Martin Center for
Nonproliferation Studies at the Monterey Institute of International Studies, Summer 2008, Divining Nuclear Intentions,
http://muse.jhu.edu/journals/international_security/v033/33.1.potter.pdf

Hymans is keenly aware of the deficiency of past proliferation projections, which he attributes in large part to
the “tendency to use the growth of nuclear capabilities, stances toward the non-proliferation regime, and a general
‘roguishness’ of the state as proxies for nuclear weapons intentions ” (p. 217). Such intentions, he
believes, cannot be discerned without reference to leadership national identity conceptions, a focus that
appears to have been absent to date in intelligence analyses devoted to forecasting proliferation.49 Hymans is equally critical
of the popular notion that “the ‘domino theory’ of the twenty-first century may well be nuclear.”50 As he points out, the
new domino theory, like its discredited Cold War predecessor , assumes an oversimplified view
about why and how decisions to acquire nuclear weapons are taken.51 Leaders’ nuclear
preferences, he maintains, “are not highly contingent on what other states decide,” and, therefore,
“ proliferation tomorrow will probably remain as rare as proliferation today , with no single
instance of proliferation causing a cascade of nuclear weapons states” (p. 225). In addition, he argues,
the domino thesis embraces “an exceedingly dark picture of world trends by lumping the truly
dangerous leaders together with the merely self assertive ones,” and equating interest in nuclear
technology with weapons intent (pp. 208209). Dire proliferation forecasts, both past and present, Hymans
believes, flow from four myths regarding nuclear decisonmaking: (1) states want the bomb as a deterrent; (2) states seek the
bomb as a “ticket to international status”; (3) states go for the bomb because of the interests of domestic groups; and (4) the
international regime protects the world from a flood of new nuclear weapons states (pp. 208216). Each of these assumptions is
faulty, Hymans contends, because of its fundamental neglect of the decisive role played by individual leaders in nuclear matters. As
discussed earlier, Hymans argues that the need for a nuclear deterrent is entirely in the eye of the beholder
—a leader with an oppositional nationalist NIC. Bythe same token, just because some leaders seek to achieve
interna tional prestige through acquisition of the bomb, it does not mean that other leaders “necessarily view
the bomb as the right ticket to punch”: witness the case of several decades of Argentine leaders, as well as the Indian
Nehruvians (pp. 211212). The case of Egypt under Anwar al-Sadat, though not discussed by Hymans, also seems to at this category.
Hymans’s focus on the individual level of analysis leads him to discount bu reaucratic political explanations for nuclear postures, as
well. Central to his argument is the assumption that decisions to acquire nuclear weapons are taken “without
the considerable vetting that political scientists typically assume precedes most important states
choices” (p. 13). As such, although he is prepared to credit nuclear energy bureaucracies as playing a
supporting role in the ef forts by Australia, France, and India to go nuclear, he does not observe their influence
to be a determining factor in root nuclear decisions by national lead ers. Moreover, contrary to a central premise
of Solingen’s model of domestic political survival, Hymans ands little evidence in his case studies of leaders pursuing nuclear
weapons to advance their political interests (p. 213). For ex ample, he argues, the 1998 nuclear tests in India were as risky
domestically for Vajpayee as they were internationally (p. 214). Most provocatively, Hymans invokes an individual-centric mode of
analysis to challenge the necessity and utility of a strong international nonproliferation regime.
As discussed in a preceding section, he finds no evidence that the NPT regime prevented any of the
leaders who desired nuclear weapons from pursuing them .

No snowball prolif- not perceived as key to security

Lynn-Jones ’10 [Sean, Editor of International Security, the International Security Program's quarterly journal, series editor of
the Belfer Center Studies in International Security, the Program's book series that is published by MIT Press, “Going Nuclear: Nuclear
Proliferation and International Security in the 21st Century,” http://mitpress.mit.edu/books/chapters/026252466Xpref2.pdf]

One way for states to exercise nuclear restraint and not develop nuclear weapons is to participate in a regional regime. Why do
some states join regional nuclear regimes? Etel Solingen, in “The Political Economy of Nuclear Restraint,” argues
that
“ruling coalitions pursuing economic liberalization” are more likely to embrace regional regimes
that limit nuclear proliferation. 7 Solingen argues that neorealist theories that explain nuclear
choices as a response to security concerns are inadequate , because such theories do not explain the
wide range of behavior across countries and across time. Variations in the level of vulnerability do not
correlate with variations in nuclear policies. Israel, South Africa, South Korea, and Taiwan are
vulnerable to conventional attacks, yet all but Israel have renounced nuclear weapons . When
one regional state acquires or seeks nuclear weapons, the others do not necessarily follow. The
certainty—or uncertainty—of security guarantees often does not influence nuclear decisions.
Such guarantees played no role in the decisions of Argentina, Brazil, Egypt, and South Africa to
abandon their quests for nuclear weapons.

No runaway proliferation- states are cutting back and NPT solves

Ho ’11 [Flora, writer for The Sydney Globalist, an international affairs magazine published at the University of Sydney, Australia
and is in her fourth year of a combined degree in International Studies and Law, “Nuclear Proliferation: Crisis, Destiny, or an Excuse
for Using Force?” http://thesydneyglobalist.org/archives/734]

Whilst all of these may truly be a legitimate basis for labelling nuclear
proliferation as an international crisis, the crisis
narratives must be used with caution. Such an attitude could breed proliferation fatalism, the
belief that any government determined to acquire nuclear weapons will eventually do so , implying
that proliferation simply cannot be checked, and there is no point in attempting to curb the inevitable. This
belief fails to
consider the fact that many countries have actually curbed their nuclear ambitions.
International instruments such as the NPT have so far achieved impressive results, and defiance ,
though sadly existent, has been limited. Rather than accepting what falsely appears to be
inevitable, the international community should work harder to prevent the unthinkable .
---Prolif- AT: Rogue prolif

Rogue proliferators can’t successfully get a bomb

Hymans ‘12 [Jacques E. C. Hymans is Associate Professor of International Relations at the University of Southern California. His
most recent book is Achieving Nuclear Ambitions: Scientists, Politicians, and Proliferation, “Botching the Bomb: Why Nuclear
Weapons Programs Often Fail on Their Own-and Why Iran's Might, Too,” Foreign Affairs91. 3 (May/Jun 2012): 44-53, Proquest]

National income is only one dimension of development, however, and in this case it is not the most important one. As the political
scientist Francis Fukuyama has stressed, despite strong rates of economic growth, most developing countries struggle to establish
high-quality state bureaucracies. And a dysfunctional bureaucracy is likely to produce a dysfunctional
nuclear weapons project. Nuclear research and development organizations depend heavily on intense commitment, creative
thinking, and a shared spirit of cooperation among large numbers of highly educated scientific and technical workers. To elicit this
positive behavior, management needs to respect their professional autonomy and facilitate their efforts,
and not simply order them around. Respect for professional autonomy was instrumental to the brilliant successes of the earliest
nuclear weapons projects. Even in Stalin's Soviet Union, as the historian David Holloway has written, "it is striking how the apparatus
of the police state fused with the physics community to build the bomb. . . . [The physics community's] autonomy was not destroyed
by the creation of the nuclear project. It continued to exist within the administrative system that was set up to manage the project."
By contrast, most rulers of recent would-be nuclear states have tended to rely on a coercive,
authoritarian management approach to advance their quest for the bomb, using appeals to scientists'
greed and fear as the primary motivators. That coercive approach is a major mistake, because it
produces a sense of alienation in the workers by removing their sense of professionalism . As a
result, nuclear programs lose their way. Moreover, underneath these bad management choices lie bad management
cultures. In developing states with inadequate civil service protections, every decision tends to become politicized, and state
bureaucrats quickly learn to keep their heads down. Not even the highly technical matters faced by nuclear scientific and technical
workers are safe from meddling politicians. The result is precisely the reverse of what the politicians intend: not heightened
efficiency but rather a mixture of bureaucratic sloth, corruption, and endless blame shifting. Although it is difficult to measure the
quality of state institutions precisely, the historical record strongly indicates that the more a state has conformed to the professional
management culture generally found in developed states, the less time it has needed to get its first bomb and the lower its chances
of failure. Conversely, the
more a state has conformed to the authoritarian management culture
typically found in developing states, the more time it has needed to get its first bomb and the higher its
chances of failure.
Prolif Cred Answers
Frontline

Current restriction regime solves credibility

NEI 12 Nuclear Energy Institute, May 2012, Issues in Focus: Nuclear Energy Exports and Nonproliferation,
www.nei.org/resourcesandstats/documentlibrary/newplants/whitepaper/issues-in-focus-nuclear-energy-exports-and-
nonproliferation

These imperatives are inextricably linked. To maintain U.S. influence over global nonproliferation policy and international nuclear
safety, the U.S. commercial nuclear energy sector must participate in the rapidly expanding global market for nuclear energy
technologies (439 commercial nuclear reactors in operation around the world, 65 under construction, 162 planned or on order).
Without U.S. commercial engagement, the United States would have substantially diminished influence over other nations’
nonproliferation policies and practices. U.S. technology and U.S. industry are a critical engine that drives U.S. nonproliferation
policies. A successful nuclear trade and export policy must be a partnership between government and industry. A Section 123
Agreement is a prerequisite for U.S. commercial nuclear exports. It is also promotes U.S. nonproliferation interests.
Section 123 Agreements already include provisions governing enrichment and reprocessing of U.S.-
controlled nuclear material, including a prohibition on enrichment or reprocessing without prior U.S. consent. Any effort in U.S. 123
agreements to impose additional restrictions on enrichment and/or reprocessing of nuclear material controlled by other
countries is seen by many countries as an overreach by the United States. It would be counterproductive to require
other nations to forswear enrichment and reprocessing in order to execute a Section 123 agreement with the United States. Most
nations would refuse to do so, and would simply turn to other commercial nuclear suppliers – France, Russia and
others that do not impose such requirements. Without a Section 123 agreement, the United States cannot engage
in commercial nuclear trade, and thus has substantially diminished influence over nonproliferation. Unilateral
requirements, imposed in the name of nonproliferation, could have the perverse effect of undermining U.S.
influence over nonproliferation policy.

No impact to non-prolif cred-countries won’t accept U.S. leadership

Karp ’12 [Regina, associate professor of political science and the director of the interdisciplinary Graduate Program in
International Studies (GPIS) at Old Dominion University. She has published books and articles on international security including
works on German security pohcy and the transformation of war, “Nuclear Disarmament: Should America Lead?” Political Science
Quarterly, Volume 127, Number 1, 2012]

Second, U.S. commitment alone, even if sustained, cannot craft an enduring new balance between
possession goals and milieu goals. From the perspective of international order, there is an
inherent tension between possession goals and milieu goals. By their very nature, possession goals are
about hierarchy. The dilemma is that in an anarchic international system, one cannot get miheu goals for all without
possession goals for some. What has changed is the price tag for claiming possession goals without attending to milieu goals. The
United States has begun to reassess the price it is willing to pay for possession goals in order to
make its milieu goals more effective and attractive. There is little evidence of other nuclear
powers willing to act on this logic .^' If the United States fails to engage other nuclear powers in re-evaluating the role of
nuclear weapons in international security, the domestic consensus could quickly unravel.^" Third, U.S. leadership in arms
control is challenged internationally . The very same security environment that is reshuffling the relationship between
national nuclear arsenals and proliferation concerns also demands that milieu goals themselves cease to reflect preferred visions of
governance. To be sure, reduced American reliance on nuclear weapons and sustained advocacy of
nonproliferation cooperation can have important milieu effects. From a perspective of world
order, however, these effects continue to represent American-crafted or, at a minimum, American-led
Western milieu goals. For those who do not share these goals, they appear as just another form
of dominance. In a world of rising powers, others will demand voice in how rules of governance
are created and implemented. It is therefore paramount that creating sustainable milieu goals for a new security order
must include rule-making power for others. This makes a twenty-first century security order a collective challenge and qualifies the
role of American leadership. The United States can lead, but only with due regard to the input of other players. More than
buying into a new security order, others win demand a seat at the table .

US nuclear leadership is strong- ensures prolif credibility and global safeguards

BPI ’12 [Bipartisan Policy Center, nonprofit research organization founded by former Senate Majority leaders, “Maintaining U.S.
Leadership in Global Nuclear Energy Markets,” July, http://bipartisanpolicy.org/sites/default/files/Leadership%20in%20Nuclear
%20Energy%20Markets.pdf]

Nuclear power already plays an important role in the U.S. energy supply mix: The nation’s existing fleet of 104 reactors currently
accounts for close to 20 percent of overall electricity production. In many parts of the country, nuclear plants help to assure grid
stability and have been a major source of cost-effective, low-carbon base-load power for decades. The
NRC, the industry’s chief
regulatory overseer, is expected to approve extension of the operating licenses for most of these
plants to 60 years while striving for improved safety and increasingly efficient operations . At
present, the domestic nuclear industry is looking at limited opportunities for expansion in terms of increasing
the number of U.S. plants. Currently, four new Generation III+ nuclear reactors have been
licensed by the NRC and are under construction in the Southeast. In addition, the Tennessee Valley Authority has
restarted construction activities at Watts Bar II. Given this near-term expansion, the United States will
continue to be a world leader in the development of advanced reactor technologies , including
Generation III+ advanced passive reactors and SMRs. International interest in developing new nuclear-
generating capacity, on the other hand, presents potentially substantial business opportunities for the
domestic nuclear industry. Commercial nuclear exports generate obvious economic benefits for
U.S. firms and for the nation’s overall balance of trade . Importantly, they also help the United States
retain a major role in the evolution and maintenance of international nuclear safety and
nonproliferation regimes. Other nations not only look to the U.S. industry for operational
expertise, they see the NRC as setting the international gold standard for safety and physical
security regulation. DOE’s National Nuclear Security Administration, meanwhile, has a great deal of influence
over the nonproliferation aspects of international fuel-cycle issues .
---Prolif Cred- Squo Solves

1NC NEI ev—current regime solves—nuclear co-op is at an all-time—diplomatic negotiations


and market integration means sufficient cred now—only a risk more regulation upsets current
balance

NPT solves rapid proliferation- sets strong international framework that bolsters the nuclear
taboo

Van der Meer ’11 [Sico, Research Fellow at the Netherlands Institute of International Relations ‘Clingendael’ and a PhD
Candidate at the Erasmus University Rotterdam; his PhD project on nuclear proliferation dynamics is financially supported by the
Dutch non-governmental organisation IKV Pax Christi. “Not that bad: Looking back on 65 years of nuclear non-proliferation efforts,”
Security and Human Rights 2011, no.1]

Although the
NPT has a history of being heavily criticized, it has been very successful in seriously slowing down
the spread of nuclear weapons. The treaty is nowadays signed and ratified by all states of the
world except three: Israel, India and Pakistan. Moreover, since the existence of the NPT only five states
have obtained nuclear weapons. Of these five, three are the non-signatory states of Israel, India
and Pakistan, as well as one state that was not a member state at the time of its nuclear weapon
production, but later dismantled its nuclear arsenal and joined the NPT: South Africa. The only state that
signed the NPT and still acquired nuclear weapons is North Korea, although serious doubts exist as to the usability of its nuclear
weapons.7 Themain success of the NPT is in general explained by the norm-setting function of the
treaty: because of the broad, almost universal support for the treaty a moral taboo against
nuclear weapons has been created, which shapes international and domestic debates about this
category of weapons. Violating these international norms will result in severe constraints to any
state, such as political, economic and possibly even military reactions.8

Global nuclear safeguards are strong and effective- NPT proves

Kidd ’10 [Steve Kidd is Director of Strategy & Research at the World Nuclear Association, where he has worked since 1995 (when
it was the Uranium Institute), “Nuclear proliferation risk – is it vastly overrated?” July 23, http://www.neimagazine.com/story.asp?
storyCode=2056931]

Nevertheless, over the past 35 years, the International Atomic Energy Agency’s (IAEA) safeguards system under
the Nuclear Non-proliferation Treaty (NPT) has been a conspicuous international success in curbing the
diversion of civil uranium into military uses. Most countries have indeed renounced nuclear
weapons, recognising that possessing of them would threaten rather than enhance national
security. They have therefore embraced the NPT as a public commitment to use nuclear
materials and technology only for peaceful purposes. Parties to the NPT agree to accept
technical safeguards measures applied by the IAEA, complemented by controls on the export of
sensitive technology from countries such as UK and USA through voluntary bodies such as the
Nuclear Suppliers’ Group (NSG). Safeguards require that operators of nuclear facilities maintain and
declare detailed accounting records of all movements and transactions involving nuclear
material. The aim is to deter the diversion of nuclear material from peaceful use by maximising the risk of early detection. At a
broader level they provide assurance to the international community that countries are honouring
their treaty commitments to use nuclear materials and facilities exclusively for peaceful
purposes. In this way safeguards are a service both to the international community and to
individual states, who recognise that it is in their own interest to demonstrate compliance with
these commitments. All NPT non-weapons states must accept these full-scope safeguards, while
facility-specific safeguards apply in the five weapons states (USA, Russia, UK, France and China) plus the non-
NPT states (India, Pakistan and Israel).

Nonproliferation regime solves

Allison 10 (Graham, Director of the Belfer Center for Science and International Affairs at Harvard, January)
After listening to a compelling briefing for a proposal or even in summarizing an argument presented by himself, Secretary of
State George Marshall was known to pause and ask, "But how could we be wrong?" In that spirit, it is important to examine the
reasons why the nonproliferation regime might actually be more robust than it appears . Start
with the bottom line. There are no more nuclear weapons states now than there were at the end
of the Cold War. Since then, one undeclared and largely unrecognized nuclear weapons state, South Africa, eliminated its
arsenal, and one new state, North Korea, emerged as the sole self-declared but unrecognized nuclear weapons state. One
hundred and eighty-four nations have forsworn the acquisition of nuclear weapons and
signed the NPT. At least 13 countries began down the path to developing nuclear weapons
with serious intent, and were technologically capable of completing the journey, but stopped short of the finish
line: Argentina, Australia, Brazil, Canada, Egypt, Iraq, Italy, Libya, Romania, South Korea,
Sweden, Taiwan, and Yugoslavia.
---Prolif Cred- US Cred High

US non-prolif leadership is strong- we control the G8 and are promoting global nuclear
safeguards

Bigongiari 12 [Jeffrey, MA in international affairs, Reporter at BioPrepWatch, “U.S. assumes leadership of G8′s non-proliferation
bodies,” 1-25 http://www.bioprepwatch.com/us_bioterror_policy/u-s-assumes-leadership-of-g8s-non-proliferation-bodies/322874/]

As the head of the G8 in 2012, the U nited States has assumed leadership of the organization’s three
non-proliferation bodies. The United States now chairs the Non-proliferation Directors Group, the Global
Partnership Against the Spread of Weapons and Materials of Mass Destruction, and the Nuclear Safety and Security Group,
which will meet throughout the year to develop international nonproliferation objectives among
the group’s members. The GP, which was launched at the Kananaskis Summit in 2002, was created initially as a 10 year,
$20 billion initiative to support non-proliferation projects in Russia and the former Soviet Union. It has since expanded into
additional regions around the world and has extended its mission beyond 2012. It now includes
projects aimed at enhancing biological and radiological security, scientific engagement and U.N. nonproliferation
efforts. Assistant Secretary of State for International Security and Non-proliferation Thomas M. Countryman plans to host the first
meeting of the NPDG at the end of January at the U.S. Department of State. The NPDG is the G8 body that primarily consults on key
issues related to nuclear weapons and delivery systems. The
NSSG supports the implementation of the
International Atomic Energy Agency’s Action Plan for Nuclear Safety, which was created as a response to
the Fukushima accident in Japan. The group aims to support the development of nuclear plant safety
worldwide, and aids the G8’s participation in the IAEA Response and Assistance network. It was also launched at the Kananaskis
summit in 2012. It will hold its first meeting in March in Washington, D.C.
---Prolif Cred- US Cred Fails

Cradle to grave fails

McGoldrick 11 Fred, CSIS, spent 30 years at the U.S. State and Energy Departments and at the U.S. mission to the IAEA,
negotiated peaceful nuclear cooperation agreements with a number of countries and helped shape the policy of the United States to
prevent the spread of nuclear weapons, May 2011, Limiting Transfers of Enrichment and Reprocessing Technology: Issues,
Constraints, Options, http://belfercenter.ksg.harvard.edu/files/MTA-NSG-report-color.pdf

The political obstacles to offering broad-based cradle-to-grave services will be formidable. With
the possible exception of Russia, no major supplier country is currently in a position to provide power
reactor fuel to other countries with a firm commitment to take back the used nuclear fuel. France
and the UK accept foreign spent fuel for reprocessing but require the return of waste and recovered material to the sending state—
although if a market for MOX emerges, they could convert such material to MOX and sell it to utilities in other European states. No
other countries have yet been willing to accept other states’ spent nuclear fuel.

Sovereignty drives prolif—proves benign tech transfer can’t solve—won’t assuage want for
weapons

Lewis 12 Jeffrey Lewis, director of the East Asia Nonproliferation Program at the James Martin Center for Nonproliferation,
8/1/12, It's Not as Easy as 1-2-3, www.foreignpolicy.com/articles/2012/08/01/it_s_not_as_easy_as_1_2_3?page=full

Creating market incentives to discourage the spread of enrichment and reprocessing seems like a
reasonable thing to do - except that most states make nuclear decisions on something other
than a cost basis . Nuclear power enthusiasts have been no strangers to wishful thinking , starting
with claims that nuclear energy would be "too cheap to meter." Government decisions about nuclear power tend to
prioritize concerns about sovereignty and keeping technological pace with neighbors. It is not hard to see national
nuclear programs as something akin to national airlines - money-losing prestige projects that barely take
market forces into account. Often, aspiring nuclear states look to countries like the United States and Japan
as models. If such countries invest heavily in fuel-cycle services, developing states might try to copy them rather than
simply become their customers.

Supply-side efforts fail miserably—countries will find work arounds

Cleary 12 Richard Cleary, American Enterprise Institute Research Assistant, 8/13/12, Richard Cleary: Persuading Countries to
Forgo Nuclear Fuel-Making, npolicy.org/article.php?aid=1192&tid=30

The examples above show the limitations of both demand and supply side efforts. Supply side diplomatic
interventions, made before the transfer of technology, have been at times effective, particularly in precluding nuclear fuel-making in
the short term and buying time for more lasting solutions. However, as the Pakistan and Brazil cases illustrated, supply
side
interventions are no substitute for demand side solutions: Countries face political choices
regarding nuclear fuel-making . A nation set upon an independent fuel-making capacity , such as
Pakistan or Brazil, is unlikely to give up efforts because of supply side controls. Multilateral fuel-
making arrangements, as proposed repeatedly by the U nited States, have not materialized and
therefore seem to have had little tangible influence .

They can easily find other countries to provide them with nuclear tech

Hibbs 12 Mark, Carnegie Nuclear Policy Program Senior Associate, 8/7/12, Negotiating Nuclear Cooperation Agreements,
carnegieendowment.org/2012/08/07/negotiating-nuclear-cooperation-agreements/d98z

U.S. resolve to include a no-ENR pledge in the body of new bilateral agreements will be seen by some countries as arrogant and
unacceptable. Incorporating ENR terms into side-letters or preambles may be less offensive. That approach would also more easily
facilitate including reciprocal commitments by the United States into its 123 bargains with foreign countries. These might include
guaranteeing nuclear fuel supply through participation in the U.S. fuel bank, facilitating the country’s access to other back-up
sources of nuclear fuel, and, in the future, perhaps even taking back U.S.-origin spent fuel. The outcome of any negotiation for a
bilateral nuclear cooperation agreement will depend on the leverage both sides bring to the table. When
the United States
negotiated most of the 22 such agreements in force today, it was the world’s leading provider of
nuclear technology, equipment, and fuel. As the examples of Jordan and Vietnam show, unlike half a century ago,
nuclear newcomers today don’t need to buy American . The vendor field is populated by firms in
Argentina, Australia, Canada, the European Union, Japan, Kazakhstan, Namibia, Niger, Russia, and South Korea, and in the future
they will be joined by others in China and India. Governments in these countries do not seek to establish a
no-ENR requirement as a condition for foreign nuclear cooperation . Some of them, Australia and Canada
for example, have strong nonproliferation track records. Countries now seeking to form foreign industrial
partnerships to set up nuclear power programs have numerous options and they will favor
arrangements that provide them the most freedom and flexibility .

Prefer our evidence—countries will support proliferators

Ford 12 Chris, Hudson Institute, served until September 2008 as United States Special Representative for Nuclear
Nonproliferation, and prior to that as Principal Deputy Assistant Secretary of State responsible for arms control, nonproliferation,
and disarmament verification and compliance policy, 6/28/12, Perilous Precedents: Proliferation as Policy in Alternative Nuclear
Futures, www.hudson.org/index.cfm?fuseaction=publication_details&id=9026

I sometimes wonder, however, whether the seeming irresistibility of the case for nonproliferation may sometimes get in
the way of our analytical acuity as we look at the geopolitical environment. It is not uncommon in our diplomatic
relations, for instance, to hear it declared with great assurance that "Country Such-and-Such shares our interest in preventing
nuclear weapons proliferation" – and to have it be assumed, in effect, that if we just remind its leaders of this shared interest, they
will see the light and come around to our point of view. If
there is a problem in obtaining someone's
cooperation on nonproliferation matters, we tend to see this as being merely due to disagreements
over "tactics," or perhaps just some lack of capacity to do be helpful despite genuinely good
intentions. At worst, we suspect merely that others are holding out in order to bargain for as high a
price as possible in return for giving us the cooperation they really do, in their hearts, agree is
important anyway. This may be unwise on their part – or perhaps on ours for commoditizing such cooperation
by trying to purchase it through concessionary inducements – but we assume that such bargaining
doesn't really bespeak a significant difference of opinion about the value of nonproliferation. Only the would-
be proliferator regimes themselves, we might think, actually want nuclear weapons to spread – and even then one usually doesn't
have to look far to find some analyst who feels their pursuit of such devices is an unfortunate but understandable choice, taken only
grudgingly in the face of real or perceived foreign threats. Almost no one, we sometimes seem to assume, really
supports proliferation. But perhaps we should take a step back from the obviousness of such conclusions, and consider the
possibility that, "proliferation as policy" is not always felt to be an inherently irrational strategy. It is a
strategy that it remains powerfully in our interest to prevent others from adopting, of course. We probably miss something
important, however, if we see proliferation as no more than some kind of aberrance or confusion.
"Proliferation-as-policy" is actually something that seems to have appealed to a number of real-world
decision-makers in the past. Many of you doubtless knows these stories at least as well as I do, but let me offer some
examples: The Soviets gave Beijing a great deal of help in its weapons development, though by all
accounts Khrushchev balked just before fulfilling the final clauses of his 1957 cooperation agreement with the Chinese, and stopped
before providing Mao Zedong with an actual weapon prototype. French scientists provided a great deal of sensitive dual-use
technology to Israel, including the reactor at Dimona and a plutonium-separation capability that many observers believe to form the
core of Israel's weapons program even today. China helped Pakistan develop its nuclear weapons program, to the point – it
has been reported – that Beijing provided actual weapons designs. Even in the midst of the ongoing nuclear crisis over Iran's
previously-secret nuclear program, Russia provided Tehran with the nuclear reactor at Bushehr. Russian
entities apparently also designed the plutonium-production reactor that Iran is constructing at Arak. China is reported to have
provided uranium hexafluoride (UF6) feedstock to Iran's secret enrichment program in the early 1990s, and to have provided the
blueprints for Iran's uranium conversion facility. North Korea is reported to have provided Libya with UF6, and – more infamously –
to have constructed a plutonium-production reactor for Bashar al-Assad in Syria, though the Israelis bombed it in 2007. The official
story about Abdul Qadeer Khan's notorious nuclear smuggling ring is that it was some kind of a rogue operation, without official
support or encouragement, but few analysts today seem really to believe this. It is widely believed that the Pakistani government, or
a significant portion of it, was indeed complicit in Khan's operations. Finally, Saudi Arabia has long been rumored to
have helped finance Pakistan's nuclear weapons program. The evidence thus suggests that proliferation isn't
just about those who want to acquire nuclear weapons: at one point or another, a number of countries have apparently
concluded that supporting proliferation is in their interest.

Can’t solve prolif- security concerns outweigh

Bergner 12 [Jonathan D. Bergner, MA – Security Studies at Georgetown, Going Nuclear: Does the Non-Proliferation Treaty
Matter?, Comparative Strategy, 31:84–102, 2012, pages 84-102, 17 Feb 2012]

The NPT appears to have been largely irrelevant in dealing with the hard cases of nuclear proliferation. The special cases of late
nuclear reversal studied in this article—which the international community generally considers nonproliferation success stories—did
not occur because of (indeed, with little or no involvement by) the NPT regime. The research
suggests that if the
nonproliferation regime is having no impact on the nuclear decisionmaking of states, it is because it
does not adequately address the security-based concerns of states. How could it, when states such as Libya
and North Korea have flagrantly flouted their commitments to the regime? Or, in the case of Japan or
Taiwan, when one of the greatest potential threats is an already “legal” nuclear weapons state? Rather, issues of
security must be resolved in order for a state to be willing to meet the obligations of the NPT regime. All
of this begs the question: why did countries like Argentina and Brazil bother to sign the NPT after they had abandoned their nuclear
weapons programs? What was the point? The likely answer is that the international community asked them to do so. It was and is a
simple cost-benefit analysis, with very little cost. As Solingen notes, in return for signing and participating in the NPT, “a state can
secure certain international economic, financial, or political benefits—such as debt relief, export markets, technology transfer, food
imports, aid, and investments.” 79

Leads to backlash
NEI 12 Nuclear Energy Institute, June 2012, H.R. 1280: A Misguided Attempt to Control Enrichment and Reprocessing
Technologies, http://www.nei.org/resourcesandstats/documentlibrary/newplants/whitepaper/white-paper--hr-1280-a-misguided-
attempt-to-control-enrichment-and-reprocessing-technologies

Recent initiatives to deny E&R technologies to countries that do not possess them have provoked strong
objections from nuclear supplier and consumer countries alike. In 2004, NSG members and the broader international
community lodged forceful complaints against President Bush’s proposal for the NSG to provide fuel
assurances only to states that forswear E&R and refrain from transferring E&R technologies to any state that does
not possess them. Many nations consider such efforts discriminatory and in violation of sovereign
rights specifically guaranteed by the Nuclear Non-Proliferation Treaty (NPT) to nonnuclear-weapons states. Uranium
producer countries like Australia and Canada have also objected on grounds that they may one day wish to enrich the uranium they
produce.

The Non-Aligned Movement (NAM), which includes many prospective partners for U.S. nuclear
cooperation, has strongly opposed restrictions on E&R in various international fora, including the IAEA Board of
Governors and NPT Review Conferences. The final document of the 2010 NPT Review Conference affirmed the
inalienable rights of parties to use nuclear energy peacefully “without jeopardizing its policies
for international cooperation agreements and arrangements for peaceful uses of nuclear energy and its fuel-cycle
choices.” The document asserted a legitimate right, particularly among developing countries, to full access to nuclear material,
equipment and technology for peaceful purposes. The
document called on parties to “eliminate in this regard any
undue constraints inconsistent with the Treaty.”
---Prolif Cred- Alt Cause

Libya intervention crushed prolif credibility.

Bergner 12 [Jonathan D. Bergner, MA – Security Studies at Georgetown, Going Nuclear: Does the Non-Proliferation Treaty
Matter?, Comparative Strategy, 31:84–102, 2012, pages 84-102, 17 Feb 2012]

U.S. policymakers would be well advised to focus on shaping the security environment of
specific problem states as best it can. This may include providing incentives and
disincentives that focus on security, or perhaps offering limited security guarantees—such
as a commitment not to use nuclear weapons in a conflict or a commitment to respect
nations’ political sovereignty. Unfortunately, the recent military intervention undertaken
by the U.S. and NATO in Libya may have dealt a powerful blow to future
nonproliferation efforts by creating a strong disincentive for potential proliferators
to follow Qadhafi’s example. Surely it is not hard to imagine how different the
international community’s response to the unrest in Libya would have been had the
regime there actually had nuclear weapons. Whether intentional or not, the lesson that
was taught was that no good nonproliferation behavior goes unpunished.

First, bureaucratic constraints

NEI ’12 ["Improved Policies for Commercial Nuclear Trade Will Create American Jobs," June,
http://www.nei.org/resourcesandstats/documentlibrary/newplants/policybrief/improved-policies-for-commercial-nuclear-trade-
will-create-american-jobs?page=1~~]

While U.S. firms offer some of the most innovative and safest nuclear energy technologies,
they are hampered by cumbersome trade regulations, lack of coordination among the federal
agencies involved, an inefficient export licensing process, limited options for financing nuclear
exports and the absence of an international liability regime . These companies face intense competition from
suppliers in nations with less restrictive policies and substantial government subsidies for their nuclear industries. To facilitate
a greater U.S. role in the global commercial nuclear market, government support must be
integrated into a seamless mechanism that includes coordination of nuclear trade policy,
creation of bilateral agreements, export control reform and enhanced export financing . It also
is vital that the United States pursue the international adoption of effective civil nuclear
liability regimes.

Second, US arsenal causes resentment


Perkovich ’08 [George, vice president for studies and director of the Nonproliferation Program at the Carnegie Endowment for
International Peace, "Abolishing Nuclear Weapons: Why the United States Should Lead," October,
http://www.carnegieendowment.org/files/abolishing_nuclear_weapons.pdf~~]

This Brief summarizes four security interests that would be served by making the longterm project of abolishing nuclear weapons a
central purpose of U.S. policy: preventing proliferation; preventing nuclear terrorism; reducing toward zero the
unique threat of nuclear annihilation; and fostering optimism regarding U.S. global leadership. Each of these objectives
can be (and has been) pursued without the larger purpose of eliminating nuclear weapons . However,
the chances of success will steadily diminish if the few nuclear-armed states try to perpetuate
a discriminatory order based on haves and have-nots and if they enforce it firmly against some
states and hollowly against others. Such inequity breeds noncooperation and resistance when
what is needed now is cooperation to prevent proliferation, nuclear terrorism, and the failure of deterrence. Why should
everyone cooperate in enforcing a system that looks like it was designed to favor just a few?

Third, spent fuel rejection undermines prolif cred

Bengelsdorf ‘07 [Harold, consultant and former director of both key State and Energy Department offices that are concerned
with international nuclear and nonproliferation affair, "THE U.S. DOMESTIC CIVIL NUCLEAR INFRASTRUCTURE AND U.S.
NONPROLIFERATION POLICY", White Paper prepared for the American Council on Global Nuclear Competitiveness, May,
http://www.nuclearcompetitiveness.org/images/COUNCIL_WHITE_PAPER_Final.pdf]

During the last several decades, the U.S. has been struggling to implement a national
policy for management of commercial spent nuclear fuel , independently of whether it will result in
direct disposal of the spent fuel or reprocessing and recycle. In fact, the U.S. Government is presently in protracted litigation
with most U.S. utilities for monetary damages associated with DOE's inability to accept their spent fuel and dispose of it as
One adverse implication that this may have
called for in contracts that it has with each of these customers.
on U.S. nuclear nonproliferation policy is that it seriously undermines the ability of the
U.S. to offer fuel leasing or cradleto-grave fuel cycle services to foreign countries. The
ability to make such offers could be a valuable tool for discouraging the spread of
sensitive nuclear technologies.
---Prolif Cred- Turn: Backlash
Catch-all regimes fail

NEI 12 Nuclear Energy Institute, June 2012, H.R. 1280: A Misguided Attempt to Control
Enrichment and Reprocessing Technologies,
http://www.nei.org/resourcesandstats/documentlibrary/newplants/whitepaper/white-paper--
hr-1280-a-misguided-attempt-to-control-enrichment-and-reprocessing-technologies

The U.S. no longer plays a dominant role in the international nuclear market and, therefore, is in no
position to insist that other countries renounce E&R capabilities . GAO figures show that, between 1994 and
2008, the U.S. share of global nuclear reactor and component exports fell from 11 percent to 7 percent, and
fuel exports dropped from 29 percent to just 10 percent.5 Many countries still value U.S. cooperation agreements as a
means to gain access to U.S. nuclear technology and trade privileges, and for the ability to handle U.S.-flagged items. But unlike in
decades past, alternative sources of reactors, components and fuel are widely available. The age of
U.S. primacy on the international nuclear market is long over, and H.R. 1280’s insistence that countries renounce
E&R as a condition of a U.S. nuclear cooperation agreement amounts to a poison pill : no other
sup- pliers demand such a concession, and these suppliers will be the ones that benefit from
nations that consider the signing away of E&R rights too steep a price for U.S. collaboration. Countries Will Not Match the UAE’s
Bilateral Commitment The H.R. 1280 report points to the legally binding commitment by the UAE to forswear E&R in its bilateral
nuclear cooperation agree- ment as the proper standard for all U.S. nu- clear cooperation agreements. But the
UAE example
involves a unique set of economic and political circumstances, and if the U.S. insists that all partners for
nuclear cooperation follow suit, it is likely that few, if any , additional nuclear cooperation
agreements will be negotiated. As the H.R. 1280 report acknowledges, the UAE had already voluntarily adopted a
national policy to renounce E&R before negotiations for a U.S.-UAE 123 agreement began. The UAE’s decision was likely made easier
by the fact that E&R facilities in the UAE would not be profitable in the absence of plans to construct a large reactor fleet. And the
UAE does not possess domestic uranium reserves that could supply facilities to enrich fuel for international markets. The UAE’s
acquiescence on E&R should be viewed in its unique context : in 2006, the U.S. Congress had expressed a strong
lack of confidence in UAE, and blocked the UAE Government-owned firm Dubai Ports World from operating U.S. ports. Two years
later, the UAE was understandably concerned that Congress would ask hard questions about its intentions in the course of
considering the U.S.-UAE 123 agreement, and the renunciation of E&R in that agreement helped mute criticism. This set of
circumstances is unlikely to be repeated in other cases .

US leadership on prolif-resistant nuclear energy cooperation fails, causes backlash that


undermines nonproliferation

Hibbs 12 Mark, Carnegie Nuclear Policy Program Senior Associate, 8/7/12, Negotiating Nuclear Cooperation Agreements,
carnegieendowment.org/2012/08/07/negotiating-nuclear-cooperation-agreements/d98z

U.S. resolve to include a no-ENR pledge in the body of new bilateral agreements will be seen
by some countries as arrogant and unacceptable . Incorporating ENR terms into side-letters or preambles may be less
offensive. That approach would also more easily facilitate including reciprocal commitments by the United States into its 123
bargains with foreign countries. These might include guaranteeing nuclear fuel supply through participation in the U.S. fuel bank,
facilitating the country’s access to other back-up sources of nuclear fuel, and, in the future, perhaps even taking back U.S.-origin
spent fuel. The outcome of any negotiation for a bilateral nuclear cooperation agreement will depend on the leverage both sides
bring to the table. When the United States negotiated most of the 22 such agreements in force today, it
was the world’s leading provider of nuclear technology , equipment, and fuel. As the examples of Jordan and
Vietnam show, unlike half a century ago, nuclear newcomers today don’t need to buy American . The
vendor field is populated by firms in Argentina, Australia, Canada, the European Union, Japan, Kazakhstan, Namibia,
Niger, Russia, and South Korea, and in the future they will be joined by others in China and India. Governments in these
countries do not seek to establish a no-ENR requirement as a condition for foreign nuclear
cooperation. Some of them, Australia and Canada for example, have strong nonproliferation track records. Countries now
seeking to form foreign industrial partnerships to set up nuclear power programs have numerous
options and they will favor arrangements that provide them the most freedom and flexibility .
Equity in international nuclear affairs matters. By negotiating with its partners voluntary political
agreements, including side benefits to limit the application of sensitive technologies, instead of trying to legally compel
them to make concessions that are politically onerous , the United States can serve its nonproliferation and
security interests while avoiding the challenge to U.S. credibility that would follow from rigid application of a one-size-
fits-all policy. The United States should show nonproliferation leadership by generally discouraging countries without enrichment
and reprocessing capabilities from embarking in this direction. But negotiators need policy guidelines that provide for flexibility and
encourage them to create incentives to get desired results. To some extent, the current policy may be informed by the insight that
trying to negotiate no-ENR terms into the operative text of an agreement may fail , and that other
approaches may be more productive. It also reflects the reality that U.S. leverage on nuclear trade is declining .

Backlash guts market cooperation which means US nuclear never gets a foothold—turns case

McGoldrick 10 Fred, CSIS, spent 30 years at the U.S. State and Energy Departments and at the U.S. mission to the IAEA,
negotiated peaceful nuclear cooperation agreements with a number of countries and helped shape the policy of the United States to
prevent the spread of nuclear weapons, 11/30/10, The U.S.-UAE Peaceful Nuclear Cooperation Agreement: A Gold Standard or
Fool’s Gold?, http://csis.org/files/publication/101130_McGoldrick_USUAENuclear.pdf

On November 14,2010, a number of experts in the nonproliferation field wrote the president urging him
not to provide “US federal energy loan guarantees, federal contracts, or other subsidies or assistance to help foreign
government-backed nuclear firms expand their nuclear business in the US unless they have committed to
apply the nonproliferation standards (including with respect to enrichment and spent fuel recycling) established in
the U.S.-United Arab Emirates (UAE) civilian nuclear cooperation agreement in all of their future civilian nuclear
cooperation agreements.”11 However, any such proposal would not only compromise our ability to rebuild
our own nuclear industry and to compete in the international market , but it would also alienate close
allies whose cooperation is essential for strengthening the global nonproliferation regime. In sum, the United States is facing
an uphill battle to compete in the international nuclear market and cannot dictate
nonproliferation conditions that others will find unacceptable. Nations embarking on new nuclear
programs do not need to rely on the U nited S tates for their nuclear fuel, equipment, components, or
technology. They have alternatives and lots of them , as other states with nuclear programs have steadily built
up their nuclear export capacities, which in some cases are state run or state supported.
---Prolif Cred- Turn: Vietnam 123

Kills the Vietnam agreement

NEI 12 Nuclear Energy Institute, June 2012, H.R. 1280: A Misguided Attempt to Control Enrichment and Reprocessing
Technologies, http://www.nei.org/resourcesandstats/documentlibrary/newplants/whitepaper/white-paper--hr-1280-a-misguided-
attempt-to-control-enrichment-and-reprocessing-technologies

The H.R. 1280 report states that there is “no evidence to support the concern” that U.S. suppliers would be disadvantaged by the
requirement for countries to forswear E&R as a condition for U.S. nuclear cooperation. But the
cases of Vietnam and
Jordan suggest otherwise: it is not clear that these states will accept the same restrictions found in the
U.S.-UAE agreement. With negotiations for U.S. cooperation long stalled over E&R concerns, both countries’ nuclear energy
programs have moved ahead in partnership with non-U.S. suppliers. Unlike UAE, Jordan possesses sizeable uranium reserves—
around 200,000 tons—and has expressed an interest in eventually enriching fuel for export to international mar kets. And while
Jordan had reportedly considered some E&R limits in 2011 negotiations with the U.S., Dr. Khaled Toukan, head of the Jordanian
Atomic Energy Commission, has now publicly stated opposition to “restrictions outside of the NPT on a regional basis or a country-
by-country basis.”6 He has also criticized the UAE commitment, saying that country “has relinquished all of its NPT rights to sensitive
nuclear technology indefinitely. Why should we give up our rights?”7 It is unclear whether Jordan will ultimately accept E&R
restrictions in exchange for a U.S. nuclear cooperation agreement; but the country is clearly not waiting for such cooperation to
move ahead with its civil nuclear aspirations. Jordan has nearly finished the technology selection process for its first nuclear power
plant, a 1,000-megawatt reactor due in service in 2019. A Japanese-French consortium, as well as Russian and Canadian groups, are
seeking to win that bid, while South Korea has loaned Jordan $70 million to help fund a 5- megawatt nuclear research reactor worth
$130 million. Meanwhile, Jordan has granted France's AREVA exclusive rights over the next 25 years to mine uranium in the
country’s central region. Although Vietnam has indicated that it has no plans to develop E&R capabilities, sources
close to its
negotiations with the U.S. say Vietnam has so far chosen not to renounce E&R rights in
exchange for a U.S. nuclear agreement. As with Jordan, Vietnam has sought alternatives to U.S.
cooperation, including a $5.6- billion deal with Russia in late 2010 to build two 1,000-MW VVER reactors. Russia will also supply
the fuel for the reactors and handle its removal and reprocessing. In 2011, Vietnam concluded a separate reactor
deal with Japan for the supply of two additional reactors .

123 agreement key to strategic cooperation—key to check China flare-ups in the South
China Seas
Jha ‘10
Saurav Jha, The Diplomat and World Politics Review Contibutor, 9/15/10, Why a US-Vietnam Nuclear Deal?,
thediplomat.com/2010/09/15/why-a-us-vietnam-nuclear-deal/?all=true

But the engagement with Vietnam that the visit also demonstrated goes deeper than just this show of force— Washington is
looking to move beyond symbolism to engage in a genuine strategic partnership , the
cornerstone of which will be the US-Vietnam 123 nuclear cooperation agreement . Unsurprisingly, the
deal has already riled China and non-proliferation proponents alike, who note that the deal being offered to Vietnam is devoid of the
standard strings that have characterised other deals with emerging nuclear nations, including the United Arab Emirates. Most
notably, Vietnam won’t have to abandon having the option to carry out nuclear fuel cycle activities on its territory as the UAE had to.
This means that Vietnam, can, at least hypothetically, establish enrichment and reprocessing (ENR) facilities in its territory. Of
course, the agreement doesn’t mean that the United States is about to transfer any ENR technology to Vietnam—or that the latter is
in any hurry to set up its own such facilities. As Vuong Huu Tan, president of the government-affiliated Vietnam Atomic Energy
Institute, has noted: ‘Vietnam doesn’t intend to enrich as of now because of expensive and very sensitive technology.’ ENR
technology is anyway a closely guarded secret that only a handful of countries have the capacity to exploit on an industrial scale. But
while any country with a nuclear energy programme would typically like to retain a certain degree of independence—and the NPT
actually entitles all of its members to engage in full nuclear co-operation—the reality for many is that commercial and proliferation
sensitivities have prompted various restrictions and regimes to be put in place denying them any such technology. In addition, such
activities are simply prohibitively expensive for small and mid-sized nuclear estates. Yet while the UAE’s willingness to forsake fuel
cycle activity on its own soil seemed to provide a gold standard Washington could use for its nuclear dealings, th e
nature of the
Vietnam deal implies that a broader technological relationship could yet be crafted between
Hanoi and Washington. With its industrial activity in the north of the country expanding rapidly, Vietnam has been
prompted to explore nuclear power as a ‘clean’ way of meeting its growing electricity demands. But a 123 agreement with
the United States is unlikely to stop at nuclear co-operation . As US Secretary of State, Hillary Clinton, said during
her visit to Hanoi in July, ‘Ties between the two countries will be taken to the next level.’ What could this
mean? Certainly US firms can be expected to play an increasing role in Vietnam’s industrial development, something that would
likely necessitate a much broader deal than Washington has arranged with other countries. For example, a number of
instrumentation technologies are classified as dual use by the US State Department, but will be required if Hanoi wishes to exploit its
offshore hydrocarbon resources. Unlike in the Middle East, US oil majors aren’t already entrenched in Vietnam’s fossil fuel sector,
and an excessively restrictive deal would adversely affect their ability to compete. Such differences mean that the Vietnam
arrangement is more akin to the India nuclear deal than the one with the UAE, a point no more evident than at the strategic level.
Indeed, although it’s on a quite different scale, the philosophy and rationale underpinning a US-Vietnam 123 are remarkably similar
to the Indo-US nuclear deal. So why is the United States so interested in making an India-like exception to its nuclear arrangements
Vietnam is a potentially sympathetic US partner in any
with Vietnam? China. As another wary neighbour of China,
attempts to keep expansionist Chinese ambitions in the S outh C hina S ea in check . With a long
maritime tradition and a knack for military upsets (the Vietnamese have managed to defeat the French, Americans
and Chinese on different occasions), combined with its very sizeable armed forces, Vietnam is potentially an
indispensable ally in any possible regional flare-up .

Extinction
Wittner ‘11 (Lawrence S. Wittner, Emeritus Professor of History at the State University of New York/Albany, Wittner is the
author of eight books, the editor or co-editor of another four, and the author of over 250 published articles and book reviews. From
1984 to 1987, he edited Peace & Change, a journal of peace research., 11/28/2011, "Is a Nuclear War With China Possible?",
www.huntingtonnews.net/14446)

While nuclear weapons exist, there remains a danger that they will be used . After all, for centuries
national conflicts have led to wars, with nations employing their deadliest weapons . The current
deterioration of U.S. relations with China might end up providing us with yet another example
of this phenomenon. The gathering tension between the United States and China is clear enough.
Disturbed by China’s growing economic and military strength, the U.S. government recently challenged China’s
claims in the South China Sea, increased the U.S. military presence in Australia, and deepened U.S.
military ties with other nations in the Pacific region. According to Secretary of State Hillary Clinton, the United
States was “asserting our own position as a Pacific power.” But need this lead to nuclear war ? Not necessarily. And
yet, there are signs that it could. After all, both the United States and China possess large
numbers of nuclear weapons. The U.S. government threatened to attack China with nuclear
weapons during the Korean War and, later, during the conflict over the future of China’s
offshore islands, Quemoy and Matsu. In the midst of the latter confrontation, President Dwight Eisenhower declared
publicly, and chillingly, that U.S. nuclear weapons would “be used just exactly as you would use a bullet or anything else.” Of course,
China didn’t have nuclear weapons then. Now that it does, perhaps the behavior of national leaders will be more temperate. But the
loose nuclear threats of U.S. and Soviet government officials during the Cold War, when both nations had vast nuclear arsenals,
should convince us that, even as the military ante is raised, nuclear saber-rattling persists. Some
pundits argue that
nuclear weapons prevent wars between nuclear-armed nations ; and, admittedly, there haven’t been very
many—at least not yet. But the Kargil War of 1999, between nuclear-armed India and nuclear-armed Pakistan,
should convince us that such wars can occur. Indeed, in that case, the conflict almost slipped into a
nuclear war. Pakistan’s foreign secretary threatened that, if the war escalated, his country felt free to use “any weapon” in its
arsenal. During the conflict, Pakistan did move nuclear weapons toward its border, while India, it is claimed, readied its own nuclear
missiles for an attack on Pakistan. At the least, though, don’t
nuclear weapons deter a nuclear attack ? Do they?
Obviously, NATO leaders didn’t feel deterred , for, throughout the Cold War, NATO’s strategy was
to respond to a Soviet conventional military attack on Western Europe by launching a Western
nuclear attack on the nuclear-armed Soviet Union. Furthermore, if U.S. government officials really
believed that nuclear deterrence worked, they would not have resorted to championing “Star
Wars” and its modern variant, national missile defense. Why are these vastly expensive—and probably unworkable—
military defense systems needed if other nuclear powers are deterred from attacking by U.S.
nuclear might? Of course, the bottom line for those Americans convinced that nuclear weapons
safeguard them from a Chinese nuclear attack might be that the U.S. nuclear arsenal is far
greater than its Chinese counterpart. Today, it is estimated that the U.S. government possesses over five thousand
nuclear warheads, while the Chinese government has a total inventory of roughly three hundred. Moreover, only about forty of
these Chinese nuclear weapons can reach the United States. Surely the United States would “win” any nuclear war with China. But
what would that “victory” entail? Anuclear attack by China would immediately slaughter at least 10
million Americans in a great storm of blast and fire, while leaving many more dying horribly of sickness and radiation
poisoning. The Chinese death toll in a nuclear war would be far higher . Both nations would be
reduced to smoldering, radioactive wastelands. Also, radioactive debris sent aloft by the nuclear
explosions would blot out the sun and bring on a “nuclear winter” around the globe—
destroying agriculture, creating worldwide famine, and generating chaos and destruction.
Moreover, in another decade the extent of this catastrophe would be far worse. The Chinese government is currently
expanding its nuclear arsenal, and by the year 2020 it is expected to more than double its
number of nuclear weapons that can hit the United States. The U.S. government, in turn, has plans to
spend hundreds of billions of dollars “modernizing” its nuclear weapons and nuclear production facilities
over the next decade. To avert the enormous disaster of a U.S.-China nuclear war , there are two obvious
actions that can be taken. The first is to get rid of nuclear weapons, as the nuclear powers have agreed to do but thus far have
resisted doing. The second, conducted while the nuclear disarmament process is occurring, is to improve
U.S.-China
relations. If the American and Chinese people are interested in ensuring their survival and that
of the world, they should be working to encourage these policies .
---Prolif Cred- SMRs Don’t Solve

Military SMR development causes international backlash -- devastates US nonprolif


leadership.

Smith, ‘11
[Terrence P., Program Coordinator and Research Assistant, William E. Simon Chair in Political Economy -- CSIS, “An Idea I Can Do
Without: “Small Nuclear Reactors for Military Installations”,” Center for Strategic & International Studies, http://csis.org/blog/idea-i-
can-do-without-small-nuclear-reactors-military-installations]

the DoD
What are the alternatives to small nuclear reactors (assuming we want to maintain a large oversees military presence)? The NDU report makes the point that

has already been experimenting with “an array of initiatives on energy efficiency and renewable
and alternative fuels.” But, according to the report, “unfortunately, even with massive investment and ingenuity, these initiatives will be insufficient to solve
DOD’s reliance on the civilian grid or its need for convoys in forward areas.” While, to my knowledge, the DoD has not seen any huge relief from what I would call its

token attempts at ‘going green,’ it hardly writes off the possibility of alternative energy supplies short of going nuclear. The

report repeatedly emphasizes the point that “DOD’s “’first mover’ pursuit of small reactors could have a profound influence on the development of the industry,” and
cautions that “if DOD does not support the U.S. small reactor industry, the industry could be
dominated by foreign companies.” The U.S. nonproliferation agenda, if there is one, stands in
opposition to this line of thinking . Pursuing a nuclear technology out of the fear that others will
get it (or have it), is what fueled the Cold War and much of the proliferation we have seen and
are seeing today. It is a mentality I think we should avoid . I do not mean to say this report ignores the risks. In fact they
explicitly say, “We acknowledge that there are many uncertainties and risks associated with these reactors.” For example it says, Some key issues that

require consideration include securing sealed modules, determining how terrorists might use
captured nuclear materials, carefully considering the social and environmental consequences of dispersing reactors.
The report also points out that “from a financial perspective, small reactors represent
substantial losses in economies of scale .” These issues, which were briefly mentioned, hardly seem like small potatoes. The reports answer
to the issues raised: “making reliable projections about these reactors’ economic and technical performance while they are still on paper is a significant challenge,” and
“Nevertheless, no issue involving nuclear energy is simple.” On the other hand, the report argues, “failing to pursue these technologies raises its own set of risks for DOD.” “First,
small reactors may fail to be commercialized in the United States; second, the designs that get locked in by the private market may not be optimal for DOD’s needs; and third,
expertise on small reactors may become concentrated in foreign countries.” Yes these are important issue for a business stand, but I don’t find them to be the primary concern.

The reactors are purely for energy purposes, but in a world that seems to be growing tired of
U.S. military intervention, the idea of ensuring our ability to do so through the proliferation of
mobile nuclear reactors will hardly quell any hostile sentiment. In addition, it can only add fire to the
“nuclear = good” flame . So, while even under best case scenario, the reactors are completely proliferation proof and pose no direct threat to the
nonproliferation cause (ignoring the spreading of nuclear tech and knowledge in general), I have a tough time seeing how it helps. The report
concludes that the DoD “should seriously consider taking a leadership role on small reactors.” Since the 1970s, the report says, “in the United States, only the military has

the plans for small


overcome the considerable barriers to building nuclear reactors. This will probably be the case with small reactors as well.” For now,

nuclear reactors are “unfortunately,” for the most part, “caught between the drawing board and
production.”My point is, maybe that is where they should stay.

SMRs can’t prevent prolif.


Doyle & Newman, ‘11
[Neal, Former Program Coordinator -- Project on Managing the Atom, Andrew, Former Research Associate -- Project on Managing
the Atom, 1-18, “Modular Nuclear Reactors Can Meet Safe, Secure, and Proliferation Resistant Energy Demands,”
http://belfercenter.hks.harvard.edu/publication/20960/modular_nuclear_reactors_can_meet_safe_secure_and_proliferation_resist
ant_energy_demands.html]

Smaller, modular reactors are a potentially important part of the nuclear future because they could potentially
reduce the risk of accidents, terrorism, and proliferation. However, there are many other risks from the
existing nuclear complex that need to be managed . As nuclear energy use spreads, making
sure it is safe, secure, and used only for peaceful purposes will require a new global
management framework. This should include new or strengthened institutions that can
effectively accomplish agreed safety, security, and nonproliferation goals while respecting
states’ interests in sovereignty and energy security . Russia and the United States, working with other countries,
should lead an international negotiation of effective global nuclear safety standards, binding on all participants. Improving
safety will also require all states to: strengthen existing regulatory approaches, and establish
effective nuclear regulation in “newcomer” states building their first nuclear power plants, to align with the
global standard; build “reporting cultures” in which all staff are encouraged to report and
resolve all problems that arise that could have an effect on safety; and commit to accepting IAEA-led peer
reviews for major civilian facilities. Leading nuclear states must also work together to forge
effective global standards for nuclear security , building on agreements already in place, such as the amended
Convention on Physical Protection of Nuclear Materials and Facilities, the International Convention on the Suppression of Nuclear
Terrorism, and the IAEA’s physical protection recommendations. Expanded exchange of international best
practice and more comprehensive and detailed reporting on safety and security-related
incidents is vital to this mission. If, on the other hand, nuclear energy is pursued  without such
measures, the result could be both dangerous and inimical to the conditions necessary to
achieve and sustain large-scale nuclear growth. Even a single catastrophe – whether a Chernobyl-scale
accident, a successful sabotage (a “security Chernobyl”), or worse yet, a terrorist nuclear bomb – would severely
undermine prospects for nuclear growth .
Rare Earth Mineral Answers
Frontline

Recent trends show China has lost monopoly on REMs- and no conflict

MT ’13 (Mining-Technoloy, Online Mining/ Mineral Investment Website, “Rare earth minerals – China loses its grip”,
http://www.mining-technology.com/features/featurerare-earth-minerals-china/, February 18, 2013)

Blockbuster video game Black Ops II last year enthralled gamers, with its premise that the world could be brought to the brink of war
over China's dominance of rare earth minerals (REM). The premise is based on the scarcity of these minerals which are used for,
among many other things, powerful batteries, camera lenses, MRI scanners, modern electronics, such as iPods, TVs and computers,
and for renewable energies, such as solar panels and wind turbines, meaning they are integral to modern life. Although obviously far
fetched, at its inception Black Op II's narrative didn't seem so implausible. For many years, China had been responsible for producing
97% of all REMs. In recent years it has been known to use its monopoly of the industry as a geopolitical weapon, and to drive up the
price of REMs. However, more recently China's dominance has diminished and in November last year, the
country closed its largest mine, Baotou Steel mine, in a bid to maintain falling prices. As a direct result of China's tactics,
the exact opposite to Black Op II's narrative has occurred - the world hasn't fought China for its REM riches,
but found its own. Just recently, two REM companies, Molycorp and Lynas, have come online and
reserves of REM have been discovered all over the world. A 2011 British Geological survey
outlines 74 potential deposits worldwide. In January, the US Department of Energy announced it
will invest $120bn in a Critical Materials Centre which is, according to its director Alex King: "Spurred by the rare
earth crisis, in 2010 - 2011, when the price went up by a factor of ten, maybe 20" and because: "They [China] also showed a
China, Baotou Mine
willingness to use the availability of rare earths as a tool of international political relations". Meanwhile, in
has announced it is expecting its 2012 profits to falls by between 50 and 60%. Economy blogger and consultant and
trader in rare earth scandium, Tim Worstall, says that, after 50-odd years, China's "dominance of basic supply is
now gone." He adds: "Once China decided to throw its weight around, of course, the incentive was to
get into the business. Which is exactly what has happened."

Market solves and China doesn’t have a monopoly

Tamny 12 (John Tamny, Forbes Staff, China's "Rare Earths", and the Hypocrisy of the Obama Administration, 3-25-12,
http://www.forbes.com/sites/johntamny/2012/03/25/chinas-rare-earths-and-the-hyprocrisy-of-the-obama-administration/2/)

The above is important considering what Sternberg also alerted readers to in his recent Journal op-ed. Sternberg observed that

Beijing doesn’t have much control over rare-earths producers in China, and to prove his
point he noted that roughly 40% of exports to Japan “weren’t registered with Chinese customs
authorities.” Translated, rare earths from China are one way or the other exiting the country, and
once they do we, along with everyone else, have access to them at whatever the prevailing
market price may be. Just as every oil producing country on earth could “embargo” the U.S. with zero impact on our access
to their oil (we’d simply buy it from those they’re not embargoing), so long as rare earths are exiting China, U.S.
producers will be able to purchase them in the marketplace in the same way they buy other
commodities. Barring a decision by Beijing that forces Chinese producers to hoard rare earths, we’ll be able to buy them. All
this raises a question centering on what if the Chinese government chooses to do the unlikely, and
restricts all rare earths exports? If so, it’s none of our business. Just as we wouldn’t want the Chinese demanding from us
“American oil”, we shouldn’t presume to tell them what to sell to American producers. Of course the scenario just
mentioned is as mentioned, highly unlikely . For one, it’s hard to imagine that a still desperately
poor country would turn its back on the dollars it would gain if it were to hoard commodities
heavily demanded by market participants. Second, markets have for centuries proven expert at
working around shortages of most anything. Assuming the Chinese choose to halt or severely restrict rare earths
exports, it’s a fair bet that alternatives will soon find their way to those in need .

No impact to rare earth minerals


Hurst ’11 (Cindy Hurst is an analyst for the US Army’s Foreign Military Studies Office, Fort Leavenworth, KS, “Common
Misconceptions of Rare Earth Elements”, http://www.ensec.org/index.php?option=com_content&view=article&id=290:common-
misconceptions-of-rare-earth-elements&catid=114:content0211&Itemid=374, March 15 ,2011, LEQ)

On September 22, 2010 word began circulating among rare earth traders that China was ‘unofficially’ suspending rare earth exports
to Japan. The suspension occurred as a result of an ongoing territorial dispute between China and Japan, which escalated after Japan
detained the captain of a Chinese trawler near the disputed Senkaku/Diaoyu islands in the East China Sea. China demanded that
Japan release the captain while Japan insisted on holding him until they completed a full investigation over the incident. By
September 23, news that China had suspended rare earth deliveries to Japan captured the attention of the global media. Within
hours, China denied all reports of withholding rare earths, and Japanese prosecutors announced that the captain would soon be
released. As of late November, there has been conflicting information on the status of rare earth exports to Japan. Some claim that
exports have recommenced. Others say China continues to withhold exports. Still others claim that exports have recommenced but
they are not clearing customs. No matter what the status, there is a lesson that can be drawn about over-dependence on one
country for a vital resource. The news that China withheld rare earth exports to Japan has renewed urgency in revitalizing the rare
earth industry. Rare earth elements are vital in the production of hundreds of modern technologies. They can be found in cell
phones, i-Pods, computer hard drives, green technologies, and critical military weapons systems to name a few. China produces over
95% of the world’s rare earth elements, and the country has been steadily cutting back its export allocations since 2006, causing the
Western world to begin scrambling for alternatives. While governments and private companies seek out alternative sources, there
is often confusion from misinformation spread through the media and other sources . With all
the fuss over rare earth elements by government officials and investors, it is important that
decision makers have a basic understanding of the industry in order to promote good
decision-making. Four Experts The following article is derived from interviews with four of the
world’s leading rare earth experts. Dudley Kingsnorth is one of the foremost authorities in the rare earths industry. He
is the Executive Director of the rare earth consulting company Industrial Minerals Company of Australia (IMCOA). Gareth Hatch has a
background in materials science and metallurgy and is the co-founder of Technology Metals Research, LLC. Mark Smith, with over 25
years experience in the energy and mining industries, is the chief executive officer of Molycorp, which owns and operates Mountain
Pass, the only rare earth mine and processing facility in the United States. Finally, Karl Gschneidner is a distinguished professor in the
Department of Materials Science and Engineering and senior metallurgist at the US Department of Energy's Ames Laboratory.
Misconception #1: If China cuts off all exports of rare earth elements, we will no longer be able to manufacture modern day
technology Reports exaggerate what might happen if China stops all exports of rare earth elements. For example, in
an article titled “Global Supply of Rare Earth Elements Could be Wiped out by 2012,” it assumed that if China stopped
exporting rare earth elements to the rest of the world, “the Western world will be crippled by the collapse of available
rare earth elements. Manufacturing of everything from computers and electronics to farm machinery will grind to a halt. Electronics
will disappear from the shelves and prices for manufactured goods that depend on these rare elements will skyrocket.” This is a
fallacy. Since the late 1990s, China has been enticing manufacturing sectors to move their facilities to China with the promise that
they will have access to rare earth elements as long as their manufacturing operations remain in China. Mark Smith describes a
three-tiered priority system within China’s rare earth industry. At the top of the list as priority one are Chinese consumers. Not only
do Chinese consumers get first dibs at rare earth elements, but costs are maintained at the lowest level. The second priority-
consumers are international companies that move their manufacturing facilities to China. These consumers pay more than domestic
consumers, but less than the rest of the world. The balance of the world’s rare earth consumers fall under priority three. Obviously,
the real danger is for countries that are heavily dependent on China for their rare earth metals if demand at the two tier priority
levels is high which may crimp supply for the remainder of world consumers. Because of this system, more and more Western
companies have been progressively moving their manufacturing to China. Control over rare earths acts as an incentive in providing
more job opportunities to Chinese citizens. Clearly, manufacturing will not grind to a halt if China cuts off its exports. Also
noteworthy is that alternatives to rare earth elements do exist. However, these alternatives are not generally as
effective as the rare earths themselves. For example, prior to the discovery of rare earth magnets, many applications relied on alnico
and ferrite magnets, neither of which contain rare earths. Both the alnico and ferrite magnets are considerably weaker than their
rare earth counterpart. In order to match the same magnetic power as a rare earth magnet, these traditional magnets would have to
be made larger. This poses a great problem for many applications. According to Smith, “If you were to use ferrite magnets, as
opposed to Neodymium-Iron-Boron (Nd-Fe-B) magnets (Note: The Nd-Fe-B rare earth magnets are the strongest magnets available),
in hybrid and electric cars, the vehicles would never meet the definitions of hybrids for electric vehicles, or the zero emissions
vehicles. The size of each unit would become so big. As the size gets bigger, of course, the battery can only operate for so long.”
Misconception #2: A rare earth is a rare earth is a rare earth This misconception is truly in the eye of the beholder. According to
Gareth Hatch, there is a tendency to talk uniformly about rare earth elements as if they were one. This is misleading because they
are all different, with different levels of demand-consumption patterns and different levels of abundance. From an electronics point
of view, there are heavy rare earth elements, medium rare earth elements, and light rare earth elements. For our purposes they will
be broken down into light and heavy rare earth elements. The light rare earth elements as generally defined by industry—
lanthanum, cerium, praseodymium, neodymium, and samarium—are more abundant in the earth’s crust than the heavy rare earth
elements. Scientifically, according to Karl Gschneidner, while rare earth elements are similar in their chemical properties, when it
comes to what is known as the 4f-electrons each element is unique. The generally accepted standard for the breakdown of rare
earth elements by their applications is that used by IMCOA and Roskill. Once a rare earth oxide, chemical or metal is transformed
into a ‘value added’ product, it is no longer part of the rare earths data base. For example, the ‘magnetic’ rare earths have an
estimated gross value of $400-500 million; the gross value of the rare earth magnets is $4-5 billion; and the electric motors in which
they are used have a value that is another order of magnitude greater. This is the reason China seeks to maximize the ‘value added’
in China. Some of the elements are used as a means to process other materials, but are not present in the ultimate end products of
that process. For example, lanthanum is used as a catalyst for cracking petroleum products. Cerium is used to polish glass. On the
other hand, Smith warned that some catalyst manufacturers are sensitive about the issue. “Those people who make the catalyst
consider that catalyst to be the end product,” said Smith. Gschneidner, pointing out that 28% of rare earths produced in 2009 were
mixed rare earth compounds. These compounds are made up of mostly light rare earths and are used in catalysts and mischmetal.
At the other end of the “misconception stick,” according to Hatch, “I encounter people every week who believe that there are
primary neodymium mines, just like there are lead and copper mines.” This isn’t true. Rare earth elements are found grouped
together within the same mineral deposits. With the exception of scandium and promethium, it would be impossible to mine one
particular rare earth element without them being mixed in with the other rare earth elements. Misconception #3: China possesses
nearly all the world’s rare earth elements Past estimates have shown China to possess more than 55% of all known global rare earth
reserves. In 2008, according to the US Geological Survey (USGS), China possessed approximately 57% of base reserves. Base reserves
include all rare earth deposits including those that are not economical enough to be mined. Today, however, according to the USGS,
Zhanheng of the Chinese
the country possesses approximately 36.5% of economically viable deposits of rare earth. Dr. Chen
explained China’s declining percentage of the world’s rare earth reserves,
Society of Rare Earths
“With the discovery of new rare earth deposits worldwide and consumption of the proved rare
earth reserves in China, this ratio is declining, estimated as 35% and will continue to decline." (Dr. Chen
Zhangheng, “The Development and Policies of China Rare Earths Industry, Presentation, April 6, 2010). Some sources within China
offer even lower estimates. For example, according to Lin Donglu, Secretary General of the Chinese Society of Rare Earths, China’s
reserves account for only 25% to 30% of the world’s rare earth resources. According to Kingsnorth, approximately 35%-40% of
overall rare earth reserves (both heavy and light rare earths) are located in China. When it comes to heavy rare earths, though, the
country probably has over 50% of global supply. Presently it is impossible to ensure complete accuracy in China’s rare earth reserves.
While professional mining associations worldwide have set standards on calculating mining reserves, China has no such standards.
What is certain, though, is that China, with what reserves it does have, is responsible for over 95% of the global production of rare
earth elements. It is also important to note that not all the reserves and resources identified by the USGS and others are economic.
However, as China has lower environmental standards, lower chemical costs, and a better established rare earths infrastructure than
the rest of the world, a greater portion of its reserves are likely to be economically viable. Misconception #4: Heavy rare earth
elements are of more concern because they are less abundant than light rare earth elements Indeed, heavy rare earth elements are
much less abundant than the light ones. However, as Kingsnorth points out, typically the ratio in which rare earth elements are used
does not necessarily match the ratio in which they are found in the earth’s crust. In fact, demand rates have varied dramatically over
the past 70 years. For example, in the early 1960s lanthanum was used in the optical glass industry, cerium was widely used to polish
media, and didymium, which is a mixture of neodymium and praseodymium, was widely used in the glass industry for coloring.
However, there was no market for samarium and europium, and large stock piles of these materials grew. Then, in 1965, the United
States began to use europium as a red phosphor in color televisions. In the 1970s, samarium became a key ingredient for the
samarium cobalt permanent magnet, and by the 1990s, demand for the magnetic rare earths was taking off. According to
Kingsnorth, “We now have a situation in this decade in which the demand for magnets is growing much more quickly than the
demand for phosphors. So, the demand for neodymium is probably, to a certain extent, going to dictate the rate at which we
produce rare earths” since the rare earth elements are found within the same deposits. Currently, the
demand for the
heavy rare earth elements is significantly lower than the demand for the light rare earths. Smith insists
that “you don’t need a whole lot of production of these heavy rare earths to take care of the needs that this country has.” For
example, the projected global demand for terbium in 2015 is estimated to be less than 500 tons a year. Smith believes that if all the
advertised and promoted mines come online and begin producing terbium and other heavy rare earth elements needed in today’s
technologies, there could actually be a surplus, which would likely lower the cost of the oxides and make them uneconomical for
companies to produce. Hatch has a different perspective and believes that if all the advertised and promoted mines come online,
there is a greater probability for a global surplus of light rare earths than heavy ones. “This is because you still have to extract and
separate out the light rare earths present in a deposit in order to get at the heavies present,” he explained. Kingsnorth tends to
agree with Hatch for the medium term, but agrees with Smith in the longer term (post 2020). Gschneidner pointed out that
“everyone is fixated on the heavies, but you have to go through the lights to get the heavies out of the ores, unless you are lucky
with a heavy rare earth deposit.” Currently, the only viable heavy rare earth deposits right now are in South China’s ionic clays.
However, due to their higher value, those deposits with a greater proportion of heavy rare earths are receiving the most attention.
Therefore, normal market forces will likely work to achieve a balance in the longer term. Misconception #5: Rare earth
elements are rare The belief that rare earth elements are rare can be attributed to their name. The term “rare earth”
however, is a result of history. Kingsnorth explained that the name “rare earth” was generated about two centuries ago. At that
time, if a mineral could not produce a metal, for example, by heating it with coal, it was called an “earth.” When the first rare earth
elements were discovered, people were unable to produce a metal from them. In addition, the stone discovered was considered
rare. Therefore, it was called “rare earth.” It would take another 100 years from the time of the initial discovery before the metals
could be separated. Rare
earth elements are not rare at all, being found in low concentrations
throughout the Earth’s crust, and in higher concentrations in certain minerals. They are found
in almost all massive rock formations. However, their concentrations range from ten to a few hundred parts per
million by weight. Therefore, finding them where they can be economically mined is rare. Misconception #6: China is the number
one exporter of rare earth elements In general it is true that China is the largest exporter of rare earth elements. However, a few
notes of clarification need to be made. While China was once a heavy exporter of rare earth oxides, its
business model has been changing. Today, the country is increasingly pushing on exporting finished- and semi-finished
products. “The biggest misconception here,” said Hatch, “is that most of the 124,000 tons of rare earth produced by China is
exported, when clearly it isn’t,” at least not in their raw forms. Hatch explained that much of the rare earth produced by China is
made into finished products, which are then either exported or used within China. Smith pointed out that “The Chinese consumer is
now becoming much more powerful and their internal consumption is growing.” Chinese officials have been open about their
ambition to become more of an exporter of end products than rare earth oxides. On 2 September 2009, speaking at the annual
Minor Metals and Rare Earth Conference in Beijing, Wang Caifeng, Deputy Director General of the Materials Department at the
Ministry of Industry and Information Technology, pointed out that China would encourage the sales of finished rare earth products,
but limit the export of semi-finished goods. According to Kingsnorth, over the past 40 years, China has experienced a major
transition in its export products. In the 1970s, China exported rare earth mineral concentrates. In the 1980s, the country began
exporting mixed rare earth chemical concentrates. By the late 1990s, China was exporting magnets, phosphors, and polishing
powders. Now, in the new millennium, China has upgraded its production capabilities and is exporting finished products, such as
electric motors, computers, batteries, LCDs, and mobile phones. One of the best explanations for China increasing its role in the
production supply chain is one offered by Kingsnorth. “If China just mined the rare earth and sold the minerals (the minerals contain
the rare earth elements), that would employ hundreds of people,” he said. “If they separated rare earths and sold those (the oxides),
that would employ thousands of people. However, if they went all the way up the value added chain, and they can produce
computers, cameras, vehicles, and those types of things, that employs millions of people. That is their goal.” Misconception #7:
Recycling is a simple alternative Every year, hundreds of millions of tons of “e-waste” end up being sold to third world and
developing countries such as Ghana, Malaysia, Vietnam, Pakistan, China and India, where they are stripped of some of their
components and reassembled into working units. Those units that are unsalvageable are stripped of their copper, iron, or even
traces of gold. Rare earth elements, on the other hand, are not normally recycled because they are usually mixed with other
materials, making it difficult to separate them out. According to Kingsnorth, most applications use such small quantities of rare earth
that it is unlikely to be economical to recycle. Misconception #8: Every wind turbine has a neodymium-iron-boron magnet in it In
September 2010 a blogger talked about the shortage of rare earths. At the bottom of the blog was a photo of several old, worn-
down wind turbines, suggesting that the neodymium from these old wind turbines really should be recycled. By their rundown
appearance and older style, it was obvious that the wind turbines pictured did not use the neodymium-iron-boron, also known as
permanent magnet generators (PMG). Today, only 5%-10% of wind turbines use PMGs. Currently, there is a trend to develop larger
wind turbines. As these wind turbines grow in size the use of PMGs will likely also rise. In a forward looking statement, Kingsnorth
estimated that over the next ten years, the use of PMG driven wind turbines is likely to increase to 20% to 25% of total annual
installations. Meanwhile, according to Hu Bo-Ping, who spoke during the 6th International Conference on Rare Earth Development
and Applications in August 2010, China is aiming to increase its production/installation of wind turbines by 50% per annum for the
next 10 years. Furthermore, within the next five years, 25% of China’s wind turbines will be using rare earth PMGs.
Alternatives are opening now—increases supply

Broadband ’11 (October 26, 2011 http://www.voanews.com/english/news/asia/Chinas-Rare-Earth-Minerals-Supply-


Manipulation-Could-Backfire-132605798.html Manipulation of China's Rare Earth Minerals Supply Could Backfire Ivan Broadband |
Hong Kong

Unlike previous occasions when Beijing has manipulated supply, the world is now looking
increasingly prepared to move away from its dependence on Chinese rare earths . Manufacturers,
including Toyota and General Motors, are already developing processes that minimize the use of rare earths in vehicle design, says
Matthew Fusarelli, head of research at AME. “Rare earths generally have a very high degree of
substitutability," Fusarelli said. "So electronics manufacturers can, over time, change their production
processes to use rare earths more sparingly, if at all.” Andrew Bloodworth says it will not take many new
suppliers to alter the dependence on China. “The amount we use compared to industrial metals is absolutely tiny," he
explained. "Last year in the world we mined about 17, 18 million tons of copper. We mined about 130,-140,000 tons of rare earth. A
couple of new mines will change the picture completely.” Alternatives New
mines are being planned in the United
States, Russia and Australia. Other mines once moth-balled or not developed on concerns about viability are
moving towards production. Among these is the Molycorp mine in Mountain Pass, California. This closed in
2002 when China swamped the market with cheap supplies. Molycorp directors say they have discovered several new rare earth
deposits at the site and expect the old mine to be one of the world’s largest rare earth suppliers by 2014.
---REM- No China Monopoly

1NC Tamny—no Chinese dependence—China doesn’t control rare earth


exports, almost half escaped country without registration—no incentive to
hoard because of market incentives—markets will work around artificial
barriers

WTO ruling checks reliance and export restriction

Ojo ’11 (Double Blows to China in Rare Earth Minerals Dispute Bolaji Ojo, Editor in Chief 7/7/2011 (13) comments

The rare earth minerals market is headed for a more normal supply condition following a
World Trade Organization (WTO) ruling against Chinese export restrictions and the discovery
by scientists of a large deposit of the raw materials used heavily in high-tech equipment
manufacturing. The WTO this week declared China was wrong to impose restrictions on rare
earth metal exports and noted measures taken by the country in recent years have constrained
supply and led to unwarranted price increases . Without saying so directly, the WTO indicated China was illegally
manipulating the supply of minerals such as bauxite, coke, fluorspar, magnesium, manganese, silicon carbide, silicon metal, yellow
phosphorus, and zinc. The average price of these rare earth metals has shot up, and supply has been constrained since China, which
supplies 90 percent of the world demand, began imposing export restrictions in the last year. (See: The Truth About Rare Earths, Part
1 and The Truth About Rare Earths, Part 2.) The WTO did not outright say China's action was illegal; the agency does not use such
terms. It merely said in a July 5 report that the country's "export duties were inconsistent with the commitments that China had
agreed to in its Protocol of Ascension," and that "certain aspects of China's export licensing regime, applicable to several of the
products at issue, restrict the export of the raw materials and so are inconsistent with WTO rules." (Click here for the summary of
the WTO report.) The WTO panel finding took two years and followed complaints by a group of countries, including the United
States and a battery of interested third-party players. The US in June 2009 alleged that China had imposed various "restraints on the
exports," adding "there appear to be additional unpublished restrictive measures," being carried out by Chinese vendors. Here's
China's defense and the WTO's response, as outlined by the organization: China had argued in its defense that some of its export
duties and quotas were justified because they related to the conservation of exhaustible natural resources for some of the raw
materials. But China was not able to demonstrate that it imposed these restrictions in conjunction with restrictions on domestic
production or consumption of the raw materials so as to conserve the raw materials. The Panel acknowledged, however, that China
appears to be heading in the right direction in adopting a framework to justify its quotas under WTO rules, but that the framework is
not yet WTO-consistent as it still has to be put into effect for domestic producers. As for other of the raw materials, China had
claimed that its export quotas and duties were necessary for the protection of the health of its citizens. China was unable to
demonstrate that its export duties and quotas would lead to a reduction of pollution in the short- or long-term and therefore
contribute towards improving the health of its people. So, what does the finding against China mean for the global electronics
industry and the demand-supply condition in the rare earth market? First, the WTO did not specifically indicate what should follow
on its finding, but I assume a
verdict against China would mean the country has to roll back whatever
restrictions it has on the export of the minerals . China has not responded to the ruling. The verdict should
be a relief also for Western electronics equipment vendors and component suppliers, many of
which had become greatly concerned about their ability to source rare earth metals from China.
But even greater relief is on the way, according to the Information Network, a market
research and consulting firm. China's restrictions sparked frenzy for alternate sourcing for the
minerals over the last year, and it appears the industry might be able to dramatically reduce its
dependence on Chinese suppliers following a huge discovery. Robert Castellano, president of The Information
Network, noted in an email: In the past few days we have learned thatvast deposits of rare earth minerals, crucial in
making high-tech electronics products, have been found on the floor of the Pacific Ocean and can be readily
extracted. Discovered by Japanese scientists, it is estimated that rare earths contained in the deposits amounted to 80 to 100
billion metric tons, compared to global reserves currently confirmed by the US Geological Survey of just 110 million tons that have
been found mainly in China, Russia and other former Soviet countries, and the United States.

Illegal rare earth mining ensures Chinese quotas are irrelevant and provide 50% of global
supply

Barsher ’10 (New york Times, 12/29/10 Keith, “In China, Illegal Rare Earth Mines Face Crackdown,”
http://www.nytimes.com/2010/12/30/business/global/30smuggle.html?_r=2&ref=business

Rogue operations in southern China produce an estimated half of the world’s supply of heavy
rare earths, which are the most valuable kinds of rare earth metals. Heavy rare earths are increasingly vital
to the global manufacture of a range of high-technology products — including iPhones, BlackBerrys, flat-panel televisions, lasers,
hybrid cars and wind-power turbines, as well as a lot of military hardware. China
mines 99 percent of the global
supply of heavy rare earths, with legal, state-owned mines mainly accounting for the rest of
China’s output. That means the Chinese government’s only effective competitors in producing
these valuable commodities are the crime rings within the country’s borders .
---REM- Alt Sources

1NC Broadband—alternative sources solve REM dependence—huge mines opening up in


Japan, Australia, and Russia—means no reliance on China—US domestic production is set to
become largest supplier—no risk of resource entanglement

Just found 1,000 times more REMs in Japan—no risk of impact

Lee ’11 (Japan's 'Rare Earth' Mineral Discovery Could Fix iPad Supply Shortages The Huffington Post Amy Lee First Posted: 7/5/11
Updated: 9/4/11

Scientists found the minerals on the bottom of the Pacific Ocean, according to the Wall Street
Journal. The trove is said to include minerals such as gadolinium, lutetium, terbium and
dysprosium. The minerals could multiply the known supply on land by 1,000 times . Currently,
China produces the vast majority of the global supply of rare earth minerals and has used its 97
percent share to threaten export blockages . The low supply of such minerals has been a cause of concern for some
time, fueling fears that electronics prices could skyrocket, or that supply of the gadgets themselves could be reduced. "The
deposits have a heavy concentration of rare earths. Just one square km (0.4 sq mile) of deposits
will be able to provide one-fifth of the current global annual consumption ," Yasuhiro Kato, an
associate professor of earth science at the University of Tokyo, said in the report, according to
The Guardian. The minerals were discovered in sea mud at 3,500 to 6,000 meters below the surface of the sea in 78 locations,
with one-third of sites displaying rare earth content. Scientists estimate that there are 80 to 100 billion tons
of the minerals under the sea. Right now, 110 million tons have been found on land.

Joint ventures with Japan solve

Casey ‘10, 12/21/10


Simon, “Molycorp, Hitachi Metals Plan Rare-Earth Ventures,” http://www.businessweek.com/news/2010-12-21/molycorp-hitachi-
metals-plan-rare-earth-ventures.html

Molycorp Inc., owner of the world’s largest non-Chinese, rare-earth metals deposit, agreed to
form joint ventures with Japan’s Hitachi Metals Ltd. to produce alloys and magnets in the U.S.
The companies are expected to sign definitive agreements on the ventures in 2011, Greenwood
Village, Colorado-based Molycorp said today in a statement. Rare earths are a group of 17 metal elements
used in batteries, turbines, cell phones and catalysts to reduce auto exhaust emissions. Hitachi Metals, Japan’s largest maker of rare-
earths magnets, uses as much as 600 tons of the metals each year . The company is among manufacturers around the world
depending on rare-earth shipments from China, which controls 97 percent of the global supply. The
planned joint ventures
with Hitachi Metals “move our company and the U.S. one step closer to realizing the strategic
goal of re-establishing a complete rare-earth manufacturing supply chain in the U.S.,” Molycorp
Chief Executive Officer Mark Smith said in the statement. Molycorp rose $1.40, or 3.6 percent, to $40.65 of
8:43 a.m. before the start of regular trading on the New York Stock Exchange. The shares have more than doubled
since the company raised $394 million in an initial public offering in July to help fund the restart
of its Mountain Pass mine in California.

US to supply 50% of world supply alone with current operations

TMR ’10 (Technology Metals research 12/27/10 [“Rare-Earth-Metals Mine Is Key To US Control Over Hi-Tech Future,”
http://www.techmetalsresearch.com/2010/12/rare-earth-metals-mine-is-key-to-us-control-over-hi-tech-future/]

“I don’t believe that China is trying to chop the west off at the knees but it has a growing internal market that is driving the
demand,” said Gareth Hatch, an analyst at Technology Metal Research. “That reduces the amount they are willing to export.” That is
where Molycorp– the frontrunner for now in a global race to develop alternative production of
rare earth materials – hopes to step in. Since going public last July, the company has raised more
than $500m (£323m) to expand its production facilities at Mountain Pas s, a collection of rusting buildings
that date from the 1950s. This month, Sumitomo Corp of Japan invested $130m in return for guaranteed supplies of rare earths for
the next seven years. The company has also applied for department of energy loans. By
mid-2012, Molycorp aims to
produce 20,000 tonnes a year of nine of the 17 rare earths or about 25% of current western
imports from China. Smith suggested the company could possibly ramp up production to 40,000
tonnes within the next 18 months. He says Molycorp has exposed just 55 acres of the 2,200
acre site.

Large accessible reserve now—only a question of increased production

Mills ‘11, Energy Intelligence with Forbes magazine, 1/1/Mark P. “Tech’s Mineral Infrastructure – Time to Emulate China’s Rare
Earth Policies,” http://blogs.forbes.com/markpmills/2011/01/01/techs-mineral-infrastructure-time-to-emulate-chinas-rare-earth-
policies/

The strange thing, and thus the opportunity, is that rare earths are not in fact rare. The nomenclature is surely an artifact
of timing. Gold, which is rare, has been mined and used by humans for thousands of years . The family of rare earth
elements – including others like gadolinium (computers), terbiuim (lighting), lutetium (medical imaging), and yttrium (cellular
communications) – are geophysically speaking roughly 1,000 times more common than gold . But rare earth
elements were not known to exist until a couple of hundred years ago, and only in more recent history have their unique properties
been put to significant use. In due course, the world-outside-of-China, even America, could start to produce more
rare earths (as Jack Lifton has noted, and the Chinese have unsurprisingly suggested). But that will take time. These are after all
mines, often big mines, and the minerals extracted require complex and challenging refining to separate the chemically similar
constellations of rare earth elements that are always collocated in various mineral concentrations and combinations. It is in short an
annoyingly difficult, messy and thus capital-intensive process. For
policy makers looking for near-term
geopolitical comfort, and investors looking for opportunity, you go with what exists today and
may be able to scale quickly. Thus the investor enthusiasm for Molycorp’s re-opening of the
world’s formerly largest rare earths mine in California. The other near-term options reside in
resource-rich Canada and Australia. The southeast Australia project of Alkane Resources
(AU:ALK) is of special interest in part because of an abundance of dysprosium (as are related comments
from Gady Epstein, fellow Forbes’ blogger). More prominently known is Lynas (AX: LYC) and its rich Mt. Weld, West Australia, mine
and Malaysian processing facility, the latter a critical bottleneck feature in the entire rare earth food chain. (J.P. Morgan’s June 2010
initiation of coverage report on Lynas has a particularly good backgrounder on the entire subject.) As usual when it comes
to resources, there is a coterie of Canadians with mineable deposit s, including Avalon [AMEX:AVL], Great
Western Minerals (TSX VENTURE: GWG-V), Quest Rare Minerals (TSX VENTURE: QRM-V), and traders and stockpilers like Dacha
Strategic Metals (TSX VENTURE: DSM-V). Neomaterials [TO: NEM] is
of particular interest because of their
processing, refining and manufacturing of rare-earth-based magnets. There are a rising number
of private exploration and putative mining players and promising finds in places like Kvanefjeld
Greenland, or Steenkamposkraal South Africa. But new deposits and mines, not to mention processing facilities, will take
significant time to develop and open.
Resource Wars Answers
Frontline

No resource wars—multiple factors check

Tertrais 12 [Bruno, Senior Research Fellow at the Fondation pour la Recherche Strat gique (FRS) The Demise of Ares: The End
of War as We Know It? The Washington Quarterly • 35:3 pp. 722]

Future resource wars are unlikely . There are fewer and fewer conquest wars . Between the
Westphalia peace and the end of World War II, nearly half of conflicts were fought over territory. Since the end of the Cold War, it
has been less than 30 percent. 61 The invasion of Kuwaita nationwide bank robberymay go down in history as being
the last great resource war. The U.S.-led intervention of 1991 was partly driven by the need to maintain the free flow
of oil, but not by the temptation to capture it. (Nor was the 2003 war against Iraq motivated by oil.) As for the current
tensions between the two Sudans over oil, they are the remnants of a civil war and an offshoot of a botched secession process, not a
desire to control new resources. China’s and India’s energy needs are sometimes seen with apprehension: in light of growing
oil and gas scarcity, is there not a risk of military clashes over the control of such resources? This seemingly consensual idea
rests on two fallacies. One is that there is such a thing as oil and gas scarcity, a notion challenged by many energy experts. 62
As prices rise, previously untapped reserves and non-conventional hydrocarbons become economically
attractive. The other is that spilling blood is a rational way to access resources. As shown by the work of historians and political
scientists such as Quincy Wright, the economic rationale for war has always been overstated. And because of
globalization, it has become cheaper to buy than to steal. We no longer live in the world of 1941, when fear of lacking
oil and raw materials was a key motivation for Japan’s decision to go to war. In an era of liberalizing trade, many natural
resources are fungible goods. (Here, Beijing behaves as any other actor: 90 percent of the oil its companies produce
outside of China goes to the global market, not to the domestic one.) 63 There may be clashes or conflicts in regions in
maritime resource-rich areas such as the South China and East China seas or the Mediterranean, but they will be driven by
nationalist passions, not the desperate hunger for hydrocarbons. Only in civil wars does the question of resources such
as oil, diamonds, minerals, and the like play a significant role; this was especially true as Cold War superpowers stopped their
financial patronage of local actors. 64 Indeed, as Mueller puts it in his appropriately titled The Remnants of War, ‘‘ Many
[existing wars] have been labeled ‘new war,’ ‘ethnic conflict,’ or, most grandly ‘clashes of civilization.’ But in fact, most. . .are
more nearly opportunistic predation by packs, often remarkably small ones, of criminals, bandits, and thugs.’’ 65 It is
the abundance of resources, not their scarcity, which fuels such conflicts. The risk is particularly high when the
export of natural resources represents at least a third of the country’s GDP. 66

No resource wars

Allouche, 11 - Research Fellow at the Institute of Development Studies at the University of Sussex (Jeremy,. "The sustainability
and resilience of global water and food systems: Political analysis of the interplay between security, resource scarcity, political
systems and global trade" Food Policy, Volume 36, Supplement 1, January 2011, Science Direct)

Water/food resources, war and conflict The question of resource scarcity has led to many debates on
whether scarcity (whether of food or water) will lead to conflict and war. The underlining reasoning behind most of
these discourses over food and water wars comes from the Malthusian belief that there is an imbalance between

the economic availability of natural resources and population growth since while food production grows linearly,
Following this reasoning, neo-Malthusians claim that finite natural resources
population increases exponentially.

place a strict limit on the growth of human population and aggregate consumption; if these limits are
exceeded, social breakdown, conflict and wars result. Nonetheless, it seems that most empirical studies do
not support any of these neo-Malthusian arguments. Technological change and greater inputs
of capital have dramatically increased labour productivity in agriculture. More generally, the neo-
Malthusian view has suffered because during the last two centuries humankind has breached
many resource barriers that seemed unchallengeable. Lessons from history: alarmist scenarios, resource wars and international
relations In a so-called age of uncertainty, a number of alarmist scenarios have linked the increasing

use of water resources and food insecurity with wars. The idea of water wars (perhaps more than food wars) is
a dominant discourse in the media (see for example Smith, 2009), NGOs (International Alert, 2007) and within international organizations (UNEP,
2007). In 2007, UN Secretary General Ban Ki-moon declared that ‘water scarcity threatens economic and social gains and is a potent fuel for wars and conflict’ (Lewis, 2007). Of

this type of discourse has an instrumental purpose; security and conflict are here used for
course,

raising water/food as key policy priorities at the international level. In the Middle East, presidents, prime
ministers and foreign ministers have also used this bellicose rhetoric. Boutrous Boutros-Gali
said; ‘the next war in the Middle East will be over water, not politics’ (Boutros Boutros-Gali in Butts, 1997, p. 65). The
question is not whether the sharing of transboundary water sparks political tension and alarmist
declaration, but rather to what extent water has been a principal factor in international
conflicts. The evidence seems quite weak . Whether by president Sadat in Egypt or King Hussein in Jordan, none of these
declarations have been followed up by military action. The governance of transboundary water has gained increased attention
these last decades. This has a direct impact on the global food system as water allocation agreements determine the amount of water that can used for irrigated agriculture. The

None of the
likelihood of conflicts over water is an important parameter to consider in assessing the stability, sustainability and resilience of global food systems.

various and extensive databases on the causes of war show water as a casus belli. Using the
International Crisis Behavior (ICB) data set and supplementary data from the University of
Alabama on water conflicts, Hewitt, Wolf and Hammer found only seven disputes where water
seems to have been at least a partial cause for conflict (Wolf, 1998, p. 251). In fact, about 80% of the incidents relating to water
were limited purely to governmental rhetoric intended for the electorate (Otchet, 2001, p. 18). As shown in The Basins At Risk (BAR) water

event database, more than two-thirds of over 1800 water-related ‘events’ fall on the
‘cooperative’ scale (Yoffe et al., 2003). Indeed, if one takes into account a much longer period, the
following figures clearly demonstrate this argument . According to studies by the United Nations Food and Agriculture Organization
(FAO), organized political bodies signed between the year 805 and 1984 more than 3600 water-

related treaties, and approximately 300 treaties dealing with water management or allocations in international basins have been negotiated since 1945 ([FAO, 1978]
and [FAO, 1984]). The fear around water wars have been driven by a Malthusian outlook which equates

scarcity with violence, conflict and war. There is however no direct correlation between water
scarcity and transboundary conflict. Most specialists now tend to agree that the major issue is not scarcity per se but rather the allocation of
water resources between the different riparian states (see for example [Allouche, 2005], [Allouche, 2007] and [Rouyer, 2000]). Water rich countries have been involved in a
number of disputes with other relatively water rich countries (see for example India/Pakistan or Brazil/Argentina). The perception of each state’s estimated water needs really
constitutes the core issue in transboundary water relations. Indeed, whether this scarcity exists or not in reality, perceptions of the amount of available water shapes people’s

scarcity drives the process of co-operation


attitude towards the environment (Ohlsson, 1999). In fact, some water experts have argued that

among riparians ([Dinar and Dinar, 2005] and [Brochmann and Gleditsch, 2006]). In terms of international relations, the threat
of water wars due to increasing scarcity does not make much sense in the light of the recent
historical record . Overall, the water war rationale expects conflict to occur over water, and
appears to suggest that violence is a viable means of securing national water supplies, an
argument which is highly contestable. The debates over the likely impacts of climate change have again popularised the idea of water wars. The
argument runs that climate change will precipitate worsening ecological conditions contributing to resource scarcities, social breakdown, institutional failure, mass migrations
and in turn cause greater political instability and conflict ([Brauch, 2002] and [Pervis and Busby, 2004]). In a report for the US Department of Defense, Schwartz and Randall
(2003) speculate about the consequences of a worst-case climate change scenario arguing that water shortages will lead to aggressive wars (Schwartz and Randall, 2003, p. 15).
Despite growing concern that climate change will lead to instability and violent conflict, the
evidence base to substantiate the connections is thin ([Barnett and Adger, 2007] and [Kevane and Gray, 2008]).

No resource wars – prefer statistical evidence

Pinker 11 (Steven, Harvard College Professor and Johnstone Family Professor in the Department of Psychology – Harvard
University, “The Better Angels of Our Nature: Why Violence Has Declined,” Google Books)

climate change can cause plenty of misery and deserves to be


Once again it seems to me that the appropriate response is "maybe, but maybe not." Though

will not necessarily lead to armed conflict. The political scientists who track
mitigated for that reason alone, it

war and peace, such as Halvard Buhaug, Idean Salehyan, Ole Theisen, and Nils Gleditsch, are skeptical of the popular idea that
people fight wars over scarce resources. Hunger and resource shortages are tragically
common in sub-Saharn countries such as Malawi, Zambia, and Tanzania, but wars involving them are not. Hurricanes,
floods, droughts, and tsunamis (such as the disastrous one in the Indian Ocean in 2004) do not generally lead to
armed conflict. The American dust bowl in the 1930s, to take another example, caused plenty of deprivation
but no civil war. And while temperatures have been rising steadily in Africa during the past fifteen years, civil
wars and war deaths have been falling. Pressures on access to land and water can certainly cause
local skirmishes, but a genuine war requires that hostile forces be organized and armed , and
that depends more on the influence of bad governments, closed economies, and militant
ideologies than on the sheer availability of land and water . Certainly any connection to terrorism is in the imagination of
the terror warriors: terrorists tend to be underemployed lower-middle-class men, not subsistence farmers. As for genocide, the Sudanese government finds it convenient

In a regression
to blame violence in Darfur on desertification, distracting the world from its own role in tolerating or encouraging the ethnic cleansing.

analysis on armed conflicts from 1980 to 1992, Theisen found that conflict was more likely if a country was
poor, populous, politically unstable, and abundant in oil , but not if it had suffered from
droughts, water shortages, or mild land degradation. (Severe land degradation did have a small effect.) Reviewing
analyses that examined a large number (N) of countries rather than cherry-picking one or
two , he concluded, "those who foresee doom, because of the relationship between resource
scarcity and violent internal conflict, have very little support in the large-N literature ."
Salehyan adds that relatively inexpensive advances in water use and agriculture practices in
the developing world can yield massive increases in productivity with a constant or even
shrinking amount of land, and that better governance can mitigate the human costs of
environmental damage, as it does in developed democracies. Since the state of the
environment is at most one ingredient in a mixture that depends far more on political and
social organization, resource wars are far from inevitable, even in a climate-changed world.

Sharing solves—no root cause


Victor 07— professor of law at Stanford Law School and the director of the Program on Energy and Sustainable
Development. He is also a senior fellow at the Council on Foreign Relations (David, Nov 14, “What resource wars?”
http://www.atimes.com/atimes/Global_Economy/IK14Dj04.html)

such as malaria that could be harder to contain if tropical conditions are more prevalent, which in turn could stress health-
care systems and lead to hot wars.
While there are many reasons to fear global warming, the risk that such dangers could
cause violent conflict ranks extremely low on the list because it is highly unlikely to
materialize. Despite decades of warnings about water wars, what is striking is that water
wars don't happen - usually because countries that share water resources have a lot
more at stake and armed conflict rarely fixes the problem. Some analysts have pointed to conflicts
over resources, including water and valuable land, as a cause in the Rwandan genocide, for example. Recently, the UN
But none of these
secretary-general suggested that climate change was already exacerbating the conflicts in Sudan.
supposed causal chains stay linked under close scrutiny - the conflicts over resources are
usually symptomatic of deeper failures in governance and other primal forces for
conflicts, such as ethnic tensions, income inequalities and other unsettled grievances.
Climate is just one of many factors that contribute to tension. The same is true for scenarios of
climate refugees, where the moniker "climate" conveniently obscures the deeper causal forces.

Empirical data concludes Neg

Salehyan 7 (Idean, Assistant Professor of Political Science – University of North Texas, “The New Myth About Climate Change”,
Foreign Policy, August, http://www.foreignpolicy.com/story/cms.php?story_id=3922)

First, aside from a few anecdotes, there is little systematic empirical evidence that resource scarcity
and changing environmental conditions lead to conflict. In fact, several studies have shown that an
abundance of natural resources is more likely to contribute to conflict. Moreover, even as the
planet has warmed, the number of civil wars and insurgencies has decreased dramatically. Data collected by
researchers at Uppsala University and the International Peace Research Institute, Oslo shows a steep decline in the
number of armed conflicts around the world. Between 1989 and 2002, some 100 armed conflicts came to an end,
including the wars in Mozambique, Nicaragua, and Cambodia. If global warming causes conflict, we should not be witnessing
this downward trend. Furthermore, if famine and drought led to the crisis in Darfur, why have scores of environmental
catastrophes failed to set off armed conflict elsewhere? For instance, the U.N. World Food Programme warns that 5 million
people in Malawi have been experiencing chronic food shortages for several years. But famine-wracked Malawi has
yet to experience a major civil war. Similarly, the Asian tsunami in 2004 killed hundreds of thousands of people,
generated millions of environmental refugees, and led to severe shortages of shelter, food, clean water, and
electricity. Yet the tsunami, one of the most extreme catastrophes in recent history, did not lead to an outbreak of
resource wars. Clearly then, there is much more to armed conflict than resource scarcity and
natural disasters.

Market adjustments solve

National Post 8 (Canada – National Edition, “Don’t Panic”, 4-26, Lexis)

The trouble with doom-and-gloom predictions -- whether they be about oil shortages, food scarcity, water wars or
population explosions --is that most are based on the linear extrapolation of short-term trends. If, say,
rice prices rise, alarmists assume they will keep rising indefinitely at the same rate -- and then
produce scary-looking graphs that show trend lines veering up into the wild-eyed blue yonder. But history shows that
human adaptation invariably intervenes --especially in parts of the world that have the benefit of a market
economy. Scarcitydrives innovations that pull the world back from the brink. Consumers take
high prices as their cue to consume less; producers take the same cue to produce more. A
new equilibrium is reached, just as college microeconomics textbooks would predict. That's why we aren't losing any sleep over
the latest predictions from Canadian Imperial Bank of Commerce chief economist Jeffrey Rubin, which were fronted
prominently on Friday's National Post. New
inventions, new oil discoveries and improvements in
existing technologies will conspire to spare us Mr. Rubin's parade of horribles, which include $2.25-a-litre gasoline
and tens of thousands of job losses in the auto-making sector. In a report entitled The Age of Scarcity, released on Thursday,
Mr. Rubin predicts that by 2012, demand for oil, gas and diesel in the rest of the world will exceed that in OECD
countries. As developing
nations get richer, they will begin competing with the current
industrialized world for diminishing resources. This will drive up the cost of everything from energy to food to
computer components. Mr. Rubin predicts this will lead to the biggest economic disruption in North America since the 1973 oil
crisis. But that same historical comparison suggests a reason Canadians should be suspicious of this ominous forecast: While
the oil shortages of the 1970s displaced millions of assembly-line workers and led to a temporary slowdown of the North
American economy, the adaptations they spurred ultimately made industry more efficient and ordinary people more
prosperous. North American manufacturing is far more productive and energy-efficient now than it was 30 years ago, as well as
producing far less pollution. (Many Canadians under 30, who have been reared on a constant diet of dire environmental claims,
may have trouble believing this, but despite the rapid growth of our economy in the last three decades, smog is actually less
toxic and our waters less polluted than in 1970.) In an interview with the National Post, Mr. Rubin fell into a common trap: He
assumed growth is a zero-sum game, whereby someone must lose ground every time someone else gains it. "I think there will
be fewer people on the road in North America in five years than there is right now," Mr. Rubin said on Thursday. "For everybody
who's about to get on the road by buying a new Tata or a Chery car in the developing world, someone's going to have to get off
the road in this part of the world. There's just not enough gasoline to go around." Anyone tempted to buy into this line of
thinking would do well to remember the famous bet between Paul R. Ehrlich, author of the apocalyptic 1968 book The
Population Bomb, and economist Julian Simon. Mr. Erlich
predicted that by the late 19 70s , the world
would begin to run out of oil and metals, and that "wide-scale famine caused by declining
food production" would cause hundreds of millions of deaths annually . Mr. Simon, on the other
hand contended, that "natural resources are not finite in any serious way; they are created by the intellect of man, an always
renewable resource." In 1980, he bet Mr. Ehrlich $1,000 that by 1990 a basket of any five commodities of his choosing would
cost less than it had 10 years earlier. By the end-is-nigh thinking embraced by Mr. Ehrlich (and, to a lesser extent, Mr. Rubin), he
should have won easily. Instead, Mr. Simon won. The five commodities chosen were, after inflation, 40% cheaper in 1990 than
they had been a decade before. The same pattern is beginning to unfold in 2008. In just a few short months, rising prices for
fuel have prompted the sort of market-driven energy efficiencies and environmental solutions that the green movement has
failed to achieve through years of hectoring, regulating and legislating. Full-sized SUV sales have plummeted, home builders are
designing smaller, low-consumption houses, airlines and railways are switching to more efficient planes and engines and car
makers are scrambling to lighten their models. Thanks to just a 30% increase in pump prices, the
automobile sector is
likely to raise fleet fuel efficiency more than all the laws demanding higher standards
passed in the past 35 years combined. There is no doubt that our society is changing because of
the scarcity in food and fuel that Mr. Rubin highlights. But it defies the principles of economics to
imagine that such scarcity will persist indefinitely. If there is one trend we can depend on, it is that the law
of supply and demand will intervene to blunt the economic shocks that even the most prosperous nations must inevitably face.
---Resource Wars- Other Factors Check

1NC Tertrais—no resource wars—freeing up reserves and nonconventional sources


fill-in for scarcity—means abundances is just as big of a link—also economics checks—
trade will balance out inequalities and it’s cheaper to buy than to fight.

And even if the occur, resource wars won’t escalate—too costly

Victor ‘8 – (David G., Victor law professor, Stanford, director of the Program on Energy and Sustainable Development, senior
fellow, CFR, “Smoke and Mirrors, Debating Disaster: The World Is Not Enough,” http://www.nationalinterest.org/Article.aspx?
id=16522)

MY ARGUMENT is that classic resource wars—hot conflicts driven by a struggle to grab resources— are
increasingly rare. Even where resources play a role, they are rarely the root cause of bloodshed.
Rather, the root cause usually lies in various failures of governance. That argument—in both its classic form
and in its more nuanced incarnation—is hardly a straw man, as Thomas Homer-Dixon asserts. Setting aside hyperbole, the
punditry increasingly points to resources as a cause of war. And so do social scientists and policy analysts, even
with their more nuanced views. I’ve triggered this debate because conventional wisdom puts too much emphasis on resources as a
cause of conflict. Getting the story right has big implications for social scientists trying to unravel cause-and-effect and often even
larger implications for public policy. Mihael Klare is right to underscore Saddam
Hussein’s invasion of Kuwait, the only
classic resource conflict in recent memory. That episode highlights
two of the reasons why classic resource wars
are becoming rare—they’re expensive and rarely work. (And even in Kuwait’s case, many other forces also
spurred the invasion. Notably, Iraq felt insecure with its only access to the sea a narrow strip of land sandwiched between Kuwait on
one side and its archenemy Iran on the other.) In the end, Saddam lost resources on the order of $100 billion
(plus his country and then his head) in his quest for Kuwait’s 1.5 million barrels per day of combined oil and gas output. By contrast,
Exxon paid $80 billion to get Mobil’s 1.7 million barrels per day of oil and gas production—a merger that has held and flourished. As
the bulging sovereign wealth funds are discovering, it is easier to get resources through the stock exchange than the gun barrel.
Klare takes me to task for failing to acknowledge the role of “lootable” resources as a motive for war. My point is that looters
loot what they can—not just natural resources, but also foreign aid and anything else that
passes within reach. (Paul Collier’s research, which Klare cites for support, finds that a sizeable share of African military
budgets is, in effect, aid money that is looted and redirected from foreign aid.) I suspect that we don’t differ much in our assessment
of the effects of lootable resources within weak and failed states, but where we do part company is in the implication for policy.
Fixing the problems in the Niger River Delta—the case he uses—requires a stronger and more accountable government. That means
making it harder to loot resources, taming official corruption, lending a hand with law enforcement in places where oil is produced
and stolen, and engaging reformist forces in the Nigerian government. Resource looting and misallocation are severe, but they are
symptoms whose cures require focusing on governance. The
realities of global resource depletion are
somewhat different from Klare’s story. It is true that primary resources, such as oil in the
ground, are now more concentrated in “armpit” countries because more readily available
resources are being depleted. That fact , though, only serves to further support my conclusion: That
we must redouble our efforts to improve governance because all oil-consuming countries have a
stake in the good governance of their oil producers.

Globalization means powers would deescalate the conflict


Barnett 9 (Thomas, visiting scholar at U Tennessee's Howard Baker Center, The Daily Sentinel, "Threat of great power war
recedes" http://www.gjsentinel.com/opin/content/news/opinion/stories/2009/03/21/barnett_power_war.html?
cxtype=rss&cxsvc=7&cxcat=9)

Remember when Cold Warriors


Why do I so casually dismiss “resource wars” as a strategic planning principle?
predicted we’d fight the Soviets across the “arc of crisis” for precious resources? Well, back
then, both sides lived within miniature versions of today’s global economy. In that bifurcated
world economy, zero-sum resource wars were entirely plausible. That bifurcated world no
longer exists, as evidenced by the recent financial contagion. In globalization, demand
determines power more than supply. Don’t believe me? Imagine a world where there’s no Chinese demand for U.S.
debt or no U.S. demand for Chinese exports. Dreaming up future “resource wars” to obviate our military’s
necessary adjustment to this era’s security tasks will not render them moot. Indeed, like
Somalia’s recent pirate epidemic, they invariably attract the collaborative efforts of other great
powers, like China and India, which have no choice but to defend their growing economic
networks.

Trading goods solves

Allouche 10 Jeremy Allouche Research Fellow in water supply and sanitation. Institute of Development Studies, Brighton, U The
sustainability and resilience of global water and food systems: Political analysis of the interplay between security, resource scarcity,
political systems and global trade. Food Policy (2010), doi:10.1016/j.foodpol.2010.11.013

Debates on resource scarcity and conflict have ignored the role of trade in both causing and addressing local and
regional shortages. In the case of food and water, this has led to conclusions that are highly
questionable. Indeed, food security has essentially been addressed through national water
availability and ignores the spectacularly successful benefits of international trade, in this
particular case food imports (Allan, 2001). Water availability is often hidden in international trade. Countries with
more water are able to trade water-intensive goods for export. Water embedded in traded
crops has been termed ‘virtual water’ and trade in virtual water has been suggested as a way to
alleviate water shortages. However, the limit of this logic should be recognized in that global trade is based on broader
political and economic factors rather than on water. Through global trade, one can observe an overall increase
in terms of food security between 1970 and 1990. The greatest improvements were in North Africa and the
Middle East, moderate change in Asia and Oceania and Latin America, and a decline in Sub-Saharan Africa. A number of specialists
emphasize the need for free international trade in order to assure global food security, as it enables supply and demand to be
balanced across regions (Godfray et al., 2010). Global
trade therefore is seen as a solution to the ‘equality’
problem as it enables food security as defined by the FAO (namely when ‘‘all people, at all times, have physical
and economic access to sufficient, safe and nutritious food to meet their dietary needs and food preferences for an active and
healthy life’’ – as defined at the 1996 World Food Summit, FAO, 1996). Although it has been acknowledged that free markets usually
penalize the poorest who have the least influence on how global markets are structured and regulated (see Anderson, 2009 and
Aksoy and Beghin, 2005), alternatives have usually been dismissed.

Market adjustments solve

National Post 8 (Canada – National Edition, “Don’t Panic”, 4-26, Lexis)


The trouble with doom-and-gloom predictions -- whether they be about oil shortages, food scarcity, water wars or
population explosions --is that most are based on the linear extrapolation of short-term trends. If, say,
rice prices rise, alarmists assume they will keep rising indefinitely at the same rate -- and then
produce scary-looking graphs that show trend lines veering up into the wild-eyed blue yonder. But history shows that
human adaptation invariably intervenes --especially in parts of the world that have the benefit of a market
economy. Scarcity drives innovations that pull the world back from the brink. Consumers take
high prices as their cue to consume less; producers take the same cue to produce more. A
new equilibrium is reached, just as college microeconomics textbooks would predict. That's why we aren't losing any sleep over
the latest predictions from Canadian Imperial Bank of Commerce chief economist Jeffrey Rubin, which were fronted
prominently on Friday's National Post. New
inventions, new oil discoveries and improvements in
existing technologies will conspire to spare us Mr. Rubin's parade of horribles, which include $2.25-a-litre gasoline
and tens of thousands of job losses in the auto-making sector. In a report entitled The Age of Scarcity, released on Thursday,
Mr. Rubin predicts that by 2012, demand for oil, gas and diesel in the rest of the world will exceed that in OECD
countries. As developing
nations get richer, they will begin competing with the current
industrialized world for diminishing resources. This will drive up the cost of everything from energy to food to
computer components. Mr. Rubin predicts this will lead to the biggest economic disruption in North America since the 1973 oil
crisis. But that same historical comparison suggests a reason Canadians should be suspicious of this ominous forecast: While
the oil shortages of the 1970s displaced millions of assembly-line workers and led to a temporary slowdown of the North
American economy, the adaptations they spurred ultimately made industry more efficient and ordinary people more
prosperous. North American manufacturing is far more productive and energy-efficient now than it was 30 years ago, as well as
producing far less pollution. (Many Canadians under 30, who have been reared on a constant diet of dire environmental claims,
may have trouble believing this, but despite the rapid growth of our economy in the last three decades, smog is actually less
toxic and our waters less polluted than in 1970.) In an interview with the National Post, Mr. Rubin fell into a common trap: He
assumed growth is a zero-sum game, whereby someone must lose ground every time someone else gains it. "I think there will
be fewer people on the road in North America in five years than there is right now," Mr. Rubin said on Thursday. "For everybody
who's about to get on the road by buying a new Tata or a Chery car in the developing world, someone's going to have to get off
the road in this part of the world. There's just not enough gasoline to go around." Anyone tempted to buy into this line of
thinking would do well to remember the famous bet between Paul R. Ehrlich, author of the apocalyptic 1968 book The
Population Bomb, and economist Julian Simon. Mr. Erlich
predicted that by the late 19 70s , the world
would begin to run out of oil and metals, and that "wide-scale famine caused by declining
food production" would cause hundreds of millions of deaths annually . Mr. Simon, on the other
hand contended, that "natural resources are not finite in any serious way; they are created by the intellect of man, an always
renewable resource." In 1980, he bet Mr. Ehrlich $1,000 that by 1990 a basket of any five commodities of his choosing would
cost less than it had 10 years earlier. By the end-is-nigh thinking embraced by Mr. Ehrlich (and, to a lesser extent, Mr. Rubin), he
should have won easily. Instead, Mr. Simon won. The five commodities chosen were, after inflation, 40% cheaper in 1990 than
they had been a decade before. The same pattern is beginning to unfold in 2008. In just a few short months, rising prices for
fuel have prompted the sort of market-driven energy efficiencies and environmental solutions that the green movement has
failed to achieve through years of hectoring, regulating and legislating. Full-sized SUV sales have plummeted, home builders are
designing smaller, low-consumption houses, airlines and railways are switching to more efficient planes and engines and car
makers are scrambling to lighten their models. Thanks to just a 30% increase in pump prices, the
automobile sector is
likely to raise fleet fuel efficiency more than all the laws demanding higher standards
passed in the past 35 years combined. There is no doubt that our society is changing because of
the scarcity in food and fuel that Mr. Rubin highlights. But it defies the principles of economics to
imagine that such scarcity will persist indefinitely. If there is one trend we can depend on, it is that the law
of supply and demand will intervene to blunt the economic shocks that even the most prosperous nations must inevitably face.

No war—3 reasons
Trade, horizontal military modernization, new reserves
Deudney 99 – (Dan, Associate Professor of Political Science, Johns Hopkins, Contested Grounds: Security and Conflict in the
New Environmental Politics, Eds. Deudney & Matthews p 205-6)

The hypothesis that states will begin fighting each other as natural resources are depleted and
degraded seems intuitively accurate. The popular metaphor of a lifeboat adrift at sea with declining supplies of clean
water and rations suggests there will be fewer opportunities for positive-sum gains between actors as resource scarcity grows. Many
fears of resource war are derived from the cataclysmic world wars of the first half of the twentieth century Influenced by geopolitical
theories that emphasized the importance of land and resources for great power status, Adolf Hitler fashioned Nazi German war aims
to achieve resource autonomy. The aggression of Japan was directly related to resource goals: lacking indigenous fuel and minerals,
and faced with a slowly tightening embargo by the Western colonial pow ers in Asia, the Japanese invaded Southeast Asia for oil, tin,
and rub ber. Although the United States had a richer resource endowment than the Axis powers, fears of shortages and industrial
strangulation played a central role in the strategic thinking of American elites about world strategy. During the Cold War, the
presence of natural resources in the Third World helped turn this vast area into an arena for East-West conflict. Given this record,
the scenario of conflicts over resources playing a powerful role in shaping international order should be taken seriously.
However, there are three strong reasons for concluding that the familiar scenarios of resource
war are of diminishing plausibility for the foreseeable future. First, the robust character of the
world trade system means that states no longer experience resource dependency as a major
threat to their military security and political autonomy. During the 1930s, the collapse of the world trading
system drove states to pursue economic autarky, but the resource needs of contemporary states are routinely met without
territorial control of the resource source. As Ronnie Lipschutz has argued, this means that re source constraints are much less likely
to generate interstate violence than in the past. Second, the prospects for resource wars are diminished by
the growing difficulty that states face in obtaining resources through territorial conquest. Although
the invention of nuclear explosives has made it easy and cheap to annihilate humans and infrastructure in extensive areas, the
spread of conventional weaponry and national consciousness has made it very costly for an
invader, even one equipped with advanced technology, to subdue a resisting population, as France
discovered in Indochina and Algeria, the United States in Vietnam, and the Soviet Union in Afghanistan. At the lower levels of
violence capability that matter most for conquering and subduing territory; the great powers have lost effective military superiority
and are unlikely soon to regain it. Third,
nonrenewable resources are, contrary to intuitive logic,
becoming less economically scarce. There is strong evidence that the world is entering what H. E. Goeller and Alvin M.
Weinberg have labeled the “age of substitutability,” in which industrial technology is increasingly capable of fashioning ubiquitous
and plentiful earth materials such as iron, aluminum, silicon, and hydrocarbons into virtually everything needed by modem societies.
The most striking manifestation of this trend is that prices for virtually every raw material have
been stagnant or falling for the last two decades despite the continued growth in world
economic output. In contrast to the expectations widely held during the 1970s that resource scarcity would drive up
commodity prices to the benefit of Third World raw material suppliers, prices have fallen.

Resources are infinite

Simon 98 (Julian, Professor of Business Administration – University of Maryland and Advisory Board – Acton Institute, “The
Ultimate Resource II: People, Materials, and the Environment,
http://www.juliansimon.com/writings/Ultimate_Resource/TINTRO.txt)

There is an almost insuperable difficulty in the definition of available "copper," "oil," and so on, because there are many
different grades of each resource in places that vary in difficulty of extracting the resource, and because (as seen in Table 2-1)
the amounts at low concentrations (such as the quantities of metals on the sea bottom and in sea water) are extraordinarily
large in contrast to the quantities we usually have in mind (the "proven reserves"). What's more, we constantly create
new supplies of resources, in the sense of discovering them where they were thought not
to exist. (In the past, the U.S. Geological Survey and others thought that there was no oil in California or Texas. Often, new
supplies of a resource come from areas outside the accustomed boundaries of our system, as resources
from other continents came to Europe in past centuries and as resources may in the future be brought from the sea or from
other planets. New
supplies also arise when a resource is created from other materials, just as
grain is grown and nuclear fuel is "bred." (Here we must avoid getting hung up on the word "natural," as in
"natural resources.")
---Resource Wars- Cooperation

1NC Victor—cooperation instead of conflict—empirically proven—their causal chains


are based on shoddy speculation—peaceful negotiations are cheaper than fighting—if
scarcity is the root cause there is low risk of conflict

Multiple case studies prove at international and state level


Dalby ‘6 (Simon, Dept. Of Geography, Carleton University, "Security and environment linkages revisited" in Globalisation and
Environmental Challenges: Reconceptualising Security in the 21st Century, www.ntu.edu.sg/idss/publications/SSIS/SSIS001.pdf)

In parallel with the focus on human security as a necessity in the face of both natural and artificial forms of vulnerability, recent
literature has emphasised the opportunities that environmental management presents for political cooperation between states and
other political actors, on both largescale infrastructure projects as well as more traditional matters of wildlife and new concerns with
biodiversity preservation (Matthew/Halle/Switzer 2002). Simultaneously ,
the discussion on water wars, and in
particular the key finding the shared resources frequently stimulate cooperation rather than
conflict, shifted focus from conflict to the possibilities of environmental action as a mode of
peacemaking. Both at the international level in terms of environmental diplomacy and
institution building, there is considerable evidence of cooperative action on the part of many
states (Conca/Dabelko 2002). Case studies from many parts of the world suggest that cooperation and
diplomatic arrangements can facilitate peaceful responses to the environmental difficulties in
contrast to the pessimism of the 1990’s where the focus was on the potential for conflicts. One
recent example of the attempts to resolve difficulties in the case of Lake Victoria suggests a
dramatic alternative to the resource war scenarios . The need to curtail over-fishing in the lake and the
importance of remediation has encouraged cooperation; scarcities leading to conflict arguments have not been common in the
region, and they have not influenced policy prescriptions (Canter/Ndegwa 2002). Many
conflicts over the allocations of
water use rights continue around the world but most of them are within states and international
disputes simply do not have a history of leading to wars.

Inevitably deescalate and lead to cooperation because of economics

Bennett and Nordstrom, 2K – department of political science at Penn State


(D Scott and Timothy, The Journal of Conflict Resolution, 44:1, “Foreign policy substitutability and internal economic problems in
enduring rivalries”, ProQuest, WEA)

Conflict settlement is also a distinct route to dealing with internal problems that leaders in rivalries may pursue when faced with
internal problems. Military
competition between states requires large amounts of resources, and rivals require
even more attention. Leaders may choose to negotiate a settlement that ends a rivalry to free up
important resources that may be reallocated to the domestic economy . In a "guns versus butter" world of
economic trade-offs, when a state can no longer afford to pay the expenses associated with
competition in a rivalry, it is quite rational for leaders to reduce costs by ending a rivalry. This gain (a peace
dividend) could be achieved at any time by ending a rivalry. However, such a gain is likely to be most important and attractive to
leaders when internal conditions are bad and the leader is seeking ways to alleviate active problems. Support
for policy
change away from continued rivalry is more likely to develop when the economic situation sours
and elites and masses are looking for ways to improve a worsening situation. It is at these times that the
pressure to cut military investment will be greatest and that state leaders will be forced to recognize the difficulty of continuing to
pay for a rivalry. Among other things, this
argument also encompasses the view that the cold war ended
because the Union of Soviet Socialist Republics could no longer compete economically with the
U nited S tates.
---Resource Wars- Best Data

1NC Salehyan—prefer our evidence—all the best empirical and statistical data goes
our way—several case studies prove point—best research shows other root causes—
their evidence isn’t based in statistical fact

Prefer data on question of resource wars—best way to evaluate

Sharp 7 (Travis, Military Policy Analyst – Center for Arms Control and Non-Proliferation, “Resource Conflict in the Twenty-First
Century”, Peace Review, 19(3), http://www.armscontrolcenter.org/policy/securityspending/articles/
resource_conflict_twenty_first_century/)

Brito and Intriligator's results have been supported more recently by the World Bank's Collier-Hoeffler (CH) model of civil
war onset. The CH model maintains that the opportunities to organize and finance a war are more significant variables than any
social or political grievances per se. Under this rubric, the CH model
predicts that the chance a nation with
limited resources will have a civil war in any five-year span is 1 in 100, but the chance that a
resource rich nation will is 1 in 5, according to the March 2006 Harper's Index. Although mathematically-
derived quantitative theories provide a rigorous and concrete demonstration of the
causal relationship between resources and conflict, the historical record should verify any theory of war. I
want to now use a specific case study to illustrate the historical link between natural resources and violence.
Russia Relations Answers
Frontline

Cooperation is impossible

Bovt ’12 (9/12 (Columnist-Moscow Times, “Whether Obama or Romney, the Reset Is Dead,”
http://www.themoscowtimes.com/opinion/article/whether-obama-or-romney-the-reset-is-dead/467947.html#ixzz274U7VOyl

During every U.S. presidential election campaign, there is a debate in Russia over whether the
Republican or Democratic candidate would be more beneficial for the Kremlin. Russian analysts and
politicians always fail to understand that Americans have shown little interest in foreign policy
since the end of the Cold War. Even when foreign policy is mentioned in the campaign, Russia is far
down the list as a priority item. The volume of U.S-Russian trade remains small. The recent Exxon-Rosneft deal
notwithstanding, U.S. interest in Russia's energy projects has fallen, particularly as the Kremlin has increased its role in this sector. To
make matters worse, the United States is determined to establish clean energy and energy independence, while Russia's gas exports
are feeling the pinch from stiff competition with the U.S. development of shale gas production. Of course, traditional areas of
cooperation remain: the transit of shipments to and from Afghanistan through Russia, Iran's nuclear program and the struggle
against international terrorism. But the transit route into Afghanistan cannot, by itself, greatly influence bilateral relations as a
whole, andprogress on the other two points seems to have reached a plateau beyond which little potential
remains for bringing the two countries into closer cooperation. On the positive side, a new visa
agreement came into force this week that will facilitate greater contact between both countries'
citizens. But it will be years before that significantly influences overall U.S.-Russian relations. A
new agreement regarding child adoptions has also been implemented after a few disturbing adoption stories prompted Russia's
media, with the help of government propaganda, to spoil the U.S. image in Russia. Meanwhile, both U.S. President Barack
Obama and Republican candidate Mitt Romney support the U.S. missile defense program in principle ,
although the exact form and scope of its deployment differ among the candidates. Even though President Vladimir Putin, during his
interview with RT state television last week, expressed guarded optimism over the prospect of reaching an agreement on missile
defense with Obama, Russia
seems to underestimate the degree to which Americans are fixated on
missile defense as a central component of their national security. It is highly unlikely that any
U.S. administration — Democratic or Republican — will ever agree to major concessions on
missile defense . It even seemed that Kremlin propagandists were happy when in March Romney
called Russia the United States' No. 1 foe . They were given another present when Obama, addressing the Democratic
National Convention last week, said Romney's comment only proved that he lacked foreign policy experience and was locked in Cold
War thinking.
For the next two months, however, the two candidates are unlikely to devote muc h
attention to Russia. Russia's internal politics will also be one of the key factors shaping future
U.S.-Russian relations. The two-year jail sentence slapped on three members of Pussy Riot for their anti-Putin prayer in
Moscow's main cathedral has already become a subject of discussion between Foreign Minister Sergei Lavrov and U.S. Secretary of
State Hillary Clinton. Even the most pragmatic "pro-reset" U.S. administration would criticize to one
degree or another Russia's poor record on human rights. It appears that Russia is moving
increasingly toward confrontation rather than rapprochement with the West. The Kremlin now
seems fully committed to spreading the myth that the U.S. State Department is the cause
behind most of Russia's domestic problems and is bent on undermining its national security by deploying missile
defense installations in Europe and by supporting the opposition. There are other disturbing signals as well. Take, for example, the
United Russia bill that would prohibit Russian officials from owning bank accounts and property
overseas, with particular attention paid to their holdings in the West. The ideological
underpinning of this bill is that assets located in the West are tantamount to betrayal of the
motherland. Then there is Russia's opposition to the U.S. Magnitsky Act. The Kremlin interprets this
initiative as yet another confirmation of its suspicions that Washington is conspiring against it and that the bill's real U.S. motive is to
blackmail Russian officials by threatening to freeze their overseas bank accounts and property. An increase in these anti-
Western attitudes does not bode well for U.S.-Russian relations, even if Obama is re-elected in November.
Regardless of which candidate wins, the reset is bound to either slowly die a natural death under Obama
or be extinguished outright under Romney. As a result, the most we can likely expect from U.S.-
Russian relations in the next four years is cooperation on a limited range of mundane issues .
Under these conditions, avoiding excessive anti-Russian or anti-U.S. rhetoric from both sides would itself be considered a major
achievement in bilateral relations.

Cooperation is impossible- alt causes and Putin

LaFranchi, 12 [Christian Science Monitor, “A cold-war chill US-Russia relations falter over Libya and Syria”,
http://www.csmonitor.com/USA/Foreign-Policy/2012/0303/A-cold-war-chill-US-Russia-relations-falter-over-Libya-and-Syria/
%28page%29/2]

Secretary of State Hillary Rodham Clinton doffed her diplomatic gloves after Russia vetoed a U nited N ations
Security Council resolution on Syria. Calling the February veto " despicable ," she laid at Moscow's feet the
"murders" of Syrian "women, children, [and] brave young men." Not to be outdone, Russian Prime Minister Vladimir Putin
railed against the U nited S tates for indulging its " bellicose itch " to get involved in other countries' internal
affairs. And he vowed that Russia will thwart American designs in the Middle East. Whatever happened to the
"reset," President Obama's ballyhooed reorientation of US-Russia relations to a more cooperative path focused on common
interests? Russia would say Libya happened – the conflict where the West and the US in particular demonstrated a zeal for
intervention that struck at Russia's sense of sovereignty and of what the UN should and shouldn't do. The US would say Syria
happened – revealing Russia's revived obstructionist tendencies on the Security Council and demonstrating Russia's determination
to protect an old ally at the expense of the Syrian people. Both countries might say that what happened is this: The common
interests that the "reset" was meant to emphasize – arms control, counterterrorism, the global
economy – have taken a back seat to awakened geopolitical rivalries and diverging international
visions. Add to this the fact that Mr. Putin is expected to return to Russia's presidency in elections Sunday,
bringing with him a blame-the-west perspective for explaining many of Russia's ills. The result is that stormy
days lie ahead for US-Russia relations, many say. Progress on issues like missile defense and NATO-
Russia relations is likely to remain stalled – and could suffer serious setbacks if the Syria and Iran
crises deteriorate further. "I foresee a tough year for US-Russia relations," says Andrew Weiss, a former
director for Russian affairs on the National Security Council under President Clinton who is now a Russia
analyst at the RAND Corp. in Arlington, Va. With little prospect for advances, he adds, the Obama administration is likely to focus on
preventing backsliding. "The emphasis will be on ensuring that these fast-moving conflicts don't put the remaining areas of
cooperation at risk," he says. Others say the current frictions demonstrate how relations, despite the efforts of three
administrations, have never overcome cold-war mistrusts to progress to a deeper level. "Under both
Clinton and Bush, the US made it look like things were moving forward with Russia by focusing
on things that were easier to do and that didn't require sacrifice from either side ," says Paul Saunders,
executive director of the Center for the National Interest in Washington. Three years ago this month, President
Obama said he hoped to promote more cooperation between the U.S. and Russia. It would be
hard to see how that may happen as Vladimir Putin approaches power once again. Host Scott
Simon speaks with the U.S. ambassador to Russia, Michael McFaul, about Sunday's elections in
Russia.
No impact to relations

Weitz ‘11 (Richard, senior fellow at the Hudson Institute and a World Politics Review senior editor 9/27/2011, “Global Insights:
Putin not a Game-Changer for U.S.-Russia Ties,” http://www.scribd.com/doc/66579517/Global-Insights-Putin-not-a-Game-Changer-
for-U-S-Russia-Ties )

Fifth, there will inevitably be areas of conflict between Russia and the United States regardless of who is in
the Kremlin. Putin and his entourage can never be happy with having NATO be Europe's most powerful security institution, since
Moscow is not a member and cannot become one. Similarly, the Russians will always object to NATO's missile
defense efforts since they can neither match them nor join them in any meaningful way. In the case of Iran, Russian officials
genuinely perceive less of a threat from Tehran than do most Americans, and Russia has more to lose from a cessation of economic
ties with Iran -- as well as from an Iranian-Western reconciliation. On the other hand, these
conflicts can be managed,
since they will likely remain limited and compartmentalized . Russia and the West do not have
fundamentally conflicting vital interests of the kind countries would go to war over. And as the
Cold War demonstrated, nuclear weapons are a great pacifier under such conditions. Another novel
development is that Russia is much more integrated into the international economy and global
society than the Soviet Union was, and Putin's popularity depends heavily on his economic
track record. Beyond that, there are objective criteria, such as the smaller size of the Russian population and
economy as well as the difficulty of controlling modern means of social communication, that
will constrain whoever is in charge of Russia.

Putin makes cooperation iumpossible

Weiss ’12 (6-19 – Founder and Chief Executive Officer of Weiss Asset Management, a Boston-based investment firm,[2] and
Professor Emeritus Boston University (Andrew, 2012, “Putin's Waiting Game”
http://www.foreignpolicy.com/articles/2012/06/19/waiting_game?page=full)

The most important yet overlooked aspect of the current situation, however, may be the cynicism and
casual indifference that Putin has displayed toward the U.S.-Russian relationship in the face of his
much bigger problems at home. At the moment, Putin appears to be preoccupied by the political
mess created by his decision to switch jobs with Medvedev and the badly flawed Duma elections last
December. He also must contend with the ripple effects of the eurozone drama and global
economic slowdown, which together have contributed to a 20 percent decline in global oil prices over the past two months
alone. Against this backdrop, the ups and downs of relations with Washington may be little more
than a distraction from the more urgent challenge of restoring the aura of invulnerability and
bezal'ternativnost' (the lack of any alternative) that bolstered Putin's authority during his first 12 years in
power. Already, he seems to have fallen back on the tried-and-true formula of portraying himself as
the protector of a Fortress Russia beset by imaginary foreign enemies and spies.   This gambit has long
helped the Kremlin cultivate support from average citizens and build up the regime's legitimacy. The chief beneficiaries of Putin's
rule -- the increasingly affluent and middle-class residents of places like Moscow -- show no signs of muffling their anger about his
return to the Kremlin despite an ongoing crackdown on political dissent. Still, Putin
knows how to cater to the two-
thirds of the Russian electorate that voted for him in March and reside primarily in Russia's
smaller cities and countryside. He may find it hard to resist the temptation to play upon their
worst fears and anti-Western stereotypes. Sacrificing the past several years of dramatic
improvement in the U.S.-Russian relationship may seem like a small price to pay if it breathes
new life and legitimacy into his rule.

No risk of total collapse—relations cyclical

Fenenko 11 (6/21/11, Alexei, leading researcher at the Russian Academy of Sciences' Institute for International Security, “The
Cyclical Nature of Russian-American Relations,” http://en.rian.ru/valdai_op/20110621/164739508.html)

There is nothing special or unusual about the current difficulties. Over


the past twenty years, both Russia and the
United States have experienced several cycles of convergence and divergence in their bilateral
relations. It seems that Moscow and Washington are doomed to repeat these cycles time and
again. Such changes in bilateral relations are no mere coincidence. Russia and the United States base their
relations on mutual nuclear deterrence. The material and technical foundations for Russian-American relations differ little from
those underpinning the Soviet-American relations of the 1980s. Thus, these cycles of Russian-American rapprochement are due to
two factors. First
comes the desire to consistently reduce aging nuclear systems so that during
disarmament neither party risked destroying the military-strategic parity. Second, the reaction
to a major military-political crisis after which the parties seek to reduce confrontation and
update the rules of conduct in the military-political sphere . After confronting these tasks, Russia and the
United States returned to a state of low intensity confrontation. The first rapprochement cycle was observed in the early 1990s.
Yeltsin’s government needed U.S. support in recognizing Russia within the 1991 borders of the RSFSR. Boris Yeltsin also needed U.S.
assistance in addressing the problem of the Soviet “nuclear legacy” and taking on the Supreme Council. The administrations of
George Bush Senior and Bill Clinton were willing to help the Kremlin solve these problems. However, the Americans demanded
major strategic concessions from Russia in return, outlined in START-III: making the elimination of heavy intercontinental ballistic
missiles a priority. The parties reached an unofficial compromise: U.S. recognition of the Russian leadership in exchange for the rapid
decrease in Russia’s strategic nuclear forces (SNF). However, the stronger Russian state institutions became, the weaker the impetus
to the rapprochement. In autumn 1994, Russia refused to ratify the original version of START-II and declared NATO’s eastward
expansion unacceptable. The United States adopted the concept of “mutually assured safety” (January
1995) under which Russia’s democratic reforms qualified as inseparable from continued
armament reduction. The “Overview of U.S. nuclear policy” in 1994 also confirmed that America
deemed Russian strategic nuclear forces a priority threat. The crises that unfolded during the late 1990s in Iran
and Yugoslavia were, like NATO expansion, the logical results of a restoration of the old approach to Soviet-American relations. It
was actually the events of 1994, not 2000, that in fact predetermined the subsequent
development of Russian-American relations. The second cycle of Russian-American rapprochement was also rooted
in strategic considerations. In 2000 START-II and the ABM Treaty collapsed. Both Washington and Moscow were faced with the
problem of their agreed decommissioning of nuclear systems dating back to the 1970s. These events pushed presidents Vladimir
Putin and George W. Bush to reach a strategic compromise at a meeting in Crawford (12 November 2001). The United States agreed
to sign a new Strategic Offensive Reductions Treaty (SORT), and Russia did not object to Washington’s withdrawal from the ABM
Treaty. Instead of the ABM Treaty, the parties signed the Moscow Declaration on May 24, 2002, under which the United States
pledged to consult with Russia on all issues pertaining to missile defense deployment. However, after the “compromise at
Crawford,” the agenda for Russian-American rapprochement was exhausted. The
disputes between Moscow and
Washington over Iraq, Iran, Georgia, Ukraine and Beslan, which had been gathering steam since
2003, necessitated a return to the traditional format for Russian-American relations. At the Bratislava
meeting (February 24, 2005) President Vladimir Putin refused to accept George W. Bush’s suggestion of including issues of fissile
material safety in the agenda.
Since then, the “rapprochement” between Russia and the U.S. has
reached a dead end, including at the official level.
---Russia Relations- Alt Causes

1NC LaFranchi—tons of alt causes—Syria, US interventionism, missile defense, NATO, and


lingering Cold War mistrust—no way aff can overcome all of them—

and their link is nonunique—Obama is promoting cooperation now but these factors
overwhelm

Lots of alt causes but they remain compartmentalized—no escalation


Weitz ‘11 (Richard, senior fellow at the Hudson Institute and a World Politics Review senior editor 9/27/2011, “Global Insights:
Putin not a Game-Changer for U.S.-Russia Ties,” http://www.scribd.com/doc/66579517/Global-Insights-Putin-not-a-Game-Changer-
for-U-S-Russia-Ties)

Fifth, there will inevitably be areas of conflict between Russia and the United States regardless of who is in the
Kremlin. Putin and his entourage can never be happy with having NATO be Europe's most powerful security institution, since
Moscow is not a member and cannot become one. Similarly, the Russians will always object to NATO's missile
defense efforts since they can neither match them nor join them in any meaningful way. In the case of Iran, Russian officials
genuinely perceive less of a threat from Tehran than do most Americans, and Russia has more to lose from a cessation of economic
ties with Iran -- as well as from an Iranian-Western reconciliation. On the other hand, these conflicts
can be managed,
since they will likely remain limited and compartmentalized. Russia and the West do not have
fundamentally conflicting vital interests of the kind countries would go to war over. And as the Cold
War demonstrated, nuclear weapons are a great pacifier under such conditions. Another novel development is that
Russia is much more integrated into the international economy and global society than the Soviet Union was,
and Putin's popularity depends heavily on his economic track record. Beyond that, there are objective criteria, such as the smaller
size of the Russian population and economy as well as the difficulty
of controlling modern means of social
communication, that will constrain whoever is in charge of Russia.

Means the US can’t broaden cooperation

Fly ‘10 – executive director of the Foreign Policy Initiative (Jamie M., 06/25, “President Obama’s Failed ‘Reset’ with Russia,”
http://www.nationalreview.com/corner/232469/president-obamas-failed-reset-russia-jamie-m-fly)

This is a concept that the Obama administration has shown itself unable or unwilling to grasp as it
has rushed to grant every possible concession to Moscow in an effort to obtain a new arms-control agreement. Despite the visit
to a burger joint, talk about Russian economic modernization, and supposed civil-society cooperation, President Obama’s

relationship with President Medvedev has been defined by one thing : arms control. If Russia
were truly ready for a “reset,” President Obama would be able to express concerns about political
repression, the rule of law, and Russia’s policies towards its neighbors without risking the collapse of the
relationship. President Obama has shown no willingness to broaden his engagement with Moscow
to include such issues. Unlike President Reagan’s engagement with Moscow on arms control, which was coupled with criticism of the
Soviet Union’s repression of its citizens, this administration has stood by while the situation in Russia deteriorates.

Laundry list of alt causes

A- NATO expansion

Cohen, 11– PhD in Russian studies from Columbia University, former professor of politics and Russian studies at Princeton
University, current professor at New York University, former advisor to President Bush, CBS News consultant, member of the Council
on Foreign Relations (Stephen, “Obama's Russia 'Reset': Another Lost Opportunity?” June 10, 2011, America-Russia.net,
http://www.america-russia.net/eng/face/278185917)

Not surprisingly, the Russian leadership entered into the reset in 2009 with expectations diametrically opposed to the unilateral
concessions expected by the Obama administration. As an unnamed Kremlin aide bluntly told a Washington Post columnist,
'America owes Russia, and it owes a lot, and it has to pay its debt.' A year later, when the head of NATO assured the international
media that the reset would 'bury the ghosts of the past,' it was another example of how little the US-led alliance understands or
cares about history. The 'ghost' barring a truly fundamental change in relations is, of course, the twelve-year expansion of NATO to
Russia's borders-the first and most fateful broken American promise. Despite
assurances of a 'NATO-Russian
friendship,' the Obama administration has not disavowed more NATO expansion and instead
reaffirmed US support for eventual membership for the former Soviet republics of Ukraine and Georgia,
Moscow's declared 'red lines.' No state that feels encircled and threatened by an encroaching military alliance-an anxiety
repeatedly expressed by Moscow, most recently by Putin in April-will, of course, ever feel itself an equal or secure partner of that
alliance. Still more, expanding NATO eastward has institutionalized a new and even larger geopolitical
conflict with Russia. Moscow's protests and countersteps against NATO encroachment, especially Medvedev's statement in
2008 that Russia is entitled to a 'sphere of strategic interests' in the former Soviet republics, have been indignantly denounced by
American officials and commentators as 'Russia's determination to re-establish a sphere of influence in neighboring countries.' Thus,
Biden stated in Moscow in March, 'We will not recognize any state having a sphere of influence.' But what
is NATO's
eastward movement other than a vast expansion of America's sphere of influence -military,
political and economic-into what had previously been Russia's? No US official or mainstream commentator
will admit as much, but Saakashvili, the Georgian leader bent on joining the alliance, feels no such constraint. In 2010, he welcomed
the growth of 'NATO's presence in the region' because it enables the United States and its allies to 'expand their sphere of influence.'
Of all the several double standards in US policy-making-"hypocrisy," Moscow charges- none has done
more to prevent an American-Russian partnership and to provoke a new cold war. * * * Given that
the new NATO states cannot now be deprived of membership, there is only one way to resolve, or at least reduce, this profound
geopolitical conflict between the US and Russia: in
return for Moscow's reaffirmation of the sovereignty of all
the former Soviet republics, Washington and its allies should honor retroactively another broken
promise-that Western military forces would not be based in any new NATO country east of
Germany. Though anathema to the US policy establishment and weapons industries, this would, in effect, demilitarize NATO's
expansion since 1999. Without diminishing the alliance's guarantee of collective security for all of its members, such a grand
accommodation would make possible a real partnership with post-Soviet Russia. First, and crucially, it would redeem one of
America's broken promises to Russia. Second, it would recognize that Moscow is entitled to at least one "strategic interest'-the
absence of a potential military threat on its borders. (Washington has long claimed this privilege for itself, defending it to the brink
of nuclear war in Cuba in 1962.) Third, the
demilitarization of NATO's expansion would alleviate Russia's
historical fear of military encirclement while bolstering its trust in Western partners . And fourth, this
would reduce the Kremlin's concerns about missile defense sites in Eastern Europe, making it more willing to contribute what may
be Russia's necessary resources to the still unproven project. Much else of essential importance both to America
and Russia could then follow, from far greater reductions in all of their weapons of mass
destruction to full cooperation against the looming dangers of  nuclear proliferation and
international terrorism. The result would be, that is, another chance to regain the historic opportunity lost in the 1990s.

B- Libya

Cohen, 11– PhD in Russian studies from Columbia University, former professor of politics and Russian studies at Princeton
University, current professor at New York University, former advisor to President Bush, CBS News consultant, member of the Council
on Foreign Relations (Stephen, “Obama's Russia 'Reset': Another Lost Opportunity?” June 10, 2011, America-Russia.net,
http://www.america-russia.net/eng/face/278185917)

Obama's recapitulations of failed American policies, along with his declared intention to pursue missile defense in Eastern Europe-
plans to put interceptor missiles in Romania and related weapons in Poland have already been announced-can only severely limit his
détente with Moscow, and possibly destroy it. Given
Russia's overriding importance for vital US interests, the
president seems to have no national security priorities. Even the wanton NATO air attacks
on Libya are eroding support for the reset in Moscow, where lessons are being drawn that
'Russia was essentially deceived' (again) and Obama's partner Medvedev was 'naïve' in  trusting
the US-backed UN resolution on a 'no-fly zone'; that nations without formidable nuclear
weapons-first Serbia, then Iraq and now Libya (Muammar el-Qaddafi relinquished his nuclear materials in 2004)
risk becoming targets of such attacks; and that NATO's slouching toward Russia is  even more
menacing than previously thought.

C- Missile defense

Cohen, 11 Stephen F. – PhD in Russian studies from Columbia University, former professor of politics and Russian studies at
Princeton University, current professor at New York University, former advisor to President Bush, CBS News consultant, member of
the Council on Foreign Relations, “Obama's Russia 'Reset': Another Lost Opportunity?”, June 10, 2011, America-Russia.net,
http://www.america-russia.net/eng/face/278185917)

In pursuing the one-way concessions implicit in 'selective cooperation,' Obama, like Clinton and Bush before him, seems
unable or unwilling to connect the strategic dots of mutual security the way Reagan and
Gorbachev did in the late 1980s. In effect, Obama is asking Moscow to substantially reduce its long-range
nuclear weapons while Russia is being surrounded by NATO bases with their superior conventional forces
and with an antimissile system potentially capable of neutralizing Russia's reduced retaliatory capability. In that crucial respect, the
new arms-reduction treaty is inherently unstable. If nothing else, Obama is undermining his own
hope of also negotiating a major reduction of Russia's enormous advantage in short-range tactical nuclear
weapons, which Moscow increasingly considers vital for its national defense. Instead, as Medvedev also warned, unless the
missile defense conflict is resolved, there will be 'another escalation of the arms race' that would, he
added on May 18, 'throw us back into the cold war era.'

Russia backsliding on multiple issues now


Sestanovich ‘7 (Stephen, George F. Kennan Senior Fellow for Russian and Eurasian Studies at the Council on Foreign Relations,
May 17th, “Russian-American Relations: Problems and Prospects,”
http://www.cfr.org/publication/13354/russianamerican_relations.html

Russian-American frictions—both specific disagreements


At the same time, it is essential to recognize that
and a more general tension—are also growing. It was only last week, after all, that President Putin, implicitly
but unmistakably, compared the United States to the Third Reich . (Please pay no attention to the pro forma
denials: Mr. Putin clearly wanted to make the comparison and to be able to deny that he had done so.) And it was only a week
earlier that an angry and sometimes violent mob in Moscow was allowed to mount a multi-day
siege of the embassy of Estonia, a treaty ally of the United States, while the police stood idly
by. Unfortunately, the negative developments of this month do not stand alone. In April, President Putin announced
that Russia would suspend its observance of the treaty on conventional forces in Europe,
negotiated in 1990, and revised in 1999, under American leadership. He also continued a campaign
of—to my mind, spurious—charges that, in planning the thinnest imaginable shield to protect Europe against a future
Iranian missile capability, the U.S. is threatening Russia n security. Other Russian officials have
suggested that they may want to pull out of the treaty on intermediate-range nuclear
missiles signed in 1987 by Ronald Reagan and Mikhail Gorbachev.

Democratization, security issues, energy, terror, G-summits, and corruption

Edwards, Kemp, and Sestanovich ‘6 (John, Senator, Jack, Former Republican Nominee for VP, and Stephen, George F.
Kennan Senior Fellow for Russian and Eurasian Studies at the CFR, March, “Task Report: Russia’s Wrong Direction – What the United
States Can and Should Do,” http://www.cfr.org/publication/9997/)

The Task Force is comprised of many of the nation’s preeminent Russia scholars and policy practitioners. It applauds recent
Russian support for containing Iran’s nuclear program and cooperative initiatives to secure nuclear materials, but cautions that
“U.S.-Russia relations are now marked by a growing number of disagreements. The
partnership is not living up to its potential.” The areas of most concern include: * De-
democratization: The report finds that Russian political institutions are becoming “corrupt and brittle.” As a result,
“Russia’s capacity to address security concerns of fundamental importance to the United States and its allies is
reduced. And many kinds of cooperation—from securing nuclear materials to intelligence sharing—are
undermined.” * Energy supplies: “Russia has used energy exports as a foreign policy weapon :
intervening in Ukraine’s politics, putting pressure on its foreign policy choices, and curtailing supplies to the rest of Europe. The
reassertion of government control over the Russian energy sector increases the risk this weapon will be used again.” * The
war on terror: The Task Force finds “a seeming Russian effort to curtail U.S. and NATO military access to Central Asian
bases,” a sign that Russia is retreating from the idea that “success in Afghanistan serves a common interest.” * Russia
hosting the G8: “A country that has in the space of a single year supported massive fraud in
the elections of its largest European neighbor and then punished it for voting wrong by
turning off its gas supply has to be at least on informal probation at a meeting of the world’s industrial
democracies.”
---Russia Relations- Doesn’t Solve

1NC Weiss—no legitimate cooperation—Putin too concerned with economic and political
stability—puts the relationship on the back burner and uses anti-American policy to galvanize
support—any concession will be pocketed

Russia views it as zero-sum game—no cooperation

Cohen 12 – Senior Research Fellow in Russian and Eurasian Studies and International Energy Policy in the Douglas and Sarah
Allison Center for Foreign Policy Studies (Ariel, 03/15, “How the U.S. Should Respond to Russia's Unhelpful Role in the Middle East,”
http://www.heritage.org/research/reports/2012/03/how-the-us-should-respond-to-russias-unhelpful-role-in-the-middle-east)

The anti-American tilt of Russian foreign policy prevents diplomatic cooperation because the U.S.
and Russia lack a shared threat assessment and mutual understanding in dealing with the changing dynamics of the Middle East.
Despite clear statements to the contrary by Prime Minister Putin and Foreign Minister Lavrov, the Obama Administration has
repeatedly declared that the U.S. is not competing with Russia for regional influence. Regrettably, the Kremlin has not received this
memo. Instead, Russian attempts to constrain U.S. policy have provoked little or no response from Washington. Lavrov habitually
invokes a “polycentric” or multipolar model of the world, with Russia working with her partners toward a
future in which U.S. power is so diminished that it cannot act without Moscow’s permission .
Russia’s vision of the Middle East is a case in point.[68] Moscow’s concept of multipolarity entails not just an uncontested Russian
sphere of influence in the Commonwealth of Independent States, but also together with Iran wielding much greater clout in the
Middle East. Moscow clearly wants to retain ties with Iran, which it regards as the rising great power in the Gulf and Middle East.
However, the Obama Administration has been deluding itself that Russia would be a genuine partner in restraining Iran.
Notwithstanding Washington’s and Riyadh’s irritation, Russia defends the Assad regime despite its bloody repression of its own
citizens. Even though the regime is teetering on collapse, Russia has signed an agreement with Syria to refurbish Soviet naval bases
in Latakiyah and Tartus and has increased sales of sophisticated weapons. Thus, Russia is obstructing U.N. resolutions censuring
Syria, while allowing its relationship with the Obama Administration to wilt.[69] Moscow’s suspicions of the U.S. and
the prevailing anti-American mindset lead it to persist in playing a zero-sum game in the
Middle East and elsewhere. The intense competition, in turn, tends to work to the advantage of third countries, such as
Iran and China, and of terrorist groups, such as Hamas and Hezbollah.[70] For instance, although Iran and nonstate or state-
sponsored Islamist radicals present long-term dangers to both states, Russia tends to ignore the Iranian threat. U.S. interests lie in a
more democratic and pro-Western environment that fosters civil society and economic opportunity. However, the Obama
Administration’s myopic laissez-faire attitude toward Islamists seems to have moved this goal further away than before the Arab
upheavals erupted.[71] International energy companies also need security for capital-intensive energy projects, which often require
investments of the tens of billions of dollars. Russia’s
zero-sum policy is preventing Washington and Moscow
from identifying and exploring areas in which U.S. and Russian interests in the region converge, such
as anti-terrorism and disrupting funding of globally active radical Islamists. The areas in which the two states are pursuing diverging
foreign policy goals, such as Russia’s trade in arms and nuclear reactors, will require special attention and, where necessary,
consistent pushback. Russia’s interests in the region—including energy and weapons trade, supporting a nuclear Iran, and
attempting to selectively legitimize anti-Israel radical Islamist organizations while fighting similar ones at home—contradict U.S.
interests. In addition, Russia is pursuing a diplomatic strategy of developing an ad hoc Sino–Russian axis to undermine U.S. priorities
around the world, particularly in the Middle East.

The U.S. goes all-in on Medvedev, but he holds no political power


Cohen 11 – Senior Research Fellow in Russian and Eurasian Studies and International Energy Policy in the Douglas and Sarah
Allison Center for Foreign Policy Studies (Ariel, 06/15, “Reset Regret: U.S. Should Rethink Relations with Russian Leaders,”
http://www.heritage.org/research/reports/2011/06/reset-regret-us-should-rethink-relations-with-russian-leaders)

For the past two years, the Obama Administration has touted its Russia “reset policy” as one of its great diplomatic achievements.
The President spent an inordinate amount of time cultivating Russian President Dmitry Medvedev and making
him his principal diplomatic interlocutor— despite the fact that Medvedev is Prime Minister Vladimir
Putin’s appointed protégé with no political base of his own. To uphold the “reset,” the Administration agreed
to cut U.S. strategic nuclear forces under New START, abandoned missile defense deployment in Poland and the Czech Republic,
engaged Russia in missile defense talks, pursued a policy of geopolitical neglect in the former Soviet Union, and toned down criticism
of political freedom violations in Russia. However, Putin
remains Russia’s “national leader” and the real power behind
—and on—the throne. Top White House and State Department officials now privately recognize that they bet

on the wrong horse , as it is unlikely that Medvedev will wield any real power beyond the spring of 2012. However, the
Administration cannot publicly admit that this bet failed, as it would undermine the very notion of
this over-personalized “reset.” Yet the reality that Medvedev has a limited capacity to deliver and is
unlikely to continue in office means that the U.S. should rethink its strategy for engaging with Russia’s leadership.

Can’t convince key officials

Cohen 11 – Senior Research Fellow in Russian and Eurasian Studies and International Energy Policy in the Douglas and Sarah
Allison Center for Foreign Policy Studies (Ariel, 06/15, “Reset Regret: U.S. Should Rethink Relations with Russian Leaders,”
http://www.heritage.org/research/reports/2011/06/reset-regret-us-should-rethink-relations-with-russian-leaders)

U.S.–Russian relations include issues such as human rights and Islamist extremism in Russia, the energy and
sovereignty concerns of U.S. friends and allies, Iran, and nuclear nonproliferation. The Obama Administration cannot
address these issues by pretending that Medvedev and his narrow circle of supporters wield the
real power. In fact, it is the Putin group—which includes the key energy, military and security services
officials, businessmen, and the leadership of the United Russia ruling party—that exercises the ultimate power .
Now Putin, no great friend of America, is likely to move back from the Prime Minister’s office to the Kremlin in the spring of 2012,
raising tough questions about Obama’s Russian policy. Putin publicly disagreed with Medvedev, his handpicked
successor, on a number of key policy issues, many of them vital to U.S. interests. These included the role of
freedom in the country, the legacy of Joseph Stalin (Putin called him “an effective manager”), and the collapse of the Soviet Union.
The two also argued on modernization, Libya, and persecution of the former oil magnate Mikhail Khodorkovsky. Putin also supports
“friendship” with China and Venezuela and good relations with Iran. At various points Putin accused the U.S. of supporting Islamist
terrorists in North Caucasus in order to dismantle Russia, illegally intervening in Iraq, being responsible for the global economic
recession, and toppling regimes in the Middle East through promotion of social media. Putin views modernization as primarily
boosting military technology, pays lip service to the fight against corruption, and directly intervenes in prominent court cases.
Putin formed his worldview in the KGB and by reading Russian nationalist philosophers. He famously considers the collapse of the
Soviet Union “the greatest geopolitical catastrophe of the 20th century.” He also does not like or trust the U nited
S tates.
No specific issue cooperation—all political lip service

Brookes 12 – Senior Fellow, National Security Affairs and Chung Ju-Yung Fellow for Policy Studies (Peter, 06/19, “US-Russia:
From ‘Reset’ to Regret,” http://www.heritage.org/research/commentary/2012/06/us-russia-from-reset-to-regret)

There’s little doubt from the reporting of the lackluster meeting between President Obama and Russian President Vladimir Putin at

the G-20 in Mexico yesterday that the White House’s Russia policy is moving from “reset” to “regret.”
Of course, we’ve seen this coming for awhile — despite lots of wishful thinking on the administration’s part. Team Obama’s
hope over the last three-plus years has been that Russia would become a partner of the United States on a range of
international issues if ties could only be “reset,” pruning away thorny tensions that have grown in the relationship. In other words, if
we could just get relations chummy enough, the Kremlin and the White House would become a dynamic duo, tackling a growing list
of world problems. So much for that plan. One key focus of the “reset” policy was getting Russia to help stop Iran’s expanding
nuclear (weapons) program. While supporting some added pressure on Tehran, Moscow hasn’t really come on board. This week’s
P5+1 meeting (the latest in a seemingly endless series) in Moscow on Iranian nukes probably won’t change that. In fact, after the
supposed “reset,” Russia finished building Iran’s first nuclear reactor and provided fuel for it. If Tehran doesn’t return the fuel rods, it
could reprocess them for plutonium, providing another avenue for making nukes. The Russians have been continually cranky about
US-led missile defense in Europe, too, seeing it as being aimed at their nuclear deterrent rather than at the growing Iranian missile
threat. The griping didn’t stop even after Obama unilaterally abrogated the deal to put antimissile sites in Poland and the Czech
Republic. Not long ago, a senior Russian general rattled a Soviet-like saber, threatening a pre-emptive strike on US-NATO missile
defenses in Eastern Europe, if necessary. Another flashpoint is Syria. Secretary of State Hillary Clinton last week blasted the Russians
with both barrels for sending weapons to their longtime friend, the Syrian regime. Even with a few words of support for a
democratic transition in Syria at the G-20, Moscow has frustrated Washington’s UN efforts to punish Syria’s
Basher, er, Bashar Assad. (The Kremlin is reportedly crabby about Libya, believing the mission crept beyond its original UN mandate.)
Russia continues to befriend countries of concern, too: Venezuela is a rapacious buyer of Russian arms; Moscow held its first-ever
naval exercises with Beijing in April in the Yellow Sea, waters China considers “sensitive” to US military operations. Team Obama has
tried to lump Russia in with its claimed foreign-policy successes, citing the New Strategic Arms Reduction Treaty and Moscow’s
provision of supply and withdrawal routes in and out of Afghanistan as proof positive of better relations. Of course, many see New
START as having advantaged the Russians, since a majority of the cuts came from us. The Russia-Afghanistan road is worrisome
because it’s much more expensive than through Pakistan (currently closed) and gives Moscow leverage over us, especially when we
pull out. Critics say we’ve also given ground on a Russian sphere of influence in some parts of the former Soviet
Union’s stompin’ grounds — and Obama’s certainly been pretty much mum on political and social liberty in Russia. Yet the president
may still be hoping for a “reset” redo in a second term. Who can forget his open-mike moment this spring, offering now-former
Russian President Dmitry Medvedev more flexibility on missile defense after the US election? Isn’t that comforting? The fact is
Putin wants the US to get out of the way of Russia’s re-emergence — but is willing to cooperate
on issues that benefit him politically or Moscow in general. That’s really nothing new. So it’s probably a good time
to forget the reset — and instead embrace a pragmatic policy that sees Russia for what it is, not what Team Obama hopes
against hope it will be.

Empirics go neg

Bendikova and Cohen 12 (Michaela, Research Assistant for Missile Defense and Foreign Policy in the Douglas and Sarah
Allison Center for Foreign Policy, Ariel, Senior Research Fellow in Russian and Eurasian Studies and International Energy Policy in the
Douglas and Sarah Allison Center for Foreign Policy Studies, 08/04, “Who Are the Real Cold War Monsters?”
http://blog.heritage.org/2011/08/04/who-are-the-real-cold-war-monsters/)

It has been over two years since the U nited S tates launched the “reset” policy. Where is it heading in view of
Russian rhetoric and threats? President Obama called the “reset” his “great achievement” only days after Putin’s “parasite”
outburst. Maybe he was encouraged by Russia’s issuing a series of postage stamps to commemorate his 50th birthday. If
history is any guide: The U nited S tates tried a policy of détente with the Soviet Union in the 1970s,
culminating in the kiss between President Jimmy Carter and Soviet Leader Leonid Brezhnev at the SALT II
Treaty signing in Vienna, Austria. The
U.S. reward for its more “constructive” stance, however, was the
Soviet invasion in Afghanistan.

Zero-sum attitude on both sides—concessions are pocketed

Korade ‘9 (Former Defense Secretary Says Nuclear Proliferation 'At a Tipping Point' Published By: CQ Homeland Security January
8, 2009 By Matt Korade, CQ Staff

"Specifically Russian leaders say publicly that the United States is the No. 1 threat," Joseph said,
"seeing us as the principal source of their national humiliation in the 1990s and perhaps the
greatest impediment to their goals. They seem to view interaction with the United States from a
zero-sum perspective. If we win they lose , if they win we lose. And I don't believe that this
calculus will change with the change in the White House ." Russia's opposition to the placement of a missile
defense shield against Iranian nuclear ballistic missiles in Eastern Europe highlights this attitude, Joseph said, providing Moscow with
an opportunity for a U.S. concession that, if allowed, could jeopardize international security. "My sense
is that we must resist the temptation of a grand gesture to Moscow , perhaps in the context of an offensive
arms control treaty that Moscow very much wants, independent of the missile defense issue," he said. "To abandon
missile defense in the face of Russian pressure would only whet the appetite for other concessions and only raise
questions about the credibility of the U.S. extended deterrent."
---Russia Relations- No Collapse

1NC Fenenko—no total relations collapse—historically marked by positive and negative


cycles, but crises push nations together—proves they’d solve impact regardless of other issues
—also means they can’t solve inevitable peaks and valleys in the relationship

US-Russian relations resilient –strategic cooperation won’t get rolled back

VOA News 11 (“Putin Presidency Unlikely to Derail US-Russia Relations” 10/7


http://www.voanews.com/english/news/europe/Putin-Presidency-Unlikely-to-Derail-US-Russia-Relations-131345683.html)

Many experts agree with Legvold that there


will not be any real change in U.S.-Russia relations with
Vladimir Putin back as president. Matthew Rojansky at the Carnegie Endowment for International
Peace, says Putin, as prime minister, if not calling all the shots, at least approved the key decisions related
to U.S. relations. “So for example, I don’t see New START [strategic arms agreement] being rolled
back," said Rojansky. "I don’t see cooperation on Afghanistan being rolled back. The Libya [U.N.]
resolution [imposing a no-fly zone] which Russia didn’t block was a difficult call and Putin certainly had
reservations and you heard him expressing those reservations. But did he ultimately come to some kind of consensus with
Medvedev? Clearly he did. I think the two of them operate as a unit.” Rojansky believes that while
the substance of the
U.S.-Russia relationship may not change, the tone might. “Obama has invested very heavily in
his relationship with Medvedev," he said. "It made sense. It was relatively easy for him because he and Medvedev come
from a similar kind of origin in the sense of both being lawyers, both being technology oriented, both being kind of globalists in their
outlook. Putin just doesn’t have that. And
I don’t see Putin and Obama pushing the relationship to be very
active by sheer force of personality and interest in one another. I just don’t think that’s going to happen.”
The analysts believe one thing is for sure: the U.S.-Russia relationship has grown over the years
to such an extent that they say a return to the tension-filled Cold War days is virtually
impossible. 

Relations resilient - empirically proven

Kortunov 96 (Andrei, Russian Science Foundation, Comparative Strategy, p. 335)

However, surprisingly enough there seemed to be no visible, qualitative deterioration in the


relationship in 1995-96. Indeed, at some points it looked as if the relations were sliding into a
mini-cold war (e.g., after the notorious Yeltsin-Clinton encounter at Budapest in fall 1994 or when
the North Atlantic Treaty Organization (NATO) started its air strikes against Bosnian Serbs without first
consulting Moscow). But, at every juncture, each side was able to bounce back minimizing
the negative impact of tensions in particular fields on the overall relationship.
---Russia Relations- AT: Afghanistan

Relations have a minimal effect on Afghanistan – only hurts the mission

Cohen 10 – Senior Research Fellow in Russian and Eurasian Studies and International Energy Policy in the Douglas and Sarah
Allison Center for Foreign Policy Studies (Ariel, 10/26, “Time to Revise Obama’s Russian “Reset” Policy,”
http://www.heritage.org/research/reports/2010/10/time-to-revise-obamas-russian-reset-policy)

Thus, the Kremlin is exploiting Obama’s “see no evil” approach in Russia’s expansion into former
Soviet space and cooperation with anti-Western regimes. Russia has also prioritized its Arctic expansion and persists in its claim to a
vast territory in the Arctic Ocean greater than Germany, France, and Italy combined. Moscow declared the Northern Sea Route
around the northern edge of Eurasia as its territorial waters and is backing up its claim under the Law of the Sea Treaty with military
force.[3] The Obama Administration believes that it needs strong international support for its
military campaigns in Afghanistan and Iraq as well as for confrontation with Iran, and North Korea, and, in the long run,
possibly China. And in doing so, the White House hopes to bring Moscow to the U.S. side. So far, any

such success is minimal . The Obama Administration’s Russia policy will inevitably produce a massive
loss of American influence in Eurasia and jeopardize the security of the U.S. and its friends and allies east
of the Oder. Jeopardizing allies while empowering strategic competitors does not equal safety.
---Russia Relations- AT: Bostrum

Bostrom changed his mind

Bostrom, ‘7 [Future of Humanity Institute, Faculty of Philosophy & James Martin 21st Century School, Oxford University, 2009
Gannon Award Recipient, The Future of Humanity, 2007, www.nickbostrom.com/papers/future.pdf]

Extinction risks constitute an especially severe subset of what could go badly wrong for humanity. There are many possible
global catastrophes that would cause immense worldwide damage, maybe even the collapse of modern
civilization, yet fall short of terminating the human species. An all-out nuclear war between Russia
and the United States might be an example of a global catastrophe that would be unlikely to result in
extinction. A terrible pandemic with high virulence and 100% mortality rate among infected individuals might be another
example: if some groups of humans could successfully quarantine themselves before being exposed, human extinction could be
avoided even if, say, 95% or more of the world’s population succumbed. What
distinguishes extinction and other
existential catastrophes is that a comeback is impossible. A non-existential disaster causing
the breakdown of global civilization is, from the perspective of humanity as a whole, a potentially recoverable
setback: a giant massacre for man, a small misstep for mankind.
---Russia Relations- AT: China Rise

Relations don’t solve China’s rise

Bhadrakumar 10 – career diplomat in the Indian Foreign Service. His assignments included the Soviet Union, South Korea, Sri
Lanka, Germany, Afghanistan, Pakistan, Uzbekistan, Kuwait and Turkey (MK, 03/27, “China wary of US-Russia nuclear embrace,”
http://www.atimes.com/atimes/China/LC27Ad02.html)

Obama was expected to pick up his "hotline" to his Russian counterpart Dmitry Medvedev on Friday to okay the first arms control
agreement of the post-Cold War era. The "reset" of US-Russia ties is under way, which is no mean achievement
considering the army of cold warriors in Washington, including within Obama's administration. However, at this historic point in
contemporary world politics, such an
arms control deal needs to be more than a bilateral Russian-
American affair. Moscow had a hugely important visitor this week - China's Vice President Xi Jinping, who is widely regarded as
the main candidate to succeed President Hu Jintao as the secretary general of the Chinese Communist Party at the 18th Party
Congress in 2012. The new Strategic Arms Reduction Treaty (START) agreement heralds an uncertain phase in the
complex US-Russia-China equation, and Beijing will watch closely because China's rise could well
be a leitmotif of the US maneuvering to "reset" ties with Russia. In a resonant statement in Moscow on
Tuesday, Xi all but suggested a Sino-Russian alliance. "Russia and China must become strategic props for each other
in the future on all questions which have a strategic interest for Russia," Xi said.
---Russia Relations- AT: Economy

Relations don’t solve the economy

Shleifer and Treisman 11 (Andrei, Professor of Economics at Harvard University, Daniel, Professor of Political Science at
the University of California, Los Angeles, and a Visiting Fellow at the Institute for Human Sciences in Vienna. He is the author of The
Return: Russia's Journey From Gorbachev to Medvedev, Jan/Feb, “Why Moscow Says No,” EBSCO)

But does Russia need Washington's help to achieve its key goals? The short answer is no. As a consumer of
Russian energy, the United States is inconsequential: during the last five years, the United States bought only two to four percent of
Russia's crude oil exports and almost none of its gas. Indeed, U.S. and Russian energy interests mostly conflict. The mining of shale
gas in the United States freed up LNG shipments to Europe, depressing prices. Washington, eager to wean its European partners
from dependence on Russia, favors construction of rival pipelines such as Nabucco. And in the long term, U.S. efforts to reduce
domestic demand for fossil fuels would lower the global price of oil, on which Moscow is dependent. More generally, Russia and
the United States share few economic interests . In 1995, six percent of Russia's exports went to
the U nited S tates; by 2009, the figure was three percent -- less than Russia exported to Poland.
Meanwhile, the U nited S tates supplies just five percent of Russia's imports. The U.S. share of capital
flows to Russia is also insignificant . In 1995, 28 percent of all inflows of foreign investment to Russia came from the
United States; by 2010, this share was just 2.5 percent. By then, the stock of Russian investment in the U.S. economy was larger than

that of U.S. investment in Russia. Russia's economic disengagement from the U nited S tates has coincided
with its growing ties to Europe and China. In 2009, Europe accounted for 52 percent of Russia's exports and 45
percent of its imports; another 14 percent of its imports came from China, up from two percent in 1995. Although Russian exports to
China currently make up less than six percent of the total, this number will rise as China's hunger for raw materials grows. Europe's
share of foreign investment in Russia increased from 41 percent in 1995 to 71 percent in 2010, although much of this is Russian
wealth repatriated from Cyprus, Luxembourg, and Switzerland.
---Russia Relations- AT: Environment

Nothing gets Russia on board for the environment

Shleifer and Treisman 11 (Andrei, Professor of Economics at Harvard University, Daniel, Professor of Political Science at
the University of California, Los Angeles, and a Visiting Fellow at the Institute for Human Sciences in Vienna. He is the author of The
Return: Russia's Journey From Gorbachev to Medvedev, Jan/Feb, “Why Moscow Says No,” EBSCO)

The New START treaty mostly ratified cuts in the Russian nuclear arsenal that were occurring anyway as the weapons aged. Further
reductions are not so clearly in Russia's interest; as antimissile systems become more accurate and powerful, Moscow will need to
maintain enough missiles and warheads to remain sure of a second-strike capability. On
climate change, the Kremlin
recognizes that global warming would impose huge costs, causing floods and destroying
infrastructure. Still, how the expense of cutting pollution should be shared among the major industrial
and industrializing countries remains contentious . Like other countries, Russia has a powerful
pro-carbon lobby.
---Russia Relations- AT: Heg

Relations wreck U.S. heg and nuclear primacy

Andersen 11 – Senior Digital Communications Associate at The Heritage Foundation (Ericka, 10/26, “Morning Bell: The Serious
Risks of the Russian Reset,” http://blog.heritage.org/2011/10/26/morning-bell-the-serious-risks-of-the-russian-reset/)

President Obama may believe that America’s “reset” policy with Russia is the correct move to cover important foreign policy bases,
but the policy is deeply flawed. It puts the United States at a disadvantage we can’t afford and forces us to lay aside fundamental
American principles of human liberty. The “reset” concessions are simply not worth the exchange of
empty promises from Russian President Dmitry Medvedev, who is merely a talking head for Prime Minister Vladimir
Putin. As Heritage’s Ariel Cohen & Kim Holmes wrote recently in a memo on U.S.–Russia Relations, Putin
would like nothing
less than a “Soviet-like superpower prestige and status through forced nuclear equality with Washington.” The
large “reset” payoff requires America put it all on the line by cutting U.S. strategic nuclear forces and
engaging in missile defense talks with Russia, in addition to abandoning missile defense deployment
in Poland and the Czech Republic and keeping quiet about political freedom violations running rampant throughout
Russia. America may never have won the Cold War 22 years ago with policies such as these. It is imperative that America lead with
the cause of freedom and justice when dealing with Russia, or any other nation for that matter. In Heritage’s Understanding America
series, Matthew Spalding explains that the United States was founded and thrives on “universal principles that appeal to a higher
standard.” Such universal principles of freedom should be the foundation of America’s foreign policy strategy—not an afterthought.
Yesterday at a Heritage Foundation conference focused on the “reset” policy, House Speaker John Boehner (R–OH) recalled the
international leadership that prompted America to victory in the Cold War not so long ago. He applauded President Ronald Reagan
and British Prime Minister Margaret Thatcher as two who “quite simply, loved freedom…[and] made their feelings well-known,
contagious, as if no one or no force could stand in their way.” Boehner urged America not forget what life was like for the Soviets
before these two warriors of freedom refused to stand for it. As Boehner said, “freedom most inspires those who remember life
without it.” While the Obama Administration may believe the “reset” policy as it stands is necessary, the deal raises a lot of red flags.
In his paper, Cohen urges that America must not tolerate Russian mischief or fail to make its priorities of freedom loud and clear. As
Boehner said, instead of negotiating with Russia, Washington should call its bluff—“Publicly, forcefully, frequently.” As the leader of
the free world, America has a responsibility to remain in control and end the idea that it is “leading from behind” when it comes to
Russia. In a recent memo, Cohen explained why the Obama Administration must stop its policy of “please Moscow” and push Russia
to “reset” its own policies. He writes: Moscow has continuously promoted in word and deed the idea that there
is or should be a multipolar world order that constrains U.S. foreign policies. A “reset” policy that ignores
Russia’s global efforts to undermine the U.S. recalls the ill-fated détente of the 1970s. As experts at yesterday’s conference attested,

the risks involved in America’s “reset” relations with Russia are many . Foreign policy dealings with any
nation—especially Russia—must be guided by America’s Founding values first and foremost. The consequences of doing otherwise
will be great.

Only a risk relations hurt U.S. influence

Cohen 10 – Senior Research Fellow in Russian and Eurasian Studies and International Energy Policy in the Douglas and Sarah
Allison Center for Foreign Policy Studies (Ariel, 10/26, “Time to Revise Obama’s Russian “Reset” Policy,”
http://www.heritage.org/research/reports/2010/10/time-to-revise-obamas-russian-reset-policy)
Thus, the Kremlin is exploiting Obama’s “see no evil” approach in Russia’s expansion into former
Soviet space and cooperation with anti-Western regimes. Russia has also prioritized its Arctic expansion and persists in its claim to a
vast territory in the Arctic Ocean greater than Germany, France, and Italy combined. Moscow declared the Northern Sea Route
around the northern edge of Eurasia as its territorial waters and is backing up its claim under the Law of the Sea Treaty with military
force.[3] The Obama Administration believes that it needs strong international support for its
military campaigns in Afghanistan and Iraq as well as for confrontation with Iran, and North Korea, and, in the long run,
possibly China. And in doing so, the White House hopes to bring Moscow to the U.S. side. So far, any

such success is minimal . The Obama Administration’s Russia policy will inevitably produce a
massive loss of American influence in Eurasia and jeopardize the security of the U.S. and its
friends and allies east of the Oder. Jeopardizing allies while empowering strategic competitors does not equal safety.
---Russia Relations- AT: Iran Prolif/Aggression

Relations don’t solve Iran prolif – Russia backs Iran and won’t cooperate

Cohen 12 – Senior Research Fellow in Russian and Eurasian Studies and International Energy Policy in the Douglas and Sarah
Allison Center for Foreign Policy Studies (Ariel, 03/15, “How the U.S. Should Respond to Russia's Unhelpful Role in the Middle East,”
http://www.heritage.org/research/reports/2012/03/how-the-us-should-respond-to-russias-unhelpful-role-in-the-middle-east)

The anti-American tilt of Russian foreign policy prevents diplomatic cooperation because the
U.S. and Russia lack a shared threat assessment and mutual understanding in dealing with the changing dynamics of the Middle East.
Despite clear statements to the contrary by Prime Minister Putin and Foreign Minister Lavrov, the Obama Administration has
repeatedly declared that the U.S. is not competing with Russia for regional influence. Regrettably, the Kremlin has not received this
memo. Instead, Russian attempts to constrain U.S. policy have provoked little or no response from Washington. Lavrov habitually
invokes a “polycentric” or multipolar model of the world, with Russia working with her partners toward a
future in which U.S. power is so diminished that it cannot act without Moscow’s
permission . Russia’s vision of the Middle East is a case in point.[68] Moscow’s concept of multipolarity entails not just an
uncontested Russian sphere of influence in the Commonwealth of Independent States, but also together with Iran wielding much
greater clout in the Middle East. Moscow clearly wants to retain ties with Iran , which it regards as the rising great
power in the Gulf and Middle East. However, the Obama Administration has been deluding itself that
Russia would be a genuine partner in restraining Iran. Notwithstanding Washington’s and Riyadh’s irritation,
Russia defends the Assad regime despite its bloody repression of its own citizens. Even though the regime is teetering on collapse,
Russia has signed an agreement with Syria to refurbish Soviet naval bases in Latakiyah and Tartus and has increased sales of
sophisticated weapons. Thus, Russia is obstructing U.N. resolutions censuring Syria, while allowing its relationship with the Obama
Administration to wilt.[69] Moscow’s suspicions of the U.S. and the prevailing anti-American mindset
lead it to persist in playing a zero-sum game in the Middle East and elsewhere. The intense competition,
in turn, tends to work to the advantage of third countries, such as Iran and China, and of terrorist groups, such as Hamas and
Hezbollah.[70] For instance, although Iran and nonstate or state-sponsored Islamist radicals present long-term dangers to both
states, Russia tends to ignore the Iranian threat. U.S. interests lie in a more democratic and pro-Western
environment that fosters civil society and economic opportunity. However, the Obama Administration’s myopic laissez-faire attitude
toward Islamists seems to have moved this goal further away than before the Arab upheavals erupted.[71] International energy
companies also need security for capital-intensive energy projects, which often require investments of the tens of billions of dollars.
Russia’s zero-sum policy is preventing Washington and Moscow from identifying and exploring
areas in which U.S. and Russian interests in the region converge, such as anti-terrorism and disrupting funding of
globally active radical Islamists. The areas in which the two states are pursuing diverging foreign policy goals, such as Russia’s trade
in arms and nuclear reactors, will require special attention and, where necessary, consistent pushback. Russia’s interests in
the region—including energy and weapons trade, supporting a nuclear Iran, and attempting to selectively legitimize
anti-Israel radical Islamist organizations while fighting similar ones at home—contradict U.S. interests. In addition, Russia is
pursuing a diplomatic strategy of developing an ad hoc Sino–Russian axis to undermine U.S. priorities around the world, particularly
in the Middle East.

Relations don’t solve – Russia directly supports Iran prolif

Brookes 12 – Senior Fellow, National Security Affairs and Chung Ju-Yung Fellow for Policy Studies (Peter, 06/19, “US-Russia:
From ‘Reset’ to Regret,” http://www.heritage.org/research/commentary/2012/06/us-russia-from-reset-to-regret)
There’s little doubt from the reporting of the lackluster meeting between President Obama and Russian President Vladimir Putin at

the G-20 in Mexico yesterday that the White House’s Russia policy is moving from “reset” to “regret.”
Of course, we’ve seen this coming for awhile — despite lots of wishful thinking on the administration’s part. Team Obama’s
hope over the last three-plus years has been that Russia would become a partner of the United States on a range of
international issues if ties could only be “reset,” pruning away thorny tensions that have grown in the relationship. In other words, if
we could just get relations chummy enough, the Kremlin and the White House would become a
dynamic duo, tackling a growing list of world problems. So much for that plan. One key focus of the “reset”

policy was getting Russia to help stop Iran’s expanding nuclear (weapons) program. While supporting some added
pressure on Tehran, Moscow hasn’t really come on board. This week’s P5+1 meeting (the latest in a seemingly
endless series) in Moscow on Iranian nukes probably won’t change that. In fact, after the supposed “reset,” Russia finished
building Iran’s first nuclear reactor and provided fuel for it. If Tehran doesn’t return the fuel
rods, it could reprocess them for plutonium, providing another avenue for making nukes. The Russians have been
continually cranky about US-led missile defense in Europe, too, seeing it as being aimed at their nuclear deterrent rather than at the
growing Iranian missile threat. The griping didn’t stop even after Obama unilaterally abrogated the deal to put antimissile sites in
Poland and the Czech Republic. Not long ago, a senior Russian general rattled a Soviet-like saber, threatening a pre-emptive strike on
US-NATO missile defenses in Eastern Europe, if necessary. Another flashpoint is Syria. Secretary of State Hillary Clinton last week
blasted the Russians with both barrels for sending weapons to their longtime friend, the Syrian regime. Even with a few words of
support for a democratic transition in Syria at the G-20, Moscow has frustrated Washington’s UN efforts to punish Syria’s Basher, er,
Bashar Assad. (The Kremlin is reportedly crabby about Libya, believing the mission crept beyond its original UN mandate.) Russia
continues to befriend countries of concern, too: Venezuela is a rapacious buyer of Russian arms; Moscow held its first-ever naval
exercises with Beijing in April in the Yellow Sea, waters China considers “sensitive” to US military operations. Team Obama has tried
to lump Russia in with its claimed foreign-policy successes, citing the New Strategic Arms Reduction Treaty and Moscow’s provision
of supply and withdrawal routes in and out of Afghanistan as proof positive of better relations. Of course, many see New START as
having advantaged the Russians, since a majority of the cuts came from us. The Russia-Afghanistan road is worrisome because it’s
much more expensive than through Pakistan (currently closed) and gives Moscow leverage over us, especially when we pull out.
Critics say we’ve also given ground on a Russian sphere of influence in some parts of the former Soviet Union’s stompin’ grounds —
and Obama’s certainly been pretty much mum on political and social liberty in Russia. Yet the president may still be hoping for a
“reset” redo in a second term. Who can forget his open-mike moment this spring, offering now-former Russian President Dmitry
Medvedev more flexibility on missile defense after the US election? Isn’t that comforting? The fact is Putin
wants the US to
get out of the way of Russia’s re-emergence — but is willing to cooperate on issues that benefit
him politically or Moscow in general. That’s really nothing new. So it’s probably a good time to forget the reset
— and instead embrace a pragmatic policy that sees Russia for what it is, not what Team Obama hopes against hope it will be.

Relations only result in weak sanctions on Iran – Russia uses this to elicit U.S. concessions –
feeds our turns

Vajdic 11 – research assistant in foreign and defense policy studies at the American Enterprise Institute (Daniel, 08/22, “Don’t
Let Russia Use Iran as a Bargaining Chip,” http://www.nationalreview.com/corner/275236/dont-let-russia-use-iran-bargaining-chip-
daniel-vajdic)

But it would be wrong to consider Moscow’s abrupt change in approach toward Iran an entirely altered policy. Russia’s recent
behavior remains consistent with its strategy in Iran over the last decade. The
Kremlin increases its involvement in
Iran in order to generate leverage that can , in turn, be used to extract concessions from the U.S. Its
present effort to engage Iran is nothing more than an attempt to convince the U.S. to alter some
of its less accommodating Russia policies. Let’s hope that the Obama administration doesn’t yield to Moscow’s
implicit blackmail, first and foremost because Russia
doesn’t have the capacity to decisively affect the
situation surrounding Iran’s nuclear program. Giving in to Russia’s demands — which include reduced U.S.
involvement in the former Soviet Union and disregard for Russia’s democratic deficiencies — will at best result in
another round of heavily diluted Security Council sanctions against Tehran. As we’ve seen with the
existing four rounds of sanctions, however, weak
economic penalties won’t convince Iran to forgo its
nuclear ambitions. There’s simply no reason to capitulate to the Kremlin’s attempts to employ Iran as an artificial bargaining
chip in its relations with the West.

Russia will never allow sanctions

Satter 9 – senior fellow at the Hudson Institute and a visiting scholar at the Johns Hopkins University School of Advanced
International Studies (David, 09/28, “Will Russia Sanction Iran? Don’t Bet on It,”
http://www.nationalreview.com/corner/187817/will-russia-sanction-iran-dont-bet-it/david-satter)

Despite the naïve exuberance of President Obama’s Russia advisers, there


is little likelihood that Russia will join the
West in imposing crippling sanctions against Iran. Since 2003, Russia has consistently frustrated or
watered down all attempts at sanctions. Russian diplomats often privately support the Western position and
Russia has officially condemned Iranian enrichment. But Russia has never cooperated tangibly in stemming Iran’s
drive to develop nuclear weapons. Unfortunately, President Medvedev’s seeming endorsement of sanctions is unlikely
to indicate a change. Words are cheap for the Kremlin and the Iranians are aware of this. The Russians,

having endorsed sanctions, will now find hundreds of reasons why any specific sanctions package is
unfair. They will say the timing is unreasonable, cultural sensitivities are not being respected, that there need to be exceptions,
and diplomacy must still be given a chance. By the time they are done confusing the issue and making genuine sanctions impossible,
the West will have forgotten that they ever said that they were in favor of them. The purpose of Medvedev’s remarks was to impress
on the Western audience that once Western governments stop behaving in an aggressive manner, i.e. in a manner which limits the
scope for Russian aggression, Russian cooperation is assured. This is nonsense, of course. The West is not aggressive, Russia does
seek to dominate its neighbors and if Russia did not have enemies, it would need to invent them. But it’s a way of assuring that the

tragic-comedy called “Reset” has a long way to run. The reason is that support for Iran is Russia’s most important
trump card in foreign relations and there is little likelihood that they will give it up.
---Russia Relations- AT: Missile Defense

Russia already agreed to cooperate – no war scenarios

Bridge, ’12 (Robert, 1/19/12, RT staff writer, http://rt.com/politics/nato-russia-makarov-2012-cooperation-213/)

Although disagreements remain between Russia and NATO over the US missile defense
project which Russia has called a threat to its national security, the two sides will continue
working together in 2012. Russian Chief of General Staff Nikolai Makarov praised Russia-NATO achievements in 2011,
which he said provided the basis for further cooperation in 2012. "On the whole, we have a positive impression from
Russia-NATO military cooperation in 2011,” Makarov told reporters following a meeting with NATO
colleagues at the Russia-NATO Council in Brussels on Thursday. “Thus, a plan of military cooperation (between
Russia and NATO) for 2012 has been approved.” Russia’s top military official picked out some of last year’s high
points in Russia’s partnership with NATO. "Vigilant Skies and Bold Monarch were the most significant joint exercises in 2011,”
Makarov noted. “We also had useful contacts in military medicine, logistics, disarming of handmade bombs and personnel training."
Russia’s highest-ranking military official also mentioned the “joint combating of pirates off the Horn of Africa,” which he said is now
entering “a new phase.” Finally, Makarov discussed the situation in Afghanistan, where Russia and NATO are “intensifying their
interaction” in the fight against dire local threats including, but not limited to, terrorism and drug trafficking. The Chief of General
Staff stressed that what happens in Afghanistan, where Allied forces have been fighting a 10-year uphill war against Taliban forces,
has a “direct influence on security along the Russian border and worries our allies in the Collective Security Treaty Organization
(CSTO).” Given the urgency of the situation, Russia will continue to “render assistance to the peacekeepers in Afghanistan, primarily
with the transit of cargo," he said.

Russia will inevitably cooperate on NATO missile defense – solves mutual deterrence

AFP, ’12 (1/13/12, http://www.google.com/hostednews/afp/article/ALeqM5iSj2oAA6NjB-unEvEv-9nzlJwS8g?


docId=CNG.1329a5298ebd1e5db375cb847cc8fb3e.581)

WASHINGTON — The United States hopes to reach a deal with Russia by the end of the year for the
deployment of a ballistic missile shield in Europe , the State Department's top arms control official said. "We will
get a missile defense agreement for cooperation with Russia," Undersecretary of State for
Arms Control Ellen Tauscher said Thursday, according to the website of Foreign Policy magazine. "This is the place
where we can begin to put aside the Cold War and mutually assured destruction' and move
toward mutually assured stability'." The United States has long wanted to deploy a missile defense system in eastern
Europe that it says would be directed at Iran, but Russia has objected, saying such a system would undercut its own nuclear
deterrent. "The only way they are going to be assured ... the system does not undercut their strategic deterrent is to sit with us in
the tent in NATO and see what we are doing. They will only be their own eyes and ears," Tauscher said. "Is it a political leap of faith?
Yes. Are they ready to do it? No. But we are hoping that these
strategic stability talks over the next eight
months will start to loosen these old ties that have been binding everybody in the old way of
thinking." President Dmitry Medvedev said in November that Moscow was prepared to deploy short-range Iskander missiles in
the Kaliningrad enclave that borders EU members Poland and Lithuania in response to the deployment of a missile shield. Romania
and Poland have agreed to host part of a revamped US missile shield, and NATO member Turkey has also decided to host an early
warning radar. The United States and Russia entered into their first nuclear arms deal in two
decades last year, agreeing to reduce warhead ceilings by 30 percent and limit each side to 700
deployed long-range missiles and heavy bombers. Medvedev and US President Barack Obama
signed the new START in Prague in 2010 as the two sides tried to "reset" relations that soured
under the eight-year Republican administration of George W. Bush.
---Russia Relations- AT: Prolif

Relations don’t solve prolif

Cohen 11 – Senior Research Fellow in Russian and Eurasian Studies and International Energy Policy in the Douglas and Sarah
Allison Center for Foreign Policy Studies (Ariel, 06/15, “Reset Regret: U.S. Should Rethink Relations with Russian Leaders,”
http://www.heritage.org/research/reports/2011/06/reset-regret-us-should-rethink-relations-with-russian-leaders)

U.S.–Russian relations include issues such as human rights and Islamist extremism in Russia, the energy and
sovereignty concerns of U.S. friends and allies, Iran, and nuclear nonproliferation . The Obama Administration cannot
address these issues by pretending that Medvedev and his narrow circle of supporters wield the
real power. In fact, it is the Putin group—which includes the key energy, military and security services
officials, businessmen, and the leadership of the United Russia ruling party—that exercises the ultimate power .
Now Putin, no great friend of America, is likely to move back from the Prime Minister’s office to the Kremlin in the spring of 2012,
raising tough questions about Obama’s Russian policy. Putin publicly disagreed with Medvedev , his handpicked
successor, on a number of key policy issues, many of them vital to U.S. interests. These included the role of
freedom in the country, the legacy of Joseph Stalin (Putin called him “an effective manager”), and the collapse of the Soviet Union.
The two also argued on modernization, Libya, and persecution of the former oil magnate Mikhail Khodorkovsky. Putin also supports
“friendship” with China and Venezuela and good relations with Iran. At various points Putin accused the U.S. of supporting Islamist
terrorists in North Caucasus in order to dismantle Russia, illegally intervening in Iraq, being responsible for the global economic
recession, and toppling regimes in the Middle East through promotion of social media. Putin views modernization as primarily
boosting military technology, pays lip service to the fight against corruption, and directly intervenes in prominent court cases.
Putin formed his worldview in the KGB and by reading Russian nationalist philosophers. He famously considers the collapse of the
Soviet Union “the greatest geopolitical catastrophe of the 20th century.” He also does not like or trust the U nited
S tates.
---Russia Relations- AT: Terror

Relations don’t solve terror – Russia backs international terrorists and won’t cooperate

Cohen 12 – Senior Research Fellow in Russian and Eurasian Studies and International Energy Policy in the Douglas and Sarah
Allison Center for Foreign Policy Studies (Ariel, 03/15, “How the U.S. Should Respond to Russia's Unhelpful Role in the Middle East,”
http://www.heritage.org/research/reports/2012/03/how-the-us-should-respond-to-russias-unhelpful-role-in-the-middle-east)

The anti-American tilt of Russian foreign policy prevents diplomatic cooperation because the
U.S. and Russia lack a shared threat assessment and mutual understanding in dealing with the changing dynamics of the Middle East.
Despite clear statements to the contrary by Prime Minister Putin and Foreign Minister Lavrov, the Obama Administration has
repeatedly declared that the U.S. is not competing with Russia for regional influence. Regrettably, the Kremlin has not received this
memo. Instead, Russian attempts to constrain U.S. policy have provoked little or no response from Washington. Lavrov habitually
invokes a “polycentric” or multipolar model of the world, with Russia working with her partners toward a
future in which U.S. power is so diminished that it cannot act without Moscow’s
permission . Russia’s vision of the Middle East is a case in point.[68] Moscow’s concept of multipolarity entails not just an
uncontested Russian sphere of influence in the Commonwealth of Independent States, but also together with Iran wielding much
greater clout in the Middle East. Moscow clearly wants to retain ties with Iran, which it regards as the rising great power in the Gulf
and Middle East. However, the Obama Administration has been deluding itself that Russia would be a genuine partner in restraining
Iran. Notwithstanding Washington’s and Riyadh’s irritation, Russia defends the Assad regime despite its bloody repression of its own
citizens. Even though the regime is teetering on collapse, Russia has signed an agreement with Syria to refurbish Soviet naval bases
in Latakiyah and Tartus and has increased sales of sophisticated weapons. Thus, Russia is obstructing U.N. resolutions censuring
Syria, while allowing its relationship with the Obama Administration to wilt.[69] Moscow’s suspicions of the U.S. and
the prevailing anti-American mindset lead it to persist in playing a zero-sum game in the
Middle East and elsewhere. The intense competition, in turn, tends to work to the advantage of third countries, such as Iran and
China, and of terrorist groups, such as Hamas and Hezbollah.[70] For instance, although Iran and nonstate or state-sponsored
Islamist radicals present long-term dangers to both states, Russia tends to ignore the Iranian threat. U.S. interests lie in a more
democratic and pro-Western environment that fosters civil society and economic opportunity. However, the Obama
Administration’s myopic laissez-faire attitude toward Islamists seems to have moved this goal further away than before the Arab
upheavals erupted.[71] International energy companies also need security for capital-intensive energy projects, which often require
investments of the tens of billions of dollars. Russia’s
zero-sum policy is preventing Washington and Moscow
from identifying and exploring areas in which U.S. and Russian interests in the region converge, such
as anti-terrorism and disrupting funding of globally active radical Islamists. The areas in which the
two states are pursuing diverging foreign policy goals, such as Russia’s trade in arms and nuclear reactors, will require special
attention and, where necessary, consistent pushback. Russia’s
interests in the region—including energy and weapons
trade, supporting a nuclear Iran, and attempting
to selectively legitimize anti-Israel radical Islamist organizations
while fighting similar ones at home—contradict U.S. interests. In addition, Russia is pursuing a diplomatic strategy of
developing an ad hoc Sino–Russian axis to undermine U.S. priorities around the world, particularly in the Middle East.
---Russia Relations- AT: Warming

Nothing gets Russia on board to solve warming

Shleifer and Treisman 11 (Andrei, Professor of Economics at Harvard University, Daniel, Professor of Political Science at
the University of California, Los Angeles, and a Visiting Fellow at the Institute for Human Sciences in Vienna. He is the author of The
Return: Russia's Journey From Gorbachev to Medvedev, Jan/Feb, “Why Moscow Says No,” EBSCO)

The New START treaty mostly ratified cuts in the Russian nuclear arsenal that were occurring anyway as the weapons aged. Further
reductions are not so clearly in Russia's interest; as antimissile systems become more accurate and powerful, Moscow will need to
maintain enough missiles and warheads to remain sure of a second-strike capability. On
climate change, the Kremlin
recognizes that global warming would impose huge costs, causing floods and destroying
infrastructure. Still, how the expense of cutting pollution should be shared among the major industrial
and industrializing countries remains contentious . Like other countries, Russia has a powerful
pro-carbon lobby.

Already working at capacity to solve

Hiskes ‘9 (Jonathon Hiskes, Grist, “Global warming is no friend to Russia, ambassador says”, http://www.grist.org/article/2009-
08-03-global-warming-is-no-friend-to-russia-ambassador-says/

August 3, 2009,

Red Square in Moscow: It gets cold there, but that doesn't mean they like global warming. Russia may be one of the coldest
nations on Earth, but it has no interest in seeing global warming continue unchecked, the Russian
ambassador to the United States said in an interview. Ambassador Sergei Kislyak said Russia is
willing to work with other countries to reduce greenhouse-gas emissions. He disagreed
sharply with recent news reports suggesting Russian leaders may welcome climate change
because it would make Arctic gas and oil deposits and northern regions more accessible. “ Climate change brings not only
the warming of Siberia, it brings many problems that we’ll have to cope with,” Kislyak said. “They will outweigh the
benefits, the perceived benefits. We have developed a lot of technologies to make even the most remote places in Siberia
accessible. It’s not the biggest problem.” Heading into climate talks in Copenhagen this December, Russia wants to
ensure that all heavy emitters are involved in an international treaty , he said. Russia is the third-
largest emitter of greenhouse gases behind China and the United States. “ We want all the countries that
contribute to climate change to be on board in cutting emissions ,” Kislyak said. “That is kind of
our guiding principle. Certainly the negotiations are going to be difficult. But I would say that, more or less, our
positions are closer and closer with the United States .” Kislyak, a veteran diplomat and a nuclear physicist
by training, acknowledged that climate change wasn’t high among his areas of expertise. He framed the issue largely in
economic terms. “We want the issue of climate change to be addressed in a way that will promote the stability of the climate,
rather than the way it is devolving now,” he said. “But I think everybody would claim that. The issue is, at what price and who is
going to do what?” We spoke after his talk on Russian-U.S. relations in Seattle last Friday, hosted by the
Foundation for Russian American Economic Cooperation. Last September Kislyak began his ambassadorship
in the aftermath of the violent conflict in South Ossetia, a time he described as the lowest
point in U.S.-Russia relations since the Cold War. In his address, he said he drew a good
deal of optimism from President Barack Obama’s meeting with Russian President Dmitri Medvedev in Moscow
last month. Yet he gave unapologetic defenses for Russia’s position in Georgia, its stance on Iran’s nuclear pursuits, and its
opposition to a U.S. missile shield in Eastern Europe. “We were told [the missile shield] is not against us, it’s against Iran, so
Russia shouldn’t be worried. People are saying Russia was consulted when the decision was made. All of this is not completely
true,” he said. He also took a confrontational tone in discussing the Kyoto climate treaty. “We are, by the way, members of the
Kyoto Protocol,” he said. “You are not. And we can afford this, easily.” What he didn’t say is that the reason Russia can afford to
meet Kyoto benchmarks is because they are based on emissions levels from 1990, two years before Russia’s economy nosedived
in the aftermath of the Soviet Union's collapse. Russia can continue growing its economy (and climate pollution with it) and stay
Kislyak had more to say about energy efficiency,
comfortably within Kyoto standards for several years.
which is a focal point of a recent report from the Center for American Progress that calls for a “reset” of U.S.-
Russia relations via climate and energy cooperation . “It is a priority for Russia because our
economy is much less efficient than many others --several times less efficient,” he said. “It’s not because we
are not technologically advanced, it’s because we have been living with the luxury of having so much fossil fuel that we simply
didn’t care too much about it. “Times have changed and we
understand that fossil fuels need to be left for
future generations. We need to be energy sufficient, but I would underline that it’s part of the
Russian economic program no matter what. Whether “here will be [an international climate] conference or not,
we are going to modernize our economy for our own people.”
Russia War Answers
Frontline

Give a Russia war impact zero probability

Graham ‘7 (Thomas Graham, senior advisor on Russia in the US National Security Council staff 2002-2007, September 2007,
"Russia in Global Affairs” July - September 2007, The Dialectics of Strength and Weakness

An astute historian of Russia, Martin Malia, wrote several years ago that “Russia
has at different times been
demonized or divinized by Western opinion less because of her real role in Europe than because of the fears and
frustrations, or hopes and aspirations, generated within European society by its own domestic problems.”
Such is the case today. To be sure, mounting Western concerns about Russia are a consequence of Russian
policies that appear to undermine Western interests, but they are also a reflection of declining confidence in
our own abilities and the efficacy of our own policies. Ironically, this growing fear and distrust of Russia
come at a time when Russia is arguably less threatening to the West, and the United States in particular,
than it has been at any time since the end of the Second World War. Russia does not
champion a totalitarian ideology intent on our destruction, its military poses no threat to sweep across
Europe, its economic growth depends on constructive commercial relations with Europe, and its
strategic arsenal – while still capable of annihilating the United States – is under more reliable control than
it has been in the past fifteen years and the threat of a strategic strike approaches zero
probability. Political gridlock in key Western countries, however, precludes the creativity, risk-taking, and subtlety needed
to advance our interests on issues over which we are at odds with Russia while laying the basis for more constructive lon-term
relations with Russia.

Even a rapid US-Russia war would end in peace negotiations before nukes were launched –
Russian generals concede.

Ivashov ‘7 (Colonel General Leonid Ivashov, President of the Academy of Geopolitical Problems. July 2007 “WILL AMERICA FIGHT
RUSSIA”. Defense and Security, No 78. LN

Ivashov: Numerous scenarios and options are possible. Everything


may begin as a local conflict that will
rapidly deteriorate into a total confrontation . An ultimatum will be sent to Russia: say, change the
domestic policy because human rights are allegedly encroached on, or give Western businesses access to oil and gas fields.
Russia will refuse and its objects (radars, air defense components, command posts, infrastructure) will be
wiped out by guided missiles with conventional warheads and by aviation. Once this phase is over, an even
stiffer ultimatum will be presented - demanding something up to the deployment of NATO "peacekeepers" on the
territory of Russia. Refusal to bow to the demands will be met with a mass aviation and missile
strike at Army and Navy assets, infrastructure, and objects of defense industry. NATO armies will invade Belarus and
western Russia. Two turns of events may follow that. Moscow may accept the ultimatum through
the use of some device that will help it save face. The acceptance will be followed by talks over the estrangement
of the Kaliningrad enclave, parts of the Caucasus and Caspian region, international control over the Russian gas and oil complex,
and NATO control over Russian nuclear forces. The second scenario involves a warning from
the Kremlin to the United States that continuation of the aggression will trigger retaliation with the
use of all weapons in nuclear arsenals. It will stop the war and put negotiations into motion .
No Russia war

Weitz 11 - senior fellow at the Hudson Institute and a World Politics Review senior editor(Richard, 9/27/2011, “Global Insights:
Putin not a Game-Changer for U.S.-Russia Ties,” http://www.scribd.com/doc/66579517/Global-Insights-Putin-not-a-Game-Changer-
for-U-S-Russia-Ties)

Fifth, there will inevitably be areas of conflict between Russia and the United States regardless of who is in
the Kremlin. Putin and his entourage can never be happy with having NATO be Europe's most powerful security institution, since
Moscow is not a member and cannot become one. Similarly, the Russians will always object to NATO's missile
defense efforts since they can neither match them nor join them in any meaningful way. In the case of Iran, Russian officials
genuinely perceive less of a threat from Tehran than do most Americans, and Russia has more to lose from a cessation of economic
ties with Iran -- as well as from an Iranian-Western reconciliation. On the other hand, these conflicts can be managed,
since they will likely remain limited and compartmentalized . Russia and the West do not have
fundamentally conflicting vital interests of the kind countries would go to war over. And as the
Cold War demonstrated, nuclear weapons are a great pacifier under such conditions. Another novel
development is thatRussia is much more integrated into the international economy and global
society than the Soviet Union was, and Putin's popularity depends heavily on his economic track
record. Beyond that, there are objective criteria, such as the smaller size of the Russian population and
economy as well as the difficulty of controlling modern means of social communication, that will
constrain whoever is in charge of Russia.

No risk of US-Russia War

Ball 5 (Desmond, Professor – Strategic Defense Studies Centre at Australian National University, “The Probabilities of ‘On the
Beach’ Assessing Armageddon Scenarios in the 21st Century, May, http://www.manningclark.org.au/papers/se05_ball.html)

The prospects of a nuclear war between the United States and Russia must now be deemed
remote. There are now no geostrategic issues that warrant nuclear competition and no
fairly
inclination in either Washington or Moscow to provoke such issues. US and Russian strategic forces have
been taken off day-to-day alert and their ICBMs ‘de-targeted’, greatly reducing the possibilities
of war by accident, inadvertence or miscalculation. On the other hand, while the US-Russia strategic competition
is in abeyance, there are several aspects of current US nuclear weapons policy which are profoundly disturbing. In December
2001 President George W. Bush officially announced that the United States was withdrawing from the Anti-Ballistic Missile
(ABM) Treaty of 1972, one of the mainstays of strategic nuclear arms control during the Cold War, with effect from June 2002,
and was proceeding to develop and deploy an extensive range of both theatre missile defence and national missile defence
(NMD) systems. The first anti-missile missile in the NMD system, designed initially to defend against limited missile attacks from
China and North Korea, was installed at Fort Greely in Alaska in July 2004. The initial system, consisting of sixteen interceptor
missiles at Fort Greely and four at Vandenberg Air Force in California, is expected to be operational by the end of 2005. The
Bush Administration is also considering withdrawal from the Comprehensive Test Ban Treaty and resuming nuclear testing. (The
last US nuclear test was on 23 September 1992). In particular, some key Administration officials believe that testing is necessary
to develop a ‘new generation’ of nuclear weapons, including low-yield, ‘bunkerbusting’, earth-penetrating weapons specifically
designed to destroy very hard and deeply buried targets (such as underground command and control centres and leadership
bunkers).
Ties check

Hoffman ’12 (David E. Hoffman, contributing editor to Foreign Policy and the author of The Dead Hand: The Untold Story of
the Cold War Arms Race and Its Dangerous Legacy, which won the 2010 Pulitzer Prize for general non-fiction, "Hey, Big Spender,"
Foreign Policy, www.foreignpolicy.com/articles/2012/10/22/hey_big_spender?page=full, October 22, 2012)

Despite tensions that flare up, the United States and Russia are no longer enemies; the chance of
nuclear war or surprise attack is nearly zero. We trade in each other's equity markets. Russia has
the largest audience of Facebook users in Europe, and is open to the world in a way the Soviet Union never
was.

U.S. first strike would decapitate Russia

Lieber and Press 6 (Keir A., Assistant Professor of Political Science – University of Notre Dame and Daryl G., Professor of
Political Science – University of Pennsylvania, “The Rise of U.S. Nuclear Primacy”, Foreign Affairs, March / April, Lexis)

Even as the United States' nuclear forces have grown stronger since the end of the Cold War, Russia's strategic nuclear
arsenal has sharply deteriorated. Russia has 39 percent fewer long-range bombers, 58 percent fewer ICBMs, and
80 percent fewer SSBNs than the Soviet Union fielded during its last days. The true extent of the Russian arsenal's decay,
however, is much greater than these cuts suggest. What nuclear forces Russia retains are hardly ready for
use. Russia's strategic bombers, now located at only two bases and thus vulnerable to a surprise attack, rarely conduct training
exercises, and their warheads are stored off-base. Over 80 percent of Russia's silo-based ICBMs have exceeded their original
service lives, and plans to replace them with new missiles have been stymied by failed tests and low rates of production.
Russia's mobile ICBMs rarely patrol, and although they could fire their missiles from inside their bases if given sufficient warning
of an attack, it appears unlikely that they would have the time to do so. The third leg of Russia's nuclear triad has weakened the
most. Since 2000, Russia's SSBNs have conducted approximately two patrols per year, down from 60 in 1990. (By contrast, the
U.S. SSBN patrol rate today is about 40 per year.) Most of the time, all nine of Russia's ballistic missile submarines are sitting in
port, where they make easy targets. Moreover, submarines require well-trained crews to be effective. Operating a ballistic
missile submarine -- and silently coordinating its operations with surface ships and attack submarines to evade an enemy's
forces – is not simple. Without frequent patrols, the skills of Russian submariners, like the submarines themselves, are decaying.
Revealingly, a 2004 test (attended by President Vladimir Putin) of several submarine-launched ballistic missiles was a total
fiasco: all either failed to launch or veered off course. The fact that there were similar failures in the summer and fall of 2005
completes this unflattering picture of Russia's nuclear forces. Compounding these problems, Russia's early warning system is a
mess. Neither Soviet nor Russian satellites have ever been capable of reliably detecting missiles launched from U.S. submarines.
(In a recent public statement, a top Russian general described his country's early warning satellite constellation as "hopelessly
outdated.") Russian commanders instead rely on ground-based radar systems to detect incoming warheads from submarine-
launched missiles. But the radar network has a gaping hole in its coverage that lies to the east of the country, toward the Pacific
Ocean. If U.S. submarines were to fire missiles from areas in the Pacific, Russian leaders probably would not know of the attack
until the warheads detonated. Russia's radar coverage of some areas in the North Atlantic is also spotty, providing only a few
minutes of warning before the impact of submarine-launched warheads. Moscow could try to reduce its vulnerability by finding
the money to keep its submarines and mobile missiles dispersed. But that would be only a short-term fix. Russia has already
extended the service life of its aging mobile ICBMs, something that it cannot do indefinitely, and its efforts to deploy new
strategic weapons continue to flounder. The Russian navy's plan to launch a new class of ballistic missile submarines has fallen
far behind schedule. It is now highly likely that not a single new submarine will be operational before 2008, and it is likely that
none will be deployed until later. Even as Russia's nuclear forces deteriorate, the United States is
improving its ability
to track submarines and mobile missiles, further eroding Russian military leaders' confidence in Russia's
nuclear deterrent. (As early as 1998, these leaders publicly expressed doubts about the ability of Russia's ballistic missile
submarines to evade U.S. detection.) Moreover, Moscow has announced plans to reduce its land-based ICBM force by another
35 percent by 2010; outside experts predict that the actual cuts will slice 50 to 75 percent off the current force, possibly leaving
Russia with as few as 150 ICBMs by the end of the decade, down from its 1990 level of almost 1,300 missiles. The more
Russia's nuclear arsenal shrinks, the easier it will become for the United States to carry out a first
strike. To determine how much the nuclear balance has changed since the Cold War, we ran a computer model of a
hypothetical U.S. attack on Russia's nuclear arsenal using the standard unclassified formulas that defense analysts have
used for decades. We assigned U.S. nuclear warheads to Russian targets on the basis of two criteria: the most accurate weapons
were aimed at the hardest targets, and the fastest-arriving weapons at the Russian forces that can react most quickly. Because
Russia is essentially blind to a submarine attack from the Pacific and would have great difficulty detecting the approach of low-
flying stealthy nuclear-armed cruise missiles, we targeted each Russian weapon system with at least one submarine-based
warhead or cruise missile. An attack organized in this manner would give Russian leaders virtually no warning. This simple plan is
presumably less effective than Washington's actual strategy, which the U.S. government has spent decades perfecting. The real
U.S. war plan may call for first targeting Russia's command and control, sabotaging Russia's radar stations, or taking other
preemptive measures -- all of which would make the actual U.S. force far more lethal than our model assumes. According to our
model, such a simplified surprise attack would have a good chance of destroying every Russian bomber base, submarine, and
ICBM. [See Footnote #1] This finding is not based on best-case assumptions or an unrealistic scenario in which U.S. missiles
perform perfectly and the warheads hit their targets without fail. Rather, we used standard assumptions to estimate the likely
inaccuracy and unreliability of U.S. weapons systems. Moreover, our model indicates that all of Russia's
strategic nuclear arsenal would still be destroyed even if U.S. weapons were 20 percent less accurate than
we assumed, or if U.S. weapons were only 70 percent reliable, or if Russian ICBM silos were 50 percent "harder" (more
reinforced, and hence more resistant to attack) than we expected. (Of course, the unclassified estimates we used may
understate the capabilities of U.S. forces, making an attack even more likely to succeed.)
---Russia War- No War

No chance of Russian first strike

Turner 97 (Stansfield, Director of the Center for Security and International Relations – University of Maryland, Caging the
Nuclear Genie, p. 57)

The probability of Russia being able to coordinate a perfect near-simultaneous attack on


even a modest force of ICBMs is very low. Enough of them would surely survive, and so the
Russians would be self-deterred . The ICBM, then, is a liability because of its image of vulnerability. That is what
drives us to large numbers and risk readiness procedures that appear to favor taking the offense.

Common ties, Russian leadership, and defense spending checks

Aron 6 (Leon, Director of Russian Studies – American Enterprise Institute, “The United States and Russia”, 6-29,
http://www.aei.org/publications/pubID.24606/pub_detail.asp)

Yet the probability of a frontal confrontation and a new Cold War remains very remote for
at least three reasons. First, despite the erosion, the countries geopolitical assets are still
very weighty, as the bedrock issues of anti-terrorism, nuclear nonproliferation, and energy
will continue to force them to seek common ground and at least limited partnership.[17]
Second, the restorationist foreign policy notwithstanding, the three basic elements of the 1992-1993 national consensus on
the foreign policy and defense doctrine remain largely the same. Russia is to stay a nuclear superpower and
the regional superpower, but it seems to have settled for the role of one of the world’s great states, rather than
a global superpower engaged in a worldwide competition with the United States. While these desiderata will continue to cause
occasional sparring with the United States, they are no
longer dedicated to the attainment of goals
inimical to the vital interests of the United States and are not likely to ignite a relentless
antagonistic struggle to the bitter end. Lastly, despite the muscular rhetoric emanating of
late from the Kremlin, unlike the Soviet Union twenty years ago and China today, Russia is
not a revisionist power. It does not seek radically to reshape the geopolitical balance of
forces in its favor. Moscow may rail at the score, but it is unlikely to endeavor to change the rules of the game. For that,
one needs a different ideology and, as a result, a different set of priorities. Yet even in today’s Russia flush with petrodollars, the
share of GDP devoted to defense (around 3 percent) is not only at least ten times smaller than in the Soviet Union, but also
below the 1992-1997 average in a Russia that inherited an empty treasury from the Soviet Union and that was, like every
revolutionary government, unable to collect taxes. Calculated in purchasing power parity, Russia’s
defense
expenditures in 2005 ($47.77 billion) were less than one-eleventh of what the U.S. spent
($522 billion).

Deterrence checks
Turner 2 (Admiral Stansfield, Former Director – Central Intelligence Agency, Fletcher Forum of World Affairs, Winter / Spring, 26
Fletcher F. World Aff. 115, Lexis)

There are, of course, other centrals question to be considered: Would Russian psychology differ from American and would
Russian society be willing to accept large numbers of nuclear detonations on their soil in order to perpetrate a nuclear war
against the United States? These are difficult questions to answer. The more pertinent concern, however, is that this is an issue
of life or death. No
head of state could contemplate plunging the world into nuclear conflict
without considering both the mortal threat to his or her citizens, and also the likelihood of his or
her own death, underground shelters notwithstanding. The presumption that heads of state prefer to
live than to die gives us one benchmark. Another is the Cuban missile crisis, in which both
Leonid Khruschev and President Kennedy quite visibly backed away from the prospect of very
limited nuclear war. Finally, Russia’s economy, being about the size of Belgium’s, is so small that its
leaders would be well aware that recovery, even from a small nuclear attack, would be a
very lengthy process. In terms of nuclear detonation threats, the United States must consider Russian deterrence as
very close to its own.
---Russia War- US Win

More evidence- we would win

Artyukov and Trukhachev ‘6 [Oleg and Vadim—Centre for Research on Globalization”


http://www.globalresearch.ca/index.php?context=va&aid=2154]

For the first time in the last 50 years the USA is


on the verge of attaining ultimate domination with
regard to nuclear weapons. This means that Russia is no longer able to keep up with the United States. If a
conflict were to break out, the USA would be able to quickly and with impunity attack Russian territory,
and Russia would have no means to mount a response. This is roughly the message of an article published
in the latest edition of the American journal Foreign Affairs. Its authors calculated that in comparison with theUSSR, the amount
of strategic bombers at Russia’s disposal has fallen by 39%, intercontinental ballistic missiles by 58% and the number of
submarines with ballisticmissiles by 80%. “However the true scale of the collapse of the Russian arsenal is much greater than
can be judged from these figures,” they write. “The strategic nuclear forces now at Russia's disposal are barely fit
to be used in battle.” Russian radar is now incapable of detecting the launch of American missiles
from submarines located in some regions of the Pacific Ocean. Russian anti-air defense systems might not
manage to intercept B-2 stealth bombers in time, which could easily mean that they are able to inflict a strike
with impunity on Russian nuclear forces. If Russian missile forces continue to decrease at the current rate, then in
about 10 years only isolated missiles, which the American anti-missile defense is capable of
intercepting, will be able to deliver a retaliatory blow. “It will probably soon be possible for the USA to destroy the
strategic nuclear potential of Russia and China with a single strike,” says the article.

War wouldn’t be close

Leiber and Press ‘6 (Keir A. Lieber and Daryl G. Press, Assistant Professor of Political Science, University of Notre Dame,
and Associate Professor of Political Science, University of Pennsylvania, “The End of MAD? The Nuclear Dimension of U.S.
Primacy,” INTERNATIONAL SECURITY, v 30 n 4, Spring 2006, p. 7-8. http://belfercenter.ksg.harvard.edu/files/is3004_pp007-
044_lieberpress.pdf)

the strategic nuclear balance has shifted profoundly. Part of the shift is
In the last fifteen years, however,
attributable to thedecline of the Russian arsenal. Compared with the Soviet force in 1990, Russia has 58
percent fewer intercontinental ballistic missiles (ICBMs), 39 percent fewer bombers, and 80 percent
fewer ballistic missile submarines (SSBNs).16 Furthermore, serious maintenance and readiness problems
plague Russia’s nuclear forces. Most of Russia’s ICBMs have exceeded their service lives, and a series of
naval accidents— highlighted by the sinking of the attack submarine Kursk in 2000— reºect the severe decay of the ºeet.17
Budgetary constraints have also dramatically reduced the frequency of Russia’s submarine and mobile ICBM patrols,
increasing the vulnerability of what would otherwise be the most survivable element of its arsenal. Since 2000, Russian SSBNs have
conducted approximately two patrols per year (with none in 2002), down from sixty in 1990, and apparently Russia often has no
mobile missiles on patrol.18 Finally,Russia has had difficulty maintaining satellite observation of U.S. ICBM
ªelds, and gaps in its radar network would leave it blind to a U.S. submarine-launched ballistic missile (SLBM) attack from
launch areas in the Paciªc Ocean.19 While the Russian strategic arsenal has eroded, the United States has continued to
modernize its weapons. U.S. strategic forces have shrunk in number since the end of the ColdWar, but they have
become more lethal. The U.S. submarine force has undergone nearly continuous improvement over the past ªfteen years.
The deployment of the highly accurate Trident II (D-5) SLBM was a Cold War decision, but the United States stuck with the
deployment plans and has steadily reªtted its entire SSBN ºeet to carry the new missile.20 Furthermore, the United States has
signiªcantly increased the lethality of the original Trident II missile against hard targets such as missile silos: the navy replaced nearly
400 of the 100-kiloton W76 warheads on these missiles with the more powerful 455-kiloton W88 warhead, creating an incredibly
lethal combination of accuracy and warhead yield. Other upgrades to Trident II include a more accurate reentry vehicle (RV) and
other improvements to increase the missile’s accuracy.21 The United States has also been upgrading its land-based missiles and
strategic bombers. Although the United States ªnished dismantling the MX Peacekeeper ICBM in 2005 in accordance with its arms
control commitments, the key elements that gave the MX exceptional lethality are being preserved. The nuclear warheads and
advanced RVs from the MX are beginning to replace the lower-yield warheads and less accurate RVs on 200 Minuteman III ICBMs. In
addition, the Minuteman guidance systems have been upgraded to roughly match the accuracy of the retired MX.22 In another
example of U.S. force modernization, the B-2 bomber has been given upgraded avionics that allow it to avoid radar by ºying at
extremely low altitude.23 At ªrst glance, this seems like a strange capability to give the B-2: the aircraft is so stealthy that it seems
hard to justify the risks of very low altitude ºight (e.g., crashing into the ter-rain) to reduce the bomber’s exposure to radar.
However, against an adversary with an extremely sophisticated air defense network (e.g., Russia today or China in the future), very
low-level ºight may be necessary to penetrate enemy airspace.
---Russia War- AT: Miscalc/Accidents
No accidents or miscalculation

Ball 6 (Desmond, Special Professor at the Strategic and Defence Studies Centre at the Australian National University, “The
Probabilities of ‘On the Beach,’” May, rspas.anu.edu.au/papers/sdsc/wp/wp_sdsc_401.pdf)

The prospects of a nuclear war between the United States and Russia must now be deemed
fairly remote. There are now no geostrategic issues that warrant nuclear competition and no
inclination in either Washington or Moscow to provoke such issues. US and Russian strategic
forces have been taken off day-to-day alert and their ICBMs ‘de-targeted’, greatly reducing the
possibilities of war by accident, inadvertence or miscalculation. On the other hand, while the US-Russia
strategic competition is in abeyance, there are several aspects of current US nuclear weapons policy which are profoundly
disturbing. In December 2001 President George W. Bush officially announced that the United States was withdrawing from the Anti-
Ballistic Missile (ABM) Treaty of 1972, one of the mainstays of strategic nuclear arms control during the Cold War, with effect from
June 2002, and was proceeding to develop and deploy an extensive range of both theatre missile defence and national missile
defence (NMD) systems. The first anti-missile missile in the NMD system, designed initially to defend against limited missile attacks
from China and North Korea, was installed at Fort Greely in Alaska in July 2004. The initial system, consisting of sixteen interceptor
missiles at Fort Greely and four at Vandenberg Air Force in California, is expected to be operational by the end of 2005. The Bush
Administration is also considering withdrawal from the Comprehensive Test Ban Treaty and resuming nuclear testing. (The last US
nuclear test was on 23 September 1992). In particular, some key Administration officials believe that testing is necessary to develop
a ‘new generation’ of nuclear weapons, including low-yield, ‘bunker-busting’, earth-penetrating weapons specifically designed to
destroy very hard and deeply buried targets (such as underground command and control centres and leadership bunkers).
---Russia War- AT: Bostrom

BOSTROM CONCLUDES US-RUSSIA WAR WON’T CAUSE EXTINCTION

Bostrom 7 (Nick, Ph.D. Professor of Applied Ethics at Oxford University, and the Director of the Oxford Future of Humanity
Institute, “The Future of Humanity”, No specific date
2007, http://www.fhi.ox.ac.uk/__data/assets/pdf_file/0007/10222/future_of_humanity.pdf)

Extinction risks constitute an especially severe subset of what could go badly wrong for humanity. There are many possible
global catastrophes that would causeimmense worldwide damage, maybe even the collapse of modern
civilization, yet fall short of terminating the human species . An all-out nuclear war between Russia
and the United States might be an example of a global catastrophe that would be unlikely to result in
extinction. A terrible pandemic with high virulence and 100% mortality rate among infected individuals might be another
example: if some groups of humans could successfully quarantine themselves before being exposed, human extinction could be
avoided even if, say, 95% or more of the world’s population succumbed. What distinguishes extinction and other
existential catastrophes is that a comeback is impossible . A non-existential disaster causing the breakdown of
global civilization is, from the perspective of humanity as a whole, a potentially recoverable setback: a giant massacre for man, a
small misstep for mankind. 
Russia Aggression Answers
Frontline

Expansionism will be contained and non-violent --- Russia won’t use force or go beyond the
FSU.

Lukyanov ’12 (Fyodor Lukyanov, 2/29/2012. Editor in chief of the journal Russia in Global Affairs (published with Foreign
Affairs). “Russia’s Changing Political Climate and Its Impact on Foreign Policy,” CSIS Public Lecture, **this is a summary prepared by
the Russia & Eurasia Program and edited by Dr. Andrew Kuchins, full audio transcript is available (http://csis.org/event/end-post-
soviet-period-russian-foreign-policy-what-next), http://csis.org/files/attachments/120229_Lukyanov_Lecture_Summary.pdf.

Putin’s recent article, “Russia and the Changing World,” held few surprises in terms of foreign policy, but offered some
insight into Putin’s worldview. Putin describes an international arena that is dangerous and unpredictable and
argues that Russia should be concerned with keeping itself safe. Thus, Russia does not have expansionist
intentions, but rather is concerned with keeping itself insulated from outside turbulences. Putin also
addressed the threat posed to Russia’s stability by what he calls “illegal soft power.” Putin does not oppose the operation of official
Western NGOs (such as the National Endowment for Democracy) in Russia, but rather the practice of funneling money into local
organizations. Putin believes this will inevitably corrupt those institutions’ agendas and turn them into agents of foreign powers.
While Putin used similar anti-American rhetoric in his Munich speech in 2007, the current approach has a different and a
more defensive tone. Putin in 2007 was frustrated with the West for failing to build an equal relationship with Russia; in 2012
Putin is frustrated that the West has proved incapable of solving threats to global stability. Putin’s current emphasis is thus
on protecting Russia from the resulting global instability. What must Russia do to deal with this changing environment?
Putin’s first article, published in January in Izvestiya, laid out one starting point: moving beyond the “post-Soviet agenda.” In the first
twenty years of Russia’s post-Soviet existence, its leaders have been preoccupied with returning Russia to the position of global
power and influence held by the Soviet Union. It is encouraging that Putin has recognized a need to move beyond these ambitions
and to focus instead on how to adapt to a more unpredictable world. There have been major changes in Russian
foreign policy since the 2008 war with Georgia, which was a watershed moment and a psychological
victory for Russia since it signaled the end of a geopolitical retreat and demonstrated to the U.S. where Russia’s red line really
lies. Because of the Georgian war, Russia now sees no major reason to encroach any further upon its near
abroad. When President Medvedev referred to the former Soviet space as Russia’s “sphere of privileged
interests,” this actually served as an admission that, unlike during Soviet times, Moscow’s interests were
now limited to a regional arena , rather than the global one. Even within Russia’s privileged zone,
Moscow’s policy has not become more aggressive. Instead, Russia withheld from intervening in the
2010 inter-ethnic clashes in Kyrgyzstan, whereas several years ago it would have felt compelled to act. Moreover, Putin no longer
refers to the Cold War with the same frequency as he used to, marking a departure from old perspectives. This has also changed the
dynamics of U.S.-Russia relations. Ideological differences can no longer be used as an excuse for non-cooperation. Instead, we are
now witnessing a misunderstanding derived from fundamentally different worldviews. Putin expressed a disappointment with the
West and particularly with the U.S. in how it has implemented its vision of democratization in countries across the globe; Putin
doesn’t believe that intervention to such ends can be successful and does not approve of the West’s efforts to do so. Putin also has a
fundamental difference with the West and other countries on purely conceptual terms; Putin believes in classical principles of
international relations, with sovereign states as the subjects of international relations and fundamental building blocks of global
society. In Putin’s view, the West’s use of normative rhetoric and appeals to values are simply covers for the pursuit of national
interests in specific contexts. Furthermore, Putin still believes that Russia should remain a global power, one that does not
necessarily need to expand but needs instead to secure its interests in a particular part of the world without engaging or interfering
in the affairs of others. Putin is seeking to secure the status quo , and in this way he is making Russia the guarantor of
a certain set of the principles of balance of power and strategic independence. In order to defend these principles, Russia can only
rely on its own, independent strength. Putin has also tried to identify partners, such as India and China, who share a similar view of
international politics.
Russia is peaceful

Ottens ‘11 (Nick Ottens, editor of the transatlantic news and commentary site the Atlantic Sentinel and contributing analyst for
the geostrategic consultancy Wikistrat, “The Myth of Russia’s Resurgence,” August 20 2011,
http://atlanticsentinel.com/2011/08/the-myth-of-russias-resurgence/)

Wikistrat‘s Thomas Barnett reminds readers of Russian fears of encirclement in his latest World Politics Review column. After
shrugging off its empire in 1991, Russia was denied a “sense of belonging,” Barnett notes, when Europe and the United States
refused to consider Russia’s entry to NATO. Instead , America moved in militarily from the south as part of its
global War on Terror while China progressively encroached, in an economic sense, on Russia’s “near abroad” in Central Asia and
the Far East. Russiahas been remarkably reluctant to counter these infringements. Although nearly all
former Warsaw Pact members belong to the European Union now, it has made only halfhearted attempts to
regain a semblance of hegemony on its western border. Old Eastern Bloc nations may still worry about Russian
antagonism, especially if Germany, which is so dependent on Russian gas imports, won’t truly protect them in the EU (which is
why they expect security from the United States in NATO)—the likelihood of Moscow deploying force against
Poland, Lithuania or even the Ukraine is close to zero. In other parts of its former empire, too,
Russia is far from belligerent. Although vying for influence there with nearby greater powers, Russia has refrained
from policing Central Asia in Soviet style despite the alluring natural resources that the region possesses. When
Kyrgyzstan asked for a Russian troop presence last year to quell political unrest, the Kremlin balked at the request. It had no
desire to become entangled in the internal power struggles of its former client state. Russian cultural and
political influence pervades especially in the northernmost of former socialist republics in Central Asia but Chinese, Iranian and
Turkish attempts at fostering stable relations in the area could set the stage for a greater power confrontation, one from which
Moscow stands nothing to gain. Russian governors in the Far East occasionally raise the specter of the “yellow menace” and talk
of the danger posed to their underpopulated provinces by unregulated Chinese labor migrants but as Dmitry Gorenburg pointed
out here last year, “this kind of talk rarely emanates from Moscow and certainly does not affect troop positioning.” Indeed, “ it
is stunning how little trouble Moscow has fomented ” since the demise of the Soviet Union, writes Barnett,
“all while engineering arguably the greatest military demobilization in human history, going from more than two
hundred army divisions to less than one hundred brigades.”

Zero risk of Russian expansionism

Trenin ‘11 – director of the Carnegie Moscow Center, chairs the Research Council and the Foreign and Security Policy Program,
former member of the Russian armed forces (Dmitri, 10/17. “RIP Russian Empire.” http://carnegieendowment.org/2011/10/17/rip-
russian-empire/5z44)

There will be no return to the Russian Empire. The Russian Empire is dead, never to return . It’s a
museum. It’s history. To some people, it’s a source of glory, like people in the United Kingdom who think about the glorious
days of the British Empire. There are Russians who recall their empiric past with a lot of pride. And indeed, empires historically have
contributed to human advancement in various ways. Of course they were oppressive creatures and in the long
run unsustainable. But that’s a different story. So there is no return. There is no will and there are no
resources. The world has turned far from where it was when the Soviet Empire, which is the
historical empire of Russia, collapsed twenty years ago. The interesting thing though, if you listen to what a lot
of people in Russia and in neighboring countries—from Tallinn to Tbilisi—are saying, they are still talking as if the empire were there
or about to be resurrected. For some people, I think this is what they actually feel. For other people, this is something that they
believe could be useful. But my judgment is clear. There’s no Russian Empire and it’s not coming back.

Russia’s too old to be any threat

Fairbanks and Hewitt ‘8 – Former US Ambassador, GAI Director @ CSIS (Richard Fairbanks and Paul S. Hewitt, 9/17.
Washington Times, “Demography vs an Imperial Impulse,” Lexis)

fears of a resurgent post-Soviet imperialism. But such


Russia's incursion into Georgia understandably evokes Cold War-era
concerns overlook a fundamental constraint: Russia is fast running out of young men . Between 2010
and 2025, Russia's pool of potential military recruits , aged 20-29, will decline by 44 percent , according to
the United Nations. This forecast is not subject to meaningful revision; it has been "written in stone" by
births that have already occurred. A collapsing supply of new workers will create not just military, but
economic headwinds. All things being equal, Russia's shrinking work force will subtract about 1 percent
a year from potential economic growth over the next two decades. The loss of the technical acumen and
dynamism that young people bring to modern economies points to even greater damage . It is a
safe bet, for example, that Russians will claim fewer patents, start fewer businesses and produce fewer artistic achievements than
could otherwise be expected. In this regard, Russia has joined Japan and much of Europe on the path of demographically driven
decline, characterized by shrinking economies of scale and falling competencies across a range of disciplines and industries. The
grinding labor shortages on Russia's horizon are a far cry from the vast population surpluses of the early Soviet period. In 1917, the
typical Russian was just 15 years old. The population had nearly doubled during the preceding generation. Exploding populations are
notorious breeding grounds of political and social strife. On any given day, a preponderance of the world's civil conflicts play out in
ultra-youthful countries - think Chad, Haiti, Nepal, Iraq - where the typical citizen is in his teens or early 20s. Unemployed young
men, with their perennial disconnect between challenge and opportunity, are especially prone to violence. The early- and mid-20th
century saw governments respond to this internal threat through regimentation that, all too often, cast surplus youth into wars and
civil conflicts marked by horrific casualty rates. World War I produced an estimated 15 million war deaths. The Soviet Union lost as
middle-aged
many as 60 million in Josef Stalin's Great Terror of the 1930s and the Second World War that followed. Yet
countries, like present-day Russia, where the median age is 37, generally eschew militarism. Diminished
labor market pressures and a citizenry that is invested in the social order provide an important
impetus to peace. So too does a greater level of adult supervision. These peaceful proclivities are reinforced by
fiscal considerations. Older societies would rather spend money on pensions than military adventures.
Such commitments put a premium on employing youth as productively as possible. Although middle-aged societies are a relatively
modern phenomenon, there are a handful of cases where conflicts have been initiated by older nations. With a median age of 32,
Germany had the world's oldest population when it started World War II. The typical Serb was 34 when Serbia embarked on its
ethnic cleansing of Bosnia. In these cases, authoritarian regimes sought to bolster their leadership by subduing or incorporating
neighbors under the pretext of righting a historical wrong. The Russo-Georgian dustup loosely fits this pattern. America's invasion of
Iraq, at the ripe median age of 35, had nobler motives. Yet, significantly, this campaign's widely publicized stumbles can be traced
directly to a lack of manpower. If the Pentagon ever contemplated putting boots on the ground in neighboring Iran, it apparently
doesn't do so now. In Russia's case, the economic, fiscal and manpower strains imposed by depopulation make its current foreign
policy costly indeed. The Commission on Global Aging - whose deliberations we oversaw from 1998 to 2001 - concluded that the
industrial world's best hope for growth lay in greater economic integration. Only by outsourcing labor-intensive work and
specializing in high-value activities can aging, depopulating societies capture the dynamism needed to sustain their economies. In
light of these imperatives, it's hard to imagine a more self-defeating stance than that of Russian Prime Minister Vladimir Putin, who
has declared, "We don't see or feel advantages from the membership [in the World Trade Organization], if they exist at all." By
shunning trade and embracing a wasteful militarism, Russia is on the verge of a strategic blunder that its diminished future
generations will surely regret. The miscalculations of earlier strongmen underscore the clear and present danger of Russia's
renewed expansionism. The sustainability of this course is another matter.
No return to Soviet-style expansionism---Russia’s leaders are coolheaded realists, not
ideologues

Barnett ‘8 (8/21. Correlli Barnett, Daily Mail London, “World Peace? Give Me a Putin Anyday!” Lexis)

SINCE the Russian Army routed the Georgian incursion into South Ossetia last week, Western politicians
and media have been hotly accusing the Russia of President Medvedev and Prime Minister Putin, of Soviet-
style expansionism by means of bullying small neighbours like Georgia. In windy rhetoric, U. S. Secretary of
State Condoleezza Rice and Britain's Foreign Secretary David Miliband pronounce that this alleged strategy
would, if continued, pose a threat to world harmony. It reminds me all too vividly of the way George W. Bush
and Tony Blair worked up war fever against Iraq in 2002 -2003, entirely on the false basis (as some of us
suspected at the time) that Saddam Hussein had weapons of mass destruction ready to use against us. Five years on, we still
have to live with the poisonous political and military residue of that particular bout of war fever ,
including the human grief on a vast scale. Back in 2002-2003, I warned in this newspaper that an attack on Iraq would result in a
protracted guerilla war bringing catastrophe on Iraqis and invaders alike. So it has proved. Today, I find Russia under Putin and
Medvedev much less alarming than America under George W. Bush in his belligerent prime. And I also find Putin and Medvedev far
less of a potential menace to world peace than today's Washington hawks, who even now are acting as intellectual seconds to John
McCain in his big fight for the presidency, and who could — heaven forfend — shape American policy if McCain wins. Hawks like
the luxuriantly mustachioed John Bolton, former U.S. ambassador to the UN, whose idea of foreign policy is to shout that he can
beat any man in the global saloon, and who now berates Condoleezza Rice for relying on diplomacy instead of picking a fight with
Putin over the Georgian crisis.. And not forgetting hawks like the ever-so-clever academic theorists in right-wing Washington think-
tanks. Exploitation These hawks proclaim that by militarily humiliating the Georgian Prime Minister ,
Mikhail Saakashvili, Russia has begun a new 'Cold War'. It's time again, we are told, to defend
'freedom and democracy' against aggression. All this is hysterical nonsense. And why? Because Prime
Minister Putin has no ideological motivation to remodel the world, unlike the Marxist leaders of
the Soviet Union or the fervently Christian George W. Bush and Tony Blair. Instead, Putin is just an oldfashioned
Russian autocrat and nationalist of the kind that would have been familiar enough to Bismarck, Palmerston and Disraeli.
Even the trappings of the current Russian regime , such as the state coat of arms or the 19th-century uniforms of
Kremlin guards of honour, are pure Czarist revival. You could say the same for the widespread business corruption and the
suppression of independent opinion. So the paramount concern of Putin and his tame President, Medvedev, is
to restore Russia's position and prestige as a great power in a world setting. They do not buy
into the fashionable Western concept of 'globalisation' , where great foreign companies are free to acquire a
nation's major industrial or energy assets. This is why BP is being gradually edged out of the exploitation of Russian oil reserves. Not
for modern Russia the British pattern whereby the electricity supply industry is largely in the hands of a German and a French
Russia's leaders are,
company. Or whereby future British nuclear plants will be built and run by French-owned EDF.
therefore,
cool-headed (and possibly coldhearted) calculators of Russia's advantage — men whose outlook
is inspired by strategy rather than by idealism or moral purpose. Hence, the Russian flag placed by
submarine on the bottom of the sea below the North Pole, so registering in the most public way Russia's claim to whatever mineral
bounty lies below the ice. Cunning These are leaders who want Russia to profit politically as well as
economically from her immense wealth in oil and gas, and will ruthlessly use the leverage which these resources give her over
all the states dependent on her pipelines. But none of this amounts to a new ideological 'Cold War', but
only to a very old-fashioned great-power rivalry pursued with cunning and caution. For instance, there is no
similarity between the present Russian action in Georgia and the Soviet invasions of Hungary in
1956 and Czechoslovakia in 1968. In both those cases, the Soviet leadership ordered the Red Army to
invade in order 'to defend Socialism'. The mind of the men in today's Kremlin is a world away
from such doctrinaire thinking — just as it is from the ideology heated emotion and belief in a great moral cause which
consumes the hawks of Washington. Just compare the short-lived and limited Russian military intervention in Georgia to secure
Russia's own border region with George W. Bush's long-distance invasion of the sovereign state of Iraq in the hope of democratising
the Middle East and then the world. Then again, Western commentators condemn Russia for making threats to Poland and the
Czech Republic because they agreed to accept American 'Star Wars' anti-missile systems on their soil. But imagine what would be
the reaction of the Washington hawks to news that Mexico had accepted a Russian anti-missile system just south of the American
border! And how would they react if Russia's strategic frontier had advanced as far west in Europe since the end of the Iron Curtain
as Nato's strategic frontier has advanced eastwards? Or for that matter, what would be the reaction of the Washington hawks (and
their acolytes in the British media) if Russia sought to recruit Mexico to a Russian military alliance — as NATO certainly intends to
recruit Georgia? I personally am only too grateful that Putin & Co. do not go in for the moralising common to Anglo-Saxon leaders
— — even those in opposition like David Cameron, who has been proclaiming after visiting Tbilisi that we must all stand by that
absurd posturer and blunderer, Saakashvili, Prime Minister of Georgia. But then, Cameron is not so much 'a liberal Conservative' (in
his own words) as simply a liberal, and a liberal in the mode of Gladstone. Crisis I dread what the foreign policy of a Conservative
government might be — what fresh far-off entanglements Britain and her Armed Forces could be involved in. So what is the final
lesson I draw from the Georgia crisis? First, that it
is pointless to denounce the actions of Putin and
Medvedev when NATO has no military means of intervenin g — neither forces in the region, nor available
reserves to fly in. Strategic reality trumps pious sermons every time . And the second and far more important
lesson: that we in the West should jettison moral indignation and global dogoodery as the basis of
policy, and instead emulate Russia's admirable reversion to 19thcentury realpolitik . Henceforth, we
should base our foreign and economic policy on hard-nosed pursuit of our own national interest.

Claims of expansionism are overly simplistic and ignore complex dynamics shaping Russian
policy

Shearman and Sussex ‘9 (Peter, CERC Fellow – U. Melbourne, Faculty of Arts and Sciences – Webster U., and Matthew,
Senior Lecturer in School of Gov. – U. Tasmania, Small Wars & Insurgencies, “The roots of Russian conduct”, 20:2, June)

Yet much of this takes at face value the inevitability of a 'Cold Peace' between Russia and the
West, and it also makes the mistaken assumption that the Kremlin is pursuing a neo-imperial
grand strategy, viewing its policies solely through the prism of great power politics. It is certainly important that we understand
Russia's choice to fight its short war with Georgia, but the immediate interpretations of the conflict from the Western media and
many specialist accounts have been remarkable for their lack of analytical depth.` Officialreactions in some countries,
especially the United States and the United Kingdom, leapt to Tbilisi's defense before the real facts were
known. One commentator claimed that Russia was reverting to 'unrelenting aggressive unilateralism', while others have stated
baldly that Russia was returning to the practices of the Cold War. 5 We demonstrate that both of these interpretations are
incorrect. The first fails to acknowledge that Russia was actually reacting to aggression in the first
place, while the second is fundamentally flawed for misunderstanding the nature of the Cold War. The idea that
Moscow's approach can be defined simply as aggressively unilateral does not take into account
the complex local, regional and global security dynamics shaping Russian foreign policy . In the
twenty-first century the main theatre of disagreement between the West and Russia has shifted eastwards to the Caucasus as the
West has itself expanded to fill the vacuum left by the collapse of the USSR. However, the main drivers of great power tension are no
longer ideology or global bipolar rivalry, but more localized geostrategic factors linked mainly to energy security. And while other
contributions in this collection assess the question of a new Cold War, it is worth noting that rather than merely reflecting a period in
which mutual antagonisms were played out, the Cold War in fact became highly structured and ordered over time. The Helsinki
Accords, which grew out of dialogues in the Conference on Security and Cooperation in Europe (the original brainchild of Leonid
Brezhnev), created three 'baskets' of agreements in the military, economic and social realms. The Third Basket essentially made
progress on security cooperation dependent upon ensuring adherence to fundamental human rights. Brezhnev was willing to sign on
to the Accords even though it left the Warsaw Pact open to pressures on human rights issues, because the US and the 'West' were
recognizing the Soviet Union's legitimate rights in its nearest neighborhood. In other words, the West recognized that the Soviet
Union had privileged interests in its own sphere of influence.
No impact—incentives for cooperation overwhelm confrontation.

Markedonov ‘9 (Sergei, Heads the Dept. of Interethnic Problems – Institute of Political and Military Analysis (Moscow),
Russian Politics and Law, “The ‘Five-Dya War’ Preliminary Results and Consequences” 47:3, May-June)

Paradoxical as it may sound, the NATO bloc took much more constructive and cautious approaches (than
the United States) toward Russia. Recent events have shown that we should not identify the North Atlantic alliance with the
United States. All the declarations made by NATO Secretary-General Jaap de Hoop Scheffer and NATO spokesman James Appathurai
were much more politically correct than the arguments prepared by representatives of the U.S. State Department. For the sake of
comparison, I provide here just two examples. During a visit to Tbilisi on 16 September, the NATO secretary-general
declared that it was not part of his organization’s brief “to judge Russia.” At the same time, Matthew Bryza,
U.S. deputy assistant secretary of state [for European and Eurasian affairs] (the desk officer in charge of the current administration’s
Caucasus policy) proposed to stop cooperating with Russia regarding the Karabakh settlement through the OSCE Minsk Group until
the Medvedev–Sarkozy plan was completely implemented. Meanwhile, the Russian propaganda machine, ignoring the positive
messages from NATO (for instance, its position on Afghanistan), identified the position of the entire bloc (far from pro-Russian but
not so unambiguous as the American approach) with the views held by U.S. leaders. In general, in August–September 2008 Russian
diplomats and politicians, instead of focusing on “dividing the West,” became carried away by demonizing it, which objectively
helped Mikheil Saakashvili by distracting the attention of European politicians from the aggressive ambitions of the Georgian
president. In any case, the conventional “West” (personified by various countries, blocs, and structures) is not ready for a new “cold
war” against Russia. Moreover, if the South Caucasus is recognized as a zone of Moscow’s “special interests” (motivated, above all,
by the security problems in the Russian South), our country would reduce its anti-Westernism, which is currently in demand.
Today, a new “cold war” is not possible. That is another result of “hot August.” B etween the
Russian Federation and the West, there are no ideological differences: Moscow was not
exporting socialism to South Ossetia and Abkhazia and was not defending any body’s dynastic
interests there. Of course, there are essential differences in the interpretation of national interests,
and some stereotypes and phobias of the past still persist. There are, however, much more serious
challenges than these: the situation in Afghanistan and Central Asia, the problems of Iran and North Korea,
energy, and international terrorism, which require joint efforts and in principle cannot be resolved without mutual participation. All
this gives us hopes, albeit weak, that a search for general rules governing the world order will
soon begin.
---Russia Aggression- Too Weak

Russia cant pursue expansionist policies – internal weakness

Cohen ‘7 [Ariel, Ph.D., Senior Research Fellow in Russian and Eurasian Studies and International Energy Security in the Douglas
and Sarah Allison Center for Foreign Policy Studies. November 19, 2007. “Domestic Factors Driving Russia's Foreign
Policy”. http://www.heritage.org/Research/RussiaandEurasia/bg2084.cfm]

Russia's foreign policy assertiveness, funded by revenues from natural resources, makes many believe that a
newenergy empire is on the rise . The country today is ruled by post-Soviet security and military elites that have internalized
the jingoistic values of the Russian Empire and the Soviet Union. These elites view the outside world almostexclusively through the
lens of economic and military might. They also use foreign policy as a tool to buttress domestic support and to foster a perception
that Russia is surrounded by enemies at a time when its democratic legitimacy is deteriorating. Despite its projected might, the
Kremlin is not capable of dealing with some of Russia's critical demographic, social, economic, and political vulnerabilities. These
flaws may well challenge the current sense of stability in Russia, especially after the 2007-2008 election cycle or if the economy
deteriorates. As the proverb states, "Russia is never as strong as she appears, and never as weak as she appears."[1] Russian
President Vladimir Putin modified this proverb in a May 2002 speech: "Russia was never as strong as it wanted to be and never as
weak as it was thought to be."[2]Russia's strengths made the authorities and the public believe that their country is still a great
power, yet Russia's
many weaknesses limit its ability to act as one. Continuing state weakness
combined with an increasingly bold foreign policy is a recipe for imperial overreach and systemic
breakdown.[3]

Declining population prevents Russian expansion

Cohen ‘7 [Arie, Ph.D., Senior Research Fellow in Russian and Eurasian Studies and International Energy Security in the Douglas
and Sarah Allison Center for Foreign Policy Studies. November 19, 2007. “Domestic Factors Driving Russia's Foreign
Policy”.http://www.heritage.org/Research/RussiaandEurasia/bg2084.cfm]

Demographic Catastrophe The
great-power ambitions of Moscow's current elites cannot be realized
without ample, developed, and highly skilled human resources . Since the 1980s, however, Russia has
experienced dramatic declines in population, fertility, and life expectancy combined with
increases in mortality and disease rates, including a rise in the rates of HIV/AIDS and
tuberculosis infection. From 1995 to mid-2007, Russia's total population dropped by 6.5 million people, down to 142
million--a decline of almost 4.4 percent.[6] Such a drop typically is the result of war or mass emigration, but it is occurring in a largely
peaceful Russia that has a growing economy and positive immigration rate. Russia's population is the world's ninth-largest but is
projected to drop to 128.5 million by 2025 and 109.4 million by 2050.[7] Because of the low birthrate and the high mortality
rate, Russia is losing an average of 700,000 peo ple per year. In 2006, the mortality rate was 15.2 deaths per
1,000 people, and the birthrate was just 10.4 births per 1,000 people. While the birthrate is low compared to other industrial states,
the death rate, particularly among working-age males, is astonishing .
Life expectancy for Russian males is only 59
years, five years below what it was 40 years ago and 13 years lower than the life expectancy
of Russian women--one of the largest gaps in the world.[8] The current solution of stimulating births by paying
over $4,000 per baby may create a hereditary welfare problem where there now is none and encourage growth among both Russia's
Muslim population and its urban and rural poor. 
---Russia Aggression- Shared Interests Check

Give a Russia war impact zero probability

Graham ‘7 (Thomas Graham, senior advisor on Russia in the US National Security Council staff 2002-2007, September 2007,
"Russia in Global Affairs” July - September 2007, The Dialectics of Strength and Weakness

An astute historian of Russia, Martin Malia, wrote several years ago that “Russia
has at different times been
demonized or divinized by Western opinion less because of her real role in Europe than because of the fears and
frustrations, or hopes and aspirations, generated within European society by its own domestic problems.”
Such is the case today. To be sure, mounting Western concerns about Russia are a consequence of Russian
policies that appear to undermine Western interests, but they are also a reflection of declining confidence in
our own abilities and the efficacy of our own policies. Ironically, this growing fear and distrust of Russia
come at a time when Russia is arguably less threatening to the West, and the United States in particular,
than it has been at any time since the end of the Second World War. Russia does not
champion a totalitarian ideology intent on our destruction, its military poses no threat to sweep across
Europe, its economic growth depends on constructive commercial relations with Europe, and its
strategic arsenal – while still capable of annihilating the United States – is under more reliable control than
it has been in the past fifteen years and the threat of a strategic strike approaches zero
probability. Political gridlock in key Western countries, however, precludes the creativity, risk-taking, and subtlety needed
to advance our interests on issues over which we are at odds with Russia while laying the basis for more constructive long-term
relations with Russia.

No risk of a revisionist Russia

ARON ‘6, Leon Aron, resident scholar and director of the Russian Studies at the American Enterprise Institute, 6/29/2006,
Ideologies, Policies, and Relations, p. www.aei.org/publications/pubID.24606/pub_detail.asp

Yet the probability of a frontal confrontation and a new Cold War remains very remote for at least
three reasons. First, despite the erosion, the countries’ geopolitical assets are still very weighty, as
the bedrock issues of anti-terrorism, nuclear nonproliferation, and energy will continue to force
them to seek common ground and at least limited partnership.[17] Second, the “restorationist” foreign policy
notwithstanding, the three basic elements of the 1992-1993 national consensus on the foreign policy and defense doctrine remain
largely the same. Russia
is to stay a nuclear superpower and the regional superpower, but it seems to
have settled for the role of one of the world’s great states , rather than a global superpower
engaged in a worldwide competition with the United States. While these desiderata will continue to cause occasional sparring with
the United States, theyare no longer dedicated to the attainment of goals inimical to the vital
interests of the U nited S tates and are not likely to ignite a relentless antagonistic struggle to the
bitter end. Lastly, despite the muscular rhetoric emanating of late from the Kremlin, unlike the Soviet Union twenty years ago
and China today, Russia is not a “revisionist” power. It does not seek radically to reshape the
geopolitical “balance of forces” in its favor. Moscow may rail at the score, but it is unlikely to endeavor to change the
rules of the game. For that, one needs a different ideology and, as a result, a different set of priorities. Yet even in today’s Russia
flush with petrodollars, the share of GDP devoted to defense (around 3 percent) is not only at least ten times smaller than in the
Soviet Union, but also below the 1992-1997 average in a Russia that inherited an empty treasury from the Soviet Union and that
was, like every revolutionary government, unable to collect taxes. Calculated in purchasing power parity, Russia’s defense
expenditures in 2005 ($47.77 billion) were less than one-eleventh of what the U.S. spent ($522 billion).[18]
Russia China War Answers
Frontline

China won't invade the Russian Far East.

Harding, ‘9 [Luke, Writer for the Observer, "Russia fears embrace of giant eastern neighbour," 8/2,
http://www.guardian.co.uk/world/2009/aug/02/china-russia-relationship]

Most experts believe China's own strategic goals do not include Russia's far east , or primitive
territorial expansion. Instead Beijing's priorities lie elsewhere. They include development, reunification
with Taiwan and internal stability , which experts suggest is more of a priority than ever
following last month's ethnic riots against Han Chinese in Xinjiang. According to Dr Bobo Lo, a lecturer on
Chinese-Russian relations at the Centre for European Reform, Beijing's real challenge to Moscow is rather different. He argues that
the rise of China will lead to the "steady marginalisation of Russia from regional and global decision-making". The Chinese do
not want to invade Russia militarily because , he points out, they would lose.

Shared interests solve

Weitz ‘11 (Richard, Director, Center for Political-Military Analysis Senior Fellow Hudson Institute, PhD in pol sci from Harvard,
China-Russia relations and the United States: At a turning point?, http://en.rian.ru/valdai_op/20110414/163523421.html, 2011)

Since the end of the Cold War, the


improved political and economic relationship between Beijing
and Moscow has affected a range of international security issues. China and Russia have expanded their
bilateral economic and security cooperation. In addition, they have pursued distinct, yet parallel,
policies regarding many global and regional issues. Yet, Chinese and Russian approaches to a range of significant
subjects are still largely uncoordinated and at times in conflict. Economic exchanges between China and Russia remain minimal
compared to those found between most friendly countries, let alone allies. Although stronger Chinese-Russian ties could
present greater challenges to other countries (e.g., the establishment of a Moscow-Beijing condominium over Central Asia),
several factors make it unlikely that the two countries will form such a bloc. The relationship between the Chinese and Russian
governments is perhaps the best it has ever been. Theleaders of both countries engage in numerous
high-level exchanges, make many mutually supportive statements, and manifest other
displays of Russian-Chinese cooperation in what both governments refer to as their developing
strategic partnership. The current benign situation is due less to common values and shared interests than to the fact that
Chinese and Russian security concerns are predominately directed elsewhere. Although both countries have experienced a
geopolitical resurgence during the past two decades, Chinese and Russian security concerns are not directed at each other but
rather focus on different areas and issues, with the notable exceptions of maintaining stability in Central Asia and constraining
North Korea’s nuclear activities. Most Chinese policy makers worry about the rise of separatist movements and Islamist
terrorism in western China and about a potential military clash with the United States in the Asia-Pacific region, especially
regarding Taiwan and the contested maritime regions of the South China and East China Seas. In contrast, most Russian analysts
see terrorism in the North Caucasus, maintaining influence in Europe, and managing security relations with Washington as the
main security challenges to their country. Neither
Chinese nor Russian military experts perceive a near-term
military threat from the other’s country. The Russian government has even provided sophisticated navy, air,
and air defense platforms to the Chinese military , confident that the People’s Liberation Army (PLA) would only
employ these systems, if at all, against other countries. In addition, China
and Russia have resolved their
longstanding border disputes as well as contained their rivalries in Central Asia, the Korean
Peninsula, and other regions. Since the Soviet Union’s disintegration in the early 1990s, China and
Russia have resolved important sources of their Cold War-era tensions. Through protracted
negotiations, the two governments have largely solved their boundary disputes, which had erupted in
armed border clashes in the late 1960s and early 1970s. The stoking of anti-Chinese sentiment by politicians in the Russian Far
East impeded the ability of Russia’s first President, Boris Yeltsin, to make substantial progress during the 1990s in demarcating
the Russia-China border. These politicians sought to rally local support by accusing Moscow of planning to surrender territory to
Beijing. By the mid-2000s, Yeltsin’s successor, Vladimir Putin, managed to centralize sufficient political power in the Kremlin to
ignore these local sentiments. Furthermore, Russia and China have demilitarized
their lengthy shared frontier
through a series of arms control and disarmament measures. Chinese and Russian leaders share a
commitment to a philosophy of state sovereignty (non-interference) and territorial integrity (against separatism). Although
Russian and Chinese leaders defend national sovereignty by appealing to international law, their opposition also reflects more
pragmatic considerations---a shared desire to shield their human rights and civil liberties practices, and those of their allies,
from Western criticism. Chinese and Russian officials refuse to criticize each other’s foreign and domestic policies in public. They
also have issued many joint statements calling for a multi-polar world in which no one country (e.g., the United States)
dominates. During the past few years, their leaders have commonly blamed American economic mismanagement for
precipitating the global recession. They regularly advocate traditional interpretations of national sovereignty that exempt a
government’s internal policies from foreign criticism. Beijing and Moscow oppose American democracy promotion efforts, U.S.
missile defense programs, and Washington’s alleged plans to militarize outer space. The two countries strive to uphold the
authority of the United Nations, where the Chinese and Russian delegations frequently collaborate to dilute resolutions seeking
to impose sanctions on Burma, Iran, Zimbabwe, and other governments they consider friendly. In July 2008, they finally
demarcated the last pieces of their 4,300-km (2,700 mile) frontier, one of the world’s longest land borders, ending a decades-
long dispute. Chinese and Russian officials have expressed concern about the efforts by the United States and its allies to
strengthen their ballistic missile defense (BMD) capabilities. Their professed fear is that these strategic defense systems, in
combination with the strong American offensive nuclear capabilities, might enable the United States to obtain nuclear
superiority over China and Russia. Both governments have also expressed unease regarding U.S. military programs in the realm
of outer space. Russian and Chinese experts claim that the United States is seeking to acquire the means to orchestrate attacks
in space against Russian and Chinese reconnaissance satellites and long-range ballistic missiles, whose trajectories passes
through the upper atmosphere. In response, the Russian and Chinese governments have proposed various arms control
initiatives purportedly aimed at preventing the militarization of space. For example, the Russian and Chinese representatives
have unsuccessfully sought for years at the UN Conference on Disarmament to negotiate a treaty on the “Prevention of an Arms
Race in Outer Space,” which would seek to prohibit the militarization of outer space. More recently, China and Russia have
submitted a joint Space Treaty to the Conference on Disarmament in Geneva, which would impose legal constraints on how the
United States could use outer space. They have sought to link progress on other international arms control initiatives to the
adoption of these space limitations. The bilateral defense relationship has evolved in recent years to become more
institutionalized and better integrated. As befits two large and powerful neighbors, the senior military leaders of Russia and
China now meet frequently in various formats. Their direct encounters include annual meetings of their defense ministers and
their armed forces chiefs of staff. Since 1997, they have also organized yearly “strategic consultations” between their deputy
chiefs of the general staff. In March 2008, the Chinese defense minister established a direct telephone line with his Russian
counterpart, the first suchministerial hotline ever created by China and another country. In December 2008, the chiefs of the
Chinese and Russian general staffs created their own direct link. Senior Russian and Chinese defense officials also typically
participate in the regular heads of government meetings between Russia and China, which occur about once a year as bilateral
summits. They also confer frequently at sessions of multinational gatherings, such as at meetings of the SCO, which host regular
sessions for defense ministers. Contacts are even more common among mid-level military officers, especially those in charge of
border security units and military units in neighboring Chinese and Russian territories. Russian and Chinese military experts also
engage in regular direct discussions related to their functional expertise such as communications, engineering, and mapping.
Substantial academic exchanges also regularly occur. More than 1,000 Chinese students have studied at over 20 Russian military
academies since 1996. The two defense communities conduct a number of larger exchanges and engagements. The best known
are the major biennial military exercises that they have been holding since 2005, but smaller-scale engagements also frequently
occur. Chinese and Russian leaders also have developed shared perspectives and independent offensive capabilities regarding
governmental activities in the cyber domain. The two governments have been developing their information warfare capabilities
and now possess an extensive variety of offensive and defensive tools in this domain. Furthermore, recent revelations regarding
Chinese cyber-espionage activities suggest the extent to which Chinese operatives have penetrated Western information
networks. In Russia’s case, cyber attacks against Estonia, Georgia, and other countries illustrate the extensive offensive
capabilities available to that country’s forces. Russia’s hybrid August 2008 campaign against Georgia was particularly effective in
disabling Georgia’s infrastructure as well as demonstrating a potential capacity to inflict widespread physical damage. Both
countries appear to have already conducted extensive surveying of U.S. digital vulnerabilities and to have prepared targeted
campaign plans to exploit U.S. network vulnerabilities if necessary. Although these offensive and defensive preparations are
being conducted independently, the Chinese and Russian governments are collaborating, along with other Eurasian allies in the
SCO, to deny Internet resources to civil liberties groups and other opponents of their regimes. Central Asia perhaps represents
the geographic region where the security interests of China and Russia most overlap. Although
China and Russia
often compete for Central Asian energy supplies and commercial opportunities, the two governments
share a desire to limit potential instability in the region. They especially fear ethnic separatism in their
border territories supported by Islamic fundamentalist movements in Central Asia. Russian authorities dread the prospect of
continued instability in the northern Caucasus, especially Chechnya and neighboring Dagestan. China’s leaders worry about
separatist agitation in the Xinjiang Uighur Autonomous Region. The shared regional security interests between
Beijing and Moscow have meant that the newly independent states of Central Asia---Kazakhstan, Kyrgyzstan, Tajikistan,
Turkmenistan, and Uzbekistan---have become a generally unifying element in Chinese-Russian
relations. Their overlapping security interests in Central Asia have manifested themselves most
visibly in the Shanghai Cooperation Organization (SCO).

No war -- mutual interests ensure cooperation.

Hille and Anderlini ‘12 (Financial Times correspondents in Beijing (Kathrin and Jamil, “Russia and China to strengthen
trade ties,” Financial Times, http://www.ft.com/cms/s/0/d8999462-af27-11e1-a8a7-00144feabdc0.html#axzz1zrBVCIMo, June 5,
2012)

Russia and China laid out ambitious plans on Tuesday to tie their countries into a
The presidents of
closer strategic and economic partnership as both Beijing and Moscow seek to use each other to
balance their relationship with the US. The two presidents set a goal of more than doubling
bilateral trade from $83.5bn last year to $200bn in 2020, Hu Jintao, China’s president, said after talks with Vladimir Putin, the
Russian president. The ambitious target was announced together with a slew of investment and trading deals. It was held up by the
two leaders as a sign of new heights for the relationship between two powers to be reckoned with on issues ranging from global
trade to the international response to Syria. Mr Hu said that through closer
co-operation China and Russia would
“set the global political and economic order in a more fair and rational direction ”. Chinese state
media feted Mr Putin, a fairly frequent visitor to Beijing who had last been in town in October 2011, praising
close and growing bilateral co-operation and consultation of the two nations on the world
stage. People’s Daily, the ruling Communist party’s mouthpiece, even ran a long piece by Mr Putin himself in which he talked up
ties with China. “Without the participation of Russia and China, without considering Russia and China’s interests, no international
matter or issue can be discussed and implemented,” he wrote. Moscow and Beijing have angered western countries with their
refusal to back an international intervention in the brewing civil war in Syria. But Russian and Chinese analysts say the two countries
are still far away from a close alliance. “The
reason the Chinese media are hyping the visit like this is that
a significant portion of the Chinese leadership, including the military . . . hope to build Russia
into an ally to help push back against the US,” said Dmitri Trenin, director of the Carnegie Moscow Center.
“But for Mr Putin the focus is mainly on strengthening the economic relationship .” And, he cautioned,
historical suspicions lingered. “There remains a lack of trust between the two sides, and none of the two wants a true alliance.”

Negotiations solve

Chicago Tribune 4 (10-15, Lexis)


China and Russia settled the last of their decades-old border disputes Thursday during a visit to Beijing by
President Vladimir Putin, signing an agreement fixing their 2,700-mile-long border for the first time .
The struggle over border areas resulted in violent clashes in the 1960s and 1970s, when
strained Sino-Soviet relations were at their most acrimonious, feeding fears abroad that the conflict could erupt
into nuclear war. Beijing and Moscow had reached agreements on individual border sections as relations warmed in the
past decade. But a stretch of river and islands along China's northeastern border with Russia's Far East had remained in dispute.

Fear of escalation checks

Moriarty 4 (Tom, Military Intelligence Analyst – U.S. Air Force, World Affairs, 9-22, Lexis)

However, the Soviet Union ultimately chose to forgo a preemptive attack and attempted to defuse tensions
through diplomatic channels. Numerous reasons led Soviet leaders to decide against preemptive attack. The main reason
was the Soviet Union's fear that even if they could destroy all of China's nuclear weapons
capability (which, in itself, was a big assumption), they feared a conventional attack by China. Like the United
States during the Cuban missile crisis, the Soviet Union understood that they would lose the ability to prevent the crisis from
escalating into a full-blown war. Soviet leaders grew concerned that China would respond with a
prolonged people's war against the Soviet Union. Knowing that a prolonged war against a country with
more than one billion people and a proven resiliency would exhaust the Soviet Union and would require forces to
be withdrawn from Eastern Europe, Soviet leaders chose to ignore the Chinese provocations and let the
confrontation defuse naturally. (12)

China will not invade Russia – they would lose

Open Democracy 8 (“Russia-China: Axis of Convenience”, http://www.cer.org.uk/articles/lo_opendemocracy_20may08.html)

The real threat is this: China's rise will lead to Russia's steady marginalisation from regional and global decision making. The
Chinese do not intend to invade Russia militarily, because they would lose . The
consequences would be too horrific to be contemplated . They are not going to fill the Russian Far East
with lots of Chinese. Those northern regions have always been considered a barbarian outland. The Chinese who go
there want to make a quick buck before returning home . Although they receive half the salary of local
Russians, it is still much more than they would get in northeastern China or in the countryside. But very few Chinese go to the
Russian Far East to live there.

China isn’t stupid enough to attack Russia

Menon ‘3 (Rajan, Rathbone Professor of International Relations at Lehigh University, The


National Interest, Fall)
By contrast, China's military, which was quite recently a giant horde of foot soldiers, is modernizing steadily-chiefly
with Russian weaponry, much of it supplied from cash-starved military industries in Khabarovsk, Komsomol'sk and
Vladivostok. It may lag far behind the United States, but in force projection, speed, accuracy and lethality it is a wholly
different force than it was a decade ago, thanks to Russian fighter jets , submarines, tanks and
missiles, many of them built in the Russian Far East. Yet the chances that China will attempt to conquer
Russia's Far East are slim. Such a brazen power play would damage China's wider interests . Taiwan
might recoil in terror and treat Beijing's proposals for a negotiated reunification with even greater skepticism and wariness. The
prevailing Western rationale for economic engagement with China-that commerce will
transform and co-opt that country-would be shredded. China would likely face a
counterbalancing, encircling coalition of the United States, India, Japan, Russia and Vietnam .
Would such setbacks justify the burdens of ruling the vast, problem-infested Russian Far East? The Chinese leaders know their Sun
Tzu: what they seek from the Russian Far East (access to resources and a benign northern front) can be had by means of silk-gloved
hegemony. Chinese interests can be served without its formal occupation of the territory . Indeed,
what may emerge could be a "reverse Manchurian" scenario, where the Russian Far East remains a titular part of Russia but is
increasingly integrated into Beijing's sphere of influence. That is precisely what the conspiracy among geography, demography,
power and time may create in Russia's Far East.
Russia Gas Leverage Answers
Russia gas leverage gone – Gazprom can’t compete

Schulz and Bidder ‘13 (Stefan and Benjamin, Russia reporters, “Under Pressure: Once Mighty Gazprom Loses Its
Clout,” – Der Spiegel, Spiegel English Online, February 1, 2013)

The principle at stake is "take or pay." According to a long-standing clause in Gazprom's supply agreement, customers are obligated
to accept a contractually-agreed minimum quantity of natural gas, and even if the customer uses less, Moscow gets paid the full
sum. It's a common practice in the energy business and indicative of Russia's energy clout. But now Ukraine
is digging in its
heels and there is a good chance it won't have to pay up. The dispute is symptomatic of the
Russian energy giant's current plight. Technological progress is threatening its business model
and the company that has long monopolized the market has failed to adjust in time. "Eat or be eaten" has been its general operating
principle when it comes to prices. For decades, many countries, including Ukraine, relied on Gazprom for its gas supply, but the
market is becoming increasingly global. With the supply of natural gas growing and prices
falling, Gazprom is beginning to lose its grip on the market. Three-Fold Pressure There are three primary
risks that are threatening the country. For one, rivals in the Middle East are constructing facilities for
liquefied natural gas (LNG) and developing a fleet of special tankers that will be able to transport LNG to destinations
thousands of kilometers away -- further than any pipeline and with far more flexible trade routes. In Europe and Asia, LNG is
increasingly competing with Gazprom. Qatar in particular has massively boosted its LNG
supply to Europe: In 2011, the emirate exported 44 billion cubic meters, compared to 5 billion cubic meters in 2006. Secondly,
Norway is expanding its gas exploration and wresting market share from Russia in Europe. According
to Eurostat, the European Union's statistical authority, Norway's gas sales in Europe rose by 16 percent in 2012, while Gazprom's fell
by 8 percent. And thirdly, thanks to new drilling methods, it has become easier to extract natural gas trapped in permafrost, dense
clay and, especially, shale, allowing for gas production in previously untapped regions. In the US particularly, fracking, as it known,
has triggered a gas bonanza, making it hard for Russia to get a foothold in the market. Gazprom was aiming to secure 10 percent
of the US market, but this goal now seems decidedly out of reach. The Kremlin is feeling the effects of
these developments in the gas market. In coming years, EU countries such as Poland are planning to
concentrate on unconventional gas extraction in order to reduce reliance of Russia. With gas and
oil accounting for 50 percent of state revenue, a drop in Russian exports will hit Moscow hard. In addition, it will lose leverage over
countries such as Poland, Ukraine, Lithuania and other states that used belong to the Soviet sphere of influence. The German
intelligence service Bundesnachrichtendienst (BND) predicts that the erstwhile energy giant will soon begin to lose power. Reality
Catches Up Gazprom, as a result, has devolved from being one of the Kremlin's biggest political
assets to becoming a problem child. Decades of market domination have made the company
lazy and it is now failing to adapt. The company continues trying to impose costly, long-term
supply contracts on its customers, even though gas prices on the spot market have long since
begun undercutting Gazprom. Furthermore, it remains inefficient, often spending up to three
times as much as its rivals on similar projects. Now, reality is quickly catching up with the company -- and it has been
brutal. Gazprom has beem forced to concede discounts to its customers with increasing frequency. Recently, Polish company PGNiG
beat the supply price of Russian natural gas down from $500 to $450 per 1,000 cubic meters. German market leader E.on Ruhrgas,
meanwhile, negotiated a price reduction of over €1 billion for 2012 alone. Customers are also buying more from Gazprom's rivals.
Gazprom is feeling the pinch. Between January and September, 2012, the company saw profits of some €20.2 billion, down 12
percent on the same period in 2011. Its turnover from gas exports dropped 8 percent to €44.9 billion. According to the former
Deputy Energy Minister Vladimir Milov, now an opposition politician, the company's gas production fell by 6.7 percent
to 478 billion cubic meters last year. Gazprom blames the crisis in the key European market, which usually is responsible for two-
thirds of the company's profits. Yet demand for Gazprom natural gas has dropped more steeply than
demand for gas in general. In Italy, for example, general demand for natural gas declined in the first three quarter of 2012
by 2.6 percent, but Gazprom supplied 11 percent less in the same period. In the Netherlands, meanwhile, total sales dropped by 9
percent, with Gazprom's supply to the country dropping by 42.6 percent. Polish demand for gas actually rose by 6.2 percent, but
imported 11.5 percent less from Gazprom. As the company loses its market share, the European Commission is preparing to clip its
wings even further. In early September, regulators launched an investigation into whether Gazprom
might be hindering competition in Central and Eastern European gas markets, in breach of EU
anti-trust rules. The investigation strikes at the heart of the Russian business model, and will examine several of its practices,
such as whether it has imposed unfair prices on its customers by linking the price of natural gas to that of oil. Changing Paradigms
Gazprom is also under pressure outside the EU. Ukraine,
a chronically cash-strapped key customer of the
Russian company, is cutting down on gas imports. The "take or pay" contracts signed in 2009 foresaw annual
deliveries of 41.6 billion cubic meters, but Kiev imported just 25.9 billion cubic meters in 2012. In 2013, it expects to import only 20
billion cubic meters. High energy costs are threatening to derail the Ukrainian economy. For the time being, the country pays an
extortionate $425 per 1,000 cubic meters of gas, with Russia tying any potential reductions to political demands. Moscow has let
Kiev know that it can only expect a discount if Ukraine joins the Russian-led Customs Union. Belarus has already caved in and now
pays just $185 per 1,000 cubic meters of gas. But Ukraine is working on ways to get by the blackmail. A terminal for LNG tankers is
planned in Odessa, and just days before Gazprom presented the $7 billion invoice, Kiev signed a shale gas production sharing
agreement with Shell. Further deals with Chevron and Exxon are also in the cards.

No impact to gas control—countermeasures empirically prevent Russian leverage

Stegen ’11 (Professor of Renewable Energy and Environmental Politics – School of Humanities & Social Sciences @ Jacobs
University,

(Karen Smith, “Deconstructing the “energy weapon”: Russia's threat to Europe as case study,” Energy Policy Vol. 39, Issue 10, p.
6505–6513, 2011)

In some cases, Russia does seem to have implemented its energy weapon successfully. Without control over natural gas and
important energy transit routes, for example, Russia could well have lost control over the symbolically significant Black Sea Fleet.
However, the evidence for the consistently successful implementation of the energy weapon by
Russia is less than overwhelming. Client states, even weak and highly dependent states such
as the Baltic countries and Georgia, were able to resist changing their policies to appease
Russia, often through the use of strategic alliances. This raises the question: why would Germany and
other European countries not be able to resist similar pressure? This more sanguine reading of Russia–
European energy relations is, of course, grounded in the present. Twenty-five years ago, few would have believed that the Soviet
Union and Warsaw Bloc would one day voluntarily dissolve themselves; what worries some policy makers and others
are the long-term scenarios of world events that, from today's perspective, seem improbable. In one such scenario, a Russia
led by the Siloviki (the Kremlin's hardliners) engages the West in a hot war and diverts Europe's supplies to China or elsewhere (Hill,
2006). The assumptions underlying such scenarios, from an energy standpoint, are that Europe's current dependency on Russia will
continue and that European states will remain “helpless”. As the response of western states to the
1970s OPEC oil embargo—
the first use of the energy weapon—demonstrated, consumer states can develop countermeasures to
supplier state manipulation. In the 1970s, these measures included the establishment of the International Energy
Agency, through which oil importers can coordinate and limit supply shocks; creation of 90-day strategic oil
reserves; significant reductions in the oil intensity of Western economies as well as the emergence of a strategic alliance
between the U.S. and Saudi Arabia, which further limited OPEC's effectiveness. Thanks to these protections, plus the rise of non-
OPEC oil production, the Middle East oil weapon lost some of its leverage (Perovic, 2009). Similar to the countermeasures of the
1970s, the
EU—after the January 2009 Russia–Ukraine gas crisis—also implemented protective
measures against gas disruptions. The EU now requires all Member States to adopt and
regularly update preventive action plans (the first action plans should be adopted by December 3, 2012) and to
identify energy security threats—for which the insights of this article are pertinent—and mitigation measures .
Unsurprisingly, suggested measures in Annex II of this regulation include diversifying gas suppliers and gas routes; investing into
network infrastructure; increasing the share of renewable gas as a supply side measure; and increasing energy efficiency and fuel-
switching as demand side measures (EU, 2010). Even prior to legislating the above protective measures, the
EU was pursuing
the establishment of a single European gas market, which is expected to significantly contribute
to European energy security as it would increase gas flows within the entire European Union and
moderate the consequences of disruptions, no matter what their cause (for example, natural hazard or political
manipulation). The recent implementation of the “third energy package”—the unbundling of energy producers from the network—
will further enhance energy security as suppliers such as Gazprom will have to relinquish their transportation infrastructure. As one
would expect, Putin has vociferously protested the third energy package (Socor, 2011b). Unbundling could stop a producer from
being able to enact the energy weapon, but many questions remain over how the regulation will actually be enforced. Moreover, a
supplier with shared borders could still own its domestic infrastructure and could still cause a disruption on its side of the border.
Russia Economy Answers
Frontline

No impact to Russia econ collapse

Friedman ‘9 (chief intelligence officer, financial overseer, and CEO of the private intelligence corporation Stratfor, 9 (George,
7-27-09, Stratfor, “The Russian Economy and Russian Power,”
http://www.stratfor.com/weekly/20090727_u_s_policy_continuity_and_russian_response, accessed 10-27-10,)

Russia has been an economic wreck for most of its history, both under the czars and under the Soviets. The
geography of Russia has a range of weaknesses, as we have explored. Russia's geography, daunting infrastructural challenges and
demographic structure all conspire against it. But the strategicpower of Russia was never synchronized to its
economic well-being. Certainly, following World War II the Russian economy was shattered and never
quite came back together. Yet Russian global power was still enormous . A look at the crushing poverty -- but
undeniable power -- of Russia during broad swaths of time from 1600 until Andropov arrived on the scene certainly gives
credence to Putin's view. The problems of the 1980s had as much to do with the weakening and corruption of the Communist Party
under former Soviet leader Leonid Brezhnev as it had to do with intrinsic economic weakness. To put it differently, the Soviet Union
was an economic wreck under Joseph Stalin as well. The Germans made a massive mistake in confusing Soviet
economic weakness with military weakness. During the Cold War, the United States did not make that mistake. It understood that
Soviet economic weakness did not track with Russian strategic power. Moscow might not be able to house its
people, but its military power was not to be dismissed. What made an economic cripple into a military giant was
political power. Both the czar and the Communist Party maintained a ruthless degree of control over society. That meant
Moscow could divert resources from consumption to the military and suppress resistance. In a state run by
terror, dissatisfaction with the state of the economy does not translate into either policy shifts or
military weakness -- and certainly not in the short term . Huge percentages of gross domestic
product can be devoted to military purposes, even if used inefficiently there. Repression and terror smooth over public
opinion.The czar used repression widely, and it was not until the army itself rebelled in World War I that the regime collapsed.
Under Stalin, even at the worst moments of World War II, the army did not rebel. In both regimes , economic dysfunction
was accepted as the inevitable price of strategic power. And dissent -- even the hint of dissent -- was dealt
with by the only truly efficient state enterprise: the security apparatus, whether called the Okhraina, Cheka, NKVD, MGB or KGB.
From the point of view of Putin, who has called the Soviet collapse the greatest tragedy of our time , the problem was not
economic dysfunction. Rather, it was the attempt to completely overhaul the Soviet Union's foreign and
domestic policies simultaneously that led to the collapse of the Soviet Union. And that collapse did not lead to an
economic renaissance. Biden might not have meant to gloat, but he drove home the point that Putin believes. For Putin, the West,
and particularly the United States, engineered the fall of the Soviet Union by policies crafted by the Reagan administration -- and
that same policy remains in place under the Obama administration. It is not clear that Putin and Russian President Dmitri Medvedev
disagree with Biden's analysis -- the Russian economy truly is "withering" -- except in one sense. Given the policies Putin has
pursued, the Russian prime minister must believe he has a way to cope with that. In the short run, Putin might well have such a
coping mechanism, and this is the temporary window of opportunity Biden alluded to. But in the long run, the solution is not
improving the economy -- that would be difficult, if not outright impossible, for a country as large and lightly populated as
Russia. Rather, the solution is accepting that Russia's economic weakness is endemic and creating a regime that
allows Russia to be a great power in spite of that. Such a regime is the one that can create military power in the face of broad
poverty, something we will call the "Chekist state." This state uses its security apparatus, now known as the FSB, to control the
public through repression, freeing the state to allocate resources to the military as needed. In other words, this is Putin coming full
circle to his KGB roots, but without the teachings of an Andropov or Gorbachev to confuse the issue. This is not an ideological
stance; it applies to the Romanovs and to the Bolsheviks. It is an operational principle embedded in Russian geopolitics and history.
Counting on Russian strategic power to track Russian economic power is risky . Certainly, it did in the
1980s and 1990s, but Putin has worked to decouple the two . On the surface, it might seem a futile gesture, but in
Russian history, this decoupling is the norm. Obama seems to understand this to the extent that he has tried to
play off Medvedev (who appears less traditional) from Putin (who appears to be the more traditional), but we do not think this is a
viable strategy -- this is not a matter of Russian political personalities but of Russian geopolitical necessity.

The Russian economy is strong and resilient

Actuarial Post 11 [“Russian recovery continues to gather momentum,” July 19 2011,


http://www.actuarialpost.co.uk/article/russian-recovery-continues-to-gather-momentum-623.htm]

Baring Asset Management (Barings), the international investment management firm, believes Russia
will become an
increasingly attractive market to investors over the next 18 months as its economic recovery
continues to gather pace on the back of higher public sector investment and consumer spending.
Matthias Siller, manager of the Baring Russia Fund also believes that over the next 18 months, Russia's political backdrop will
have a positive influence on investment opportunities in the country. This year, Russia's Parliamentary elections will take place
and in 2012, the Presidential election. Siller explains: "The elections will naturally result in an increase in social spending on
infrastructure and on housing as the government tries to secure support. Aggressive fiscal loosening will also put more money in
people's pockets and boost consumer confidence, supporting growth." In terms of GDP growth Barings believes this will remain
solid, although Russia's economic recovery has been slower to gain momentum compared to other emerging European nations.
Siller says: "Russia's late cyclical recovery means that whilst consumer spending is only just starting to pick up, its monetary
pressures are less strained than other European countries. Consumer spending, supported by a revival in retail loan growth
since early 2010, points toward a strong, sustained recovery. Evidence of growth in consumption can be seen in rising new car

sales which for example are well above Turkey's." Barings believes the Russian economy has been relatively

resilient to the financial crisis, and currently its budget deficit forecasts are significantly
better than some other European emerging economies. While other European governments'
support of growth via deficit spending comes to an end, Russia is an exception to the trend. A
deficit spending increase in Russia will continue to underpin wage growth and consumption. Barings also expects privatisation
efforts to increase and generate more growth for businesses.

Econ resilient and impact empirically denied

Stokes ‘8 (Bruce Stokes. "Don't Ignore the Russian Bear." Council on Foreign Relations,
http://www.cfr.org/publication/3225/dont_ignore_the_russian_bear.html)

A little less than a year ago, August 17 to be precise, the post-Cold War Russian economic experiment
imploded. The ruble collapsed and debt payments to foreigners were frozen. Wall Street lost billions of dollars. Long Term
Capital Management, one of the world's biggest hedge funds, had to be taken over by its bankers. Once burned, international
investors yanked their capital out of all emerging markets— from Latin America to East Asia— causing world interest rates to
spike. The global economy teetered on the edge of depression.   But, much to the surprise of
most economic pundits, international markets quickly righted themselves. The Russian
economy proved far more resilient than anticipated . And, in retrospect, the events of August,
1998 were little more than a very large bump in the road.   The lessons of this "crisis that
wasn't" are now clear: Russia is not too big to fail (the volume of its debts do not dictate special treatment by
its creditors); the financial world can cope with such failure; and the Russian economy can
bounce back without much overt help from the West. But the impending $4.5 billion loan to Russia by the
International Monetary Fund— reflecting Washington's gratitude for Moscow's help in Kosovo, continued fear of Russian
nuclear proliferation and concern about Russia's internal political stability— demonstrates that Russia still remains too
important for the world to ignore.   This contradiction— not too big to fail, but still too big to flounder— highlights the friction
inherent when economic policy is used to further geo-political goals. Up until a year ago, the Clinton Administration argued that
aid to Russia was needed, in part, to avoid global economic collapse. August, 1998 exposed that rationale as a charade. Now
American support for assistance to Russia can only be justified for two reasons: to reinforce Russia's transition to a market
economy or as ransom in Moscow's continued strategic blackmail of the West. Evidence to justify the former is dubious. Its time
to own up to the latter.   Last summer's fleeting economic
fright reflected Russia's staggering economic
collapse. The ruble fell by more than 70 per cent in a couple of weeks. The economy shrank
by 4.3 per cent. Real wages fell 41 per cent.   But the crisis was cathartic. "The shock
accomplished what reform was intended to achieve ," said Anders Aslund, a senior associate at the Carnegie
Endowment for International Peace in Washington. The banking system now functions better. Barter is
declining. Most important, there has been no reversion to central planning, government-directed
lending, industrial subsidies or government reliance on simply printing money.

Way too many alt causes

Passell 12 (Peter, the Economics Editor of Democracy Lab, is a Senior Fellow at the Milken Institute. Why Putinomics Isn't Worth
Emulating http://www.foreignpolicy.com/articles/2012/01/27/why_putinomics_isnt_worth_emulating?page=0,0)

Some foreign investors have braved the hostile business climate. But most have been speculators hoping to
get in and out before the next stock market bubble bursts , and such "hot money" only
exacerbates economic instability . Most of the rest are multinational corporations investing in oil and hard minerals;
they are only willing to take enormous business risks in return for the possibility of enormous profits. The sorts of investors
Russia needs most -- long-term investors in manufacturing and services who bring advanced process technology along with
management and marketing skills -- are far more likely [to] end up in China or Brazil. Home-grown enterprise
that might help to fill the gap is sadly lacking. Perhaps that's a reaction to the barriers created by official
corruption , bad regulation , inefficient capital markets and organized crime . Perhaps it's a
consequence of long authoritarian rule that discouraged fresh thinking. Perhaps it's because the most
entrepreneurial Russians have emigrated to the United States, Europe and Israel. Probably it's all of the
above

No impact to Russian economy

Blackwill ‘9 – former associate dean of the Kennedy School of Government and Deputy Assistant to the President and Deputy
National Security Advisor for Strategic Planning (Robert, RAND, “The Geopolitical Consequences of the World Economic Recession—
A Caution”, http://www.rand.org/pubs/occasional_papers/2009/RAND_OP275.pdf, WEA)

Now on to Russia. Again, five years from today. Did the global recession and Russia’s present serious
economic problems substantially modify Russian foreign policy? No . (President Obama is beginning his
early July visit to Moscow as this paper goes to press; nothing fundamental will result from that visit). Did it produce a
serious weakening of Vladimir Putin’s power and authority in Russia? No, as recent polls in Russia make
clear. Did it reduce Russian worries and capacities to oppose NATO enlargement and defense measures eastward? No. Did it
affect Russia’s willingness to accept much tougher sanctions against Iran? No. Russian Foreign Minister Lavrov has said there is
no evidence that Iran intends to make a nuclear weapon.25 In sum, Russian foreign policy is today on a steady,
consistent path that can be characterized as follows: to resurrect Russia’s standing as a great power; to reestablish Russian
primary influence over the space of the former Soviet Union; to resist Western eff orts to encroach on the space of the former
Soviet Union; to revive Russia’s military might and power projection; to extend the reach of Russian diplomacy in Europe, Asia,
and beyond; and to oppose American global primacy. For
Moscow, these foreign policy first principles are
here to stay, as they have existed in Russia for centuries . 26 None of these enduring objectives
of Russian foreign policy are likely to be changed in any serious way by the economic crisis.
---Russia Econ- Resilient

Russian economy resilient – and impact is empirically denied

Post Magazine 8 (3-31, Lexis)

In late 1998 Russia's economy suffered a major downturn that saw its financial markets
collapse. As a result of world commodity prices sinking, Russia's slick black oil and gas-driven economy turned
into a red sea of disasters - and the insurance market followed suit in a crash of its own. But, slowly, Russia
began to rebuild itself. And like any country depending on exports, the win-lose battle suddenly started to improve. With
oil prices skyrocketing to a new all-time high of close to $105 (£52.6) per barrel this month, it seems likely the country will
stay in the black. "It is impossible to think of any event that might cause a similar fall in
country's insurance market," says Nikolay Galushin, deputy chief executive officer for corporate development of Ingosstrakh,
one of the country's largest domestic insurance companies.
---Russia Econ- Alt Causes

Alt causes overwhelm and mean nothing can solve the Russian econ

Kramer 12 (Economist, staff writer for NYT “Global Investor Doubts Constrain A Russian Oil Powerhouse”
http://www.nytimes.com/2012/05/29/business/global/investor-doubts-constrain-a-russian-oil-powerhouse.html?
_r=3&pagewanted=all&)

The Russian market this spring fell faster than other so-called BRIC countries of Brazil, Russia, India
and China and since mid-March is down 18.8 percent. Global oil prices have slumped, reducing expected
earnings. But even taking earnings into account, investors take a dim view of Russian equities.
The Russian stock exchange now trades at an average price to estimated earnings ratio of 4.28, compared with the MSCI Emerging-
Markets Index average. It is a glum statistic for Russia, particularly as President Vladimir V. Putin is
planning a wide-ranging sale of state assets to raise money for increased military and social spending
promised during his campaign. The price-to-earnings ratio comparison means that, statistically, a company that mines gold or
pumps oil in Russia is worth less than half as much as a company that extracts the same
amount of gold or oil just as efficiently in Brazil or Indonesia. For all the value in the Russian economy,
this wealthy industrial superpower cannot convince investors that it is safe place to put money —
even an oil company is a hard sell. _NYT No wonder. When wealthy Russia cannot convince insurance companies
to insure joint projects inside Russia, of course it will not be able to convince most investors to take the
huge risks of exposing valuable assets to the kleptocratic Russian bear. The Russian
government treats all assets -- public or private -- as its own little treasure chest of goodies

Alt causes prevent solvency- especially corruption

Passell 12 (Peter, the Economics Editor of Democracy Lab, is a Senior Fellow at the Milken Institute. Why Putinomics Isn't Worth
Emulating http://www.foreignpolicy.com/articles/2012/01/27/why_putinomics_isnt_worth_emulating?page=0,0)

It's not clear how Russia could cleanse itself of the toxic economic legacy left by communist rule and
further entrenched by the new plutocracy. What is clear, though, is that Vladimir Putin has been part
of the problem to date, and offers no reason to believe he will be part of the solution in the
future. In emerging-market countries ranging from China to Egypt to Malaysia, apologists for autocrats have argued
- sometimes plausibly -- that the uncertainty associated with the transition to political
pluralism undermines growth. But in Russia, where Putin has apparently tied his fate to the
status quo, real democracy just might be what it takes to bring the economy into the new
century.
---Russia Econ- No Impact

Empirics disprove Russian collapse

WPJ 3 (World Policy Journal, 12-22)

Using extensive interviews with participants in all three administrations, and memoirs by former officials, they paint a
compelling picture of officials often over-whelmed by the challenge of an entirely new reality. The unexpected collapse of
communism and of the Soviet Union, coming just after the GulfWar, left them with no road map to understand how Russia and
other post-Soviet states might develop. Nightmare scenarios suggested themselves: nuclear war between Russia
and Ukraine; weapons proliferation on a terrifying scale; Yugoslav-type ethnically based civil
war on the territory of the
former Soviet Union; mass starvation; economic collapse--the ominous possibilities were endless. That these
"dogs did not bark" is testimony to the unwillingness of people in the post-Soviet space to
engage in armed conflict and to Western assistance that staved off famine and economic
collapse. The failure of catastrophic scenarios to come about is one indicator of success--but if one were to measure
America's contribution to transforming Russia in more positive ways, the evidence is more mixed. If a minimalist definition of
success was the absence of catastrophe, the maximalist definition was the creation of a fully functioning democracy in Russia
with a transparent market economy and the rule of law. That has not happened yet, and it is unclear when it will. So far, there is
no consensus about what would constitute a realistic timetable for Russia's democratic development.

Russian economic decline doesn’t cause war- military is too weak

Mankoff 10 - specialist in Eurasian/Russian affairs, adjunct fellow for Russia studies at the Council on Foreign Relations

(Jeffrey, March, “Internal and External Impact of Russia's Economic Crisis”)

The crisis will affect Russian foreign policy in other ways as well. The military, which during the boom
years became accustomed to yearly funding increases, will have to confront a period of
renewed austerity.27 This austerity comes at a bad time: the war with Georgia in August 2008 exposed a
number of shortcomings that the infusion of cash during the Putin years had failed to stem . In
particular, the Russian armed forces were slow to react and struggled to mount combined operations, even against a much weaker
foe. Even though the war was ultimately successful, the military‘s less than impressive showing strengthened the conviction among
Russia‘s political leaders that reform had to be accelerated. In this way the war provided momentum for implementing a scheme to
downsize and modernize the army that experts had been discussing since the 1990s. In essence, to deal with the kind of threats it
confronts in the contemporary world, the Russian military needs to move away from the conscription-based mass army of the 20th
century to a smaller, more mobile, and more professional force. Yet because such a fundamental re-conceptualization of the
military‘s structure and goals would require significant downsizing, the officer corps has been strongly opposed to the reform.
Anatoly Serdyukov, a one-time furniture salesman, was named Minister of Defense by Putin in 2007 largely because of the perceived
necessity of having a non-military figure overseeing the reform efforts. With strong backing from both Putin and Medvedev,
Serdyukov has been doggedly pursuing his program of transformation. However, lack of funding has imperiled these
efforts, because of the need to provide social support (including housing) for laid off officers. Moscow, meanwhile, insists
that spending on military procurement will not be affected by the crisis as it seeks to both upgrade the
conventional forces in the aftermath of the Georgia conflict and maintain a sufficient strategic deterrence capability— though
whether it can actually afford to do so remains uncertain . Meanwhile, the Russian navy is also
encountering problems. Difficulties with the new Bulava submarine-launched ballistic missile (which has failed the majority
of its operational tests), delays in procuring new ships, and the mounting obsolescence of much of the existing fleet have created
major difficulties for the navy. Given
the lack of resources and decline of the military-industrial complex
since the Soviet days, these developments may be difficult to reverse .28 The struggles of the
military and military-industrial complex have given the Kremlin a powerful incentive to avoid
the risk of further military conflicts, especially while the fate of the Serdyukov-Medvedev reforms remains uncertain.

Russian decline doesn’t cause war

New Republic 2k (2-7, Lexis)

At the time and since, observers of the events of 1989-1990 in Europe have been properly amazed at what happened and at
what did not happen. The Soviet empire collapsed in Eastern Europe. Divided Germany was unified. Democratic governments
replaced communist dictatorships in Eastern Europe. The Soviet Union itself imploded and was reincarnated as quasi-
democratic, quasi- authoritarian Russia. Yet a unified Germany has not become a menacing Fourth Reich, and Russia,
despite a collapse of its economy and the spectacular loss of the Cold War, did not turn in bitterness
and frustration to the alliance of nationalists and communists who were seeking to reverse
the humiliations of a decade ago.
---Russia Econ- Strong Now

Russian economy growing and resilient – assumes current global crisis

Reuters ’11 (“Update 1-Russian economic growth gains speed in Q3” http://www.reuters.com/article/2011/10/25/russia-
economy-idUSL5E7LP46T20111025)

Russia's gross domestic product (GDP) grew 5.1 percent in the third-quarter , the economy ministry said
on Tuesday, with the figure meeting analyst expectations and suggesting economic expansion has gained speed in
recent months. In September, GDP grew 5.7 percent year-on-year, Deputy Economy Minister Andrei
Klepach said, which follows a 5.2 percent rise in August . "The third quarter was fairly positive for the economy ... we
can talk about growth gaining in pace," Klepach told reporters. Reuters most recent poll showed that economists
expect third-quarter economic expansion to reach 5.1 percent in annual terms. The data suggests that
the country is on track to achieve official forecasts of 4.1 percent GDP growth this year. Klepach said that the ministry expects GDP
growth to slow down in the fourth quarter to 3.8-3.9 percent. The
GDP data follows largely positive news from
last week that showed Russia's economy remains seemingly resilient despite global economic
turmoil and disappointing industrial output in September.

Consumption and investment growth at 3-year high

Businessweek ‘11 (“Russian Consumption Drives economic growth to 3-year high”


http://www.businessweek.com/news/2011-10-26/russian-consumption-drives-economic-growth-to-3-year-high.html)

Russian domestic demand, fueled by consumer loans, propelled economic growth to the fastest
rate in three years in the third quarter, Deputy Economy Minister Andrei Klepach said . Gross
domestic product grew 5.1 percent from a year earlier, the highest rate since the third quarter
of 2008, compared with 3.4 percent in April-June, Klepach told reporters in Moscow late yesterday. That brought nine-
month growth to 4.2 percent, or 0.1 percentage point faster than the government’s full-year
forecast. “The main factors behind the acceleration in GDP growth were household
consumption and fixed investment growth, supported by more vibrant construction activity,”
Yaroslav Lissovolik, head of research at Deutsche Bank in Moscow, in an e-mailed note today. “We expect growth in the
final quarter to stay above 4 percent, which is likely to be supported by higher budget spending
at the end of this year and the still-high growth in household consumption. ”

Russian economy growing and resilient – assumes current global crisis

Reuters ‘11 (“Update 1-Russian economic growth gains speed in Q3” http://www.reuters.com/article/2011/10/25/russia-
economy-idUSL5E7LP46T20111025)
Russia's gross domestic product (GDP) grew 5.1 percent in the third-quarter , the economy ministry said
on Tuesday, with the figure meeting analyst expectations and suggesting economic expansion has gained speed in
recent months. In September, GDP grew 5.7 percent year-on-year, Deputy Economy Minister Andrei
Klepach said, which follows a 5.2 percent rise in August . "The third quarter was fairly positive for the economy ... we
can talk about growth gaining in pace," Klepach told reporters. Reuters most recent poll showed that economists
expect third-quarter economic expansion to reach 5.1 percent in annual terms. The data suggests that
the country is on track to achieve official forecasts of 4.1 percent GDP growth this year. Klepach said that the ministry expects GDP
growth to slow down in the fourth quarter to 3.8-3.9 percent. The
GDP data follows largely positive news from
last week that showed Russia's economy remains seemingly resilient despite global economic
turmoil and disappointing industrial output in September.
---Russia Econ- AT: Oil Solves

Even with increased oil- Russia can’t lift itself out of econ rut

Passell 12 (Peter, the Economics Editor of Democracy Lab, is a Senior Fellow at the Milken Institute. Why Putinomics Isn't Worth
Emulating http://www.foreignpolicy.com/articles/2012/01/27/why_putinomics_isnt_worth_emulating?page=0,0)

Consider, too, that while the economy would be damned without the energy revenues, it is
damned with them as well . The behemoth industrial complexes that stood as cathedrals to Stalinist faith
are mostly shuttered. Yet Russia's productivity problems linger: little that Russia makes is
competitive outside domestic markets. Now, economists happily explain that international competitiveness is largely
a function of exchange rates: If rubles were cheap enough to buy with dollars or euros, the cost of
production at Russian factories would be sufficiently low to make Russian products
competitive in global markets. That's where the oil exports become a liability. Hefty earnings from
foreign sales prevent the ruble from depreciating to the point that, say, Russian cars and farm
machinery could compete with similar products made in Korea and the United States. As a result, roughly
four-fifths of the country's export revenues derive from oil, gas and other natural resources .
Much of the rest comes from weapons sales produced in government-owned factories at only-Putin-knows-what cost. Actually,
Russian oil exports are doubly cursed. The only time the post-Soviet Kremlin has felt
compelled to listen to calls for growth-enhancing economic liberalization has been when the
government needs foreign loans to contain inflation and keep the financial system afloat . So oil
exports at high prices effectively insulate Putin from pressure to deregulate markets at the
expense of his plutocratic allies and vast bureaucratic patronage machine . Other economies (think
India) have recovered from the hangover of central planning and long isolation from global markets. But Russia's economic
malaise runs very deep.

Social statistics proves

Passell 12 (Peter, the Economics Editor of Democracy Lab, is a Senior Fellow at the Milken Institute. Why Putinomics Isn't Worth
Emulating http://www.foreignpolicy.com/articles/2012/01/27/why_putinomics_isnt_worth_emulating?page=0,0)

But the view is decidedly less tidy from close up. For starters, there's the not-so-small matter of
inequality. The economist's favorite yardstick of household income inequality, the GINI index,
is middling by comparison with other countries in Russia's income range. But the impact of
poverty is all too visible in health statistics. Life expectancy is eight years less than in China and
barely above that of Papua New Guinea. Infant mortality is triple that of Spain. Arguably the
most telling statistic: the total fertility rate (the number of children the average woman will have in a lifetime) is
1.4 -- a shockingly low rate that reflects, at the very least, housing scarcity and, more likely,
deep pessimism about the future.

Corruption proves
Passell 12 (Peter, the Economics Editor of Democracy Lab, is a Senior Fellow at the Milken Institute. Why Putinomics Isn't Worth
Emulating http://www.foreignpolicy.com/articles/2012/01/27/why_putinomics_isnt_worth_emulating?page=0,0)

Corruption is notorious. Transparency International ranks Russia 154th out of 176 countries on
its Corruption Perceptions Index - behind Yemen and the Central African Republic. And a
bureaucracy that still takes its inspiration from Leonid Brezhnev makes it exceptionally
difficult for those lacking political juice to make and sell goods. Russia rates 120th on the World
Bank's Ease of Doing Business Index. No wonder: on average, it takes 281 days to get an
electricity hookup and 423 days to get a construction permit.
---Russia Econ- AT: Reactor Sales

Russia’s getting crushed by other producers

WNN ‘10, World Nuclear News, “India’s Nuclear Push”, September 23, http://csis.org/blog/india%E2%80%99s-nuclear-push

It’s happening– second-tier


nuclear suppliers from China, South Korea, and now India are waking up
to the opportunities that may emerge from intensified interest in nuclear power . India is
entering the nuclear supply business at a time when new nuclear states are looking for
alternatives to the huge, expensive reactors sold by the French, Russians, Japanese, Canadians,
and Americans. Last year, Korea won the plum contract in the Middle East – a $20 billion agreement to
build 4 nuclear power reactors in the United Arab Emirates. The UAE plans to construct a total of 10 reactors, using one contractor.
China, while busily constructing nuclear power plants at home, will build a few new reactors in Pakistan and
reportedly is interested in Turkish and Arab state plans to import. India will be next off the
starting block of this export race. There’s no way to predict how price-competitive India’s export reactors will be. NPCIL
is a public enterprise under the control of the government’s Department of Atomic Energy. One of the suggested virtues of the U.S.-
India nuclear deal was that the Indian nuclear sector would be forced to clean up its act as foreign competition grew in India. One
way for the NPCIL to become more self-sustaining is through exports. What will motivate nuclear power newcomers to buy Indian,
Korean or Chinese? First, the
reactor vendors from the advanced nuclear states are in disarray. AREVA
has its much-publicized cost overruns in Olkiluoto ; Japanese vendors do not have an export
history; and Russian reactors were previously sold only in the Eastern bloc countries or allies.
Russia will expand from reactors in India and Iran to potential contracts with Turkey and
Vietnam.
Saudi Oil Answers
Zero risk of Saudi Oil disruption – security is tight, damage would be minimal, spare capacity
solves

CNN ’11 (“Saudi oil supplies are safe and secure,” 4/7/2011, http://articles.cnn.com/2011-04-
07/opinion/obaid.saudi.energy_1_abqaiq-saudi-oil-saudi-arabia?_s=PM:OPINION

We are now facing another, more dangerous illusion: the vulnerability of Saudi Arabia's energy
infrastructure. In recent weeks some pundits, oil traders and journalists have peddled a deceptive
assessment of the threats facing the kingdom's ability to supply oil to the world , and this fear-
mongering has had very real consequences, driving the price of oil to irrational heights, adding skittishness to the markets and
threatening the nascent global economic recovery. No system as vast as the Saudi oil complex -- with its scores of
rigs, refineries, export terminals and pipelines -- is perfectly protected. But a brief overview of the safeguards
built into this infrastructure -- the forces that protect it and the demographics of the region --
sheds light on the actual security situation and reveals the risks are much less serious than
widely disseminated. The first layer of security in the Saudi energy infrastructure is simply the
design and construction of the facilities. Two failed attacks show this well. The first happened at the Yanbu
petrochemical plant in 2004, where, because of the structure's layout, Saudi security agents were able to quickly
cordon the industrial portions of the facility and isolate and neutralize the terrorists . The
complex itself was never in danger, although several people were killed. Another example is the failed al
Qaeda attack on the Abqaiq oil processing facility, one of the largest and most important in the world, in February 2006.
The terrorists were able to breach the outer perimeter and overrun the guards, but never made it to the operational areas of the
plant itself. Beyond the first guard post is a large "no-man's land," designed as a moat to protect the functioning elements of the
facility. The terrorists were trapped and forced to detonate their car bomb there. Although people
tragically lost their lives at the first gate and the exterior portions of the center suffered some damage, at no time was the
facility or its ability to process oil under serious threat . In addition to safeguards and design
elements such as multiple barriers and perimeters, thousands of sensors, cameras, sophisticated
computers and world-class surveillance and security systems protect the sprawling energy
facilities. It is worth pointing out that the elements most difficult to protect, such as the thousands of
miles of pipeline, are also the easiest to repair and quickly get back online . Saudi authorities have
estimated that in a worst case scenario -- where an entire section of pipeline is destroyed -- repair
teams could bring the pipeline back to normal operation within days. The key processing points
and bottlenecks in the system are, by their centralized nature, much easier to defend. It is also important to note that
even though Abqaiq withstood the 2006 attack, since then the Saudi government has invested
more than $10 billion to improve its energy security even further. A key element of this initiative
has been the creation of a 35,000-strong "facilities security force ." These troops come from across the
kingdom and receive extensive training through a U.S. technical assistance program. This specialized force,
which did not exist before 2005, has the exclusive responsibility of guarding all energy installations against both internal and
external threats. Also, the Saudi government has stockpiled considerable quantities of oil through its
Foreign Reserve Initiative. A sizable portion is in floating containment facilities near the kingdom's main
export markets and can be released, if necessary, in emergencies, such as the tsunami in Japan or the civil war in
Libya.
Strong security improvements now

Hill and Nonneman 11 (Ginny and Gerd, Associate Fellows of the Middle East and North Africa Programme at Chatham
House, “Yemen, Saudi Arabia and the Gulf States: Elite Politics, Street Protests and Regional Diplomacy”, 5/11,
http://humansecuritygateway.com/documents/CH_YemenSaudiArabiaandGulfStatesPolPrtsRegDip.pdf)

However, the campaign also enjoyed significant public support in Saudi Arabia and was ‘spun’ by the media as ‘a heroic and
successful struggle to protect Saudi sovereignty’. 80 Some satisfaction is derived from the fact that there
have been no
further incursions since the intervention, and that Saudi Arabia was able to turn the episode to its
advantage by securing the border area. There is now a semi-permanent military complex
around the southern Saudi city of Najran. Nearly 80 border villages have been evacuated and the villagers are
being re-housed in 10,000 purpose-built units. Visible security improvements have been reported , including
earthen berms, concertina wire, floodlights and thermal cameras. 81 These measures serve Saudi Arabia’s longer-
term objective of containing AQAP, as well as constraining cross-border flows of drugs, weapons
and illegal migrants.

Impossible to strike oil facilities effectively- dispersed and redundant facilities

Shifrinson and Priebe 11 (Joshua R. Itzkowitz and Miranda, doctoral candidates in the Political Science Department at the
Massachusetts Institute of Technology, “A Crude Threat; The Limits of an Iranian Missile Campaign against Saudi Arabian Oil,”
International Security, Vol 36, No 1, Summer 2011, Project Muse)

Finally, this article carries implications for understanding the vulnerability of oil infrastructure and oil networks. Simply put, oil is
not an easy military target. Individual production facilities are large and physically robust,
requiring a significant military effort to disable. Facilities are also geographically dispersed,
necessitating systematic targeting. Given the presence of redundant facilities, some oil networks
may have few, if any, targets that can incapacitate an entire system. Even if there are critical nodes, states
can add facilities to limit the vulnerability of an oil network to disruption . If an attack is feared,
meanwhile, states can stockpile replacement equipment to mitigate prospective damage. In sum, oil is a lucrative target,
but it is not universally vulnerable. [End Page 201]
Saudi Yemen Spillover Answers
No spillover – the Saudis are successfully containing conflict.

Hill and Nonneman 11 (Ginny and Gerd, Associate Fellows of the Middle East and North Africa Programme at Chatham
House, “Yemen, Saudi Arabia and the Gulf States: Elite Politics, Street Protests and Regional Diplomacy”, 5/11,
http://humansecuritygateway.com/documents/CH_YemenSaudiArabiaandGulfStatesPolPrtsRegDip.pdf)

However, the campaign also enjoyed significant public support in Saudi Arabia and was ‘spun’ by
the media as ‘a heroic and successful struggle to protect Saudi sovereignty’. 80 Some satisfaction is
derived from the fact that there have been no further incursions since the intervention, and that Saudi
Arabia was able to turn the episode to its advantage by securing the border area . There is now a
semi-permanent military complex around the southern Saudi city of Najran. Nearly 80 border
villages have been evacuated and the villagers are being re-housed in 10,000 purpose-built
units. Visible security improvements have been reported , including earthen berms, concertina
wire, floodlights and thermal cameras. 81 These measures serve Saudi Arabia’s longer-term
objective of containing AQAP, as well as constraining cross-border flows of drugs, weapons and
illegal migrants.
Saudi Collapse Answers
Unrest can’t topple the regime and doesn’t threaten oil output

CNBC ’11 (3/11, 2011, Saudi Arabia Regime Will Survive Protests: Eurasia Group,
http://www.cnbc.com/id/42024487/Saudi_Arabia_Regime_Will_Survive_Protests_Eurasia_Group)

Police and protesters clashed in Saudi Arabia Thursday and the country faces a day of possible mass protests Friday, but even
heavy demonstrations will not succeed in removing the current regime, according to analysts at the Eurasia Group.
“The house of Saud will survive the short term … (Thursday’s skirmish) does not threaten the regime and will not
impact oil output,” the analysts said in a note. Friday’s protest participants “unlike protesters in Tunisia, Egypt, and Bahrain – will
not be chanting, ‘the people demand the fall of the system,’” the Eurasia report said. "The majority of Saudi opposition
figures want reform within the existing system," the analysts added. One of the areas of planned protests, Qatif, is located
close to a major pipeline. But “it is unlikely… protesters will try to sabotage energy infrastructure … doing
so would not cut output for long and would invite a devastating response from the
government,” the analysts said. “Things could get complicated if the Saudi security forces overreact,” but this is
unlikely as the majority of protesters are asking for change to the existing system, not total reform. “The Saudi
government has both resources and determination… there are steps the regime can take to placate some
protesters,” the analysts said, adding, “mass protests do not automatically equal regime demise.”

Economic and social concessions stop escalation

Moubayed ’11 (Syrian university professor, editor-in-chief – Forward Magazine (Syria), 9/28/’11 (Sami, A peek under Saudi
Arabia's veil, http://www.atimes.com/atimes/middle_east/mi28ak03.html)

King Abdullah, aged 87, knows what it takes to please young Saudis . With swift moves, he has managed, for now,
to keep a job bequeathed to him and his brothers by their father, the kingdom's founder, King Abdul Aziz. In March, shortly after the
collapse of Saudi Arabia's ally, Hosni Mubarak of Egypt, in what was seen as a pre-emptive measure, King Abdullah
ordered a
massive increase in spending, up to $130 billion over the next decade, on measures like affordable housing
for young Saudis. Many young newly married couples have complained that it was becoming increasingly difficult to buy a house in
Saudi Arabia. Abdullah additionally raised wages in the public sector and pledged more public sector jobs. Critics immediately
accused him of "bribing" his citizens, but the king did what was required from him to keep his people happy. Abdullah realized that
times had changed since he was a young man in the 1940s, and that social media networks like Facebook and Twitter had
completely revolutionized not only Saudi Arabia but the Arab and Muslim World at large. The handwriting had been on the wall,
after all, since demonstrations began in Tunisia last December. In Saudi Arabia, it immediately triggered an online campaign
demanding major political and economic reform. In early February, 40 women demonstrated for the release of prisoners held
without trial in Saudi jail. This was repeated in March in al-Qatif, al-Awamiyah, and Riyadh. Demonstrators called for a "day of rage"
on March 11, but it was severely suppressed by authorities. It
is always painful for any leader to grant
concessions under pressure, and Abdullah managed to make those concessions at the right
time, before they became too painful and before riots snowballed, taking Saudi Arabia down the path of
Egypt and Bahrain. Abdullah wanted the Saudis to see him as part of the solution in Saudi Arabia, rather than
as part of the problem, as was the case with Mubarak, for example. At the weekend, King Abdullah took his reforms a step forward
by granting Saudi women the right to vote and become members in the shura council, for the first time ever in Saudi history. The
Riyadh-based council is a powerful 150-member assembly, appointed by the king to advise on legislation. Opening it to women must
have taken a lot of thinking, and courage, from the king, knowing that this would unleash hell on him from traditional and
conservatives both within his family and in the clerical community of Saudi Arabia. "We refuse to marginalize the role of women in
Saudi society in every field of work," King Abdullah said on state television, adding that empowering
women would help
empower the nation as a whole. This in itself is groundbreaking in the world's strongest theocracy , where
women are strictly segregated in public, forced to cover themselves when out, are separated from men at schools, restaurants,
banks and work.

Saudi Arabia buys off dissent

Haykel ‘11 (Bernard, professor of near eastern studies at Princeton, “Saudi Arabia’s revolt busters,” The Sunday Times, 8/21/11,
http://sundaytimes.lk/110821/Timestwo/int012.html)

They also played the Shia card, an effective trump in Sunni-majority Saudi Arabia. The rulers argued
that public protests throughout the region were being orchestrated by Shia Iran, and were anti-Sunni and
sectarian. The threat of chaos, evident now in Libya, Syria, and Yemen, also weighed in the royal family's favour. The House of Saud
has a long historical claim on rule in Arabia, and its promise of stability remains key to its durability. A
massive government
subsidy package also accounted for domestic calm. Abruptly, some $130 billion was added to
spending projections over the next five years. Salaries for all public servants, a majority of the national work
force, were raised, as was the total number of public-sector jobs . King Abdullah pledged large numbers of
new housing units, an important gesture in a country where young people, especially young married couples, cannot easily
access the housing market. In neighbouring Bahrain, the Saudis also moved quickly to bolster the Sunni-
minority regime against a rising tide of protest led by the island kingdom's Shia majority. Saudi troops marched into Bahrain
under the banner of the Gulf Cooperation Council (GCC), and the Saudi rulers issued clear instructions to adopt an iron-fisted policy
with the demonstrators, again arguing that Iran's nefarious hand was at play in subverting the country. No doubt, the Saudis believe
that a Shia-led Bahrain would lead to Iranian dominance at their very doorstep. Here, too, the
Kingdom employed its
policy of largesse through the GCC, promising Bahrain $10 billion over the next decade . Other large-
scale financial commitments were made to Oman and Jordan, both Saudi allies that have managed to silence early whispers of mass
protest.

No Saudi collapse

Bronson 6 (Rachel, Former Adjunct Senior Fellow for Middle East Studies – Council on Foreign Relations, “5 Myths About U.S.-
Saudi Relations”, Washington Post, 5-21, http://www.cfr.org/publication/10728/)

The House of Saud is about to collapse. Not likely. Since the Saudi monarchy's earliest days, observers have
anticipated its demise. However, it has shown a remarkable ability to overcome such challenges as
palace infighting, assassination and incapacitated leaders. There are still many sons of kingdom founder
Abdel Aziz waiting in an orderly queue for their chance to reign. This hardly means the Saudi rulers will have an easy time of it.
Osama bin Laden has made toppling the House of Saud one of his key goals, and there have been a series of al-Qaeda attacks
since May 2003. Also, Saudi Arabia faces demographic challenges: Sixty percent of the population is younger than 25, and jobs
for them are scarce. Meanwhile, insurgent fighters eventually will return from Iraq, trained and determined, and the Sunni-
Shiite battles ofIraq can easily spill into Saudi Arabia, where Shiites make up 10 to 15 percent of the population. But the
cleavages common before a revolution are not visible in Saudi Arabia. The kingdom is now
aggressively pursuing terrorists on its soil, and reform-minded Saudis view King Abdullah as an ally. Washington would be better
off planning on the royal family enduring. It's also the best chance Washington has to realize its oil and counterterrorism goals --
and avoid alternatives that could be worse.

Coup unlikely – loyal clearics can blunt opposition

Bremmer 4 (Ian, President – Eurasia Group and Senior Fellow – World Policy Institute, “The Saudi Paradox”, CIAO Net)

How Serious Are the Risks of Implosion? Recent events have quickened the concern that the Saudi regime might collapse from
within. It is worth recalling that scarcely anybody in intelligence or in academia foresaw the scale of the Iranian crisis of 1978–
79, and the latent strength of the ayatollahs. Might not Saudi Arabia produce the next Islamic revolution? Not this year. A
look at the differences between Iran in 1979 and Saudi Arabia today is instructive. The security
services permeate Saudi society more broadly and deeply than the Shah’s Savak penetrated Iran in the
late 1970s. And unlike the Shah, the Saudi royal family has co-opted the radicals . There is currently
enough royal money flowing through traditional tribal patronage net- works to buy loyalty .
Perhaps more impor- tantly, the Saudi royals have essentially handed over education and cultural
policy to influential Wahhabi clerics. This “unholy alliance” irks the United States and may ultimately destabilize
the country but, so far, this system has worked as the royals in- tended . In Teheran during the late 1970s, the
Shah actively alienated Shiite mullahs with his globalizing policies and aggressive secularism. By contrast, Saudi clerics are
allowed enough freedom and influence to blunt serious Islamist challenges to the
monarchy, at least for now.
Saudi Prolif Answers
No Saudi nukes

Doherty, ’12 (Ben, Sydney Morning Herald staff, 1/30/12, http://www.smh.com.au/world/iran-nuclear-bid-tipped-to-provoke-


saudi-bomb-20120129-1qo21.html#ixzz1lAI58I3Y

''For all its wealth, Saudi


Arabia does not have the technical and scientific base to create a nuclear
infrastructure,'' Pervez Hoodbhoy, a nuclear physicist at Islamabad's Quaid-e-Azam University, told the Herald.
''Too weak to defend itself and too rich to be left alone, the country has always been surrounded by those who eye its wealth.'' But
despite being ''enormously indebted'' to Saudi Arabia, Islamabad cannot simply sell bombs ''off the shelf'' to
Riyadh, Professor Hoodbhoy said. ''Deterrence becomes effective once you advertise you have a weapon in hand,'' he said. ''But if
a country buys weapons surreptitiously, it cannot flaunt them as a nuclear deterrent because of
the obvious question, 'Where did you get them from?''' Saudi Arabia, a close ally of the US,
cannot be seen to be buying nuclear weapons from Pakistan, and Pakistan, already a nuclear
pariah, cannot afford to be cast, again, as a proliferator of arms. A secret weapons program would
put Saudi Arabia in breach of a memorandum of understanding with the US that promises
American assistance for a civilian nuclear program in return for the Saudis not pursuing
''sensitive nuclear technologies''. Even with assistance, building nuclear weapons would take
Saudi Arabia 10 to 15 years, Professor Hoodbhoy said.

Saudi prolif requires Chinese assistance, but that won’t happen – aging Saudi delivery systems,
jeopardizes U.S.-China nuclear co-op agreement, sanctions, no incentive, Chinese dependence
on Middle Eastern oil

Kahl 13 – Senior Fellow at the Center for a New American Security and an associate professor in the Security Studies Program
at Georgetown University’s Edmund A. Walsh School of Foreign Service (Colin H., Melissa G. Dalton, Visiting Fellow at the Center for
a New American Security, Matthew Irvine, Research Associate at the Center for a New American Security, February, “If Iran Builds
the Bomb, Will Saudi Arabia Be Next?” http://www.cnas.org/files/documents/publications/CNAS_AtomicKingdom_Kahl.pdf)

Finally, if Pakistan provided Saudi Arabia with a nuclear warhead (or help in developing one) without accompanying aircraft or
missiles, Riyadh would probably require China’s assistance to upgrade or replace the Kingdom’s aging
CSS-2s as potential delivery systems. China has close relations with Islamabad, seeing Pakistan as a useful counterweight to
India, and China is a major customer for Saudi oil, potentially providing an incentive to cooperate.152 Allegations have also surfaced
that the Saudis have approached China (perhaps with Pakistani mediation) to provide more advanced nuclear-capable missiles.153
Nevertheless, Beijing is highly unlikely to provide the required assistance. Abetting a proliferation
deal between Saudi Arabia and Pakistan could jeopardize China’s highly valued bilateral nuclear
cooperation agreement with the United States and potentially make China the target of Western
sanctions. Furthermore, although China was willing to aid Pakistan’s nuclear program in the past and provide Saudi Arabia with
its existing CSS-2 missiles, 154 Beijing would have no compelling strategic imperative to facilitate Saudi
efforts to balance a nuclear-armed Iran. Indeed, given China’s dependence on Middle Eastern oil supplies
and interest in stability, it
seems highly unlikely that Chinese leaders would want to help accelerate a
destabilizing nuclear arms race in the region.155

Tech hurdles

Hoodbhoy, ’12 (Pervesz teaches physics and political science at LUMS. He taught at Quaid-i-Azam University for 36 years and
was head of the physics department. He received a doctorate in nuclear physics from the Massachusetts Institute of Technology,
1/23/12, http://www.sananews.net/english/2012/01/the-bomb-iran-saudi-arabia-and-pakistan/)

For all its wealth, Saudi Arabia does not have the technical and scientific base to create a nuclear
infrastructure. Too weak to defend itself and too rich to be left alone, the country has always been surrounded by those who
eye its wealth. It has many universities staffed by highly paid expatriates and tens of thousands of Saudi students have been sent to
universities overseas. But because of an ideological attitude unsuited to the acquisition of modern
scientific skills, there has been little success in producing a significant number of accomplished
Saudi engineers and scientists. Perforce, Saudi Arabia will turn to Pakistan for nuclear help. This
does not mean outright transfer of nuclear weapons by Pakistan to Saudi Arabia. One cannot
put credence on rumours that the Saudis have purchased nuclear warheads stocked at Kamra air force
base, to be flown out at the opportune time. Surely, this would certainly lead to extreme reaction from the US
and Europe, with no support offered by China or Russia. Moreover, even if a few weapons were
smuggled out, Saudi Arabia could not claim to have them. Thus they could not serve as a nuclear
deterrent.

No Saudi Prolif from Pakistan

Lippman 11 – senior adjunct scholar at the Council on Foreign Relations in Washington. His career in journalism at the
Washington Post included four years as the Washington Post’s Middle East bureau chief, three years as the Post’s oil and energy
reporter and a decade as the newspaper’s national security and diplomatic correspondent, he traveled extensively to Saudi Arabia.
He is the author of “Arabian Knight: Colonel Bill Eddy USMC and the Rise of American Power in the Middle East,” “Inside the Mirage:
America’s Fragile Partnership with Saudi Arabia,” ” Madeleine Albright and the New American Diplomacy,” ” Understanding Islam,
and Egypt After Nasser”. A writer and journalist specializing in U.S. foreign policy and Middle Eastern affairs (Thomas W., 08/05,
“Saudi Arabia’s Nuclear Policy,” http://www.susris.com/2011/08/05/saudi-arabia%E2%80%99s-nuclear-policy-lippman/)

But times have changed and Pakistan


is less likely now to undertake such a risky venture as helping
Saudi Arabia develop nuclear weapons. The A.Q. Khan network of off-the-books proliferation,
which supplied Libya and other countries, has been exposed and dismantled. And Pakistan is in
such a state of domestic political upheaval that it seems unlikely that anyone would be willing
or able to authorize such a deal. Some U.S. government officials believe that Pakistan and Saudi
Arabia have an understanding by which Pakistan’s nuclear capabilities would be made available
on demand to Saudi Arabia if the Saudis found themselves in extremis , a guarantee purchased,
in effect, by Saudi funding of Pakistan’s nuclear program. No known evidence supports this
theory and some experts openly discount it. Among them is Gary Samore, a long-time student of Saudi Arabian
security policy who is now the senior arms control and nonproliferation specialist at the National Security Council in the Clinton
Administration. “I don’t believe there’s a deal that the Saudis already paid and could take delivery on demand and if I were the
Saudis I wouldn’t trust the Pakistani to deliver on such a deal,” Samore said. “There’s no doubt the Saudis
have delivered a lot of money to Pakistan, and some went to support the nuclear weapons program, but I don’t believe any such
quid pro quo exists. What would be more likely would be that Pakistan would [again] station troops on Saudi soil, and those could
include nuclear-armed forces.” These could be attack aircraft carrying bombs, missile squadrons deploying nuclear-tipped warheads,
or ground troops, such as Pakistan previously sent to Saudi Arabia, now equipped with tactical — as opposed to strategic — nuclear
weapons. But the purpose of Pakistan’s nuclear arsenal is to deter India. Against which potential foe of Saudi Arabia would Pakistan
put its own interests at risk by deploying nuclear weapons in the Arabian Peninsula? The Pakistanis know as well as anyone that the
principal threats to the security and stability of Saudi Arabia come from the turmoil among its neighbors, and against those threats
nuclear weapons have no value. They might actually stir up more trouble than they would alleviate.

No Saudi Prolif (even if Iran develops nukes)

Lippman 11—Thomas W., senior adjunct scholar at the Council on Foreign Relations, August 5th, "Saudi Arabia's Nuclear
Policy", Saudi-US Relations Information Service, 2011, http://www.susris.com/2011/08/05/saudi-arabia%E2%80%99s-nuclear-policy-
lippman/

It is highly unlikely, however, that Saudi Arabia would wish to acquire its own nuclear arsenal or that
it is capable of doing so. King Abdullah’s comments should not be taken as a dispositive statement of considered policy.
There are compelling reasons why Saudi Arabia would not undertake an effort to develop or acquire nuclear weapons, even in
the unlikely event that Iran achieves a stockpile and uses this arsenal to threaten the Kingdom. Money is not an
issue — if destitute North Korea can develop nuclear weapons, Saudi Arabia surely has the resources to pursue such a program.
With oil prices above $90 a barrel, Riyadh is flush with cash. But the acquisition or development
of nuclear weapons
would be provocative, destabilizing, controversial and extremely difficult for Saudi Arabia, and
ultimately would be more likely to weaken the kingdom than strengthen it. The kingdom has
committed itself to an industrialization and economic development program that depends on
open access to global markets and materials; becoming a nuclear outlaw would be fatal to those
plans. Pursuing nuclear weapons would be a flagrant violation of Saudi Arabia’s commitments
under the Nuclear Nonproliferation Treaty (NPT), and would surely cause a serious breach with the United
States. Saudi Arabia lacks the industrial and technological base to develop such weapons on its own.
An attempt to acquire nuclear weapons by purchasing them, perhaps from Pakistan, would launch
Saudi Arabia on a dangerously inflammatory trajectory that could destabilize the entire region,
which Saudi Arabia’s leaders know would not be in their country’s best interests. The Saudis
always prefer stability to turmoil. Their often-stated official position is that the entire Middle
East should become an internationally supervised region free of all [wmd] weapons of mass destruction.

Long timeframe and U.S. diplomacy solves – no technical infrastructure, no country would
refuse the U.S. nuclear umbrella

Jakobsen 10 – Associate Professor at the Department of Political Science, University of Copenhagen (Peter Viggo, Ali Rahigh-
Aghsan, Assistant Professor at the Department of Society and Globalisation, Roskilde University, Autumn, “The Rise of Iran: How
Durable, How Dangerous?” ProQuest)
The danger of an arms race is not imminent, however, as the current
aspirations for nuclear energy in the region will take
10-15 years to fulfill.60 This leaves considerable time for the US to persuade Saudi Arabia, Egypt, and
Turkey not to go nuclear, and the US is in a position to give them an offer they cannot refuse. Since
all these states and the rest of the GCC countries effectively depend upon the US for their security in the interim until they can
develop their own nuclear deterrent, the US can offer to extend its nuclear umbrella to them and provide
continued military, economic, and political support contingent on their not developing nuclear weapons. This
tactic worked in similar circumstances with Taiwan and South Korea during the Cold War, and there is no reason why it should not
work in the Middle East as well.61 The
penalties the US can impose if they renege on this promise are
prohibitive and since none of these countries have an interest in appeasing Iran unless forced to do so for lack of an
alternative, it is hard to see why they should not accept such an offer. It is also inconceivable that
the US would refrain from extending such an offer to its allies given the strategic importance that it
attaches to the region. Since Iran is not capable of hitting the US mainland with nuclear missiles, the US would not have to sacrifice
New York to protect Amman and this makes extended nuclear deterrence inherently more credible in the Middle East than it was in
Western Europe during the Cold War.

No Pakistani nuclear transfers to Saudi Arabia – arrangement is unconfirmed, hurts Pakistan’s


strategic position vis-à-vis India, harsh international backlash, violation of Saudi NPT
commitments, sanctions, Pakistan focus on domestic instability

Kahl 13 – Senior Fellow at the Center for a New American Security and an associate professor in the Security Studies Program
at Georgetown University’s Edmund A. Walsh School of Foreign Service (Colin H., Melissa G. Dalton, Visiting Fellow at the Center for
a New American Security, Matthew Irvine, Research Associate at the Center for a New American Security, February, “If Iran Builds
the Bomb, Will Saudi Arabia Be Next?” http://www.cnas.org/files/documents/publications/CNAS_AtomicKingdom_Kahl.pdf)

A Pakistani Option? Consequently, if Saudi Arabia decides to proliferate in reaction to Iran’s nuclear program, many analysts contend
that it is more likely to develop the technical capability with substantial foreign assistance or seek to acquire a nuclear weapon from
another country, with Pakistan being the most likely source. Islamabad could provide Riyadh with fuel-cycle technology, fissile
materials or other sensitive assistance that might enable the Kingdom to develop weapons in a matter of years, rather than the
decade or longer that it would take Saudi Arabia on its own. It is also possible, and some believe probable, that Pakistan could
provide Saudi Arabia with operational nuclear weapons and delivery systems.121 These claims have been buttressed by
longstanding allegations that Saudi Arabia bankrolled the Pakistani nuclear program and engaged in other forms of sensitive nuclear
cooperation in exchange for a commitment from Islamabad to provide nuclear weapons to the Kingdom in extremis (see Alleged
Saudi-Pakistani Nuclear Cooperation text box). This decades-long nuclear relationship has contributed to persistent claims by
(usually unnamed) Saudi and Western sources that Pakistan would provide Saudi Arabia with a nuclear bomb “the next day” after
Iran becomes a nucleararmed state.122 Some reports even suggest that the Saudi Air Force has a small number of aircraft
permanently stationed in Pakistan to deliver nuclear weapons to the Kingdom on short notice.123 Allegations of a Riyadh-
Islamabad nuclear arrangement remain unconfirmed by publicly available information. But even if such a
deal exists, there are good reasons to believe that neither side would follow through with the
arrangement. For the Saudis, the same disincentives influencing a possible decision to indigenously develop nuclear weapons
would discourage the illicit acquisition of a Pakistani bomb or other sensitive technologies at odds with the Kingdom’s NPT
commitments. Nor is Islamabad likely to provide a weapon or sensitive assistance aimed at rapidly accelerating Saudi nuclear efforts.
Indeed, although considerable attention has been placed on the motivations underlying the Saudi “demand side” of a possible
nuclear transfer deal with Pakistan, the Pakistani “supply side” of the equation is often taken for granted. Yet it is precisely here that
claims of a grand Saudi-Pakistani nuclear conspiracy become particularly tenuous. The rhetoric of an “Islamic bomb”
notwithstanding, Pakistan did not develop its nuclear arsenal to help defend Saudi Arabia or the wider Muslim world. To be sure,
Pakistanis take great pride in being the first Muslim nation to develop nuclear weapons, and the country has long had a special
relationship with the Kingdom, rooted in common strategic interests, Wahhabi religious ties to Pakistan’s Sunni population and
mountains of Saudi cash. In February 2012, after fresh rumors surfaced of a possible nuclear arrangement between Islamabad and
Riyadh, the Pakistani Ambassador to Saudi Arabia even declared that “each Pakistani considers [the] security of Saudi Arabia as his
personal matter,” adding that the Saudi leadership also considered Pakistan and Saudi Arabia to be one country.124 Yet none of this
changes the fact that Islamabad’s nuclear arsenal serves the very specific purpose of countering
archrival India’s nuclear and conventional capabilities, and therefore, Gawdat Bahghat argues, “ Pakistan … is not likely
to ‘sell’ [the bomb] to any other country” in order to advance another objective. 125 Or, put somewhat less
definitively, Pakistan is unlikely to provide or sell nuclear weapons or other sensitive technologies to any other country unless the
strategic imperatives for doing so – especially with regard to balancing India and maintaining relationships with key states – clearly
outweigh the expected costs. With regard to a potential transfer of operational weapons to Saudi Arabia, they do not. There is no
Iran-centric strategic rationale for Pakistani leaders to transfer nuclear weapons to the Kingdom. Islamabad competes with Tehran
for influence in Afghanistan, and Sunni-majority Pakistan shares the view of its longtime Saudi ally that a nuclear-armed Iran would
be more assertive in promoting radical Shia ideology and militancy throughout the Middle East and Central Asia. Pakistan also
worries that India intends to forge a closer strategic relationship with Iran, contributing to Pakistan’s encirclement. For all these
reasons, Islamabad has cooperated with Riyadh to minimize Iranian influence in the region.138 However, despite a shared land
border with Iran, Pakistani leaders do not view Tehran as a direct or existential security threat. Moreover, if Iran crosses the nuclear
threshold, Islamabad already possesses a nuclear deterrent to neutralize the threat; providing nuclear weapons to the Kingdom
would not make the Pakistani homeland more secure. Given the estimated size of Pakistan’s current nuclear arsenal, it is also not
clear whether Islamabad has sufficient weapons to spare, at least in the near future. The country is estimated to possess
approximately 100 nuclear warheads, which it likely can deliver via F-16 and Mirage V aircraft, and solid- and liquid-fuelled ballistic
missiles. Pakistan is also in the process of significantly expanding its nuclear and ballistic missile arsenal, shifting from highly enriched
uranium-based weapons to plutonium-based weapons, and Islamabad has refused to sign the Fissile Material Cutoff Treaty for this
reason.139 Some suggest that this will easily provide “spare” weapons, including older uranium devices, which would enable a
Pakistani sale or transfer to Saudi Arabia.140 Although conceivable, this should not be taken as a given. Pakistan’s race to acquire
more nuclear weapons is a result of the Pakistani leadership’s deep anxiety over maintaining even a “minimal deterrent” in the face
of Indian plans to increase their stock of nuclear materials and weapons. Islamabad also sees a larger nuclear arsenal as essential to
check India’s conventional modernization efforts – including New Delhi’s growing air, sea and missile capabilities; emerging space-
based systems; and ballistic missile defenses – as well as the emergence of India’s “Cold Start” doctrine, which envisions the rapid
defeat of Pakistani forces. In this context, giving the Saudis a portion of the Pakistani nuclear stockpile
anytime soon would probably aggravate, not alleviate, Islamabad’s perceived strategic dilemma vis-à-vis
India.141 Of course, much would depend on how many weapons the Saudis required. If Riyadh only asked for a handful of
symbolic weapons, the net risk to India-related equities might be small, especially as Islamabad’s stockpile grows. However, if the
Kingdom requested sufficient nuclear weapons to ensure a viable second-strike capability against a nuclear-armed Iran – which
seems more likely than being satisfied with a symbolic force – the potential trade-off of a transfer with Islamabad’s quest to attain a
minimal deterrent against New Delhi would be more acute. Additionally, Pakistani leaders would have to factor in
the likely international response. A transfer of nuclear weapons or other sensitive nuclear technology to Saudi Arabia would
undoubtedly produce a very harsh response from the United States and other Western countries. Although Pakistan is
not an NPT signatory, the transfer of operational nuclear weapons to the Kingdom would likely be seen as one of the most
provocative transactions in history. If the arrangement included the transfer of weapons into the sole control of the Saudis, it
would represent a gross violation of Riyadh’s NPT commitments. Irrespective of legal technicalities, the
United States, European nations and Israel would see any transfer as an extraordinarily dangerous proliferation precedent for the
future, not to mention a potential trigger for an immediate nuclear crisis in the world’s most volatile region. Western countries, and
perhaps other members of the international community, would likely target Pakistan with sweeping
economic and military sanctions – and the Pakistanis have a lot to lose. In Washington, the debate regarding Pakistan’s
ambiguous status as a “friendly” vs. “enemy” state would likely be definitively settled in favor of the latter interpretation, and the
United States would probably terminate over $2 billion in annual U.S. economic and security assistance to
Pakistan.142 Moreover, as Christopher Clary and Mara Karlin note, “if the United States used its leverage at the World Bank or
International Monetary Fund to attenuate the support of those institutions [to Pakistan], Pakistan would be thrown into a major
fiscal crisis.”143 To be sure, previous U.S. attempts to condition Islamabad’s aid have fallen flat.144 However, as U.S. forces
complete their drawdown from Afghanistan, Washington’s willingness to come down hard on Islamabad may increase. Given acute
European concerns over Middle Eastern proliferation and European reactions to Pakistan’s 1998 nuclear tests, European states
would likely to join in whatever harsh measures the United States imposed. The ongoing
domestic instability in Pakistan
and the potential impact of sanctions on the country’s fragile economy 145 make it difficult to
imagine Pakistani leaders risking an international economic backlash by giving nuclear
weapons to Saudi Arabia.146 The broader geopolitical implications for Islamabad could also be significant. In particular,
Pakistani leaders would have to factor in the very real prospect of growing diplomatic isolation and pushing Washington into an even
closer economic and military embrace of India.147 Pakistan’s history of shady nuclear dealings would, paradoxically, make
Islamabad more sensitive to a potential Western backlash. Pakistan acquired its own nuclear weapons capabilities on the grey
market and then allowed those technologies to proliferate to some of the world’s most dangerous regimes via the A.Q. Khan
network. As Feroz Hassan Khan, a former director in the Pakistani Strategic Plans Division,148 notes, this legacy “is a scarlet letter
that Pakistan has been unable to overcome.” 149 However, it is precisely because
of this past behavior – and lingering
concerns in the West – that the Pakistani government has become more concerned about the likely
negative international consequences of further illicit nuclear transfers. Pakistani leaders know that any additional
proliferation in the Muslim world will immediately generate speculation about Islamabad’s involvement. The Pakistani government
has therefore gone out of its way in recent years to demonstrate its commitment to nonproliferation and countersmuggling. Not
only has Pakistan taken aggressive steps to shut down the A.Q. Khan network, but it has also reorganized its security bureaucracy to
tighten control over its nuclear weapons and has placed stringent export controls on technology, material and equipment that might
contribute to designing, developing, stockpiling or using nuclear weapons.150 As a result, there have been no known deliberate
Pakistani transfers of sensitive nuclear technology to other states or foreign actors in recent years, and “there is little risk of a
sudden radical change in Pakistani policy.”151
Saudi US Relations Answers
Relations impact empirically denied

Bronson 6, Rachel, Vice President, Studies at The Chicago Council on Global Affairs [“ 5 Myths About U.S.-Saudi Relations,” May
21st http://www.washingtonpost.com/wp-dyn/content/article/2006/05/19/AR2006051901758.html]

A major reason for the close ties between the two nations was their common Cold War fight against
communism. Both countries worried about the Soviet Union, and that solidified their oil and defense interests, and
minimized differences. In hindsight, by supporting religious zealots in the battle against communism, the two countries
contributed to the rise of radical Islamic movements. 2 The 9/11 hijackers undermined otherwise strong U.S.- Saudi ties. Actually,
things were never that smooth. Historians refer to the "special relationship" established when Saudi Arabia's King Abdel Aziz and
President Franklin D. Roosevelt met in 1945. But since then the relationship has endured oil embargoes , U.S.
restrictions on arms sales to Saudi Arabia, and tensions around Israel and Palestine. Dissension
permeates the entire history of U.S.-Saudi relations. Since the end of the Cold War, relations have become
particularly fraught, with the 9/11 attacks being the most recent issue. Oil, defense and some regional interests keep
the countries together, but both sides have made clear that the relationship is less special today. In 2005, Rice stated that
"for 60 years . . . the United States pursued stability at the expense of democracy in this region here in the Middle East -- and we
achieved neither."

Oil not key

Palmquist 8, citing Rachel Bronson, former director of Middle East studies at the Council on Foreign Relations, vice president
for programs and studies at the Chicago Council on Global Affairs, Matt, graduate of Northwestern University's Medill School of
Journalism [“An Inconvenient Alliance,” August 26th, http://www.miller-mccune.com/politics/the-inconvenient-alliance-4304/]

With gas prices climbing past $4 a gallon, the media, Congress and the public are blaming oil companies and questioning U.S.
energy policy, particularly vis-à-vis Saudi Arabia, the world’s leading oil exporter. And, indeed, the story of the complex
relationship between the United States and the kingdom has in recent years often been viewed through the dark
prism of petroleum. When you scan the rather jaded titles of recent books on the subject — from Sleeping With the Devil: How
Washington Sold Our Soul for Saudi Crude, by former Central Intelligence Agency case officer Robert Baer, to America’s Kingdom:
Mythmaking on the Saudi Oil Frontier, by University of Pennsylvania political scientist Robert Vitalis — the conclusion is plain: Oil is
the binding force in the unique, uncomfortable U.S.-Saudi alliance, which has been so polluted by profit and political quid pro quos
that it’s unclear where the true power lies. In these times of $125-a-barrel oil, though, the more accurate lens for looking at the
Saudi-American relationship may be inside Rachel Bronson’s book, Thicker Than Oil: America’s Uneasy Partnership With Saudi
Arabia, issued in 2006 to short but generally positive reviews and now out in paperback. Bronson, formerly the director of
Middle East studies at the Council on Foreign Relations and now vice president for programs and studies at the Chicago Council on
Global Affairs, presents a comprehensive history of the United States’ far-from-consistent policy toward Saudi Arabia and
argues against the conventional wisdom that oil forms the basis of relations. Instead, Bronson suggests, it was
primarily the shared (and expensive) commitment to resisting communism, whenever and wherever Moscow-
backed beliefs seemed to spread, that aligned America and Saudi Arabia. In Bronson’s analysis, the end of the Cold
War and the disappearance of common communist enemies created a void between the two countries,
which has been increasingly filled and exploited by the forces of religious extremism and terrorism.
Saudi China Relations Answers
China-Saudi rels are high- been stable for decades

Arab News ‘12 [“Working together for brighter future of Saudi-China relations,” http://www.arabnews.com/working-
together-brighter-future-saudi-china-relations]

Over the
This year marks the 22nd anniversary of the establishment of diplomatic relations between China and Saudi Arabia.
past two decades, despite the complex and profound changes of the regional and international situations, China-Saudi
strategic friendly relations maintained the momentum of rapid development, and bilateral
cooperation in various fields has yielded fruitful results. China and Saudi Arabia have had frequent
exchanges of high-level visits, which served to enhance mutual political trust. Both leaders have been
attaching great importance to the development of China-Saudi relationship. With a view to deepening mutually beneficial
cooperation and safeguarding the common benefits of the two peoples, both
countries took full advantage of these
high-level visits to promote mutual understanding and trust, broaden and deepen bilateral
cooperation in all fields and continuously advance China-Saudi relationship to new high .

China-Saudi relations are strong on oil

Denning ‘12 [Dan, editor of The Daily Reckoning Australia and the Publisher of Port Phillip Publishing, 1-16, “The Growing
Energy and Oil Alliance Between China and Saudi Arabia,” http://www.dailyreckoning.com.au/how-the-energy-and-oil-alliance-
between-china-and-saudi-is-growing/2012/01/16/]

The oil and energy alliance between China and Saudi Arabia has just been strengthened. On
Saturday - in the wake of Europe's debt crisis - Saudi state oil company Aramco signed a deal with China's
Sinopec to build an oil refinery in the Red Sea city of Yanbu. The refinery will process 400,000 barrels of oil per day,
some of which will presumably end up in China. "Saudi Aramco will hold a 62.5 percent stake with Sinopec holding the balance in the
venture that highlights China's growing role as an infrastructure developer in the oil rich kingdom," according to AFP. As an intriguing
sidenote, Chinese Premier, Wen Jiabao - who made Saudi Arabia the first stop on his Middle East tour - signed a series of
"agreements and cooperation programs" with Saudi King Abdullah. One of those was, "An agreement between
the two governments on the peaceful usage of nuclear energy was also signed. Signatories for the two parties were Dr. Hashem
Yamani, President of King Abdullah City for Atomic and Renewable Energy, and Chang Peng, President of the National Commission
for Development and Reforms of China." Hmm. Isn't that interesting? This confirms our idea that thanks to America's shale gas
revolution - which makes Middle East oil imports a lot less important to the United States - the
Saudis are shopping for a
new strategic patron in the region and the Chinese are keen to be that patron . Aramco CEO Khalid al-Falih
said the deal "Represents a strategic partnership in the refining industry between one of the main energy
producers in Saudi Arabia and one of the world's most important consumers."
Science Diplomacy Answers
Science diplomacy fails

Dickson 9 – Director of Science Development online (David, “The limits of science diplomacy”,
http://www.scidev.net/en/editorials/the-limits-of-science-diplomacy.html)

Using science for diplomatic purposes has obvious attractions and several benefits. But there are limits to
what it can achieve. The scientific community has a deserved reputation for its international perspective — scientists often ignore
national boundaries and interests when it comes to exchanging ideas or collaborating on global problems. So it is not surprising that
science attracts the interest of politicians keen to open channels of communication with other states. Signing agreements on
scientific and technological cooperation is often the first step for countries wanting to forge closer working relationships. More
significantly, scientists have formed key links behind-the-scenes when more overt dialogue has been impossible. At the height of the
Cold War, for example, scientific organisations provided a conduit for discussing nuclear weapons control. Only so much science can
do Recently, the Obama administration has given this field a new push, in its desire to pursue "soft diplomacy" in regions such as the
Middle East. Scientific agreements have been at the forefront of the administration's activities in countries such as Iraq and Pakistan.
But — as emerged from a meeting entitled New Frontiers in Science Diplomacy, held in London this week (1–2 June) — using
science for diplomatic purposes is not as straightforward as it seems. Some scientific collaboration clearly demonstrates
what countries can achieve by working together. For example, a new synchrotron under construction in Jordan is rapidly becoming a
symbol of the potential for teamwork in the Middle East. But
whether scientific cooperation can become a
precursor for political collaboration is less evident . For example, despite hopes that the Middle East
synchrotron would help bring peace to the region, several countries have been reluctant to
support it until the Palestine problem is resolved. Indeed, one speaker at the London meeting (organised by the
UK's Royal Society and the American Association for the Advancement of Science) even suggested that the changes scientific
innovations bring inevitably lead to turbulence and upheaval. In such a context, viewing science as a driver for peace may be wishful
thinking. Conflicting ethos Perhaps the most contentious area discussed at the meeting was how science diplomacy can frame
developed countries' efforts to help build scientific capacity in the developing world. There is little to quarrel with in collaborative
efforts that are put forward with a genuine desire for partnership. Indeed, partnership — whether between individuals, institutions
or countries — is the new buzzword in the "science for development" community. But true
partnership requires
transparent relations between partners who are prepared to meet as equals. And that goes
against diplomats' implicit role: to promote and defend their own countries' interests. John
Beddington, the British government's chief scientific adviser, may have been a bit harsh when he told the meeting that a diplomat is
someone who is "sent abroad to lie for his country". But he touched a raw nerve. Worlds apart yet co-dependent The truth is
that science and politics make an uneasy alliance. Both need the other. Politicians need science to achieve their
goals, whether social, economic or — unfortunately — military; scientists need political support to fund their research. But they
also occupy different universes. Politics is, at root, about exercising power by one means or
another. Science is — or should be — about pursuing robust knowledge that can be put to
useful purposes. A strategy for promoting science diplomacy that respects these differences deserves support. Particularly so if
it focuses on ways to leverage political and financial backing for science's more humanitarian goals, such as tackling climate change
commitment to science diplomacy that ignores the differences — acting
or reducing world poverty. But a
for example as if science can substitute politics (or perhaps more worryingly, vice versa), is dangerous. The
Obama administration's commitment to "soft power" is already faltering. It faces challenges ranging
from North Korea's nuclear weapons test to domestic opposition to limits on oil consumption. A taste of reality
may be no bad thing.

Future crises make it impossible


Dickson 11 – Director of Science Development online (David, “Science diplomacy: easier said than done”,
http://scidevnet.wordpress.com/2010/06/24/science-diplomacy-easier-said-than-done/)

But, as rapidly become clear in the opening session of the three-day meeting onscience diplomacy being held at Wilton Park
in Sussex, UK, putting the principle of such diplomacy into action presents many practical problems, some of which
SciDev.Net aired last week (see Science diplomacy must be more ambitious). As several participants pointed out, this is
particularly the case at a time when science budgets are under pressure, and scientists are being
asked to justify their support from the public purse in terms of the practical contributions they make to
national – rather than international – well-being. The dilemma was highlighted by the very first speaker at the meeting,
Peter Fletcher, chair of panel that seeks to co-ordinate the international activities of Britain’s research councils. Fletcher outlined the
many ways in which science can be effectively used as a diplomatic tool. He pointed out, for example, that scientific cooperation
offered countries such as Britain an opportunity to establish good relations with the Muslim world in just the same way that it had
helped them build bridges with China in the 1990s. “Science is a way of building relationships, sometimes even before politicians
have agreed to talk.” Fletcher said. “Researchers are used to working across national boundaries. They understand people who are
thinking about the same things as they are, and are used to working together in ways in which other people are not.” But he also
pointed out that, with the UK having just announced a 25% reduction in its science budget,
governments were
increasingly requiring scientists to demonstrate the value of their work for those who paid for it.
“How much are we prepared to commit to solving global challenges for mutual benefit [in this context]?”
he asked. Other challenges were highlighted by Vaughan Turekian, director of the Center for Science Diplomacy, American
Association for the Advancement of Science (AAAS), Washington DC Turekian pointed out that part of the attraction of using science
for diplomatic purposes was its apolitical nature. In addition, the United States, for example, was well placed to exploit the fact that
its science was held in much higher regard around the world that many of its other activities. He quoted a recent visit to Syria by a
US scientific delegation that had met with President Assad – an ophthalmologist – as an example of how science diplomacy could
help promote political engagement in situations where official relations were limited. “Science cooperation has provided a
barriers that
wonderful way to have a dialogue on issues of mutual interest,” Turekian said. But he also pointed to some of the
prevent science diplomacy from operating effectively, such as asymmetries in scientific
capabilities, economic or security concerns over providing access to certain types of key
technologies, and a general lack of funding. In the discussion that followed, it became clear that these barriers
are likely to become an important focus of attention over the next two days. Several participants, for example, pointed to the
obstacles to international scientific exchange presented by the increasing restrictions on entrance visas being
placed by countries such as the United States. “It becomes so difficult for someone to get into the US
that once they are there, they cannot afford to go home, even for a short visit, because they have no idea
whether they will be able to get back in,” was one typical comment. Others pointed to the broader issue of an
apparent conflict between the supposed goal of science to promote international interests, and the goal of
diplomacy, namely to advance the national interests of the country that the diplomat is serving. There has been
much talk of the need to find a way of achieving a balance between these two tendencies. Reaching agreement on where that
balance should lie is a major challenge. Achieving that balance will be even harder. Alread y it is clear from this meeting that
science diplomacy is easier said than done.

Cooperation inevitable

Leifert and Wagner, ‘8 (Harvey, American Geophysical Union Public Information Manager) and Caroline S. Wagner
(Researcher at the Center for International Science and Technology Policy @ George Washington U), 1/16/2008. Article by Leifert,
Cites Wagner, “Author Caroline Wagner Urges More Inclusive Global Science Cooperation”, American Association for the
Advancement of Science, http://www.aaas.org/news/releases/2008/0116stls_wagner.shtml)
Wagner told the participants that science has evolved considerably since the era of the original Invisible
College. She cited the rise of professionalism in the 18th century, the expansion of distinct scientific disciplines in the 19th
century, and the era of big, nationalistic science in the 20th century, when some scholars suggested that 80 to 90% of all scientists in
human history (up until that time) lived. National science reached its epitome during the Cold War, she
said, when the U.S. and Soviet Union built redundant scientific systems on a competitive basis .
This national innovation system, which operated through the 1990s, has become dysfunctional from the
point of view of the governance of science , Wagner said, because knowledge creation does not honor
national borders. In the 21st century, science mostly self-organizes, she emphasized, including and
most influentially on the international level. It functions through networks that are generally
informal, she said, but they do have structure, norms, and rules that scientists must understand if they are to participate
successfully. Although large, government-funded, international science projects are highly visible,
they are just the tip of an iceberg in terms of overall international scientific activities , Wagner said;
they are not in fact typical of international collaboration. "The bulk of international collaboration
in science and technology happens below the waterline ," she told the seminar, as scientists organize
themselves into teams and conduct research of mutual interest, without regard to national
boundaries or government agencies. Wagner identified four kinds of collaborative scientific activities, based on a
combination of how projects are initiated and where they take place. In the first category, she described top-down and bottom-up
projects: * Top-down science includes large, directed programs , usually based in an office that one can visit,
such as NASA programs, CERN, and polar research. * Bottom-up science is based on the interests
of individual researchers who contact colleagues all over the world to work ad hoc on a self-
generated project; their partnerships are often invisible to outsiders . In the second category, there are
centralized and distributed projects: * Centralized projects depend on specialized laboratories or
installations, such as the earthquake shake table in Japan, or the Rain Forest Research Institute
in Costa Rica. * Distributed projects take place all around the world by individuals or small
teams, such as happened in the Human Genome Project ; they do not depend on large facilities. Each area of
scientific research combines one aspect of each of the two categories. Policy-makers must understand each of the resulting four
types of international collaboration in their efforts to assure that taxpayers, who fund most of the research, get their money's worth
and that the knowledge produced "comes home" and is usable at the local level, Wagner said. Science operates on a reward system,
in which researchers are seeking to enhance their reputation and gain recognition from their colleagues in their fields, their own
countries and worldwide. As a result, Wagner said, in many developing countries where she has worked, government officials
complain that local scientists are better connected with colleagues abroad than with the needs of their own society. For example,
she was amazed at how much "great science" is being conducted in ethnobotany in Mozambique—and how little it impacts
Mozambique. Mozambican scientists are working with colleagues at Stanford University in the U.S., but their research has little
impact on local farmers. The scientists are not against improving the lives of their countrymen, Wagner said, but they realize that
"the more you gain recognition within science, the more you are able to access the resources of science. And ultimately,
scientists the world over are seeking freedom—the freedom to pursue their own interests ."
Wagner gave the example of the Department of Energy's Center for Nanoscale Materials at its Argonne National
Laboratory in Illinois. Argonne had recently hired 15 scientists recruited on a worldwide basis for this
cutting edge nanotechnology laboratory. They were seeking the best scientists they could find,
regardless of nationality, and 10 of the 15 chosen were from outside the United States , Wagner
said. This select group includes Yugang Sun, a 29-year-old Chinese postdoctoral scientist who, in his first year at Argonne, published
a seminal paper that has already been cited in the literature over 1,500 times. The Department of Energy offered Sun additional
laboratory resources he needed to continue his research, Wagner said, "but these people are free agents; they can go anywhere."
Sun is, at 29, "an unbelievable superstar in nanosciences," she said, and is—so far—still at Argonne.

Empirics prove – politics precede


Dickson 10 – Director of Science Development online (David, June 28, “Science in diplomacy: ‘On tap but not on top’”,
http://scidevnet.wordpress.com/2010/06/28/the-place-of-science-in-diplomacy-%E2%80%9Con-tap-but-not-on-top%E2%80%9D/)

The broadest gaps in understanding the potential of scientific diplomacy lay in the third
category, namely the use of science as a channel of international diplomacy, either as a way of
helping to forge consensus on contentious issues, or as a catalyst for peace in situations of
conflict.On the first of these, some pointed to recent climate change negotiations , and in particular the work of the
Intergovernmental Panel on Climate Change, as a good example, of the way that the scientific community can provide a strong rationale for joint
international action.But others
referred to the failure of the Copenhagen climate summit last December
to come up with a meaningful agreement on action as a demonstration of the limitations of this
way of thinking.It was argued that this failure had been partly due to a misplaced belief that scientific
consensus would be sufficient to generate a commitment to collective action, without taking
into account the political impact that scientific ideas would have .Another example that received considerable
attention wasthe current construction of a synchrotron facility SESAMEin Jordan, a project that is already is bringing together researchers in a range of
scientific disciplines from various countries in the Middle East (including Israel, Egypt and Palestine, as well as both Greece and Turkey). The

promoters of SESAME hope that – as with the building of CERN 60 years ago, and its operation as a research centre involving, for
example, physicists from both Russia and the United States – SESAME will become a symbol of what regional

collaboration can achieve. In that sense, it would become what one participant described as a “beacon
of hope” for the region.But others cautioned that, however successful SESAME may turn out to be in
purely scientific terms, its potential impact on the Middle East peace process should not be
exaggerated.Political conflicts have deep roots that cannot easily be papered over,however
open-minded scientists may be to professional colleagues coming from other political
contexts.Indeed, there was even a warning that in the developing world, high profile scientific
projects, particular those with explicit political backing, could end up doing damage by
inadvertently favouring one social group over another.Scientists should be wary of having their
prestige used in this way; those who did so could come over as patronising, appearing unaware
of political realities.Similarly, those who hold science in esteem as a practice committed to
promoting the causes of peace and development were reminded of the need to take into
account how advances in science – whether nuclear physics or genetic technology – have also
led to new types of weaponry. Nor did science automatically lead to the reduction of global inequalities.“Science for diplomacy”
therefore ended up with a highly mixed review. The consensus seemed to be that science can prepare the ground

for diplomatic initiatives – and benefit from diplomatic agreements – but cannot provide the
solutions to either.

Science diplomacy fails – it doesn’t unilaterally resolve issues

Dickson 9 (David, Director at SciDev.Net, “The Limits of Science Diplomacy” SciDev.Net June 2009)

Recently, the Obama administration has given this field a new push, in its desire to pursue "soft diplomacy" in regions such as the
Middle East. Scientific agreements have been at the forefront of the administration's activities in countries such as Iraq and Pakistan.
But — as emerged from a meeting entitled New Frontiers in Science Diplomacy, held in London this week (1–2 June) — using science
for diplomatic purposes is not as straightforward as it seems. Some scientific collaboration clearly demonstrates what countries can
achieve by working together. For example, a new synchrotron under construction in Jordan is rapidly becoming a symbol of the
potential for teamwork in the Middle East. But whether scientific cooperation can become a precursor for
political collaboration is less evident. For example, despite hopes that the Middle East synchrotron
would help bring peace to the region, several countries have been reluctant to support it until
the Palestine problem is resolved. Indeed, one speaker at the London meeting (organised by the UK's Royal Society and
the American Association for the Advancement of Science) even suggested that the changes scientific
innovations bring inevitably lead to turbulence and upheaval . In such a context, viewing science
as a driver for peace may be wishful thinking .

Realism means science diplomacy is irrelevant

Dickson 9 (David, Director at SciDev.Net, “The Limits of Science Diplomacy” SciDev.Net June 2009)

Perhaps the most contentious area discussed at the meeting was how science diplomacy can
frame developed countries' efforts to help build scientific capacity in the developing world . There
is little to quarrel with in collaborative efforts that are put forward with a genuine desire for partnership. Indeed, partnership —
whether between individuals, institutions or countries — is the new buzzword in the "science for development" community. But
true partnership requires transparent relations between partners who are prepared to meet as
equals. And that goes against diplomats' implicit role: to promote and defend their own
countries' interests. John Beddington, the British government's chief scientific adviser, may have been a bit harsh
when he told the meeting that a diplomat is someone who is "sent abroad to lie for his country".
But he touched a raw nerve.

Science diplomacy alone can’t solve conflict

Dickson 9 (David, Director at SciDev.Net, “The Limits of Science Diplomacy” SciDev.Net June 2009)

The truth is that science and politics make an uneasy alliance. Both need the other. Politicians need science to achieve their goals,
whether social, economic or — unfortunately — military; scientists need political support to fund their research. But they also
occupy different universes. Politics is, at root, about exercising power by one means or another. Science is — or should be — about
pursuing robust knowledge that can be put to useful purposes. A
strategy for promoting science diplomacy that
respects these differences deserves support . Particularly so if it focuses on ways to leverage
political and financial backing for science's more humanitarian goals , such as tackling climate change or
reducing world poverty. But a commitment to science diplomacy that ignores the differences — acting
for example as if science can substitute politics (or perhaps more worryingly, vice versa), is
dangerous. The Obama administration's commitment to "soft power" is already faltering . It faces
challenges ranging from North Korea's nuclear weapons test to domestic opposition to limits on oil consumption. A taste of reality
may be no bad thing.
Ship Building Answers
The plan is too slow to solve shipbuilding- the industry is cyclical and slow-reacting

Petters ’12 [Mike Petters is president and chief executive officer of Huntington Ingalls Industries (HII), America's largest military
shipbuilder, “How long would it take the shipbuilding industry to grow capacity and throughput if the nation faced a naval crisis or
conflict?” June 5, http://www.informationdissemination.net/2012/06/how-long-would-it-take-shipbuilding.html]

The fact is the


shipbuilding industry is not designed to respond rapidly to a crisis. Whatever the
conflict, the nature of our business dictates that we play with the team we’ve got .¶ Ours is a
business of anticipation, not reaction. There is nothing magical about it. To meet tomorrow’s crisis or conflict requires
continuous investment today to ensure we can deliver capability critical to our nation and economic security. It requires that we
have robust, stable and efficient operations and a supplier base that allow us to build ships as quickly and as affordably as possible. I
know one thing for certain: Further budget cuts in defense could have a potentially devastating impact to our healthy but fragile
industrial base.¶ The shipbuilding business operates on extended cycles. Ships take years to
construct. For example, it takes eight years to build a nuclear-powered aircraft carrier. A stable,
strategy-driven shipbuilding plan is crucial. We are sizing ourselves today to support the Navy's
30-year shipbuilding plan. If the Navy decided to double the numbers in the plan—for whatever reason—it
wouldn’t necessarily take twice as many people or twice as long for us to build those ships, but it would take a long time
(on the order of years) for us to hire and train the shipbuilders and create the infrastructure to do it.¶ By
the same token, we can’t cut ships from the 30-year plan and expect to be able to quickly “ramp up”
production years down the road when we decide we need more ships after all. That’s why maintaining the industrial base is so
critical. I look at it in terms of software (hiring, training and retaining the right employees) and hardware (facilities, tools and
equipment).

Shipbuilding industry is resilient and intervening actors will stabilize it - empirics

MTD ’13 [The Maritime Trades Department (MTD) is a constitutionally mandated department of the AFL-CIO, “AMERICA’S
SHIPYARDS,” accessed 1-24-13, http://maritimetrades.org/americas-shipyards/]

Even before the day America proclaimed its independence, the domestic shipbuilding industry has generated
important national economic and security benefits.¶ Leading analysts and defense figures consistently speak about the
irreplaceable role that U.S. shipyards and their workers play in enhancing America’s defense capability. From California to Maine,
from the Great Lakes down to the Gulf of Mexico, U.S. shipyards generate decent employment opportunities with benefits, not to
mention the opportunity for advanced training. These are not just jobs, they are skilled jobs with benefits and futures that help build
and sustain communities.¶ Throughout America’s history, the industry has proven its resiliency ,
adapting from wooden ships with sails to the rise of the steam engine in the 19th century
through its revitalization prior to and during World War II to the present push for double-hull
tankers and barges. Time and time again, the industry has rebounded and led the world .¶ In the
1950s, American shipbuilders produced the first nuclear powered commercial vessel. Over the past two decades,
domestic shipbuilding succeeded through joint government-public actions like the one that led
to the renaissance of the historic Philadelphia Shipyard, not to mention innovative programs like
the Title XI Shipbuilding Loan Guarantee, which has been the product of bipartisan efforts to
revive this critically important segment of America’s manufacturing base .
Alt causes to shipbuilding outweigh

Hansen ’12 [Michael, Hawaii Shippers Council, “US-Build requirement for ships: Dilemma for Hawaii, Guam, Alaska, and Puerto
Rico,” 1-26-12, http://www.hawaiifreepress.com/ArticlesMain/tabid/56/articleType/ArticleView/articleId/5974/USBuild-
requirement-for-ships-Dilemma-for-Hawaii-Guam-Alaska-and-Puerto-Rico.aspx]

Inefficiency is the principal reason the major U.S. shipyards are not competitive. There are
management and facilities issues. The U.S.-Build requirement is highly protectionist and
prevents competitive pressures from being brought to bear on the major U.S. shipyard managements and trade unions
to achieve the efficiencies of the international yards. Most yards are saddled with legacy labor contracts and
managements often deal with multiple unions in the same yard organized along craft lines. The
yards themselves also tend to be inefficient older facilities that haven’t been fully upgraded as their
counterparts in East Asia and Europe.

Naval dominance is overpowering- will survive far into the future

Work ‘12 [Robert O. Work, United States Under Secretary of the Navy, “The Coming Naval Century,”
http://www.usni.org/magazines/proceedings/2012-05/coming-naval-century]

The Navy and Marine Corps will be the long arm of a National Fleet central to U.S. military
power.¶ With the protracted campaigns in Iraq and Afghanistan either already complete or winding down, 25 years of sustained,
high-tempo military activity will soon come to a close. Frankly, our all-volunteer joint force, which has given so much to the nation, is
tired. The force—its platforms, equipment, and personnel, including families—needs time to recover, reset, reconstitute, and
prepare for future challenges.¶ This period of reconstitution and transition will be made more challenging due to the fiscal
circumstances in which we now find ourselves. President Barack Obama has developed a ten-year plan to bring our growing deficits
under control and to renew our economy. In preparing and outlining this plan, the President has emphasized the inextricable
connection between our national security and our economy, and the fact that our economic prosperity provides the foundation for
all elements of our national power—including the military. The President’s plan therefore calls for spending cuts across all national
programs, and the Department of Defense must and will contribute its fair share.¶ For those in the military concerned about the
impact of such cuts, I would simply say four things:¶ • Any grand strategy starts with an assumption that all resources are scarce,
requiring a balancing of commitments and resources. As political commentator Walter Lippmann wrote: “The nation must maintain
its objectives and its power in equilibrium, its purposes within its means, and its means equal to its purposes.” ¶ • The
upcoming defense drawdown will be less severe than past post–World War II drawdowns.
Accommodating cuts will be hard, but manageable.¶ • At the end of the drawdown, the United States
will still have the best and most capable armed forces in the world . The President well appreciates the
importance of a world-class military. “The United States remains the only nation able to project and sustain
large-scale military operations over extended distances,” he said. “We maintain superior
capabilities to deter and defeat adaptive enemies and to ensure the credibility of security
partnerships that are fundamental to regional and global security . In this way our military
continues to underpin our national security and global leadership, and when we use it
appropriately, our security and leadership is reinforced .”¶ • Most important, as the nation prioritizes what is
most essential and brings into better balance its commitments and its elements of national power, we will see the beginning of a
Naval Century—a new golden age of American sea power. ¶ Those who judge U.S. naval power solely by the
number of vessels in the Navy’s battle force are not seeing the bigger picture. Our battle force is
just one component—albeit an essential one—of a powerful National Fleet that includes the broad range of capabilities,
capacities, and enablers resident in the Navy, Marine Corps, and Coast Guard. It encompasses our special-mission,
prepositioning, and surge-sealift fleets; the ready reserve force; naval aviation , including the
maritime-patrol and reconnaissance force; Navy and Marine special operations and cyber
forces; and the U.S. Merchant Marine . Moreover, it is crewed and operated by the finest sailors ,
Marines, Coast Guardsmen, civilian mariners, and government civilians in our history , and supported
by a talented and innovative national industrial base.¶ If this were not enough, the heart of the National Fleet is a Navy–Marine
Corps team that is transforming itself from an organization focused on platforms to a total-force battle network that interconnects
sensors, manned and unmanned platforms with modular payloads, combat systems, and network-enabled weapons, as well as tech-
savvy, combat-tested people into a cohesive fighting force. ThisFleet and its network would make short work of any past U.S.
Fleet—and of any potential contemporary naval adversary.¶ It will only get better for the Navy–Marine Corps
team. In a seminal essay in the May 1954 issue of the U.S. Naval Institute Proceedings , “National Policy and the Transoceanic
Navy,” political scientist Samuel P. Huntington divided the history of American national security policy into three distinct phases—
the earlier Continental and Oceanic phases and the emerging Eurasian one. He argued that the service with the strategic concept
and organizational structure most able to answer the dominant national-security challenges of the two earlier policy eras was the
one most rewarded when it came time to allocate the country’s scarce national resources. ¶ Huntington specified that if the Navy–
Marine Corps team expected its claims for resources to be answered in the new Eurasian phase, it would need to change its strategic
concept and organizational structure. This was well suited to the previous Oceanic era, but not to the contemporary problem of
containing a continental peer located across the world’s oceans. In short, Huntington’s basic argument was that the service with a
strategic concept and organization best aligned with the country’s national-security policy would reap the benefits in terms of
strategic prestige and resources. Without a doubt, the
Navy–Marine Corps team’s strategic vision is very well
aligned with the new strategic guidance for sustaining U.S. global leadership, and with the
attendant priorities for 21st-century defense .

Alt cause to shipbuilding- skilled worker shortage

Mitchell ’12 [Ellen, Press-Register, “Gulf Coast shipbuilders struggle to replace aging workforce,” August 4,
http://www.jobkeeperalliance.org/2012/08/gulf-coast-shipbuilders-struggle-to-replace-aging-workforce/]

As John Lotshaw sees it, there’s a difficult job ahead of him.¶ The director of operations, workforce training and development at
Huntington-Ingalls Industries, the parent company of Ingalls Shipbuilding in Pascagoula, has to hire up to 2,000 new workers for the
shipyard in the next 12 months.¶ It’s not new work behind the number of positions available, rather, it’s older
workers who
are retiring and leaving unfilled spaces behind in an industry where the average age of the
workforce just keeps climbing.¶ “The ability to support the industry base for those skill sets at
this time is very concerning,” said Vic Rhoades, director of BAE Systems Southeast Shipyards Alabama in Mobile.
“The industry really hasn’t done a good job in getting the word out to younger generations about
this industry. As a result, those of us who are baby boomers want our kids to go to college.¶ “Young people don’t see
this as a viable career path.Ӧ Many in the industry cite a weak educational infrastructure and
lack of advertising as the biggest hurdles in trying to get the attention of young workers. The average age of an
employee at a major shipyard in southern Alabama or Mississippi is mid-forties.¶ “We have to make sure people understand the full
picture of what we’re offering,” Lotshaw said. “There’s a very clear and proven career path in shipbuilding that leads to a very good,
achievable, transferable career in the manufacturing industry. It is a skill set that a person can fall back on no matter what he does
for the rest of his life. We need to make sure we clearly explain that case to government officials and schools.”¶ The nation’s
commercial shipyards employ more than 50,000 workers to build and maintain non-Navy vessels, according to the Shipbuilders
Council of America, the national trade association representing the U.S. shipyard industry.¶ Include the workers building ships for
the U.S. Navy, and the number exceeds 100,000.¶ Lotshaw estimates thousands of those workers along the Gulf Coast will retire
within the next decade. Adding to the headache of Baby Boomers leaving the manufacturing workforce, skilled craftsmen are
already in short supply, he said.¶ Large shipyards such as Ingalls and Mobile’s Austal USA and BAE Systems Southeast Shipyards
Alabama have turned to their own, sometimes extensive, internal training to combat the diminishing number of skilled craftsman in
the workforce.¶ “If you talk to people in the shipbuilding industry, this problems exists everywhere —
finding skilled workers,” Lotshaw said. “It’s broader than just the maritime industry. With contraction in industry and lack of
incentive for people to enter the craft for a number a years, the result has been a stagnant workforce.Ӧ Ingalls employs about 9,500
people in Pascagoula and 500 in Gulfport, building destroyers and amphibious transport ships for the Navy and national security
cutters for the Coast Guard. Lotshaw said the average age of the Ingalls Mississippi workforce is about 43.¶ He said the company
finds workers through an active partnership with the Mississippi community college system as well as a broader network of technical
schools as far away as Tulsa, Oklahoma, and Jacksonville, Florida. Ingalls also brings in up to 2,000 people a year through its own
internal training program.¶ “Because some of the skill sets for shipbuilding are shipbuilding unique, we’ve worked with the schools
to develop the right kind of curriculum,” he said. “We have a need for about 2,000 employees a year.”¶ Ingalls is also anticipating
the completion of the Haley Reeves Barbour Maritime Training Academy training facility, set to be finished in March in Pascagoula.
The $20 million, 76,000-square-foot academy is funded through Hurricane Katrina recovery money and will help Ingalls more than
double its two- to four-year apprenticeship program from the 300 students it currently has.¶ BAE, one of the largest defense
contractors in the world, has 650 full-time employees in Mobile and is in the midst of ramping up to 800 for its mostly commercial
orderbook. Rhoades said only 14 percent of its workforce in Mobile is under 30, with the average employee age around 45.¶
Rhoades said the hardest skills to find are pipe fitters and welders, a problem that is not unique to the Gulf Coast. He said BAE
invests heavily in workforce development programs and on-the-job training to grow its own stock of workers, a practice that’s one of
the most important aspects of the business.

Alt cause to shipbuilding- the Jones Act makes any new efforts too expensive

Hansen ’12 [Michael, Hawaii Shippers Council, “US-Build requirement for ships: Dilemma for Hawaii, Guam, Alaska, and Puerto
Rico,” 1-26-12, http://www.hawaiifreepress.com/ArticlesMain/tabid/56/articleType/ArticleView/articleId/5974/USBuild-
requirement-for-ships-Dilemma-for-Hawaii-Guam-Alaska-and-Puerto-Rico.aspx]

A dilemma is facing the domestic noncontiguous jurisdictions of the United States – Alaska, Guam, Hawaii and Puerto Rico – in
regards to interstate surface transportation. On the one hand, their geography imposes an exclusive reliance on ocean shipping and
federal maritime laws require vessels engaged in the noncontiguous trades be built in the United States. While on the other hand,
the prohibitively high cost of new construction at the major shipbuilding yards on the United States mainland nearly precludes all
new ship orders. This situation has resulted in an ageing noncontiguous trade fleet of oceangoing deep draft ships that now averages
over 28 years old; making ship replacement a critical economic issue for the noncontiguous jurisdictions.¶ The
navigation
laws of the United States, popularly known as the Jones Act, require that vessels transporting cargo
and passengers between domestic points must be U.S.-Flag, U.S.-Crewed, U.S.-Owned and U.S.-
Built. These requirements pertain to all the domestic noncontiguous jurisdictions with the single
exception of the application of the U.S.-Build requirement to the Territory of Guam . Although exempt
from the U.S.-Build requirement, Guam obtains very little advantage from this more liberal provision of the law. This is primarily
because the Westbound domestic container trade to Guam depends on its linkage to Hawaii, which requires U.S.-Built ships, to be
operated successfully.¶ The Jones Act requirements most acutely affect the domestic noncontiguous jurisdictions because they have
no surface transportation alternatives to ocean shipping. In comparison, on the contiguous United States, railroad, inland barge,
pipeline and road truck carriage is readily available. The high cost of domestic ship construction coupled to the availability of these
alternatives has resulted in a limited number of oceangoing ships (as opposed to tugs and barges) engaged in the domestic coastal
and inter-coastal trades. Virtually all the deep draft ships in the coastal trades are product tankers that carry distillate fuels such as
gasoline and diesel fuel. In contrast, approximately half of the self-propelled domestic oceangoing fleet – largely containerships -- is
employed in the relatively small noncontiguous trades, making those jurisdictions uniquely sensitive to the economics of deep draft
commercial U.S. shipbuilding.¶ The ability to access new ships at a reasonable cost profoundly affects the capacity of ship operators
to provide shipping services. Clearly, the acquisition cost of a ship is a key component of any shipping company’s capital structure.
However, it is also essential for operators to regularly replace their fleets at realistic intervals with technically modern ships at
internationally competitive prices. New ships allow operators to keep abreast of new designs and technology, avoid higher operating
and maintenance costs incurred by older ships, and achieve the necessary efficiencies to charge reasonable freight rates and provide
adequate service levels.¶ The major U.S. shipbuilding yards do not deliver new ships meeting these
specifications because they have become uncompetitive under the protectionist shield of the
Jones Act. The cost of building large oceangoing ships in the United States is at least three times
greater than at the internationally competitive shipyards in Japan and South Korea. The process
of contracting for a commercial oceangoing ship from a major U.S. shipbuilding yard is
cumbersome, fraught with difficulties and subject to delays in delivery and significant cost
overruns. These contracting practices reflect the U.S. shipyards heavy reliance on military construction. While, the few deep
draft commercial ships constructed each year in the United States are typically built under license to a foreign shipbuilder, and not
to a domestic design.
Sinai Answers
No internal link to Sinai – instability there is inevitable but won’t escalate – it’s based
on Israeli-Palestinian conflict
El Amrani ‘11 – Middle Eastern affairs. He is a former North Africa analyst at the International Crisis Group  (Issandr, “THE
SITUATION IN SINAI AND EGYPT-ISRAEL RELATIONS.” http://www.arabist.net/blog/2011/8/24/the-situation-in-sinai-and-egypt-
israel-relations.html)

The Eilat attack: Israel both immediately claimed that the perpetrators of the Eilat had come in from Gaza through Egypt and that
Hamas were responsible for them, although Hamas denies this and Israel presented scant evidence. The Netanyahu government also
used them as a diversion from protests against their economic policy, and used them to justify a new bombing campaign in Gaza
that has already killed at least 15. It might very well be the case that the Eilat attackers came from Gaza into Egypt and then into
Israel — but much of the coverage of the issue suggests this is a new phenomenon due to the situation in Sinai post-revolution. In
fact, previous attacks in Israel’s south-west were probably also conducted via Sinai. So unlike Barry Rubin1 argues, this is not just
“the bitter fruit of the U.S-backed downfall of the government of President Husni Mubarak in Egypt, opening the Egypt-Israel border
as a new front in the war.” Of course, that it’s not the first time is little consolation to Israelis. But it means that
has relatively
little to do with the post-revolutionary situation. Egypt has a long border with Israel that has
been porous to human and drug trafficking for a long time. It has a limited ability to deploy
military personnel and helicopters. And it has a situation with smuggling and other illegal
activities in Eastern Sinai that has been exarcebated by the blockade on Gaza. In other words, the
core problem is not a temporary reduction in Egyptian control of Sinai. It’s the Israeli-Palestinian
conflict, the pressures on neighbors created by the blockade on Gaza, and the international
community’s endorsement of of it.
Soft Power
Frontline

Alt causes swamp soft power effectiveness

Afrasiabi, 7 (PhD and author on Iran (Kaveh, Asia Times, “The illusion of American 'smart power'”
http://www.atimes.com/atimes/Middle_East/IK13Ak02.html)

Over the years, Nye


has been anything but shy about claiming credit for his singular contributions to
the theories of power, yet much of it is undeserved, as any competent sociologist probing the history of thoughts, running
from Max Weber to Antonio Gramsci to Michel Foucault, regarding the subtleties and complexities of power, would readily attest.
Nye's theory is an excellent theory that can never be refuted precisely because it cannot be
pinned down, its core assumptions too nebulous to lend themselves to scientific parsimony . Aside
from contradictory notions and simplistic truisms, eg, "strengthen America" by "bolstering its soft power", the report is distinguished
by its unabashed glorification of the American military - that has "never been put in the service of building a colonial empire in the
manner of European militaries". A little micro-focusing on post September 11, 2001, American interventionism, curiously absent in
the whole report, would arguably lead to a diametrically different conclusion. Too much focus on power actually distracts from
conscious policies. To be sure, the authors of the "smart power" report are not void of praise for European imperialism, particularly
the 19th-century British imperialism that, they claim, contains precious lessons for the "smarter" America of the 21st century. Their
point - about "legitimized British power in the eyes of others" - is clearly Eurocentric and blind to the perception of the colonized
populations who eventually removed the chains one way or another. But that is a separate story. Tightly packed into the report is
the incontrovertible fact that American standing in the world has suffered. Yet, any
report focused on "how America
wields power in the world" that omits a serious consideration of the multiple causes, such as
the American quagmire in Iraq, cannot possibly be taken seriously. The trouble is, however, that both
authors of the report are on record supporting the 2003 invasion, although in fairness to Nye, he did criticize it as the "right war at
the wrong time", and targeted President George W Bush's failure to "neglect of allies and institutions" that have created a "a sense
of illegitimacy". [3] The problem with Nye's approach, however, is the failure to recognize that the "pretextual" war against a
sovereign nation in the Middle East, which bypassed the UN, could not possibly have the required legitimacy even if professor Nye
and his arsenal of "soft power" pills were in order at the White House; in a word, contrary to Nye, it was the wrong war at the wrong
time. Formerly of the US State Department, Francis Fukuyama has agreed that procuring legitimacy has to do with "justice". In other
words, an unjust war cannot be called legitimate no matter what the verbal acrobatics by the likes
of Nye and others, who pay lip service to the "de-legitimating" US Middle East policies , ie, neglecting the
Middle East peace process, mentioned only in passing in the above-said report, without due consideration of the
serious ramification of such neglects with respects to the threats facing the US today . While side-
stepping the Iraq issue with the lame excuse of "broader" perspectives that need to "replace the narrow lens focused on Iraq", the
report gives several other reasons for the waning influence of US, ie, reactions to American-led globalization, US's "angry" response
to September 11, perception of incompetence, and the side-effects of Cold War success as a lone superpower. Here, the authors
conflate the long-term causes of power decline with the negative fall outs of questionable
policies, such as with respect to US unilateralism. Regarding the latter, Robert Jervis has correctly pointed at the
structural causes of American unilateralism, chiefly the absence of external restraints to American power. In comparison, Nye and
Armitage mention other nations resorting to the UN to "constrain" the US power, yet provide no analysis of why the
US has fallen astray from "norm-based internationalism", the fact that it has to do with power
dynamism and America's "totalizing" power grab at the global level , to borrow a term from the French
philosophers Gilles Deleuze and Pierre-Felix Guattari. In light of their benign, tolerant attitude toward the exercise of American
power, even under the Bush administration, which is said to have used "elements of smart power", Nye
and Armitage
never really get to the bottom of their own frank admission that today many nations "resent
US's unbounded dominance". Instead of drawing from this insight the necessity of a multi-polar world order, the report
on "smart power" is keen on maintaining America's "preeminence" in the world and the various ways to ensure it, simultaneously
throwing ideas such as "shared leadership" and "accommodating rising powers". True to its contradictory nature, the report on the
one hand admits that global politics is not a "zero-sum game" and, yet, in the same breath sends the message that "China can only
become preeminent if the US continues to allow its own power of attraction to atrophy". Flawed,
inadequate diagnosis
of the problems behind America's waning influence go hand-in-hand with equally inadequate
prescriptions for a new US foreign policy, no matter how useful the insights on increasing
foreign aid, closing down Guantanamo detention center, focusing on public diplomacy, that is, the usual panoply of
"neo-liberal" recipes for action, with the sole exception of omitting the word "interdependence" previously highlighted in Nye's own
writings. Theserecommendations are not far-reaching enough, often tackling the symptoms
rather than the real causes of problems, overall denoting a mindset that reflects policy
continuity (with the past and the present) when discontinuity should have the upper hands signaling a
real foreign policy reorientation away from the disastrous policies of the Bush presidency . Clearly,
such a reorientation is impossible short of a paradigmatic shift away from the core
assumptions of the American hegemonic model (which are only superficially questioned in this report). Devoid of
such a radical shift, the report's "smart power" has nested in it the elements of a vicious policy circle, bound to reintroduce failed US
policies under new guises.

Alt causes will always come before the plan—world focuses on failures not success

Rachman 9 [Gideon Rachman is the Economist's bureau chief in Brussels, June 1 http://www.ft.com/cms/s/0/e608b556-4ee0-
11de-8c10-00144feabdc0.html]

Barack Obama is a soft power president. But the world keeps asking him hard power questions.
From North Korea to Guantánamo Bay, from Iran to Afghanistan, Mr Obama is confronting a
range of vexing issues that cannot be charmed out of existence. The problem is epitomised by the US
president’s trip to the Middle East this week. Its focal point will be a much-trailed speech in Cairo on Thursday June 4, in which he
will directly address the Muslim world. The Cairo speech is central to Mr Obama’s efforts to rebuild America’s global popularity and
its ability to persuade – otherwise known as soft power. The president has been trying out potential themes for the speech on aides
and advisers for months. He is likely to emphasise his respect for Islamic culture and history, and his personal links to the Muslim
world. He will suggest to his audience that both the US and the Islamic world have, at times, misjudged and mistreated each other –
and he will appeal for a new beginning. George W. Bush launched a military offensive in the Middle East. Mr Obama is launching a
charm offensive. There is plenty to be said for this approach. Mr Bush embroiled America in a bloody war in Iraq that strengthened
Iran and acted as a recruiting sergeant for America’s enemies. Mr Obama’s alternative strategy is based on diplomacy, engagement
and empathy. Mr Bush had a shoe thrown at him in his last appearance in the Middle East. So if Mr Obama receives his customary
standing ovation in Cairo, that will send a powerful symbolic message. But the president should not let the applause go to his head.
Even if his speech is a success, the same foreign-policy problems will be sitting in his in-tray when he gets back to the Oval Office –
and they will be just as dangerous as before. In particular, there is chatter in official Washington that the Israelis may be gearing up
to attack Iran’s nuclear facilities before the end of the year. The Obama administration is against any such move and it is normally
assumed that Israel would not dare to pull the trigger without the go-ahead from Washington – not least because the Israelis would
have to fly across US-controlled airspace to get to their targets. But the Americans do not have a complete veto over Israel’s actions.
One senior US official asks rhetorically: “What are we going to do? Shoot down their planes?” A conflict between Israel and Iran
would scatter the Obama administration’s carefully laid plans for Middle East peace to the winds. It would also make talk of
improving American soft power around the world seem beside the point. The immediate task would be to prevent a wider regional
war. In the meantime, the US will press on with the effort to achieve peace between the Israelis and the
Palestinians. But even that goal is unlikely to be advanced much by Mr Obama’s trip to the Middle East. Many in the
audience in Cairo and in the wider Islamic world will want and even expect the new president to lay out a complete vision for a
peace settlement and to apply unambiguous pressure on Israel. For reasons of domesticpolitics, diplomacy and
timing, Mr Obama is highly unlikely to do this. Yet while his Arab audience may be disappointed by what he has to say
about the Middle East peace process, Mr Obama is already facing an increasingly tense relationship with the new Israeli
government. The administration has now clashed openly with the Israelis over the Netanyahu government’s tolerance of expanded
settlements in occupied Palestinian land. Mr Obama is also running up against the limits of soft power elsewhere. Closing the prison
camp at Guantánamo was meant to be the ultimate tribute to soft power over hard power. The Obama team argued
consistently that the damage that Guantánamo did to America’s image in the world outweighed any
security gains from holding al-Qaeda prisoners there. Yet, faced with the backlash against releasing the remaining 240
prisoners or imprisoning them in the US, the Obama administration has back-tracked. It is not clear whether Guantánamo
will be closed on schedule or what will happen to the riskier-sounding prisoners, who may still be held indefinitely. The much-
criticised military trials are likely to be revived. In Afghanistan, Mr Obama is trying a mixture of hard and soft
power. There will be a military surge – but also a “civilian surge”, designed to build up civil society and governance in Afghanistan.
Old hands in Washington are beginning to shake their heads and mutter about Vietnam. Mr Obama’s preferred tools of diplomacy,
engagement and charm do not seem to be of much use with Kim Jong-il of North Korea, either.
The North Koreans have just tested a nuclear weapon – leaving the Obama administration scratching its head about what to do. The
president’s charisma and rhetorical skill are real diplomatic assets. If Mr Obama can deploy them to improve America’s image and
influence around the world, that is all to the good. There is nothing wrong with trying to re-build American “soft power”. The danger
is more subtle. It is that President Yes-we-can has raised exaggerated hopes about the pay-off from engagement and diplomacy. In
the coming months it will become increasingly obvious that soft power also has its limits.

No impact to soft power—countries will still work with us

Kagan, 6 (Robert, senior fellow at the Carnegie Endowment for International Peace, The Washington Post, 1/15,
http://www.carnegieendowment.org/publications/index.cfm?fa=view&id=17894&prog=zgp&proj=zusr)

The striking thing about the present international situation is the degree to which America remains what Bill Clinton once
called "the indispensable nation." Despite global opinion polls registering broad hostility to George W.
Bush's United States, the behavior of governments and political leaders suggests America's position
in the world is not all that different from what it was before Sept. 11 and the Iraq war. The
much-anticipated global effort to balance against American hegemony -- which the realists have been
anticipating for more than 15 years now -- has simply not occurred . On the contrary, in Europe the idea has all
but vanished. European Union defense budgets continue their steady decline, and even the project of creating a common
foreign and defense policy has slowed if not stalled. Both trends are primarily the result of internal European politics. But if they
really feared American power, Europeans would be taking more urgent steps to strengthen the
European Union's hand to check it. Nor are Europeans refusing to cooperate, even with an
administration they allegedly despise . Western Europe will not be a strategic partner as it was during the Cold War,
because Western Europeans no longer feel threatened and therefore do not seek American protection. Nevertheless, the
current trend is toward closer cooperation . Germany's new government, while still dissenting
from U.S. policy in Iraq, is working hard and ostentatiously to improve relations. It is bending
over backward to show support for the mission in Afghanistan, most notably by continuing to supply a
small but, in German terms, meaningful number of troops. It even trumpets its willingness to train Iraqi soldiers. Chancellor
Angela Merkel promises to work closely with Washington on the question of the China arms embargo, indicating agreement with the
American view that China is a potential strategic concern. For Eastern and Central Europe, the growing threat is Russia, not America,
and the big question remains what it was in the 1990s: Who will be invited to join NATO?

Empirically fails to address specific flashpoints

Greenwald ’10 (Abe is policy adviser and online editor with the Foreign Policy Initiative in Washington, July/August
http://www.commentarymagazine.com/viewarticle.cfm/the-soft-power-fallacy-15466?page=all

Like Francis Fukuyama’s essay “The End of History,” soft-power


theory was a creative and appealing attempt
to make sense of America’s global purpose. Unlike Fukuyama’s theory, however, which the new global order seemed to
support for nearly a decade, Nye’s was basically refuted by world events in its very first year. In the summer of 1990,
a massive contingent of Saddam Hussein’s forces invaded Kuwait and effectively annexed it as a province of Iraq. Although months
earlier Nye had asserted that “geography, population, and raw materials are becoming somewhat less important,” the fact is that
Saddam invaded Kuwait because of its geographic proximity, insubstantial military, and plentiful oil reserves. Despite Nye’s claim
that “the definition of power is losing its emphasis on military force,” months of concerted international pressure, including
the passage of a UN resolution, failed to persuade Saddam to withdraw. In the end, only overwhelming
American military power succeeded in liberating Kuwait. The American show of force also succeeded in establishing
the U.S. as the single, unrivaled post–Cold War superpower. Following the First Gulf War, the 1990s saw brutal acts of
aggression in the Balkans: the Bosnian War in 1992 and the Kosovo conflicts beginning in 1998. These raged on despite
international negotiations and were quelled only after America took the lead in military actions. It is also worth noting that attempts
to internationalize these efforts made them more costly in time, effectiveness, and manpower than if the U.S. had acted unilaterally.
Additionally, the 1990s left little mystery as to how cataclysmic events unfold when the U.S. declines to apply traditional tools of
power overseas. In April 1994, Hutu rebels began the indiscriminate killing of Tutsis in Rwanda. As the violence
escalated, the United Nations’s peacekeeping forces stood down so as not to violate a UN mandate prohibiting intervention in a
country’s internal politics. Washington followed suit, refusing even to consider deploying forces to East-Central Africa. By the time
the killing was done, in July of the same year, Hutus had slaughtered between half a million and 1 million Tutsis. And in the 1990s,
Japan’s economy went into its long stall, making the Japanese model of a scaled down military seem rather less relevant. All this is
to say that during the presidency of Bill Clinton, Nye’s
“intangible forms of power” proved to hold little sway
in matters of statecraft, while modes of traditional power remained as critical as ever in coercing
other nations and affirming America’s role as chief protector of the global order.
---Soft Power- Alt Causes

1NC Afrasiabi—no amount of spin or small victories can overcome American foreign policy
negativity—their analysis is flawed and doesn’t take all factors into account—soft power
theory to vague to overcome structural barriers to success—the aff isn’t far-reaching enough
—total paradigm shift key

Mid East, Russia, China, and demo promo policy all overwhelm

Ford 12 (Christopher, Senior Fellow at the Hudson Institute in Washington D.C., He Previously Served as US Special
Representative for Nuclear Nonproliferation, “Soft On Soft Power” SAID Review, Vol 32, No 1. Winter/Spring)

The Obama Administration then approached Iran’s “Green Revolution” with painful rhetorical
reticence in 2009–2010, sacrificed candor about Russia’s retreat into autocracy on the altar of
an expedient nuclear disarmament-focused “reset” of relations with Moscow, and explicitly
promised not to let human rights concerns “interfere” with America’s economic relationship
with Beijing.13 The American role in promoting democracy in Egypt in 2011 was also for a time
decidedly ambivalent, with U.S. officials still calling for President Hosni Mubarak to stay in office
until just before his resignation.14 After a long period of embarrassing silence in which U.S. officials bizarrely quoted
assessments describing Syrian dictator Bashar al-Asad as a “reformer,”15 the Obama Administration finally spoke out against his
bloody efforts to repress Syria’s pro-democracy movement, ultimately calling on Assad to step down.16 But the contribution of U.S.
pronouncements to effecting change in Syria is, at the time of writing, unclear at best. So far, the
Obama Administration’s
most conspicuous democracy-promotion [End Page 95] effort was a very “hard power” affair:
the war that led to the overthrow and execution of Libyan dictator Muammar Qaddafi.

Misguided policies with Israel, Iran, and Syria

Rubin 12 (Jennifer, Right Turn Blog for the Washington Post, “Obama’s Foreign Policy Flubs” 5/27)

You can look at President Obama’s foreign policy record as a series of discrete failures. Syria’s
butcher Bashar al-Assad is still in place. Iran’s nuclear weapons program is progressing. The “peace process” is in
tatters. China’s military and economic aggression are on the upswing; its human rights abuses are more widespread. Russia’s
thugocracy’s brutality is increasing, and its leaders sound like a bad parody of their Communist predecessors. You can
look at each episode as a unique debacle. For example, Obama’s fixation on Israel’s West Bank settlements
doomed the “peace process.” Alternatively, one can look at similarities among these approaches and reach some
conclusions about the president’s systemic failure to devise a coherent and effective foreign policy. These include over-
reliance on international bodies, failure to link hard and soft power, misdiagnosis of our
opponents’ motives, and marginalization of human rights.
Economics and china outweigh

Layne ‘9 Christopher is Professor, and Robert M. Gates Chair in Intelligence and National Security, at Texas A&M University's
George H.W. Bush School of Government and Public Service, International Security, Summer, The Waning of U.S. Hegemony--Myth
or Reality?; p. 147

Like many U.S. international relations scholars and foreign policy analysts, Zakaria believes that by using its soft power the United States can preserve its "pivotal" status in
international politics. As the NIC and Mahbubani argue, however,soft power may be significantly less potent a force for
bolstering U.S. preponderance than Zakaria (and others believe). This is so for two reasons. First, the
global financial and economic crisis has discredited one of the pillars of U.S. soft power: American
free-market capitalism and, more generally, liberalism itself (economically and institutionally). As former U.S. Deputy Treasury
Secretary Roger Altman puts it, the meltdown has "put the American model of free market capitalism under a cloud." Second, as Mahbubani rightly notes, the

United States is not the only country that possesses soft power. China, especially, has become
increasingly adept in this regard. 66 If China weathers the economic storm better than the United States, it will be in a position to expand its role in the
developing world. Even before the meltdown, China was taking advantage of the United States' preoccupation with the "war on terror" to project its soft power into East and
Southeast Asia. China also is making inroads in Latin America, Africa, and Central Asia, by providing development assistance without strings and increasing its weapons sales.

Similarly, China is using its financial clout to buy up huge quantities of raw materials and natural
resources worldwide, thereby bringing states into its political orbit
---Soft Power- Resilient

1NC Quinn—soft power is resilient—relies of many other factors of power like military and
economy—soft power will always be around as long as America is—proves that if it is in
decline, the aff is too small to solve

Soft power resilient regardless of popularity

Nye 6 – IR Professor, Harvard (Joseph, 6/25, Why Do They Hate Us?, http://www.washingtonpost.com/wp-
dyn/content/article/2006/06/22/AR2006062200972_pf.html)

Fortunately, even when the U.S. government's foreign policies are unattractive to others, our culture and our
open political processes can produce a "meta" form of soft power -- winning grudging admiration for
our freedoms at the same time that our policies are unpopular. After all, anti-American protests were
rampant around the world during the Vietnam War, but the protesters did not sing "The Internationale"; they sang
the American civil rights anthem "We Shall Overcome." Today, the fact that America remains democratic
and self-critical, that its free press exposes governmental flaws and that the legislative and judicial branches can act against the
executive, means that anti-American critics of U.S. foreign policies can still feel a residual attraction to our
society. As Sweig puts it, "The best antidote to Anti-America may well come not from how we fight (or prevent) the next war but
from the degree to which we keep intact the social contract and international appeal of American society." She also urges
Washington to adopt a changed foreign policy style that develops empathy for foreign cultures, practices better manners and pays
more attention to rules and fairness. Anti-Americanism will not go away, but it need not dominate the
21st century if Americans follow the advice of this well-reasoned book.
---Soft Power- Fails

1NC Albro—soft power can’t solve—nature of cultural power lies in its neutrality—means that
policy makers can’t direct it towards a goal and even if they could, it would fail to influence
other countries—explicitly criticizes State Department efforts to shape goals

Can’t shape into policy outcomes


Blechman, 5 (Barry M, founder and president of DFI International Inc., a research and consulting company in Washington, DC
(frequent consultant to the US Government),Winter 2004/ “Soft Power: The Means to Success in World Politics,” Political Science
Quarterly, Vol. 119, Iss 4; pg. 680-681, proquest)

Soft Power: The Means to Success in World Politics by Joseph S. Nye. New York, Public Affairs, 2004. 208 pp. $25.00.

Joseph Nye has done his usual masterful job in this elegant monograph, describing the many sources of influence in international
relations and reminding readers that excessive reliance on military or economic instruments of policy can often trigger backlashes
that harm the nation's interests in the longer term. Nye points out that rather than either coercing others to share our objectives or
buying their agreement with economic incentives, it is better for the United States to get what it wants because others share our
goals. Soft power, he says, is more than influence or persuasion, "it is also the ability to attract, and attraction often leads to
acquiescence" (p. 6). Much of the book is devoted to descriptions of the sources of soft power in the United States and other
countries, including the nation's values and the styles of individual behavior expressed in the dominant culture and transmitted
through both commercial activities (Hollywood movies, for example) and personal contacts, and the nation's policies, particularly
when they reflect values that are widely shared around the world. Thus, Nye argues, the United States won the Cold War in part
because of the attractiveness of the American form of government and economy, and because American values, or American soft
power, eventually came to dominate global perceptions of the two superpowers and induced others to want to share in our vision of
Although Nye makes a persuasive case, in the end, the book is unsatisfying because of
the world.
inherent limitations in the concept of soft power. It is a form of power, yes, but not an
instrument of power that can be deployed in specific situations or even one that can be shaped
in a meaningful way by the government. Soft power exists, and may be influenced by
governmental choices, but it is more an existential factor in the policy environment than
something policy makers can utilize to their advantage. A nation's "attractiveness" to others
is not a factor that can be exploited in any coherent way. Indeed, the chapter "Wielding Soft Power" is
devoted solely to public diplomacy-the various means available to the government to communicate the nation's policies and values.
But in our interdependent and interactive world, government-inspired communications of all types are only a tiny fraction of the
information received by people around the world about the United States. Even if the United States spent a more
reasonable amount on its public diplomacy than it does now, as Nye rightfully suggests, it's
diplomacy would still be dwarfed by the private sources of information in the United States and
abroad, and by the huge volume of interactions among citizens of all nations that take place independent of government actions
or inactions

The data is on our side

Brooks and Wohlforth 8 Stephen G. William C., Associate Professor and Professor of Government @ Dartmouth College,
08, World Out of Balance: International Relations and the Challenge of American Primacy, pg. 149,

if states have an overall propensity to cooperate and Lheir general


The first pattern of evidence concerns overall compliance rates.. Downs and Jones emphasize,

reputation greatly influences the prospects for sustaining international cooperation, then we should
expect to observe (1) uniform compliance rates across international institutions and/or high overall
compliance rates. h^1 their survey of the ^1itera^h~rc on con pliance with h^lternati^Qna^l institutions, they find that neither of these patterns
holds. As they stress: ^nWhile compliance rates are relativel good hi general, they are often considerably lower than one would e: pect them to be if every defection had
important implications for eve current and future agreement . .. It is common for a given state to ev dence very different rates of compliance reliability—the building bloci of
reputational inference—in connection with different agreements. Looking beyond compliance rates, a second pattern of evidence Coil cerns linkages across issue areas.

there is a dearth of
Although there are nur.^ie~nus rea.! examples in which poor compliance levels infh^le^l^lce the level of coop. ation within specific institutions,

documented cases in which a state's lack of compliance in one area has led to a more general decline
in a state's ability to attain cooperation in different areas For the United Statcs, the key concern in this regard is whether nth, states
engage in linkage across economic institutions and security ins tufions: the United States clearly wants increased flexibility in the tall area, but it also does not want to

Odell finds no empirical


undermine the prospects for cooper lion in the former area Surveying the literature on bargaining and intc , national negotiations,

support for the notion that states make linkages during negotiations between security issues and
economic issues (in the sense of changh^lg thei r underlying ne^On^li ~ lien positions regarding international h^lstit^tutions in one issue area do ! to the dynamics of
cooperation that exist in the other). l-le stresses tl !, is the case for states across the board.' To explain why states refra from altering their stance in economic negotiations in r
esponse lo ung a'! ing security dynamics, he ernphasi;~es the significance of interest grog if pressures: nThr proposal to bend trade or financial c~nc^Lssivns :,~ threats to
security goals is likely to meet at Icast some opposition frc i., the producers, lenders, and investors who would pay the price, cxcc ~~ when commercial moves or opportu^luties
arc insignificant ecvnonn if; ally or when these constituents believe war is imminent.

Can’t solve key crises or generate necessary cooperation

Ford 12(Christopher, Senior Fellow at the Hudson Institute in Washington D.C., He Previously Served as US Special Representative
for Nuclear Nonproliferation, “Soft On Soft Power” SAID Review, Vol 32, No 1. Winter/Spring)

In its enthusiasm for the romantically unmilitarist potential of “soft power,” therefore, the Obama Administration seems to have
picked a fight on troublesome terrain. If
“soft power” is conceived as a form of power usable by national
leaders, it is not clear that this terrain is one that favors a free democracy in dealing with an
economically vibrant Leninist autocracy. In such circumstances, “soft power” competition may actually
be an asymmetric conflict in which the advantage , at least in the short run, lies with the unfree. We
should be wary of modern Washington’s politically-correct idealization of “soft power” as a
panacea for our country’s foreign policy and national security challenges, or as a rationalization for the
relinquishment of “hard power” capabilities. “Soft power” approaches are surely worth something, but they
are being oversold in contemporary Washington.

Can’t be specifically wielded towards goals

Ford 12(Christopher, Senior Fellow at the Hudson Institute in Washington D.C., He Previously Served as US Special Representative
for Nuclear Nonproliferation, “Soft On Soft Power” SAID Review, Vol 32, No 1. Winter/Spring)

I have no particular quarrel with Nye’s original notion of “soft power ” as an analytical construct, for it is
indeed useful in helping one understand the aggregate effect that a nation has on others and
the international environment. I urge caution, however, lest one get too carried away, in overselling how usable “soft
power” might be in international policymaking. The problem here is twofold, and does not necessarily entail
any criticism of the idea that in some meaningful sense, a nation’s aggregate impact in the world
is significantly dependent upon economic, cultural, moral, and political factors . Rather, the problem lies
with the assumptions that “soft power” per se can be relied upon by U.S. policymakers to advance
their agenda and that the United States enjoys a “soft power” advantage in perhaps the world’s most
important bilateral relationship, that with the People’s Republic of China. Given Washington’s finite political capital, high-level
attention, budget dollars, and the degree to which either assumption proves faulty, the current U.S. focus on “soft power”
approaches, may be a mistaken priority. The
concept of “soft power” as a sort of aggregate measurement
of a country’s overall socio-political clout in the world retains some utility as an analytical tool.
Billions of people worldwide recognize iconic American brands such as McDonald’s and Coca-Cola, watch movies made in
Hollywood, and the United States remains the world’s largest economy in an age of profound globalization. Such pervasive
contact with the non-American world surely counts for something in assessing the United
States’ impact in global affairs. It is less clear, however, what to make of “soft power” from the
perspective of a U.S. policymaker, whose natural inclination will presumably be to ask what he
or she can do with all this “power.” It is one thing, after all, to posit that the appeal of American
values, the model offered by its political system, the broad presence of its brands overseas, and its pop cultural exports help in
some vague and very general way predispose foreign publics toward things of which U.S. policymakers approve (e.g., consumerist
democracy) and
ultimately lead to a greater convergence of interests in world affairs. It is quite
another to hold out reliance upon “soft power” as a means by which an American policymaker
can accomplish any specific policy objective.

Democracy constrains soft power’s ability to implicate policy

Ford 12(Christopher, Senior Fellow at the Hudson Institute in Washington D.C., He Previously Served as US Special Representative
for Nuclear Nonproliferation, “Soft On Soft Power” SAID Review, Vol 32, No 1. Winter/Spring)

This problem has received too little attention. To what extent is “soft power” actually something
that our national leadership can use to influence other countries or shape the international
environment? This question is, central to distinguishing “soft power” from mere impact. If one considers “soft
power” analogous to and at least partially substitutable for the “hard power” of military force,
its ability to be used is inescapably important. Many things about the United States may have great impact on
others, but unless our national leadership can shape or steer that impact, it is difficult to describe it as a tool of national power. The
distinction is important because when viewed through this prism of usability, much of what is
considered the United States’ “soft power” fails the test. Indeed, in a developed democracy such
as our own, some of the most important and dynamic aspects of society frequently associated
with our strengths, including the media, business, financial and cultural sectors, are conspicuous
because they cannot be purposefully “steered” by the U. S. government in a significant way.

Impact of soft power strategies are exaggerated – doesn’t translate into policy success

Ford 12(Christopher, Senior Fellow at the Hudson Institute in Washington D.C., He Previously Served as US Special Representative
for Nuclear Nonproliferation, “Soft On Soft Power” SAID Review, Vol 32, No 1. Winter/Spring)

In the last three years, U.S. officials have rushed to associate themselves with the idea of “soft power ,”
believing that it had received insufficient emphasis, and urging that it become a more prominent part of Washington’s efforts to
A new prioritization of “soft” approaches, it was said, would complement
influence global events.
residual “hard” capabilities and produce a hybrid, smugly termed “smart power” that would transform American foreign
policy and give the United States new clout and stature on the world stage. This essay examines some of the conceptual and
practical foundations of this “soft power” focus and considers how large a competitive advantage it is likely to offer for [End Page 89]
advancing U.S. interests and policy objectives in international affairs. It is particularly in the context of Sino-American relations—the
most important bilateral dyad of the mid-twenty-first century global security environment. As we will see, “ soft
power”
stands up less well than its prominent U.S. advocates would have one believe. “Softness,” it
turns out, is not always what it is cracked up to be . While it has some value both as an analytical construct and a
guide to policymaking, soft power is frequently confused with mere impact on the world, ignoring how
effectively national leaders can manipulate a state’s “soft” interactions with others in support of
policy ends. When “soft power” is misconceived and reified as a sort of magical balm for all sorts
of policy problems, reliance upon “softness” can become a maladaptive recipe for evading
difficult choices, and neglecting the “hard” capabilities that remain important to security and policy success in a
complicated world. When properly understood through the prism of “usability,” soft power can indeed be a valuable component of
national policymaking. A comparison of U.S. and Chinese approaches to using soft power, however, suggests that modern
Washington’s evangelists for “soft” approaches may be greatly overselling its advantages .

Soft power is an empty tool without hardpower to enforce it

Rubin 12 (Jennifer, Right Turn Blog for the Washington Post, “Obama’s Foreign Policy Flubs” 5/27)

Even more troubling, is Obama’s belief that we can effectively wield soft power (e.g., diplomacy,
economic leverage) without hard power to back it up. He continually signals that he is unwilling to use the latter,
thereby giving encouragement to adversaries. Whether it is talking down the military option for Iran ,
telegraphing that we won’t use force in Syria or bugging out of Iraq prematurely (signaling to
Iran our lack of staying power), we have undercut our own credibility , both in the immediate conflict and
to other powers trying to gauge our resolve. There is no more disastrous and dangerous undertaking in this regard than our massive
defense cuts. Obama also has consistently taken despotic regimes’ propaganda as sincere expression of their national interest. We
conclude that Russia’s Vladimir Putin wants to democratize and modernize. We imagine that the Palestinians want to end, not
merely pause, their effort to expunge a Jewish state. The entire administration is stricken with State
Department-itis, the ailment usually afflicting only Foggy Bottom and characterized by the inability to distinguish between
discussion and progress, or results and process. The notion that some difference are unbridgeable because one side harbors motives
that are incompatible with our security and values is unimaginable

Soft power can’t solve anything

Fugiel, 5 [ Michele, Sept. 15, “U.S. Public Diplomacy and the American Experience: A theoretical evolution from consent to
engagement”, MA International Studies and Diplomacy School of Oriental and African Studies,p 32-33]

A public diplomacy process that emanates from top-down directives and that is caught within traditional diplomacy
misses the essence its public nature. Instead, it becomes a tool for national security rather than a mode
of communication. The “Shared Values Initiative” and the “War on Terror” were both conceived of as strategic
messages deployed at a calculated time to garner support for U.S. operations in the Middle East. While
both campaigns may have caused initial sensation with certain audiences, they did little to increase American popularity or stability .
Both examples show how strategy does not always ensure a success. Indeed, public diplomacy is not a magic bullet for communication.80 The government’s deployment

the use of soft power to deliver messages is not as precise as a laser guided missile.81
strategy backfired because

There is nothing to ensure a direct hit, no easy way to abort the mission, and no way to guard against
retaliation. Even when a government is using soft power sub-optimally, the flow does not cease. Rather
all images, messages, and realities from all parts of society continue to be created. What hinders current U.S. public diplomacy is that these voices are not being heard

official public diplomacy for all of its functions and messages has rested within
internationally. Since its inception,

the power of the state. It cannot actually communicate the entirety of the American experience
simply because the government cannot occupy every area of the American experience. For public diplomacy to
truly be conceived of as using a softer power it must be an effort that grows from the entire American experience; it must be an organic attempt at creating reality. Nye does not
discuss the broad participation necessary for soft power. His primary conception of soft power relates to its contrast to military and economic power. He discusses the

Simply trying to
governments role in connection to these forms of power, but does not thoroughly examine how power flows throughout all of the sectors of society.

amass and deploy power does not take into account that power is always flowing. Michel Foucault
better describes the way in which power can be seen as “capillary” or in terms of networks. In this way, power is seen as
something that circulates and is never confined within anyone’s hands82. It is a set of relations and strategies circulated throughout society and passed on at every moment of

The U.S. government, or any government, even while seeming supreme cannot inhabit the whole
interaction83.

field of power relations that are dispersed throughout society 84.


---Soft Power- Squo Solves
Soft power is high now -

Tourism and immigration -

Bev 12 (Jennie S., Columnist to Forbes Indonesia, Associate Partner of Fortune PR Indonesia, The Power of American Soft Power”
5/23)

Joseph Nye of Harvard University Kennedy School of Government says “soft power” refers to the ability to get through attraction
rather than coercion or payments. By “to get” it means to receive favorable treatments based upon attractiveness of a country’s
culture, ideals, and policies. For instance, inspired by TV series about medical doctors, some children in Taiwan aspire to study
medicine at an American university. Infatuated by the idea of a fair trial, an Indonesian dissident aspires to become a lawyer. “ Soft
power” can be hardcore power. And the American brand is still the best out there . Also, thanks
to low US dollar value, a record 62 million foreign tourists visited USA in 2011. In 2010, some
1.04 million immigrants applied for permanent residency , following 1.13 million in the previous
year, which reflects the world’s insatiable faith in the US brand. The people of the world still
believe that the USA is the place to visit, to reside, and to prosper.

Branding –

Bev 12 (Jennie S., Columnist to Forbes Indonesia, Associate Partner of Fortune PR Indonesia, The Power of American Soft Power”
5/23)

US brands, such as automobile giants Buick, GM, and Ford, continue to grow outside of the USA.
US brands continue to influence socio-political-economic wellbeing of people of the world:
Facebook, Twitter, and Youtube are vital in demonstrations and social unrests. US brands continue to serve people’s
mobility and communication: Apple, Microsoft, CISCO, Oracle, and Boeing. People of the world is a market of seven-billion, and most
of them have occasionally consumed black soda drinks called Coca-Cola and Pepsi. The
US government has lost its
geopolitical epicenter, yet American brands keep the legend alive . And the shift has occurred from public
power to private power, from political power to economic power, from hard power to soft power, with the end of the Cold War as
the turning point. The
recent approval of the JOBS Act in April 2012 may as well pick up where the
failure of previous policies have left, as its intention is creating an encouraging environment for
growth of startup companies through more efficient and lenient procedures of capital raising, including crowdsourcing,
venture capitalizing, and angel investing. And it is expected that every new investment would create at least six new jobs. I can see
the greatness of American brands supported by the JOBS Act creating another shift in economic recovery, as once again a policy is
providing a conducive environment for growth, just like when Glass-Steagall Act of 1933 was repealed by Gramm-Leach-Bliley Act in
1999. Now the question is: How far will the JOBS Act’s ripple effect go? And which direction does it go? North or south? Growth,
stagnation, or decadence?

Immigration proves America’s cultural openness

Bisk 12 (Tsvi, Independent Israeli/American Futurist, Social Researcher and Strategy Planning Consultant, “The Second American
Century”, The Future Society, 5/4)
The Economist captured this truth in a recent article entitled " The Greatest Strength of America is that People
Want to Live There." The article described the overwhelming attraction of the United States and the ease
with which people of different cultures feel so quickly at home . For example, 77% of US immigrants
said it took less than five years to feel "part of the community " while 58% of 2nd and 3rd generation Germans
of Turkish descent say that they still feel unwelcome. Given the tremendous global competition for mobile talent, the inherent
openness and pluralism of American culture and society gives the United States a remarkable
competitive advantage over the rest of the world as we move deeper into the 21st century. The proactive promotion of
American cultural influence, or "soft power", will not only make for greater national security in the long run, it will be a blessing for
all of humanity. The vision of the United States as "a light unto the nations" instead of policeman to the world would be the ultimate
realization of the hopes and dreams of many of the early pioneers and Founding Fathers.
---Soft Power- AT: Backlash

American smart power impossible – US hegemony and hard power


Layne ‘7
Christopher Layne, Associate Professor in the Bush School of Government and Public Service at Texas A&M University, 2007 (“American Empire: A Debate,” pg. 68)

American primacy has its dimension of benevolence, but a state as powerful as the United
Doubtless,

States can never be benevolent enough to offset the fear that other states have of its unchecked
power. In international politics, benevolent hegemons are like unicorns—there is no such animal. Hegemons
love themselves, but others mistrust and fear them—and for good reason. In today's world, others dread both
the overconcentration of geopolitical weight in America's favor and the purposes for which it may be
used. After all, "No great power has a monopoly on virtue and, although some may have a great deal more virtue than others, virtue imposed on others is not seen as such by
them. All great powers are capable of exercising a measure of self-restraint, but they are tempted not to and the choice to practice restraint is made easier by the existence of

While Washington's self-proclaimed benevolence is inherently


countervailing power and the possibility of it being exercised."

ephemeral, the hard fist of American power is tangible. Others must worry constantly that if U.S. intentions change, bad things may
happen to them. In a one-superpower world, the overconcentration of power in America's hands is an

omnipresent challenge to other states' security, and Washington's ability to reassure others of its
benevolence is limited by the very enormity of its power.

Soft power doesn’t solve resentment – still risks backlash.


Joffe ‘6
Joffe, publisher-editor of the German weekly Die Zeit and associate of the Olin Institute for Strategic Studies at Harvard University, 2006 (Josef, New York Times, May 14,
http://www.nytimes.com/2006/05/14/magazine/14wwln_lede.html?_r=2&pagewanted=print&oref=slogin)

In recent years, a number of American thinkers, led by Joseph S. Nye Jr. of Harvard, have argued that the United
States should rely more on what he calls its "soft power" — the contagious appeal of its ideas, its culture and its way of life — and so
rely less on the "hard power" of its stealth bombers and aircraft carriers . There is one problem with this argument: soft power does

not necessarily increase the world's love for America. It is still power, and it can still make enemies.
America's soft power isn't just pop and schlock; its cultural clout is both high and low. It is grunge and Google, Madonna and MoMA, Hollywood and Harvard. If two-thirds of the
movie marquees carry an American title in Europe (even in France), dominance is even greater when it comes to translated books. The figure for Germany in 2003 was 419
versus 3,732; that is, for every German book translated into English, nine English-language books were translated into German. It used to be the other way around. A hundred
years ago, Humboldt University in Berlin was the model for the rest of the world. Tokyo, Johns Hopkins, Stanford and the University of Chicago were founded in conscious
imitation of the German university and its novel fusion of teaching and research. Today Europe's universities have lost their luster, and as they talk reform, they talk American.
Indeed, America is one huge global "demonstration effect," as the sociologists call it. The Soviet Union's cultural presence in Prague, Budapest and Warsaw vanished into thin air

There may be little or no relationship


the moment the last Russian soldier departed. American culture, however, needs no gun to travel.

between America's ubiquity and its actual influence. Hundreds of millions of people around the
world wear, listen, eat, drink, watch and dance American, but they do not identify these
accouterments of their daily lives with America. A Yankees cap is the epitome of things American, but it hardly signifies knowledge of, let
alone affection for, the team from New York or America as such. The same is true for American films, foods or songs. Of the 250 top-grossing movies around the world, only four
are foreign-made: "The Full Monty" (U.K.), "Life Is Beautiful" (Italy) and "Spirited Away" and "Howl's Moving Castle" (Japan); the rest are American, including a number of co-
productions. But these American products shape images, not sympathies, and there is little, if any, relationship between artifact and affection. If the relationship is not neutral, it
is one of repulsion rather than attraction — the dark side of the "soft power" coin. The European student movement of the late 1960's took its cue from the Berkeley free-
speech movement of 1964, the inspiration for all post-1964 Western student revolts. But it quickly turned anti-American; America was reviled while it was copied. Now shift
forward to the Cannes Film Festival of 2004, where hundreds of protesters denounced America's intervention in Iraq until the police dispersed them. The makers of the movie
"Shrek 2" had placed large bags of green Shrek ears along the Croisette, the main drag along the beach. As the demonstrators scattered, many of them put on free Shrek ears.

the enormous pull of American


"They were attracted," noted an observer in this magazine, "by the ears' goofiness and sheer recognizability." And so

imagery went hand in hand with the country's, or at least its government's, condemnation. Between Vietnam and
Iraq, America's cultural presence has expanded into ubiquity, and so has the resentment of
America's soft power. In some cases, like the French one, these feelings harden into governmental
policy. And so the French have passed the Toubon law, which prohibits on pain of penalty the use of English words — make that D.J. into a disque-tourneur. In 1993, the
French coaxed the European Union into adding a "cultural exception" clause to its commercial treaties exempting cultural products, high or low, from normal free-trade rules.
Other European nations impose informal quotas on American TV fare.
---Soft Power- AT: Conflicts

Soft power cant solve conflicts


Hampson and Oliver 98
(Fen O and Dean F, Pulpic diplomacy: a critical assessment of the Axworthy doctrine, p. lexis)

Perhaps the two best examples of the continued utility of military force are the Persian Gulf conflict of
1990-1 and the coalition deployment to the same region, led by the United States (and supported by the United Nations), in early
1998 to ensure Iraq's compliance with the 1991 ceasefire agreement. Both missions have occasioned much debate in the scholarly community, and deservedly so, but we

take it as axiomatic that for both sides on each occasion the role of military force was critical in the
evolution -- and resolution -- of the crisis. In 1990-1, this would appear to be self-evident, while in 1998 no less a commentator than Kofi Annan, in
the wake of Iraq's decision to again permit weapons inspectors access to its presidential palaces, dubbed the United States and Britain 'the perfect UN peacekeepers' for their

in each case soft power proved singularly unable to affect the


show of force in support of UNSCOM. It is important to note that

actions of a single, isolated, pariah state, albeit one that possessed considerable military
wherewithal and a modicum of regional legitimacy. It is certainly dangerous to generalize from the
Iraqi example, but one might at least question the applicability of soft power to powerful rogue
states in bold defiance of international law and international agreements .

Soft power doesn’t solve conflict – can’t be harnessed for foreign policy goals.

Nye ‘7, -Professor at Harvard University, 2007 (Joseph, Leashing the Dogs of War, ed. Crocker, p.396

Soft power can play an important role in managing conflicts, but one must not oversell it. For one thing, as
mentioned earlier, soft power is often difficult for governments to use directly. Much of it is produced and

controlled by civil society outside the control of government. To some extent, national soft power in the form of values is almost
an inadvertent by-product of domestic political life, and American popular cultural exports are controlled more by Hollywood than by Washington. Even in countries with more
central political control than in the United States, the importance of credibility limits the extent to which governments can manipulate their soft power in an information age.

setting an example does not provide power unless others choose to follow it.
Moreover, as mentioned,

Sometimes examples are ignored; and sometimes, when cultural values differ dramatically, examples
can be counterproductive. Thus soft power is not simply another “tool” to be added to the peacekeeper’s “tool kit” like an additional battalion of troops. But
attraction to the values for which peacekeepers stand can facilitate their tasks.
---Soft Power- AT: Heg

Soft power doesn’t increase hegemony


Reus-Smit ‘4 Prof. International Rels @ Australian National Univ.
(Christian, American Power and World Order, pg. 54-55)
We see here yet another expression of the assumed causal connection between power resources
and political influence, this time applied to culture. If anything, however, the problems are multiplied. Let
us assume that in one sense at least neo-conservatives are right, that many people around the globe do indeed
covet certain ‘American’ (now globalized) cultural values and artifacts, from Ivy League education to Disneyland and
Nikes. It is the height of blind chauvinism, however, to think that this necessarily translates into
American political influence. To begin with, many of the world’s citizens appear quite capable of
holding at least two ideas in their heads at the same time. ON the one hand, they might want to do a
doctorate at Harvard, watch a Hollywood movie or wear Nike runners, but at the same time they can be deeply
worried about the nature and consequences of American foreign policy . Second, even if it made
sense to speak of culture as a power ‘resource’ , it is a resource that defies control. Culture – in the
form of values or artifacts – is inherently intersubjective, and, even if it does ‘radiate outward’ from a particular society, it
is never passively received; it is always reinterpreted, grafted to other values and turned to new
purposes. Both of these points are nicely illustrated by the example of the British empire and
Indian nationalists. In the late nineteenth and early twentieth centuries, young Indian elites flocked to
Cambridge, Oxford and London universities. This did not mean, however, that they uncritically accepted
imperial rule; on the contrary, they took the ideas of liberalism and democracy they imbibed in the imperial heartland and
fashioned them into anti-imperialist nationalism.

Soft power can’t solve- hard power overwhelms

Quinn, 11 – Lecturer in International Studies at the University of Birmingham, having previously worked at the University of
Leicester and the University of Westminster alongside his graduate studies at the LSE. His chief area of interest is the role of national
history and ideology in shaping US grand strategy (Adam, “The art of declining politely: Obama’s prudent presidency and the waning
of American power”, International Affairs 87:4 (2011) 803–824 http://www.chathamhouse.org/sites/default/files/87_4quinn.pdf

Nevertheless, this qualification demands two further qualifications of its own. The first is that if
we consider ‘soft power’
as a national attribute then it is difficult to separate it with confidence from the economic and
military dimensions of power. Is it really likely that America’s ideological and cultural influence will endure undiminished
in the absence of the platform of military and economic primacy upon which it has been constructed? It may be overstatement to
suggest that, borrowing Marxist terminology, hard power represents the ‘base’ and soft power mere ‘superstructure’. But one could
plausibly argue that even
America’s non-coercive power and political appeal are inextricably entwined
with the status conferred upon it by possession of a preponderance of material resources.
While vestigial soft power may delay or mitigate the consequences of relative material decline,
it is surely unrealistic to expect it to override them such as to allow the US to continue to
exercise the same influence in a multipolar or non-polar world as it did in a unipolar one.

Soft power breeds resentment and counterbalancing


Joffe ‘6
(Josef, Editor of the Germany weekly Die Zeit, New York Times, “The Perils of Soft Power”, May 14, L/N)

In recent years, a number of American


thinkers, led by Joseph S. Nye Jr. of Harvard, have argued that the
United States should rely more on what he calls its ''soft power'' -- the contagious appeal of its ideas, its
culture and its way of life -- and so rely less on the ''hard power'' of its stealth bombers and aircraft carriers. There is one problem
with this argument: soft
power does not necessarily increase the world's love for America. It is still
power, and it can still make enemies. America's soft power isn't just pop and schlock; its cultural clout is both high
and low. It is grunge and Google, Madonna and MoMA, Hollywood and Harvard. If two-thirds of the movie marquees carry an
American title in Europe (even in France), dominance is even greater when it comes to translated books. The figure for Germany in
2003 was 419 versus 3,732; that is, for every German book translated into English, nine English-language books were translated into
German. It used to be the other way around. A hundred years ago, Humboldt University in Berlin was the model for the rest of the
world. Tokyo, Johns Hopkins, Stanford and the University of Chicago were founded in conscious imitation of the German university
and its novel fusion of teaching and research. Today Europe's universities have lost their luster, and as they talk reform, they talk
American. Indeed, America is one huge global ''demonstration effect,'' as the sociologists call it. The Soviet Union's cultural presence
in Prague, Budapest and Warsaw vanished into thin air the moment the last Russian soldier departed. American culture, however,
needs no gun to travel. There
may be little or no relationship between America's ubiquity and its actual
influence. Hundreds of millions of people around the world wear, listen, eat, drink, watch and
dance American, but they do not identify these accouterments of their daily lives with America.
A Yankees cap is the epitome of things American, but it hardly signifies knowledge of, let alone
affection for, the team from New York or America as such . The same is true for American films, foods or songs.
Of the 250 top-grossing movies around the world, only four are foreign-made: ''The Full Monty'' (U.K.), ''Life Is Beautiful'' (Italy) and
''Spirited Away'' and ''Howl's Moving Castle'' (Japan); the rest are American, including a number of co-productions. But these
American products shape images, not sympathies, and there is little, if any, relationship
between artifact and affection. If the relationship is not neutral, it is one of repulsion rather than
attraction -- the dark side of the ''soft power'' coin. The European student movement of the late
1960's took its cue from the Berkeley free-speech movement of 1964, the inspiration for all
post-1964 Western student revolts. But it quickly turned anti-American; America was reviled
while it was copied. Now shift forward to the Cannes Film Festival of 2004, where hundreds of protesters denounced
America's intervention in Iraq until the police dispersed them. The makers of the movie ''Shrek 2'' had placed large bags of green
Shrek ears along the Croisette, the main drag along the beach. As the demonstrators scattered, many of them put on free Shrek ears.
''They were attracted,'' noted an observer in this magazine, ''by the ears' goofiness and sheer recognizability.'' And so the
enormous pull of American imagery went hand in hand with the country's, or at least its
government's, condemnation. Between Vietnam and Iraq, America's cultural presence has
expanded into ubiquity, and so has the resentment of America's soft power . In some cases, like
the French one, these feelings harden into governmental policy . And so the French have passed the Toubon
law, which prohibits on pain of penalty the use of English words -- make that D.J. into a disque-tourneur. In 1993, the French coaxed
the European Union into adding a ''cultural exception'' clause to its commercial treaties exempting cultural products, high or low,
Nor is America's high
from normal free-trade rules. Other European nations impose informal quotas on American TV fare.
culture more easily accepted than its pop -- at least not by the cultural elites . A fine example is how the
art critics of two distinguished German newspapers, Suddeutsche Zeitung (leftish) and Frankfurter Allgemeine Zeitung (centrist),
dealt with an exhibit of 200 pieces from the Museum of Modern Art in Berlin in 2004. More than a million visitors stood in line, many
for up to nine hours, to view the objets from across the Atlantic. Yet the
fervor of the hoi polloi mattered little to
their betters, whose comments ran the gamut from contempt to conspiracy. The opening shots
were fired by the Suddeutsche Zeitung of Munich . Without having seen the collection, its critic aimed his
volley straight against imperial America. Regurgitating a standard piece of European ressentiment, the author
insinuated that what America has in the way of culture is not haute, and what is haute is not
American. (Or as Adolf Hitler is said to have declared, ''A single Beethoven symphony contains more culture than all that America
has ever created.'') After World War II, the critic contended, America had wrested ''artistic hegemony'' from Europe in two sleazy
ways. One culprit was ''a new abstract school of painting'' -- Abstract Expressionism -- ''that had hyped itself into high heaven.'' The
other was American mammon: ''Everything still available in old Europe was bought up.'' And this ''stolen idea of modern art will now
be presented in Berlin.'' Thus were pilferage and grand theft added to the oldest of indictments: America's cultural inferiority. The
critic of Frankfurter Allgemeine went one worse. If his colleague claimed that America's art was either hyped or heisted, the man
from Frankfurt thundered that MoMA's Berlin show was a mendacious ploy, indeed, an imperialist conspiracy. It was done by
''concealment'' and ''censorship'' in a game full of ''marked cards,'' and its aim was not only to blank out Europe's greats but also to
suppress their magnificent contribution to American art in the second half of the 20th century. This was an instance of the selective
perception that suffuses anti-Americanism or any other ''anti-ism,'' for the exhibit contained an impressive number of European
works: Matisse, Picasso, Manet, Rousseau, Brancusi and Mondrian, plus assorted Expressionists and Surrealists. That did not count.
What about contemporary Germans like Beuys, Baselitz and Kiefer? the critic huffed. But even here, MoMA had done its duty,
capping the progression with Gerhard Richter's ''18 October 1977'' cycle, which depicts dead members of the Baader-Meinhof
terrorist gang. That MoMA would display these German works enraged the feuilletoniste from Frankfurt even more. That particular
choice, he fumed, was the final proof of American perfidy. The terrorist motif was insidiously selected to finger Europe as a ''creepy''
There is a moral in this tale of two critics: the curse of soft power. In
place, as a messenger of ''bad news.''
the affairs of nations, too much hard power ends up breeding not submission but resistance.
Likewise, great soft power does not bend hearts; it twists minds in resentment and rage . And the
target of Europe's cultural guardians is not just America, the Great Seductress. It is also all those ''little people,'' a million in all, many
of whom showed up in the wee hours to snag an admissions ticket to MoMA's Berlin exhibit. By yielding to America-the-beguiling,
they committed cultural treason -- and worse: they ignored the stern verdict of their own priesthood. So America's soft power is not
only seductive but also subversive. Hard power can by defanged by coalitions and alliances. But how do you balance against soft
power? No confederation of European universities can dethrone Harvard and Stanford. Neither can all the subsidies fielded by
European governments crack the hegemony of Hollywood. To breach the bastions of American soft power, the Europeans will first
have to imitate, then improve on, the American model. Imitation and leapfrogging is the oldest game in the history of nations. But
competition has barely begun to drive the cultural contest. Europe, mourning the loss of its centuries-old supremacy, either
resorts to insulation (by quotas and ''cultural exception'' clauses) or seeks
solace in the disparagement of American
culture as vulgar, inauthentic or stolen . If we could consult Dr. Freud, he would take a deep drag on his cigar and
pontificate about inferiority feelings being compensated by hauteur and denigration.

Soft power does not lead to hegemony- instead, it is a product of hegemony.

Namasivayam ‘1 Reesha Namasivayam, M.A. Candidate, Conflict Analysis, Carleton University, 2001 ("Soft Power At The
United Nations," Http://Www.Cdacdai. Ca/Pdf/Namasivayampaper.Pdf)

Even the 'father' of soft power, Joseph Nye, cautioned that although 'Canada has always been good at punching
above its weight in world politics…. to keep doing so in the global information age requires not just
good ideas in speeches but also an extraordinary degree of political and diplomatic coordination . Nonetheless, it
is important to note that during a speech in Boston on May 2, 2000, Nye asserted, "the US is not the only country with
soft power-think of the moral authority of the Vatican, or of Canada on human rights issues." However
despite this disclaimer, the extent to which Canada could impel soft power in the Security Council remained

questionable. Nye and Keohane assert that "more often soft power is an inadvertent byproduct," as
opposed to a reflection of deliberate policies.
---Soft Power- AT: Human Rights

Soft power doesn’t solve human rights or democracy – many examples.

Krauthammer ‘8, Pulitzer-Prize winning syndicated columnist, 2008 (Charles, National Review, July 11, Lexis Academic)
This in foreign policy establishment circles is called "hard power." In the Bush years, hard power is terribly out of fashion, seen as a mere obsession of cowboys and neocons.

Both in Europe and America, the sophisticates worship at the altar of "soft power" -- the use of diplomatic and
moral resources to achieve one's ends. Europe luxuriates in soft power, nowhere more than in l'affaire Betancourt in

which Europe's repeated gestures of solidarity hovered somewhere between the fatuous and the
destructive. Europe had been pressing the Colombian government to negotiate for the hostages. Venezuela's Hugo Chavez offered to mediate. Of course, we know from
documents captured in a daring Colombian army raid into Ecuador in March -- your standard hard-power operation duly denounced by that perfect repository of soft power, the
Organization of American States -- that Chavez had been secretly funding and pulling the strings of the FARC. These negotiations would have been Chavez's opportunity to gain
recognition and legitimacy for his terrorist client. Colombia's President Alvaro Uribe, a conservative and close ally of President Bush, went instead for the hard stuff. He has for
years. As a result, he has brought to its knees the longest-running and once-strongest guerrilla force on the continent by means of "an intense military campaign (that)
weakened the FARC, killing seasoned commanders and prompting 1,500 fighters and urban operatives to desert" (Washington Post). In the end, it was that campaign -- and its

Solemn condemnations have


agent, the Colombian military -- that freed Betancourt. She was, however, only one of the high-minded West's many causes.

been issued from every forum of soft-power fecklessness -- the EU, the U.N., the G-8 foreign ministers -- demanding that
Robert Mugabe of Zimbabwe stop butchering his opponents and step down. Before that, the cause du jour

was Burma, where a vicious dictatorship allowed thousands of cyclone victims to die by denying them independently delivered foreign aid, lest it weaken the junta's
grip on power. And then there is Darfur, a perennial for which myriad diplomats and foreign-policy experts

have devoted uncountable hours at the finest five-star hotels to deplore the genocide and urgently
urge relief. What is done to free these people? Nothing. Everyone knows it will take the hardest of hard power to remove the
oppressors in Zimbabwe, Burma, Sudan, and other godforsaken places where the bad guys have the guns and use them. Indeed, as the Zimbabwean opposition leader suggested
(before quickly retracting) from his hideout in the Dutch embassy -- Europe specializes in providing haven for those fleeing the evil that Europe does nothing about -- the only
solution is foreign intervention.
---Soft Power- AT: Iran

Obama’s soft power leads to concessions to Iran, threatening proliferation


BBC ‘9
(BBC Monitoring Trans Caucasus Unit, April 24, 2009, “Iranian academician urges caution over US participation in nuclear talks”, Lexis)

America was trying with the nuclear dossier,


Commenting on America's change of mind on engaging in nuclear talks with Iran, a university lecturer has said that

to first "enter with soft power, and then relying on soft power, use hard power at
Palestine and Afghanistan

later stages", ISNA reported. Mehdi Mottaharnia said the Obama administration was a "neo-Democrat" government
and said "the neo-Democrats rely on intelligent power, which means a redefinition of the Democrats' approach in the political arena. Basically the Americans have concluded on
the basis of intelligent power that while their rivals do not have a corresponding weight in power, they use provocation against their rival and win energy from this, and use the

the neo-Democrats believe they must not insist on suspension or


reaction of their rival, which is America. So,

preconditions in Iran's nuclear issue." He referred to Iran's conduct in new conditions that "in the face of the intelligent
power of America, which is a much more serious layer than the layers of power used by America in
previous periods of history, one has to act with sense and caution. Thus diplomatic subtleties must
be used and a radical discourse and hasty conduct must be avoided. While Obama is an opportunity for the world, he
may also become a threat worse than the neo-conservatives in the world." The international affairs analyst said Iran
could use [America's] entry into talks to "increase the level of concessions it can obtain, while
considering the rival's capability to accept these concessions. The lack of a precise perspective can
turn winning concessions into a dangerous game. Iran's nuclear dossier must be looked at from a broad perspective, because the nuclear
issue is part of the puzzle of issues between Iran and America."

Soft Power won’t deter North Korea and Iran


Holmes ‘9, former assistant secretary of state, 2009
(Kim, also vice president at the Heritage Foundation and author of "Liberty's Best Hope: American Leadership for the 21st Century", June 12, 2009, Heritage Foundation, “The
importance of hard power”, http://www.heritage.org/Press/Commentary/ed061109.cfm)

Many, if not most , Europeans credit "soft power" for the peace they've enjoyed for decades . Thinking their version of a
Kantian universal peace arose from the committee chambers of the European Union - and not from the victories of the Western powers in World War II and the Cold War -

they hold up soft power as a model for the rest of the world. In their view, bridging the often hardened differences between states
and shaping their decisions requires mainly negotiation and common understanding. The importance of our military strength is

downplayed and sometimes even seen as the main obstacle to peace. Even when its importance is acknowledged, it's a
perfunctory afterthought. Many liberals are now pressing the U.S. government to adopt this vision, too. But the

futility of it can be seen everywhere, from the failure of negotiations to deter both Iran and North
Korea from their nuclear programs over the past five years - a period in which their efforts have only matured - to the
lackluster response to Russia's invasion of Georgian territory. The limits of soft power have not only
bedeviled Mr. Obama but George W. Bush as well. After applying pressure on North Korea so diligently in 2006, the Bush administration relaxed its posture in early
2007, and North Korea concluded that it was again free to backslide on its commitments. Two years later, this effort to "engage" North Korea,

which the Obama administration continued even after North Korea's April 5 missile test, has only led
North Korea to believe that it can get away with more missile tests and nuclear weapons
detonations. And so far, it has.
---Soft Power- AT: Korea

Soft power emboldens North Korea.


Kuhner ‘9, president of the Edmund Burke Institute, a Washington think tank., 2009
(Jeffrey T., June 6, 2009, Washington Times, “Another Korean War?”, http://www.washingtontimes.com/news/2009/jun/06/another-korean-war/)

North Korea threatens to engulf the Korean Peninsula in an all-out war. Pyongyang's recent test of a
nuclear bomb poses a serious threat to international security and regional stability. Dictator Kim Jong-il
continues to thumb his nose at global leaders, especially President Obama. The ailing strongman has denuded
Mr. Obama on the world stage, revealing his soft-power strategy to be ineffective and reckless. Washington's
emphasis on diplomacy was supposed to facilitate rogue states into increased cooperation. Instead, it
has only emboldened the likes of North Korea (and Iran) to press ahead with their nuclear-weapons
programs. Mr. Obama's "open hand" has been met with Mr. Kim's iron fist - one that has smashed
Uncle Sam in the face.

Soft power can’t solve North Korea – they perceive it as weakness.

Washington Times ‘9
(Wednesday, May 27, 2009, Washington Times, “North Korea tests Obama”, http://www.washingtontimes.com/news/2009/may/27/north-korea-tests-obama/)

While President Obama pushes soft power, the North Korean dictator plays hardball. North Korea's underground
nuclear test and missile trials show that the regime is probing Mr. Obama's resolve. Pyongyang apparently has
concluded that the president's rhetoric of conciliation and understanding betrays serious weakness
as a global leader. Like all tyrants, Kim Jong-il sees an open hand as a weak one. North Korea is determined
to be a nuclear power. Pyongyang has vowed to continue missile tests and uranium enrichment. The
Korean Central News Agency, the communist regime's mouthpiece, declared the regime's goal: to "further [increase] the power of nuclear weapons and steadily [develop]

nuclear technology." This comes in the face of a string of goodwill gestures by the United States and its allies. America
removed North Korea from the list of states that support terrorism in October and pointedly has overlooked the North's shipment of illegal drugs, counterfeiting, money

Pyongyang thanked us by
laundering and abduction of Japanese nationals. How did North Korea respond to these open-handed, friendly gestures?

conducting a ballistic missile test (under the cover of a satellite launch), restarting a plutonium-producing reactor at
Yongbyon, taking two American women hostage and now testing what it calls its "self-defensive
nuclear deterrent." This proves that no good deed goes unpunished.

Soft Power won’t deter North Korea and Iran


Holmes ‘9, former assistant secretary of state, 2009
(Kim, also vice president at the Heritage Foundation and author of "Liberty's Best Hope: American Leadership for the 21st Century", June 12, 2009, Heritage Foundation, “The
importance of hard power”, http://www.heritage.org/Press/Commentary/ed061109.cfm)

Many, if not most , Europeans credit "soft power" for the peace they've enjoyed for decades . Thinking their version of a
Kantian universal peace arose from the committee chambers of the European Union - and not from the victories of the Western powers in World War II and the Cold War -
they hold up soft power as a model for the rest of the world. In their view, bridging the often hardened differences between states
and shaping their decisions requires mainly negotiation and common understanding. The importance of our military strength is

downplayed and sometimes even seen as the main obstacle to peace. Even when its importance is acknowledged, it's a
perfunctory afterthought. Many liberals are now pressing the U.S. government to adopt this vision, too. But the

futility of it can be seen everywhere, from the failure of negotiations to deter both Iran and North
Korea from their nuclear programs over the past five years - a period in which their efforts have only matured - to the
lackluster response to Russia's invasion of Georgian territory. The limits of soft power have not only
bedeviled Mr. Obama but George W. Bush as well. After applying pressure on North Korea so diligently in 2006, the Bush administration relaxed its posture in early
2007, and North Korea concluded that it was again free to backslide on its commitments. Two years later, this effort to "engage" North Korea,

which the Obama administration continued even after North Korea's April 5 missile test, has only led
North Korea to believe that it can get away with more missile tests and nuclear weapons
detonations. And so far, it has.
---Soft Power- AT: Prolif

1. Soft power can’t stop proliferation – countries that want weapons are least likely to be
swayed by softpower
The Jankarta Post ‘6, January 11, 2006
[“'Soft power' a hard course for RI foreign policy”, http://skyscrapercity.com/archive/index.php/t-236097.html, farhan]

Hence, Indonesia's employment of soft power may by applicable in engaging countries of a similar character like
Brunei. But it is unlikely to have much impact on those with a strong socio-political divide , like Myanmar. It can also

be effective if there really is a linear continuum of democratization in the region, something which China and Myanmar have proved is not occurring. Soft power

diplomacy does not make countries run by despots, plutocracies and regimes bent on monopolizing
power or rifling their nation's wealth, more humane! Even Nye himself conceded that "it may be that
you have to deal with those (despotic regimes) through hard power. It's important to realize that soft
power doesn't solve all problems".

Other nations won’t join


Ross ‘99 Professor of Political Science Trent University and the University of British Columbia
(Douglas, International Journal, January)
Only the United states in conjunction with the NATO countries could practically apply an aggressive counter-proliferation policy. But political opinion in the United States has
not been terribly receptive. If counter-proliferation I s to be realized, it will have to be co-operative. Following advice from analysts like Samuel Huntington and Ted Galen

Part of the problem with finding willing partners and


Carpenter, Americans may well decide to stay out of the nuclear crossfire.

deputies for the sheriffs posse is that the goal of counter-proliferation is so demanding and
aggressive.(f. 29) It is akin to the doctrine of rollback in the 1950s and 1960s.(f. 30) Certainly allied
countries are not inspiring the American people with their willingness to share the looming burden
of international security.

Soft power cant solve conflicts


Hampson and Oliver ‘98
(Fen O and Dean F, Pulpic diplomacy: a critical assessment of the Axworthy doctrine, p. lexis)

Perhaps the two best examples of the continued utility of military force are the Persian Gulf conflict of
1990-1 and the coalition deployment to the same region, led by the United States (and supported by the United Nations), in early
1998 to ensure Iraq's compliance with the 1991 ceasefire agreement. Both missions have occasioned much debate in the scholarly community, and deservedly so, but we

take it as axiomatic that for both sides on each occasion the role of military force was critical in the
evolution -- and resolution -- of the crisis. In 1990-1, this would appear to be self-evident, while in 1998 no less a commentator than Kofi Annan, in
the wake of Iraq's decision to again permit weapons inspectors access to its presidential palaces, dubbed the United States and Britain 'the perfect UN peacekeepers' for their

in each case soft power proved singularly unable to affect the


show of force in support of UNSCOM. It is important to note that

actions of a single, isolated, pariah state, albeit one that possessed considerable military
wherewithal and a modicum of regional legitimacy. It is certainly dangerous to generalize from the
Iraqi example, but one might at least question the applicability of soft power to powerful rogue
states in bold defiance of international law and international agreements .
---Soft Power- AT: Terror

1. Soft power cannot solve terrorism—to pacifist


Kling ‘4, [Arnold Kling, Contributing Editor, “The Battle of the Mosque,” TechCentalStation, http://techcentralstation.com/072704B.html] [Dexter Zhuang]
After articulating the threat in no uncertain terms, the Commission's recommendations for dealing with militant Islam amount to
proposals for the international equivalent of midnight basketball programs. These recommendations are contained in a section of
the Report called "Preventing the Continued Growth of Islamist Terrorism," on pages 391-400. The flavor of the proposals can be
tasted from the following excerpt (p. 393): "How can the United States and its friends help moderate Muslims
combat the extremist ideas?...We should offer an example of moral leadership in the world, committed to treat people
humanely, abide by the rule of law, and be generous and caring to our neighbors." To see what is wrong with this approach to what
the Commission calls "the struggle of ideas," imagine
if we had used it to fight World War II. Instead of
bombing Tokyo or Berlin, we would have have tried to stop Japanese and German aggression by
offering "an example of moral leadership." In my view, moderate Muslims today are in a position that is analogous to
that of ordinary Germans and Japanese in World War II. Although they may not be personally committed to the rabid ideology that is
behind the behavior of the warmongers, they are in awe of it. For all practical purposes,most of the Muslim world is
undecided between Islamism and America. If we adopt a more aggressive approach, some of these
Muslims will jump off the fence and onto the other side. But passivity and weakness on our part
would be even worse. To regain support of moderate Muslims in the long run, we will have to take
steps in the short run that risk upsetting them. The Commission would like to see us win the hearts and minds of
moderate Muslims. That is certainly a laudable objective, but it could easily become an excuse for pacifism and paralysis . We
could not have won World War II with "soft power," trying to win the hearts and minds of ordinary Germans as a
way of defeating the Nazis. By 1945, we had in fact won the hearts and minds of ordinary Germans, to the point where very few of
them admitted to ever having supported Hitler. But we achieved that result only after obliterating the Nazi military and, incidentally,
killing a large number of ordinary Germans. The Commission rightly says, in the paragraph quoted at the beginning of this essay,
that calling this a "war on terrorism" with no mention of Islamist ideology serves to blur our strategy. But it equally blurs our strategy
to say that the way to stop the spread of Islamist ideology is to "be generous and caring to our neighbors."

2. Softpower coalitions fail—terrorists remain unhindered


Miller ‘2 [Steven E. Miller, director of the International Security Program, Belfer Center for Science and International Affairs, at the John F. Kennedy School of
Government, Harvard University, Winter 2002, The Washington Quarterly, http://www.twq.com/02winter/miller.pdf] [Dexter Zhuang]

Among those nations prepared to play an active role in the fight, many are prepared to stand with the United States
only in limited ways. Some are grudging or reluctant participants in the coalition, feeling that they have no choice but to go
along with the United States while harboring no real passion for the cause and having no reason to run great risks. Some key
participants have powerful incentives to tread cautiously and to circumscribe their roles as much as
possible because the potential costs to them of entanglement  with the United States are enormous.
Some are willing to cooperate, provided the rewards are large enough. Some key states in Central and South Asia are fragile and
politically unstable, raising the risk that U.S. military activities in the region “could turn the area into a shatter zone of
collapsing states.”19 This coalition is not NATO writ large. It is not a collection of states bound by common values or a tight alliance
forged by the binding effects of a large common threat. It is a mélange of states hastily assembled through insistent U.S. diplomatic
maneuverings for the immediate purpose of facilitating U.S. retaliation in Afghanistan and for the longerterm purpose of facilitating
No other state has
the U.S. campaign against terrorism. This coalition will not be easy for Washington to preserve or manage.
quite the same stake in this fight as does the United States. Other states inevitably will have and will
act on their own interests, perceptions, preferences, and constraints. As the emotions associated with
the September 11 tragedy diminish, differences are likely to become more visible. As politics as usual regains force, others will likely
be more willing to voice their contrary views. As the
war against terrorism unfolds, the United States is bound
to make missteps and to take actions with unfortunate, undesired consequences—the accidental killing of
civilians, for example—that provoke criticism, weaken support, and undermine the U.S. position. The Bush administration appears to
be utterly resolved to wage the war against terrorism indefinitely, but the commitment of others is likely to wane. Indeed, not
much more than a week after air strikes commenced in Afghanistan, Washington’s partners may already have been losing their
patience with U.S. military operations.20 Washington is likely to view the coalition as a source of support and
an instrument of U.S. policy, but others are likely to see it as a mechanism for influencing U.S.
decisions or restraining U.S. action—a possibility that is mirrored in Bush administration concerns that the coalition
might “shackle” the United States.21 Further, the United States will not find it easy to push its coalition
partners to do things they do not want to do or feel that they cannot do. Managing this coalition will
be a demanding, messy, vexing, and occasionally fruitless exercise. 
---Soft Power- AT: Chinese Soft Power

China isn’t effectively using soft power now – public opinion polls prove

Seib ‘12 (Philip, Prof of Journalism and Public Diplomacy and Prof of International Relations at the University of Southern
California, Director of USC Center on Public Diplomacy “The First Soft-Power Superpower”)

During the 20 years since the demise of the Soviet Union , and after a unipolar moment for the United States,
China has emerged as the newest superpower. All its predecessors at this exalted level, going back even before
Rome, have established their positions by amassing formidable military strength. But China is going about matters
differently. Recognizing that it would require budget-wrecking spending to quickly catch up with the
United States as a wielder of military strength, China is, at least for now, emphasizing soft power
– trying to extend its influence through attraction rather than coercion. Although it certainly retains the capability to strong-arm
other nations with its economic weapons, China has become the world’s most active exponent of public diplomacy. It has spent an
estimated US$8 billion on its international broadcasting efforts, many millions more on its worldwide network of Confucius
Institutes, and additional large sums on projects as significant as educational exchanges and as trivial as advertising on electronic
billboards in New York’s Times Square. Further, some of China’s best universities are embracing public diplomacy as an academic
discipline, training the country’s next generation of experts in this field. Butwhat is China getting for all this money
and effort? Public opinion polls from around the world indicate decidedly mixed results . In parts of
Africa, where China has built roads and stadiums, its popularity has risen. Elsewhere, however, China is viewed warily as
heavy-handed and insensitive to the political and economic lives of countries where it is
expanding its presence. In two weeks of discussions with Chinese public diplomacy practitioners and scholars in Beijing and
Shanghai, I found no consensus about what China’s public diplomacy strategy should be or what China wants from its public
diplomacy efforts. The
enthusiasm is there, but an overarching plan is not. Part of this lack of
coherence is due to China’s slow acceptance of the realities of being a superpower . I heard
complaints from many quarters about how unfairly China is being treated by the international news media, and claims that China is
not receiving the respect that it deserves. When I said that mistreatment – real or imagined, deserved or not – is something
superpowers must learn to live with, my Chinese colleagues did not seem to understand this facet of political reality. Chinese public
diplomacy leaders need a better appreciation of the give-and-take of superpower diplomacy. As a first step, they should understand
that reciprocity is important if multilateral relationships are to take shape. If China wants to export its Confucius Institutes, it must
allow the United States and others to set up comparable cultural centers (in comparable numbers) in China. If China wants to extend
the reach of its international broadcasting, it must allow other countries to have broadcast and online access to the Chinese public.
Such goals may seem far-fetched, given China’s reluctance to allow substantive political debate within its borders, much less permit
outsiders to contribute to any such debate. But more than anything else, China is determined to be a global player. Its embrace of
public diplomacy, rather than endangering the world with another superpower arms race, is encouraging. If the United States and
other nations persist in engaging with China within the realm of public diplomacy, China might be nudged toward increased
openness. This could enable the newest superpower to continue to rely on soft power.

It’s low now –

Ford ‘12(Christopher, Senior Fellow at the Hudson Institute in Washington D.C., He Previously Served as US Special
Representative for Nuclear Nonproliferation, “Soft On Soft Power” SAID Review, Vol 32, No 1.)

This is not to say that China’s “soft power” has no limits, of course. CCP officials are right that China
lags almost
immeasurably far behind the United States and the West in many aspects of global socio-cultural
clout. We will know when Beijing has really arrived in the big leagues of “soft” influence when American
students try to learn Mandarin by the millions and clamor desperately for admission to
Chinese universities, U.S. distribution of Chinese films has to be limited in order to protect Hollywood from extinction,
Americans adopt Sinic forenames in order to facilitate smoother interaction with Chinese who find Western ones difficult to
pronounce, throngs of young musicians in the West strive for international fame and fortune playing Chinese instruments in recitals
of traditional Chinese music, and citizens in the world’s democracies have to be watched carefully by the police in order to keep
them from demanding a Leninist political system. Today, of course, these
things occur only in reverse. If soft power is,
as Joseph Nye has argued, about the power of “attraction” rather than “coercion ,”64 it is hard to avoid
the conclusion that despite Beijing’s ongoing efforts to promote Chinese culture as a form of “soft
power,” the rest of the world is not much interested, and indeed finds itself repelled by
Chinese politics.
---Soft Power- AT: Nye

Nye’s crazy – Nye’s theory is compromised by his realist framework and dependence on US
supremacy
Cammack ‘8, - Paul -Head of the Department of Politics and Philosophy at Manchester Metropolitan University, Autumn (“Smart Power and US Leadership: A Critique
of Joseph Nye,” 49th Parallel, Vol 22, pg. 7-9

Nye has never embraced the implications of his own ‘hypothesis of power diffusion’. Within
As is evident,

the uncompromisingly realist framework to which he clings, he cannot imagine any situation other
than one in which the US unequivocally takes the lead—and this despite a youthful flirtation with the notion of ‘multiple leadership’.16
In successive prefaces to the hardback and paperback editions of Bound to Lead he argued that ‘if the most powerful country fails to lead, the consequences for the rest of the

His argument at that time, that


world may be disastrous’;17 and ‘if the largest power does not lead in organizing multilateral action, no one will’.18

interdependence could only be managed by continued US leadership, was no aberration .19 It is consistent
with his more recent insistence that the need to cooperate does not preclude the claim to lead: ‘we are not only bound to lead, but bound to cooperate’.20 Elsewhere, however,

he goes further, adopting rhetorical formulations that overlook cooperation altogether to represent
the choice as being between American leadership on the one hand, and abstention or isolationism
on the other: [H]ow will the only superpower guide its foreign policy after the experience of the Iraq War? Will it provide global leadership or conclude that the best
course in world affairs is to remain uninvolved?21 Again, this is no aberration, but a reflection of an enduring cast of mind .
The same thought was expressed as follows in Bound to Lead: Although polyarchy rests in part on the diffusion of power to nonstate actors and small states, its implications for
stability and welfare will depend heavily on whether the largest state takes a lead in organizing collective action among other states or if it simply allows a new feudalism to
develop [emphasis mine].22 Given Nye’s various remarks on multiple leadership, the diffusion of power, and cooperation (not to mention the everyday understanding of
cooperation as association for common benefit), one might have expected a middle term—engagement with and active contribution to the leadership of others when

his
appropriate. As it is, own definition of cooperation seems to be ‘my way or the highway’. To see why, we need to turn briefly
to the theoretical foundation on which his position rests.
South China Seas Answers
Frontline

No SCS conflict—tensions cause conflict management

Gupta ’11 (10/23 Rukmani Gupta, Associate Fellow at the Institute for Defence Studies and Analyses,10/23/11, South China Sea
Conflict? No Way, the-diplomat.com/2011/10/23/south-china-sea-conflict-no-way/

These suggestions to recalibrate Indian policy towards the South China Sea and its relationship with Vietnam are premature at best.
Despite the rhetoric, conflict in the South China Sea may well not be inevitable. If the history of
dialogue between the parties is any indication, then current tensions are likely to result in forward
movement. In the aftermath of statements by the United States, and skirmishes over fishing vessels, ASEAN and
China agreed upon the Guidelines on the Implementation of the Declaration on the Conduct of Parties in the South China
Sea at the Bali Summit in July 2010. And recent tensions may well prod the parties towards a more binding code of
conduct. This isn’t to suggest that territorial claims and sovereignty issues will be resolved, but certainly they can
become more manageable to prevent military conflict. There’s a common interest in making the
disputes more manageable, essentially because, nationalistic rhetoric notwithstanding, the parties to the dispute
recognize that there are real material benefits at stake . A disruption of maritime trade through the
South China Sea would entail economic losses – and not only for the littoral states. No party to the dispute,
including China, has thus far challenged the principle of freedom of navigation for global trade through the
South China Sea. The states of the region are signatories to the UNCLOS , which provides that ‘Coastal
States have sovereign rights in a 200-nautical mile exclusive economic zone (EEZ) with respect to natural resources and certain
economic activities, and exercise jurisdiction over marine science research and environmental protection’ but that ‘All other States
have freedom of navigation and over flight in the EEZ, as well as freedom to lay submarine cables and pipelines.’ The prospect of
threats to SLOCS thus seems somewhat exaggerated.

No war over Senkaku or the SCS

Carlson ’13 (Allen Carlson is an Associate Professor in Cornell University’s Government Department. He was granted his PhD
from Yale University’s Political Science Department. His undergraduate degree is from Colby College. In 2005 his Unifying China,
Integrating with the World: Securing Chinese Sovereignty in the Reform Era was published by Stanford University Press. He has
also written articles that appeared in the Journal of Contemporary China, Pacific Affairs, Asia Policy, and Nations and Nationalism.
In addition, he has published monographs for the National Committee on U.S.-China Relations and the East-West Center
Washington. Carlson was a Fulbright-Hays scholar at Peking University during the 2004-2005 academic year. In 2005 he was
chosen to participate in the National Committee’s Public Intellectuals Program, and he currently serves as an adviser to Cornell’s
China Asia Pacific Studies program and its East Asia Program. Carlson is currently working on a project exploring the issue of
nontraditional security in China’s emerging relationship with the rest of the international system. His most recent publications are
the co-edited Contemporary Chinese Politics: New Sources, Methods and Field Strategies (Cambridge University Press, 2010) and
New Frontiers in China’s Foreign Relations (Lexington, 2011). China Keeps the Peace at Sea China Keeps the Peace at Sea Why the
Dragon Doesn't Want War Allen Carlson February 21, 2013

At times in the past few months, China and Japan have appeared almost ready to do battle over the
Senkaku (Diaoyu) Islands --which are administered by Tokyo but claimed by both countries -- and to ignite a war that could be
bigger than any since World War II. Although Tokyo and Beijing have been shadowboxing over the territory for years, the standoff
reached a new low in the fall, when the Japanese government nationalized some of the islands by purchasing them from a private
owner. The decision set off a wave of violent anti-Japanese demonstrations across China. In the wake of these events, the conflict
quickly reached what political scientists call a state of equivalent retaliation -- a situation in which both countries believe that it is
imperative to respond in kind to any and all perceived slights. As a result, it may have seemed that armed engagement was
imminent.
Yet, months later, nothing has happened . And despite their aggressive posturing in the
disputed territory, both sides now show glimmers of willingness to dial down hostilities and to
reestablish stability . Some analysts have cited North Korea's recent nuclear test as a factor in the countries' reluctance to
engage in military conflict. They argue that the detonation, and Kim Jong Un's belligerence, brought China and Japan together,
unsettling them and placing their differences in a scarier context. Rory Medcalf, a senior fellow at the Brookings Institution,
explained that "the nuclear test gives the leadership in both Beijing and Tokyo a chance to focus on a foreign and security policy
challenge where their interests are not diametrically at odds." The nuclear test, though, is a red herring in terms of the conflict over
In truth, the roots of the conflict -- and the reasons it has not yet exploded -- are
the disputed islands.
much deeper . Put simply, China cannot afford military conflict with any of its Asian neighbors. It
is not that China believes it would lose such a spat; the country increasingly enjoys strategic
superiority over the entire region, and it is difficult to imagine that its forces would be beaten
in a direct engagement over the islands , in the South China Sea or in the disputed regions
along the Sino-Indian border. However, Chinese officials see that even the most pronounced
victory would be outweighed by the collateral damage that such a use of force would cause to
Beijing's two most fundamental national interests -- economic growth and preventing the
escalation of radical nationalist sentiment at home. These constraints, rather than any
external deterrent , will keep Xi Jinping, China's new leader, from authorizing the use of deadly
force in the Diaoyu Islands theater. For over three decades , Beijing has promoted peace and stability
in Asia to facilitate conditions amenable to China's economic development. The origins of the policy
can be traced back to the late 1970s, when Deng Xiaoping repeatedly contended that to move beyond the economically
debilitating Maoist period, China would have to seek a common ground with its neighbors. Promoting cooperation in the region
would allow China to spend less on military preparedness, focus on making the country a more welcoming destination for foreign
investment, and foster better trade relations. All of this would strengthen the Chinese economy. Deng was right. Today, China's
economy is second only to that of the United States. The fundamentals of Deng's grand economic strategy are still revered in
Beijing. But any war in the region would erode the hard-won, and precariously held, political capital that China has gained in the
last several decades. It would also disrupt trade relations, complicate efforts to promote the yuan as an international currency,
and send shock waves through the country's economic system at a time when it can ill afford them. There is thus little reason to
specter of rising Chinese nationalism,
think that China is readying for war with Japan. At the same time, the
although often seen as a promoter of conflict, further limits the prospects for armed
engagement . This is because Beijing will try to discourage nationalism if it fears it may lose control or be forced by popular
sentiment to take an action it deems unwise. Ever since the Tiananmen Square massacre put questions
about the Chinese Communist Party's right to govern before the population, successive
generations of Chinese leaders have carefully negotiated a balance between promoting nationalist
sentiment and preventing it from boiling over. In the process, they cemented the legitimacy of their rule. A war with Japan could
easily upset that balance by inflaming nationalism that could blow back against China's leaders. Consider a hypothetical scenario
in which a uniformed Chinese military member is killed during a firefight with Japanese soldiers. Regardless of the specific
circumstances, the casualty would create a new martyr in China and, almost as quickly, catalyze popular protests against Japan.
Demonstrators would call for blood, and if the government (fearing economic instability) did not extract enough, citizens would
agitate against Beijing itself. Those in Zhongnanhai, the Chinese leadership compound in Beijing, would find themselves between a
rock and a hard place. It is possible that Xi lost track of these basic facts during the fanfare of his rise to power and in the face of
renewed Japanese assertiveness. It is also possible that the Chinese state is more rotten at the core than is understood. That is, party
elites believe that a diversionary war is the only way to hold on to power -- damn the economic and social consequences. But Xi does
not seem blind to the principles that have served Beijing so well over the last few decades. Indeed, although he recently warned
unnamed others about infringing upon China's "national core interests" during a foreign policy speech to members of the Politburo,
he also underscored China's commitment to "never pursue development at the cost of sacrificing other country's interests" and to
never "benefit ourselves at others' expense or do harm to any neighbor." Of course, wars do happen -- and still could in the East
China Sea. Should either side draw first blood through accident or an unexpected move, Sino-Japanese relations would be pushed
into terrain that has not been charted since the middle of the last century. However, understanding that war would be a no-win
situation, China has avoided rushing over the brink. This relative restraint seems to have surprised everyone. But it shouldn't.
Beijing will continue to disagree with Tokyo over the sovereign status of the islands, and will not budge in its negotiating
position over disputed territory. However, it cannot take the risk of going to war over a few rocks in the sea.
On the contrary, in the coming months it will quietly seek a way to shelve the dispute in return for
securing regional stability, facilitating economic development, and keeping a lid on the Pandora's box
of rising nationalist sentiment. The ensuing peace , while unlikely to be deep, or especially conducive to improving
Sino-Japanese relations, will be enduring.

Wont escalate—tensions are bluffs for negotiating leverage

Gupta ’11 (10/23 Rukmani Gupta, Associate Fellow at the Institute for Defence Studies and Analyses,10/23/11, South China Sea
Conflict? No Way, the-diplomat.com/2011/10/23/south-china-sea-conflict-no-way/

Despite what opinion pieces in the Global Times may say, there’s reason to suspect that China
doesn’t want to escalate
conflict in the region. Although commentary from the United States has suggested that China considers
the South China Sea a ‘core interest,’ no official Chinese writing can be found to corroborate this . In
addition, China’s caution can also be seen as a reflection on Chinese military capabilities , which
aren’t seen as strong enough to win a war over the South China Sea. In fact, the China National
Defence News, published by the Chinese People’s Liberation Army’s General Political Department, has likened the use of
force by China in the South China Sea to shooting one’s own foot. Not only would the use of force
bring ASEAN together on the issue, it could conceivably involve the United States and Japan, derail China’s
plans for continued economic growth and undo China’s diplomacy . Chinese declarations on the
South China Sea can therefore be seen as attempts to exaggerate claims so as to secure a better
negotiating stance.

No escalation – US wins decisively

AP ’11
(“China challenges U.S. edge in Asia-Pacific”)

The U.S. PacificCommand has 325,000 personnel, five aircraft-carrier strike groups, 180 ships and
nearly 2,000 aircraft. Tens of thousands of forces stay on China's doorstep at long-established bases in
South Korea and Japan. China's defense spending is still dwarfed by the United States. Even if
China really invests twice as much in its military as its official $91.5 billion budget, that would still be only
about a quarter of U.S. spending. It has no aircraft carriers and lags the United States in defense
technology. Some of its most vaunted recent military advances will take years to reach operation. For
example, China test-flew its stealth fighter in January, months earlier than U.S. intelligence expected, but U.S. Defense Secretary
Robert M. Gates says China
will still only have a couple of hundred of these "fifth-generation" jets by
2025. The United States should have 1,500 by then.

South China Seas are stable – China lacks capability and interdependence checks
Rosenberg 9 (David, Professor of Political Science – Middlebury College and Research Fellow at the Research School of Pacific
and Asian Studies – Australian National University, “Dire Straits: Competing Security Priorities in the South China Sea”, The Asia-
Pacific Journal, 3-20, http://japanfocus.org/-David-Rosenberg/1773)

From the Taiwan Strait to the Strait of Malacca, security concerns are growing around the South China
Sea. While the Bush Administration sees a resurgent Chinese military threat across the Taiwan Strait and a terrorist threat in
the Strait of Malacca, many countries between the Straits are more concerned about security for their maritime resources from
the threats of competitors, traffickers, poachers, and pirates. Security Concerns in the South China Sea Several recent
statements and appointments highlight the current Bush administration view of China's threat to Taiwan. Porter Goss, director
of the U.S. Central Intelligence Agency, warned that improved Chinese capabilities not only threaten Taiwan but also U.S. forces
in the (western Pacific) region. U.S. Defense Secretary Donald Rumsfeld worried that the Chinese navy was building some
amphibious landing ships for possible use across the Taiwan Strait. The appointment of combative neoconservative John Bolton
as U.S. ambassador to the United Nations sends a clear and ominous signal: formerly a paid consultant to the Taiwanese
government, Bolton has advocated Taiwan's independence and its full U.N. membership. Then, in February 2005, Secretary of
State Condoleezza Rice, Defense Secretary Donald Rumsfeld and their Japanese counterparts announced a significant alteration
in the U.S.-Japan Security Alliance by identifying security in the Taiwan Strait as a "common strategic objective." Has there been
any big shift in the balance of power around the Taiwan Strait that warrants this U.S. response? The Chinese defense
budget has grown by double-digit increases for the past fourteen years. This year it's up by 12 percent. But that is not
significantly faster than the Chinese economy as a whole is growing. China is modernizing its defenses -- adding anti-ship
missiles to aircraft, acquiring AWACS-airborne early warning and control systems, guided missile destroyers and frigates.
However, its power projection capabilities are limited. It lacks any long-range amphibious
capability or support infrastructure to supply forces over long distances for a protracted
period. It also lacks heavy cargo-carrying aircraft, comprehensive air defenses, seaworthy
ships, and aircraft carriers. Given the current state of Chinese equipment and training, the
Chinese have no capability to pursue an expansionist maritime policy in the Taiwan Strait or the
South China Sea. [1] By contrast, the U.S. has overwhelming military superiority and an
expansive network of military bases across the Asia-Pacific. The U.S. Pacific Fleet is the world's
largest naval command, including approximately 190 ships, about 1,400 Navy and Marine Corps aircraft and 35 shore
installations. Over 300,000 Navy, Army, Air Force, Marine Corps, Special Operations, and Intelligence military personnel are
integrated under the unified command of PACOM, the U.S. Pacific Command. What are China's strategic goals between the
Straits? China'sDefense White Paper of 2002 emphasizes the importance of pursuing peaceful
external relations initiatives through multilateral, cooperative approaches to promote domestic
development. The most recent Defense White Paper, published in December of 2004, reiterates this priority. More important
than statements of good intentions, however, China has taken significant steps to implement this goal. It
was evident in the Framework Agreement on ASEAN-China Comprehensive Economic Cooperation, negotiated in November
2002. That led to the agreement signed in November 2004 to implement an ASEAN-China Free Trade Area (FTA) by 2010.
Following the 10th Summit Meeting of the Association of Southeast Asian Nations (ASEAN), in Vientiane, Laos in November
2004, Beijing held its own summit with ASEAN leaders (ASEAN Plus One) and then joined Japan and the Republic of Korea in
discussions with ASEAN leaders (ASEAN Plus Three, or APT). Beijing had earlier in November hosted the first Security Policy
Conference of the ASEAN Regional Forum. It featured an anti-piracy drill and a workshop on countering terrorism. Regional
Economic and Financial Agreements Regional economic agreements were the main achievements of these meetings. However,
the ASEAN Plus Three sessions identified other areas for cooperation, including deeper cooperation in investment and finance,
expanded security dialogue and cooperation, expanded cultural exchanges, and periodic progress reviews. Perhaps the most
dramatic developments have occurred in regional financial cooperation. Finance ministers of the ASEAN+3 countries have
launched an Asian Bond Markets Initiative and the regional central bankers group set up two Asian Bond Funds in early 2005.
These are key steps in addressing one of the major weaknesses in the region's development as indicated by the currency and
financial crisis that struck large parts of the region in 1997: the heavy reliance by firms on short-term bank loans for financing.
As Jennifer Amyx notes, many countries in East Asia maintain high savings rates but, because of the absence of stable long-term
debt markets, the savings deposited into local banks tended to be funneled out to international financial centers and then back
into the region as short-term foreign currency loans. This situation creates a problem referred to as a "double mismatch" -- that
is, a mismatch between debt maturities (short-term borrowing for long-term investments) and the denomination of this debt (in
foreign rather than local currencies). [2] The ASEAN+3 finance ministers had earlier set up a network of bilateral currency swaps
to permit a country beset by a speculative attack to draw on reserves of other nations. The program -- the Chiang Mai Initiative
(CMI) -- went into effect at the end of 2003. Japan, with the largest reserves in the region, led negotiations over swap
arrangements and will play the role of arbitrator for currency loans. China, another potential lender with substantial reserves in
excess of potential needs, also lent its support to the CMI. Widespread participation by ASEAN Plus Three members in these
initiatives encourages smooth financial liberalization processes and thereby bolsters regional stability. It also reinforces the
efforts of various working groups to improve transparency and information dissemination and to strengthen settlement systems
and regulatory reforms. China'sshift to a more proactive position on regional financial cooperation
has greatly facilitated these recent financial developments. As a result, interdependence
between the Chinese economy and other economies in the region has deepened significantly in recent
years. Today, trade by ASEAN member nations with China far exceeds trade conducted within the ASEAN grouping, while
China is predicted to soon overtake the United States as Japan's top trading partner. Levels of investment in China by countries
in the region are also extremely high. The worst case scenario is not Chinese domination but a Chinese
meltdown, as many regional monetary authorities are quick to note.

Many factors check (Geography, ASEAN, Shipping Lanes, Political Cost)

Joyner 98 (Chris, Professor of International Relations – Georgetown University, New England Law Review, Spring, Lexis)

Nevertheless, several factors suggest the unlikelihood of large-scale military conflict over the
Spratlys in the foreseeable future. For one, there is the geography: These islands are
scattered over an immense area, nearly 200,000 square kilometers. Considerable room is available
for naval patrols to maneuver and miss contact with one another. Relatedly, the Spratlys are
more than 300 kilometers (185 miles) from [*837] the Philippine and Vietnamese coasts, and more than 1000
kilometers (600 miles) from mainland China. This distance presents serious difficulties for any
claimant government to patrol more than a small area of the Spratly archipelago at any one time,
especially given these states’ relatively weak capabilities for projecting armed forces. No claimant state possesses sufficient
logistical support capabilities to ensure effective occupation and maintain extended control over these islands, which
underscores the importance of relative naval size. Even so, these conditions presumably should permit
greater
opportunities for confidence building measures to be considered as alternative strategies.
50 The Cold Wars passing has also fostered a sense of rapprochement throughout Asia, which
makes the political costs of a large-scale military conflict in Spratlys less acceptable to the
PRC or Taiwan. 51 The dynamic economic expansion of ASEAN counties, increasingly close
links with the international community, and strategically significant shipping lanes through
the South China Sea -- all converge to dissuade overt attempts by any state, including the
PRC, to strive for regional military domination. That the economies of both the PRC and Taiwan have become
increasingly interdependent with those of Southeast Asian states, including other claimants to the Spratlys, underscores that
reluctance.

Your impact is empirically denied and military intelligence proves no risk of war

Kyodo News 99 (“China unlikely to launch war in S. China Sea.,” 9/24,


http://www.thefreelibrary.com/China+unlikely+to+launch+war+in+S.+China+Sea.-a055944319,

China will continue to strengthen its military presence in the disputed Spratly Islands but is
not likely to clash militarily with five Southeast Asian countries that also have claims on the island chain in the South
China Sea, according to a classified U.S. military document obtained Friday by Kyodo News. The U.S. Pacific
Command document says Beijing will continue to fortify its military structures and facilities in the Spratlys but that " because
of potential damage to China's international position, China probably does not have immediate
plans to engage in military activities." "While minor clashes remain a distinct possibility," the
document adds, "a major conflict is unlikely in the Spratlys or the South China Sea." Like the Korean Peninsula and
the Taiwan Strait, the South China Sea is a potential flash point in the Asia-Pacific region. China's move to flex its military muscle
China began beefing up its military structures and facilities
in the area has worried countries in the region.
in the Spratlys in 1988. The U.S. document says China now has seven permanent military outposts in the disputed
archipelago, comprised of more than 100 small coral islands and reefs scattered in the southern half of the South China Sea.
---South China Seas- No Escalation

No escalation

Bush and O’Hanlon 7 (Richard and Michael, Senior Fellows – Brookings Institution, “U.S. Grapples With China’s Rise,
Taiwan”, The Daily Yomiuri (Tokyo), 5-3, Lexis)

But most of the issues and frictions that accompany China's rise can be managed. The good news
is that China and the United States, not to mention other key regional players like Japan, now have politicians and bureaucracies
that are relatively good at preventing serious problems from becoming grounds for war. China will want to flex its
military muscle more in the future, but it also wants economic prosperity for the political stability that
comes with it. In addition, the United States and its regional partners know how to maintain open dialogue with Beijing while
also sustaining vigorous defense alliances. China has enough reason to worry about nuclear weapons and global instability
that it will
not be totally oblivious to our concerns about proliferating countries such as Iran and North Korea.
Conflict with the littoral nations of Japan, the Philippines or Vietnam over disputed seabed
resources (like oil in the East China Sea or small islets in the South China Sea ) is highly
unlikely.

U.S. won’t intervene

Betts 97 (Prof. Political Science and International Security Policy Studies, Vietnam Joins The World)

Neither Hanoi nor the other claimants who oppose Beijing are likely to be able to prevent the Chinese from consolidating
control of the Spratlys if the decision is taken in Beijing to do so. Only the U.S. Navy could do that, and Washington has
no commitment to go to war with a nuclear-armed power over those reefs and rocks.
Although some in the American naval circles invoke the importance of protecting transit rights as a
reason to contest Chinese claims to the Spratlys, these is no plausible reason to expect a
strategic consensus for doing anything. Washington even failed to back Malaysia or its ally the
Philippines in the mid-1970s when China took the Paracals, and more recently it tacitly agreed to the joint
venture in the South China Sea between an American company, Crestone, and China.
Space Colonization
All previous human spaceflight is insignificant – long term colonization is still infeasible.

Launius 10 – (2010, Roger, PhD, Curator, Planetary Exploration Programs, National Air and Space Museum, expert on
Aerospace history, fellow and board member of the American Astronautical Society, “Can we colonize the solar system? Human
biology and survival in the extreme space environment,” Endeavour Volume 34, Issue 3, September 2010, Pages 122-129, science
direct, )

Although microbial life might survive the extreme conditions of space, for
Homo sapien sapiens the space
environment remains remarkably dangerous to life. One space life scientist, Vadim Rygalov, remarked that
ensuring human life during spaceflight was largely about providing the basics of human
physiological needs. From the most critical – meaning that its absence would cause immediate death, to the
least critical – these include such constants available here on Earth of atmospheric pressure, breathable
oxygen, temperature, drinking water, food, gravitational pull on physical systems, radiation
mitigation, and others of a less immediate nature. As technologies, and knowledge about them, stand at this time,
humans are able to venture into space for short periods of less than a year only by supplying all of these
needs either by taking everything with them (oxygen, food, air, etc.) or creating them artificially (pressurized vehicles, centrifugal
force to substitute for gravity, etc.).10 Spaceflight would be much easier if humans could go into hibernation during the extremes of
spaceflight, as did the Streptococcus mitis bacteria. Resolvingthese issues has proven difficult but not
insurmountable for such basic spaceflight activities as those undertaken during the heroic age of space exploration
when the United States and the Soviet Union raced to the Moon. Overcoming the technological hurdles
encountered during the Mercury, Gemini, and Apollo programs were child's play in comparison
to the threat to human life posed by long duration, deep space missions to such places as Mars.
Even the most sophisticated of those, the lunar landings of Project Apollo, were relatively short camping
trips on an exceptionally close body in the solar system , and like many camping trips undertaken by Americans
the astronauts took with them everything they would need to use while there. This approach will continue to work
well until the destination is so far away that resupply from Earth becomes highly problematic if
not impossible if the length of time to be gone is so great that resupply proves infeasible. There is no question that
the U.S. could return to the Moon in a more dynamic and robust version of Apollo ; it could also build
a research station there and resupply it from Earth while rotating crews and resupplying from Earth on a regular basis. In this
instance, the lunar research station might look something like a more sophisticated and difficult to
support version of the Antarctic research stations. A difficult challenge, yes; but certainly it is something that
could be accomplished with presently envisioned technologies.11 The real difficulty is that at the point a lunar
research station becomes a colony profound changes to the manner in which humans interact
with the environment beyond Earth must take place. Countermeasures for core challenges –
gravity, radiation, particulates, and ancillary effects – provide serious challenges for humans engaged in space
colonization (Figure 4).

Colonization Impossible – Multiple Barriers


Brandt 7
(David Brandt, The Hard SF, articles focusing on delineating science from science fiction, 5-10-7, “Can Space Colonization Guarantee
Human Survival?”, http://www.hardsf.org/IssuSpac.htm)

Can Space Colonization Guarantee Human Survival? Many people have argued that as long as
humans live only on Earth, we have "all our eggs in one basket". They suggest we need space
colonies to insure the future of the species. There are many current and potential threats to the human race.
However, considering the human source of many of the threats and the timescales involved, I'm
not sure that space colonization should be the top priority in preempting those threats. Timescales
To consider how well space colonization is likely to solve our problems we need to ask what the
timescales of sustainable, independent space colonies are. If, after disaster strikes Earth, Earth is
still able to supplement the needs of space colonies, then those space colonies aren't necessarily
essential to continuing the human race. We have to ask when spaces colonies would be
functioning without need of any assistance from Earth. Truly independent space colonies must
not simply provide bare nutrition, air, heat, and habitat repair for 100 years. They should have a
non-traumatizing environment with enough people to protect against dangerous levels of
inbreeding – able to last and progress indefinitely. There will also be a minimum number of
people required for any space colony in order to provide needed manpower in various
occupations (one person with multiple occupations doesn’t help if you need two of those occupations in different places at the
same time). How does that compare to the timescales of threats from climate change, environmental crisis, nuclear / bio weapons
and accidents, possible nanotech weapons or accidents, overpopulation, etc.? We also have to consider threats to the global
economy, since an economic collapse would presumably at least interrupt efforts towards establishing space colonies. Economic
crises also increase risks of war, which could have apocalyptic consequences. Even assuming the ultimate solution of human survival
is space colonization, we may need to find a way to extend the lifespan of human civilization and economy on Earth in order to have
time to accomplish sustainable space colonization. Consider the possible habitats. Space stations in orbit around Earth or at L5 have
little natural resources at their location other than solar energy. The
Moon has no atmosphere, a limited amount
of water at best, which part of the Moon has access to solar energy varies during the month,
and it's not considered one of the solar system's better sources of minerals. Venus is extremely
hot, the atmosphere is dangerous and with the cloud cover I'm not sure how practical solar
energy would be at the surface. Mars has too little atmosphere and accessible water is
questionable, etc. Some of the outer planet's moons may have enough ice and raw materials,
but are very cold, lack usable atmospheres and get limited solar energy. And so on. We may be able
to establish bases at some of these places in a realistically short amount of time, but not independent ones. Any colony that wants
to get resources from post-apocalyptic Earth will need to have spaceships that can land on Earth and later achieve escape velocity
the needed resources may not be available
from Earth while carrying cargo without help from Earth. Otherwise,
from a single astronomical body. That could require longer distance travel between bodies - whether that's between
asteroids, between moons, between planets or some other combination. Significant space travel ability may be essential. A colony
would need an industrial base capable of extracting and refining raw materials, and making useful things from them. Interstellar
colonies and terraforming of planets in our solar system are longer range goals. Colonies in any
place other than an Earth-like planet will require a substantial infrastructure to allow humans to exist
in an otherwise deadly environment. The colony needs to be able to maintain and repair that infrastructure... There is a significant
difference between an enormous disaster on Earth and one at any space colony we can expect for at least a century. Even something
on the scale of a "dinosaur killer" asteroid impact won't necessarily kill all humans on Earth. (However, if the world economy /
technology is setback too much it may not be possible to re-achieve a hi-tech civilization. We've extracted most minerals / fossil
fuels that can be gotten without hi-tech, a post-disaster society may be left unable to get these.) It will be a long time before an
independent space colony could grow to the point some of its people could survive after a major disaster. Meanwhile, we have not
yet solved the physical and psychological problems that develop during months of low gravity. Most of the physical issues may not
be significant for those who never intend to return to Earth-type gravities. Psychological issues remain. Some physical issues may
arise when dealing with years and decades in low gravity. Even in shorter spans of time, weakening bones may have serious
consequences in low gravity situations. Weakened hip bones may be a problem for women giving birth in low gravity. Other stressful
activities may also be problematic. We need to find out how low gravity will effect a fetus during pregnancy and child growth
afterwards. Identifying and resolving all the issues is likely to take many years. Currently, our society is not inclined to invest that
much in either stopping global warming (and other threats) or space habitats. It strikes me as improbable that we will see a heavy
investment in both of them at the same time in the next period of time. My impression is the best chance for human survival is
focusing as much as possible on one or the other of the two paths, and that space colonization will not solve the problem within the
limited time-frame. Of course, if governments refuse to fund solutions to the environmental crisis, but budget money for space
habitats we should use that money. Hopefully, governments will respond to the crisis before it’s too late and the problems will be
brought under control and within safe limits. Then there will be no reason not to expand out into the universe. Postscript For those
who still believe space colonization should be the priority, I would like to suggest one piece of advice. The known threats to human
survival in the next century or so are not vast earthquakes and volcanoes, asteroid impacts, supernovas or other natural disasters.
Most of them are at least partly man-made. If
the same problems are not to threaten survival of humans on
space colonies, we either have to make humans on Earth act more responsibly to ensure
survival before we colonize, or we need to know how to insure that those people who colonize
are not so prone to make the same mistakes their Earthly brothers do. If space colonization ends
up amounting to running away from our problems , we will not have changed the odds of human
survival by much. Space colonies would need to be planned in a way to avoid this fate.

Space colonization is impossible --- humans can’t adjust

Piersma 10, Theunis ,professor of animal ecology at the University of Groningen in the Netherlands and senior research scientist
at the Royal Netherlands Institute for Sea Research in Den Burg, “Why space is the impossible frontier,” NewScientist, 11-16-10,
http://www.newscientist.com/article/mg20827860.100-why-space-is-the-impossible-frontier.html

Hawking, Obama and other proponents of long-term space travel are making a grave error. Humans cannot leave Earth
for the several years that it takes to travel to Mars and back, for the simple reason that our
biology is intimately connected to Earth. To function properly, we need gravity. Without it, the
environment is less demanding on the human body in several ways, and this shows upon the
return to Earth. Remember the sight of weakened astronauts emerging after the Apollo missions?
That is as nothing compared with what would happen to astronauts returning from Mars. One of
the first things to be affected is the heart, which shrinks by as much as a quarter after just one week in
orbit (The New England Journal of Medicine, vol 358, p 1370). Heart atrophy leads to decreases in blood
pressure and the amount of blood pushed out by the heart. In this way heart atrophy leads to
reduced exercise capacity. Astronauts returning to Earth after several months in the International Space Station experience
dizziness and blackouts because blood does not reach their brains in sufficient quantities. Six weeks in bed
leads to about as much atrophy of the heart as one week in space, suggesting that the atrophy is caused by both weightlessness and
the concomitant reduction in exercise. Other
muscle tissue suffers too. The effects of weightlessness on
the muscles of the limbs are easy to verify experimentally. Because they bear the body's weight, the "anti-
gravity" muscles of the thighs and calves degenerate significantly when they are made
redundant during space flight. Despite the best attempts to give replacement exercise to crew members on the
International Space Station, after six months they had still lost 13 per cent of their calf muscle volume and 32 per cent of the
maximum power that their leg muscles could deliver (Journal of Applied Physiology, vol 106, p 1159). Various metabolic
changes also occur, including a decreased capacity for fat oxidation , which can lead to the build-up of
fat in atrophied muscle. Space travellers also suffer deterioration of immune function both during and after their missions
(Aviation, Space, and Environmental Medicine, vol 79, p 835). Arguably the most fearsome effect on bodies is bone
loss (The Lancet, vol 355, p 1569). Although the hardness and strength of bone, and the relative ease
with which it fossilises, give it an appearance of permanence, bone is actually a living and
remarkably flexible tissue. In the late 19th century, the German anatomist Julius Wolff discovered that bones adjust to the
loads that they are placed under. A decrease in load leads to the loss of bone material, while an increase leads to thicker bone. It is
no surprise, then, that in
the microgravity of space bones demineralise, especially those which
normally bear the greatest load. Cosmonauts who spent half a year in space lost up to a quarter
of the material in their shin bones, despite intensive exercise (The Lancet, vol 355, p 1607). Although
experiments on chicken embryos on the International Space Station have established that bone formation does continue in
microgravity, formation rates are overtaken by bone loss. What is of greatest concern here is that, unlike muscle loss which levels off
with time, bone
loss seems to continue at a steady rate of 1 to 2 per cent for every month of
weightlessness. During a three-year mission to Mars, space travellers could lose around 50 per
cent of their bone material, which would make it extremely difficult to return to Earth and its
gravitational forces. Bone loss during space travel certainly brings home the maxim "use it or lose it". Bone loss is not permanent.
Within six months of their return to Earth, those cosmonauts who spent half a year in space did show partial recovery of bone mass.
However, even after a year of recovery, men who had been experimentally exposed to three months of total bed rest had not fully
regained all the lost bone, though their calf muscles had recovered much earlier (Bone, vol 44, p 214). Space agencies will have to
become very creative in addressing the issue of bone loss during flights to Mars. There are concepts in development for spacecraft
with artificial gravity, but nobody even knows what gravitational force is needed to avoid the problems. So far, boneless creatures
such as jellyfish are much more likely than people to be able to return safely to Earth after multi-year space trips. For humans,
gravity is a Mars bar. The impossibility of an escape to space is just one of many examples of how our
bodies, and those of our fellow organisms, are inseparable from the environments in which we
live. In our futuristic ambitions we should not forget that our minds and bodies are connected to Earth as by an umbilical cord.
Strait of Hormuz Answers
Frontline

No Strait of Hormuz Impact

Yadlin and Guzansky ’12 [Gen. (ret.) Amos Yadlin is the Director of INSS. Yoel Guzansky is a research associate at INSS, “The
Strait of Hormuz: Assessing and Neutralizing the Threat”, Strategic Assessment, Volume 14, No. 4, January 2012,
http://www.inss.org.il/upload/(FILE)1329728172.pdf]

Iran will likely try to avoid a comprehensive campaign in the Gulf that could well cost it dearly in military,
political, and economic terms. However, it will continue to threaten to close the Strait of Hormuz, a threat that serves its
strategic deterrence well, while taking advantage of the unique geographical conditions of the Strait and the global sensitivity to
every tremor in the world’s energy market. As evidence that Iran will likely actually not block the passageway, consider
that even at the height of the tanker war (1984-1987) the Strait remained open. Moreover, blocking the
Strait violates international law and may justifiably be considered grounds for going to war against Iran. 34 Thus
while discussion of this issue in recent years has often assumed that Iran intends to block the Strait, this intention
seems far from self-evident , as it is not in keeping with Iran’s true interests . Furthermore, Iran’s
ability to block the Strait effectively over a long period of time is not assured , because any such attempt would
immediately generate US military intervention designed to open the international shipping lane to oil and
gas tankers. Within the limitations of uncertainty regarding any forecast of a military confrontation, one could say that in light
of its superior military capabilities, the US could open a blockade at a tolerable cost. While the
importance of the Strait and its relative vulnerability, as well as the global economic state and the sensitive energy market
(especially in the northern hemisphere in the winter), are liable to amplify any event, the global energy market is in the
long run affected primarily by supply and demand , certainly more than by psychological
factors alone. Use of alternate land-based pipelines can compensate for the loss of a significant portion of the
oil exported from the Gulf through the Strait. This capability will grow if flow-enhancing additives are used and existing pipelines
are rehabilitated and put into action. Together with the oil that can be released and transported from the strategic reserves ,
and taking advantage of Saudi Arabia’s swing capacity, this amount may compensate for the amount of oil normally
moving through the Strait of Hormuz and significantly mitigate the ramifications of a blocked
passageway .

Iran won’t and can’t close the Strait

Singh ’12 (1/3 Michael Singh is managing director of The Washington Institute, 1/3/12, The Real Iranian Threat in the Gulf ,
www.washingtoninstitute.org/templateC06.php?CID=1789

Iran's bellicose rhetoric and Gulf wargames in recent days have given rise to the question of whether Tehran could close the
Strait of Hormuz. As many analysts have observed, the answer is no -- not for a meaningful period of time. Less
frequently addressed, however, is whether Iran would even try. The answer to that question is also "no" -- even the attempt
would have devastating strategic consequences for Iran. The presumable target of an Iranian effort to close the
Strait would be the United States. However, while we would of course be affected by any resulting rise in global oil
prices, the U.S. gets little of our petroleum from the Gulf. The U.S. imports only about 49 percent of the
petroleum we consume, and over half of those imports come from the Western Hemisphere. Less than 25 percent of U.S.
imports came from all the Gulf countries combined in October 2011 -- far less than is available in
the U.S. Strategic Petroleum Reserve, were Gulf supplies to be interrupted . China, on the other hand,
would find its oil supplies significantly threatened by an Iranian move against the Strait. China's most significant oil supplier is Saudi
Arabia. China also happens, however, to be Iran's primary oil customer and perhaps its most important ally: Beijing provides Iran
with its most sophisticated weaponry and with diplomatic cover at the United Nations. Thus a move to close the Strait would
backfire strategically by harming the interests of -- and likely alienating -- Iran's most important patron and cutting off Iran's own
economic lifeline, while doing little to imperil U.S. supplies of crude. It is perhaps no coincidence, then, that China quickly dispatched
Vice Foreign Minister Zhai Jun to Tehran in the wake of Iran's bellicose statements. In typically opaque fashion, the Chinese Foreign
Ministry said only that "China hopes that peace and stability can be maintained in the Strait"; this is essentially diplo-speak for "Cool
it." Even if Iran ignored these considerations and proceeded with an effort to close the Strait, the U.S. and
others would move to keep it open, and would be unlikely to stop there . As Iran has crept closer
to a nuclear weapons capability, the possibility of military action against Iran has also become
more imminent. President Obama has been reluctant to threaten Iran militarily, and any U.S. president would think long and
hard before engaging in another armed conflict in the Middle East. An effort by Iran to shut down the oil trade in
the Gulf, however, would make such a decision straightforward . The U.S. would react with force, and once
engaged in hostilities with Iran, would likely take the opportunity to target Iran's nuclear facilities and other military targets. It is
difficult to envision any scenario beginning with an Iranian effort to close the Strait of Hormuz
that does not end in a serious strategic setback for the Iranian regime.

Backups prevents economic shock

Abu-Nasser, ‘12 (Donna, Bloomberg, http://www.bloomberg.com/news/2012-01-26/u-a-e-can-export-oil-if-iran-


closes-strait-official-says.html)

The United Arab Emirates will be able to use ports on the Gulf of Oman for oil exports if Iran closes
the Strait of Hormuz, transit point for about a fifth of the global oil supply, Dahi Khalfan, Dubai’s chief of Police, said in an
interview with Bahrain TV last night. “Iran is capable of fomenting tension in the region,” he said in the interview, which was
rebroadcast today. “We
in the Gulf have cards in our hands that allow us to marginalize the role of
the strait and undermine its importance.” “We tell Iran if you try to close this place, we will open the other
gates and nullify the importance of the Strait of Hormuz,” he added. Iran has threatened to block the strait if
an embargo against its oil exports is implemented. The European Union announced Jan. 23 that it would ban oil imports from Iran
starting July 1 to pressure the Persian Gulf nation over its nuclear program.
---Strait of Hormuz- Can’t Close

Contingency plan solves the impact

Dahan ‘12 (Maha, Reuters, “Gulf Arabs have plans against Hormuz closure: official”,
http://www.reuters.com/article/2012/01/30/us-gulf-hormuz-gcc-idUSTRE80T0UT20120130)

(Reuters) - Coastguards
and naval forces of the Gulf Cooperation Council (GCC) group of Arab countries have
contingency plans for a possible attempt by Iran to shut the Strait of Hormuz , a Kuwaiti maritime official
said on Monday. Five of the six GCC members - Saudi Arabia, Bahrain, the United Arab Emirates (UAE), Qatar and Kuwait - rely on
the world's most important energy shipping lane being open to export most of their oil or gas. Tehran has threatened to close the
narrow shipping lane between Oman, the only GCC member which does not depend on Hormuz, and Iran if Western sanctions
aimed at starving Iran's disputed nuclear program of funds stop it from selling oil. The GCC members, which also rely on the four-
mile-wide (6.4 kilometer) channel being open to import food for their growing populations, has now drawn up a contingency plan in
case Iran acts on its threats. "Exporting oil or importing goods and cargo through Hormuz is a main concern for the GCC,"
Commander Mubarak Ali Al-Sabah chief of maritime operations at Kuwait's Coast Guard told Reuters in an interview. "The GCC has a
plan as a body - not just Kuwait separately or Bahrain or Saudi Arabia - we have a plan we just hope that everything stays safe," Al-
Sabah said, without giving details of the plans. "Awareness
and understanding of the consequences of it has
increased," he said. "We have plans how to deal with this but didn't do field exercises on it." Al-Sabah said the
planning included coordinating both between coastguards and navies of GCC countries and with
Western naval forces patrolling the area -- including U.S., Australian and French navies . Kuwaiti and
Iranian coastguards hold regular meetings on how to manage their shared maritime border, with the next one scheduled for next
month. "We don't go into politics or speak about other issues just what concerns the coastguards and how we can work it out," he
said. Oil tanker flows through the Strait of Hormuz are estimated at around 16 million barrels per day (bpd), or just under a fifth of
global oil supplies. A
new pipeline from the UAE's oilfields to the Gulf of Oman could carry most of the Gulf OPEC
oil producer's exports if Hormuz were to be blocked . But even a brief disruption to shipping could stop most of
the oil exported from Saudi Arabia, Iran, Kuwait and Iraq from leaving the Gulf, along with liquefied natural gas (LNG) from leading
supplier Qatar. In December, the
U.S. Fifth Fleet said it would not tolerate any disruption of traffic in
Hormuz but analysts say Iran might be able to hinder traffic transiting the Strait by scattering mines in it. "In any navy plan
that exists there would be plans for swift coordination to de-mine areas that might have been
mined... Or act in coordination preemptively or reactively to prevent Iranian small vessels
disrupting shipping," Christian Le Miere, research fellow for naval forces and maritime security at the International Institute
for Strategic Studies said. Earlier this month, Iran's foreign minister warned Arab neighbors not to side with the United States in the
escalating dispute over Tehran's nuclear activities which the West says includes weapons development and Tehran insists are limited
to electricity production.
---Strait of Hormuz- Won’t Close

Iran wont’ close the strait


Russell and Boot ‘12
Bradley Russell, a navy captain, is a visiting fellow at the Council on Foreign Relations, Max Boot,
CFR National Security Fellow, 1/4/12, Iran Won't Close the Strait of Hormuz ,
online.wsj.com/article/SB10001424052970204632204577130834200656156.html?
mod=googlenews_wsj

Iran threatened to close the Strait of Hormuz last week, in response to U.S. and European Union moves to apply sanctions on its oil
industry. Only 21 miles wide at its narrowest point, the strait sees the passage of roughly 28 tanker ships a day, half loaded, half
empty. Some 17 million barrels of oil—20% of oil traded in the world—go through this chokepoint. If Iran really could close the
strait, it
would do great damage to the world economy. But it would also damage its own already
shaky economy because Iran relies on the strait to deliver oil exports to China and other
customers. In any case, closing the strait is not nearly as easy as Adm. Habibollah Sayari, commander of the
Iranian Navy, would have it. He said that closing the strait is "as easy as drinking a glass of water." Actually it would be
about as easy as drinking an entire bucket of water in one gulp. Iran tried this trick before and
failed miserably. In 1984, during the Iran-Iraq War, Saddam Hussein attacked Iranian oil tankers and the Iranian oil-processing
facility at Kharq Island. Iran struck back by attacking Kuwaiti tankers carrying Iraqi crude and then other tankers in the Persian Gulf.
In 1987, after years of growing disruptions in this vital waterway, President Ronald Reagan
responded by offering to
reflag Kuwaiti tankers with the U.S. flag and provide U.S. naval escort. Iran shied away from
direct attacks on U.S. warships but continued sowing mines, staging attacks with small patrol boats, and firing a variety of
missiles at tankers. On April 14, 1988, the guided-missile frigate USS Samuel B. Roberts struck an Iranian mine; no sailors were killed
but several were injured and the ship nearly sank. The U.S. Navy responded by launching Operation Praying Mantis, its biggest
surface combat action since World War II. Half a dozen U.S. warships in two separate Surface Action Groups moved in to destroy two
Iranian oil platforms. The Iranians responded by sending armed speedboats, frigates and F-4 aircraft to fire at the U.S. warships. In
defending themselves, the American vessels sank at least three Iranian speedboats, one gunboat and one frigate; other Iranian ships
and aircraft were damaged. The only major U.S. loss occurred when a Marine Corps Sea Cobra helicopter crashed, apparently by
accident, killing two crewmen. The war all but ended less than three months later when the guided missile cruiser USS Vincennes
mistakenly fired a surface-to-air missile at an Iranian passenger airliner that it had mistaken for a fighter jet. The plane was
destroyed and 290 people killed. Although this was an accident, the Iranian regime was convinced that Washington was escalating
the conflict and decided to reach a truce with Iraq. The greatest loss suffered by U.S. forces during this whole conflict occurred in
1987 when an Iraqi aircraft fired an Exocet missile that hit the frigate USS Stark, killing 37 sailors and injuring 21. (Saddam Hussein
claimed this was an accident.) The Iranians had little to show for their efforts: Lloyd's of London estimated that the
Tanker War resulted in damage to 546 commercial vessels and the deaths of 430 civilian mariners but many of those losses were
caused by Iraq, not Iran. While these attacks temporarily disrupted the free passage of oil, they did not come close to closing the
strait. Despite the unveiling of a new antiship cruise missile called the Qader, Iran's
conventional naval and air forces
—on display during the Veleyat 90 naval exercises in the Persian Gulf which ended Monday— are still no match for the
U.S. and its allies in the region. The U.S. alone has in the area two carrier strike groups, an
expeditionary strike force (centered around an amphibious assault ship that is in essence a small aircraft carrier), and
numerous land-based aircraft at bases such as Al Udied in Qatar, Al Dafra in the United Arab Emirates,
and Isa Air Base in Bahrain. The U.S. and our Arab allies (which are equipped with a growing array of modern
American-made equipment such as F-15s and F-16s) could use overwhelming force to destroy Iran's
conventional naval forces in very short order. Iran's real ability to disrupt the flow of oil lies in its asymmetric war-
fighting capacity. Iran has thousands of mines(and any ship that can carry a mine is by definition a mine-layer), a small number of
midget submarines, thousands of small watercraft that could be used in swarm attacks, and antiship cruise missiles. If the Iranians
lay mines, it will take a significant amount of time to clear them. It took several months to clear all mines after the Tanker War, but a
much shorter period to clear safe passages through the Persian Gulf to and from oil shipping terminals. Antiship cruise missiles are
mobile, yet those can also be found and destroyed. Yono submarines are short-duration threats—they eventually have to come to
port for resupply, and when they do they will be sitting ducks. U.S. forces may take losses, as they did with the hits on the USS Stark
and Samuel B. Roberts, but they will prevail and in fairly short order. The Iranians must realize
that the balance of
forces does not lie in their favor. By initiating hostilities they risk American retaliation against
their most prized assets—their covert nuclear-weapons program. The odds are good, then, that the
Iranians will not follow through on their saber-rattling threats .
Suez Canal Answers
Frontline

No risk of shutdown – military insiders

Krauss, 11 – Toronto bureau chief for the New York Times (Clifford, 2/2. “Shippers Concerned Over Possible Suez Canal
Disruptions.” http://www.nytimes.com/2011/02/03/world/middleeast/03suez.html)

As violence has broken out in Egypt, concern has turned to the risk of the blocking of the Suez Canal or
nearby pipelines, which could pose a threat to world energy supplies. So far, oil and gas flows through Egypt have
not been interrupted, and the army has stepped up security around the canal and pipelines . But
rising tensions in the port of Suez have led several shipping companies to order their ships not to change crews in Egypt. Meanwhile,
disruptions in government port services have slowed the discharge of some crude oil cargo at the Red Sea port of Ain Sukhna, at the
southern entrance to the Suez Canal. “Potentially, we see attacks on employees of shipping companies and attempted attacks on
vessels docked in ports if you see more violent demonstrations around the ports,” said Helima L. Croft, a director and geopolitical
strategist at Barclays Capital. “It could be very problematic.” Egypt is not a major oil producer, and Western oil and gas companies
have halted most drilling in the country. But it is a crucial link for oil and gas headed to Europe, Asia and the United States. More
than two million barrels of oil and petroleum products traverse the Suez Canal daily, speeding the transit of crude and gas supplies
that shippers would otherwise have to send around the Horn of Africa, prolonging delivery times and expenses. About 4.5 percent of
global oil supplies flow through the canal and the Sumed pipeline, which connects the Red Sea with the Mediterranean. About 14
percent of the global liquefied natural gas trade is shipped through the canal as well, providing critical supplies to Spain, and to a
lesser extent, South Korea and the United States. Egypt also produces about 3 percent of the world’s liquefied natural gas supplies at
two export terminals, which were reported to be operating normally. Gen.
James Mattis, head of the United States
Central Command responsible for military operations in the region, said on Tuesday that it was
inconceivable that anyone would want to disrupt the canal. “Were it to happen, obviously, we
would have to deal with it diplomatically, economically, militarily, whatever, but that to me is a hypothetical,”
Reuters quoted him as saying to a policy group in London. There are already signs of slowing cargo operations at the Alexandria and
Damietta Ports, which handle container and bulk shipments of grains and other goods. Canal traffic has not yet been affected,
although employees must leave their posts early to comply with the government curfew that begins in the late afternoon. Egypt
has historically taken strong measures to assure oil and gas traffic, and interruptions have been
rare. The army is trained to operate the canal in case of any strikes . The canal was closed for several
months during the 1956 Suez crisis, and it was blockaded by the military for eight years after the 1967 war with Israel. In recent
years, it has been considered a safe passageway.

Alternatives and new freighters dramatically lower the importance of the Suez

Maritime Sun, 11 (2/7, “How Important Is the Suez Canal to the US and Europe?” http://www.maritimesun.com/news/how-
important-is-the-suez-canal-to-the-us-and-europe/)

Europe is in a much tighter spot and, insofar as it wants to sell stuff to Europe, so is Asia. Lately, congestion of the Panama Canal and
the west ports of the US have led Asian exporters to increasingly use the Suez route even for the east American destinations. It is
estimated that more than 22% of this trade is now using the Suez route. All
in all it’s estimated that the Suez Canal
sees 8% of total world trade. For Europe, the Suez Canal is a vital waterway for oil and trade with Asia. This is not to
say that it is without alternative, since the less traveled Cape of Good Hope could be used, but
closure of the canal would have several consequences: * Higher costs for European imports (and exports) * Tight supply of super
tankers and super container vessels to do the long South African route * Possible disruptions of supplies in the short run All things
considered, it is much more convenient for Europe (and just about everyone) to guarantee safe passage of
the Suez Canal, although new freighters have considerably lowered the strategic importance of
the canal, probably much below the casus belli it used to be .

No impact to Suez cutoff – oil can be redirected, the tanker market is flexible, and the possible
impact on prices is negligible

Newell, 11 – Administrator for the Energy Information Administration, U.S. Department of Energy, and Gendell Associate
Professor of Energy and Environmental Economics at Duke University’s Nicholas School of the Environment (Richard, 2/10.
STATEMENT before the COMMITTEE ON ENERGY AND COMMERCE SUBCOMMITTEE ON ENERGY AND POWER, U.S. HOUSE OF
REPRESENTATIVES. http://democrats.energycommerce.house.gov/sites/default/files/image_uploads/Newell_Testimony.pdf)

Second, EIA
took an in-depth look at a concern more directly related to Egypt involving the possibility of
disruption of the Suez Canal and/or the Sumed (Suez-Mediterranean) pipeline. EIA estimates that roughly 3.1
million bbl/d (January-November 2010 average, 2.9 million bbl/d in 2009) of crude oil and oil products transit the Suez Canal or the
Sumed pipeline, representing about 6 percent of total daily global waterborne oil movements. About 20-25 percent of global
liquefied natural gas (LNG) shipments also pass through the canal, and several European countries are heavily dependent on those
shipments. Information available to EIA as of February 8, when this written testimony was finalized, suggests that the canal and oil
pipeline are both operating normally. However, a gas pipeline in Egypt’s Sinai Peninsula exploded on February 5. Jordan relies on
Egyptian gas to generate around 80% of its electricity. Israel receives about 40 percent of its natural gas imports from Egypt, and 15
percent of Israeli electricity generation is met by this natural gas. Reports suggest the gas pipeline may remain shut down for about a
week. While we are not aware of any reports of any current disruptions to canal or oil pipeline activity, we recognize that
policymakers may want to understand the possible implications for energy markets of a disruption of these routes. For reasons
outlined below, however, we would expect the direct effects of any such closures to be manageable,
although there would undoubtedly be an adjustment period. ƒ Full diversion of all Suez Canal/Sumed flows
around Africa is an extreme worst-case scenario, since it is likely that some crude or product
streams would be redirected to reduce the need for such movements if the canal and the
pipeline are disrupted. ƒ Even assuming a scenario where all 3.1 million barrels of crude and
product that flow through the Suez Canal and Sumed daily (January-November 2010 data) were diverted
around Africa, the increase in tanker requirements traffic would be modest in the context of current
global oil shipment flows. Oil diverted around Africa could require an extra 6000 miles and 12 days of transit, with the
actual values depending upon the exact destination. ƒ Our contacts in the tanker community suggest that the tanker market
remains relatively relaxed. About 45 million bbl/d of waterborne oil shipments are moving daily
with a spare capacity margin of roughly 10 percent or 4 to 5 million barrels per day. ƒ Even if tanker rates
were to increase significantly in a disruption scenario involving closure of the Suez Canal /Sumed
transport route, which is again contrary to the indications above, tanker costs represent a very small
component of overall delivered crude costs . For example, tanker costs from the Persian Gulf to the Gulf of Mexico
generally fall within the range of $1 to $2 per barrel, so even a major increase in tanker rates would have little
impact on delivered oil prices.

Their impact’s overblown – shutdown is highly unlikely and most oil doesn’t flow through the
Suez

Kelly, 11 – Web Producer, CNBC (Gennine, 1/31. “Suez Canal Concerns are Overblown: Shipping Analyst.” CNBC.
http://www.cnbc.com/id/41351580/Suez_Canal_Concerns_are_Overblown_Shipping_Analyst)
Concerns about supply disruptions in the Suez Canal are an "overreaction ," Natasha Boyden, senior
managing director and shipping analyst at Cantor Fitzgerald, told CNBC on Monday. "The main thing to
realize is that the canal is under military control. They're keeping a very close watch on what goes
on there to make sure there are no disruptions to the canal," Boyden said. As social unrest
continues to reach new levels throughout Egypt, many shipping analysts are trying to determine whether there are any real
threats to one of the world's most heavily used shipping routes. "The main thing that will concern us is if we stop seeing military
control of the canal and that would enable other factions to take over and start causing disruptions to the canal," she said.
"Although to be fair if there were disruptions that wouldn't necessarily be a bad thing for oil tankers because we would see a rate
spike if the disruptions were to happen," Boyden said. "Egypt earned about 3.2 percent of its GDP from the
canal, which was the third largest source of revenue for the country, so it's really not in its interest to have it
closed down by any stretch of the imagination," she said. Also, if they were to close the canal it would have diplomatic
implications throughout the rest of the world because there are so many different countries that rely on the canal, Boyden said.

Despite that, mostMiddle Eastern oil does not go north through the Suez Canal, but rather goes
east to China, she said. "The eastbound route is seeing a lot more traction than the westbound
route," Boyden said.

No impact – the oil could go around Africa and there’s inventory to cover the shortage in the
meantime

Hargreaves, 11 – senior writer for CNNMoney (Steve, 2/1. “Egypt oil threat may be overblown.”
http://money.cnn.com/2011/01/31/news/international/egypt_oil_supply/index.htm)

But some traders believe the run-up is an overreaction, with the major Gulf oil states protected
from such conflict. The price retreated 81 cents to $91.38 a barrel in Tuesday trading. Egypt is not an oil exporter, and only
about 1.8 million barrels a day move through the Suez Canal , which it controls. That's just 2% of the
world's oil supply. Another two million barrels a day can go through the Sumed pipeline, which also
runs through Egypt, connecting the Red Sea to the Mediterranean. The pipeline is used by ships too big to pass through the Suez .
So far, both the pipeline and the canal are operating . But traders fear workers may not be able to get
to their jobs due to the violence, which would disrupt supplies. That would be a pain for oil
refiners, and prices would probably rise. But the oil wouldn't be lost. It could ultimately go
around Africa. Analysts say there are more than enough inventories to cover a brief shortage
while the crude was in route.
---Suez- No Impact

No impact – worst case boosts oil prices a tiny amount

Platts, 11 (global energy supplier, 2/10. “Closure of Suez Canal would have minimal impact on US: EIA.”
http://www.platts.com/RSSFeedDetailedNews/RSSFeed/Oil/6821601)

A total closure of the Suez Canal and the Sumed Pipeline would have minimal impact on supply
or crude oil prices, Richard Newell, Director of the US Energy Information Administration, told a House
subcommittee Thursday. Newell, testifying at a hearing on the effects of Middle East events on US energy markets, said a
total shutdown of key routes for oil and liquid natural gas is highly unlikely. But in the worst case,
diverting supplies would add a minimal number of days to delivery and $1 to $2/barrel to the
price of crude. EIA estimates that about 3.1 million b/d of crude oil passes through the Suez Canal or the Sumed pipeline,
representing about 6% of all daily global waterborne oil movements. About 20% to 25% of all global LNG also pass through the canal,
the EIA estimates. "Even assuming a scenario where all 3.1 million barrels of crude and product that
flow through the Suez Canal and Sumed daily were diverted around Africa, the increase in
tanker requirements traffic would be modest in the context of current global oil shipment
flows," Newell wrote in his prepared testimony. "Oil diverted around Africa could require an extra 6,000 miles and 12 days of
transit, with the actual values depending upon the exact destination." Newell said the tanker community has spare
capacity of about 10%, or 4 million b/d to 5 million b/d. But even if tanker rates were to increase
significantly as a result of a total closure, tanker rates constitute a small portion of the price of
oil, Newell said. "For example, tanker costs from the Persian Gulf to the Gulf of Mexico generally fall within the range of $1 to $2
per barrel, so even a major increase in tanker rates would have little impact on delivered oil prices,"
Newell wrote in his prepared testimony.

No economic impact to shutdown of the Suez

Learsy, 11 – member of the Woodrow Wilson International Center for Scholars and author of Oil and Finance: The Epic
Corruption 2006-2010 (Raymond J, 2/6. “Risks to the Suez Canal Set the Stage for Falsely Hyping the Price of Oil.”
http://www.huffingtonpost.com/raymond-j-learsy/the-risks-to-the-suez-can_b_819309.html)

Over the past days, the airwaves and talking heads have been frightening us with somber predictions of what would happen to the
price of oil should current events in Egypt shutter the canal. The oil boys and their allies can barely contain themselves in their
appearances of concern and like minded predictions of calamity, such as today's Reuters report quoting Imad al-Atiqi, member of
Kuwait's Supreme Petroleum Council -- "I expect oil to reach $110 during the first half of 2011... A huge amount of oil passes through
the Suez Canal..." thereby ever nudging oil prices skyward with Brent Crude already surpassing $100 a barrel. Yet has
anyone
stopped to determine what the closure of the Suez Canal would actually mean to the oil market
in dollars and cents? In the shipping world the type of vessel that can transit the Suez Canal has
its own designation, named a "Suezmax" category. The typical deadweight of a Suezmax oil tanker
is about 240,000 tonnes. Now, approximately 7.1 barrels of oil make up one metric tonne. Therefore, a 240,000
tonnes deadweight tanker carries some 1.7 million barrels of oil . According to the New York Times,
"Taking cargo around Africa would add about 16 days time to delivering oil to world markets ."
Calculating a per diem charter rate for a Suezmax tanker at $50,000 per day (and probably less),
brings the additional cost of transporting a cargo of oil, lifting 1.7 million barrels around Africa,
to $800,000 per voyage. More to the point, the additional cost per barrel of oil would be 47 cents per
barrel. And these 47 cents would apply only to the some 1.8 million barrels of crude oil that are
transported through the canal (an additional 2mm plus barrels can be transported through Egypt overland via the Sumed
pipeline). The additional cost of $800,000 for transporting these 1.8 million barrels around the
horn of Africa, distributed over the world's daily consumption of oil of 85 million barrels, would
settle out at just under a penny per barrel . All said, the additional 16 days would be a problem if the oil market were
in a state of hand to mouth. Fortuitously, oil stocks are bulging throughout the world and the sixteen days
additional steaming time can be easily accommodated with ample leeway to alter delivery
schedules factoring in these changed logistics. Clearly, the closing of the Suez Canal to the oil trade
would be a hindrance but hardly the disaster portrayed in the media and our friends at OPEC.
---Suez- No Shutdown

No chance of shutdown—Egypt will ensure the Suez stays open

Cook and Cohen, 11 – * Hasib J. Sabbagh Senior Fellow For Middle Eastern Studies, Council On Foreign Relations and
**Adjunct Fellow, Council On Foreign Relations (Steven and Jared, 1/20. “Media Conference Call: Tunisia - Repercussions for the
Region.” http://www.cfr.org/democracy-and-human-rights/media-conference-call-tunisia---repercussions-region/p23849)

As far as disrupting oil, very unlikely that this kind of situation is going to have an effect on the
supply, that it's going to have an effect on politics in the Gulf region .  You -- of course, if there is some sort
of disaster in Egypt, it might have an effect on the -- on the canal.  But as I said, it's unlikely that that's going to happen. And even
if there were some sort of mass uprising in Egypt and the military did intervene, whether on
behalf of the regime or in support of society, as the Tunisian military has apparently done, one of the things
that they would be sure to keep open and keep operating is the Suez Canal .

Egypt wouldn’t shut down the Suez – fears international response

Innocent and Bouasria, 11 – Foreign Policy Analyst at the Cato Institute and professor of Arab Studies at the International
Center for Language Studies (Malou and Abdelilah, 4/21. “End U.S. Aid to Egypt.” https://store.cato.org/pub_display.php?
pub_id=13047)

In addition, ending
U.S. aid would not automatically lead to Egypt's closure of Suez . Given past
experience — the 1956 Suez Crisis — Egyptian officials know full well that closing that important
strategic and economic waterway is a provocative action. More importantly, they know that the
regional and international response would be inevitable, swift and devastating.

Egypt can’t shut down the canal – it’s under the control of the US Navy

Mayton, 10 – American journalist based in Cairo and founder/editor of Bikyamasr.com (Joseph, 12/15. “The US role in the Suez
Canal.” http://bikyamasr.com/22094/the-us-role-in-the-suez-canal/)

According to an American military official who recently spoke to Bikya Masr on condition of anonymity due to the
sensitivity of the information, the American Navy is responsible for the safe passage of ships through the
canal, into the Red Sea and across the Gulf of Aden. What he said occurs is very simple. When a ship enters the Suez Canal Zone,
either north in the Mediterranean Sea or south via the Gulf of Aden, the captain must radio the Egyptian authorities in order to ask
for permission to cross the waterway. Then, it gets a bit tricky and the American officer said at this point, upon approval to
enter the canal, the Americans take over. “Most ships will be waiting some distance away from the entrance of the
canal and will need to be guided into the canal and through the passage without hitting rocks or other barriers in the water,” he
began, “so we, the American military is responsible for this caretaking mission .” But, what happened to the
Egyptian side? He said that the Egyptians do not have the tech nology needed to guide and direct larger
vessels from the waters off the canal and throughout the waterway. “It is expensive technology that we
haven’t given them because they don’t have the manpower to take control of it.”
No risk of shutdown

Vaughan, 11 – writer for Express News (Vicki, 2/4. “Suez Canal skirts unrest in Egypt.”
http://www.mysanantonio.com/business/article/Suez-Canal-skirts-unrest-in-Egypt-997497.php)

But economists say the jitters, while understandable, are a bit misplaced. “The possibility of a halt in
shipping on the Suez Canal is remote, to say the least,” said economist Travis Tullos of TXP, an Austin-
based consulting firm. It would not and should not be tolerated.” About 4.5 percent of the world’s oil supply moves
through the canal, and OPEC nations in the Middle East “would lean hard on any group” seeking to
slow down traffic. David Hightower, a commodity expert and editor of The Hightower Report in Chicago, agreed, saying the
Suez Canal “is sort of a national treasure” for Egypt. “It’s hugely important for the Egyptians to
have the canal open,” he said. “So indications are that it would have to be a very serious conditions
there” for shipping to be curtailed. Should conditions worsen , the presence of the U.S. Fifth Fleet’s
presence in the region could aid in heading off a serious disruption, said Jim Kee, chief economist
with South Texas Money Management in San Antonio. At San Antonio-based refiner Tesoro Corp., “we are in
frequent, daily contact with our business partners in Egypt,” spokesman Mike Marcy said, and so far they’ve
told the company there’s been no disruption of shipments through the Suez Canal or the Red Sea ports.
---Suez- No Al Qeada Attack

Al-Qaeda can’t do any damage in the Suez – it’s too difficult to blow up or sink a ship, and
safeguards check

USA Today, 11 (5/21, “Oil tanker terror hijacks easy, attacks complex.” Associated Press.
http://www.usatoday.com/news/world/2011-05-21-terror-plot-oil-tankers_n.htm)

Al-Qaida operatives with enough training could easily manage to capture ships carrying millions of gallons of
oil or liquefied natural gas. All they would have to do is imitate the tactics of Somali pirates who already use small boats to
overpower tanker crews in mostly remote locations, the experts said. Few supertankers have armed guards, due to gun import laws
and the risk of accidental gunfire igniting explosive cargos. But once terrorists captured a supertanker, it wouldn't
be so easy to sow the economic chaos and costly environmental destruction bin Laden desired
and outlined in secret files captured from his Pakistan hideout. It's actually extremely complex to blow up a
supertanker or even sink it near heavily guarded oil shipping lanes like the Suez Canal, the Panama Canal
or the Strait of Hormuz at the end of the Persian Gulf. "It would only be a risk if they could sail it
undetected and had worked out how to blow it up, which is pretty complicated," said Graeme
Gibbon-Brooks, the head of Dryad Maritime Intelligence . The FBI and the U.S. Department of Homeland Security
issued a confidential warning to authorities and the energy industry Friday that al-Qaida was seeking information on the size and
construction of tankers. The newly revealed plot showed that while bin Laden was scheming about the next strike to kill thousands
of Americans, he also believed an attack on the oil industry in "non-Muslim waters" could create a worldwide economic panic that
would send oil prices soaring and hurt Westerners at the gas pump. Other bin Laden documents revealed that the terror group
identified New York, Washington, Los Angeles and Chicago as important cities that should be attacked. Al-Qaida also identified key
dates for those attacks, including the 10th anniversary of the Sept. 11, 2001 attacks, Christmas, July 4th and during Obama's State of
the Union address in January. Oil already is a known target. On Saturday, a truck tanker carrying oil for NATO forces in Afghanistan
exploded in northwestern Pakistan as people tried to siphon off fuel, killing 15. Fourteen other NATO oil trucks were damaged in a
bombing at a nearby border town, but no one was hurt. The hundreds of seafaring oil tankers that travel across the planet daily are
theoretically capable of igniting massive fires with the capability for extensive destruction. Intelligence gathered from bin Laden's
hideout revealed that al-Qaida
realized the tankers would have to be boarded so explosives could be
planted inside them. Security experts say , however, blowing them up would be difficult because
the tankers have double hulls and compartmentalized holds that prevent oil spills in groundings
and can withstand direct hits from rocket propelled grenades. Plus, getting enough explosives
aboard the tankers would mean using more speedboats than Somali hijackers normally do to take
over the ships and hold crews hostage, Gibbon-Brooks said. Somali pirates have already captured five supertankers, proving that
men with little training and basic weapons can easily seize the giant ships. Supertankers move slowly when fully loaded, can be
longer than three football fields and generally only have around 20 unarmed crew onboard. Although the size of the ships makes
them vulnerable, their slow
speed also makes it harder for terrorists to sneak one into a port or a
narrow shipping passage. Ships are closely tracked via satellite and any unexplained deviations
from their travel plans would immediately raise alarms. While al-Qaida's most brazen sea attack was on the USS
Cole in Yemen, the explosives that knocked a hole in the destroyer and killed 17 sailors in 2000 would not sink a double-hulled oil
tanker.Other marine attacks have been botched. An attempt to blow up the USS The Sullivans
warship failed in 2000 when the plotters overloaded their speedboat with explosives and it sank
en route to the mission. The 2002 suicide bombing of a tanker off the coast of Yemen damaged the
ship but didn't sink it. And last year a suicide bomber only slightly damaged the Japanese tanker
M. Star in the Straits of Hormuz, which handles 40 percent of the world's tanker traffic. An obscure al-Qaida-linked group
claimed responsibility. After those attempts, al-Qaida decided attackers have to board a ship and blow it up from the inside,
according to documents seized by U.S. special forces from the compound where bin Laden was killed nearly three weeks ago.
Syria CBW Answers
Frontline

Their evidence is overblown- Syria doesn’t have any biological weapons post-dates all of their
evidence

NTI ‘11 (Nuclear Threat Initiative is a nonprofit, nonpartisan organization with a mission to strengthen global security by reducing
the risk of use and preventing the spread of nuclear, biological, and chemical weapons), Syria: Biological
http://www.nti.org/country-profiles/syria/biological/

In the past, unclassified statements by U.S. officials occasionally claimed reason to suspect Syria
of maintaining an offensive BW program. [2] However, in contrast to discussions of Syrian chemical warfare (CW)
capabilities, such claims have not included any details on the size and scale of Syria’s potential BW
program, and are not presented alongside supporting evidence . Instead, discussions on this topic have
focused on speculative extrapolations of Syrian dual-capable industry and on Syrian political motivations. Such analysis can
be neither detailed nor comprehensive. Although the existence of a biotechnology industrial
base would suggest that Syria has some indigenous expertise useful for developing a biological
weapons capability, it does not imply and cannot confirm the existence of an offensive biological
weapons program. Furthermore, given that Israel, a state that is understood to possess a nuclear arsenal and continues to
occupy the Golan Heights, remains Syria’s primary security concern, and given the risk of “blowback” when deploying biological
weapons, such weapons would be of questionable tactical desirability from a Syrian perspective. While
public sources on
the nature of Syria’s chemical and nuclear programs are limited, even less exists about Syria’s
biological program, and “there is no hint of its existence from open sources.” [3]

Even if Syria has some infrastructure for limited bioweapon use they don’t have the capability
to make them effective weapons

NTI ‘11 (Nuclear Threat Initiative is a nonprofit, nonpartisan organization with a mission to strengthen global security by reducing
the risk of use and preventing the spread of nuclear, biological, and chemical weapons), Syria: Biological
http://www.nti.org/country-profiles/syria/biological/

Open sources provide no indications that Syria currently possesses an offensive BW capability .
Syria has invested heavily in its pharmaceutical industry in the past 20 years, and this sector has grown substantially both in size and
in output. This suggests that Syria perhaps has an organizational capacity to develop modern scientific production facilities and also
that indigenous scientific expertise in the biosciences exists. However, there is no information on whether this infrastructure or
expertise could lend themselves to an offensive BW program. Further, the
geographic proximity of Israel, Syria’s
most likely military adversary, would undermine the desirability of biological weapons in Syria’s
strategic planning. Official government reports from the United States continue to mention a hypothetical BW threat from
Syria. In March 2009 DIA Director Lt General Michael Maples testified that "we judge some elements of the program may have
advanced beyond the research and development stage and may be capable of limited agent production. Syria
is not known
to have successfully weaponized biological agents in an effective delivery system, but it possesses a
number of conventional and chemical weapon systems that could easily be modified for biological agent delivery." [17] The Director
of National Intelligence delivered a similar message in the 2008 721 report asserting that: " Syria's
biotechnical
infrastructure is capable of supporting limited BW agent development, but the Syrians are not
believed to have achieved a capability to put BW agents into effective weapons ." [18] However,
there is no open source discussion of evidence that would support such assessments, and few
insights if any exist to elucidate whether these claims reflect political goals or technical realities.
Syria Lashout Answers
No CBW lashout

Rubin ’11 (Sept.6/2011 ([“Israel's Strategic Situation: Plenty to Worry About but Little to Fear”,2011 Barry Rubin is director of
the Global Research in International Affairs (GLORIA) Center, editor of the Middle East Review of International Affairs (MERIA)
Journal, and Middle East editor and featured columnist at PajamasMedia]

Might Syria fire some chemical weapons at Israel as a last ditch to save itself? --There is no
indication of that happening and the Iraqis--who had much more motive to do it--didn't follow
that path in 2003. Israel would have plenty of warning time and could take countermeasures.
Not a realistic scenario.

Syria won’t lashout against Israel

Rubin ’11 (Sept.6/2011 ([“Israel's Strategic Situation: Plenty to Worry About but Little to Fear”,2011 Barry Rubin is director of
the Global Research in International Affairs (GLORIA) Center, editor of the Middle East Review of International Affairs (MERIA)
Journal, and Middle East editor and featured columnist at PajamasMedia]

What will Israel do should Hamas, Hizbullah, Syria all start firing missiles into Israel at same time ? Syria
won't. If Damascus
attacked Israel it would be the end of the regime. Rather than mobilize mass support, a s in the
past, Syria's regime would have its armed forces heavily damaged and fall within a week or two,
with very poor prospects for the life spans of its elite . As for Hizballah-Hamas cooperation, that's
less likely than ever. Those two groups are now on different sides of the Sunni-Shia divide and
didn't even work together in 2008-2009 when they were much closer to each other. Reportedly,
Hamas asked for help and Hizballah refused, unwilling to risk its own interests to help its
"brothers" in the Gaza Strip.
Terrorism Answers
Frontline
No terror threat

Walt ‘12 Stephen M. Robert and Renée Belfer professor of international relations at Harvard University, "'America the brittle?'"
September 10, Foreign Policy, http://walt.foreignpolicy.com/posts/2012/09/09/inflating_the_terrorist_threat_again

According to yesterday's New York Times, assorted "senior American officials" are upset that adversaries like al Qaeda,
the Taliban, or the Somali pirates are not simply rolling over and dying. Instead, these foes are
proving to be "resilient,"
"adaptable," and "flexible." These same U.S. officials are also worried that the United States isn't demonstrating the same
grit, as supposedly revealed by high military suicide rates, increased reports of PTSD, etc. According to Times reporters Thom
Shanker and Eric Schmitt, these developments¶ "raise concerns that the United States is losing ground in the New Darwinism of
security threats, in which an agile enemy evolves in new ways to blunt America's vast technological prowess with clever homemade
bombs and anti-American propaganda that helps supply a steady stream of fighters."¶ Or as Shanker and Schmitt put it (cue the
scary music): "Have we become America the brittle?" ¶ This
sort of pop sociology is not very illuminating,
especially when there's no evidence presented to support the various officials' gloomy
pronouncements. In fact, the glass looks more than half-full. Let's start by remembering that the Somali pirates and
al Qaeda have been doing pretty badly of late . Piracy in the Gulf of Aden is down sharply, Osama bin Laden is dead,
and his movement's popularity is lower than ever. Whatever silly dreams he might have had about restoring the caliphate have
proven to be just hollow fantasies. And as John Mueller and Mark Stewart showed in an article I linked to a few weeks ago, the
actual record of post-9/11 plots against the United States suggests that these supposedly "agile"
and "resilient" conspirators are mostly bumbling incompetents . In fact, Lehman Bros. might be the only
major world organization that had a worse decade than al Qaeda did.

No terrorism impact – groups are too weak and law enforcement solves

Zenko and Cohen 12, (Fellow in the Center for Preventive Action at the Council on Foreign Relations, *Fellow at the Century
Foundation, (Micah and Michael, "Clear and Present Safety," March/April, Foreign Affairs,
www.foreignaffairs.com/articles/137279/micah-zenko-and-michael-a-cohen/clear-and-present-safety

NONE OF this is meant to suggest that the United States faces no major challenges today. Rather, the point is that the problems
confronting the country are manageable and pose minimal risks to the lives of the overwhelming majority of Americans. None of
them -- separately or in combination -- justifies the alarmist rhetoric of policymakers and politicians or should lead to the conclusion
that Americans live in a dangerous world. Take terrorism. Since 9/11, no security threat has been hyped more. Considering the
horrors of that day, that is not surprising. But the result has been a level of fear that is completely out of proportion to both the
On 9/11, al Qaeda got tragically lucky .
capabilities of terrorist organizations and the United States' vulnerability.
Since then, the United States has been preparing for the one percent chance (and likely even less) that it might get lucky again.
But al Qaeda lost its safe haven after the U.S.-led invasion of Afghanistan in 2001, and further military, diplomatic,
intelligence, and law enforcement efforts have decimated the organization , which has essentially lost
whatever ability it once had to seriously threaten the United States. According to U.S. officials, al Qaeda's leadership has been
reduced to two top lieutenants: Ayman al-Zawahiri and his second-in-command, Abu Yahya al-Libi. Panetta has even said that the
defeat of al Qaeda is "within reach." The near collapse of the original al Qaeda organization is one reason why, in the decade since
9/11, the U.S. homeland has not suffered any large-scale terrorist assaults . All subsequent
attempts have failed or been thwarted , owing in part to the incompetence of their perpetrators. Although there are
undoubtedly still some terrorists who wish to kill Americans, their dreams will likely continue to be frustrated by
their own limitations and by the intelligence and law enforcement agencies of the United States and its
allies.
Zero risk of terrorism- their impact is alarmism (No nuclear, chemical, biological wpns)

Mueller ’12 (John, Senior Research Scientist at the Mershon Center for International Security Studies and Adjunct Professor
in the Department of Political Science, both at Ohio State University, and Senior Fellow at the Cato Institute. Mark G. Stewart is
Australian Research Council Professorial Fellow and Professor and Director at the Centre for Infrastructure Performance and
Reliability at the University of Newcastle in Australia, The Terrorism Delusion, International Security, Vol. 37, No. 1, pp. 81–110,
Summer 2012)

Over the course of time, such essentially delusionary thinking has been internalized and institutionalized in
a great many ways. For example, an extrapolation of delusionary proportions is evident in the common
observation that, because terrorists were able, mostly by thuggish means, to crash airplanes into buildings, they might
therefore be able to construct a nuclear bomb. In 2005 an FBI report found that, despite years of well-
funded sleuthing, the Bureau had yet to uncover a single true al-Qaida sleeper cell in the U nited
States. The report was secret but managed to be leaked. Brian Ross, “Secret FBI Report Questions Al Qaeda Capabilities: No ‘True’ Al
Qaeda Sleeper Agents Have Been Found in U.S.,” ABC News, March 9, 2005. Fox News reported that the FBI, however, observed that
“just because there’s no concrete evidence of sleeper cells now, doesn’t mean they don’t exist.” “FBI Can’t Find Sleeper Cells,” Fox
News, March 10, 2005. Jenkins has run an internet search to discover how often variants of the term “al-Qaida” appeared within ten
words of “nuclear.” There were only seven hits in 1999 and eleven in 2000, but the number soared to 1,742 in 2001 and to 2,931 in
2002. 47 By 2008, Defense Secretary Robert Gates was assuring a congressional committee that what keeps every senior
government leader awake at night is “the thought of a terrorist ending up with a weapon of mass destruction, especially nuclear.” 48
Few of the sleepless, it seems, found much solace in the fact that an
al-Qaida computer seized in Afghanistan in
2001 indicated that the group’s budget for research on weapons of mass destruction (almost all of it focused
on primitive chemical weapons work) was $2,000 to $4,000. 49 In the wake of the killing of Osama bin Laden,
officials now have many more al-Qaida computers, and nothing in their content appears to suggest that
the group had the time or inclination, let alone the money, to set up and staff a uranium-seizing operation, as
well as a fancy, super-high-technology facility to fabricate a bomb . This is a process that requires
trusting corrupted foreign collaborators and other criminals, obtaining and transporting highly
guarded material, setting up a machine shop staffed with top scientists and technicians, and
rolling the heavy, cumbersome, and untested finished product into position to be detonated
by a skilled crew—all while attracting no attention from outsiders. 50 If the miscreants in the American
cases have been unable to create and set off even the simplest conventional bombs, it stands to reason that none of them
were very close to creating, or having anything to do with, nuclear weapons—or for that matter
biological, radiological, or chemical ones. In fact, with perhaps one exception, none seems to have even
dreamed of the prospect; and the exception is José Padilla (case 2), who apparently mused at one point about creating a
dirty bomb—a device that would disperse radiation—or even possibly an atomic one. His idea about isotope separation was to put
uranium into a pail and then to make himself into a human centrifuge by swinging the pail around in great arcs. Even
if a
weapon were made abroad and then brought into the United States, its detonation would require
individuals in-country with the capacity to receive and handle the complicated weapons and then to set
them off. Thus far, the talent pool appears, to put mildly, very thin. There is delusion, as well, in the legal
expansion of the concept of “weapons of mass destruction.” The concept had once been taken as a synonym for nuclear
weapons or was meant to include nuclear weapons as well as weapons yet to be developed that might have similar destructive
capacity. After the Cold War, it was expanded to embrace chemical, biological, and radiological weapons even though those
weapons for the most part are incapable of committing destruction that could reasonably be considered “massive,” particularly in
comparison with nuclear ones.
No terrorism- groups are too weak and attack strategy has shifted to local focus

Sofer ’11 (The Evolution of Terrorism Since 9/11 From Hierarchical Organizations to Small Groups and Individuals SOURCE:
AP/TV2 Norway By Ken Sofer | September 9, 2011 Ken Sofer is the Special Assistant for National Security at American Progress.

When Osama bin Laden’s body was buried at sea, many observers believed an era in
transnational terrorism was buried with him. In truth, the era of transnational terrorism reached
its pinnacle in the atrocities of September 11 a full decade ago. Over the last 10 years, the structure of
terrorist groups has evolved, in part because of American and allied policies, and in part because new technologies have
opened up a new model of terrorism. Gone are the days of a centralized, hierarchical international terrorist movement with Al
Qaeda clearly in the lead. That system has been replaced by a much more diffuse network of regional terrorist groups and individual
actors connected to terrorist leaders only by the Internet. The breakdown of the hierarchical system of terror When bin Laden
organized Al Qaeda in the late 1980s, he envisioned the group as an army of the faithful, which he could train and mobilize to fight
kuffar, or nonbelievers, throughout Muslim lands. He financed training camps in tribal Pakistan, developed a system of recruitment
for would-be suicide bombers, and planned complex operations, which required adherence to a strict chain of command. The
attacks on the Twin Towers in New York represented the pinnacle of organized, hierarchical terrorism and would have been
impossible to execute without Al Qaeda’s deep pockets and operational expertise. Since that day the
United States has
eliminated Al Qaeda’s operational safe haven in Afghanistan and decimated its core leadership . A
combination of raids, police stings, and the increased use of drone strikes under President Barack Obama have
led to the capture or killing of many of Al Qaeda’s mid- to senior-level leaders, most notably bin
Laden, and most recently Atuyah Abd al Rahman, a key figure in the organization’s operations .
The death of Al Qaeda’s core leadership and its loss of a safe haven in Afghanistan puts the organization close to strategic defeat,
according to White House counterterrorism chief John Brennan. While Brennan’s comments on Al Qaeda’s imminent demise are
likely overly optimistic, the organization is clearly weaker than it was a decade ago and has become increasingly reliant on a variety
of ideologically sympathetic affiliates in Yemen, Algeria, and Iraq who have adopted the Al Qaeda name brand. While these affiliates,
most notably the Yemen-based Al Qaeda in the Arabian Peninsula, or AQAP, maintain close ties to bin Laden’s Pakistan-based core
and adhere to its central message, these organizations operate independently of Al Qaeda Central and do not generally coordinate
with one another. Further, the new leadership of Ayman al Zawahiri, an extremely divisive figure in the jihadist community, likely
means Al Qaeda Central will have a more difficult time controlling operatives and affiliates around the world. The increasingly
confederate nature of Al Qaeda has broken down the hierarchical system bin Laden built in the late 1970s and 1980s. Lower barriers
of access to terrorism Al Qaeda has adapted to the increasing difficulty of maintaining a physical organization in an identifiable safe
haven such as Afghanistan by relying on the Internet and public media to spread its ideology and give individuals the tools to
become terrorists. Just as Twitter and blogs made everyone a potential journalist, Al Qaeda and its affiliates launched a variety of
media outlets and websites with the hope of making everyone a potential terrorist. Publications such as AQAP’s English-language
magazine Inspire feature interviews with prominent leaders and how-to articles such as “Make a Bomb in the Kitchen of Your Mom.”
Meanwhile, Al Qaeda’s media production house As Sahab produces “documentary-quality films, iPod files and cellphone video” for
distribution across terrorist-sympathetic message boards and blogs. The effect of this propaganda boom and the proliferation of
easily attainable bomb-making instructions has been a further decentralization of international terrorism. While members of
terrorist cells still actively recruit radicals to carry out attacks, such as the failed Times Square bomb plot last year, terrorism has
become increasingly reliant on volunteers who are inspired by Al Qaeda’s ideology. One example is Fort Hood shooter Nidal Malik
Hasan, who killed 13 people in 2009 and was inspired by AQAP’s Anwar al Awlaki. Hasan regularly emailed Awlaki for spiritual
guidance and justification in the lead-up to the attack. To an extent, the increasing decentralization of terrorism represents a loss in
Al Qaeda’s operational capabilities. This means that they are less likely to pull off another expensive and complex attack like 9/11.
But the decentralization of terrorism also poses a variety of new threats. For one, it makes it significantly harder for the intelligence
community to track would-be terrorists and thwart their efforts, which is why the only successful attacks in the United States since
9/11 have been gunmen acting alone inspired by the Al Qaeda ideology. Al Qaeda’s ability to communicate and spread its ideology
to a constituency of radicals is likely its most powerful remaining tool since 9/11, and now that a potential terrorist can Google an
inspirational sermon and bomb-making instructions instead of needing to fly to a training camp in Kandahar, this tool has become
even more potent. The near enemy vs. the far enemy One of the truly unique and dangerous elements of Al Qaeda’s brand of
terrorism is its transnational nature. Bin Laden and many of his followers derided the governments of most Muslim-majority nations,
in particular Saudi Arabia and Egypt, as apostates. Yet
instead of targeting these governments, often referred
to as the “near enemy,” Al Qaeda believed that destroying their U.S. and Western allies, the “far
enemy,” would more effectively lead to the downfall of apostate Arab regimes. The group’s
transnational aims and focus on the United States made it unique among terrorist organizations
and brought jihadist terrorism to American soil. Over the last decade, the United States has
demonstrated the enormous costs associated with making it a target. When coupled with the
death of bin Laden , the most effective advocate for this strategy, the near enemy/far enemy
balance has shifted decidedly in favor of the near enemy. Al Qaeda affiliates, with the possible exception of
AQAP, seem much more concerned about attacking domestic targets as opposed to spending their
resources on a much more difficult attack on the other side of the planet . Additionally, Al Qaeda’s
membership now frequently loses recruits to organizations such as the Afghan Taliban, Hamas, Hezbollah, or Lashkar-e-Taiba who
have purely national and not transnational aspirations. These organizations may be similar to Al Qaeda in that they use violence to
kill civilians and seek to establish a conservative Islamist caliphate, but their goals only apply to the country they operate in. Of
the
48 groups designated by the Department of State as Foreign Terrorist Organizations, Al Qaeda
is the only group left with truly global operations and aspirations. The remaining groups, such as the Kurdish
PKK, the Colombian FARC, the Sri Lankan Tamil Tigers, and the Japanese Aum Shinrikyo have a distinctly national or
semiregional focus. Many of these groups frequently target American citizens, soldiers, and interests in their countries, but they
either do not possess the capabilities or desire to launch an attack on U.S. or European soil. What
do these changes mean for terrorism in America today? Since 9/11 we have braced for the possibility of
another catastrophic attack on U.S. soil and pursued policies that have thankfully prevented such an attack from
happening again. But largely because of our success in decapitating and dismantling terrorist
networks and organizations, the landscape of terrorism looks very different than it did 10 years ago.
Today we are less likely to face a large, complex attack from an enemy organization abroad such as Al Qaeda. But we remain
vulnerable to a smaller, less traceable attack from an individual or small group of individuals here in the United States. Incidents
such as the Oklahoma City bombing in 1995, the Fort Hood shooting in 2009, or the Oslo attacks earlier this
year are likely to become the dominant strain of terrorism entering the next decade after 9/11. While many
would-be terrorists are inspired by the ideology of Al Qaeda and Anwar al Awlaki, as we have seen, lone-wolf terrorists can draw
their inspiration from antigovernment or xenophobic ideologies as well.

Terrorists strategy has shifted- new attacks will be small scale and local

Schofield ’12 (McClatchy Washington Bureau Print This Article Posted on Thu, Apr. 26, 2012 After Osama bin Laden, al Qaida
still a many-headed threat By Matthew Schofield | McClatchy Newspapers last updated: April 29, 2012 08:55:55 AM WASHINGTON
-- ]

“What we’re facing today is a much, much larger global threat,” said Seth Jones, an expert at the RAND Corp. who’s advised the
Pentagon on Afghanistan and Pakistan. “It’s
a more dispersed threat. The threat is decentralizing to a broad
network of groups. Al Qaida inspires, but doesn’t control, and they work with locals.” The
meaning of that threat:
Massive attacks such as those on 9/11 are unlikely to be repeated. But expect smaller-scale
attacks — the “strategy of a thousand cuts,” it was called in AQAP’s slick online propaganda magazine Inspire. A
deadly example came in 2009 with the rampage at Fort Hood, Texas, where Army psychiatrist
Maj. Nidal Malik Hasan, allegedly radicalized online by AQAP, is accused of shooting dead 13
soldiers. His trial is scheduled to begin in August. Experts note that these groups have largely
localized agendas. Generally, they’re looking to impose Islamic Sharia law and, if not overthrow a local government, carve out
a space in which to operate in their home country.
No risk of nuclear terror—means and motive

Chapman 12 [Stephen, columnist and editorial writer for the Chicago Tribune “The Implausibility of Nuclear Terrorism” May
17 http://reason.com/archives/2012/05/17/the-implausibility-of-nuclear-terrorism]

Given their inability to do something simple — say, shoot up a shopping mall or set off a truck bomb — it’s
reasonable to ask whether they have a chance at something much more ambitious. Far from being
plausible, argued Ohio State University professor John Mueller in a presentation at the University of Chicago, “the
likelihood that a terrorist group will come up with an atomic bomb seems to be vanishingly
small .” The events required to make that happen comprise a multitude of Herculean tasks . First, a
terrorist group has to get a bomb or fissile material, perhaps from Russia’s inventory of decommissioned warheads. If
that were easy, one would have already gone missing. Besides, those devices are probably no longer a
danger, since weapons that are not maintained quickly become what one expert calls “ radioactive scrap metal .” If
terrorists were able to steal a Pakistani bomb, they would still have to defeat the arming codes and other
safeguards designed to prevent unauthorized use. As for Iran, no nuclear state has ever given a bomb to an
ally — for reasons even the Iranians can grasp. Stealing some 100 pounds of bomb fuel would require help from
rogue individuals inside some government who are prepared to jeopardize their own lives. Then comes
the task of building a bomb. It’s not something you can gin up with spare parts and power tools in
your garage. It requires millions of dollars , a safe haven and advanced equipment — plus people with
specialized skills, lots of time and a willingness to die for the cause. Assuming the jihadists vault over those
Himalayas, they would have to deliver the weapon onto American soil. Sure, drug smugglers bring in contraband
all the time — but seeking their help would confront the plotters with possible exposure or extortion. This, like
every other step in the entire process, means expanding the circle of people who know what’s going on, multiplying
the chance someone will blab, back out or screw up. That has heartening implications. If al-Qaida
embarks on the project, it has only a minuscule chance of seeing it bear fruit. Given the formidable odds, it
probably won’t bother.

Won’t use nukes- they are rational

Kapur ‘8 [S. Paul, associate professor in the Department of National Security Affairs at the Naval Postgraduate School, The Long
Shadow: Nuclear Weapons and Security in 21st Century Asia. pg. 32]

Before a terrorist group can attempt to use nuclear weapons, it must meet two basic requirements. First, the group must decide that
it wishes to engage in nuclear terrorism. Analysts and policy makers often assume that terrorist groups necessarily want to do so
(Carter 2004; U.S. Government 2002). However, itis not clear that terrorist organizations would necessarily
covet nuclear devices. Although analysts often characterize terrorism as an irrational activity (Laqeuer 1999: 4-5),
extensive empirical evidence indicates that terrorist groups in fact behave rationally , adopting
strategies designed to achieve particular ends (Crenshaw 1995: 4; Pape 2003: 344). Thus whether terrorists
would use nuclear weapons is contingent on whether doing so is likely to further their goals .
Under what circumstances could nuclear weapons fail to promote terrorists' goals? For certain types of terrorist objectives,
nuclear weapons could be too destructive. Large-scale devastation could negatively influence
audiences important to the terrorist groups. Terrorists often rely on populations sympathetic to
their cause for political, financial, and military support. The horrific destruction of a nuclear
explosion could alienate segments of this audience. People who otherwise would sympathize with the terrorists
may conclude that in using a nuclear device terrorists had gone too far and were no longer deserving of support . The
catastrophic effects of nuclear weapons could also damage or destroy the very thing that the
terrorist group most values. For example, if a terrorist orga- nization were struggling with another group for control of
their common home- land, the use of nuclear weapons against the enemy group would devastate the
terrorists' own home territory . Using nuclear weapons would be extremely counter-
productive for the terrorists in this scenario. It is thus not obvious that all terrorist groups would use nuclear weapons. Some
groups would probably not. The propensity for nuclear acquisition and use by ter- rorist groups must be assessed on a case-by-case
basis.
---EXT Al Qaeda Weak

No impact

Zakaria ’11 (May 2nd, 2011 04:11 PM ET Al Qaeda is over Editor’s Note: This is the first of three posts from Fareed Zakaria on
the death of Osama bin Laden. The other posts are Vindication of Obama's strategy and What did Pakistan know? You can follow
Fareed on Facebook and Twitter or timely analysis of global events. By Fareed Zakaria, CNN

In the wake of Osama bin Laden’s death, a number of people are saying that this does not mean
that al Qaeda has been destroyed. Some argue that the organization may, in fact, be thriving.
Front-page articles in both The New York Times and The Washington Post make this claim. Many
officials from Obama downward are saying this. I understand why officials have to say this. They
want to be cautious. They don’t want to overpromise. But the truth is this is a huge,
devastating blow to al Qaeda, which had already been crippled by the Arab Spring . It is not an
exaggeration to say that this is the end of al Qaeda in any meaningful sense of the word. Al Qaeda is not
an organization that commands massive resources. It doesn’t have a big army. It doesn’t have vast reservoirs of
funds that it can direct easily across the world. Al Qaeda was an idea and an ideology, symbolized by an extremely charismatic figure
in Osama bin Laden. Bin Laden was this Saudi prince-like figure who had gone into the mountains of Afghanistan forsaking the riches
of a multibillion-dollar fortune, fought against the Soviets, demonstrated personal bravery and then crafted a seductive message
about Islam and Islamic extremism as a path to destroy the corrupt regimes of the Middle East. History teaches us that the loss of
the charismatic leader - of the symbol - is extraordinarily damaging for the organization. It is very difficult to keep such an
organization together, particularly in the absence of great power backers. In the case of al Qaeda, this is a virtual organization held
together by its message and the inspiration it provided. A large part of that inspiration was bin Laden. Ayman Zawahiri may have
been the brains behind the outfit, but he did not excite people. When people volunteered for jihad, they were volunteering to be bin
Laden’s foot soldiers, not Ayman Zawahiri’s or Khalid Sheikh Mohammed’s. The loss of bin Laden’s personality is hugely important
because it was so much part of al Qaeda’s appeal. In addition, we must remember that the death of bin Laden is not occurring in a
vacuum. The Arab Awakening has already crippled the basic rationale of al Qaeda. Al Qaeda existed because bin Laden argued that
the regimes of the Arab world were dictatorial and oppressive. He argued that the United States was supporting those regimes and,
as a result, Muslims had to engage in terrorism against the United States and those regimes. He claimed that the only way to achieve
change was through violence, terrorism and Islamic extremism. In the past few months, we have seen democratic, peaceful, non-
Islamic revolutions transform Egypt and Tunisia. We are seeing these forces changing almost every government in the Arab world. Al
Qaeda is not in the picture. So when you combine the Arab Spring with bin Laden’s death, you have a very powerful one-two punch
to al Qaeda. Certainly, there are groups of terrorists around the world, some of which now call themselves al Qaeda. These groups
are loosely affiliated in some sense. But gangs of bad guys have always been around. With the death of bin Laden, the
central
organizing ideology that presented an existential seduction to the Muslim world and an existential threat to the
Western world is damaged beyond repair. We’re left with free-lance terrorists who will, of course, be able to inflict some
harm. But the Somali pirates are able to inflict harm on civilians, and that doesn’t turn them into an existential threat to the Western
world. That existential threat is gone.

Al Qaeda can’t launch attacks

Ismail ’11 (Ismail Nehad Ismail, UK based writer/broadcaster and commentator on Middle Eastern Affairs GET UPDATES FROM
NEHAD ISMAIL Like 7 Another 9/11 Unlikely; Al Qaeda is doomed Posted: 10/09/11 18:18
Al-Qaeda is too weak to mount a major attack against USA in future. The terror group has been
emasculated and is not in a position to carry out spectacular operations. It is important to
note that the group has failed to carry out any major attacks since July 7th 2005 bombing in
London. The war on terror is not over yet. Ten years down the line, $3.2 trillion later, and over 6000 American soldiers killed with
44000 injured, the fight against Al-Qaeda is far from finished. Why has it taken so long to nearly defeat Al Qaeda. Despite suffering
huge losses in its ranks and the decapitating of its iconic symbol Osama Bin Laden and his most senior Operational Chiefs, Al Qaeda
is still alive but not kicking. It has not been completely destroyed. I would say Al Qaeda is severely weakened but not dead .
It is
now scattered and splintered and has presence in Somalia, Yemen, Nigeria, Sub-Saharan Africa and an assortment of
dormant cells all over the world. What it lacks is an effective central command and a co-ordinating body .
Al Qaeda is short of cash. Worst of all they are running out of recruits willing to sacrifice
themselves for some redundant Jihadist ideas.

Al-Qaeda is dead – attacks fail and ideology dead

Bergen 12 (Peter Bergen, CNN national security analyst, is the author of "Manhunt: The Ten-Year Search for bin Laden, From
9/11 to Abbottabad.", 6/6/2012, "And now, only one senior al Qaeda leader left", edition.cnn.com/2012/06/05/opinion/bergen-al-
qaeda-whos-left/index.html)

Washington (CNN) -- The news that Abu Yahya al-Libi,


the No.2 leader of al Qaeda, is now confirmed to
have been killed in a CIA drone strike in Pakistan's tribal region along the border with Afghanistan further underlines
that the terrorist group that launched the 9/11 attacks is now more or less out of business. Under President
Barack Obama, CIA drone strikes have killed 15 of the most important players in al Qaeda, according to a count maintained by the
New America Foundation (a nonpartisan think tank where I am a director). Similarly, President George W. Bush also authorized
drone strikes that killed 16 important al Qaeda operatives in Pakistan while he was in office. As a result, according
to senior
U.S. counterterrorism officials, there now remains only one leader of any consequence in al
Qaeda and that is Ayman al-Zawahiri, the tetchy Egyptian surgeon who became the head of the group following the
death of its founder, Osama bin Laden, in a U.S. Navy SEAL raid in Pakistan in May 2011. Zawahiri, presumably, is keenly
aware of the fate of so many of his longtime colleagues in al Qaeda . He will be expending considerable
energy not to end up on the business end of a missile fired by a CIA drone if he, too, is hiding in the Pakistani tribal regions where
the drone strikes have been concentrated. Meanwhile, Zawahiri faces an almost impossible task to follow
through on al Qaeda's main mission: attacking the United States, or failing that, one of its close allies. Al
Qaeda hasn't conducted a successful attack in the West since the bombings on London's transportation
system on July 7, 2005, and of course, the group hasn't succeeded in attacking the United States for
more than a decade. There are, however, al Qaeda's regional affiliates still to contend with. The
most virulent of those is the Yemen-based Al Qaeda in the Arabian Peninsula. It was AQAP that tried to bring down
Northwest Flight 253 over Detroit on Christmas Day 2009 using a Nigerian recruit who had secreted a hard-to-detect bomb
in his underwear, and it was AQAP that smuggled bombs in printer cartridges onto cargo planes bound for the U.S. in October 2010.
Last month came news that a spy had penetrated AQAP and had retrieved a new generation of underwear bomb that the group's
bomb maker had apparently recently designed to bring down a commercial jet. But all of AQAP's plots to bring down
planes have had one thing in common: They failed. Some might say that that while al Qaeda the
organization may be basically dead, its ideology continues to thrive and to inspire "lone
wolves" to attack the United States. In fact, lone wolves inspired by jihadist ideology have managed to
kill a total of 17 Americans in the United States since 9/11, according to a tally maintained by the New America
Foundation. Meanwhile, 54 Americans are reported to be killed every year by lightning , according to the
National Weather Service. In other words, to the average American, lightning is about 30 times more deadly than
jihadist terrorism. Few Americans harbor irrational fears about being killed by a lightning bolt. Abu Yahya al-Libi's death on
Monday should remind them that fear of al Qaeda in its present state is even more irrational.
---EXT Strategy Shift

Strategy has shifted- will be small scale terrorism

Finn ‘10 (Risk of small-scale attacks by al-Qaeda and its allies is rising, officials say By Peter Finn Washington Post Staff Writer
Wednesday, September 22, 2010; 10:50 PM

Al-Qaeda and its allies are likely to attempt small-scale, less sophisticated terrorist attacks in the
United States, senior Obama administration officials said Wednesday, noting that it's extremely difficult to detect such threats in
advance. "Unlike
large-scale, coordinated, catastrophic attacks, executing smaller-scale attacks
requires less planning and fewer pre-operational steps," said Homeland Security Secretary
Janet Napolitano, testifying before the Senate Homeland Security and Governmental Affairs Committee. "Accordingly,
there are fewer opportunities to detect such an attack before it occurs." Terrorism experts have puzzled over al-Qaeda's apparent
unwillingness after the Sept. 11, 2001, attacks to use car bombs, improvised explosives and small arms to conduct assaults in the
United States. The group appeared fixated on orchestrating another dramatic mass-casualty event, such as the simultaneous
downing of several commercial airliners. Indeed, attacks inspired by al-Qaeda in Madrid in 2004 and London in 2005 involved
multiple, coordinated bombings targeting mass-transit systems. But the risk of a single-target bombing or an attack by a lone
gunman has increased, officials say, with the rise of al-Qaeda-affiliated groups in the tribal areas of Pakistan, in Yemen and in
Somalia, and with the emergence of radicalized Americans inspired by the ideology of violent jihad. " The
impact of the
attempted attacks during the past year suggests al-Qaeda, and its affiliates and allies, will attempt to
conduct smaller-scale attacks targeting the homeland but with greater frequency," said Michael Leiter, director of the
National Counterterrorism Center, pointing to plots against the subway system in New York, the attempt to down a commercial
airliner approaching Detroit and the failed car bombing in Times Square.

AQAP proves- small scale attacks are the new strategy

Riedel 11 – Senior Fellow in the Saban Center for Middle East Policy at the Brookings Institution and a professor at Georgetown
University (Bruce, 08/01, “AQAP’s ‘Great Expectations’ for the Future,” http://www.ctc.usma.edu/posts/aqap%E2%80%99s-
%E2%80%98great-expectations%E2%80%99-for-the-future)

Strategy of a Thousand Cuts

AQAP devoted the third issue of Inspire to the parcel bomb plot and to outlining its strategy for defeating the United States. It
expanded beyond traditional al-Qa`ida strategic thinking. AQAP claims it now has a team of crafty bombmakers producing its
wares that can supposedly get through the most sophisticated airport surveillance equipment in the world. It says
its goal is to
“hemorrhage” the U.S. economy by conducting
waves of small-scale attacks similar to the parcel bombs (a
“thousand cuts”) and the Christmas Day plot that force added security countermeasures . The cover proudly
proclaimed that the parcel plot cost just $4,200 to execute. The Detroit operation has already produced expensive new security
measures at airports from Amsterdam to Auckland. These new attacks are notable for their relatively small
footprint. They are harder to defeat because they are less complex. Unlike the 9/11 plot or the 2006 failed attempt to blow
up 10 airliners en route from London to North America, these efforts are conducted by a small number of people. Only a few
participated in the planning and execution, and the plots went from concept to action in a few months. Abdulmutallab, for
example, was recruited and trained for his mission in Shabwa Province only a few weeks before his attack.[6] The recent intelligence
on AQAP’s attempts to acquire ricin fit within this strategy. According to U.S. officials cited in the New York Times, “evidence points
to efforts to secretly concoct batches of the [ricin] poison, pack them around small explosives, and then try to explode them in
contained spaces, like a shopping mall, an airport or a subway station.”[7] As part of this strategy, AQAP is using its propaganda
message to inspire American Muslims to act on their own to attack targets on U.S. territory. Al-`Awlaqi says he encouraged Major
Nidal Malik Hasan to conduct his carnage at Fort Hood in Texas on November 5, 2009, an attack that killed 13 people. More recently,
another U.S. soldier of Palestinian descent, Naser Abdo, tried to carry out an attack at the same base.[8] The police found a copy of
an article from Inspire in his possession.[9]
---EXT No Means

1NC Miller—nuclear terrorism is laughable—Mueller’s study proves it’s defunct—


material can’t be made into a bomb, can’t bypass safeguards, no state sponsor, no
tech, no funding, and necessarily large networks increase risk of capture and failure—
all reasons groups wouldn’t pursue in first place

1 in 3.5 million risk—most recent data proves

Mueller ‘11—IR prof at Ohio State. PhD in pol sci from UCLA (2 August 2011, John, The Truth about Al Qaeda,
http://www.foreignaffairs.com/articles/68012/john-mueller/the-truth-about-al-qaeda?page=show)

As a misguided Turkish proverb holds, "If your enemy be an ant, imagine him to be an elephant." The new information
unearthed in Osama bin Laden's hideout in Abbottabad, Pakistan, suggests that the United States has been doing so
for a full decade. Whatever al Qaeda's threatening rhetoric and occasional nuclear fantasies, its potential as a
menace, particularly as an atomic one, has been much inflated. The public has now endured a
decade of dire warnings about the imminence of a terrorist atomic attack . In 2004, the former CIA spook
Michael Scheuer proclaimed on television's 60 Minutes that it was "probably a near thing," and in 2007, the physicist Richard Garwin
assessed the likelihood of a nuclear explosion in an American or a European city by terrorism or other means in the next ten years to
be 87 percent. By 2008, Defense Secretary Robert Gates mused that what keeps every senior government leader awake at night is
"the thought of a terrorist ending up with a weapon of mass destruction, especially nuclear." Few, it seems, found much solace in
the fact that an al Qaeda computer seized in Afghanistan in 2001 indicated that the group's budget for research on
weapons of mass destruction (almost all of it focused on primitive chemical weapons work) was some $2,000 to
$4,000. In the wake of the killing of Osama bin Laden, officials now have more al Qaeda computers, which reportedly contain
a wealth of information about the workings of the organization in the intervening decade. A multi-agency task force has completed
its assessment, and according to first reports, it has found that al
Qaeda members have primarily been engaged
in dodging drone strikes and complaining about how cash-strapped they are . Some reports suggest
they've also been looking at quite a bit of pornography. The full story is not out yet, but it seems breathtakingly
unlikely that the miserable little group has had the time or inclination, let alone the money, to
set up and staff a uranium-seizing operation, as well as a fancy, super-high-tech facility to
fabricate a bomb. It is a process that requires trusting corrupted foreign collaborators and other
criminals, obtaining and transporting highly guarded material , setting up a machine shop staffed
with top scientists and technicians, and rolling the heavy, cumbersome, and untested finished
product into position to be detonated by a skilled crew, all the while attracting no attention
from outsiders. The documents also reveal that after fleeing Afghanistan, bin Laden maintained what one member of the task
force calls an "obsession" with attacking the United States again, even though 9/11 was in many ways a disaster for the group. It led
to a worldwide loss of support, a major attack on it and on its Taliban hosts, and a decade of furious and dedicated harassment. And
indeed, bin Laden did repeatedly and publicly threaten an attack on the United States. He assured Americans in 2002 that "the youth
of Islam are preparing things that will fill your hearts with fear"; and in 2006, he declared that his group had been able "to breach
your security measures" and that "operations are under preparation, and you will see them on your own ground once they are
finished." Al Qaeda's animated spokesman, Adam Gadahn, proclaimed in 2004 that "the streets of America shall run red with blood"
and that "the next wave of attacks may come at any moment." The obsessive desire notwithstanding, such fulminations
have clearly lacked substance. Although hundreds of millions of people enter the United States legally every year, and
countless others illegally, no true al Qaeda cell has been found in the country since 9/11 and exceedingly few
people have been uncovered who even have any sort of "link" to the organization. The closest effort
at an al Qaeda operation within the country was a decidedly nonnuclear one by an Afghan-American, Najibullah Zazi, in 2009.
Outraged at the U.S.-led war on his home country, Zazi attempted to join the Taliban but was persuaded by al Qaeda operatives in
Pakistan to set off some bombs in the United States instead. Under surveillance from the start, he was soon arrested, and, however
"radicalized," he has been talking to investigators ever since, turning traitor to his former colleagues. Whatever training Zazi received
was inadequate; he repeatedly and desperately sought further instruction from his overseas instructors by phone. At one point, he
purchased bomb material with a stolen credit card, guaranteeing that the purchase would attract attention and that security video
recordings would be scrutinized. Apparently, his handlers were so strapped that they could not even advance him a bit of cash to
purchase some hydrogen peroxide for making a bomb. For al Qaeda, then, the operation was a failure in every way -- except for the
ego boost it got by inspiring the usual dire litany about the group's supposedly existential challenge to the United States, to the
civilized world, to the modern state system. Indeed,no Muslim extremist has succeeded in detonating even a
simple bomb in the United States in the last ten years, and except for the attacks on the London Underground in
2005, neither has any in the United Kingdom. It seems wildly unlikely that al Qaeda is remotely ready to go nuclear. Outside of
war zones, the amount of killing carried out by al Qaeda and al Qaeda linkees, maybes, and
wannabes throughout the entire world since 9/11 stands at perhaps a few hundred per year.
That's a few hundred too many, of course, but it scarcely presents an existential, or elephantine,
threat. And the likelihood that an American will be killed by a terrorist of any ilk stands at one in
3.5 million per year, even with 9/11 included.

Threat assessment prove no focus on nuclear—ideology and capabilities

Basit ’11 (12/23/11 – writer for Islamabad Pulse (Abdul, “A threat assessment of Al-Qaeda’s strength and weaknesses-IV.”
http://www.weeklypulse.org/details.aspx?contentID=1641&storylist=2)

A threat assessment of Al-Qaeda’s current status looking into its capabilities, intentions and
opportunities would reveal that currently it clearly lacks capabilities and has fewer opportunities
at its disposal; however, it still has the intentions to carry forward its agenda of transnational jihad. Effective and
efficient responses to Al-Qaeda’s threat at political and ideological level have isolated the terror
network. As mentioned in previous pieces Al-Qaeda’s staunchest ally, the Afghan Taliban, have distanced
themselves from its ideology of global jihad and portray themselves as nationalist resistant movements. Currently, Al-
Qaeda’s closest ally in Pakistan, the Tehrik-e-Taliban Pakistan (TTP), is also suffering desertions and
dissentions. Internal divisions are quite visible within the TTP ranks and it is finding it difficult to keep its act together. In such a
scenario, it will be doubly difficult for the TTP to protect and shelter Al-Qaeda in Pak-Afghan tribal region.  Starting with
capabilities, Al-Qaeda does not possess both manpower and firepower to carry out large-scale
terror attacks against its target. It is weak at the center but strong at the fringes. Out of the network’s 10 main
leaders listed after the September 11, 2001 attacks in the United States, only two are still alive: Ayman al-Zawahiri, Osama bin
Laden’s one-time deputy who took over after his boss was killed in May; and Abu Yahya Al-Libi. However, due to continuous threat
of CIA-led predator drone strikes and fear of being spotted they
remain underground. Most of the time their
focus is on how to survive and keep themselves alive and, every now and then, to appear in a video message to
address their followers and operatives. This in turn weakens their ability to manage operational matters of Al-Qaeda and actively
coordinate with its world-wide cells. Even before his death former Al-Qaeda chief Osama Bin Laden was also leading life of a recluse
and he was hardly in touch with the leadership of his group.  After
9/11 Al-Qaeda has failed to target any major
attacks beyond Gulf. Most of its terror plots were foiled or averted by the security agencies.
According to a research carried out by the Heritage Foundation since the terrorist attacks of September 11, 2001 at least 39 terror
plots against the United States have been foiled. Moreover, few lone-wolf style terror acts by Al-Qaeda
operatives were also unsuccessful. The abortive attempts of Richard Reid and Faisal Shahzad are two cases in point.
Richard Reid abortively tried to light a fuse protruding from his shoe on a Paris bound American Airlines Flight No. 63. Reid was
overpowered by fight crew and passengers and the flight was diverted to Boston. While Faisal Shahzad, who tried to blow up a car
bomb in New York’ Time Square, failed to detonate the explosive material and was arrested by security authorities. Looking at
intentions the
group still harbors its espoused vision of global jihad and target its enemies around the globe. However, it has
not been able to recover from various setbacks it has suffered in the last decade. Various attempts,
abortive or otherwise, establish this fact beyond any doubt that against all odds and difficulties Al-Qaeda has not given up on its
stated goals and objectives. The like-minded terror networks which Al-Qaeda has built, highlights its aims. A worrying factor in this
regard is the breakdown of state institutions in different Muslim countries of Africa and Middle East as well as spread of
radicalization in Muslim Diaspora communities of the West and US. Al-Qaeda has thrived in failed or failing Muslim states. The
erosion of incompetent and corrupt Muslim leaders and poor governance created huge vacuums which Al-Qaeda masterly exploited
and furthered its own interests. Currently, the abysmal state of affairs in several Muslim countries provides an ideal opportunity to
Al-Qaeda and its like-minded groups to re-cultivate their influence. A
heartening thing to notice in Arab Spring was
‘minimal’ to ‘no’ Al-Qaeda influence in these movements . Though these protests varied from country to
country in their agendas and motivations, one thing common in these mass movements was that they sprouted from their own set
of problems in local contexts. The major demands in these movements were better job opportunities, right to elect their
representatives and end to decades of dictatorial rules and monarchies. None of these moments attributed the ills to external forces
(read America) and demanded solutions which do not provide Al-Qaeda any groundswell. However, a concerning thing in this
situation is the transition phase. If the public mandate is not respected and peaceful transfer of power to elected public-
representatives is not facilitated by interim set-ups then prolonged transition phases can provide Al-Qaeda with an opportunity to
inject its influence to manipulate the process of power transition from old to new setups.  Another lesson learnt from Arab Spring is
the rise of Islamist forces in elections. Any attempt to sideline these Islamist forces would pave way for Al-Qaeda to manipulate the
circumstances to its benefit. A case in point is suppression of Islamic Salvation Front (FIS) in Algeria by Algerian military which over
the years brought Algeria’s Islamists closer to Al-Qaeda. Al-Qaeda in the Islamic Maghrib (AQIM) grew out of a conflict in Algeria
between the government and Islamist militants. 

Can’t build it

Mueller, 10 Ohio State University Political Science Professor,

[John, "Nuclear weapons are the greatest threat to humankind", http://www.lebanonwire.com/1001MLN/10011006FP.asp]

"Fabricating a Bomb Is 'Child's Play.'"

Hardly. An editorialist in Nature, the esteemed scientific journal, did apply that characterization to the manufacture of uranium
bombs, as opposed to plutonium bombs, last January, but even that seems an absurd exaggeration. Younger, the former Los
Alamos research director, has expressed his amazement at how "self-declared 'nuclear weapons
experts,' many of whom have never seen a real nuclear weapon," continue to "hold forth on
how easy it is to make a functioning nuclear explosive ." Uranium is "exceptionally difficult to
machine," he points out, and "plutonium is one of the most complex metals ever discovered, a
material whose basic properties are sensitive to exactly how it is processed." Special technology
is required, and even the simplest weapons require precise tolerances. Information on the general idea for building a bomb
is available online, but none of it, Younger says, is detailed enough to "enable the confident
assembly of a real nuclear explosive." A failure to appreciate the costs and difficulties of a
nuclear program has led to massive overestimations of the ability to fabricate nuclear weapons .
As the 2005 Silberman-Robb commission, set up to investigate the intelligence failures that led to the Iraq war, pointed out, it is
"a fundamental analytical error" to equate "procurement activity with weapons system
capability." That is, "simply because a state can buy the parts does not mean it can put them together and make them work."
For example, after three decades of labor and well over $100 million in expenditures, Libya was
unable to make any progress whatsoever toward an atomic bomb . Indeed, much of the country's nuclear
material, surrendered after it abandoned its program, was still in the original boxes.

Insider sources confirm they aren't going for a nuke seriously.


Mueller ’10 (John, Prof. Pol. Sci. – Ohio State U., American Conservative, “Nuclear Bunkum”, 1-1,
http://www.amconmag.com/article/2010/jan/01/00020/)

To show al-Qaeda’s desire to obtain atomic weapons, many have focused on a set of
conversations that took place in Afghanistan in August 2001 between two Pakistani nuclear scientists , bin
Laden, and three other al-Qaeda officials. Pakistani intelligence officers characterize the
discussions as “academic.” Reports suggest that bin Laden may have had access to some radiological material—acquired
for him by radical Islamists in Uzbekistan—but the scientists told him that he could not manufacture a weapon with it. Bin
Laden’s questions do not seem to have been very sophisticated . The scientists were incapable of
providing truly helpful information because their expertise was not in bomb design but in
processing fissile material, which is almost certainly beyond the capacities of a non-state group .
Nonetheless, some U.S. intelligence agencies convinced themselves that the scientists provided al-Qaeda with a “blueprint” for
constructing nuclear weapons. Khalid Sheikh Mohammed, the apparent mastermind behind the 9/11 attacks,
reportedly said that al-Qaeda’s atom-bomb efforts never went beyond searching the Internet .
After the fall of the Taliban in 2001, technical experts from the CIA and the Department of
Energy examined information uncovered in Afghanistan and came to similar conclusions. They
found no credible proof that al-Qaeda had obtained fissile material or a nuclear weapon and no
evidence of “any radioactive material suitable for weapons .” They did uncover, however, a “nuclear related”
document discussing “openly available concepts about the nuclear fuel cycle and some weapons related issues.” Physicist and
weapons expert David Albright concludes that any al-Qaeda atomic efforts were “seriously
disrupted”—indeed, “nipped in the bud”—by the invasion of Afghanistan in 2001 . After that, the
“chance of al-Qaeda detonating a nuclear explosive appears on reflection to be low.”
---EXT No Motive/Conventional

1NC Kapur—no motive to pursue nuclear—conventional weapons fit with terrorist group
structure and belief—large scale alienates support and seen as counterproductive—wouldn’t
pursue a nuke even if they had the means—conventional accomplishes all their goals

Prefer conventional weapons—structure and ideology

Craig 11 [Campbell, professor of international relations at the University of Southampton Special Issue: Bringing Critical Realism
and Historical Materialism into Critical Terrorism Studies Atomic obsession: nuclear alarmism from Hiroshima to al-Qaeda Critical
Studies on Terrorism Volume 4, Issue 1, 2011, April, pages 115-124]

Let us address each of his claims, in reverse order. Mueller suggests that the risk of an act of major nuclear terrorism is exceptionally
small, along the lines of an asteroid hitting the earth. Drawing upon his powerful book against terrorism alarmism, Overblown
(2006), he shows that serious anti-Western terrorist groups are today widely scattered and disorganized –
precisely the wrong kind of arrangement for the sustained and centralized project of building an
atomic bomb. Looking for immediate results, terrorist groups are likely to go with what works today, rather
than committing to a long-term and likely futile project . He points out, as have other authors, that so-called
‘rogue’ nations, even if they obtain a bomb, are never going to hand it over to terrorists : to do so would
utterly negate everything they had worked so hard for . A nation such as Iran that somehow decided to give
its bomb to al-Qaeda (leaving aide their completely different objectives) would not only be handing over a weapon
that it had spent years and billions to build , and giving up the prestige and deterrence the bomb supposedly confers,
it would also be putting itself at acute risk of being on the receiving end of a retaliatory strike
once the terrorists did their work. By what rationale would any leader make such a move? The potential costs would be
astronomical, the benefits non-existent.

Easy and cheap—no reason to prefer otherwise

Whitlock ‘7 Craig Whitlock, Washington Post Foreign Service, July 5, 2007, The Washington Post, “Homemade, Cheap and
Dangerous: Terror Cells Favor Simple Ingredients In Building Bombs,” http://www.washingtonpost.com/wp-
dyn/content/article/2007/07/04/AR2007070401814_pf.html

Counterterrorism officials have warned for years that Osama bin Laden and his lieutenants have
tried to obtain weapons of mass destruction, such as a nuclear device or chemical or biological weapons. In
response, U.S. military and intelligence agencies have invested vast amounts of money to block their acquisition. So far,
however, al-Qaeda and its affiliates have relied almost solely on simple, homemade bombs crafted
from everyday ingredients -- such as nail-polish remover and fertilizer -- when plotting attacks in Europe and the
United States. The makeshift bombs lack the destructive potential of the conventional explosives that rake Iraq on a
daily basis. They are also less reliable, as demonstrated by the car bombs that failed to go off in London last week after the culprits
tried to ignite them with detonators wired to cellphones. But other attempts have generated plenty of mayhem and
damage, including the kitchen-built backpack bombs that killed 52 people in the London public transit system on July 7, 2005. "It
makes no difference to your average person if somebody puts a car bomb out there that is crude
or one that is sophisticated," said Chris Driver-Williams, a retired British major and military
intelligence officer who studies explosive devices used by terrorist groups. "If it detonates, all of a sudden
you've got a very serious device and one that has achieved exactly what the terrorists wanted."
The advantages of homemade explosives are that they are easy and cheap to manufacture, as well as
difficult for law enforcement agencies to detect. According to one expert, the peroxide-based liquid explosives that an al-Qaeda cell
allegedly intended to use to blow up nine transatlantic airliners last summer would have cost as little as $15 a bomb.

View mass casualty attacks as counter productive

Roberts and Moodie ‘2 Brad Roberts, Inst Dfnse Analyses, and Michael Moodie, Chem & Bio Arms Cntrl Inst, ‘2 (Defense
Horizons 15, July)

The argument about terrorist motivation is also important. Terrorists


generally have not killed as many as they
have been capable of killing. This restraint seems to derive from an understanding of mass
casualty attacks as both unnecessary and counterproductive. They are unnecessary because terrorists, by
and large, have succeeded by conventional means. Also, they are counterproductive because they might
alienate key constituencies, whether among the public, state sponsors, or the terrorist leadership group. In Brian Jenkins'
famous words, terrorists want a lot of people watching, not a lot of people dead . Others have argued that
the lack of mass casualty terrorism and effective exploitation of BW has been more a matter of accident and good fortune than
capability or intent. Adherents of this view, including former Secretary of Defense William Cohen, argue that "it's not a matter of if
but when." The attacks of September 11 would seem to settle the debate about whether terrorists have both the motivation and
sophistication to exploit weapons of mass destruction for their full lethal effect. After all, those were terrorist attacks of
unprecedented sophistication that seemed clearly aimed at achieving mass casualties--had the World Trade Center towers collapsed
as the 1993 bombers had intended, perhaps as many as 150,000 would have died. Moreover, Osama bin Laden's constituency would
appear to be not the "Arab street" or some other political entity but his god. And terrorists answerable only to their deity have
proven historically to be among the most lethal. But this debate cannot be considered settled. Bin
Laden and his followers
could have killed many more on September 11 if killing as many as possible had been their primary objective. They now
face the core dilemma of asymmetric warfare: how to escalate without creating new interests for the stronger power and thus the
incentive to exploit its power potential more fully. Asymmetric
adversaries want their stronger enemies
fearful, not fully engaged--militarily or otherwise. They seek to win by preventing the stronger partner from exploiting its
full potential. To kill millions in America with biological or other weapons would only commit the
United States--and much of the rest of the international community-- to the annihilation of the
perpetrators.

9/11 is as big as they’ll ever aim


Mueller ‘6 (John Mueller, Professor of Political Science @ Ohio State University, 2006, Foreign Affairs, Sep/Oct, p. lexis)

One reason al Qaeda and "al Qaeda types" seem not to be trying very hard to repeat 9/11 may
be that that dramatic act of destruction itself proved counterproductive by massively
heightening concerns about terrorism around the world. No matter how much they might
disagree on other issues (most notably on the war in Iraq), there is a compelling incentive for states -- even ones
such as Iran, Libya, Sudan, and Syria -- to cooperate in cracking down on al Qaeda, because they know that they could
easily be among its victims. The FBI may not have uncovered much of anything within the United States since 9/11, but thousands of
apparent terrorists have been rounded, or rolled, up overseas with U.S. aid and encouragement .
Although some Arabs
and Muslims took pleasure in the suffering inflicted on 9/11 -- Schadenfreude in German, shamateh in Arabic -- the
most common response among jihadists and religious nationalists was a vehement rejection of
al Qaeda's strategy and methods.

Studies prove-
Parachini ‘3 John Parachini, policy analyst at RAND, Fall 2003 (Washington Quarterly, l/n)

An apparent lack of interest on the part of terrorist groups in acquiring unconventional weapons also
helps explain why unconventional weapons attacks are so rare. In the case studies on the Irish
Republican Army (IRA), the FARC, and Hamas, political vision, practical military utility, and moral codes
all restrained them in part from seeking and using unconventional weapons. In some cases, group leaders
indicated to members that the use of chemical or biological weapons would not be legitimate to their struggle. Hamas leader Abu
Shannab, for one, stated that the use of poison was contrary to Islamic teachings . n22 Although Hamas is a
religiously based organization, its struggle to establish a Palestinian state on Israeli territory and to eliminate Israel as a state is
decidedly political.
---EXT No Retaliation
1NC Jenks-Smith and Herron—no retaliation even if attack is successful—no institutional
support and culpability uncertainty mean governments would err on the side of caution—be
skeptical of their escalation and miscalc claims—history proves governments collaborate and
talk to solve crises

It’s not an option on the table

Schmitt and Shanker ’11 ( BY ERIC SCHMITT, THOMAS SHANKER | SEPTEMBER 6, 2011 Eric Schmitt is a terrorism and
national security correspondent for the New York Times. Thomas Shanker is a Pentagon and national security correspondent for the
Times.

3. The Threat to Bomb Mecca As


fears of a second attack mounted following the 9/11 strikes, U.S.
government planners frantically cast about for strategies to protect the country. Even the most
far-fetched ideas had a hearing, however briefly. In one case, some government planners
proposed that if al Qaeda appeared ready to attack America again , the United States should
publicly threaten to bomb the city of Mecca in Saudi Arabia, the holiest site in all of Islam, in
retaliation. "Just nuts!" one Pentagon aide wrote to himself when he heard the proposal. The
idea was quickly and permanently shelved.

No public or international support.

Bremmer ‘4 Ian, president of Eurasia Group, senior fellow at the World Policy Institute,
September 13, New Statesman, “Suppose a new 9/11 hit America…,” p. Lexis

What would happen if there were a new terrorist attack inside the U nited S tates on 11 September 2004? How
would it affect the presidential election campaign? The conventional wisdom is that Americans - their patriotic defiance aroused -
would rally to President George W Bush and make him an all but certain winner in November. But consider the differences between
the context of the original 9/11 and that of any attack which might occur this autumn. In 2001, the public reaction was one of
disbelief and incomprehension. Many Americans realised for the first time that large-scale terrorist attacks on US soil were not only
conceivable; they were, perhaps, inevitable. A majority focused for the first time on the threat from al-Qaeda, on the Taliban and on
the extent to which Saudis were involved in terrorism. This time, the public response would move much more quickly from shock to
anger; debate over how America should respond would begin immediately. Yet it
is difficult to imagine how the Bush
administration could focus its response on an external enemy. Should the US send 50,000 troops to the
Afghan-Pakistani border to intensify the hunt for Osama Bin Laden and 'step up' efforts to attack the heart of al-Qaeda? Many would
wonder if that wasn't what the administration pledged to do after the attacks three years ago. The president would face intensified
criticism from those who have argued all along that Iraq was a distraction from 'the real war on terror'. And what if a significant
number of the terrorists responsible for the pre-election attack were again Saudis? The Bush administration could
hardly
take military action against the Saudi government at a time when crude-oil prices are already
more than $45 a barrel and global supply is stretched to the limit. While the Saudi royal family might
support a co-ordinated attack against terrorist camps, real or imagined, near the Yemeni border – where recent
searches for al-Qaeda have concentrated – that would seem like a trivial, insufficient retaliation for an attack on the US
mainland. Remember how the Republicans criticised Bill Clinton's administration for ineffectually 'bouncing the rubble' in
Afghanistan after the al-Qaeda attacks on the US embassies in Kenya and Tanzania in the 1990s. So what kind of response might be
credible? Washington's concerns about Iran are rising. The 9/11 commission report noted evidence of co-operation between Iran
and al-Qaeda operatives, if not direct Iranian advance knowledge of the 9/11 hijacking plot. Over the past few weeks, US officials
have been more explicit, too, in declaring Iran's nuclear programme 'unacceptable'. However, in
the absence of an official
Iranian claim of responsibility for this hypothetical terrorist attack, the domestic opposition to such a war
and the international outcry it would provoke would make quick action against Iran unthinkable.
In short, a decisive response from Bush could not be external. It would have to be domestic.
Instead of Donald Rumsfeld, the defence secretary, leading a war effort abroad, Tom Ridge, the homeland security
secretary, and John Ashcroft, the attorney general, would pursue an anti-terror campaign at home. Forced to
use legal tools more controversial than those provided by the Patriot Act, Americans would experience stepped-up domestic
surveillance and border controls, much tighter security in public places and the detention of a large number of suspects. Many
Americans would undoubtedly support such moves. But concern
for civil liberties and personal freedom would ensure
that the government would have nowhere near the public support it enjoyed for the invasion of
Afghanistan.

No strategic targets and can’t trace weapons.

Dowle. 5 (Mark, Teaches at the Graduate School of Journalism at Berkeley, California Monthly, September,
http://www.alumni.berkeley.edu/Alumni/Cal_Monthly/September_2005/COVER_STORY-_Berkeleys_Big_Bang_Project_.asp)

Because terrorists tend to be stateless and well hidden , immediate retaliation in kind is almost
impossible. But some nuclear explosions do leave an isotopic signature, a DNA-like fingerprint that allows forensic physicists
such as Naval Postgraduate School weapons systems analyst Bob Harney to possibly determine the origin of the fissile material in
the bomb. Nuclear forensics is not a precise science, Harney warns. Post-attack sites are almost
certain to be contaminated with unrelated or naturally occurring radioactivity, and there are
numerous, highly enriched uranium stashes in the world with unknown signatures. But there is no question,
according to Peter Huessy, a member of the Committee on the Present Danger and consultant to the National Defense University in
Washington, D.C., that Russian forensic experts could quickly detect Russian isotopes, and that highly enriched uranium (HEU) from,
say, France could readily be differentiated from American HEU. But, Huessy warns, distinguishing
post-blast residues of
Pakistani uranium from North Korean uranium would be more challenging, probably impossible.
Because neither country is a member of the International Atomic Energy Agency, IAEA inspectors have been
unable to collect from their facilities reliable isotope samples that could be compared to post-attack residues. Even if
the uranium were traced, the source nation could claim that the material had been stolen.

Democracy checks.

Pilat 99 Joseph F., Research Assoc. – Int’l Inst. For Strat. Studies, (Survival 40.4, “WMD Terrorism: An Exchange”)

The belief that NBC terrorist acts will have an impact disproportionate to their real physical damage because of public health
hysteria about WMD is not groundless, but it is exaggerated. Falkenrath himself recognises that NBC attacks are unlikely to be
apocalyptic, but he does not follow his own logic to its conclusion. The belief that widespread panic or perhaps
the collapse of public order would result from such attacks is speculative and ill-founded. The
societal responses to accidents or natural disasters that produce massive destruction or
disruption have been on the whole measured and reasonable: public order in Japan did not
collapse in the wake of the Kobe earthquake of 1995 or the subway gassing in Tokyo. The
strength and resilience of liberal-democratic societies in the face of such threats tends to be
underestimated.
History proves

Mueller ‘5 John, Professor Political Science - Ohio State University (Conflict Studies Conference, psweb.sbs.ohio-
state.edu/faculty/jmueller/NB.PDF)

However, history clearly demonstrates that overreaction is not necessarily inevitable. Sometimes, in fact, leaders
have been able to restrain their instinct to overreact. Even more important, restrained reaction--or even
capitulation to terrorist acts--has often proved to be entirely acceptable politically . That is, there are
many instances where leaders did nothing after a terrorist attack (or at least refrained from
overreacting) and did not suffer politically or otherwise.

Limited scope

Mueller 99, John professor of political science at the University of Rochester, and Karl Mueller, assistant professor of
Comparative Military Studies at the School of Advanced Airpower Studies at Maxwell Air Force Base, May/June 19, Foreign Affairs,
“Sanctions of Mass Destruction,” p. Lexis

Nuclear weapons clearly deserve the “weapons of mass destruction” designation because they can indeed destroy masses of people
in a single blow. Even so, it is worth noting that any
nuclear weapons acquired by terrorist groups or rogue states,
at least initially, are likely to be small. Contrary to exaggerated Indian and Pakistani claims, for example, independent
analyses of their May 1998 nuclear tests have concluded that the yields were Hiroshima-sized or smaller. Such
bombs can cause horrible though not apocalyptic damage. Some 70,000 people died in Hiroshima and
40,000 in Nagasaki. People three miles away from the blast sites received only superficial
wounds even when fully exposed, and those inside bomb shelters at Nagasaki were uninjured even though they were close to
ground zero. Some buildings of steel and concrete survived, even when they were close to the blast centers, and most municipal
services were restored within days. A
Hiroshima-sized bomb exploded in a more fire-resistant modern city
would likely be considerably less devastating. Used against well-prepared , dug-in, and dispersed troops,
a small bomb might actually cause only limited damage. If a single such bomb or even a few of
them were to fall into dangerous hands , therefore, it would be terrible, though it would hardly threaten
the end of civilization.
---Terrorism- LNG

Multiple checks prevent LNG terrorism

Quoddy 8 (Bay LLC, “Safety & Security”, http://www.quoddylng.com/safety.html)

Are LNG tankers and storage facilities likely terrorist targets? All parts of our critical energy infrastructure have been reassessed
since the terrorist attacks of September 11, 2001. Security consciousness throughout the United States is heightened.
Shippers have redoubled their already-stringent efforts to ensure security of transportation and the safety
of terminals. There is no indication that LNG facilities or ships are more likely terrorist targets than
other cargo ships or higher visibility political targets such as federal or state landmarks, public gatherings or bridges and tunnels.
Nonetheless, LNG suppliers work closely with U.S. agencies charged with national security, and many developers contract with
international experts who test their plans, procedures, people, and training to ensure they are sound. First, stringent
access controls exist at both the point of origin and the point of destination. Both the liquefaction and re-gasification
terminals have gated security access and continuous surveillance monitoring. Next, highly
specialized, well-trained personnel serve as crewmembers. Before an LNG ship enters U.S. waters, the
immigration service validates the crew. There is a buffer zone required between tankers and other
traffic, and tugboats control the direction of tankers as they approach a terminal. Oversight is handled by the U.S.
Coast Guard and host port autho

No impact to LNG explosion

Styles 4 (Geoffrey SW, Managing Director – GSW Strategy Group, LLC, “Energy Outlook”, 5-14,
http://energyoutlook.blogspot.com/2004/05/lng-disaster-movie-front-page-of-last.html)

The other remarkable feature of this situation is the degree of fear being instilled by those opposed to the LNG terminals.
Although I don't fault communities for wanting a say in the kind of industrial facilities that will be in close proximity to them,
those discussions should still be based on fact and not wild ravings. The Wall Street Journal cited one
LNG opponent who
claimed that the destructive potential of an LNG tanker was equivalent to 55 Hiroshima bombs (see
analysis below). This reflects an irrational fear, bolstered by junk science. It's hard to argue with, but we
cannot base the nation's energy policies on paranoia. Many have picked up on the explosion at the LNG plant in
Skikda, Algeria (see my blog of January 21) as evidence of the risks of handling LNG, but even if that were a fair comparison--and
there are good reasons why it is not--it is actually a pretty good illustration that the risks are similar to those associated with
many kinds of industrial facilities and not orders of magnitude greater, as activists assert. Having recently seen prosaic and
trusted objects turned into deadly weapons, it is natural to worry a bit more about LNG than we might have a few years ago.
Every LNG tanker--along with every crude oil or gasoline tanker, tank truck, or rail car--has the potential for destructive misuse.
Yet we have not grounded all airplanes for fear they will be turned into cruise missiles, nor can we shun every link in the energy
chain on which we all rely. While we can minimize risk, we cannot eliminate it. And if you don't want the LNG terminal in your
neighborhood, for reasons that seem perfectly valid to you, just exactly whose neighborhood are you proposing as an
alternative? Or are you and your neighbors prepared to take your houses off the gas grid and heat them with something else?
Finally, for anyone interested in the atomic bomb comparison, a few facts: 1. A fully loaded LNG tanker of 120,000 cubic meters
capacity holds about 50,000 tons of methane. 2. The yield of the Hiroshima bomb was equivalent to 21,000 tons of TNT. 3.
Conservatively assuming that TNT and methane have the same energy content gives you a ratio
of 2.5, not 55, but we are not done yet. 4. An atomic bomb releases its energy (from the conversion of
matter into energy, via our old friend e=mc^2) in 1/1000th of a second. This makes for a stupendous flash and
explosion, with a surface temperature comparable to that of the sun. This is why every H-bomb has an A-bomb trigger.5. A
chemical explosion of methane requires a narrow range of air/fuel mix (5-15%) that could not be achieved all
at once for the entire volume of an LNG tanker. In the real world, it would take many seconds and
probably minutes to consume all the available fuel. 6. The difference between points 4 and 5 above is analogous
to the difference between going from 60-0 mph by hitting a brick wall, compared to a panic
stop using the brakes. The same energy is released, but in very different ways . 7. If it were easy
to liberate nuclear weapon yields from large quantities of fuel, people would be doing this routinely. The closest we get is
something like this. And note that there is an enormous distinction between achieving A-bomb-like overpressures
in a very limited radius with a fuel/air device vs. the kind of wide-scale effects of an actual nuclear explosion.

Accidents won’t cause LNG explosions

Quoddy 8 (Bay LLC, “Safety & Security”, http://www.quoddylng.com/safety.html)

What is the likelihood of explosion at the storage tanks? An explosion is highly unlikely because LNG is
stored under atmospheric pressure. LNG is never flammable and natural gas cannot explode if
it is not confined under pressure. Immediately after being released into the surrounding air, LNG starts to
warm up and convert into a gas. Since initially the gas is colder and heavier than the surrounding air, it creates an icy
fog - freezing the moisture in the air, as when a freezer door is opened. However, as the gas warms up, it blends with the air
and begins to disperse and rise upward. The cloud could ignite close to the ground only if there is something to ignite it during a
narrow window when the right mixture of gas and air exists for combustion. If released on water, LNG floats and
vaporizes, leaving no residue.
Terrorists won’t attack LNG tankers – no incentive.

Farrell, 7, Richard- analyst for Chamber Corporation, “Maritime Terrorism,” Naval War College Review, Vol 60 No 3, EBSCO

A recent study by the ioMosaic Corporation draws upon field measurements, operational information, and engineering information
on LNG vessels gathered over the last sixty years."' It takes into account terrorism and other twenty-first- century threats. The
overall conclusion is straightforward—that in the highly unlikely event of a very large scale release of liquified
natural gas on land or water, significant effects will be felt in the immediate vicinity.'"50 However, the zone of impact
would not extend anywhere close to the thirty miles predicted by some groups." As long as an LNG
vapor cloud is unconfined, it will not explode. A cloud reaching a populated area would
quickly find an ignition source and burn back to the spill site before it could cover large
numbers of people. If inflicting mass casualties is the terrorist goal, LNG facilities and tankers
are not good targets. CONTINUED. TEXT OMITTED. THIS IS FOOTNOTE 50. 50 According to
Kalelkar et al. (p. 4), available data and explosion dynamics indicate that it is not possible to
detonate LNG vapors, even with an explosive charge on a storage tank, unless the LNG vapors
contain high fractions of ethane and propane (more than 20 per- cent). They claim that the
likelihood of this scenario is equivalent to winning the Powerball or Megabucks lottery several
times simultaneously. For impact, p. 22

No risk of an LNG explosion – chemical properties of LNG and structural countermeasures


within LNG tankers prevent.

Raj, 7 Dr. Phani K-, President, Technology & Management Systems, Inc., Testimony to Committee on Homeland Security,
“Department of Homeland Security: LNG Tanker Security,” http://hsc.house.gov/hearings/index.asp?ID=25

Other researchers have postulated the failure of LNG ship tanks due to fire heat exposure leading to a LNG tank explosion (due to
BLEVE). As indicated above, no such explosions have occurred in tanks of ships carrying refrigerated liquefied petroleum gas (LPG)
even after being subjected to intense fires on the outside. For a BLEVE to occur the liquid in a tank must be heated to temperatures
well beyond its normal boiling temperature (and therefore higher pressure in the tank) and the tank has to be suddenly
depressurized by, say, the sudden rupture of the tank wall. The rapid depressurization results in the production of large volumes of
vapor, which may ignite and form a large fireball and the pressure waves created may hurl pieces of the tank to some distance. The
higher the pressure in the tank at rupture the worse will be the effect of a BLEVE including the throw of the pieces of the tank to
distances up to ¼ mile. BLEVE incidents have occurred in pressurized (LPG) rail tank cars or relatively small LPG storage tanks in
which the pressure is normally about 105 psig and the tank will withstand pressures up to 375 psig before rupturing. Large tanks
have not exhibited the BLEVE type of explosive rupture; the smaller the tank and the higher the pressure it can
withstand the greater is the likelihood for the occurrence of a BLEVE. One can conclude that a BLEVE is extremely unlikely
in a LNG ship tank when one considers the conditions necessary for such an event to occur .
First, the volume of liquid in each tank of a LNG carrier is large (25, 000 m3) . To heat such a massive amount of
liquid to any temperature significantly higher than its normal boiling temperature requires significant amount of
heat to be input. The calculated lifetime of a LNG pool fire (caused by the rupture of another tank) generally ranges
from a few minutes to, at best, 15 minutes. Over this burning time it is difficult to transfer significant
quantity of heat to the LNG in the tanks. Second, the LNG tanks are well insulated and separated from
the outer hull by a large (at least 2 m wide) inter-hull ballast space, which impedes heat transfer from
the fire to the tank wall. Third, the tanks are provided with relief valves, which will ensure that no significant rise
in the pressure occurs. Furthermore, because of the size of the LNG tank the roof of the tank will not be able to withstand
any significant increase in pressure () before being damaged. Last but not the least, actual experience with large ships carrying
refrigerated fuels (LPG and butane) in tanks similar to those in LNG carriers indicates that even though they were subject to fires
of BLEVE as a potential public hazard
lasting several hours no BLEVE resulted. Therefore, in my opinion, the consideration
phenomenon in the scenario of an accidental or intentional release from a LNG ship is addressing a non-problem.

Tankers aren’t terrorist targets and the impact will be limited

Melhem et al 6 – PHD Professor of Structural Engineering


(Dr. G. A. Melhem, Dr. A. S. Kalelkar, Dr. S. Saraf “Managing LNG Risks: Separating the Facts from the Myths” updated 2006,
http://archives1.iomosaic.com/whitepapers/Managing%20LNG%20Risks.pdf)

Myth No. 2 LNG tankers and land based facilities are vulnerable to terrorism ; An LNG potential disaster
(explosion of an LNG tanker) is greater today because of the threat of terrorism. The gigantic quantity of energy stored in huge
cryogenic tanks is what makes LNG a desirable terrorist target. Tankers may be physically attacked in a variety of ways to destroy
their cargo or used as weapons against coastal targets. Fact As discussed earlier, LNG
ships are not attractive “mass
casualties” terrorist targets. Any explosive charge used on an LNG ship will cause immediate
ignition of the LNG vapors. The subsequent LNG pool fire will have a potentially significant
impact on the immediate release area only. This will significantly limit the extent of impact. There
are also new Coast Guard security regulations (33 CFR Part 105) for LNG tanker movements and terminals. In addition, IMO and the
USCG have established stringent security requirements for vessels in international and United States waters.

5. Our impact outweighs

Kaplan ‘6 Eben, Associate Editor, CFR, “Liquefied Natural Gas: A Potential Terrorist Target?” 2-27,
http://www.cfr.org/publication/9810/)

But an attack
on an LNG terminal might not be so damaging. Terminals are equipped with
emergency fire detection mechanisms designed to minimize the impact of fires resulting from terrorist
attacks or accidents. The most attractive targets are the boats: 1,000-foot tankers with double hulls and specially constructed
storage tanks that keep the LNG cold. A report, put out by Good Harbor Consulting assessing the risk of a proposed LNG terminal in
Providence, Rhode Island, concluded that a successful
terrorist attack on a tanker could result in as many as
8,000 deaths and upwards of 20,000 injuries. It is important to keep in mind that this is the
worst case scenario. A report on LNG safety and security by the University of Texas' Center for Energy and Economics
explains LNG "tanks require exceptionally large amounts of force to cause damage. Because the amount of energy required to
breach containment is so large, in almost all cases the major hazard presented by terrorists is a fire, not an explosion."

-- No accidents –

A) Double hulls
Quoddy 8 (Bay LLC, “Safety & Security”, http://www.quoddylng.com/safety.html)

The shipswill employ both double containment of their contents and double hulls, ensuring a
very low risk of any spills or accidents. This full containment ensures that if leaks or spills do
occur, the LNG will be contained and isolated. The double hulls ensure a very low risk that any breach would even
reach the hull containment tanks. The vessels are designed with a double hull to ensure minimization of leakage in the event of
a collision or grounding, as well as separate ballast.

B) Safety systems

Quoddy 8 (Bay LLC, “Safety & Security”, http://www.quoddylng.com/safety.html)

LNG facilities have extensive, state-of-the-art warning systems, including gas detectors,
ultraviolet or infrared fire detectors, smoke or combustion product detectors, low temperature detectors ,
and detectors to monitor LNG levels and vapor pressures. Codes and standards from state, national, and
international agencies and institutions insure the chances of any releases are very small,
and if there are releases, the volume of the release is minimal. In addition to warning systems, LNG facilities
have automated firefighting systems, including foam, dry chemical, or water dispersal and automatic
shutdown systems.
---Terrorism- AQIM

AQIM not a threat – too weak, can’t carry out attacks, squo counterterror solves.

PN ‘12. [Peace Newspaper, accredited internationally by the United Nations, IMF/World Bank, and nationally by the Ministry of
Communication, internally citing numerous security experts, “Al-Qaeda seeks help from Maghreb affiliate” --
http://www.peacenewspaper.net/?p=1584]

Al-Qaeda’s central command sees AQIMas capable of sustaining the global terror group. Analysts say
it’s a poor bet. It is a desperate time for the once-dreaded al-Qaeda. Funds are depleted, long-time
figurehead Osama Bin Laden is dead and counter-terror operations in Afghanistan and Pakistan keep
operatives on the run or in hiding. The embattled terror group is now counting on al-Qaeda in Islamic Maghreb (AQIM) and
other organisations for restored credibility and rescue. When Ayman al-Zawahiri needed to overcome a shortfall of funds after the
death of Bin Laden, he looked to the Sahara. “After Osama Bin Laden was killed, it became likely that the future of al-Qaeda would
involve Maghreb countries,” university professor Ely al-Sheikh Ould Bah said. Arms and drug trafficking, along with abductions for
ransoms, have produced a revenue stream for al-Qaeda’s North African branch. But all
is not as it appears, security experts
caution. The high-profile kidnappings of westerners in the Sahel-Saharan region in recent months do not necessarily
indicate “an increase in the organisation’s capacity to strike”, a UN Security Council report noted in
February. The abductions “might just imply a need to raise money and get international attention, or be the result of internal power
struggles”. Security analyst Hamadi Ould Dah agrees that al-Qaeda “had no other option but to resort to its Arab Maghreb
branch”. “However, this group seems to be unable to provide any support for the main al-Qaeda organisation
because of the internal problems facing it, and also because of crippling security crackdowns by Sahel
countries,” he tells Magharebia. Given significant attrition among its ranks, as well as its poor media skills,
the Maghreb branch appears unlikely to live up to Zawahiri’s expectations. The Madrid-based Institute of
Studies on Conflicts and Humanitarian Action (IECAH) recently confirmed that al-Qaeda’s Maghreb offshoot faces problems of its
own. “Internal fractures within AQIM lead us to question the real cohesion of the organisation ,”
Madrid-based Institute of Studies on Conflicts and Humanitarian Action (IECAH) observed in a February report. “Its activities in the
Sahel region seem to better reflect the action of different cells with varied driving forces rather than a co-ordinated action with a
clear identity and rationale,” the IECAH said. And as al-Qaeda gets more connected with criminal activities “and disconnects with the
ideological discourse as justification of its actions, its tactics are subject to increasing criticism and rejection by religious leaders in
the Maghreb and Sahel”, the report added. Al-Qaeda has long prepared its branch in the Islamic Maghreb to assume the mantle
when it collapses. To celebrate the 5th anniversary of the September 11th, 2001 attacks, the terror group posted a video in which al-
Zawahiri announced that Algeria’s “Salafist Group for Preaching and Combat” (GSPC) had become part of al-Qaeda. GSPC emir
Abdelmalek Droukdel was eager to mount a serious media campaign. The only problem: no one knew how to do it. Abu Yasser
Sayyaf, GSPC’s webmaster, had to issue an online plea for help uploading content and using different programmes, “which shows
how far behind GSPC was technologically”, terrorism analyst and academic Manuel Torres Soriano explained in a January analysis,
“The Road to Media Jihad: The Propaganda Actions of al-Qaeda in the Islamic Maghreb”. Sayyaf’s excuse for the second-rate video
and audio was the group’s isolated location in the mountains of Algeria. Things changed in 2007. The GSPC’s adoption of the name
“al-Qaeda in the Islamic Maghreb” (AQIM) coincided with a media outreach initiative modelled after the parent group to which
AQIM had become allied. More media production was compulsory for GSPC if it were to merge with al-Qaeda as an official branch.
The newest members of the global terror organisation quickly recognised the merit of visual and audio messages, Torres Soriano
says. More media production was compulsory for GSPC if it were to merge with Al-Qaeda as an official branch. For example, in 2007
— the year of the name change — AQIM released six videos, almost double the number of all videos it produced during the previous
eight years. Al-Qaeda had been trying for years to solidify its support in the North Africa desert, but the
GSPC was still a
long way from performing like an affiliate of a global terror group . “Although the GSPC started its
activities at the information era par excellence, its method was more that of a traditional gang than that of a group ‘apprenticed’ by
Osama Bin Laden,” Torres Soriano points out. After AQIM’s merger with al-Qaeda, however, “it turned into a group with its own
media strategy”. “Ayman al-Zawahiri urged AQIM to develop its own propaganda machine,” journalist Mohamed Ould Sid al-Moktar
tells Magharebia. Now that AQIM has met the challenge, Bin Laden’s successor and his top aides believe it to be a group upon which
al-Qaeda can depend, al-Moktar adds. They forget that the
Maghreb branch is under siege from Sahel security
services. “Al-Qaeda’s dependence on AQIM is similar to a weak entity leaning on another
weak entity; something that will take both of them down,” al-Moktar says. Abdel-Rahim Al-Manar Slimi of
Mohammed V University in Rabat noted that “al-Qaeda’s branches are prospering in areas where the state is absent or is failing; a
situation which didn’t exist in North Africa until the fall of Kadhafi’s regime, and Libya’s entry into a transitional status”. Meanwhile,
AQIM is trying to expand relations with terrorist movements in Africa, especially Boko Haram. Al-Qaeda’s Maghreb branch may also
try to exploit the recent clashes between Touareg rebels and Malian army. Mauritanian President Mohamed Ould Abdel Aziz
recently suggested in an interview with Le Monde that al-Qaeda was involved in the northern Mali conflict. A spokesperson for the
Touareg rebels countered that the National Movement for the Liberation of Azaouad (MNLA) “neither has interests nor shares any
policies with that terrorist organisation”. Instead of turning to AQIM, Ayman Al-Zawahiri could have chosen to depend on al-Qaeda
branches, in the Arabian Peninsula or Iraq, Boko Haram or Shabab al-Mujahideen. But these branches are not based in the Sahara.
Analysts believe that “al-Qaeda Central” sees the desert as a place where its leaders can escape, and where rampant criminal activity
keeps the money flowing. Mohamed Ould Zain of the Sahara Media network points out, however, that both al-Qaeda and its
offshoot AQIM face difficult times in the wake of the Arab Spring. “There is growing recognition that peaceful struggle and
democracy are the most effective way to make progress, not violence and killing,” Ould Zain says.

AQIM sucks

Stratfor 10, (“AQIM: The Devolution of al Qaeda's North African Node,” 8-10)

From AQIM’s official founding in 2006 to the present, our research indicates a few discernable patterns regarding the group’s
operational capacity inside Algeria. First, the majority of attacks have produced low casualty counts, from zero to
three. Attacks that did achieve a higher degree of lethality (which we define as two or more people killed), were restricted mostly to
Algiers and slightly to the east of the capital. Second, after GSPC’s September 2006 merger with al Qaeda, the number of violent
attacks and threats against foreign/international targets within Algeria’s borders increased significantly. This was particularly evident
in the spring of 2008 and continues to date. The attack and casualty rates were highest between mid-2008 and late 2009. Indeed,
during the last six months of 2009 there was a noteworthy spike in the number of attacks. However, tracing the geographical
distribution of attacks last year, we noticed that AQIM had zeroed in on softer, more vulnerable targets closer to its base in the east,
strongly suggesting that the group’s operational capacity had been crippled by Algerian
counterterrorism efforts and that AQIM was likely trying to defend its base. The uptick in attacks appears to have been an
effort on the part of the North African al Qaeda node to prove that it remained a security threat and relevant actor on the
international jihadist stage. It was not a verifiable indicator that the group’s strength was surging. It could well have been nothing
more than a last gasp that will not likely be repeated, unless AQIM is given room to rest and regroup. Also, since the group’s merger
with al Qaeda in 2006, research shows an increase in attacks in September of each year, near the end of or directly after the Muslim
holy month of Ramadan. The more recent increase of abductions of Westerners and clashes with security forces in
the Sahara-Sahel is
not, as some observers believe, an indication of AQIM’s ability to effectively strike targets
at a much longer range. Kidnapping and executing a 78-year-old aid worker in the Sahel simply does not make the same
forceful statement as a coordinated multiple VBIED attack in Algiers. We believe this expanded activity in the south is more likely the
result of a rivalry between sub-commanders seeking to raise funds for the organization and an overall indication of the weakness
and lack of cohesion within the group. It could also be the result of increased initiative on the part of countries in the Sahara-Sahel
region to go on the offensive against AQIM. A joint military base operated by Algeria, Mauritania, Mali and Niger was set up in April
in the southern Algerian town of Tamanrasset to coordinate counterterrorism activities and clamp down on one of AQIM’s main
smuggling routes. According to a report July 25 in the Algerian newspaper El Watan, Algeria will be in charge of air support, with
Mali covering ground operations, Mauritania heading up communications, Niger handling logistics and Burkina Faso serving in an
observation role. However, as recent events have demonstrated, the joint effort has failed to advance beyond vocal commitments
and formalities. Moreover, the North African al Qaeda node has failed in its original objective of unifying North African
militants in the Sahara-Sahel and Maghreb, remaining an Algerian-run organization by location and leadership. Despite numerous
attempts to recruit militants and organize cells of Europeans of North African heritage, it also has failed to strike Europe —
namely France and Spain, its preferred targets — and other Western countries. Indeed, AQIM has failed to live up to al-Zawahiri’s
promise when he announced the formation of al Qaeda’s new North African node, that it would “be a bone in the throat of the
American and French crusaders and their allies.” And pressure against the group is intensifying . The military
operations by French-backed Mauritanian troops in Mauritania and Mali in July were likely a harbinger of a more aggressive
counterterrorism stance against the group by countries in the region. Paris’ open declaration of war on AQIM after the death of the
French hostage will certainly add energy to the effort. However, instead of putting French troops on the ground in Algeria, an idea
that Algeria openly rejected (probably because of the sensitive colonial history between the two countries), France’s declaration will
likely lead to enhanced military and intelligence efforts against the North African al Qaeda node. Joining France’s call, Niger’s military
leader, whose remarks were conveyed by French Foreign Minister Bernard Kouchner, said July 28 his government is ready to “take
necessary action” against terrorism and AQIM in the Sahara-Sahel. Meanwhile, Algeria itself is continuing its assault against AQIM.
The Algerian daily newspaper El Khabar reported July 26 that Algerian security forces, responding to a number of small attacks
against army patrols in the region, launched an operation July 21 that included heavy air strikes against suspected AQIM hideouts in
Tizi Ouzou and Bouira provinces. This followed an announcement by the Ministry of Defense in June that it was reinforcing its
National Gendarmerie police force by adding 9,000 members in an effort to take the offensive against AQIM. According to El Khabar,
citing official sources in the ministry, Algeria has dispatched an additional 16,000 police to the southern Sahara-Sahel region of the
country to confront AQIM and combat cross-border crime and smuggling. This would increase the security coverage in the south
five-fold compared to the previous three years. The coverage has been further expanded by a recent doubling of the number of air
patrols conducted unilaterally by the Algerian police and jointly by the police and the Algerian army. As part of the overall build-up,
Algerian security forces also have incorporated a new communications network known as “Ronital.” Set up in the Tizi Ouzou region
of the Kabylie Mountains, where Algiers is concentrating its fight against AQIM, Ronital serves as a unified communications network
operated by Algeria’s central command to ensure the secure and reliable transmission of electronic messages, including sound and
images. As the government offensive continues, AQIM’s future seems bleak. In all likelihood, attacks involving small
arms and IEDs against military and civilian convoys and slightly more hardened symbols of the Algerian state such as police stations
will continue to be concentrated in Algeria, near AQIM’s eastern stronghold in Blida and Boumerdes provinces. It
does not appear that AQIM has the operational freedom to conduct large VBIED attacks against hard targets in Algiers, as it has done
in the past. If the regional security momentum continues at its current pace, 2011 may see al Qaeda’s North African
node further reduced and fragmented, its remnants pushed farther south into the Sahara-Sahel and the northern
portions of Mali, Mauritania and Niger. Indeed, abductions of Westerners and clashes with security forces in that region may even
increase, but only because the group is unable to secure the propaganda victories and financial resources it needs due to the success
of Algerian security operations. Like the Islamic State of Iraq, if criminal enterprises like smuggling and kidnapping-for-ransom
operations become AQIM’s predominant focus, it may find its credibility among jihadists and appeal to potential recruits eroded,
making its already tenuous position even more difficult.

AQIM Isn’t a threat.

Bauer ‘12 – Middle-Eastern Studies and Arabic at the University Exeter, with a specialisation in North-African politics (William,
“Al-Qaeda in North Africa Doesn't Scare Anyone,” http://www.policymic.com/article/show?id=1363)

Since the death of Osama bin Laden, talk has been rife in intelligence circles about the next move Al-Qaeda will make and in which
theater of operations this will be in. One thing is for sure, that Al-Qaeda in the Islamic Maghreb (AQIM) will not be one to
watch, for in Hugh Roberts' words: "They haven't done anything spectacular." AQIM has been around since 2007,
primarily active in Algeria, Niger, Mali, and Mauritania, the group has committed a few attacks,
kidnapped a few people and killed a few Algerian soldiers. But, in terms of the threat they pose
to the Maghreb and to Europe, AQIM is essentially a non-entity. Founded in 2007 by elements linked to the defunct GIA, a
group infamously involved in Algeria's civil war of the 1990s, AQIM was given the full blessing of Al-Qaeda's central command as
part of a new strategy of Glocal Jihad. Glocal Jihad is a strategy designed to wage a global war on local lines, targets
and objectives: in other words, decentralized jihad. AQIM was supposed to operate in the Maghreb, bringing down regimes which
were viewed as illegitimate, Western puppets; but they have failed to do so. They failed because in a region largely unaffected by
foreign intervention, unlike the Middle East or Central Asia, they lacked a galvanizing cause for raising money, volunteers,
and equipment. With no foreign oppressor to fight, most in the Maghreb balked at the idea of needlessly killing their own
countrymen. AQIM is also unable to fight off the singularly effective counter-intelligence service of the Algerian regime (Algeria is
AQIM's main theater of operations), which, with the help of the U.S., has fought them and cornered them in the Saharan desert,
where few, if any, real targets can be found. Also, with
Algeria as its main hub of operations, AQIM does not
achieve the popular support needed to achieve any of its objectives. After the bloody and indiscriminate civil
war that shook Algeria during the 1990s, Algerians have no stomach for a group bent on violent acts of terror, especially when this
group is linked to the former GIA. Equally, AQIM frequently blunders and causes some spectacular accidents
which decimate their group; including one that in 2009 allegedly wiped out 40 of its operatives during an experiment with
a chemical or biological weapon. AQIM does, however, excel at seizing foreign nationals in the Sahel region,
especially in Mauritanian sovereign territory as detailed by the British FCO in its travel advice. This shows AQIM to be an
organization that thrives on opportunism rather than planning, which makes it incapable of carrying
out longer range, more complex attacks. Finally, because of the Arab Spring and nascent social movements in the
Maghreb, AQIM is largely a relic of another age, when violence seemed the only means for people of the Maghreb to
express discontent. Now, with popular and often peaceful protests becoming the norm, AQIM will become
inconsequential in the region and to the world. Whether, in light of the events of the past year, this same fate awaits the Al-
Qaeda group itself is yet to be seen but its affiliate in the Maghreb has definitely become an irrelevant non-entity.

AQIM has no ability to carry out a foreign operation

Keaten 11 – Associated Press (Jamey, 08/06, “Judge: AQIM won’t attack Europe,”
http://www.nwherald.com/2011/08/06/judge-aqim-wont-attack-europe/ab4wl3r/)

PARIS – France’s top judge in the fight against Islamic terrorism said Friday that al-Qaida’s North African wing had
shown no ability to strike in Europe or elsewhere beyond its zone of operations. Al-Qaida in the Islamic Maghreb,
born of a former insurgent group in Algeria, remains motivated largely out of a desire to attack former colonial power France. It
holds four French hostages, and French officials have called the group the biggest terrorist threat to France and its interests. In an
interview, anti-terrorism judge Marc Trevidic suggested AQIM is being forced to work hard to control parts of its
traditional territory in the Sahel region along the southern Sahara. “It’s been shown that AQIM is only able to strike in
its own zone, by wanting to kill tourists – and we have seen nothing emerge as a significant foreign
operation in Europe that was really organized by AQIM,” he said. Still, AQIM has been active in offering statements of support
through the Internet to would-be terrorists in Europe, Trevidic said, citing his recent case files. “ It’s incitation without a
structure behind it,” he said. The group is “holed up, and already has trouble s controlling its zone ...
Only when a terror group is very strong in its own territory will it begin exporting.”

Empirics

CFR 9 – Council on Foreign Relations (07/21, “Al-Qaeda in the Islamic Maghreb (AQIM),” http://www.cfr.org/north-africa/al-
qaeda-islamic-maghreb-aqim/p12717#p6)

Is the group capable of carrying out global attacks?

Some experts warn the group's growing confidence could increase its willingness to target Westerners both inside and outside
Algeria. The group has called for jihadis who can't reach the battlefields of Iraq to target Jews, Christians, and apostates (PDF) in
their own regions. AQIM has taken over, and some say revitalized, many Europe-based cells of the former GIA for both fundraising
and recruiting. In spite of its growing global presence, some experts doubt AQIM's ability to carry out a
Qaeda-scale attack. "They haven't done anything spectacular," says Hugh Roberts, an expert on North
African politics and former head of the International Crisis Group's North Africa project. "They have not actually pulled off a
single terrorist attack in Europe in the eight years they've existed. And that's a fact that you have
to put in balance against European security services that say the group is a major threat. "
Exaggerated.

Thurston 11 – Masters of Arts in Arab Studies program at Georgetown University, Ph.D. candidate in the Religion Department at
Northwestern University in the Islam in Africa track (Alex, 08/19, “Response to NYT Article on Boko Haram,”
http://sahelblog.wordpress.com/2011/08/19/response-to-nyt-article-on-boko-haram/)

Finally, the notion that we should fear a scenario where “extremists bent on jihad are spreading their
reach across the continent and planting roots in a major, Western-allied state that had not been seen as a hotbed of
global terrorism” seems overblown to me. AQIM has suffered setbacks this summer in Mauritania and Mali
(and it conducted fewer kidnappings in 2010 than in 2009), al Shabab recently abandoned Mogadishu, and Boko Haram’s primary
goals remain oriented to altering Nigerian politics (spreading shari’a, removing hated leaders, etc.). The formation of a pan-
African jihadist movement is, it seems to me, still a remote possibility.

Can’t Escalate.

Lebovich 11 – Policy Analyst for the National Security Studies Program at the New America Foundation (Andrew, 07/18,
“Analyzing AQIM Jihad Recruitment Propaganda,” http://thewasat.wordpress.com/2011/07/18/analyzing-aqim-jihad-recruitment-
propaganda/)

Are shared local grievances enough to sway the Muslim masses – or even more than a radical fringe – to adopt global jihad? AQIM
has gained a limited local following since emerging from the Algerian GSPC Groupe Salafiste pour le Predication et le
Combat; its membership now consists of hundreds from across the Maghreb and Sahel, with geographical concentration
in the border area lining Mali, Mauritania, and Algeria. “What had started out as an Algerian problem is now engulfing
Mali and Mauritania. They are the weak link,” said Zakaria Ould Ahmed Salem, a specialist on political Islam at the University of
Nouakchott.[14] But I would argue that AQIM’s recruitment strategy is intrinsically limited by what has
been called “confrontational dialectics between global and local jihad ”.[15] There is no direct
path from point A, the common ground of enduring pain and shame at the colonial experience – a wound reopened
by ongoing militarization of the Sahel region – and frustrations with weak local states, to point B, waging war upon France,
which Shanqiti explicitly references with a direct threat to Nicolas Sarkozy.[16] Shanqiti’s message is atypical of AQIM
communications, affording rare insight into an inward-oriented recruitment campaign tailor-made for the Sahel and Maghreb. Since
the 2007 merger with Al Qaeda, AQIM communications have increasingly featured videos and images to accompany written texts,
allowing increased impact in mass media and raising the disturbing impact of kidnappings of Westerners with graphic materials.[17]
Communications also often come with French translations to facilitate the task of foreign media, with transcriptions of video and
audio content often added. Torriano posits that these trends stem from a desire to merge with Al Qaeda and please its leaders, or
perhaps were even conditions of joining the Al Qaeda franchise imposed by central leadership. Improvements in the quantity and
quality of AQIM propaganda following the 2007 merger also reflect the influence of Al Qaeda in Iraq, who effectively used media to
exaggerate its image and capacities, “enabling it to lead Iraqi insurgency even when said image did not correspond to its actual size
and representativeness”.[18] Shanqiti’s message is a window into a different kind of propaganda campaign, one that does not seek
to impress foreign jihadis or foreign media but rather to build a political and spiritual community grounded in local Sahelian-
Maghrebi discourses. It is unaccompanied by images and videos that make superficial impressions, nor by a transcription to simplify
the task for foreign media. It contains poetry and allusions to local traditions such as tribalism, pastoralism[19], and the treachery of
a Muslim army collaborating as “agents” of the former colonial power. Poetic communications by groups like Hamas similarly use
poetic verses as a medium for “political mobilization and ideological consolidation”.[20] But whereas Hamas is first and foremost
committed to Palestinian sovereignty and independence, AQIM has no positive goals to offer recruitees – only the
negative one of opposing foreign ‘Crusaders’.

Algeria is a huge alt cause

Cristiani ‘12 – PhD Candidate in Middle East and Mediterranean Studies at King’s College in London, Senior Analyst at the Global
Governance Institute in Brussels (Dario, “Al-Qaeda in the Islamic Maghreb’s Operational Revival in Northern Algeria,”
http://www.jamestown.org/single/?
no_cache=1&tx_ttnews[tt_news]=38441&tx_ttnews[backPid]=7&cHash=e80be7885e1e7689a5137ccc70ee5c65)

In the past few months, there has been a rather remarkable operational return of Al Qaeda in the Islamic Maghreb
(AQIM) in Algeria. On July 16, a double suicide attack rocked a security compound in the small town of Bordj Menail, 60 km
east of Algiers, with two people killed and 14 injured (Algérie Plus, Jul 16). A few days later, Algerian security forces in the area of
Thénia intercepted an AQIM team alleged to be on its way to carry out an attack in Algiers (Tout Sur l'Algérie, Jul 26). The most
important attack occurred in late August, when another double suicide attack hit the military academy of Cherchell, about 100 km
west of Algiers, killing 18 people and injuring another 20 (Algerian Press Service, August 27; El Watan, August 27). AQIM
claimed responsibility for both attacks, blaming also the Algerian government for its support of Muammar Qaddafi
(Afrik.com, July 20; AFP, August 20). An AQIM statement described the Cherchell attack as “a small gift to the families of the martyrs,
the injured and the weak and subdued prisoners of Algeria, who have been suffering the worst of atrocities at the hands of France,
making them subject, with metal and fire, to a criminal gang of Algerian army generals, transgressing across the land spreading much
corruption. In fact, their corruption and crime even reached across the borders to neighboring countries, conspiring against the
revolutions of our brothers in Tunisia and Libya” (al-Andalus Media/al-Fajr Media Center, August 27). Following an overall decline in
operations since 2008, AQIM’s profile remained low in Algeria until April 2011. The causes for this were several:
The “Sahelization” of AQIM as the geographical center of its activity shifted from Algeria to the Sahel. The increasing focus on illegal
business activities rather than on terrorist actions. The consistent and effective counter-terrorist efforts of Algeria. The waning
appeal of violence in a country that is still recovering from the psychological burden of two decades of violence. An internally divided
and fragmented leadership, in which power is rather diffused, with its Salehian factions enjoying a strong autonomy and the formal
leadership, based in Kabylia, exercising a rather loose control over the various AQIM units. However, this situation has been
changing since April. As well as the major attacks described above, there have also been strikes on individuals of the military,
policemen and gendarmes, killing more than 50 people according to official sources (Jeune Afrique, August 5). What does this
recrudescence mean and what are the reasons behind it? Worsening of the Regional Security Picture: The conflict in Libya has had a
negative impact on the security of almost all its neighbors, though it was also overestimated to further national security interests
and assist the survival of the political elites. Support for Mua’mmar Qaddafi was equated with preserving regional stability, although
this policy proved unsustainable in the long term. Though the regime was wary of the fact that the Algerian
protests might indicate the beginning of a spillover effect from the Libyan conflict, it is clear that protests,
strikes and mass rallies are constant elements of the Algerian political and social landscape. Nevertheless,
the fear of a spillover effect has strongly influenced the Algerian regime’s reaction to the Libyan conflict, and regional security
has worsened through the spread of weapons from Libya n arsenals and ineffective control of Algeria’s porous
borders. The possibility that weapons and explosives have reached the country is high , and a nexus between
the rising number of attacks in Algeria and the conflict in Libya can be identified. The National Political Juncture: Although Algeria’s
resilience to the Arab Spring has been greater than that of some other Arab countries, the protests in Algeria were still regarded as
presenting a menace to the survival of the regime. Moreover, Algeria will likely experience an increase in political tension and
divisions over the next few months. The major political personalities are already eyeing the 2014 presidential elections and many of
them have begun creating alliances and strategies for this fundamental date (Algérie 360, May 20). The major party, the Front de
Libération Nationale (FNL), is facing increasing internal fragmentation. Most notable of the factions to spin off from the FNL is the
Mouvement de Redressement et de l’Authenticité, which is harshly critical of FNL secretary general Abdelaziz Belkhadem, one of the
closest politicians to President Abdelaziz Bouteflika (Le Journal d’Algérie, August 15). In narrow security terms, the protests mean a
greater focus by the government on maintaining domestic political stability, hence a devotion of greater quantities of financial
resources and security forces to control these protests. Subsequently, AQIM could have an interest in seizing the political
momentum by exploiting this shift of focus in security on increasing its operational profile prior to destabilizing the state should the
overall socio-political picture enter a precipitate decline. External and Internal Symbolic Meanings: These latest attacks could also
have external and internal symbolic meanings. The external meaning encompasses the international dimension as well as a national
one. Internationally, the attacks aim at showing that the group is alive even though its leader, Osama Bin Laden, has been killed. In
the national dimension, they show that AQIM still has the capability to attack the most visible elements of state control -- the
military installations. The internal meaning of the attacks could be a response to allegations of a decline in influence of the Algerian-
based leadership over other factions of the group. Carrying out successful attacks against Algerian military installations could
represent a means for AQIM Amir Abdelmalek Droukdel to boost his weakened leadership after the “Sahelization” of the movement
entailed a shift in the group’s internal balance of power. Whether this acceleration in AQIM operations in Algeria will be effective in
reviving the fortunes of the Algerian militants is unclear. The
worsening regional picture and the increasing domestic
troubles facing by the Algerian government could represent a major opportunity for AQIM to further increase
its operational profile in Algeria. Internal rivalries could also push some factions to act more vigorously to reaffirm their
power and influence within the organization. The ability of AQIM to return to its 2007-2008 levels of violence in Algeria remains
weak; however, it is undisputable that the strategic context in Algeria has changed slightly in favor of AQIM in the last few months.
---Terrorism- AQAP

AQAP isn’t a large risk to the US

Derrick ’11 (A False Foundation? AQAP, Tribes and Ungoverned Spaces in Yemen Edited by: Gabriel Koehler-Derrick THE
COMBATING TERRORISM CENTER AT WEST POINT www.ctc.usma.edu September 2011 © Combating Terrorism Center at West Point
(September 2011)

frequently been marred by tactical failure. AQAP


Al-Qa`ida in the Arabian Peninsula’s use of terrorist violence has
boasts a low success rate against hardened targets. While operationally innovative, the group has
consistently failed to match the tactical skill of other al-Qa`ida affiliates. 445 The group too often
reuses failed tactics and has not capitalized on real or perceived successes as often as would be
expected of an organization that demonstrates such strategic discipline . Although the increasing influence
of foreign members does extend AQAP’s reach deeper into the West than at any other point in the group’s history, attacks on
the U.S. homeland remain an issue of low salience to a vast majority of Yemenis, preoccupied as
they are with far more pressing local concerns.

Prefer our evidence- based on on-the-ground fieldwork

Derrick ’11 (A False Foundation? AQAP, Tribes and Ungoverned Spaces in Yemen Edited by: Gabriel Koehler-Derrick THE
COMBATING TERRORISM CENTER AT WEST POINT www.ctc.usma.edu September 2011 © Combating Terrorism Center at West Point
(September 2011)

This project is based on twelve months of fieldwork completed by the author from 2008 to 2009 and
subsequent phone interviews with contacts in Marib and al-Jawf through the spring of 2011. The author’s
name has been withheld from this report because of his continued research in the region. Nevertheless, the project would not have
been possible without the assistance of a number of individuals who deserve very public thanks for helping to bring this report to
fruition

AQAP is only conducting small-scale attacks that result in security countermeasures, not war

Riedel 11 – Senior Fellow in the Saban Center for Middle East Policy at the Brookings Institution and a professor at Georgetown
University (Bruce, 08/01, “AQAP’s ‘Great Expectations’ for the Future,” http://www.ctc.usma.edu/posts/aqap%E2%80%99s-
%E2%80%98great-expectations%E2%80%99-for-the-future)

Strategy of a Thousand Cuts

AQAP devoted the third issue of Inspire to the parcel bomb plot and to outlining its strategy for defeating the United States. It
expanded beyond traditional al-Qa`ida strategic thinking. AQAP claims it now has a team of crafty bombmakers producing its
wares that can supposedly get through the most sophisticated airport surveillance equipment in the world. It says
its goal is to
“hemorrhage” the U.S. economy by conducting waves of small-scale attacks similar to the parcel bombs (a
“thousand cuts”) and the Christmas Day plot that force added security countermeasures . The cover proudly
proclaimed that the parcel plot cost just $4,200 to execute. The Detroit operation has already produced expensive new security
measures at airports from Amsterdam to Auckland. These new attacks are notable for their relatively small
footprint. They are harder to defeat because they are less complex. Unlike the 9/11 plot or the 2006 failed attempt to blow
up 10 airliners en route from London to North America, these efforts are conducted by a small number of people. Only a few
participated in the planning and execution, and the plots went from concept to action in a few months. Abdulmutallab, for
example, was recruited and trained for his mission in Shabwa Province only a few weeks before his attack.[6] The recent intelligence
on AQAP’s attempts to acquire ricin fit within this strategy. According to U.S. officials cited in the New York Times, “evidence points
to efforts to secretly concoct batches of the [ricin] poison, pack them around small explosives, and then try to explode them in
contained spaces, like a shopping mall, an airport or a subway station.”[7] As part of this strategy, AQAP is using its propaganda
message to inspire American Muslims to act on their own to attack targets on U.S. territory. Al-`Awlaqi says he encouraged Major
Nidal Malik Hasan to conduct his carnage at Fort Hood in Texas on November 5, 2009, an attack that killed 13 people. More recently,
another U.S. soldier of Palestinian descent, Naser Abdo, tried to carry out an attack at the same base.[8] The police found a copy of
an article from Inspire in his possession.[9]

AQAP is not a threat

Spencer 11 – retired British infantry commander, strategic analyst on political, security, and trade issues of the Middle East and
North Africa and a specialist on Yemen (James, 06/08, “A False Dawn for Yemen's Militants,”
http://www.foreignaffairs.com/ARTICLES/67883/james-spencer/a-false-dawn-for-yemens-militants?page=show)

Although AQAP describes itself as one organization, in practice the group is split in two: the domestic Yemeni group and
those terrorists who fled Saudi Arabia. The two wings operate in separate areas using different methods. Indeed,
their different tactics, and particularly their different weapons, suggest that they have little contact with
each other. The more nationally focused faction of AQAP , based in the Abyan-Shabwa-Marib area, is of little
threat to the West. Its cause is largely directed against the Saleh regime, meaning that with the demise
of Saleh and his system of patronage, much of its raison d'être will end; where it does not, the tribes will drive
it out since it will have outlived its usefulness. The special operations group of AQAP, mostly constituted of transnational jihadis,
remains a danger -- although more so to Saudi Arabia than to the West, for reasons of geography. Nevertheless, Saleh's political
demise is likely to lead to the reform of the Yemeni state's many overlapping security organizations; this, in
turn, will lead to the end of much of the unofficial cooperation between AQAP and sympathizers in the state
security apparatus on which AQAP has depended to plan and mount its operations, such as the follow-up
attack against the South Koreans. As a consequence, AQAP will find it far harder to achieve its attacks. From
AQAP's limited use of suicide bombers to date, it would appear that AQAP does not have access to a large cadre of
volunteers; similarly, the group's relatively slow pace of attacks suggests that it does not have many
competent bomb-makers. Clearly, if Ibrahim al-Asiri, AQAP's master bomb-maker, can be removed from the picture, much
of the special operations group's capability will be lost. Of course, Western policymakers must consider the worst-case scenario:
Yemen as a failed state. Pirates based in Somalia are already the scourge of the Gulf of Aden; additional bases for piracy on Yemen's
southern coast would pose only a marginally increased threat. More important, Somalia is a haven for some transnational jihadis,
but a poor operating base: there are few flights into or out of Somalia, and a limited number of attractive terrorist targets in the
region. Yemenmight descend into Somalia-style chaos, but the difficulties of sustaining the
organization in such an environment would complicate AQAP's operation rather than enable it.

AQAP is weak
Derrick ’11 (A False Foundation? AQAP, Tribes and Ungoverned Spaces in Yemen Edited by: Gabriel Koehler-Derrick THE
COMBATING TERRORISM CENTER AT WEST POINT www.ctc.usma.edu September 2011 © Combating Terrorism Center at West Point
(September 2011)

Yet there is reason for optimism. Despite


uncommonly pragmatic leadership, excellent strategic
communications and widespread animosity toward U.S. foreign policy in Yemen, AQAP is not a
popular movement. It boasts no formal safe haven in Marib or al-Jawf. Nor has it successfully
mobilized tribes to its cause. The group is a mediocre military practitioner at best, and to date
does not have the power to overthrow the Yemeni state. Reports widely attributing the recent
violence in the south to AQAP alone are misleading . Rumors of an ill-defined Islamic Emirate in Abyan’s Zinjibar in
the spring of 2011 are a far better indication of the range of actors vying for political power than of AQAP’s expanding influence. 444
Islamists of varying stripes appear to have established a presence in Ja`ar and Zinjibar during the recent political unrest sweeping
through the country, some apparently using the same name as an ambiguously defined AQAP outreach branch referenced by the
group in 2010 and 2011. Yet as with violence tied to the al-Qa`ida in the Arabian Peninsula generally, it remains incumbent on the
group to prove its ties to the events occurring in Abyan. Shared organizational names and statements of congratulations do not
suffice for evidence of AQAP’s move toward governance and insurgency. AQAP
has never proven itself capable of
holding territory, and there are significant operational risks inherent in its trying to do so. In the
past, attempts to establish an open presence have left AQAP vulnerable to air assaults, and
there can be no doubt that formal control over rural territory risks provoking hostilities with
southern tribesmen.

Instability isn’t key to AQAP- helps their enemies as well

Derrick ’11 (A False Foundation? AQAP, Tribes and Ungoverned Spaces in Yemen Edited by: Gabriel Koehler-Derrick THE
COMBATING TERRORISM CENTER AT WEST POINT www.ctc.usma.edu September 2011 © Combating Terrorism Center at West Point
(September 2011)

Successfully navigating the current policy debate is particularly important given Yemen’s highly
dynamic political environment. While the political unrest currently destabilizing Yemen has
given AQAP more operational space in certain parts of the country, it has also created
opportunities for other opposition actors, virtually all of whom enjoy far more public support
than al-Qa`ida in the Arabian Peninsula. These groups function as natural competitors to AQAP,
not as allies. As broad-based political movements, their successful inclusion in the political process stands to further marginalize
the relevancy of AQAP’s message, which claims that change can come only through jihad. While it is true that neither a more
representative Yemeni government nor the potential departure of President Salih will have any significant impact on the ability of al-
Qa`ida in the Arabian Peninsula to strike the United States in the short term, a more accountable and transparent Yemeni
government presents a serious strategic challenge to the group’s long-term survival.

AQAP is not the lynchpin of global al-Qaeda, and it’s active outside of Yemen

Riedel 11 – Senior Fellow in the Saban Center for Middle East Policy at the Brookings Institution and a professor at Georgetown
University (Bruce, 08/01, “AQAP’s ‘Great Expectations’ for the Future,” http://www.ctc.usma.edu/posts/aqap%E2%80%99s-
%E2%80%98great-expectations%E2%80%99-for-the-future)

Great Expectations: Local and Regional


Although AQAP is plotting against the U.S. homeland, it does not want to replace the al-Qa`ida core in
Pakistan as the leader of the global jihad. AQAP leader Nasir al-Wihayshi publicly proclaimed his group’s allegiance
to Ayman al-Zawahiri as the new amir of al-Qa`ida in July 2011. The most recent edition of Inspire was dedicated to Bin Ladin’s
memory and martyrdom, and it clearly endorses al-Zawahiri as the leader of global jihad, saying “now Shaykh Ayman carries the
banner” of jihad. Yet AQAP
does aspire to play a much larger role in Yemen, the Arabian Peninsula and the
Arab world as a whole.
---Terrorism- AT: Loose Nukes
No missing nukes – they would have been used already

Associated Press 12 – Vladimir Isachenkov, reporter for the Associated Press, January 9,
2012, "How Threat of Loose Soviet Nukes Was Avoided,"
http://www.military.com/news/article/how-threat-of-loose-soviet-nukes-was-avoided.html

There have been gnawing fears that a few Soviet nukes still might have gone missing, but
experts with inside knowledge say that if it were true, the world would already know. " If
somebody or a terrorist group got hold of a nuclear weapon, they would probably use it
as quickly as possible," said Steven Pifer, who served as U.S. ambassador to Ukraine, held other senior State
Department posts and is now director of the Brookings Institute's Arms Control Initiative. "So the
fact that you haven't seen a nuclear detonation ... reflects the fact that the nuclear
weapons have been maintained in a secure way."
---Terrorism- Nuclear

No risk of nuclear terror—means and motive

Chapman 12 [Stephen, columnist and editorial writer for the Chicago Tribune “The Implausibility of Nuclear Terrorism” May
17 http://reason.com/archives/2012/05/17/the-implausibility-of-nuclear-terrorism]

Given their inability to do something simple — say, shoot up a shopping mall or set off a truck bomb — it’s
reasonable to ask whether they have a chance at something much more ambitious. Far from being
plausible, argued Ohio State University professor John Mueller in a presentation at the University of Chicago, “the
likelihood that a terrorist group will come up with an atomic bomb seems to be vanishingly
small .” The events required to make that happen comprise a multitude of Herculean tasks . First, a
terrorist group has to get a bomb or fissile material, perhaps from Russia’s inventory of decommissioned warheads. If
that were easy, one would have already gone missing. Besides, those devices are probably no longer a
danger, since weapons that are not maintained quickly become what one expert calls “ radioactive scrap metal .” If
terrorists were able to steal a Pakistani bomb, they would still have to defeat the arming codes and other
safeguards designed to prevent unauthorized use. As for Iran, no nuclear state has ever given a bomb to an
ally — for reasons even the Iranians can grasp. Stealing some 100 pounds of bomb fuel would require help from
rogue individuals inside some government who are prepared to jeopardize their own lives. Then comes
the task of building a bomb. It’s not something you can gin up with spare parts and power tools in
your garage. It requires millions of dollars , a safe haven and advanced equipment — plus people with
specialized skills, lots of time and a willingness to die for the cause. Assuming the jihadists vault over those
Himalayas, they would have to deliver the weapon onto American soil. Sure, drug smugglers bring in contraband
all the time — but seeking their help would confront the plotters with possible exposure or extortion. This, like
every other step in the entire process, means expanding the circle of people who know what’s going on, multiplying
the chance someone will blab, back out or screw up. That has heartening implications. If al-Qaida
embarks on the project, it has only a minuscule chance of seeing it bear fruit. Given the formidable odds, it
probably won’t bother.

Won’t use nukes- they are rational

Kapur ‘8 [S. Paul, associate professor in the Department of National Security Affairs at the Naval Postgraduate School, The Long
Shadow: Nuclear Weapons and Security in 21st Century Asia. pg. 32]

Before a terrorist group can attempt to use nuclear weapons, it must meet two basic requirements. First, the group must decide that
it wishes to engage in nuclear terrorism. Analysts and policy makers often assume that terrorist groups necessarily want to do so
(Carter 2004; U.S. Government 2002). However, it is not clear that terrorist organizations would necessarily
covet nuclear devices. Although analysts often characterize terrorism as an irrational activity (Laqeuer 1999: 4-5),
extensive empirical evidence indicates that terrorist groups in fact behave rationally , adopting
strategies designed to achieve particular ends (Crenshaw 1995: 4; Pape 2003: 344). Thus whether terrorists
would use nuclear weapons is contingent on whether doing so is likely to further their goals .
Under what circumstances could nuclear weapons fail to promote terrorists' goals? For certain types of terrorist objectives,
nuclear weapons could be too destructive. Large-scale devastation could negatively influence
audiences important to the terrorist groups. Terrorists often rely on populations sympathetic to
their cause for political, financial, and military support. The horrific destruction of a nuclear
explosion could alienate segments of this audience. People who otherwise would sympathize with the terrorists
may conclude that in using a nuclear device terrorists had gone too far and were no longer deserving of support . The
catastrophic effects of nuclear weapons could also damage or destroy the very thing that the
terrorist group most values. For example, if a terrorist orga- nization were struggling with another group for control of
their common home- land, the use of nuclear weapons against the enemy group would devastate the
terrorists' own home territory . Using nuclear weapons would be extremely counter-
productive for the terrorists in this scenario. It is thus not obvious that all terrorist groups would use nuclear weapons. Some
groups would probably not. The propensity for nuclear acquisition and use by ter- rorist groups must be assessed on a case-by-case
basis.

No Retaliation.

Jenks-Smith and Herron 5 (Hank and Kerry, professor and adjunct professor at George Bush School of Government and
Public Service at Texas A&M University. “United States Public Response to Terrorism: Fault Lines or Bedrock?” Review of Policy
Research. September. Lexis)

Our final contrasting set of expectations relates to the degree to which the public will support or demand retribution against
terrorists and supporting states. Here our data show that support for using conventional United States military
force to retaliate against terrorists initially averaged above midscale, but did not reach a high level of
demand for military action. Initial support declined significantly across all demographic and belief categories by the time
of our survey in 2002. Furthermore, panelists both in 2001 and 2002 preferred that high levels of certainty about
culpability (above 8.5 on a scale from zero to ten) be established before taking military action . Again, we find the
weight of evidence supporting revisionist expectations of public opinion. Overall, these results are inconsistent with
the contention that highly charged events will result in volatile and unstructured responses
among mass publics that prove problematic for policy processes. The initial response to the terrorist strikes demonstrated a
broad and consistent shift in public assessments toward a greater perceived threat from terrorism, and greater willingness to
support policies to reduce that threat. But even
in the highly charged context of such a serious attack on the
American homeland, the overall public response was quite measured. On average, the public showed very little
propensity to undermine speech protections, and initial willingness to engage in military retaliation moderated
significantly over the following year.
---Terrorism- Cyber

No cyber attacks- capability or motivation, and no impact

Knake, 10 – fellow at the Council on Foreign Relations [2/16, Robert, “Cyberterrorism Hype v. Fact”,
http://www.cfr.org/terrorism-and-technology/cyberterrorism-hype-v-fact/p21434, AL]

Director of National Intelligence Dennis Blair caught the media's attention recently with two major headlines when he presented this
year's Annual Threat Assessment (PDF) of the U.S. Intelligence Community. The first was his statement that the United States is
"severely threatened" by cyberattacks of "extraordinary sophistication." The second was that al-Qaeda is intent on striking within
the United States in the next six months. Both sections of the assessment are chilling, but they are unrelated. Blair
said that
the United States faces challenges in cyberspace from nation states, terrorist networks,
organized criminal groups, individuals, and other cyberactors. He went on to say, "Terrorist groups and their
sympathizers have expressed interest in using cybermeans to target the United States and its citizens." Fortunately, interest does
not equal capability. After raising the specter of cyberterrorism, Blair never mentioned the cyberthreat from al-Qaeda anywhere in
the five pages he devoted to their plans to strike the United States. Here's why. While
the United States' critical
infrastructure, from the electric grid to the financial sector, is vulnerable to attack through
cyberspace, al-Qaeda lacks the capability and motivation to exploit these vulnerabilities. To
penetrate, map, and damage the networks that control the industrial base requires a large team
of experienced hackers, a lot of time, and advanced infrastructure . Only a handful of groups,
mostly nation state actors, possess this level of capability, and al-Qaeda is not one of them. In
the last ten years, according to the National Counterterrorism Center's Worldwide Incidents
Tracking Database, there have been 63,192 incidents of terrorism. Not one was an incident of
cyberterrorism. As Irving Lachow at NDU has pointed out, the jihadist community heavily relied on one London-based hacker
known by the moniker Irhabi 007, who at best had moderate ability. Since his arrest in 2005, indications are that al-Qaeda's
cybercapabilities have only eroded. Whilecontinuing to rely on petty crime to fund many plots, al-Qaeda
has been unable to capitalize on the explosion of cybercrime , lacking the technical capability to
do so. For al-Qaeda to do any real damage with cyberattacks, it would need to make a multi-year investment in developing
offensive cybercapabilities and would need a secure facility and advance test bed from which to do it. Understanding the control
software for an electric grid is not a widely available skill. It is one thing to find a way to hack into a network and quite another to
know what to do once you're inside. Beyond the technical hurdles, al-Qaeda's primary goal has always
been to generate large numbers of casualties in addition to inflicting economic damage. But
cyberattacks are largely weapons of mass disruption, not destruction . Causing a blackout or
destroying airline reservations systems won't kill many people , if any at all. The worst-case scenario
is that a cyberattack could override controls at a chemical or nuclear plant and cause a chemical release
or nuclear meltdown. Such an incident could kill thousands if not millions. Thankfully, the control systems for plants
that could cause that kind of harm are still "air gapped," disconnected from networks that
connect to the Internet. In attempting to attack the homeland, the organization has relied on decidedly low-tech means. Of
the twenty-two plots disrupted since 9/11, all involved the use of improvised explosives or small arms, and all were aimed at killing
large numbers of people. In its twenty-year existence, al-Qaeda has never carried out a plot intended to do economic harm without
also causing large numbers of casualties. Concerns
about cyberterrorism arise from the fact that al-Qaeda
has expressed interest in devastating the U.S. economy and that Bin Laden has spoken of
"bleeding America to the point of bankruptcy." But the context for these quotes is important,
and has nothing to do with cyberterror aspirations. Bin Laden has articulated a goal of forcing the withdrawal of
U.S. forces from the Muslim world by raising the costs of these deployments both politically and economically to the point that they
are no longer sustainable. To do this, Bin Laden is borrowing a play from the mujahedeen, who pinned down the Soviets in
Afghanistan for over a decade before forcing their withdrawal and, ultimately, the collapse of the Soviet Union. For less than
$500,000 and using box cutters as the primary weapon, al-Qaeda was able to create a military response that to date has cost
between $1 trillion to $2.5 trillion. What
kind of results could al-Qaeda get from hacking? If al-Qaeda
were able to cause a power blackout by hacking SCADA systems, they couldn't do much better
than the tree limbs that caused the 2003 Northeast Blackout. That event put 50 million people
in the United States and Canada in the dark for up to four days. Economists place the cost of
that event between $4.5 and $10 billion, a blip in the $14.2 trillion economy. One thing the United
States has learned about the cost of disruption to the economy is that disruption causes pain that is short lived and minimal. A two-
day snow storm doesn't eliminate two days of economic activity, it only delays it. The same holds true for port closures and other
disruptive activities. For now, the United States has little to fear from al-Qaeda on the cyberfront. Only a handful of sophisticated
nation states currently have the ability to carry out a devastating cyberstrike. In his assessment of the People's Liberation Army
Modernization program, Blair briefly noted that "China's aggressive cyberactivities" pose challenges, and it's true that China, Russia,
and other countries' capabilities do pose a real threat. Luckily, these countries also have vulnerable systems, as well as a lot to lose,
in any conflict, cyber or otherwise. The United States' reliance on the Internet and dependence on automated systems connected to
it represent a massive vulnerability to the United States, but it
is not one that terrorist organizations are likely to
be able to exploit anytime soon. As with any developing technology, the cost and other barriers to developing an
advanced cyberoffensive are declining each year.

No cyberterror – empirics, hollow threats, no capabilities, no motive, disincentives, and focus on


other strategies

Healey ‘11 Director of the Cyber Statecraft Initiative at the Atlantic Council of the United States (Jason, “Cyberterror is
Aspirational Blather,” http://www.acus.org/new_atlanticist/cyberterror-aspirational-blather10/3/11)

The Atlantic Council’s Cyber Statecraft Initiative recently hosted a conference call to discuss the terrorist use of the Internet and how
it has evolved in the ten years since 9/11. The call featured Matt Devost of FusionX, Neal Pollard of PriceWaterhouseCooper and Rick
Howard of VeriSign/iDefense – who have been tracking this and many other online threats for years. While this conversation was off
the record, this blog attempts to capture the spirit. Terms such as “cyber 9/11” and “cyber terrorism” have been used frequently to
describe the security threats posed by terrorists online. Cyber technologies, like any other, enable terrorist groups to do their
terrorizing more effectively and efficiently. In the past few years it is increasingly common for them to use the Internet for
propaganda, fundraising, general support, and convergence. Videos and anonymous discussion forums allow for the dissemination
of training information and the call to arms for more individuals to participate and join groups. Importantly, the panelists agreed
that these groups have not yet used cyber attack capabilities in any significant way to cause casualties or
actually terrorize anyone. While Ibrahim Samudra or Irahabi 007 hacked to raise funds through credit-card fraud, this is a traditional
support activity, not “cyber terror”. The US government was a relatively early advocate of a strict definition of cyber terrorism, as
nearly a decade ago they were calling it as “a criminal act perpetrated through computers resulting in violence, death and/or
destruction, and creating terror for the purpose of coercing a government to change its policies.” Not defacing a webpage, not
flooding a website (even of the South Korean president) and not stealing credit card information. Some
terrorists groups
may talk about waging an e-Jihad, but such talk remains, for now, aspirational blather. For decades, the
rule of thumb for intelligence analysts has been that adversaries with motives for damaging cyber attacks do not
have the capabilities, while those with the capabilities do not yet have the motives. A large-scale
cyber attacks is more difficult than is generally believed and few adversaries have both the motive and capability.
Additionally, terrorist groups have many disincentives for pursing cyber capabilities. For example, their
leadership tends to be conservative and they tend to stick with what they know will work – suicide bombers,
road-side bombs, and kinetic assaults. These actually kill and terrorize people which, as yet, no cyber attack has
accomplished. The C ongressional R esearch S ervice summed this up as “lower risk, but less drama.”

Zero risk of cyberterror and no impact – small-scale attacks, zero incidents, no al-Qaeda
capabilities, and cybersecurity solves

Bambauer ‘11 – fellow at the Berkman Center for Internet & Society at Harvard Law School and a researcher for the OpenNet
Initiative (Derek, 06/08, “Cybersecurity Theory and Myths,” http://blogs.law.harvard.edu/infolaw/2011/06/08/cybersecurity-theory-
and-myths/)

David Opderbeck put together a terrific cybersecurity conference at Seton Hall today. I was on a panel discussing cybersecurity
policy and legal theory. The audience was primarily law enforcement and practicing attorneys, so I asked, “What are you doing
here?” In good academic fashion, I proceeded to (try to) answer my own question – why is a theory of cybersecurity useful?
Currently, “cybersecurity” is a term that utterly lacks coherence. It encompasses threats including malware,
identity theft, hacking, intellectual property infringement, denial of service attacks, espionage, and
acts of war by nation-states. It tries to address risks to end users, administrators, ISPs, utility companies, financial institutions,
defense contractors, and the government. In short, we’re unhelpfully subsuming a congeries of technical and legal policy concerns
under a single rubric. They don’t fit. This
is a significant reason for the disparity between apocalyptic rhetoric
about cyber-threats (reminiscent of doomsday predictions about the Y2K bug) and the admitted lack of meaningful progress
on cybersecurity in the last decade. Our current conception of cybersecurity hinders us in prioritizing among these issues and
then focusing on the most relevant threats. This is the role of theory: it provides an organizing framework to rank competing
concerns, to measure progress in addressing them, and to make and justify the inevitable trade-offs that occur. Theory helps guide
policymakers to the most pressing problems, and helps us assess how they’re doing at resolving those issues. My suggestion for a
cybersecurity theory is to focus on information – in particular, on access to, alteration of, and integrity of information. I spell this out
in Conundrum, which is forthcoming in the Minnesota Law Review, and which I’ve blogged about before. We need a new theory of
cybersecurity, and I think this one could be helpful in practical ways. The conference also reminded me of how new this field is, and
the degree to which multiple professions and disciplines are struggling with its challenges. Sometimes this is frustrating, as one
keeps hearing the same tropes raised again and again. So, I present you with today’s Top Four Cybersecurity Myths: Cyber
terrorism: it does not exist. There are no – repeat, zero – documented incidents of cyber-terrorism.
The idea that al-Qaeda will use virtual reality technology to train terrorists here in America (which I heard today)
is simply ludicrous. Bin Laden didn’t even use e-mail! And anyone who’s tried streaming Hulu over a wireless connection will
appreciate just how hard it is to use high-bandwidth apps even in a broadband environment. Supply chain exploits: again, there’s
simply no evidence that China, or anyone else for that matter, is planting covert code in computer chips or laptops. This
is
conspiracy theory stuff. Cybersecurity can be solved: several speakers today talked of getting to the point where we “solve”
cybersecurity. Given our success at solving IP infringement, identity theft, and even physical security, I think it’s safe to say that we
will at best manage cybersecurity. It’s
like the flu: we can turn it into a nuisance with occasional bad
outbreaks. “No one cares more about civil liberties than Cyber Command and the National Security Agency.” Yes. The NSA would
never illegally eavesdrop on U.S. telephone calls and e-mails, in violation of the Foreign Intelligence Surveillance Act and the Fourth
Amendment. (I stopped listening to Mark Young, who works for Cyber Command, after this gem.)

No economic impact
Knake ’10 (Robert K. Knake, an internationally-recognized expert on security, including homeland security, national security,
cyber security, and counterterrorism. He is currently an on-air consultant for ABC News and teaches at Harvard's Kennedy School of
Government. Clarke served the last three Presidents as a senior White House Advisor. Over the course of an unprecedented 11
consecutive years of White House service, he held the titles of: - Special Assistant to the President for Global Affairs - National
Coordinator for Security and Counterterrorism - Special Advisor to the President for Cyber Security Prior to his White House years,
Clarke served for 19 years in the Pentagon, the Intelligence Community, and State Department. During the Reagan Administration,
he was Deputy Assistant Secretary of State for Intelligence. During the Bush (41) Administration, he was Assistant Secretary of State
for Political-Military Affairs and coordinated diplomatic efforts to support the 1990–1991 Gulf War and the subsequent security
arrangements. As a Partner in Good Harbor Consulting, LLC, Clarke advises clients on a range of issues including: - Corporate
security risk management - Information security technology - Dealing with the Federal Government on security and IT issues -
Counterterrorism In a Special Report by Foreign Policy Magazine, Clarke was chosen as one of The Top 100 Global Thinkers of 2010,
Council on Foreign Relations, Former international affairs fellow in residence, “Cyberterrorism Hype v. Fact”,
http://www.cfr.org/terrorism-and-technology/cyberterrorism-hype-v-fact/p21434 , February 16,
2010, LEQ)

For less than $500,000 and using box cutters as the primary weapon, al-Qaeda was able to create a military response that to
date has cost between $1 trillion to $2.5 trillion. What kind of results could al-Qaeda get from hacking? If
al-Qaeda were
able to cause a power blackout by hacking SCADA systems, they couldn't do much better than
the tree limbs that caused the 2003 Northeast Blackout. That event put 50 million people in
the United States and Canada in the dark for up to four days. Economists place the cost of that event
between $4.5 and $10 billion, a blip in the $14.2 trillion economy. One thing the United States has learned
about the cost of disruption to the economy is that disruption causes pain that is short lived and
minimal. A two-day snow storm doesn't eliminate two days of economic activity, it only
delays it. The same holds true for port closures and other disruptive activities.

It’s media hype

Conway, 11 – Lecturer in International Security in the School of Law and Government at Dublin City University [February,
Maura, “Against Cyberterrorism: Why cyber-based terrorist attacks are unlikely to occur”, Communications of the ACM, Volume 54,
Number 2, p. 26-28, AL]

Given the high cost—not just in terms of money, but also time, commitment, and effort— and
the high possibility of failure on the basis of manpower issues, timing, and complexity of a
potential cyberterrorist attack, the costs appear to me to still very largely outweigh the
potential publicity benefits. The publicity aspect is crucial for potential perpetrators of terrorism
and so the possibility that an attack may be apprehended or portrayed as an accident , which
would be highly likely with regard to cyberterrorism, is detrimental. Add the lack of spectacular
moving images and it is my belief that cyberterrorism, regardless of what you may read in
newspapers, see on television, or obtain via other media sources, is not in our near future. So
why then the persistent treatment of cyberterrorism on the part of journalists? Well, in this instance,
science fiction-type fears appear to trump rational calculation almost every time. And I haven’t
even begun to discuss how the media discourse has clearly influenced the pronouncements of
policymakers.
---Terrorism- Nuclear Reactor

-- Nuclear reactor terrorism unlikely – no capability or motive

Ferguson and Potter 4 (Charles, Science-in-Residence – Monterey Institute of International Studies, and William, Professor
and Director of the Center for Nonproliferation Studies – Monterey Institute of International Studies, The Four Faces of Nuclear
Terrorism, p. 192-193)

Despite these benefits to the attackers, causing a significant radioactive release from a nuclear
installation would be a daunting challenge , requiring considerable technical, organizational,
and financial resources. Technical skills would be needed to identify relevant buildings and
equipment within what are typically large and complex industrial installations; to identify and implement
the actions needed to cause a radioactive release; and to defeat all backup safety systems.
Organizational requirements would also be very substantial. A ground assault on a nuclear facility
would require a sizeable number of assailants, probably divided into teams, a cadre roughly comparable to the 19-man
group that executed the 9/11 attacks. Since all U.S. nuclear reactor facilities, except research reactors, are protected
by armed guard forces , the assaulting group also would need military-style training to mount a successful attack.
Appropriate plan personnel would have to be identified and strategies devised and implemented to gain
insider support through ideological indoctrination , bribery, or coercion. Aerial attacks on nuclear facilities
would require equally sophisticated planning. If a group of terrorists were to succeed in gaining
control of an aircraft, they would also have to be capable of precisely targeting vital plant safety
systems, such as the reactor's containment structure, or the spent fuel pools in order to generate substantial off-site release of
radioactivity. Significant financial resources would be needed to meet the foregoing technical and organizational
requirements. However, the group would not necessarily require the multinational capabilities necessary for nuclear weapon and
IND plots involving the transportation of a nuclear weapon or fissile material from locations abroad to the United States. A
relatively small number of terrorist organizations are likely to possess the motivations and
capabilities to mount an attack on a nuclear facility. The 9/11 attacks are a strong reminder, however, that
these abilities could be within the grasp of a well-organized and well-trained terrorist group.

-- Containment structures solve

Ferguson and Potter 4 (Charles, Science-in-Residence – Monterey Institute of International Studies, and William, Professor
and Director of the Center for Nonproliferation Studies – Monterey Institute of International Studies, The Four Faces of Nuclear
Terrorism, p. 235-236)

Despite fears that most nuclear accidents or even terrorist attacks would result in destruction and harm
comparable to the Chernobyl accident, that accident was exceptional. One reason for the extremely severe
consequence was the lack of a containment structure . As discussed earlier, all U.S. commercial nuclear power
plants use containment structures that would very likely prevent the release of substantial
amounts of radioactivity to the environment during an accident or attack, just as the containment protected the
public during the 1979 Three Mile Island accident discussed above. Furthermore, U.S. nuclear power
plants employ inherently safer designs than that of the Chernobyl plant. Therefore, it is highly
improbable that the consequences of a terrorist attack on a U.S. nuclear power plant would
approach that of the Chernobyl accident.

-- Safety systems check

Ferguson and Potter 4 (Charles, Science-in-Residence – Monterey Institute of International Studies, and William, Professor
and Director of the Center for Nonproliferation Studies – Monterey Institute of International Studies, The Four Faces of Nuclear
Terrorism, p. 210-211)

Nuclear power plant operators have accumulated a vast amount of safety experience through
nuclear accident prevention training and planning. In the past several decades, nuclear power has grown into a mature
industry. As part of this maturation process, the industry carefully examined the 1979 TMI accident, which
resulted in negligible off-site release of radioactivity (as mentioned previously) and the 1986 Chernobyl accident, which
resulted in a massive off-site release of radioactivity mainly because the reactor unit was not enclosed inside a containment
structure. These
accidents provided valuable lessons about nuclear safety. Through study of actual plant
operations and
numerous computer simulations of plant performance, engineers can predict the
likelihood that a particular power plant component would fail due to malfunction. Such failures can occur due to
normal wear and tear as well as operator error. To protect against failures arising from equipment malfunction, plant personnel
perform preventive maintenance, and to defend against human error, they train thoroughly and frequently. Moreover, the
nuclear industry generally has come to embrace a safety culture mentality , which strives to keep safety a
high priority. In contrast to a nuclear accident in which one or perhaps two plant component failures initiate an accident, a terrorist
attack could target numerous plant components, thereby potentially damaging more than one vital plan system in a short time
period. Thus, defending against a terrorist attack might be more demanding than preventing nuclear accidents. However,
because hitting multiple targets simultaneously would challenge terrorist capabilities and
because redundant safety systems could further obstruct successful terrorist attacks, most nuclear
power plants would likely be resilient to terrorist attack or sabotage.
---Terrorism- Agro

Successful agroterror is impossible and doesn’t have a spillover

Stratfor 8 (Global Forecasting, “Placing the Terrorist Threat to the Food Supply in Perspective”, April 22,
http://www.stratfor.com/weekly/terrorism_weekly_april_22)

While attacks against the food supply may appear simple in theory, they have occurred infrequently
and for good reason: When one considers the sheer size of the U.S. agricultural sector, conducting a
productive assault proves difficult. As seen by the coca and marijuana eradication efforts by the United States and
its partners in Mexico, Central America and the Andes, the logistical effort needed to make any substantial
dent in agricultural production is massive. Even the vast resources the United States has dedicated to drug
eradication tasks in small countries –- overt plane flights spraying untold thousands of gallons of herbicides for decades — have
failed to create more than a limited effect on marijuana and coca crops. Obviously, any sort of meaningful chemical attack on
U.S. agriculture would have to be so massive that it is simply not logistically feasible. This is where pathogens — agents that can,
at least in theory, be introduced in limited amounts, reproduce and then rapidly spread to infect a far larger area — enter the
picture. In order to be effective, however, a pathogen must be one that is easily spread and very deadly and
has a long incubation period (in order to ensure it is passed along before the host dies). It is also very helpful to the propagation
of a disease if it is difficult to detect and/or difficult to treat. While a pathogen that possesses all of the aforementioned traits
could be devastating, finding such an agent is difficult. Few diseases have all the requisite
characteristics. Some are very deadly, but act too quickly to be passed, while others are more readily passed but do not
have a long incubation period or are not as virulent. Other pathogens, such as the Ug99 wheat fungus, are easy to detect and
kill. There is also the problem of mutation, meaning that many pathogens tend to mutate into less
virulent actors. It is also important to note that genetically engineering a super bug — one that possess all
the characteristics to make it highly effective — is still much harder in real life than it is on television. Even
if such an effective pathogen is found, someone intending to use it in an attack must isolate the virulent strain,
manufacture it in sufficient quantities to be effective, ship it to the place of the planned attack and then distribute it in a manner
whereby it is effectively dispersed. The infrastructure required to undertake such an endeavor is both large
and expensive. Even in past cases where groups possessed the vast monetary resources to fund biological weapons efforts
and amassed the scientific expertise to attempt such a program — Aum Shinrikyo comes to mind — virulent pathogens have
proven very difficult to produce and effectively disperse in large quantities. Another factor making these sorts of attacks
difficult to orchestrate is the very nature of farming . For thousands of years, farmers have been battling
plant and animal diseases. Most of the pathogens that are mentioned in connection with attacks against agriculture
include elements already existing in nature such as hoof-and-mouth disease , H5N1 bird flu or a
fungus like Ug99. As a result, farmers and governmental organizations such as the Animal and Plant Health
Inspection Service have systems in place to monitor crops and animals for signs of
pathogens. When these pathogens appear, action is taken and diseased crops are treated or eradicated.
Animals are treated or culled. Even in past cases where massive eradication and culling efforts occurred — BSE in the United
Kingdom, citrus canker in Florida or the many bird flu outbreaks over the past few years –- the
measures have not
crippled or affected the country’s agricultural sector or the larger economy .

No motivation
Stratfor 8 (Global Forecasting, “Placing the Terrorist Threat to the Food Supply in Perspective”, April 22,
http://www.stratfor.com/weekly/terrorism_weekly_april_22)

Such attacks could potentially be conducted by a wide array of actors, ranging from a single mentally disturbed individual on
one end of the spectrum to sovereign nations on the other end. Cults and domestic or transnational terrorist groups fall
somewhere in the middle. The motivation behind these diverse actors could range from monetary extortion or attempts to
commit mass murder to acts of war designed to cripple the U.S. economy or the nation’s ability to project power. Of these
actors, however, there are very few who possess the ability to conduct attacks that could have a
substantial impact on the U.S. food supply. In fact, most of the actors are only capable of
contaminating finished food products. While they all have this rudimentary capability, there is also the question
of intent. Documents and manuals found in Afghanistan after the 2001 U.S.-led invasion revealed an al
Qaeda interest in conducting chemical and biological attacks, although this interest was not a well-
developed program. From a cost-benefit standpoint, it would be much cheaper and easier to use explosives to create
disruption than it would be to execute a complicated plot against the food supply. Besides, such a target would not produce the
type of spectacular imagery the group enjoys. While we do not foresee any huge attempt by the Russians or Chinese, and food
supply is not a part of al Qaeda’s preferred target set, it is possible that a lone wolf or a smaller extremist
organization could attempt to conduct such an attack. While any such offensive will likely have limited success, it could have far
wider societal repercussions. At the present time, the public has become somewhat accustomed to food scares and recalls over
things such as contaminated spinach, ground beef and green onions. Even warnings over lead and other harmful chemicals in
food imported from China have caused concern. However, if even a relatively unsuccessful attack on the food supply were
conducted by a terrorist group, it could create significant hysteria — especially if the media sensationalized the event. In such a
case, even an ineffective terror plot could result in a tremendous amount of panic and economic loss.

No retaliation

Chalk 1 (Peter, Senior Policy Analyst – RAND Corporation, “The US Agricultural Sector: A New Target For Terrorism?”, 2-9,
http://www.janes.com/security/international_security/news/jir/jir010209_1_n.shtml)

Quite apart from their relative ease, attacks against agriculture are comparatively risk free in the sense that they neither cross
the threshold of mass destruction, nor, in most cases, do they represent a direct threat to those carrying them out.
Destroying pig or cattle production would be unlikely to attract the same response as a
more 'conventional' bioattack against a heavily populated centre such as Los Angeles or
San Francisco. Equally, because there is no large-scale loss of human life , perpetrators are unlikely to
be affected by residual feelings of moral guilt or, indeed, substantially weakened by reduced popular support - both potential
Also, biological attacks against livestock can be carried out in
costs of civilian-oriented operations.
such a way that they imitate natural or common disease occurrences. This complicates
accurate epidemiological investigation and greatly reduces risks to the perpetrators of possible
detection.

Tech barriers check

Kohnen 2k (Anne Kohnen. “Responding to the Threat of Agroterrorism: Specific


Recommendations for the United States Department of Agriculture.” BCSIA Discussion Paper 2000-29,

ESDP Discussion Paper ESDP-2000-04, John F. Kennedy School of Government, Harvard University,
October 2000)

Crop diseases, in general, do not travel airborne as fast or as far as animal diseases such as FMD. Plant
pathogens are also highly sensitive to environmental factors such as temperature,
humidity, and sunlight; even if a pathogen were released, it would not necessarily cause
disease. Thus it would be technically very difficult to produce an agent that was
guaranteed to cause disease. Spores would have to be protected from ultraviolet light, and the agent would
have to be formulated to prevent clumping, allowing for airborne dispersal.49 Nevertheless, mere exposure to a pathogen is, in
some cases, grounds for export restrictions, so the technical barriers to deliberately spreading crop diseases are not necessarily
relevant.
Terrorists won’t target agriculture - doesn’t trigger a media response

Chalk 4 (Peter, Senior Policy Analyst – RAND Corporation, Hitting America’s Soft Underbelly: The Potential Threat of Deliberate
Biological Attacks Against the US Agricultural and Food Industries, RAND, p. 28)

Despite the ease with which an act of agroterrorism could be carried out and the serious impact of a
successful assault (especially the economic and political fallout), it is unlikely to constitute a primary form of
terrorist aggression. Indeed, agroterrorism would probably be viewed as being too "mundane"
in comparison with traditional terrorist tactics (which focus on more-spectacular, human-directed atrocities) because it does
not produce immediate, visible effects. The impact of bio-assaults on livestock and the food chain,
although significant, is delayed and lacks a single focal point for media attention. More
specifically, there is no drama of the sort that results from a suicide bombing or a September l1
style attack, which is absolutely essential to creating the hostility and panic that such acts are
designed to elicit." In this light, it is perhaps understandable why biological attacks against agriculture
have not been more of a problem. In fact, since 1912 there have been only 12 documented cases
involving the substate use of pathogenic agents to infect livestock or contaminate food produce. Of those 12
incidents, only two could in any way be termed terrorist in nature : the 1984 Rajneeshee salmonella food
poisoning in Oregon and the 1952 Mau Mau plant toxin incident in Kenya (see Table 3.2).

Agroterror is impossible – firebreaks within the ag industry prevent damage

Pate and Cameron ‘1 (Jason Pate, senior research associate at the Monterey Institute, and Gavin Cameron, post
doctoral fellow at the Belfer Center for Science Affairs, August, 2001, Covert Biological Weapons Attacks Against Agricultural
Targets, BCSIA Discussion Paper

It would be extremely difficult for a terrorist group to perpetrate a significant biological attack
against the agricultural economy in the United States, however, for several reasons. First, obtaining and
effectively delivering a biological agent against an agricultural target is a task fraught with technical
hurdles. Although some agricultural agents can be obtained relatively easily and crudely delivered, to cause a
catastrophic incident would require a more sophisticated approach. Second, because crops and
livestock in the United States are generally not concentrated, eliminating a segment of the agricultural
economy would require a multipronged attack and a sophisticated understanding of the economy. Although not
impossible, this type of attack presents significant obstacles. Third, the U.S. agricultural economy has in place
networks and plans to respond to an attack once detected , and surveillance of crop and animal disease in
the United States is extraordinarily sophisticated. Even if a terrorist group managed to deliver a biological agent
effectively against a target, the effects of the attack would likely be severely limited by the U.S.
response. Fourth, although a determined group could conceivably carry out a devastating attack, there is no evidence
of terrorist groups with the motivation to carry out a catastrophic attack against U.S.
agriculture. It is clear however that more research is required before an accurate assessment can be made of the threat
terrorism poses to the U.S. agricultural economy.
---Terrorism- Chemical Weapons

No impact to chemical terrorism.

Mueller ‘5, John -Professor of Political Science at Ohio State, May (International Studies Perspectives, Volume 6 Issue 2 Page
208-234)

Biologist Matthew Meselson calculates that it would take a ton of nerve gas or 5 tons of mustard gas to
produce heavy casualties among unprotected people in an open area of 1 km2. Even for nerve gas this would take the concentrated
delivery into a rather small area of about 300 heavy artillery shells or seven 500-lb bombs (1991:13). And, this would usually
require a considerable amount of time, allowing many people to evacuate the targeted area
a ton of Sarin
(McNaugher, 1990:31). A 1993 analysis by the Office of Technology Assessment of the U.S. Congress finds that
nerve gas perfectly delivered under absolutely ideal conditions over a heavily populated area
against unprotected people could cause between 3,000 and 8,000 deaths. Under slightly less
ideal circumstances—if there was a moderate wind or if the sun was out, for example— the death rate would be
only 1/10th as great.21 Nuclear weapons are considered weapons of mass destruction because a single bomb can generate
great devastation. For chemical weapons to cause extensive damage, by contrast, many of them must be used, just like conventional
ones.

Terrorists will kill themselves while producing chemical weapons.


Stimson Center ‘5 (Henry L. Stimson Center, Frequently Asked Questions: Likelihood of Terrorists Acquiring and Using
Chemical or Biological Weapons, http://www.stimson.org/cwc/terror.htm)

However, two factors stand in the way of manufacturing chemical agents for the purpose of mass
casualty. First, the chemical reactions involved with the production of agents are dangerous :
precursor chemicals can be volatile and corrosive, and minor misjudgments or mistakes in processing could
easily result in the deaths of would-be weaponeers . Second, this danger grows when the
amount of agent that would be needed to successfully mount a mass casualty attack is
considered. Attempting to make sufficient quantities would require either a large, well-financed operation that would increase
the likelihood of discovery or, alternatively, a long, drawn-out process of making small amounts incrementally. These small
It
quantities would then need to be stored safely in a manner that would not weaken the agent's toxicity before being released.
would take 18 years for a basement-sized operation to produce the more than two tons of sarin
gas that the Pentagon estimates would be necessary to kill 10,000 people , assuming the sarin was
manufactured correctly at its top lethality.

Safeguards solve the impact, and technical difficulties prevent mass casualties.

Stimson Center ‘5 (Henry L. Stimson Center, Frequently Asked Questions: Likelihood of Terrorists Acquiring and Using
Chemical or Biological Weapons, http://www.stimson.org/cwc/terror.htm)
The options for delivering poison gas range from high to low tech. Theoretically, super toxic chemicals could be
employed to foul food or water supplies , put into munitions, or distributed by an aerosol or spray method.
Because of safeguards on both our food and water supplies as well as the difficulty of covertly disbursing sufficient
quantities of agent, this method is unlikely to be an effective means to achieving terrorist aims.
Chemical agents could also be the payload of any number of specially designed or modified conventional
munitions, from bombs and grenades to artillery shells and mines. However designing munitions that reliably
produce vapor and liquid droplets requires a certain amount of engineering skill . Finally,
commercial sprayers could be mounted on planes or other vehicles . In an outdoor attack such as this,
however, 90 percent of the agent is likely to dissipate before ever reaching its target . Effective
delivery, which entails getting the right concentration of agent and maintaining it long enough for inhalation to occur, is quite
difficult to achieve because chemical agents are highly susceptible to weather conditions .

No impact to chemical terrorism- it’s all hype.


Mueller ‘99 (John- Professor of Political Science at the University of Rochester, Karl- Assistant Professor of Comparative
Military Studies at the School of Advanced Airpower Studies, Foreign Affairs, May/June 1999)

Chemical weapons, meanwhile, are virtually incapable of killing masses of people in open areas
except when used in vast quantities, and so their inclusion in the WMD category is highly dubious unless the
concept is so diluted that bullets or machetes could be included as well. As with terrorism, the problem here is
primarily fear rather than actual consequences. Matthew Meselson, a biologist at Harvard University, calculates
that it would take a ton of nerve gas or five tons of mustard gas to produce heavy casualties among unprotected people in an open
area one kilometer square. Even for nerve gas this would require the concentrated delivery into a rather small area of about 300
heavy artillery shells or seven 500-pounds bombs. A 1993 analysis by Congress' Office of Technology Assessment concluded that a
ton of sarin perfectly delivered under absolutely ideal conditions over a heavily populated area against unprotected people might
cause between 3,000 and 8,000 deaths. Under slightly less ideal circumstances -- if there were a moderate wind or if the sun were
out, for example -- the death rate would be a tenth as great .
Discussions of chemical weapons often stress their
ability to cause many casualties -- both dead and wounded -- glossing over the fact that
historically most of those incapacitated by such weapons have not actually died.
---Terrorism- Bioterrorism

No threatening bioterror programs and current defenses solve

Orent 9 [Wendy, Ph.D. in anthropology from the University of Michigan, leading freelance science writer, and author of Plague:
The Mysterious Past and Terrifying Future of the World's Most Dangerous Disease, "America's Bioterror Bugaboo." Los Angeles
Times (Los Angeles, CA) 17 Jul 2009: A.29. SIRS Researcher. Web. 29 January 2010]

After the anthrax letter attacks of October 2001, the Bush administration pledged $57 billion to
keep the nation safe from bioterror. Since then, the government has created a vast network of
laboratories and institutions to track down and block every remotely conceivable form of
bioterror threat. The Obama administration seems committed to continuing the biodefense
push, having just appointed a zealous bioterror researcher as undersecretary of science and technology in the Department of
Homeland Security. But is the threat really as great as we've been led to believe? Last summer, the FBI concluded that the anthrax
letters that killed five Americans came not from abroad but from an American laboratory, the United States Army Medical Research
Russian bioweapons program was officially shut down in
Institute of Infectious Diseases. Meanwhile, the
1992, and it's unlikely that anything remaining of it could pose much of a threat. Iraq, it has
turned out, had no active program. And Al Qaeda's rudimentary explorations were interrupted,
according to an Army War College report, by the U.S. invasion of Afghanistan.

No impact to bioterror- laundry list of reasons

Fettweis 10 [Christopher J., fifth year doctoral student in the University of Maryland's Department of Government and Politics.
His primary interests include US foreign and national security policies. His dissertation, currently titled The Geopolitics of Energy and
the Obsolescence of Major War, focuses on the relationship between oil and conflict. Mr. Fettweis has a BA in History from the
University of Notre Dame, Threat and Anxiety in US Foreign Policy, April 2010 Survival, 52:2, 59 - 82]

Even terrorists equipped with nuclear, biological or chemical weapons would be incapable of
causing damage so cataclysmic that it would prove fatal to modern states. Though the prospect
of terrorists obtaining and using such weapons is one of the most consistently terrifying scenarios of
the new era, it is also highly unlikely and not nearly as dangerous as sometimes portrayed. As
the well-funded, well-staffed Aum Shinrikyo cult found out in the 1990s, workable forms of
weapons of mass destruction are hard to purchase, harder still to synthesise without state help,
and challenging to use effectively. The Japanese group managed to kill a dozen people on the
Tokyo subway system at rush hour. While tragic, the attack was hardly the stuff of apocalyptic
nightmares. Super-weapons are simply not easy for even the most sophisticated non-state
actors to use.31 If terrorists were able to overcome the substantial obstacles and use the most
destructive weapons in a densely populated area, the outcome would of course be terrible for
those unfortunate enough to be nearby. But we should not operate under the illusion that
doomsday would arrive. Modern industrialised countries can cope with disasters, both natural
and man-made. As unpleasant as such events would be, they do not represent existential
threats.
No risk of a bioterror attack, and there won’t be retaliation - your evidence is hype

Matishak 10 (Martin, Global Security Newswire, “U.S. Unlikely to Respond to Biological Threat With Nuclear Strike, Experts
Say,” 4-29, http://www.globalsecuritynewswire.org/gsn/nw_20100429_7133.php)

WASHINGTON -- The United States is not likely to use nuclear force to respond to a biological
weapons threat, even though the Obama administration left open that option in its recent update to the nation's nuclear
weapons policy, experts say (See GSN, April 22). "The notion that we are in imminent danger of
confronting a scenario in which hundreds of thousands of people are dying in the streets of New York as a
consequence of a biological weapons attack is fanciful," said Michael Moodie, a consultant who served as
assistant director for multilateral affairs in the U.S. Arms Control and Disarmament Agency during the George H.W. Bush
administration. Scenarios in which the United States suffers mass casualties as a result of such an event
seem "to be taking the discussion out of the realm of reality and into one that is
hypothetical and that has no meaning in the real world where this kind of exchange is just not going to
happen," Moodie said this week in a telephone interview. "There are a lot of threat mongers who talk about
devastating biological attacks that could kill tens of thousands, if not millions of Americans," according to Jonathan
Tucker, a senior fellow with the James Martin Center for Nonproliferation Studies. "But in fact, no country out
there today has anything close to what the Soviet Union had in terms of mass-casualty biological
warfare capability. Advances in biotechnology are unlikely to change that situation, at least for the
foreseeable future." No terrorist group would be capable of pulling off a massive biological
attack, nor would it be deterred by the threat of nuclear retaliation, he added. The biological threat provision was addressed
in the Defense Department-led Nuclear Posture Review, a restructuring of U.S. nuclear strategy, forces and readiness. The
Obama administration pledged in the review that the United States would not conduct nuclear strikes on non-nuclear states
that are in compliance with global nonproliferation regimes. However, the 72-page document contains a caveat that would
allow Washington to set aside that policy, dubbed "negative security assurance," if it appeared that biological weapons had
been made dangerous enough to cause major harm to the United States. "Given the catastrophic potential of biological
weapons and the rapid pace of biotechnology development, the United States reserves the right to make any adjustment in the
assurance that may be warranted by the evolution and proliferation of the biological weapons threat and U.S. capacities to
counter that threat," the posture review report says. The caveat was included in the document because "in theory, biological
weapons could kill millions of people," Gary Samore, senior White House coordinator for WMD counterterrorism and arms
control, said last week after an event at the Carnegie Endowment for International Peace. Asked if the White House had
identified a particular technological threshold that could provoke a nuclear strike, Samore replied: "No, and if we did we
obviously would not be willing to put it out because countries would say, 'Oh, we can go right up to this level and it won't
change policy.'" "It's deliberately ambiguous," he told Global Security Newswire. The document's key qualifications have
become a lightning rod for criticism by Republican lawmakers who argue they eliminate the country's previous policy of
"calculated ambiguity," in which U.S. leaders left open the possibility of executing a nuclear strike in response to virtually any
hostile action against the United States or its allies (see GSN, April 15). Yet experts
say there are a number of
reasons why the United States is not likely to use a nuclear weapon to eliminate a non-
nuclear threat. It could prove difficult for U.S. leaders to come up with a list of appropriate
targets to strike with a nuclear warhead following a biological or chemical event, former Defense Undersecretary for Policy
Walter Slocombe said during a recent panel discussion at the Hudson Institute. "I don't think nuclear weapons are necessary to
deter these kinds of attacks given U.S. dominance in conventional military force," according to Gregory Koblentz, deputy
director of the Biodefense Graduate Program at George Mason University in Northern Virginia. "There's a bigger downside to
the nuclear nonproliferation side of the ledger for threatening to use nuclear weapons in those circumstances than there is the
benefit of actually deterring a chemical or biological attack," Koblentz said during a recent panel discussion at the James Martin
Center. Thenonproliferation benefits for restricting the role of strategic weapons to deterring
nuclear attacks outweigh the "marginal" reduction in the country's ability to stem the use of
biological weapons, he said. In addition, the United States has efforts in place to defend against chemical and biological
attacks such as vaccines and other medical countermeasures, he argued. "We
have ways to mitigate the
consequences of these attacks," Koblentz told the audience. "There's no way to mitigate the effects
of a nuclear weapon." Regardless of the declaratory policy, the U.S. nuclear arsenal will always provide a "residual
deterrent" against mass-casualty biological or chemical attacks, according to Tucker. "If a biological or chemical attack against
the United States was of such a magnitude as to potentially warrant a nuclear response, no attacker could be confident that the
U.S. -- in the heat of the moment -- would not retaliate with nuclear weapons, even if its declaratory policy is not to do so," he
told GSN this week during a telephone interview. Political Benefits Experts are unsure what, if any, political benefit the country
or President Barack Obama's sweeping nuclear nonproliferation agenda will gain from the posture review's biological weapons
caveat. The report's reservation "was an unnecessary dilution of the strengthened negative security and a counterproductive
elevation of biological weapons to the same strategic domain as nuclear weapons," Koblentz told GSN by e-mail this week.
"The United States has nothing to gain by promoting the concept of the biological weapons
as 'the poor man's atomic bomb,'" he added.

Terrorists won’t use bioweapons

Stern 99 (Jessica, Member of the Council on Foreign Relations, “The Prospect of Domestic Bioterrorism,”
http://www.cdc.gov/ncidod/EID/vol5no4/stern.htm CDC Emerging Infectious Diseases--Vol 5 # 4 July)

Would domestic terrorists use biological weapons?1 The conventional wisdom among experts has been that terrorists
"want a lot of people watching, not a lot of people dead" and are unlikely to turn to
weapons of mass destruction.2 A new school of thought proposes that improved technology
has made biological attacks resulting in hundreds of thousands or millions of deaths all but
inevitable. While terrorists are increasingly interested in weapons of mass destruction, proponents of the latter
view exaggerate the threat. Using biological weapons to create mass casualties would require more than having
biological agents in hand. The terrorists would need to disseminate the agent, which presents technical and organizational
obstacles that few domestic groups could surmount. In addition, relatively few
terrorists would want to kill
millions of people, even if they could. For most terrorists, the costs of escalation to
biological weapons would seem to outweigh the benefits. Most modern terrorists have
had substantively rational goals, such as attaining national autonomy or establishing a
government purportedly more representative of the people's will. Escalating to such
frightening weapons would result in a massive government crackdown and could alienate
the group's supporters. Biological weapons are also dangerous to produce. A number of Aum Shinrikyo members
reportedly damaged their own health while working on biological agents. Additionally, some terrorists may perceive moral
constraints.3

Your evidence is massively exaggerated

Leitenberg 5 (Milton, Senior Research Scholar @ University of Maryland, “ASSESSING THE BIOLOGICAL WEAPONS AND
BIOTERRORISM THREAT,” December,

Framing “the threat” and setting the agenda of public perceptions and policy prescriptions. For the past decade the
risk and
immanence of the use of biological agents by nonstate actors/terrorist organizations—“ bioterrorism”—has been
systematically and deliberately exaggerated. It became more so after the combination of the 9/11 events and
the October- November 2001 anthrax distribution in the United States that followed immediately afterwards. U.S. Government
officials worked hard to spread their view to other countries. An
edifice of institutes, programs, conferences,
and publicists has grown up which continue the exaggeration and scare-mongering. In the last year or
two, the drumbeat had picked up. It may however become moderated by the more realistic
assessment of the likelihood of the onset of a natural flu pandemic, and the accompanying realization that the U.S.
Government has been using the overwhelming proportion of its relevant resources to prepare for the
wrong contingency.

No extinction - history proves

Easterbrook 3 (Gregg, Senior Fellow – New Republic, “We’re All Gonna Die!”, Wired Magazine, July,
http://www.wired.com/wired/archive/11.07/doomsday.html?pg=1&topic=&topic_set=)

3. Germ warfare!Like chemical agents, biological weapons have never lived up to their billing in popular
culture. Consider the 1995 medical thriller Outbreak, in which a highly contagious virus takes out entire towns. The reality is
quite different. Weaponized smallpox escaped from a Soviet laboratory in Aralsk, Kazakhstan, in 1971;
three people died, no epidemic followed. In 1979, weapons-grade anthrax got out of a Soviet
facility in Sverdlovsk (now called Ekaterinburg); 68 died, no epidemic. The loss of life was tragic, but no
greater than could have been caused by a single conventional bomb. In 1989, workers at a US
government facility near Washington were accidentally exposed to Ebola virus. They walked around
the community and hung out with family and friends for several days before the mistake was discovered. No
one died. The fact is, evolution has spent millions of years conditioning mammals to resist
germs. Consider the Black Plague. It was the worst known pathogen in history , loose in a Middle
Ages society of poor public health, awful sanitation, and no antibiotics. Yet it didn’t kill off humanity. Most
people who were caught in the epidemic survived. Any superbug introduced into today’s Western
world would encounter top-notch public health, excellent sanitation, and an array of
medicines specifically engineered to kill bioagents. Perhaps one day some aspiring Dr. Evil will invent a bug
that bypasses the immune system. Because it is possible some novel superdisease could be invented, or that existing pathogens
like smallpox could be genetically altered to make them more virulent (two-thirds of those who contract natural smallpox
survive), biological agents are a legitimate concern. They may turn increasingly troublesome as time passes and knowledge of
biotechnology becomes harder to control, allowing individuals or small groups to cook up nasty germs as readily as they can buy
guns today. But no
superplague has ever come close to wiping out humanity before, and it
seems unlikely to happen in the future.

Tech hurdles check

Mueller 6 (John, Chair of National Security Studies – Mershon Center and Professor of Political Science – Ohio State University,
Overblown, p. 24)

Not only has the science about chemical and biological weapons been quite sophisticated for more than a century, but that
science has become massively more developed over that period. Moreover, governments (not just small terrorist groups)
have spent a great deal of money over decades in an effort to make the weapons more effective. Yet, although
there have been great improvements in the lethality, effectiveness, and deployment of conventional and nuclear weapons
during that time, the difficulties of controlling and dispersing chemical and biological substances
seem to have persisted. Perhaps dedicated terrorists will, in time, figure it out. However, the experience in the 1990s
of the Japanese cult Aum Shinrikyo suggests there are great difficulties. The group had some 300 scientists in its
employ and an estimated budget of $1 billion, and it reportedly tried at least nine times over five years to set
off biological weapons by spraying pathogens from trucks and wafting them from rooftops, hoping fancifully to ignite an
apocalyptic war. These efforts failed to create a single fatality ; in fact, nobody even noticed that the attacks
had taken place. It was at that point that the group abandoned its biological efforts in frustration and instead turned to the
biological (and chemical) terrorist
infamous sarin chemical attack.29 As two analysts stress, there have been so few
attacks because they would require overcoming several major technological hurdles. Among them:
gaining access to specialized ingredients, acquiring equipment and know-how to produce and dis-
perse the agents, and creating an organization that can resist infiltration or early detection by law
enforcement." In the meantime, the science with respect to detecting and ably responding to such attacks is likely to grow.
Although acknowledging that things could change in the future, the Gilmore Commission has concluded, "As easy as some argue
effective
that it may be for terrorists to culture anthrax spores or brew up a concoction of deadly nerve gas, the
dissemination or dispersal of these viruses and poisons still presents serious technological hurdles that
greatly inhibit their effective use.

No risk of bioterror – it’s too technically difficult

Washington Post ‘4 (12/30, “Technical Hurdles Separate Terrorists From Biowarfare.” John Mintz, staff writer. Lexis.)

In 2002, a panel of biowarfare experts concluded in a report co-published by the National Defense University
(NDU) that while terrorists could mount some small-scale bioattacks, larger assaults would require
them to overcome many technical hurdles . Some key biotechnologies would be achievable only three to four years
from then, the panel found. "When we sent out the report for review to [hands-on] bench scientists, we got the response, 'What do
you mean we can't do this? We're doing it now,' " said Raymond Zilinskas, a co-author of the report who heads biowarfare studies at
the Center for Nonproliferation Studies, a California think tank. "It shows how fast the field is moving." Those
skeptical of
the prospect of large-scale bioattacks cite the tiny number of biological strikes in recent
decades. Members of the Rajneeshee cult sickened 750 people in 1984 when they contaminated salad bars in 10 Oregon
restaurants with salmonella. Among the few others were the 2001 anthrax attacks through the U.S. mail that killed five people.
One reason for the small number of attacks is that nearly every aspect of a bioterrorist's job is
difficult. The best chance of acquiring the anthrax bacterium, Bacillus anthracis, is either from
commercial culture collections in countries with lax security controls, or by digging in soil where
livestock recently died of the disease -- a tactic Aum Shinrikyo tried unsuccessfully in the Australian Outback. Once
virulent stocks of anthrax have been cultured, it is no trivial task to propagate pathogens with
the required attributes for an aerosolized weapon: the hardiness to survive in an enclosed
container and upon release into the atmosphere, the ability to lodge in the lungs, and the
toxicity to kill. The particles' size is crucial: If they are too big, they fall to the ground, and if they
are too small, they are exhaled from the body. If they are improperly made, static electricity can
cause them to clump. Making a bug that defeats antibiotics, a desired goal for any bioweaponeer, is relatively
simple but can require laborious trial and error, because conferring antibiotic resistance often
reduces a bioweapon's killing power . Field-testing germ weapons is necessary even for experienced weapons makers,
and that is likely to require open spaces where animals or even people can be experimentally infected. Each bioagent
demands specific weather conditions and requires unforgiving specifications for the spraying
device employed. "Dry" anthrax is harder to make -- it requires special equipment, and scientists
must perform the dangerous job of milling particles to the right size. "Wet" anthrax is easier to
produce but not as easily dispersed. Experts agree that anthrax is the potential mass-casualty agent
most accessible to terrorists. The anthrax letter sent in 2001 to then-Senate Minority Leader Thomas A. Daschle (D-S.D.)
contained one gram of anthrax, or 1 trillion spores.

Weather blocks and solves death toll


Laquer 99 (Walter, Cochair of the International Research Council at The Center for Strategic and International Studies, “The New
Terrorism”)

Ironically, the major factor retarding the use of gases and germs by states and terrorists is no the revulsion or moral constraints
but technical difficulties. “Ideal”
conditions for an attack seldom if ever exist, and the possibility of
things going wrong is almost unlimited, aerosols may nor function, the wind may blow in
the wrong direction, missiles carrying a deadly load may land in the wrong place or
neutralize the germs on impact. In the course of time these technical difficulties may be overcome, but it is still
very likely that roughly nine out of ten of the early attempts by terrorists to wage chemical
or biological warfare will fail. But they will not pass unnoticed; the authorities and the public will be alerted, and
the element of surprise lost. The search for perpetrators may begin even before the first
successful attack. And what has just been said with regard to terrorists may also be to state terrorism.

Even if terrorists have bioweapons, there’s no way they can disperse them

Smithson ‘5 (Amy E., PhD, project director for biological weapons at the Henry L. Stimson Center. “Likelihood of Terrorists
Acquiring and Using Chemical or Biological Weapons”. http://www.stimson.org/cbw/?SN=CB2001121259]

Terrorists cannot count on just filling the delivery system with agent, pointing the device, and
flipping the switch to activate it. Facets that must be deciphered include the concentration of
agent in the delivery system, the ways in which the delivery system degrades the potency of the agent ,
and the right dosage to incapacitate or kill human or animal targets. For open-air delivery, the meteorological
conditions must be taken into account. Biological agents have extreme sensitivity to sunlight,
humidity, pollutants in the atmosphere, temperature, and even exposure to oxygen, all of which
can kill the microbes. Biological agents can be dispersed in either dry or wet forms. Using a dry agent can boost effectiveness
because drying and milling the agent can make the particles very fine, a key factor since particles must range between 1 to 10 ten
microns, ideally to 1 to 5, to be breathed into the lungs. Drying
an agent, however, is done through a complex and
challenging process that requires a sophistication of equipment and know-how that terrorist
organizations are unlikely to possess. The alternative is to develop a wet slurry , which is much easier to
produce but a great deal harder to disperse effectively . Wet slurries can clog sprayers and undergo mechanical
stresses that can kill 95 percent or more of the microorganisms.

Multiple technical barriers prevent bioterrorism.

Non-Proliferation Review ‘4, Fall-Winter (http://www.biosecurity.sandia.gov/subpages/papersBriefings/2004/gaud3.pdf )

In particular, obtaining
a pathogen or toxin does not ensure production of a biological weapon that will
produce a high-consequence event. First of all, if
an a virulent strain of a pathogen is chosen, no significant
outbreak of disease will occur.81 Even after the right strain is selected, additional challenges face the
bioterrorist, including isolation, amplification, protection against environmental degradation,
and development of an effective dissemination method. Most pathogens and toxins are not dermally active.
Therefore, to cause incapacitation or death, these agents must enter a susceptible host either through ingestion or inhalation. Since
most pathogens and toxins would not survive human digestive processes, aerosolization of agents for inhalation has been
acknowledged to be the most effective method for achieving a mass-casualty biological attack.82 Thus, the perpetrator
needs to master the skills to optimize particle size and to decrease degradation from
environmental stressors.83 The perpetrator must then be able to select and use an appropriate delivery system. In general,
these steps require moderate to high levels of scientific expertise and moderate to high levels of financial
resources.84 Thus, well-developed states are judged to have the technical sophistication and financial resources to cause a high-
consequence biological weapon event. Less-developed states, which include nations currently identified as rogue states,
generally do not have the technical sophistication and financial resources that well-developed states have
and are assessed to be capable of causing BW events with moderate consequences. Bioterrorists, who generally lack the technical
skills and financial resources of states, are currently considered capable of perpetrating low to moderate consequences with
biological weapons. Even
a low-consequence event requires a considerable level of expertise to execute. In
addition, the objectives of the perpetrators may be more easily realized by other means, such as conventional
weapons. However, certain types of low-consequence events may be more likely than others because they require little
organization, funding, or expertise. Because these acts are generally carried out by one or a few individuals, they are less likely to be
detected or prevented. These “biocrimes” are generally targeted attacks, such as assassinations or murders. Although biocrimes are
omitted from this study’s discussion of nonstate actors because they do not constitute terrorist acts, they are nonetheless worthy of
note. These attacks require a very low level of organization and expertise and, to date, have involved a limited class of agents, such
as botulinum toxin and ricin. However, they may also include nonlethal agents. Because biocrime attacks can be carried out by a
lone actor with a moderate level of technical expertise, they are considered fairly probable. However the consequences would be
minimal, resulting in a single death or temporary illness. Generally, biocrimes cannot inflict mass casualties or the other forms of
damage that designate an event as high consequence. Although biocrimes have a high probability of occurring, by definition
biocrimes are limited to low consequences. From the BW perspective, biocrimes are low risk.

Bioterrorism is impossible and civil defense measures solve.

Mueller ‘5, John - Professor Poli Sci - Ohio State, (International Studies, “Perspectives, Simplicity and Spook”)

Belligerents have eschewed such weapons with good reason: biological weapons are extremely
difficult to deploy and to control. Terrorist groups or rogue states may be able to solve such problems in the future with
advances in technology and knowledge, but the record thus far is unlikely to be very encouraging to them. For example, Japan
reportedly infected wells in Manchuria and bombed several Chinese cities with plague-infested fleas before and during the Second
World War. These ventures may have killed thousands of Chinese, but they apparently also caused thousands of unintended
casualties among Japanese troops and seem to have had little military impact.18 In the 1990s, Aum Shinrikyo, a Japanese cult that
had some 300 scientists in its employ and an estimated budget of $1 billion, reportedly tried at least nine times over 5 years to set
off biological weapons by spraying pathogens from trucks and wafting them from rooftops, hoping fancifully to ignite an apocalyptic
war. These efforts failed to create a single fatality—in fact, nobody even noticed that the attacks had taken place (Broad, 1998;
Rapoport, 1999:57). For the most destructive results, biological weapons need to be dispersed in very low-altitude aerosol clouds:
aerosols do not appreciably settle, and anthrax (which is not easy to spread or catch and is not contagious) would probably have to
be sprayed near nose level (Meselson, 1995; Panofsky, 1998; Terry, 1998). Explosive methods of dispersion may
destroy the organisms. Moreover, except for anthrax spores, long-term storage of lethal organisms in bombs or
warheads is difficult, and, even if refrigerated, most of the organisms have a limited lifetime. The effects of such weapons
can take days or weeks to have full effect, during which time they can be countered with civil
defense measures. And their impact is very difficult to predict and in combat situations may spread
back on the attacker (OTA, 1993:48–49, 62; Broad and Miller, 1998; Easterbrook, 2002).

Even if terrorists have bioweapons, there’s no way they can disperse them

Smithson 05 (Amy E., PhD, project director for biological weapons at the Henry L. Stimson Center. “Likelihood of Terrorists
Acquiring and Using Chemical or Biological Weapons”. http://www.stimson.org/cbw/?SN=CB2001121259]
Terrorists cannot count on just filling the delivery system with agent, pointing the device, and
flipping the switch to activate it. Facets that must be deciphered include the concentration of
agent in the delivery system, the ways in which the delivery system degrades the potency of the agent ,
and the right dosage to incapacitate or kill human or animal targets. For open-air delivery, the meteorological
conditions must be taken into account. Biological agents have extreme sensitivity to sunlight,
humidity, pollutants in the atmosphere, temperature, and even exposure to oxygen, all of which
can kill the microbes. Biological agents can be dispersed in either dry or wet forms. Using a dry agent can boost effectiveness
because drying and milling the agent can make the particles very fine, a key factor since particles must range between 1 to 10 ten
microns, ideally to 1 to 5, to be breathed into the lungs. Drying
an agent, however, is done through a complex and
challenging process that requires a sophistication of equipment and know-how that terrorist
organizations are unlikely to possess. The alternative is to develop a wet slurry , which is much easier to
produce but a great deal harder to disperse effectively . Wet slurries can clog sprayers and undergo mechanical
stresses that can kill 95 percent or more of the microorganisms.
---Terrorism- Economy

Their economy impact is flawed- two reasons.


- based on disproven speculation

- terrorism is already factored into stock prices


Simon ‘6., Ellen AP Business Writer, Sept 4, (“Terrorism Hurts Markets, But How Much?” AP Online, lexis)

How much does terrorism affect stock prices? To find the answer, economists
have parsed market reaction to
events dating back to the 1915 sinking of the Lusitania, academics have studied how attacks on McDonald's restaurants
have affected the company's stock price and researchers in Israel have chronicled what 13 years of suicide bombings did to stocks
there. Their unsurprising general conclusion is that terror
attacks hurt stock prices but some of the details are
unexpected. For instance, after Sept. 11, 2001, the stocks in the Standard & Poor's 500 rebounded to their pre-attack
prices faster than every other major world index except Japan's Nikkei, according to research by Andrew H. Chen
of Southern Methodist University and Thomas F. Siems of the Federal Reserve Bank of Dallas. What none of the research
has answered is the degree to which the fear of another terrorist attack on the U nited States has
been factored into stock prices. David Sowerby, chief market analyst, Loomis, Sayles & Co., estimates that the threat of
terrorism represents a 5 percent "tether" on stocks. How did he get that number? The overhang is "more than zero, less than 10.
Five seems reasonable. If I said 4, I'd be fine-tuning it too much," he said. The price-to-earnings ratio on the S&P 500 is around 17. A
traditional model pegs the price-to-earnings ratio, one of the most popular methods of gauging a stock's value, to Treasury bond
yields. Under that formula, the price-to-earnings ratio should be closer to 20, said David Wyss, chief economist for Standard &
Poor's. What's holding prices back? "Is it terrorist attacks, distrust of earnings, expectations of a slowdown?" he said. "I don't know."
Similarly, "people keep saying there's a war premium in the price of oil," said Robert Streed, portfolio manager of Northern Trust
Select Equity Fund in Chicago. "People keep throwing out numbers, but I can't see any objective way they come up with those
numbers." Those who say some threat of a future attack is already baked into stock prices hasten to add that another attack in the
United States would still send prices lower. "We would likely see a drop in stock prices of significant proportion, but investors
seem to believe there would be a subsequent recovery ," said Lynn Reaser, chief economist at the investment
strategies group of Bank of America.

The economy will bounce back from terrorist attacks.

Becker and Murphy ‘1, - Gary -Nobel Laureate in Economics -Kevin Economics and Sociology at Chicago, (“Prosperity Will
Rise Out of The Ashes,” Wall Street Journal, October 29)

Yet history shows that economies adjust . The effect of the oil price increase fell greatly over time as the U.S. reduced
its dependence on oil. Sectors of the economy that are less energy-dependent grew relative to those that are highly so, and
consumers and producers conserved. As a result of these shifts, and the subsequent decline in oil prices, oil imports accounted for
only about 0.7% of GDP in 1999, vs. 2.8% in 1980. More recent energy shocks have had a much smaller impact on the economy. The
effect of the terrorist threat is likely to follow a similar pattern . Even if the external threat remains fixed over
costs imposed on air
time, our ability to deal with it in an effective and efficient manner will improve, perhaps greatly. The
travelers in terms of long lines and schedule disruptions will be reduced as we find more efficient ways to ensure security, and
as potential travelers move toward video-conferencing and other means of communication. In the absence of further incidents, the
psychological impact of the attacks will also wane. Already, air travel has recovered to about 80% of its pre-attack level,
after falling to less than 50% in the first week after air travel resumed. In justifying airline subsidies, some political leaders pointed to
the disastrous effects on the economy of eliminating air travel. But the relevant question is not what dire consequences would result
from elimination, but what will be the damage from a higher effective price for air travel due to the terrorist threat? Air travel, taken
in its entirety, may be an "indispensable" element of the economy, but marginal adjustments are much less costly. This is one reason
why the federal airline bailout was hasty and excessive. Consumers have made a rational reaction to the uncertainty and ongoing
threats. They cut back on purchases of big-ticket items as they husband resources and maintain flexibility to deal with contingencies.
reluctance to spend
Similarly, businesses cut back on investment until they have a better idea of what is to come. But this
has hardly been universal or long term . In fact, purchases of key staples like food and medicines initially
increased, while consumption that required individuals to go out in public places, like restaurants and theaters, collapsed.
However, publicly consumed goods have already rebounded strongly; attendance on Broadway, for instance, has
returned to close to pre-attack levels. Quantifying the impact of the attacks is instructive, even though estimates are imprecise. The
destroyed World Trade Center was worth $3 billion to $4 billion. The lost assets of the building’s tenants, and the cleanup cost,
might add another $10 billion. Including the damage to surrounding buildings and the Pentagon, the planes lost, and the lost
productive capacity of those killed would raise the total economic loss to somewhere between $25 billion and $60 billion.

Worst case scenario is only a loss of 0.3% of the GDP.

Becker and Murphy ‘1, - Gary -Nobel Laureate in Economics -Kevin Economics and Sociology at Chicago, (“Prosperity Will
Rise Out of The Ashes,” Wall Street Journal, October 29)

To put this in perspective, total physical assets in the U.S. are about $30 trillion, and total productive assets that also include human
capital are on the order of $100 trillion. So even a $60 billion loss is only 0.2% of physical assets and 0.06% of total productive assets.
In contrast, the $114 billion of physical assets destroyed in Kobe was four times as large when compared to the Japanese economy.
The impact of an ongoing threat is harder to quantify. To gain a feel for how large that might be, we use a pessimistic
scenario – namely, that attacks will be attempted each year for the foreseeable future , but that security
measures will reduce the likelihood of success. The direct cost of increased airport security has been estimated at about $4 per
passenger per flight segment. We assume that flight delays and security checks will force travelers to spend an additional half-hour
per flight segment. (However, bad policies could greatly raise waiting times, as when gasoline rationing caused long lines at gas
stations in the 1970s.) If the average passenger values time at $20 per hour, increased security would cost about $10 billion per year.
We further assume that even with enhanced security, terrorists would destroy one plane each year, resulting in up to 100 deaths.
With a generous value of $10 million per life lost, this would add another $1 billion to the annual perceived cost of flying. This gives a
total added cost to the airline industry of about $11 billion per year. Under this pessimistic scenario, the terrorist threat would add
about 11% to the cost of air travel, and impose a cost on the economy of about 0.1% of GDP . Increased security
would also reduce the likelihood of successful attacks on physical assets. But to err again on the high side, suppose the sustained
annual loss from continuing attacks equals $15 billion. That annual loss in assets would reduce net national product by about $15
The
billion for a given capital stock, and the percentage point reduction in the net return to capital would be five basis points.
investment response to the lower return to capital would reduce long-run capital stock by about 0.8%,
resulting in a loss of about 0.2% in long-run GDP . This is similar to our estimate of the direct impact of the costs
imposed on air travel. Adding these two estimates gives a total impact of about 0.3% of GDP .
Trade Answers
Frontline

Trade is strong and resilient

Ikenson, 9 [Daniel, associate director of the Center for Trade Policy Studies at the Cato Institute, “ A Protectionism Fling: Why
Tariff Hikes and Other Trade Barriers Will Be Short-Lived,” March 12, 2009, http://www.cato.org/pub_display.php?pub_id=10651]

Although some governments will dabble in some degree of protectionism, the combination of a
sturdy rules-based system of trade and the economic self interest in being open to participation
in the global economy will limit the risk of a protectionist pandemic. According to recent estimates from
the International Food Policy Research Institute, if all WTO members were to raise all of their applied tariffs
to the maximum bound rates, the average global rate of duty would double and the value of
global trade would decline by 7.7 percent over five years.8 That would be a substantial decline relative to the
5.5 percent annual rate of trade growth experienced this decade.9 But, to put that 7.7 percent decline in historical
perspective, the value of global trade declined by 66 percent between 1929 and 1934, a period
mostly in the wake of Smoot Hawley's passage in 1930.10 So the potential downside today from what
Bergsten calls "legal protectionism" is actually not that "massive," even if all WTO members raised
all of their tariffs to the highest permissible rates . If most developing countries raised their tariffs to their bound
rates, there would be an adverse impact on the countries that raise barriers and on their most important trade partners. But most
developing countries that have room to backslide (i.e., not China) are not major importers, and thus
the impact on global trade flows would not be that significant. OECD countries and China account for the top twothirds of global
import value.11
Backsliding from India, Indonesia, and Argentina (who collectively account for 2.4 percent of
is not going to be the spark that ignites a global trade war. Nevertheless, governments
global imports)
are keenly aware of the events that transpired in the 1930s , and have made various pledges to
avoid protectionist measures in combating the current economic situation. In the United States, after President
Obama publicly registered his concern that the "Buy American" provision in the American
Recovery and Reinvestment Act might be perceived as protectionist or could incite a trade war,
Congress agreed to revise the legislation to stipulate that the Buy American provision "be applied in a manner
consistent with United States obligations under international agreements." In early February, China's vice commerce minister, Jiang
Zengwei, announced that China would not include "Buy China" provisions in its own $586 billion stimulus bill.12 But even more
promising than pledges to avoid trade provocations are actions taken to reduce existing trade
barriers. In an effort to "reduce business operating costs, attract and retain foreign investment, raise business productivity, and
provide consumers a greater variety and better quality of goods and services at competitive prices," the Mexican
government initiated a plan in January to unilaterally reduce tariffs on about 70 percent of the
items on its tariff schedule. Those 8,000 items, comprising 20 different industrial sectors, accounted for about half of all
Mexican import value in 2007. When the final phase of the plan is implemented on January 1, 2013, the average industrial tariff rate
in Mexico will have fallen from 10.4 percent to 4.3 percent.13v And Mexico is not alone. In February, the Brazilian
government suspended tariffs entirely on some capital goods imports and reduced to 2 percent duties on
a wide variety of machinery and other capital equipment, and on communications and information technology products.14 That
decision came on the heels of late-January decision in Brazil to scrap plans for an import licensing program that would have affected
60 percent of the county's imports.15 Meanwhile, on February 27, a new free trade agreement was signed
between Australia, New Zealand, and the 10 member countries of the Association of Southeast Asian
Nations to reduce and ultimately eliminate tariffs on 96 percent of all goods by 2020. While the media and members of
the trade policy community fixate on how various protectionist measures around the world
might foreshadow a plunge into the abyss, there is plenty of evidence that governments remain
interested in removing barriers to trade. Despite the occasional temptation to indulge
discredited policies, there is a growing body of institutional knowledge that when people are free to
engage in commerce with one another as they choose, regardless of the nationality or location of the other parties, they can
leverage that freedom to accomplish economic outcomes far more impressive than when governments attempt to limit choices
through policy constraints.

No trade war impact

Fletcher 11 Ian Fletcher is Senior Economist of the Coalition for a Prosperous America, former Research Fellow at the U.S.
Business and Industry Council M.A. and B.A. from Columbia and U Chicago, "Avoid Trade War? We're Already In One!" August 29
2011 www.huffingtonpost.com/ian-fletcher/avoid-trade-war-were-alre_b_939967.html

trade war is that, unlike actual shooting war, it has no historical precedent . In
The curious thing about the concept of
fact, there has never been a significant trade war, "significant" in the sense of having done serious economic damage.
All history records are minor skirmishes at best.¶ Go ahead. Try and name a trade war. The Great Trade War of
1834? Nope. The Great Trade War of 1921? Nope Again. There isn't one.¶ The standard example free traders give is that
America's Smoot-Hawley tariff of 1930 either caused the Great Depression or made it spread around the world. But this canard
does not survive serious examination, and has actually been denied by almost every economist
who has actually researched the question in depth -- a group ranging from Paul Krugman on the left to Milton Friedman on the
right.¶ The Depression's cause was monetary . The Fed allowed the money supply to balloon during the late 1920s,
piling up in the stock market as a bubble. It then panicked, miscalculated, and let it collapse by a third by 1933, depriving the
economy of the liquidity it needed to breathe. Trade had nothing to do with it .¶ As for the charge that Smoot caused
the Depression to spread worldwide: it was too small a change to have plausibly so large an effect. For a start, it only applied to
about one-third of America's trade: about 1.3 percent of our GDP. Our average tariff on dutiable goods went from 44.6 to 53.2
percent -- not a terribly big jump. Tariffs were higher in almost every year from 1821 to 1914. Our tariff went up in 1861, 1864, 1890,
and 1922 without producing global depressions, and the recessions of 1873 and 1893 managed to spread worldwide without tariff
increases.¶ As the economic historian (and free trader!) William Bernstein puts it in his book A Splendid Exchange: How Trade
Shaped the World,¶ Between 1929 and 1932, real GDP fell 17 percent worldwide, and by 26 percent in the United States, but most
economic historians now believe that only a miniscule part of that huge loss of both world GDP and the United States' GDP can be
ascribed to the tariff wars. .. At the time of Smoot-Hawley's passage, trade volume accounted for only about 9 percent of world
economic output. Had all international trade been eliminated, and had no domestic use for the previously exported goods been
found, world GDP would have fallen by the same amount -- 9 percent. Between 1930 and 1933, worldwide trade volume fell off by
one-third to one-half. Depending on how the falloff is measured, this computes to 3 to 5 percent of world GDP, and these losses
were partially made up by more expensive domestic goods. Thus, the damage done could not possibly have exceeded 1 or 2 percent
of world GDP -- nowhere near the 17 percent falloff seen during the Great Depression... The inescapable conclusion: contrary to
public perception, Smoot-Hawley did not cause, or even significantly deepen, the Great Depression.¶ The oft-bandied idea
that Smoot-Hawley started a global trade war of endless cycles of tit-for-tat retaliation is also
mythical . According to the official State Department report on this very question in 1931:¶ With the exception of discriminations
in France, the extent of discrimination against American commerce is very slight...By far the largest number of countries do not
discriminate against the commerce of the United States in any way.¶ That is to say, foreign nations did indeed raise their tariffs after
the passage of Smoot, but this was a broad-brush response to the Depression itself, aimed at all other foreign nations without
tit-for-tat retaliation between
distinction, not a retaliation against the U.S. for its own tariff. The doom-loop of spiraling
trading partners that paralyzes free traders with fear today simply did not happen. ¶ "Notorious" Smoot-Hawley is a
deliberately fabricated myth , plain and simple. We should not allow this myth to paralyze our policy-making in the present
day.¶ There is a basic unresolved paradox at the bottom of the very concept of trade war. If, as free traders
insist, free trade is beneficial whether or not one's trading partners reciprocate, then why would any rational nation
start one, no matter how provoked? The only way to explain this is to assume that major national governments like the Chinese
and the U.S. -- governments which, whatever bad things they may have done, have managed to hold nuclear weapons for decades
without nuking each other over trivial spats -- are not players of realpolitik, but schoolchildren.¶ When the moneymen in
Beijing, Tokyo, Berlin, and the other nations currently running trade surpluses against the U.S. start to ponder the financial
realpolitik of exaggerated retaliation against the U.S. for any measures we may employ to bring our trade back into balance,
they will discover the advantage is with us, not them . Because they are the ones with trade surpluses to lose,
not us.¶ So our present position of weakness is, paradoxically, actually a position of strength.¶ Likewise, China can supposedly
suddenly stop buying our Treasury Debt if we rock the boat. But this would immediately reduce the value of the trillion or so they
already hold -- not to mention destroying, by making their hostility overt, the fragile (and desperately-tended) delusion in the U.S.
that America and China are still benign economic "partners" in a win-win economic relationship.¶ At the end of the day, China
cannot force us to do anything economically that we don't choose to. America is still a nuclear power. We can -- an
irresponsible but not impossible scenario -- repudiate our debt to them (or stop paying the interest) as the ultimate counter-move to
anything they might contemplate. More plausibly, we might simply restore the tax on the interest on foreign-held bonds that was
repealed in 1984 thanks to Treasury Secretary Donald Regan.¶ Thus a certain amount of back-and-forth token retaliation (and loud
squealing) is indeed likely if America starts defending its interests in trade as diligently as our trading partners have been defending
theirs, but that's it. The rest of the world engages in these struggles all the time without doing much
harm; it will be no different if we join the party.

No correlation between trade and peace

MARTIN, et al ‘8 (Phillipe, University of Paris 1 Pantheon—Sorbonne, Paris School of Economics, and Centre for Economic
Policy Research; Thierry MAYER, University of Paris 1 Pantheon—Sorbonne, Paris School of Economics, CEPII, and Centre for
Economic Policy Research, Mathias THOENIG, University of Geneva and Paris School of Economics, The Review of Economic Studies
75)

Does globalization pacify international relations? The “liberal” view in political science argues that increasing trade
flows and the spread of free markets and democracy should limit the incentive to use military force in interstate relations. This
vision, which can partly be traced back to Kant’s Essay on Perpetual Peace (1795), has been very influential: The main objective of
the European trade integration process was to prevent the killing and destruction of the two World Wars from ever happening
again.1 Figure 1 suggests2 however, that during the 1870–2001 period, the correlation between trade
openness and military conflicts is not a clear cut one. The first era of globalization, at the end of the 19th
century, was a period of rising trade openness and multiple military conflicts, culminating with
World War I. Then, the interwar period was characterized by a simultaneous collapse of world
trade and conflicts. After World War II, world trade increased rapidly, while the number of conflicts
decreased (although the risk of a global conflict was obviously high). There is no clear evidence that
the 1990s, during which trade flows increased dramatically, was a period of lower prevalence of
military conflicts, even taking into account the increase in the number of sovereign states.

Trade wars don’t escalate

Ikenson, 12 [March 5th, Daniel, Daniel Ikenson is director of the Herbert A. Stiefel Center for Trade Policy Studies at the Cato
Institute,

http://www.cato.org/publications/free-trade-bulletin/trade-policy-priority-one-averting-uschina-trade-war]
. Nature of the U.S.-China Trade War It
should not be surprising that the increasing number of commercial
exchanges between entities in the world's largest and second largest economies produce
frictions on occasion. But the U.S.-China economic relationship has not descended into an
existential call to arms. Rather, both governments have taken protectionist actions that are legally
defensible or plausibly justifiable within the rules of global trade. That is not to say that those
measures have been advisable or that they would withstand closer legal scrutiny, but to make
the distinction that, unlike the free-for-all that erupted in the 1930s, these trade "skirmishes"
have been prosecuted in a manner that speaks to a mutual recognition of the primacy of — if
not respect for — the rules-based system of trade. And that suggests that the kerfuffle is
containable and the recent trend reversible.1
---EXT No Trade Wars

1NC Ikenson and Fletcher —interdependence and self interest mean no trade war—the
system is stable—major economies can’t backslide and smaller ones don’t have an impact—
governments are committed to reducing barriers now proves their issue is a non-starter

Hold them to a high threshold—only risk of 7% decrease in trade value compared to 66%
decrease before world war two

And even if they access trade wars, we’ll win they don’t escalate—Ikenson 12 says countries
will err on the side of peaceful resolution because war only risks worsening economic
situation—flashpoints are containable and trends reversible

Alt causes outweigh now and prove impact is empirically denied

Zappone, 12 [January, Chris, Sydney Morning Herald, 'Murky protectionism' on the rise - but no trade war,
http://www.smh.com.au/business/world-business/murky-protectionism-on-the-rise--but-no-trade-war-20120110-1pt3t.html]

At the outset of the global financial crisis, the world’s leaders pledged to resist calls to shield their local
economies in order to prevent a trade war that could further damage global growth. Four years
on, with China slowing, Europe heading into recession and a political environment soured by successive financial crises, the question
arises: how long will policymakers be able to resist those calls for more protectionism? “Free trade is
going to be under pressure,” said Lowy Institute international economy program director Mark Thirlwell. “Since 2007-08 the case for
moving to greater trade liberalisation has got tougher and the demands for protection have increased.” Only last week, China, which
is grappling with a slowdown, raised the prospect of a trade war with the European Union in response to the EU's implementation of

a carbon emissions tax on air travel to and from Europe. Earlier last month China imposed tariffs up to 21 per cent on
US-made cars , affecting about $US 4 billion imports a year . Advertisement Across the Pacific, US

politicians in the throes of an election year with 8.5 per cent unemployment have issued more
strident calls for China to “play by the rules” and allow the yuan to appreciate faster against the
US dollar. The US has also asked the World Trade Organisation to probe China's support for its
solar panel industry and the restrictions Beijing has placed on US poultry imports. In fact, the most
recent WTO data shows that the number of trade restrictive measures enacted by members rose 53 per cent to 339 occurrences
over the year to October. Yet the WTO admits that the motives behind the spate of actions aren’t always simply to protect local jobs.
“Not all measures categorised as trade restrictive may have been adopted with such an intention,” the body said. In Brazil, for
example, the steep rise in the value of its currency, the real, has sparked a torrent of car imports into the country - similar to the
online-overseas shopping boom in Australia. Brazil has in turn put a one-year provisional 30 per cent increase on auto imports, to
counterbalance the effects of their strong currency. In the US, China and Australia, infrastructure spending measures contain “buy
local” requirements to stoke domestic growth, not necessary punish foreign businesses. The federal government in September
streamlined its anti-dumping system that eases the way for companies to ask for investigations into imported goods that come in
below market value to Australia. Again, well within the rules. “What we’ve seen is a gradual ratcheting up of trade intervention,”
said Mr Thirlwell, amounting to what he calls “murky protectionism” or government intervention through support for industries or
complaints to global trade authorities. To date, observers such as Mr Thirlwell say most countries have remained remarkably
resistant to throwing up significant trade barriers. For example, in November, the US, Australia and seven other Asian-Pacific nations
including Japan, outlined the plan for an ambitious multilateral Trans-Pacific Partnership trade block worth 40 per cent of the world’s
trade, in an effort to increase the flow of cross-border goods and investment. Japan, China and South Korea are also in the later
stages of negotiation over a free trade deal between those three nations. Australian National University
international trade lecturer John Tang doesn’t believe the world is on the edge a new round of
protectionism. “I don’t see a general sea change towards protectionism for major trading blocks
but that may be because so much of the industrialised world is relying on developing countries
to sustain their exports,” he said. Nevertheless, a shift in the political reality of the US, China or elsewhere could change
that, he said. Washington DC-based Brookings Institution fellow Joshua Meltzer said that if the euro zone broke up, elevating the
crisis to a new stage, nations may switch to much more protective measures. ‘‘ I
wouldn’t go so far to say the global
economy is so integrated that we could never have anything that would approach a trade war,” said
Washington DC-based Brookings Institution fellow Joshua Meltzer. “But I don’t think we’re on that track .”

System resilient—no risk one issue collapses it

Lamy ’11(Pascal Lamy is the Director-General of the World Trade Organization. Lamy is Honorary President of Paris-based think
tank Notre Europe. Lamy graduated from the prestigious Sciences Po Paris, from HEC and ÉNA, graduating second in his year of
those specializing in economics. “System Upgrade” BY PASCAL LAMY | APRIL 18, 2011)

Any doubts about the stability and importance of the global trade architecture should have been
dispelled by the remarkable manner in which the system has endured the devastating
economic crisis that shook the world from 2008 to 2009 . That durability stands in stark contrast
with many other elements in the international architecture , which proved too brittle to
withstand this shock. For example, governments have yet to devise an international system for
managing climate change or currency volatility. Other issues heavily tinged by domestic politics,
such as migration, are not even on the international agenda and face fire even at the regional
level, as we have seen with the influx of immigrants to Europe following recent events in North
Africa. By now, it should be clear that the failure to establish a system of global governance in the area of international finance
unquestionably blunted governments' ability to respond effectively to the crisis. Yet even while a great many things went wrong in
the crisis, the
trading system responded precisely as it was intended to. Compare that with the
1930s, when the last great global economic calamity unfolded. No such system was in place, and
the global economy paid a heavy price. The United States passed the Smoot-Hawley Tariff Act in
1930, quadrupling tariffs and raising duties to at least 60 percent on more than 3,000 types of imported products. Faced with this
provocation, America's trading partners did not sit idly by; tit-for-tat retaliation rapidly ensued, closing markets and throttling trade.
Between 1929 and 1934, world trade contracted 66 percent. The bulk of this collapse was due to disintegrating demand and the
drying-up of credit. But the imposition
of prohibitive duties not only pushed the economy further into depression -- it
also fostered a profound sense of ill will between governments and contributed to the international tensions that
led to World War II. To prevent this from happening again, leaders of great vision and wisdom agreed to
create a rules-based, transparent, and nondiscriminatory trading system. Men like James Meade and
Cordell Hull succeeded in encouraging 23 countries to accept a compact known as the General Agreement on Tariffs and Trade
(GATT). Since
the GATT came into force in 1948 and since the World Trade Organization ( WTO)
opened its doors in 1995, the world has not seen protectionist outbreaks like those of the early
1930s. This is not to say that the trading system has not been tested. Protectionist pressures
surged in 1971, for example, when the gold standard for currency conversion was abandoned,
and during the 1997-1998 Asian economic crisis, when Pacific Rim countries saw their
economies contract by double-digit margins. In each case, markets stayed open to the flow of
goods and services from the affected countries, enabling them to trade their way back to
stability and prosperity.

Threats of trade wars are hype—systemic barriers prevent now

Forbes 10 [Steve, President and Chief Executive Officer of Forbes and Editor-in-Chief of Forbes magazine, and Pascal Lamy,
World Trade Organization Director General, Intelligent Investing WTO Head Pooh-Poohs World Trade War Possibility Steve Forbes,
07.26.10, 06:00 PM EDT http://www.forbes.com/2010/07/26/trade-war-wto-intelligent-investing-china.html?
boxes=Homepagechannels]

World Trade Organization Director General Pascal Lamy says that all-out, fists-swinging trade wars don't
happen anymore . Steve Forbes: U.S.,China relationships. We've avoided a trade war so far this year. Put that in
perspective. Are we doomed to have more contentious negotiations or can we get something on a level where we don't always have
to worry this thing's going to blow up? Pascal Lamy: Well, it's inevitable that the growing importance of China in the world economy
and in trade creates frictions. Remember what happened in the '60s, '70s with Japan at the time. Forbes: Or even Germany. Lamy:
Or even Germany. So there will be frictions. There are frictions. The problem is whether these frictions
remain in a reasonably corralled procedure , which is what WTO is about. If U.S. and China disagree on
trade, if China believes U.S. is doing too much anti-dumping or if U.S. believes China is not doing according to its commitment, we
have a litigation system. We have a dispute settlement system that allows these two members
to run this peacefully. So we are not in a planet where there can be trade wars anymore . And by the
way, that's a great historical achievement because if you look at the economic history of humanity, we've had trade wars. We've
had trades frictions that generated, degenerated in trade wars. This is not possible anymore
thanks to the system. Forbes: So you can have small wars like the chicken wars we had way back in the '60s and '70s. Lamy:
Media will call them wars because it sells better than "dispute" or "friction." But the legal reality
is that these are disputes or frictions. And true, the U.S., China relationship has a sort of specific color given the huge
trade deficit of U.S. and the huge China trade surplus. Now, whether the U.S. trade deficit is generated by China or whether China
surplus is generated by the U.S. is a different question. The fundamental reality is that the reasons why the U.S. has a trade deficit do
not have much to do with trade. It has a lot to do with the fact that the U.S. economy doesn't save a lot and consumes a lot and the
other way around.

Their Doha brink claims go our way—proves even fast collapse doesn’t culminate in trade war

Seattle Times ‘8 7-31

Economists disagree on the Doha round's potential benefits ; estimates of economic gain that could have been reaped
through additional trade range from $4 billion to $100 billion. Set against the rapid expansion of global trade to $13.6 trillion

last year from $7.6 trillion five years ago, however, the bottom-line loss from Doha's failure is "not a

market issue," said Julian Callow, an economist at Barclays Capital in London. Nor is the world on the edge of the kind of
protectionist wave that ended the last period of globalization in the early 20th century and
contributed to two world wars, analysts say. Countries are likely to go on negotiating bilateral
trade deals with each other, such as the U.S.-South Korea free-trade deal earlier this year.

Numerous constraints prevent protectionism

Kumar ’8, Rajiv -Protectionism and Obama, p. http://www.mydigitalfc.com/opinion/protectionism-and-obama


President designate Obama, while he can, of course, take the protectionist route, is unlikely to do so for
several reasons. First, he appears to be strongly committed to reversing the decline in US's global
prestige and leadership that has happened during the Bush presidency, especially over the last four years. He cannot hope
to achieve this by leading the US away from globalisation and turning his back to US’s long-standing
commitment to free market for goods and services. This will seriously erode the legitimacy that Pax American enjoys at present. A
protectionist move by Obama presidency must surely imply the beginning of the end of the US economic hegemony in the world and
accelerate the shift away from the Atlantic basin to Asia. Second, US
firms with overseas operations, especially in Asia,
will for good reason, resist these moves as their competitiveness and indeed survival will be
threatened. The loss of competitiveness and eventual shutting down of these firms will also result in job losses within the US.
Third, any unilateral protectionist moves by the US will raise the spectre of competitive tariff escalation by
its trading partners, which will surely exacerbate the current crisis and make a world-wide depression that much more possible.
There are more than enough people within the US academia, and hopefully also within the
administration, who can hammer home the dangers inherent in such an approach and thus stop the
Obama administration from going ahead in the protectionist direction. Fourth and last, higher protection levels
will imply ringing the death knell of the Doha Round and effectively also the near complete loss of World Trade Organisation's
credibility and indeed legitimacy. I doubt if any US president can precipitate such an eventuality. These factors will hopefully
ensure that while there will be plenty of threats, and perhaps even some calls from voluntary export restraints
from the incoming administration, these will not be carried forward to actual imposition of higher tariff or non-
tariff walls by the US.

Counterproductive and trade court solves

Ziemba ‘9 (The Re-Emergence of Global Protectionism: A Newer Version of Smoot-Hawley?


Rachel Ziemba | Mar 4, 2009

probability of these measures becoming significant enough to lead to a trade war


However, the
like the 1930s might be low given that counties understand that retaliation effects will counter-
productive for domestic growth and jobs. Moreover, the WTO surveillance mechanism, absent
during the 1930s, will help countries go to the WTO court if they face import barriers and thus
prevent trade wars.
---EXT Doesn’t Solve War

1NC Martin—No link between trade and peace— 130 years of case studies process no
correlation—high trade before world war one and during the instability of the 90s proves their
analysis is snapshot—prefer concrete historical economic data—theirs is just rhetorical moves
by trade hacks

Interdependence causes as much conflict as it solves—US-Russian and Germany-France prove

Friedman and Friedman ’98 (George Friedman, PhD, Chairman of STRATFOR, internationally recognized expert in security
and intelligence, and Meredith Friedman, senior writer and co-founders of STRATFOR, freelance international affairs writer, 1998,
The Future of War: Power, Technology and American World Dominance in the Twenty-First Century, p. 3-5

The argument that interdependence gives rise to peace is flawed in theory as well as in practice. Conflicts
arise from friction, particularly friction involving the fundamental interests of different nations. The less
interdependence there is, the fewer the areas of serious friction. The more interdependence there is, the
greater the areas of friction, and, therefore, the greater the potential for conflict. Two widely separated nations that
trade little with each other are unlikely to go to war—Brazil is unlikely to fight Madagascar precisely
because they have so little to do with each other. France and Germany, on the other hand, which
have engaged in extensive trade and transnational finance, have fought three wars with each
other over about seventy years. Interdependence was the root of the conflicts , not the deterrent.
There are, of course, cases of interdependence in which one country effectively absorbs the other or in which their interests match
so precisely that the two countries simply merge. In other cases, interdependence remains peaceful because the economic, military,
and political power of one country is overwhelming and inevitable. In relations between advanced industrialized countries and third-
world countries, for example, this sort of asymmetrical relationship can frequently be seen. All such relationships have a quality of
unease built into them, particularly when the level of interdependence is great. When one or both nations attempt, intentionally or
unintentionally, to shift the balance of power, the result is often tremendous anxiety and, sometimes, real pain. Each side sees the
other’s actions as an attempt to gain advantage and becomes frightened. In the end, precisely because the level of interdependence
is so great, the relationship can, and frequently does, spiral out of control. Consider
the seemingly miraculous ability
of the United States and Soviet Union to be rivals and yet avoid open warfare. These two powers
could forgo extreme measures because they were not interdependent . Neither relied on the
other for its economic well-being, and therefore, its social stability. This provided considerable room for
maneuvering. Because there were few economic linkages, neither nation felt irresistible pressure to bring
the relationship under control; neither felt any time constraint. Had one country been dependent on the
other for something as important as oil or long-term investment, there would have been
enormous fear of being held hostage economically . Each would have sought to dominate the relationship, and
the result would have been catastrophic. In the years before World War I, as a result of European
interdependence, control of key national issues fell into the hands of foreign governments . Thus,
decisions made in Paris had tremendous impact on Austria , and decisions made in London determined growth
rates in the Ruhr. Each government sought to take charge of its own destiny by shift ing the pattern of
interdependence in its favor. Where economic means proved insufficient, political and military
strategies were tried. T he int er nati onal syst em following the C old War resem bles the pre -World War I
syste m in som e fundam ental ways. F ir st , there is a gen er al pr ospe rity. That is t o say, t he int er nati onal
ec onom ic syst em appe ar s to be functi oning extr em ely well, in spit e of t he nor mal c yc lical down t ur ns of the
e ar ly 1990 s. Se cond, alm ost no fundame nt al ide olog ic al issues divide t he m ajor powe rs; one could say there
is ge ne ral agr eem ent on m att e rs of politi cal pr inciple. T hird, there is a long -standing patt er n of
int er de pende nce , me asur ed in both tr ade and fi nanc ial fl ows—c apit al has become t ransnati onal. F ourth,
beneath the apparent prosperity and stability there is a sense within
and per haps m ost im port ant,
each great power of a real and growing vulnerability to the actions of others . S ome nati ons fe ar
t hat growing pr ot ecti onism will shift t he balance of the sys te m ag ainst the m, while others ar e convince d
Today, observers focus on the first
t hat maintaining the cur r ent syste m will be devastati ng to t heir int ere st s.
three phenomena, as they did prior to World War I, and argue
that there is no economic basis for polit ical
conflict. What they miss is that the subsurface sense of insecurity —experienced by Japan, the United States,
and Europe—marks the beginning of such conflict. Thus, the argument that war is obsolete
because of growing interdependence is unsupportable . War may be obsolete, but, if it is, it is not
because of interdependence. As we have seen, World
War I broke out at a time when interdependence was
substantially higher than it is today; indeed, in all likelihood war broke out because
interdependence was so high. Today, war remains not only possible but, as a simple statistical
matter, highly likely.
Turkey Soft Power Answers
Turkey can’t use its soft power – also means they can’t solve
Grunstein 11
Judah, “Turkey and the Arab Spring”, World Politics Review's editor-in-chief. His coverage of French politics, foreign policy and
national security has appeared in World Politics Review, the American Prospect online, French Politics, the Small Wars Journal and
Foreign Policy online

http://www.worldpoliticsreview.com/trend-lines/8764/turkey-and-the-arab-spring

Moreover, and supporting Cook's argument, as much as I've often admired Turkey's regional approach, I've
previously noted that there remains a wide gap between the rhetoric used by Ankara -- and outside
observers -- to describe Turkey's regional pull and the reality of it . This was on prominent display following the
Turkey- and Brazil-mediated nuclear fuel swap agreement with Iran, and is also characteristic of most of Turkey's "successes" in the
region and beyond. The fact is, Ankara is pretty good at getting people to the table, as its many past and
present mediation efforts -- Israel and Syria, Serbia and Bosnia-Herzegovina, the Iran deal -- attest. But the
difference between mediation and influence lies in the ability to get everyone , including interested
third parties, to sign on the dotted line. And that's where Turkey bumps up against the limits of its
diplomatic weight. Ankara has delivered some concrete results in resolving tensions with its own
neighbors: Syria, Iraq, Iraqi Kurdistan, Iran and Armenia, as well as a more distant Russia. But on a regional basis, it has
not yet managed to convert the soft power that comes with increased trust and connectivity
into the hard power of getting others to adopt its preferred line . In part that is because it just doesn't have the
muscle necessary both to twist people's arms and to guarantee the resulting compromises. In part it is because the rise in its
regional influence remains a work in progress. For Turkey, the Arab Spring comes 10 years too soon. But it is also
worth noting the regional realities: Ankara is operating in a very challenging neighborhood. If Turkey has failed in the Middle East,
and during the Arab Spring, who exactly has succeeded? As for the Turkish Model that emphasizes connectivity and autonomy over
containment and alignment, Ankara's current challenges do not necessarily invalidate it, but they do demonstrate two
vulnerabilities. First, connectivity is only as stable as the surrounding network. In almost all circumstances, the redundancy it offers
serves to minimize the destabilizing impact of local perturbations. However, connectivity shifts from being a dampening effect to
being a magnifying effect when the entire network experiences generalized perturbations. That's what's
happening in the
Middle East right now, and the threat -- or challenge -- it poses to Turkey is a clear illustration of
connectivity's double-edged sword. Second, the diplomatic value of connectivity is inversely proportional to the
surrounding network's reach. Turkey's refusal to exclusively align with either side of the region's many divides -- Saudi-Iranian,
Sunni-Shiite, Western-Arab -- was more valuable in the context of a region where trust and communication were low, but where the
cost of conflict was high. In that kind of environment, being the only party that has relationships with all sides can be more easily
converted into influence. But Turkey's
status as the only the only game in town when it comes to
bridging divides and resolving conflicts is no longer guaranteed . Egypt's emerging reorientation, including
tentative contacts with Iran and an increasing willingness to buck U.S. policy, suggests that, in a best-case scenario for the Arab
Turkey might find that it is no longer the only regional actor pursuing a policy of
Spring,
connectivity and nonalignment. That will be good news for the region, but it could very mean that the
high-point of Ankara's regional influence is behind it .

Your authors misread signals – Turkey’s clearly aligned with Western interests
Saunders, 12 – chief of the Globe and Mail's London-based European Bureau; card is internally qualified (Doug, originally
published 9/16/11, updated 2/17/12. “The Middle East’s new emperor: Why the West quietly cheers Turkey’s rise.”
http://www.theglobeandmail.com/news/world/africa-mideast/the-middle-easts-new-emperor-why-the-west-quietly-cheers-
turkeys-rise/article2169665/page2/)

But it is exactly those sort of imperial


analogies that have Westerners worried about Turkey’s new
assertiveness. It has become popular to call Mr. Erdogan’s tactics “neo-Ottoman ,” after the Turk-led
Muslim empire that conquered much of Europe, the Middle East and north Africa between the 14th and the 19th centuries. The
worry is that Turkey is now turning away from its European roots – after being shunned by the European
Union in its bid for membership – and using the power vacuums to its south to link up with the region’s
Islamist parties and form a network of Islamic power to threaten the West . It is a misleading
analogy, mistaking Mr. Erdogan’s bold but self-interested mission for some sort of Islamic
imperialism, but it is a popular and understandable one. After all, Mr. Erdogan, a former Islamist and devout believer, launched
this tour after turning a minor spat with Israel, Turkey’s traditional staunch ally, into an outright conflagration. After Israel refused to
apologize earlier this month for its army’s killing of nine Turkish civilians aboard the controversial aid flotilla to Gaza in May of 2010,
Mr. Erdogan responded furiously by withdrawing Turkey’s ambassador, suspending its military co-operation with Israel, and freezing
all trade ties with the Jewish state. His Arab tour has been laced with fiery criticisms of Israel, glowing support for the Palestinian
cause, and macho statements suggesting a military showdown: “Israel will no longer be able to do what it wants in the
Mediterranean,” he told an audience in Tunis on Thursday, “and you’ll be seeing Turkish warships in this sea.” That was only one of
several such alarming sabre-rattling statements, suggesting that the spat with Israel is in large part intended to send a message of
solidarity to his country’s Arab neighbours. That, combined with the warm mutual embrace between Mr. Erdogan and leaders of the
Muslim Brotherhood (who tend to tell Western reporters that they see Turkey as their role model), has led some to fear that
Europe’s largest Muslim state is turning to the dark side. It’s an alarming prospect, given that Turkey has Europe’s largest standing
army and has become wealthy enough, from gas pipelines and industrial exports to Europe, to become a major power. Yet most
informed observers of Turkish diplomacy would say that’s a serious misreading . “Yes, Turkey has
been engaging its neighbourhood, and not just in the Middle East, and building its influence with Muslim states,”
said Joshua W. Walker, a Turkey specialist who is a fellow at Harvard’s Kennedy School of Government, “but it would be a
mistake to think that this is a move away from the West or from democracy or secularism.
They’re making a shift from a French style of secularism to an American one, where you’re allowed to
be religious and still be in government, but there’s no sign that Turkey is moving away from the West .” Indeed,
many Western diplomats, including those from the United States, quietly say that Mr. Erdogan’s
eastern turn is a welcome and beneficial development – in good part because it could herald the eclipse of Saudi
Arabia’s and Iran’s much more dangerous influence over the Arab states, but also because what Mr. Erdogan is doing is hardly
imperial or Islamist. His key message to Egyptians, delivered in a national TV interview, is that they should get rid of their old sharia-
based constitution and become a secular state. “In Turkey, constitutional secularism is defined as the state remaining equidistant to
all religions,” he said. “In a secular regime people are free to be religious or not.” And if there was any ambiguity, he then told
Egyptians that the most important thing Arabs should learn from Turkey is secularism – a word that is close to unmentionable in
Egypt these days. “I recommend a secular constitution for Egypt,” he said. “Do not fear secularism because it does not mean being
an enemy of religion. I hope the new regime in Egypt will be secular. I hope that after these remarks of mine the way the Egyptian
people look at secularism will change.” That message was heard loudly across the Arab world, and provoked angry responses from
the Brotherhood and other Islamist groups (who nevertheless still sought to associate themselves with the Turkish leader). And it
came alongside a number of other signs that Turkey is far from turning eastward. Just as he was falling into his feud with Israel, Mr.
Erdogan struck a bold deal with the United States to use his country as the staging ground for a missile defence system that uses
huge radar installations to protect against Iranian missile attacks. A senior U.S. official told the New York Times that it is “the most
significant military co-operation between Washington and Ankara since 2003” and it was widely seen as part of a major boost in the
country’s 59-year-old membership in NATO. At the same time, Turkey joined a major antiterrorism initiative with the United States.
And its trade and political relations with European states have been growing strongly. “ If
you actually examine what is
happening,” says Fadi Hakura, head of the Turkey Project at London’s Chatham House, “you
realize that this is the best possible situation for the United States and Europe – you have a
strongly allied country that can exercise a tough position with Israel without promoting the kind
of violence that other regional actors like Iran did.” In other words, Turkey may play the bad cop with Israel, but
unlike Tehran, it won’t be interested in bankrolling terrorists groups like Hezbollah.

Iraq proves Turkish influence is inevitable – it’s a question of efficacy – requires successful
economic integration in the Arab world

Shadid, 11 – 2-time Pulitzer Prize-winning foreign correspondent, died in Syria  (Anthony, 1/4. “Resurgent Turkey Flexes Its
Muscles Around Iraq.” The New York Times. http://www.nytimes.com/2011/01/05/world/middleeast/05turkey.html?
pagewanted=all)

A Turkey as resurgent as at any time since its Ottoman glory is projecting influence through a
turbulent Iraq, from the boomtowns of the north to the oil fields near southernmost Basra, in a
show of power that illustrates its growing heft across an Arab world long suspicious of it. Its ascent
here, in an arena contested by the United States and Iran, may prove its greatest success so far, as it emerges from
the shadow of its alliance with the West to chart an often assertive and independent foreign policy. Turkey’s influence is greater in
northern Iraq and broader, though not deeper, than Iran’s in the rest of the country. While the United States invaded and occupied
Iraq, losing more than 4,400 troops there, Turkey now exerts what may prove a more lasting legacy — so-called soft power,
the assertion of influence through culture, education and business. “This is the trick — we are very much welcome here,” said
Ali Riza Ozcoskun, who heads Turkey’s consulate in Basra, one of four diplomatic posts it has in Iraq. Turkey’s newfound influence
here has played out along an axis that runs roughly from Zakho in the north to Basra, by way of the capital, Baghdad. For a country
that once deemed the Kurdish region in northern Iraq an existential threat, Turkey has embarked on the beginning of what might be
called a beautiful friendship. In the Iraqi capital, where politics are not for the faint-hearted, it promoted a secular coalition that it
helped build, drawing the ire of Iraq’s prime minister, Nuri Kamal al-Maliki, along the way. For
Iraq’s abundant oil and
gas, it has positioned itself as the country’s gateway to Europe , while helping to satisfy its own
growing energy needs. Just as the Justice and Development Party of Prime Minister Recep Tayyip Erdogan has reoriented
politics in Turkey, it is doing so in Iraq, with repercussions for the rest of the region. While some Turkish officials recoil at the notion
of neo-Ottomanism — an orientation of Turkey away from Europe and toward an empire that once included parts of three
continents — the country’s process of globalization and attention to the markets of the Middle East
is upsetting assumptions that only American power is decisive . Turkey has committed itself here
to economic integration, seeing its future in at least an echo of its past . “No one is trying to overtake Iraq
or one part of Iraq,” said Aydin Selcen, who heads the consulate in Erbil, which opened this year. “But we are going to
integrate with this country. Roads, railroads, airports, oil and gas pipeline s — there will be a free flow of
people and goods between the two sides of the border.” By the border, he meant Zakho and the 26-lane checkpoint of Ibrahim
Khalil, where 1,500 trucks pass daily, bringing Turkish building materials, clothes, furniture, food and pretty much everything else
that fills shops in northern Iraq. The
economic boom they have helped propel has reverberated across
Iraq. Trade between the two countries amounted to about $6 billion in 2010, almost double what it was in 2008, Turkish officials
say. They project that, in two or three years, Iraq may be Turkey’s biggest export market. “This is the very beginning,” said Rushdi
Said, the flamboyant Iraqi Kurdish chairman of Adel United, a company involved in everything from mining to sprawling housing
projects. “All of the world has started fighting over Iraq. They’re fighting for the money.”

All impact turns are empirically denied – Turkey’s rise has been peaceful

Cagaptay, 11 – senior fellow and director of the Turkish Research Program at The Washington Institute (Soner, 11/16. “Ties to
Neighbors, Not Their Dictators.” http://www.nytimes.com/roomfordebate/2011/11/15/why-turkey-turned-away-from-syria/ties-to-
the-neighbors-not-their-dictators)
Accordingly, Turkey now has a chance to promote democracy in the Middle East, build ties with
its neighboring peoples, and rise to leadership in the region , all at the same time. Turkey’s time to
become a Middle East power seems to have arrived . Challenges remain, of course, including the future of
Turkish-Israeli ties and Iran’s hostile attitude to Ankara’s rising influence in the region. If Turkey and Israel can come to some accord,
this would help Ankara’s ambitions to become a regional leader, respected and liked by the peoples of the region. Iran is a more
tricky case: Tehran envisions itself as the Middle East hegemon and will do all it can to undermine Turkey’s ambitions to be the
leader of a democratic Middle East. CTurkey is already rising to the challenge to lead the region by
example. For instance, its prime minister recently called for secular democracy during his trip to
Egypt, upsetting that country's Islamists. Not since the heyday of the Ottoman Empire have the Turks had
this much clout in the Middle East . The sultans must be green with envy.
Turkey Prolif Answers
Long timeframe and U.S. diplomacy solves – no technical infrastructure, no country would
refuse the U.S. nuclear umbrella

Jakobsen 10 – Associate Professor at the Department of Political Science, University of Copenhagen (Peter Viggo, Ali Rahigh-
Aghsan, Assistant Professor at the Department of Society and Globalisation, Roskilde University, Autumn, “The Rise of Iran: How
Durable, How Dangerous?” ProQuest)

The danger of an arms race is not imminent, however, as the current


aspirations for nuclear energy in the region will take
10-15 years to fulfill.60 This leaves considerable time for the US to persuade Saudi Arabia, Egypt, and
Turkey not to go nuclear, and the US is in a position to give them an offer they cannot refuse.
Since all these states and the rest of the GCC countries effectively depend upon the US for their security in the interim until they can
develop their own nuclear deterrent, the US can offer to extend its nuclear umbrella to them and provide
continued military, economic, and political support contingent on their not developing nuclear weapons. This
tactic worked in similar circumstances with Taiwan and South Korea during the Cold War, and there is no reason why it should not
work in the Middle East as well.61 The penalties the US can impose if they renege on this promise are
prohibitive and since none of these countries have an interest in appeasing Iran unless forced to do so for lack of an
alternative, it is hard to see why they should not accept such an offer. It is also inconceivable that
the US would refrain from extending such an offer to its allies given the strategic importance that it
attaches to the region. Since Iran is not capable of hitting the US mainland with nuclear missiles, the US would not have to sacrifice
New York to protect Amman and this makes extended nuclear deterrence inherently more credible in the Middle East than it was in
Western Europe during the Cold War.
UN Answers
UN fails and not key to global problems—coalitions solves

Haas, 10
Richard Haas, CFR President, 1/5/2010, The Case for Messy Multilateralism,
http://www.cfr.org/publication/21132/case_for_messy_multilateralism.html?breadcrumb=%2Fissue%2F42%2Fun

But to acknowledge that we are all multilateralists now (or at least need to be) is only to start the conversation. Multilateralism is
not one thing but many. The issue takes on a new urgency in the aftermath of the recent Copenhagen conference, which brought
together representatives of 193 governments in an unsuccessful effort to reach a formal, binding and comprehensive accord.
Whatever its consequences for climate change, Copenhagen is but the most recent reminder that classic
multilateralism is increasingly difficult to achieve. This same reality also helps to account for the
world's inability to agree to a new global trade accord. Launched in Qatar nearly a decade ago, the Doha round
of negotiations has stalled. There are simply too many participants, too many contentious issues and
too many domestic political concerns to discuss. This problem also explains the near-total irrelevance of
the United Nations General Assembly. "One man, one vote" may provide a sound basis for domestic politics, but on a
global scale democracy (or, more precisely, democratic multilateralism) is a prescription for doing nothing. It is not
simply the large number of participants but the fact that it makes little sense to give countries with minuscule
populations and economies equal standing with, say, China or the US. The UN's founders predicted as much
when they created the Security Council. The idea was to establish an elite body to tackle the world's most important issues. The
problem is that the composition of the Security Council reflects what the world looked like after the
second world war. That world is now more than 60 years old. Missing from the ranks of permanent members are India, Japan,
Germany, Brazil and representatives of a more integrated Europe. It was this weakness (along with the inability to agree on the
make-up of a reformed Security Council) that in part led to the creation of the Group of Seven and the trilateral
process in the 1970s. Japan and the European Commission gained a seat at this important table. Yet over the decades, the G7 also
proved inadequate, as it left out such critical countries as China and India. Hence the emergence of the Group of 20 in the midst of
the global financial crisis and the Major Economies Forum as concerns over climate change mounted. It is too soon to judge the
impact of these latest versions of elite multilateralism. In the meantime, we are seeing the emergence of
multiple innovations. One is regionalism. The proliferation of bilateral and regional trade pacts (most recently in Asia) is
in part a reaction to the failure to conclude a global trade accord. Such arrangements are inferior - they do not, for example,
normally deal with subsidies, much less cover all products and services. They can also have the perverse effect of retarding trade by
discriminating against non-members. But some trade expansion is preferable to none. A second alternative is functional
multilateralism - coalitions of the willing and relevant. A global accord on climate will prove elusive for some time
to come. But that need not translate into international inaction. A useful step would be to conclude a global pact to discourage the
cutting down and burning of forests, something that accounts for a fifth of the world's carbon output. Copenhagen made some
limited progress here, but more needs to be done to assist such countries as Brazil and Indonesia. Yet another alternative
might be described as informal multilateralism . In many cases it will prove impossible to negotiate international
accords that will be approved by national parliaments. Instead, governments would sign up to implementing, as best they can, a
series of measures consistent with agreed-upon international norms. We are most likely to see this in the financial realm, where
setting standards for the capital requirements of banks, accounting systems and credit ratings would facilitate global economic
growth. None of this - not elitism or regionalism or functionalism or informalism - is a panacea. Such collective action is
invariably less inclusive, less comprehensive and less predictable than formal global accords. It can suffer from a lack of legitimacy.
But it is doable and desirable, and can lead to or complement classic multilateralism . Multilateralism in
the 21st century is, like the century itself, likely to be more fluid and, at times, messy than what we are used to.
UN is beyond repair---lack of cred means it can’t solve conflicts

Ogwyn, 5
(Columnist-Tomorrows World, Sept/Oct, http://www.tomorrowsworld.org/magazines/2005/sep-oct/the-united-nations-mans-last-
best-hope)

It quickly became evident that the UN was as powerless as the League of Nations had been to stop the
regional conflicts that were fueled by superpower rivalry. Whether it was the long-standing Vietnam conflict or Israel's Six Day
War against the Arab League, the UN was neither able to prevent conflict nor to resolve it effectively
once it had begun. United Nations Peacekeepers From the Congo to Bosnia to Somalia, UN
peacekeeping operations have earned a reputation of being far worse than ineffective.
Notorious for their lack of discipline, UN troops have regularly been accused of rape and
brutality. Generally drawn from third-world countries, these "peacekeepers" have never been much of a
force for peace. In 1960 the Congo gained independence from Belgium under the leadership of Patrice Lumumba, a self-
proclaimed communist. Within weeks, the nation descended into anarchy as rape and looting became widespread. Belgium
responded by sending in troops to protect its nationals, and a firestorm was set off at the UN. Russia and its allies demanded that UN
troops be sent to protect Congo's independence from the "imperialists." At this point, Katanga province leader Moise Tshombe
declared independence, announcing that he was "seceding from chaos." In the ensuing months, there were coups and counter
coups in the Congo. Lumumba was finally assassinated. However, it was against Katanga province that the ire of the UN forces was
directed. From the pitched battles launched by UN forces in the fall, to the infamous UN bombing of the Elizabethtown Hospital in
Katanga shortly before Christmas 1961, the situation went from bad to worse. Over the next year, there were a succession of battles
and ceasefires. By January 1963, Katanga was finally subjugated by UN peacekeepers. By that time, the UN's record of atrocities was
well documented by both civilian physicians and western journalists present in Katanga during the fighting. Sadly, what happened
there was not an isolated case of UN misconduct, though it was one of the most egregious examples. How about a much more
recent example? The UN mission to Eritrea and Ethiopia was launched in 2000. Barely three months into its mission, reports
surfaced of Danish peacekeepers sexually abusing a 13-year-old Eritrean girl. No corrective actions were ever taken. Three-and-a-
half years after the launch of the UN mission, the Eritrean government declared that all the UN had accomplished was to efficiently
squander about a billion dollars! Accused
of sexual exploitation of women and children, torture and
murder of civilians, and using diplomatic immunity for trafficking in everything from narcotics to
people, the reputation of the UN's peacekeeping forces had reached a nadir. Such accusations
against the UN do not come merely from obscure sources or right-wing critics. Notice excerpts from an
article, "The U.N., Preying on the Weak," found in the April 12, 2005 issue of the Washington Post: "Anyone who was shocked by the
most recent revelation of sexual misconduct by United Nations staff has never set foot in a U.N.-sponsored refugee camp. Sex
crimes are only one especially disturbing symptom of a culture of abuse that exists in the United
Nations precisely because the United Nations and its staff lack accountability." Corrupt and Ineffective
Conduct of "Peacekeeper" troops in country after country is only one small part of the problem. A culture of arrogance
and corruption has permeated the institution and brought the UN into widespread disrepute. The
scandal involving the oil-for-food program in pre-war Iraq is one of the more recent cases in point. Investigations have shown
involvement reaching to the highest levels of the UN, yet no real disciplinary actions took place. U.S. Senator Norm Coleman led a
seven-month investigation into the UN-sponsored oil-for-food program and ended up calling for the resignation of UN Secretary
General Kofi Annan because of the corruption associated with this program. Senator Coleman claimed that Saddam Hussein had
defrauded the program of $21 billion and that this was made possible by a combination of both passive and active help from UN
officials. Writing in the Wall Street Journal, Senator Coleman declared, "We have obtained evidence that Saddam doled out lucrative
oil allotments to foreign officials, sympathetic journalists and even one senior U. N. official. We are gathering evidence that Saddam
gave hundreds of thousands—maybe even millions—of Oil-for-Food dollars to terrorists and terror organizations… under the
supposedly vigilant eye of the U.N." Where
it is not found corrupt, the U.N. is consistently found
ineffective. The crisis in Bosnia, back in the mid-1990s, illustrated the UN's utter inability to deal
with the very problems it was set up to prevent. Newsweek magazine, in its July 14, 1995 issue, detailed some of
the atrocities that illustrated the genocide taking place in Bosnia. One graphic example was of a Bosnian mother sitting at the table
eating a meal with her family when Serbian troops forced their way into the house. A soldier pulled a large knife and killed her oldest
son before her eyes, then arrested her 15-year-old son and took him away. "Later that afternoon, the Serbs rounded up all males in
their teens and older and took them off for 'interrogation'—leaving 10,000 or more people unaccounted for and raising fears of a
mass atrocity." Where was the UN when all of this was going on? Most often, it was denying the seriousness of what was happening!
Note these comments by Jeanne Kirkpatrick, U.S. Ambassador to the UN during the Bosnian genocide: "Almost everyone
understands that this war is not a conflict of limited importance, as the U.N. Secretary-General and selected European leaders have
tried to pretend it is. It is a particularly heinous example of aggression that seeks not merely to conquer a country but to destroy,
expel and dispossess peoples in a process of 'ethnic cleansing'" ("The U.N. Emasculation of NATO," San Diego Union-Tribune, July 14,
1995). For decades, critics of every kind have decried the corruption and ineffectiveness that seems to
permeate the UN. However, they all seem to come to the conclusion that since there is no viable alternative to the UN, the
best hope seems to be some sort of institutional reform. Even a bad UN is better than no UN, appears to be the thinking. Most
recently a reform program has been floated that would enlarge the UN Security Council and give permanent seats to Germany and
Japan as well as perhaps India and Brazil. Would this really solve anything?

UN structurally fails

Posner and Yoo 5. (Eric A., Teacher – U Chicago Law, and John C., Prof UC Berkeley Law, LA Times, 4-19, Lexis)

By design, the U.N. inhibits multilateral action. The five permanent members of the Security
Council -- Britain, France, Russia, China and the U.S. -- all have an absolute veto over authorizing military action, applying
sanctions and sending peacekeepers. Because these states rarely can reach a consensus, the Security
Council rarely acts. In the past, the U.S. accomplished its foreign policy goals by working around
the U.N., not through it. Washington's successful anti-Soviet containment policy, implemented with allies
as diverse as France, Germany, Turkey and Japan, proceeded independently of the U.N . U.S.-led efforts to stop
British and French seizure of the Suez Canal and to end the Israeli-Arab wars occurred with little help
from the U.N. NATO's intervention during the wars in the former Yugoslavia was, in large part, a violation of the U.N. Charter.
More recently, efforts to contain North Korea , to limit conflict between India and Pakistan , to keep the
peace between Taiwan and China and to remove Saddam Hussein have owed little or nothing to
the U.N. Historically, then, the U.N. has been mostly irrelevant, but with the end of the Cold War, it acquired new
prestige, based mainly on the hope that the relaxation of superpower tensions would finally allow it to act. The foreign veto-holders
on the Security Council have an interest in maintaining the U.N.'s new popularity because it gives them a kind of equality with the
U.S. that they lack in reality. Exploiting these rules, a single opponent can prevent action that a group
may prefer. Russia vetoed U.N. action in Kosovo. During the recent Iraq conflict, the delay caused by seeking U.N. approval
interfered with the timetable for the invasion, and the lack of approval gave some nations a free ride -- they had an excuse for not
joining the effort, a forum for objecting to it, and yet they could still benefit from it. If the U.N. continues its rise as a political forum
for opposition to U.S. military force, American diplomats should be marginalizing it, not strengthening it. The
U.N. also
undermines the advance of international law. Its charter outlaws war except in self-defense or
with the authorization of the Security Council -- a quixotic, unenforceable rule . There have been dozens
of wars since 1945 and the U.N's birth. Pretending that nations will not engage in war, and that the U.N.
can be the world's policeman, guaranteeing the safety of all, only breeds cynicism. It works against using
international law realistically to prevent humanitarian disasters, eliminate threats to regional peace, and stop state supporters of
terrorism. Not every U.N.-sponsored treaty has been a failure, but it is notable that the world's most successful treaties -- GATT and
the WTO, the European Union, arms control agreements and NATO -- have had little to do with the U.N. These treaties were
negotiated by states for well-defined purposes, and they have generated clear gains even if they also involve trade-offs reflecting
balance of power realities. By contrast, the U.N. is less effective because it is committed to an unattainable
ideal of equality among states, when it is not immobilized by the vetoes held by five of them.
The U.N.'s structure reflects post-World War II hopes that have not materialized . As long as ordinary
people continue to put faith in it, and governments play politics through it, the U.N. can only hinder international
cooperation and the advance of international law. The U.S. and its envoy must understand that demolition is the
order of the day -- and sometimes demolition is best accomplished from within.

Slow decision making

Perle 3. (Richard, Chair – Defense Policy Board, The Guardian, 3-21, Lexis)

But is the Security Council capable of ensuring order and saving us from anarchy? History suggests not. The
UN arose from the ashes of a war that the League of Nations was unable to avert. It was simply not up to confronting Italy in
Abyssinia, much less - had it survived that debacle - to taking on Nazi Germany. In the heady aftermath of the allied victory, the
hope that security could be made collective was embodied in the UN Security Council - with
abject results. During the Cold War the Security Council was hopelessly paralysed. The Soviet
empire was wrestled to the ground, and eastern Europe liberated, not by the UN, but by the mother of all
coalitions, NATO. Apart from minor skirmishes and sporadic peacekeeping missions, the only case
of the Security Council acting during the Cold War was its use of force to halt the invasion of South
Korea - and that was only possible because the Soviets were not in the chamber to veto it. It was a
mistake they did not make again. Facing Milosevic's multiple aggressions, the UN could not stop the Balkan
wars or even protect its victims. It took a coalition of the willing to save Bosnia from extinction. And when the war was
over, peace was made in Dayton, Ohio, not in the UN. The rescue of Muslims in Kosovo was not a UN action:
their cause never gained Security Council approval . The United Kingdom, not the United Nations, saved the
Falklands. This new century now challenges the hopes for a new world order in new ways. We will not defeat or even contain
fanatical terror unless we can carry the war to the territories from which it is launched. This will sometimes require that we use force
against states that harbour terrorists, as we did in destroying the Taliban regime in Afghanistan. The most dangerous of these states
are those that also possess weapons of mass destruction. Iraq is one, but there are others. Whatever hope there is that they can be
persuaded to withdraw support or sanctuary from terrorists rests on the certainty and effectiveness with which they are confronted.
The chronic failure of the Security Council to enforce its own resolutions is unmistakable: it is
simply not up to the task. We are left with coalitions of the willing. Far from disparaging them as a threat to a new world
order, we should recognise that they are, by default, the best hope for that order, and the true alternative to the anarchy of
the abject failure of the UN.

Recent developments

Tillawi 11. (Hesham, Host – Current Issues TV Program, “No Credibility Left for UN”, Press TV, 9-22,
http://presstv.com/detail/200556.html)

Since the UN does not have the power to stop these imperialist powers from controlling the world and from killing people wherever
they want and protecting Israel and being a cover for the Zionists, I think the United Nations days
are over with. We should
have another organization. After all, the world looks totally different than what it did back in 1945. Most
of these countries were not even countries back in 1945 . To revise the UN, I think the UN should be a
new organization taking into consideration all the countries that exist in the world. We should
totally have another organization. It is not realistic now but it should be in the future.

The UN is useless – outdated, slow, ineffective.


Schaefer 11. [Brett, Heritage International Regulatory Affairs Fellow "United Nations: Urgent Problems That Need
Congressional Action", February 3 -- http://www.heritage.org/research/lecture/2011/02/united-nations-urgent-problems-that-
need-congressional-action]

The United Nations was created to maintain international peace and security, promote self-determination and
basic human rights, and protect fundamental freedoms. Regrettably, the past 65 years have yielded more
disappointment than success in realizing these high aspirations. A great deal of the blame for this failure
is due to divergent interests among the member states that have prevented the organization from taking
decisive, timely action. However, the U.N. system itself is partly to blame. The U.N. and many of its
affiliated organizations are beset by outdated or duplicative missions and mandates, poor
management practices, ineffectual oversight, and a general lack of accountability. As former U.N.
Deputy Secretary-General Mark Malloch Brown recently stated to The New York Times, “There’s a huge redundancy and
lack of efficiency” in the U.N. system, and the budget is “utterly opaque, untransparent and
completely in shadow.”[1] These problems waste resources and undermine the U.N.’s ability to
discharge its responsibilities effectively. The past six decades have seen dozens of initiatives from governments, think
tanks, foundations, and panels of experts aimed at reforming the U.N. to make it more effective in meeting its responsibilities.[2]
Although these reform efforts have seen rare success, for the most part they have failed due to opposition from the majority of the
U.N. member states. Indeed, the U.S. almost always finds itself on the losing side in U.N. debates and votes when it proposes
reforms to improve U.N. management, oversight, and accountability. This happens because the bulk of the U.N. member
states simply do not pay enough to the U.N. for inefficiency , waste, or corruption to trouble them. For
instance, Sierra Leone is assessed 0.001 percent of the U.N.’s regular budget and 0.0001 percent of the peacekeeping budget. In
contrast, the U.S. is assessed 22 percent and 27.1 percent, respectively. Therefore, while Sierra Leone and the dozens of other
countries with the same assessments pay less than $35,000 per year to these budgets, the U.S. pays billions. With this in mind, is it
surprising that the U.S. cares about how the U.N. is managed and how the funding is used, while most countries do not? Yet these
are the countries that control most of the votes. The combined assessment of the 128 least-assessed countries—two-thirds of the
General Assembly—totals less than 1 percent of the U.N.’s regular budget and less than one-third of 1 percent of the peacekeeping
budget, even though that group alone can, according to U.N. rules, pass the budget. These countries, combined with influential
voting blocs in the U.N., can and do block U.S. attempts to implement reforms and curtail budgets.[3] There
is another
problem. American administrations are often interested in pressing for reform, but frequently
that long-term agenda is abandoned in favor of achieving more immediate political objectives.
Pressing for reform ruffles feathers at the U.N . When the U.S. is lobbying for votes on a resolution, the last thing
U.S. diplomats want to do is anger a mission by pressing for budgetary cuts or other reforms .

Global collective security fails

Carpenter 97
(Ted, head of foreign policy studies at CATO, “The Mirage of Global Collective Security Ted Galen Carpenter,” Delusions of grandeur:
the United Nations and global intervention, CATO, google books)

Any collective security system would be hard-pressed to succeed, given the extent of
disintegrative trends. But a global collective security system is especially impractical. It assumes that militarily
capable nations will choose to expend financial resources and put their troops at risk to deter or
suppress conflicts that have little or no relevance to their national security interests. Such manifestations of
altruism are exceedingly rare in the long history of international affairs. Moreover, it is not clear
that it would be wise to encourage great powers to meddle in local conflicts outside their own
regions even if they were so inclined. Among other problems, the globalist strategy increases the danger of friction
among those powers unless their agendas are fully compatible. Even if friction does not arise at the start of a UN-sponsored mission,
it can occur with a change of policy or regime in one of the major powers. Pat Buchanan rightly warns of the danger
of making minor conflicts the concern of outside great powers . Referring to the intervention in Bosnia, he
states that such a policy has already "made the Balkan war what anyone with a sense of history
hoped it would not again become: a playground for great power rivalry."'2

Alt cause—Security Council reform

Patrick, 9
Stewart Patrick, CFR Global Governance Program Director and Senior Fellow, October 2009, "Prix Fixe and a la Carte: Avoiding False
Multilateralism" http://www.twq.com/09october/docs/09oct_Patrick.pdf

Fifth, simply expanding international institutions is no panacea . Proponents of enlarging the Security Council,
adding members to the Group of 8 (G-8), or Five major obstacles impede a massive overhaul of the world’s bedrock institutions.
adjusting the voting shares of IFIs typically argue that such steps would make the relevant institution not only more representative
but also more effective. But as analyst Thomas Wright notes, these two goals often pull in opposite directions.17 All things being
equal, a larger body reflecting a diversity of global perspectives and interests will have higher international legitimacy, whereas a
more selective body will be capable of agile decisionmaking. Susan E. Rice, U.S. ambassador to the UN, alluded to these tensions in
February 2009, declaring: ‘‘The United States believes that the long-term legitimacy and viability of the United Nations Security
Council depends on its reflecting the world of the twenty-first century. At the same time, any expansion must not diminish its
effectiveness or efficiency.’’18 Therein lays the rub for Security Council expansion. Rather than facilitating
consensus on global flashpoints such as Darfur, Iran, and North Korea, council enlargement
might simply exacerbate overall cacophony and gridlock. Yet, failing to update leading multilateral
institutions is hardly attractive. It may be easier to make rules and take decisions in smaller bodies. But efficiency is not
the same as effectiveness, which implies giving decisions practical effect. Increasingly, global governance requires that emerging
market economies implement the decisions of authoritative bodies . If Western-dominated institutions fail to
adjust to geopolitical realities, they court a legitimacy crisis that will cripple global regimes. There
is already evidence of ‘‘soft balancing’’ by dissatisfied rising powers, which are creating competing regional or like-minded groupings
such as the Asian Monetary Fund, the East Asian Summit, and the Shanghai Cooperation Organization (SCO). In sum, many global
institutions suffer from a dual crisis: they are not particularly representative, nor especially
effective. But addressing one gap will not necessarily address the other.

UN fails – consensus

Krause 4 (Joachim, Professor of International Relations – University of Kiel and Member – Council of the International Institute
for Strategic Studies, “Multilateralism: Behind European Views” – Washington Quarterly, Spring,
http://www.twq.com/04spring/docs/04spring_krause.pdf)

U.S. critics of collective security do not just argue that the UN’s lack of enforcement capabilities are the
heart of the problem. The lack of unity among Security Council members is often cited as contributing
to the UN’s inability to ensure collective security , particularly in cases where astute dictators have
managed to pit various major powers against one another. This problem was evident in the cases
of Bosnia-Herzegovina and Kosovo, in which Russia made it clear that, irrespective of how serious Serb atrocities became,
it would never authorize the use of military force against Serbia. Even in cases when the Security Council was able
to agree on measures and sanctions against individual states, implementation usually was considered
inconsequential and halfhearted. For example, in the case of Bosnia-Herzegovina, the Muslim towns of Srebenica and
Zepa were declared “safe areas,” but the forces needed to protect them were not provided. Moreover, sanctions imposed
on countries usually have been subject to heavy erosion. Violators have often been able to
evade whatever sanctions are put in place, further undermining the credibility of collective security.
UUV Answers
Status quo solves- renewables

Quick, 10 -- GizMag writer


[Darren, "First robotic underwater vehicle to be powered entirely by natural, renewable, ocean thermal energy," 4-8-10,
www.gizmag.com/robotic-underwater-vehicle-powered-ocean-thermal-energy/14763/, accessed 1-27-13, mss]

We’ve covered a few underwater autonomous robots designed to make exploring the murky depths easier here on Gizmag, such as
Snookie and the Talisman, but none that can generate its own power – until now. NASA, US Navy and university researchers
have successfully demonstrated the first underwater vehicle to be powered entirely by natural,
renewable, ocean thermal energy. Scalable for use on most robotic oceanographic vehicles, this technological
breakthrough could usher in a new generation of autonomous underwater vehicles capable of
virtually indefinite ocean monitoring for climate and marine animal studies, exploration and surveillance. The Sounding
Oceanographic Lagrangrian Observer Thermal RECharging ( SOLO-TREC) autonomous underwater vehicle uses a novel
thermal recharging engine powered by the natural temperature differences found at different
ocean depths.

Alt cause- political will

Martin, 12 -- industry consultant and principal of Unmanned Vehicles Systems Consulting LLC [Antoine, "U.S. Expands Use Of
Underwater Unmanned Vehicles," April 2012,
www.nationaldefensemagazine.org/archive/2012/April/Pages/USExpandsUseOfUnderwaterUnmannedVehicles.aspx, accessed 1-27-
13, mss]

The Navy released a UUV “master plan” in 2004, and it is still relevant. Nine missions are identified: Intelligence,
surveillance and reconnaissance; mine countermeasures; anti-submarine warfare; inspection/identification; oceanography;
communication/navigation network nodes; payload delivery; information operations; and time-critical strike. Several
recommendations from the master plan have been initiated. Among them is the development of four UUV classes including one that
weighs less than 100 pounds, a lightweight vehicle at 500 pounds, a 3,000-pound heavy weight submersible and large submarine at
around 20,000 pounds. The plan also called for the development of standards and modularity, increased experimentation in the
technology, coordination with other unmanned vehicle programs, and the fielding of systems in the fleet. Those
recommendations have not been executed yet, which might explain why an updated roadmap has not been made
public. One reason for the delays is there is no major threat at hand for which underwater drones
are needed, such as roadside bombs that drove the rapid procurement of ground robots, or the
demands for intelligence gathering that fueled purchases of aerial surveillance drones.

Tech not viable

Martin, 12 -- industry consultant and principal of Unmanned Vehicles Systems Consulting LLC [Antoine, "U.S. Expands Use Of
Underwater Unmanned Vehicles," April 2012,
www.nationaldefensemagazine.org/archive/2012/April/Pages/USExpandsUseOfUnderwaterUnmannedVehicles.aspx, accessed 1-27-
13, mss]
The Office of Naval Research’s “Science & Strategy Plan 2011” projects continued development of unmanned systems. The priorities
are increasing their endurance and power, and becoming more reliable in harsh maritime environments. These goals
are
particularly challenging as sensitive electronics must operate for days or weeks at sea. The robots
are subjected to extreme pressure, corrosion , waves and currents. Poorly integrated technology
does not survive long under these circumstances . ONR will seek to develop underwater distributive networks,
through the use of unmanned drones, which will in turn provide information on perception and environmental changes. Increasing
the perception and intelligence of UUVs are recurring themes in Defense Department documents. A major hurdle for the
technology is the launch and recovery from other vehicles because of low speed, relatively low endurance and short-range
communication. Underwater robots are covert by nature because of their small size and low sonar signature. If the host platform has
to alter its operation to launch and recover one, it can be put at risk. This is especially sensitive when the launching and recovering is
done from submarines. With limited launch tubes, trading torpedoes for UUVs is a tough choice .

Renewables solve- Navy will use

Quick, 10 -- GizMag writer


[Darren, "First robotic underwater vehicle to be powered entirely by natural, renewable, ocean thermal energy," 4-8-10,
www.gizmag.com/robotic-underwater-vehicle-powered-ocean-thermal-energy/14763/, accessed 1-27-13, mss]

"Energy harvesting from the natural environment opens the door for a tremendous expansion in
the use of autonomous systems for naval and civilian applications," said Thomas Swean, the Office of Naval
Research program manager for SOLO-TREC. "This is particularly true for systems that spend most of their time submerged below the
sea surface, where mechanisms for converting energy are not as readily available. The JPL/Scripps concept is unique in that its
stored energy gets renewed naturally as the platform traverses ocean thermal gradients, so, in theory, the system has unlimited
range and endurance. This is a very significant advance ." SOLO-TREC is now in an extended mission with the JPL/Scripps
team planning to operate SOLO-TREC for many more months, if not years. The public can even keep tabs on SOLO-TREC's travels via
an online map. "The present thermal engine shows great promise in harvesting ocean thermal energy," said Russ Davis, a Scripps
oceanographer. "With further engineering refinement, SOLO-TREC has the potential to augment ocean monitoring currently done by
the 3,200 battery-powered Argo floats." The international Argo array measures temperature, salinity and velocity across the world's
ocean. NASA and the
US Navy also plans to apply this thermal recharging technology to the next
generation of submersible vehicles.
Venezuela UPDATE Threat Answers
Chavez is dead…

No Venezuela threat

Bandow, ’11 (Doug is a Senior Fellow at the Cato Institute. Juan Carlos Hidalgo is Project Coordinator for Latin America at the
Cato Institute's Center for Global Liberty and Prosperity, 1/12/11, “ Defusing Venezuela's Nuclear Threat?”
http://www.cato.org/pub_display.php?pub_id=12700)

Venezuela's close relationship with Iran and plans to build nuclear facilities with Russian help are raising
fears in Washington of another nuclear crisis. The incoming Republican House majority may place increased pressure on the Obama
administration to confront Caracas. Washington need not panic. A "Chávez bomb" is but a distant
possibility and much will happen in Venezuela in the meantime. The U.S. should work with other interested
states to discourage Caracas from pursuing nuclear weapons. Venezuela suffers from severe energy shortages —
primarily due to the Chávez government's mismanagement — and there's reason to doubt Chávez's claim that
his nuclear program is for purely peaceful purposes. For one — Chávez's arms purchases far outstrip his nation's security needs.
Over the last decade Caracas has purchased fighters, attack helicopters, antiaircraft missiles, and 100,000 assault rifles. Yet
Venezuela has been at peace since 1823 and faces no external threats. Yet even if Venezuela chooses to pursue
nuclear weapons, it's far from certain that Caracas will succeed. The difficult process requires
time, money, technology, and science. Developing nuclear weapons is even harder in the face of
international opposition. Moreover, creating weapons of deliverable size poses another
significant challenge. Despite Chávez's pretensions of global leadership, his corruption-ridden and inept
regime may be the biggest obstacle to a Venezuelan nuclear bomb. Worst is his gross economic
mismanagement despite the government's receipt of billions in oil revenues. The country's infrastructure is
crumbling. Last April an offshore drilling rig rented by PDVSA, Venezuela's state-owned oil
company, sank. The deal involved a questionable rental contract with former PDVSA executives and the accident was never
properly investigated. Earlier this year power blackouts caused by a series of explosions at electrical plants and inadequate
maintenance at the Guri hydro-electrical dam forced the government to impose electricity rationing. Venezuela's
transportation infrastructure is literally falling apart. The government agency that manages the country's food
supply let 120,000 tons of imported food rot in port while its own supermarkets suffered shortages of basic staples. Chávez's anti-
business policies discourage private investment. Although Caracas is a major oil supplier, it cannot easily
afford an expensive nuclear program. With the days of skyrocketing oil prices over, at least in the foreseeable future,
the government faces serious financial difficulties. For example, Chávez's regime owes
Colombian businesses approximately $500 million for past exports . PDVSA has delayed payments to its
contractors. After Chávez's allies lost the legislative elections in October, his government launched an expropriation spree but only
9 percent of the confiscated industries have been paid for. Moreover, Chávez is not certain to
retain power in the face of a contracting economy, staggering crime rate, unbridled corruption
and an increasingly united opposition. Even if he wins reelection in 2012, Chávez likely will find it
more difficult to achieve his international ambitions . Obviously, it would be foolish to dismiss the possibility of
Venezuela becoming a nuclear power, but it is equally mistaken to speak of "an over-the-horizon Cuban
Missile Crisis," in the words of the Heritage Foundation's Peter Brookes. Venezuela is nowhere close to or
certain of becoming a threat to the U.S.

Chavez is doomed – low oil prices destroyed his influence

Walter 8 (Matthew, Staff Writer – Bloomberg, “Chavez Ambitions in Venezuela May Fade With Oil Price”, Bloomberg News, 10-
27, http://www.bloomberg.com/apps/news?pid=20601109&sid=a4nOsWIX3sWs&refer=home)

The same tumbling oil prices that led OPEC to slash output last week threaten to send Venezuela's
economy into a tailspin, and put an end to President Hugo Chavez's ambitions to expand his
socialist revolution at home and abroad. To cope with plummeting oil revenue, the source of half the
government's spending, Chavez may have to cut domestic handouts and foreign aid. The first items likely
to go will be arms purchases from Russia, oil subsidies for Cuba, and job-creating local projects such as bridges
and subways, economists say. ``You have a country with an oil boom, that doesn't know how to save, doesn't know how to set
up productive industries that generate jobs, and goes into debt,'' said Elsa Cardozo, a professor of political science and
international relations at the Universidad Central de Venezuela. ``Then oil
prices fall and the party ends.''
Venezuela may be poised to repeat the economic collapse it suffered in the 1980s at the end of
its last oil boom. Former President Carlos Andres Perez, employing policies similar to Chavez's, lavished petrodollars on public
works projects, foreign aid and nationalizations in the late 1970s, setting the stage for a 1983 currency devaluation and
spending cuts that sent millions of Venezuelans into poverty. 'Most Vulnerable' `` Venezuela is now more dependent
than ever on oil,'' said Jose Toro Hardy, a former board member of state oil company Petroleos de Venezuela SA. ``Venezuela
is the most vulnerable country in all of Latin America to a falling oil price.'' Chavez
is already spending beyond his
means, posting a $7 billion budget deficit in the first half of 2008, a period of unprecedented oil prices, on a $63.9 billion
budget for the year. Economists' estimates of the minimum oil price Chavez needs to sustain his economic policies range from
$120 a barrel to $65. Oil fell 93 cents, or 1.4 percent, to a 17-month low of $63.22 a barrel today on the New York Mercantile
Exchange. Below $80 a barrel, it's likely that Chavez will devalue the bolivar for the first time since 2005, sparking a surge of
inflation and a drop in real wages because of Venezuela's reliance on imports, said Gustavo Garcia, an economics and public
finance professor at the Instituto de Estudios Superiores de Administracion, a Caracas business school. Oil Shock ``Depending on
the intensity of the shock, it could be a situation without precedent in Venezuelan history,'' said
Tamara Herrera, managing director at Caracas-based economic consulting company Sintesis Financiera. Venezuela's benchmark
9.25 bond due in 2027 fell 20.5 cents to 50.79 cents on the dollar from 79 cents a month earlier, pushing the yield to 18.83
percent, according to JPMorgan Chase & Co. The government, which has historically calculated its spending plans based on a
conservative forecast for oil prices, is projecting a $60-a-barrel average for the 2009 budget, and output well above today's
level. Oil options contracts to sell crude at $50 by December almost tripled Oct. 24. Venezuela, which pledged last week to trim
129,000 barrels a day from its production as part of the Organization of Petroleum Exporting Countries' 1.5 million barrel-a-day
cut, is the fourth-largest supplier of oil to the U.S. and the biggest petroleum exporter in the Americas. Aid, Arms Slashing
foreign aid and arms purchases, while diminishing Chavez's influence in the region, will
have the smallest political cost domestically, said Alejandro Grisanti, director of Latin American analysis at
Barclays Capital Inc.

Regional initiatives will check Chavez

Goodman 8 (Joshua, “Brazil, Colombia Plan Accord to Fortify Military Ties”, Bloomberg News, 7-17,
http://www6.miami.edu/UMH/CDA/UMH_Main/0,1770,39370-1;63355-3,00.html)
Brazil and Colombia plan to sign a defense accord to share military information, promote the
sale of weapons and conduct more joint training exercises, a Colombian diplomat taking part in the
negotiations said. The nine-page agreement, a draft copy of which was given to Bloomberg News, will be signed as early as this
weekend when Brazil's President Luiz Inacio Lula da Silva travels to Colombia tomorrow for two days of meetings with his
Colombian counterpart Alvaro Uribe, the diplomat said. He spoke on the condition that he not be identified because the plan
hasn't been made public. The initiative, which is being led by Brazil, would facilitate surveillance of the mostly lawless,
1,661-kilometer shared Amazon border, a transit zone for drug traffickers and guerrillas. It may curb the regional
influence of Venezuelan President Hugo Chavez, who threatened war with Colombia in retaliation for its
attack on a guerrilla camp in Ecuador. ``By strengthening its relations with Colombia , Brazil is trying to
reposition its relations with Chavez,'' said Susan Purcell, director of the Center for Hemispheric Policy at the
University of Miami. ``It sends a signal to other democratic governments in the region that they might
want to do the same.'' The Revolutionary Armed Forces of Colombia, or FARC, regularly cross into Brazil to smuggle
cocaine and weaponry fueling their insurgency, said Cesar Restrepo, an analyst at Bogota's Security and Democracy Foundation.
Justice Minister Tarso Genro in March said Brazil would build a new police base along the border to intensify controls. Broad
Blueprint The 10 articles of the agreement are a broad blueprint for cooperation in the areas of defense technology, training,
intelligence sharing, and weapons procurement. Article 6 would allow defense officials to sign ``complementary protocols in
specific areas of defense cooperation.'' One area of cooperation could be the Amazon, with the establishment of a joint military
base or check points along the border, said Tony Jozame, Colombia's ambassador to Brazil. Jozame, in a phone interview,
declined to confirm the countries are negotiating a defense agreement. ``With a willing partner like Brazil we'll always seek ways
to improve cooperation,'' he said. Calls for regional security cooperation took on new urgency after Colombia on March 1
crossed into Ecuador to kill a senior FARC commander. Chavez, in retaliation, ordered troops to the border. He parried back only
after Colombia alleged computers seized during the raid pointed to his longstanding financial and military support for the rebels.
Chavez denied the charge. `Brush Off' After an emergency session of the Washington-based Organization of American States
failed to defuse the crisis, Brazil, with Venezuela's support, started pushing a NATO-styled South American Defense Council. The
defense council got off to a slow start after Uribe refused to join, saying Colombia couldn't trust all of its neighbors. Chavez's
saber-rattling made it easier for Lula, a former trade unionist, to edge away from Chavez, says political analyst
Alexandre Barros. Thomas Shannon, U.S. assistant secretary of State for Western Hemisphere affairs, said in congressional
testimony today that ``Venezuela
has hit the limits of its international influence'' as traditional
regional leaders re-emerge. Barros, head of Early Warning, a Brasilia-based political risk research firm, said ``Brazil
will never publicly abandon Chavez, but a defense agreement with Colombia and other
recent actions are a clear brush-off.''

Chavez doesn’t sponsor terrorism – their ev is a myth

Ruyssenaars 5 (Henk, Senior Foreign Correspondent, FPF, 10-13, http://internetsamizdat.blogspot.com/2005/10/fair-parade-


magazines-smear-chavez.html)

As for scott's claim that chavez is funding "revolutionaries and terrorists throughout latin america," parade should provide
evidence for this charge or issue a retraction. The
US State Department, hardly a pro-Chavez source,
does not include Venezuela among the list of state sponsors of terrorism in its most recent
"Country Reports on Terrorism" (4/05). The report's section on Venezuela only alleges that "it is unclear to what extent"
Venezuela might support any of the principal combatants in Colombia's long-running civil war. The report goes on to mention,
however, that the Colombia and Venezuela governments have cooperated on some terrorism-
related cases, and that several Venezuelan National Guard officers were reportedly killed by FARC forces near the border
between the two countries. While some anti-Chavez partisans have claimed that Chavez supports
various terrorist groups, they have failed to back up these allegations with evidence .

Venezuela won’t cut off oil


Hansen 6 (Stephanie I., News Editor – CFR, “Hugo Chavez’s World Tour”, Council on Foreign Relations, 8-16,
http://www.cfr.org/publication/11285/hugo_chavezs_world_tour.html)

What would happen if Venezuela stopped selling the U nited States oil? It could result in a crude oil
price spike of up to $11 per barrel, according to a new study from the U.S. Government Accountability Office (PDF). Yet some
experts say the likelihood of such a move is low, given the asymmetry of the relationshi p
(Venezuelan oil comprises roughly 11 percent of U.S. oil imports and roughly half of Venezuelan
oil exports).

Chavez threat is totally overblown

Hansen 6 (Stephanie I., News Editor – CFR, “Hugo Chavez’s World Tour”, Council on Foreign Relations, 8-16,
http://www.cfr.org/publication/11285/hugo_chavezs_world_tour.html)

Some say warnings about Chavez's growing power are overstated. As CFR Senior Fellow Julia E. Sweig
writes in the Washington Post: "Theprospect of Chavez supplanting the ailing Castro as the new Latin
threat of the post-Cold War world is vastly overblown." And despite President Bush's own concern that Chavez is
"undermining democracy" in Venezuela and Latin America, some experts say his administration has yet to
translate that concern into concrete policy. But perhaps the appropriate stance on Chavez is somewhere in
between. "There is a view that dismisses him as a clown and sees him as a folkloric figure and I don't share that view," says
Shifter. "On the other hand, it's
a serious mistake to inflate his importance and say that he poses a
serious threat to the United States."
Warming Answers
Frontline

U.S can’t solve warming

Grose 13
(Thomas K., National Geographic News Writer, “As U.S. Cleans Its Energy Mix, It Ships Coal Problems Abroad”

Ready for some good news about the environment? Emissions of carbon dioxide in the United States are declining.
But don't celebrate just yet. A major side effect of that cleaner air in the U.S. has been the further darkening of skies over Europe
and Asia. The United States essentially is exporting a share of its greenhouse gas emissions in the form of coal, data
show. If the trend continues, the dramatic changes in energy use in the United States—in particular, the switch from coal to newly
abundant natural gas for generating electricity—will have only a modest impact on global warming, observers warn. The Earth's
atmosphere will continue to absorb heat-trapping CO2, with a similar contribution from U.S. coal. It will simply be burned overseas
instead of at home. "Switching from coal to gas only saves carbon if the coal stays in the ground," said John Broderick, lead author of
a study on the issue by the Tyndall Center for Climate Change Research at England's Manchester University. The U.S. Energy
Information Administration (EIA) released data this week showing that United States coal exports hit a record 126 million
short tons in 2012, a 17 percent increase over the previous year. Overseas shipments surpassed the previous high mark set in
1981 by 12 percent. The United States clearly is using less coal: Domestic consumption fell by about 114 million tons, or 11 percent,
largely due to a decline in the use of coal for electricity. But U.S. coal production fell just 7 percent. The United States, with the
world's largest coal reserves, continued to churn out the most carbon-intensive fuel, producing 1 billion tons of coal from its mines
in 2012. Emissions Sink The EIA estimates that due largely to the drop in coal-fired electricity, U.S. carbon emissions from burning
fossil fuel declined 3.4 percent in 2012. If the numbers hold up, it will extend the downward trend that the U.S. Environmental
Protection Agency (EPA) outlined last month in its annual greenhouse gas inventory, which found greenhouse gas emissions in 2011
had fallen 8 percent from their 2007 peak to 6,703 million metric tons of CO2 equivalent (a number that includes sources other than
energy, like methane emissions from agriculture). In fact, if you don't count the recession year of 2009, U.S. emissions in 2011
dropped to their lowest level since 1995. President Barack Obama counted the trend among his environmental accomplishments in
his State of the Union address last month: "Over the last four years, our emissions of the dangerous carbon pollution that threatens
our planet have actually fallen." The reason is clear: Coal, which in 2005 generated 50 percent of U.S. electricity, saw its share erode
to 37.4 percent in 2012, according to EIA's new short-term energy outlook. An increase in U.S. renewable energy certainly played a
role; renewables climbed in those seven years from 8.7 percent to 13 percent of the energy mix, about half of it hydropower. But
the big gain came from natural gas, which climbed from 19 percent to 30.4 percent of U.S. electricity during that time frame,
primarily because of abundant supply and low prices made possible by hydraulic fracturing, or fracking. The trend appears on track
to continue, with U.S. coal-fired plants being retired at a record pace. But U.S. coal producers haven't been standing still as their
domestic market has evaporated. They've been shipping their fuel to energy-hungry markets overseas, from the ports of Norfolk,
Baltimore, and New Orleans. Although demand is growing rapidly in Asia—U.S. coal exports to China were
on track to double last year—Europe was the biggest customer, importing more U.S. coal last year than all other countries
combined. The Netherlands, with Europe's largest port, Rotterdam, accepted the most shipments, on pace for a 24 jump in U.S. coal
imports in 2012. TheUnited Kingdom, the second largest customer, saw its U.S. coal imports jump more than 70
percent. The hike in European coal consumption would appear to run counter to big government initiatives across the Continent
to cut CO2 emissions. But in the European Union, where fracking has made only its initial forays and natural gas is still expensive,
American coal is, well, dirt cheap. European utilities are now finding that generating power from coal is a profitable
gambit. In the power industry, the profit margin for generating electricity from coal is called the "clean dark spread"; at the end of
December in Great Britain, it was going for about $39 per megawatt-hour, according to Argus. By contrast, the profit margin for gas-
fired plants—the "clean spark spread"—was about $3. Tomas Wyns, director of the Center for Clean Air Policy-Europe, a nonprofit
organization in Brussels, Belgium, said those kinds of spreads are typical across Europe right now. The
EU has a cap-and-
trade carbon market, the $148 billion, eight-year-old Emissions Trading System (ETS). But it's in the doldrums because
of a huge oversupply of permits. That's caused the price of carbon to fall to about 4 euros ($5.23). A plan called
"backloading" that would temporarily extract allowances from the market to shore up the price has faltered so far in the European
Parliament. "A better carbon price could make a difference" and even out the coal and gas spreads, Wyns said. He estimates a price
of between 20 and 40 euros would do the trick. "But a structural change to the Emissions Trading System is not something that will
happen very quickly. A solution is years off." The Tyndall Center study estimates that the burning of all that exported coal
could erase fully half the gains the United States has made in reducing carbon emissions. For huge reserves of
shale gas to help cut CO2 emissions, "displaced fuels must be reduced globally and remain suppressed indefinitely," the report said.
Future Emissions It is not clear that the surge in U.S. coal exports will continue. One reason for the uptick in coal-fired generation in
Europe has been the looming deadline for the EU's Large Combustion Plant Directive, which will require older coal plants to meet
lower emission levels by the end of 2015 or be mothballed. Before that phaseout begins, Wyns says, " there
is a bit of a binge
going on." Also, economic factors are at work. Tyndall's Broderick said American coal companies have been essentially selling
surplus fuel overseas at low profit margins, so there is a likelihood that U.S. coal production will decrease further. The U.S.
government forecasters at EIA expect that U.S. coal exports will fall back to about 110 million tons per year over the next two years,
due to economic weakness in Europe, falling international prices, and competition from other coal-exporting countries. The Paris-
based International Energy Agency (IEA) calls Europe's "coal renaissance" a temporary phenomenon; it forecasts an increasing use of
renewables, shuttering of coal plants, and a better balance between gas and coal prices in the coming years. But IEA
does not
expect that the global appetite for coal will slacken appreciably. The agency projects that, by 2017, coal will
rival oil as the world's primary energy source, mainly because of skyrocketing demand in Asia.
U.S. coal producers have made clear that they aim to tap into that growing market.

They have no effect on CO2

Carnegie Institute 12
Carnegie Institute of Science, February 16, 2012, "Only the lowest CO2 emitting technologies can avoid a hot end-of-century",
http://carnegiescience.edu/news/only_lowest_co2_emitting_technologies_can_avoid_hot_endofcentury

Washington, D.C.— Could replacing coal-fired electricity plants with generators fueled by natural gas bring global warming to a halt
in this century? What about rapid construction of massive numbers of solar or wind farms, hydroelectric dams, or nuclear reactors—
or the invention of new technology for capturing the carbon dioxide produced by fossil-fueled power plants and storing it
permanently underground? Nathan Myhrvold of Intellectual Ventures teamed up with Carnegie Institution’s Ken Caldeira to
calculate the expected climate effects of replacing the world’s supply of electricity from coal plants with any of eight cleaner options.
The work was published online by Environmental Research Letters on February 16. When published, it will be available at
http://iopscience.iop.org/1748-9326/7/1/014019. In each case, Myhrvold and Caldeira found that to achieve substantial benefit this
century, we would need to engage in a rapid transition to the lowest emitting energy technologies such as solar, wind, or nuclear
power – as well as conserve energy where possible. The researchers found that it takes much longer to curtail the
warming of the Earth than one might expect. And in the case of natural gas—increasingly the power industry’s
fuel of choice, because gas reserves have been growing and prices have been falling— the study finds that warming would
continue even if over the next 40 years every coal-fired power plant in the world were replaced with a
gas-fueled plant. “There is no quick fix to global warming,” Caldeira said. “Shifting from one energy system to
another is hard work and a slow process. Plus, it takes several decades for the climate system to fully
respond to reductions in emissions. If we expect to see substantial benefits in the second half of this century, we had better get
started now.” Researchers have previously conducted studies projecting the long-term climate effects of rolling out a single new
energy technology. But this work from Myhrvold and Caldeira is the first to examine all the major candidate technologies for
replacing coal power—including conservation—and to examine wide ranges of possible assumptions about both the emissions each
technology generates and also the scope and duration of the build-out. “It takes a lot of energy to make new power plants—and it
generally takes more energy to make those that use cleaner technology--like nuclear, solar, and wind--than it does to make dirty
ones that burn coal and gas,” Myhrvold added. “You have to use the energy system of today to build the new-and-improved energy
system of tomorrow, and unfortunately that means creating more emission in the near-term than we would otherwise. So we incur
a kind of ‘emissions debt’ in making the transition to a better system, and it can take decades to pay that off. Meanwhile, the
temperature keeps rising.” The
study used widely accepted models relating emissions to temperature. The two
researchers also drew on a rich literature of studies, called life-cycle analyses, that total up all the greenhouse
gases produced during the construction and operation of, say, a natural gas plant or a hydroelectric dam or a solar photovoltaic
farm. It also examined the potential that technological improvements, such as advances in carbon capture and storage or in solar
panel efficiency, could have on outcomes. “It was surprising to us just how long it takes for the benefit of a switch from coal to
something better to show up in the climate in the form of a slowdown in global warming,” Caldeira said. “If countries were to
start right away and build really fast, so that they installed a trillion watts of gas-fired electricity generation
steadily over the next 40 years,” Myhrvold said, “that would still add about half a degree Fahrenheit to the
average surface temperature of the Earth in 2112—that’s within a tenth of a degree of the warming
that coal-fired plants would produce by that year .”

Can’t solve warming

AP 9 (Associated Press, Six Degree Temperature Rise by 2100 is Inevitable: UNEP, September 24, http://www.speedy-
fit.co.uk/index2.php?option=com_content&do_pdf=1&id=168)

Earth's temperature is likely to jump six degrees between now and the end of the century even
if every country cuts greenhouse gas emissions as proposed, according to a United Nations update.
Scientists looked at emission plans from 192 nations and calculated what would happen to global
warming. The projections take into account 80 percent emission cuts from the U.S. and Europe by
2050, which are not sure things. The U.S. figure is based on a bill that passed the House of Representatives but is running
into resistance in the Senate, where debate has been delayed by health care reform efforts. Carbon dioxide, mostly from the burning
of fossil fuels such as coal and oil, is the main cause of global warming, trapping the sun's energy in the atmosphere. The world's
average temperature has already risen 1.4 degrees since the 19th century. Much
of projected rise in temperature is
because of developing nations, which aren't talking much about cutting their emissions , scientists
said at a United Nations press conference Thursday. China alone adds nearly 2 degrees to the projections. "We are
headed toward very serious changes in our planet," said Achim Steiner, head of the U.N.'s environment program, which issued the
update on Thursday. The review looked at some 400 peer-reviewed papers on climate over the last three years. Even if the
developed world cuts its emissions by 80 percent and the developing world cuts theirs in half
by 2050, as some experts propose, the world is still facing a 3-degree increase by the end of the
century, said Robert Corell, a prominent U.S. climate scientist who helped oversee the update. Corell said the most likely
agreement out of the international climate negotiations in Copenhagen in December still translates into a nearly 5-degree increase
in world temperature by the end of the century. European leaders and the Obama White House have set a goal to limit warming to
just a couple degrees. The U.N.'s environment program unveiled the update on peer-reviewed climate change science to tell
diplomats how hot the planet is getting. The last big report from the Nobel Prize-winning Intergovernmental Panel on Climate
Change came out more than two years ago and is based on science that is at least three to four years old, Steiner said. Global
warming is speeding up, especially in the Arctic, and that means that some top-level science projections from 2007 are already out
of date and overly optimistic. Corell, who headed an assessment of warming in the Arctic, said global warming "is accelerating in
ways that we are not anticipating." Because Greenland and West Antarctic ice sheets are melting far faster than thought, it looks
like the seas will rise twice as fast as projected just three years ago, Corell said. He said seas should rise about a foot every 20 to 25
years.

Warming won’t cause extinction


Barrett ‘7 professor of natural resource economics – Columbia University, (Scott, Why Cooperate? The Incentive to Supply
Global Public Goods, introduction)

First, climate change does not threaten the survival of the human species.5 If unchecked, it will cause
other species to become extinction (though biodiversity is being depleted now due to other reasons ). It will
alter critical ecosystems (though this is also happening now, and for reasons unrelated to climate
change). It will reduce land area as the seas rise, and in the process displace human populations. “Catastrophic” climate
change is possible, but not certain. Moreover, and unlike an asteroid collision, large changes (such as sea level
rise of, say, ten meters) will likely take centuries to unfold, giving societies time to adjust. “Abrupt”
climate change is also possible, and will occur more rapidly, perhaps over a decade or two. However, abrupt climate change
(such as a weakening in the North Atlantic circulation), though potentially very serious, is unlikely to be ruinous. Human-
induced climate change is an experiment of planetary proportions, and we cannot be sur of its consequences. Even in a worse
case scenario, however, global climate change is not the equivalent of the Earth being hit by mega-
asteroid. Indeed, if it were as damaging as this, and if we were sure that it would be this harmful ,
then our incentive to address this threat would be overwhelming. The challenge would still be more difficult
than asteroid defense, but we would have done much more about it by now.

CO2 isn’t key


Watts ’12 25-year climate reporter, works with weather technology, weather stations, and weather data processing systems in
the private sector, 7/25/

(Anthony, http://wattsupwiththat.com/2012/07/25/lindzen-at-sandia-national-labs-climate-models-are-flawed/)

ALBUQUERQUE, N.M. — Massachusetts Institute of Technology professor Richard Lindzen, a global warming skeptic, told about
70 Sandia researchers in June that too much is being made of climate change by researchers seeking
government funding. He said their data and their methods did not support their claims. “ Despite concerns over the last
decades with the greenhouse process, they oversimplify the effect,” he said. “Simply cranking up CO2
[carbon dioxide] (as the culprit ) is not the answer” to what causes climate change. Lindzen, the ninth
speaker in Sandia’s Climate Change and National Security Speaker Series, is Alfred P. Sloan professor of meteorology in
MIT’s department of earth, atmospheric and planetary sciences. He has published more than
200 scientific papers and is the lead author of Chapter 7 (“Physical Climate Processes and Feedbacks”) of the
International Panel on Climate Change’s (IPCC) Third Assessment Report. He is a member of the National
Academy of Sciences and a fellow of the American Geophysical Union and the American
Meteorological Society. For 30 years, climate scientists have been “locked into a simple-minded
identification of climate with greenhouse-gas level. … That climate should be the function of a
single parameter (like CO2) has always seemed implausible. Yet an obsessive focus on such an
obvious oversimplification has likely set back progress by decades,” Lindzen said. For major
climates of the past, other factors were more important than carbon dioxide. Orbital
variations have been shown to quantitatively account for the cycles of glaciations of the past 700,000
years, he said, and the elimination of the arctic inversion, when the polar caps were ice-free, “is
likely to have been more important than CO2 for the warm episode during the Eocene 50 million
years ago.” There is little evidence that changes in climate are producing extreme weather events, he said. “Even the IPCC says there
is little if any evidence of this. In fact, there are important physical reasons for doubting such anticipations.” Lindzen’s views run
counter to those of almost all major professional societies. For example, the American Physical Society statement of Nov. 18, 2007,
read, “The evidence is incontrovertible: Global warming is occurring.” But he doesn’t feel they are necessarily right. “Why did the
American Physical Society take a position?” he asked his audience. “Why did they find it compelling? They never answered.”
Speaking methodically with flashes of humor — “I always feel that when the conversation turns to weather, people are bored.” —
he said a basic problem with current computer climate models that show disastrous increases in temperature is that relatively small
increases in atmospheric gases lead to large changes in temperatures in the models. But, he said, “ predictions based on
high (climate) sensitivity ran well ahead of observations.” Real-world observations do not support
IPCC models, he said: “We’ve already seen almost the equivalent of a doubling of CO2 (in radiative
forcing) and that has produced very little warming.” He disparaged proving the worth of models by applying
their criteria to the prediction of past climatic events, saying, “The models are no more valuable than answering a
test when you have the questions in advance.” Modelers , he said, merely have used aerosols as a
kind of fudge factor to make their models come out right. (Aerosols are tiny particles that reflect sunlight. They
are put in the air by industrial or volcanic processes and are considered a possible cause of temperature change at Earth’s surface.)
Then there is the practical question of what can be done about temperature increases even if they are occurring, he said. “China,
India, Korea are not going to go along with IPCC recommendations, so … the only countries punished will be those who go along with
the recommendations.” He discounted mainstream opinion that climate change could hurt national security, saying that “historically
there is little evidence of natural disasters leading to war, but economic conditions have proven much more serious. Almost all
proposed mitigation policies lead to reduced energy availability and higher energy costs. All studies of human benefit and national
security perspectives show that increased energy is important.” He showed a graph that demonstrated that more energy
consumption leads to higher literacy rate, lower infant mortality and a lower number of children per woman. Given that proposed
policies are unlikely to significantly influence climate and that lower energy availability could be considered a significant threat to
national security, to continue with a mitigation policy that reduces available energy “would, at the least, appear to be irresponsible,”
he argued. Responding to audience questions about rising temperatures, he said a
0.8 of a degree C change in
temperature in 150 years is a small change. Questioned about five-, seven-, and 17-year averages that
seem to show that Earth’s surface temperature is rising, he said temperatures are always
fluctuating by tenths of a degree.

We’ll adapt

Kenny 12 [April 9, 2012, Charles, senior fellow at the Center for Global Development, a Schwartz fellow at the New America
Foundation, and author, most recently, of Getting Better: Why Global Development Is Succeeding and How We Can Improve the
World Even More., “Not Too Hot to Handle,” http://www.foreignpolicy.com/articles/2012/04/09/not_too_hot_to_handle?
print=yes&hidecomments=yes&page=full]

But for all international diplomats appear desperate to affirm the self-worth of pessimists and doomsayers worldwide, it is
important to put climate change in a broader context. It is a vital global issue -- one that threatens to slow the
worldwide march toward improved quality of life. Climate change is already responsible for more extreme weather and an
accelerating rate of species extinction -- and may ultimately kill off as many as 40 percent of all living species. But it
is also a
problem that we know how to tackle, and one to which we have some time to respond before it
is likely to completely derail progress. And that's good news, because the fact that it's manageable is the best reason
to try to tackle it rather than abandon all hope like a steerage class passenger in the bowels of the Titanic.

Start with the economy. The Stern Review, led by the distinguished British economist Nicholas Stern, is the most comprehensive look
to date at the economics of climate change. It suggests that, in
terms of income, greenhouse gasses are a threat to
global growth, but hardly an immediate or catastrophic one. Take the impact of climate change on the
developing world. The most depressing forecast in terms of developing country growth in Stern's paper is the "A2 scenario" -- one of
a series of economic and greenhouse gas emissions forecasts created for the U.N.'s Intergovernmental Panel on Climate Change
(IPCC). It's a model that predicts slow global growth and income convergence (poor countries catching up to rich countries). But
even under this model, Afghanistan's GDP per capita climbs sixfold over the next 90 years, India
and China ninefold, and Ethiopia's income increases by a factor of 10. Knock off a third for the most
pessimistic simulation of the economic impact of climate change suggested by the Stern report, and people in those
countries are still markedly better off -- four times as rich for Afghanistan, a little more than six times as rich for
Ethiopia.
It's worth emphasizing that the
Stern report suggests that the costs of dramatically reducing
greenhouse-gas emissions is closer to 1 (or maybe 2) percent of world GDP -- in the region of $600 billion to
$1.2 trillion today. The economic case for responding to climate change by pricing carbon and investing in alternate energy sources
is a slam dunk. But for all the likelihood that the world will be a poorer, denuded place than it would be if we responded rapidly to
reduce greenhouse gases, the global economy is probably not going to collapse over the next century
even if we are idiotic enough to delay our response to climate change by a few years. For all the flooding, the drought,
and the skyrocketing bills for air conditioning, the economy would keep on expanding , according to the data that
Stern uses.

And what about the impact on global health? Suggestions that malaria has already spread as a result of climate change and that
malaria deaths will expand dramatically as a result of warming in the future don't fit the evidence of declining deaths and reduced
malarial spread over the last century. The authors of a recent study published in the journal Nature conclude that the forecasted
future effects of rising temperatures on malaria "are at least one order of magnitude smaller than the changes observed since about
1900 and about two orders of magnitude smaller than those that can be achieved by the effective scale-up of key control measures."
In other words, climate change is and will likely remain a small factor in the toll of malaria deaths into the foreseeable future.

What about other diseases? Christian Zimmermann at the University of Connecticut and Douglas Gollin at Williams
evaluate the likely impact of a 3-degree rise in temperatures on tropical diseases like dengue fever,
which causes half a million cases of hemorrhagic fever and 22,000 deaths each year. Most of the vectors for such diseases --
mosquitoes, biting flies, and so on -- do poorly in frost. So if the weather stays warmer, these diseases are likely to spread. At the
same time, there are existing tools to prevent or treat most tropical diseases, and Zimmerman and Gollin
suggest "rather modest improvements in protection efficacy could compensate for the consequences of
climate change." We can deal with this one.

It's the same with agriculture. Global warming will have many negative (and a few positive) impacts on food supply, but it
is likely that other impacts -- both positive, including technological change , and negative, like the exhaustion of
aquifers-- will have far bigger effects. The 2001 IPCC report suggested that climate change over the long term could
reduce agricultural yields by as much as 30 percent. Compare that with the 90 percent increase in rice yields in Indonesia between
1970 and 2006, for example.

Again, while climate change will make extreme weather events and natural disasters like flooding and
hurricanes more common, the negative effect on global quality of life will be reduced if economies
continue to grow. That's because, as Matthew Kahn from Tufts University has shown, the safest place to suffer a
natural disaster is in a rich country. The more money that people and governments have, the more they can both afford
and enforce building codes, land use regulations, and public infrastructure like flood defenses that lower death tolls.

Let's also not forget how human psychology works. Too many environmentalists suggest that dealing with climate change
will take immediate and radical retooling of the global economy. It won't. It is affordable, practical, and wouldn't take
a revolution. Giving out the message that the only path to sustainability will require medieval standards of living only puts
everyone else off. And once you've convinced yourself the world is on an inevitable course to disaster if some corner of the U.S.
Midwest is fracked once more or India builds another three coal-fueled power plants, the only logical thing to do when the fracking
or the building occurs is to sit back, put your Toms shoes on the couch, and drink micro-brewed herbal tea until civilization collapses.
Climate change isn't like that -- or at the very least, isn't like that yet.

So, if
you're really just looking for a reason to strap on the "end of the world is nigh" placards and
go for a walk, you can find better excuses -- like, say, the threat of global thermonuclear war or a rogue
asteroid. The fight to curb greenhouse gas emissions is one for the hard-nosed optimist.
----------dumb args below.

No warming and c02 doesn’t cause it- newest data

Taylor ’11 (7/27- senior fellow for environment policy at the Heartland Institute (2011, “New NASA Data Blow Gaping Hole In
Global Warming Alarmism,” Forbes, http://blogs.forbes.com/jamestaylor/2011/07/27/new-nasa-data-blow-gaping-hold-in-global-
warming-alarmism/)

NASA satellite data from the years 2000 through 2011 show the Earth’s atmosphere is allowing far
more heat to be released into space than alarmist computer models have predicted, reports a
new study in the peer-reviewed science journal Remote Sensing . The study indicates far less
future global warming will occur than United Nations computer models have predicted, and supports
prior studies indicating increases in atmospheric carbon dioxide trap far less heat than alarmists have claimed. Study co-author Dr.
Roy Spencer, a principal research scientist at the University of Alabama in Huntsville and U.S. Science Team Leader for the Advanced
Microwave Scanning Radiometer flying on NASA’s Aqua satellite, reports that real-world data from NASA’s Terra satellite contradict
multiple assumptions fed into alarmist computer models. “The satellite observations suggest there is much
more energy lost to space during and after warming than the climate models show ,” Spencer said in a
July 26 University of Alabama press release. “ There is a huge discrepancy between the data and the
forecasts that is especially big over the oceans.” In addition to finding that far less heat is being trapped than
alarmist computer models have predicted, the NASA satellite data show the atmosphere begins shedding
heat into space long before United Nations computer models predicted. The new findings are
extremely important and should dramatically alter the global warming debate . Scientists on all sides
of the global warming debate are in general agreement about how much heat is being directly trapped by human emissions of
carbon dioxide (the answer is “not much”). However, the
single most important issue in the global warming
debate is whether carbon dioxide emissions will indirectly trap far more heat by causing large
increases in atmospheric humidity and cirrus clouds . Alarmist computer models assume human carbon dioxide
emissions indirectly cause substantial increases in atmospheric humidity and cirrus clouds (each of which are very effective at
trapping heat),but real-world data have long shown that carbon dioxide emissions are not causing
as much atmospheric humidity and cirrus clouds as the alarmist computer models have
predicted. The new NASA Terra satellite data are consistent with long-term NOAA and NASA
data indicating atmospheric humidity and cirrus clouds are not increasing in the manner predicted by alarmist computer models.
The Terra satellite data also support data collected by NASA’s ERBS satellite showing far more longwave radiation (and thus, heat)
escaped into space between 1985 and 1999 than alarmist computer models had predicted. Together, the NASA ERBS and Terra
satellite data show that for 25 years and counting, carbon dioxide emissions have directly and indirectly trapped far less heat than
alarmist computer models have predicted. In short, the central premise of alarmist global warming theory is that carbon dioxide
emissions should be directly and indirectly trapping a certain amount of heat in the earth’s atmosphere and preventing it from
escaping into space. Real-world measurements, however, show far less heat is being trapped in the earth’s atmosphere than the
When
alarmist computer models predict, and far more heat is escaping into space than the alarmist computer models predict.
objective NASA satellite data, reported in a peer-reviewed scientific journal, show a “huge
discrepancy” between alarmist climate models and real-world facts, climate scientists , the
media and our elected officials would be wise to take notice . Whether or not they do so will tell us a great
deal about how honest the purveyors of global warming alarmism truly are.

Negative feedbacks solve and no tipping point


McShane 8 (Owen, the chairman of the policy panel of the New Zealand Climate Science Coalition and director of the Center
for Resource Management Studies, 4-4-8, The National Business Review (New Zealand), “Climate change confirmed but global
warming is cancelled”, Lexis)

Atmospheric scientists generally agree that as carbon dioxide levels increase there is a law
of "diminishing returns" - or more properly " diminishing effects " - and that ongoing
increases in CO2 concentration do not generate proportional increases in temperature. The
common analogy is painting over window glass. The first layers of paint cut out lots of light but
subsequent layers have diminishing impact . So, you might be asking, why the panic? Why does Al Gore talk
about temperatures spiraling out of control, causing mass extinctions and catastrophic rises
in sea-level, and all his other disastrous outcomes when there is no evidence to support it?
The alarmists argue that increased CO2 leads to more water vapour - the main greenhouse gas - and this provides positive
feedback and hence makes the overall climate highly sensitive to small increases in the concentration of CO2. Consequently, the
IPCC argues that while carbon dioxide may well "run out of puff" the consequent evaporation of water vapour provides the
This assumption that
positive feedback loop that will make anthropogenic global warming reach dangerous levels.
water vapour provides positive feedback lies behind the famous "tipping point," which
nourishes Al Gore's dreams of destruction, and indeed all those calls for action now - "before it is too late!" But
no climate models predict such a tipping point . However, while the absence of hot spots has refuted one
important aspect of the IPCC models we lack a mechanism that fully explains these supposed outcomes. Hence the IPCC, and its
supporters, have been able to ignore this "refutation." So by the end of last year, we were in a similar situation to the 19th
century astronomers, who had figured out that the sun could not be "burning" its fuel - or it would have turned to ashes long
ago - but could not explain where the energy was coming from. Then along came Einstein and E=mc2. Hard to explain Similarly,
the climate sceptics have had to explain why the hotspots are not where they should be - not
just challenge the theory with their observations . This is why I felt so lucky to be in the right place at the right
time when I heard Roy Spencer speak at the New York conference on climate change in March. At first I thought this was just
another paper setting out observations against the forecasts, further confirming Evans' earlier work. But as the argument
unfolded I realised Spencer was drawing on observations
and measurements from the new Aqua
satellites to explain the mechanism behind this anomaly between model forecasts and
observation. You may have heard that the IPCC models cannot predict clouds and rain with any
accuracy. Their models assume water vapour goes up to the troposphere and hangs around
to cook us all in a greenhouse future. However, there is a mechanism at work that "washes
out" the water vapour and returns it to the oceans along with the extra CO2 and thus turns
the added water vapour into a NEGATIVE feedback mechanism. The newly discovered mechanism is a
combination of clouds and rain (Spencer's mechanism adds to the mechanism earlier identified by Professor Richard Lindzen
called the Iris effect). The IPCC models assumed water vapour formed clouds at high altitudes
that lead to further warming. The Aqua satellite observations and Spencer's analysis show
water vapour actually forms clouds at low altitudes that lead to cooling . Furthermore, Spencer
shows the extra rain that falls from these clouds cools the underlying oceans, providing a
second negative feedback to negate the CO2 warming . Alarmists' quandary This has struck the alarmists
like a thunderbolt, especially as the
lead author of the IPCC chapter on feedback has written to
Spencer agreeing that he is right! There goes the alarmist neighbourhood!

Natural variability explains warming trends


Idso ‘12, director of envt science – Peabody Energy, PhD Geography – ASU, Idso, professor –
Maricopa County Community College, and Idso, PhD botany – ASU,

(Craig, Sherwood, and Keith, “Northern Scandinavian Temperatures: It's a Whole New Ball
Game,” CO2 Science Vol. 15, No. 30, July)

In a game-changing paper published in the online version of Nature Climate Change, Esper et al.
(8 July 2012) provide convincing evidence that both the Medieval and Roman Warm Periods of
1000 and 2000 years ago, respectively, were warmer than the Current Warm Period has been to
date, in spite of the fact that today's atmospheric CO2 concentration is some 40% greater than
it was during those two earlier periods.

In setting the stage for their paradigm-altering work, the twelve researchers - hailing from
Finland, Germany, Scotland and Switzerland - write that "solar insolation changes, resulting from
long-term oscillations of orbital configurations (Milankovitch, 1941), are an important driver of
Holocene climate," referencing the studies of Mayewski et al. (2004) and Wanner et al. (2008).
In addition, they state that this forcing has been "substantial over the past 2000 years, up to
four times as large as the 1.6 W/m2 net anthropogenic forcing since 1750," as suggested by
the work of Berger and Loutre (1991). And on the basis of "numerous high-latitude proxy
records," as they describe it, they note that "slow orbital changes have recently been shown to
gradually force boreal summer temperature cooling over the common era," citing Kaufman et al.
(2009).

Fast-forwarding to the present, Esper et al. describe how they developed "a 2000-year summer
temperature reconstruction based on 587 high-precision maximum latewood density (MXD)
series from northern Scandinavia," which feat was accomplished "over three years using living
and subfossil pine (Pinus sylvestris) trees from 14 lakes and 3 lakeshore sites above 65°N,
making it not only longer but also much better replicated than any existing MXD time series."
Then, after calibrating the pine MXD series against regional June-July-August mean temperature
over the period 1876-2006, they obtained their final summer temperature history for the period
stretching from 138 BC to AD 2006, as depicted in the graph below.

As determined from the relationship depicted in the figure above, Esper et al. calculate a long-
term cooling trend of -0.31 ± 0.03°C per thousand years, which cooling they say is "missing in
published tree-ring proxy records" but is "in line with coupled general circulation models (Zorita
et al., 2005; Fischer and Jungclaus, 2011)," which computational results portray, as they describe
it: substantial summer cooling over the past two millennia in northern boreal and Arctic
latitudes.
"These findings," as the European researchers continue, "together with the missing orbital
signature in published dendrochronological records, suggest that large-scale near-surface air
temperature reconstructions (Mann et al., 1999; Esper et al., 2002; Frank et al., 2007; Hegerl et
al., 2007; Mann et al., 2008) relying on tree-ring data may underestimate pre-instrumental
temperatures including warmth during Medieval and Roman times," although they suggest that
the impacts of the omitted long-term trend in basic tree-ring data may "diminish towards lower
Northern Hemisphere latitudes, as the forcing and radiative feedbacks decrease towards
equatorial regions."

And so it is that the question for our day ought to be: Why was much of the CO2-starved world
of Medieval and Roman times decidedly warmer (by about 0.3 and 0.5°C, respectively) than it
was during the peak warmth of the 20th century? Clearly, the greenhouse effect of atmospheric
CO2 - if it has not been grossly over-estimated - must currently be being significantly tempered
by some unappreciated CO2- and/or warming-induced negative-feedback phenomenon
(possibly of biological origin) to the degree that the basic greenhouse effect of earth's rising
atmospheric CO2 concentration cannot fully compensate for the decrease in solar insolation
experienced over the past two millennia as a result of the "long-term oscillations of orbital
configurations" cited by Esper et al. (2012).
---EXT Can’t Solve

Asia pollution offsets any US action – global warming is inevitable

Knappenberger ‘12 (Paul Knappenberger, Assistant Director of the Cato Institute’s Center for the Study of Science. He
holds an M.S. degree in Environmental Sciences (1990) from the University of Virginia as well as a B.A. degree in Environmental
Sciences (1986) from the same institution.His over 20 years of experience as a climate researcher have included 10 years with the
Virginia State Climatology Office and 13 years with New Hope Environmental Services, Inc., "Asian Air Pollution Warms U.S More
than Our GHG Emissions (More futility for U.S. EPA)", www.masterresource.org/2012/06/asian-air-pollution-warming/, June 7, 2012)

“The whims of foreign nations, not to mention Mother Nature, can completely offset any climate changes induced by U.S.
greenhouse gas emissions reductions…. So, what’s the point of forcing Americans into different energy choices?” A
new study
provides evidence that air pollution emanating from Asia will warm the U.S. as much or more
than warming from U.S. greenhouse gas (GHG) emissions. The implication? Efforts by the U.S.
Environmental Protection Agency (and otherwise) to mitigate anthropogenic climate change is moot.¶ If
the future temperature rise in the U.S. is subject to the whims of Asian environmental and energy policy, then what sense does it
make for Americans to have their energy choices regulated by efforts aimed at mitigating future temperature increases across the
country—efforts which will have less of an impact on temperatures than the policies enacted across Asia? Maybe the
EPA
should reconsider the perceived effectiveness of its greenhouse gas emission regulations—at least
when it comes to impacting temperatures across the U.S. New Study A new study just published in the scientific
journal Geophysical Research Letters is authored by a team led by Haiyan Teng from the
National Center for Atmospheric Research, in Boulder, Colorado. The paper is titled “Potential Impacts of Asian
Carbon Aerosols on Future US Warming.” Skipping the details of this climate modeling study and cutting to the chase, here is the
abstract of the paper: This
study uses an atmosphere-ocean fully coupled climate model to investigate
possible remote impacts of Asian carbonaceous aerosols on US climate change. We took a 21st century
mitigation scenario as a reference, and carried out three sets of sensitivity experiments in which the prescribed carbonaceous
aerosol concentrations over a selected Asian domain are increased by a factor of two, six, and ten respectively during the period of
2005–2024. The resulting enhancement of atmospheric solar absorption (only the direct effect of
aerosols is included) over Asia induces tropospheric heating anomalies that force large-scale circulation changes
which, averaged over the twenty-year period, add as much as an additional 0.4°C warming over the eastern US
during winter and over most of the US during summer. Such remote impacts are confirmed by an atmosphere stand-alone
experiment with specified heating anomalies over Asia that represent the direct effect of the carbon aerosols. Usually, when
considering the climate impact from carbon aerosol emissions (primarily in the form of black carbon, or soot), the effect is thought
to be largely contained to the local or regional scale because the atmospheric lifetime of these particulates is only on the order of a
week (before they are rained out). Since Asia lies on the far side of the Pacific Ocean—a distance which requires about a week for air
masses to navigate—we usually aren’t overly concerned about the quality of Asian air or the quantity of junk that they emit into it.
By the time it gets here, it has largely been naturally scrubbed clean. But in the Teng et al. study, the authors find that, according to
their climate model, the local heating of the atmosphere by the Asiancarbon aerosols (which are quite good at absorbing
sunlight) can impart changes to the character of the larger-scale atmospheric circulation patterns . And these
changes to the broader atmospheric flow produce an effect on the weather patterns in the U.S . and thus induce a change in the
climate here characterized by “0.4°C [surface air temperature] warming on average over the eastern US during winter and over
almost the entire US during summer” averaged over the 2005–2024 period. While most of the summer warming doesn’t start to kick
in until Asian carbonaceous aerosol emissions are upped in the model to 10 times what they are today, the winter warming over the
eastern half of the country is large (several tenths of a °C) even at twice the current rate of Asian emissions. Now let’s revisit just
how much “global warming” that stringent U.S. greenhouse gas emissions reductions may avoid averaged across the country. In my
Master Resource post “Climate Impacts of Waxman-Markey (the IPCC-based arithmetic of no gain)” I calculated that a more
than 80% reduction of greenhouse gas emissions in the U.S. by the year 2050 would result in a
reduction of global temperatures (from where they otherwise would be) of about 0.05°C. Since
the U.S. is projected to warm slightly more than the global average (land warms faster than the
oceans), a 0.05°C of global temperature reduction probably amounts to about 0.075°C of
temperature “savings” averaged across the U.S., by the year 2050. Comparing the amount of
warming in the U.S. saved by reducing our greenhouse gas emissions by some 80% to the
amount of warming added in the U.S. by increases in Asian black carbon (soot) aerosol
emissions (at least according to Teng et al.) and there is no clear winner. Which points out the
anemic effect that U.S. greenhouse gas reductions will have on the climate of the U.S. and just
how easily the whims of foreign nations, not to mention Mother Nature, can completely offset
any climate changes induced by our greenhouse gas emissions reductions. And even if the traditional
form of air pollution (e.g., soot) does not increase across Asia (a slim chance of that), greenhouse gases emitted there certainly will.
emissions from China will completely subsume an
For example, at the current growth rate, new greenhouse gas
80% reduction in U.S. greenhouse gas emission in just over a decade. Once again, pointing out that a
reduction in domestic greenhouse gases is for naught, at least when it comes to mitigating climate change. So, what’s the point,
really, of forcing Americans into different energy choices? As I have repeatedly pointed out, nothing
we do here (when it
will make any difference either domestically, or globally, when it
comes to greenhouse gas emissions)
comes to influences on the climate. What the powers-that-be behind emissions reduction schemes in the U.S. are hoping
for is that 1) it doesn’t hurt us too much, and 2) that China and other large developing nations will follow our lead. Both outcomes
seem dubious at time scales that make a difference

China is a greater cause of warming- destroys all solvency

Wortzel ‘8 (Former Director of Asian Studies at the Heritage Foundation (Larry et al, Report to Congress of the U.S.-China
Economic and Security Review Commission, Nov, p. google)

China argues that developed countries are the primary cause of climate change and therefore places primary responsibility for re
ducing emissions on those countries rather than on China and other developing countries, a concept identified as ‘‘common but
differentiated responsibilities.’’ 190 The United States is the largest historical greenhouse gas emitter and far exceeds China in
emissions per capita.191 However, in the past two years China
has overtaken the United States in total production
of greenhouse gas emissions. All projections indicate that, in the absence of major energy
consumption changes in China, both China’s aggregate emissions and its share of global emissions will continue
to increase dramatically for the foreseeable future. The consequent reality is that it will be impossible
for the international community to resolve the climate change problem by sufficiently reducing
emissions unless China contributes to the effort. The solution also is unachievable unless the United States—as
currently the world’s second largest emitter and the largest historical emitter of greenhouse gases— makes a substantial
contribution. Any efforts to address this problem will require global participation by developed and developing nations.

No modeling or momentum

Mead '10 (Walter Russell, senior fellow for U.S. foreign policy at the Council on Foreign Relations, The Death of Global
Warming, http://blogs.the-american-interest.com/wrm/2010/02/01/the-death-of-global-warming/, February 1, 2010)
The global warming movement as we have known it is dead. Its health had been in steady decline during the last year
as the once robust hopes for a strong and legally binding treaty to be agreed upon at the Copenhagen Summit faded away. By the
time that summit opened, campaigners were reduced to hoping for a ‘politically binding’ agreement to be agreed that would set the
stage for the rapid adoption of the legally binding treaty. After the failure of the summit to agree to even that much, the movement
went into a rapid decline. The movement died from two causes: bad science and bad politics . After years
in which global warming activists had lectured everyone about the overwhelming nature of the scientific evidence, it turned out that
the most prestigious agencies in the global warming movement were breaking laws, hiding
data, and making inflated, bogus claims resting on, in some cases, no scientific basis at all. This latest
story in the London Times is yet another shocker; the IPCC’s claims that the rainforests were going to
disappear as a result of global warming are as bogus and fraudulent as its claims that the
Himalayan glaciers would melt by 2035. It seems as if a scare story could grab a headline, the IPCC
simply didn’t care about whether it was reality-based. With this in mind, ‘climategate’ — the scandal over hacked
emails by prominent climate scientists — looks sinister rather than just unsavory. The British government has concluded that
University of East Anglia, home of the research institute that provides the global warming with much of its key data, had violated
Britain’s Freedom of Information Act when scientists refused to hand over data so that critics could check
their calculations and methods. Breaking the law to hide key pieces of data isn’t just ‘science as usual,’ as the global
warming movement’s embattled defenders gamely tried to argue. A cover-up like that suggests that you indeed have something to
conceal. The urge to make the data better than it was didn’t just come out of nowhere. The
global warmists were
trapped into the necessity of hyping the threat by their realization that the actual evidence they had — which, let me
emphasize, all hype aside, is serious, troubling and establishes in my mind the need for intensive additional research and
investigation, as well as some prudential steps that would reduce CO2 emissions by enhancing fuel use efficiency and promoting
alternative energy sources — was not sufficient to get the world’s governments to do what they thought needed to be done.
Hyping the threat increasingly doesn’t look like an accident: it looks like it was a conscious
political strategy. Now it has failed. Not everything that has come out of the IPCC and the East Anglia Climate Unit is
false, but enough of their product is sufficiently tainted that these institutions can best serve the cause of fighting climate change by
stepping out of the picture. New leadership might help, but everything these two agencies have done will now have to be re-
checked by independent and objective sources. The global warming campaigners got into this mess because they had
a deeply flawed political strategy. They were never able to develop a pragmatic approach that could reach its goals in
the context of the existing international system. The global warming movement proposed a complex set of international agreements
involving vast transfers of funds, intrusive regulations in national economies, and substantial changes to the domestic political
economies of most countries on the planet. As it happened, the movement never got to the first step — it never got
the world’s countries to agree to the necessary set of treaties, transfers and policies that would constitute, at least on paper, a
program for achieving its key goals. Even if that first step had been reached, the second and third would almost surely not have
been. The United States Congress is unlikely to pass the kind of legislation these agreements would require before the midterm
elections, much less ratify a treaty. (It takes 67 senate votes to ratify a treaty and only 60 to overcome a filibuster.) After the
midterms, with the Democrats expected to lose seats in both houses, the chance of passage would be even more remote —
especially as polls show that global warming ranks at or near the bottom of most voters’ priorities. American public opinion supports
‘doing something’ about global warming, but not very much; support for specific measures and sacrifices will erode rapidly as
commentators from Fox News and other conservative outlets endlessly hammer away. Without a commitment from the United
States to pay its share of the $100 billion plus per year that poor countries wanted as their price for compliance, and without US
participation in other aspects of the proposed global approach, the intricate global deals fall apart. Since the United States was never
very likely to accept these agreements and ratify these treaties, and is even less prepared to do so in a recession with the Democrats
in retreat, even “success” in Copenhagen would not have brought the global warming movement the kind of victory it sought —
although it would have created a very sticky and painful political problem for the United States. But even
if somehow,
miraculously, the United States and all the other countries involved not only accepted the
agreements but ratified them and wrote domestic legislation to incorporate them into law, it
is extremely unlikely that all this activity would achieve the desired result. Countries would
cheat, either because they chose to do so or because their domestic systems are so weak, so corrupt or so both
that they simply wouldn’t be able to comply. Governments in countries like China and India
aren’t going to stop pushing for all the economic growth they can get by any means that will
work — and even if central governments decided to move on global warming, state and local authorities have
agendas of their own. The examples of blatant cheating would inevitably affect compliance in
other countries; it would also very likely erode what would in any case be an extremely fragile
consensus in rich countries to keep forking over hundreds of billions of dollars to poor countries — many of whom would not be
in anything like full compliance with their commitments. For better or worse, the global political system isn’t capable
of producing the kind of result the global warming activists want . It’s like asking a jellyfish to climb a flight
of stairs; you can poke and prod all you want, you can cajole and you can threaten. But you are asking for something that you just
can’t get — and at the end of the day, you won’t get it. The grieving friends and relatives aren’t ready to pull the plug; in a typical,
whistling-past-the-graveyard comment, the BBC first acknowledges that even
if the current promises are kept,
temperatures will rise above the target level of two degrees Celsius — but let’s not despair! The BBC quotes one
of its own reporters: “BBC environment reporter Matt McGrath says the accord lacks teeth and does not include any clear targets on
cutting emissions. But if most countries at least signal what they intend to do to cut their emissions, it will mark the first time that
the UN has a comprehensive written collection of promised actions, he says.”

Tech strategies insufficient

Revkin ’12 (Andrew C., Environment and energy blogger – NYT, “Can China Follow U.S. Shift from Coal to Gas?”, July 4, 2012)

Fourth, there is growing interest in so-called “technology strategies” to address climate change .
The gas revolution is a good poster child for the importance of technological innovation. Most of the key advances that make today’s
gas revolution possible—not just fracking but across the production and transmission of gas as well as in the ultra-efficient turbines
that are today’s best way to make electricity from gas—trace their origins back to publicly funded R&D in tandem with lots of private
sector investment. Some people have unwisely taken that logic to the extreme and suggested that
if the US and other innovating nations just pushed hard on technology that there wouldn’t be
much need for emission limits, cap and trade or carbon taxes. That’s too simplistic. There’s no
question that we need a big push on technology and that all nations, collectively, massively under-invest in
energy R&D. But a technology push with no pull from the market’s a recipe for waste. I like the carbon tax like
the one Australia introduced this week to create an incentive not just to invent new low-carbon
technologies but also to deploy them. One implication for technology R&D policy is that in a world of cheap gas there’s
probably a lot of value in looking carbon capture and storage (CCS) technologies for use on gas-fired power plants. To date, most CCS
investment has focused on coal on the assumption that coal is cheap and that the technologies needed for CCS on gas are too
expensive. That conventional view could change in a world where the full cost of burning coal is high and gas is cheap. Some of the
technologies for CCS are generic—they work whether the original fuel is coal or gas—but others (including the costliest parts of CCS
systems) must be tailored to the fuel. I’ve always thought that CCS was an inelegant way to lick the carbon problem—because it
involves burning fuels and then corralling a huge mass of pollution rather than avoiding the pollution in the first place—but if gas is
to be a real “bridge” to a low emission future rather than a nice-looking dead end then we must seriously explore ways to further cut
emissions from gas plants. [Here's a link to an article by Jesse Ausubel on one such technology.]. Fifth, all these surprises are a
reminder of how much we don’t know about how technology and markets will unfold . Earlier this year the
Energy Information Administration published a rather brave study: a retrospective on how well its
forecasters have done predicting things like demand for energy, the cost of oil and such. One lesson
from that study is that a lot of forecasting is done by looking in the rear view mirror—forecasts
typically start with current conditions, and as facts on the ground change radically so do the
forecasts. Another lesson from that study is that the record of forecasting energy prices—gas in particular but also
oil—is pretty abysmal. Since so much, even CO2, depends on relative energy prices we should be
sober about what we can realistically predict for the future. Sixth, I see the gas revolution as just one of a
large class of strategies for getting serious about climate change in ways that are politically expedient. In a few countries and
jurisdictions—such as Europe, California, and Vermont—people will invest lots of their own money to control emissions in an effort
to slow global warming. But most of the world isn’t so keen, yet, to spend handsomely on this global
goal. I’ve always thought that the way to make progress on climate change, especially in “reluctant” countries like China and even
the U.S., is to start by focusing on places where climate goals overlap with other national priorities—like clearing the air or making
energy supplies more reliable. (For another example, focused on the tremendous potential for slowing climate change through
action on soot, see the last issue of foreign affairs for an article co-authored with two colleagues here in La Jolla, V. Ramanathan and
C. Kennel.) We probably can’t lick global warming with self-interested actions alone, but at least we can point countries in the right
direction and build political support for the deeper and more expensive cuts that will be essential. As Victor notes, simply
moving from coal to gas is hardly a climate solution on its own, and others challenge the idea
that natural gas can serve as a bridge along the road to a post-fossil energy future. And certainly if
China’s gas push comes with the same wasteful, leaky practices that American oil and gas companies have only
slowly abandoned (and that still abound in Russia and elsewhere ), that’s not a reasonable bridge at
all. Nothing I, or anyone else writes, will change the reality that the gas age is here for many years to come. But my hope is
that progress in avoiding environmental regrets can come through constructive discussion of ways to cut risks and
waste and to sustain a long-term energy quest that extends beyond fossil fuels even while they remain
abundant and cheap. That’s no easy task.
---EXT Irreversible

1NC—warming inevitable—top scientists agree—no amount of reductions solve because


carbon is already absorbed—thousand year threshold

More evidence- there’s too much CO2 in the air even if we stop

Hansen ‘8 (Hansen, head of NASA Goddard Institute and professor of Environmental Sciences, Columbia University, 2008 (James
E. Hanson. Head of the NASA Goddard Institute for Space Studies in New York City and adjunct professor in the Department of Earth
and Environmental Science at Columbia University. Al Gore’s science advisor. Introductory chapter for the book State of the Wild.
“Tipping point: Perspective of a Scientist.” April. http://www.columbia.edu/~jeh1/2008/StateOfWild_20080428.pdf)

The upshot of the combination of inertia and feedbacks is that additional climate change is
already “in the pipeline”: even if we stop increasing greenhouse gases today, more
warming will occur. This is sobering when one considers the present status of Earth’s climate. Human civilization
developed during the Holocene (the past 12,000 years). It has been warm enough to keep ice sheets off North America and
Europe, but cool enough for ice sheets to remain on Greenland and Antarctica. With rapid warming of 0.6°C in the past 30 years,
global temperature is at its warmest level in the Holocene.3 The warming that has already
occurred, the positive feedbacks that have been set in motion, and the additional warming in
the pipeline together have brought us to the precipice of a planetary tipping point. We are
at the tipping point because the climate state includes large, ready positive feedbacks provided
by the Arctic sea ice, the West Antarctic ice sheet, and much of Greenland’s ice. Little additional forcing is needed
to trigger these feedbacks and magnify global warming. If we go over the edge, we will
transition to an environment far outside the range that has been experienced by humanity,
and there will be no return within any foreseeable future generation. Casualties would include more than the loss of
indigenous ways of life in the Arctic and swamping of coastal cities. An intensified hydrologic cycle will produce both greater
floods and greater droughts. In the US, the semiarid states from central Texas through Oklahoma and both Dakotas would
become more drought-prone and ill suited for agriculture, people, and current wildlife. Africa would see a great expansion of
dry areas, particularly southern Africa. Large populations in Asia and South America would lose their primary dry season
freshwater source as glaciers disappear. A major casualty in all this will be wildlife.

Too late

Rahn 11 (Richard W. Rahn, 1/25/2011 (senior fellow at the Cato Institute, The Washington Times, “Obama's regulatory reform
test,” Lexis)

The Obama Environmental Protection Agency (EPA) has ruled that carbon dioxide is a pollutant and, as a result, has been
holding up the permitting of new power and manufacturing plants. If this continues, it will cause a significant drop in U.S.
economic growth and job creation, yet it will
have no measurable benefit. China, India and many other
countries are rapidly increasing CO2 emissions, overwhelming whatever actions the U nited
States may take. Even if all new CO2 emissions were stopped globally , it would be decades
before there would be even a minor effect on global temperatures. Now, new research is
indicating that sunspot activity is much more important than CO2 when it comes to
influencing the earth's temperature. The EPA ban is nothing more than national economic suicide. Let us see if
Mr. Obama has the courage to tell the EPA to stop.
Ice melting irreversibly now

FP 11 (Foreign Policy, “Beating a retreat”, http://www.economist.com/node/21530079, September 24, 2011, LEQ)


ON SEPTEMBER 9th, at the height of its summertime shrinkage, ice covered 4.33m square km, or 1.67m square miles, of the Arctic
Ocean, according to America’s National Snow and Ice Data Centre (NSIDC). That is not a record low—not quite. But the actual
record, 4.17m square km in 2007, was the product of an unusual combination of sunny days, cloudless skies and warm currents
flowing up from mid-latitudes. This year has seen no such opposite of a perfect storm, yet the summer sea-ice minimum is a mere
4% bigger than that record. Add in the fact that the thickness of the ice, which is much harder to measure, is estimated to have
fallen by half since 1979, when satellite records began, and there is probably less ice floating on the Arctic Ocean now than at any
time since a particularly warm period 8,000 years ago, soon after the last ice age. That Arctic sea ice is disappearing has been known
for decades. The underlying cause is believed by all but a handful of climatologists to be global warming brought about by
greenhouse-gas emissions. Yet the rate the ice is vanishing confounds these climatologists’ models. These predict that if the level of
carbon dioxide, methane and so on in the atmosphere continues to rise, then the Arctic
Ocean will be free of floating
summer ice by the end of the century. At current rates of shrinkage, by contrast, this looks likely to happen
some time between 2020 and 2050. The reason is that Arctic air is warming twice as fast as the
atmosphere as a whole. Some of the causes of this are understood, but some are not. The darkness of land and water compared
with the reflectiveness of snow and ice means that when the latter melt to reveal the former, the area exposed absorbs more heat
from the sun and reflects less of it back into space. The result is a feedback loop that accelerates local
warming. Such feedback, though, does not completely explain what is happening. Hence the search for other things that might
assist the ice’s rapid disappearance. Forcing the issue One is physical change in the ice itself. Formerly a solid mass that
melted and refroze at its edges, it is now thinner, more fractured, and so more liable to melt. But that is (literally and
figuratively) a marginal effect. Filling the gap between model and reality may need something besides this. The latest candidates are
“short-term climate forcings”. These are pollutants, particularly ozone and soot, that do not hang around in the atmosphere as
carbon dioxide does, but have to be renewed continually if they are to have a lasting effect. If they are so renewed, though, their
impact may be as big as CO2’s. At the moment, most eyes are on soot (or “black carbon”, as jargon-loving researchers refer to it). In
the Arctic, soot is a double whammy. First, when released into the air as a result of incomplete combustion (from sources as varied
as badly serviced diesel engines and forest fires), soot particles absorb sunlight, and so warm up the atmosphere. Then, when snow
or rain wash them onto an ice floe, they darken its surface and thus cause it to melt faster. Reducing soot (and also ozone, an
industrial pollutant that acts as a greenhouse gas) would not stop the summer sea ice disappearing, but it might delay the process by
a decade or two. According to a recent report by the United Nations Environment Programme, reducing black carbon and ozone in
the lower part of the atmosphere, especially in the Arctic countries of America, Canada, Russia and Scandinavia, could cut warming
in the Arctic by two-thirds over the next three decades. Indeed, the report suggests, if such measures—preventing crop burning and
forest fires, cleaning up diesel engines and wood stoves, and so on—were adopted everywhere they could halve the wider rate of
warming by 2050. Without corresponding measures to cut CO2 emissions, this would be but a temporary fix. Nonetheless, it is an
attractive idea because it would have other benefits (soot is bad for people’s lungs) and would not require the wholesale rejigging of
energy production which reducing CO2 emissions implies. Not everyone agrees it would work, though. Gunnar Myhre of the Centre
for International Climate and Environmental Research in Oslo, for example, notes that the amount of black carbon in the Arctic is
small and has been falling in recent decades. He does not believe it is the missing factor in the models. Carbon dioxide, in his view, is
the main culprit. Black carbon deposited on the Arctic snow and ice, he says, will have only a minimal effect on its reflectivity. The
rapid melting of the Arctic sea ice, then, illuminates the difficulty of modelling the climate—but not in a way that brings much
comfort to those who hope that fears about the future climate might prove exaggerated. When reality is changing faster than theory
suggests it should, a certain amount of nervousness is a reasonable response. It’s an ill wind… The direct consequences of changes in
the Arctic are mixed. They should not bring much rise in the sea level, since floating ice obeys Archimedes’s principle and displaces
its own mass of water. A
darker—and so more heat-absorbent—Arctic, though, will surely accelerate global
warming and may thus encourage melting of the land-bound Greenland ice sheet. That certainly would raise sea levels (though
not as quickly as News Corporation’s cartographers suggest in the latest edition of the best-selling “Times Atlas”, which claims that
15% of the Greenland sheet has melted in the past 12 years; the true figure is more like 0.05%). Wildlife will also suffer. Polar bears,
which hunt for seals along the ice’s edge, and walruses, which fish there, will both be hard-hit.

All their impacts are inevitable


Sterlicchi ‘9 (John Sterlicchi, BusinessGreen, “US report says rising sea levels inevitable”,
http://www.businessgreen.com/bg/news/1805554/us-report-rising-sea-levels-inevitable , January
29, 2009, LEQ)

Some of the effects of global warming will be irreversible and others may last for at least 1,000 years, according
to a new US government-sponsored report. The study, led by researchers at the US National Oceanic and Atmospheric
Administration and paid for by the Department of Energy, paints a depressing picture of the future caused by increased CO2
emissions. If CO2 is allowed to peak at 450-600 parts per million, said the report, the result will be less dry-season rainfall that
will be reminiscent of the 1930s Dust Bowl era in the US. There
will be decreases in drinking water supplies,
increased fire frequency and an end to dry-season farming of wheat and maize. Regions that
will be affected are southern Europe, northern and southern Africa, southwestern US, and western Australia. Also, if CO2 peaks
at 600ppm, global water
levels will rise by as much as one metre. If it peaks at 1,000ppm, the rise will double.
Rising sea levels would cause "irreversible commitments to future changes in the
geography of the Earth, since many coastal and island features would ultimately become submerged,"
the report said. Those grim predictions of rising sea levels also did not take into account the melting of ice at both poles, as the
result of that was unpredictable. "People have imagined that if we stopped emitting carbon dioxide, the
climate would go back to normal in 100 years, 200 years; that is not true," the head of the research
team, Susan Solomon, said in a teleconference. This is because of the role played by the world's oceans. Currently the
oceans are absorbing the CO2 and keeping the planet cool but in the future they will become saturated.
---EXT No Impact

Consensus of experts agree that there is no impact to warming

Hsu 10
Jeremy, Live Science Staff, July 19, pg. http://www.livescience.com/culture/can-humans-survive-extinction-doomsday-
100719.html

His views deviate sharply from those of most


experts, who don't view climate change as the end for
humans. Even the worst-case scenarios discussed by the Intergovernmental Panel on Climate Change don't
foresee human extinction. "The scenarios that the mainstream climate community are
advancing are not end-of-humanity, catastrophic scenarios," said Roger Pielke Jr., a climate
policy analyst at the University of Colorado at Boulder. Humans have the technological tools to begin tackling
climate change, if not quite enough yet to solve the problem, Pielke said. He added that doom-mongering did little to encourage
people to take action. "My view of politics is that the long-term, high-risk scenarios are really difficult to use to motivate short-
term, incremental action," Pielke explained. "The rhetoric of fear and alarm that some people tend toward is
counterproductive." Searching for solutions One technological solution to climate change already exists through carbon
capture and storage, according to Wallace Broecker,
a geochemist and renowned climate scientist at
Columbia University's Lamont-Doherty Earth Observatory in New York City. But Broecker remained skeptical that
governments or industry would commit the resources needed to slow the rise of carbon dioxide (CO2) levels, and predicted that
more drastic geoengineering might become necessary to stabilize the planet. " The rise in CO2 isn't going to kill
many people, and it's not going to kill humanity ," Broecker said. "But it's going to change the entire wild
ecology of the planet, melt a lot of ice, acidify the ocean, change the availability of water and change crop yields, so we're
essentially doing an experiment whose result remains uncertain."

No impact to warming – history and scientific study prove

Jaworowski 8 (Professor Zbigniew, Chairman of the Scientific Council of the Central Laboratory for Radiological Protection in
Warsaw and former chair of the United Nations Scientific Committee on the Effects of Atomic Radiation, “Fear Propaganda,”
http://www.ourcivilisation.com/aginatur/cycles/chap3.htm)

Doomsayers preaching the horrors of warming are not troubled by the fact that in
the Middle Ages, when for a few
hundred years it was warmer than it is now, neither the Maldive atolls nor the Pacific
archipelagos were flooded. Global oceanic levels have been rising for some hundreds or
thousands of years (the causes of this phenomenon are not clear). In the last 100 years, this increase amounted to 10 cm
to 20 cm, (24) but it does not seem to be accelerated by the 20th Century warming . It turns out that in
warmer climates, there is more water that evaporates from the ocean (and subsequently falls as snow on the Greenland and
Antarctic ice caps) than there is water that flows to the seas from melting glaciers. (17) Since
the 1970s, the glaciers
of the Arctic, Greenland, and the Antarctic have ceased to retreat, and have started to grow.
On January 18, 2002, the journal Science published the results of satellite-borne radar and ice core studies performed by
scientists from CalTech's Jet Propulsion Laboratory and the University of California at Santa Cruz. These results indicate that the
Antarctic ice flow has been slowed, and sometimes even stopped, and that this has resulted in the
thickening of the continental glacier at a rate of 26.8 billion tons a year. (25) In 1999, a Polish Academy of Sciences paper was
prepared as a source material for a report titled "Forecast of the Defense Conditions for the Republic of Poland in 2001-2020."
The paper implied that the increase of atmospheric precipitation by 23 percent in Poland, which was presumed to be caused by
global warming, would be detrimental. (Imagine stating this in a country where 38 percent of the area suffers from permanent
surface water deficit!) The same paper also deemed an extension of the vegetation period by 60 to 120 days as a disaster. Truly,
a possibility of doubling the crop rotation, or even prolonging by four months the harvest of radishes, makes for a horrific vision
in the minds of the authors of this paper. Newspapers
continuously write about the increasing frequency
and power of the storms. The facts, however, speak otherwise. I cite here only some few
data from Poland, but there are plenty of data from all over the world. In Cracow, in 1896-
1995, the number of storms with hail and precipitation exceeding 20 millimeters has
decreased continuously, and after 1930, the number of all storms decreased. (26) In 1813 to
1994, the frequency and magnitude of floods of Vistula River in Cracow not only did not
increase but, since 1940, have significantly decreased. (27) Also, measurements in the
Kolobrzeg Baltic Sea harbor indicate that the number of gales has not increased between
1901 and 1990. (28) Similar observations apply to the 20th Century hurricanes over the Atlantic
Ocean (Figure 4,) and worldwide.
---EXT Adaptation
Tech and adaptive advances prevent all climate impacts---warming won’t cause war

Singer et al 11, Dr. S. Fred Research Fellow at The Independent Institute, Professor Emeritus of Environmental Sciences at the University
of Virginia, President of the Science and Environmental Policy Project, a Fellow of the American Association for the Advancement of Science, and a
Member of the International Academy of Astronautics; Robert M. Carter, Research Professor at James Cook University (Queensland) and the University
of Adelaide (South Australia), palaeontologist, stratigrapher, marine geologist and environmental scientist with more than thirty years professional
experience; and Craig D. Idso, founder and chairman of the board of the Center for the Study of Carbon Dioxide and Global Change, member of the
American Association for the Advancement of Science, American Geophysical Union, American Meteorological Society, Arizona-Nevada Academy of
Sciences, and Association of American Geographers, et al, 2011, “Climate Change Reconsidered: 2011 Interim Report,” online:
http://www.nipccreport.org/reports/2011/pdf/FrontMatter.pdf

Decades-long empirical trends of climate -sensitive measures of human well-being, including the
percent of developing world population suffering from chronic hunger, poverty rates, and deaths due to extreme weather events,
reveal dramatic improvement during the twentieth century, notwithstanding the historic increase in
atmospheric CO2 concentrations. The magnitude of the impacts of climate change on human well-being depends on
society's adaptability (adaptive capacity), which is determined by, among other things, the wealth and
human resources society can access in order to obtain, install, operate, and maintain technologies necessary to cope
with or take advantage of climate change impacts. The IPCC systematically underestimates
adaptive capacity by failing to take into account the greater wealth and technological
advances that will be present at the time for which impacts are to be estimated . Even accepting
the IPCC's and Stern Review's worst-case scenarios, and assuming a compounded annual growth rate of per-capita GDP of
only 0.7 percent, reveals that net GDP per capita in developing countries in 2100 would be double the
2006 level of the U.S. and triple that level in 2200. Thus, even developing countries' future
ability to cope with climate change would be much better than that of the U.S. today . The IPCC's
embrace of biofuels as a way to reduce greenhouse gas emissions was premature, as many researchers have found "even the best
biofuels have the potential to damage the poor, the climate, and biodiversity" (Delucchi, 2010). Biofuel production consumes nearly
as much energy as it generates, competes with food crops and wildlife for land, and is unlikely to ever meet more than a small

fraction of the world's demand for fuels. The notion that global warming might cause war and social unrest is not
only wrong, but even backwards - that is, global cooling has led to wars and social unrest in the
past, whereas global warming has coincided with periods of peace, prosperity, and social stability .
---EXT No Warming

1NC 1- There is no warming- Prefer our Taylor evidence because it is citing a new
groundbreaking peer-reviewed study that takes into account new NASA satellite data.

More energy is actually lost to space than predicted- this means climate models are flawed
because actual measurements have disproven the predictions.

Their quals argument don’t apply here

Wilson ’12 (GLOBAL WARMING: THE SATELLITES DON'T LIE March 3, 2012 7:48 AM | 7 Comments James A. Wilson

Over the summer Forbes Magazine published NASA satellite data indicating the alarmist predictions -
even the UN computer models on which they were based - are dead wrong. The study, reported in the peer
reviewed journal , Remote Sensing, correlates data from 2000 through 2011. It shows two
phenomena surprising to the apostles of doom in the scientific and political community. There is much less heat being
trapped in the atmosphere by greenhouse gases - or any other cause - than the models portend, and a lot
more of it is being released naturally into space. This is especially true over the oceans. James M.
Taylor, a senior fellow for environmental policy at The Heartland Institute and managing editor of
Environment and Climate News authored the Forbes article. Credentials don't get any more
impeccable.

Consensus of NASA and NOAA satellite data shows no warming

Wilson ’12 (GLOBAL WARMING: THE SATELLITES DON'T LIE March 3, 2012 7:48 AM | 7 Comments James A. Wilson

The latest satellite gathered information is consistent with NOAA and NASA data showing
humidity and the formation of cirrus clouds has lagged far behind alarmist predictions as well.
These findings, and those of NASA's ERBS satellite show similar patterns of heat exhange for the
years 1985 to 1999. In other words, we are simply not going to hell in a climate change hand
basket.
---EXT Negative Feedbacks Solves

Prefer this evidence—based on new measurements that models don't take into account—the
lead author of the IPCC on feedbacks agrees

History proves that water vapor is a negative feedback- this renders their evidence obselete

McShane 8 (Owen, the chairman of the policy panel of the New Zealand Climate Science Coalition and director of the Center
for Resource Management Studies, 4-4-8, The National Business Review (New Zealand), “Climate change confirmed but global
warming is cancelled”, Lexis)

The climate is not highly sensitive to CO2 warming because water vapour is a damper
against the warming effect of CO2. That is why history is full of Ice Ages - where other effects, such
as increased reflection from the ice cover, do provide positive feedback - while we do not hear about Heat Ages. The
Medieval Warm Period, for example, is known for being benignly warm - not dangerously
hot. We live on a benign planet - except when it occasionally gets damned cold. While I have
done my best to simplify these developments they remain highly technical and many people distrust their own ability to assess
competing scientific claims. However, in this case the tipping point theories are based on models that
do not include the effects of rain and clouds. The new Nasa Aqua satellite is the first to measure
the effects of clouds and rainfall. Spencer's interpretation of the new data means all
previous models and forecasts are obsolete. Would anyone trust long-term forecasts of farm production that
were hopeless at forecasting rainfall? The implications of these breakthroughs in measurement and
understanding are dramatic to say the least. The responses will be fun to watch.
---Warming- Squo Solves

SQ solves emissions- Obama solves warming through leadership now

Goldenberg 2-1-13 [Suzanne, the US environment correspondent of the Guardian and is based in Washington DC. She has
won several awards for her work in the Middle East, “US carbon emissions fall to lowest levels since 1994,”
http://www.guardian.co.uk/environment/2013/feb/01/us-carbon-emissions-lowest-levels]

America's carbon dioxide emissions last year fell to their lowest levels since 1994 , according to a new
report.¶ Carbon dioxide emissions fell by 13% in the past five years, because of new energy-saving
technologies and a doubling in the take-up of renewable energy, the report compiled by Bloomberg New
Energy Finance (BNEF) for the Business Council for Sustainable Energy (BCSE) said. ¶ The reduction in climate pollution – even as
Congress failed to act on climate change – brings America more than halfway towards Barack Obama's target of cutting emissions by
17% from 2005 levels over the next decade, the Bloomberg analysts said. ¶ By the end of last year, America's emissions of carbon
dioxide and other greenhouse gas emissions had fallen 10.7% from the 2005 baselines. ¶ That drop puts Obama in a
better position to defend his environmental achievements , which have often gone overlooked in the bitter
rows over climate science.¶ It may also buoy up America's standing in the global climate negotiations .¶
"There have certainly been some solid results on the board in the US as a result of all these
changes," Ethan Zindler, a BNEF analyst said.¶ A report last year by the independent thinktank Resources for the Future also
suggested America was on course to meet those targets.¶ Lisa Jacobson, president of the BCSE, said the Bloomberg findings exposed
the conservative argument that acting on climate change would be a drag on the economy. Instead, carbon
emissions
declined even as GDP was going up, she noted.¶ As described by Bloomberg, the US is in the throes of a
major shift in energy production. Coal fell to just 18.1% of America's energy mix last year, down
from 22.5% in 2007. Oil use also declined .¶ The explosion of natural gas production, thanks to fracking, filled much of
the gap. America got 31% of its electricity from gas-fired power plants last year.¶ But the report found steadily expanding installation
of wind, solar, hydro and geothermal energy. Renewables represented the largest single source of new
growth last year, reaching $44bn in 2012, the report said, the report said.¶ Over the same time span, total energy use
fell since 2007, by 6.4%, the report said. Most of the emissions cuts were due to installing more efficient heating and cooling systems
in commercial building.¶ Other cuts in emission came from transport, with 488,000 Americans last year opting for hybrid and plug-in
vehicles.

Emissions are on the decline- here’s a chart

Bump 2-1-13 [Philip, writer for Grist, an environmental news service, Managing Editor of Daily Download. He has contributed to
The Atlantic, The Daily, and Mediaite, covering the intersection of technology and the media, “CO2 emissions from energy
production drop to 1994 levels in the U.S.,” http://grist.org/news/co2-emissions-from-energy-production-drop-to-1994-levels-in-
the-u-s/]

The headline at The Guardian says almost everything you need to know: U.S. carbon emissions fall to lowest levels
since 1994.¶ Carbon dioxide emissions fell by 13% in the past five years, because of new energy-
saving technologies and a doubling in the take-up of renewable energy, the report compiled by
Bloomberg New Energy Finance for the Business Council for Sustainable Energy (BCSE) [PDF] said. ¶ The reduction in climate
pollution — even as Congress failed to act on climate change — brings America more than halfway towards Barack
Obama’s target of cutting emissions by 17% from 2005 levels over the next decade , the Bloomberg
analysts said.¶ By the end of last year, America’s emissions of carbon dioxide and other greenhouse gas emissions had fallen 10.7%
from the 2005 baselines.¶ The caveat: The carbon emissions discussed are those related to energy production. Energy production
isn’t all CO2 emission, but it’s a lot of it.¶ So here’s what that reduction looks like. Since 1974, levels of energy-related carbon
emissions have seen two peaks. As indicated above, we’re on a downward trend, something David Roberts explained last year.

Over the past few years, individual energy sources have played a fluctuating role in the reduction. In
2009, the collapsing
economy meant lower emissions from all sources. That coal figure in 2012 is remarkable .¶ This
morning, the U.S. Energy Information Administration released state-by-state data on CO2 emissions through 2010. We put together
this map showing net increase or reducton in CO2 emissions by state between 1994 and 2010. The darker brown a state is, the more
its emissions rose; the darker green, the more emissions fell. Most states went up. But go Delaware!
---Warming- AT: Ocean Acidification

Ocean acidification will be slow and stable, proven by 1000 studies- it improves ocean
resiliency

Codling ‘11 [Jo, received a Bachelor of Science first class and won the FH Faulding and the Swan Brewery prizes at the
University of Western Australia. Her major was microbiology, molecular biology. Nova received a Graduate Certificate in Scientific
Communication from the Australian National University in 1989,[4] and she did honours research in 1990, prize-winning science
graduate, Jo has has done over 200 radio interviews, many on the Australian ABC.  She was formerly an associate lecturer in Science
Communication at the ANU and is based in Perth, Western Australia, , “Ocean Acidification — a little bit less alkalinity could be a
good thing,” Sept. 11, http://joannenova.com.au/2011/09/ocean-acidification-a-little-bit-less-alkalinity-could-be-a-good-thing/]

Studies of how marine life copes with less alkaline conditions include many experiments with water at pH values in a range beyond
anything that is likely on planet Earth — they go beyond the bounds of what’s possible. There are estimates that the pH of the ocean
has shifted about 0.1 pH unit in the last 200 years, yet some studies consider the effects of water that is shifted
by 2 or even 4 entire pH units. Four pH units means 10,000 fold change in the concentration of hydrogen ions). That’s
a shift so large, it’s not going to occur in the next few thousand years, even under the worst of
the worst case scenarios by the most sadistic models. Indeed, it’s virtually impossible for CO2
levels to rise high enough to effect that kind of change , even if we burned every last fossil,
every tree, plant microbe, and vaporized life on earth. (Yet still someone thought it was worth studying what
would happen if, hypothetically, that happened. Hmm.)¶ 1103 studies on acidification say there’s no need to panic¶ CO2
science has an extraordinary data base of 1103 studies of the effects of “acidification” on marine
life. They reason that any change beyond 0.5 pH units is “far far beyond the realms of reality”
even if you are concerned about coral reefs in the year 2300 (see Tans 2009). Even the IPCC’s highest end “scenario
A2″ estimate predicts a peak change in the range of 0 .6 units by 2300.¶ Many of the headlines
forecasting “Death to Reefs” come from studies of ocean water at extreme pH’s that will never
occur globally, and that are beyond even what the IPCC is forecasting. Some headlines come from studies of hydrothermal
vents where CO2 bubbles up from the ocean floor. Not surprisingly they find changes to marine life near the vents, but then, the pH
of these areas ranges right down to 2.8. They are an extreme environment, nothing like what we might expect to convert the worlds
oceans too.¶ Marine life, quite happy about a bit more CO2?¶ Studies
of growth, calcification, metabolism,
fertility and survival show that, actually, if things were a little less alkaline, on average, marine
life would benefit. There will be winners and losers, but on the whole, using those five measures of health,
the reefs are more likely to have more life on and around them, than they are to shrink .¶ Figure
12. Percent change in the five measured life characteristics (calcification, metabolism, growth, fertility and survival) vs. decline of
seawater pH from its present (control treatment) value to ending values extending up to the beginning pH value of "the warped
world of the IPCC" for all individual data points falling within this pH decline range.¶ How can this be?¶ First, marine
life
evolved under conditions where most of the time the world was warmer and had more CO2 in
the atmosphere than it does today. Second, like life above the water, life-below-water is based on carbon,
and putting more carbon into the water is not necessarily a bad thing . That said, the dots in the graph
above represent study results, and the ones below zero tell us there will be some losers , even though there will be
more winners (above zer0). Thirdly, watch out for some of the more devastating headlines which also come from studies where
researchers changed the pH by tossing hydrochloric acid into the tank. Chlorine, as they say, is not the same as the gas nature
breathes — CO2. (The strange thing about the studies with hydrochloric acid, is that it doesn’t seem to be bad as we might have
expected– nonetheless, it seems like a dubious practice to use in studying the health of corals.)¶ The Ocean Acidification Database is
housed at CO2 science.¶ The graph above is just one of many on their results and conclusions page.¶ The bottom line:¶ Yes, we
should watch and monitor the oceans careful. No, there is no chance the Great Barrier Reef will be gone in the
next 100 years: 1103 studies show that if the worlds oceans were slightly less basic then marine
life as a whole will be slightly more likely to grow, survive, and be fertile.

Climate change proves Oceans and marine bioD are resilient – alarmist predictions empirically
denied

Taylor ‘10 (James M. Taylor is a senior fellow of The Heartland Institute and managing editor of Environment & Climate News.,
“Ocean Acidification Scare Pushed at Copenhagen,” Feb 10 http://www.heartland.org/publications/environment
%20climate/article/26815/Ocean_Acidification_Scare_Pushed_at_Copenhagen.html)

With global temperatures continuing their decade-long decline and United Nations-sponsored global warming talks falling apart in
Copenhagen, alarmists at the U.N. talks spent
considerable time claiming carbon dioxide emissions will
cause catastrophic ocean acidification, regardless of whether temperatures rise. The latest scientific data,
however, show no such catastrophe is likely to occur. Food Supply Risk Claimed The United Kingdom’s
environment secretary, Hilary Benn, initiated the Copenhagen ocean scare with a high-profile speech and numerous media
interviews claiming ocean acidification threatens the world’s food supply. “The fact is our seas absorb CO2. They absorb
about a quarter of the total that we produce, but it is making our seas more acidic,” said Benn in his speech. “If this continues as a
problem, then it can affect the one billion people who depend on fish as their principle source of protein, and we have to feed
another 2½ to 3 billion people over the next 40 to 50 years.” Benn’s
claim of oceans becoming “more acidic” is
misleading, however. Water with a pH of 7.0 is considered neutral. pH values lower than 7.0 are
considered acidic, while those higher than 7.0 are considered alkaline. The world’s oceans have a pH of 8.1,
making them alkaline, not acidic. Increasing carbon dioxide concentrations would make the oceans
less alkaline but not acidic. Since human industrial activity first began emitting carbon dioxide into the
atmosphere a little more than 200 years ago, the pH of the oceans has fallen merely 0.1 , from 8.2 to 8.1. Following
Benn’s December 14 speech and public relations efforts, most of the world’s major media outlets produced stories claiming ocean
acidification is threatening the world’s marine life. An Associated Press headline, for example, went so far as to call ocean
acidification the “evil twin” of climate change. Studies Show CO2 Benefits Numerous recent scientific studies show higher
carbon dioxide levels in the world’s oceans have the same beneficial effect on marine life as
higher levels of atmospheric carbon dioxide have on terrestrial plant life . In a 2005 study
published in the Journal of Geophysical Research, scientists examined trends in chlorophyll
concentrations, critical building blocks in the oceanic food chain. The French and American scientists reported “an overall
increase of the world ocean average chlorophyll concentration by about 22 percent” during the prior two decades of increasing
carbon dioxide concentrations. In a 2006 study published in Global Change Biology, scientists observed higher CO2 levels are
correlated with better growth conditions for oceanic life. Thehighest CO2 concentrations produced “higher
growth rates and biomass yields” than the lower CO2 conditions . Higher CO2 levels may well
fuel “subsequent primary production, phytoplankton blooms, and sustaining oceanic food-
webs,” the study concluded. Ocean Life ‘Surprisingly Resilient’ In a 2008 study published in Biogeosciences,
scientists subjected marine organisms to varying concentrations of CO2, including abrupt
changes of CO2 concentration. The ecosystems were “surprisingly resilient” to changes in
atmospheric CO2, and “the ecosystem composition, bacterial and phytoplankton abundances and productivity, grazing rates and
total grazer abundance and reproduction were not significantly affected by CO2-induced effects.” In a 2009 study published in
Proceedings of the National Academy of Sciences, scientists reported, “Sea star growth and feeding rates increased with water
temperature from 5ºC to 21ºC. A doubling of current [CO2] also increased growth rates both with and without a concurrent
temperature increase from 12ºC to 15ºC.” Another False CO2 Scare “ Far
too many predictions of CO2-induced
catastrophes are treated by alarmists as sure to occur, when real-world observations show
these doomsday scenarios to be highly unlikely or even virtual impossibilities,” said Craig Idso, Ph.D.,
author of the 2009 book CO2, Global Warming and Coral Reefs. “The phenomenon of CO2-induced ocean acidification appears to be
no different.
---Warming- AT: Sea Level Rise

Sea level rise is junk science—models empirically fail

Gupta ’12 (Alexander Sen, Climate Change Research Centre @ University of New South Wales, et al., “Climate Drift in the CMIP3
Models,” Journal of Climate Vol. 25, Issue 13, p. 4621-4640, 2012)

As discussed above, drift


in temperature and salinity dominates 20C3M trends throughout most of the
subsurface ocean. In the calculation of steric sea level rise, a given temperature or salinity change
will generally have less effect at depth than near the surface. As the amount of expansion for a given change in
temperature or salinity is itself a function of temperature, salinity, and pressure (in particular warmer water expands more than
colder water for the same increase in heat content), the changes
in temperature near the warm surface ocean
have a proportionally larger influence on steric sea level rise than temperature changes in the cold deeper
ocean (at least away from the wellmixed high-latitude regions). Nevertheless, given that the global warming signal over the
twentieth century is predominantly limited to the top few hundred meters, in most regions, while ocean drift extends through the
entire water column, drift still introduces considerable bias
into both regional and global sea level rise. The
CMIP3 models show a broad range of estimates for steric sea level rise over 1950–2000 (Fig. 10a).
The spread in the raw 20C3M estimates is considerable (standard deviation ;0.76 mm yr21 with a multimodel mean of
0.45 mm yr21). In addition a number of the models indicate a lowering of sea level over the period. For
the drift-corrected sea level rise (i.e., by using drift corrected temperature and salinity) values become considerably more consistent
(standard deviation ;0.36 mm yr21) and all models now indicate a rise in sea level. While considerable intermodel variability still
exists the driftcorrected multimodel mean (;0.59 mm yr21) is consistent with the Domingues et al. (2008) observational estimate
(0.526 0.08 mm yr21, for 0–700 m, 1950–2003). Figure 10a shows raw 20C3M trends and drift-corrected estimates of forced trend
for steric sea level rise, including multiple ensemble members where available; ensemble members for a given model are generally
initialized from the same PICNTRL experiment but from different points in time, usually separated by multiple years (Table 1).
Nevertheless the
drift, which is derived from different time periods from a single PICNTRL simulation,
is very similar across ensemble members , suggesting that the linear drift approximation is valid and that natural
variability is not having a major effect on the drift estimates. Figure 10b shows a scatter of the raw 20C3M trend magnitudes versus
drift magnitudes. The drift-related error varies considerably across the models from less than 10% to over 200% for the ECHAM4
model (see previous discussion of this model). As with surface drift, subsurface drift in temperature and salinity is
spatially heterogeneous and so can result in a larger bias on regional scales. This is particularly
important for assessing twentieth-century regional changes, where the steric component of sea level rise is a
major component of the total (e.g., Domingues et al. 2008). Figure 11 shows both the raw 20C3M and driftcorrected
1950–2000 trends for three models (calculated from the surface to the bottom). A few models (e.g., MRI-CGCM2.3.2) have a well-
equilibrated preindustrial control throughout the ocean and so are essentially untroubled by drift. However, most
models are
significantly affected in certain regions. In fact for many models and regions the sign of the sea
level trend is changed by the spurious drift. For instance in the CSIRO Mk3.0 model the steric sea level anomaly
over much of the tropics and midlatitudes, estimated from the raw 20C3M temperature and salinity, changes sign once the drift is
taken into account.
---Warming- AT: Chalko

Chalko’s claims are wrong and he’s an idiot

Hube 8 (D.S., Member of the National Association of Scholars and Former Chair of the Delaware Textbook Assessment
Committee, “CBS, MSNBC Websites Promote Earthquakes' Tie to Global Warming,” NewsBuster, June 19 th,
http://newsbusters.org/blogs/d-s-hube/2008/06/19/cbs-msnbc-websites-promote-earthquakes-tie-global-warming,

An article at CBSNews.com and MSNBC.com utilizes the claims of a scientist with some very questionable ideas and theories. In
this case it's "seismic activity is five times more energetic than 20 years ago" and the reason is due to -- you guessed it -- global
warming. The research proves that destructive ability of earthquakes on Earth increases alarmingly fast and that this trend is
set to continue, unless the problem of "global warming" is comprehensively and urgently addressed. The analysis of more
than 386,000 earthquakes between 1973 and 2007 recorded on the US Geological Survey database proved that the global
annual energy of earthquakes on Earth began increasing very fast since 1990. Dr. [Tom] Chalko said that global seismic
activity was increasing faster than any other global warming indicator on Earth and that this increase is extremely alarming.
Who exactly is this Dr. Tom Chalko? And how precisely are earthquakes related to global warming? The blog SansPretense has
done a pretty thorough job dissecting Chalko's claims, as well as investigating some of Chalko's other "research." For example,
Chalko contributes to a "scientific" journal that's ... not what many would consider scientific. It's called the NU
Journal of Discovery -- the "NU" standing for Natural University. There's just one problem: There is
no Natural University. Take a look at the "university's" website to see what I mean: * NU is a University in which
there are no professors, only Autonomous Individual Intellects that respect each other's Freedom of Choice. * NU is a
University in which everyone is a perpetual STUDENT and everyone learns from everyone else. * NU is a University in which
there are no secrets, no project is ever banned and information is always open for discussion for those who can comprehend it.
* NU is a University that accepts anyone who has enough motivation to develop his/her individual intellect, regardless of age,
gender, nationality, belief system and social status. * NU is a University in which the study of the limitations of the material
reality will be only a part of exploring the Universal Law and the Purpose of Existence. * NU is a University in which there is
nothing to steal, simply because knowledge is given away free to start with. * NU is a University that is nowhere in particular
and yet everywhere on Earth. * NU is a University that will never be referred to as a building or a business. * NU is a
University that will be impossible to close down. * NU is a University in which the distribution of information will resemble the
fractal distribution of the information in the entire Universe, so by studying any fragment in sufficient detail you can eventually
learn the lot * NU is a University whose Purpose precisely coincides with The Purpose of the entire Universe - maximizing the
satisfaction from conscious existence. So, Chalko is a professor, er, um, "facilitator of knowledge" (since Natural University has
no professors) at a university that doesn't exist/yet exists everywhere. Right. Got it. Some
of Chalko's other
"scientific" interests/workshops are the study of "auras" which includes "How to see your own
Aura," "Meaning of the Aura and its colours," "Amplifying your Aura vibration," and "Matching your Aura with the
environment." Then there's "Healing and Consciousness" which includes the "Role of Aura, bio-energy and chakras," the use of
"colour to enhance your well-being," and "Choosing garments, decorating home and office." MSNBC's version of the article lists
[another of] Chalko's "credentials" at bottom -- he
is "Head of Geophysics Division at Scientific
Engineering Research, Mt Best, Australia." If you visit the site, it appears to be nothing
more than Chalko's own webpage (or one of his webpages). There are no other individuals listed at
the Division that I could find; in other words, Scientific Engineering Research is Tom Chalko, and Tom Chalko is Scientific
Engineering Research. But Chalko's claims in the CBS/MSNBC articles are tame compared to the assertions in one of his
papers linked to at the SER: The
Earth could explode as a result of global warming! I kid you not. Funny
how, as the blog SansPretense points out, the planet didn't explode long ago when the planet was
much hotter than it is now. In addition, SansPretense provides links to two articles that refute Chalko's overall thesis
about earthquakes.
---Warming- AT: Tickell

Warming doesn't cause extinction- their authors are hacks

Lomborg ‘8 (Director of the Copenhagen Consensus Center and adjunct professor at the Copenhagen Business School, Bjorn,
“Warming warnings get overheated”, The Guardian, 8/15,
http://www.guardian.co.uk/commentisfree/2008/aug/15/carbonemissions.climatechange

These alarmist predictions are becoming quite bizarre, and could be dismissed as sociological oddities, if it
weren’t for the fact that they get such big play in the media. Oliver Tickell, for instance, writes that a global warming causing
a 4C temperature increase by the end of the century would be a “catastrophe” and the beginning of the “extinction” of the
human race. This is simply silly. His evidence? That 4C would mean that all the ice on the planet would melt, bringing the
long-term sea level rise to 70-80m, flooding everything we hold dear, seeing billions of people die. Clearly, Tickell has maxed out
the campaigners’ scare potential (because there is no more ice to melt, this is the scariest he could ever conjure). But he is
wrong. Let us just remember that the UN climate panel, the IPCC, expects a temperature rise by the end of the century
between 1.8 and 6.0C. Within this range, the IPCC predicts that, by the end of the century, sea levels will rise
18-59 centimetres – Tickell [he] is simply exaggerating by a factor of up to 400. Tickell will undoubtedly
claim that he was talking about what could happen many, many millennia from now. But this is disingenuous. First, the 4C
temperature rise is predicted on a century scale – this is what we talk about and can plan for. Second, although sea-level rise
will continue for many centuries to come, the models
unanimously show that Greenland’s ice shelf will be reduced,
but Antarcticice will increase even more (because of increased precipitation in Antarctica) for the
next three centuries. What will happen beyond that clearly depends much more on emissions in future centuries. Given
that CO2 stays in the atmosphere about a century, what happens with the temperature, say, six centuries from now mainly
depends on emissions five centuries from now (where it seems unlikely non-carbon emitting technology such as solar panels will
not have become economically competitive). Third, Tickell tells us how the 80m sea-level rise would wipe out all the world’s
coastal infrastructure and much of the world’s farmland – “undoubtedly” causing billions to die. But to cause billions to die, it
would require the surge to occur within a single human lifespan. This sort of scare tactic is insidiously wrong
and misleading, mimicking a firebrand preacher who claims the earth is coming to an end and we need to repent. While it
is probably true that the sun will burn up the earth in 4-5bn years’ time, it does give a slightly different perspective on the need
for immediate repenting. Tickell’s claim that 4C will be the beginning of our extinction is again
many times beyond wrong and misleading, and, of course, made with no data to back it up. Let us
just take a look at the realistic impact of such a 4C temperature rise. For the Copenhagen Consensus , one of the
lead economists of the IPCC, Professor Gary Yohe, did a survey of all the problems and all
the benefits accruing from a temperature rise over this century of about approximately 4C.
And yes, there will, of course, also be benefits: as temperatures rise, more people will die from heat, but fewer
from cold; agricultural yields will decline in the tropics, but increase in the temperate zones, etc. The
model evaluates the impacts on agriculture, forestry, energy, water, unmanaged ecosystems, coastal zones, heat and cold
deaths and disease. The bottom line is that benefits from global warming right now outweigh the costs
(the benefit is about 0.25% of global GDP). Global warming will continue to be a net benefit until about 2070, when the
damages will begin to outweigh the benefits, reaching
a total damage cost equivalent to about 3.5% of
GDP by 2300. This is simply not the end of humanit y. If anything, global warming is a net
benefit now; and even in three centuries, it will not be a challenge to our civilisation. Further,
the IPCC expects the average person on earth to be 1,700% richer by the end of this
century.
Water Wars Answers
Frontline

No water wars—their ev is hype.

Katz 11—Lecturer of Geography and Environmental Studies @ University of Haifa [Dr. David Katz (PhD in Natural Resource Policy
& MA in Applied Economics @ University of Michigan), “Hydro-Political Hyperbole: Examining Incentives for Overemphasizing the
Risks of Water Wars,” Global Environmental Politics, Volume 11, Number 1, February 2011, pp. 12-35]

Evidence and Perception

In sum, despite some instances of violent conflict over water, there is little systematic evidence of war over water resources.
Evidence for a deterministic relationship between water scarcity and the outbreak of armed conflict is
particularly weak . Less ambitious claims that water shortages will contribute to insecurity, which can, in turn, lead to violent
conflict, have more empirical support. Even here, however, the
importance of water as a causal variable is
questionable. Several studies have found that variables such as regime type and institutional
capacity are much more important indicators of conflict potential, 43 and may have mitigating effects
on any water-conflict link. As a consequence of accumulated research , many scholars have
concluded that risks of water wars are low , 44 and others have toned down or qualified their statements about
the likelihood of future water wars.45 Some governmental reports have limited their contentions to highlighting that water scarcity
can aggravate conflicts and increase insecurity,46 and many studies now emphasize water as a tool for
cooperation .47 Warnings and predictions of imminent water wars continue to be
commonplace, however. In a review of published academic literature, Gupta and van der Zaag find that articles on water
conflict outnumber those on cooperation by nearly three to one, and are five times more likely to be cited.48 This article will now
turn to offering possible explanations for the persistence and popularity of such declarations despite the bulk of expert opinion
downplaying the risks of water wars. Incentives to Stress a Water War Scenario Incentives Presented in Existing Literature Observers

have noted that various actors may have incentives to stress or even exaggerate the risks of water wars.
Lonergan notes, for instance, that in “many cases, the comments are little more than media hype ; in
others, statements have been made for political reasons .”49 Beyond mere acknowledgement of the possibility
of such incentives, however, little research has attempted to understand what these incentives are and how they may differ
between actors. An understanding of the different motivations
of various groups of actors to stress the
possibility of imminent water wars can help explain the continued seemingly disproportionate popularity
of such messages and help to evaluate such warnings more critically.pg. 17-18 //1nc

Water Wars are a myth- water scarcity causes peace

Wolf et al. ‘5 (State of the World 2005 Global Security Brief #5: Water Can Be a Pathway to Peace, Not War by A. T. Wolf - A.
Kramer - A. Carius - G. ... on June 1, 2005 About the authors: Aaron T. Wolf is Associate Professor of Geography in the Department of
Geosciences at Oregon State University and Director of the Transboundary Freshwater Dispute Database. Annika Kramer is Research
Fellow and Alexander Carius is Director of Adelphi Research in Berlin. Geoffrey D. Dabelko is the Director of the Environmental
Change and Security Project at the Woodrow Wilson International Center for Scholars in Washington, D.C.

“Water wars are coming!” the newspaper headlines scream. It seems obvious—rivalries over water have
been the source of disputes since humans settled down to cultivate food. Even our language reflects these ancient roots: “rivalry”
comes from the Latin rivalis, or “one using the same river as another.” Countries or provinces bordering the same river (known as
“riparians”) are often rivals for the water they share. As
the number of international river basins (and impact of
water scarcity) has grown so do the warnings that these countries will take up arms to ensure
their access to water. In 1995, for example, World Bank Vice President Ismail Serageldin claimed
that “the wars of the next century will be about water.” These apocalyptic warnings fly in the
face of history : no nations have gone to war specifically over water resources for thousands of
years . International water disputes— even among fierce enemies —are resolved peacefully,
even as conflicts erupt over other issues . In fact, instances of cooperation between riparian
nations outnumbered conflicts by more than two to one between 1945 and 1999. Why?
Because water is so important, nations cannot afford to fight over it . Instead, water fuels
greater interdependence . By coming together to jointly manage their shared water resources,
countries build trust and prevent conflict . Water can be a negotiating tool, too: it can offer a
communication lifeline connecting countries in the midst of crisis . Thus, by crying “water
wars,” doomsayers ignore a promising way to help prevent war: cooperative water resources
management. Of course, people compete—sometime violently—for water. Within a nation, users—farmers, hydroelectric
dams, recreational users, environmentalists—are often at odds, and the probability of a mutually acceptable solution falls as the
number of stakeholders rises. Water is never the single—and hardly ever the major—cause of conflict. But it can exacerbate existing
tensions. History is littered with examples of violent water conflicts: just as Californian farmers bombed pipelines moving water from
Owens Valley to Los Angeles in the early 1900s, Chinese farmers in Shandong clashed with police in 2000 to protest government
plans to divert irrigation water to cities and industries. But these conflicts usually break out within nations. International rivers are a
different story. The world’s 263 international river basins cover 45.3 percent of Earth’s land surface, host about 40 percent of the
world’s population, and account for approximately 60 percent of global river flow. And the number is growing, largely due to the
“internationalization” of basins through political changes like the breakup of the Soviet Union, as well as improved mapping
technology. Strikingly, territory in 145 nations falls within international basins, and 33 countries are located almost entirely within
these basins. As many as 17 countries share one river basin, the Danube. Contrary to received wisdom, evidence proves this
interdependence does not lead to war. Researchers at Oregon State University compiled a dataset of every reported interaction
(conflictive or cooperative) between two or more nations that was driven by water in the last half century. They found that the rate
of cooperation overwhelms the incidence of acute conflict. In the last 50 years, only 37 disputes involved violence, and 30 of those
occurred between Israel and one of its neighbors. Outside of the Middle East, researchers found only 5 violent events while 157
treaties were negotiated and signed. The total number of water-related events between nations also favors cooperation: the 1,228
cooperative events dwarf the 507 conflict-related events. Despite the fiery rhetoric of politicians—aimed more
often at their own constituencies than at the enemy—most actions taken over water are mild.
Of all the events, 62 percent are verbal, and more than two-thirds of these were not official statements. Simply put, water
is a greater pathway to peace than conflict in the world’s international river basins.
International cooperation around water has a long and successful history; some of the world’s
most vociferous enemies have negotiated water agreements. The institutions they have
created are resilient, even when relations are strained . The Mekong Committee, for example,
established by Cambodia, Laos, Thailand, and Viet Nam in 1957, exchanged data and
information on the river basin throughout the Viet Nam War. Israel and Jordan held secret
“picnic table” talks to manage the Jordan River since 1953, even though they were officially at
war from 1948 until the 1994 treaty. The Indus River Commission survived two major wars
between India and Pakistan. And all 10 Nile Basin riparian countries are currently involved in
senior government–level negotiations to develop the basin cooperatively, despite the verbal
battles conducted in the media. Riparians will endure such tough, protracted negotiations to
ensure access to this essential resource and its economic and social benefits. Southern African
countries signed a number of river basin agreements while the region was embroiled in a
series of wars in the 1970s and 1980s, including the “people’s war” in South Africa and civil
wars in Mozambique and Angola. These complex negotiations produced rare moments of
peaceful cooperation. Now that most of the wars and the apartheid era have ended, water
management forms one of the foundations for cooperation in the region, producing one of the
first protocols signed within the Southern African Development Community (SADC).

No water wars- best database proves

Tertrais ’11 (Senior Research Fellow at the Fondation pour la Recherche Stratégique, Paris) 11
(Bruno, The Climate Wars Myth, The Washington Quarterly, Volume 34, Issue 3, pages 17-29)

And water crises do not mean water wars. The issue of access to water resources is undoubtedly a major dimension
of numerous regional crises, in particular in the Greater Middle East, as testified by decades-old disputes between Turkey and Syria,
or Egypt and Sudan. The value of strategic locations such as the Golan Heights or Kashmir is not a small part of tensions between
Syria and Israel, or India and Pakistan. And water sharing can be the cause of local disputes sometimes degenerating into small-scale
collective violence in Africa or Asia. However, experts from the University of Oregon, who maintain the
most complete database on this topic , state that there has never been a “war over water ” (that
is, large-scale collective violence for the sake of a water resource) in the past 4,500 years.35 The last war over water
opposed two Sumerian cities in the middle of the third millennium B.C.E., about sharing the waters of the Tigris and Euphrates.
There are good reasons for such a scant record. Any country seeking to control the upstream of
a river would need to ensure complete and permanent domination over it, which would be an
ambitious goal. In the modern era, resorting to arms over water (like resorting to arms over oil)
is just not worth the cost. Especially for those whose geographical location and budget can afford to build desalination
plants—which is the case for some of the most water-stressed countries, those located on the Arabian Peninsula. One should
therefore not be surprised that access to water has always generated more cooperation than
conflict. Since antiquity, thousands of agreements and treaties have been signed for water-
sharing. And cooperation between adversaries has stood the test of wartime , as was seen
during the 20th century in the Middle East, South Asia, or Southeast Asia.

No Water Wars

Barnaby ‘9 (Wendy, editor of People & Science, the magazine published by the British Science Association, “Do nations go to
war over water?,”, http://www.nature.com/nature/journal/v458/n7236/full/458282a.html, March 19, 2009)

The United Nations warned as recently as last week that climate change harbours the potential for serious conflicts over water. In its
World Water Development Report1 of March 2009, it quotes UN Secretary-General Ban Ki-moon noting the risk of water scarcity
"transforming peaceful competition into violence". It is statements such as this that gave birth to popular notions of 'water wars'. It
is time we dispelled this myth. Countries do not go to war over water, they solve their water shortages
through trade and international agreements. Cooperation, in fact, is the dominant response to
shared water resources. There are 263 cross-boundary waterways in the world. Between 1948
and 1999, cooperation over water, including the signing of treaties, far outweighed conflict
over water and violent conflict in particular. Of 1,831 instances of interactions over
international freshwater resources tallied over that time period (including everything from unofficial
verbal exchanges to economic agreements or military action), 67% were cooperative, only 28% were conflictive,
and the remaining 5% were neutral or insignificant. In those five decades, there were no formal declarations of
war over water.
Their Ev is bad scholarhsip

Barnaby ‘9 (Wendy, editor of People & Science, the magazine published by the British Science Association, “Do nations go to
war over water?,”, http://www.nature.com/nature/journal/v458/n7236/full/458282a.html, March 19, 2009)

Yet the myth of water wars persists. Climate change, we are told, will cause water shortages. The Intergovernmental Panel on
Climate Change estimates that up to 2 billion people may be at risk from increasing water stress by the 2050s, and that this number
could rise to 3.2 billion by the 2080s7. Water management will need to adapt. But the mechanisms of trade, international
agreements and economic development that currently ease water shortages will persist. Researchers, such as Aaron Wolf at Oregon
State University, Corvallis, and Nils Petter Gleditsch at the International Peace Research Institute in Oslo, point out that
predictions of armed conflict come from the media and from popular , non-peer-reviewed
work. There is something other than water for which shortages, or even the perceived threat of future shortages, does cause war
— oil. But the strategic significance of oil is immeasurably higher than that of water. Serious interruptions of oil supplies would stop
highly developed economies in their tracks. Oil is necessary for a developed economy, and a developed economy provides for all the
needs of its citizens, including water. People in developed economies do not die of thirst. My encounter with Allan's work killed my
book. I offered to revise its thesis, but my publishers pointed out that predicting an absence of war over water
would not sell.
---EXT- No Water Wars

Recent research proves- water scarcity is more likely to cause cooperation than war

Deen 7 (Thalif, Staff – IPS, “Water Wars A Myth”, Inter Press Service, 8-25, Lexis)

history has demonstrated that cooperation, rather than conflict, is


"Despite the potential problem,
likely in shared basins," UNESCO concludes. The Stockholm International Water Institute (SIWI) says that 10- to 20-
year-old arguments about conflict over water are still being recycled. "Such arguments ignore
massive amounts of recent research which shows that water-scarce states that share a water
body tend to find cooperative solutions rather than enter into violent conflict, " the institute says. SIWI
says that during the entire "intifada" -- the ongoing Palestinian uprising against Israel in the occupied territories of West Bank
and Gaza -- the only thing on which the two warring parties continued to cooperate at a basic level was their shared waters.
"Thus, rather than reaching for arguments for the 'water war hypotheses,' the facts seem to support the idea that water is a
uniting force and a potential source of peace rather than violent conflict." SIWI said. Ghosh, co-author of
the UNDP study, pointed out several agreements which were "models of cooperation", including the Indus Waters Treaty, the
Israel-Jordan accord, the Senegal River Development Organisation and the Mekong River Commission. A study sponsored by the
Washington-based Woodrow Wilson International Centre for Scholars points that despite newspaper headlines screaming
"water wars are coming!", these apocalyptic warnings fly in the face of history. "No nations have gone to war specifically over
water resources for thousands of years. International water disputes -- even among fierce enemies -- are resolved peacefully,
even as conflicts erupt over other issues," it says. The study also points out instances of cooperation between riparian nations --
countries or provinces bordering the same river -- that outnumbered conflicts by more than two to one between 1945 and
1999. Why? "Because water is so important, nations cannot afford to fight over it. Instead, water fuels greater interdependence.
By coming together to jointly manage their shared water resources, countries can build trust and prevent conflict," argues the
study, jointly co-authored by Aaron Wolf, Annika Kramer, Alexander Carius and Geoffrey Dabelko.
---Water Wars- AT: Middle East

No Middle East water wars- instead scarcity causes cooperation

Deen ‘5 (Are Water Wars a Fantasy, or a Future Reality? By Thalif Deen Inter Press Service August 29, 2005

The Middle East, one of the world's perennial war zones, has traditionally been blessed with a
surfeit of oil and cursed by a scarcity of water . The irony, says one Arab diplomat half-jokingly, is
that whenever energy-rich Gulf states dig for water, they invariably strike oil. The longstanding
speculation among some political experts is that the world's future wars will be fought over
water, not oil. Asked whether she subscribes to this view, Sunita Narain, the winner of the 2005 Stockholm Water Prize, said:
"Water wars are not inevitable. It lies in our hands -- and in our minds." The award, including 150,000 dollars in cash and a crystal
sculpture, was presented by King Carl XVI Gustaf of Sweden at a formal ceremony in Stockholm last week. Narain, director of the
Centre for Science and Environment in New Delhi and publisher of the widely-acclaimed environmental magazine Down to Earth,
said water is very different from oil. "Water is a replenishable commodity. The question is society's relationship to live with water.
The management of water is critical. Water wars or water peace is in our hands," Narain told IPS. She admits that "water stress"
leads to tension and conflicts -- as evidenced by a recent police shooting of farmers in Rajasthan, India. The farmers were protesting
the release of water from their lands to neighboring cities. "It was a very violent agitation," said Narain, recounting two other
incidents of violence over water that resulted in the deaths of Indian farmers. Narain believes that water "is one thing that is
crippling India's growth". "I am not here as a pessimist saying that India is doomed and that water wars are going to happen, and we
are going to destroy ourselves. I am saying very clearly that if India continues to go in this route, yes there will be water wars and
there will be water conflicts. And we will be more and more crippled in our growth," she warned. Narain noted that India has
political leaders who are listening to this message. "They are recognizing the need for a new paradigm. But this new paradigm
unfortunately demands good politics, because it demands decentralization of power, and it demands the involvement of people."
The Stockholm International Water Institute (SIWI), which bestows the water prize every year, points out that most parts of the
Middle East are already facing severe water scarcities and stress, making future prospects for food security bleak. "The availability of
water resources to secure sufficient food production for growing populations is one of the biggest challenges faced by water and
In a newly-released publication titled "Liquid Assets: An Economic
agricultural managers," SIWI said.
Approach for Water Management and Conflict Resolution in the Middle East and Beyond",
Franklin Fisher and Annette Huber-Lee argue that the common view of water as an inevitable
cause of future wars is neither rational nor necessary . "Typically, two or more parties with claim to the same
water sources are thought to play a zero-sum game, with each side placing a high emotional and political value on the ownership of
the water," they point out. However,
say the authors, when disputes in ownership are expressed as
disputes about money values, in most cases, the benefits of ownership will be surprisingly small.
"By assigning an economic value to water and treating it as a tradable source, parties see that
the gains from cooperation exceed the costs, resulting from the change in ownership . A zero-
sum game becomes a win-win situation," they add. In a paper about the Tigris and Euphrates
rivers, which flow through Turkey, Syria, Iraq, Iran, Kuwait and even northern Saudi Arabia,
Prof. Olcay Unver of the Ohio-based Kent State University says that despite the political volatility
of the issue, shared water resource management between Turkey, Syria and Iraq may promote
international cooperation, as opposed to interstate conflict, in the coming decade. In a recent
presentation to the Washington-based Centre for Strategic and International Studies, Unver said
that during a time of great upheaval and transformation in the Middle East, the Tigris and
Euphrates river basins "could bring about a unique rebuttal to worries over 'water wars' in
one of the most conflicted regions on earth". With the change of regime in Iraq -- and the
potential opening of Syria -- now may be an appropriate time to focus on cross-border water
issues as a catalyst for regional cooperation and economic development, he argues.
---Water Wars- AT: Indo-Pak

No Indo-Pak water wars- instead scarcity causes cooperation

Alam ‘2 (Questioning the water wars rationale: a case study of the Indus Waters Treaty Geographical Journal, The, Dec, 2002 by
Undala Z. Alam, Professor at Queen’s University, Belfast

Despite bellicose statements, even at the highest levels, India and Pakistan persisted in
negotiating over the Indus basin. This suggests that though statements made by key decisionmakers
in public may suggest a move towards war, the statements are used to generate domestic support for a political
position. As seen in the Indus basin, the political rhetoric did not match the governments' actions which
sought to resolve an international water dispute through cooperation. In winter 2001-2002,
against a backdrop of deteriorating Indo-Pakistan relations, notably the attack on the Indian
parliament in December 2001, statements were made in India that it would unilaterally
abrogate the Indus Waters Treaty (Dawn 2001 2002a; The Tribune 2002). Despite considerable
discussion within the Indian and Pakistani press, the Permanent Indus Commission, set up as
part of the Treaty, met for the 37th time in New Delhi on 29 May 2002 for three days (Dawn 2002b;
The Hindu 2002; The Indian Express 2002; Paukert 2002). This can be seen as a clear indication of the countries
commitment to the Treaty despite statements by senior decisionmakers and speculation in the press.
The experience from the Indus basin, therefore, throws into question whether public statements made for a domestic audience are
The water wars rationale would
truly indicative of a country's intent to go to war over shared waters. Conclusion
suggest that India and Pakistan should have gone to war over the Indus basin. Despite water
scarcity, competitive use, Pakistan's absolute dependency upon the basin and the wider dispute
involving a series of issues including Kashmir, the two countries cooperated instead. With the
good offices of the World Bank, India and Pakistan negotiated the Indus Waters Treaty over a
period of nine years, signing it in September 1960. The principal explanation for this Indo-Pakistan cooperation is based upon
the finances that the Treaty brought with it in the Indus Basin Development Fund. However, the financial explanation is inadequate,
as it does not explain why the countries cooperated over nine years so that there was a treaty to finance. Nor does it explain the
repeated negotiations for the temporary ad hoc agreements that supplied water to Pakistan during the larger negotiations for a
comprehensive agreement. An alternative explanation is that India
and Pakistan cooperated because it was
water rational. In other words, cooperation was needed to safeguard the countries' long-term
access to shared water. This suggests that the issues of water scarcity , competitive use and a wider
conflict do not necessarily lead to war, since war cannot guarantee a country's water supply in
the long term.
US Water Shortage Answers
No US water shortages

UPI ‘13
(Study: U.S. water shortage is not so dire http://www.upi.com/Science_News/2013/01/30/Study-US-water-shortage-is-not-so-
dire/UPI-86981359590529/#ixzz2ONSLwj5f)

GAINESVILLE, Fla., Jan. 30 (UPI) -- While drought and water restrictions paint a bleak picture of the U.S. water supply,
researchers at the University of Florida say conditions aren't so bad. Previous assessments of urban water
supplies typically used what is known as a "runoff-based approach" that takes into account factors such as river flows
and rainfall amounts, researchers said, but other factors should also be taken into consideration. Those
previous assessments did not consider the infrastructure used to maintain urban water supplies, such as water stored in
aquifers, lakes, reservoirs or water that's pumped in to an area and stored, they said. Jim Jawitz, a
University of Florida soil and water science professor, and Julie Padowski, who earned her doctoral degree from UF and is now a
postdoctoral researcher at Stanford University, said using the runoff-based approach suggests 47 percent of the total U.S. population
is vulnerable to water scarcity issues. However, they said, when infrastructure was accounted for the number
dropped to just 17 percent of the population.
Water Pollution Answers
Water quality improving

Hayward 4 (Steven F., Senior Fellow – Pacific Research Institute, Index of Leading Indicators,
http://www.pacificresearch.org/pub/sab/enviro/04_enviroindex/Enviro_2004.pdf)

The EPA reports that the percentage of the U.S. population served by water systems that have
reported no violations of any health-based standards has risen from 79 percent in 1993 to 94 percent in 2002.
· While water quality monitoring is much less reliable than that of air quality, there have been improvements in
recent years. Indiana, Maryland, and at least 17 other states are developing statistical sampling systems that
promise to produce more useful trend data. · Private efforts such as Ducks Unlimited have been enormously
successful in conserving habitat for waterfowl and other wildlife, and private water trusts have also
helped further conservation efforts in a number of states. A lack of consistent, comprehensive trend data for water
quality in the U.S. remains one of the largest frustrations in the development of meaningful environmental indicators.a circumstance
we have commented upon in past editions. The EPA.s Draft Report on the Environment last year also noted this problem: .At this
time there is not sufficient information to provide a national answer to this question [of water quality] with confidence and scientific
credibility.. It
is certain that water quality has improved in many ways since the passage of the
Clean Water Act in 1972, but there are no data-sets or measures that are commensurate with our national findings on air
quality. And without national benchmarks, it can be difficult to make judgments about local conditions, assign priorities to clean up
efforts, or even to enforce major water pollution statutes.

Multiple alt causes to water pollution

Lenntech 8 (Water Treatment, “Sources of Water Pollution”, http://www.lenntech.com/groundwater/pollution-sources.htm)

Saltwater encroachment associated with over drafting of aquifers or natural leaching from natural occurring deposits are
natural sources of groundwater pollution. Most concern over groundwater contamination has centered on pollution
associated with human activities. Human groundwater contamination can be related to waste disposal
(private sewage disposal systems, land disposal of solid waste, municipal wastewater, wastewater impoundments, land spreading of
sludge, brine disposal from the petroleum industry, mine wastes, deep-well disposal of liquid wastes, animal feedlot wastes,
radioactive wastes) or not directly related to waste disposal (accidents, certain agricultural activities, mining, highway
deicing, acid rain, improper well construction and maintenance, road salt).
Yemen Civil War Answers
Uprising will stay non-violent and geography and Saudi Arabia prevent escalation

Phillip 2011 (Chapter Seven: Yemen and the 2011 Arab Uprisings, Sarah Phillip, Special Issue: Yemen and the Politics of
Permanent Crisis,  Available online: 04 Jul 2011,
http://www.tandfonline.com.proxy1.cl.msu.edu/doi/pdf/10.1080/19445571.2011.600540)

The outbreak of civil war is a possibility under any scenario, particularly if the longstanding personal acrimony between the al-Ahmar
family and President Saleh’s family increases. Despite the obvious combustibility of the situation, there are some factors
that may help to reduce the chances of a long-term civil war. Firstly, while Saleh’s son commands the
Republican Guard, many of the Guardsmen have family and tribal kinsmen in both Ali Muhsin’s 1st Armoured Division and in the
tribes that support the al-Ahmar family. The relatively narrow geographical and tribal origins of these three
key groups could help to at least limit the potential for resorting to deadly force over an
extended period. Secondly, although the availability of arms is often cited as a factor that increases
the risk of civil war, it could also work the other way. Ordinary Yemenis are acutely aware that violence can
spiral exponentially as a result of small miscalculations. The fact that the protesters were resolutely nonviolent
despite the regime’s violence against them was just one indication of how well this is
understood. In tribal conflicts, for example, the goal is less to vanquish an opponent than to demonstrate the ability to apply
symbolic force in defence of one’s position and then to negotiate a solution in which both sides retain honour. While this
tends to lay a foundation for theatrical brinkmanship in which the cost of miscalculation is both
real and high, it also means that violent outbursts tend to be relatively short-lived . As of June 2011,
the casualties caused by the fighting between the Ahmars and those loyal to Saleh were lower
than one might expect, considering the amount of firepower used.
Yucca Answers
Yucca is safe

Senate Report 6
Committee on Environment and Public Works, “Yucca Mountain: The Most Studied Real Estate on the Planet”, March,
http://epw.senate.gov/repwhitepapers/YuccaMountainEPWReport.pdf

From the science of 1986, which indicated that Yucca Mountain was the best of nine potential U.S. sites for geologic
disposal of nuclear waste, to the science of 1998, which validated the expectation that “remote, desert regions of the
Southwest are well-suited for a geologic repository”, to the science of 2002, demonstrating that a geologic repository at
Yucca Mountain can protect public health and safety in accordance with stringent regulations – confidence in this
particular site as the solution to meeting U.S. nuclear waste disposal needs has been continually
on the increase. More is known about Yucca Mountain than any other parcel of real estate on
the planet. This knowledge extends well below the surface through miles of tunnels and dozens
of drillings. It has been confirmed in the laboratory, reviewed by independent experts , and
validated against information from analogous sites around the world. Through all that has been gained by
20 years and $8.6 billion dollars of world leading scientific research, one thing has remained constant – the more we
examine Yucca Mountain, the better it looks. There is certainly no reason in science not to
move forward directly with this project .

You might also like